From owner-obm-l@sucuri.mat.puc-rio.br Sun Sep 1 00:15:43 2002 Return-Path: Received: (from majordom@localhost) by sucuri.mat.puc-rio.br (8.9.3/8.9.3) id AAA19520 for obm-l-MTTP; Sun, 1 Sep 2002 00:14:34 -0300 Received: from gorgo.centroin.com.br (gorgo.centroin.com.br [200.225.63.128]) by sucuri.mat.puc-rio.br (8.9.3/8.9.3) with ESMTP id AAA19516 for ; Sun, 1 Sep 2002 00:14:32 -0300 Received: from centroin.com.br (du42c.rjo.centroin.com.br [200.225.58.42]) (authenticated bits=0) by gorgo.centroin.com.br (8.12.2/8.12.1) with ESMTP id g813FFCR026934 for ; Sun, 1 Sep 2002 00:15:16 -0300 (BRT) Message-ID: <3D7186D6.4060107@centroin.com.br> Date: Sun, 01 Sep 2002 00:17:42 -0300 From: Augusto =?ISO-8859-1?Q?C=E9sar?= Morgado User-Agent: Mozilla/5.0 (Windows; U; Win98; en-US; rv:0.9.4.1) Gecko/20020508 Netscape6/6.2.3 X-Accept-Language: en-us MIME-Version: 1.0 To: obm-l@mat.puc-rio.br Subject: Re: [obm-l] esclarecimento References: Content-Type: multipart/alternative; boundary="------------050901000500000101080902" Sender: owner-obm-l@sucuri.mat.puc-rio.br Precedence: bulk Reply-To: obm-l@mat.puc-rio.br --------------050901000500000101080902 Content-Type: text/plain; charset=ISO-8859-1; format=flowed Content-Transfer-Encoding: 8bit Eh 3/8. Adoraria saber qual o raciocinio que conduziu a resposta 1/4. Supondo naturalmente as moedas nao-tendenciosas. Morgado Lltmdrtm@aol.com wrote: > Ao jogar três moedas, qual a probabilidade de dar duas caras e uma coroa? > Alguns colegas acham que é 1 / 4 outros acham que é 3 / 8. Por que a > confusão? > É possível as duas respostas estarem corretas? --------------050901000500000101080902 Content-Type: text/html; charset=us-ascii Content-Transfer-Encoding: 7bit Eh 3/8. Adoraria saber qual o raciocinio que conduziu a resposta 1/4.
Supondo naturalmente as moedas nao-tendenciosas.
Morgado

Lltmdrtm@aol.com wrote:
Ao jogar três moedas, qual a probabilidade de dar duas caras e uma coroa?
Alguns colegas acham que é 1 / 4 outros acham que é 3 / 8. Por que a confusão?
É possível as duas respostas estarem corretas?

--------------050901000500000101080902-- ========================================================================= Instruções para entrar na lista, sair da lista e usar a lista em http://www.mat.puc-rio.br/~nicolau/olimp/obm-l.html O administrador desta lista é ========================================================================= From owner-obm-l@sucuri.mat.puc-rio.br Sun Sep 1 01:05:13 2002 Return-Path: Received: (from majordom@localhost) by sucuri.mat.puc-rio.br (8.9.3/8.9.3) id BAA20267 for obm-l-MTTP; Sun, 1 Sep 2002 01:04:55 -0300 Received: from smtp.ieg.com.br (stone.protocoloweb.com.br [200.226.139.11]) by sucuri.mat.puc-rio.br (8.9.3/8.9.3) with ESMTP id BAA20263 for ; Sun, 1 Sep 2002 01:04:53 -0300 Received: from localhost (wh1stler@200-158-118-125.dsl.telesp.net.br [200.158.118.125]) by smtp.ieg.com.br (IeG relay/8.9.3) with SMTP id g8141GfE097337 for ; Sun, 1 Sep 2002 01:01:18 -0300 (BRT) From: Tonik To: obm-l@mat.puc-rio.br Date: Sun, 01 Sep 2002 01:04:25 -0300 X-Priority: 3 (Normal) Organization: Tonik In-Reply-To: <3D7186D6.4060107@centroin.com.br> Message-Id: Subject: Re: [obm-l] esclarecimento MIME-Version: 1.0 Content-Type: text/plain; charset="windows-1252" X-Mailer: Opera 6.04 build 1135 Sender: owner-obm-l@sucuri.mat.puc-rio.br Precedence: bulk Reply-To: obm-l@mat.puc-rio.br 01/09/02 00:17:42, Augusto César Morgado wrote: > Ao jogar três moedas, qual a probabilidade de dar duas caras e uma coroa? Isso eh um acontecimento binario, ou seja, acontecimentos individuais com probabilidades individuais iguais, em que vc deseja saber a probabilidade de acontecer determinado numero de vezes uma das partes do acontecimento. Existe uma formulinha para isso, em q eu andei mudando o nome das variaveis para facilitar a memorizacao: Pbin = C(t,a) * Pa^a * Pe^e em que C(n,p) = combinacao, binomio de newton ou ainda n!/(p!*(n-p)!) t = tentativas, a = acertos, e = erros Pa = probabilidade de acerto em cada tentativa Pe = probabilidade de erro em cada tentativa No caso das 3 moedas, vamos chamar cara de acerto e coroa de erro, temos t=3, a=2, e=1, e as probabilidades de erro e acerto sao iguais a 1/2. Pbin = C(3,2) * (1/2)^2 * (1/2)^1 resolvendo a conta temos Pbin = 3!/2! * 1/4 * 1/2 Pbin = 3 * 1/8 Pbin = 3/8 ------- Agora dexa eu perguntar, eu to com quatro duvidas, ou melhor, conceitos, pendurados, e preciso que alguem me ilumine: 1) "Pelamordedeus", alguem escreva a teoria das equacoes reciprocas, primeira e segunda especie, sua teoria e principalmente suas propriedades. Nao deve ser longo, se eu tivesse uma boa biblioteca perto daqui eu ja teria ido e nao teria perguntado para 3 professores diferentes q por 2 semanas estao enrolando q estao meio sem tempo. Entao por favor, digam-me o que esta escrito, nao aonde esta escrito. Ja tentei procurar na internet e se acha apenas um unico site (fora os seus mirrors) e eu sei q ele esta incompleto. 2) Funcao f: R -> R tal que f(x) = (2+x)/(2-x). Esta obvio como calcular o conjunto Dominio da funcao (que é R-{2}) da mesma forma como deve estar obvio calcular o seu conjunto imagem, só que eu nao sei como se nao montando o grafico... 3) Será que um livro deixa de circulacao porque é bom? Cade as formulas para se achar o volume de setor esferico, calota de esfera e tronco de esfera (ou algo analogo)? Alguem pode me passar essas formulinhas? Sera q existe um jeito de calcular o volume de um solido de revolucao dado a area do corte transversal e o eixo de giro? 4) Sem calculadora, como calculo cos(arcsen(1/3)) ? Ou eh impossivel/inviavel? Obrigado, Tonik ========================================================================= Instruções para entrar na lista, sair da lista e usar a lista em http://www.mat.puc-rio.br/~nicolau/olimp/obm-l.html O administrador desta lista é ========================================================================= From owner-obm-l@sucuri.mat.puc-rio.br Sun Sep 1 01:58:47 2002 Return-Path: Received: (from majordom@localhost) by sucuri.mat.puc-rio.br (8.9.3/8.9.3) id BAA20977 for obm-l-MTTP; Sun, 1 Sep 2002 01:57:32 -0300 Received: from puma.unisys.com.br (ns2.unisys.com.br [200.220.64.7]) by sucuri.mat.puc-rio.br (8.9.3/8.9.3) with ESMTP id BAA20972 for ; Sun, 1 Sep 2002 01:57:30 -0300 Received: from jf (riohiper01p148.uninet.com.br [200.220.2.148]) by puma.unisys.com.br (8.12.3/8.12.3) with SMTP id g814vvjR028849 for ; Sun, 1 Sep 2002 01:57:58 -0300 (EST) X-Spam-Filter: check_local@puma.unisys.com.br by digitalanswers.org Message-ID: <00a401c25174$b6f71ee0$9402dcc8@jf> From: "Jose Francisco Guimaraes Costa" To: References: Subject: Re: [obm-l] fisica Date: Sun, 1 Sep 2002 02:02:02 -0300 MIME-Version: 1.0 Content-Type: text/plain; charset="iso-8859-1" Content-Transfer-Encoding: 8bit X-Priority: 3 X-MSMail-Priority: Normal X-Mailer: Microsoft Outlook Express 6.00.2600.0000 X-MimeOLE: Produced By Microsoft MimeOLE V6.00.2600.0000 Sender: owner-obm-l@sucuri.mat.puc-rio.br Precedence: bulk Reply-To: obm-l@mat.puc-rio.br Vá até http://scienceworld.wolfram.com/physics/ e especifique o que quer saber: angular momentum (momento angular), moment of inertia (momento de inércia), torque (torque)... JF ----- Original Message ----- From: "diegoalonsoteixeira" To: Cc: Sent: Saturday, August 31, 2002 8:12 AM Subject: [obm-l] fisica Alguem poderia me indicar algum site na internet que explique detalhadamente momento angular,momento de inércia ,torque.. obrigado ========================================================================= Instruções para entrar na lista, sair da lista e usar a lista em http://www.mat.puc-rio.br/~nicolau/olimp/obm-l.html O administrador desta lista é ========================================================================= From owner-obm-l@sucuri.mat.puc-rio.br Sun Sep 1 02:18:12 2002 Return-Path: Received: (from majordom@localhost) by sucuri.mat.puc-rio.br (8.9.3/8.9.3) id CAA21288 for obm-l-MTTP; Sun, 1 Sep 2002 02:17:48 -0300 Received: from puma.unisys.com.br (ns2.unisys.com.br [200.220.64.7]) by sucuri.mat.puc-rio.br (8.9.3/8.9.3) with ESMTP id CAA21284 for ; Sun, 1 Sep 2002 02:17:46 -0300 Received: from jf (riohiper01p148.uninet.com.br [200.220.2.148]) by puma.unisys.com.br (8.12.3/8.12.3) with SMTP id g815IEjR000562 for ; Sun, 1 Sep 2002 02:18:14 -0300 (EST) X-Spam-Filter: check_local@puma.unisys.com.br by digitalanswers.org Message-ID: <00af01c25177$8c2e5540$9402dcc8@jf> From: "Jose Francisco Guimaraes Costa" To: References: Subject: Re: [obm-l] esclarecimento Date: Sun, 1 Sep 2002 02:22:19 -0300 MIME-Version: 1.0 Content-Type: text/plain; charset="Windows-1252" Content-Transfer-Encoding: 8bit X-Priority: 3 X-MSMail-Priority: Normal X-Mailer: Microsoft Outlook Express 6.00.2600.0000 X-MimeOLE: Produced By Microsoft MimeOLE V6.00.2600.0000 Sender: owner-obm-l@sucuri.mat.puc-rio.br Precedence: bulk Reply-To: obm-l@mat.puc-rio.br Para ver as fórmulas pedidas, vá até http://mathworld.wolfram.com/ e especifique: spherical sector (setor esférico), spherial cap (calota de esfera), spherical segment (tronco de esfera). O volume de um sólido de revolução está em http://mathworld.wolfram.com/SolidofRevolution.html JF ----- Original Message ----- From: "Tonik" To: Sent: Sunday, September 01, 2002 1:04 AM Subject: Re: [obm-l] esclarecimento > Agora dexa eu perguntar, eu to com quatro duvidas, ou melhor, conceitos, > pendurados, e preciso que alguem me ilumine: > > 3) Será que um livro deixa de circulacao porque é bom? Cade as formulas para se achar > o volume de setor esferico, calota de esfera e tronco de esfera (ou algo analogo)? > Alguem pode me passar essas formulinhas? Sera q existe um jeito de calcular o volume > de um solido de revolucao dado a area do corte transversal e o eixo de giro? > > Obrigado, > Tonik > ========================================================================= Instruções para entrar na lista, sair da lista e usar a lista em http://www.mat.puc-rio.br/~nicolau/olimp/obm-l.html O administrador desta lista é ========================================================================= From owner-obm-l@sucuri.mat.puc-rio.br Sun Sep 1 02:34:25 2002 Return-Path: Received: (from majordom@localhost) by sucuri.mat.puc-rio.br (8.9.3/8.9.3) id CAA21652 for obm-l-MTTP; Sun, 1 Sep 2002 02:34:08 -0300 Received: from puma.unisys.com.br (ns2.unisys.com.br [200.220.64.7]) by sucuri.mat.puc-rio.br (8.9.3/8.9.3) with ESMTP id CAA21648 for ; Sun, 1 Sep 2002 02:34:06 -0300 Received: from jf (riohiper01p148.uninet.com.br [200.220.2.148]) by puma.unisys.com.br (8.12.3/8.12.3) with SMTP id g815YYjR002309 for ; Sun, 1 Sep 2002 02:34:34 -0300 (EST) X-Spam-Filter: check_local@puma.unisys.com.br by digitalanswers.org Message-ID: <00c001c25179$d43aa120$9402dcc8@jf> From: "Jose Francisco Guimaraes Costa" To: "obm-l" Subject: Fw: [obm-l] esclarecimento Date: Sun, 1 Sep 2002 02:38:40 -0300 MIME-Version: 1.0 Content-Type: text/plain; charset="Windows-1252" Content-Transfer-Encoding: 8bit X-Priority: 3 X-MSMail-Priority: Normal X-Mailer: Microsoft Outlook Express 6.00.2600.0000 X-MimeOLE: Produced By Microsoft MimeOLE V6.00.2600.0000 Sender: owner-obm-l@sucuri.mat.puc-rio.br Precedence: bulk Reply-To: obm-l@mat.puc-rio.br COMPLEMENTAÇÃO Para ver as fórmulas pedidas, vá até http://mathworld.wolfram.com/ e especifique: spherical sector (setor esférico), spherial cap (calota de esfera), spherical segment (tronco de esfera). O volume de um sólido de revolução está em http://mathworld.wolfram.com/SolidofRevolution.html Para uma solução mais genérica, use o Teorema da Centroide de Pappus, descrito em http://mathworld.wolfram.com/PappussCentroidTheorem.html JF ----- Original Message ----- From: "Tonik" To: Sent: Sunday, September 01, 2002 1:04 AM Subject: Re: [obm-l] esclarecimento > > Agora dexa eu perguntar, eu to com quatro duvidas, ou melhor, conceitos, > > pendurados, e preciso que alguem me ilumine: > > > > > 3) Será que um livro deixa de circulacao porque é bom? Cade as formulas > para se achar > > o volume de setor esferico, calota de esfera e tronco de esfera (ou algo > analogo)? > > Alguem pode me passar essas formulinhas? Sera q existe um jeito de > calcular o volume > > de um solido de revolucao dado a area do corte transversal e o eixo de > giro? > > > > Obrigado, > > Tonik > > > > ========================================================================= Instruções para entrar na lista, sair da lista e usar a lista em http://www.mat.puc-rio.br/~nicolau/olimp/obm-l.html O administrador desta lista é ========================================================================= From owner-obm-l@sucuri.mat.puc-rio.br Sun Sep 1 07:56:56 2002 Return-Path: Received: (from majordom@localhost) by sucuri.mat.puc-rio.br (8.9.3/8.9.3) id HAA26795 for obm-l-MTTP; Sun, 1 Sep 2002 07:55:58 -0300 Received: from hotmail.com (oe14.pav0.hotmail.com [64.4.32.94]) by sucuri.mat.puc-rio.br (8.9.3/8.9.3) with ESMTP id HAA26791 for ; Sun, 1 Sep 2002 07:55:55 -0300 Received: from mail pickup service by hotmail.com with Microsoft SMTPSVC; Sun, 1 Sep 2002 03:56:23 -0700 X-Originating-IP: [200.227.231.54] From: "e isso mesmo" To: Subject: [obm-l] Existe?? Date: Sun, 1 Sep 2002 07:56:17 -0300 MIME-Version: 1.0 X-Mailer: MSN Explorer 7.00.0021.1900 Content-Type: multipart/alternative; boundary="----=_NextPart_001_0000_01C2518D.0D6B2740" Message-ID: X-OriginalArrivalTime: 01 Sep 2002 10:56:23.0818 (UTC) FILETIME=[365B76A0:01C251A6] Sender: owner-obm-l@sucuri.mat.puc-rio.br Precedence: bulk Reply-To: obm-l@mat.puc-rio.br ------=_NextPart_001_0000_01C2518D.0D6B2740 Content-Type: text/plain; charset="iso-8859-1" Content-Transfer-Encoding: quoted-printable Companheiros, ajudem-me: -Mostre que existe um n=FAmero x entre 0 e Pi/2 tal que x=3Dcos x. ObrigadoAproveite melhor a Web. Fa=E7a o download GR=C1TIS do MSN Explore= r : http://explorer.msn.com.br/intl.asp#po ------=_NextPart_001_0000_01C2518D.0D6B2740 Content-Type: text/html; charset="iso-8859-1" Content-Transfer-Encoding: quoted-printable
 
<= DIV>Companheiros, ajudem-me:
 
-Mostre que exi= ste um n=FAmero x entre  0 e Pi/2 tal que x=3Dcos x.
&nbs= p;
Obrigado


Aproveite m= elhor a Web. Fa=E7a o download GR=C1TIS do MSN Explorer : http://explorer.msn.com.br/intl.asp#p= o

------=_NextPart_001_0000_01C2518D.0D6B2740-- ========================================================================= Instruções para entrar na lista, sair da lista e usar a lista em http://www.mat.puc-rio.br/~nicolau/olimp/obm-l.html O administrador desta lista é ========================================================================= From owner-obm-l@sucuri.mat.puc-rio.br Sun Sep 1 09:05:33 2002 Return-Path: Received: (from majordom@localhost) by sucuri.mat.puc-rio.br (8.9.3/8.9.3) id JAA27504 for obm-l-MTTP; Sun, 1 Sep 2002 09:03:52 -0300 Received: from traven10.uol.com.br (200-221-4-45.portais-uolinc.uol.com.br [200.221.4.45] (may be forged)) by sucuri.mat.puc-rio.br (8.9.3/8.9.3) with ESMTP id JAA27500 for ; Sun, 1 Sep 2002 09:03:50 -0300 Received: from ui.uol.com.br ([200.160.246.234]) by traven10.uol.com.br (8.9.1/8.9.1) with ESMTP id JAA17174; Sun, 1 Sep 2002 09:03:48 -0300 (BRT) Message-Id: <5.1.0.14.2.20020901090036.00b142d0@pop3.uol.com.br> X-Sender: cavictor@pop3.uol.com.br X-Mailer: QUALCOMM Windows Eudora Version 5.1 Date: Sun, 01 Sep 2002 09:04:12 -0300 To: obm-l@mat.puc-rio.br, From: Carlos Victor Subject: Re: [obm-l] Existe?? In-Reply-To: Mime-Version: 1.0 Content-Type: multipart/alternative; boundary="=====================_5863541==_.ALT" Sender: owner-obm-l@sucuri.mat.puc-rio.br Precedence: bulk Reply-To: obm-l@mat.puc-rio.br --=====================_5863541==_.ALT Content-Type: text/plain; charset="iso-8859-1"; format=flowed Content-Transfer-Encoding: quoted-printable Ol=E1 Rdrql, Observe tal fato tra=E7ando os dois gr=E1ficos , ok ? []=B4s Carlos Victor At 07:56 1/9/2002 -0300, e isso mesmo wrote: > >Companheiros, ajudem-me: > >-Mostre que existe um n=FAmero x entre 0 e Pi/2 tal que x=3Dcos x. > >Obrigado > > >---------- >Aproveite melhor a Web. Fa=E7a o download GR=C1TIS do MSN Explorer :=20 >http://explorer.msn.com.br/intl.asp#po --=====================_5863541==_.ALT Content-Type: text/html; charset="iso-8859-1" Content-Transfer-Encoding: quoted-printable Ol=E1  Rdrql,
Observe  tal fato tra=E7ando  os  dois  gr=E1ficos , ok ?

[]=B4s  Carlos  Victor




At 07:56 1/9/2002 -0300, e isso mesmo wrote:
 
Companheiros, ajudem-me:
 
-Mostre que existe um n=FAmero x entre  0 e Pi/2 tal que x=3Dcos=20 x.
 
Obrigado


Aproveite melhor a Web. Fa=E7a o download GR=C1TIS do MSN Explorer : http://explorer.msn.com= .br/intl.asp#po
--=====================_5863541==_.ALT-- ========================================================================= Instruções para entrar na lista, sair da lista e usar a lista em http://www.mat.puc-rio.br/~nicolau/olimp/obm-l.html O administrador desta lista é ========================================================================= From owner-obm-l@sucuri.mat.puc-rio.br Sun Sep 1 09:05:33 2002 Return-Path: Received: (from majordom@localhost) by sucuri.mat.puc-rio.br (8.9.3/8.9.3) id JAA27504 for obm-l-MTTP; Sun, 1 Sep 2002 09:03:52 -0300 Received: from traven10.uol.com.br (200-221-4-45.portais-uolinc.uol.com.br [200.221.4.45] (may be forged)) by sucuri.mat.puc-rio.br (8.9.3/8.9.3) with ESMTP id JAA27500 for ; Sun, 1 Sep 2002 09:03:50 -0300 Received: from ui.uol.com.br ([200.160.246.234]) by traven10.uol.com.br (8.9.1/8.9.1) with ESMTP id JAA17174; Sun, 1 Sep 2002 09:03:48 -0300 (BRT) Message-Id: <5.1.0.14.2.20020901090036.00b142d0@pop3.uol.com.br> X-Sender: cavictor@pop3.uol.com.br X-Mailer: QUALCOMM Windows Eudora Version 5.1 Date: Sun, 01 Sep 2002 09:04:12 -0300 To: obm-l@mat.puc-rio.br, From: Carlos Victor Subject: Re: [obm-l] Existe?? In-Reply-To: Mime-Version: 1.0 Content-Type: multipart/alternative; boundary="=====================_5863541==_.ALT" Sender: owner-obm-l@sucuri.mat.puc-rio.br Precedence: bulk Reply-To: obm-l@mat.puc-rio.br --=====================_5863541==_.ALT Content-Type: text/plain; charset="iso-8859-1"; format=flowed Content-Transfer-Encoding: quoted-printable Ol=E1 Rdrql, Observe tal fato tra=E7ando os dois gr=E1ficos , ok ? []=B4s Carlos Victor At 07:56 1/9/2002 -0300, e isso mesmo wrote: > >Companheiros, ajudem-me: > >-Mostre que existe um n=FAmero x entre 0 e Pi/2 tal que x=3Dcos x. > >Obrigado > > >---------- >Aproveite melhor a Web. Fa=E7a o download GR=C1TIS do MSN Explorer :=20 >http://explorer.msn.com.br/intl.asp#po --=====================_5863541==_.ALT Content-Type: text/html; charset="iso-8859-1" Content-Transfer-Encoding: quoted-printable Ol=E1  Rdrql,
Observe  tal fato tra=E7ando  os  dois  gr=E1ficos , ok ?

[]=B4s  Carlos  Victor




At 07:56 1/9/2002 -0300, e isso mesmo wrote:
 
Companheiros, ajudem-me:
 
-Mostre que existe um n=FAmero x entre  0 e Pi/2 tal que x=3Dcos=20 x.
 
Obrigado


Aproveite melhor a Web. Fa=E7a o download GR=C1TIS do MSN Explorer : http://explorer.msn.com= .br/intl.asp#po
--=====================_5863541==_.ALT-- ========================================================================= Instruções para entrar na lista, sair da lista e usar a lista em http://www.mat.puc-rio.br/~nicolau/olimp/obm-l.html O administrador desta lista é ========================================================================= From owner-obm-l@sucuri.mat.puc-rio.br Sun Sep 1 09:11:47 2002 Return-Path: Received: (from majordom@localhost) by sucuri.mat.puc-rio.br (8.9.3/8.9.3) id JAA27606 for obm-l-MTTP; Sun, 1 Sep 2002 09:11:46 -0300 Received: from gorgo.centroin.com.br (gorgo.centroin.com.br [200.225.63.128]) by sucuri.mat.puc-rio.br (8.9.3/8.9.3) with ESMTP id JAA27602 for ; Sun, 1 Sep 2002 09:11:44 -0300 Received: from centroin.com.br (du4c.rjo.centroin.com.br [200.225.58.4]) (authenticated bits=0) by gorgo.centroin.com.br (8.12.2/8.12.1) with ESMTP id g81CCbCR017563 for ; Sun, 1 Sep 2002 09:12:38 -0300 (BRT) Message-ID: <3D7204C7.60603@centroin.com.br> Date: Sun, 01 Sep 2002 09:15:03 -0300 From: Augusto =?ISO-8859-1?Q?C=E9sar?= Morgado User-Agent: Mozilla/5.0 (Windows; U; Win98; en-US; rv:0.9.4.1) Gecko/20020508 Netscape6/6.2.3 X-Accept-Language: en-us MIME-Version: 1.0 To: obm-l@mat.puc-rio.br Subject: Re: [obm-l] Existe?? References: Content-Type: multipart/alternative; boundary="------------060509020603020100080905" Sender: owner-obm-l@sucuri.mat.puc-rio.br Precedence: bulk Reply-To: obm-l@mat.puc-rio.br --------------060509020603020100080905 Content-Type: text/plain; charset=ISO-8859-1; format=flowed Content-Transfer-Encoding: 8bit F(x) = x-cosx eh continua em [0, pi/2]. F(0) = -1 e F(pi/2) = pi/2. Logo, pelo teorema de Bolzano... e isso mesmo wrote: > > > Companheiros, ajudem-me: > > > > -Mostre que existe um número x entre 0 e Pi/2 tal que x=cos x. > > > > Obrigado > > > ------------------------------------------------------------------------ > Aproveite melhor a Web. Faça o download GRÁTIS do MSN Explorer : > http://explorer.msn.com.br/intl.asp#po --------------060509020603020100080905 Content-Type: text/html; charset=us-ascii Content-Transfer-Encoding: 7bit F(x) = x-cosx eh continua em [0, pi/2].
F(0) = -1  e  F(pi/2) = pi/2.
Logo, pelo teorema de Bolzano...


e isso mesmo wrote:
 
Companheiros, ajudem-me:
 
-Mostre que existe um número x entre  0 e Pi/2 tal que x=cos x.
 
Obrigado


Aproveite melhor a Web. Faça o download GRÁTIS do MSN Explorer : http://explorer.msn.com.br/intl.asp#po

--------------060509020603020100080905-- ========================================================================= Instruções para entrar na lista, sair da lista e usar a lista em http://www.mat.puc-rio.br/~nicolau/olimp/obm-l.html O administrador desta lista é ========================================================================= From owner-obm-l@sucuri.mat.puc-rio.br Sun Sep 1 09:23:38 2002 Return-Path: Received: (from majordom@localhost) by sucuri.mat.puc-rio.br (8.9.3/8.9.3) id JAA28125 for obm-l-MTTP; Sun, 1 Sep 2002 09:23:30 -0300 Received: from gorgo.centroin.com.br (gorgo.centroin.com.br [200.225.63.128]) by sucuri.mat.puc-rio.br (8.9.3/8.9.3) with ESMTP id JAA28121 for ; Sun, 1 Sep 2002 09:23:28 -0300 Received: from centroin.com.br (du4c.rjo.centroin.com.br [200.225.58.4]) (authenticated bits=0) by gorgo.centroin.com.br (8.12.2/8.12.1) with ESMTP id g81COFCR018074 for ; Sun, 1 Sep 2002 09:24:16 -0300 (BRT) Message-ID: <3D720783.8080406@centroin.com.br> Date: Sun, 01 Sep 2002 09:26:43 -0300 From: Augusto =?windows-1252?Q?C=E9sar?= Morgado User-Agent: Mozilla/5.0 (Windows; U; Win98; en-US; rv:0.9.4.1) Gecko/20020508 Netscape6/6.2.3 X-Accept-Language: en-us MIME-Version: 1.0 To: obm-l@mat.puc-rio.br Subject: Re: [obm-l] esclarecimento References: Content-Type: multipart/alternative; boundary="------------050103080509080409040908" Sender: owner-obm-l@sucuri.mat.puc-rio.br Precedence: bulk Reply-To: obm-l@mat.puc-rio.br --------------050103080509080409040908 Content-Type: text/plain; charset=windows-1252; format=flowed Content-Transfer-Encoding: 8bit 4) Chame arc sen 1/3 de x. Entao, senx = 1/3 e x eh do primeiro quadrante. Voce quer calcular cosx. Ora, (senx)^2 + (cosx)^2=1, (cosx)^2 = 8/9 e cos x = 2 (raiz de 2)/3 2) Resolva a equação y = (2+x)/(2-x) ao contrário. Dah x = (2y-2) / (y+1) , y diferente de -1. A imagem é o conjunto dos reais diferentes de -1. PS: Mensagens como a sua (biblioteca...) deveriam, na minha opiniao, vir acompanhadas da cidade do remetente. Morgado Tonik wrote: >01/09/02 00:17:42, Augusto César Morgado wrote: > >> Ao jogar três moedas, qual a probabilidade de dar duas caras e uma coroa? >> > >Isso eh um acontecimento binario, ou seja, acontecimentos individuais com >probabilidades individuais iguais, em que vc deseja saber a probabilidade >de acontecer determinado numero de vezes uma das partes do acontecimento. > >Existe uma formulinha para isso, em q eu andei mudando o nome das variaveis >para facilitar a memorizacao: > >Pbin = C(t,a) * Pa^a * Pe^e > >em que >C(n,p) = combinacao, binomio de newton ou ainda n!/(p!*(n-p)!) >t = tentativas, a = acertos, e = erros >Pa = probabilidade de acerto em cada tentativa >Pe = probabilidade de erro em cada tentativa > >No caso das 3 moedas, vamos chamar cara de acerto e coroa de erro, >temos t=3, a=2, e=1, e as probabilidades de erro e acerto sao iguais a 1/2. > >Pbin = C(3,2) * (1/2)^2 * (1/2)^1 > >resolvendo a conta temos > >Pbin = 3!/2! * 1/4 * 1/2 >Pbin = 3 * 1/8 >Pbin = 3/8 > >------- > >Agora dexa eu perguntar, eu to com quatro duvidas, ou melhor, conceitos, >pendurados, e preciso que alguem me ilumine: > >1) "Pelamordedeus", alguem escreva a teoria das equacoes reciprocas, primeira e >segunda especie, sua teoria e principalmente suas propriedades. Nao deve ser longo, >se eu tivesse uma boa biblioteca perto daqui eu ja teria ido e nao teria perguntado >para 3 professores diferentes q por 2 semanas estao enrolando q estao meio sem tempo. >Entao por favor, digam-me o que esta escrito, nao aonde esta escrito. Ja tentei >procurar na internet e se acha apenas um unico site (fora os seus mirrors) e eu sei >q ele esta incompleto. > >2) Funcao f: R -> R tal que f(x) = (2+x)/(2-x). Esta obvio como calcular o conjunto >Dominio da funcao (que é R-{2}) da mesma forma como deve estar obvio calcular o seu >conjunto imagem, só que eu nao sei como se nao montando o grafico... > >3) Será que um livro deixa de circulacao porque é bom? Cade as formulas para se achar >o volume de setor esferico, calota de esfera e tronco de esfera (ou algo analogo)? >Alguem pode me passar essas formulinhas? Sera q existe um jeito de calcular o volume >de um solido de revolucao dado a area do corte transversal e o eixo de giro? > >4) Sem calculadora, como calculo cos(arcsen(1/3)) ? Ou eh impossivel/inviavel? > >Obrigado, >Tonik > > > > > > > > > > >========================================================================= >Instruções para entrar na lista, sair da lista e usar a lista em >http://www.mat.puc-rio.br/~nicolau/olimp/obm-l.html >O administrador desta lista é >========================================================================= > > --------------050103080509080409040908 Content-Type: text/html; charset=windows-1252 Content-Transfer-Encoding: 8bit 4) Chame arc sen 1/3 de x. Entao, senx = 1/3 e x eh do primeiro quadrante. Voce quer calcular cosx. Ora, (senx)^2 + (cosx)^2=1,  (cosx)^2 = 8/9  e cos x = 2 (raiz de 2)/3

2) Resolva a equação   y  = (2+x)/(2-x) ao contrário.
Dah   x = (2y-2) / (y+1) , y diferente de -1.
A imagem é o conjunto dos reais diferentes de  -1.
PS: Mensagens como a sua (biblioteca...) deveriam, na minha opiniao, vir acompanhadas da cidade do remetente.
Morgado
 

Tonik wrote:
01/09/02 00:17:42, Augusto César Morgado <morgado@centroin.com.br> wrote:
   Ao jogar três moedas, qual a probabilidade de dar duas caras e uma coroa?

Isso eh um acontecimento binario, ou seja, acontecimentos individuais com
probabilidades individuais iguais, em que vc deseja saber a probabilidade
de acontecer determinado numero de vezes uma das partes do acontecimento.

Existe uma formulinha para isso, em q eu andei mudando o nome das variaveis
para facilitar a memorizacao:

Pbin = C(t,a) * Pa^a * Pe^e

em que
C(n,p) = combinacao, binomio de newton ou ainda n!/(p!*(n-p)!)
t = tentativas, a = acertos, e = erros
Pa = probabilidade de acerto em cada tentativa
Pe = probabilidade de erro em cada tentativa

No caso das 3 moedas, vamos chamar cara de acerto e coroa de erro,
temos t=3, a=2, e=1, e as probabilidades de erro e acerto sao iguais a 1/2.

Pbin = C(3,2) * (1/2)^2 * (1/2)^1

resolvendo a conta temos

Pbin = 3!/2! * 1/4 * 1/2
Pbin = 3 * 1/8
Pbin = 3/8

-------

Agora dexa eu perguntar, eu to com quatro duvid as, ou melhor, conceitos,
pendurados, e preciso que alguem me ilumine:

1) "Pelamordedeus", alguem escreva a teoria das equacoes reciprocas, primeira e
segunda especie, sua teoria e principalmente suas propriedades. Nao deve ser longo,
se eu tivesse uma boa biblioteca perto daqui eu ja teria ido e nao teria perguntado
para 3 professores diferentes q por 2 semanas estao enrolando q estao meio sem tempo.
Entao por favor, digam-me o que esta escrito, nao aonde esta escrito. Ja tentei
procurar na internet e se acha apenas um unico site (fora os seus mirrors) e eu sei
q ele esta incompleto.

2) Funcao f: R -> R tal que f(x) = (2+x)/(2-x). Esta obvio como calcular o conjunto
Dominio da funcao (que é R-{2}) da mesma forma como deve estar obvio calcular o seu
conjunto imagem, só que eu nao sei como se nao montando o grafico...

3) Será que um livro deixa de circulacao porque é bom? Cade as formulas para se achar
o volume de setor esferico, calota de esfera e tronco de esfera (ou algo analogo)?
Alguem pode me passar essas formulinhas? Sera q existe um jeito de calcular o volume
de um solido de revolucao dado a area do corte transversal e o eixo de giro?

4) Sem calculadora, como calculo cos(arcsen(1/3)) ? Ou eh impossivel/inviavel?

Obrigado,
Tonik










=========================================================================
Instruções para entrar na lista, sair da lista e usar a lista em
http://www.mat.puc-rio.br/~nicolau/olimp/obm-l.html
O administrador desta lista é <nicolau@mat.puc-rio.br>
=========================================================================



--------------050103080509080409040908-- ========================================================================= Instruções para entrar na lista, sair da lista e usar a lista em http://www.mat.puc-rio.br/~nicolau/olimp/obm-l.html O administrador desta lista é ========================================================================= From owner-obm-l@sucuri.mat.puc-rio.br Sun Sep 1 11:04:05 2002 Return-Path: Received: (from majordom@localhost) by sucuri.mat.puc-rio.br (8.9.3/8.9.3) id LAA29723 for obm-l-MTTP; Sun, 1 Sep 2002 11:03:31 -0300 Received: from sidney1.bol.com.br (sidney1.bol.com.br [200.221.24.206]) by sucuri.mat.puc-rio.br (8.9.3/8.9.3) with ESMTP id LAA29719; Sun, 1 Sep 2002 11:03:29 -0300 Received: from bol.com.br (200.221.24.134) by sidney1.bol.com.br (5.1.071) id 3D5D7EAA003048E4; Sun, 1 Sep 2002 11:03:26 -0300 Date: Sun, 1 Sep 2002 11:02:31 -0300 Message-Id: Subject: Re: [obm-l] fisica MIME-Version: 1.0 Content-Type: multipart/mixed; boundary="_=__=_XaM3_Boundary.1030888951.2A.312400.42.11002.52.42.101010.613961217" From: "diegoalonsoteixeira" To: obm-l@mat.puc-rio.br Cc: obm-l@mat.puc-rio.br X-XaM3-API-Version: 2.4.3.4.4 X-SenderIP: 200.206.236.239 Sender: owner-obm-l@sucuri.mat.puc-rio.br Precedence: bulk Reply-To: obm-l@mat.puc-rio.br --_=__=_XaM3_Boundary.1030888951.2A.312400.42.11002.52.42.101010.613961217 Content-Type: text/plain;charset="iso-8859-1" Content-Transfer-Encoding: quoted-printable ogrigado pela indica=E7=E3o, mas meu ingles n=E3o =E9 muito bom,vc conhece algum em portugues? __________________________________________________________________________ AcessoBOL, s=F3 R$ 9,90! O menor pre=E7o do mercado! Assine j=E1! http://www.bol.com.br/acessobol --_=__=_XaM3_Boundary.1030888951.2A.312400.42.11002.52.42.101010.613961217 Content-Type: text/plain; name="000008C6" Content-Transfer-Encoding: base64 VuEgYXTpIGh0dHA6Ly9zY2llbmNld29ybGQud29sZnJhbS5jb20vcGh5c2ljcy8gZSBlc3Bl Y2lmaXF1ZSBvIHF1ZSBxdWVyDQpzYWJlcjogYW5ndWxhciBtb21lbnR1bSAobW9tZW50byBh bmd1bGFyKSwgbW9tZW50IG9mIGluZXJ0aWEgKG1vbWVudG8gZGUNCmlu6XJjaWEpLCB0b3Jx dWUgKHRvcnF1ZSkuLi4NCg0KSkYNCg0KLS0tLS0gT3JpZ2luYWwgTWVzc2FnZSAtLS0tLQ0K RnJvbTogImRpZWdvYWxvbnNvdGVpeGVpcmEiIDxkaWVnb2Fsb25zb3RlaXhlaXJhQGJvbC5j b20uYnI+DQpUbzogPG9ibS1sQG1hdC5wdWMtcmlvLmJyPg0KQ2M6IDxvYm0tbEBtYXQucHVj LXJpby5icj4NClNlbnQ6IFNhdHVyZGF5LCBBdWd1c3QgMzEsIDIwMDIgODoxMiBBTQ0KU3Vi amVjdDogW29ibS1sXSBmaXNpY2ENCg0KDQpBbGd1ZW0gcG9kZXJpYSBtZSBpbmRpY2FyIGFs Z3VtIHNpdGUgbmEgaW50ZXJuZXQgcXVlDQpleHBsaXF1ZSBkZXRhbGhhZGFtZW50ZSBtb21l bnRvIGFuZ3VsYXIsbW9tZW50byBkZQ0KaW7pcmNpYSAsdG9ycXVlLi4NCm9icmlnYWRvDQoN Cg0KDQo9PT09PT09PT09PT09PT09PT09PT09PT09PT09PT09PT09PT09PT09PT09PT09PT09 PT09PT09PT09PT09PT09PT09PT09PT09DQpJbnN0cnXn9WVzIHBhcmEgZW50cmFyIG5hIGxp c3RhLCBzYWlyIGRhIGxpc3RhIGUgdXNhciBhIGxpc3RhIGVtDQpodHRwOi8vd3d3Lm1hdC5w dWMtcmlvLmJyL35uaWNvbGF1L29saW1wL29ibS1sLmh0bWwNCk8gYWRtaW5pc3RyYWRvciBk ZXN0YSBsaXN0YSDpIDxuaWNvbGF1QG1hdC5wdWMtcmlvLmJyPg0KPT09PT09PT09PT09PT09 PT09PT09PT09PT09PT09PT09PT09PT09PT09PT09PT09PT09PT09PT09PT09PT09PT09PT09 PT09PQ0K --_=__=_XaM3_Boundary.1030888951.2A.312400.42.11002.52.42.101010.613961217-- ========================================================================= Instruções para entrar na lista, sair da lista e usar a lista em http://www.mat.puc-rio.br/~nicolau/olimp/obm-l.html O administrador desta lista é ========================================================================= From owner-obm-l@sucuri.mat.puc-rio.br Sun Sep 1 11:04:14 2002 Return-Path: Received: (from majordom@localhost) by sucuri.mat.puc-rio.br (8.9.3/8.9.3) id LAA29750 for obm-l-MTTP; Sun, 1 Sep 2002 11:04:11 -0300 Received: from hotmail.com (oe23.law10.hotmail.com [64.4.14.80]) by sucuri.mat.puc-rio.br (8.9.3/8.9.3) with ESMTP id LAA29746 for ; Sun, 1 Sep 2002 11:04:09 -0300 Received: from mail pickup service by hotmail.com with Microsoft SMTPSVC; Sun, 1 Sep 2002 07:04:37 -0700 X-Originating-IP: [200.191.100.8] From: "Rubens Vilhena" To: Subject: Re: [obm-l] Existe?? Date: Sun, 1 Sep 2002 11:04:27 -0300 MIME-Version: 1.0 X-Mailer: MSN Explorer 7.00.0021.1900 Content-Type: multipart/alternative; boundary="----=_NextPart_001_0000_01C251A7.56A271B0" Message-ID: X-OriginalArrivalTime: 01 Sep 2002 14:04:37.0223 (UTC) FILETIME=[81C04F70:01C251C0] Sender: owner-obm-l@sucuri.mat.puc-rio.br Precedence: bulk Reply-To: obm-l@mat.puc-rio.br ------=_NextPart_001_0000_01C251A7.56A271B0 Content-Type: text/plain; charset="iso-8859-1" Content-Transfer-Encoding: quoted-printable Prezado colega: Seja f(x)=3Dx -cos x. Ent=E3o f(0)=3D -1 < 0 < Pi/2 =3D f(Pi/2). Como f =E9= cont=EDnua, f(k)=3D0 para algum n=FAmero k entre 0 e Pi/2. Isto =E9, k- = cos k =3D 0, ent=E3o k=3Dcos k. Espero que tenha ajudado. -----Mensagem Original----- De: e isso mesmo Enviado: domingo, 1 de setembro de 2002 08:26 Para: obm-l@mat.puc-rio.br Assunto: [obm-l] Existe?? Companheiros, ajudem-me: -Mostre que existe um n=FAmero x entre 0 e Pi/2 tal que x=3Dcos x. Obrigado Aproveite melhor a Web. Fa=E7a o download GR=C1TIS do MSN Explorer : http= ://explorer.msn.com.br/intl.asp#poAproveite melhor a Web. Fa=E7a o downlo= ad GR=C1TIS do MSN Explorer : http://explorer.msn.com.br/intl.asp#po ------=_NextPart_001_0000_01C251A7.56A271B0 Content-Type: text/html; charset="iso-8859-1" Content-Transfer-Encoding: quoted-printable
Prezado colega= :
 
Seja f(x)=3Dx -cos x. Ent=E3o f(0)=3D -1 &= lt; 0 < Pi/2 =3D f(Pi/2). Como f =E9 cont=EDnua, f(k)=3D0 para algum n= =FAmero k entre 0 e Pi/2. Isto =E9, k- cos k =3D 0, ent=E3o k=3Dcos = k.
 
Espero que tenha ajudado.
&nbs= p;
-----Mensagem Original-----
De: e isso me= smo
Enviado: domingo, 1 de s= etembro de 2002 08:26
Para: = obm-l@mat.puc-rio.br
Assunto: [obm-l] Existe??
 
 
Companh= eiros, ajudem-me:
 
-Mostre que existe um n=FA= mero x entre  0 e Pi/2 tal que x=3Dcos x.
 
Obrigado

Aproveite melhor a Web. Fa=E7a o d= ownload GR=C1TIS do MSN Explorer : http://explorer.msn.com.br/intl.asp#po



Aproveite melhor a Web. Fa=E7a o= download GR=C1TIS do MSN Explorer : http://explorer.msn.com.br/intl.asp#po

------=_NextPart_001_0000_01C251A7.56A271B0-- ========================================================================= Instruções para entrar na lista, sair da lista e usar a lista em http://www.mat.puc-rio.br/~nicolau/olimp/obm-l.html O administrador desta lista é ========================================================================= From owner-obm-l@sucuri.mat.puc-rio.br Sun Sep 1 11:15:45 2002 Return-Path: Received: (from majordom@localhost) by sucuri.mat.puc-rio.br (8.9.3/8.9.3) id LAA30110 for obm-l-MTTP; Sun, 1 Sep 2002 11:15:29 -0300 Received: from imo-r09.mx.aol.com (imo-r09.mx.aol.com [152.163.225.105]) by sucuri.mat.puc-rio.br (8.9.3/8.9.3) with ESMTP id LAA30106 for ; Sun, 1 Sep 2002 11:15:27 -0300 From: Lltmdrtm@aol.com Received: from Lltmdrtm@aol.com by imo-r09.mx.aol.com (mail_out_v34.10.) id z.14b.13597f4e (4539) for ; Sun, 1 Sep 2002 10:15:51 -0400 (EDT) Message-ID: <14b.13597f4e.2aa37b16@aol.com> Date: Sun, 1 Sep 2002 10:15:50 EDT Subject: [obm-l] Ajuda To: obm-l@mat.puc-rio.br MIME-Version: 1.0 Content-Type: multipart/alternative; boundary="part1_14b.13597f4e.2aa37b16_boundary" X-Mailer: AOL 7.0 for Windows BR sub 10501 Sender: owner-obm-l@sucuri.mat.puc-rio.br Precedence: bulk Reply-To: obm-l@mat.puc-rio.br --part1_14b.13597f4e.2aa37b16_boundary Content-Type: text/plain; charset="ISO-8859-1" Content-Transfer-Encoding: quoted-printable Disp=F5e-se de 6 cores distintas, 3 das quais ser=E3o escolhidas para pintar= as=20 faces de um cubo. De quantos modos a pintura poder=E1 ser feita se faces=20 opostas devem ter a mesma cor? --part1_14b.13597f4e.2aa37b16_boundary Content-Type: text/html; charset="ISO-8859-1" Content-Transfer-Encoding: quoted-printable Disp=F5e-se de 6 cores distintas, 3 das quais ser=E3o=20= escolhidas para pintar as faces de um cubo. De quantos modos a pintura poder= =E1 ser feita se faces opostas devem ter a mesma cor? --part1_14b.13597f4e.2aa37b16_boundary-- ========================================================================= Instruções para entrar na lista, sair da lista e usar a lista em http://www.mat.puc-rio.br/~nicolau/olimp/obm-l.html O administrador desta lista é ========================================================================= From owner-obm-l@sucuri.mat.puc-rio.br Sun Sep 1 11:55:32 2002 Return-Path: Received: (from majordom@localhost) by sucuri.mat.puc-rio.br (8.9.3/8.9.3) id LAA31414 for obm-l-MTTP; Sun, 1 Sep 2002 11:53:35 -0300 Received: from cairu.terra.com.br (cairu.terra.com.br [200.176.3.19]) by sucuri.mat.puc-rio.br (8.9.3/8.9.3) with ESMTP id LAA31410 for ; Sun, 1 Sep 2002 11:53:33 -0300 Received: from pavuna.terra.com.br (pavuna.terra.com.br [200.176.3.41]) by cairu.terra.com.br (Postfix) with ESMTP id B612C4711A for ; Sun, 1 Sep 2002 11:54:03 -0300 (EST) Received: from ig (nrce02-1054.dial.rce.embratel.net.br [200.249.120.54]) (authenticated user renato_l) by pavuna.terra.com.br (Postfix) with ESMTP id 1341468291 for ; Sun, 1 Sep 2002 11:53:51 -0300 (EST) Message-ID: <008401c251c7$b6b5ba60$3678f9c8@ig> From: "Renato Lira" To: References: Subject: Re: [obm-l] fisica Date: Sun, 1 Sep 2002 11:55:49 -0300 MIME-Version: 1.0 Content-Type: text/plain; charset="iso-8859-1" Content-Transfer-Encoding: 8bit X-Priority: 3 X-MSMail-Priority: Normal X-Mailer: Microsoft Outlook Express 5.50.4807.1700 X-MimeOLE: Produced By Microsoft MimeOLE V5.50.4807.1700 Sender: owner-obm-l@sucuri.mat.puc-rio.br Precedence: bulk Reply-To: obm-l@mat.puc-rio.br www.fisica.net é muito completo.. coloca na busca q vc acha. ----- Original Message ----- From: "diegoalonsoteixeira" To: Cc: Sent: Sunday, September 01, 2002 11:02 AM Subject: Re: [obm-l] fisica ogrigado pela indicação, mas meu ingles não é muito bom,vc conhece algum em portugues? __________________________________________________________________________ AcessoBOL, só R$ 9,90! O menor preço do mercado! Assine já! http://www.bol.com.br/acessobol ---------------------------------------------------------------------------- ---- > Vá até http://scienceworld.wolfram.com/physics/ e especifique o que quer > saber: angular momentum (momento angular), moment of inertia (momento de > inércia), torque (torque)... > > JF > > ----- Original Message ----- > From: "diegoalonsoteixeira" > To: > Cc: > Sent: Saturday, August 31, 2002 8:12 AM > Subject: [obm-l] fisica > > > Alguem poderia me indicar algum site na internet que > explique detalhadamente momento angular,momento de > inércia ,torque.. > obrigado > > > > ========================================================================= > Instruções para entrar na lista, sair da lista e usar a lista em > http://www.mat.puc-rio.br/~nicolau/olimp/obm-l.html > O administrador desta lista é > ========================================================================= > ========================================================================= Instruções para entrar na lista, sair da lista e usar a lista em http://www.mat.puc-rio.br/~nicolau/olimp/obm-l.html O administrador desta lista é ========================================================================= From owner-obm-l@sucuri.mat.puc-rio.br Sun Sep 1 12:45:45 2002 Return-Path: Received: (from majordom@localhost) by sucuri.mat.puc-rio.br (8.9.3/8.9.3) id MAA32298 for obm-l-MTTP; Sun, 1 Sep 2002 12:44:53 -0300 Received: from smtp.ieg.com.br (sharon.protocoloweb.com.br [200.226.139.12]) by sucuri.mat.puc-rio.br (8.9.3/8.9.3) with ESMTP id MAA32294 for ; Sun, 1 Sep 2002 12:44:51 -0300 Received: from igor (IDENT:hahaha@[200.222.144.154]) by smtp.ieg.com.br (IeG relay/8.9.3) with SMTP id g81FkiVq054083 for ; Sun, 1 Sep 2002 12:47:00 -0300 (BRT) Message-ID: <000801c251ce$5ba75140$9a90dec8@igor> From: "Igor Castro" To: "obm-lista" Subject: [obm-l] Estadual-RJ Date: Sun, 1 Sep 2002 12:43:44 -0300 MIME-Version: 1.0 Content-Type: multipart/alternative; boundary="----=_NextPart_000_0005_01C251B5.352BF980" X-Priority: 3 X-MSMail-Priority: Normal X-Mailer: Microsoft Outlook Express 5.00.2615.200 X-MimeOLE: Produced By Microsoft MimeOLE V5.00.2615.200 Sender: owner-obm-l@sucuri.mat.puc-rio.br Precedence: bulk Reply-To: obm-l@mat.puc-rio.br This is a multi-part message in MIME format. ------=_NextPart_000_0005_01C251B5.352BF980 Content-Type: text/plain; charset="iso-8859-1" Content-Transfer-Encoding: quoted-printable Algu=E9m tem a resposta do primeiro problema do nivel 3 da segunda fase = da Estadual-RJ que teve na puc ontem??????????? ------=_NextPart_000_0005_01C251B5.352BF980 Content-Type: text/html; charset="iso-8859-1" Content-Transfer-Encoding: quoted-printable Algu=E9m tem a = resposta do primeiro=20 problema do nivel 3 da segunda fase da Estadual-RJ que teve na puc=20 ontem??????????? ------=_NextPart_000_0005_01C251B5.352BF980-- ========================================================================= Instruções para entrar na lista, sair da lista e usar a lista em http://www.mat.puc-rio.br/~nicolau/olimp/obm-l.html O administrador desta lista é ========================================================================= From owner-obm-l@sucuri.mat.puc-rio.br Sun Sep 1 12:59:11 2002 Return-Path: Received: (from majordom@localhost) by sucuri.mat.puc-rio.br (8.9.3/8.9.3) id MAA32606 for obm-l-MTTP; Sun, 1 Sep 2002 12:58:56 -0300 Received: from mail.gmx.net (mail.gmx.net [213.165.64.20]) by sucuri.mat.puc-rio.br (8.9.3/8.9.3) with SMTP id MAA32591 for ; Sun, 1 Sep 2002 12:58:51 -0300 Received: (qmail 1049 invoked by uid 0); 1 Sep 2002 15:59:19 -0000 Received: from ad190-1-2.cta.gvt.net.br (HELO ?200.175.4.190?) (200.175.4.190) by mail.gmx.net (mp005-rz3) with SMTP; 1 Sep 2002 15:59:19 -0000 From: "Fernando Henrique Ferraz P. da Rosa" Subject: Re:[obm-l] Re: [obm-l] Re: [obm-l] Achar raizes "na mão" To: obm-l@mat.puc-rio.br Date: Sun, 1 Sep 2002 12:57:23 -0300 Lines: 49 Message-ID: Mime-Version: 1.0 Content-Transfer-Encoding: 7bit X-Mailer: Kaufman Mail Warrior 3,61 Final Sender: owner-obm-l@sucuri.mat.puc-rio.br Precedence: bulk Reply-To: obm-l@mat.puc-rio.br Você pode utilizar o algoritmo de Newton para aproximação de raízes de funções.. Seja a funcao f(x) = x^2 - a. A raiz positiva dessa funcao é a raiz quadrada que você está procurando e a é o numero de qual voce quer tirar a raiz quadrada (f(x) = x^2 - 8 para calcular a raiz quadrada de 8 por exemplo). Primeiro voce escolhe uma aproximacao x0 arbitrária (pode ser a/2, a, etc. de preferencia um valor positivo). Dai com algumas iteracoes de acordo com a funcao abaixo voce acha x1, x2, x3, x_i.... quanto maior i melhor a aproximacao. Em particular para a funcao f(x) = x^2 - a, quanto maior i melhor sua aproximacao. Usando o metodo de newton para a funcao f(x) acima: g(x_k+1) = x_k - f(x_k)/f'(x_k) g(x_k+1) = x_k - (x_k^2 - a)/(2*x_k) g(x_k+1) = (2x_k^2 - x_k^2 + a)2x_k g(x_k+1) = (x_k^2 +a)2x_k g(x_k+1) = 1/2 ((x_k^2 +a)/x_k) g(x_k+1) = 1/2 (x_k + a/x_k) Voce pode extender o metodo para a raiz enésima, fazendo f(x) = x^n - a, a demonstracao é análoga. Um exemplo númerico... vamos calcular a raiz quadrada de 1774 Isso equivale a achar a raiz da funcao f(x) = x^2 - 1774... Escolhemos x0 = 20. (poderia ser 30, 40, 0, voce só precisaria de mais ou menos iteracoes). x1 = 1/2 (x0 + 1774/x0) => x1 = 1/2 (20 + 1774/20) => x1 = 54.35 x2 = 1/2 (x1 + 1774/x1) => x2 = 1/2(54.35 + 1774/54.35) => x2 = 43.49... x3 = (idem acima) => x3 = 42,14... x4 = (ibidem) => x4 = 42,12 (arredondando) Em geral em 4 iteracoes voce já consegue uma boa aproximacao. 42.12^2 = 1774.0944.... Conforme voce quiser uma aproximacao melhor voce pode usar mais casas decimais entre as iteracoes ou aumentar o numero de iteracoes. "Jeremias de Paula Eduardo" wrote on 31/08/02 16:07:11: > >Laurito, >Obrigado por ter respondido minha primeira pergunta, e aí vai minha primeira >resposta. > >Não sei bem. Eu tava estudando algo sobre números irracionais e percebi que >não podia calcular raízes, lembrei de um tio meu que ficava criticando o >ensino de hoje, dezendo que os alunos não sabem mais fazer as coisas. Eu >retruquei e ele perguntou se eu sabia efetuar calculo de raízes e eu disse >que não. > >Sei lá, é algo que eu quero saber, quem sabe não consigo pegar alguma >demonstração de cálculo e aprimorá-lo ao meu gosto e até fazer de cabeça. >Seria interessante calcular raízes de cabeça... É isso, nada demais. "Wir müssen wissen. Wir würden wissen." David Hilbert ========================================================================= Instruções para entrar na lista, sair da lista e usar a lista em http://www.mat.puc-rio.br/~nicolau/olimp/obm-l.html O administrador desta lista é ========================================================================= From owner-obm-l@sucuri.mat.puc-rio.br Sun Sep 1 17:07:33 2002 Return-Path: Received: (from majordom@localhost) by sucuri.mat.puc-rio.br (8.9.3/8.9.3) id RAA02251 for obm-l-MTTP; Sun, 1 Sep 2002 17:06:25 -0300 Received: from web40308.mail.yahoo.com (web40308.mail.yahoo.com [66.218.78.87]) by sucuri.mat.puc-rio.br (8.9.3/8.9.3) with SMTP id RAA02247 for ; Sun, 1 Sep 2002 17:06:22 -0300 Message-ID: <20020901200651.76020.qmail@web40308.mail.yahoo.com> Received: from [200.248.101.128] by web40308.mail.yahoo.com via HTTP; Sun, 01 Sep 2002 17:06:51 ART Date: Sun, 1 Sep 2002 17:06:51 -0300 (ART) From: =?iso-8859-1?q?Sharon=20Guedes?= Subject: [obm-l] Ajuda em fatorial. To: obm-l@mat.puc-rio.br MIME-Version: 1.0 Content-Type: multipart/alternative; boundary="0-1979419228-1030910811=:75039" Content-Transfer-Encoding: 8bit Sender: owner-obm-l@sucuri.mat.puc-rio.br Precedence: bulk Reply-To: obm-l@mat.puc-rio.br --0-1979419228-1030910811=:75039 Content-Type: text/plain; charset=iso-8859-1 Content-Transfer-Encoding: 8bit Olá pessoal ! Alguém poderia me ajudar nestas questões? 1) (UnB)Sendo m . (m + 3) . (m - 1)! = 6 e m > 0, o valor de m é: (m - 2) . (m + 1)! 35 a) 9 b) 12 c) 15 d) 18 2)(URCAMP)Resolva as equações: a) [(m + 2)! - (m + 1)!] m! = 24 ² b) (m + 3)! + (m + 2)! = 6 (m + 3)! - (m + 2)! c) (n + 2)! + (n + 1)! = 21 1 . 2 . 3 ... (n - 3) (n - 2) (n - 1)! 3)(URCAMP)Dê o domínio de cada uma destas expresões: a) n! b) (n - 5)! c) (n + 1)! (n + 1)! (URCAMP)Escreva os produtos empregando a notação fatorial. a)(n + 2) . (n + 1) . n . (n - 1) . ... . 2 .1 b)(n -7). (n - 8). (n - 9). ... . 2 .1 At. Sharon. --------------------------------- Yahoo! PageBuilder - O super editor para criação de sites: é grátis, fácil e rápido. --0-1979419228-1030910811=:75039 Content-Type: text/html; charset=iso-8859-1 Content-Transfer-Encoding: 8bit

Olá pessoal !

Alguém poderia me ajudar nestas questões?

1) (UnB)Sendo m . (m + 3) . (m - 1)! = 6 e m > 0, o valor de m é:

                          (m - 2) . (m + 1)!    35

a) 9

b) 12

c) 15

d) 18

2)(URCAMP)Resolva as equações:

a) [(m + 2)! - (m + 1)!] m! = 24 ²

b) (m + 3)! + (m + 2)! = 6

(m + 3)! - (m + 2)!

c) (n + 2)! + (n + 1)! = 21

1 . 2 . 3 ... (n - 3) (n - 2) (n - 1)!

3)(URCAMP)Dê o domínio de cada uma destas expresões:

a) n!

b) (n - 5)!

c) (n + 1)! (n + 1)!

(URCAMP)Escreva os produtos empregando a notação fatorial.

a)(n + 2) . (n + 1) . n . (n - 1) . ... . 2 .1

b)(n -7). (n - 8). (n - 9). ... . 2 .1

At. Sharon.

 


Yahoo! PageBuilder - O super editor para criação de sites: é grátis, fácil e rápido. --0-1979419228-1030910811=:75039-- ========================================================================= Instruções para entrar na lista, sair da lista e usar a lista em http://www.mat.puc-rio.br/~nicolau/olimp/obm-l.html O administrador desta lista é ========================================================================= From owner-obm-l@sucuri.mat.puc-rio.br Sun Sep 1 17:30:48 2002 Return-Path: Received: (from majordom@localhost) by sucuri.mat.puc-rio.br (8.9.3/8.9.3) id RAA02712 for obm-l-MTTP; Sun, 1 Sep 2002 17:29:12 -0300 Received: from traven.uol.com.br (200-221-4-39.portais-uolinc.uol.com.br [200.221.4.39] (may be forged)) by sucuri.mat.puc-rio.br (8.9.3/8.9.3) with ESMTP id RAA02708 for ; Sun, 1 Sep 2002 17:29:10 -0300 Received: from emmanuela ([200.150.128.234]) by traven.uol.com.br (8.9.1/8.9.1) with SMTP id RAA13370 for ; Sun, 1 Sep 2002 17:17:05 -0300 (BRT) Message-ID: <002401c251f6$2171b560$ea8096c8@emmanuela> From: "Pauloemanu" To: Subject: [obm-l] OCM 2002 Date: Sun, 1 Sep 2002 17:28:23 -0300 MIME-Version: 1.0 Content-Type: multipart/alternative; boundary="----=_NextPart_000_0021_01C251DC.F975D820" X-Priority: 3 X-MSMail-Priority: Normal X-Mailer: Microsoft Outlook Express 6.00.2600.0000 X-MimeOLE: Produced By Microsoft MimeOLE V6.00.2600.0000 Sender: owner-obm-l@sucuri.mat.puc-rio.br Precedence: bulk Reply-To: obm-l@mat.puc-rio.br This is a multi-part message in MIME format. ------=_NextPart_000_0021_01C251DC.F975D820 Content-Type: text/plain; charset="iso-8859-1" Content-Transfer-Encoding: quoted-printable As provas da XXII Olimp=EDada Cearense de Matem=E1tica foram realizadas = hoje, 01 de setembro. Confira os enunciados em http://br.groups.yahoo.com/group/teoremalista/files/ocm02enu.pdf=20 Paulo ------=_NextPart_000_0021_01C251DC.F975D820 Content-Type: text/html; charset="iso-8859-1" Content-Transfer-Encoding: quoted-printable
As provas da XXII Olimp=EDada Cearense de = Matem=E1tica foram=20 realizadas hoje, 01 de setembro. Confira os enunciados em
 
http://br.groups.yahoo.com/group/teoremalista/files/ocm02enu.pdf=20

Paulo
------=_NextPart_000_0021_01C251DC.F975D820-- ========================================================================= Instruções para entrar na lista, sair da lista e usar a lista em http://www.mat.puc-rio.br/~nicolau/olimp/obm-l.html O administrador desta lista é ========================================================================= From owner-obm-l@sucuri.mat.puc-rio.br Sun Sep 1 23:10:32 2002 Return-Path: Received: (from majordom@localhost) by sucuri.mat.puc-rio.br (8.9.3/8.9.3) id XAA04999 for obm-l-MTTP; Sun, 1 Sep 2002 23:09:25 -0300 Received: from smtp-3.ig.com.br (smtp-3.ig.com.br [200.226.132.152]) by sucuri.mat.puc-rio.br (8.9.3/8.9.3) with SMTP id XAA04995 for ; Sun, 1 Sep 2002 23:09:23 -0300 Received: (qmail 24990 invoked from network); 2 Sep 2002 02:09:32 -0000 Received: from shasta051118.ig.com.br (HELO jat) (200.151.51.118) by smtp-3.ig.com.br with SMTP; 2 Sep 2002 02:09:32 -0000 Message-ID: <001401c25225$c9712be0$763397c8@jat> From: "Jose Augusto" To: References: <20020901200651.76020.qmail@web40308.mail.yahoo.com> Subject: Re: [obm-l] Ajuda em fatorial.(1) Date: Sun, 1 Sep 2002 23:09:33 -0300 MIME-Version: 1.0 Content-Type: multipart/alternative; boundary="----=_NextPart_000_0011_01C2520C.A224C880" X-Priority: 3 X-MSMail-Priority: Normal X-Mailer: Microsoft Outlook Express 5.50.4133.2400 X-MIMEOLE: Produced By Microsoft MimeOLE V5.50.4133.2400 Sender: owner-obm-l@sucuri.mat.puc-rio.br Precedence: bulk Reply-To: obm-l@mat.puc-rio.br This is a multi-part message in MIME format. ------=_NextPart_000_0011_01C2520C.A224C880 Content-Type: text/plain; charset="iso-8859-1" Content-Transfer-Encoding: quoted-printable Pressupondo que se saiba que fatorial de um n=FAmero eh o produto de n ( = natural e > =3D a dois )com seus consecutivos , ou , o produto de n com = os antecessores ate 1 : n.(n-1).(n-2). ... 2.1 =3D n! e que 0!=3D1 : 1) (UnB)Sendo m . (m + 3) . (m - 1)! =3D 6 e m > 0, o valor de m =E9: (m - 2) . (m + 1)! 35 m(m-1)!(m+3) / (m+1)(m)!(m-2)=3D 6 / 35 m!(m+3) / (m+1)(m!)(m-2)=3D 6 / 35 (m+3) / (m +1)(m-2)=3D6/35,=20 ai fica : (m+3)35=3D6(m+1)(m-2) que resolvendo m=3D9. J=E1 os outros eh so questao de aplicar a definicao.=20 O 2)a)[(m + 2)! - (m + 1)!] m! =3D 24 =B2 Como (m+2)! =3D (m+2)(m+1)(m)...1=3D(m + 2)(m+1)! [(m+2)(m+1)! - (m+1)!]m!=3D24.24, [(m+1)!(m+2 - 1)]m!=3D24.24 (m+1)!(m+1)m!=3D24.24, (m+1)!.(m+1)!=3D24.24 , (m+1)!=3D24, m=3D3 :) Abracos,=20 jose A. Tavares. ------=_NextPart_000_0011_01C2520C.A224C880 Content-Type: text/html; charset="iso-8859-1" Content-Transfer-Encoding: quoted-printable
Pressupondo que se saiba que = fatorial de um=20 n=FAmero eh o produto de n ( natural e > =3D a dois )com seus = consecutivos ,=20 ou , o produto de n com os antecessores ate 1 :
n.(n-1).(n-2). ... 2.1 =3D = n!  e que 0!=3D1=20 :

1) (UnB)Sendo m . (m + 3) . (m - 1)! =3D 6 e m > = 0, o valor=20 de m =E9:

=

           &nbs= p;            = ;  (m=20 - 2) . (m + 1)!    35

 m(m-1)!(m+3) / (m+1)(m)!(m-2)=3D 6 / = 35

m!(m+3) / (m+1)(m!)(m-2)=3D 6 / 35

(m+3) / (m +1)(m-2)=3D6/35,

ai fica : (m+3)35=3D6(m+1)(m-2) que resolvendo=20 m=3D9.

 

J=E1 os outros eh so questao de aplicar a = definicao.=20

O 2)a)[(m = + 2)! - (m +=20 1)!] m! =3D 24 =B2

Como (m+2)! =3D (m+2)(m+1)(m)...1=3D(m + 2)(m+1)!

[(m+2)(m+1)! - (m+1)!]m!=3D24.24, [(m+1)!(m+2 - 1)]m!=3D24.24

(m+1)!(m+1)m!=3D24.24, (m+1)!.(m+1)!=3D24.24 , (m+1)!=3D24, = m=3D3

:)  Abracos,

        =    =20 jose A. Tavares.

 

 

 

------=_NextPart_000_0011_01C2520C.A224C880-- ========================================================================= Instruções para entrar na lista, sair da lista e usar a lista em http://www.mat.puc-rio.br/~nicolau/olimp/obm-l.html O administrador desta lista é ========================================================================= From owner-obm-l@sucuri.mat.puc-rio.br Mon Sep 2 10:37:51 2002 Return-Path: Received: (from majordom@localhost) by sucuri.mat.puc-rio.br (8.9.3/8.9.3) id KAA09695 for obm-l-MTTP; Mon, 2 Sep 2002 10:34:07 -0300 Received: from sidney1.bol.com.br (sidney1.bol.com.br [200.221.24.206]) by sucuri.mat.puc-rio.br (8.9.3/8.9.3) with ESMTP id KAA09691; Mon, 2 Sep 2002 10:34:05 -0300 Received: from bol.com.br (200.221.24.137) by sidney1.bol.com.br (5.1.071) id 3D5D7EAA0032C9D4; Mon, 2 Sep 2002 10:34:04 -0300 Date: Mon, 2 Sep 2002 10:33:01 -0300 Message-Id: Subject: Re: [obm-l] fisica MIME-Version: 1.0 Content-Type: multipart/mixed; boundary="_=__=_XaM3_Boundary.1030973581.2A.508830.42.28354.52.42.101010.1213119245" From: "diegoalonsoteixeira" To: obm-l@mat.puc-rio.br Cc: obm-l@mat.puc-rio.br X-XaM3-API-Version: 2.4.3.4.4 X-SenderIP: 200.206.236.239 Sender: owner-obm-l@sucuri.mat.puc-rio.br Precedence: bulk Reply-To: obm-l@mat.puc-rio.br --_=__=_XaM3_Boundary.1030973581.2A.508830.42.28354.52.42.101010.1213119245 Content-Type: text/plain;charset="iso-8859-1" Content-Transfer-Encoding: quoted-printable ogrigado pela indica=E7=E3o __________________________________________________________________________ AcessoBOL, s=F3 R$ 9,90! O menor pre=E7o do mercado! Assine j=E1! http://www.bol.com.br/acessobol --_=__=_XaM3_Boundary.1030973581.2A.508830.42.28354.52.42.101010.1213119245 Content-Type: text/plain; name="00000LHP" Content-Transfer-Encoding: base64 d3d3LmZpc2ljYS5uZXQNCg0K6SBtdWl0byBjb21wbGV0by4uIGNvbG9jYSBuYSBidXNjYSBx IHZjIGFjaGEuDQoNCg0KLS0tLS0gT3JpZ2luYWwgTWVzc2FnZSAtLS0tLQ0KRnJvbTogImRp ZWdvYWxvbnNvdGVpeGVpcmEiIDxkaWVnb2Fsb25zb3RlaXhlaXJhQGJvbC5jb20uYnI+DQpU bzogPG9ibS1sQG1hdC5wdWMtcmlvLmJyPg0KQ2M6IDxvYm0tbEBtYXQucHVjLXJpby5icj4N ClNlbnQ6IFN1bmRheSwgU2VwdGVtYmVyIDAxLCAyMDAyIDExOjAyIEFNDQpTdWJqZWN0OiBS ZTogW29ibS1sXSBmaXNpY2ENCg0KDQpvZ3JpZ2FkbyBwZWxhIGluZGljYefjbywgbWFzIG1l dSBpbmdsZXMgbuNvIOkgbXVpdG8NCmJvbSx2YyBjb25oZWNlIGFsZ3VtIGVtIHBvcnR1Z3Vl cz8NCg0KDQoNCl9fX19fX19fX19fX19fX19fX19fX19fX19fX19fX19fX19fX19fX19fX19f X19fX19fX19fX19fX19fX19fX19fX19fX19fX19fDQpBY2Vzc29CT0wsIHPzIFIkIDksOTAh IE8gbWVub3IgcHJl528gZG8gbWVyY2FkbyENCkFzc2luZSBq4SEgaHR0cDovL3d3dy5ib2wu Y29tLmJyL2FjZXNzb2JvbA0KDQoNCg0KLS0tLS0tLS0tLS0tLS0tLS0tLS0tLS0tLS0tLS0t LS0tLS0tLS0tLS0tLS0tLS0tLS0tLS0tLS0tLS0tLS0tLS0tLS0tLS0tLS0tLQ0KLS0tLQ0K DQoNCj4gVuEgYXTpIGh0dHA6Ly9zY2llbmNld29ybGQud29sZnJhbS5jb20vcGh5c2ljcy8g ZSBlc3BlY2lmaXF1ZSBvIHF1ZSBxdWVyDQo+IHNhYmVyOiBhbmd1bGFyIG1vbWVudHVtICht b21lbnRvIGFuZ3VsYXIpLCBtb21lbnQgb2YgaW5lcnRpYSAobW9tZW50byBkZQ0KPiBpbuly Y2lhKSwgdG9ycXVlICh0b3JxdWUpLi4uDQo+DQo+IEpGDQo+DQo+IC0tLS0tIE9yaWdpbmFs IE1lc3NhZ2UgLS0tLS0NCj4gRnJvbTogImRpZWdvYWxvbnNvdGVpeGVpcmEiIDxkaWVnb2Fs b25zb3RlaXhlaXJhQGJvbC5jb20uYnI+DQo+IFRvOiA8b2JtLWxAbWF0LnB1Yy1yaW8uYnI+ DQo+IENjOiA8b2JtLWxAbWF0LnB1Yy1yaW8uYnI+DQo+IFNlbnQ6IFNhdHVyZGF5LCBBdWd1 c3QgMzEsIDIwMDIgODoxMiBBTQ0KPiBTdWJqZWN0OiBbb2JtLWxdIGZpc2ljYQ0KPg0KPg0K PiBBbGd1ZW0gcG9kZXJpYSBtZSBpbmRpY2FyIGFsZ3VtIHNpdGUgbmEgaW50ZXJuZXQgcXVl DQo+IGV4cGxpcXVlIGRldGFsaGFkYW1lbnRlIG1vbWVudG8gYW5ndWxhcixtb21lbnRvIGRl DQo+IGlu6XJjaWEgLHRvcnF1ZS4uDQo+IG9icmlnYWRvDQo+DQo+DQo+DQo+ID09PT09PT09 PT09PT09PT09PT09PT09PT09PT09PT09PT09PT09PT09PT09PT09PT09PT09PT09PT09PT09 PT09PT09PT09PT0NCj4gSW5zdHJ15/VlcyBwYXJhIGVudHJhciBuYSBsaXN0YSwgc2FpciBk YSBsaXN0YSBlIHVzYXIgYSBsaXN0YSBlbQ0KPiBodHRwOi8vd3d3Lm1hdC5wdWMtcmlvLmJy L35uaWNvbGF1L29saW1wL29ibS1sLmh0bWwNCj4gTyBhZG1pbmlzdHJhZG9yIGRlc3RhIGxp c3RhIOkgPG5pY29sYXVAbWF0LnB1Yy1yaW8uYnI+DQo+ID09PT09PT09PT09PT09PT09PT09 PT09PT09PT09PT09PT09PT09PT09PT09PT09PT09PT09PT09PT09PT09PT09PT09PT09PT0N Cj4NCg0KDQo9PT09PT09PT09PT09PT09PT09PT09PT09PT09PT09PT09PT09PT09PT09PT09 PT09PT09PT09PT09PT09PT09PT09PT09PT09DQpJbnN0cnXn9WVzIHBhcmEgZW50cmFyIG5h IGxpc3RhLCBzYWlyIGRhIGxpc3RhIGUgdXNhciBhIGxpc3RhIGVtDQpodHRwOi8vd3d3Lm1h dC5wdWMtcmlvLmJyL35uaWNvbGF1L29saW1wL29ibS1sLmh0bWwNCk8gYWRtaW5pc3RyYWRv ciBkZXN0YSBsaXN0YSDpIDxuaWNvbGF1QG1hdC5wdWMtcmlvLmJyPg0KPT09PT09PT09PT09 PT09PT09PT09PT09PT09PT09PT09PT09PT09PT09PT09PT09PT09PT09PT09PT09PT09PT09 PT09PT09PQ0K --_=__=_XaM3_Boundary.1030973581.2A.508830.42.28354.52.42.101010.1213119245-- ========================================================================= Instruções para entrar na lista, sair da lista e usar a lista em http://www.mat.puc-rio.br/~nicolau/olimp/obm-l.html O administrador desta lista é ========================================================================= From owner-obm-l@sucuri.mat.puc-rio.br Mon Sep 2 11:16:51 2002 Return-Path: Received: (from majordom@localhost) by sucuri.mat.puc-rio.br (8.9.3/8.9.3) id LAA10686 for obm-l-MTTP; Mon, 2 Sep 2002 11:14:36 -0300 Received: from web21307.mail.yahoo.com (web21307.mail.yahoo.com [216.136.128.232]) by sucuri.mat.puc-rio.br (8.9.3/8.9.3) with SMTP id LAA10682 for ; Mon, 2 Sep 2002 11:14:33 -0300 Message-ID: <20020902141504.59499.qmail@web21307.mail.yahoo.com> Received: from [200.19.153.1] by web21307.mail.yahoo.com via HTTP; Mon, 02 Sep 2002 11:15:04 ART Date: Mon, 2 Sep 2002 11:15:04 -0300 (ART) From: =?iso-8859-1?q?Marcos=20Reynaldo?= Subject: Re: [obm-l] esclarecimento To: obm-l@mat.puc-rio.br In-Reply-To: MIME-Version: 1.0 Content-Type: text/plain; charset=iso-8859-1 Content-Transfer-Encoding: 8bit Sender: owner-obm-l@sucuri.mat.puc-rio.br Precedence: bulk Reply-To: obm-l@mat.puc-rio.br Bom nao precisa de formulas mais elaboradas pra resolver o problema, eh soh verificar o espaco amostral (admitindo moedas nao viciadas). K=Cara C=coroa Temos {CCC, CCK, CKC, CKK, KCC, KCK, KKC, KKK} dos quais nos interessam apenas os que apresentam duas caras e um coroa. Entao sao 3 casos em oito (CCK, CKC, KCC), e dai a probabilidade ser 3/8. Eu acho que esse eh o raciocinio mais simples (claro que os outros estao certos). Agora, também como o Morgado disse, nao entendi de onde vc tirou o 1/4, nao consegui ver nenhuma logica (errada evidentemente) que leve a esse resultado. Tambem estou curioso. []'s Marcos --- Lltmdrtm@aol.com escreveu: > Ao jogar três moedas, qual a probabilidade de dar > duas caras e uma coroa? > Alguns colegas acham que é 1 / 4 outros acham que é > 3 / 8. Por que a > confusão? > É possível as duas respostas estarem corretas? > _______________________________________________________________________ Yahoo! PageBuilder O super editor para criação de sites: é grátis, fácil e rápido. http://br.geocities.yahoo.com/v/pb.html ========================================================================= Instruções para entrar na lista, sair da lista e usar a lista em http://www.mat.puc-rio.br/~nicolau/olimp/obm-l.html O administrador desta lista é ========================================================================= From owner-obm-l@sucuri.mat.puc-rio.br Mon Sep 2 12:20:34 2002 Return-Path: Received: (from majordom@localhost) by sucuri.mat.puc-rio.br (8.9.3/8.9.3) id MAA12032 for obm-l-MTTP; Mon, 2 Sep 2002 12:18:18 -0300 Received: from imo-d07.mx.aol.com (imo-d07.mx.aol.com [205.188.157.39]) by sucuri.mat.puc-rio.br (8.9.3/8.9.3) with ESMTP id MAA12027 for ; Mon, 2 Sep 2002 12:18:16 -0300 From: Lltmdrtm@aol.com Received: from Lltmdrtm@aol.com by imo-d07.mx.aol.com (mail_out_v34.10.) id z.53.1be2e47d (25508) for ; Mon, 2 Sep 2002 11:18:45 -0400 (EDT) Message-ID: <53.1be2e47d.2aa4db54@aol.com> Date: Mon, 2 Sep 2002 11:18:44 EDT Subject: Re: [obm-l] esclarecimento To: obm-l@mat.puc-rio.br MIME-Version: 1.0 Content-Type: multipart/alternative; boundary="part1_53.1be2e47d.2aa4db54_boundary" X-Mailer: AOL 7.0 for Windows BR sub 10501 Sender: owner-obm-l@sucuri.mat.puc-rio.br Precedence: bulk Reply-To: obm-l@mat.puc-rio.br --part1_53.1be2e47d.2aa4db54_boundary Content-Type: text/plain; charset="ISO-8859-1" Content-Transfer-Encoding: quoted-printable A indaga=E7=E3o surgiu a partir de uma coloca=E7=E3o de uma colega:=20 Ele disse que ao jogarmos as tr=EAs moedas, o que pode ocorrer =E9: V=EA duas k,k e uma c ou kkk ou cc e uma k ou ccc, por isso a probabilidade=20= 1 /=20 4. =C9 como se n=E3o importa-se a ordem de caras e coroas e sim quantas caras e= =20 coroas podemos ver num lan=E7amento. Valeu!!!! --part1_53.1be2e47d.2aa4db54_boundary Content-Type: text/html; charset="ISO-8859-1" Content-Transfer-Encoding: quoted-printable A indaga=E7=E3o surgiu a partir de uma coloca=E7=E3o d= e uma colega:
Ele disse que ao jogarmos as tr=EAs moedas, o que pode ocorrer =E9:
V=EA duas k,k e uma c ou kkk ou cc e uma k ou ccc, por isso a probabilidade=20= 1 / 4.
=C9 como se n=E3o importa-se a ordem de caras e coroas e sim quantas caras e= coroas podemos ver num lan=E7amento.

Valeu!!!!
--part1_53.1be2e47d.2aa4db54_boundary-- ========================================================================= Instruções para entrar na lista, sair da lista e usar a lista em http://www.mat.puc-rio.br/~nicolau/olimp/obm-l.html O administrador desta lista é ========================================================================= From owner-obm-l@sucuri.mat.puc-rio.br Mon Sep 2 13:17:38 2002 Return-Path: Received: (from majordom@localhost) by sucuri.mat.puc-rio.br (8.9.3/8.9.3) id NAA13328 for obm-l-MTTP; Mon, 2 Sep 2002 13:17:01 -0300 Received: from web12901.mail.yahoo.com (web12901.mail.yahoo.com [216.136.174.68]) by sucuri.mat.puc-rio.br (8.9.3/8.9.3) with SMTP id NAA13324 for ; Mon, 2 Sep 2002 13:16:58 -0300 Message-ID: <20020902161730.74711.qmail@web12901.mail.yahoo.com> Received: from [200.206.103.3] by web12901.mail.yahoo.com via HTTP; Mon, 02 Sep 2002 13:17:30 ART Date: Mon, 2 Sep 2002 13:17:30 -0300 (ART) From: =?iso-8859-1?q?Johann=20Peter=20Gustav=20Lejeune=20Dirichlet?= Subject: Re: [obm-l] Achar_raizes_"na_mão" To: obm-l@mat.puc-rio.br In-Reply-To: <003201c2508c$8beffe80$0201a8c0@x> MIME-Version: 1.0 Content-Type: multipart/alternative; boundary="0-1926601694-1030983450=:73943" Content-Transfer-Encoding: 8bit Sender: owner-obm-l@sucuri.mat.puc-rio.br Precedence: bulk Reply-To: obm-l@mat.puc-rio.br --0-1926601694-1030983450=:73943 Content-Type: text/plain; charset=iso-8859-1 Content-Transfer-Encoding: 8bit Tente Taylor ou Fourier Jeremias de Paula Eduardo escreveu: Estou acostumado a apertar a raiz da calculadora, mas gostaria de aprender a calcular-las manualmente e não encontrei como. Obrigado por toda ajuda Jeremias de Paula Eduardo --------------------------------- Yahoo! PageBuilder - O super editor para criação de sites: é grátis, fácil e rápido. --0-1926601694-1030983450=:73943 Content-Type: text/html; charset=iso-8859-1 Content-Transfer-Encoding: 8bit

Tente Taylor ou Fourier

  Jeremias de Paula Eduardo <jeremias@brturbo.com> escreveu:

Estou acostumado a apertar a raiz da calculadora, mas gostaria de aprender a calcular-las manualmente e não encontrei como.
 
Obrigado por toda ajuda
 
Jeremias de Paula Eduardo


Yahoo! PageBuilder - O super editor para criação de sites: é grátis, fácil e rápido. --0-1926601694-1030983450=:73943-- ========================================================================= Instruções para entrar na lista, sair da lista e usar a lista em http://www.mat.puc-rio.br/~nicolau/olimp/obm-l.html O administrador desta lista é ========================================================================= From owner-obm-l@sucuri.mat.puc-rio.br Mon Sep 2 13:19:10 2002 Return-Path: Received: (from majordom@localhost) by sucuri.mat.puc-rio.br (8.9.3/8.9.3) id NAA13373 for obm-l-MTTP; Mon, 2 Sep 2002 13:18:48 -0300 Received: from smtp.ieg.com.br (sharon.protocoloweb.com.br [200.226.139.12]) by sucuri.mat.puc-rio.br (8.9.3/8.9.3) with ESMTP id NAA13369 for ; Mon, 2 Sep 2002 13:18:45 -0300 Received: from dois (200-161-155-49.dsl.telesp.net.br [200.161.155.49]) by smtp.ieg.com.br (IeG relay/8.9.3) with SMTP id g82GKcVo030397 for ; Mon, 2 Sep 2002 13:20:54 -0300 (BRT) Message-ID: <003201c2529c$f2084800$0200a8c0@dois> From: =?iso-8859-1?Q?Gabriel_P=E9rgola?= To: "Obm-l" Subject: [obm-l] =?iso-8859-1?Q?N=FAmeros_Complexos?= Date: Mon, 2 Sep 2002 13:22:18 -0300 MIME-Version: 1.0 Content-Type: text/plain; charset="iso-8859-1" Content-Transfer-Encoding: 8bit X-Priority: 3 X-MSMail-Priority: Normal X-Mailer: Microsoft Outlook Express 5.50.4807.1700 X-MIMEOLE: Produced By Microsoft MimeOLE V5.50.4807.1700 Sender: owner-obm-l@sucuri.mat.puc-rio.br Precedence: bulk Reply-To: obm-l@mat.puc-rio.br Ops! Mandei a mensagem pelo meu outro e-mail que nao eh cadastrado. Mas agora tah aí com o certo! E aí pessoal, Gostaria de ver a resolução destes problemas de números complexos que não consegui fazer: 1) Obtenha o argumento de sen 40º + i cos 40º 2) Determine o menor valor inteiro e positivo de n para o qual (1 + i sqrt[3])^n é um numero real 3) Determine o menor valor inteiro e positivo de n para o qual (1 + i sqrt[3])^n é um numero real positivo. 4) Obtenha as raizes complexas das equacoes: a) x^5 = 1 b) x^6 = 1 5) Representando, no plano, as raizes complexas da equacao z^3 + 8 = 0, obtem-se um triangulo. Calcule a area desse triangulo. 6) A quantidade de numeros complexos que tem o seu quadrado igual ao seu conjugado é? É isso! Agradeço qualquer ajuda. Gabriel Pérgola ========================================================================= Instruções para entrar na lista, sair da lista e usar a lista em http://www.mat.puc-rio.br/~nicolau/olimp/obm-l.html O administrador desta lista é ========================================================================= From owner-obm-l@sucuri.mat.puc-rio.br Mon Sep 2 13:56:05 2002 Return-Path: Received: (from majordom@localhost) by sucuri.mat.puc-rio.br (8.9.3/8.9.3) id NAA14429 for obm-l-MTTP; Mon, 2 Sep 2002 13:54:50 -0300 Received: from web12908.mail.yahoo.com (web12908.mail.yahoo.com [216.136.174.75]) by sucuri.mat.puc-rio.br (8.9.3/8.9.3) with SMTP id NAA14425 for ; Mon, 2 Sep 2002 13:54:47 -0300 Message-ID: <20020902165520.76115.qmail@web12908.mail.yahoo.com> Received: from [200.206.103.3] by web12908.mail.yahoo.com via HTTP; Mon, 02 Sep 2002 13:55:20 ART Date: Mon, 2 Sep 2002 13:55:20 -0300 (ART) From: =?iso-8859-1?q?Johann=20Peter=20Gustav=20Lejeune=20Dirichlet?= Subject: Re: [obm-l] polinomio To: obm-l@mat.puc-rio.br In-Reply-To: MIME-Version: 1.0 Content-Type: multipart/alternative; boundary="0-637745560-1030985720=:75007" Content-Transfer-Encoding: 8bit Sender: owner-obm-l@sucuri.mat.puc-rio.br Precedence: bulk Reply-To: obm-l@mat.puc-rio.br --0-637745560-1030985720=:75007 Content-Type: text/plain; charset=iso-8859-1 Content-Transfer-Encoding: 8bit Eu nao sei direito mas acho que usa complexos "adr.scr.m" escreveu: gostaria de uma ajuda nessa questao, P(x) eh um polinomio de grau 3n tal que P(0)=P(3)=...=P(3n)=2 P(1)=P(4)=...=P(3n-2)=1 P(2)=P(5)=...=P(3n-1)=0 e P(3n+1)=730 Determine n. []'s. Obrigado. Adriano. __________________________________________________________________________ AcessoBOL, só R$ 9,90! O menor preço do mercado! Assine já! http://www.bol.com.br/acessobol ========================================================================= Instruções para entrar na lista, sair da lista e usar a lista em http://www.mat.puc-rio.br/~nicolau/olimp/obm-l.html O administrador desta lista é ========================================================================= --------------------------------- Yahoo! PageBuilder - O super editor para criação de sites: é grátis, fácil e rápido. --0-637745560-1030985720=:75007 Content-Type: text/html; charset=iso-8859-1 Content-Transfer-Encoding: 8bit

Eu nao sei direito mas acho que usa complexos

  "adr.scr.m" <adr.scr.m@bol.com.br> escreveu:

gostaria de uma ajuda nessa questao,
P(x) eh um polinomio de grau 3n tal que
P(0)=P(3)=...=P(3n)=2
P(1)=P(4)=...=P(3n-2)=1
P(2)=P(5)=...=P(3n-1)=0

e P(3n+1)=730

Determine n.

[]'s.
Obrigado.
Adriano.


__________________________________________________________________________
AcessoBOL, só R$ 9,90! O menor preço do mercado!
Assine já! http://www.bol.com.br/acessobol


=========================================================================
Instruções para entrar na lista, sair da lista e usar a lista em
http://www.mat.puc-rio.br/~nicolau/olimp/obm-l.html
O administrador desta lista é
=========================================================================


Yahoo! PageBuilder - O super editor para criação de sites: é grátis, fácil e rápido. --0-637745560-1030985720=:75007-- ========================================================================= Instruções para entrar na lista, sair da lista e usar a lista em http://www.mat.puc-rio.br/~nicolau/olimp/obm-l.html O administrador desta lista é ========================================================================= From owner-obm-l@sucuri.mat.puc-rio.br Mon Sep 2 15:17:49 2002 Return-Path: Received: (from majordom@localhost) by sucuri.mat.puc-rio.br (8.9.3/8.9.3) id PAA16284 for obm-l-MTTP; Mon, 2 Sep 2002 15:16:25 -0300 Received: from puma.unisys.com.br (ns2.unisys.com.br [200.220.64.7]) by sucuri.mat.puc-rio.br (8.9.3/8.9.3) with ESMTP id PAA16272 for ; Mon, 2 Sep 2002 15:16:07 -0300 Received: from josefran (riopm18p108.uninet.com.br [200.220.16.108]) by puma.unisys.com.br (8.12.3/8.12.3) with SMTP id g82IG8jR003236 for ; Mon, 2 Sep 2002 15:16:22 -0300 (EST) X-Spam-Filter: check_local@puma.unisys.com.br by digitalanswers.org Message-ID: <009501c252ad$81c84200$6c10dcc8@josefran> From: "Jose Francisco Guimaraes Costa" To: "obm-l" Subject: En: [obm-l] esclarecimento Date: Mon, 2 Sep 2002 14:42:11 -0300 MIME-Version: 1.0 Content-Type: multipart/alternative; boundary="----=_NextPart_000_008E_01C2528E.EBDDA3C0" X-Priority: 3 X-MSMail-Priority: Normal X-Mailer: Microsoft Outlook Express 5.00.2314.1300 X-MimeOLE: Produced By Microsoft MimeOLE V5.00.2314.1300 Sender: owner-obm-l@sucuri.mat.puc-rio.br Precedence: bulk Reply-To: obm-l@mat.puc-rio.br This is a multi-part message in MIME format. ------=_NextPart_000_008E_01C2528E.EBDDA3C0 Content-Type: text/plain; charset="iso-8859-1" Content-Transfer-Encoding: quoted-printable Segundo essa linha de racioc=EDnio, a probabilidade de se obter tr=EAs = caras =E9 1/4. Topas jogar comigo? V aposta em tr=EAs caras, e eu pago 4 = por 1. JF -----Mensagem Original-----=20 De: Lltmdrtm@aol.com=20 Para: obm-l@mat.puc-rio.br=20 Enviada em: Segunda-feira, 2 de Setembro de 2002 12:18 Assunto: Re: [obm-l] esclarecimento A indaga=E7=E3o surgiu a partir de uma coloca=E7=E3o de uma colega:=20 Ele disse que ao jogarmos as tr=EAs moedas, o que pode ocorrer =E9: V=EA duas k,k e uma c ou kkk ou cc e uma k ou ccc, por isso a = probabilidade 1 / 4. =C9 como se n=E3o importa-se a ordem de caras e coroas e sim quantas = caras e coroas podemos ver num lan=E7amento. Valeu!!!!=20 ------=_NextPart_000_008E_01C2528E.EBDDA3C0 Content-Type: text/html; charset="iso-8859-1" Content-Transfer-Encoding: quoted-printable
Segundo essa linha de racioc=EDnio, a probabilidade de se obter = tr=EAs caras =E9=20 1/4. Topas jogar comigo? V aposta em tr=EAs caras, e eu pago 4 por = 1.
 
JF
 
-----Mensagem Original-----=20
De: Lltmdrtm@aol.com=20
Enviada em: Segunda-feira, 2 de Setembro de 2002 12:18
Assunto: Re: [obm-l] esclarecimento

A indaga=E7=E3o surgiu a partir de uma = coloca=E7=E3o de uma colega:=20
Ele disse que ao jogarmos as tr=EAs moedas, o que pode ocorrer = =E9:
V=EA duas=20 k,k e uma c ou kkk ou cc e uma k ou ccc, por isso a probabilidade 1 / = 4.
=C9=20 como se n=E3o importa-se a ordem de caras e coroas e sim quantas caras e = coroas=20 podemos ver num lan=E7amento.

Valeu!!!!
=
------=_NextPart_000_008E_01C2528E.EBDDA3C0-- ========================================================================= Instruções para entrar na lista, sair da lista e usar a lista em http://www.mat.puc-rio.br/~nicolau/olimp/obm-l.html O administrador desta lista é ========================================================================= From owner-obm-l@sucuri.mat.puc-rio.br Mon Sep 2 16:34:43 2002 Return-Path: Received: (from majordom@localhost) by sucuri.mat.puc-rio.br (8.9.3/8.9.3) id QAA17827 for obm-l-MTTP; Mon, 2 Sep 2002 16:33:25 -0300 Received: from traven10.uol.com.br (200-221-4-45.portais-uolinc.uol.com.br [200.221.4.45] (may be forged)) by sucuri.mat.puc-rio.br (8.9.3/8.9.3) with ESMTP id QAA17822 for ; Mon, 2 Sep 2002 16:33:22 -0300 Received: from u2z7z2 ([200.158.145.22]) by traven10.uol.com.br (8.9.1/8.9.1) with ESMTP id QAA13046 for ; Mon, 2 Sep 2002 16:33:23 -0300 (BRT) Message-ID: <001e01c252b7$ce50f4a0$2101a8c0@u2z7z2> From: "Wagner" To: Subject: [obm-l] =?iso-8859-1?Q?O_problema_das_infinitas_solu=E7=F5es?= Date: Mon, 2 Sep 2002 16:34:50 -0300 Organization: Wagner MIME-Version: 1.0 Content-Type: multipart/alternative; boundary="----=_NextPart_000_001B_01C2529E.A8B8F800" X-Priority: 3 X-MSMail-Priority: Normal X-Mailer: Microsoft Outlook Express 5.50.4133.2400 X-MimeOLE: Produced By Microsoft MimeOLE V5.50.4133.2400 Sender: owner-obm-l@sucuri.mat.puc-rio.br Precedence: bulk Reply-To: obm-l@mat.puc-rio.br This is a multi-part message in MIME format. ------=_NextPart_000_001B_01C2529E.A8B8F800 Content-Type: text/plain; charset="iso-8859-1" Content-Transfer-Encoding: quoted-printable Esse =E9 o meu primeiro problema na lista Nota=E7=E3o: - a^(b) =3D a elevado a pot=EAncia b=20 - PI =3D o n=BA pi Prove que a equa=E7=E3o: x^(PI)-5x^(PI-1)+3=3D0. Possui infinitas = solu=E7=F5es complexas. Andr=E9 T. ------=_NextPart_000_001B_01C2529E.A8B8F800 Content-Type: text/html; charset="iso-8859-1" Content-Transfer-Encoding: quoted-printable
Esse =E9 o meu primeiro problema na=20 lista
 
Nota=E7=E3o:
- a^(b) =3D a elevado a = pot=EAncia b
-=20 PI =3D o n=BA pi
 
Prove que a equa=E7=E3o: = x^(PI)-5x^(PI-1)+3=3D0. Possui=20 infinitas solu=E7=F5es complexas.
 

 Andr=E9 = T.
------=_NextPart_000_001B_01C2529E.A8B8F800-- ========================================================================= Instruções para entrar na lista, sair da lista e usar a lista em http://www.mat.puc-rio.br/~nicolau/olimp/obm-l.html O administrador desta lista é ========================================================================= From owner-obm-l@sucuri.mat.puc-rio.br Mon Sep 2 16:49:59 2002 Return-Path: Received: (from majordom@localhost) by sucuri.mat.puc-rio.br (8.9.3/8.9.3) id QAA18165 for obm-l-MTTP; Mon, 2 Sep 2002 16:49:26 -0300 Received: from traven9.uol.com.br (200-221-4-35.portais-uolinc.uol.com.br [200.221.4.35] (may be forged)) by sucuri.mat.puc-rio.br (8.9.3/8.9.3) with ESMTP id QAA18160 for ; Mon, 2 Sep 2002 16:49:23 -0300 Received: from u2z7z2 ([200.158.145.22]) by traven9.uol.com.br (8.9.1/8.9.1) with ESMTP id QAA09283 for ; Mon, 2 Sep 2002 16:51:31 -0300 (BRT) Message-ID: <001801c252ba$0a9dbea0$2101a8c0@u2z7z2> From: "Wagner" To: Subject: [obm-l] =?iso-8859-1?Q?Re:_Re:_=5Bobm-l=5D_esclarecimento_=28To_:Augusto_C=E9sar_?= =?iso-8859-1?Q?Morgado_=29?= Date: Mon, 2 Sep 2002 16:50:51 -0300 Organization: Wagner MIME-Version: 1.0 Content-Type: multipart/alternative; boundary="----=_NextPart_000_0015_01C252A0.E51E2C00" X-Priority: 3 X-MSMail-Priority: Normal X-Mailer: Microsoft Outlook Express 5.50.4133.2400 X-MimeOLE: Produced By Microsoft MimeOLE V5.50.4133.2400 Sender: owner-obm-l@sucuri.mat.puc-rio.br Precedence: bulk Reply-To: obm-l@mat.puc-rio.br This is a multi-part message in MIME format. ------=_NextPart_000_0015_01C252A0.E51E2C00 Content-Type: text/plain; charset="iso-8859-1" Content-Transfer-Encoding: quoted-printable -Se voc=EA quer q os 3 eventos aconte=E7am em sequ=EAncia (cara, cara e = depois coroa), a probabilidade =E9 (1/2) ao cubo =3D 1/8 -Se a ordem dos eventos n=E3o importa, a probabilidade =E9 C(3,2)/8 =3D = 3/8, ou C(3,1)/8 =3D 3/8. OBS:C(a,b)=3D combina=E7=F5es de a elementos tomados p a p. Andr=E9 T. ------=_NextPart_000_0015_01C252A0.E51E2C00 Content-Type: text/html; charset="iso-8859-1" Content-Transfer-Encoding: quoted-printable
-Se voc=EA quer q os 3 eventos = aconte=E7am em sequ=EAncia=20 (cara, cara e depois coroa), a probabilidade =E9 (1/2) ao cubo =3D = 1/8
-Se a ordem dos eventos n=E3o importa, = a=20 probabilidade =E9 C(3,2)/8 =3D 3/8, ou C(3,1)/8 =3D 3/8.
 
OBS:C(a,b)=3D combina=E7=F5es de a = elementos tomados p a=20 p.
 
Andr=E9 T.
------=_NextPart_000_0015_01C252A0.E51E2C00-- ========================================================================= Instruções para entrar na lista, sair da lista e usar a lista em http://www.mat.puc-rio.br/~nicolau/olimp/obm-l.html O administrador desta lista é ========================================================================= From owner-obm-l@sucuri.mat.puc-rio.br Mon Sep 2 17:40:16 2002 Return-Path: Received: (from majordom@localhost) by sucuri.mat.puc-rio.br (8.9.3/8.9.3) id RAA19587 for obm-l-MTTP; Mon, 2 Sep 2002 17:39:32 -0300 Received: from jurere.terra.com.br (jurere.terra.com.br [200.176.3.22]) by sucuri.mat.puc-rio.br (8.9.3/8.9.3) with ESMTP id RAA19583 for ; Mon, 2 Sep 2002 17:39:29 -0300 Received: from engenho.terra.com.br (engenho.terra.com.br [200.176.3.42]) by jurere.terra.com.br (Postfix) with ESMTP id 2CA1448A5E for ; Mon, 2 Sep 2002 17:40:03 -0300 (EST) Received: from Itautec.terra.com.br (200-158-60-43.dsl.telesp.net.br [200.158.60.43]) (authenticated user bruleite) by engenho.terra.com.br (Postfix) with ESMTP id 490096848C for ; Mon, 2 Sep 2002 17:40:02 -0300 (EST) Message-Id: <5.1.0.14.0.20020902173358.00a58700@pop.sao.terra.com.br> X-Sender: bruleite@pop.sao.terra.com.br X-Mailer: QUALCOMM Windows Eudora Version 5.1 Date: Mon, 02 Sep 2002 17:36:29 -0300 To: obm-l@mat.puc-rio.br From: "Bruno F. C. Leite" Subject: Re: En: [obm-l] esclarecimento In-Reply-To: <009501c252ad$81c84200$6c10dcc8@josefran> Mime-Version: 1.0 Content-Type: text/plain; charset="iso-8859-1"; format=flowed Content-Transfer-Encoding: 8bit X-MIME-Autoconverted: from quoted-printable to 8bit by sucuri.mat.puc-rio.br id RAA19584 Sender: owner-obm-l@sucuri.mat.puc-rio.br Precedence: bulk Reply-To: obm-l@mat.puc-rio.br Pensando assim, você pode morrer amanhã com probablilidade 1/2, pois só há dois eventos: 1)morrer amanhã 2)não morrer amanhã É claro que isto está errado. O problema é que os eventos não são equiprováveis... Bruno Leite http://www.ime.usp.br/~brleite At 14:42 02/09/02 -0300, you wrote: >Segundo essa linha de raciocínio, a probabilidade de se obter três caras é >1/4. Topas jogar comigo? V aposta em três caras, e eu pago 4 por 1. > >JF > >-----Mensagem Original----- >De: Lltmdrtm@aol.com >Para: obm-l@mat.puc-rio.br >Enviada em: Segunda-feira, 2 de Setembro de 2002 12:18 >Assunto: Re: [obm-l] esclarecimento > >A indagação surgiu a partir de uma colocação de uma colega: >Ele disse que ao jogarmos as três moedas, o que pode ocorrer é: >Vê duas k,k e uma c ou kkk ou cc e uma k ou ccc, por isso a probabilidade >1 / 4. >É como se não importa-se a ordem de caras e coroas e sim quantas caras e >coroas podemos ver num lançamento. > >Valeu!!!! ========================================================================= Instruções para entrar na lista, sair da lista e usar a lista em http://www.mat.puc-rio.br/~nicolau/olimp/obm-l.html O administrador desta lista é ========================================================================= From owner-obm-l@sucuri.mat.puc-rio.br Mon Sep 2 18:31:49 2002 Return-Path: Received: (from majordom@localhost) by sucuri.mat.puc-rio.br (8.9.3/8.9.3) id SAA20705 for obm-l-MTTP; Mon, 2 Sep 2002 18:31:44 -0300 Received: from smtp.ieg.com.br (stone.protocoloweb.com.br [200.226.139.11]) by sucuri.mat.puc-rio.br (8.9.3/8.9.3) with ESMTP id SAA20701 for ; Mon, 2 Sep 2002 18:31:42 -0300 Received: from localhost (wh1stler@200-158-118-125.dsl.telesp.net.br [200.158.118.125]) by smtp.ieg.com.br (IeG relay/8.9.3) with SMTP id g82LSDfE067536 for ; Mon, 2 Sep 2002 18:28:13 -0300 (BRT) From: Tonik To: obm-l@mat.puc-rio.br Date: Mon, 02 Sep 2002 18:31:23 -0300 X-Priority: 3 (Normal) Organization: Tonik In-Reply-To: <003201c2529c$f2084800$0200a8c0@dois> Message-Id: Subject: =?windows-1252?Q?Re:=20[obm-l]=20N=FAmeros=20Complexos?= MIME-Version: 1.0 Content-Type: text/plain; charset="iso-8859-1" X-Mailer: Opera 6.04 build 1135 Sender: owner-obm-l@sucuri.mat.puc-rio.br Precedence: bulk Reply-To: obm-l@mat.puc-rio.br 02/09/02 13:22:18, Gabriel Pérgola wrote: >E aí pessoal, > >Gostaria de ver a resolução destes problemas de números complexos que não >consegui fazer: Sao exercicios simples, q vou fazer mais para me exercitar, pois sao mais trabalhosos do q desafiantes... cheque as contas! >1) Obtenha o argumento de sen 40º + i cos 40º obviamente, 40º >2) Determine o menor valor inteiro e positivo de n para o qual (1 + i >sqrt[3])^n é um numero real para (1 + i*sqrt(3))^n ser real, seu argumento devera ser 0º ou 180º ou k180º, k E Z, passando (1+isqrt(3)) para a forma trigonometrica, temos: modulo = sqrt(1^2 + sqrt(3)^2) = sqrt(1+3) = 2 argumento = arccos(1/2) = 60º entao temos (2*(cos60º+isen60º))^n = = 2^n*(cos(60º*n)+isen(60º*n) para que o argumento (60º*n) de 0º ou 180º com n>0, n E Z: 60*n=360º, n=6 60*n=180º, n=3 Logo a resposta eh 3. >3) Determine o menor valor inteiro e positivo de n para o qual (1 + i >sqrt[3])^n é um numero real positivo. a mesma coisa, só que agora 180º nao serve (pois eh real negativo) 60*n=360, n=6 >4) Obtenha as raizes complexas das equacoes: > a) x^5 = 1 > b) x^6 = 1 x^5 = 1 x= raizquintupla(1*(cos0+isen0)) x= cos(0/5 + 360k/5) + isen(0/5 + 360k/5), 0<=k<5, k E Z as raizes: x= cos0º+isen0º = 1 (nao eh complexa) x= cos72º+isen72º x= cos144º+isen144º x= cos216º+isen216º x= cos288º+isen288º >5) Representando, no plano, as raizes complexas da equacao z^3 + 8 = 0, >obtem-se um triangulo. Calcule a area desse triangulo. z^3=-8 tem 3 raizes, por 3 ser impar, uma das raizes eh real z= raizcubica(-8) z= raizcubica( 8*(cos180º+isen180º) ) z= 2*(cos(180/3+360k/3)+isen(180/3+360k/3)), 0<=k<3 z= 2*(cos(60+120k)+isen(60+120k)) as raizes: k=0, z=2*(cos60º+isen60º) = 2*(1/2 + i*sqrt(3)/2) = 1+i*sqrt(3) k=1, z=2*(cos180º+isen180º) = 2*(-1 + i*0) = -2 k=2, z=2*(cos300º+isen300º) = (sabe-se que 1+isqrt(3) eh raiz, entao seu conjugado, 1-isqrt(3) eh a terceira raiz) entao temos os pontos do triangulo ABC no plano complexo: A(1, sqrt(3)) B(-2, 0) C(1, -sqrt(3)) Seja D a matriz: |Ax Ay 1| |Bx By 1| |Cx Cy 1| Area = modulo do determinante de D sobre 2 Area = |sqrt(3)+2sqrt(3)-(-sqrt(3)-2sqrt(3))|/2 Area = 3sqrt(3) >6) A quantidade de numeros complexos que tem o seu quadrado igual ao seu >conjugado é? Seja z um numero complexo, vc quer a qtde de nº complexos que z^2 = conjugado de z pela forma trigonometrica, seja m o modulo e a o argumento: m^2*(cos(2a)+isen(2a)) = m*(cos(a)-isen(a)) sabemos que -sen(x) = sen(-x) m^2*(cos(2a)+isen(2a)) = m*(cos(a)+isen(-a)) sabemos que m eh diferente de zero, pois senao o numero nao sera complexo. dividimos ambos os lados por m. m*(cos(2a)+isen(2a)) = (cos(a)+isen(-a)) pela identidade: (1) mcos2a = cosa (2) msen2a = sen-a (1) 2m*cos^2(a) - m = cosa (2) 2m*cosa*sena = -sena sabemos que sena nao eh zero, pois senao o numero nao sera complexo. (1) m = cosa / (2cos^2(a) - 1) (2) 2mcosa = -1 (2) cosa = -1/2m (1) m = (-1 / 2m) / ( 2*(1/4m^2) - 1) (1) m = (-1 / 2m) / ( 1/(2m^2) - 1) (1) m = (-1 / 2m) / ((1-2m^2)/(2m^2)) os extremos pelos meios (1) m = (-1 * 2m^2) / (2m * (1-2m^2)) (1) m = -2m^2 / (2m - 4m^3)) (1) 2m^2 -4m^4 = -2m^2 (1) -4m^4 + 4m^2 = 0 (1) m^4 -m^2 = 0 y = m^2 (1) y^2-y=0 (1) y(y-1) = 0 (1) ou y=0 --> m^2=0 --> m=0 o que nao devemos levar em conta no exercicio (1) ou y=1 --> m^2=1 --> m=1 ou m=-1 para m=1: (2) cosa = -1/2 (2) a = 120º formando o numero complexo: z = 1*(cos120º+isen120º) z = -1/2 + i*sqrt(3)/ 2 para m=-1: (2) cosa = 1/2 (2) a = 60º formando: z = -1*(cos60º+isen60º) z = 1*(cos240º+isen240º) z = -1/2 - i*sqrt(3)/2 Entao temos dois numeros complexos z tal que z^2 = conjugado de z e eles sao cos60º+isen60º e cos240º+isen240º ufa, quanta conta, acho melhor tirar a prova... z^2 = conjugado de z (isqrt(3)/2 - 1/2)^2 = -1/2 -isqrt(3)/2 -3/4 - isqrt(3)/2 +1/4 = -1/2 -isqrt(3)/2 -1/2 - isqrt(3)/2 = -1/2 -isqrt(3)/2 (verdade) z^2 = conjugado de z (-1/2 -isqrt(3)/2)^2 = -1/2 +isqrt(3)/2 1/4 + isqrt(3)/2 - 3/4 = -1/2 +isqrt(3)/2 -1/2 + isqrt(3)/2 = -1/2 +isqrt(3)/2 (verdade) >É isso! Agradeço qualquer ajuda. > >Gabriel Pérgola ========================================================================= Instruções para entrar na lista, sair da lista e usar a lista em http://www.mat.puc-rio.br/~nicolau/olimp/obm-l.html O administrador desta lista é ========================================================================= From owner-obm-l@sucuri.mat.puc-rio.br Mon Sep 2 19:32:10 2002 Return-Path: Received: (from majordom@localhost) by sucuri.mat.puc-rio.br (8.9.3/8.9.3) id TAA21628 for obm-l-MTTP; Mon, 2 Sep 2002 19:31:35 -0300 Received: from pina.terra.com.br (pina.terra.com.br [200.176.3.17]) by sucuri.mat.puc-rio.br (8.9.3/8.9.3) with ESMTP id TAA21624 for ; Mon, 2 Sep 2002 19:31:33 -0300 Received: from smtp4-poa.terra.com.br (smtp4-poa.terra.com.br [200.176.3.35]) by pina.terra.com.br (Postfix) with ESMTP id D4FD852FBA for ; Mon, 2 Sep 2002 19:32:07 -0300 (EST) Received: from xt (200-148-65-147.dsl.telesp.net.br [200.148.65.147]) (authenticated user macwad) by smtp4-poa.terra.com.br (Postfix) with ESMTP id 6AE31AC5B2 for ; Mon, 2 Sep 2002 19:32:07 -0300 (EST) Message-ID: <006401c252d0$9d23a7b0$0401010a@xt> From: =?iso-8859-1?Q?Vinicius_Jos=E9_Fortuna?= To: References: Subject: [obm-l] =?iso-8859-1?Q?Re:_=5Bobm-l=5D_N=FAmeros_Complexos?= Date: Mon, 2 Sep 2002 19:32:25 -0300 MIME-Version: 1.0 Content-Type: text/plain; charset="iso-8859-1" Content-Transfer-Encoding: 8bit X-Priority: 3 X-MSMail-Priority: Normal X-Mailer: Microsoft Outlook Express 6.00.2600.0000 X-MimeOLE: Produced By Microsoft MimeOLE V6.00.2600.0000 Sender: owner-obm-l@sucuri.mat.puc-rio.br Precedence: bulk Reply-To: obm-l@mat.puc-rio.br ----- Original Message ----- From: "Tonik" Subject: Re: [obm-l] Números Complexos > >1) Obtenha o argumento de sen 40º + i cos 40º > > obviamente, 40º Não seria 50 graus? Ângulos em graus: sen 40 + i cos 40 = cos(90-40) + i sen(90-40) = cos 50 + i sen 50 Logo, 50 graus. Até mais Vinicius Fortuna ========================================================================= Instruções para entrar na lista, sair da lista e usar a lista em http://www.mat.puc-rio.br/~nicolau/olimp/obm-l.html O administrador desta lista é ========================================================================= From owner-obm-l@sucuri.mat.puc-rio.br Mon Sep 2 20:21:20 2002 Return-Path: Received: (from majordom@localhost) by sucuri.mat.puc-rio.br (8.9.3/8.9.3) id UAA22276 for obm-l-MTTP; Mon, 2 Sep 2002 20:20:40 -0300 Received: from gorgo.centroin.com.br (gorgo.centroin.com.br [200.225.63.128]) by sucuri.mat.puc-rio.br (8.9.3/8.9.3) with ESMTP id UAA22271 for ; Mon, 2 Sep 2002 20:20:38 -0300 Received: from centroin.com.br (du13c.rjo.centroin.com.br [200.225.58.13]) (authenticated bits=0) by gorgo.centroin.com.br (8.12.2/8.12.1) with ESMTP id g82NLXCR029960 for ; Mon, 2 Sep 2002 20:21:35 -0300 (BRT) Message-ID: <3D73F310.5090903@centroin.com.br> Date: Mon, 02 Sep 2002 20:24:00 -0300 From: Augusto =?ISO-8859-1?Q?C=E9sar?= Morgado User-Agent: Mozilla/5.0 (Windows; U; Win98; en-US; rv:0.9.4.1) Gecko/20020508 Netscape6/6.2.3 X-Accept-Language: en-us MIME-Version: 1.0 To: obm-l@mat.puc-rio.br Subject: [Fwd: Re: [obm-l] =?ISO-8859-1?Q?N=FAmeros?= Complexos] Content-Type: multipart/alternative; boundary="------------010207020005060601010000" Sender: owner-obm-l@sucuri.mat.puc-rio.br Precedence: bulk Reply-To: obm-l@mat.puc-rio.br --------------010207020005060601010000 Content-Type: text/plain; charset=ISO-8859-1; format=flowed Content-Transfer-Encoding: 8bit 5) Representando, no plano, as raizes complexas da equacao z^3 + 8 = 0, >obtem-se um triangulo. Calcule a area desse triangulo. z^3 = -8 modulo de z = 2 As imagens das raizes da equaçao sao vertices de um triangulo equilatero inscrito num circulo de centro na origem e raio 2. O lado vale 2raiz de3 e a area vale 3raiz de 3. 6) (x+yi)^2 = x-yi x^2-y^2 +2xyi = x-yi x^2-y^2 = x e 2xy = -y A segunda equaçao dah y=0 ou x = -(1/2) Substituindo na primeira, x=0 ou x=1 no primeiro caso, y = (+-) [raiz de3]/2 no segundo. Ha quatro soluçoes: 0 ; 1 ; - 1/2 + (sqrt3)/2 ; - 1/2 - (sqrt3)/2 Desde quando 0 nao eh complexo? Morgado -------- Original Message -------- From: - Mon Sep 02 20:06:02 2002 X-UIDL: F5;!!GlU!!\?e"!I:m!! X-Mozilla-Status: 0001 X-Mozilla-Status2: 00000000 Return-Path: Received: from sucuri.mat.puc-rio.br (sucuri.mat.puc-rio.br [139.82.27.7]) by trex.centroin.com.br (8.12.5/8.12.1) with ESMTP id g82LZC4B009660 for ; Mon, 2 Sep 2002 18:35:12 -0300 (BRT) Received: (from majordom@localhost) by sucuri.mat.puc-rio.br (8.9.3/8.9.3) id SAA20705 for obm-l-MTTP; Mon, 2 Sep 2002 18:31:44 -0300 Received: from smtp.ieg.com.br (stone.protocoloweb.com.br [200.226.139.11]) by sucuri.mat.puc-rio.br (8.9.3/8.9.3) with ESMTP id SAA20701 for ; Mon, 2 Sep 2002 18:31:42 -0300 Received: from localhost (wh1stler@200-158-118-125.dsl.telesp.net.br [200.158.118.125]) by smtp.ieg.com.br (IeG relay/8.9.3) with SMTP id g82LSDfE067536 for ; Mon, 2 Sep 2002 18:28:13 -0300 (BRT) From: Tonik To: obm-l@mat.puc-rio.br Date: Mon, 02 Sep 2002 18:31:23 -0300 X-Priority: 3 (Normal) Organization: Tonik In-Reply-To: <003201c2529c$f2084800$0200a8c0@dois> Message-Id: Subject: Re: [obm-l] Números Complexos MIME-Version: 1.0 Content-Type: text/plain; charset="iso-8859-1" X-Mailer: Opera 6.04 build 1135 Sender: owner-obm-l@sucuri.mat.puc-rio.br Precedence: bulk Reply-To: obm-l@mat.puc-rio.br X-UIDL: F5;!!GlU!!\?e"!I:m!! Status: U 02/09/02 13:22:18, Gabriel Pérgola wrote: >E aí pessoal, > >Gostaria de ver a resolução destes problemas de números complexos que não >consegui fazer: Sao exercicios simples, q vou fazer mais para me exercitar, pois sao mais trabalhosos do q desafiantes... cheque as contas! >1) Obtenha o argumento de sen 40º + i cos 40º obviamente, 40º >2) Determine o menor valor inteiro e positivo de n para o qual (1 + i >sqrt[3])^n é um numero real para (1 + i*sqrt(3))^n ser real, seu argumento devera ser 0º ou 180º ou k180º, k E Z, passando (1+isqrt(3)) para a forma trigonometrica, temos: modulo = sqrt(1^2 + sqrt(3)^2) = sqrt(1+3) = 2 argumento = arccos(1/2) = 60º entao temos (2*(cos60º+isen60º))^n = = 2^n*(cos(60º*n)+isen(60º*n) para que o argumento (60º*n) de 0º ou 180º com n>0, n E Z: 60*n=360º, n=6 60*n=180º, n=3 Logo a resposta eh 3. >3) Determine o menor valor inteiro e positivo de n para o qual (1 + i >sqrt[3])^n é um numero real positivo. a mesma coisa, só que agora 180º nao serve (pois eh real negativo) 60*n=360, n=6 >4) Obtenha as raizes complexas das equacoes: > a) x^5 = 1 > b) x^6 = 1 x^5 = 1 x= raizquintupla(1*(cos0+isen0)) x= cos(0/5 + 360k/5) + isen(0/5 + 360k/5), 0<=k<5, k E Z as raizes: x= cos0º+isen0º = 1 (nao eh complexa) x= cos72º+isen72º x= cos144º+isen144º x= cos216º+isen216º x= cos288º+isen288º >5) Representando, no plano, as raizes complexas da equacao z^3 + 8 = 0, >obtem-se um triangulo. Calcule a area desse triangulo. z^3=-8 tem 3 raizes, por 3 ser impar, uma das raizes eh real z= raizcubica(-8) z= raizcubica( 8*(cos180º+isen180º) ) z= 2*(cos(180/3+360k/3)+isen(180/3+360k/3)), 0<=k<3 z= 2*(cos(60+120k)+isen(60+120k)) as raizes: k=0, z=2*(cos60º+isen60º) = 2*(1/2 + i*sqrt(3)/2) = 1+i*sqrt(3) k=1, z=2*(cos180º+isen180º) = 2*(-1 + i*0) = -2 k=2, z=2*(cos300º+isen300º) = (sabe-se que 1+isqrt(3) eh raiz, entao seu conjugado, 1-isqrt(3) eh a terceira raiz) entao temos os pontos do triangulo ABC no plano complexo: A(1, sqrt(3)) B(-2, 0) C(1, -sqrt(3)) Seja D a matriz: |Ax Ay 1| |Bx By 1| |Cx Cy 1| Area = modulo do determinante de D sobre 2 Area = |sqrt(3)+2sqrt(3)-(-sqrt(3)-2sqrt(3))|/2 Area = 3sqrt(3) >6) A quantidade de numeros complexos que tem o seu quadrado igual ao seu >conjugado é? Seja z um numero complexo, vc quer a qtde de nº complexos que z^2 = conjugado de z pela forma trigonometrica, seja m o modulo e a o argumento: m^2*(cos(2a)+isen(2a)) = m*(cos(a)-isen(a)) sabemos que -sen(x) = sen(-x) m^2*(cos(2a)+isen(2a)) = m*(cos(a)+isen(-a)) sabemos que m eh diferente de zero, pois senao o numero nao sera complexo. dividimos ambos os lados por m. m*(cos(2a)+isen(2a)) = (cos(a)+isen(-a)) pela identidade: (1) mcos2a = cosa (2) msen2a = sen-a (1) 2m*cos^2(a) - m = cosa (2) 2m*cosa*sena = -sena sabemos que sena nao eh zero, pois senao o numero nao sera complexo. (1) m = cosa / (2cos^2(a) - 1) (2) 2mcosa = -1 (2) cosa = -1/2m (1) m = (-1 / 2m) / ( 2*(1/4m^2) - 1) (1) m = (-1 / 2m) / ( 1/(2m^2) - 1) (1) m = (-1 / 2m) / ((1-2m^2)/(2m^2)) os extremos pelos meios (1) m = (-1 * 2m^2) / (2m * (1-2m^2)) (1) m = -2m^2 / (2m - 4m^3)) (1) 2m^2 -4m^4 = -2m^2 (1) -4m^4 + 4m^2 = 0 (1) m^4 -m^2 = 0 y = m^2 (1) y^2-y=0 (1) y(y-1) = 0 (1) ou y=0 --> m^2=0 --> m=0 o que nao devemos levar em conta no exercicio (1) ou y=1 --> m^2=1 --> m=1 ou m=-1 para m=1: (2) cosa = -1/2 (2) a = 120º formando o numero complexo: z = 1*(cos120º+isen120º) z = -1/2 + i*sqrt(3)/ 2 para m=-1: (2) cosa = 1/2 (2) a = 60º formando: z = -1*(cos60º+isen60º) z = 1*(cos240º+isen240º) z = -1/2 - i*sqrt(3)/2 Entao temos dois numeros complexos z tal que z^2 = conjugado de z e eles sao cos60º+isen60º e cos240º+isen240º ufa, quanta conta, acho melhor tirar a prova... z^2 = conjugado de z (isqrt(3)/2 - 1/2)^2 = -1/2 -isqrt(3)/2 -3/4 - isqrt(3)/2 +1/4 = -1/2 -isqrt(3)/2 -1/2 - isqrt(3)/2 = -1/2 -isqrt(3)/2 (verdade) z^2 = conjugado de z (-1/2 -isqrt(3)/2)^2 = -1/2 +isqrt(3)/2 1/4 + isqrt(3)/2 - 3/4 = -1/2 +isqrt(3)/2 -1/2 + isqrt(3)/2 = -1/2 +isqrt(3)/2 (verdade) >É isso! Agradeço qualquer ajuda. > >Gabriel Pérgola ========================================================================= Instruções para entrar na lista, sair da lista e usar a lista em http://www.mat.puc-rio.br/~nicolau/olimp/obm-l.html O administrador desta lista é ========================================================================= --------------010207020005060601010000 Content-Type: text/html; charset=us-ascii Content-Transfer-Encoding: 7bit
5) Representando, no plano, as raizes complexas da equacao z^3 + 8 = 0,
>obtem-se um triangulo. Calcule a area desse triangulo.
z^3 = -8
modulo de z = 2
As imagens das  raizes da equaçao sao vertices de um triangulo equilatero inscrito num circulo de centro na origem e raio 2. O lado vale 2raiz de3 e a area vale 3raiz de 3.

6) (x+yi)^2 = x-yi
x^2-y^2 +2xyi = x-yi
x^2-y^2 = x  e  2xy = -y
A segunda equaçao dah  y=0 ou  x = -(1/2)
Substituindo na primeira, x=0 ou x=1 no primeiro caso,  y = (+-) [raiz de3]/2 no segundo.
Ha quatro soluçoes:  0 ;     1 ;     - 1/2 + (sqrt3)/2   ;   - 1/2 - (sqrt3)/2

Desde quando 0 nao eh complexo?
Morgado

-------- Original Message --------
From: - Mon Sep 02 20:06:02 2002
X-UIDL: F5;!!GlU!!\?e"!I:m!!
X-Mozilla-Status: 0001
X-Mozilla-Status2: 00000000
Return-Path: <owner-obm-l@sucuri.mat.puc-rio.br>
Received: from sucuri.mat.puc-rio.br (sucuri.mat.puc-rio.br [139.82.27.7]) by trex.centroin.com.br (8.12.5/8.12.1) with ESMTP id g82LZC4B009660 for <morgado@centroin.com.br>; Mon, 2 Sep 2002 18:35:12 -0300 (BRT)
Received: (from majordom@localhost) by sucuri.mat.puc-rio.br (8.9.3/8.9.3) id SAA20705 for obm-l-MTTP; Mon, 2 Sep 2002 18:31:44 -0300
Received: from smtp.ieg.com.br (stone.protocoloweb.com.br [200.226.139.11]) by sucuri.mat.puc-rio.br (8.9.3/8.9.3) with ESMTP id SAA20701 for <obm-l@mat.puc-rio.br>; Mon, 2 Sep 2002 18:31:42 -0300
Received: from localhost (wh1stler@200-158-118-125.dsl.telesp.net.br [200.158.118.125]) by smtp.ieg.com.br (IeG relay/8.9.3) with SMTP id g82LSDfE067536 for <obm-l@mat.puc-rio.br>; Mon, 2 Sep 2002 18:28:13 -0300 (BRT)
From: Tonik <tonik@ieg.com.br>
To: obm-l@mat.puc-rio.br
Date: Mon, 02 Sep 2002 18:31:23 -0300
X-Priority: 3 (Normal)
Organization: Tonik
In-Reply-To: <003201c2529c$f2084800$0200a8c0@dois>
Message-Id: <BA4WNB9ED86BAVRE0VQLYSC7HC7RM.3d73d8ab@localhost>
Subject: Re: [obm-l] Números Complexos
MIME-Version: 1.0
Content-Type: text/plain; charset="iso-8859-1"
X-Mailer: Opera 6.04 build 1135
Sender: owner-obm-l@sucuri.mat.puc-rio.br
Precedence: bulk
Reply-To: obm-l@mat.puc-rio.br
X-UIDL: F5;!!GlU!!\?e"!I:m!!
Status: U


02/09/02 13:22:18, Gabriel Pérgola  wrote:

>E aí pessoal,
>
>Gostaria de ver a resolução destes problemas de números complexos que não
>consegui fazer:

Sao exercicios simples, q vou fazer mais para me exercitar, pois sao mais 
trabalhosos do q desafiantes... cheque as contas!

>1) Obtenha o argumento de sen 40º + i cos 40º

obviamente, 40º

>2) Determine o menor valor inteiro e positivo de n para o qual (1 + i
>sqrt[3])^n é um numero real

para (1 + i*sqrt(3))^n ser real, seu argumento devera ser 0º ou 180º
ou k180º, k E Z, passando (1+isqrt(3)) para a forma trigonometrica, 
temos:
modulo = sqrt(1^2 + sqrt(3)^2) = sqrt(1+3) = 2
argumento = arccos(1/2) = 60º

entao temos (2*(cos60º+isen60º))^n =
= 2^n*(cos(60º*n)+isen(60º*n)
para que o argumento (60º*n) de 0º ou 180º com n>0, n E Z:

60*n=360º, n=6
6
0*n=180º, n=3

Logo a resposta eh 3.

>3) Determine o menor valor inteiro e positivo de n para o qual (1 + i
>sqrt[3])^n é um numero real positivo.

a mesma coisa, só que agora 180º nao serve (pois eh real negativo)
60*n=360, n=6

>4) Obtenha as raizes complexas das equacoes:
>    a) x^5 = 1
>    b) x^6 = 1

x^5 = 1
x= raizquintupla(1*(cos0+isen0))
x= cos(0/5 + 360k/5) + isen(0/5 + 360k/5), 0<=k<5, k E Z

as raizes:
x= cos0º+isen0º = 1 (nao eh complexa)
x= cos72º+isen72º
x= cos144º+isen144º
x= cos216º+isen216º
x= cos288º+isen288º

>5) Representando, no plano, as raizes complexas da equacao z^3 + 8 = 0,
>obtem-se um triangulo. Calcule a area desse triangulo.

z^3=-8 tem 3 raizes, por 3 ser impar, uma das raizes eh real
z= raizcubica(-8)
z= raizcubica( 8*(cos180º+isen180º) )
z= 2*(cos(180/3+360k/3)+isen(180/3+360k/3)), 0<=k<3
z= 2*(cos(60+120k)+isen(60+120k))
as 
raizes:
k=0, z=2*(cos60º+isen60º) = 2*(1/2 + i*sqrt(3)/2) = 1+i*sqrt(3)
k=1, z=2*(cos180º+isen180º) = 2*(-1 + i*0) = -2
k=2, z=2*(cos300º+isen300º) =
(sabe-se que 1+isqrt(3) eh raiz, entao seu conjugado, 1-isqrt(3) eh a 
terceira raiz)

entao temos os pontos do triangulo ABC no plano complexo:
A(1,  sqrt(3))
B(-2, 0)
C(1,  -sqrt(3))

Seja D a matriz:
|Ax Ay 1| 
|Bx By 1|
|Cx Cy 1|

Area = modulo do determinante de D sobre 2
Area = |sqrt(3)+2sqrt(3)-(-sqrt(3)-2sqrt(3))|/2
Area = 3sqrt(3)

>6) A quantidade de numeros complexos que tem o seu quadrado igual ao seu
>conjugado é?

Seja z um numero complexo, vc quer a qtde de nº complexos que
z^2 = conjugado de z

pela forma trigonometrica, seja m o modulo e a o argumento:

m^2*(cos(2a)+isen(2a)) = m*(cos(a)-isen(a))
sabemos que -sen(x) = sen(-x)
m^2*(cos(2a)+isen(2a)) = m*(cos(a)+isen(-a))
sabemos que m eh diferente de zero, pois senao o numero nao sera complexo. 
dividimos ambos os l
ados por m.
m*(cos(2a)+isen(2a)) = (cos(a)+isen(-a))
pela identidade:
(1) mcos2a = cosa
(2) msen2a = sen-a

(1) 2m*cos^2(a) - m = cosa
(2) 2m*cosa*sena = -sena

sabemos que sena nao eh zero, pois senao o numero nao sera complexo.

(1) m = cosa / (2cos^2(a) - 1)
(2) 2mcosa = -1
(2) cosa = -1/2m
(1) m = (-1 / 2m) / ( 2*(1/4m^2) - 1)
(1) m = (-1 / 2m) / ( 1/(2m^2) - 1)
(1) m = (-1 / 2m) / ((1-2m^2)/(2m^2))
os extremos pelos meios
(1) m = (-1 * 2m^2) / (2m * (1-2m^2))
(1) m = -2m^2 / (2m - 4m^3))
(1) 2m^2 -4m^4 = -2m^2
(1) -4m^4 + 4m^2 = 0
(1) m^4 -m^2 = 0
y = m^2
(1) y^2-y=0
(1) y(y-1) = 0
(1) ou y=0 --> m^2=0 --> m=0 o que nao devemos levar em conta no exercicio
(1) ou y=1 --> m^2=1 --> m=1 ou m=-1

para m=1:
(2) cosa = -1/2
(2) a = 120º
formando o numero complexo:
z = 1*(cos120º+isen120º)
z = -1/2 + i*sqrt(3)/ 2

para m=-1:
(2) cosa = 1/2
(2) a = 60º
formando:
z = -1*(cos60º+isen60º)
z = 1*(cos240º+isen240º)
z = -1/2 - i*sqrt
(3)/2

Entao temos dois numeros complexos z tal que z^2 = conjugado de z 
e eles sao cos60º+isen60º e cos240º+isen240º

ufa, quanta conta, acho melhor tirar a prova...
z^2 = conjugado de z
(isqrt(3)/2 - 1/2)^2 = -1/2 -isqrt(3)/2
-3/4 - isqrt(3)/2 +1/4 = -1/2 -isqrt(3)/2
-1/2 - isqrt(3)/2 = -1/2 -isqrt(3)/2 (verdade)

z^2 = conjugado de z
(-1/2 -isqrt(3)/2)^2 = -1/2 +isqrt(3)/2
1/4 + isqrt(3)/2 - 3/4 = -1/2 +isqrt(3)/2
-1/2 + isqrt(3)/2 = -1/2 +isqrt(3)/2 (verdade)

>É isso! Agradeço qualquer ajuda.
>
>Gabriel Pérgola


=========================================================================
Instruções para entrar na lista, sair da lista e usar a lista em
http://www.mat.puc-rio.br/~nicolau/olimp/obm-l.html
O administrador desta lista é 
================================================
=========================


--------------010207020005060601010000-- ========================================================================= Instruções para entrar na lista, sair da lista e usar a lista em http://www.mat.puc-rio.br/~nicolau/olimp/obm-l.html O administrador desta lista é ========================================================================= From owner-obm-l@sucuri.mat.puc-rio.br Mon Sep 2 22:05:20 2002 Return-Path: Received: (from majordom@localhost) by sucuri.mat.puc-rio.br (8.9.3/8.9.3) id WAA23622 for obm-l-MTTP; Mon, 2 Sep 2002 22:05:07 -0300 Received: from smtp.estaminas.com.br (smtp.estaminas.com.br [200.188.191.51]) by sucuri.mat.puc-rio.br (8.9.3/8.9.3) with ESMTP id WAA23618 for ; Mon, 2 Sep 2002 22:05:04 -0300 Received: from uai.com.br (pop.uai.com.br [200.188.191.2]) by smtp.estaminas.com.br (8.11.0/8.11.0) with ESMTP id g831D9b05109 for ; Mon, 2 Sep 2002 22:13:09 -0300 Received: from mesantos (dial-tm-mp-200-165-28-170.estaminas.com.br [200.165.28.170]) by uai.com.br (8.12.4/8.12.4) with SMTP id g831D91Q012894 for ; Mon, 2 Sep 2002 22:13:09 -0300 (BRT) Message-ID: <001801c865f8$d4de6aa0$aa1ca5c8@mesantos> From: "Marcos Eike Tinen dos Santos" To: References: <001e01c252b7$ce50f4a0$2101a8c0@u2z7z2> Subject: [obm-l] =?iso-8859-1?Q?Re:_=5Bobm-l=5D_O_problema_das_infinitas_solu=E7=F5es?= Date: Sat, 2 Feb 2008 22:07:56 -0200 MIME-Version: 1.0 Content-Type: text/plain; charset="iso-8859-1" Content-Transfer-Encoding: 8bit X-Priority: 3 X-MSMail-Priority: Normal X-Mailer: Microsoft Outlook Express 5.50.4133.2400 X-MimeOLE: Produced By Microsoft MimeOLE V5.50.4133.2400 Sender: owner-obm-l@sucuri.mat.puc-rio.br Precedence: bulk Reply-To: obm-l@mat.puc-rio.br Eu vou prestar vestibular neste ano, porém estou na dúvida em qual curso. Minhas dúvidas são: devo prestar para ciencia da computação ou para matematica computacional? Meus objetivos é fazer uma pós de computação gráfica. Ats, Marcos Eike ========================================================================= Instruções para entrar na lista, sair da lista e usar a lista em http://www.mat.puc-rio.br/~nicolau/olimp/obm-l.html O administrador desta lista é ========================================================================= From owner-obm-l@sucuri.mat.puc-rio.br Mon Sep 2 23:40:13 2002 Return-Path: Received: (from majordom@localhost) by sucuri.mat.puc-rio.br (8.9.3/8.9.3) id XAA24863 for obm-l-MTTP; Mon, 2 Sep 2002 23:39:55 -0300 Received: from shannon.bol.com.br (shannon.bol.com.br [200.221.24.13]) by sucuri.mat.puc-rio.br (8.9.3/8.9.3) with ESMTP id XAA24859 for ; Mon, 2 Sep 2002 23:39:52 -0300 Received: from bol.com.br (200.221.24.138) by shannon.bol.com.br (5.1.071) id 3D60E4DC0040940B for obm-l@mat.puc-rio.br; Mon, 2 Sep 2002 23:40:09 -0300 Date: Mon, 2 Sep 2002 23:38:57 -0300 Message-Id: Subject: [obm-l] circuito IME MIME-Version: 1.0 Content-Type: text/plain;charset="iso-8859-1" From: "rafaelc.l" To: obm-l@mat.puc-rio.br X-XaM3-API-Version: 2.4.3.4.4 X-SenderIP: 200.176.166.139 Content-Transfer-Encoding: 8bit X-MIME-Autoconverted: from quoted-printable to 8bit by sucuri.mat.puc-rio.br id XAA24860 Sender: owner-obm-l@sucuri.mat.puc-rio.br Precedence: bulk Reply-To: obm-l@mat.puc-rio.br Olá, gostaria de uma luz pra resolver esse problema do IME. http://www.ime.eb.br/~sd3/vestibular/provas20012002/fisica /fis7.htm Obrigado OBS: Não quero solução com cálculo integral, já que não consta no programa do IME. __________________________________________________________________________ AcessoBOL, só R$ 9,90! O menor preço do mercado! Assine já! http://www.bol.com.br/acessobol ========================================================================= Instruções para entrar na lista, sair da lista e usar a lista em http://www.mat.puc-rio.br/~nicolau/olimp/obm-l.html O administrador desta lista é ========================================================================= From owner-obm-l@sucuri.mat.puc-rio.br Tue Sep 3 09:32:47 2002 Return-Path: Received: (from majordom@localhost) by sucuri.mat.puc-rio.br (8.9.3/8.9.3) id JAA29025 for obm-l-MTTP; Tue, 3 Sep 2002 09:31:53 -0300 Received: from smtp.ieg.com.br (sharon.protocoloweb.com.br [200.226.139.12]) by sucuri.mat.puc-rio.br (8.9.3/8.9.3) with ESMTP id JAA29021 for ; Tue, 3 Sep 2002 09:31:51 -0300 Received: from localhost (200-158-118-125.dsl.telesp.net.br [200.158.118.125]) by smtp.ieg.com.br (IeG relay/8.9.3) with SMTP id g83CY2fx065699 for ; Tue, 3 Sep 2002 09:34:02 -0300 (BRT) From: Tonik To: obm-l@mat.puc-rio.br Date: Tue, 03 Sep 2002 09:31:37 -0300 X-Priority: 3 (Normal) Organization: Tonik In-Reply-To: <006401c252d0$9d23a7b0$0401010a@xt> Message-Id: Subject: =?windows-1252?Q?Re:=20[obm-l]=20=20N=FAmeros=20Complexos?= MIME-Version: 1.0 Content-Type: text/plain; charset="iso-8859-1" X-Mailer: Opera 6.04 build 1135 Sender: owner-obm-l@sucuri.mat.puc-rio.br Precedence: bulk Reply-To: obm-l@mat.puc-rio.br >----- Original Message ----- >From: "Tonik" >> >1) Obtenha o argumento de sen 40º + i cos 40º >> obviamente, 40º > >Não seria 50 graus? >Ângulos em graus: >sen 40 + i cos 40 = cos(90-40) + i sen(90-40) = cos 50 + i sen 50 >Logo, 50 graus. >Até mais >Vinicius Fortuna Muito bem notado, sabe aqueles exercicios que de tao simples lhe enganam por ilusao? entao, esse eh um deles para mim >5) Representando, no plano, as raizes complexas da equacao z^3 + 8 = 0, >obtem-se um triangulo. Calcule a area desse triangulo. >z^3 = -8 >modulo de z = 2 >As imagens das raizes da equaçao sao vertices de um triangulo equilatero >inscrito num circulo de centro na origem e raio 2. O lado vale 2raiz >de3 e a area vale 3raiz de 3. eh, uma forma facil de resolver :) sabendo-se do raio da circunferencia acha-se o volume do poligono inscrito, sendo este sempre regular pois originou das raizes de um numero complexo >6) (x+yi)^2 = x-yi >x^2-y^2 +2xyi = x-yi >x^2-y^2 = x e 2xy = -y >A segunda equaçao dah y=0 ou x = -(1/2) >Substituindo na primeira, x=0 ou x=1 no primeiro caso, y = (+-) [raiz de3]/2 no segundo. >Ha quatro soluçoes: 0 ; 1 ; - 1/2 + (sqrt3)/2 ; - 1/2 - (sqrt3)/2 Eu tinha comecado a resolver pela forma polar da equacao e comecei a notar que a forma algebrica teria chegado ao mesmo sistema muito mais rapidamente e sem funcoes trigonometricas.. mas como ja tava la, iria seguir até o final >Desde quando 0 nao eh complexo? >Morgado Bom, 0 e 1 pertencem aos complexos, mas quando se pergunta "quantas solucoes complexas tem certa equacao", acho que vc nao deve contar as solucoes que pertencem aos reais, ou eu estou errado? Alguem me corrija? Obrigado pelas observacoes. Tonik. ========================================================================= Instruções para entrar na lista, sair da lista e usar a lista em http://www.mat.puc-rio.br/~nicolau/olimp/obm-l.html O administrador desta lista é ========================================================================= From owner-obm-l@sucuri.mat.puc-rio.br Tue Sep 3 10:35:06 2002 Return-Path: Received: (from majordom@localhost) by sucuri.mat.puc-rio.br (8.9.3/8.9.3) id KAA30182 for obm-l-MTTP; Tue, 3 Sep 2002 10:34:23 -0300 Received: from gorgo.centroin.com.br (gorgo.centroin.com.br [200.225.63.128]) by sucuri.mat.puc-rio.br (8.9.3/8.9.3) with ESMTP id KAA30177 for ; Tue, 3 Sep 2002 10:34:21 -0300 Received: from centroin.com.br (du150c.rjo.centroin.com.br [200.225.58.150]) (authenticated bits=0) by gorgo.centroin.com.br (8.12.2/8.12.1) with ESMTP id g83DZJCR010064 for ; Tue, 3 Sep 2002 10:35:21 -0300 (BRT) Message-ID: <3D74BB29.1080607@centroin.com.br> Date: Tue, 03 Sep 2002 10:37:45 -0300 From: Augusto =?ISO-8859-1?Q?C=E9sar?= Morgado User-Agent: Mozilla/5.0 (Windows; U; Win98; en-US; rv:0.9.4.1) Gecko/20020508 Netscape6/6.2.3 X-Accept-Language: en-us MIME-Version: 1.0 To: obm-l@mat.puc-rio.br Subject: Re: [obm-l] Re: [obm-l] O problema das infinitas =?ISO-8859-1?Q?solu=E7=F5es?= References: <001e01c252b7$ce50f4a0$2101a8c0@u2z7z2> <001801c865f8$d4de6aa0$aa1ca5c8@mesantos> Content-Type: text/plain; charset=ISO-8859-1; format=flowed Content-Transfer-Encoding: 8bit Sender: owner-obm-l@sucuri.mat.puc-rio.br Precedence: bulk Reply-To: obm-l@mat.puc-rio.br Marcos, mensagens como a sua deveriam sempre vir acompanhadas da cidade do autor. Morgado Marcos Eike Tinen dos Santos wrote: >Eu vou prestar vestibular neste ano, porém estou na dúvida em qual curso. >Minhas dúvidas são: devo prestar para ciencia da computação ou para >matematica computacional? Meus objetivos é fazer uma pós de computação >gráfica. > >Ats, >Marcos Eike > > >========================================================================= >Instruções para entrar na lista, sair da lista e usar a lista em >http://www.mat.puc-rio.br/~nicolau/olimp/obm-l.html >O administrador desta lista é >========================================================================= > > ========================================================================= Instruções para entrar na lista, sair da lista e usar a lista em http://www.mat.puc-rio.br/~nicolau/olimp/obm-l.html O administrador desta lista é ========================================================================= From owner-obm-l@sucuri.mat.puc-rio.br Tue Sep 3 13:05:57 2002 Return-Path: Received: (from majordom@localhost) by sucuri.mat.puc-rio.br (8.9.3/8.9.3) id NAA00426 for obm-l-MTTP; Tue, 3 Sep 2002 13:02:40 -0300 Received: from smtp.estaminas.com.br (smtp.estaminas.com.br [200.188.191.51]) by sucuri.mat.puc-rio.br (8.9.3/8.9.3) with ESMTP id NAA00421 for ; Tue, 3 Sep 2002 13:02:37 -0300 Received: from uai.com.br (pop.uai.com.br [200.188.191.2]) by smtp.estaminas.com.br (8.11.0/8.11.0) with ESMTP id g83GAcb25189 for ; Tue, 3 Sep 2002 13:10:39 -0300 Received: from mesantos (dial-tm-mp-200-149-114-194.estaminas.com.br [200.149.114.194]) by uai.com.br (8.12.4/8.12.4) with SMTP id g83GAP1U015599 for ; Tue, 3 Sep 2002 13:10:35 -0300 (BRT) Message-ID: <008201c86676$367572e0$c27295c8@mesantos> From: "Marcos Eike Tinen dos Santos" To: References: <001e01c252b7$ce50f4a0$2101a8c0@u2z7z2> <001801c865f8$d4de6aa0$aa1ca5c8@mesantos> <3D74BB29.1080607@centroin.com.br> Subject: [obm-l] =?iso-8859-1?Q?Re:_=5Bobm-l=5D_Re:_=5Bobm-l=5D_O_problema_das_infinitas?= =?iso-8859-1?Q?_solu=E7=F5es?= Date: Sun, 3 Feb 2008 12:58:15 -0200 MIME-Version: 1.0 Content-Type: text/plain; charset="iso-8859-1" Content-Transfer-Encoding: 8bit X-Priority: 3 X-MSMail-Priority: Normal X-Mailer: Microsoft Outlook Express 5.50.4133.2400 X-MimeOLE: Produced By Microsoft MimeOLE V5.50.4133.2400 Sender: owner-obm-l@sucuri.mat.puc-rio.br Precedence: bulk Reply-To: obm-l@mat.puc-rio.br Morgado, eu moro em BH. Caso ajude a resolver meu problema profissional. Obrigado! Ats, Marcos Eike ----- Original Message ----- From: "Augusto César Morgado" To: Sent: Tuesday, September 03, 2002 11:37 AM Subject: Re: [obm-l] Re: [obm-l] O problema das infinitas soluções > Marcos, mensagens como a sua deveriam sempre vir acompanhadas da cidade > do autor. > Morgado > > Marcos Eike Tinen dos Santos wrote: > > >Eu vou prestar vestibular neste ano, porém estou na dúvida em qual curso. > >Minhas dúvidas são: devo prestar para ciencia da computação ou para > >matematica computacional? Meus objetivos é fazer uma pós de computação > >gráfica. > > > >Ats, > >Marcos Eike > > > > > >========================================================================= > >Instruções para entrar na lista, sair da lista e usar a lista em > >http://www.mat.puc-rio.br/~nicolau/olimp/obm-l.html > >O administrador desta lista é > >========================================================================= > > > > > > > ========================================================================= > Instruções para entrar na lista, sair da lista e usar a lista em > http://www.mat.puc-rio.br/~nicolau/olimp/obm-l.html > O administrador desta lista é > ========================================================================= > ========================================================================= Instruções para entrar na lista, sair da lista e usar a lista em http://www.mat.puc-rio.br/~nicolau/olimp/obm-l.html O administrador desta lista é ========================================================================= From owner-obm-l@sucuri.mat.puc-rio.br Tue Sep 3 13:18:14 2002 Return-Path: Received: (from majordom@localhost) by sucuri.mat.puc-rio.br (8.9.3/8.9.3) id NAA00644 for obm-l-MTTP; Tue, 3 Sep 2002 13:17:35 -0300 Received: from shannon.bol.com.br (shannon.bol.com.br [200.221.24.13]) by sucuri.mat.puc-rio.br (8.9.3/8.9.3) with ESMTP id NAA00631 for ; Tue, 3 Sep 2002 13:17:33 -0300 Received: from bol.com.br (200.221.24.133) by shannon.bol.com.br (5.1.071) id 3D60E4DC0042EDFF for obm-l@mat.puc-rio.br; Tue, 3 Sep 2002 13:17:50 -0300 Date: Tue, 3 Sep 2002 13:16:21 -0300 Message-Id: Subject: Re:[obm-l] circuito IME MIME-Version: 1.0 Content-Type: multipart/mixed; boundary="_=__=_XaM3_Boundary.1031069781.2A.620901.42.19840.52.42.101010.963279460" From: "rafaelc.l" To: obm-l@mat.puc-rio.br X-XaM3-API-Version: 2.4.3.4.4 X-SenderIP: 200.132.35.120 Sender: owner-obm-l@sucuri.mat.puc-rio.br Precedence: bulk Reply-To: obm-l@mat.puc-rio.br --_=__=_XaM3_Boundary.1031069781.2A.620901.42.19840.52.42.101010.963279460 Content-Type: text/plain;charset="iso-8859-1" Content-Transfer-Encoding: quoted-printable A=ED vai o endere=E7o certinho da quest=E3o: http://www.ime.eb.br/~sd3/vestibular/provas20012002/fisic a/fis7.htm __________________________________________________________________________ AcessoBOL, s=F3 R$ 9,90! O menor pre=E7o do mercado! Assine j=E1! http://www.bol.com.br/acessobol --_=__=_XaM3_Boundary.1031069781.2A.620901.42.19840.52.42.101010.963279460 Content-Type: text/plain; name="00000D6S" Content-Transfer-Encoding: base64 ICANCiBPbOEsIGdvc3RhcmlhIGRlIHVtYSBsdXogcHJhIHJlc29sdmVyIGVzc2UgcHJvYmxl bWEgZG8gDQpJTUUuDQoNCmh0dHA6Ly93d3cuaW1lLmViLmJyL35zZDMvdmVzdGlidWxhci9w cm92YXMyMDAxMjAwMi9maXNpY2ENCi9maXM3Lmh0bQ0KICAgICAgICAgICAgICAgICAgDQpP YnJpZ2Fkbw0KDQogT0JTOiBO428gcXVlcm8gc29sdefjbyBjb20gY+FsY3VsbyBpbnRlZ3Jh bCwgauEgcXVlIG7jbyANCmNvbnN0YSBubyBwcm9ncmFtYSBkbyBJTUUuDQoNCiANCl9fX19f X19fX19fX19fX19fX19fX19fX19fX19fX19fX19fX19fX19fX19fX19fX19fX19fX19fX19f X19fX19fX19fX19fX19fDQpBY2Vzc29CT0wsIHPzIFIkIDksOTAhIE8gbWVub3IgcHJl528g ZG8gbWVyY2FkbyENCkFzc2luZSBq4SEgaHR0cDovL3d3dy5ib2wuY29tLmJyL2FjZXNzb2Jv bA0KDQoNCj09PT09PT09PT09PT09PT09PT09PT09PT09PT09PT09PT09PT09PT09PT09PT09 PT09PT09PT09PT09PT09PT09PT09PT09PT0NCkluc3Rydef1ZXMgcGFyYSBlbnRyYXIgbmEg bGlzdGEsIHNhaXIgZGEgbGlzdGEgZSB1c2FyIGEgbGlzdGEgZW0NCmh0dHA6Ly93d3cubWF0 LnB1Yy1yaW8uYnIvfm5pY29sYXUvb2xpbXAvb2JtLWwuaHRtbA0KTyBhZG1pbmlzdHJhZG9y IGRlc3RhIGxpc3RhIOkgPG5pY29sYXVAbWF0LnB1Yy1yaW8uYnI+DQo9PT09PT09PT09PT09 PT09PT09PT09PT09PT09PT09PT09PT09PT09PT09PT09PT09PT09PT09PT09PT09PT09PT09 PT09PT09DQo= --_=__=_XaM3_Boundary.1031069781.2A.620901.42.19840.52.42.101010.963279460-- ========================================================================= Instruções para entrar na lista, sair da lista e usar a lista em http://www.mat.puc-rio.br/~nicolau/olimp/obm-l.html O administrador desta lista é ========================================================================= From owner-obm-l@sucuri.mat.puc-rio.br Tue Sep 3 15:26:46 2002 Return-Path: Received: (from majordom@localhost) by sucuri.mat.puc-rio.br (8.9.3/8.9.3) id PAA03473 for obm-l-MTTP; Tue, 3 Sep 2002 15:26:07 -0300 Received: from toole.uol.com.br (toole.uol.com.br [200.221.4.26]) by sucuri.mat.puc-rio.br (8.9.3/8.9.3) with ESMTP id PAA03458 for ; Tue, 3 Sep 2002 15:25:58 -0300 Received: from xxx ([200.191.170.241]) by toole.uol.com.br (8.9.1/8.9.1) with ESMTP id PAA21864 for ; Tue, 3 Sep 2002 15:21:05 -0300 (BRT) From: "haroldo" To: Subject: [obm-l] =?iso-8859-1?Q?Gabarito_da_olimp=EDada_carioca.?= Date: Tue, 3 Sep 2002 15:27:51 -0300 Message-ID: <000301c25377$9d9386a0$f1aabfc8@xxx> MIME-Version: 1.0 Content-Type: multipart/alternative; boundary="----=_NextPart_000_0004_01C2535E.78464EA0" X-Priority: 3 (Normal) X-MSMail-Priority: Normal X-Mailer: Microsoft Outlook, Build 10.0.2627 X-MimeOLE: Produced By Microsoft MimeOLE V5.00.2919.6600 Importance: Normal Sender: owner-obm-l@sucuri.mat.puc-rio.br Precedence: bulk Reply-To: obm-l@mat.puc-rio.br This is a multi-part message in MIME format. ------=_NextPart_000_0004_01C2535E.78464EA0 Content-Type: text/plain; charset="iso-8859-1" Content-Transfer-Encoding: quoted-printable Gostaria de ver o gabarito oficial da olimp=EDada carioca, realizada na PUC, dia 31/08. Pois tenho dois alunos que participaram da competi=E7=E3o. Grato. Haroldo . ------=_NextPart_000_0004_01C2535E.78464EA0 Content-Type: text/html; charset="iso-8859-1" Content-Transfer-Encoding: quoted-printable

Gostaria de ver o gabarito oficial da olimp=EDada = carioca, realizada na PUC, dia 31/08.

Pois tenho dois alunos que participaram da = competi=E7=E3o.

Grato.

Haroldo .

------=_NextPart_000_0004_01C2535E.78464EA0-- ========================================================================= Instruções para entrar na lista, sair da lista e usar a lista em http://www.mat.puc-rio.br/~nicolau/olimp/obm-l.html O administrador desta lista é ========================================================================= From owner-obm-l@sucuri.mat.puc-rio.br Tue Sep 3 16:56:31 2002 Return-Path: Received: (from majordom@localhost) by sucuri.mat.puc-rio.br (8.9.3/8.9.3) id QAA05342 for obm-l-MTTP; Tue, 3 Sep 2002 16:55:50 -0300 Received: from home.iis.com.br (mail.iis.com.br [200.202.96.2]) by sucuri.mat.puc-rio.br (8.9.3/8.9.3) with ESMTP id QAA05338 for ; Tue, 3 Sep 2002 16:55:45 -0300 Received: from Marcio (rio-tc0-tty404.iis.com.br [200.202.99.150]) by home.iis.com.br (8.11.6/8.11.6/1.1.1.16) with SMTP id g83JuC406038 for ; Tue, 3 Sep 2002 16:56:12 -0300 Message-ID: <000d01c25384$19d56880$9663cac8@epq.ime.eb.br> From: "Marcio" To: References: <000301c25377$9d9386a0$f1aabfc8@xxx> Subject: [obm-l] =?iso-8859-1?Q?Re:_=5Bobm-l=5D_Gabarito_da_olimp=EDada_carioca.?= Date: Tue, 3 Sep 2002 16:56:16 -0300 MIME-Version: 1.0 Content-Type: multipart/alternative; boundary="----=_NextPart_000_000A_01C2536A.D15F07A0" X-Priority: 3 X-MSMail-Priority: Normal X-Mailer: Microsoft Outlook Express 5.50.4133.2400 X-MimeOLE: Produced By Microsoft MimeOLE V5.50.4133.2400 X-AntiVirus: Antivirus for sendmail by Petr Rehor Sender: owner-obm-l@sucuri.mat.puc-rio.br Precedence: bulk Reply-To: obm-l@mat.puc-rio.br This is a multi-part message in MIME format. ------=_NextPart_000_000A_01C2536A.D15F07A0 Content-Type: text/plain; charset="iso-8859-1" Content-Transfer-Encoding: quoted-printable Nao eh oficial nem completo.. Me mostraram a prova e eu pensei sobre = algumas questoes ...=20 Ainda nao pensei nas questoes 1 e 2, mas elas parecem mais faceis.. 4a Q: chata de fazer sem figura, pra entender o q eu escrevi deve ser = necessario papel.. Pra facilitar, vou mudar os nomes.. Pra mim, A_b eh o ponto que sai do = vertice A e fica oposto ao B (no contexto do problema). Equivale ao A_2 = do enunciado.. analagomante para os outros. Fiz o caso ABC acutangulo, = mas o outro deve ser bem parecido... (1) B_c, A_c, B_a, C_a sao cociclicos. (1a) Como A,B_c,B_a,C sao cociclicos, ang =3D ang. (1b) Note que A,C,C1,A1 sao cociclicos e (A1,C1,C_a,A_c) tmb, donde = ang =3Dang =3D ang.=20 (2) C_b, B_c, C_a, B_a sao cociclicos. (2a)Desenhe uma figura soh com C_1, C_a e C_b. Note que C_b, C1, C_a, C = sao cociclicos, e portanto=20 ang =3D ang =3D 90 - A e portanto ang = =3D A.=20 (2b)Analogamente, ang =3D A. Por (1b), ang =3D ang e portanto (2) vale. 3aQ:=20 a) Prove por inducao q (a1,a2,a3,a4) =3D (0,1,1,0) mod 2, onde os = indices sao olhados modulo 4.Em particular, como 2001 =3D 1 mod 4, a1 eh = par. b) Prove por inducao q (a1, a2, a3, ..., a12, a13) =3D = (0,1,1,2,1,1,1,0,2,0,2,1,0) mod 3 (indices olhados mod 13), e como = 2003=3D13mod13, a2002 eh multiplo de 3. c) Sim. Comece com (499,1,0) por exemplo. d) Nao. Um modo de explicar eh q comecando com 0,1,1 o 15o termo ja da = 1705. Com 1,1,1 ja passa de 2002, e entao eh soh testar com termos = iniciais menores (claro que aumentar um termo inicial soh aumenta a15).. = Uma maneira diferente eh vc notar que comecar com (a,b,c) equivale a = comecar com (1,0,0); (0,1,0) e (0,0,1) e multiplicar o a_15 por a,b,c = respectivamente. Fica algo como a15 =3D 504a + 778b + 927c, e ai vc = testa a15=3D2002 como na outra solucao. A proposito, essa prova pareceu bem mais dificil do que a do ano = passado.. Como q o pessoal que fez a prova foi? O que acharam?=20 []'s Marcio ----- Original Message -----=20 From: haroldo=20 To: obm-l@mat.puc-rio.br=20 Sent: Tuesday, September 03, 2002 3:27 PM Subject: [obm-l] Gabarito da olimp=EDada carioca. Gostaria de ver o gabarito oficial da olimp=EDada carioca, realizada = na PUC, dia 31/08. Pois tenho dois alunos que participaram da competi=E7=E3o. Grato. Haroldo . ------=_NextPart_000_000A_01C2536A.D15F07A0 Content-Type: text/html; charset="iso-8859-1" Content-Transfer-Encoding: quoted-printable
Nao eh oficial nem completo.. Me = mostraram a prova=20 e eu pensei sobre algumas questoes ...
 
Ainda nao pensei nas questoes 1 e 2, = mas elas=20 parecem mais faceis..
4a Q: chata de fazer sem figura, pra = entender o q=20 eu escrevi deve ser necessario papel..
Pra facilitar, vou mudar os nomes.. Pra = mim, A_b eh=20 o ponto que sai do vertice A e fica oposto ao B (no contexto do = problema).=20 Equivale ao A_2 do enunciado.. analagomante para os outros. Fiz o caso = ABC=20 acutangulo, mas o outro deve ser bem parecido...
 
(1) B_c, A_c, B_a, C_a sao = cociclicos.
(1a) Como A,B_c,B_a,C sao cociclicos,=20 ang<B_a,B_c,B> =3D ang<C>.
(1b) Note que A,C,C1,A1 sao cociclicos = e=20 (A1,C1,C_a,A_c) tmb, donde = ang<C_a,A_c,B>=20 =3Dang<A1,C1,B> =3D ang<C>.
 
(2) C_b, B_c, C_a, B_a sao = cociclicos.
(2a)Desenhe uma figura soh com C_1, C_a = e C_b. Note=20 que C_b, C1, C_a, C sao cociclicos, e portanto
ang<C,C_a,C_b> =3D = ang<C1,C,C_b> =3D 90 - A=20 e portanto ang<C_b,C_a,C> =3D A.
(2b)Analogamente, ang<B_c,B_a,B> = =3D=20 A.
Por (1b), ang<C_b,B_c,B_a> =3D=20 ang<B_c,B_a,B> e portanto (2) vale.
 
3aQ:
a) Prove por inducao q =  (a1,a2,a3,a4) =3D=20 (0,1,1,0) mod 2, onde os indices sao olhados modulo 4.Em=20 particular, como 2001 =3D 1 mod 4, a1 eh par.
b) Prove por inducao q (a1, a2, a3, = ..., a12, a13)=20 =3D (0,1,1,2,1,1,1,0,2,0,2,1,0) mod 3 (indices olhados mod 13), e como=20 2003=3D13mod13, a2002 eh multiplo de 3.
c) Sim. Comece com (499,1,0) por=20 exemplo.
d) Nao. Um modo de explicar eh q = comecando com=20 0,1,1 o 15o termo ja da 1705. Com 1,1,1 ja passa de 2002, e entao = eh soh=20 testar com termos iniciais menores (claro que aumentar um termo inicial = soh=20 aumenta a15)..
Uma maneira diferente eh vc notar = que comecar=20 com (a,b,c) equivale a comecar com (1,0,0); (0,1,0) e (0,0,1) e = multiplicar o=20 a_15 por a,b,c respectivamente. Fica algo como a15 =3D 504a + 778b + = 927c, e ai vc=20 testa a15=3D2002 como na outra solucao.
 
 
 
A proposito, essa prova pareceu bem = mais dificil do=20 que a do ano passado.. Como q o pessoal que fez a prova foi? O que = acharam?=20
 
[]'s
Marcio
 
----- Original Message -----
From:=20 haroldo=20
Sent: Tuesday, September 03, = 2002 3:27=20 PM
Subject: [obm-l] Gabarito da = olimp=EDada=20 carioca.

Gostaria de ver o = gabarito oficial=20 da olimp=EDada carioca, realizada na PUC, dia=20 31/08.

Pois tenho dois alunos = que=20 participaram da competi=E7=E3o.

Grato.

Haroldo=20 .

------=_NextPart_000_000A_01C2536A.D15F07A0-- ========================================================================= Instruções para entrar na lista, sair da lista e usar a lista em http://www.mat.puc-rio.br/~nicolau/olimp/obm-l.html O administrador desta lista é ========================================================================= From owner-obm-l@sucuri.mat.puc-rio.br Wed Sep 4 00:49:54 2002 Return-Path: Received: (from majordom@localhost) by sucuri.mat.puc-rio.br (8.9.3/8.9.3) id AAA10560 for obm-l-MTTP; Wed, 4 Sep 2002 00:49:19 -0300 Received: from hotmail.com (f58.pav1.hotmail.com [64.4.31.58]) by sucuri.mat.puc-rio.br (8.9.3/8.9.3) with ESMTP id AAA10556 for ; Wed, 4 Sep 2002 00:49:16 -0300 Received: from mail pickup service by hotmail.com with Microsoft SMTPSVC; Tue, 3 Sep 2002 20:49:53 -0700 Received: from 200.199.185.152 by pv1fd.pav1.hotmail.msn.com with HTTP; Wed, 04 Sep 2002 03:49:53 GMT X-Originating-IP: [200.199.185.152] From: "Adherbal Rocha Filho" To: obm-l@mat.puc-rio.br Subject: [obm-l] ajudem na boa Date: Wed, 04 Sep 2002 03:49:53 +0000 Mime-Version: 1.0 Content-Type: text/plain; charset=iso-8859-1; format=flowed Message-ID: X-OriginalArrivalTime: 04 Sep 2002 03:49:53.0553 (UTC) FILETIME=[209C2810:01C253C6] Sender: owner-obm-l@sucuri.mat.puc-rio.br Precedence: bulk Reply-To: obm-l@mat.puc-rio.br >Por favor,alguem pode dar uma luz nestes problemas? >Um triangulo acutangulo ABC está inscrito numa circunferencia de centro >O.As alturas do triangulo são AD,BE, CF. A reta EF intersecta a >circunferencia em P e Q. Prove q OA eh perpendicular a PQ. Prove q se M eh >pnt. medio de BC,entãoAP^2=2AD*OM > >Encontre todos os inteiros positivos l,m,n primos 2 a 2 ,tais q >(n+l+m)(1/n + 1/l + 1/m) é um inteiro positivo. > >Obrigado! >Adherbal > _________________________________________________________________ Converse com seus amigos online, faça o download grátis do MSN Messenger: http://messenger.msn.com.br ========================================================================= Instruções para entrar na lista, sair da lista e usar a lista em http://www.mat.puc-rio.br/~nicolau/olimp/obm-l.html O administrador desta lista é ========================================================================= From owner-obm-l@sucuri.mat.puc-rio.br Wed Sep 4 08:56:37 2002 Return-Path: Received: (from majordom@localhost) by sucuri.mat.puc-rio.br (8.9.3/8.9.3) id IAA14146 for obm-l-MTTP; Wed, 4 Sep 2002 08:56:09 -0300 Received: from hotmail.com (f44.sea2.hotmail.com [207.68.165.44]) by sucuri.mat.puc-rio.br (8.9.3/8.9.3) with ESMTP id IAA14142 for ; Wed, 4 Sep 2002 08:56:07 -0300 Received: from mail pickup service by hotmail.com with Microsoft SMTPSVC; Wed, 4 Sep 2002 04:56:45 -0700 Received: from 32.94.119.253 by sea2fd.sea2.hotmail.msn.com with HTTP; Wed, 04 Sep 2002 11:56:45 GMT X-Originating-IP: [32.94.119.253] From: "Paulo Santa Rita" To: obm-l@mat.puc-rio.br Subject: [obm-l] =?iso-8859-1?B?UmU6IFtvYm0tbF0gR2FiYXJpdG8gZGEgb2xpbXDtYWRhIGNhcmlvY2Eu?= Date: Wed, 04 Sep 2002 11:56:45 +0000 Mime-Version: 1.0 Content-Type: text/plain; charset=iso-8859-1; format=flowed Message-ID: X-OriginalArrivalTime: 04 Sep 2002 11:56:45.0502 (UTC) FILETIME=[2449D5E0:01C2540A] Sender: owner-obm-l@sucuri.mat.puc-rio.br Precedence: bulk Reply-To: obm-l@mat.puc-rio.br Ola Marcio e demais colegas desta lista ... OBM-L, Pela primeira vez eu participei - POR E-MAIL'S - da elaboracao das questoes e das discussoes. Havia quase um consenso de que a prova do ano anterior ( e provavelmente de outros anos ) foi muito facil, assemelhando-se mais a um BOM exame de vestibular, tradicional, que a uma olimpiada. Eu olhei a prova passada e fiquei com a mesma impressao ... Parece que num BOM exame vestibular exige-se a DESENVOLTURA com o conhecimento; numa Olimpiada, pressupondo-se a desenvoltura, exige-se tambem - e sobretudo - a CRIATIVIDADE. Neste sentido, uma olimpiada deve exigir INTELIGENCIA e INOVACAO, nao ATIVIDADE BRACAL e APLICACAO BUROCRATICA. A sua observacao SUGERE que os objetivos da banca foram atingidos ... Eu penso que deve ser assim, doravante. Um abraco Paulo Santa Rita 4,0855,040902 >From: "Marcio" >Reply-To: obm-l@mat.puc-rio.br >To: >Subject: [obm-l] Re: [obm-l] Gabarito da olimpíada carioca. >Date: Tue, 3 Sep 2002 16:56:16 -0300 > >A proposito, essa prova pareceu bem mais dificil do que a do ano passado.. >Como q o pessoal que fez a prova foi? O que acharam? > >[]'s >Marcio > > ----- Original Message ----- > From: haroldo > To: obm-l@mat.puc-rio.br > Sent: Tuesday, September 03, 2002 3:27 PM > Subject: [obm-l] Gabarito da olimpíada carioca. > > > Gostaria de ver o gabarito oficial da olimpíada carioca, realizada na >PUC, dia 31/08. > > Pois tenho dois alunos que participaram da competição. > > Grato. > > Haroldo . _________________________________________________________________ MSN Photos é a maneira mais fácil e prática de editar e compartilhar sua fotos: http://photos.msn.com.br ========================================================================= Instruções para entrar na lista, sair da lista e usar a lista em http://www.mat.puc-rio.br/~nicolau/olimp/obm-l.html O administrador desta lista é ========================================================================= From owner-obm-l@sucuri.mat.puc-rio.br Wed Sep 4 09:42:28 2002 Return-Path: Received: (from majordom@localhost) by sucuri.mat.puc-rio.br (8.9.3/8.9.3) id JAA15271 for obm-l-MTTP; Wed, 4 Sep 2002 09:42:15 -0300 Received: from hotmail.com (f142.sea2.hotmail.com [207.68.165.142]) by sucuri.mat.puc-rio.br (8.9.3/8.9.3) with ESMTP id JAA15267 for ; Wed, 4 Sep 2002 09:42:12 -0300 Received: from mail pickup service by hotmail.com with Microsoft SMTPSVC; Wed, 4 Sep 2002 05:42:51 -0700 Received: from 32.94.119.253 by sea2fd.sea2.hotmail.msn.com with HTTP; Wed, 04 Sep 2002 12:42:50 GMT X-Originating-IP: [32.94.119.253] From: "Paulo Santa Rita" To: obm-l@mat.puc-rio.br Subject: [obm-l] =?iso-8859-1?B?UmU6IFtvYm0tbF0gTyBwcm9ibGVtYSBkYXMgaW5maW5pdGFzIHNvbHXn?= =?iso-8859-1?B?9WVz?= Date: Wed, 04 Sep 2002 12:42:50 +0000 Mime-Version: 1.0 Content-Type: text/plain; charset=iso-8859-1; format=flowed Message-ID: X-OriginalArrivalTime: 04 Sep 2002 12:42:51.0276 (UTC) FILETIME=[94D174C0:01C25410] Sender: owner-obm-l@sucuri.mat.puc-rio.br Precedence: bulk Reply-To: obm-l@mat.puc-rio.br Ola Wagner e demais colegas desta lista ... OBM-L, Eu nao entendi bem a sua questao, pois PARECE-ME que voce esta se referindo ao conjunto "C - R". Mas nao tenho certeza. Talvez voce esteja pensando em X^pi - 5*[X^(pi-1)] + 3 = 0 X^pi - [5*(X^pi)]/X + 3 = 0 X^pi(1 - 5/X) = -3 X^pi = 3X/(5-X) ... (A) X=a*[e^(Ti)] => X^pi = (a^pi)*{[e^(pi*i)]^T} X^pi=(a^pi)*{[cos(pi)+i*sen(pi)]^T} X^pi=(a^pi)*[(-1)^T] ... (B) (B) em (A) : (a^pi)*[(-1)^T]=3X/(5-X) (a^pi)*[i^2T]=3X/(5-X) X={[5*(a^pi)]*[i^(2T)]}/{3+[(a^pi)*[i^(2T)]]} Variando "a" e "T" convenientemente teremos uma infinidade de numeros que satisfazem a equacao proposta. Um abraco Paulo Santa Rita 4,0941,040902 >From: "Wagner" >Reply-To: obm-l@mat.puc-rio.br >To: >Subject: [obm-l] O problema das infinitas soluções >Date: Mon, 2 Sep 2002 16:34:50 -0300 > >Esse é o meu primeiro problema na lista > >Notação: >- a^(b) = a elevado a potência b >- PI = o nº pi > >Prove que a equação: x^(PI)-5x^(PI-1)+3=0. Possui infinitas soluções >complexas. > > > André T. _________________________________________________________________ Tenha você também um MSN Hotmail, o maior webmail do mundo: http://www.hotmail.com/br ========================================================================= Instruções para entrar na lista, sair da lista e usar a lista em http://www.mat.puc-rio.br/~nicolau/olimp/obm-l.html O administrador desta lista é ========================================================================= From owner-obm-l@sucuri.mat.puc-rio.br Wed Sep 4 13:45:08 2002 Return-Path: Received: (from majordom@localhost) by sucuri.mat.puc-rio.br (8.9.3/8.9.3) id NAA20599 for obm-l-MTTP; Wed, 4 Sep 2002 13:41:52 -0300 Received: from lia.ufc.br (iracema [200.19.177.4] (may be forged)) by sucuri.mat.puc-rio.br (8.9.3/8.9.3) with ESMTP id NAA20595 for ; Wed, 4 Sep 2002 13:41:49 -0300 From: latino@lia.ufc.br Received: from lia.ufc.br (latino@localhost [127.0.0.1]) by lia.ufc.br (8.12.5/8.12.5/Debian-1) with ESMTP id g84GgS0o030726 for ; Wed, 4 Sep 2002 13:42:28 -0300 Received: from localhost (latino@localhost) by lia.ufc.br (8.12.5/8.12.5/Debian-1) with ESMTP id g84GgR70030722 for ; Wed, 4 Sep 2002 13:42:28 -0300 Date: Wed, 4 Sep 2002 13:42:27 -0300 (BRT) To: obm-l@mat.puc-rio.br Subject: Re: [obm-l] ajudem na boa In-Reply-To: Message-ID: MIME-Version: 1.0 Content-Type: TEXT/PLAIN; charset=X-UNKNOWN Content-Transfer-Encoding: 8bit X-MIME-Autoconverted: from QUOTED-PRINTABLE to 8bit by sucuri.mat.puc-rio.br id NAA20596 Sender: owner-obm-l@sucuri.mat.puc-rio.br Precedence: bulk Reply-To: obm-l@mat.puc-rio.br > >Por favor,alguem pode dar uma luz nestes problemas? > >Um triangulo acutangulo ABC está inscrito numa circunferencia de centro > >O.As alturas do triangulo são AD,BE, CF. A reta EF intersecta a > >circunferencia em P e Q. Prove q OA eh perpendicular a PQ. Prove q se M eh > >pnt. medio de BC,entãoAP^2=2AD*OM Para a primeira parte, observe que provar que OA e perpendicular a PQ e equivalente a mostrar que arc(AP) = arc(AQ). Chame o ortocentro do triangulo de H. Como o quadrilatero AFHE e inscritivel, temos AP^2 = AD.2OM => AP^2 = AD.AH. (Nao e dificil mostrar que AH = 2OM... tente semelhanca entre os triangulos ABH e OMN, onde N e ponto medio de AC) Os triangulos AFH e ABD sao semelhantes, logo: AB/AH = AD/AF => AD.AH = AB.AF A relacao a ser provada se transforma, entao, em: AP^2 = AB.AF Observe que os triangulos APF e APB sao semelhantes, pois AP^2 = AB.AF, que era o que queriamos mostrar. abracos, ##################################### # MSc. Edson Ricardo de A. Silva # # Computer Graphics Group - CRAB # # Federal University of Ceara - UFC # ##################################### ========================================================================= Instruções para entrar na lista, sair da lista e usar a lista em http://www.mat.puc-rio.br/~nicolau/olimp/obm-l.html O administrador desta lista é ========================================================================= From owner-obm-l@sucuri.mat.puc-rio.br Wed Sep 4 15:33:19 2002 Return-Path: Received: (from majordom@localhost) by sucuri.mat.puc-rio.br (8.9.3/8.9.3) id PAA23704 for obm-l-MTTP; Wed, 4 Sep 2002 15:32:20 -0300 Received: from traven9.uol.com.br (200-221-4-35.portais-uolinc.uol.com.br [200.221.4.35] (may be forged)) by sucuri.mat.puc-rio.br (8.9.3/8.9.3) with ESMTP id PAA23698 for ; Wed, 4 Sep 2002 15:32:11 -0300 Received: from u2z7z2 ([200.158.145.125]) by traven9.uol.com.br (8.9.1/8.9.1) with ESMTP id PAA03203 for ; Wed, 4 Sep 2002 15:34:11 -0300 (BRT) Message-ID: <000701c25441$93b7d120$2101a8c0@u2z7z2> From: "Wagner" To: References: Subject: [obm-l] =?iso-8859-1?Q?Re:_=5Bobm-l=5D_Re:_=5Bobm-l=5D_O_problema_das_infinitas?= =?iso-8859-1?Q?_solu=E7=F5es?= Date: Wed, 4 Sep 2002 15:33:33 -0300 Organization: Wagner MIME-Version: 1.0 Content-Type: text/plain; charset="iso-8859-1" Content-Transfer-Encoding: 8bit X-Priority: 3 X-MSMail-Priority: Normal X-Mailer: Microsoft Outlook Express 5.50.4133.2400 X-MimeOLE: Produced By Microsoft MimeOLE V5.50.4133.2400 Sender: owner-obm-l@sucuri.mat.puc-rio.br Precedence: bulk Reply-To: obm-l@mat.puc-rio.br Oi pra todo mundo Muito bem Paulo você achou a resposta (o conjunto universo da equação é "C"). Mas quando eu imaginei o problema eu pensei numa resposta mais simples: Imagine uma equação do tipo: x^(a/b)+cx^((a/b)-1)+dx((a/b)-2)+...+n=0. Em que a e b são números inteiros e a/b é uma fração irredutível. Se y=x^(1/b). Logo: y^(a)+cy^(a-b)+...+n=0. Logo existem a valores complexos para y que satisfazem a equação e consequentemente, a valores para x. Considerando pi/1 como uma fração irredutível e n o nº de casas decimais de pi. Logo: (pi)(10^n)/(10^n) é uma fração irredutível e portanto existem (pi)(10^n) valores de x que satisfazem: x^pi - 5^pi + 3 = 0. Como pi é um nº irracional, ele tem infinitas casas decimais e portanto a equação do problema possui infinitas soluções complexas. OBS: Isso acontece com qualquer equação em que o índice a que x esta elevado é um nº irracional em pelo menos um de seus termos. André T. ----- Original Message ----- From: "Paulo Santa Rita" To: Sent: Wednesday, September 04, 2002 9:42 AM Subject: [obm-l] Re: [obm-l] O problema das infinitas soluções > Ola Wagner e demais > colegas desta lista ... OBM-L, > > Eu nao entendi bem a sua questao, pois PARECE-ME que voce esta se referindo > ao conjunto "C - R". Mas nao tenho certeza. Talvez voce esteja pensando em > > X^pi - 5*[X^(pi-1)] + 3 = 0 > X^pi - [5*(X^pi)]/X + 3 = 0 > X^pi(1 - 5/X) = -3 > X^pi = 3X/(5-X) ... (A) > > X=a*[e^(Ti)] => X^pi = (a^pi)*{[e^(pi*i)]^T} > X^pi=(a^pi)*{[cos(pi)+i*sen(pi)]^T} > X^pi=(a^pi)*[(-1)^T] ... (B) > > (B) em (A) : > > (a^pi)*[(-1)^T]=3X/(5-X) > (a^pi)*[i^2T]=3X/(5-X) > X={[5*(a^pi)]*[i^(2T)]}/{3+[(a^pi)*[i^(2T)]]} > > Variando "a" e "T" convenientemente teremos uma infinidade de numeros que > satisfazem a equacao proposta. > > Um abraco > Paulo Santa Rita > 4,0941,040902 > > >From: "Wagner" > >Reply-To: obm-l@mat.puc-rio.br > >To: > >Subject: [obm-l] O problema das infinitas soluções > >Date: Mon, 2 Sep 2002 16:34:50 -0300 > > > >Esse é o meu primeiro problema na lista > > > >Notação: > >- a^(b) = a elevado a potência b > >- PI = o nº pi > > > >Prove que a equação: x^(PI)-5x^(PI-1)+3=0. Possui infinitas soluções > >complexas. > > > > > > André T. > > > > > _________________________________________________________________ > Tenha você também um MSN Hotmail, o maior webmail do mundo: > http://www.hotmail.com/br > > ========================================================================= > Instruções para entrar na lista, sair da lista e usar a lista em > http://www.mat.puc-rio.br/~nicolau/olimp/obm-l.html > O administrador desta lista é > ========================================================================= > ========================================================================= Instruções para entrar na lista, sair da lista e usar a lista em http://www.mat.puc-rio.br/~nicolau/olimp/obm-l.html O administrador desta lista é ========================================================================= From owner-obm-l@sucuri.mat.puc-rio.br Wed Sep 4 18:54:05 2002 Return-Path: Received: (from majordom@localhost) by sucuri.mat.puc-rio.br (8.9.3/8.9.3) id SAA27412 for obm-l-MTTP; Wed, 4 Sep 2002 18:53:39 -0300 Received: from hotmail.com (f105.sea2.hotmail.com [207.68.165.105]) by sucuri.mat.puc-rio.br (8.9.3/8.9.3) with ESMTP id SAA27407 for ; Wed, 4 Sep 2002 18:53:37 -0300 Received: from mail pickup service by hotmail.com with Microsoft SMTPSVC; Wed, 4 Sep 2002 14:54:16 -0700 Received: from 32.94.119.253 by sea2fd.sea2.hotmail.msn.com with HTTP; Wed, 04 Sep 2002 21:54:16 GMT X-Originating-IP: [32.94.119.253] From: "Paulo Santa Rita" To: obm-l@mat.puc-rio.br Subject: [obm-l] =?iso-8859-1?B?UmU6IFtvYm0tbF0gUmU6IFtvYm0tbF0gUmU6IFtvYm0tbF0gTyBwcm9i?= =?iso-8859-1?B?bGVtYSBkYXMgaW5maW5pdGFzIHNvbHXn9WVz?= Date: Wed, 04 Sep 2002 21:54:16 +0000 Mime-Version: 1.0 Content-Type: text/plain; charset=iso-8859-1; format=flowed Message-ID: X-OriginalArrivalTime: 04 Sep 2002 21:54:16.0404 (UTC) FILETIME=[9D175940:01C2545D] Sender: owner-obm-l@sucuri.mat.puc-rio.br Precedence: bulk Reply-To: obm-l@mat.puc-rio.br Ola Wagner e demais colegas desta lista ... OBM-L, O conceito de simplicidade e subjetivo... mas e bonita a sua solucao ! Todavia, e bom que se diga, e uma demonstracao de existencia, nao exibindo a "cara" ou "forma" da solucoes. Em verdade, esse jeito foi a primeira coisa que veio a minha cabeca, mas eu prefiri uma outra via, construtiva, porque assim eu forneceria elementos para verificacoes posteriores, coisa que uma simples prova de existencia nao concede ... A respeito de equacoes nao triviais existe uma questao bonita : Seja y=f(x) uma equacao do 5 GRAU, INCOMPLETA, isto e, na qual um ou mais dos coeficientes da equacao geral e(sao) nulo. Em que casos ela admite uma solucao algebrica, isto e, quando as solucoes podem ser expressas como operacoes algebricas sobre os seus coeficientes ? Um abraco Paulo Santa Rita 4,1852,040902 >From: "Wagner" >Reply-To: obm-l@mat.puc-rio.br >To: >Subject: [obm-l] Re: [obm-l] Re: [obm-l] O problema das infinitas soluções >Date: Wed, 4 Sep 2002 15:33:33 -0300 > >Oi pra todo mundo > >Muito bem Paulo você achou a resposta (o conjunto universo da equação é >"C"). Mas quando eu imaginei o problema eu pensei numa resposta mais >simples: > >Imagine uma equação do tipo: x^(a/b)+cx^((a/b)-1)+dx((a/b)-2)+...+n=0. Em >que a e b são números inteiros e a/b é uma fração >irredutível. Se y=x^(1/b). Logo: y^(a)+cy^(a-b)+...+n=0. Logo existem a >valores complexos para y que satisfazem a equação e consequentemente, a >valores para x. Considerando pi/1 como uma fração irredutível e n o nº de >casas decimais de pi. Logo: (pi)(10^n)/(10^n) é uma fração irredutível e >portanto existem (pi)(10^n) valores de x que satisfazem: x^pi - 5^pi + 3 = >0. Como pi é um nº irracional, ele tem infinitas casas decimais e portanto >a >equação do problema possui infinitas soluções complexas. > >OBS: Isso acontece com qualquer equação em que o índice a que x esta >elevado >é um nº irracional em pelo menos um de seus termos. > >André T. > > >----- Original Message ----- >From: "Paulo Santa Rita" >To: >Sent: Wednesday, September 04, 2002 9:42 AM >Subject: [obm-l] Re: [obm-l] O problema das infinitas soluções > > > > Ola Wagner e demais > > colegas desta lista ... OBM-L, > > > > Eu nao entendi bem a sua questao, pois PARECE-ME que voce esta se >referindo > > ao conjunto "C - R". Mas nao tenho certeza. Talvez voce esteja pensando >em > > > > X^pi - 5*[X^(pi-1)] + 3 = 0 > > X^pi - [5*(X^pi)]/X + 3 = 0 > > X^pi(1 - 5/X) = -3 > > X^pi = 3X/(5-X) ... (A) > > > > X=a*[e^(Ti)] => X^pi = (a^pi)*{[e^(pi*i)]^T} > > X^pi=(a^pi)*{[cos(pi)+i*sen(pi)]^T} > > X^pi=(a^pi)*[(-1)^T] ... (B) > > > > (B) em (A) : > > > > (a^pi)*[(-1)^T]=3X/(5-X) > > (a^pi)*[i^2T]=3X/(5-X) > > X={[5*(a^pi)]*[i^(2T)]}/{3+[(a^pi)*[i^(2T)]]} > > > > Variando "a" e "T" convenientemente teremos uma infinidade de numeros >que > > satisfazem a equacao proposta. > > > > Um abraco > > Paulo Santa Rita > > 4,0941,040902 > > > > >From: "Wagner" > > >Reply-To: obm-l@mat.puc-rio.br > > >To: > > >Subject: [obm-l] O problema das infinitas soluções > > >Date: Mon, 2 Sep 2002 16:34:50 -0300 > > > > > >Esse é o meu primeiro problema na lista > > > > > >Notação: > > >- a^(b) = a elevado a potência b > > >- PI = o nº pi > > > > > >Prove que a equação: x^(PI)-5x^(PI-1)+3=0. Possui infinitas soluções > > >complexas. > > > > > > > > > André T. > > > > > > > > > > _________________________________________________________________ > > Tenha você também um MSN Hotmail, o maior webmail do mundo: > > http://www.hotmail.com/br > > > > >========================================================================= > > Instruções para entrar na lista, sair da lista e usar a lista em > > http://www.mat.puc-rio.br/~nicolau/olimp/obm-l.html > > O administrador desta lista é > > >========================================================================= > > > > >========================================================================= >Instruções para entrar na lista, sair da lista e usar a lista em >http://www.mat.puc-rio.br/~nicolau/olimp/obm-l.html >O administrador desta lista é >========================================================================= O ========================================================================= Instruções para entrar na lista, sair da lista e usar a lista em http://www.mat.puc-rio.br/~nicolau/olimp/obm-l.html O administrador desta lista é ========================================================================= From owner-obm-l@sucuri.mat.puc-rio.br Wed Sep 4 21:20:12 2002 Return-Path: Received: (from majordom@localhost) by sucuri.mat.puc-rio.br (8.9.3/8.9.3) id VAA29370 for obm-l-MTTP; Wed, 4 Sep 2002 21:19:23 -0300 Received: from smtp-3.ig.com.br (smtp-3.ig.com.br [200.226.132.152]) by sucuri.mat.puc-rio.br (8.9.3/8.9.3) with SMTP id VAA29366 for ; Wed, 4 Sep 2002 21:19:20 -0300 Received: (qmail 30413 invoked from network); 5 Sep 2002 00:19:37 -0000 Received: from shasta054248.ig.com.br (HELO araujoimeig) (200.151.54.248) by smtp-3.ig.com.br with SMTP; 5 Sep 2002 00:19:37 -0000 Message-ID: <003601c25471$77a16a20$df0597c8@com.br> From: =?iso-8859-1?B?QW5kcuk=?= To: "OBM" References: Subject: [obm-l] =?iso-8859-1?Q?Re:_=5Bobm-l=5D_sele=E7=E3o_cone_sul?= Date: Wed, 4 Sep 2002 21:15:35 -0300 MIME-Version: 1.0 Content-Type: text/plain; charset="iso-8859-1" Content-Transfer-Encoding: 8bit X-Priority: 3 X-MSMail-Priority: Normal X-Mailer: Microsoft Outlook Express 5.00.2615.200 X-MimeOLE: Produced By Microsoft MimeOLE V5.00.2615.200 Sender: owner-obm-l@sucuri.mat.puc-rio.br Precedence: bulk Reply-To: obm-l@mat.puc-rio.br > ae pessoal,será q alguem pode ajudar nestas questões: > > 3.A bissetriz do angulo B em um triangulo ABC intercepta o lado AC > em D.Seja E um ponto sobre o lado BC tal q 3CÂE=2BÂE.Os segmentos BD > e AE interseptam-se no ponto P.Se ED=AD=AP,determine os angulos do > triangulo > Caminho das pedras... 1) Triangulo ABE semelhante ao triangulo ABC, assim: AE = AC*AB/BC 2) Do teorema da bissetriz interna temos: AD = AC*AB/(BC+AB) 3) Trace por D uma paralela a AE que corta BC em F. O triangulo CDF semelhante ao triangulo CAE, assim: DF = AC*AB/BC ==> DF = AD = DE. Dai concluimos que ang(CAE) = ang(ABC)/2. Logo: A = 75, B = 60 e C = 45. Andre. ========================================================================= Instruções para entrar na lista, sair da lista e usar a lista em http://www.mat.puc-rio.br/~nicolau/olimp/obm-l.html O administrador desta lista é ========================================================================= From owner-obm-l@sucuri.mat.puc-rio.br Wed Sep 4 21:20:12 2002 Return-Path: Received: (from majordom@localhost) by sucuri.mat.puc-rio.br (8.9.3/8.9.3) id VAA29352 for obm-l-MTTP; Wed, 4 Sep 2002 21:18:40 -0300 Received: from Euler.impa.br (euler.impa.br [147.65.1.3]) by sucuri.mat.puc-rio.br (8.9.3/8.9.3) with ESMTP id VAA29347 for ; Wed, 4 Sep 2002 21:18:38 -0300 Received: from [147.65.11.3] (dial03.impa.br [147.65.11.3]) by Euler.impa.br (8.11.6/8.11.6) with ESMTP id g850JI717622 for ; Wed, 4 Sep 2002 21:19:18 -0300 (EST) Message-Id: <200209050019.g850JI717622@Euler.impa.br> X-Mailer: Microsoft Outlook Express Macintosh Edition - 4.5 (0410) Date: Fri, 06 Sep 2002 21:17:13 -0300 Subject: Re: [obm-l] Re: [obm-l] Re: [obm-l] Re: [obm-l] O problema das infinitas solu=?ISO-8859-1?B?5/U=?=es From: "Eduardo Wagner" To: obm-l@mat.puc-rio.br Mime-version: 1.0 X-Priority: 3 Content-type: text/plain; charset="ISO-8859-1" Content-Transfer-Encoding: 8bit X-MIME-Autoconverted: from quoted-printable to 8bit by sucuri.mat.puc-rio.br id VAA29348 Sender: owner-obm-l@sucuri.mat.puc-rio.br Precedence: bulk Reply-To: obm-l@mat.puc-rio.br Caros amigos da lista: So para esclarecer, o Wagner(timpa@uol) nao eh Eduardo Wagner, que alias sou eu. Trata-se de um outro participante da lista que gostaria de conhecer. Abracos E. Wagner ---------- >From: "Paulo Santa Rita" >To: obm-l@mat.puc-rio.br >Subject: [obm-l] Re: [obm-l] Re: [obm-l] Re: [obm-l] O problema das infinitas soluções >Date: Wed, Sep 4, 2002, 6:54 PM > > Ola Wagner e demais > colegas desta lista ... OBM-L, > > O conceito de simplicidade e subjetivo... mas e bonita a sua solucao ! > Todavia, e bom que se diga, e uma demonstracao de existencia, nao exibindo a > "cara" ou "forma" da solucoes. > > Em verdade, esse jeito foi a primeira coisa que veio a minha cabeca, mas eu > prefiri uma outra via, construtiva, porque assim eu forneceria elementos > para verificacoes posteriores, coisa que uma simples prova de existencia nao > concede ... > > A respeito de equacoes nao triviais existe uma questao bonita : > > Seja y=f(x) uma equacao do 5 GRAU, INCOMPLETA, isto e, na qual um ou mais > dos coeficientes da equacao geral e(sao) nulo. Em que casos ela admite uma > solucao algebrica, isto e, quando as solucoes podem ser expressas como > operacoes algebricas sobre os seus coeficientes ? > > Um abraco > Paulo Santa Rita > 4,1852,040902 > >>From: "Wagner" >>Reply-To: obm-l@mat.puc-rio.br >>To: >>Subject: [obm-l] Re: [obm-l] Re: [obm-l] O problema das infinitas soluções >>Date: Wed, 4 Sep 2002 15:33:33 -0300 >> >>Oi pra todo mundo >> >>Muito bem Paulo você achou a resposta (o conjunto universo da equação é >>"C"). Mas quando eu imaginei o problema eu pensei numa resposta mais >>simples: >> >>Imagine uma equação do tipo: x^(a/b)+cx^((a/b)-1)+dx((a/b)-2)+...+n=0. Em >>que a e b são números inteiros e a/b é uma fração >>irredutível. Se y=x^(1/b). Logo: y^(a)+cy^(a-b)+...+n=0. Logo existem a >>valores complexos para y que satisfazem a equação e consequentemente, a >>valores para x. Considerando pi/1 como uma fração irredutível e n o nº de >>casas decimais de pi. Logo: (pi)(10^n)/(10^n) é uma fração irredutível e >>portanto existem (pi)(10^n) valores de x que satisfazem: x^pi - 5^pi + 3 = >>0. Como pi é um nº irracional, ele tem infinitas casas decimais e portanto >>a >>equação do problema possui infinitas soluções complexas. >> >>OBS: Isso acontece com qualquer equação em que o índice a que x esta >>elevado >>é um nº irracional em pelo menos um de seus termos. >> >>André T. >> >> >>----- Original Message ----- >>From: "Paulo Santa Rita" >>To: >>Sent: Wednesday, September 04, 2002 9:42 AM >>Subject: [obm-l] Re: [obm-l] O problema das infinitas soluções >> >> >> > Ola Wagner e demais >> > colegas desta lista ... OBM-L, >> > >> > Eu nao entendi bem a sua questao, pois PARECE-ME que voce esta se >>referindo >> > ao conjunto "C - R". Mas nao tenho certeza. Talvez voce esteja pensando >>em >> > >> > X^pi - 5*[X^(pi-1)] + 3 = 0 >> > X^pi - [5*(X^pi)]/X + 3 = 0 >> > X^pi(1 - 5/X) = -3 >> > X^pi = 3X/(5-X) ... (A) >> > >> > X=a*[e^(Ti)] => X^pi = (a^pi)*{[e^(pi*i)]^T} >> > X^pi=(a^pi)*{[cos(pi)+i*sen(pi)]^T} >> > X^pi=(a^pi)*[(-1)^T] ... (B) >> > >> > (B) em (A) : >> > >> > (a^pi)*[(-1)^T]=3X/(5-X) >> > (a^pi)*[i^2T]=3X/(5-X) >> > X={[5*(a^pi)]*[i^(2T)]}/{3+[(a^pi)*[i^(2T)]]} >> > >> > Variando "a" e "T" convenientemente teremos uma infinidade de numeros >>que >> > satisfazem a equacao proposta. >> > >> > Um abraco >> > Paulo Santa Rita >> > 4,0941,040902 >> > >> > >From: "Wagner" >> > >Reply-To: obm-l@mat.puc-rio.br >> > >To: >> > >Subject: [obm-l] O problema das infinitas soluções >> > >Date: Mon, 2 Sep 2002 16:34:50 -0300 >> > > >> > >Esse é o meu primeiro problema na lista >> > > >> > >Notação: >> > >- a^(b) = a elevado a potência b >> > >- PI = o nº pi >> > > >> > >Prove que a equação: x^(PI)-5x^(PI-1)+3=0. Possui infinitas soluções >> > >complexas. >> > > >> > > >> > > André T. >> > >> > >> > >> > >> > _________________________________________________________________ >> > Tenha você também um MSN Hotmail, o maior webmail do mundo: >> > http://www.hotmail.com/br >> > >> > >>========================================================================= >> > Instruções para entrar na lista, sair da lista e usar a lista em >> > http://www.mat.puc-rio.br/~nicolau/olimp/obm-l.html >> > O administrador desta lista é >> > >>========================================================================= >> > >> >> >>========================================================================= >>Instruções para entrar na lista, sair da lista e usar a lista em >>http://www.mat.puc-rio.br/~nicolau/olimp/obm-l.html >>O administrador desta lista é >>========================================================================= > > > O > > > ========================================================================= > Instruções para entrar na lista, sair da lista e usar a lista em > http://www.mat.puc-rio.br/~nicolau/olimp/obm-l.html > O administrador desta lista é > ========================================================================= ========================================================================= Instruções para entrar na lista, sair da lista e usar a lista em http://www.mat.puc-rio.br/~nicolau/olimp/obm-l.html O administrador desta lista é ========================================================================= From owner-obm-l@sucuri.mat.puc-rio.br Wed Sep 4 22:36:39 2002 Return-Path: Received: (from majordom@localhost) by sucuri.mat.puc-rio.br (8.9.3/8.9.3) id WAA30782 for obm-l-MTTP; Wed, 4 Sep 2002 22:35:31 -0300 Received: from smtp-4.ig.com.br (smtp-4.ig.com.br [200.226.132.153]) by sucuri.mat.puc-rio.br (8.9.3/8.9.3) with SMTP id WAA30778 for ; Wed, 4 Sep 2002 22:35:29 -0300 Received: (qmail 6441 invoked from network); 5 Sep 2002 01:35:49 -0000 Received: from shasta091238.ig.com.br (HELO araujoimeig) (200.151.91.238) by smtp-4.ig.com.br with SMTP; 5 Sep 2002 01:35:49 -0000 Message-ID: <001901c2547c$1b4ffce0$ee5b97c8@com.br> From: =?iso-8859-1?B?QW5kcuk=?= To: "OBM" References: Subject: [obm-l] =?iso-8859-1?Q?Re:_=5Bobm-l=5D_sele=E7=E3o_cone_sul?= Date: Wed, 4 Sep 2002 22:30:52 -0300 MIME-Version: 1.0 Content-Type: text/plain; charset="iso-8859-1" Content-Transfer-Encoding: 8bit X-Priority: 3 X-MSMail-Priority: Normal X-Mailer: Microsoft Outlook Express 5.00.2615.200 X-MimeOLE: Produced By Microsoft MimeOLE V5.00.2615.200 Sender: owner-obm-l@sucuri.mat.puc-rio.br Precedence: bulk Reply-To: obm-l@mat.puc-rio.br > ae pessoal,será q alguem pode ajudar nestas questões: > > 2.Seja ABCD um paralelogramo ,H o ortocentro do triangulo ABD e O o > circuncentro do triangulo BCD.Prove q H,O e C são colineares. > Caminho das pedras... 1) Seja M o ponto de encontro das diagonais, assim: AM = CM; 2) Seja O' o circuncentro do triangulo ABD, como os triangulos ABD e BCD sao congruentes; entao: OM = O'M; 3) Eh sabido q O'M = AH/2 (pois ABH eh semelhante OMN, N medio de AD); Dai concluimos que AHC eh semelhante MOC, com razao de semelhanca 1/2, assim H, O e C sao colineares. Andre. ========================================================================= Instruções para entrar na lista, sair da lista e usar a lista em http://www.mat.puc-rio.br/~nicolau/olimp/obm-l.html O administrador desta lista é ========================================================================= From owner-obm-l@sucuri.mat.puc-rio.br Wed Sep 4 23:23:04 2002 Return-Path: Received: (from majordom@localhost) by sucuri.mat.puc-rio.br (8.9.3/8.9.3) id XAA31672 for obm-l-MTTP; Wed, 4 Sep 2002 23:21:28 -0300 Received: from imo-r10.mx.aol.com (imo-r10.mx.aol.com [152.163.225.106]) by sucuri.mat.puc-rio.br (8.9.3/8.9.3) with ESMTP id XAA31669 for ; Wed, 4 Sep 2002 23:21:25 -0300 From: DEOLIVEIRASOU@aol.com Received: from DEOLIVEIRASOU@aol.com by imo-r10.mx.aol.com (mail_out_v34.10.) id z.26.2d484e6b (3310) for ; Wed, 4 Sep 2002 22:22:03 -0400 (EDT) Message-ID: <26.2d484e6b.2aa819cb@aol.com> Date: Wed, 4 Sep 2002 22:22:03 EDT Subject: [obm-l] (nenhum assunto) To: obm-l@mat.puc-rio.br MIME-Version: 1.0 Content-Type: multipart/alternative; boundary="part1_26.2d484e6b.2aa819cb_boundary" X-Mailer: AOL 7.0 for Windows BR sub 10501 Sender: owner-obm-l@sucuri.mat.puc-rio.br Precedence: bulk Reply-To: obm-l@mat.puc-rio.br --part1_26.2d484e6b.2aa819cb_boundary Content-Type: text/plain; charset="ISO-8859-1" Content-Transfer-Encoding: quoted-printable Valeu Edilon...valeu mesmo pelo esfor=E7o em elucidar minhas duvidas...s=E3o= =20 caras como voc=EA que fazem dessa lista algo grande....Voc=EA foi magn=E2nim= o!! Um Abra=E7o! Crom --part1_26.2d484e6b.2aa819cb_boundary Content-Type: text/html; charset="ISO-8859-1" Content-Transfer-Encoding: quoted-printable Valeu Edilon...valeu mesmo pelo esfor=E7o em elucidar=20= minhas duvidas...s=E3o caras como voc=EA que fazem dessa lista algo grande..= ..Voc=EA foi magn=E2nimo!!
   Um Abra=E7o!
            &nbs= p;  Crom
--part1_26.2d484e6b.2aa819cb_boundary-- ========================================================================= Instruções para entrar na lista, sair da lista e usar a lista em http://www.mat.puc-rio.br/~nicolau/olimp/obm-l.html O administrador desta lista é ========================================================================= From owner-obm-l@sucuri.mat.puc-rio.br Thu Sep 5 02:06:03 2002 Return-Path: Received: (from majordom@localhost) by sucuri.mat.puc-rio.br (8.9.3/8.9.3) id CAA00975 for obm-l-MTTP; Thu, 5 Sep 2002 02:04:38 -0300 Received: from shannon.bol.com.br (shannon.bol.com.br [200.221.24.13]) by sucuri.mat.puc-rio.br (8.9.3/8.9.3) with ESMTP id CAA00971 for ; Thu, 5 Sep 2002 02:04:36 -0300 Received: from bol.com.br (200.221.24.135) by shannon.bol.com.br (5.1.071) id 3D60E4DC004A7FBC for obm-l@mat.puc-rio.br; Thu, 5 Sep 2002 02:05:00 -0300 Date: Thu, 5 Sep 2002 02:03:34 -0300 Message-Id: Subject: [obm-l] =?iso-8859-1?q?quest=E3o_da_2=AAfase=28rj=29?= MIME-Version: 1.0 Content-Type: text/plain;charset="iso-8859-1" From: "ponciomineiro" To: obm-l@mat.puc-rio.br X-XaM3-API-Version: 2.4.3.4.4 X-SenderIP: 200.222.249.51 Content-Transfer-Encoding: 8bit X-MIME-Autoconverted: from quoted-printable to 8bit by sucuri.mat.puc-rio.br id CAA00972 Sender: owner-obm-l@sucuri.mat.puc-rio.br Precedence: bulk Reply-To: obm-l@mat.puc-rio.br Caros amigos, alguém conseguiu fazer a 1ª questão do 2º nível (o problema das formigas)? Achei o enunciado meio estranho... Um abraço a todos, Poncio __________________________________________________________________________ AcessoBOL, só R$ 9,90! O menor preço do mercado! Assine já! http://www.bol.com.br/acessobol ========================================================================= Instruções para entrar na lista, sair da lista e usar a lista em http://www.mat.puc-rio.br/~nicolau/olimp/obm-l.html O administrador desta lista é ========================================================================= From owner-obm-l@sucuri.mat.puc-rio.br Thu Sep 5 08:12:55 2002 Return-Path: Received: (from majordom@localhost) by sucuri.mat.puc-rio.br (8.9.3/8.9.3) id IAA03320 for obm-l-MTTP; Thu, 5 Sep 2002 08:11:12 -0300 Received: from siegel.bol.com.br (siegel.bol.com.br [200.221.24.20]) by sucuri.mat.puc-rio.br (8.9.3/8.9.3) with ESMTP id IAA03316 for ; Thu, 5 Sep 2002 08:11:09 -0300 Received: from proxyvr (200.221.24.191) by siegel.bol.com.br (5.1.071) id 3D5D1C9F006B9DA8 for obm-l@mat.puc-rio.br; Thu, 5 Sep 2002 08:11:37 -0300 Message-ID: <002801c254cd$4abb1a40$2932a8c0@redeconora.com.br> From: "Hely Jr." To: Subject: [obm-l] =?iso-8859-1?Q?Expans=E3o?= Date: Thu, 5 Sep 2002 08:09:12 -0300 MIME-Version: 1.0 Content-Type: multipart/alternative; boundary="----=_NextPart_000_0014_01C254B3.84D92E20" X-Priority: 3 X-MSMail-Priority: Normal X-Mailer: Microsoft Outlook Express 6.00.2600.0000 X-MIMEOLE: Produced By Microsoft MimeOLE V6.00.2600.0000 X-Sender-IP: 200.222.64.218 Sender: owner-obm-l@sucuri.mat.puc-rio.br Precedence: bulk Reply-To: obm-l@mat.puc-rio.br This is a multi-part message in MIME format. ------=_NextPart_000_0014_01C254B3.84D92E20 Content-Type: text/plain; charset="iso-8859-1" Content-Transfer-Encoding: quoted-printable Se alguem puder ajudar agrade=E7o: Mostre que a expans=E3o de (x+y)^10 pode ser escrita como a soma de ( 10! x^a y^b ) / a!b! onde a+b =3D 10 ------=_NextPart_000_0014_01C254B3.84D92E20 Content-Type: text/html; charset="iso-8859-1" Content-Transfer-Encoding: quoted-printable
Se alguem puder ajudar = agrade=E7o:
 
Mostre que a expans=E3o de (x+y)^10 = pode ser escrita=20 como a soma de
 
( 10!  x^a y^b ) / = a!b!
 
onde a+b =3D = 10
------=_NextPart_000_0014_01C254B3.84D92E20-- ========================================================================= Instruções para entrar na lista, sair da lista e usar a lista em http://www.mat.puc-rio.br/~nicolau/olimp/obm-l.html O administrador desta lista é ========================================================================= From owner-obm-l@sucuri.mat.puc-rio.br Thu Sep 5 14:42:07 2002 Return-Path: Received: (from majordom@localhost) by sucuri.mat.puc-rio.br (8.9.3/8.9.3) id OAA09979 for obm-l-MTTP; Thu, 5 Sep 2002 14:39:57 -0300 Received: from sr1.terra.com.br (sr1.terra.com.br [200.176.3.16]) by sucuri.mat.puc-rio.br (8.9.3/8.9.3) with ESMTP id OAA09975 for ; Thu, 5 Sep 2002 14:39:55 -0300 Received: from pavuna.terra.com.br (pavuna.terra.com.br [200.176.3.41]) by sr1.terra.com.br (Postfix) with ESMTP id 3A8106F556 for ; Thu, 5 Sep 2002 14:40:31 -0300 (EST) Received: from stabel (unknown [200.203.38.231]) (authenticated user dudasta) by pavuna.terra.com.br (Postfix) with ESMTP id CFCD568410 for ; Thu, 5 Sep 2002 14:40:30 -0300 (EST) Message-ID: <007101c25503$554c6cd0$0301a8c0@stabel> From: "Eduardo Casagrande Stabel" To: References: <002801c254cd$4abb1a40$2932a8c0@redeconora.com.br> Subject: [obm-l] =?iso-8859-1?Q?Re:_=5Bobm-l=5D_Expans=E3o?= Date: Thu, 5 Sep 2002 14:40:31 -0300 MIME-Version: 1.0 Content-Type: text/plain; charset="iso-8859-1" Content-Transfer-Encoding: 8bit X-Priority: 3 X-MSMail-Priority: Normal X-Mailer: Microsoft Outlook Express 6.00.2600.0000 X-MIMEOLE: Produced By Microsoft MimeOLE V6.00.2600.0000 Sender: owner-obm-l@sucuri.mat.puc-rio.br Precedence: bulk Reply-To: obm-l@mat.puc-rio.br Hely Jr., esse é um caso particular (particular pois n=natural) do famoso Binômio de Newton. Se n=natural então (x + y)^n vai ser uma soma de termos x^iy^(n-i) multiplicado por constantes. Quais constantes? Considere a forma clássica de fazer o produto de somas (x + y)^n = (x + y)(x + y)(x + y)...(x + y). Em cada parcela a gente deve escolher, ou x ou y, e ir multiplicando pela seguinte. A quantidade de formas de se obter x^iy^(n-i) é a quantidade de formas de escolher i x's dum bolo de n caixas que contêm cada uma um x e um y, ou ainda, a quantidade de formas de se escolher (n-i) y's dum bolo de n caixas que contêm cada uma um x e um y. Essa fórmula é clássica e chama-se COMBINAÇÃO de n elementos i a i, ou n elementos n-i a n-i, e vale COMB(n ; i) = n! / i! (n-i)! = COMB(n ; n-i). Na fórmula temos (x + y)^n = somatório de COMB(n ; i) x^i y^(n-i) onde i varia de 0 (inclusive) até n (inclusive). De onde vem a fórmula COMB(n ; i)? Essa tenho certeza que tem no arquivo da lista. Eduardo. Porto Alegre, RS. From: Hely Jr. >Se alguem puder ajudar agradeço: >Mostre que a expansão de (x+y)^10 pode ser escrita como a soma de >( 10! x^a y^b ) / a!b! >onde a+b = 10 ========================================================================= Instruções para entrar na lista, sair da lista e usar a lista em http://www.mat.puc-rio.br/~nicolau/olimp/obm-l.html O administrador desta lista é ========================================================================= From owner-obm-l@sucuri.mat.puc-rio.br Thu Sep 5 17:28:29 2002 Return-Path: Received: (from majordom@localhost) by sucuri.mat.puc-rio.br (8.9.3/8.9.3) id RAA13437 for obm-l-MTTP; Thu, 5 Sep 2002 17:27:54 -0300 Received: from hotmail.com (oe162.pav0.hotmail.com [64.4.33.155]) by sucuri.mat.puc-rio.br (8.9.3/8.9.3) with ESMTP id RAA13426 for ; Thu, 5 Sep 2002 17:27:51 -0300 Received: from mail pickup service by hotmail.com with Microsoft SMTPSVC; Thu, 5 Sep 2002 13:28:33 -0700 X-Originating-IP: [200.217.188.1] From: "e isso mesmo" To: Subject: [obm-l] =?iso-8859-1?Q?Turma_Do_Software_Matem=E1tico?= Date: Thu, 5 Sep 2002 17:31:23 -0300 MIME-Version: 1.0 X-Mailer: MSN Explorer 6.10.0016.1624 Content-Type: multipart/alternative; boundary="----=_NextPart_001_0000_01C25502.0E5BFBC0" Message-ID: X-OriginalArrivalTime: 05 Sep 2002 20:28:33.0792 (UTC) FILETIME=[CE44C800:01C2551A] Sender: owner-obm-l@sucuri.mat.puc-rio.br Precedence: bulk Reply-To: obm-l@mat.puc-rio.br ------=_NextPart_001_0000_01C25502.0E5BFBC0 Content-Type: text/plain; charset="iso-8859-1" Content-Transfer-Encoding: quoted-printable Caros colegas, gostaria de entrar em contato com usu=E1rios do MATLAB, M= ATHEMATICA, DERIVE, MATHCAD e MAPLE, etc. Seria bom trocarmos id=E9ias de= como resolver alguns problemas dessa lista ( e outros ) utilizando soft= s. Quem sabe criarmos uma turma de usu=E1rios que se ajudem mutuamente. O= site http://www.matematicas.net/php/main.php possui uma programoteca =F3= tima para iniciarmos. Um abra=E7o a todosAproveite melhor a Web. Fa=E7a o download GR=C1TIS do = MSN Explorer : http://explorer.msn.com.br/intl.asp#po ------=_NextPart_001_0000_01C25502.0E5BFBC0 Content-Type: text/html; charset="iso-8859-1" Content-Transfer-Encoding: quoted-printable
 
<= DIV>Caros colegas, gostaria de entrar em contato com usu=E1rios do  = MATLAB, MATHEMATICA, DERIVE, MATHCAD e MAPLE, etc. Seria bom tro= carmos id=E9ias de como resolver alguns problemas dessa lista ( e outros = ) utilizando  softs. Quem sabe criarmos uma turma de usu=E1rios que = se ajudem mutuamente. O site http://www.matematicas.net/php/main.php possui uma progr= amoteca =F3tima para iniciarmos.
Um abra=E7o a todos


Aproveite melhor a Web. Fa=E7a o download G= R=C1TIS do MSN Explorer : http://explorer.msn.com.br/intl.asp#po

------=_NextPart_001_0000_01C25502.0E5BFBC0-- ========================================================================= Instruções para entrar na lista, sair da lista e usar a lista em http://www.mat.puc-rio.br/~nicolau/olimp/obm-l.html O administrador desta lista é ========================================================================= From owner-obm-l@sucuri.mat.puc-rio.br Thu Sep 5 17:38:01 2002 Return-Path: Received: (from majordom@localhost) by sucuri.mat.puc-rio.br (8.9.3/8.9.3) id RAA13731 for obm-l-MTTP; Thu, 5 Sep 2002 17:37:52 -0300 Received: from traven9 (traven9.uol.com.br [200.221.4.35]) by sucuri.mat.puc-rio.br (8.9.3/8.9.3) with ESMTP id RAA13720 for ; Thu, 5 Sep 2002 17:37:50 -0300 Received: from franklin ([200.214.41.193]) by traven9 (8.9.1/8.9.1) with SMTP id RAA14890 for ; Thu, 5 Sep 2002 17:40:09 -0300 (BRT) From: "Franklin de Lima Marquezino" To: Subject: [obm-l] =?iso-8859-1?Q?Re:_1_=E9_primo=3F?= Date: Thu, 5 Sep 2002 17:38:48 -0300 Message-ID: <01c2551c$3ce856e0$c129d6c8@franklin> MIME-Version: 1.0 Content-Type: multipart/alternative; boundary="----=_NextPart_000_002A_01C25503.179B1EE0" X-Priority: 3 X-MSMail-Priority: Normal X-Mailer: Microsoft Outlook Express 4.71.1712.3 X-MIMEOLE: Produced By Microsoft MimeOLE V4.71.1712.3 Sender: owner-obm-l@sucuri.mat.puc-rio.br Precedence: bulk Reply-To: obm-l@mat.puc-rio.br This is a multi-part message in MIME format. ------=_NextPart_000_002A_01C25503.179B1EE0 Content-Type: text/plain; charset="iso-8859-1" Content-Transfer-Encoding: quoted-printable >Quando eu comecei a estudar os primos gaussianos fiquei felicissimo. De = cara=20 >descobri que 5=3D(2-i)(2+i) e que, portanto, por este angulo, 5 nao e = primo.=20 >Pensava que a balburdia ia acabar ... Ledo engano ! Afinal, em que = sentido=20 >os inteiros gaussianos esclarecem melhor a natureza dos numeros primos = ? O=20 >livro nao dizia ! Ninguem me respondeu ! Ol=E1! Algu=E9m pode comentar alguma coisa sobre esses primos gaussianos. = Pelo menos uma defini=E7=E3o e alguma refer=EAncia. =C9 que eu estudo = Ci=EAncia da Computa=E7=E3o e n=E3o tive isso na gradua=E7=E3o ainda, = nem sei se vou ter. At=E9 mais, Franklin -------------------------------------------------------------------------= --------------------------------------------------------------------- CBPF - Centro Brasileiro de Pesquisas F=EDsicas (Bolsista PIBIC/CNPq) LNCC - Laborat=F3rio Nacional de Computa=E7=E3o Cient=EDfica (Grupo de = Computa=E7=E3o Qu=E2ntica) GFT - Grupo de F=EDsica Te=F3rica Jos=E9 Leite Lopes http://virtual01.lncc.br/dcs/links/dcs_quantum_comp.html http://gft.ucp.br/staff/franklin -------------------------------------------------------------------------= -------------------------------------------------------------------- ------=_NextPart_000_002A_01C25503.179B1EE0 Content-Type: text/html; charset="iso-8859-1" Content-Transfer-Encoding: quoted-printable
>Quando eu comecei a estudar os primos gaussianos fiquei = felicissimo. De=20 cara
>descobri que 5=3D(2-i)(2+i) e que, portanto, por este = angulo, 5 nao e=20 primo.
>Pensava que a balburdia ia acabar ... Ledo engano ! = Afinal, em=20 que sentido
>os inteiros gaussianos esclarecem melhor a natureza = dos=20 numeros primos ? O
>livro nao dizia ! Ninguem me respondeu=20 !
 
Olá!
 
  Alguém pode comentar alguma coisa sobre esses primos=20 gaussianos. Pelo menos uma definição e alguma = referência.=20 É que eu estudo Ciência da Computação e = não=20 tive isso na graduação ainda, nem sei se vou ter.
 
     Até mais,
       Franklin
 
 
 
 
 
----------------------------------------------------------------= -------------------------------------------------------------------------= -----
CBPF=20 - Centro Brasileiro de Pesquisas Físicas (Bolsista = PIBIC/CNPq)
LNCC -=20 Laboratório Nacional de Computação = Científica (Grupo=20 de Computação Quântica)
 
GFT - Grupo de Física Teórica = José Leite=20 Lopes
http://= virtual01.lncc.br/dcs/links/dcs_quantum_comp.html
=
http://gft.ucp.br/staff/frankli= n
----------------------------------------------------------------= -------------------------------------------------------------------------= ----
 
------=_NextPart_000_002A_01C25503.179B1EE0-- ========================================================================= Instruções para entrar na lista, sair da lista e usar a lista em http://www.mat.puc-rio.br/~nicolau/olimp/obm-l.html O administrador desta lista é ========================================================================= From owner-obm-l@sucuri.mat.puc-rio.br Thu Sep 5 17:48:30 2002 Return-Path: Received: (from majordom@localhost) by sucuri.mat.puc-rio.br (8.9.3/8.9.3) id RAA14319 for obm-l-MTTP; Thu, 5 Sep 2002 17:48:19 -0300 Received: from zeus.opendf.com.br (zeus.opengate.com.br [200.181.71.10]) by sucuri.mat.puc-rio.br (8.9.3/8.9.3) with ESMTP id RAA14315 for ; Thu, 5 Sep 2002 17:48:16 -0300 Received: from localhost (localhost.opengate.com.br [127.0.0.1]) by zeus.opendf.com.br (Postfix) with ESMTP id 95D313EB19; Thu, 5 Sep 2002 17:48:56 -0300 (BRT) Received: by zeus.opendf.com.br (Postfix, from userid 48) id 191213EB26; Thu, 5 Sep 2002 17:48:56 -0300 (BRT) From: "498 - Artur Costa Steiner" To: obm-l@mat.puc-rio.br, artur@opendf.com.br Subject: [obm-l] O caráter não enumerável de R X-Mailer: NeoMail 1.25 X-IPAddress: 200.252.155.2 MIME-Version: 1.0 Content-Type: text/plain; charset=iso-8859-1 Message-Id: <20020905204856.191213EB26@zeus.opendf.com.br> Date: Thu, 5 Sep 2002 17:48:56 -0300 (BRT) X-Virus-Scanned: by AMaViS new-20020517 Sender: owner-obm-l@sucuri.mat.puc-rio.br Precedence: bulk Reply-To: obm-l@mat.puc-rio.br Um abraço a todos os amigos deste grupo no qual acabei de me inscrever! O assunto que mencionei sempre me intriga um pouco. Há uma clássica demonstração de que R (o conjunto dos reais)não é numerável e que pode ser encontrada na maioria dos livros sobre Análise. Estas provas baseiam-se no fato de que, nos espaços euclidianos, conjuntos perfeitos não são numeráveis. Logo, um ponto chave em tais provas é que os elementos do espaço são pontos de acumulação do mesmo. Sabemos que todo elemento de R é ponto de acumulação. Mas, e este é o ponto que me intriga, tal conclusão depende da métrica definida em R. Na métrica euclidiana usual tal fato é demonstrado (admitindo-se que R seja completo). Mas, se tomarmos, por exemplo, a chamada métrica discreta (d(x,y)=1, se x<>y e d(x,y)=0 se x=y))então nenhum elemento de R (ou do espaço métrico em questão) é ponto de acumulação. A provas que conheço sobre a não enumerabilidade de R (que consistem em se construir uma seqüência de intervalos fechados aninhados) não mais se aplicam na métrica discreta. Não me parece plausível que um espaço métrico seja enumerável numa métrica (ou topologia) e não numerável em outra, mas será que existe uma prova de que R (ou um espaço métrico qualquer) não é numerável a qual seja independente da forma segundo a qual definamos seus conjuntos abertos? Artur ========================================================================= Instruções para entrar na lista, sair da lista e usar a lista em http://www.mat.puc-rio.br/~nicolau/olimp/obm-l.html O administrador desta lista é ========================================================================= From owner-obm-l@sucuri.mat.puc-rio.br Thu Sep 5 17:58:37 2002 Return-Path: Received: (from majordom@localhost) by sucuri.mat.puc-rio.br (8.9.3/8.9.3) id RAA14974 for obm-l-MTTP; Thu, 5 Sep 2002 17:58:31 -0300 Received: (from nicolau@localhost) by sucuri.mat.puc-rio.br (8.9.3/8.9.3) id RAA14969 for obm-l@mat.puc-rio.br; Thu, 5 Sep 2002 17:58:30 -0300 Date: Thu, 5 Sep 2002 17:58:30 -0300 From: "Nicolau C. Saldanha" To: obm-l@mat.puc-rio.br Subject: [obm-l] Re: =?iso-8859-1?Q?=5Bobm-l=5D_O_car=E1ter_n=E3o_enumer=E1vel_de_R?= Message-ID: <20020905175830.A14459@sucuri.mat.puc-rio.br> References: <20020905204856.191213EB26@zeus.opendf.com.br> Mime-Version: 1.0 Content-Type: text/plain; charset=iso-8859-1 Content-Disposition: inline Content-Transfer-Encoding: 8bit User-Agent: Mutt/1.2.5i In-Reply-To: <20020905204856.191213EB26@zeus.opendf.com.br>; from artur@opendf.com.br on Thu, Sep 05, 2002 at 05:48:56PM -0300 Sender: owner-obm-l@sucuri.mat.puc-rio.br Precedence: bulk Reply-To: obm-l@mat.puc-rio.br On Thu, Sep 05, 2002 at 05:48:56PM -0300, 498 - Artur Costa Steiner wrote: > Um abraço a todos os amigos deste grupo no qual acabei de me inscrever! > > O assunto que mencionei sempre me intriga um pouco. Há uma clássica > demonstração de que R (o conjunto dos reais)não é numerável e que pode > ser encontrada na maioria dos livros sobre Análise. Estas provas > baseiam-se no fato de que, nos espaços euclidianos, conjuntos perfeitos > não são numeráveis. Logo, um ponto chave em tais provas é que os > elementos do espaço são pontos de acumulação do mesmo. > > Sabemos que todo elemento de R é ponto de acumulação. Mas, e este é o > ponto que me intriga, tal conclusão depende da métrica definida em R. > Na métrica euclidiana usual tal fato é demonstrado (admitindo-se que R > seja completo). Mas, se tomarmos, por exemplo, a chamada métrica > discreta (d(x,y)=1, se x<>y e d(x,y)=0 se x=y))então nenhum elemento de > R (ou do espaço métrico em questão) é ponto de acumulação. A provas que > conheço sobre a não enumerabilidade de R (que consistem em se construir > uma seqüência de intervalos fechados aninhados) não mais se aplicam na > métrica discreta. > > Não me parece plausível que um espaço métrico seja enumerável numa > métrica (ou topologia) e não numerável em outra, mas será que existe > uma prova de que R (ou um espaço métrico qualquer) não é numerável a > qual seja independente da forma segundo a qual definamos seus conjuntos > abertos? De fato, o fato de um conjunto ser ou n~ao enumer'avel independe da m'etrica, topologia, ou de qualquer outra estrutura: depende apenas do conjunto. Considere o seguinte teorema/demonstra,c~ao: Seja A um conjunto e P(A) o conjunto das partes de A. Ent~ao toda fun,c~ao de A em P(A) 'e n~ao sobrejetora. De fato, X = { x in A | x n~ao pertence a f(x) } se X = f(x) temos x pertence a X <==> x n~ao pertence a X um absurdo. Isso demonstra que o conjunto das partes de um conjunto infinito 'e sempre n~ao-enumer'avel. Mas podemos criar uma c'opia de P(N) (partes de N, o conjunto dos naturais) dentro de R, basta representar cada subconjunto de N pela cadeia infinita correspondente de 0 e 1, interpretada como uma expans~ao decimal infinita. 'E bem verdade que o que eu acabo de construir 'e um conjunto de Cantor, mas n~ao usei a m'etrica de R... []s, N. ========================================================================= Instruções para entrar na lista, sair da lista e usar a lista em http://www.mat.puc-rio.br/~nicolau/olimp/obm-l.html O administrador desta lista é ========================================================================= From owner-obm-l@sucuri.mat.puc-rio.br Thu Sep 5 19:17:53 2002 Return-Path: Received: (from majordom@localhost) by sucuri.mat.puc-rio.br (8.9.3/8.9.3) id TAA17221 for obm-l-MTTP; Thu, 5 Sep 2002 19:17:24 -0300 Received: from hotmail.com (f100.sea2.hotmail.com [207.68.165.100]) by sucuri.mat.puc-rio.br (8.9.3/8.9.3) with ESMTP id TAA17214 for ; Thu, 5 Sep 2002 19:17:21 -0300 Received: from mail pickup service by hotmail.com with Microsoft SMTPSVC; Thu, 5 Sep 2002 15:18:04 -0700 Received: from 32.94.119.254 by sea2fd.sea2.hotmail.msn.com with HTTP; Thu, 05 Sep 2002 22:18:03 GMT X-Originating-IP: [32.94.119.254] From: "Paulo Santa Rita" To: obm-l@mat.puc-rio.br Subject: [obm-l] Primos Gaussianos Date: Thu, 05 Sep 2002 22:18:03 +0000 Mime-Version: 1.0 Content-Type: text/plain; charset=iso-8859-1; format=flowed Message-ID: X-OriginalArrivalTime: 05 Sep 2002 22:18:04.0271 (UTC) FILETIME=[1A944FF0:01C2552A] Sender: owner-obm-l@sucuri.mat.puc-rio.br Precedence: bulk Reply-To: obm-l@mat.puc-rio.br Ola Franklin e demais colegas desta lista ... OBM-L, Um complexo z=a+bi e dito ser um INTEIRO GAUSSIANO se "a" e "b" sao inteiros. Dentre os inteiros gaussianos, sao chamados PRIMOS GAUSSIANOS aqueles em que : 1) "a" e "b" nao sao nulos e "a^2 + b^2" e um primo ordinario. 2) "b=0", "a" e primo ordinario e a==3(mod 4) 3) "a=0", modulo(b) e primo ordinario e b==3(mod 4) Entenda X==Y(mod Z) como "X e congruente a Y modulo Z". Voce destacou um fragmento de uma mensagem minha ... Realmente, existem muitas perguntas sobre primos e outras indiretamente ligadas a eles que ainda nao tiveram uma resposta satisfatoria ... Por que isso ocorre ? Pode ser que seja por pura inabilidade nossa. Pode ser ! ... mas pode ser tambem porque nos ainda nao fizemos as perguntas corretas ... e pode ser que ainda nao fizemos as perguntas corretas porque ainda nao dispomos do instrumental conceitual com o qual haveremos de faze-las ... Um abraco Paulo Santa Rita 5,1916,050902 >From: "Franklin de Lima Marquezino" >Reply-To: obm-l@mat.puc-rio.br >To: >Subject: [obm-l] Re: 1 é primo? >Date: Thu, 5 Sep 2002 17:38:48 -0300 > > >Quando eu comecei a estudar os primos gaussianos fiquei felicissimo. De >cara > >descobri que 5=(2-i)(2+i) e que, portanto, por este angulo, 5 nao e >primo. > >Pensava que a balburdia ia acabar ... Ledo engano ! Afinal, em que >sentido > >os inteiros gaussianos esclarecem melhor a natureza dos numeros primos ? >O > >livro nao dizia ! Ninguem me respondeu ! > >Olá! > > Alguém pode comentar alguma coisa sobre esses primos gaussianos. Pelo >menos uma definição e alguma referência. É que eu estudo Ciência da >Computação e não tive isso na graduação ainda, nem sei se vou ter. > > Até mais, > Franklin > > > > > >---------------------------------------------------------------------------------------------------------------------------------------------- >CBPF - Centro Brasileiro de Pesquisas Físicas (Bolsista PIBIC/CNPq) >LNCC - Laboratório Nacional de Computação Científica (Grupo de Computação >Quântica) >GFT - Grupo de Física Teórica José Leite Lopes >http://virtual01.lncc.br/dcs/links/dcs_quantum_comp.html >http://gft.ucp.br/staff/franklin >--------------------------------------------------------------------------------------------------------------------------------------------- _________________________________________________________________ MSN Photos é a maneira mais fácil e prática de editar e compartilhar sua fotos: http://photos.msn.com.br ========================================================================= Instruções para entrar na lista, sair da lista e usar a lista em http://www.mat.puc-rio.br/~nicolau/olimp/obm-l.html O administrador desta lista é ========================================================================= From owner-obm-l@sucuri.mat.puc-rio.br Thu Sep 5 20:45:47 2002 Return-Path: Received: (from majordom@localhost) by sucuri.mat.puc-rio.br (8.9.3/8.9.3) id UAA18378 for obm-l-MTTP; Thu, 5 Sep 2002 20:44:41 -0300 Received: from traven9 (traven9.uol.com.br [200.221.4.35]) by sucuri.mat.puc-rio.br (8.9.3/8.9.3) with ESMTP id UAA18374 for ; Thu, 5 Sep 2002 20:44:38 -0300 Received: from u2z7z2 ([200.158.145.46]) by traven9 (8.9.1/8.9.1) with ESMTP id UAA05153 for ; Thu, 5 Sep 2002 20:46:59 -0300 (BRT) Message-ID: <003d01c25536$730caa40$2101a8c0@u2z7z2> From: "Wagner" To: References: <200209050019.g850JI717622@Euler.impa.br> Subject: [obm-l] =?iso-8859-1?Q?Re:_=5Bobm-l=5D_Re:_=5Bobm-l=5D_Re:_=5Bobm-l=5D_Re:_?= =?iso-8859-1?Q?=5Bobm-l=5D_O_problema_das_infinitas_solu=E7=F5es?= Date: Thu, 5 Sep 2002 20:46:26 -0300 Organization: Wagner MIME-Version: 1.0 Content-Type: text/plain; charset="iso-8859-1" Content-Transfer-Encoding: 8bit X-Priority: 3 X-MSMail-Priority: Normal X-Mailer: Microsoft Outlook Express 5.50.4133.2400 X-MimeOLE: Produced By Microsoft MimeOLE V5.50.4133.2400 Sender: owner-obm-l@sucuri.mat.puc-rio.br Precedence: bulk Reply-To: obm-l@mat.puc-rio.br Alo pessoal Só pra esclarecer o mau entendido (só percebi quando recebi o e-mail do leonardo mattos em que ele elogiava um livro de geometria que eu nunca escrevi). O meu nome é André Timpanaro e Wagner é o nome do meu pai (não quis criar um e-mail só pra mim). Notação: log n (a) = logaritmo natural de a Queria aproveitar para perguntar para o Paulo porque na solução dele : e^(PI)i = -1. Isso implicaria que e^i = (i.sen1 + cos1) ? Então um nº real a poderia ser elevado a i, tal que a^i = e^(log n (a))(i). O que implicaria que a^i = (i.sen (log n (a)) + cos (log n (a))) ? André T. ----- Original Message ----- From: "Eduardo Wagner" To: Sent: Friday, September 06, 2002 9:17 PM Subject: Re: [obm-l] Re: [obm-l] Re: [obm-l] Re: [obm-l] O problema das infinitas soluções > Caros amigos da lista: > > So para esclarecer, o Wagner(timpa@uol) > nao eh Eduardo Wagner, que alias sou eu. > Trata-se de um outro participante da lista que > gostaria de conhecer. > > Abracos > > E. Wagner > > ---------- > >From: "Paulo Santa Rita" > >To: obm-l@mat.puc-rio.br > >Subject: [obm-l] Re: [obm-l] Re: [obm-l] Re: [obm-l] O problema das infinitas > soluções > >Date: Wed, Sep 4, 2002, 6:54 PM > > > > > Ola Wagner e demais > > colegas desta lista ... OBM-L, > > > > O conceito de simplicidade e subjetivo... mas e bonita a sua solucao ! > > Todavia, e bom que se diga, e uma demonstracao de existencia, nao exibindo a > > "cara" ou "forma" da solucoes. > > > > Em verdade, esse jeito foi a primeira coisa que veio a minha cabeca, mas eu > > prefiri uma outra via, construtiva, porque assim eu forneceria elementos > > para verificacoes posteriores, coisa que uma simples prova de existencia nao > > concede ... > > > > A respeito de equacoes nao triviais existe uma questao bonita : > > > > Seja y=f(x) uma equacao do 5 GRAU, INCOMPLETA, isto e, na qual um ou mais > > dos coeficientes da equacao geral e(sao) nulo. Em que casos ela admite uma > > solucao algebrica, isto e, quando as solucoes podem ser expressas como > > operacoes algebricas sobre os seus coeficientes ? > > > > Um abraco > > Paulo Santa Rita > > 4,1852,040902 > > > >>From: "Wagner" > >>Reply-To: obm-l@mat.puc-rio.br > >>To: > >>Subject: [obm-l] Re: [obm-l] Re: [obm-l] O problema das infinitas soluções > >>Date: Wed, 4 Sep 2002 15:33:33 -0300 > >> > >>Oi pra todo mundo > >> > >>Muito bem Paulo você achou a resposta (o conjunto universo da equação é > >>"C"). Mas quando eu imaginei o problema eu pensei numa resposta mais > >>simples: > >> > >>Imagine uma equação do tipo: x^(a/b)+cx^((a/b)-1)+dx((a/b)-2)+...+n=0. Em > >>que a e b são números inteiros e a/b é uma fração > >>irredutível. Se y=x^(1/b). Logo: y^(a)+cy^(a-b)+...+n=0. Logo existem a > >>valores complexos para y que satisfazem a equação e consequentemente, a > >>valores para x. Considerando pi/1 como uma fração irredutível e n o nº de > >>casas decimais de pi. Logo: (pi)(10^n)/(10^n) é uma fração irredutível e > >>portanto existem (pi)(10^n) valores de x que satisfazem: x^pi - 5^pi + 3 = > >>0. Como pi é um nº irracional, ele tem infinitas casas decimais e portanto > >>a > >>equação do problema possui infinitas soluções complexas. > >> > >>OBS: Isso acontece com qualquer equação em que o índice a que x esta > >>elevado > >>é um nº irracional em pelo menos um de seus termos. > >> > >>André T. > >> > >> > >>----- Original Message ----- > >>From: "Paulo Santa Rita" > >>To: > >>Sent: Wednesday, September 04, 2002 9:42 AM > >>Subject: [obm-l] Re: [obm-l] O problema das infinitas soluções > >> > >> > >> > Ola Wagner e demais > >> > colegas desta lista ... OBM-L, > >> > > >> > Eu nao entendi bem a sua questao, pois PARECE-ME que voce esta se > >>referindo > >> > ao conjunto "C - R". Mas nao tenho certeza. Talvez voce esteja pensando > >>em > >> > > >> > X^pi - 5*[X^(pi-1)] + 3 = 0 > >> > X^pi - [5*(X^pi)]/X + 3 = 0 > >> > X^pi(1 - 5/X) = -3 > >> > X^pi = 3X/(5-X) ... (A) > >> > > >> > X=a*[e^(Ti)] => X^pi = (a^pi)*{[e^(pi*i)]^T} > >> > X^pi=(a^pi)*{[cos(pi)+i*sen(pi)]^T} > >> > X^pi=(a^pi)*[(-1)^T] ... (B) > >> > > >> > (B) em (A) : > >> > > >> > (a^pi)*[(-1)^T]=3X/(5-X) > >> > (a^pi)*[i^2T]=3X/(5-X) > >> > X={[5*(a^pi)]*[i^(2T)]}/{3+[(a^pi)*[i^(2T)]]} > >> > > >> > Variando "a" e "T" convenientemente teremos uma infinidade de numeros > >>que > >> > satisfazem a equacao proposta. > >> > > >> > Um abraco > >> > Paulo Santa Rita > >> > 4,0941,040902 > >> > > >> > >From: "Wagner" > >> > >Reply-To: obm-l@mat.puc-rio.br > >> > >To: > >> > >Subject: [obm-l] O problema das infinitas soluções > >> > >Date: Mon, 2 Sep 2002 16:34:50 -0300 > >> > > > >> > >Esse é o meu primeiro problema na lista > >> > > > >> > >Notação: > >> > >- a^(b) = a elevado a potência b > >> > >- PI = o nº pi > >> > > > >> > >Prove que a equação: x^(PI)-5x^(PI-1)+3=0. Possui infinitas soluções > >> > >complexas. > >> > > > >> > > > >> > > André T. > >> > > >> > > >> > > >> > > >> > _________________________________________________________________ > >> > Tenha você também um MSN Hotmail, o maior webmail do mundo: > >> > http://www.hotmail.com/br > >> > > >> > > >>========================================================================= > >> > Instruções para entrar na lista, sair da lista e usar a lista em > >> > http://www.mat.puc-rio.br/~nicolau/olimp/obm-l.html > >> > O administrador desta lista é > >> > > >>========================================================================= > >> > > >> > >> > >>========================================================================= > >>Instruções para entrar na lista, sair da lista e usar a lista em > >>http://www.mat.puc-rio.br/~nicolau/olimp/obm-l.html > >>O administrador desta lista é > >>========================================================================= > > > > > > O > > > > > > ========================================================================= > > Instruções para entrar na lista, sair da lista e usar a lista em > > http://www.mat.puc-rio.br/~nicolau/olimp/obm-l.html > > O administrador desta lista é > > ========================================================================= > ========================================================================= > Instruções para entrar na lista, sair da lista e usar a lista em > http://www.mat.puc-rio.br/~nicolau/olimp/obm-l.html > O administrador desta lista é > ========================================================================= > ========================================================================= Instruções para entrar na lista, sair da lista e usar a lista em http://www.mat.puc-rio.br/~nicolau/olimp/obm-l.html O administrador desta lista é ========================================================================= From owner-obm-l@sucuri.mat.puc-rio.br Thu Sep 5 21:00:52 2002 Return-Path: Received: (from majordom@localhost) by sucuri.mat.puc-rio.br (8.9.3/8.9.3) id VAA18634 for obm-l-MTTP; Thu, 5 Sep 2002 21:00:35 -0300 Received: from traven10 (traven10.uol.com.br [200.221.4.45]) by sucuri.mat.puc-rio.br (8.9.3/8.9.3) with ESMTP id VAA18630 for ; Thu, 5 Sep 2002 21:00:33 -0300 Received: from u2z7z2 ([200.158.145.46]) by traven10 (8.9.1/8.9.1) with ESMTP id VAA25417 for ; Thu, 5 Sep 2002 21:00:44 -0300 (BRT) Message-ID: <001101c25538$acd27b40$2101a8c0@u2z7z2> From: "Wagner" To: Subject: [obm-l] A volta do futebol arte ! Date: Thu, 5 Sep 2002 21:02:21 -0300 Organization: Wagner MIME-Version: 1.0 Content-Type: multipart/alternative; boundary="----=_NextPart_000_000E_01C2551F.87208E00" X-Priority: 3 X-MSMail-Priority: Normal X-Mailer: Microsoft Outlook Express 5.50.4133.2400 X-MimeOLE: Produced By Microsoft MimeOLE V5.50.4133.2400 Sender: owner-obm-l@sucuri.mat.puc-rio.br Precedence: bulk Reply-To: obm-l@mat.puc-rio.br This is a multi-part message in MIME format. ------=_NextPart_000_000E_01C2551F.87208E00 Content-Type: text/plain; charset="iso-8859-1" Content-Transfer-Encoding: quoted-printable Oi pessoas!!! =20 Como hoje n=E3o estou muito inspirado hoje, vou propor um problema = simples ( para n=E3o dizer rid=EDculo ): Se voc=EA pegar uma bola de futebol e achatar todas as suas faces de = modo que elas fiquem retas, voc=EA ter=E1 um poliedro com 60 v=E9rtices (em uma bola de futebol de qualidade e que n=E3o tenha = sido comprada na 25 de mar=E7o, =E9 claro) . Como a=20 maioria sabe, a costura da bola de futebol forma pent=E1gonos e = hex=E1gonos regulares, arranjados de forma que em volta de cada = pent=E1gono existem 5 hex=E1gonos e em volta de cada hex=E1gono existem 3 = pent=E1gonos e 3 hex=E1gonos. Logo quantas faces de uma bola de futebol s=E3o pentagonais e quantas s=E3o hex=E1gonais ? Andr=E9 T. ------=_NextPart_000_000E_01C2551F.87208E00 Content-Type: text/html; charset="iso-8859-1" Content-Transfer-Encoding: quoted-printable
    Oi = pessoas!!!  =20
 
    Como hoje n=E3o = estou muito=20 inspirado hoje, vou propor um problema simples ( para n=E3o dizer = rid=EDculo=20 ):
 
    Se voc=EA pegar uma = bola de=20 futebol e achatar todas as suas faces de modo que elas fiquem retas, = voc=EA ter=E1=20 um poliedro
com 60 v=E9rtices (em uma bola de = futebol de=20 qualidade e que n=E3o tenha sido comprada na 25 de mar=E7o, =E9 = claro) . Como a=20
maioria sabe, a costura da bola de = futebol forma=20 pent=E1gonos e hex=E1gonos regulares, arranjados de forma que em volta = de cada=20 pent=E1gono
existem 5 hex=E1gonos e em volta de = cada hex=E1gono=20 existem 3 pent=E1gonos e 3 hex=E1gonos. Logo quantas faces de uma bola = de=20 futebol
s=E3o pentagonais e quantas s=E3o = hex=E1gonais=20 ?
 
    Andr=E9=20 T.
------=_NextPart_000_000E_01C2551F.87208E00-- ========================================================================= Instruções para entrar na lista, sair da lista e usar a lista em http://www.mat.puc-rio.br/~nicolau/olimp/obm-l.html O administrador desta lista é ========================================================================= From owner-obm-l@sucuri.mat.puc-rio.br Thu Sep 5 21:58:51 2002 Return-Path: Received: (from majordom@localhost) by sucuri.mat.puc-rio.br (8.9.3/8.9.3) id VAA19969 for obm-l-MTTP; Thu, 5 Sep 2002 21:58:25 -0300 Received: from pina.terra.com.br (pina.terra.com.br [200.176.3.17]) by sucuri.mat.puc-rio.br (8.9.3/8.9.3) with ESMTP id VAA19965 for ; Thu, 5 Sep 2002 21:58:23 -0300 Received: from penha.terra.com.br (penha.terra.com.br [200.176.3.43]) by pina.terra.com.br (Postfix) with ESMTP id 2663453025; Thu, 5 Sep 2002 21:59:08 -0300 (EST) Received: from abc (unknown [200.251.38.123]) (authenticated user ida.portella) by penha.terra.com.br (Postfix) with ESMTP id 8B8496829A; Thu, 5 Sep 2002 21:59:06 -0300 (EST) Message-ID: <00ab01c25540$ad93aec0$c18cd6c8@abc> From: "Margarida Lanna" To: , Subject: [obm-l] AJUDA Date: Thu, 5 Sep 2002 21:54:13 -0300 MIME-Version: 1.0 Content-Type: multipart/alternative; boundary="----=_NextPart_000_00A5_01C25526.C59527C0" X-Priority: 3 X-MSMail-Priority: Normal X-Mailer: Microsoft Outlook Express 5.50.4133.2400 X-MimeOLE: Produced By Microsoft MimeOLE V5.50.4133.2400 Sender: owner-obm-l@sucuri.mat.puc-rio.br Precedence: bulk Reply-To: obm-l@mat.puc-rio.br This is a multi-part message in MIME format. ------=_NextPart_000_00A5_01C25526.C59527C0 Content-Type: text/plain; charset="iso-8859-1" Content-Transfer-Encoding: quoted-printable PRECISO DE AJUDA, ALGU=C9M PODE ME ENVIAR CHARGES, CARTOONS, PIADAS OU ADIVINHAS = ENVOLVENDO MATEM=C1TICA? ABRA=C7OS, IDA ------=_NextPart_000_00A5_01C25526.C59527C0 Content-Type: text/html; charset="iso-8859-1" Content-Transfer-Encoding: quoted-printable
PRECISO DE AJUDA,
 
ALGU=C9M PODE ME ENVIAR CHARGES, = CARTOONS, PIADAS OU=20 ADIVINHAS ENVOLVENDO MATEM=C1TICA?
 
ABRA=C7OS,
 
IDA
------=_NextPart_000_00A5_01C25526.C59527C0-- ========================================================================= Instruções para entrar na lista, sair da lista e usar a lista em http://www.mat.puc-rio.br/~nicolau/olimp/obm-l.html O administrador desta lista é ========================================================================= From owner-obm-l@sucuri.mat.puc-rio.br Thu Sep 5 22:52:14 2002 Return-Path: Received: (from majordom@localhost) by sucuri.mat.puc-rio.br (8.9.3/8.9.3) id WAA20844 for obm-l-MTTP; Thu, 5 Sep 2002 22:52:10 -0300 Received: from ginsberg.uol.com.br (ginsberg.uol.com.br [200.221.4.48]) by sucuri.mat.puc-rio.br (8.9.3/8.9.3) with ESMTP id WAA20840 for ; Thu, 5 Sep 2002 22:52:07 -0300 Received: from u2z7z2 ([200.158.145.46]) by ginsberg.uol.com.br (8.9.1/8.9.1) with ESMTP id WAB27045 for ; Thu, 5 Sep 2002 22:51:13 -0300 (BRT) Message-ID: <002d01c25548$41a2f4c0$2101a8c0@u2z7z2> From: "Wagner" To: Subject: [obm-l] Resposta da pergunta do Paulo Santa Rita Date: Thu, 5 Sep 2002 22:53:53 -0300 Organization: Wagner MIME-Version: 1.0 Content-Type: multipart/alternative; boundary="----=_NextPart_000_002A_01C2552F.1BD89D80" X-Priority: 3 X-MSMail-Priority: Normal X-Mailer: Microsoft Outlook Express 5.50.4133.2400 X-MimeOLE: Produced By Microsoft MimeOLE V5.50.4133.2400 Sender: owner-obm-l@sucuri.mat.puc-rio.br Precedence: bulk Reply-To: obm-l@mat.puc-rio.br This is a multi-part message in MIME format. ------=_NextPart_000_002A_01C2552F.1BD89D80 Content-Type: text/plain; charset="iso-8859-1" Content-Transfer-Encoding: quoted-printable Alo Paulo, pessoal! PERGUNTA: Se f(x) =E9 uma fun=E7=E3o de 5=BA grau incompleta, quando = =E9 poss=EDvel encontrar as solu=E7=F5es algebricamente? RESPOSTA: Existem varias situa=E7=F5es em que isso =E9 poss=EDvel. = Considerando f(x) =3D ax^5 + bx^4 + ... + ex + f : -1=AA situa=E7=E3o: f =3D 0. Nesse caso zero =E9 solu=E7ao e pode-se = encontrar as outras ra=EDzes atrav=E9s de ax^4 + bx^3 + ... + e =3D 0, = que pode ser resolvida algebricamente pelo m=E9todo de Ferrari (observe que a = situa=E7=E3o e,f =3D 0 =3D> f =3D 0) -2=AA situa=E7=E3o: Sendo x =3D y + z. x^5=3D y^5 + 5(y^4)z + = 10(y^3)(z^2) + 10(y^2)(z^3) + 5y(z^4) + z^5 =3D y^5 + z^5 + 5(y^3+z^3)(yz) + 10 (y+z)((yz)^2) =3D x^5 =3D> Se a=3D1, e = =3D -10(yz)^2 e f =3D -(y^5 + z^5 + 5(y^3 + z^3)(yz)) =3D> Se f =E9 = diferente de zero, a equa=E7=E3o s=F3 pode ser resolvida algebricamente se e somente = se b,c,d =3D 0. (Ou quando essa condi=E7=E3o for satisfeita pela = substitui=E7=E3o da inc=F3gnita x por g-(b/a) em que g passa a ser a nova inc=F3gnita, = como na resposta das equa=E7=F5es de 3=BA e 4=BA grau) (N=E3o tenho muita certeza se a dedu=E7=E3o na 2=AA situa=E7=E3o esta = correta)=20 Andr=E9 T. ------=_NextPart_000_002A_01C2552F.1BD89D80 Content-Type: text/html; charset="iso-8859-1" Content-Transfer-Encoding: quoted-printable
    Alo Paulo, = pessoal!
 
    PERGUNTA: Se f(x) = =E9 uma fun=E7=E3o=20 de 5=BA grau incompleta, quando =E9 poss=EDvel encontrar as solu=E7=F5es = algebricamente?
 
    RESPOSTA: Existem = varias=20 situa=E7=F5es em que isso =E9 poss=EDvel. Considerando f(x) =3D = ax^5 + bx^4 + ... +=20 ex + f :
 
    -1=AA situa=E7=E3o: = f =3D 0. Nesse caso=20 zero =E9 solu=E7ao e pode-se encontrar as outras ra=EDzes atrav=E9s de = ax^4 + bx^3 + ...=20 + e =3D 0, que pode ser
resolvida algebricamente pelo m=E9todo = de Ferrari=20 (observe que a situa=E7=E3o e,f =3D 0 =3D> f =3D 0)
    -2=AA situa=E7=E3o: = Sendo x =3D y + z.=20 x^5=3D y^5 + 5(y^4)z + 10(y^3)(z^2) + 10(y^2)(z^3) + 5y(z^4) + z^5 = =3D
y^5 + z^5 + 5(y^3+z^3)(yz) + 10 = (y+z)((yz)^2) =3D x^5=20 =3D> Se a=3D1, e =3D -10(yz)^2 e f =3D -(y^5 + z^5 + 5(y^3 + = z^3)(yz)) =3D> Se f =E9=20 diferente de
zero, a equa=E7=E3o s=F3 pode ser = resolvida=20 algebricamente se e somente se b,c,d =3D 0. (Ou quando essa condi=E7=E3o = for=20 satisfeita pela substitui=E7=E3o
da inc=F3gnita x por g-(b/a) em que g = passa a ser a=20 nova inc=F3gnita, como na resposta das equa=E7=F5es de 3=BA e 4=BA = grau)
 
(N=E3o tenho muita certeza se a = dedu=E7=E3o na 2=AA=20 situa=E7=E3o esta correta)
 
    Andr=E9=20 T.
------=_NextPart_000_002A_01C2552F.1BD89D80-- ========================================================================= Instruções para entrar na lista, sair da lista e usar a lista em http://www.mat.puc-rio.br/~nicolau/olimp/obm-l.html O administrador desta lista é ========================================================================= From owner-obm-l@sucuri.mat.puc-rio.br Fri Sep 6 00:20:55 2002 Return-Path: Received: (from majordom@localhost) by sucuri.mat.puc-rio.br (8.9.3/8.9.3) id AAA21967 for obm-l-MTTP; Fri, 6 Sep 2002 00:20:39 -0300 Received: from sr1.terra.com.br (sr1.terra.com.br [200.176.3.16]) by sucuri.mat.puc-rio.br (8.9.3/8.9.3) with ESMTP id AAA21963 for ; Fri, 6 Sep 2002 00:20:37 -0300 Received: from penha.terra.com.br (penha.terra.com.br [200.176.3.43]) by sr1.terra.com.br (Postfix) with ESMTP id C2FA86F06C for ; Fri, 6 Sep 2002 00:21:22 -0300 (EST) Received: from stabel (unknown [200.203.38.231]) (authenticated user dudasta) by penha.terra.com.br (Postfix) with ESMTP id 4FCA4683BE for ; Fri, 6 Sep 2002 00:21:22 -0300 (EST) Message-ID: <001b01c25554$7950d6b0$0301a8c0@stabel> From: "Eduardo Casagrande Stabel" To: References: <002d01c25548$41a2f4c0$2101a8c0@u2z7z2> Subject: Re: [obm-l] Resposta da pergunta do Paulo Santa Rita Date: Fri, 6 Sep 2002 00:21:21 -0300 MIME-Version: 1.0 Content-Type: text/plain; charset="iso-8859-1" Content-Transfer-Encoding: 8bit X-Priority: 3 X-MSMail-Priority: Normal X-Mailer: Microsoft Outlook Express 6.00.2600.0000 X-MimeOLE: Produced By Microsoft MimeOLE V6.00.2600.0000 Sender: owner-obm-l@sucuri.mat.puc-rio.br Precedence: bulk Reply-To: obm-l@mat.puc-rio.br Caro André T., considere a equação de 5. grau incompleta x^5 + 4x^4 + 4x^3 + x + 2 = 0. Se fizermos a substituição x = y - 4, teremos y^5 - 16y^4 + 100y^3 - 304y^2 + 449y - 258 = 0. Portanto nem f=0, nem b,c,d=0 nas duas equações, logo ela não teria soluções algébricas pelo seu critério, mas veja que a primeira pode ser escrita assim x^3(x^2 + 4x + 4) + (x + 2) = x^3(x + 2)^2 + (x + 2) = (x + 2)(x^4 + 2x^3 + 1), logo todas as raízes são algébricas. Para alguns casos simples que eu testei o seu critério funcionava, por que a subst. x = y - (b/a), transformava a eq. numa fácil de resolver; deve ser algo parecido com o que você falou. Mas não sei se vai ser fácil de demonstrar que um tal critério funciona sem saber a teoria de Galois, a qual eu não conheço. Quanto à sua solução para o problema: mostrar que existem infinitos x complexos tais que x^(PI)-5x^(PI-1)+3=0. Você fala em considerar (pi)/1 como uma fração irredutível, o que quer dizer isso? Afinal (pi) não é inteiro. Depois você fala em (pi)10^n valores de x, mas como isso é possível se (pi) não é inteiro? Qual o sentido de 1.5 soluções? Pelo que compreendi a sua solução está baseada em aproximações de pi por números racionais, ou algo assim, mas não saquei como funciona de fato. É interessante que às vezes um método informal esconde muito mais coisa que um todo talhado e bonitinho. Um grande abraço! Eduardo. Porto Alegre, RS. From: Wagner Alo Paulo, pessoal! PERGUNTA: Se f(x) é uma função de 5º grau incompleta, quando é possível encontrar as soluções algebricamente? RESPOSTA: Existem varias situações em que isso é possível. Considerando f(x) = ax^5 + bx^4 + ... + ex + f : -1ª situação: f = 0. Nesse caso zero é soluçao e pode-se encontrar as outras raízes através de ax^4 + bx^3 + ... + e = 0, que pode ser resolvida algebricamente pelo método de Ferrari (observe que a situação e,f = 0 => f = 0) -2ª situação: Sendo x = y + z. x^5= y^5 + 5(y^4)z + 10(y^3)(z^2) + 10(y^2)(z^3) + 5y(z^4) + z^5 = y^5 + z^5 + 5(y^3+z^3)(yz) + 10 (y+z)((yz)^2) = x^5 => Se a=1, e = -10(yz)^2 e f = -(y^5 + z^5 + 5(y^3 + z^3)(yz)) => Se f é diferente de zero, a equação só pode ser resolvida algebricamente se e somente se b,c,d = 0. (Ou quando essa condição for satisfeita pela substituição da incógnita x por g-(b/a) em que g passa a ser a nova incógnita, como na resposta das equações de 3º e 4º grau) (Não tenho muita certeza se a dedução na 2ª situação esta correta) André T. ========================================================================= Instruções para entrar na lista, sair da lista e usar a lista em http://www.mat.puc-rio.br/~nicolau/olimp/obm-l.html O administrador desta lista é ========================================================================= From owner-obm-l@sucuri.mat.puc-rio.br Fri Sep 6 02:05:53 2002 Return-Path: Received: (from majordom@localhost) by sucuri.mat.puc-rio.br (8.9.3/8.9.3) id CAA23286 for obm-l-MTTP; Fri, 6 Sep 2002 02:04:59 -0300 Received: from salmon.bol.com.br (salmon.bol.com.br [200.221.24.15]) by sucuri.mat.puc-rio.br (8.9.3/8.9.3) with ESMTP id CAA23282 for ; Fri, 6 Sep 2002 02:04:56 -0300 Received: from bol.com.br (200.221.24.137) by salmon.bol.com.br (5.1.071) id 3D63EEF60050273B for obm-l@mat.puc-rio.br; Fri, 6 Sep 2002 02:05:00 -0300 Date: Fri, 6 Sep 2002 02:03:52 -0300 Message-Id: Subject: [obm-l] =?iso-8859-1?q?3_problemas_ol=EDmpicos?= MIME-Version: 1.0 Content-Type: multipart/mixed; boundary="_=__=_XaM3_Boundary.1031288632.2A.773740.42.5330.52.42.101010.1321857246" From: "fredericogomes" To: obm-l@mat.puc-rio.br X-XaM3-API-Version: 2.4.3.4.4 X-SenderIP: 150.161.2.4 Sender: owner-obm-l@sucuri.mat.puc-rio.br Precedence: bulk Reply-To: obm-l@mat.puc-rio.br --_=__=_XaM3_Boundary.1031288632.2A.773740.42.5330.52.42.101010.1321857246 Content-Type: text/plain;charset="iso-8859-1" Content-Transfer-Encoding: quoted-printable 1-(Ucr=E2nia 1992)- Demonstrar que n=E3o existem solu=E7=F5es reais do sistema: { x^2 + 4yz + 2z =3D0 { x + 2xy + 2z^2 =3D0 { 2xz + y^2 + y + 1 =3D0 2-(China 1993) Achar todas as ternas (x,y,z) de inteiros n=E3o negativos tais que: 7^x + 1 =3D 3^y + 5^z. obs: =E9 =F3bvio que (0,0,0) e (1,1,1) s=E3o solu=E7=F5es e que n=E3o temos mais nenhuma solu=E7=E3o que envolva inteiro(s) nulo(s), neste caso podemos admitir x,y,z >=3D1 3-(Iran 1993) Encontrar todos os primos =EDmpares p tais que [ 2^(p-1) - 1 ] / p =E9 um quadrado perfeito Ficarei imensamente grato se tiver pelo menos um destes tr=EAs resolvidos. []=B4s Frederico. __________________________________________________________________________ AcessoBOL, s=F3 R$ 9,90! O menor pre=E7o do mercado! Assine j=E1! http://www.bol.com.br/acessobol --_=__=_XaM3_Boundary.1031288632.2A.773740.42.5330.52.42.101010.1321857246 Content-Type: text/plain; name="000000GR" Content-Transfer-Encoding: base64 UFJFQ0lTTyBERSBBSlVEQSwNCg0KQUxHVclNIFBPREUgTUUgRU5WSUFSIENIQVJHRVMsIENB UlRPT05TLCBQSUFEQVMgT1UgQURJVklOSEFTIEVOVk9MVkVORE8gTUFURU3BVElDQT8NCg0K QUJSQcdPUywNCg0KSURBDQo= --_=__=_XaM3_Boundary.1031288632.2A.773740.42.5330.52.42.101010.1321857246-- ========================================================================= Instruções para entrar na lista, sair da lista e usar a lista em http://www.mat.puc-rio.br/~nicolau/olimp/obm-l.html O administrador desta lista é ========================================================================= From owner-obm-l@sucuri.mat.puc-rio.br Fri Sep 6 03:30:08 2002 Return-Path: Received: (from majordom@localhost) by sucuri.mat.puc-rio.br (8.9.3/8.9.3) id DAA24385 for obm-l-MTTP; Fri, 6 Sep 2002 03:29:47 -0300 Received: from smtp-6.ig.com.br (smtp-6.ig.com.br [200.226.132.155]) by sucuri.mat.puc-rio.br (8.9.3/8.9.3) with SMTP id DAA24372 for ; Fri, 6 Sep 2002 03:29:45 -0300 From: ezer@ig.com.br Received: (qmail 1366 invoked from network); 6 Sep 2002 06:30:12 -0000 Received: from shasta084253.ig.com.br (HELO almirfam) (200.151.84.253) by smtp-6.ig.com.br with SMTP; 6 Sep 2002 06:30:12 -0000 To: obm-l@mat.puc-rio.br Date: Wed, 1 Jan 1997 07:38:50 -0200 MIME-Version: 1.0 Subject: [obm-l] [Outro OFF??] Computacoes.. Message-ID: <32CA148A.28674.75BD9A@localhost> X-mailer: Pegasus Mail for Windows (v4.01) Content-type: text/plain; charset=ISO-8859-1 Content-description: Mail message body Content-Transfer-Encoding: 8bit X-MIME-Autoconverted: from Quoted-printable to 8bit by sucuri.mat.puc-rio.br id DAA24375 Sender: owner-obm-l@sucuri.mat.puc-rio.br Precedence: bulk Reply-To: obm-l@mat.puc-rio.br Eu sei que muitas das pessoas aqui da lista estao envolvidas com Computacao, programacao, etc. Por isso, achei conveniente fazer tal pergunta nesta lista, aonde, tenho certeza, terei um bom feedback. Eu gostaria que me fossem indicadas boas listas de discussao sobre Computacao e/ou programacao C/C++, que fossem utilizadas por voces. Por exemplo, se conhecer uma boa lista sobre Inteligencia Artificial, sera mais do que bem-vindo! (Obs.: Pode ser por lista por e-mail ou via web) Desde já o meu obrigado, Ezer ========================================================================= Instruções para entrar na lista, sair da lista e usar a lista em http://www.mat.puc-rio.br/~nicolau/olimp/obm-l.html O administrador desta lista é ========================================================================= From owner-obm-l@sucuri.mat.puc-rio.br Fri Sep 6 03:30:08 2002 Return-Path: Received: (from majordom@localhost) by sucuri.mat.puc-rio.br (8.9.3/8.9.3) id DAA24376 for obm-l-MTTP; Fri, 6 Sep 2002 03:29:45 -0300 Received: from smtp-6.ig.com.br (smtp-6.ig.com.br [200.226.132.155]) by sucuri.mat.puc-rio.br (8.9.3/8.9.3) with SMTP id DAA24370 for ; Fri, 6 Sep 2002 03:29:43 -0300 From: ezer@ig.com.br Received: (qmail 1166 invoked from network); 6 Sep 2002 06:30:09 -0000 Received: from shasta084253.ig.com.br (HELO almirfam) (200.151.84.253) by smtp-6.ig.com.br with SMTP; 6 Sep 2002 06:30:09 -0000 To: obm-l@mat.puc-rio.br Date: Wed, 1 Jan 1997 07:38:51 -0200 MIME-Version: 1.0 Subject: [obm-l] [Meio OFF meio ON] Perguntas Message-ID: <32CA148B.20245.75BFD6@localhost> X-mailer: Pegasus Mail for Windows (v4.01) Content-type: text/plain; charset=ISO-8859-1 Content-description: Mail message body Content-Transfer-Encoding: 8bit X-MIME-Autoconverted: from Quoted-printable to 8bit by sucuri.mat.puc-rio.br id DAA24371 Sender: owner-obm-l@sucuri.mat.puc-rio.br Precedence: bulk Reply-To: obm-l@mat.puc-rio.br Ola caros colegas da lista, Envio-os abaixo algumas duvidas e questoes, que me interessaram muito, ou por saber se tratar de assunto fascinante, ou pelo meu desconhecimento completo dos fatos :c). 1) Stephen Wolfram ficou rico vendendo o seu software "Mathematica" para empresas como a Nasa, etc. Em que consistia este programa (e o que o tornava tão valioso?) 2) Em que consiste a lógica Paraconsistente? Quais as suas aplicações? (se possível, gostaria de receber algum exemplo prático de seu funcionamento interno) 3) O que sao os teoremas de Godel? O que eh a teoria geral da recursao? E analise nao-standard? De que modo tais resultados da logica se constituem alternativa ao Calculo, e como podem conter alem dos numeros infinitesimos, outros, infinitamente grandes? Agradeço Àquele que puder silenciar alguma(s) destas dúvidas. ========================================================================= Instruções para entrar na lista, sair da lista e usar a lista em http://www.mat.puc-rio.br/~nicolau/olimp/obm-l.html O administrador desta lista é ========================================================================= From owner-obm-l@sucuri.mat.puc-rio.br Fri Sep 6 07:30:51 2002 Return-Path: Received: (from majordom@localhost) by sucuri.mat.puc-rio.br (8.9.3/8.9.3) id HAA26843 for obm-l-MTTP; Fri, 6 Sep 2002 07:30:38 -0300 Received: from hotmail.com (f200.law7.hotmail.com [216.33.237.200]) by sucuri.mat.puc-rio.br (8.9.3/8.9.3) with ESMTP id HAA26812 for ; Fri, 6 Sep 2002 07:30:35 -0300 Received: from mail pickup service by hotmail.com with Microsoft SMTPSVC; Fri, 6 Sep 2002 03:31:20 -0700 Received: from 200.241.240.154 by lw7fd.law7.hotmail.msn.com with HTTP; Fri, 06 Sep 2002 10:31:19 GMT X-Originating-IP: [200.241.240.154] From: "marcelo oliveira" To: obm-l@mat.puc-rio.br Subject: [obm-l] 0 é imaginário puro? Date: Fri, 06 Sep 2002 10:31:19 +0000 Mime-Version: 1.0 Content-Type: text/plain; format=flowed Message-ID: X-OriginalArrivalTime: 06 Sep 2002 10:31:20.0377 (UTC) FILETIME=[8A4D5290:01C25590] Sender: owner-obm-l@sucuri.mat.puc-rio.br Precedence: bulk Reply-To: obm-l@mat.puc-rio.br Esta dúvida surgiu durante a última prova de matemática da AFA. Finalmente, pode-se considerar 0 como imaginário puro? Claramente a primeira idéia é não considerar 0 como imaginário puro, por pensamentos puramente algébricos. Entretanto pense no plano imaginário (plano de Argand-Gauss) e note que 0 (a origem do sistema) pertence ao eixo imaginário (e real também?!). Gostaria também de saber uma justificativa (se houver, caso não seja uma simples convenção) para o fato de 0 ser ou não ser imaginário puro. Até mais, Marcelo Rufino de Oliveira _________________________________________________________________ MSN Photos is the easiest way to share and print your photos: http://photos.msn.com/support/worldwide.aspx ========================================================================= Instruções para entrar na lista, sair da lista e usar a lista em http://www.mat.puc-rio.br/~nicolau/olimp/obm-l.html O administrador desta lista é ========================================================================= From owner-obm-l@sucuri.mat.puc-rio.br Fri Sep 6 08:15:49 2002 Return-Path: Received: (from majordom@localhost) by sucuri.mat.puc-rio.br (8.9.3/8.9.3) id IAA27591 for obm-l-MTTP; Fri, 6 Sep 2002 08:15:29 -0300 Received: from gorgo.centroin.com.br (gorgo.centroin.com.br [200.225.63.128]) by sucuri.mat.puc-rio.br (8.9.3/8.9.3) with ESMTP id IAA27587 for ; Fri, 6 Sep 2002 08:15:27 -0300 Received: from centroin.com.br (du128c.rjo.centroin.com.br [200.225.58.128]) (authenticated bits=0) by gorgo.centroin.com.br (8.12.2/8.12.1) with ESMTP id g86BGZnH016354 for ; Fri, 6 Sep 2002 08:16:36 -0300 (BRT) Message-ID: <3D788F2D.4040103@centroin.com.br> Date: Fri, 06 Sep 2002 08:19:09 -0300 From: Augusto =?ISO-8859-1?Q?C=E9sar?= Morgado User-Agent: Mozilla/5.0 (Windows; U; Win98; en-US; rv:0.9.4.1) Gecko/20020508 Netscape6/6.2.3 X-Accept-Language: en-us MIME-Version: 1.0 To: obm-l@mat.puc-rio.br Subject: Re: [obm-l] 0 =?ISO-8859-1?Q?=E9=20imagin=E1rio?= puro? References: Content-Type: text/plain; charset=ISO-8859-1; format=flowed Content-Transfer-Encoding: 8bit Sender: owner-obm-l@sucuri.mat.puc-rio.br Precedence: bulk Reply-To: obm-l@mat.puc-rio.br Pegue qualquer livro decente (por exemplo, Churchill "Complex Variables" ou Boas "Mathematical Methods in the Physical Sciences") e você verá que eles definem imaginário puro como um complexo x+yi cuja parte real x é igual a 0. Agora, ser ou não ser não é uma questão de pensamentos. A questão é usar os nomes no sentido que a comunidade matemática emprega (afinal, é a comunidade que define a "norma culta"). Em relação ao primeiro livro que citei, estimo que 3 em cada 4 matemáticos com mais de 40 anos de idade tenham-no lido. marcelo oliveira wrote: > Esta dúvida surgiu durante a última prova de matemática da AFA. > > Finalmente, pode-se considerar 0 como imaginário puro? > > Claramente a primeira idéia é não considerar 0 como imaginário puro, > por pensamentos puramente algébricos. > > Entretanto pense no plano imaginário (plano de Argand-Gauss) e note > que 0 (a origem do sistema) pertence ao eixo imaginário (e real > também?!). > > Gostaria também de saber uma justificativa (se houver, caso não seja > uma simples convenção) para o fato de 0 ser ou não ser imaginário puro. > > > Até mais, > Marcelo Rufino de Oliveira > > _________________________________________________________________ > MSN Photos is the easiest way to share and print your photos: > http://photos.msn.com/support/worldwide.aspx > > ========================================================================= > Instruções para entrar na lista, sair da lista e usar a lista em > http://www.mat.puc-rio.br/~nicolau/olimp/obm-l.html > O administrador desta lista é > ========================================================================= > > ========================================================================= Instruções para entrar na lista, sair da lista e usar a lista em http://www.mat.puc-rio.br/~nicolau/olimp/obm-l.html O administrador desta lista é ========================================================================= From owner-obm-l@sucuri.mat.puc-rio.br Fri Sep 6 09:54:38 2002 Return-Path: Received: (from majordom@localhost) by sucuri.mat.puc-rio.br (8.9.3/8.9.3) id JAA29372 for obm-l-MTTP; Fri, 6 Sep 2002 09:54:15 -0300 Received: from Euler.impa.br (euler.impa.br [147.65.1.3]) by sucuri.mat.puc-rio.br (8.9.3/8.9.3) with ESMTP id JAA29369 for ; Fri, 6 Sep 2002 09:54:12 -0300 Received: from obm-01 (obm-01.impa.br [147.65.2.170]) by Euler.impa.br (8.11.6/8.11.6) with SMTP id g86Csw719340 for ; Fri, 6 Sep 2002 09:54:58 -0300 (EST) Message-Id: <3.0.5.32.20020906095528.007bece0@pop.impa.br> X-Sender: obm@pop.impa.br X-Mailer: QUALCOMM Windows Eudora Light Version 3.0.5 (32) Date: Fri, 06 Sep 2002 09:55:28 -0300 To: obm-l@mat.puc-rio.br From: Olimpiada Brasileira de Matematica Subject: [obm-l] Revista No. 3 da OIM. Mime-Version: 1.0 Content-Type: text/plain; charset="iso-8859-1" Content-Transfer-Encoding: 8bit X-MIME-Autoconverted: from quoted-printable to 8bit by sucuri.mat.puc-rio.br id JAA29370 Sender: owner-obm-l@sucuri.mat.puc-rio.br Precedence: bulk Reply-To: obm-l@mat.puc-rio.br Caros(as) amigos(as) da lista, Ya esta en linea el numero 3 de la Revista Escolar de la Olimpiada Iberoamericana de Matematica. La direccion es: http://www.campus-oei.org/oim/revistaoim/numero3.htm Los contenidos son: Articulos, Notas y Lecciones de preparacion olimpica Abderrahim Ouardini. Máximos y mínimos sin derivación. Tres ejemplos (versión española de F.Bellot) Problemas para alumnos de Educación Media y de Olimpiadas Problemas de la Fase nacional de la Olimpiada de Croacia 2002, correspondientes a los grados 3 y 4. Problemas para los más jóvenes Presentamos los problemas propuestos en la X Olimpiada Provincial de E.S.O. (Enseñanza Secundaria Obligatoria). Problemas resueltos Soluciones a los problemas propuestos para los más jovenes en nuestro número anterior. Problemas propuestos En este apartado se invita a los lectores a resolver cinco problemas y enviarnos sus soluciones. Las más originales serán publicadas. Divertimentos matemáticos El Administrato. Transcripción de Francisco Bellot. Abracos, Nelly. ========================================================================= Instruções para entrar na lista, sair da lista e usar a lista em http://www.mat.puc-rio.br/~nicolau/olimp/obm-l.html O administrador desta lista é ========================================================================= From owner-obm-l@sucuri.mat.puc-rio.br Fri Sep 6 10:01:38 2002 Return-Path: Received: (from majordom@localhost) by sucuri.mat.puc-rio.br (8.9.3/8.9.3) id KAA29525 for obm-l-MTTP; Fri, 6 Sep 2002 10:01:19 -0300 Received: from smtp.brturbo.com (smtp3.brturbo.com [200.199.201.47]) by sucuri.mat.puc-rio.br (8.9.3/8.9.3) with ESMTP id KAA29522 for ; Fri, 6 Sep 2002 10:01:17 -0300 Received: from x (unknown [200.181.93.165]) by smtp.brturbo.com (Postfix) with SMTP id 2252B11E595 for ; Fri, 6 Sep 2002 09:43:21 -0300 (BRT) Message-ID: <002401c255a5$a48a7f60$0201a8c0@x> From: "Jeremias de Paula Eduardo" To: Subject: [obm-l] um sistema Date: Fri, 6 Sep 2002 10:02:23 -0300 MIME-Version: 1.0 Content-Type: multipart/alternative; boundary="----=_NextPart_000_0021_01C2558C.7F1BB5A0" X-Priority: 3 X-MSMail-Priority: Normal X-Mailer: Microsoft Outlook Express 5.00.2919.6600 X-MimeOLE: Produced By Microsoft MimeOLE V5.00.2919.6600 Sender: owner-obm-l@sucuri.mat.puc-rio.br Precedence: bulk Reply-To: obm-l@mat.puc-rio.br This is a multi-part message in MIME format. ------=_NextPart_000_0021_01C2558C.7F1BB5A0 Content-Type: text/plain; charset="iso-8859-1" Content-Transfer-Encoding: quoted-printable Sabendo que ax-by=3D1 e que ay+bx=3D0, prove que x=3D a/a^2 +b^2 e y =3D = -b/a^2+b^2 ------=_NextPart_000_0021_01C2558C.7F1BB5A0 Content-Type: text/html; charset="iso-8859-1" Content-Transfer-Encoding: quoted-printable
Sabendo que ax-by=3D1 e que ay+bx=3D0, = prove que x=3D=20 a/a^2 +b^2 e y =3D -b/a^2+b^2
------=_NextPart_000_0021_01C2558C.7F1BB5A0-- ========================================================================= Instruções para entrar na lista, sair da lista e usar a lista em http://www.mat.puc-rio.br/~nicolau/olimp/obm-l.html O administrador desta lista é ========================================================================= From owner-obm-l@sucuri.mat.puc-rio.br Fri Sep 6 10:41:05 2002 Return-Path: Received: (from majordom@localhost) by sucuri.mat.puc-rio.br (8.9.3/8.9.3) id KAA31078 for obm-l-MTTP; Fri, 6 Sep 2002 10:40:52 -0300 Received: from gorgo.centroin.com.br (gorgo.centroin.com.br [200.225.63.128]) by sucuri.mat.puc-rio.br (8.9.3/8.9.3) with ESMTP id KAA31074 for ; Fri, 6 Sep 2002 10:40:48 -0300 Received: from centroin.com.br (du133c.rjo.centroin.com.br [200.225.58.133]) (authenticated bits=0) by gorgo.centroin.com.br (8.12.2/8.12.1) with ESMTP id g86DfsnH024707 for ; Fri, 6 Sep 2002 10:41:54 -0300 (BRT) Message-ID: <3D78B134.9010503@centroin.com.br> Date: Fri, 06 Sep 2002 10:44:20 -0300 From: Augusto =?ISO-8859-1?Q?C=E9sar?= Morgado User-Agent: Mozilla/5.0 (Windows; U; Win98; en-US; rv:0.9.4.1) Gecko/20020508 Netscape6/6.2.3 X-Accept-Language: en-us MIME-Version: 1.0 To: obm-l@mat.puc-rio.br Subject: Re: [obm-l] Ajuda em fatorial. References: <20020901200651.76020.qmail@web40308.mail.yahoo.com> Content-Type: multipart/alternative; boundary="------------010305000903060304020401" Sender: owner-obm-l@sucuri.mat.puc-rio.br Precedence: bulk Reply-To: obm-l@mat.puc-rio.br --------------010305000903060304020401 Content-Type: text/plain; charset=ISO-8859-1; format=flowed Content-Transfer-Encoding: 8bit 1) m(m-1)!=m! m!/(m+1)!=1/(m+1) A equaçao fica (m+3)/[(m-2)(m+1)] = 6/35 6m^2 -41m -117=0 A unica soluçao inteira eh 9. 2)a) [(m + 2)! - (m + 1)!] m! = 24 ² (m+1)![(m+2)-1]m!=24^2 (m+1)! (m+1) m! = 24^2 (m+1)! (m+1)! = 24^2 (m+1)! = 24 m+1 = 4 m=3 b) (m + 3)! + (m + 2)! = 6 (m + 3)! - (m + 2)! Divida numerador e denominador por (m+2)! Fica [(m+3)+1]/ [(m+3)-1] = 6 (m+4)/(m+2) = 6 Esta equação não possui solução inteira. c)????? 4) a) (n+2)! b) (n-7)! 3) n inteiro e também a) maior que ou igual a 0 b) maior que ou igual a 5 c) maior que ou igual a -1 Sharon Guedes wrote: > Olá pessoal ! > > Alguém poderia me ajudar nestas questões? > > 1) (UnB)Sendo m . (m + 3) . (m - 1)! = 6 e m > 0, o valor de m é: > > (m - 2) . (m + 1)! 35 > > a) 9 > > b) 12 > > c) 15 > > d) 18 > > 2)(URCAMP)Resolva as equações: > > a) [(m + 2)! - (m + 1)!] m! = 24 ² > > b) (m + 3)! + (m + 2)! = 6 > > (m + 3)! - (m + 2)! > > c) (n + 2)! + (n + 1)! = 21 > > 1 . 2 . 3 ... (n - 3) (n - 2) (n - 1)! > > 3)(URCAMP)Dê o domínio de cada uma destas expresões: > > a) n! > > b) (n - 5)! > > c) (n + 1)! (n + 1)! > > (URCAMP)Escreva os produtos empregando a notação fatorial. > > a)(n + 2) . (n + 1) . n . (n - 1) . ... . 2 .1 > > b)(n -7). (n - 8). (n - 9). ... . 2 .1 > > At. Sharon. > > > > ------------------------------------------------------------------------ > Yahoo! PageBuilder - O super > editor para criação de sites: é grátis, fácil e rápido. --------------010305000903060304020401 Content-Type: text/html; charset=us-ascii Content-Transfer-Encoding: 7bit 1) m(m-1)!=m!
m!/(m+1)!=1/(m+1)
A equaçao fica  (m+3)/[(m-2)(m+1)] = 6/35
6m^2 -41m -117=0
A unica soluçao inteira eh  9.

2)a) [(m + 2)! - (m + 1)!] m! = 24 ²
(m+1)![(m+2)-1]m!=24^2
(m+1)! (m+1) m! = 24^2
(m+1)! (m+1)! = 24^2
(m+1)! = 24
m+1 = 4
m=3

b) (m + 3)! + (m + 2)! = 6

(m + 3)! - (m + 2)!

Divida numerador e denominador por  (m+2)!
Fica
[(m+3)+1]/ [(m+3)-1] = 6
(m+4)/(m+2) = 6
Esta equação não possui solução inteira.

c)?????

4) a) (n+2)!
b) (n-7)!

3) n inteiro e também
a) maior que ou igual a 0
b) maior que ou igual a 5
c) maior que ou igual a -1

Sharon Guedes wrote:

Olá pessoal !

Alguém poderia me ajudar nestas questões?

1) (UnB)Sendo m . (m + 3) . (m - 1)! = 6 e m > 0, o valor de m é:

                          (m - 2) . (m + 1)!    35

a) 9

b) 12

c) 15

d) 18

2)(URCAMP)Resolva as equações:

a) [(m + 2)! - (m + 1)!] m! = 24 ²

b) (m + 3)! + (m + 2)! = 6

(m + 3)! - (m + 2)!

c) (n + 2)! + (n + 1)! = 21

1 . 2 . 3 ... (n - 3) (n - 2) (n - 1)!

3)(URCAMP)Dê o domínio de cada uma destas expresões:

a) n!

b) (n - 5)!

c) (n + 1)! (n + 1)!

(URCAMP)Escreva os produtos empregando a notação fatorial.

a)(n + 2) . (n + 1) . n . (n - 1) . ... . 2 .1

b)(n -7). (n - 8). (n - 9). ... . 2 .1

At. Sharon.

 


Yahoo! PageBuilder - O super editor para criação de sites: é grátis, fácil e rápido.

--------------010305000903060304020401-- ========================================================================= Instruções para entrar na lista, sair da lista e usar a lista em http://www.mat.puc-rio.br/~nicolau/olimp/obm-l.html O administrador desta lista é ========================================================================= From owner-obm-l@sucuri.mat.puc-rio.br Fri Sep 6 11:17:03 2002 Return-Path: Received: (from majordom@localhost) by sucuri.mat.puc-rio.br (8.9.3/8.9.3) id LAA31932 for obm-l-MTTP; Fri, 6 Sep 2002 11:16:20 -0300 Received: from puma.unisys.com.br (ns2.unisys.com.br [200.220.64.7]) by sucuri.mat.puc-rio.br (8.9.3/8.9.3) with ESMTP id LAA31924 for ; Fri, 6 Sep 2002 11:16:11 -0300 Received: from josefran (riohiper01p194.uninet.com.br [200.220.2.194]) by puma.unisys.com.br (8.12.3/8.12.3) with SMTP id g86EGtjR024443 for ; Fri, 6 Sep 2002 11:16:56 -0300 (EST) X-Spam-Filter: check_local@puma.unisys.com.br by digitalanswers.org Message-ID: <006501c25555$e0e859a0$c202dcc8@josefran> From: "Jose Francisco Guimaraes Costa" To: References: <32CA148B.20245.75BFD6@localhost> Subject: [obm-l] =?iso-8859-1?Q?Perguntas_=28o_que_=E9_o_Mathematica=3F=29?= Date: Fri, 6 Sep 2002 00:31:18 -0300 MIME-Version: 1.0 Content-Type: text/plain; charset="iso-8859-1" Content-Transfer-Encoding: 8bit X-Priority: 3 X-MSMail-Priority: Normal X-Mailer: Microsoft Outlook Express 5.00.2314.1300 X-MimeOLE: Produced By Microsoft MimeOLE V5.00.2314.1300 Sender: owner-obm-l@sucuri.mat.puc-rio.br Precedence: bulk Reply-To: obm-l@mat.puc-rio.br Para saber em que consiste - e não "consistia" - o Mathematica, vá até http://www.wolfram.com/ JF -----Mensagem Original----- De: Para: Enviada em: Quarta-feira, 1 de Janeiro de 1997 06:38 Assunto: [obm-l] [Meio OFF meio ON] Perguntas > Ola caros colegas da lista, > > Envio-os abaixo algumas duvidas e questoes, que me interessaram > muito, > ou por saber se tratar de assunto fascinante, ou pelo meu > desconhecimento > completo dos fatos :c). > > > 1) Stephen Wolfram ficou rico vendendo o seu software > "Mathematica" para > empresas como a Nasa, etc. Em que consistia este programa (e o > que o > tornava tão valioso?) > ========================================================================= Instruções para entrar na lista, sair da lista e usar a lista em http://www.mat.puc-rio.br/~nicolau/olimp/obm-l.html O administrador desta lista é ========================================================================= From owner-obm-l@sucuri.mat.puc-rio.br Fri Sep 6 11:17:04 2002 Return-Path: Received: (from majordom@localhost) by sucuri.mat.puc-rio.br (8.9.3/8.9.3) id LAA31908 for obm-l-MTTP; Fri, 6 Sep 2002 11:15:56 -0300 Received: (from nicolau@localhost) by sucuri.mat.puc-rio.br (8.9.3/8.9.3) id LAA31903 for obm-l@mat.puc-rio.br; Fri, 6 Sep 2002 11:15:55 -0300 Date: Fri, 6 Sep 2002 11:15:54 -0300 From: "Nicolau C. Saldanha" To: obm-l@mat.puc-rio.br Subject: Re: [obm-l] A volta do futebol arte ! Message-ID: <20020906111554.A31784@sucuri.mat.puc-rio.br> References: <001101c25538$acd27b40$2101a8c0@u2z7z2> Mime-Version: 1.0 Content-Type: text/plain; charset=iso-8859-1 Content-Disposition: inline Content-Transfer-Encoding: 8bit User-Agent: Mutt/1.2.5i In-Reply-To: <001101c25538$acd27b40$2101a8c0@u2z7z2>; from timpa@uol.com.br on Thu, Sep 05, 2002 at 09:02:21PM -0300 Sender: owner-obm-l@sucuri.mat.puc-rio.br Precedence: bulk Reply-To: obm-l@mat.puc-rio.br On Thu, Sep 05, 2002 at 09:02:21PM -0300, Wagner wrote: > Oi pessoas!!! > > Como hoje não estou muito inspirado hoje, vou propor um problema simples > ( para não dizer ridículo ): > > Se você pegar uma bola de futebol e achatar todas as suas faces de modo > que elas fiquem retas, você terá um poliedro com 60 vértices (em uma bola > de futebol de qualidade e que não tenha sido comprada na 25 de março, é > claro) . Como a maioria sabe, a costura da bola de futebol forma > pentágonos e hexágonos regulares, arranjados de forma que em volta de > cada pentágono existem 5 hexágonos e em volta de cada hexágono existem 3 > pentágonos e 3 hexágonos. Logo quantas faces de uma bola de futebol são > pentagonais e quantas são hexágonais ? Sem responder, este é um exemplo de poliedro arquimediano. Os poliedros arquimedianos são os que têm faces regulares e todos os vértices iguais (mais precisamente, dados dois vértices v1 e v2, existe uma isometria do poliedro levando v1 em v2). O leitor pode tentar listar todos os poliedros arquimedianos. Outra coisa legal sobre a bola de futebol: ela é um diagrama de Cayley para o grupo simples A5. Dados dois vértices da bola (!) existe uma única isometria preservando orientação levando um vértice no outro: este grupo de simetrias é isomorfo ao grupo das permutações pares de um conjunto de 5 elementos. Dá para pintar os hexágonos de 5 cores para ver isso... []s, N. ========================================================================= Instruções para entrar na lista, sair da lista e usar a lista em http://www.mat.puc-rio.br/~nicolau/olimp/obm-l.html O administrador desta lista é ========================================================================= From owner-obm-l@sucuri.mat.puc-rio.br Fri Sep 6 11:17:04 2002 Return-Path: Received: (from majordom@localhost) by sucuri.mat.puc-rio.br (8.9.3/8.9.3) id LAA31917 for obm-l-MTTP; Fri, 6 Sep 2002 11:15:58 -0300 Received: from web2.poli.usp.br (web2.poli.usp.br [143.107.106.102]) by sucuri.mat.puc-rio.br (8.9.3/8.9.3) with ESMTP id LAA31901 for ; Fri, 6 Sep 2002 11:15:53 -0300 Received: from web03.poli.usp.br ([143.107.106.18]) by web2.poli.usp.br with Microsoft SMTPSVC(5.0.2195.5329); Fri, 6 Sep 2002 11:16:35 -0300 Received: from apl03.poli.usp.br ([143.107.106.15]) by web03.poli.usp.br with Microsoft SMTPSVC(5.0.2195.5329); Fri, 6 Sep 2002 11:16:35 -0300 X-MimeOLE: Produced By Microsoft Exchange V6.0.5762.3 content-class: urn:content-classes:message MIME-Version: 1.0 Content-Type: multipart/mixed; boundary="----_=_NextPart_001_01C255B0.0157EAF2" Subject: RES: [obm-l] [Outro OFF??] Computacoes.. Date: Fri, 6 Sep 2002 11:16:34 -0300 Message-ID: <2B184DFE97456744924ACF58987D941D019352FB@apl03.poli.usp.br> X-MS-Has-Attach: X-MS-TNEF-Correlator: <2B184DFE97456744924ACF58987D941D019352FB@apl03.poli.usp.br> Thread-Topic: [obm-l] [Outro OFF??] Computacoes.. Thread-Index: AcJVc3FP8V8zaGc5S7qvqtFL1wvH5AAOTC+f From: "Edilon Ribeiro da Silva" To: X-OriginalArrivalTime: 06 Sep 2002 14:16:35.0032 (UTC) FILETIME=[01AA0580:01C255B0] Sender: owner-obm-l@sucuri.mat.puc-rio.br Precedence: bulk Reply-To: obm-l@mat.puc-rio.br This is a multi-part message in MIME format. ------_=_NextPart_001_01C255B0.0157EAF2 Content-Type: text/plain; charset="iso-8859-1" Content-Transfer-Encoding: quoted-printable No site do professor Gabriek Torres (www.gadrieltorres.com.br = ou www.clubedohardware.com.br = ) h=E1 um excelente f=F3rum sobre = assuntos da Computa=E7=E3o. O f=F3rum tem a seguinte divis=E3o por = assuntos: =20 * Hardware - Overclocking & Case; - Perif=E9ricos; - Hardware - Geral; - Not=EDcias;=20 - Recomenda=E7=F5es de Produtos Servi=E7os e Lojas; =20 - Placa M=E3e; =20 - Processadores e Chipsets; =20 - Placas de V=EDdeo; =20 - Multim=EDdia e Som Professional; =20 - Mem=F3rias; =20 - Armazenamento; =20 - Modems; =20 - Grava=E7=E3o de Cd's, DVD's, DivX, Mp3 e Edi=E7=E3o de = v=EDdeos; =20 - Port=E1teis; =20 - Drives e Manuais; =20 - Macintosh; =20 - Eletr=F4nica; =20 * Redes - Redes - Software; - Redes - Equipamentos; - Seguran=E7a; - Webmaster; - Internet; * Sistemas Operacionais - Registro do Windows - Regedit; - DOS e Windows 3.x; - Windows 9x/ME; - Windows NT/2000; - Windows XP; - Unix & GNU\Linux; - MacOS; * Programas - Praga Digital; - Softwares em Geral; - Office; - Internet - Softwares; - Jogos; - Programa=E7=E3o; =20 -----Mensagem original-----=20 De: ezer@ig.com.br [mailto:ezer@ig.com.br]=20 Enviada: qua 1/1/1997 07:38=20 Para: obm-l@mat.puc-rio.br=20 Cc:=20 Assunto: [obm-l] [Outro OFF??] Computacoes.. =09 =09 Eu sei que muitas das pessoas aqui da lista estao envolvidas com Computacao, programacao, etc. Por isso, achei conveniente fazer tal pergunta nesta lista, aonde, tenho certeza, terei um bom feedback. =09 Eu gostaria que me fossem indicadas boas listas de discussao sobre Computacao e/ou programacao C/C++, que fossem utilizadas por voces. Por exemplo, se conhecer uma boa lista sobre Inteligencia Artificial, sera mais do que bem-vindo! (Obs.: Pode ser por lista por e-mail ou via web) =09 =09 Desde j=E1 o meu obrigado, =09 Ezer =09 =09 ------_=_NextPart_001_01C255B0.0157EAF2 Content-Type: application/ms-tnef; name="winmail.dat" Content-Transfer-Encoding: base64 eJ8+IiMOAQaQCAAEAAAAAAABAAEAAQeQBgAIAAAA5AQAAAAAAADoAAEIgAcAGAAAAElQTS5NaWNy b3NvZnQgTWFpbC5Ob3RlADEIAQ2ABAACAAAAAgACAAEEgAEAKQAAAFJFUzogW29ibS1sXSBbT3V0 cm8gT0ZGPz9dIENvbXB1dGFjb2VzLi4APA0BBYADAA4AAADSBwkABgALABAAIgAFACoBASCAAwAO AAAA0gcJAAYACwAQACIABQAqAQEJgAEAIQAAAEJDQzJGMURDODNCODJENEM4RkZEMkYzOUYwNDM3 RjMxAHQHAQOQBgDAJQAANgAAAAMANgAAAAAAQAA5APLqVwGwVcIBHgA9AAEAAAAGAAAAUkVTOiAA AAACAUcAAQAAAC0AAABjPUJSO2E9IDtwPUVQVVNQO2w9QVBMMDMtMDIwOTA2MTQxNjM0Wi02NjQy NgAAAAAeAEkAAQAAACQAAABbb2JtLWxdIFtPdXRybyBPRkY/P10gQ29tcHV0YWNvZXMuLgBAAE4A AHHKl8f3uwEeAFoAAQAAAA8AAABlemVyQGlnLmNvbS5icgAAAgFbAAEAAAA7AAAAAAAAAIErH6S+ oxAZnW4A3QEPVAIAAAAAZXplckBpZy5jb20uYnIAU01UUABlemVyQGlnLmNvbS5icgAAAgFcAAEA AAAUAAAAU01UUDpFWkVSQElHLkNPTS5CUgAeAF0AAQAAAA8AAABlemVyQGlnLmNvbS5icgAAAgFe AAEAAAA7AAAAAAAAAIErH6S+oxAZnW4A3QEPVAIAAAAAZXplckBpZy5jb20uYnIAU01UUABlemVy QGlnLmNvbS5icgAAAgFfAAEAAAAUAAAAU01UUDpFWkVSQElHLkNPTS5CUgAeAGYAAQAAAAUAAABT TVRQAAAAAB4AZwABAAAADwAAAGV6ZXJAaWcuY29tLmJyAAAeAGgAAQAAAAUAAABTTVRQAAAAAB4A aQABAAAADwAAAGV6ZXJAaWcuY29tLmJyAAAeAHAAAQAAACQAAABbb2JtLWxdIFtPdXRybyBPRkY/ P10gQ29tcHV0YWNvZXMuLgACAXEAAQAAABsAAAABwlVzcU/xXzNoZzlLuq+q0UvXC8fkAA5ML58A HgB0AAEAAAAVAAAAb2JtLWxAbWF0LnB1Yy1yaW8uYnIAAAAAHgAaDAEAAAAYAAAARWRpbG9uIFJp YmVpcm8gZGEgU2lsdmEAHgAdDgEAAAAkAAAAW29ibS1sXSBbT3V0cm8gT0ZGPz9dIENvbXB1dGFj b2VzLi4AAgEJEAEAAAA/HwAAOx8AABO0AABMWkZ1DYHkBAMACgByY3BnMTI1gjIDQ2h0bWwxAzA/ AQMB9wqAAqQD4wIAY2jBCsBzZXQwIAcTAoD/EAMAUARWCFUHshHVDlEDAd0Q1zIGAAbDEdUzBEYQ 2fkS72Y0EG8XHBHjCO8J9/Y7Gx8OMDUcPx1DEeEMYM5jAFALCQFkMzYRYAulZDQgEAIqXA6yAZBn LxTwCqMR4yGINBTwPCEARE9DVFlQRSAASFRNTCBQVUIATElDICItLy+IVzNDJSBEVEQkNAgzLjIl IEVOIj7nIo0iLydhMTgjkCRCJq8bJ78qMDMhICkQRUFEXyltDvEqjy0PKJQ2DvA8UE1FVEEHsEEw AD1MIkcJ8ASQYXQFsCLRFxBPTlQmcFQwkAXhREV4EPFuZ2UGUnYPEzEy4QCQAiAgNi4wkC41Nzcz wDkxJp4PLm8oozPwI5BUSVRMCkUpbjQO8FtvYm2ILWxdOGBPdXQDYMAgT0ZGPz840AhQLG1wORAA 0G8HkC4u9SgONSOQLzb/NS8p5ThBLzvAK+8qPz+UNRFgPEIAT0RZIGRpcj19P7ByPwA/cwAhAzBC EWTObwDgQhEKsVxxGwBCEX8Q8AMwQnURYD8rIJFAL2fEOTYjkERJVkJJAAAzRIc/STY0R79E3zE0 pSjxRjFxIGYA0GUwkNcXKTFAAJB6TMAyR6sYMLcDsgHQRGlOOUAAkHQykKlDMCBwA2BmB5BzBbHG RwGgCIFrIFQFsBsgVwQgPaxF9SgoHDVHQS9vTEJCSUJXRP04K7EwMGiNGyBmMJAOsHRwOiUgwndY kC5nYWQIgT+w5VIDLgWgbS5RkCaRTko1WRFkP0JmWyALgHN0lHtIJAFSJMBOSyTw/1gvWT0aMAHx WyARID+wGoD9AEF1AyFWD0uvTL8Ko03P/07fYEJdL1mlU89U31XpaO/Paf9gL0cRI4EvQTw+YW8f Yn9jj2SfVm8hICZuYsxzcAKAQmgnYQFAcr/3c89033XiIAhgdi93P3hLv2dfaG9r/1b7CqNXz2MK QEpiCYBvEQFkdwrAZf9Zr1q/W8+A/4INXr9fz25P/29fcG9xf2Uehj9nD3zvam//kg+TH4kvbW+J /4sPjG+Nf+2VcimAYHVAZQ7wUm9GIvB1bSBlMjBZIAnwUHH2ZnVAFdByndFRMFGQMpAqYQQQdQIw bwQgZGGLOcacMTecMTNvLjlQ/56XeT96T3VqUHCiD6MfdVvPoDCcj0YiETBndQuAUHJ8aXYEAKED UMAFsZ+WOv+Qb5F/lQk/SQHAQlcKopUI+wqAKBwwK7ElgEeLlQ9Ff/9Gj0efrf9Jv7Mfl7+Yz5nf /5rvfB+sj7K/pX+mj7b/rv+/sA+xH8APwR+0T7VWVSlR/0JoCrEQAsjPyd8i2Xjhzs3fuBAjkCTA tgo/QnALkNJQ+Gx2bAJgiEDSUBPx0lD/C4ABAAIwIHEAANJQDNACUE91QAGwGjCT2WxpIHRp/i0g ctQTngAFQD9gAZGW/++6j7ufvK9PW0iCBb6Pv5//k8/Ff8aMAcA7sdEyxp87wL/Lu0JQzJPM/84P 0FxQtg//5W/YP4tP2g/bH+k/wh/DL//u3+/v8P/yD/Mf9C/1P/ZP//df+G/5f/qP+5/8r/2//s// /98A7wH/Aw8EHwUvBj8HT38IXwlvCn8Ljwyfe0+oX31eLTlQMuGPAENAax/gZ1kObyZhOfAPySZ4 aUP5n5BlO9z/3g/fH+Av5Qf/zHUN3+G56BHlv+bP598b7//p/+sP7B/tLyEPDu8P/yav/ye/KM8p 3yrvK/8tDy4fLy//MD8xTzJfM280fzWPNp83r/84vznPOt877zz/Pg8/H0Avf0E/Qk9DX0RvRX8R TxJWUGUS8GmhkWU5UaCQAHP/Fp8Xrxi/Gc8a30X/HP8eD/8fHyAvUV8iTyNfJG8lf1Z//0c/SE9c H10vXj9fT2BfYW//Yn9jj2SfZa9mv2fPaN9p7/9q/2wPbR9uL28/cE9xX3Jv/3N/dI91n3avd794 z3nfeu//e/99D34YErBJj1Qi3HaBgfJHEvBhbEvfTO9N/08P/1Affh9SP1NPVF9Vb4mPV4//WJ9Z r1q/jq9/X4BvlE+VX/+Wb5d/mI+Zn5qvm7+cz53f/57vn/+hD6Ifoy+kP6VPpl//p2+of6mPqp+r r6y/rc+u3z+v77D/sg9Ij4HPEoNOb+PS8EtQZGNpFmCEDLTf/7MPhC+FP4ZPh1+Ib7uPio//i5/K vsfMjmnQS8Xfwa+P7/+Q/5IPkx/JD7n/uw/Or8+//9DP0d/S79P/1Q/WH9cv2D//2U/aX9tv3H/d j96f36/gv//hz+Lf4+/k/+YP5x/oL+k/P+pP61/sb7Yvtz+BkFJlQUugbWVuZGFLQTeB7VBmNWVz IGSDcEBQcm9kdXRLsCByU8tgdmnzIvRhg3BM/G9quP68373twgvMp79e9clMMI1xL8WcvinDP8RP /8Vfxm/Hf8iPyZ/Kr8u/zM//zd/uj++fCL8JzwrfC+8M//8ODw8fEC8RPxJPE18UbxV//xaPF58Y rxm/Gs8b3xzvHf//Hw8gHyEvIj8jTyRfJW8mf/8nj/DvSlb13/bv9//5D/of//sv/D8oT/5f/28C nwGPN6//A68EvwXPBt8H7yifKa8n3/9Af0GPQp9Dr0S/Rc9G30fv/0j/Sg9LH0wvTT9OT09fUG// UX9Sj1OfVK9Vv1bPV99Y7/9Z/1sPXB9dL14/X09gX2FvE4E/t4pQbDyAYSBN+biRM2X1zy4PLx8w LzE//zJPM19iTzV/No85vzivco//b6873zzqPf8/D3W/aM9u//9i/2QPcz9rP2xPbV9ub31f/3CP cZ90z3O/iI9133bvPQ//eL95z32vfr9h/5Ffkm+Tf/+Uj5Wflq+Xv5jPmd+a75v//50Pnh+fL6A/ oU+iX6NvpH//pY+mn6evqL+pz6rfq++s//+uD68fsC+xP7JPZW9mevQQgY1gc3NhZG9y86EB9VBD aGlwc2V0//W/e+9p/4Dvgf+DD4Qfsw//hj+HT4hfiW+Kf8EvjJ+Nr/+Ov4/Pxz+0b7V/zN/N787/ /9AP0R/SL9M/1E/VX9Zv13//2I/Zn9qv27/cz93f3u/f///hD+IfKv+537rvxQ+9D74f/78v6F/B T8Jfw2/Ef8WPxp//x6/Iv8nPyt/L7+P/5Q/5f//6j/uf/K/9v/7P/98A7wH//wMPBB8FLwY/B08I XwlvCn//C48Mnw2vDr8PzxDfEe8S//8UDxUfFi8XPxhP5l9mebaAq2cjuLBkuNBWZ5FkHoD+b7l/ 58/o3+nv6v/sD+0f/xkv7z/wT/N/8m8p3yb/9Z//d9/4D/kfH18gbzGvGi8bP/8hzyLfI+8k/zPv Jx8oLyk//ypPK18sby1/Lo/3HzBfMW//NU82X0fvSP9KD0sfTC9NP/9OT09fUG9Rf1KPU59Ur1W/ /1bPV99Y71n/Ww9cH10vXj//X09gX2FvYn9jj2SfZa9mv5McTx1aTXW3MGltHsPEaWG4wVNvbR4A uACCZrgxaW9uYWwfP/8zTyFfOE85XzpvO39njz2f/z6vQd9Az3kfdj9D/y8vRm//R39un2+vgO9o j2mfcQ9yH/9zL3Q/gy92X3dveH95j3qf/3uvfL99z0V/f5+Ar4SPhZ//ly+YP5lPml+bb5x/nY+e n/+fr6C/oc+i36PvpP+mD6cf/6gvqT+qT6tfrG+tf66Pr5//sK+xv7LPs9+077X/aq9ru/JlbMFm M5RgHlBuf4KP/3Cfh4+In4mvir+2z4zfje//kR+QD8cPxC+TP35vla+Wv/+8j72fzt+3z7jfvv/A D8Ef/8Iv0R/ET8Vfxm/Hf8iPyZ//yq/Lv5S/zY/On9J/04/lH//mL+c/6E/pX+pv63/sj+2f/+6v 77/wz/Hf8u/z//UP9h//9y/4P/lP+l/7b/x//Y/+n3//rwC/Ac8C3wPvue8dWkFYcm1h42BuQG0J 8HT/Hy/Qb75/1W/Wf9eP2J8Er//av9vP3N/d797/Ei/hH8xP/+OP5J8KjwufEc8Frwa/Fg//Dg8P HxAvHx8STxNfFG8Vf/8WjxefGK8Zv+KfG48cnyB//yGPMx80LzU/Nk83XzhvOX//Oo87nzyvPb8+ zz/fQO9B//9DD0QfRS9GP0dPSF9Jb0p//0uPTJ9Nr06/T89Q31HvB8/5a6xvZLvAvF8efwyPI3// JI8lnyavUr8ozynfKu8r//8tD1/PLy8aXzGfMq9YL1k//19vU79Uz2OvW69cv13PbL//X+9g/2IP Yx9kL2U/Zk9nX/8wr2kvaj9uH28vgL+Bz4Lf/4PvhP+GD4cfiC+JP4pPi1//jG+Nf46Pj5+Qr5G/ ks+T3/+U75X/lw+YH5kvmj+bT5xfH51vnn+fj1XfuutHcmEkdmGgEGU3paEzbwIgV9AgQ2QncywQ IERWRKaTaXZYIaawTXAzIKZQRWSqaaWrdqWhZFfQb1gP/2wfWi9xH3Ivcz90T6Bfdm//d394j3mf eq+xT3zPZ/9/P/+AT6mvqr+w76Ffom+1L60v/64/r0++P7Fvsn+zj7Sfta//tr+3z7jffk+6r7u/ v5/Ar//SP9NP1F/Vb9Z/14/Yn9mv/9q/28/c393v3v/gD+Ef4i//4z/kT+Vf5m/nf+iP6Z/qr//r v+zP7d/u7+//8Q/yH/MvnfQ/IFeQo+/IglBvw4DhpaExdGVpqY+9n6uv/8Kfw6/Ev8XP9L/H78j/ yg//yx/MLwIfzk+5f9C/0c/6f//7jwG/9b/2zwX//f//DwAf/w8PAj8DTwRfBW8GfwePCJ//Ca/P zwt/DI8QbxF/Iw8kH/8lLyY/J08oXylvKn8rjyyf/y2vLr8vzzDfMe8y/zQPNR//Ni83PzhPOV86 bzt/PI89n/8+rz+/QM9B30LvQ/9FD/gvRRklRCBQdmVzp9FN8GFudWH6Tw5f/G8TX/8UbxV/Fo8X nxivGb8az03//xzvUx8fDwo/IX8ijw1vTI//Ur9Gf0ePVv9O/1APUR9gD/9hH1RPVV9Wb1d/WI9Z n1qv/yCPXH9dj2FvYn9Ff3Ufdi//dz94T3lfem97f3yPfZ9+r/9/v4DPgd+C74P/hQ+GH4cv/4g/ iU+KX4tvjH+Nj46fj68/kL+Rz5Lfo2+kfEsAY2n0bnSpcGhLf1+PTZ9kj/9ln2avZ7+Tz2nfau9u H20P/6PvoQ9wP1tvcq9zv5lvmn//q7+Uz5Xfm9+c753/nw+t//+hL6I/o0+kX6Vvpn+nj6if/3G/ qm+rf69fsG/B/8MPxB//xS/GP8dPyF/Jb8p/y4/Mn//Nr86/z8/Q39Hv0v/UD9Uf/9Yv1z/YT9lf 2m/bf9yP3Z//3q/fv+DP4d/i7+P/SQ+4FGBFbGV0cuQgwVBu+GljYZlPrV+bb7Jfs2//tH+1j+R/ t6+4v7vvut/0z//x774PqT/Af8GP6k/rX/yf/+V/5o/sv+3P7t/v7/7f8g/z8x/0JFVM+zm/0ggC B3/zCI/oRzI0lxEJPfbA/jBMTEn7OugicG4Q8WyYdmxi9oAQ8WYyEPGJmOBkZZjwMzYwC5AhEQB0 eHRi5CBiN2J97FlcbGkSkfwhadYtFHISw2UTECDoQAzg/mIJTPkP+h/7L/w/AB8BL+nkWVJlElBz /T/+TxOa/wPfBO8Dch9RD+IFTx9gCmvfHIALQxqPDL8PDFD0vybP/xbvvw8YvxnPKp8b7xz/MD// MU8yXzNvNH81jzafN684v/85zzrfO+88/z4PPx9AL0E//0JPQ19Eb0V/Ro9Hn0ivSb//Ss9L30zv Tf9PD1AfUS/nbwfofB4DVYFTb2Z0d/0toGXqLx9fIG8hfyZYCyX/Uh8jGSlxJx8oLyk/XV8rX/8s by1/Lo9if1MvVD9oH2kv/2o/a09sX21vbn9vj3Cfca//cr9zz3Tfde92/3gPeR96L/97P3xPfV9+ b39/gI+Bn4Kvf4O/hM+F34bvly+YNlcGRehxdWllwG0SYZkQWA//WR9aL1s/XE+Hb15vX39gj/9h n5MvY79kz2XfZu+YT4iv/4m/ne+e/6APoR+iL6M/pE//pV+mb6d/qI+pn6qvq7+sz/+t367vr/+x D7Ifsy+0P7VP/7Zft2+4f7mPup+7r4n/iw9RjBNTZWeaoGERECf8ZTfqH46vj7+Qz5Hfku//k/+V D5Yfly/IP5lPml+bb/+cf81fvj+/T9L/1A/VH9Yv/9c/2E/ZX9pv23/cj92f3q//37/gz+Hf4u/j /+UP5h/nL//oP+lP6l/rb+x/7Y/un++vP/C/8c/y3/PvVO/A31dl0GJtYXMVcHLCz8Pf/8Tvxf/H D/UPyS/KP8tPzF//AA/Of8+P0J/RrwUv9k/3X/8KzwvfDO8N/w8PEB8RLxI//xNPFF8VbxZ/F48Y nxmvGr//G88c3x3vHv8gDyEfIi8jP/8kTyVfJm8nfyiPKZ8qryu/G/fvwO5JjWAHsG5ldP/6j/uf /K/9v/7PLM8A7wH/5QMMOTMxVUwEPzffOj85OKMyNMBhAv8zQExJGwjqMIJwwmBCEGx2bCZiOLBC AWYyQgFpbmNXII1gMzYwPKBCEHSIeHRiLIBiN300asxsaUOhCdFpLUWCQ9PaZUQgIDCgPfBiP9wG b/8HegiPCZ89Hy4/LG9MT01fz05vT39Qj1GUU2n6QfohzzBfPl9Ssj+xT3AHsEiw8GlvbmFU4DJP M180b7U1fwo4Ly9A8kc8MAPx/i8721NQNz89fz/PBA9g7/9Hzwc/CE9Kr2S/Uo9Tn2pf/2tvbH9t j26fb69wv3HPct//c+90/3YPdx94L3k/ek97X/98b31/fo9/n4Cvgb+Cz4Pf/4Tvhf+HD4gfiS+K P4tPL58NVa9SwhBU4XJvIGTlkcBXQ0Fvd1VAweCRUfhlZGkyL1mfWq9bvzZv/2C/OI9hj2KfY6+Y n2XPZt//Z+9o/52/jZ+Or6NfpG+lf/+mj6efqK+pv6rPq9+s763//68PsB+xL7I/s0+0X7Vvtn// t4+4n7mvur+7z7zfve++/z/AD8Efwi6P/5tjj+BET4hTIGWSBzMueJMv/5Q/lU+WX5dvwo+Zj5qf m6//nL/OH57fn++g/6IP0z/Dz//E39jf2e/a/9wP3R/eL98//+BP4V/ib+N/5I/ln+av57//6M/p 3+rv6//tD+4f7y/wP//xT/Jf82/0f/WP9p/FH8Yvg8czkhY5eC9NRcif/8mvyr/Lz8zf9//O/9AP 0R//0i8DT9RP1V/Wb9d/CG/5P//6Tw4PDx8QLxE/Ek8TXxRv/xV/Fo8XnxivGb8azxvfHO//Hf8f DyAfIS8iPyNPJF8lb/8mfyePKJ8pryq/K8/6j/uf+/yrChAvDVAy4P3P/t//7/8A/wIPLS8ELwU/ Bk8HXzif/wl/Co8LnwyvPb8uby9/Q1//RG9Ff0aPR59Ir0m/Ss9L3/9M703/Tw9QH1EvUj9TT1Rf /1VvVn9Xj1ifWa9av1vPXN//Xe9e/2APYR9iL2M/ZE+PT/OQXpIWWFAzHzQvNT82T/83X2VfOX86 jzufPK9wbz7P/z/fQO9B/3WPZp9nr3svfD//fU9+X39vgH+Bj4Kfg6+Ev/+Fz4bfh++I/4oPix+M L40//45Pj1+Qb5F/ko+Tn5Svlb//ls+X35jvmf+bD5wfaD9pThBVbml4nb8mYW0FnwomMAlHTlVc XPpMx/B1yI9r72z/bg9vH/9wL3E/ck9zX3Rvqy92j3ef/3iveb+wT55fn2+177b/uA//uR+6L7s/ vE+9X75vv3/Aj//Bn8Kvw7/Ez8Xfxu/H/8kP/8ofyy/MP81Pzl/Pb9B/0Y9f0p/Try+voM/8lE2y QE/+U6W/ps+n36jvqf/VH6wfy60vrjw522FVTK9v4A9z4m/g0zI02HGuL9twTCZJtBrYwnBu6jFs dsxsYuDg6jFmMuox/QHAZW50MzYw5NDqQBB0eHRi1NBiN32Z3JpsaevRtQFpLe2ytewDZexQINjg 5iBi6Ax/sZ+yqrO/tM/lT9aP1L5QkHJvZ3KjcGFz2n/X24/cn92vCuBfL+ki72z6MK8hL+QL9cDf b+Wv5///rz8AH+//sm+zf/LfA+/0///2DwmPCp8Lrwy/Dc8O3w/v/xD/Eg8THxQvFT8WTxdfGG// GX8ajxufHK8dvx7PH98g7/8h/yMPJB8lLyY/J0/Xn9ivBzHE90DhAGEgRGln+mkBMGzab/jf+e/6 /96v/yi/4M8AzwHfAu80DwUPBh//By8IPzkvKf8rDz7PP99A7/9B/0MPRB9FL0Y/R09IX0lv/0p/ S49Mn02vTr9Pz1DfUe//Uv9UD1UfVi9XP1hPWV9abx9bf1yPK08sX9nDU29mlHR3PFBlaqBlbaUA /zuwLm8vfzCPMZ8yrzO/NM//Nd827zf/aW86HzsvPD89T/9uj18PYB90L3U/dk93X3hv/3l/eo97 n3yvfb9+z3/fgO//gf+DD4QfhS+GP4dPiF+Jb/+Kf4uPjJ+Nr46/j8+Q35Hv55L/YW/ZpU9m7gBw oGQP/2UfZi9nP2hPk29qb2t/bI//bZ+eT2+/cM9x33Lvo2+Ur/+Vv6kPqh+rL6w/rU+uX69v/7B/ sY+yn7OvtL+1z7bft+//uP+6D7sfvC+9P75Pv1/Ab//Bf8KPw5/Er8W/xs/H38jvX8n/ywjZ8Jb/ oRJJ7qFy/G5l7uBiyZjPmd+a75v//50Pyw+fL6A/oU+iX9a/pH//pY+mn6ev29/MT81f4X/ij//j n+Sv5b/mz+ff6O/p/+sP/+wf7S/uP+9P8F/xb/J/84//9J/1r/a/98/43/nv+v/8D//9H/4v/z8A TwFfAm/N75dO8EpvZ2/RL9I/00/UX//Vb9Z/14/Yn9mv2r8OL9zcf98/4EHgvxP/3YHe492/Iv/f nxbPBJ8FrxufHK8dvx7P/x/fIO8h/yMPJB8lLyY/J0//KF8pbyp/K48sny2vLr8vz/8w3zHvMv80 DzUfNi83PzhPPzlfOm8HD2J1F1/dVGNvwGxvcj0jMEGxmHA5GjtjZg+AOt875zxTVFRS3XBHGjti QxlQJnIIcGPQbWE6kGU36UdBM28IvDkPgkTfCun/CM8J30ofSy9MP01PTl8Kn/8Lrwy/Dc8O3w/v EP8SDxMf/0QvRT9a70BvQX9Cj14/UU//T49IH0kvZJ87/z0PUv9UDz9VH1YvVz9YT2WVY0FCTABP Q0tRVU9URTggZGlgEG6A3xF0eQJs3eFNQVJHSU4ALVJJR0hUOiB4MHB4Gixykd7xdSJjcGhccWxh r2+vPokyjjQ+MXB/Y1BESVZaDw92g1kLer9qo2xpMzYfXk8UvxXPBneCIk1lbiJzbqBlbSBgAGln sGluYWyCI36NMXFyvlIaOX4h0EBtP1u6Qlzf0YZmRGU6ZS8vh/pSiaQgZYAgckCDIC5f0PhtLmLe cD5vd+9o0XkxSltHIGlugG86jGxdP4PfhO+F/4cPiB+T5UVu4HZpYWRhib+Kzz3XQHF1YSAxL5ph OcQ5N3QQNzoz3TCNX/+Ob490kV+Sb5N/lI+Vn0ZlD98Al2+Yfz3Ib2JtLQRsQEcgdC5wdWP6Ld5A b40fnq+fv6DPod/5qihDY6PPpN8936hPqV+Hqm+rf0XdQXNzddAA/5Awra+uv6/Pm++O/aZzkTAI W091cqBvIE9G2EY/P5Ew3hBtpxB3cPNf0NEQLi6wT7Ffsm+9//+/D8AZt8973G7AaWdrbn6c+jBZ wS96fGdfeIfdMWN1/3bfui94/1mPbN/DuH1/fo+Xf5+Ar5aWddQgZWmaEaBlIG11acxQc3JApddB cL2Ac2/XQWGaIPvWoJdQINJgctCaQLkfzM/3j3S9gMxQb8DPwd+/7IKAfnZf4Jcg13KM4bzn28As x9egRtXgQ2V0Y72/3O/1v91QYAAg2MDX4OBwGSD+aNaR2R/aL490X9CXEIKA3xmwtjDW4BkQjHEg zFB2EPnXsHJntiGaQL/g2NHhr5/iv7/s2LPlIegwZGXgcPnosG5ovGAZMMYwjGDtofXosHLWkXWC 4OWv5r+PdIJi36FmZWVkYhkg/mvhn+tvv9/zP/RP9VrWUf8IgcxQGaCaQNbD6MEIkNaA/4Lgg0By UOBA13LyQNdB2LM/11HW4O/P8N+PdHJQc2P+dbYA28/3D/Ue1+CNINbgO9/YjFAvGYDgiq1gL0P8 KyvgcNbC+kX8T/1fj3R/vDCQANRA+wL/XwBv9Txwn+Sx3wAZML2Q5JNleILQvnBf8OBw1oBfwe5g Ze6h/++BmkD7UdivBj+PCwJTCL87Cc/1PEnoodJggsBuYy+XMBGvEr/1PEHGMGlm+/rglzBsDRJH ANbwj/DXUWu8YNbCYoLQLZcg7fBvTiEVbxZ/9TwoT2kgLv90AA7fD++PdOSg/BEZIQuj8w50C7Jl LY/igvDWYJch4CB3ZWIpGt8b7/U/vyOPJJ8lrya/J8+JGHP8EfpqR0ExgvD6AdZgHk8fX/+PdAJh gyCXQOBgKV8qbyh//zC/Mc/4aoxxM480nzKvNv//OA85H2K/yM9qn2uvz99tz/9u38gHcag+P0Jf zH9Hry23wjXLEkJPRFnLnWGRn0l/zcCa0D3Qc+BNTNSQAn1OIAAeADUQAQAAAD0AAAA8MkIxODRE RkU5NzQ1Njc0NDkyNEFDRjU4OTg3RDk0MUQwMTkzNTJGQkBhcGwwMy5wb2xpLnVzcC5icj4AAAAA HgBHEAEAAAAPAAAAbWVzc2FnZS9yZmM4MjIAAAsA8hABAAAAHwDzEAEAAABmAAAAUgBFAFMAJQAz AEEAIABbAG8AYgBtAC0AbABdACAAWwBPAHUAdAByAG8AIABPAEYARgAlADMARgAlADMARgBdACAA QwBvAG0AcAB1AHQAYQBjAG8AZQBzAC4ALgAuAEUATQBMAAAAAAALAPYQAAAAAEAABzCo0wqirFXC AUAACDAmwm8BsFXCAQMA3j+vbwAAAwDxPxYEAAAeAPg/AQAAABgAAABFZGlsb24gUmliZWlybyBk YSBTaWx2YQACAfk/AQAAAFAAAAAAAAAA3KdAyMBCEBq0uQgAKy/hggEAAAAAAAAAL089RVBVU1Av T1U9RElSRVRPUklBL0NOPVJFQ0lQSUVOVFMvQ049RURJTE9OLlNJTFZBAB4A+j8BAAAAFQAAAFN5 c3RlbSBBZG1pbmlzdHJhdG9yAAAAAAIB+z8BAAAAHgAAAAAAAADcp0DIwEIQGrS5CAArL+GCAQAA AAAAAAAuAAAAAwD9P+QEAAADABlAAAAAAAMAGkAAAAAAAwAdQAAAAAADAB5AAAAAAB4AMEABAAAA DQAAAEVESUxPTi5TSUxWQQAAAAAeADFAAQAAAA0AAABFRElMT04uU0lMVkEAAAAAHgAyQAEAAAAP AAAAZXplckBpZy5jb20uYnIAAB4AM0ABAAAADwAAAGV6ZXJAaWcuY29tLmJyAAAeADhAAQAAAA0A AABFRElMT04uU0lMVkEAAAAAHgA5QAEAAAACAAAALgAAAAsAKQAAAAAACwAjAAAAAAADAAYQPrW+ WwMABxBrBQAAAwAQEAAAAAADABEQAAAAAB4ACBABAAAAZQAAAE5PU0lURURPUFJPRkVTU09SR0FC UklFS1RPUlJFUyhXV1dHQURSSUVMVE9SUkVTQ09NQlI8SFRUUDovL1dXV0dBRFJJRUxUT1JSRVND T01CUk9VV1dXQ0xVQkVET0hBUkRXQVIAAAAAAgF/AAEAAAA9AAAAPDJCMTg0REZFOTc0NTY3NDQ5 MjRBQ0Y1ODk4N0Q5NDFEMDE5MzUyRkJAYXBsMDMucG9saS51c3AuYnI+AAAAAJ2s ------_=_NextPart_001_01C255B0.0157EAF2-- ========================================================================= Instruções para entrar na lista, sair da lista e usar a lista em http://www.mat.puc-rio.br/~nicolau/olimp/obm-l.html O administrador desta lista é ========================================================================= From owner-obm-l@sucuri.mat.puc-rio.br Fri Sep 6 11:37:31 2002 Return-Path: Received: (from majordom@localhost) by sucuri.mat.puc-rio.br (8.9.3/8.9.3) id LAA00555 for obm-l-MTTP; Fri, 6 Sep 2002 11:37:08 -0300 Received: from videira.terra.com.br (videira.terra.com.br [200.176.3.5]) by sucuri.mat.puc-rio.br (8.9.3/8.9.3) with ESMTP id LAA00538 for ; Fri, 6 Sep 2002 11:37:03 -0300 Received: from pavuna.terra.com.br (pavuna.terra.com.br [200.176.3.41]) by videira.terra.com.br (Postfix) with ESMTP id 14533E1230 for ; Fri, 6 Sep 2002 11:37:43 -0300 (EST) Received: from terra.com.br (webmail4.terra.com.br [200.176.3.179]) (authenticated user mgmelo) by pavuna.terra.com.br (Postfix) with ESMTP id 1B56E684D3; Fri, 6 Sep 2002 11:37:43 -0300 (EST) Date: Fri, 6 Sep 2002 12:37:37 -0200 Message-Id: Subject: [obm-l] =?iso-8859-1?q?Re=3A_=5Bobm=2Dl=5D_0_=E9_imagin=E1rio_puro=3F?= MIME-Version: 1.0 Content-Type: text/plain;charset="iso-8859-1" From: "Marcos Melo" To: "obm-l" Cc: "obm-l" X-XaM3-API-Version: 2.4 R5 B5 JSC SMTP AUTH X-SenderIP: 200.228.200.3 Content-Transfer-Encoding: 8bit X-MIME-Autoconverted: from quoted-printable to 8bit by sucuri.mat.puc-rio.br id LAA00550 Sender: owner-obm-l@sucuri.mat.puc-rio.br Precedence: bulk Reply-To: obm-l@mat.puc-rio.br Zero é o ponto de encontro entre o imaginário e o real. SDS, Marcos Melo. > ---------- Mensagem original ----------- > > De : owner-obm-l@sucuri.mat.puc-rio.br > Para : obm-l@mat.puc-rio.br > Cc : > Data : Fri, 06 Sep 2002 08:19:09 -0300 > Assunto : Re: [obm-l] 0 é imaginário puro? > > Pegue qualquer livro decente (por exemplo, Churchill "Complex Variab les" > ou Boas "Mathematical Methods in the Physical Sciences") e você ver á > que eles definem imaginário puro como um complexo x+yi cuja parte re al x > é igual a 0. > Agora, ser ou não ser não é uma questão de pensamentos. A questão é usar > os nomes no sentido que a comunidade matemática emprega (afinal, é a > comunidade que define a "norma culta"). > Em relação ao primeiro livro que citei, estimo que 3 em cada 4 > matemáticos com mais de 40 anos de idade tenham-no lido. > > marcelo oliveira wrote: > > > Esta dúvida surgiu durante a última prova de matemática da AFA. > > > > Finalmente, pode-se considerar 0 como imaginário puro? > > > > Claramente a primeira idéia é não considerar 0 como imaginário pur o, > > por pensamentos puramente algébricos. > > > > Entretanto pense no plano imaginário (plano de Argand- Gauss) e note > > que 0 (a origem do sistema) pertence ao eixo imaginário (e real > > também?!). > > > > Gostaria também de saber uma justificativa (se houver, caso não se ja > > uma simples convenção) para o fato de 0 ser ou não ser imaginário puro. > > > > > > Até mais, > > Marcelo Rufino de Oliveira > > > > _________________________________________________________________ > > MSN Photos is the easiest way to share and print your photos: > > http://photos.msn.com/support/worldwide.aspx > > > > ================================================================== ======= > > Instruções para entrar na lista, sair da lista e usar a lista em > > http://www.mat.puc-rio.br/~nicolau/olimp/obm-l.html > > O administrador desta lista é > > ================================================================== ======= > > > > > > > ==================================================================== ===== > Instruções para entrar na lista, sair da lista e usar a lista em > http://www.mat.puc-rio.br/~nicolau/olimp/obm-l.html > O administrador desta lista é > ==================================================================== ===== > > ========================================================================= Instruções para entrar na lista, sair da lista e usar a lista em http://www.mat.puc-rio.br/~nicolau/olimp/obm-l.html O administrador desta lista é ========================================================================= From owner-obm-l@sucuri.mat.puc-rio.br Fri Sep 6 12:14:02 2002 Return-Path: Received: (from majordom@localhost) by sucuri.mat.puc-rio.br (8.9.3/8.9.3) id MAA02334 for obm-l-MTTP; Fri, 6 Sep 2002 12:13:20 -0300 Received: from hotmail.com (oe50.law9.hotmail.com [64.4.8.22]) by sucuri.mat.puc-rio.br (8.9.3/8.9.3) with ESMTP id MAA02330 for ; Fri, 6 Sep 2002 12:13:18 -0300 Received: from mail pickup service by hotmail.com with Microsoft SMTPSVC; Fri, 6 Sep 2002 08:14:03 -0700 X-Originating-IP: [200.174.48.209] From: "Ana Carolina Boero" To: "obm-l" Subject: [obm-l] =?iso-8859-1?Q?D=FAvida?= Date: Fri, 6 Sep 2002 12:13:59 -0300 MIME-Version: 1.0 Content-Type: multipart/alternative; boundary="----=_NextPart_000_009A_01C2559E.E1A97C90" X-Priority: 3 X-MSMail-Priority: Normal X-Mailer: Microsoft Outlook Express 6.00.2600.0000 X-MimeOLE: Produced By Microsoft MimeOLE V6.00.2600.0000 Message-ID: X-OriginalArrivalTime: 06 Sep 2002 15:14:03.0710 (UTC) FILETIME=[093C85E0:01C255B8] Sender: owner-obm-l@sucuri.mat.puc-rio.br Precedence: bulk Reply-To: obm-l@mat.puc-rio.br This is a multi-part message in MIME format. ------=_NextPart_000_009A_01C2559E.E1A97C90 Content-Type: text/plain; charset="iso-8859-1" Content-Transfer-Encoding: quoted-printable Ol=E1 colegas da lista, Estou com uma d=FAvida e gostaria de saber se algu=E9m poderia me = ajudar: Suponha uma seq=FC=EAncia de n=FAmeros reais, crescente, tal que a = diferen=E7a entre termos sucessivos vai a zero.=20 Ser=E1 que existe um limite finito para essa seq=FC=EAncia? Muito obrigada, Carol ------=_NextPart_000_009A_01C2559E.E1A97C90 Content-Type: text/html; charset="iso-8859-1" Content-Transfer-Encoding: quoted-printable
Ol=E1 colegas da lista,
 
Estou com uma d=FAvida e gostaria de saber se algu=E9m poderia me = ajudar:
 
Suponha uma seq=FC=EAncia de n=FAmeros reais, crescente, tal que a = diferen=E7a=20 entre termos sucessivos vai a zero.
Ser=E1 que existe um limite finito para essa seq=FC=EAncia?
 
Muito obrigada,
 
Carol
------=_NextPart_000_009A_01C2559E.E1A97C90-- ========================================================================= Instruções para entrar na lista, sair da lista e usar a lista em http://www.mat.puc-rio.br/~nicolau/olimp/obm-l.html O administrador desta lista é ========================================================================= From owner-obm-l@sucuri.mat.puc-rio.br Fri Sep 6 12:26:28 2002 Return-Path: Received: (from majordom@localhost) by sucuri.mat.puc-rio.br (8.9.3/8.9.3) id MAA02774 for obm-l-MTTP; Fri, 6 Sep 2002 12:25:53 -0300 Received: from zeus.opendf.com.br (zeus.opengate.com.br [200.181.71.10]) by sucuri.mat.puc-rio.br (8.9.3/8.9.3) with ESMTP id MAA02769 for ; Fri, 6 Sep 2002 12:25:50 -0300 Received: from localhost (localhost.opengate.com.br [127.0.0.1]) by zeus.opendf.com.br (Postfix) with ESMTP id A592A3EA39 for ; Fri, 6 Sep 2002 12:26:32 -0300 (BRT) Received: by zeus.opendf.com.br (Postfix, from userid 48) id 3991D3EA6A; Fri, 6 Sep 2002 12:26:32 -0300 (BRT) From: "498 - Artur Costa Steiner" To: obm-l@mat.puc-rio.br Subject: Re: [obm-l] Re: [obm-l] 0 é imaginário puro? X-Mailer: NeoMail 1.25 X-IPAddress: 200.252.155.2 MIME-Version: 1.0 Content-Type: text/plain; charset=iso-8859-1 Message-Id: <20020906152632.3991D3EA6A@zeus.opendf.com.br> Date: Fri, 6 Sep 2002 12:26:32 -0300 (BRT) X-Virus-Scanned: by AMaViS new-20020517 Sender: owner-obm-l@sucuri.mat.puc-rio.br Precedence: bulk Reply-To: obm-l@mat.puc-rio.br > Zero é o ponto de encontro entre o imaginário e o real. > SDS, > > Marcos Melo. Eu acho que esta termo "imaginário", embora consagrado, é bastante infeliz. Os reais são tão imginários quanto os imaginários ou, caso se prefira, os imaginários são tão reais quanto os reais. Embora isto não seja importante, eu acho mais razoável admitir que zero é simultaneamente real e imaginário. O que efetivamente importa é que o conjunto dos complexos é um corpo e zero é seu elemento neutro da adição. Artur > > > ---------- Mensagem original ----------- > > > > De : owner-obm-l@sucuri.mat.puc-rio.br > > Para : obm-l@mat.puc-rio.br > > Cc : > > Data : Fri, 06 Sep 2002 08:19:09 -0300 > > Assunto : Re: [obm-l] 0 é imaginário puro? > > > > Pegue qualquer livro decente (por exemplo, Churchill "Complex Variab > les" > > ou Boas "Mathematical Methods in the Physical Sciences") e você ver > á > > que eles definem imaginário puro como um complexo x+yi cuja parte re > al x > > é igual a 0. > > Agora, ser ou não ser não é uma questão de pensamentos. A questão é > usar > > os nomes no sentido que a comunidade matemática emprega (afinal, é a > > > comunidade que define a "norma culta"). > > Em relação ao primeiro livro que citei, estimo que 3 em cada 4 > > matemáticos com mais de 40 anos de idade tenham-no lido. > > > > marcelo oliveira wrote: > > > > > Esta dúvida surgiu durante a última prova de matemática da AFA. > > > > > > Finalmente, pode-se considerar 0 como imaginário puro? > > > > > > Claramente a primeira idéia é não considerar 0 como imaginário pur > o, > > > por pensamentos puramente algébricos. > > > > > > Entretanto pense no plano imaginário (plano de Argand- > Gauss) e note > > > que 0 (a origem do sistema) pertence ao eixo imaginário (e real > > > também?!). > > > > > > Gostaria também de saber uma justificativa (se houver, caso não se > ja > > > uma simples convenção) para o fato de 0 ser ou não ser imaginário > puro. > > > > > > > > > Até mais, > > > Marcelo Rufino de Oliveira > > > > > > _________________________________________________________________ > > > MSN Photos is the easiest way to share and print your photos: > > > http://photos.msn.com/support/worldwide.aspx > > > > > > ================================================================== > ======= > > > Instruções para entrar na lista, sair da lista e usar a lista em > > > http://www.mat.puc-rio.br/~nicolau/olimp/obm-l.html > > > O administrador desta lista é > > > ================================================================== > ======= > > > > > > > > > > > > ==================================================================== > ===== > > Instruções para entrar na lista, sair da lista e usar a lista em > > http://www.mat.puc-rio.br/~nicolau/olimp/obm-l.html > > O administrador desta lista é > > ==================================================================== > ===== > > > > > > ======================================================================== = > Instruções para entrar na lista, sair da lista e usar a lista em > http://www.mat.puc-rio.br/~nicolau/olimp/obm-l.html > O administrador desta lista é > ======================================================================== = > > -- OPEN Internet - o 1º Provedor do DF com anit-virus no servidor de e- mail! ========================================================================= Instruções para entrar na lista, sair da lista e usar a lista em http://www.mat.puc-rio.br/~nicolau/olimp/obm-l.html O administrador desta lista é ========================================================================= From owner-obm-l@sucuri.mat.puc-rio.br Fri Sep 6 12:39:48 2002 Return-Path: Received: (from majordom@localhost) by sucuri.mat.puc-rio.br (8.9.3/8.9.3) id MAA03378 for obm-l-MTTP; Fri, 6 Sep 2002 12:39:30 -0300 Received: from hotmail.com (f13.sea2.hotmail.com [207.68.165.13]) by sucuri.mat.puc-rio.br (8.9.3/8.9.3) with ESMTP id MAA03373 for ; Fri, 6 Sep 2002 12:39:27 -0300 Received: from mail pickup service by hotmail.com with Microsoft SMTPSVC; Fri, 6 Sep 2002 08:40:13 -0700 Received: from 32.94.119.254 by sea2fd.sea2.hotmail.msn.com with HTTP; Fri, 06 Sep 2002 15:40:12 GMT X-Originating-IP: [32.94.119.254] From: "Paulo Santa Rita" To: obm-l@mat.puc-rio.br Subject: [obm-l] =?iso-8859-1?B?UmU6IFtvYm0tbF0gRPp2aWRh?= Date: Fri, 06 Sep 2002 15:40:12 +0000 Mime-Version: 1.0 Content-Type: text/plain; charset=iso-8859-1; format=flowed Message-ID: X-OriginalArrivalTime: 06 Sep 2002 15:40:13.0211 (UTC) FILETIME=[B0BB5EB0:01C255BB] Sender: owner-obm-l@sucuri.mat.puc-rio.br Precedence: bulk Reply-To: obm-l@mat.puc-rio.br Ola Ana e demais colegas desta lista ... OBM-L, Voce deve estar querendo dizer que a diferenca entre dois termos consecutivos TENDE A ZERO. Se for isso, voce esta diante de uma SEQUENCIA DE CAUCHY. Exite um teorema muito conhecido que afirma o seguinte : "Toda sequencia de Cauchy e convergente e toda sequencia convergente e de Cauchy." Um abraco Paulo Santa Rita 6,1239,060902 >From: "Ana Carolina Boero" >Reply-To: obm-l@mat.puc-rio.br >To: "obm-l" >Subject: [obm-l] Dúvida >Date: Fri, 6 Sep 2002 12:13:59 -0300 > >Olá colegas da lista, > >Estou com uma dúvida e gostaria de saber se alguém poderia me ajudar: > >Suponha uma seqüência de números reais, crescente, tal que a diferença >entre termos sucessivos vai a zero. >Será que existe um limite finito para essa seqüência? > >Muito obrigada, > >Carol _________________________________________________________________ Tenha você também um MSN Hotmail, o maior webmail do mundo: http://www.hotmail.com/br ========================================================================= Instruções para entrar na lista, sair da lista e usar a lista em http://www.mat.puc-rio.br/~nicolau/olimp/obm-l.html O administrador desta lista é ========================================================================= From owner-obm-l@sucuri.mat.puc-rio.br Fri Sep 6 12:44:07 2002 Return-Path: Received: (from majordom@localhost) by sucuri.mat.puc-rio.br (8.9.3/8.9.3) id MAA03599 for obm-l-MTTP; Fri, 6 Sep 2002 12:43:57 -0300 Received: from hotmail.com (f164.sea2.hotmail.com [207.68.165.164]) by sucuri.mat.puc-rio.br (8.9.3/8.9.3) with ESMTP id MAA03595 for ; Fri, 6 Sep 2002 12:43:54 -0300 Received: from mail pickup service by hotmail.com with Microsoft SMTPSVC; Fri, 6 Sep 2002 08:44:39 -0700 Received: from 32.94.119.253 by sea2fd.sea2.hotmail.msn.com with HTTP; Fri, 06 Sep 2002 15:44:39 GMT X-Originating-IP: [32.94.119.253] From: "Paulo Santa Rita" To: obm-l@mat.puc-rio.br Subject: Re: [obm-l] [Outro OFF??] Computacoes.. Date: Fri, 06 Sep 2002 15:44:39 +0000 Mime-Version: 1.0 Content-Type: text/plain; charset=iso-8859-1; format=flowed Message-ID: X-OriginalArrivalTime: 06 Sep 2002 15:44:39.0899 (UTC) FILETIME=[4FB0BAB0:01C255BC] Sender: owner-obm-l@sucuri.mat.puc-rio.br Precedence: bulk Reply-To: obm-l@mat.puc-rio.br Ola Ezer e demais colegas desta lista ... OBM-L, Use o GOOGLE e voce vai descobrir dezenas destas listas. Eu participo de algumas ( sobre C e sobre Linux ), mas com psedonimo. Um abraco Paulo Santa Rita 6,1243,060902 >From: ezer@ig.com.br >Reply-To: obm-l@mat.puc-rio.br >To: obm-l@mat.puc-rio.br >Subject: [obm-l] [Outro OFF??] Computacoes.. >Date: Wed, 1 Jan 1997 07:38:50 -0200 > >Eu sei que muitas das pessoas aqui da lista estao >envolvidas com Computacao, programacao, etc. >Por isso, achei conveniente fazer tal pergunta nesta >lista, aonde, tenho certeza, terei um bom feedback. > >Eu gostaria que me fossem indicadas boas listas de discussao >sobre Computacao e/ou programacao C/C++, que fossem utilizadas >por voces. Por exemplo, se conhecer uma boa lista sobre >Inteligencia >Artificial, sera mais do que bem-vindo! >(Obs.: Pode ser por lista por e-mail ou via web) > > >Desde já o meu obrigado, > >Ezer >========================================================================= >Instruções para entrar na lista, sair da lista e usar a lista em >http://www.mat.puc-rio.br/~nicolau/olimp/obm-l.html >O administrador desta lista é >========================================================================= _________________________________________________________________ Converse com seus amigos online, faça o download grátis do MSN Messenger: http://messenger.msn.com.br ========================================================================= Instruções para entrar na lista, sair da lista e usar a lista em http://www.mat.puc-rio.br/~nicolau/olimp/obm-l.html O administrador desta lista é ========================================================================= From owner-obm-l@sucuri.mat.puc-rio.br Fri Sep 6 14:28:53 2002 Return-Path: Received: (from majordom@localhost) by sucuri.mat.puc-rio.br (8.9.3/8.9.3) id OAA06782 for obm-l-MTTP; Fri, 6 Sep 2002 14:28:05 -0300 Received: from sr1.terra.com.br (sr1.terra.com.br [200.176.3.16]) by sucuri.mat.puc-rio.br (8.9.3/8.9.3) with ESMTP id OAA06778 for ; Fri, 6 Sep 2002 14:28:02 -0300 Received: from pavuna.terra.com.br (pavuna.terra.com.br [200.176.3.41]) by sr1.terra.com.br (Postfix) with ESMTP id CECB56E84C for ; Fri, 6 Sep 2002 14:28:48 -0300 (EST) Received: from Itautec.terra.com.br (200-158-60-118.dsl.telesp.net.br [200.158.60.118]) (authenticated user bruleite) by pavuna.terra.com.br (Postfix) with ESMTP id C7DCF6834E for ; Fri, 6 Sep 2002 14:28:47 -0300 (EST) Message-Id: <5.1.0.14.0.20020906142414.00a04dd0@pop.sao.terra.com.br> X-Sender: bruleite@pop.sao.terra.com.br X-Mailer: QUALCOMM Windows Eudora Version 5.1 Date: Fri, 06 Sep 2002 14:25:12 -0300 To: obm-l@mat.puc-rio.br From: "Bruno F. C. Leite" Subject: =?iso-8859-1?Q?Re:_[obm-l]_D=FAvida?= In-Reply-To: Mime-Version: 1.0 Content-Type: text/plain; charset="iso-8859-1"; format=flowed Content-Transfer-Encoding: 8bit X-MIME-Autoconverted: from quoted-printable to 8bit by sucuri.mat.puc-rio.br id OAA06779 Sender: owner-obm-l@sucuri.mat.puc-rio.br Precedence: bulk Reply-To: obm-l@mat.puc-rio.br At 12:13 06/09/02 -0300, you wrote: >Olá colegas da lista, > >Estou com uma dúvida e gostaria de saber se alguém poderia me ajudar: > >Suponha uma seqüência de números reais, crescente, tal que a diferença >entre termos sucessivos vai a zero. >Será que existe um limite finito para essa seqüência? Nem sempre. Tome por exemplo a série (a_n) onde a_n=1+1/2+1/3+...+1/n. Bruno Leite http://www.ime.usp.br/~brleite > >Muito obrigada, > >Carol ========================================================================= Instruções para entrar na lista, sair da lista e usar a lista em http://www.mat.puc-rio.br/~nicolau/olimp/obm-l.html O administrador desta lista é ========================================================================= From owner-obm-l@sucuri.mat.puc-rio.br Fri Sep 6 14:55:04 2002 Return-Path: Received: (from majordom@localhost) by sucuri.mat.puc-rio.br (8.9.3/8.9.3) id OAA07540 for obm-l-MTTP; Fri, 6 Sep 2002 14:52:53 -0300 Received: from studer.bol.com.br (studer.bol.com.br [200.221.24.21]) by sucuri.mat.puc-rio.br (8.9.3/8.9.3) with ESMTP id OAA07536 for ; Fri, 6 Sep 2002 14:52:50 -0300 Received: from seguro1 (200.221.24.99) by studer.bol.com.br (5.1.071) id 3D5D1D430070F53A for obm-l@mat.puc-rio.br; Fri, 6 Sep 2002 14:53:08 -0300 Message-ID: <001601c255ce$a5a122e0$0100a8c0@seguro1> From: "roberto-garcia" To: Subject: [obm-l] ECONOMIA Date: Fri, 6 Sep 2002 14:55:54 -0300 MIME-Version: 1.0 Content-Type: multipart/alternative; boundary="----=_NextPart_000_0013_01C255B5.800C3380" X-Priority: 3 X-MSMail-Priority: Normal X-Mailer: Microsoft Outlook Express 6.00.2600.0000 X-MimeOLE: Produced By Microsoft MimeOLE V6.00.2600.0000 X-Sender-IP: 200.158.194.223 Sender: owner-obm-l@sucuri.mat.puc-rio.br Precedence: bulk Reply-To: obm-l@mat.puc-rio.br This is a multi-part message in MIME format. ------=_NextPart_000_0013_01C255B5.800C3380 Content-Type: text/plain; charset="iso-8859-1" Content-Transfer-Encoding: quoted-printable Quem souber pode mandar a resposta para mim > >Matem=E1tica > > >Eu, Tu e Ele.... fomos comer no restaurante e no final a=20 conta deu R$30,00. > >Fizemos o seguinte: cada um deu dez mangos... > >Eu: R$ 10,00 >Tu: R$ 10,00 >Ele:R$ 10,00 >O gar=E7om levou o dinheiro at=E9 o caixa e o dono do=20 restaurante disse >seguinte: > >- Esses tr=EAs s=E3o clientes antigos do restaurante, ent=E3o=20 vou devolver R$ >5,00 para eles! >E entregou ao gar=E7om cinco notas de R$ 1,00. >O gar=E7om, muito esperto, fez o seguinte: pegou R$ 2,00=20 para ele e deu >R$1,00 para cada um de n=F3s. >No final ficou assim: >Eu: R$ 10,00 (- R$ 1,00 que foi devolvido) =3D Eu gastei=20 R$ 9,00. >Tu: R$ 10,00 (- R$ 1,00 que foi devolvido) =3D Tu gastaste=20 R$ 9,00. >Ele: R$ 10,00 (- R$ 1,00 que foi devolvido) =3D Ele gastou=20 R$ 9,00. >Logo, se cada um de n=F3s gastou R$ 9,00, o que n=F3s tr=EAs=20 gastamos juntos,foi >R$ 27,00. >E se o gar=E7om pegou R$ 2,00 para ele, temos: >N=F3s: R$ 27,00 >Gar=E7om: R$ 2,00 >TOTAL: R$ 29,00 >Pergunta-se: Onde foi parar a droga do outro R$ 1,00? >* Enviado pelo Departamento de Economia da PUC --- Outgoing mail is certified Virus Free. Checked by AVG anti-virus system (http://www.grisoft.com). Version: 6.0.384 / Virus Database: 216 - Release Date: 21/08/2002 ------=_NextPart_000_0013_01C255B5.800C3380 Content-Type: text/html; charset="iso-8859-1" Content-Transfer-Encoding: quoted-printable
Quem souber pode mandar a resposta para=20 mim
>
>Matem=E1tica
>
>
>Eu, Tu e Ele.... = fomos=20 comer no restaurante e no final a
conta deu = R$30,00.
>
>Fizemos=20 o seguinte: cada um deu dez mangos...
>
>Eu: R$ = 10,00
>Tu: R$=20 10,00
>Ele:R$ 10,00
>O gar=E7om levou o dinheiro at=E9 o = caixa e o dono=20 do
restaurante disse
>seguinte:
>
>- Esses tr=EAs = s=E3o=20 clientes antigos do restaurante, ent=E3o
vou devolver R$
>5,00 = para=20 eles!
>E entregou ao gar=E7om cinco notas de R$ 1,00.
>O = gar=E7om, muito=20 esperto, fez o seguinte: pegou R$ 2,00
para ele e deu
>R$1,00 = para=20 cada um de n=F3s.
>No final ficou assim:
>Eu: R$ 10,00 (- R$ = 1,00 que=20 foi devolvido) =3D Eu gastei
R$ 9,00.
>Tu: R$ 10,00 (- R$ 1,00 = que foi=20 devolvido) =3D Tu gastaste
R$ 9,00.
>Ele: R$ 10,00 (- R$ 1,00 = que foi=20 devolvido) =3D Ele gastou
R$ 9,00.
>Logo, se cada um de n=F3s = gastou R$=20 9,00, o que n=F3s tr=EAs
gastamos juntos,foi
>R$ = 27,00.
>E se o=20 gar=E7om pegou R$ 2,00 para ele, temos:
>N=F3s: R$ = 27,00
>Gar=E7om: R$=20 2,00
>TOTAL: R$ 29,00
>Pergunta-se: Onde foi parar a droga = do outro=20 R$ 1,00?
>* Enviado pelo Departamento de Economia da PUC
 

---
Outgoing mail is certified Virus Free.
Checked by AVG = anti-virus system (http://www.grisoft.com).
Version: = 6.0.384 /=20 Virus Database: 216 - Release Date: 21/08/2002
------=_NextPart_000_0013_01C255B5.800C3380-- ========================================================================= Instruções para entrar na lista, sair da lista e usar a lista em http://www.mat.puc-rio.br/~nicolau/olimp/obm-l.html O administrador desta lista é ========================================================================= From owner-obm-l@sucuri.mat.puc-rio.br Fri Sep 6 15:10:47 2002 Return-Path: Received: (from majordom@localhost) by sucuri.mat.puc-rio.br (8.9.3/8.9.3) id PAA08061 for obm-l-MTTP; Fri, 6 Sep 2002 15:09:59 -0300 Received: from studer.bol.com.br (studer.bol.com.br [200.221.24.21]) by sucuri.mat.puc-rio.br (8.9.3/8.9.3) with ESMTP id PAA08054 for ; Fri, 6 Sep 2002 15:09:55 -0300 Received: from seguro1 (200.221.24.99) by studer.bol.com.br (5.1.071) id 3D5D1D43007109DD for obm-l@mat.puc-rio.br; Fri, 6 Sep 2002 15:10:14 -0300 Message-ID: <003c01c255d1$09396f40$0100a8c0@seguro1> From: "roberto-garcia" To: References: Subject: [obm-l] ECONOMIA Date: Fri, 6 Sep 2002 15:12:58 -0300 MIME-Version: 1.0 Content-Type: text/plain; charset="iso-8859-1" Content-Transfer-Encoding: 8bit X-Priority: 3 X-MSMail-Priority: Normal X-Mailer: Microsoft Outlook Express 6.00.2600.0000 X-MimeOLE: Produced By Microsoft MimeOLE V6.00.2600.0000 X-Sender-IP: 200.158.194.223 Sender: owner-obm-l@sucuri.mat.puc-rio.br Precedence: bulk Reply-To: obm-l@mat.puc-rio.br Quem souber pode mandar a resposta para mim > >Matemática > > >Eu, Tu e Ele.... fomos comer no restaurante e no final a conta deu R$30,00. > >Fizemos o seguinte: cada um deu dez mangos... > >Eu: R$ 10,00 >Tu: R$ 10,00 >Ele:R$ 10,00 >O garçom levou o dinheiro até o caixa e o dono do restaurante disse >seguinte: > >- Esses três são clientes antigos do restaurante, então vou devolver R$ >5,00 para eles! >E entregou ao garçom cinco notas de R$ 1,00. >O garçom, muito esperto, fez o seguinte: pegou R$ 2,00 para ele e deu >R$1,00 para cada um de nós. >No final ficou assim: >Eu: R$ 10,00 (- R$ 1,00 que foi devolvido) = Eu gastei R$ 9,00. >Tu: R$ 10,00 (- R$ 1,00 que foi devolvido) = Tu gastaste R$ 9,00. >Ele: R$ 10,00 (- R$ 1,00 que foi devolvido) = Ele gastou R$ 9,00. >Logo, se cada um de nós gastou R$ 9,00, o que nós três gastamos juntos,foi >R$ 27,00. >E se o garçom pegou R$ 2,00 para ele, temos: >Nós: R$ 27,00 >Garçom: R$ 2,00 >TOTAL: R$ 29,00 >Pergunta-se: Onde foi parar a droga do outro R$ 1,00? >* Enviado pelo Departamento de Economia da PUC --- Outgoing mail is certified Virus Free. Checked by AVG anti-virus system (http://www.grisoft.com). Version: 6.0.384 / Virus Database: 216 - Release Date: 21/08/2002 ========================================================================= Instruções para entrar na lista, sair da lista e usar a lista em http://www.mat.puc-rio.br/~nicolau/olimp/obm-l.html O administrador desta lista é ========================================================================= From owner-obm-l@sucuri.mat.puc-rio.br Fri Sep 6 15:35:42 2002 Return-Path: Received: (from majordom@localhost) by sucuri.mat.puc-rio.br (8.9.3/8.9.3) id PAA08926 for obm-l-MTTP; Fri, 6 Sep 2002 15:34:34 -0300 Received: from zeus.opendf.com.br (zeus.opengate.com.br [200.181.71.10]) by sucuri.mat.puc-rio.br (8.9.3/8.9.3) with ESMTP id PAA08922 for ; Fri, 6 Sep 2002 15:34:31 -0300 Received: from localhost (localhost.opengate.com.br [127.0.0.1]) by zeus.opendf.com.br (Postfix) with ESMTP id CD4C63EB19 for ; Fri, 6 Sep 2002 15:35:13 -0300 (BRT) Received: by zeus.opendf.com.br (Postfix, from userid 48) id 4AE9C3EB25; Fri, 6 Sep 2002 15:35:13 -0300 (BRT) From: "498 - Artur Costa Steiner" To: obm-l@mat.puc-rio.br Subject: Re: [obm-l] Dúvida X-Mailer: NeoMail 1.25 X-IPAddress: 200.252.155.2 MIME-Version: 1.0 Content-Type: text/plain; charset=iso-8859-1 Message-Id: <20020906183513.4AE9C3EB25@zeus.opendf.com.br> Date: Fri, 6 Sep 2002 15:35:13 -0300 (BRT) X-Virus-Scanned: by AMaViS new-20020517 Sender: owner-obm-l@sucuri.mat.puc-rio.br Precedence: bulk Reply-To: obm-l@mat.puc-rio.br > > > Olá colegas da lista, > > Estou com uma dúvida e gostaria de saber se alguém poderia me ajudar: > > Suponha uma seqüência de números reais, crescente, tal que a diferença entre termos sucessivos vai a zero. > Será que existe um limite finito para essa seqüência? > > Muito obrigada, > > Carol Não Carol, não necessariamente. Considere, como contra-exemplo, a sequência de números reais dada por (L(n)) one L é o log natural. É imediato que esta sequência tende ao infinito. No entanto, temos que L (n+1) - L(n) = L(1+1/n) Quando n vai para infinito, 1+1/n tende a 1 e L (1+1/n) tende a ZERO. Observe qe a condição que vc apresentou NÃO garante que tenhamos uma seq. de Cauchy. Não basta que a diferença entre termos consecutivos tenda a zero. Por definição, (x(n) é de Cuchy se, dado qualquer eps >0, existir um natural k (em princípio, dependente de eps) tal que |xm - xn| = k. Logo, não podemos admitir qualquer relação pré-fixada entre m e n. No caso que vc apresentou, a condição só vale se m = n+1, logo, NÃO temos uma seq. de Cauchy. Lembro que, nos espaços métricos completos, uma seq. é convergente se, e somente se, for de Cauchy. Abraços Artur PEN Internet - o 1º Provedor do DF com anit-virus no servidor de e- mail! ========================================================================= Instruções para entrar na lista, sair da lista e usar a lista em http://www.mat.puc-rio.br/~nicolau/olimp/obm-l.html O administrador desta lista é ========================================================================= From owner-obm-l@sucuri.mat.puc-rio.br Fri Sep 6 15:35:43 2002 Return-Path: Received: (from majordom@localhost) by sucuri.mat.puc-rio.br (8.9.3/8.9.3) id PAA08990 for obm-l-MTTP; Fri, 6 Sep 2002 15:35:29 -0300 Received: from gorgo.centroin.com.br (gorgo.centroin.com.br [200.225.63.128]) by sucuri.mat.puc-rio.br (8.9.3/8.9.3) with ESMTP id PAA08986 for ; Fri, 6 Sep 2002 15:35:26 -0300 Received: from centroin.com.br (du7c.rjo.centroin.com.br [200.225.58.7]) (authenticated bits=0) by gorgo.centroin.com.br (8.12.2/8.12.1) with ESMTP id g86IaZnH012858 for ; Fri, 6 Sep 2002 15:36:35 -0300 (BRT) Message-ID: <3D78F64B.6020009@centroin.com.br> Date: Fri, 06 Sep 2002 15:39:07 -0300 From: Augusto =?ISO-8859-1?Q?C=E9sar?= Morgado User-Agent: Mozilla/5.0 (Windows; U; Win98; en-US; rv:0.9.4.1) Gecko/20020508 Netscape6/6.2.3 X-Accept-Language: en-us MIME-Version: 1.0 To: obm-l@mat.puc-rio.br Subject: Re: [obm-l] =?ISO-8859-1?Q?D=FAvida?= References: Content-Type: multipart/alternative; boundary="------------050601070806080000060602" Sender: owner-obm-l@sucuri.mat.puc-rio.br Precedence: bulk Reply-To: obm-l@mat.puc-rio.br --------------050601070806080000060602 Content-Type: text/plain; charset=ISO-8859-1; format=flowed Content-Transfer-Encoding: 8bit Que significa diferença entre termos sucessivos? Se é a diferença entre dois termos sucessivos, a resposta é não. A sequencia an = log n é crescente e tal que a n+1 - an = log(n+1) - log(n) = log [ 1+ 1/n] tende a log 1 = 0 e nao tem limite finito. Ana Carolina Boero wrote: > Olá colegas da lista, > > > > Estou com uma dúvida e gostaria de saber se alguém poderia me ajudar: > > > > Suponha uma seqüência de números reais, crescente, tal que a diferença > entre termos sucessivos vai a zero. > > Será que existe um limite finito para essa seqüência? > > > > Muito obrigada, > > > > Carol > --------------050601070806080000060602 Content-Type: text/html; charset=us-ascii Content-Transfer-Encoding: 7bit Que significa diferença entre termos sucessivos?
Se é a diferença entre dois termos sucessivos, a resposta é não.
A sequencia  an = log n  é crescente e tal que  a n+1  -  an = log(n+1) - log(n) =  log [ 1+ 1/n]  tende a
log 1 = 0  e nao tem limite finito.
Ana Carolina Boero wrote:
Olá colegas da lista,
 
Estou com uma dúvida e gostaria de saber se alguém poderia me ajudar:
 
Suponha uma seqüência de números reais, crescente, tal que a diferença entre termos sucessivos vai a zero.
Será que existe um limite finito para essa seqüência?
 
Muito obrigada,
 
Carol

--------------050601070806080000060602-- ========================================================================= Instruções para entrar na lista, sair da lista e usar a lista em http://www.mat.puc-rio.br/~nicolau/olimp/obm-l.html O administrador desta lista é ========================================================================= From owner-obm-l@sucuri.mat.puc-rio.br Fri Sep 6 15:39:36 2002 Return-Path: Received: (from majordom@localhost) by sucuri.mat.puc-rio.br (8.9.3/8.9.3) id PAA09148 for obm-l-MTTP; Fri, 6 Sep 2002 15:39:23 -0300 Received: from zeus.opendf.com.br (zeus.opengate.com.br [200.181.71.10]) by sucuri.mat.puc-rio.br (8.9.3/8.9.3) with ESMTP id PAA09144 for ; Fri, 6 Sep 2002 15:39:20 -0300 Received: from localhost (localhost.opengate.com.br [127.0.0.1]) by zeus.opendf.com.br (Postfix) with ESMTP id A82BF3EB19 for ; Fri, 6 Sep 2002 15:40:02 -0300 (BRT) Received: by zeus.opendf.com.br (Postfix, from userid 48) id 284743EB25; Fri, 6 Sep 2002 15:40:02 -0300 (BRT) From: "498 - Artur Costa Steiner" To: obm-l@mat.puc-rio.br Subject: Re: [obm-l] Re: [obm-l] Dúvida X-Mailer: NeoMail 1.25 X-IPAddress: 200.252.155.2 MIME-Version: 1.0 Content-Type: text/plain; charset=iso-8859-1 Message-Id: <20020906184002.284743EB25@zeus.opendf.com.br> Date: Fri, 6 Sep 2002 15:40:02 -0300 (BRT) X-Virus-Scanned: by AMaViS new-20020517 Sender: owner-obm-l@sucuri.mat.puc-rio.br Precedence: bulk Reply-To: obm-l@mat.puc-rio.br > Ola Ana e demais colegas > desta lista ... OBM-L, > > Voce deve estar querendo dizer que a diferenca entre dois termos > consecutivos TENDE A ZERO. Se for isso, voce esta diante de uma SEQUENCIA DE > CAUCHY. Não, não. Não é suficiente. Definição: (x(n)) é uma seq. de Cauchy se, dados qualquer eps>o, existir um natural k, dependente de eps, tal que |x_m - x_n| =k. Não basta que esta condição se verifique apenas para termos consecutivos de (x(n)). Não podemos assumir qualquer relação enter me e n. Abraços Artur > > Exite um teorema muito conhecido que afirma o seguinte : > > "Toda sequencia de Cauchy e convergente e toda sequencia convergente e de > Cauchy." Nos espaços métricos completos... > > Um abraco > Paulo Santa Rita > 6,1239,060902 > > > > >From: "Ana Carolina Boero" > >Reply-To: obm-l@mat.puc-rio.br > >To: "obm-l" > >Subject: [obm-l] Dúvida > >Date: Fri, 6 Sep 2002 12:13:59 -0300 > > > >Olá colegas da lista, > > > >Estou com uma dúvida e gostaria de saber se alguém poderia me ajudar: > > > >Suponha uma seqüência de números reais, crescente, tal que a diferença > >entre termos sucessivos vai a zero. > >Será que existe um limite finito para essa seqüência? > > > >Muito obrigada, > > > >Carol > > > > > _________________________________________________________________ > Tenha você também um MSN Hotmail, o maior webmail do mundo: > http://www.hotmail.com/br > > ======================================================================== = > Instruções para entrar na lista, sair da lista e usar a lista em > http://www.mat.puc-rio.br/~nicolau/olimp/obm-l.html > O administrador desta lista é > ======================================================================== = > > -- OPEN Internet - o 1º Provedor do DF com anit-virus no servidor de e- mail! ========================================================================= Instruções para entrar na lista, sair da lista e usar a lista em http://www.mat.puc-rio.br/~nicolau/olimp/obm-l.html O administrador desta lista é ========================================================================= From owner-obm-l@sucuri.mat.puc-rio.br Fri Sep 6 15:41:21 2002 Return-Path: Received: (from majordom@localhost) by sucuri.mat.puc-rio.br (8.9.3/8.9.3) id PAA09260 for obm-l-MTTP; Fri, 6 Sep 2002 15:41:03 -0300 Received: from gorgo.centroin.com.br (gorgo.centroin.com.br [200.225.63.128]) by sucuri.mat.puc-rio.br (8.9.3/8.9.3) with ESMTP id PAA09255 for ; Fri, 6 Sep 2002 15:41:00 -0300 Received: from centroin.com.br (du7c.rjo.centroin.com.br [200.225.58.7]) (authenticated bits=0) by gorgo.centroin.com.br (8.12.2/8.12.1) with ESMTP id g86Ig9nH013107 for ; Fri, 6 Sep 2002 15:42:10 -0300 (BRT) Message-ID: <3D78F798.2060307@centroin.com.br> Date: Fri, 06 Sep 2002 15:44:40 -0300 From: Augusto =?ISO-8859-1?Q?C=E9sar?= Morgado User-Agent: Mozilla/5.0 (Windows; U; Win98; en-US; rv:0.9.4.1) Gecko/20020508 Netscape6/6.2.3 X-Accept-Language: en-us MIME-Version: 1.0 To: obm-l@mat.puc-rio.br Subject: Re: [obm-l] ECONOMIA References: <001601c255ce$a5a122e0$0100a8c0@seguro1> Content-Type: multipart/alternative; boundary="------------080302060203070000000103" Sender: owner-obm-l@sucuri.mat.puc-rio.br Precedence: bulk Reply-To: obm-l@mat.puc-rio.br --------------080302060203070000000103 Content-Type: text/plain; charset=ISO-8859-1; format=flowed Content-Transfer-Encoding: 8bit roberto-garcia wrote: > Quem souber pode mandar a resposta para mim > > > >Matemática > > > > > >Eu, Tu e Ele.... fomos comer no restaurante e no final a > conta deu R$30,00. > > > >Fizemos o seguinte: cada um deu dez mangos... > > > >Eu: R$ 10,00 > >Tu: R$ 10,00 > >Ele:R$ 10,00 > >O garçom levou o dinheiro até o caixa e o dono do > restaurante disse > >seguinte: > > > >- Esses três são clientes antigos do restaurante, então > vou devolver R$ > >5,00 para eles! > >E entregou ao garçom cinco notas de R$ 1,00. > >O garçom, muito esperto, fez o seguinte: pegou R$ 2,00 > para ele e deu > >R$1,00 para cada um de nós. > >No final ficou assim: > >Eu: R$ 10,00 (- R$ 1,00 que foi devolvido) = Eu gastei > R$ 9,00. > >Tu: R$ 10,00 (- R$ 1,00 que foi devolvido) = Tu gastaste > R$ 9,00. > >Ele: R$ 10,00 (- R$ 1,00 que foi devolvido) = Ele gastou > R$ 9,00. > >Logo, se cada um de nós gastou R$ 9,00, o que nós três > gastamos juntos,foi > >R$ 27,00. > >E se o garçom pegou R$ 2,00 para ele, temos: > >Nós GASTAMOS: R$ 27,00 > > MENOS O QUE FICOU PARA O Garçom: R$ 2,00 > >TOTAL QUE FOI PARA O RESTAURANTE : R$ 25,00 > tÁ FALTANDO NADA. > > > > > --- > Outgoing mail is certified Virus Free. > Checked by AVG anti-virus system (http://www.grisoft.com ). > Version: 6.0.384 / Virus Database: 216 - Release Date: 21/08/2002 > --------------080302060203070000000103 Content-Type: text/html; charset=us-ascii Content-Transfer-Encoding: 7bit

roberto-garcia wrote:
Quem souber pode mandar a resposta para mim
>
>Matemática
>
>
>Eu, Tu e Ele.... fomos comer no restaurante e no final a
conta deu R$30,00.
>
>Fizemos o seguinte: cada um deu dez mangos...
>
>Eu: R$ 10,00
>Tu: R$ 10,00
>Ele:R$ 10,00
>O garçom levou o dinheiro até o caixa e o dono do
restaurante disse
>seguinte:
>
>- Esses três são clientes antigos do restaurante, então
vou devolver R$
>5,00 para eles!
>E entregou ao garçom cinco notas de R$ 1,00.
>O garçom, muito esperto, fez o seguinte: pegou R$ 2,00
para ele e deu
>R$1,00 para cada um de nós.
>No final ficou assim:
>Eu: R$ 10,00 (- R$ 1,00 que foi devolvido) = Eu gastei
R$ 9,00.
>Tu: R$ 10,00 (- R$ 1,00 que foi devolvido) = Tu gastaste
R$ 9,00.
>Ele: R$ 10,00 (- R$ 1,00 que foi devolvido) = Ele gastou
R$ 9,00.
>Logo, se cada um de nós gastou R$ 9,00, o que nós três
gastamos juntos,foi
>R$ 27,00.
>E se o garçom pegou R$ 2,00 para ele, temos:
>Nós GASTAMOS: R$ 27,00
> MENOS O QUE FICOU PARA O Garçom: R$ 2,00
>TOTAL QUE FOI PARA O RESTAURANTE : R$ 25,00  
tÁ FALTANDO NADA.
 

---
Outgoing mail is certified Virus Free.
Checked by AVG anti-virus system (http://www.grisoft.com ).
Version: 6.0.384 / Virus Database: 216 - Release Date: 21/08/2002

--------------080302060203070000000103-- ========================================================================= Instruções para entrar na lista, sair da lista e usar a lista em http://www.mat.puc-rio.br/~nicolau/olimp/obm-l.html O administrador desta lista é ========================================================================= From owner-obm-l@sucuri.mat.puc-rio.br Fri Sep 6 15:59:32 2002 Return-Path: Received: (from majordom@localhost) by sucuri.mat.puc-rio.br (8.9.3/8.9.3) id PAA10863 for obm-l-MTTP; Fri, 6 Sep 2002 15:59:13 -0300 Received: from zeus.opendf.com.br (zeus.opengate.com.br [200.181.71.10]) by sucuri.mat.puc-rio.br (8.9.3/8.9.3) with ESMTP id PAA10850 for ; Fri, 6 Sep 2002 15:59:08 -0300 Received: from localhost (localhost.opengate.com.br [127.0.0.1]) by zeus.opendf.com.br (Postfix) with ESMTP id 249A33EB14 for ; Fri, 6 Sep 2002 15:59:46 -0300 (BRT) Received: by zeus.opendf.com.br (Postfix, from userid 48) id 657273EB27; Fri, 6 Sep 2002 15:59:44 -0300 (BRT) From: "498 - Artur Costa Steiner" To: obm-l@mat.puc-rio.br Subject: Re: [obm-l] ECONOMIA X-Mailer: NeoMail 1.25 X-IPAddress: 200.252.155.2 MIME-Version: 1.0 Content-Type: text/plain; charset=iso-8859-1 Message-Id: <20020906185944.657273EB27@zeus.opendf.com.br> Date: Fri, 6 Sep 2002 15:59:44 -0300 (BRT) X-Virus-Scanned: by AMaViS new-20020517 Sender: owner-obm-l@sucuri.mat.puc-rio.br Precedence: bulk Reply-To: obm-l@mat.puc-rio.br Isso é um sofisma. Basta observar que os fregueses pagaram o total de 3x9= 27, dos quais s 25 ficaram no caixa e 2 com o garçom. > This is a multi-part message in MIME format. > > > Quem souber pode mandar a resposta para mim > > > >Matemática > > > > > >Eu, Tu e Ele.... fomos comer no restaurante e no final a > conta deu R$30,00. > > > >Fizemos o seguinte: cada um deu dez mangos... > > > >Eu: R$ 10,00 > >Tu: R$ 10,00 > >Ele:R$ 10,00 > >O garçom levou o dinheiro até o caixa e o dono do > restaurante disse > >seguinte: > > > >- Esses três são clientes antigos do restaurante, então > vou devolver R$ > >5,00 para eles! > >E entregou ao garçom cinco notas de R$ 1,00. > >O garçom, muito esperto, fez o seguinte: pegou R$ 2,00 > para ele e deu > >R$1,00 para cada um de nós. > >No final ficou assim: > >Eu: R$ 10,00 (- R$ 1,00 que foi devolvido) = Eu gastei > R$ 9,00. > >Tu: R$ 10,00 (- R$ 1,00 que foi devolvido) = Tu gastaste > R$ 9,00. > >Ele: R$ 10,00 (- R$ 1,00 que foi devolvido) = Ele gastou > R$ 9,00. > >Logo, se cada um de nós gastou R$ 9,00, o que nós três > gastamos juntos,foi > >R$ 27,00. > >E se o garçom pegou R$ 2,00 para ele, temos: > >Nós: R$ 27,00 > >Garçom: R$ 2,00 > >TOTAL: R$ 29,00 > >Pergunta-se: Onde foi parar a droga do outro R$ 1,00? > >* Enviado pelo Departamento de Economia da PUC > > > > --- > Outgoing mail is certified Virus Free. > Checked by AVG anti-virus system (http://www.grisoft.com). > Version: 6.0.384 / Virus Database: 216 - Release Date: 21/08/2002 > > -- OPEN Internet - o 1º Provedor do DF com anit-virus no servidor de e- mail! ========================================================================= Instruções para entrar na lista, sair da lista e usar a lista em http://www.mat.puc-rio.br/~nicolau/olimp/obm-l.html O administrador desta lista é ========================================================================= From owner-obm-l@sucuri.mat.puc-rio.br Fri Sep 6 16:05:05 2002 Return-Path: Received: (from majordom@localhost) by sucuri.mat.puc-rio.br (8.9.3/8.9.3) id QAA11071 for obm-l-MTTP; Fri, 6 Sep 2002 16:04:59 -0300 Received: from videira.terra.com.br (videira.terra.com.br [200.176.3.5]) by sucuri.mat.puc-rio.br (8.9.3/8.9.3) with ESMTP id QAA11058 for ; Fri, 6 Sep 2002 16:04:55 -0300 Received: from engenho.terra.com.br (engenho.terra.com.br [200.176.3.42]) by videira.terra.com.br (Postfix) with ESMTP id E58DAE2300 for ; Fri, 6 Sep 2002 15:34:36 -0300 (EST) Received: from stabel (unknown [200.203.39.93]) (authenticated user dudasta) by engenho.terra.com.br (Postfix) with ESMTP id 2F0D368180 for ; Fri, 6 Sep 2002 15:34:36 -0300 (EST) Message-ID: <005d01c255d4$0d348a50$0301a8c0@stabel> From: "Eduardo Casagrande Stabel" To: References: <001601c255ce$a5a122e0$0100a8c0@seguro1> Subject: Re: [obm-l] ECONOMIA Date: Fri, 6 Sep 2002 15:34:35 -0300 MIME-Version: 1.0 Content-Type: text/plain; charset="iso-8859-1" Content-Transfer-Encoding: 8bit X-Priority: 3 X-MSMail-Priority: Normal X-Mailer: Microsoft Outlook Express 6.00.2600.0000 X-MimeOLE: Produced By Microsoft MimeOLE V6.00.2600.0000 Sender: owner-obm-l@sucuri.mat.puc-rio.br Precedence: bulk Reply-To: obm-l@mat.puc-rio.br From: roberto-garcia >Quem souber pode mandar a resposta para mim > >Matemática > > >Eu, Tu e Ele.... fomos comer no restaurante e no final a conta deu R$30,00. > >Fizemos o seguinte: cada um deu dez mangos... > >Eu: R$ 10,00 >Tu: R$ 10,00 >Ele:R$ 10,00 >O garçom levou o dinheiro até o caixa e o dono do restaurante disse >seguinte: > >- Esses três são clientes antigos do restaurante, então vou devolver R$ >5,00 para eles! >E entregou ao garçom cinco notas de R$ 1,00. >O garçom, muito esperto, fez o seguinte: pegou R$ 2,00 para ele e deu >R$1,00 para cada um de nós. >No final ficou assim: >Eu: R$ 10,00 (- R$ 1,00 que foi devolvido) = Eu gastei R$ 9,00. >Tu: R$ 10,00 (- R$ 1,00 que foi devolvido) = Tu gastaste R$ 9,00. >Ele: R$ 10,00 (- R$ 1,00 que foi devolvido) = Ele gastou R$ 9,00. >Logo, se cada um de nós gastou R$ 9,00, o que nós três gastamos juntos,foi >R$ 27,00. >E se o garçom pegou R$ 2,00 para ele, temos: >Nós: R$ 27,00 >Garçom: R$ 2,00 >TOTAL: R$ 29,00 >Pergunta-se: Onde foi parar a droga do outro R$ 1,00? É bom evitar esse tipo de linguagem para não ofender ninguém e começar um conflito na lista. Ok? Esse problema é muito maldoso. A conta certa é a seguinte: cada um deu 10 reais e recebeu 1 de volta, portanto cada um deu 9 reais e ficou com 1 real. Ora dos 27 reais que foram dados, 25 ficaram com o vendedor e 2 ficaram com o garçom. Ou seja, a conta fecha direitinho, 25 para o vendedor, 2 para o garçom e 3 para os clientes, total 30 reais. Por que a aparente falha? por que não se está considerando os 3 reais que ficaram com os clientes e está se repetindo na conta duas vezes a gorjeta ao garçom. Explico. Nós demos 27 reais, e ficamos com 3. Desses 27 reais, 25 foram ao vendedor e 2 ao garçom, se a gente soma 27 com 2 a gente vai ter 25 + 2 + 2 = 29, uma repetição do dinheiro do garçom e a falta do dinheiro (3) que ficou com os clientes. Portanto a frase certa é: cada um deu 10 reais e recebeu 1 de volta, portanto cada um deu 9 reais (27 dos três) e os clientes ficaram com 3 reais, total 30 reais. Eduardo. Porto Alegre, RS. ========================================================================= Instruções para entrar na lista, sair da lista e usar a lista em http://www.mat.puc-rio.br/~nicolau/olimp/obm-l.html O administrador desta lista é ========================================================================= From owner-obm-l@sucuri.mat.puc-rio.br Fri Sep 6 16:10:07 2002 Return-Path: Received: (from majordom@localhost) by sucuri.mat.puc-rio.br (8.9.3/8.9.3) id QAA11275 for obm-l-MTTP; Fri, 6 Sep 2002 16:09:49 -0300 Received: from zeus.opendf.com.br (zeus.opengate.com.br [200.181.71.10]) by sucuri.mat.puc-rio.br (8.9.3/8.9.3) with ESMTP id QAA11270 for ; Fri, 6 Sep 2002 16:09:44 -0300 Received: from localhost (localhost.opengate.com.br [127.0.0.1]) by zeus.opendf.com.br (Postfix) with ESMTP id BD6783EA9C for ; Fri, 6 Sep 2002 16:10:25 -0300 (BRT) Received: by zeus.opendf.com.br (Postfix, from userid 48) id 0515F3EAE4; Fri, 6 Sep 2002 16:10:24 -0300 (BRT) From: "498 - Artur Costa Steiner" To: obm-l@mat.puc-rio.br, Subject: [obm-l] Uma prova simples para a seguinte afirmação X-Mailer: NeoMail 1.25 X-IPAddress: 200.252.155.2 MIME-Version: 1.0 Content-Type: text/plain; charset=iso-8859-1 Message-Id: <20020906191024.0515F3EAE4@zeus.opendf.com.br> Date: Fri, 6 Sep 2002 16:10:24 -0300 (BRT) X-Virus-Scanned: by AMaViS new-20020517 Sender: owner-obm-l@sucuri.mat.puc-rio.br Precedence: bulk Reply-To: obm-l@mat.puc-rio.br Este teorema é um daqueles em que a intuição funciona, mas para o qual não consigo dar uma prova simples, conforme um garoto de 15 anos me pediu: Seja A um subconjunto de um espaço euclidiano Rn. Se A não possuir pontos de acumulação, então A é numerável. (Observemos que em outros espaços isto pode ser falso, mas estou me detendo em Rn com a métrica euclidiana.) De fato, parece óbvio, mas a prova que conheço exige o conhecimento de alguns conceitos um pouco mais avançados de espaços métricos, como separabilidade. Obrigado Artur ========================================================================= Instruções para entrar na lista, sair da lista e usar a lista em http://www.mat.puc-rio.br/~nicolau/olimp/obm-l.html O administrador desta lista é ========================================================================= From owner-obm-l@sucuri.mat.puc-rio.br Fri Sep 6 16:10:07 2002 Return-Path: Received: (from majordom@localhost) by sucuri.mat.puc-rio.br (8.9.3/8.9.3) id QAA11275 for obm-l-MTTP; Fri, 6 Sep 2002 16:09:49 -0300 Received: from zeus.opendf.com.br (zeus.opengate.com.br [200.181.71.10]) by sucuri.mat.puc-rio.br (8.9.3/8.9.3) with ESMTP id QAA11270 for ; Fri, 6 Sep 2002 16:09:44 -0300 Received: from localhost (localhost.opengate.com.br [127.0.0.1]) by zeus.opendf.com.br (Postfix) with ESMTP id BD6783EA9C for ; Fri, 6 Sep 2002 16:10:25 -0300 (BRT) Received: by zeus.opendf.com.br (Postfix, from userid 48) id 0515F3EAE4; Fri, 6 Sep 2002 16:10:24 -0300 (BRT) From: "498 - Artur Costa Steiner" To: obm-l@mat.puc-rio.br, Subject: [obm-l] Uma prova simples para a seguinte afirmação X-Mailer: NeoMail 1.25 X-IPAddress: 200.252.155.2 MIME-Version: 1.0 Content-Type: text/plain; charset=iso-8859-1 Message-Id: <20020906191024.0515F3EAE4@zeus.opendf.com.br> Date: Fri, 6 Sep 2002 16:10:24 -0300 (BRT) X-Virus-Scanned: by AMaViS new-20020517 Sender: owner-obm-l@sucuri.mat.puc-rio.br Precedence: bulk Reply-To: obm-l@mat.puc-rio.br Este teorema é um daqueles em que a intuição funciona, mas para o qual não consigo dar uma prova simples, conforme um garoto de 15 anos me pediu: Seja A um subconjunto de um espaço euclidiano Rn. Se A não possuir pontos de acumulação, então A é numerável. (Observemos que em outros espaços isto pode ser falso, mas estou me detendo em Rn com a métrica euclidiana.) De fato, parece óbvio, mas a prova que conheço exige o conhecimento de alguns conceitos um pouco mais avançados de espaços métricos, como separabilidade. Obrigado Artur ========================================================================= Instruções para entrar na lista, sair da lista e usar a lista em http://www.mat.puc-rio.br/~nicolau/olimp/obm-l.html O administrador desta lista é ========================================================================= From owner-obm-l@sucuri.mat.puc-rio.br Fri Sep 6 16:34:51 2002 Return-Path: Received: (from majordom@localhost) by sucuri.mat.puc-rio.br (8.9.3/8.9.3) id QAA13405 for obm-l-MTTP; Fri, 6 Sep 2002 16:34:08 -0300 Received: from zeus.opendf.com.br (zeus.opengate.com.br [200.181.71.10]) by sucuri.mat.puc-rio.br (8.9.3/8.9.3) with ESMTP id QAA13401 for ; Fri, 6 Sep 2002 16:34:05 -0300 Received: from localhost (localhost.opengate.com.br [127.0.0.1]) by zeus.opendf.com.br (Postfix) with ESMTP id BB99B3EA9C for ; Fri, 6 Sep 2002 16:34:47 -0300 (BRT) Received: by zeus.opendf.com.br (Postfix, from userid 48) id F20BA3EAE9; Fri, 6 Sep 2002 16:34:46 -0300 (BRT) From: "498 - Artur Costa Steiner" To: obm-l@mat.puc-rio.br, Subject: [obm-l] Algumas definições X-Mailer: NeoMail 1.25 X-IPAddress: 200.252.155.2 MIME-Version: 1.0 Content-Type: text/plain; charset=iso-8859-1 Message-Id: <20020906193446.F20BA3EAE9@zeus.opendf.com.br> Date: Fri, 6 Sep 2002 16:34:46 -0300 (BRT) X-Virus-Scanned: by AMaViS new-20020517 Sender: owner-obm-l@sucuri.mat.puc-rio.br Precedence: bulk Reply-To: obm-l@mat.puc-rio.br Alguns conceitos em matemática parecem varia um pouco conforme seja o autor, principalmente naqueles nativos da lígua inglesa. Gostaria de saber se esu tenho as definições mais comumentes usada para os seguintes conceitos: 1) Ponto de acumulação - x é ponto de acumulação de A se qualquer vizinhança de x contiver pelo menos um elemento de A distinto de x. Ém inglês, geralmente isto se chama accumulation point ou, menos comumente, cluster point. 2) Ponto limite - há certa confusão. Em Português, acho que sempre dizemos que x é ponto limite de A se qualquer vizinhança de x contiver elementos de A e de seu complementar C(A). Mas há autores , principalmente em Inglês, (aliás, muitos), que entendem por limit point o mesmo que accumulation point. Para o que defini comoponto limite, este autores usam boundary point. Em Português, já vi ponto de fronteira, mas apenas em livros de programação matemática, e não em livros básicos sobre Análise. 3) Estou em dúvida sobre qual é termo que, em Português, é mais empregado para definir o que em Inglês se define como a closure de um conjunto, ou seja, o menor conjuntio fechado que contém o dado conjunto. Há umtermo específico? 4) Vizinhança - entendo por vizinhança de um ponto qualquer conjunto aberto que contenha o ponto. Mas alguns autores entendem por este termo qualquer conjunto que contenha um conjunto aberto que contenha o ponto. Acho que esta definição mais geral só cria confusão. No caso de espaços métricos, há ainda alguns autores, principalmente em Inglês, que confundem vizinhança, no caso neighborhood, com bola aberta (open ball). Alguns chegan a usar a epressão r - neighborhood, para significar a bola aberta de raio r. Obrigado Artur ========================================================================= Instruções para entrar na lista, sair da lista e usar a lista em http://www.mat.puc-rio.br/~nicolau/olimp/obm-l.html O administrador desta lista é ========================================================================= From owner-obm-l@sucuri.mat.puc-rio.br Fri Sep 6 16:34:51 2002 Return-Path: Received: (from majordom@localhost) by sucuri.mat.puc-rio.br (8.9.3/8.9.3) id QAA13405 for obm-l-MTTP; Fri, 6 Sep 2002 16:34:08 -0300 Received: from zeus.opendf.com.br (zeus.opengate.com.br [200.181.71.10]) by sucuri.mat.puc-rio.br (8.9.3/8.9.3) with ESMTP id QAA13401 for ; Fri, 6 Sep 2002 16:34:05 -0300 Received: from localhost (localhost.opengate.com.br [127.0.0.1]) by zeus.opendf.com.br (Postfix) with ESMTP id BB99B3EA9C for ; Fri, 6 Sep 2002 16:34:47 -0300 (BRT) Received: by zeus.opendf.com.br (Postfix, from userid 48) id F20BA3EAE9; Fri, 6 Sep 2002 16:34:46 -0300 (BRT) From: "498 - Artur Costa Steiner" To: obm-l@mat.puc-rio.br, Subject: [obm-l] Algumas definições X-Mailer: NeoMail 1.25 X-IPAddress: 200.252.155.2 MIME-Version: 1.0 Content-Type: text/plain; charset=iso-8859-1 Message-Id: <20020906193446.F20BA3EAE9@zeus.opendf.com.br> Date: Fri, 6 Sep 2002 16:34:46 -0300 (BRT) X-Virus-Scanned: by AMaViS new-20020517 Sender: owner-obm-l@sucuri.mat.puc-rio.br Precedence: bulk Reply-To: obm-l@mat.puc-rio.br Alguns conceitos em matemática parecem varia um pouco conforme seja o autor, principalmente naqueles nativos da lígua inglesa. Gostaria de saber se esu tenho as definições mais comumentes usada para os seguintes conceitos: 1) Ponto de acumulação - x é ponto de acumulação de A se qualquer vizinhança de x contiver pelo menos um elemento de A distinto de x. Ém inglês, geralmente isto se chama accumulation point ou, menos comumente, cluster point. 2) Ponto limite - há certa confusão. Em Português, acho que sempre dizemos que x é ponto limite de A se qualquer vizinhança de x contiver elementos de A e de seu complementar C(A). Mas há autores , principalmente em Inglês, (aliás, muitos), que entendem por limit point o mesmo que accumulation point. Para o que defini comoponto limite, este autores usam boundary point. Em Português, já vi ponto de fronteira, mas apenas em livros de programação matemática, e não em livros básicos sobre Análise. 3) Estou em dúvida sobre qual é termo que, em Português, é mais empregado para definir o que em Inglês se define como a closure de um conjunto, ou seja, o menor conjuntio fechado que contém o dado conjunto. Há umtermo específico? 4) Vizinhança - entendo por vizinhança de um ponto qualquer conjunto aberto que contenha o ponto. Mas alguns autores entendem por este termo qualquer conjunto que contenha um conjunto aberto que contenha o ponto. Acho que esta definição mais geral só cria confusão. No caso de espaços métricos, há ainda alguns autores, principalmente em Inglês, que confundem vizinhança, no caso neighborhood, com bola aberta (open ball). Alguns chegan a usar a epressão r - neighborhood, para significar a bola aberta de raio r. Obrigado Artur ========================================================================= Instruções para entrar na lista, sair da lista e usar a lista em http://www.mat.puc-rio.br/~nicolau/olimp/obm-l.html O administrador desta lista é ========================================================================= From owner-obm-l@sucuri.mat.puc-rio.br Fri Sep 6 17:01:14 2002 Return-Path: Received: (from majordom@localhost) by sucuri.mat.puc-rio.br (8.9.3/8.9.3) id RAA14505 for obm-l-MTTP; Fri, 6 Sep 2002 17:00:31 -0300 Received: from pina.terra.com.br (pina.terra.com.br [200.176.3.17]) by sucuri.mat.puc-rio.br (8.9.3/8.9.3) with ESMTP id RAA14501 for ; Fri, 6 Sep 2002 17:00:29 -0300 Received: from engenho.terra.com.br (engenho.terra.com.br [200.176.3.42]) by pina.terra.com.br (Postfix) with ESMTP id 4EDE75305C for ; Fri, 6 Sep 2002 17:01:16 -0300 (EST) Received: from stabel (unknown [200.203.39.93]) (authenticated user dudasta) by engenho.terra.com.br (Postfix) with ESMTP id 3D32B68120 for ; Fri, 6 Sep 2002 17:01:15 -0300 (EST) Message-ID: <000e01c255e0$27c607c0$0301a8c0@stabel> From: "Eduardo Casagrande Stabel" To: References: <20020906193446.F20BA3EAE9@zeus.opendf.com.br> Subject: [obm-l] =?iso-8859-1?Q?Re:_=5Bobm-l=5D_Algumas_defini=E7=F5es?= Date: Fri, 6 Sep 2002 17:01:13 -0300 MIME-Version: 1.0 Content-Type: text/plain; charset="iso-8859-1" Content-Transfer-Encoding: 8bit X-Priority: 3 X-MSMail-Priority: Normal X-Mailer: Microsoft Outlook Express 6.00.2600.0000 X-MimeOLE: Produced By Microsoft MimeOLE V6.00.2600.0000 Sender: owner-obm-l@sucuri.mat.puc-rio.br Precedence: bulk Reply-To: obm-l@mat.puc-rio.br Olá Artur! From: "498 - Artur Costa Steiner" > Alguns conceitos em matemática parecem varia um pouco conforme seja o > autor, principalmente naqueles nativos da lígua inglesa. Gostaria de > saber se esu tenho as definições mais comumentes usada para os > seguintes conceitos: > > 1) Ponto de acumulação - x é ponto de acumulação de A se qualquer > vizinhança de x contiver pelo menos um elemento de A distinto de x. Ém > inglês, geralmente isto se chama accumulation point ou, menos > comumente, cluster point. Nunca vi definição diferente dessa. > 2) Ponto limite - há certa confusão. Em Português, acho que sempre > dizemos que x é ponto limite de A se qualquer vizinhança de x contiver > elementos de A e de seu complementar C(A). Mas há autores , > principalmente em Inglês, (aliás, muitos), que entendem por limit point > o mesmo que accumulation point. Para o que defini comoponto limite, > este autores usam boundary point. Em Português, já vi ponto de > fronteira, mas apenas em livros de programação matemática, e não em > livros básicos sobre Análise. Eu sempre vi ponto limite (em português) com o a mesma definição de ponto de aderência = toda vizinhança de a contém pontos de X (pode ser só o a). Vem de: a é ponto limite se existir seq. {x_n} em X que converge a a, sem precisar que os {x_n} sejam distintos. Portanto todo ponto de a é ponto limite. Mas em Inglês, eles geralmente chamam limit point = accumulation point. Ou seja, eles não tem um termo para o nosso ponto limite. A definição de ponto aderente a X ao complementar de X é ponto de fronteira, e está por toda parte dos livros de matemática. > 3) Estou em dúvida sobre qual é termo que, em Português, é mais > empregado para definir o que em Inglês se define como a closure de um > conjunto, ou seja, o menor conjuntio fechado que contém o dado > conjunto. Há umtermo específico? closure = fecho > 4) Vizinhança - entendo por vizinhança de um ponto qualquer conjunto > aberto que contenha o ponto. Mas alguns autores entendem por este termo > qualquer conjunto que contenha um conjunto aberto que contenha o ponto. > Acho que esta definição mais geral só cria confusão. No caso de espaços > métricos, há ainda alguns autores, principalmente em Inglês, que > confundem vizinhança, no caso neighborhood, com bola aberta (open > ball). Alguns chegan a usar a epressão r - neighborhood, para > significar a bola aberta de raio r. É comum ver se chamar de neighborhood à bola aberta, eu já vi isso também. Acho que o termo mais usado é neighborhood para um conjunto que contenha uma bola aberta de a, e open neighborhood para um conjunto aberto que contenha a. Acho que em português vale o mesmo para vizinhança e vizinhança aberta. Eduardo. Porto Alegre, RS. > > Obrigado > Artur > ========================================================================= > Instruções para entrar na lista, sair da lista e usar a lista em > http://www.mat.puc-rio.br/~nicolau/olimp/obm-l.html > O administrador desta lista é > ========================================================================= > > ========================================================================= Instruções para entrar na lista, sair da lista e usar a lista em http://www.mat.puc-rio.br/~nicolau/olimp/obm-l.html O administrador desta lista é ========================================================================= From owner-obm-l@sucuri.mat.puc-rio.br Fri Sep 6 17:31:10 2002 Return-Path: Received: (from majordom@localhost) by sucuri.mat.puc-rio.br (8.9.3/8.9.3) id RAA15726 for obm-l-MTTP; Fri, 6 Sep 2002 17:30:53 -0300 Received: (from nicolau@localhost) by sucuri.mat.puc-rio.br (8.9.3/8.9.3) id RAA15721 for obm-l@mat.puc-rio.br; Fri, 6 Sep 2002 17:30:52 -0300 Date: Fri, 6 Sep 2002 17:30:52 -0300 From: "Nicolau C. Saldanha" To: obm-l@mat.puc-rio.br Subject: [obm-l] Re: =?iso-8859-1?Q?=5Bobm-l=5D_D=FAvida?= Message-ID: <20020906173052.A15682@sucuri.mat.puc-rio.br> References: Mime-Version: 1.0 Content-Type: text/plain; charset=iso-8859-1 Content-Disposition: inline Content-Transfer-Encoding: 8bit User-Agent: Mutt/1.2.5i In-Reply-To: ; from acboero@hotmail.com on Fri, Sep 06, 2002 at 12:13:59PM -0300 Sender: owner-obm-l@sucuri.mat.puc-rio.br Precedence: bulk Reply-To: obm-l@mat.puc-rio.br On Fri, Sep 06, 2002 at 12:13:59PM -0300, Ana Carolina Boero wrote: > Olá colegas da lista, > > Estou com uma dúvida e gostaria de saber se alguém poderia me ajudar: > > Suponha uma seqüência de números reais, crescente, tal que a diferença entre > termos sucessivos vai a zero. Será que existe um limite finito para essa > seqüência? Não, considere a seqüência a_n = log(n). []s, N. ========================================================================= Instruções para entrar na lista, sair da lista e usar a lista em http://www.mat.puc-rio.br/~nicolau/olimp/obm-l.html O administrador desta lista é ========================================================================= From owner-obm-l@sucuri.mat.puc-rio.br Fri Sep 6 18:03:02 2002 Return-Path: Received: (from majordom@localhost) by sucuri.mat.puc-rio.br (8.9.3/8.9.3) id SAA17150 for obm-l-MTTP; Fri, 6 Sep 2002 18:02:30 -0300 Received: from hotmail.com (f182.pav2.hotmail.com [64.4.37.182]) by sucuri.mat.puc-rio.br (8.9.3/8.9.3) with ESMTP id SAA17017 for ; Fri, 6 Sep 2002 18:02:05 -0300 Received: from mail pickup service by hotmail.com with Microsoft SMTPSVC; Fri, 6 Sep 2002 14:00:46 -0700 Received: from 200.199.193.135 by pv2fd.pav2.hotmail.msn.com with HTTP; Fri, 06 Sep 2002 21:00:45 GMT X-Originating-IP: [200.199.193.135] From: "ricardo matos" To: obm-l@mat.puc-rio.br Subject: Re: [obm-l] Primos Gaussianos Date: Fri, 06 Sep 2002 18:00:45 -0300 Mime-Version: 1.0 Content-Type: text/plain; charset=iso-8859-1; format=flowed Message-ID: X-OriginalArrivalTime: 06 Sep 2002 21:00:46.0461 (UTC) FILETIME=[78A46AD0:01C255E8] Sender: owner-obm-l@sucuri.mat.puc-rio.br Precedence: bulk Reply-To: obm-l@mat.puc-rio.br Na verdade os primos Gaussianos tem a mesma definição que os primos inteiros(ou quase), uma definição mais geral é que um primo em um domínio de integridade(um conjunto com uma série de propridades parecidas com as dos inteiros como os números Gaussianos) é aquele elementos(não inversíveis) que se são o produto de dois elementos do conjunto um deles é inversível. Nos inteiros os únicos números inversíveis são 1 e -1 agora em números Gaussianos são 1,-1,i e -i. O que ele enunciou(não confirmei) é uma consequência desta definição(não obviamente). Quando fatoramos um número desejamos quebrá-lo ao máximo até(possivelmente)chegar em seus fatores irredutíveis, os primos, os fatores inversíveis como 1 e -1 não acrescentam informação como em 1=(-1).(-1) ou 7=(-1).(-7) 14=2.7=(-2)(-7) por isso(eu penso) a definição. Uma coisa que podemos não reparar(por parecer natural) é que nos inteiros é crucial o fato de termos fatoração única em primos(a menos de inversíveis), agora em domínios de integridade mesmo que o número seja completamente fatorável podemos não ter unicidade de fatoração(desculpa não lembro contra-exemplo) mas em vários domínios de integridade nós temos fatoração única como nos Gaussianos. Quanto a utilidade dos Gaussianos me lempbro que com eles era mais fácil demonstrar que x^4+y^4=z^4 em inteiros então um deles é zero. Sei que a teoria de domínios de fatoração única tem várias aplicações em teoria dos números, bem dê uma olhada em algum livro de teoria dos números como o do Hardy ou outro que tenha. _________________________________________________________________ MSN Photos é a maneira mais fácil e prática de editar e compartilhar sua fotos: http://photos.msn.com.br ========================================================================= Instruções para entrar na lista, sair da lista e usar a lista em http://www.mat.puc-rio.br/~nicolau/olimp/obm-l.html O administrador desta lista é ========================================================================= From owner-obm-l@sucuri.mat.puc-rio.br Fri Sep 6 18:10:11 2002 Return-Path: Received: (from majordom@localhost) by sucuri.mat.puc-rio.br (8.9.3/8.9.3) id SAA17409 for obm-l-MTTP; Fri, 6 Sep 2002 18:10:05 -0300 Received: from ginsberg.uol.com.br (ginsberg.uol.com.br [200.221.4.48]) by sucuri.mat.puc-rio.br (8.9.3/8.9.3) with ESMTP id SAA17405 for ; Fri, 6 Sep 2002 18:10:03 -0300 Received: from ui.uol.com.br ([200.160.246.132]) by ginsberg.uol.com.br (8.9.1/8.9.1) with ESMTP id SAA17326 for ; Fri, 6 Sep 2002 18:09:11 -0300 (BRT) Message-Id: <5.1.0.14.2.20020906174140.018336b0@pop3.uol.com.br> X-Sender: fnicks@uol.com.br@pop3.uol.com.br (Unverified) X-Mailer: QUALCOMM Windows Eudora Version 5.1 Date: Fri, 06 Sep 2002 18:10:46 -0300 To: obm-l@mat.puc-rio.br From: Nicks Subject: [obm-l] probabilidade Mime-Version: 1.0 Content-Type: text/plain; charset="iso-8859-1"; format=flowed Content-Transfer-Encoding: 8bit X-MIME-Autoconverted: from quoted-printable to 8bit by sucuri.mat.puc-rio.br id SAA17406 Sender: owner-obm-l@sucuri.mat.puc-rio.br Precedence: bulk Reply-To: obm-l@mat.puc-rio.br Olá , Poderiam me ajudar no seguinte problema ? Um inspetor sabe que o chefe de 5 bandidos é o mais baixo de todos e que todas as alturas são diferentes . Sabe -se também que eles estarão presentes numa reunião em um edifício . Depois da reunião , os bandidos por medida de precaução deixam o edifício em um intervalo de 15 minutos .Como o inspetor não sabe qual deles é o mais baixo , decide deixar sair os dois primeiros bandidos , e prender o primeiro dos seguintes que seja mais baixo do que os que até esse momento sairam .Qual a probabilidade do inspetor prender a pessoa certa ? []´s Nick ========================================================================= Instruções para entrar na lista, sair da lista e usar a lista em http://www.mat.puc-rio.br/~nicolau/olimp/obm-l.html O administrador desta lista é ========================================================================= From owner-obm-l@sucuri.mat.puc-rio.br Fri Sep 6 18:12:50 2002 Return-Path: Received: (from majordom@localhost) by sucuri.mat.puc-rio.br (8.9.3/8.9.3) id SAA17540 for obm-l-MTTP; Fri, 6 Sep 2002 18:12:38 -0300 Received: from zeus.opendf.com.br (zeus.opengate.com.br [200.181.71.10]) by sucuri.mat.puc-rio.br (8.9.3/8.9.3) with ESMTP id SAA17535 for ; Fri, 6 Sep 2002 18:12:35 -0300 Received: from localhost (localhost.opengate.com.br [127.0.0.1]) by zeus.opendf.com.br (Postfix) with ESMTP id D05D03EA39 for ; Fri, 6 Sep 2002 18:13:17 -0300 (BRT) Received: by zeus.opendf.com.br (Postfix, from userid 48) id 5C3A33EA50; Fri, 6 Sep 2002 18:13:17 -0300 (BRT) From: "498 - Artur Costa Steiner" To: obm-l@mat.puc-rio.br Subject: Re: [obm-l] Re: [obm-l] Algumas definições X-Mailer: NeoMail 1.25 X-IPAddress: 200.252.155.2 MIME-Version: 1.0 Content-Type: text/plain; charset=iso-8859-1 Message-Id: <20020906211317.5C3A33EA50@zeus.opendf.com.br> Date: Fri, 6 Sep 2002 18:13:17 -0300 (BRT) X-Virus-Scanned: by AMaViS new-20020517 Sender: owner-obm-l@sucuri.mat.puc-rio.br Precedence: bulk Reply-To: obm-l@mat.puc-rio.br Obrigado, Eduardo! Artur > Olá Artur! > > From: "498 - Artur Costa Steiner" > > Alguns conceitos em matemática parecem varia um pouco conforme seja o > > autor, principalmente naqueles nativos da lígua inglesa. Gostaria de > > saber se esu tenho as definições mais comumentes usada para os > > seguintes conceitos:... ========================================================================= Instruções para entrar na lista, sair da lista e usar a lista em http://www.mat.puc-rio.br/~nicolau/olimp/obm-l.html O administrador desta lista é ========================================================================= From owner-obm-l@sucuri.mat.puc-rio.br Fri Sep 6 18:40:54 2002 Return-Path: Received: (from majordom@localhost) by sucuri.mat.puc-rio.br (8.9.3/8.9.3) id SAA19221 for obm-l-MTTP; Fri, 6 Sep 2002 18:40:47 -0300 Received: from bidu.ime.usp.br (bidu.ime.usp.br [143.107.45.12]) by sucuri.mat.puc-rio.br (8.9.3/8.9.3) with SMTP id SAA19217 for ; Fri, 6 Sep 2002 18:40:44 -0300 Received: (qmail 26120 invoked from network); 6 Sep 2002 21:41:32 -0000 Received: from jaca.ime.usp.br (143.107.45.56) by bidu.ime.usp.br with SMTP; 6 Sep 2002 21:41:32 -0000 Received: (qmail 24338 invoked by uid 216); 6 Sep 2002 21:41:31 -0000 Date: Fri, 6 Sep 2002 18:41:31 -0300 (EST) From: Angelo Barone Netto X-Sender: barone@jaca To: obm-l@mat.puc-rio.br Subject: [obm-l] =?ISO-8859-1?Q?Re=3A_=5Bobm-l=5D_Uma_prova_simples_para_a_s?= =?ISO-8859-1?Q?eguinte_afirma=E7=E3o?= In-Reply-To: <20020906191024.0515F3EAE4@zeus.opendf.com.br> Message-ID: MIME-Version: 1.0 Content-Type: TEXT/PLAIN; charset=US-ASCII Sender: owner-obm-l@sucuri.mat.puc-rio.br Precedence: bulk Reply-To: obm-l@mat.puc-rio.br Caro Artur. Para cada ponto de A tome um aberto que so encontra A nesse ponto. Em cada um dos abertos tome um ponto com todas as coordenadas racionais. Pronto. Ja de enumeravel. Angelo Barone{\ --\ }Netto Universidade de Sao Paulo Departamento de Matematica Aplicada Instituto de Matematica e Estatistica Rua do Matao, 1010 Butanta - Cidade Universitaria Caixa Postal 66 281 phone +55-11-3091-6162/6224/6136 05311-970 - Sao Paulo - SP fax +55-11-3091-6131 Agencia Cidade de Sao Paulo . ========================================================================= Instruções para entrar na lista, sair da lista e usar a lista em http://www.mat.puc-rio.br/~nicolau/olimp/obm-l.html O administrador desta lista é ========================================================================= From owner-obm-l@sucuri.mat.puc-rio.br Fri Sep 6 18:52:45 2002 Return-Path: Received: (from majordom@localhost) by sucuri.mat.puc-rio.br (8.9.3/8.9.3) id SAA19626 for obm-l-MTTP; Fri, 6 Sep 2002 18:52:36 -0300 Received: from hotmail.com (f212.pav2.hotmail.com [64.4.37.212]) by sucuri.mat.puc-rio.br (8.9.3/8.9.3) with ESMTP id SAA19622 for ; Fri, 6 Sep 2002 18:52:33 -0300 Received: from mail pickup service by hotmail.com with Microsoft SMTPSVC; Fri, 6 Sep 2002 14:53:20 -0700 Received: from 200.199.193.135 by pv2fd.pav2.hotmail.msn.com with HTTP; Fri, 06 Sep 2002 21:53:20 GMT X-Originating-IP: [200.199.193.135] From: "ricardo matos" To: obm-l@mat.puc-rio.br Subject: [obm-l] =?iso-8859-1?B?UmU6IFtvYm0tbF0gVW1hIHByb3ZhIHNpbXBsZXMgcGFyYSBhIHNlZ3Vp?= =?iso-8859-1?B?bnRlIGFmaXJtYefjbw==?= Date: Fri, 06 Sep 2002 18:53:20 -0300 Mime-Version: 1.0 Content-Type: text/plain; charset=iso-8859-1; format=flowed Message-ID: X-OriginalArrivalTime: 06 Sep 2002 21:53:20.0358 (UTC) FILETIME=[D082E860:01C255EF] Sender: owner-obm-l@sucuri.mat.puc-rio.br Precedence: bulk Reply-To: obm-l@mat.puc-rio.br Tem um professor meu que fala que quando alguma coisa parece verdade mas não temos nenhuma idéia como demonstrar tentamos fazer por absurdo. Aqui vai a demonstração Suponha que ele seja não-enumerável logo se dividirmos o R^n em enúmeráveis cubos de lado 1(os de coordenadas inteiras) temos que empelo menos algum deles possui um conjunto não-enumerável pois, caso contrário teríamos um absurdo agora pegamos este cubo e dividimos em cubos menores, analogamente existe algum cubinho com um conjunto não enumerável de pontos e assim por diante ponto que está contido emtos esse cubinhos é um ponto de acumulação. Na verdade no primeiro cubo se usarmos que em todo compacto toda sequência, possui subsequência convergente ou(que é a mesma coisa) que todo conjunto enumerável em um compacto tem ponto de acumulação temos o resulado. Esses resultados em R^n são demonstrados de maneira análoga a nossa resolução. (não tive esta idéia fantástica vi em algum lugar) _________________________________________________________________ MSN Photos é a maneira mais fácil e prática de editar e compartilhar sua fotos: http://photos.msn.com.br ========================================================================= Instruções para entrar na lista, sair da lista e usar a lista em http://www.mat.puc-rio.br/~nicolau/olimp/obm-l.html O administrador desta lista é ========================================================================= From owner-obm-l@sucuri.mat.puc-rio.br Fri Sep 6 18:55:43 2002 Return-Path: Received: (from majordom@localhost) by sucuri.mat.puc-rio.br (8.9.3/8.9.3) id SAA19799 for obm-l-MTTP; Fri, 6 Sep 2002 18:55:42 -0300 Received: from hotmail.com (f80.pav2.hotmail.com [64.4.37.80]) by sucuri.mat.puc-rio.br (8.9.3/8.9.3) with ESMTP id SAA19795 for ; Fri, 6 Sep 2002 18:55:39 -0300 Received: from mail pickup service by hotmail.com with Microsoft SMTPSVC; Fri, 6 Sep 2002 14:56:25 -0700 Received: from 200.199.193.135 by pv2fd.pav2.hotmail.msn.com with HTTP; Fri, 06 Sep 2002 21:56:25 GMT X-Originating-IP: [200.199.193.135] From: "ricardo matos" To: obm-l@mat.puc-rio.br Subject: [obm-l] =?iso-8859-1?B?UmU6IFtvYm0tbF0gVW1hIHByb3ZhIHNpbXBsZXMgcGFyYSBhIHNlZ3Vp?= =?iso-8859-1?B?bnRlIGFmaXJtYefjbw==?= Date: Fri, 06 Sep 2002 18:56:25 -0300 Mime-Version: 1.0 Content-Type: text/plain; charset=iso-8859-1; format=flowed Message-ID: X-OriginalArrivalTime: 06 Sep 2002 21:56:25.0890 (UTC) FILETIME=[3F18D820:01C255F0] Sender: owner-obm-l@sucuri.mat.puc-rio.br Precedence: bulk Reply-To: obm-l@mat.puc-rio.br Tem um professor meu que fala que quando alguma coisa parece verdade mas não temos nenhuma idéia como demonstrar tentamos fazer por absurdo. Aqui vai a demonstração: Suponha que ele seja não-enumerável logo se dividirmos o R^n em enúmeráveis cubos de lado 1(os de coordenadas inteiras) temos que em pelo menos algum deles possui um conjunto não-enumerável pois, caso contrário teríamos um absurdo. Agora pegamos este cubo e dividimos em cubos menores, analogamente existe algum cubinho com um conjunto não enumerável de pontos, e assim por diante o ponto que está contido em todos esse cubinhos é um ponto de acumulação. Na verdade no primeiro cubo se usarmos que em todo compacto toda sequência, possui subsequência convergente ou(que é a mesma coisa) que todo conjunto enumerável em um compacto tem ponto de acumulação temos o resulado. Esses resultados em R^n são demonstrados de maneira análoga a nossa resolução. (não tive esta idéia fantástica vi em algum lugar) garoto de quinze anos???? _________________________________________________________________ MSN Photos é a maneira mais fácil e prática de editar e compartilhar sua fotos: http://photos.msn.com.br ========================================================================= Instruções para entrar na lista, sair da lista e usar a lista em http://www.mat.puc-rio.br/~nicolau/olimp/obm-l.html O administrador desta lista é ========================================================================= From owner-obm-l@sucuri.mat.puc-rio.br Fri Sep 6 19:06:22 2002 Return-Path: Received: (from majordom@localhost) by sucuri.mat.puc-rio.br (8.9.3/8.9.3) id TAA20319 for obm-l-MTTP; Fri, 6 Sep 2002 19:05:58 -0300 Received: from trex.centroin.com.br (trex.centroin.com.br [200.225.63.134]) by sucuri.mat.puc-rio.br (8.9.3/8.9.3) with ESMTP id TAA20314 for ; Fri, 6 Sep 2002 19:05:56 -0300 Received: from trex.centroin.com.br (localhost [127.0.0.1]) by trex.centroin.com.br (8.12.5/8.12.1) with ESMTP id g86M77Ev014608 for ; Fri, 6 Sep 2002 19:07:07 -0300 (BRT) Received: by trex.centroin.com.br (8.12.5/8.12.5/Submit) id g86M77Be014606; Fri, 6 Sep 2002 19:07:07 -0300 (BRT) Message-Id: <200209062207.g86M77Be014606@trex.centroin.com.br> Received: from 200.225.58.146 by trex.centroin.com.br (CIPWM versao 1.4C1) with HTTPS for ; Fri, 6 Sep 2002 19:07:06 -0300 (BRT) Date: Fri, 6 Sep 2002 19:07:06 -0300 (BRT) From: Augusto Cesar de Oliveira Morgado To: obm-l@mat.puc-rio.br Subject: =?iso-8859-1?q?Re: [obm-l] Algumas defini=E7=F5es?= MIME-Version: 1.0 X-Mailer: CentroIn Internet Provider WebMail v. 1.4C1 (http://www.centroin.com.br/) Content-Type: text/plain; charset="iso-8859-1" Content-Transfer-Encoding: 8bit X-MIME-Autoconverted: from quoted-printable to 8bit by sucuri.mat.puc-rio.br id TAA20315 Sender: owner-obm-l@sucuri.mat.puc-rio.br Precedence: bulk Reply-To: obm-l@mat.puc-rio.br 3) Fecho 2) Ponto de fronteira parece-me mais comum. Ponto limite me parece mais usado para sequencias: x eh ponto limite da sequencia a_n sse qualquer que seja a vizinhança V de x ha infinitos n para os quais a_n pertence a V. 4) Parece-me que a maioria está contigo. Em Fri, 6 Sep 2002 16:34:46 -0300 (BRT), 498 - Artur Costa Steiner disse: > Alguns conceitos em matemática parecem varia um pouco conforme seja o > autor, principalmente naqueles nativos da lígua inglesa. Gostaria de > saber se esu tenho as definições mais comumentes usada para os > seguintes conceitos: > > 1) Ponto de acumulação - x é ponto de acumulação de A se qualquer > vizinhança de x contiver pelo menos um elemento de A distinto de x. Ém > inglês, geralmente isto se chama accumulation point ou, menos > comumente, cluster point. > > 2) Ponto limite - há certa confusão. Em Português, acho que sempre > dizemos que x é ponto limite de A se qualquer vizinhança de x contiver > elementos de A e de seu complementar C(A). Mas há autores , > principalmente em Inglês, (aliás, muitos), que entendem por limit point > o mesmo que accumulation point. Para o que defini comoponto limite, > este autores usam boundary point. Em Português, já vi ponto de > fronteira, mas apenas em livros de programação matemática, e não em > livros básicos sobre Análise. > > 3) Estou em dúvida sobre qual é termo que, em Português, é mais > empregado para definir o que em Inglês se define como a closure de um > conjunto, ou seja, o menor conjuntio fechado que contém o dado > conjunto. Há umtermo específico? > > 4) Vizinhança - entendo por vizinhança de um ponto qualquer conjunto > aberto que contenha o ponto. Mas alguns autores entendem por este termo > qualquer conjunto que contenha um conjunto aberto que contenha o ponto. > Acho que esta definição mais geral só cria confusão. No caso de espaços > métricos, há ainda alguns autores, principalmente em Inglês, que > confundem vizinhança, no caso neighborhood, com bola aberta (open > ball). Alguns chegan a usar a epressão r - neighborhood, para > significar a bola aberta de raio r. > > Obrigado > Artur > ========================================================================= > Instruções para entrar na lista, sair da lista e usar a lista em > http://www.mat.puc-rio.br/~nicolau/olimp/obm-l.html > O administrador desta lista é > ========================================================================= > > ========================================================================= Instruções para entrar na lista, sair da lista e usar a lista em http://www.mat.puc-rio.br/~nicolau/olimp/obm-l.html O administrador desta lista é ========================================================================= From owner-obm-l@sucuri.mat.puc-rio.br Fri Sep 6 20:13:16 2002 Return-Path: Received: (from majordom@localhost) by sucuri.mat.puc-rio.br (8.9.3/8.9.3) id UAA22131 for obm-l-MTTP; Fri, 6 Sep 2002 20:12:58 -0300 Received: from hotmail.com (f156.pav2.hotmail.com [64.4.37.156]) by sucuri.mat.puc-rio.br (8.9.3/8.9.3) with ESMTP id UAA22127 for ; Fri, 6 Sep 2002 20:12:54 -0300 Received: from mail pickup service by hotmail.com with Microsoft SMTPSVC; Fri, 6 Sep 2002 16:13:41 -0700 Received: from 200.199.193.135 by pv2fd.pav2.hotmail.msn.com with HTTP; Fri, 06 Sep 2002 23:13:41 GMT X-Originating-IP: [200.199.193.135] From: "ricardo matos" To: obm-l@mat.puc-rio.br Subject: [obm-l] =?iso-8859-1?B?UmU6IFtvYm0tbF0gUmU6IFtvYm0tbF0gVW1hIHByb3ZhIHNpbXBsZXMg?= =?iso-8859-1?B?cGFyYSBhIHNlZ3VpbnRlIGFmaXJtYefjbw==?= Date: Fri, 06 Sep 2002 20:13:41 -0300 Mime-Version: 1.0 Content-Type: text/plain; charset=iso-8859-1; format=flowed Message-ID: X-OriginalArrivalTime: 06 Sep 2002 23:13:41.0659 (UTC) FILETIME=[0A3B0EB0:01C255FB] Sender: owner-obm-l@sucuri.mat.puc-rio.br Precedence: bulk Reply-To: obm-l@mat.puc-rio.br >From: Angelo Barone Netto >Reply-To: obm-l@mat.puc-rio.br >To: obm-l@mat.puc-rio.br >Subject: [obm-l] Re: [obm-l] Uma prova simples para a seguinte afirmação >Date: Fri, 6 Sep 2002 18:41:31 -0300 (EST) > >Caro Artur. >Para cada ponto de A tome um aberto que so encontra A nesse ponto. >Em cada um dos abertos tome um ponto com todas as coordenadas racionais. >Pronto. Ja de enumeravel. > >Angelo Barone{\ --\ }Netto Universidade de Sao Paulo >Departamento de Matematica Aplicada Instituto de Matematica e Estatistica >Rua do Matao, 1010 Butanta - Cidade Universitaria >Caixa Postal 66 281 phone +55-11-3091-6162/6224/6136 >05311-970 - Sao Paulo - SP fax +55-11-3091-6131 >Agencia Cidade de Sao Paulo >. > > > > > > > > > > >========================================================================= >Instruções para entrar na lista, sair da lista e usar a lista em >http://www.mat.puc-rio.br/~nicolau/olimp/obm-l.html >O administrador desta lista é >========================================================================= Essa demonstração é massa só que falta garantir que os abertos são disjuntos para não escolhermos o mesmo ponto racional duas vezes(a princípio garantimos que para cada ponto existe um aberto que só contém aquele ponto do conjunto A mas não está garantido que estes abertos não tem intercessões entre si), para isso em cada ponto medimos o ínfimo das distâncias para o resto do conjunto e escolhemos uma bola aberta com metade desse raio e assim está claro que podemos escolher abertos disjuntos. ========================================================================= Instruções para entrar na lista, sair da lista e usar a lista em http://www.mat.puc-rio.br/~nicolau/olimp/obm-l.html O administrador desta lista é ========================================================================= From owner-obm-l@sucuri.mat.puc-rio.br Fri Sep 6 22:17:28 2002 Return-Path: Received: (from majordom@localhost) by sucuri.mat.puc-rio.br (8.9.3/8.9.3) id WAA24357 for obm-l-MTTP; Fri, 6 Sep 2002 22:16:48 -0300 Received: from mail.vetor.com.br (wool.vetor.com.br [200.160.244.7]) by sucuri.mat.puc-rio.br (8.9.3/8.9.3) with ESMTP id WAA24353 for ; Fri, 6 Sep 2002 22:16:46 -0300 Received: from rodrigo (dl-ip182.wb.com.br [200.160.244.182] (may be forged)) by mail.vetor.com.br (8.12.5) with SMTP id g871DKqM019026 for ; Fri, 6 Sep 2002 22:13:21 -0300 Message-ID: <003e01c2560c$24639ee0$b6f4a0c8@rodrigo> From: "Rodrigo Villard Milet" To: Subject: [obm-l] =?iso-8859-1?Q?Re:_=5Bobm-l=5D_3_problemas_ol=EDmpicos?= Date: Fri, 6 Sep 2002 22:14:59 -0300 MIME-Version: 1.0 Content-Type: text/plain; charset="iso-8859-1" Content-Transfer-Encoding: 8bit X-Priority: 3 X-MSMail-Priority: Normal X-Mailer: Microsoft Outlook Express 4.72.3110.5 X-MimeOLE: Produced By Microsoft MimeOLE V4.72.3110.3 X-Virus-Scanned: by amavis (http://amavis.org/) Sender: owner-obm-l@sucuri.mat.puc-rio.br Precedence: bulk Reply-To: obm-l@mat.puc-rio.br Eu enviei a solução do 3 pra eureka 12. Dê uma olhada em www.obm.org.br . -----Mensagem original----- De: fredericogomes Para: obm-l@mat.puc-rio.br Data: Sexta-feira, 6 de Setembro de 2002 02:21 Assunto: [obm-l] 3 problemas olímpicos 1-(Ucrânia 1992)- Demonstrar que não existem soluções reais do sistema: { x^2 + 4yz + 2z =0 { x + 2xy + 2z^2 =0 { 2xz + y^2 + y + 1 =0 2-(China 1993) Achar todas as ternas (x,y,z) de inteiros não negativos tais que: 7^x + 1 = 3^y + 5^z. obs: é óbvio que (0,0,0) e (1,1,1) são soluções e que não temos mais nenhuma solução que envolva inteiro(s) nulo(s), neste caso podemos admitir x,y,z >=1 3-(Iran 1993) Encontrar todos os primos ímpares p tais que [ 2^(p-1) - 1 ] / p é um quadrado perfeito Ficarei imensamente grato se tiver pelo menos um destes três resolvidos. []´s Frederico. __________________________________________________________________________ AcessoBOL, só R$ 9,90! O menor preço do mercado! Assine já! http://www.bol.com.br/acessobol ========================================================================= Instruções para entrar na lista, sair da lista e usar a lista em http://www.mat.puc-rio.br/~nicolau/olimp/obm-l.html O administrador desta lista é ========================================================================= From owner-obm-l@sucuri.mat.puc-rio.br Fri Sep 6 22:51:27 2002 Return-Path: Received: (from majordom@localhost) by sucuri.mat.puc-rio.br (8.9.3/8.9.3) id WAA25112 for obm-l-MTTP; Fri, 6 Sep 2002 22:51:15 -0300 Received: from hotmail.com (f77.sea2.hotmail.com [207.68.165.77]) by sucuri.mat.puc-rio.br (8.9.3/8.9.3) with ESMTP id WAA25108 for ; Fri, 6 Sep 2002 22:51:12 -0300 Received: from mail pickup service by hotmail.com with Microsoft SMTPSVC; Fri, 6 Sep 2002 18:51:59 -0700 Received: from 200.220.36.213 by sea2fd.sea2.hotmail.msn.com with HTTP; Sat, 07 Sep 2002 01:51:58 GMT X-Originating-IP: [200.220.36.213] From: "leonardo mattos" To: obm-l@mat.puc-rio.br Subject: [obm-l] Circulo de 9 pontos e reta de Simson Date: Sat, 07 Sep 2002 01:51:58 +0000 Mime-Version: 1.0 Content-Type: text/plain; charset=iso-8859-1; format=flowed Message-ID: X-OriginalArrivalTime: 07 Sep 2002 01:51:59.0260 (UTC) FILETIME=[273DF5C0:01C25611] Sender: owner-obm-l@sucuri.mat.puc-rio.br Precedence: bulk Reply-To: obm-l@mat.puc-rio.br Ola pessoal, Gostaria muito de saber quais seriam as propriedades do circulo de 9 pontos e da reta de Simson. Um abraço,Leonardo _________________________________________________________________ MSN Photos é a maneira mais fácil e prática de editar e compartilhar sua fotos: http://photos.msn.com.br ========================================================================= Instruções para entrar na lista, sair da lista e usar a lista em http://www.mat.puc-rio.br/~nicolau/olimp/obm-l.html O administrador desta lista é ========================================================================= From owner-obm-l@sucuri.mat.puc-rio.br Sat Sep 7 00:11:15 2002 Return-Path: Received: (from majordom@localhost) by sucuri.mat.puc-rio.br (8.9.3/8.9.3) id AAA26260 for obm-l-MTTP; Sat, 7 Sep 2002 00:10:53 -0300 Received: from hotmail.com (f21.law3.hotmail.com [209.185.241.21]) by sucuri.mat.puc-rio.br (8.9.3/8.9.3) with ESMTP id AAA26256 for ; Sat, 7 Sep 2002 00:10:51 -0300 Received: from mail pickup service by hotmail.com with Microsoft SMTPSVC; Fri, 6 Sep 2002 20:11:38 -0700 Received: from 200.151.33.97 by lw3fd.law3.hotmail.msn.com with HTTP; Sat, 07 Sep 2002 03:11:38 GMT X-Originating-IP: [200.151.33.97] From: "Marcelo Souza" To: obm-l@mat.puc-rio.br Subject: Re: [obm-l] Gabarito da olimpíada carioca. Date: Sat, 07 Sep 2002 03:11:38 +0000 Mime-Version: 1.0 Content-Type: text/plain; format=flowed Message-ID: X-OriginalArrivalTime: 07 Sep 2002 03:11:38.0613 (UTC) FILETIME=[47F55A50:01C2561C] Sender: owner-obm-l@sucuri.mat.puc-rio.br Precedence: bulk Reply-To: obm-l@mat.puc-rio.br Bom, eu fiz as questoes e: 1) A primeira deu 8/15 2) Era só fazer o gráfico das funções log(x) e 2log(5)senx que cortava em 7 pontos. 3)a) a_2001 é par (pela congruencia modulo 2) b) a_2002 era multiplo de 3 (congr. mod 3) c) Sim d) Não 4)Saía por quadrilateros inscritíveis, notando que A_2B_3 era paralelo a AB (ou A_2B_2....naum me liguei muito em qual dos lados ele mandou colocar B_2 ou B_3...tanto faz) falow Abracos Marcelo obs.:Tenho muita certeza dos resultados, mas posso estar errado. Caso esteja, peço que o pessoal da lista corrija. >From: "haroldo" >Reply-To: obm-l@mat.puc-rio.br >To: >Subject: [obm-l] Gabarito da olimpíada carioca. >Date: Tue, 3 Sep 2002 15:27:51 -0300 > >Gostaria de ver o gabarito oficial da olimpíada carioca, realizada na >PUC, dia 31/08. >Pois tenho dois alunos que participaram da competição. >Grato. >Haroldo . _________________________________________________________________ Chat with friends online, try MSN Messenger: http://messenger.msn.com ========================================================================= Instruções para entrar na lista, sair da lista e usar a lista em http://www.mat.puc-rio.br/~nicolau/olimp/obm-l.html O administrador desta lista é ========================================================================= From owner-obm-l@sucuri.mat.puc-rio.br Sat Sep 7 01:06:03 2002 Return-Path: Received: (from majordom@localhost) by sucuri.mat.puc-rio.br (8.9.3/8.9.3) id BAA27138 for obm-l-MTTP; Sat, 7 Sep 2002 01:05:50 -0300 Received: from web40307.mail.yahoo.com (web40307.mail.yahoo.com [66.218.78.86]) by sucuri.mat.puc-rio.br (8.9.3/8.9.3) with SMTP id BAA27134 for ; Sat, 7 Sep 2002 01:05:47 -0300 Message-ID: <20020907040635.23024.qmail@web40307.mail.yahoo.com> Received: from [200.248.101.14] by web40307.mail.yahoo.com via HTTP; Sat, 07 Sep 2002 01:06:35 ART Date: Sat, 7 Sep 2002 01:06:35 -0300 (ART) From: =?iso-8859-1?q?Sharon=20Guedes?= Subject: [obm-l] Geodesias To: obm-l@mat.puc-rio.br In-Reply-To: MIME-Version: 1.0 Content-Type: multipart/alternative; boundary="0-1572309853-1031371595=:15268" Content-Transfer-Encoding: 8bit Sender: owner-obm-l@sucuri.mat.puc-rio.br Precedence: bulk Reply-To: obm-l@mat.puc-rio.br --0-1572309853-1031371595=:15268 Content-Type: text/plain; charset=iso-8859-1 Content-Transfer-Encoding: 8bit Olá pessoal, alguém poderia me ajudar, com algum material sobre geodesia? É para um trabalho que eu tenho que apresentar. At. Sharon. --------------------------------- Yahoo! PageBuilder - O super editor para criação de sites: é grátis, fácil e rápido. --0-1572309853-1031371595=:15268 Content-Type: text/html; charset=iso-8859-1 Content-Transfer-Encoding: 8bit

 Olá pessoal, alguém poderia me ajudar, com algum material sobre geodesia?  É para um trabalho que eu tenho que apresentar.

At. Sharon.


Yahoo! PageBuilder - O super editor para criação de sites: é grátis, fácil e rápido. --0-1572309853-1031371595=:15268-- ========================================================================= Instruções para entrar na lista, sair da lista e usar a lista em http://www.mat.puc-rio.br/~nicolau/olimp/obm-l.html O administrador desta lista é ========================================================================= From owner-obm-l@sucuri.mat.puc-rio.br Sat Sep 7 01:06:32 2002 Return-Path: Received: (from majordom@localhost) by sucuri.mat.puc-rio.br (8.9.3/8.9.3) id BAA27164 for obm-l-MTTP; Sat, 7 Sep 2002 01:06:30 -0300 Received: from sporus.bol.com.br (sporus.bol.com.br [200.221.24.23]) by sucuri.mat.puc-rio.br (8.9.3/8.9.3) with ESMTP id BAA27160 for ; Sat, 7 Sep 2002 01:06:28 -0300 Received: from bol.com.br (200.221.24.132) by sporus.bol.com.br (5.1.071) id 3D63DB28004CAE7B for obm-l@mat.puc-rio.br; Sat, 7 Sep 2002 01:07:13 -0300 Date: Sat, 7 Sep 2002 01:05:26 -0300 Message-Id: Subject: [obm-l] listas de treinamento interncionais MIME-Version: 1.0 Content-Type: text/plain;charset="iso-8859-1" From: "basketboy_igor" To: obm-l@mat.puc-rio.br X-XaM3-API-Version: 2.4.3.4.4 X-SenderIP: 200.253.226.107 Content-Transfer-Encoding: 8bit X-MIME-Autoconverted: from quoted-printable to 8bit by sucuri.mat.puc-rio.br id BAA27161 Sender: owner-obm-l@sucuri.mat.puc-rio.br Precedence: bulk Reply-To: obm-l@mat.puc-rio.br Gostaria de ser informando de sites ou receber lista de treinamentos intenacionais p/ IMO, USAMO, Ibero, torneio das cidades e outras olimpíadas internacionais de matemática, ou de sites que contenham questões vaiadas de matemática vindas de várias partes do mundo. __________________________________________________________________________ AcessoBOL, só R$ 9,90! O menor preço do mercado! Assine já! http://www.bol.com.br/acessobol ========================================================================= Instruções para entrar na lista, sair da lista e usar a lista em http://www.mat.puc-rio.br/~nicolau/olimp/obm-l.html O administrador desta lista é ========================================================================= From owner-obm-l@sucuri.mat.puc-rio.br Sat Sep 7 11:05:14 2002 Return-Path: Received: (from majordom@localhost) by sucuri.mat.puc-rio.br (8.9.3/8.9.3) id KAA30700 for obm-l-MTTP; Sat, 7 Sep 2002 10:57:52 -0300 Received: from shen.bol.com.br (shen.bol.com.br [200.221.24.14]) by sucuri.mat.puc-rio.br (8.9.3/8.9.3) with ESMTP id KAA30696 for ; Sat, 7 Sep 2002 10:57:49 -0300 Received: from bol.com.br (200.221.24.134) by shen.bol.com.br (5.1.071) id 3D63D22F0059F53F for obm-l@mat.puc-rio.br; Sat, 7 Sep 2002 10:57:59 -0300 Date: Sat, 7 Sep 2002 10:56:53 -0300 Message-Id: Subject: [obm-l] funcao MIME-Version: 1.0 Content-Type: text/plain;charset="iso-8859-1" From: "adr.scr.m" To: obm-l@mat.puc-rio.br X-XaM3-API-Version: 2.4.3.4.4 X-SenderIP: 200.151.88.130 Content-Transfer-Encoding: 8bit X-MIME-Autoconverted: from quoted-printable to 8bit by sucuri.mat.puc-rio.br id KAA30697 Sender: owner-obm-l@sucuri.mat.puc-rio.br Precedence: bulk Reply-To: obm-l@mat.puc-rio.br li num livro e gostaria de saber porque eh errado falar " seja uma funcao f(x)",e o certo eh " seja uma funcao f " . []'s. Adriano. __________________________________________________________________________ AcessoBOL, só R$ 9,90! O menor preço do mercado! Assine já! http://www.bol.com.br/acessobol ========================================================================= Instruções para entrar na lista, sair da lista e usar a lista em http://www.mat.puc-rio.br/~nicolau/olimp/obm-l.html O administrador desta lista é ========================================================================= From owner-obm-l@sucuri.mat.puc-rio.br Sat Sep 7 11:23:15 2002 Return-Path: Received: (from majordom@localhost) by sucuri.mat.puc-rio.br (8.9.3/8.9.3) id LAA31014 for obm-l-MTTP; Sat, 7 Sep 2002 11:19:20 -0300 Received: from traven9 (traven9.uol.com.br [200.221.4.35]) by sucuri.mat.puc-rio.br (8.9.3/8.9.3) with ESMTP id LAA31010 for ; Sat, 7 Sep 2002 11:19:18 -0300 Received: from u2z7z2 ([200.158.144.190]) by traven9 (8.9.1/8.9.1) with ESMTP id LAA23998 for ; Sat, 7 Sep 2002 11:21:44 -0300 (BRT) Message-ID: <000a01c25679$d0c99aa0$2101a8c0@u2z7z2> From: "Wagner" To: Subject: [obm-l] =?iso-8859-1?Q?Resposta_para_Eduardo_Casagrande_sobre_o_problem_das_infin?= =?iso-8859-1?Q?itas_solu=E7=F5es?= Date: Sat, 7 Sep 2002 11:21:10 -0300 Organization: Wagner MIME-Version: 1.0 Content-Type: multipart/alternative; boundary="----=_NextPart_000_0007_01C25660.AAEE7A80" X-Priority: 3 X-MSMail-Priority: Normal X-Mailer: Microsoft Outlook Express 5.50.4133.2400 X-MimeOLE: Produced By Microsoft MimeOLE V5.50.4133.2400 Sender: owner-obm-l@sucuri.mat.puc-rio.br Precedence: bulk Reply-To: obm-l@mat.puc-rio.br This is a multi-part message in MIME format. ------=_NextPart_000_0007_01C25660.AAEE7A80 Content-Type: text/plain; charset="iso-8859-1" Content-Transfer-Encoding: quoted-printable Alo, alo pessoal !!! -Eu quis dizer para considerar PI como o resultado da divis=E3o de dois = n=FAmeros inteiros (isso =E9 poss=EDvel pela defini=E7=E3o dada por = Leibniz do n=BA PI, em que PI =E9 o limite da sequ=EAncia : 4 - 4/3 + = 4/5 - 4/7 + 4/9 - ... + 4/n , quando n tende ao infinito e n =E9 = =EDmpar), logo embora isso n=E3o seja poss=EDvel na pr=E1tica, faz = sentido. Se voc=EA considerar um n=BA a de x casas decimais (em que x = =E9 o menor valor poss=EDvel), logo a.10^x =E9 inteiro. Eu apliquei o = mesmo racioc=EDnio no n=BA PI, o problema =E9 que pela defini=E7=E3o, PI = tem infinitas casas decimais. Pela dedu=E7=E3o que eu apresentei, o n=BA = de ra=EDzes seria o numerador da fra=E7=E3o irredut=EDvel entre 2 = n=FAmeros inteiros. Omenor valor poss=EDvel para esse numerador =E9 PI = vezes 10 elevado ao n=BA de casas decimais de PI, que =E9 infinito, logo = x^(PI)-5x^(PI-1)+3=3D0, possui infinitas solu=E7=F5es. -Outra solu=E7=E3o al=E9m dessa e a apresentada pelo Paulo Santa Rita = =E9 a seguinte: Atribuindo aos polin=F4mios uma propriedade chamada semi-grau (acabei de = inventar esse nome e esse conceito), parecida com o grau. Na equa=E7=E3o: x^a + bx^c + dx^e + ... + n =3D 0, a =E9 o semigrau = dela, se e somente se a>c,a>e,...(ou seja, somente se a =E9 o maior = expoente a que x =E9 elevado em qualquer de seus termos). Pela = dedu=E7=E3o, equa=E7=F5es com semi-graus iguais possuem um mesmo n=BA de = solu=E7=F5es complexas, note que a n=E3o =E9 necessariamente um n=BA = inteiro. Logo x^(PI)-5x^(PI-1)+3=3D0, possui o mesmo n=BA de raizes que = x^Pi =3D 1. Pelo 2=BA teorema de De Moivre: x =3D |1^(1/Pi)|.(i.sen(2k) + cos(2k)) = =3D 1.(i.sen(2k) + cos(2k)), com k inteiro. Como Pi =E9 irracional, = n=E3o existem 2 ra=EDzes iguais para 2 valores diferentes de k, logo = existem tantas ra=EDzes quanto forem os valores poss=EDveis de k, ou = seja existem infinitas ra=EDzes. Espero ter esclarecido. Andr=E9 T. S=E3o Paulo-SP Quanto =E0 sua solu=E7=E3o para o problema: mostrar que existem infinitos x complexos tais que = x^(PI)-5x^(PI-1)+3=3D0. Voc=EA fala em considerar (pi)/1 como uma fra=E7=E3o irredut=EDvel, o = que quer dizer isso? Afinal (pi) n=E3o =E9 inteiro. Depois voc=EA fala em (pi)10^n = valores de x, mas como isso =E9 poss=EDvel se (pi) n=E3o =E9 inteiro? Qual o sentido = de 1.5 solu=E7=F5es? Pelo que compreendi a sua solu=E7=E3o est=E1 baseada em = aproxima=E7=F5es de pi por n=FAmeros racionais, ou algo assim, mas n=E3o saquei como = funciona de fato. =C9 interessante que =E0s vezes um m=E9todo informal esconde muito = mais coisa que um todo talhado e bonitinho. Um grande abra=E7o! Eduardo. Porto Alegre, RS. ------=_NextPart_000_0007_01C25660.AAEE7A80 Content-Type: text/html; charset="iso-8859-1" Content-Transfer-Encoding: quoted-printable
Alo, alo pessoal !!!
 
-Eu quis dizer para considerar PI como o resultado da divis=E3o de = dois=20 n=FAmeros inteiros (isso =E9 poss=EDvel pela defini=E7=E3o dada por = Leibniz do n=BA PI, em=20 que PI =E9 o limite da sequ=EAncia : 4 - 4/3 + 4/5 - 4/7 + 4/9 - ... + = 4/n , quando=20 n tende ao infinito e n =E9 =EDmpar), logo embora isso n=E3o seja = poss=EDvel na pr=E1tica,=20 faz sentido. Se voc=EA considerar um n=BA a de x casas decimais (em = que x =E9 o=20 menor valor poss=EDvel), logo a.10^x =E9 inteiro. Eu apliquei o mesmo = racioc=EDnio no=20 n=BA PI, o problema =E9 que pela defini=E7=E3o, PI tem infinitas casas = decimais. Pela=20 dedu=E7=E3o que eu apresentei, o n=BA de ra=EDzes seria o numerador = da fra=E7=E3o=20 irredut=EDvel entre 2 n=FAmeros inteiros. Omenor valor poss=EDvel = para esse=20 numerador =E9 PI vezes 10 elevado ao n=BA de casas decimais de PI, que = =E9 infinito,=20 logo x^(PI)-5x^(PI-1)+3=3D0, possui infinitas solu=E7=F5es.
-Outra solu=E7=E3o al=E9m dessa e a apresentada pelo Paulo Santa = Rita =E9 a=20 seguinte:
Atribuindo aos polin=F4mios uma propriedade chamada semi-grau = (acabei de=20 inventar esse nome e esse conceito), parecida com o grau.
Na equa=E7=E3o: x^a + bx^c + dx^e + ... + n =3D 0, a =E9 o semigrau = dela, se e=20 somente se a>c,a>e,...(ou seja, somente se a =E9 o maior expoente = a que x =E9=20 elevado em qualquer de seus termos). Pela dedu=E7=E3o, equa=E7=F5es com = semi-graus=20 iguais possuem um mesmo n=BA de solu=E7=F5es complexas, note que a n=E3o = =E9=20 necessariamente um n=BA inteiro. Logo x^(PI)-5x^(PI-1)+3=3D0, possui o = mesmo n=BA de=20 raizes que x^Pi =3D 1.
Pelo 2=BA teorema de De Moivre: x =3D = |1^(1/Pi)|.(i.sen(2k) +=20 cos(2k)) =3D 1.(i.sen(2k) + cos(2k)), com k inteiro. Como Pi =E9 = irracional, n=E3o=20 existem 2 ra=EDzes iguais para 2 valores diferentes de k, logo existem = tantas=20 ra=EDzes quanto forem os valores poss=EDveis de k, ou seja existem = infinitas=20 ra=EDzes.
 
Espero ter esclarecido.
Andr=E9 T.
S=E3o Paulo-SP
 
 
Quanto =E0 sua solu=E7=E3o para o problema:
mostrar que existem = infinitos x=20 complexos tais que x^(PI)-5x^(PI-1)+3=3D0.
Voc=EA fala em considerar = (pi)/1 como=20 uma fra=E7=E3o irredut=EDvel, o que quer dizer
isso? Afinal (pi) = n=E3o =E9 inteiro.=20 Depois voc=EA fala em (pi)10^n valores de x,
mas como isso =E9 = poss=EDvel se (pi)=20 n=E3o =E9 inteiro? Qual o sentido de 1.5
solu=E7=F5es? Pelo que = compreendi a sua=20 solu=E7=E3o est=E1 baseada em aproxima=E7=F5es de
pi por n=FAmeros = racionais, ou algo=20 assim, mas n=E3o saquei como funciona de
fato. =C9 interessante que = =E0s vezes um=20 m=E9todo informal esconde muito mais
coisa que um todo talhado e=20 bonitinho.

Um grande abra=E7o!

Eduardo.
Porto Alegre,=20 RS.
------=_NextPart_000_0007_01C25660.AAEE7A80-- ========================================================================= Instruções para entrar na lista, sair da lista e usar a lista em http://www.mat.puc-rio.br/~nicolau/olimp/obm-l.html O administrador desta lista é ========================================================================= From owner-obm-l@sucuri.mat.puc-rio.br Sat Sep 7 11:41:10 2002 Return-Path: Received: (from majordom@localhost) by sucuri.mat.puc-rio.br (8.9.3/8.9.3) id LAA31439 for obm-l-MTTP; Sat, 7 Sep 2002 11:37:16 -0300 Received: from ginsberg.uol.com.br (ginsberg.uol.com.br [200.221.4.48]) by sucuri.mat.puc-rio.br (8.9.3/8.9.3) with ESMTP id LAA31435 for ; Sat, 7 Sep 2002 11:37:14 -0300 Received: from u2z7z2 ([200.158.144.190]) by ginsberg.uol.com.br (8.9.1/8.9.1) with ESMTP id LAA03173 for ; Sat, 7 Sep 2002 11:36:25 -0300 (BRT) Message-ID: <001101c2567c$54076760$2101a8c0@u2z7z2> From: "Wagner" To: Subject: [obm-l] Pergunta para Paulo Santa Rita Date: Sat, 7 Sep 2002 11:39:10 -0300 Organization: Wagner MIME-Version: 1.0 Content-Type: multipart/alternative; boundary="----=_NextPart_000_000E_01C25663.2E87D4C0" X-Priority: 3 X-MSMail-Priority: Normal X-Mailer: Microsoft Outlook Express 5.50.4133.2400 X-MimeOLE: Produced By Microsoft MimeOLE V5.50.4133.2400 Sender: owner-obm-l@sucuri.mat.puc-rio.br Precedence: bulk Reply-To: obm-l@mat.puc-rio.br This is a multi-part message in MIME format. ------=_NextPart_000_000E_01C25663.2E87D4C0 Content-Type: text/plain; charset="iso-8859-1" Content-Transfer-Encoding: quoted-printable Bom dia pra todos! -Nota=E7=E3o log n (a) =3D logaritmo natural de a -(a,b) =3D a + bi Caro Paulo, na sua resposta para o meu problema = (x^(PI)-5x^(PI-1)+3=3D0), voc=EA diz que : - e^Pi.i =3D -1 =3D> (estou considerando que o e da resposta seja o = n=BA neperiano)=20 e^Pi.i =3D (i.sen(Pi) + cos(Pi)), isso implicaria que: e^i(i.sen1 + = cos1), certo? Ent=E3o a^i =3D e^log n (a).i =3D (i.sen(log n (a)) + = cos(log n (a))). Ent=E3o : a^(x,y) =3D a^x.(i.sen(y.log n (a)) + cos(y.log n (a))) ? Ou = seja um n=BA real pode ser elevado a um expoente imagin=E1rio ? Ent=E3o quanto seria (a,b)^(c,d) ? E tamb=E9m qual a dedu=E7=E3o de que = e^Pi.i =3D -1 ? - Tamb=E9m queria saber porque x =3D a.e^T.i e consequentemente x^Pi = =3D a(-1)T. Andr=E9 T. ------=_NextPart_000_000E_01C25663.2E87D4C0 Content-Type: text/html; charset="iso-8859-1" Content-Transfer-Encoding: quoted-printable
Bom dia pra todos!
 
-Nota=E7=E3o log n (a) =3D logaritmo natural de a
-(a,b) =3D a + bi
 
Caro Paulo, na sua resposta para o meu problema = (x^(PI)-5x^(PI-1)+3=3D0),=20 voc=EA diz que :
 
-  e^Pi.i =3D -1 =3D> (estou considerando que o e da = resposta seja=20 o n=BA neperiano) 
e^Pi.i =3D (i.sen(Pi) + cos(Pi)), isso implicaria que: e^i(i.sen1 + = cos1),=20 certo? Ent=E3o a^i =3D e^log n (a).i =3D (i.sen(log n (a)) + cos(log n = (a))).
Ent=E3o : a^(x,y) =3D a^x.(i.sen(y.log n (a)) + cos(y.log n (a))) ? = Ou seja um=20 n=BA real pode ser elevado a um expoente imagin=E1rio ?
Ent=E3o quanto seria (a,b)^(c,d) ? E tamb=E9m qual a dedu=E7=E3o de = que e^Pi.i =3D -1=20 ?
-  Tamb=E9m queria saber porque x =3D a.e^T.i e = consequentemente x^Pi =3D=20 a(-1)T.
 
Andr=E9 T.
------=_NextPart_000_000E_01C25663.2E87D4C0-- ========================================================================= Instruções para entrar na lista, sair da lista e usar a lista em http://www.mat.puc-rio.br/~nicolau/olimp/obm-l.html O administrador desta lista é ========================================================================= From owner-obm-l@sucuri.mat.puc-rio.br Sat Sep 7 12:12:12 2002 Return-Path: Received: (from majordom@localhost) by sucuri.mat.puc-rio.br (8.9.3/8.9.3) id MAA32427 for obm-l-MTTP; Sat, 7 Sep 2002 12:08:20 -0300 Received: from ginsberg.uol.com.br (ginsberg.uol.com.br [200.221.4.48]) by sucuri.mat.puc-rio.br (8.9.3/8.9.3) with ESMTP id MAA32423 for ; Sat, 7 Sep 2002 12:08:18 -0300 Received: from u2z7z2 ([200.158.144.190]) by ginsberg.uol.com.br (8.9.1/8.9.1) with ESMTP id MAA20567 for ; Sat, 7 Sep 2002 12:07:29 -0300 (BRT) Message-ID: <000b01c25680$aaf6d160$2101a8c0@u2z7z2> From: "Wagner" To: References: Subject: Re: [obm-l] funcao Date: Sat, 7 Sep 2002 12:10:13 -0300 Organization: Wagner MIME-Version: 1.0 Content-Type: text/plain; charset="iso-8859-1" Content-Transfer-Encoding: 8bit X-Priority: 3 X-MSMail-Priority: Normal X-Mailer: Microsoft Outlook Express 5.50.4133.2400 X-MimeOLE: Produced By Microsoft MimeOLE V5.50.4133.2400 Sender: owner-obm-l@sucuri.mat.puc-rio.br Precedence: bulk Reply-To: obm-l@mat.puc-rio.br Oi para todos Outro jeito de falar é f em função de x. Se você falar em função f(x), você esta se referindo a (f(x))(a), ou seja função de x em função de uma outra variavel qualquer, ou seja a função muda em função de x e a.(se eu entendi direito a sua pergunta) -exemplo: (f(x))(a) = ax. Se a = 1, (f(x))(1)= x, se a = 2, (f(x))(2)=2x. Esse "tipo de função" não pode ser representado em planos bidimensionais(x,y), mas pode ser representado em planos tridimensionais(x,y,z). André T. ----- Original Message ----- From: "adr.scr.m" To: Sent: Saturday, September 07, 2002 10:56 AM Subject: [obm-l] funcao > li num livro e gostaria de saber porque eh > errado falar " seja uma funcao f(x)",e o > certo eh " seja uma funcao f " . > []'s. > Adriano. > > > __________________________________________________________________________ > AcessoBOL, só R$ 9,90! O menor preço do mercado! > Assine já! http://www.bol.com.br/acessobol > > > ========================================================================= > Instruções para entrar na lista, sair da lista e usar a lista em > http://www.mat.puc-rio.br/~nicolau/olimp/obm-l.html > O administrador desta lista é > ========================================================================= ========================================================================= Instruções para entrar na lista, sair da lista e usar a lista em http://www.mat.puc-rio.br/~nicolau/olimp/obm-l.html O administrador desta lista é ========================================================================= From owner-obm-l@sucuri.mat.puc-rio.br Sat Sep 7 12:13:05 2002 Return-Path: Received: (from majordom@localhost) by sucuri.mat.puc-rio.br (8.9.3/8.9.3) id MAA32442 for obm-l-MTTP; Sat, 7 Sep 2002 12:09:18 -0300 Received: from shen.bol.com.br (shen.bol.com.br [200.221.24.14]) by sucuri.mat.puc-rio.br (8.9.3/8.9.3) with ESMTP id MAA32437 for ; Sat, 7 Sep 2002 12:09:16 -0300 Received: from bol.com.br (200.221.24.135) by shen.bol.com.br (5.1.071) id 3D63D22F005A24EC for obm-l@mat.puc-rio.br; Sat, 7 Sep 2002 12:09:26 -0300 Date: Sat, 7 Sep 2002 12:08:13 -0300 Message-Id: Subject: Re:[obm-l] circuito IME MIME-Version: 1.0 Content-Type: text/plain;charset="iso-8859-1" From: "adr.scr.m" To: obm-l@mat.puc-rio.br X-XaM3-API-Version: 2.4.3.4.4 X-SenderIP: 200.151.91.31 Content-Transfer-Encoding: 8bit X-MIME-Autoconverted: from quoted-printable to 8bit by sucuri.mat.puc-rio.br id MAA32438 Sender: owner-obm-l@sucuri.mat.puc-rio.br Precedence: bulk Reply-To: obm-l@mat.puc-rio.br um tempo atras um garoto perguntou,e eu respondi,veja em listas anteriores. so que ele perguntou como questao 3. obs. mesmo que so pudesse ser feita com integral,nao iria interessar se estava no programa,ja que a questao 10 nao estava no programa,e nem foi anulada. []'s. Adriano. __________________________________________________________________________ AcessoBOL, só R$ 9,90! O menor preço do mercado! Assine já! http://www.bol.com.br/acessobol ========================================================================= Instruções para entrar na lista, sair da lista e usar a lista em http://www.mat.puc-rio.br/~nicolau/olimp/obm-l.html O administrador desta lista é ========================================================================= From owner-obm-l@sucuri.mat.puc-rio.br Sat Sep 7 12:34:43 2002 Return-Path: Received: (from majordom@localhost) by sucuri.mat.puc-rio.br (8.9.3/8.9.3) id MAA00967 for obm-l-MTTP; Sat, 7 Sep 2002 12:30:54 -0300 Received: from silva5.uol.com.br (silva5.uol.com.br [200.221.4.52]) by sucuri.mat.puc-rio.br (8.9.3/8.9.3) with ESMTP id MAA00963 for ; Sat, 7 Sep 2002 12:30:52 -0300 Received: from slap ([200.207.152.104]) by silva5.uol.com.br (8.9.1/8.9.1) with SMTP id MAA07605 for ; Sat, 7 Sep 2002 12:34:03 -0300 (EST) Message-ID: <008401c25684$5c609320$6898cfc8@slap> From: "Afemano" To: Subject: [obm-l] =?iso-8859-1?Q?Teste_dif=EDcil...?= Date: Sat, 7 Sep 2002 12:36:40 -0300 MIME-Version: 1.0 Content-Type: multipart/alternative; boundary="----=_NextPart_000_0081_01C2566B.36E10080" X-Priority: 3 X-MSMail-Priority: Normal X-Mailer: Microsoft Outlook Express 6.00.2600.0000 X-MimeOLE: Produced By Microsoft MimeOLE V6.00.2600.0000 Sender: owner-obm-l@sucuri.mat.puc-rio.br Precedence: bulk Reply-To: obm-l@mat.puc-rio.br This is a multi-part message in MIME format. ------=_NextPart_000_0081_01C2566B.36E10080 Content-Type: text/plain; charset="iso-8859-1" Content-Transfer-Encoding: quoted-printable Oi, sou novo por aqui e n=E3o sei se este teste j=E1 foi respondido.... = falou ... "A forma decimal do n=FAmero 4444 elevado a 4444 possui um certo = n=FAmero de algarismos. A soma desses algarismos =E1 A. A soma dos = algarismos de A =E9 B. Qual a soma dos algarismos de B ???" Isso est=E1 num caderno de exerc=EDcios de um amigo meu do cursinho. Se = algu=E9m puder ajudar ae valeu !! ------=_NextPart_000_0081_01C2566B.36E10080 Content-Type: text/html; charset="iso-8859-1" Content-Transfer-Encoding: quoted-printable
Oi, sou novo por aqui e n=E3o sei se = este teste j=E1=20 foi respondido.... falou ...
 
"A forma decimal do n=FAmero 4444 = elevado a 4444=20 possui um certo n=FAmero de algarismos. A soma desses algarismos =E1 A. = A soma dos=20 algarismos de A =E9 B. Qual a soma dos algarismos de B ???"
 
Isso est=E1 num caderno de exerc=EDcios = de um amigo meu=20 do cursinho. Se algu=E9m puder ajudar ae valeu = !!
------=_NextPart_000_0081_01C2566B.36E10080-- ========================================================================= Instruções para entrar na lista, sair da lista e usar a lista em http://www.mat.puc-rio.br/~nicolau/olimp/obm-l.html O administrador desta lista é ========================================================================= From owner-obm-l@sucuri.mat.puc-rio.br Sat Sep 7 13:14:27 2002 Return-Path: Received: (from majordom@localhost) by sucuri.mat.puc-rio.br (8.9.3/8.9.3) id NAA02159 for obm-l-MTTP; Sat, 7 Sep 2002 13:10:10 -0300 Received: from hotmail.com (f149.sea2.hotmail.com [207.68.165.149]) by sucuri.mat.puc-rio.br (8.9.3/8.9.3) with ESMTP id NAA02155 for ; Sat, 7 Sep 2002 13:10:07 -0300 Received: from mail pickup service by hotmail.com with Microsoft SMTPSVC; Sat, 7 Sep 2002 09:10:56 -0700 Received: from 200.216.9.183 by sea2fd.sea2.hotmail.msn.com with HTTP; Sat, 07 Sep 2002 16:10:55 GMT X-Originating-IP: [200.216.9.183] From: "Paulo Santa Rita" To: obm-l@mat.puc-rio.br Subject: [obm-l] =?iso-8859-1?B?UmU6IFtvYm0tbF0gMyBwcm9ibGVtYXMgb2ztbXBpY29z?= Date: Sat, 07 Sep 2002 16:10:55 +0000 Mime-Version: 1.0 Content-Type: text/plain; charset=iso-8859-1; format=flowed Message-ID: X-OriginalArrivalTime: 07 Sep 2002 16:10:56.0440 (UTC) FILETIME=[25CB8B80:01C25689] Sender: owner-obm-l@sucuri.mat.puc-rio.br Precedence: bulk Reply-To: obm-l@mat.puc-rio.br Ola Frederico e demais colegas desta lista ... OBM-L, ja que o Vilaard fez o terceiro, vou dar uma ideia pros dois primeiro. Voce completa os detalhes : 1) Problema bonito ! Das duas primeiras equacoes tiramos : 2z(2y+1)=-x^2 x(2y+1)=-2z^2 Isto mostra que "x" e "z" devem ter o mesmo sinal ... Logo ... Da terceira equacao, tiramos : y^2+y+1=-2xz. como y^2+y+1 e sempre positivo, isto implica -2xz > 0 segue que xz < 0 e portanto "x" e "z" devem ter sinais diferentes ... Logo ... 2)Esse e trivial, tipo uma imlicacao depois da outra. Suponha (x,y,z) uma solucao : 7^x + 1=3^y + 5^z => 7^x - 1=3^y + 5^z - 2 => (7^x - 1) - 3^y = 5^z - 2. Como 7==1(mod 3) entao 7^x==1(mod 3) => 7^x - 1 e multiplo de 3. Por outro lado, 3^y e multiplo de 3, logo 5^z - 2 e multiplo de 3 => 5^z==2(mod 3) => 5^(z+2)=50(mod 3) => 5^(z+2)==2(mod3) => z e impar ... Seja z=2p+1 Logo ... Um abraco Paulo Santa Rita 7,1308,070902 >From: "fredericogomes" >Reply-To: obm-l@mat.puc-rio.br >To: obm-l@mat.puc-rio.br >Subject: [obm-l] 3 problemas olímpicos >Date: Fri, 6 Sep 2002 02:03:52 -0300 > >1-(Ucrânia 1992)- Demonstrar que não existem soluções >reais do sistema: > { x^2 + 4yz + 2z =0 > { x + 2xy + 2z^2 =0 > { 2xz + y^2 + y + 1 =0 > > >2-(China 1993) Achar todas as ternas (x,y,z) de inteiros >não negativos tais que: 7^x + 1 = 3^y + 5^z. > >obs: é óbvio que (0,0,0) e (1,1,1) são soluções e que >não temos mais nenhuma solução que envolva inteiro(s) >nulo(s), neste caso podemos admitir x,y,z >=1 > >3-(Iran 1993) Encontrar todos os primos ímpares p tais >que [ 2^(p-1) - 1 ] / p é um quadrado perfeito > >Ficarei imensamente grato se tiver pelo menos um destes >três resolvidos. > > []´s Frederico. > > >__________________________________________________________________________ >AcessoBOL, só R$ 9,90! O menor preço do mercado! >Assine já! http://www.bol.com.br/acessobol >PRECISO DE AJUDA, > >ALGUÉM PODE ME ENVIAR CHARGES, CARTOONS, PIADAS OU ADIVINHAS ENVOLVENDO >MATEMÁTICA? > >ABRAÇOS, > >IDA _________________________________________________________________ Converse com seus amigos online, faça o download grátis do MSN Messenger: http://messenger.msn.com.br ========================================================================= Instruções para entrar na lista, sair da lista e usar a lista em http://www.mat.puc-rio.br/~nicolau/olimp/obm-l.html O administrador desta lista é ========================================================================= From owner-obm-l@sucuri.mat.puc-rio.br Sat Sep 7 13:41:15 2002 Return-Path: Received: (from majordom@localhost) by sucuri.mat.puc-rio.br (8.9.3/8.9.3) id NAA02583 for obm-l-MTTP; Sat, 7 Sep 2002 13:38:36 -0300 Received: from www.zipmail.com.br (smtp.zipmail.com.br [200.187.242.10]) by sucuri.mat.puc-rio.br (8.9.3/8.9.3) with ESMTP id NAA02579 for ; Sat, 7 Sep 2002 13:38:34 -0300 From: ghaeser@zipmail.com.br Received: from [200.158.6.170] by www.zipmail.com.br with HTTP; Sat, 7 Sep 2002 13:39:23 -0300 Message-ID: <3D7939C600001DAD@www.zipmail.com.br> Date: Sat, 7 Sep 2002 13:39:23 -0300 In-Reply-To: Subject: [obm-l] =?iso-8859-1?Q?Fibonacci=20generalizado?= To: obm-l@mat.puc-rio.br MIME-Version: 1.0 Content-Type: text/plain; charset="iso-8859-1" Content-Transfer-Encoding: 8bit X-MIME-Autoconverted: from quoted-printable to 8bit by sucuri.mat.puc-rio.br id NAA02580 Sender: owner-obm-l@sucuri.mat.puc-rio.br Precedence: bulk Reply-To: obm-l@mat.puc-rio.br Considere o polinômio P(x)=x^(n+1)-2.x^n+1 que também pode ser escrito como: P(x)=(x-1).(x^n-x^(n-1)-..-x-1) este polinômio (a menos da raiz x=1) gera as sequencias de Fibonacci generalizadas, isto é, uma sequencia de Fibonacci generalizada de ordem n, é tal que cada elemento é a soma dos n anteriores, com uma escolha arbitrária dos n primeiros (pode ser todos iguais a 1 por exemplo.) este polinômio, se n for ímpar, possui uma raiz real positiva que será o limite de x->oo F(x+1)/F(x), que é a razão de ouro para o caso n=2 e as outras raizes sao todas complexas. se n for par, o polinomio possui uma raiz real positiva (que será o limite de x->oo F(x+1)/F(x)), e uma raiz real negativa.. sendo as demais complexas. seja x1,..,xn as n raizes do polinomio, entao podemos escrever a sequencia de Fibonacci de ordem n como sendo: F(t)=Re[A(1).(x1)^t+..+A(n).(xn)^t] onde F(t) é o t-ésimo número da sequencia, e as constantes A(n) podem ser determinadas através das n condições iniciais de F. .. o método funciona muito bem até n=4 porém com n=5, só consegui encontrar as raizes por métodos numéricos, e mesmo com 300 casas decimais de precisão aparecem erros até mesmo nas primeiras componentes da sequencia. se alguem quiser ver o programa, ele está disponível em: http://sites.uol.com.br/ghaeser/fibgen.nb .. a minha pergunta é: será que não consigo encontrar exatamente todas as raízes do polinômio P(x)=x^(n+1)-2.x^n+1 O interessante é que o limite da razão entre os termos consecutivos tende a 2, quando n tende a infinito.. o que é fácil de ver pelo polinomio. Agradeço à comentários Gabriel Haeser www.gabas.cjb.net "Mathematicus nascitur, non fit" Matemáticos não são feitos, eles nascem --------------------------------------- Gabriel Haeser www.gabas.cjb.net ------------------------------------------ Use o melhor sistema de busca da Internet Radar UOL - http://www.radaruol.com.br ========================================================================= Instruções para entrar na lista, sair da lista e usar a lista em http://www.mat.puc-rio.br/~nicolau/olimp/obm-l.html O administrador desta lista é ========================================================================= From owner-obm-l@sucuri.mat.puc-rio.br Sat Sep 7 13:54:59 2002 Return-Path: Received: (from majordom@localhost) by sucuri.mat.puc-rio.br (8.9.3/8.9.3) id NAA02980 for obm-l-MTTP; Sat, 7 Sep 2002 13:52:11 -0300 Received: from ginsberg.uol.com.br (ginsberg.uol.com.br [200.221.4.48]) by sucuri.mat.puc-rio.br (8.9.3/8.9.3) with ESMTP id NAA02976 for ; Sat, 7 Sep 2002 13:52:08 -0300 Received: from ui.uol.com.br ([200.160.246.142]) by ginsberg.uol.com.br (8.9.1/8.9.1) with ESMTP id NAA01856 for ; Sat, 7 Sep 2002 13:51:19 -0300 (BRT) Message-Id: <5.1.0.14.2.20020907135136.0173fcf0@pop3.uol.com.br> X-Sender: paciniv@uol.com.br@pop3.uol.com.br (Unverified) X-Mailer: QUALCOMM Windows Eudora Version 5.1 Date: Sat, 07 Sep 2002 13:52:41 -0300 To: obm-l@mat.puc-rio.br From: Pacini Mime-Version: 1.0 Content-Type: text/plain; charset="iso-8859-1"; format=flowed Content-Transfer-Encoding: 8bit X-MIME-Autoconverted: from quoted-printable to 8bit by sucuri.mat.puc-rio.br id NAA02977 Sender: owner-obm-l@sucuri.mat.puc-rio.br Precedence: bulk Reply-To: obm-l@mat.puc-rio.br To: obm-l@mat.puc-rio.br Subject: [obm-l] probabilidade Olá , Poderiam me ajudar no seguinte problema ? Um inspetor sabe que o chefe de 5 bandidos é o mais baixo de todos e que todas as alturas são diferentes . Sabe -se também que eles estarão presentes numa reunião em um edifício . Depois da reunião , os bandidos por medida de precaução deixam o edifício em um intervalo de 15 minutos .Como o inspetor não sabe qual deles é o mais baixo , decide deixar sair os dois primeiros bandidos , e prender o primeiro dos seguintes que seja mais baixo do que os que até esse momento sairam .Qual a probabilidade do inspetor prender a pessoa certa ? []´s Nick ========================================================================= Instruções para entrar na lista, sair da lista e usar a lista em http://www.mat.puc-rio.br/~nicolau/olimp/obm-l.html O administrador desta lista é ========================================================================= From owner-obm-l@sucuri.mat.puc-rio.br Sat Sep 7 14:13:44 2002 Return-Path: Received: (from majordom@localhost) by sucuri.mat.puc-rio.br (8.9.3/8.9.3) id OAA03639 for obm-l-MTTP; Sat, 7 Sep 2002 14:11:02 -0300 Received: from ginsberg.uol.com.br (ginsberg.uol.com.br [200.221.4.48]) by sucuri.mat.puc-rio.br (8.9.3/8.9.3) with ESMTP id OAA03635 for ; Sat, 7 Sep 2002 14:11:00 -0300 Received: from ui.uol.com.br ([200.160.246.142]) by ginsberg.uol.com.br (8.9.1/8.9.1) with ESMTP id OAA19573 for ; Sat, 7 Sep 2002 14:10:11 -0300 (BRT) Message-Id: <5.1.0.14.2.20020907135312.017450d8@pop3.uol.com.br> X-Sender: paciniv@uol.com.br@pop3.uol.com.br (Unverified) X-Mailer: QUALCOMM Windows Eudora Version 5.1 Date: Sat, 07 Sep 2002 14:11:36 -0300 To: obm-l@mat.puc-rio.br From: Pacini Subject: [obm-l] Re-probabilidade Mime-Version: 1.0 Content-Type: text/plain; charset="iso-8859-1"; format=flowed Content-Transfer-Encoding: 8bit X-MIME-Autoconverted: from quoted-printable to 8bit by sucuri.mat.puc-rio.br id OAA03636 Sender: owner-obm-l@sucuri.mat.puc-rio.br Precedence: bulk Reply-To: obm-l@mat.puc-rio.br Olá , Tenho uma dúvida com relação a este problema . Enumerando as alturas por 1,2,3,4 e5 ; o número de maneiras de sair os dois primeiros é C5,2 = 10 . Como o inspetor irá prender o mais baixo que saiu até o momento , o Espaço Amostral será 10 X 3! = 60 ? ; pois se dentre os dois primeiros que sairem estiver o mais baixo , o inspetor não irá prender alguém ou , seja , a minha dúvida é a seguinte : a sequencia 12345 ,12453 por exemplo não deverá entrar duas vezes para o Espaço Amostral . O que vocês acham ? Pacini To: obm-l@mat.puc-rio.br Subject: [obm-l] probabilidade Olá , Poderiam me ajudar no seguinte problema ? Um inspetor sabe que o chefe de 5 bandidos é o mais baixo de todos e que todas as alturas são diferentes . Sabe -se também que eles estarão presentes numa reunião em um edifício . Depois da reunião , os bandidos por medida de precaução deixam o edifício em um intervalo de 15 minutos .Como o inspetor não sabe qual deles é o mais baixo , decide deixar sair os dois primeiros bandidos , e prender o primeiro dos seguintes que seja mais baixo do que os que até esse momento sairam .Qual a probabilidade do inspetor prender a pessoa certa ? []´s Nick ========================================================================= Instruções para entrar na lista, sair da lista e usar a lista em http://www.mat.puc-rio.br/~nicolau/olimp/obm-l.html O administrador desta lista é ========================================================================= From owner-obm-l@sucuri.mat.puc-rio.br Sat Sep 7 14:58:02 2002 Return-Path: Received: (from majordom@localhost) by sucuri.mat.puc-rio.br (8.9.3/8.9.3) id OAA04562 for obm-l-MTTP; Sat, 7 Sep 2002 14:53:32 -0300 Received: from toole.uol.com.br (toole.uol.com.br [200.221.4.26]) by sucuri.mat.puc-rio.br (8.9.3/8.9.3) with ESMTP id OAA04558 for ; Sat, 7 Sep 2002 14:53:30 -0300 Received: from slap ([200.207.152.104]) by toole.uol.com.br (8.9.1/8.9.1) with SMTP id OAA02610 for ; Sat, 7 Sep 2002 14:48:53 -0300 (BRT) Message-ID: <002d01c25698$49e5a640$6898cfc8@slap> From: "Afemano" To: References: <5.1.0.14.2.20020907135312.017450d8@pop3.uol.com.br> Subject: Re: [obm-l] Re-probabilidade Date: Sat, 7 Sep 2002 14:59:18 -0300 MIME-Version: 1.0 Content-Type: text/plain; charset="iso-8859-1" Content-Transfer-Encoding: 8bit X-Priority: 3 X-MSMail-Priority: Normal X-Mailer: Microsoft Outlook Express 6.00.2600.0000 X-MimeOLE: Produced By Microsoft MimeOLE V6.00.2600.0000 Sender: owner-obm-l@sucuri.mat.puc-rio.br Precedence: bulk Reply-To: obm-l@mat.puc-rio.br Acho que o começo está errado por que se sair o 4 e o 5 por exemplo ele poderá prender o 1 , o 2 ou o 3... ----- Original Message ----- From: "Pacini" To: Sent: Saturday, September 07, 2002 2:11 PM Subject: [obm-l] Re-probabilidade > > Olá , > Tenho uma dúvida com relação a este problema . Enumerando > as alturas por 1,2,3,4 e5 ; o número de maneiras de sair os dois > primeiros é C5,2 = 10 . Como o inspetor irá prender o mais baixo que > saiu até o momento , o Espaço Amostral será 10 X 3! = 60 ? ; pois > se dentre os dois primeiros que sairem estiver o mais baixo , o > inspetor não irá prender alguém ou , seja , a minha dúvida é a seguinte > : a sequencia 12345 ,12453 por exemplo não deverá entrar duas vezes > para o Espaço Amostral . O que vocês acham ? > Pacini > > > > > > > > To: > obm-l@mat.puc-rio.br Subject: [obm-l] probabilidade > Olá , > Poderiam me ajudar no seguinte problema ? > Um inspetor sabe que o chefe de 5 bandidos é o mais baixo de todos e que > todas as alturas são diferentes . Sabe -se também que eles estarão > presentes numa reunião em um edifício . Depois da reunião , os bandidos por > medida de precaução deixam o edifício em um intervalo de 15 minutos .Como o > inspetor não sabe qual deles é o mais baixo , decide deixar sair os dois > primeiros bandidos , e prender o primeiro dos seguintes que seja mais baixo > do que os que até esse momento sairam .Qual a probabilidade do inspetor > prender a pessoa certa ? > []´s Nick > > > ========================================================================= > Instruções para entrar na lista, sair da lista e usar a lista em > http://www.mat.puc-rio.br/~nicolau/olimp/obm-l.html > O administrador desta lista é > ========================================================================= > ========================================================================= Instruções para entrar na lista, sair da lista e usar a lista em http://www.mat.puc-rio.br/~nicolau/olimp/obm-l.html O administrador desta lista é ========================================================================= From owner-obm-l@sucuri.mat.puc-rio.br Sat Sep 7 15:04:13 2002 Return-Path: Received: (from majordom@localhost) by sucuri.mat.puc-rio.br (8.9.3/8.9.3) id PAA04635 for obm-l-MTTP; Sat, 7 Sep 2002 15:00:26 -0300 Received: from toole.uol.com.br (toole.uol.com.br [200.221.4.26]) by sucuri.mat.puc-rio.br (8.9.3/8.9.3) with ESMTP id PAA04631 for ; Sat, 7 Sep 2002 15:00:23 -0300 Received: from Eder ([200.211.161.217]) by toole.uol.com.br (8.9.1/8.9.1) with SMTP id OAA07841 for ; Sat, 7 Sep 2002 14:55:43 -0300 (BRT) Message-ID: <002d01c25698$37561000$7ac1fea9@Eder> From: "Eder" To: Subject: [obm-l] ??? Date: Sat, 7 Sep 2002 14:58:43 -0300 MIME-Version: 1.0 Content-Type: multipart/alternative; boundary="----=_NextPart_000_002A_01C2567F.0EFD1EA0" X-Priority: 3 X-MSMail-Priority: Normal X-Mailer: Microsoft Outlook Express 5.00.2314.1300 X-MimeOLE: Produced By Microsoft MimeOLE V5.00.2314.1300 Sender: owner-obm-l@sucuri.mat.puc-rio.br Precedence: bulk Reply-To: obm-l@mat.puc-rio.br This is a multi-part message in MIME format. ------=_NextPart_000_002A_01C2567F.0EFD1EA0 Content-Type: text/plain; charset="iso-8859-1" Content-Transfer-Encoding: quoted-printable Gostaria de ajuda nestes problemas: 1)Encontre todas as solu=E7=F5es reais de cosx+(cosx)^5+cos7x=3D3. Bom,eu tranformei cosx+cos7x em produto,depois sa=ED fazendo = simplifica=E7=F5es para ficar somente com cosx.Substituindo cosx po = m,cheguei a um polin=F4mio bem "estranho".Foi f=E1cil checar que m=3D1 = =3D=3D>cosx=3D1=3D=3D>x=3D2kpi =E9 solu=E7=E3o (na verdade =E9 f=E1cil = de ver isso no enunciado),por=E9m n=E3o soube mais o que fazer depois = disso,para ver se h=E1 outras solu=E7=F5es. 2)Considere um polin=F4mio de coeficientes inteiros.Sabe-se que = p(21)=3D17,p(32)=3D-247 e p(37)=3D33.Prove que se para algum N tivermos = p(N)=3D51.Ent=E3o N=3D26. Obrigado pela ajuda. Eder ------=_NextPart_000_002A_01C2567F.0EFD1EA0 Content-Type: text/html; charset="iso-8859-1" Content-Transfer-Encoding: quoted-printable
Gostaria de ajuda nestes = problemas:
 
1)Encontre todas as solu=E7=F5es reais = de=20 cosx+(cosx)^5+cos7x=3D3.
 
Bom,eu tranformei cosx+cos7x em = produto,depois sa=ED=20 fazendo simplifica=E7=F5es para ficar somente com cosx.Substituindo cosx = po=20 m,cheguei a um polin=F4mio bem "estranho".Foi f=E1cil checar que m=3D1=20 =3D=3D>cosx=3D1=3D=3D>x=3D2kpi  =E9 solu=E7=E3o (na verdade = =E9 f=E1cil de ver isso no=20 enunciado),por=E9m n=E3o soube mais o que fazer depois disso,para ver se = h=E1 outras=20 solu=E7=F5es.
 
2)Considere um polin=F4mio de = coeficientes=20 inteiros.Sabe-se que  p(21)=3D17,p(32)=3D-247 e p(37)=3D33.Prove = que se para=20 algum N tivermos p(N)=3D51.Ent=E3o N=3D26.
 
 
Obrigado pela ajuda.
 
Eder
 
 
------=_NextPart_000_002A_01C2567F.0EFD1EA0-- ========================================================================= Instruções para entrar na lista, sair da lista e usar a lista em http://www.mat.puc-rio.br/~nicolau/olimp/obm-l.html O administrador desta lista é ========================================================================= From owner-obm-l@sucuri.mat.puc-rio.br Sat Sep 7 15:10:56 2002 Return-Path: Received: (from majordom@localhost) by sucuri.mat.puc-rio.br (8.9.3/8.9.3) id PAA04791 for obm-l-MTTP; Sat, 7 Sep 2002 15:07:08 -0300 Received: from pina.terra.com.br (pina.terra.com.br [200.176.3.17]) by sucuri.mat.puc-rio.br (8.9.3/8.9.3) with ESMTP id PAA04787 for ; Sat, 7 Sep 2002 15:07:06 -0300 Received: from engenho.terra.com.br (engenho.terra.com.br [200.176.3.42]) by pina.terra.com.br (Postfix) with ESMTP id 4B0AC5309E for ; Sat, 7 Sep 2002 15:07:56 -0300 (EST) Received: from stabel (unknown [200.203.38.221]) (authenticated user dudasta) by engenho.terra.com.br (Postfix) with ESMTP id 9515C68172 for ; Sat, 7 Sep 2002 15:07:54 -0300 (EST) Message-ID: <009301c25699$806a9990$0301a8c0@stabel> From: "Eduardo Casagrande Stabel" To: References: <008401c25684$5c609320$6898cfc8@slap> Subject: [obm-l] =?iso-8859-1?Q?Re:_=5Bobm-l=5D_Teste_dif=EDcil...?= Date: Sat, 7 Sep 2002 15:07:43 -0300 MIME-Version: 1.0 Content-Type: multipart/alternative; boundary="----=_NextPart_000_008E_01C25680.512B9AD0" X-Priority: 3 X-MSMail-Priority: Normal X-Mailer: Microsoft Outlook Express 6.00.2600.0000 X-MimeOLE: Produced By Microsoft MimeOLE V6.00.2600.0000 Sender: owner-obm-l@sucuri.mat.puc-rio.br Precedence: bulk Reply-To: obm-l@mat.puc-rio.br This is a multi-part message in MIME format. ------=_NextPart_000_008E_01C25680.512B9AD0 Content-Type: text/plain; charset="iso-8859-1" Content-Transfer-Encoding: quoted-printable J=E1 veio =E0 lista. Vou recomendar a solu=E7=E3o de = http://www.kalva.demon.co.uk/imo/isoln/isoln754.html do John Scholes. Se voc=EA quiser tentar sozinho, v=E3o a=ED umas dicas: se n =E9 um = n=FAmero natural e s(n) =E9 a soma dos algarismos de n ent=E3o n - s(n) = =E9 divis=EDvel por 9 (demonstre!); tente estimar o B, com desigualdades = A < ..., B < ... e tente juntar os dois fatos para encontrar as somas = dos algarismos de B. Um abra=E7=E3o! Eduardo. Porto Alegre, RS. ----- Original Message -----=20 From: Afemano=20 To: obm-l@mat.puc-rio.br=20 Sent: Saturday, September 07, 2002 12:36 PM Subject: [obm-l] Teste dif=EDcil... Oi, sou novo por aqui e n=E3o sei se este teste j=E1 foi = respondido.... falou ... "A forma decimal do n=FAmero 4444 elevado a 4444 possui um certo = n=FAmero de algarismos. A soma desses algarismos =E1 A. A soma dos = algarismos de A =E9 B. Qual a soma dos algarismos de B ???" Isso est=E1 num caderno de exerc=EDcios de um amigo meu do cursinho. = Se algu=E9m puder ajudar ae valeu !! ------=_NextPart_000_008E_01C25680.512B9AD0 Content-Type: text/html; charset="iso-8859-1" Content-Transfer-Encoding: quoted-printable
J=E1 veio =E0 lista.
 
Vou recomendar a solu=E7=E3o = de http://www.= kalva.demon.co.uk/imo/isoln/isoln754.html do=20 John Scholes.
 
Se voc=EA quiser tentar sozinho, v=E3o = a=ED umas dicas:=20 se n =E9 um n=FAmero natural e s(n) =E9 a soma dos algarismos de n = ent=E3o n - s(n) =E9=20 divis=EDvel por 9 (demonstre!); tente estimar o B, com desigualdades A = < ..., B=20 < ... e tente juntar os dois fatos para encontrar as somas dos = algarismos de=20 B.
 
Um abra=E7=E3o!
 
Eduardo.
Porto Alegre, RS.
 
----- Original Message -----
From:=20 Afemano=20
Sent: Saturday, September 07, = 2002 12:36=20 PM
Subject: [obm-l] Teste = dif=EDcil...

Oi, sou novo por aqui e n=E3o sei se = este teste j=E1=20 foi respondido.... falou ...
 
"A forma decimal do n=FAmero 4444 = elevado a 4444=20 possui um certo n=FAmero de algarismos. A soma desses algarismos =E1 = A. A soma dos=20 algarismos de A =E9 B. Qual a soma dos algarismos de B = ???"
 
Isso est=E1 num caderno de = exerc=EDcios de um amigo=20 meu do cursinho. Se algu=E9m puder ajudar ae valeu=20 !!
------=_NextPart_000_008E_01C25680.512B9AD0-- ========================================================================= Instruções para entrar na lista, sair da lista e usar a lista em http://www.mat.puc-rio.br/~nicolau/olimp/obm-l.html O administrador desta lista é ========================================================================= From owner-obm-l@sucuri.mat.puc-rio.br Sat Sep 7 15:46:08 2002 Return-Path: Received: (from majordom@localhost) by sucuri.mat.puc-rio.br (8.9.3/8.9.3) id PAA06224 for obm-l-MTTP; Sat, 7 Sep 2002 15:44:50 -0300 Received: from seki.bol.com.br (seki.bol.com.br [200.221.24.26]) by sucuri.mat.puc-rio.br (8.9.3/8.9.3) with ESMTP id PAA06220 for ; Sat, 7 Sep 2002 15:44:47 -0300 Received: from lba (200.221.24.192) by seki.bol.com.br (5.1.071) id 3D6143DE005593C1 for obm-l@mat.puc-rio.br; Sat, 7 Sep 2002 15:45:17 -0300 Message-ID: <001101c2569f$3fc418c0$9606c3c8@lba> From: "Leonardo Borges Avelino" To: Subject: [obm-l] =?iso-8859-1?Q?Quest=E3o_de_Geometria?= Date: Fri, 6 Sep 2002 10:56:42 -0300 MIME-Version: 1.0 Content-Type: multipart/alternative; boundary="----=_NextPart_000_000F_01C25594.1565A960" X-Priority: 3 X-MSMail-Priority: Normal X-Mailer: Microsoft Outlook Express 5.00.2615.200 X-MimeOLE: Produced By Microsoft MimeOLE V5.00.2615.200 X-Sender-IP: 200.195.6.150 Sender: owner-obm-l@sucuri.mat.puc-rio.br Precedence: bulk Reply-To: obm-l@mat.puc-rio.br This is a multi-part message in MIME format. ------=_NextPart_000_000F_01C25594.1565A960 Content-Type: text/plain; charset="iso-8859-1" Content-Transfer-Encoding: quoted-printable Ei pessoal Acho que todos conhecem a quest=E3o de geometria que existe uma = circunfer=EAncia e duas tangentes. As tangentes se encontram num ponto = P. Ligamos os pontos de tang=EAncia que determinam os pontos A e B. Do = segmento AB levantamos uma perpendicular que toca a circunfer=EAncia no = ponto C. Dados os valores das dist=E2ncias C at=E9 a reta AP; e de C = at=E9 BP. Calcule o valor da perpendicular. (N=E3o est=E1 muito claro de entender o enunciado pois eu escrevi com = pressa.) Essa quest=E3o jah caiu no Colegio Naval/2001 e me disseram que ela caiu = no IME, e tamb=E9m ela est=E1 num livrinho preto do prof. Eduardo = Wagner. Para todos que conhecem tal quest=E3o e principalmente ao Prof. Eduardo = Wagner, pergunto: * Essa quest=E3o eh n=EDvel IMO? * Quem eh o autor dessa quest=E3o? Valeu!! Leonardo Borges Avelino ------=_NextPart_000_000F_01C25594.1565A960 Content-Type: text/html; charset="iso-8859-1" Content-Transfer-Encoding: quoted-printable
Ei pessoal
Acho que todos conhecem a quest=E3o de = geometria que=20 existe uma circunfer=EAncia e duas tangentes. As tangentes se encontram = num ponto=20 P. Ligamos os pontos de tang=EAncia que determinam os pontos A e B. = Do=20 segmento AB levantamos uma  perpendicular que toca a = circunfer=EAncia no=20 ponto C. Dados os valores das dist=E2ncias C at=E9 a reta AP; e de C = at=E9 BP. Calcule=20 o valor da perpendicular.
(N=E3o est=E1 muito claro de entender o = enunciado pois=20 eu escrevi com pressa.)
 
Essa quest=E3o jah caiu no Colegio = Naval/2001 e me=20 disseram que ela caiu no IME, e tamb=E9m ela est=E1 num livrinho preto = do prof.=20 Eduardo Wagner.
Para todos que conhecem tal quest=E3o e = principalmente ao Prof. Eduardo Wagner, pergunto:
* Essa quest=E3o eh n=EDvel = IMO?
* Quem eh o autor dessa = quest=E3o?
 
Valeu!!
Leonardo Borges = Avelino
------=_NextPart_000_000F_01C25594.1565A960-- ========================================================================= Instruções para entrar na lista, sair da lista e usar a lista em http://www.mat.puc-rio.br/~nicolau/olimp/obm-l.html O administrador desta lista é ========================================================================= From owner-obm-l@sucuri.mat.puc-rio.br Sat Sep 7 15:46:08 2002 Return-Path: Received: (from majordom@localhost) by sucuri.mat.puc-rio.br (8.9.3/8.9.3) id PAA06213 for obm-l-MTTP; Sat, 7 Sep 2002 15:44:45 -0300 Received: from seki.bol.com.br (seki.bol.com.br [200.221.24.26]) by sucuri.mat.puc-rio.br (8.9.3/8.9.3) with ESMTP id PAA06209 for ; Sat, 7 Sep 2002 15:44:42 -0300 Received: from lba (200.221.24.192) by seki.bol.com.br (5.1.071) id 3D6143DE005593AF for obm-l@mat.puc-rio.br; Sat, 7 Sep 2002 15:45:12 -0300 Message-ID: <001001c2569f$3e977fa0$9606c3c8@lba> From: "Leonardo Borges Avelino" To: Subject: [obm-l] =?iso-8859-1?B?RXF1YefjbyBQb2xpbm9taWFs?= Date: Fri, 6 Sep 2002 10:47:31 -0300 MIME-Version: 1.0 Content-Type: multipart/alternative; boundary="----=_NextPart_000_0005_01C25592.CCFFD660" X-Priority: 3 X-MSMail-Priority: Normal X-Mailer: Microsoft Outlook Express 5.00.2615.200 X-MimeOLE: Produced By Microsoft MimeOLE V5.00.2615.200 X-Sender-IP: 200.195.6.150 Sender: owner-obm-l@sucuri.mat.puc-rio.br Precedence: bulk Reply-To: obm-l@mat.puc-rio.br This is a multi-part message in MIME format. ------=_NextPart_000_0005_01C25592.CCFFD660 Content-Type: text/plain; charset="iso-8859-1" Content-Transfer-Encoding: quoted-printable Ei pessoal Eu aprendi a resolver equa=E7=F5es polinomiais atrav=E9s do m=E9todo de = consulta de ra=EDzes. Aquele que vc pega os divisores do termo = independente e divide pelos ... , enfim. Uma d=FAvida: Quando o termo = independente =E9 primo e utilizarmos deste m=E9todo, como saberemos = quais s=E3o as ra=EDzes se nenhuma das poss=EDveis ra=EDzes servir como = raiz. Para ficar clara minha pergunta vai um exemplo. x^3 + 5x^2 - 4x + 7=3D0 Consult=E1r=EDamos + ou - 1 , e + ou - 7. Mas n=E3o conseguir=EDamos = achar raiz alguma. Desculpem-me se falo besteira, mas foi algo que naum descobri ainda. Valeu!! Leonardo Borges Avelino ------=_NextPart_000_0005_01C25592.CCFFD660 Content-Type: text/html; charset="iso-8859-1" Content-Transfer-Encoding: quoted-printable
Ei pessoal
Eu aprendi a resolver equa=E7=F5es = polinomiais atrav=E9s=20 do m=E9todo de consulta de ra=EDzes. Aquele que vc pega os divisores do = termo=20 independente e divide pelos ... , enfim. Uma d=FAvida: Quando o = termo=20 independente =E9 primo e utilizarmos deste m=E9todo, como saberemos = quais s=E3o as=20 ra=EDzes se nenhuma das poss=EDveis ra=EDzes servir como = raiz.
Para ficar clara minha pergunta vai um=20 exemplo.
 
x^3 + 5x^2 - 4x = + 7=3D0
 
Consult=E1r=EDamos + ou - 1   = , e + ou - 7.=20 Mas n=E3o conseguir=EDamos achar raiz alguma.
 
Desculpem-me se falo besteira, mas foi = algo que=20 naum descobri ainda.
 
Valeu!!
Leonardo Borges = Avelino
------=_NextPart_000_0005_01C25592.CCFFD660-- ========================================================================= Instruções para entrar na lista, sair da lista e usar a lista em http://www.mat.puc-rio.br/~nicolau/olimp/obm-l.html O administrador desta lista é ========================================================================= From owner-obm-l@sucuri.mat.puc-rio.br Sat Sep 7 15:57:51 2002 Return-Path: Received: (from majordom@localhost) by sucuri.mat.puc-rio.br (8.9.3/8.9.3) id PAA06715 for obm-l-MTTP; Sat, 7 Sep 2002 15:56:23 -0300 Received: from traven.uol.com.br (traven.uol.com.br [200.221.4.39]) by sucuri.mat.puc-rio.br (8.9.3/8.9.3) with ESMTP id PAA06708 for ; Sat, 7 Sep 2002 15:56:21 -0300 Received: from slap ([200.207.152.104]) by traven.uol.com.br (8.9.1/8.9.1) with SMTP id PAA03171 for ; Sat, 7 Sep 2002 15:44:24 -0300 (BRT) Message-ID: <000d01c256a1$0f1bd1c0$6898cfc8@slap> From: "Afemano" To: References: <008401c25684$5c609320$6898cfc8@slap> <009301c25699$806a9990$0301a8c0@stabel> Subject: [obm-l] =?iso-8859-1?Q?Re:_=5Bobm-l=5D_Re:_=5Bobm-l=5D_Teste_dif=EDcil...?= Date: Sat, 7 Sep 2002 16:01:50 -0300 MIME-Version: 1.0 Content-Type: multipart/alternative; boundary="----=_NextPart_000_000A_01C25687.E0B49560" X-Priority: 3 X-MSMail-Priority: Normal X-Mailer: Microsoft Outlook Express 6.00.2600.0000 X-MimeOLE: Produced By Microsoft MimeOLE V6.00.2600.0000 Sender: owner-obm-l@sucuri.mat.puc-rio.br Precedence: bulk Reply-To: obm-l@mat.puc-rio.br This is a multi-part message in MIME format. ------=_NextPart_000_000A_01C25687.E0B49560 Content-Type: text/plain; charset="iso-8859-1" Content-Transfer-Encoding: quoted-printable Nossa te amo ahuahuaah brigad=E3o !!! Fui ----- Original Message -----=20 From: Eduardo Casagrande Stabel=20 To: obm-l@mat.puc-rio.br=20 Sent: Saturday, September 07, 2002 3:07 PM Subject: [obm-l] Re: [obm-l] Teste dif=EDcil... J=E1 veio =E0 lista. Vou recomendar a solu=E7=E3o de = http://www.kalva.demon.co.uk/imo/isoln/isoln754.html do John Scholes. Se voc=EA quiser tentar sozinho, v=E3o a=ED umas dicas: se n =E9 um = n=FAmero natural e s(n) =E9 a soma dos algarismos de n ent=E3o n - s(n) = =E9 divis=EDvel por 9 (demonstre!); tente estimar o B, com desigualdades = A < ..., B < ... e tente juntar os dois fatos para encontrar as somas = dos algarismos de B. Um abra=E7=E3o! Eduardo. Porto Alegre, RS. ----- Original Message -----=20 From: Afemano=20 To: obm-l@mat.puc-rio.br=20 Sent: Saturday, September 07, 2002 12:36 PM Subject: [obm-l] Teste dif=EDcil... Oi, sou novo por aqui e n=E3o sei se este teste j=E1 foi = respondido.... falou ... "A forma decimal do n=FAmero 4444 elevado a 4444 possui um certo = n=FAmero de algarismos. A soma desses algarismos =E1 A. A soma dos = algarismos de A =E9 B. Qual a soma dos algarismos de B ???" Isso est=E1 num caderno de exerc=EDcios de um amigo meu do cursinho. = Se algu=E9m puder ajudar ae valeu !! ------=_NextPart_000_000A_01C25687.E0B49560 Content-Type: text/html; charset="iso-8859-1" Content-Transfer-Encoding: quoted-printable
Nossa te amo ahuahuaah brigad=E3o = !!!
Fui
----- Original Message -----
From:=20 Eduardo=20 Casagrande Stabel
Sent: Saturday, September 07, = 2002 3:07=20 PM
Subject: [obm-l] Re: [obm-l] = Teste=20 dif=EDcil...

J=E1 veio =E0 lista.
 
Vou recomendar a = solu=E7=E3o de http://www.= kalva.demon.co.uk/imo/isoln/isoln754.html do=20 John Scholes.
 
Se voc=EA quiser tentar sozinho, = v=E3o a=ED umas dicas:=20 se n =E9 um n=FAmero natural e s(n) =E9 a soma dos algarismos de n = ent=E3o n - s(n) =E9=20 divis=EDvel por 9 (demonstre!); tente estimar o B, com desigualdades A = < ...,=20 B < ... e tente juntar os dois fatos para encontrar as somas dos = algarismos=20 de B.
 
Um abra=E7=E3o!
 
Eduardo.
Porto Alegre, RS.
 
----- Original Message -----
From:=20 Afemano=20
Sent: Saturday, September 07, = 2002=20 12:36 PM
Subject: [obm-l] Teste = dif=EDcil...

Oi, sou novo por aqui e n=E3o sei = se este teste=20 j=E1 foi respondido.... falou ...
 
"A forma decimal do n=FAmero 4444 = elevado a 4444=20 possui um certo n=FAmero de algarismos. A soma desses algarismos =E1 = A. A soma=20 dos algarismos de A =E9 B. Qual a soma dos algarismos de B = ???"
 
Isso est=E1 num caderno de = exerc=EDcios de um amigo=20 meu do cursinho. Se algu=E9m puder ajudar ae valeu=20 !!
------=_NextPart_000_000A_01C25687.E0B49560-- ========================================================================= Instruções para entrar na lista, sair da lista e usar a lista em http://www.mat.puc-rio.br/~nicolau/olimp/obm-l.html O administrador desta lista é ========================================================================= From owner-obm-l@sucuri.mat.puc-rio.br Sat Sep 7 16:11:48 2002 Return-Path: Received: (from majordom@localhost) by sucuri.mat.puc-rio.br (8.9.3/8.9.3) id QAA07385 for obm-l-MTTP; Sat, 7 Sep 2002 16:10:22 -0300 Received: from silva5.uol.com.br (silva5.uol.com.br [200.221.4.52]) by sucuri.mat.puc-rio.br (8.9.3/8.9.3) with ESMTP id QAA07380 for ; Sat, 7 Sep 2002 16:10:19 -0300 Received: from ui.uol.com.br ([200.191.180.44]) by silva5.uol.com.br (8.9.1/8.9.1) with ESMTP id QAA00043; Sat, 7 Sep 2002 16:13:30 -0300 (EST) Message-Id: <5.1.0.14.2.20020907160914.00b14518@pop3.uol.com.br> X-Sender: cavictor@pop3.uol.com.br X-Mailer: QUALCOMM Windows Eudora Version 5.1 Date: Sat, 07 Sep 2002 16:10:47 -0300 To: obm-l@mat.puc-rio.br, From: Carlos Victor Subject: Re: [obm-l] Re-probabilidade In-Reply-To: <002d01c25698$49e5a640$6898cfc8@slap> References: <5.1.0.14.2.20020907135312.017450d8@pop3.uol.com.br> Mime-Version: 1.0 Content-Type: text/plain; charset="iso-8859-1"; format=flowed Content-Transfer-Encoding: 8bit X-MIME-Autoconverted: from quoted-printable to 8bit by sucuri.mat.puc-rio.br id QAA07381 Sender: owner-obm-l@sucuri.mat.puc-rio.br Precedence: bulk Reply-To: obm-l@mat.puc-rio.br Poderíamos ouvir a opinião do nosso mestre Morgado ,não é verdade ? Pacini At 14:59 7/9/2002 -0300, Afemano wrote: >Acho que o começo está errado por que se sair o 4 e o 5 por exemplo ele >poderá prender o 1 , o 2 ou o 3... > >----- Original Message ----- >From: "Pacini" >To: >Sent: Saturday, September 07, 2002 2:11 PM >Subject: [obm-l] Re-probabilidade > > > > > > Olá , > > Tenho uma dúvida com relação a este problema . Enumerando > > as alturas por 1,2,3,4 e5 ; o número de maneiras de sair os dois > > primeiros é C5,2 = 10 . Como o inspetor irá prender o mais baixo que > > saiu até o momento , o Espaço Amostral será 10 X 3! = 60 ? ; pois > > se dentre os dois primeiros que sairem estiver o mais baixo , o > > inspetor não irá prender alguém ou , seja , a minha dúvida é a seguinte > > : a sequencia 12345 ,12453 por exemplo não deverá entrar duas vezes > > para o Espaço Amostral . O que vocês acham ? > > Pacini > > > > > > > > > > > > > > > > To: > > obm-l@mat.puc-rio.br Subject: [obm-l] probabilidade > > Olá , > > Poderiam me ajudar no seguinte problema ? > > Um inspetor sabe que o chefe de 5 bandidos é o mais baixo de todos e que > > todas as alturas são diferentes . Sabe -se também que eles estarão > > presentes numa reunião em um edifício . Depois da reunião , os bandidos >por > > medida de precaução deixam o edifício em um intervalo de 15 minutos .Como >o > > inspetor não sabe qual deles é o mais baixo , decide deixar sair os dois > > primeiros bandidos , e prender o primeiro dos seguintes que seja mais >baixo > > do que os que até esse momento sairam .Qual a probabilidade do inspetor > > prender a pessoa certa ? > > []´s Nick > > > > > > ========================================================================= > > Instruções para entrar na lista, sair da lista e usar a lista em > > http://www.mat.puc-rio.br/~nicolau/olimp/obm-l.html > > O administrador desta lista é > > ========================================================================= > > > >========================================================================= >Instruções para entrar na lista, sair da lista e usar a lista em >http://www.mat.puc-rio.br/~nicolau/olimp/obm-l.html >O administrador desta lista é >========================================================================= ========================================================================= Instruções para entrar na lista, sair da lista e usar a lista em http://www.mat.puc-rio.br/~nicolau/olimp/obm-l.html O administrador desta lista é ========================================================================= From owner-obm-l@sucuri.mat.puc-rio.br Sat Sep 7 16:11:48 2002 Return-Path: Received: (from majordom@localhost) by sucuri.mat.puc-rio.br (8.9.3/8.9.3) id QAA07385 for obm-l-MTTP; Sat, 7 Sep 2002 16:10:22 -0300 Received: from silva5.uol.com.br (silva5.uol.com.br [200.221.4.52]) by sucuri.mat.puc-rio.br (8.9.3/8.9.3) with ESMTP id QAA07380 for ; Sat, 7 Sep 2002 16:10:19 -0300 Received: from ui.uol.com.br ([200.191.180.44]) by silva5.uol.com.br (8.9.1/8.9.1) with ESMTP id QAA00043; Sat, 7 Sep 2002 16:13:30 -0300 (EST) Message-Id: <5.1.0.14.2.20020907160914.00b14518@pop3.uol.com.br> X-Sender: cavictor@pop3.uol.com.br X-Mailer: QUALCOMM Windows Eudora Version 5.1 Date: Sat, 07 Sep 2002 16:10:47 -0300 To: obm-l@mat.puc-rio.br, From: Carlos Victor Subject: Re: [obm-l] Re-probabilidade In-Reply-To: <002d01c25698$49e5a640$6898cfc8@slap> References: <5.1.0.14.2.20020907135312.017450d8@pop3.uol.com.br> Mime-Version: 1.0 Content-Type: text/plain; charset="iso-8859-1"; format=flowed Content-Transfer-Encoding: 8bit X-MIME-Autoconverted: from quoted-printable to 8bit by sucuri.mat.puc-rio.br id QAA07381 Sender: owner-obm-l@sucuri.mat.puc-rio.br Precedence: bulk Reply-To: obm-l@mat.puc-rio.br Poderíamos ouvir a opinião do nosso mestre Morgado ,não é verdade ? Pacini At 14:59 7/9/2002 -0300, Afemano wrote: >Acho que o começo está errado por que se sair o 4 e o 5 por exemplo ele >poderá prender o 1 , o 2 ou o 3... > >----- Original Message ----- >From: "Pacini" >To: >Sent: Saturday, September 07, 2002 2:11 PM >Subject: [obm-l] Re-probabilidade > > > > > > Olá , > > Tenho uma dúvida com relação a este problema . Enumerando > > as alturas por 1,2,3,4 e5 ; o número de maneiras de sair os dois > > primeiros é C5,2 = 10 . Como o inspetor irá prender o mais baixo que > > saiu até o momento , o Espaço Amostral será 10 X 3! = 60 ? ; pois > > se dentre os dois primeiros que sairem estiver o mais baixo , o > > inspetor não irá prender alguém ou , seja , a minha dúvida é a seguinte > > : a sequencia 12345 ,12453 por exemplo não deverá entrar duas vezes > > para o Espaço Amostral . O que vocês acham ? > > Pacini > > > > > > > > > > > > > > > > To: > > obm-l@mat.puc-rio.br Subject: [obm-l] probabilidade > > Olá , > > Poderiam me ajudar no seguinte problema ? > > Um inspetor sabe que o chefe de 5 bandidos é o mais baixo de todos e que > > todas as alturas são diferentes . Sabe -se também que eles estarão > > presentes numa reunião em um edifício . Depois da reunião , os bandidos >por > > medida de precaução deixam o edifício em um intervalo de 15 minutos .Como >o > > inspetor não sabe qual deles é o mais baixo , decide deixar sair os dois > > primeiros bandidos , e prender o primeiro dos seguintes que seja mais >baixo > > do que os que até esse momento sairam .Qual a probabilidade do inspetor > > prender a pessoa certa ? > > []´s Nick > > > > > > ========================================================================= > > Instruções para entrar na lista, sair da lista e usar a lista em > > http://www.mat.puc-rio.br/~nicolau/olimp/obm-l.html > > O administrador desta lista é > > ========================================================================= > > > >========================================================================= >Instruções para entrar na lista, sair da lista e usar a lista em >http://www.mat.puc-rio.br/~nicolau/olimp/obm-l.html >O administrador desta lista é >========================================================================= ========================================================================= Instruções para entrar na lista, sair da lista e usar a lista em http://www.mat.puc-rio.br/~nicolau/olimp/obm-l.html O administrador desta lista é ========================================================================= From owner-obm-l@sucuri.mat.puc-rio.br Sat Sep 7 17:45:05 2002 Return-Path: Received: (from majordom@localhost) by sucuri.mat.puc-rio.br (8.9.3/8.9.3) id RAA09191 for obm-l-MTTP; Sat, 7 Sep 2002 17:44:03 -0300 Received: from hotmail.com (f82.pav2.hotmail.com [64.4.37.82]) by sucuri.mat.puc-rio.br (8.9.3/8.9.3) with ESMTP id RAA09187 for ; Sat, 7 Sep 2002 17:44:00 -0300 Received: from mail pickup service by hotmail.com with Microsoft SMTPSVC; Sat, 7 Sep 2002 13:44:50 -0700 Received: from 200.199.193.135 by pv2fd.pav2.hotmail.msn.com with HTTP; Sat, 07 Sep 2002 20:44:49 GMT X-Originating-IP: [200.199.193.135] From: "ricardo matos" To: obm-l@mat.puc-rio.br Subject: Re: [obm-l] Circulo de 9 pontos e reta de Simson Date: Sat, 07 Sep 2002 17:44:49 -0300 Mime-Version: 1.0 Content-Type: text/plain; charset=iso-8859-1; format=flowed Message-ID: X-OriginalArrivalTime: 07 Sep 2002 20:44:50.0326 (UTC) FILETIME=[69278B60:01C256AF] Sender: owner-obm-l@sucuri.mat.puc-rio.br Precedence: bulk Reply-To: obm-l@mat.puc-rio.br >From: "leonardo mattos" >Reply-To: obm-l@mat.puc-rio.br >To: obm-l@mat.puc-rio.br >Subject: [obm-l] Circulo de 9 pontos e reta de Simson >Date: Sat, 07 Sep 2002 01:51:58 +0000 > >Ola pessoal, > >Gostaria muito de saber quais seriam as propriedades do circulo de 9 pontos >e da reta de Simson. > Um abraço,Leonardo > > > > > >_________________________________________________________________ >MSN Photos é a maneira mais fácil e prática de editar e compartilhar sua >fotos: http://photos.msn.com.br > >========================================================================= >Instruções para entrar na lista, sair da lista e usar a lista em >http://www.mat.puc-rio.br/~nicolau/olimp/obm-l.html >O administrador desta lista é >========================================================================= Da reta de Simson só sei a definição(dado um ponto no circuncírculo tome os três pés com relação a cada um dos lados ou extensões estes três pontos são sempre colineares e a reta que os contém é chamada reta de Simson). Conheço melhor as propriedades do círculo dos nove pontos( na verdade estava estudando isto justamente agora), bem para começar o círculo dos nove pontos é o círculo que contém os três pontos médios dos lados e acaba que esse círculo também contém os três pés das alturas e os pontos médios do ortocentro com os vértices. No triâgulo ABC, sejam H é o Ortocentro, O o centro do circuncírculo; Ma, Mb e Mc os pontos médios dos lados a,b,c(opostos aos vértices A,B e C respectivamente);Pa, Pb e Pc os pés das alturas; Ha, Hb e Hc os pontos médios de HA, HB e HC. 1) Seu raio é metade do raio do circuncírculo. 2) Seu centro está no ponto médio do circuncentro com o ortocentro (a reta que contémestes pontos se chama linha de Euler e contém também o Baricentro). 3) Se H é o Ortocentro. Os triângulos ABC, AHB, BHC e CHA tem o mesmo círculo dos nove pontos (na verdade os mesmos nove pontos). 4) (Teorema de Feuerbach's) Ele é tangente internamente ao incírculo e externamente aos três ex-circuncírculos. a) Com ele é possível demonstrar que HA=2OMa. b) O centro do circuncírculo de AHB é o reflexo de O pelo lado AB.(por 3) e 1) eles tem o mesmo raio). c) O reflexo do H por qualquer lado pertence ao circuncírculo. Deve ter dado pra ver que este círculo é bem mágico. Quanto a demonstração desses fatos a menos do 4) não são muito difíceis(se você já viu antes). Tem uma demonstração do teorema dos nove pontos que eu vi no livro do Coxeter "Introduction to Geometry" que demonstra dum jeito bem simples(e mágico) mostrando primeiro que Ma, Mb, Mc, Ha, Hb e Hc estão num mesmo círculo e que este cículo contém os pés das alturas é imediato(na demonstração), não sei se já conhece mas vou colocar mesmo assim pra quem quiser conhecer. (// significa paralelo e A-B significa o segmento AB) Demonstração: Primeiro Ma-Mb // A-B (pontos médios) da mesma forma Ha-Hb // A-B. Com a mesma idéia teremos Ma-Ha // H-C e Hb-Mb // H-C como H-C é perpendicular à A-B temos que HaHbMaMb é um retângulo logo estes pontos estão num mesmo círculo de diâmetros Ma-Ha e Mb-Hb(diagonais do retângulo) e claro que seu centro está no ponto médio de Ma-Ha e Mb-Hb. Se repetirmos esta idéia para os outros lados teremos outros três retângulos com diagonais Ma-Ha, Mb-Hb e Mc-Hc que se cruzam em seus pontos médios logo este ponto é o centro do círculo que passa por Ma, Mb, Mc, Ha, Hb e Hc. Agora o golpe final o diâmetro Hc-Mc é a hipotenusa do triângulo retângulo PcMcHc logo Pc também pertence ao círculo, e analogamente para os outros pés. Para um teorema desses não podia ter demonstração mais simples. Como isto se relaciona com os outros fatos principalmente 1), 2), a), b) e c). Não fica claro, apesar de a partir daí ser demonstrável. No entanto conheço outra demonstração que demonstra tudo isso junto e que me parece menos mágico e mais natural. Se quiserem ela me peçam que eu posso mandar para a lista. O fato 4) tem uma demonstração por inversão(transformação geométrica legal) no livro do Coxeter "Geometry Revisited" e neste livro também tem uma demostração fácil das linhas de Simson . Se você sabe um pouco de inglês recomendo os sites: http://www.cut-the-knot.com/ Este site é o máximo tem muita geometria junto com as demonstracões e está cheio de aplicativos interativos. http://mathworld.wolfram.com/ Este é um site enciclopédico(provavelmete o melhor lugar pra procurar propriedades e teoremas em geral mas em geral não tem demontrações mas ele indica referências). _________________________________________________________________ MSN Photos é a maneira mais fácil e prática de editar e compartilhar sua fotos: http://photos.msn.com.br ========================================================================= Instruções para entrar na lista, sair da lista e usar a lista em http://www.mat.puc-rio.br/~nicolau/olimp/obm-l.html O administrador desta lista é ========================================================================= From owner-obm-l@sucuri.mat.puc-rio.br Sat Sep 7 17:47:23 2002 Return-Path: Received: (from majordom@localhost) by sucuri.mat.puc-rio.br (8.9.3/8.9.3) id RAA09269 for obm-l-MTTP; Sat, 7 Sep 2002 17:47:21 -0300 Received: from traven.uol.com.br (traven.uol.com.br [200.221.4.39]) by sucuri.mat.puc-rio.br (8.9.3/8.9.3) with ESMTP id RAA09265 for ; Sat, 7 Sep 2002 17:47:19 -0300 Received: from ui.uol.com.br ([200.160.246.88]) by traven.uol.com.br (8.9.1/8.9.1) with ESMTP id RAA12832; Sat, 7 Sep 2002 17:35:28 -0300 (BRT) Message-Id: <5.1.0.14.2.20020907172144.0258a8a8@pop3.uol.com.br> X-Sender: fnicks@uol.com.br@pop3.uol.com.br (Unverified) X-Mailer: QUALCOMM Windows Eudora Version 5.1 Date: Sat, 07 Sep 2002 17:46:44 -0300 To: obm-l@mat.puc-rio.br, From: Nick Subject: Re: [obm-l] Re-probabilidade In-Reply-To: <002d01c25698$49e5a640$6898cfc8@slap> References: <5.1.0.14.2.20020907135312.017450d8@pop3.uol.com.br> Mime-Version: 1.0 Content-Type: text/plain; charset="iso-8859-1"; format=flowed Content-Transfer-Encoding: 8bit X-MIME-Autoconverted: from quoted-printable to 8bit by sucuri.mat.puc-rio.br id RAA09266 Sender: owner-obm-l@sucuri.mat.puc-rio.br Precedence: bulk Reply-To: obm-l@mat.puc-rio.br Inicialmente fiz o seguinte :1) na situação em que saem os dois primeiros (1,2)ou(1,3)ou(1,4)ou(1,5) teremos quatro possibilidades em que o inspetor não prenderá alguém , independente da ordem que saem os outros . 2)na situação em que os dois primeiros a sairem são (2,3)ou (2,4)ou (2,5) teremos tres possibilidades em que o inspetor prenderá o mais baixo , independente de qual ordem saem os outros do edifício.3)Nas situações em que os dois primeiros a sairem são : (3,4)ou (3,5) teremos um total de 8 ( 4 para cada) possibilidades em o inspetor prenderá alguém , 1 ou 2. 4)na situação (4,5) teremos tres casos em o inspetor prenderá alguém . Como há entre todos esses casos 8 possibilidades em em ele prenderá o mais baixo ,teremos como probabildade 8/18 . Ou será que na situação (1) deverei contar 24 casos ? Estopu errado ? []´s Nick At 14:59 7/9/2002 -0300, Afemano wrote: >Acho que o começo está errado por que se sair o 4 e o 5 por exemplo ele >poderá prender o 1 , o 2 ou o 3... > >----- Original Message ----- >From: "Pacini" >To: >Sent: Saturday, September 07, 2002 2:11 PM >Subject: [obm-l] Re-probabilidade > > > > > > Olá , > > Tenho uma dúvida com relação a este problema . Enumerando > > as alturas por 1,2,3,4 e5 ; o número de maneiras de sair os dois > > primeiros é C5,2 = 10 . Como o inspetor irá prender o mais baixo que > > saiu até o momento , o Espaço Amostral será 10 X 3! = 60 ? ; pois > > se dentre os dois primeiros que sairem estiver o mais baixo , o > > inspetor não irá prender alguém ou , seja , a minha dúvida é a seguinte > > : a sequencia 12345 ,12453 por exemplo não deverá entrar duas vezes > > para o Espaço Amostral . O que vocês acham ? > > Pacini > > > > > > > > > > > > > > > > To: > > obm-l@mat.puc-rio.br Subject: [obm-l] probabilidade > > Olá , > > Poderiam me ajudar no seguinte problema ? > > Um inspetor sabe que o chefe de 5 bandidos é o mais baixo de todos e que > > todas as alturas são diferentes . Sabe -se também que eles estarão > > presentes numa reunião em um edifício . Depois da reunião , os bandidos >por > > medida de precaução deixam o edifício em um intervalo de 15 minutos .Como >o > > inspetor não sabe qual deles é o mais baixo , decide deixar sair os dois > > primeiros bandidos , e prender o primeiro dos seguintes que seja mais >baixo > > do que os que até esse momento sairam .Qual a probabilidade do inspetor > > prender a pessoa certa ? > > []´s Nick > > > > > > ========================================================================= > > Instruções para entrar na lista, sair da lista e usar a lista em > > http://www.mat.puc-rio.br/~nicolau/olimp/obm-l.html > > O administrador desta lista é > > ========================================================================= > > > >========================================================================= >Instruções para entrar na lista, sair da lista e usar a lista em >http://www.mat.puc-rio.br/~nicolau/olimp/obm-l.html >O administrador desta lista é >========================================================================= ========================================================================= Instruções para entrar na lista, sair da lista e usar a lista em http://www.mat.puc-rio.br/~nicolau/olimp/obm-l.html O administrador desta lista é ========================================================================= From owner-obm-l@sucuri.mat.puc-rio.br Sat Sep 7 17:47:23 2002 Return-Path: Received: (from majordom@localhost) by sucuri.mat.puc-rio.br (8.9.3/8.9.3) id RAA09269 for obm-l-MTTP; Sat, 7 Sep 2002 17:47:21 -0300 Received: from traven.uol.com.br (traven.uol.com.br [200.221.4.39]) by sucuri.mat.puc-rio.br (8.9.3/8.9.3) with ESMTP id RAA09265 for ; Sat, 7 Sep 2002 17:47:19 -0300 Received: from ui.uol.com.br ([200.160.246.88]) by traven.uol.com.br (8.9.1/8.9.1) with ESMTP id RAA12832; Sat, 7 Sep 2002 17:35:28 -0300 (BRT) Message-Id: <5.1.0.14.2.20020907172144.0258a8a8@pop3.uol.com.br> X-Sender: fnicks@uol.com.br@pop3.uol.com.br (Unverified) X-Mailer: QUALCOMM Windows Eudora Version 5.1 Date: Sat, 07 Sep 2002 17:46:44 -0300 To: obm-l@mat.puc-rio.br, From: Nick Subject: Re: [obm-l] Re-probabilidade In-Reply-To: <002d01c25698$49e5a640$6898cfc8@slap> References: <5.1.0.14.2.20020907135312.017450d8@pop3.uol.com.br> Mime-Version: 1.0 Content-Type: text/plain; charset="iso-8859-1"; format=flowed Content-Transfer-Encoding: 8bit X-MIME-Autoconverted: from quoted-printable to 8bit by sucuri.mat.puc-rio.br id RAA09266 Sender: owner-obm-l@sucuri.mat.puc-rio.br Precedence: bulk Reply-To: obm-l@mat.puc-rio.br Inicialmente fiz o seguinte :1) na situação em que saem os dois primeiros (1,2)ou(1,3)ou(1,4)ou(1,5) teremos quatro possibilidades em que o inspetor não prenderá alguém , independente da ordem que saem os outros . 2)na situação em que os dois primeiros a sairem são (2,3)ou (2,4)ou (2,5) teremos tres possibilidades em que o inspetor prenderá o mais baixo , independente de qual ordem saem os outros do edifício.3)Nas situações em que os dois primeiros a sairem são : (3,4)ou (3,5) teremos um total de 8 ( 4 para cada) possibilidades em o inspetor prenderá alguém , 1 ou 2. 4)na situação (4,5) teremos tres casos em o inspetor prenderá alguém . Como há entre todos esses casos 8 possibilidades em em ele prenderá o mais baixo ,teremos como probabildade 8/18 . Ou será que na situação (1) deverei contar 24 casos ? Estopu errado ? []´s Nick At 14:59 7/9/2002 -0300, Afemano wrote: >Acho que o começo está errado por que se sair o 4 e o 5 por exemplo ele >poderá prender o 1 , o 2 ou o 3... > >----- Original Message ----- >From: "Pacini" >To: >Sent: Saturday, September 07, 2002 2:11 PM >Subject: [obm-l] Re-probabilidade > > > > > > Olá , > > Tenho uma dúvida com relação a este problema . Enumerando > > as alturas por 1,2,3,4 e5 ; o número de maneiras de sair os dois > > primeiros é C5,2 = 10 . Como o inspetor irá prender o mais baixo que > > saiu até o momento , o Espaço Amostral será 10 X 3! = 60 ? ; pois > > se dentre os dois primeiros que sairem estiver o mais baixo , o > > inspetor não irá prender alguém ou , seja , a minha dúvida é a seguinte > > : a sequencia 12345 ,12453 por exemplo não deverá entrar duas vezes > > para o Espaço Amostral . O que vocês acham ? > > Pacini > > > > > > > > > > > > > > > > To: > > obm-l@mat.puc-rio.br Subject: [obm-l] probabilidade > > Olá , > > Poderiam me ajudar no seguinte problema ? > > Um inspetor sabe que o chefe de 5 bandidos é o mais baixo de todos e que > > todas as alturas são diferentes . Sabe -se também que eles estarão > > presentes numa reunião em um edifício . Depois da reunião , os bandidos >por > > medida de precaução deixam o edifício em um intervalo de 15 minutos .Como >o > > inspetor não sabe qual deles é o mais baixo , decide deixar sair os dois > > primeiros bandidos , e prender o primeiro dos seguintes que seja mais >baixo > > do que os que até esse momento sairam .Qual a probabilidade do inspetor > > prender a pessoa certa ? > > []´s Nick > > > > > > ========================================================================= > > Instruções para entrar na lista, sair da lista e usar a lista em > > http://www.mat.puc-rio.br/~nicolau/olimp/obm-l.html > > O administrador desta lista é > > ========================================================================= > > > >========================================================================= >Instruções para entrar na lista, sair da lista e usar a lista em >http://www.mat.puc-rio.br/~nicolau/olimp/obm-l.html >O administrador desta lista é >========================================================================= ========================================================================= Instruções para entrar na lista, sair da lista e usar a lista em http://www.mat.puc-rio.br/~nicolau/olimp/obm-l.html O administrador desta lista é ========================================================================= From owner-obm-l@sucuri.mat.puc-rio.br Sat Sep 7 18:32:04 2002 Return-Path: Received: (from majordom@localhost) by sucuri.mat.puc-rio.br (8.9.3/8.9.3) id SAA10688 for obm-l-MTTP; Sat, 7 Sep 2002 18:31:55 -0300 Received: from gorgo.centroin.com.br (gorgo.centroin.com.br [200.225.63.128]) by sucuri.mat.puc-rio.br (8.9.3/8.9.3) with ESMTP id SAA10684 for ; Sat, 7 Sep 2002 18:31:53 -0300 Received: from centroin.com.br (du134c.rjo.centroin.com.br [200.225.58.134]) (authenticated bits=0) by gorgo.centroin.com.br (8.12.2/8.12.1) with ESMTP id g87LX7nH014874 for ; Sat, 7 Sep 2002 18:33:08 -0300 (BRT) Message-ID: <3D7A712D.5010507@centroin.com.br> Date: Sat, 07 Sep 2002 18:35:41 -0300 From: Augusto =?ISO-8859-1?Q?C=E9sar?= Morgado User-Agent: Mozilla/5.0 (Windows; U; Win98; en-US; rv:0.9.4.1) Gecko/20020508 Netscape6/6.2.3 X-Accept-Language: en-us MIME-Version: 1.0 To: obm-l@mat.puc-rio.br Subject: Re: [obm-l] ??? References: <002d01c25698$37561000$7ac1fea9@Eder> Content-Type: multipart/alternative; boundary="------------060804040703010901010802" Sender: owner-obm-l@sucuri.mat.puc-rio.br Precedence: bulk Reply-To: obm-l@mat.puc-rio.br --------------060804040703010901010802 Content-Type: text/plain; charset=ISO-8859-1; format=flowed Content-Transfer-Encoding: 8bit 1) como os co-senos têm o saudável hábito de serem menores que ou iguais a 1, o unico jeito dessa soma dar 3 eh cosx=1 e cos 7x = 1. Logo, as unicas soluçoes sao as da forma 2kpi. Eder wrote: > Gostaria de ajuda nestes problemas: > > > > 1)Encontre todas as soluções reais de cosx+(cosx)^5+cos7x=3. > > > > Bom,eu tranformei cosx+cos7x em produto,depois saí fazendo > simplificações para ficar somente com cosx.Substituindo cosx po > m,cheguei a um polinômio bem "estranho".Foi fácil checar que m=1 > ==>cosx=1==>x=2kpi é solução (na verdade é fácil de ver isso no > enunciado),porém não soube mais o que fazer depois disso,para ver se > há outras soluções. > > > > 2)Considere um polinômio de coeficientes inteiros.Sabe-se que > p(21)=17,p(32)=-247 e p(37)=33.Prove que se para algum N tivermos > p(N)=51.Então N=26. > > > > > > Obrigado pela ajuda. > > > > Eder > > > > > --------------060804040703010901010802 Content-Type: text/html; charset=us-ascii Content-Transfer-Encoding: 7bit 1) como os co-senos têm o saudável hábito de serem menores que ou iguais a 1, o unico jeito dessa soma dar 3  eh  cosx=1 e cos 7x = 1.
Logo, as unicas soluçoes sao as da forma 2kpi.

Eder wrote:
Gostaria de ajuda nestes problemas:
 
1)Encontre todas as soluções reais de cosx+(cosx)^5+cos7x=3.
 
Bom,eu tranformei cosx+cos7x em produto,depois saí fazendo simplificações para ficar somente com cosx.Substituindo cosx po m,cheguei a um polinômio bem "estranho".Foi fácil checar que m=1 ==>cosx=1==>x=2kpi  é solução (na verdade é fácil de ver isso no enunciado),porém não soube mais o que fazer depois disso,para ver se há outras soluções.
 
2)Considere um polinômio de coeficientes inteiros.Sabe-se que  p(21)=17,p(32)=-247 e p(37)=33.Prove que se para algum N tivermos p(N)=51.Então N=26.
 
 
Obrigado pela ajuda.
 
Eder
 
 

--------------060804040703010901010802-- ========================================================================= Instruções para entrar na lista, sair da lista e usar a lista em http://www.mat.puc-rio.br/~nicolau/olimp/obm-l.html O administrador desta lista é ========================================================================= From owner-obm-l@sucuri.mat.puc-rio.br Sat Sep 7 18:38:48 2002 Return-Path: Received: (from majordom@localhost) by sucuri.mat.puc-rio.br (8.9.3/8.9.3) id SAA10796 for obm-l-MTTP; Sat, 7 Sep 2002 18:38:46 -0300 Received: from siegel.bol.com.br (siegel.bol.com.br [200.221.24.20]) by sucuri.mat.puc-rio.br (8.9.3/8.9.3) with ESMTP id SAA10792 for ; Sat, 7 Sep 2002 18:38:44 -0300 Received: from lba (200.221.24.191) by siegel.bol.com.br (5.1.071) id 3D5D1C9F007980DF for obm-l@mat.puc-rio.br; Sat, 7 Sep 2002 18:39:22 -0300 Message-ID: <007401c256b7$8f17c9e0$a306c3c8@lba> From: "Leonardo Borges Avelino" To: Subject: [obm-l] Reta de Simson Date: Sat, 7 Sep 2002 18:43:03 -0300 MIME-Version: 1.0 Content-Type: multipart/alternative; boundary="----=_NextPart_000_0071_01C2569E.66389180" X-Priority: 3 X-MSMail-Priority: Normal X-Mailer: Microsoft Outlook Express 5.00.2615.200 X-MimeOLE: Produced By Microsoft MimeOLE V5.00.2615.200 X-Sender-IP: 200.195.6.163 Sender: owner-obm-l@sucuri.mat.puc-rio.br Precedence: bulk Reply-To: obm-l@mat.puc-rio.br This is a multi-part message in MIME format. ------=_NextPart_000_0071_01C2569E.66389180 Content-Type: text/plain; charset="iso-8859-1" Content-Transfer-Encoding: quoted-printable Caro Leonardo recomendo baixar este arquivo que =E9 um material usado na V Semana = Ol=EDmpica www.obm.org.br/semana/retasimson.doc Valeu!! Leonardo Borges Avelino ------=_NextPart_000_0071_01C2569E.66389180 Content-Type: text/html; charset="iso-8859-1" Content-Transfer-Encoding: quoted-printable
Caro Leonardo
recomendo baixar este arquivo que =E9 = um material=20 usado na V Semana Ol=EDmpica
 
www.obm.org.br/seman= a/retasimson.doc
 
Valeu!!
Leonardo Borges = Avelino
------=_NextPart_000_0071_01C2569E.66389180-- ========================================================================= Instruções para entrar na lista, sair da lista e usar a lista em http://www.mat.puc-rio.br/~nicolau/olimp/obm-l.html O administrador desta lista é ========================================================================= From owner-obm-l@sucuri.mat.puc-rio.br Sat Sep 7 19:11:30 2002 Return-Path: Received: (from majordom@localhost) by sucuri.mat.puc-rio.br (8.9.3/8.9.3) id TAA11937 for obm-l-MTTP; Sat, 7 Sep 2002 19:10:55 -0300 Received: from gorgo.centroin.com.br (gorgo.centroin.com.br [200.225.63.128]) by sucuri.mat.puc-rio.br (8.9.3/8.9.3) with ESMTP id TAA11932 for ; Sat, 7 Sep 2002 19:10:52 -0300 Received: from centroin.com.br (du74c.rjo.centroin.com.br [200.225.58.74]) (authenticated bits=0) by gorgo.centroin.com.br (8.12.2/8.12.1) with ESMTP id g87MC2nH016342 for ; Sat, 7 Sep 2002 19:12:07 -0300 (BRT) Message-ID: <3D7A7A42.8030304@centroin.com.br> Date: Sat, 07 Sep 2002 19:14:26 -0300 From: Augusto =?ISO-8859-1?Q?C=E9sar?= Morgado User-Agent: Mozilla/5.0 (Windows; U; Win98; en-US; rv:0.9.4.1) Gecko/20020508 Netscape6/6.2.3 X-Accept-Language: en-us MIME-Version: 1.0 To: obm-l@mat.puc-rio.br Subject: Re: [obm-l] funcao References: Content-Type: text/plain; charset=ISO-8859-1; format=flowed Content-Transfer-Encoding: 8bit Sender: owner-obm-l@sucuri.mat.puc-rio.br Precedence: bulk Reply-To: obm-l@mat.puc-rio.br Uma funçao eh uma correspondencia. A funçao que associa a cada real o seu dobro pode ser descrita como associando a cada real x o valor de 2x (ou seja, f(x)=2x), mas pode ser descrita como associando a cada real y o valor de 2y (ou seja, f(y) = 2y), etc. A funçao, ou seja a correspondencia, eh f. f(x) eh o valor que f associa a x. Deu pra entender? Se deu, otimo. Agora, ca entre nos e que os muito rigorosos nao nos leiam. Todo mundo fala seja uma funcao f(x) querendo dizer seja uma funçao f que a cada x associa f(x) Portanto, nao se preocupe em demasia com esse erro, que eh na verdade apenas um abuso de linguagem. Morgado adr.scr.m wrote: >li num livro e gostaria de saber porque eh >errado falar " seja uma funcao f(x)",e o >certo eh " seja uma funcao f " . >[]'s. >Adriano. > > >__________________________________________________________________________ >AcessoBOL, só R$ 9,90! O menor preço do mercado! >Assine já! http://www.bol.com.br/acessobol > > >========================================================================= >Instruções para entrar na lista, sair da lista e usar a lista em >http://www.mat.puc-rio.br/~nicolau/olimp/obm-l.html >O administrador desta lista é >========================================================================= > > ========================================================================= Instruções para entrar na lista, sair da lista e usar a lista em http://www.mat.puc-rio.br/~nicolau/olimp/obm-l.html O administrador desta lista é ========================================================================= From owner-obm-l@sucuri.mat.puc-rio.br Sat Sep 7 19:14:12 2002 Return-Path: Received: (from majordom@localhost) by sucuri.mat.puc-rio.br (8.9.3/8.9.3) id TAA12063 for obm-l-MTTP; Sat, 7 Sep 2002 19:14:10 -0300 Received: from gorgo.centroin.com.br (gorgo.centroin.com.br [200.225.63.128]) by sucuri.mat.puc-rio.br (8.9.3/8.9.3) with ESMTP id TAA12057 for ; Sat, 7 Sep 2002 19:14:07 -0300 Received: from centroin.com.br (du74c.rjo.centroin.com.br [200.225.58.74]) (authenticated bits=0) by gorgo.centroin.com.br (8.12.2/8.12.1) with ESMTP id g87MFMnH016436 for ; Sat, 7 Sep 2002 19:15:22 -0300 (BRT) Message-ID: <3D7A7B11.80809@centroin.com.br> Date: Sat, 07 Sep 2002 19:17:53 -0300 From: Augusto =?ISO-8859-1?Q?C=E9sar?= Morgado User-Agent: Mozilla/5.0 (Windows; U; Win98; en-US; rv:0.9.4.1) Gecko/20020508 Netscape6/6.2.3 X-Accept-Language: en-us MIME-Version: 1.0 To: obm-l@mat.puc-rio.br Subject: Re: [obm-l] listas de treinamento interncionais References: Content-Type: text/plain; charset=ISO-8859-1; format=flowed Content-Transfer-Encoding: 8bit Sender: owner-obm-l@sucuri.mat.puc-rio.br Precedence: bulk Reply-To: obm-l@mat.puc-rio.br Acesse www.obm.org.br basketboy_igor wrote: >Gostaria de ser informando de sites ou receber lista de >treinamentos intenacionais p/ IMO, USAMO, Ibero, torneio >das cidades e outras olimpíadas internacionais de >matemática, ou de sites que contenham questões vaiadas >de matemática vindas de várias partes do mundo. > > > >__________________________________________________________________________ >AcessoBOL, só R$ 9,90! O menor preço do mercado! >Assine já! http://www.bol.com.br/acessobol > > >========================================================================= >Instruções para entrar na lista, sair da lista e usar a lista em >http://www.mat.puc-rio.br/~nicolau/olimp/obm-l.html >O administrador desta lista é >========================================================================= > > ========================================================================= Instruções para entrar na lista, sair da lista e usar a lista em http://www.mat.puc-rio.br/~nicolau/olimp/obm-l.html O administrador desta lista é ========================================================================= From owner-obm-l@sucuri.mat.puc-rio.br Sat Sep 7 19:15:21 2002 Return-Path: Received: (from majordom@localhost) by sucuri.mat.puc-rio.br (8.9.3/8.9.3) id TAA12132 for obm-l-MTTP; Sat, 7 Sep 2002 19:15:19 -0300 Received: from ginsberg.uol.com.br (ginsberg.uol.com.br [200.221.4.48]) by sucuri.mat.puc-rio.br (8.9.3/8.9.3) with ESMTP id TAA12127 for ; Sat, 7 Sep 2002 19:15:16 -0300 Received: from Eder ([200.211.159.126]) by ginsberg.uol.com.br (8.9.1/8.9.1) with SMTP id TAA10756 for ; Sat, 7 Sep 2002 19:14:27 -0300 (BRT) Message-ID: <000a01c256bb$d2743760$3758fea9@Eder> From: "Eder" To: References: <002d01c25698$37561000$7ac1fea9@Eder> <3D7A712D.5010507@centroin.com.br> Subject: Re: [obm-l] ??? Date: Sat, 7 Sep 2002 19:13:34 -0300 MIME-Version: 1.0 Content-Type: multipart/alternative; boundary="----=_NextPart_000_0007_01C256A2.A9843620" X-Priority: 3 X-MSMail-Priority: Normal X-Mailer: Microsoft Outlook Express 5.00.2314.1300 X-MimeOLE: Produced By Microsoft MimeOLE V5.00.2314.1300 Sender: owner-obm-l@sucuri.mat.puc-rio.br Precedence: bulk Reply-To: obm-l@mat.puc-rio.br This is a multi-part message in MIME format. ------=_NextPart_000_0007_01C256A2.A9843620 Content-Type: text/plain; charset="iso-8859-1" Content-Transfer-Encoding: quoted-printable Puxa!Eu n=E3o reparei nissso...Valeu! ----- Original Message -----=20 From: Augusto C=E9sar Morgado=20 To: obm-l@mat.puc-rio.br=20 Sent: Saturday, September 07, 2002 6:35 PM Subject: Re: [obm-l] ??? 1) como os co-senos t=EAm o saud=E1vel h=E1bito de serem menores que = ou iguais a 1, o unico jeito dessa soma dar 3 eh cosx=3D1 e cos 7x =3D = 1. Logo, as unicas solu=E7oes sao as da forma 2kpi.=20 Eder wrote: Gostaria de ajuda nestes problemas: 1)Encontre todas as solu=E7=F5es reais de cosx+(cosx)^5+cos7x=3D3. Bom,eu tranformei cosx+cos7x em produto,depois sa=ED fazendo = simplifica=E7=F5es para ficar somente com cosx.Substituindo cosx po = m,cheguei a um polin=F4mio bem "estranho".Foi f=E1cil checar que m=3D1 = =3D=3D>cosx=3D1=3D=3D>x=3D2kpi =E9 solu=E7=E3o (na verdade =E9 f=E1cil = de ver isso no enunciado),por=E9m n=E3o soube mais o que fazer depois = disso,para ver se h=E1 outras solu=E7=F5es. 2)Considere um polin=F4mio de coeficientes inteiros.Sabe-se que = p(21)=3D17,p(32)=3D-247 e p(37)=3D33.Prove que se para algum N tivermos = p(N)=3D51.Ent=E3o N=3D26. Obrigado pela ajuda. Eder ------=_NextPart_000_0007_01C256A2.A9843620 Content-Type: text/html; charset="iso-8859-1" Content-Transfer-Encoding: quoted-printable
Puxa!Eu n=E3o reparei = nissso...Valeu!
----- Original Message -----
From:=20 Augusto=20 C=E9sar Morgado
Sent: Saturday, September 07, = 2002 6:35=20 PM
Subject: Re: [obm-l] ???

1) como os co-senos t=EAm o saud=E1vel h=E1bito de = serem menores que=20 ou iguais a 1, o unico jeito dessa soma dar 3  eh  cosx=3D1 = e cos 7x =3D=20 1.
Logo, as unicas solu=E7oes sao as da forma 2kpi.

Eder = wrote:
Gostaria de ajuda nestes=20 problemas:
 
1)Encontre todas as solu=E7=F5es = reais de=20 cosx+(cosx)^5+cos7x=3D3.
 
Bom,eu tranformei cosx+cos7x em = produto,depois=20 sa=ED fazendo simplifica=E7=F5es para ficar somente com = cosx.Substituindo cosx po=20 m,cheguei a um polin=F4mio bem "estranho".Foi f=E1cil checar que = m=3D1=20 =3D=3D>cosx=3D1=3D=3D>x=3D2kpi  =E9 solu=E7=E3o (na = verdade =E9 f=E1cil de ver isso no=20 enunciado),por=E9m n=E3o soube mais o que fazer depois disso,para = ver se h=E1=20 outras solu=E7=F5es.
 
2)Considere um polin=F4mio de = coeficientes=20 inteiros.Sabe-se que  p(21)=3D17,p(32)=3D-247 e = p(37)=3D33.Prove que se para=20 algum N tivermos p(N)=3D51.Ent=E3o N=3D26.
 
 
Obrigado pela ajuda.
 
Eder
 
 

------=_NextPart_000_0007_01C256A2.A9843620-- ========================================================================= Instruções para entrar na lista, sair da lista e usar a lista em http://www.mat.puc-rio.br/~nicolau/olimp/obm-l.html O administrador desta lista é ========================================================================= From owner-obm-l@sucuri.mat.puc-rio.br Sat Sep 7 19:24:00 2002 Return-Path: Received: (from majordom@localhost) by sucuri.mat.puc-rio.br (8.9.3/8.9.3) id TAA12557 for obm-l-MTTP; Sat, 7 Sep 2002 19:23:51 -0300 Received: from mtasjc.directnet.com.br (mtasjc.directnet.com.br [200.152.0.15]) by sucuri.mat.puc-rio.br (8.9.3/8.9.3) with ESMTP id TAA12552 for ; Sat, 7 Sep 2002 19:23:48 -0300 Received: from meu ([200.152.17.12]) by pop.directnet.com.br (iPlanet Messaging Server 5.1 HotFix 1.3 (built Jul 16 2002)) with SMTP id <0H230031ZA444B@pop.directnet.com.br> for obm-l@mat.puc-rio.br; Sat, 07 Sep 2002 19:21:49 -0300 (EST) Date: Sat, 07 Sep 2002 19:39:39 -0300 From: Daniel Subject: [obm-l] =?iso-8859-1?Q?Re:_=5Bobm-l=5D_Equa=E7=E3o_Polinomial?= To: obm-l@mat.puc-rio.br Message-id: <007c01c256bf$74bd4ae0$0c1198c8@directnet.com.br> MIME-version: 1.0 X-MIMEOLE: Produced By Microsoft MimeOLE V5.00.2615.200 X-Mailer: Microsoft Outlook Express 5.00.2615.200 Content-type: multipart/alternative; boundary="Boundary_(ID_qRbaqnCQKQN5S62PaUU+eQ)" X-Priority: 3 X-MSMail-priority: Normal References: <001001c2569f$3e977fa0$9606c3c8@lba> Sender: owner-obm-l@sucuri.mat.puc-rio.br Precedence: bulk Reply-To: obm-l@mat.puc-rio.br This is a multi-part message in MIME format. --Boundary_(ID_qRbaqnCQKQN5S62PaUU+eQ) Content-type: text/plain; charset=iso-8859-1 Content-transfer-encoding: quoted-printable O teorema das ra=EDzes racionais diz que SE equa=E7=E3o polinomial de = coeficientes inteiros admite uma raiz racional da forma p/q, p, q = inteiros, q n=E3o nulo, ENT=C3O p =E9 divisor do termo independente e q = =E9 divisor do termo de maor grau. Ou seja nem toda a equa=E7=E3o vai ser resolvida dessa forma. Daniel From: Leonardo Borges Avelino=20 To: obm-l@mat.puc-rio.br=20 Sent: Friday, September 06, 2002 10:47 AM Subject: [obm-l] Equa=E7=E3o Polinomial Ei pessoal Eu aprendi a resolver equa=E7=F5es polinomiais atrav=E9s do m=E9todo = de consulta de ra=EDzes. Aquele que vc pega os divisores do termo = independente e divide pelos ... , enfim. Uma d=FAvida: Quando o termo = independente =E9 primo e utilizarmos deste m=E9todo, como saberemos = quais s=E3o as ra=EDzes se nenhuma das poss=EDveis ra=EDzes servir como = raiz. Para ficar clara minha pergunta vai um exemplo. =20 x^3 + 5x^2 - 4x + 7=3D0 Consult=E1r=EDamos + ou - 1 , e + ou - 7. Mas n=E3o conseguir=EDamos = achar raiz alguma. Desculpem-me se falo besteira, mas foi algo que naum descobri ainda. Valeu!! Leonardo Borges Avelino --Boundary_(ID_qRbaqnCQKQN5S62PaUU+eQ) Content-type: text/html; charset=iso-8859-1 Content-transfer-encoding: quoted-printable  
O=20 teorema das ra=EDzes racionais diz que SE equa=E7=E3o polinomial de = coeficientes=20 inteiros admite uma raiz racional da forma p/q, p, q inteiros, q n=E3o = nulo,=20 ENT=C3O p =E9 divisor do termo independente e q =E9 divisor do termo = de maor=20 grau.
Ou seja=20 nem toda a equa=E7=E3o vai ser resolvida dessa forma.
     =20             =    =20         Daniel         From:=20 Leonardo=20 Borges Avelino
Sent: Friday, September 06, = 2002 10:47=20 AM
Subject: [obm-l] Equa=E7=E3o = Polinomial

Ei pessoal
Eu aprendi a resolver equa=E7=F5es = polinomiais=20 atrav=E9s do m=E9todo de consulta de ra=EDzes. Aquele que vc pega os = divisores do=20 termo independente e divide pelos ... , enfim. Uma d=FAvida: = Quando o termo=20 independente =E9 primo e utilizarmos deste m=E9todo, como saberemos = quais s=E3o as=20 ra=EDzes se nenhuma das poss=EDveis ra=EDzes servir como = raiz.
Para ficar clara minha pergunta vai = um=20 exemplo.
 
x^3 + 5x^2 - 4x = + 7=3D0
 
Consult=E1r=EDamos + ou - = 1   , e + ou - 7.=20 Mas n=E3o conseguir=EDamos achar raiz alguma.
 
Desculpem-me se falo besteira, mas = foi algo que=20 naum descobri ainda.
 
Valeu!!
Leonardo Borges=20 Avelino
--Boundary_(ID_qRbaqnCQKQN5S62PaUU+eQ)-- ========================================================================= Instruções para entrar na lista, sair da lista e usar a lista em http://www.mat.puc-rio.br/~nicolau/olimp/obm-l.html O administrador desta lista é ========================================================================= From owner-obm-l@sucuri.mat.puc-rio.br Sat Sep 7 20:45:06 2002 Return-Path: Received: (from majordom@localhost) by sucuri.mat.puc-rio.br (8.9.3/8.9.3) id UAA14960 for obm-l-MTTP; Sat, 7 Sep 2002 20:44:50 -0300 Received: from hotmail.com (f140.pav2.hotmail.com [64.4.37.140]) by sucuri.mat.puc-rio.br (8.9.3/8.9.3) with ESMTP id UAA14956 for ; Sat, 7 Sep 2002 20:44:47 -0300 Received: from mail pickup service by hotmail.com with Microsoft SMTPSVC; Sat, 7 Sep 2002 16:45:37 -0700 Received: from 200.199.182.11 by pv2fd.pav2.hotmail.msn.com with HTTP; Sat, 07 Sep 2002 23:45:37 GMT X-Originating-IP: [200.199.182.11] From: "Fernanda Medeiros" To: obm-l@mat.puc-rio.br Subject: [obm-l] violencia Date: Sat, 07 Sep 2002 23:45:37 +0000 Mime-Version: 1.0 Content-Type: text/plain; charset=iso-8859-1; format=flowed Message-ID: X-OriginalArrivalTime: 07 Sep 2002 23:45:37.0981 (UTC) FILETIME=[AADC56D0:01C256C8] Sender: owner-obm-l@sucuri.mat.puc-rio.br Precedence: bulk Reply-To: obm-l@mat.puc-rio.br Olá, alguém pode dar uma ajuda nestas questões? 1.a)uma "gang" tem infinitos bandidos e cada um dos meliantes tem um único inimigo no interior da "gang",que ele quer matar.Prove q é possivel reunir uma quantidade infinita de bandidos desta "gang", semq haja o risco de q um bandido mate outro durante a reunião. b)Se cada bandido tiver um nº finito mas indefinido de inimigos(um bandido pode ter 2 inimigos, outro somente 1, um terceiro pode ter 20 e assim por diante).Será sempre possivel promover uma reunião com infinitos bandidos sem risco de derramamento de sangue? 2.Encontre todas as soluções em inteiros não negativos para: 2^x +3^y =z^2 3.Encontre todos os inteiros positivos (x,y) tais q 7^x - 3^y =4 Valeu!! FÊ _________________________________________________________________ Converse com seus amigos online, faça o download grátis do MSN Messenger: http://messenger.msn.com.br ========================================================================= Instruções para entrar na lista, sair da lista e usar a lista em http://www.mat.puc-rio.br/~nicolau/olimp/obm-l.html O administrador desta lista é ========================================================================= From owner-obm-l@sucuri.mat.puc-rio.br Sat Sep 7 21:03:37 2002 Return-Path: Received: (from majordom@localhost) by sucuri.mat.puc-rio.br (8.9.3/8.9.3) id VAA15398 for obm-l-MTTP; Sat, 7 Sep 2002 21:03:29 -0300 Received: from gorgo.centroin.com.br (gorgo.centroin.com.br [200.225.63.128]) by sucuri.mat.puc-rio.br (8.9.3/8.9.3) with ESMTP id VAA15394 for ; Sat, 7 Sep 2002 21:03:26 -0300 Received: from centroin.com.br (du99c.rjo.centroin.com.br [200.225.58.99]) (authenticated bits=0) by gorgo.centroin.com.br (8.12.2/8.12.1) with ESMTP id g8804fnH021002 for ; Sat, 7 Sep 2002 21:04:42 -0300 (BRT) Message-ID: <3D7A94B0.3050802@centroin.com.br> Date: Sat, 07 Sep 2002 21:07:12 -0300 From: Augusto =?ISO-8859-1?Q?C=E9sar?= Morgado User-Agent: Mozilla/5.0 (Windows; U; Win98; en-US; rv:0.9.4.1) Gecko/20020508 Netscape6/6.2.3 X-Accept-Language: en-us MIME-Version: 1.0 To: obm-l@mat.puc-rio.br Subject: Re: [obm-l] ??? References: <002d01c25698$37561000$7ac1fea9@Eder> Content-Type: multipart/alternative; boundary="------------070801050407030504010406" Sender: owner-obm-l@sucuri.mat.puc-rio.br Precedence: bulk Reply-To: obm-l@mat.puc-rio.br --------------070801050407030504010406 Content-Type: text/plain; charset=ISO-8859-1; format=flowed Content-Transfer-Encoding: 8bit Ha alguma coisa errada no problema 2. Em um polinomio de coeficientes inteiros, P(a) - P(b) eh divisivel por a-b (a, b inteiros, naturalmente). Entao, 247 - 17 = 230 deveria ser divisivel por 32 - 21 = 11. Eder wrote: > Gostaria de ajuda nestes problemas: > > > > 1)Encontre todas as soluções reais de cosx+(cosx)^5+cos7x=3. > > > > Bom,eu tranformei cosx+cos7x em produto,depois saí fazendo > simplificações para ficar somente com cosx.Substituindo cosx po > m,cheguei a um polinômio bem "estranho".Foi fácil checar que m=1 > ==>cosx=1==>x=2kpi é solução (na verdade é fácil de ver isso no > enunciado),porém não soube mais o que fazer depois disso,para ver se > há outras soluções. > > > > 2)Considere um polinômio de coeficientes inteiros.Sabe-se que > p(21)=17,p(32)=-247 e p(37)=33.Prove que se para algum N tivermos > p(N)=51.Então N=26. > > > > > > Obrigado pela ajuda. > > > > Eder > > > > > --------------070801050407030504010406 Content-Type: text/html; charset=us-ascii Content-Transfer-Encoding: 7bit Ha alguma coisa errada no problema 2.
Em um polinomio de coeficientes inteiros, P(a) - P(b) eh divisivel por a-b (a, b inteiros, naturalmente).
Entao, 247 - 17 = 230  deveria ser divisivel por  32 - 21 = 11.

Eder wrote:
Gostaria de ajuda nestes problemas:
 
1)Encontre todas as soluções reais de cosx+(cosx)^5+cos7x=3.
 
Bom,eu tranformei cosx+cos7x em produto,depois saí fazendo simplificações para ficar somente com cosx.Substituindo cosx po m,cheguei a um polinômio bem "estranho".Foi fácil checar que m=1 ==>cosx=1==>x=2kpi  é solução (na verdade é fácil de ver isso no enunciado),porém não soube mais o que fazer depois disso,para ver se há outras soluções.
 
2)Considere um polinômio de coeficientes inteiros.Sabe-se que  p(21)=17,p(32)=-247 e p(37)=33.Prove que se para algum N tivermos p(N)=51.Então N=26.
 
 
Obrigado pela ajuda.
 
Eder
 
 

--------------070801050407030504010406-- ========================================================================= Instruções para entrar na lista, sair da lista e usar a lista em http://www.mat.puc-rio.br/~nicolau/olimp/obm-l.html O administrador desta lista é ========================================================================= From owner-obm-l@sucuri.mat.puc-rio.br Sat Sep 7 21:40:24 2002 Return-Path: Received: (from majordom@localhost) by sucuri.mat.puc-rio.br (8.9.3/8.9.3) id VAA16281 for obm-l-MTTP; Sat, 7 Sep 2002 21:40:15 -0300 Received: from silva5.uol.com.br (silva5.uol.com.br [200.221.4.52]) by sucuri.mat.puc-rio.br (8.9.3/8.9.3) with ESMTP id VAA16277 for ; Sat, 7 Sep 2002 21:40:13 -0300 Received: from Eder ([200.211.154.146]) by silva5.uol.com.br (8.9.1/8.9.1) with SMTP id VAA18910 for ; Sat, 7 Sep 2002 21:43:24 -0300 (EST) Message-ID: <003701c256cf$da426ac0$3758fea9@Eder> From: "Eder" To: References: <002d01c25698$37561000$7ac1fea9@Eder> <3D7A94B0.3050802@centroin.com.br> Subject: Re: [obm-l] ??? Date: Sat, 7 Sep 2002 21:36:06 -0300 MIME-Version: 1.0 Content-Type: multipart/alternative; boundary="----=_NextPart_000_0034_01C256B6.931BBB60" X-Priority: 3 X-MSMail-Priority: Normal X-Mailer: Microsoft Outlook Express 5.00.2314.1300 X-MimeOLE: Produced By Microsoft MimeOLE V5.00.2314.1300 Sender: owner-obm-l@sucuri.mat.puc-rio.br Precedence: bulk Reply-To: obm-l@mat.puc-rio.br This is a multi-part message in MIME format. ------=_NextPart_000_0034_01C256B6.931BBB60 Content-Type: text/plain; charset="iso-8859-1" Content-Transfer-Encoding: quoted-printable Vou checar se n=E3o distorci o problema.Ele caiu na olimp=EDada interna = do meu col=E9gio. ----- Original Message -----=20 From: Augusto C=E9sar Morgado=20 To: obm-l@mat.puc-rio.br=20 Sent: Saturday, September 07, 2002 9:07 PM Subject: Re: [obm-l] ??? Ha alguma coisa errada no problema 2. Em um polinomio de coeficientes inteiros, P(a) - P(b) eh divisivel por = a-b (a, b inteiros, naturalmente). Entao, 247 - 17 =3D 230 deveria ser divisivel por 32 - 21 =3D 11. Eder wrote: Gostaria de ajuda nestes problemas: 1)Encontre todas as solu=E7=F5es reais de cosx+(cosx)^5+cos7x=3D3. Bom,eu tranformei cosx+cos7x em produto,depois sa=ED fazendo = simplifica=E7=F5es para ficar somente com cosx.Substituindo cosx po = m,cheguei a um polin=F4mio bem "estranho".Foi f=E1cil checar que m=3D1 = =3D=3D>cosx=3D1=3D=3D>x=3D2kpi =E9 solu=E7=E3o (na verdade =E9 f=E1cil = de ver isso no enunciado),por=E9m n=E3o soube mais o que fazer depois = disso,para ver se h=E1 outras solu=E7=F5es. 2)Considere um polin=F4mio de coeficientes inteiros.Sabe-se que = p(21)=3D17,p(32)=3D-247 e p(37)=3D33.Prove que se para algum N tivermos = p(N)=3D51.Ent=E3o N=3D26. Obrigado pela ajuda. Eder ------=_NextPart_000_0034_01C256B6.931BBB60 Content-Type: text/html; charset="iso-8859-1" Content-Transfer-Encoding: quoted-printable
Vou checar se n=E3o distorci o = problema.Ele caiu na=20 olimp=EDada interna do meu col=E9gio.
----- Original Message -----
From:=20 Augusto=20 C=E9sar Morgado
Sent: Saturday, September 07, = 2002 9:07=20 PM
Subject: Re: [obm-l] ???

Ha alguma coisa errada no problema 2.
Em um = polinomio de=20 coeficientes inteiros, P(a) - P(b) eh divisivel por a-b (a, b = inteiros,=20 naturalmente).
Entao, 247 - 17 =3D 230  deveria ser divisivel = por=20  32 - 21 =3D 11.

Eder wrote:
Gostaria de ajuda nestes=20 problemas:
 
1)Encontre todas as solu=E7=F5es = reais de=20 cosx+(cosx)^5+cos7x=3D3.
 
Bom,eu tranformei cosx+cos7x em = produto,depois=20 sa=ED fazendo simplifica=E7=F5es para ficar somente com = cosx.Substituindo cosx po=20 m,cheguei a um polin=F4mio bem "estranho".Foi f=E1cil checar que = m=3D1=20 =3D=3D>cosx=3D1=3D=3D>x=3D2kpi  =E9 solu=E7=E3o (na = verdade =E9 f=E1cil de ver isso no=20 enunciado),por=E9m n=E3o soube mais o que fazer depois disso,para = ver se h=E1=20 outras solu=E7=F5es.
 
2)Considere um polin=F4mio de = coeficientes=20 inteiros.Sabe-se que  p(21)=3D17,p(32)=3D-247 e = p(37)=3D33.Prove que se para=20 algum N tivermos p(N)=3D51.Ent=E3o N=3D26.
 
 
Obrigado pela ajuda.
 
Eder
 
 

------=_NextPart_000_0034_01C256B6.931BBB60-- ========================================================================= Instruções para entrar na lista, sair da lista e usar a lista em http://www.mat.puc-rio.br/~nicolau/olimp/obm-l.html O administrador desta lista é ========================================================================= From owner-obm-l@sucuri.mat.puc-rio.br Sat Sep 7 21:50:33 2002 Return-Path: Received: (from majordom@localhost) by sucuri.mat.puc-rio.br (8.9.3/8.9.3) id VAA16479 for obm-l-MTTP; Sat, 7 Sep 2002 21:50:23 -0300 Received: from data2.poli.usp.br (data2.poli.usp.br [143.107.106.108]) by sucuri.mat.puc-rio.br (8.9.3/8.9.3) with ESMTP id VAA16475 for ; Sat, 7 Sep 2002 21:50:21 -0300 Received: from web2.poli.usp.br ([143.107.106.102]) by data2.poli.usp.br with Microsoft SMTPSVC(5.0.2195.4453); Sat, 7 Sep 2002 21:51:12 -0300 Received: from web05.poli.usp.br ([143.107.106.19]) by web2.poli.usp.br with Microsoft SMTPSVC(5.0.2195.5329); Sat, 7 Sep 2002 21:50:51 -0300 Received: from apl03.poli.usp.br ([143.107.106.15]) by web05.poli.usp.br with Microsoft SMTPSVC(5.0.2195.5329); Sat, 7 Sep 2002 21:50:51 -0300 X-MimeOLE: Produced By Microsoft Exchange V6.0.5762.3 content-class: urn:content-classes:message MIME-Version: 1.0 Content-Type: multipart/mixed; boundary="----_=_NextPart_001_01C256D1.C709349E" Subject: RE: [obm-l] ??? Date: Sat, 7 Sep 2002 21:50:50 -0300 Message-ID: <2B184DFE97456744924ACF58987D941D019352FE@apl03.poli.usp.br> X-MS-Has-Attach: X-MS-TNEF-Correlator: <2B184DFE97456744924ACF58987D941D019352FE@apl03.poli.usp.br> Thread-Topic: [obm-l] ??? Thread-Index: AcJWz+7VfM4V3uRDT9a76p7fjaYLfQAAUuUB From: "Edilon Ribeiro da Silva" To: X-OriginalArrivalTime: 08 Sep 2002 00:50:51.0098 (UTC) FILETIME=[C7429BA0:01C256D1] Sender: owner-obm-l@sucuri.mat.puc-rio.br Precedence: bulk Reply-To: obm-l@mat.puc-rio.br This is a multi-part message in MIME format. ------_=_NextPart_001_01C256D1.C709349E Content-Type: text/plain; charset="iso-8859-1" Content-Transfer-Encoding: quoted-printable Morgado,=20 =20 Observe que P(32) =3D - 247 e P(21) =3D 17, logo P(32) - = p(21) =3D - 247 - 17 =3D -264, que =E9 divis=EDvel por 32 - 21 =3D 11. =20 Edilon Ribeiro. =20 -----Original Message-----=20 From: Augusto C=E9sar Morgado [mailto:morgado@centroin.com.br]=20 Sent: Sat 9/7/2002 9:07 PM=20 To: obm-l@mat.puc-rio.br=20 Cc:=20 Subject: Re: [obm-l] ??? =09 =09 Ha alguma coisa errada no problema 2. Em um polinomio de coeficientes inteiros, P(a) - P(b) eh divisivel por = a-b (a, b inteiros, naturalmente). Entao, 247 - 17 =3D 230 deveria ser divisivel por 32 - 21 =3D 11. =09 Eder wrote: =09 Gostaria de ajuda nestes problemas: =20 1)Encontre todas as solu=E7=F5es reais de cosx+(cosx)^5+cos7x=3D3. =20 Bom,eu tranformei cosx+cos7x em produto,depois sa=ED fazendo = simplifica=E7=F5es para ficar somente com cosx.Substituindo cosx po = m,cheguei a um polin=F4mio bem "estranho".Foi f=E1cil checar que m=3D1 = =3D=3D>cosx=3D1=3D=3D>x=3D2kpi =E9 solu=E7=E3o (na verdade =E9 f=E1cil = de ver isso no enunciado),por=E9m n=E3o soube mais o que fazer depois = disso,para ver se h=E1 outras solu=E7=F5es. =20 2)Considere um polin=F4mio de coeficientes inteiros.Sabe-se que = p(21)=3D17,p(32)=3D-247 e p(37)=3D33.Prove que se para algum N tivermos = p(N)=3D51.Ent=E3o N=3D26. =20 =20 Obrigado pela ajuda. =20 Eder =20 =20 ------_=_NextPart_001_01C256D1.C709349E Content-Type: application/ms-tnef; name="winmail.dat" Content-Transfer-Encoding: base64 eJ8+IjMAAQaQCAAEAAAAAAABAAEAAQeQBgAIAAAA5AQAAAAAAADoAAEIgAcAGAAAAElQTS5NaWNy b3NvZnQgTWFpbC5Ob3RlADEIAQ2ABAACAAAAAgACAAEEgAEAEAAAAFJFOiBbb2JtLWxdID8/PwBd BAEFgAMADgAAANIHCQAHABUAMgAyAAYAaAEBIIADAA4AAADSBwkABwAVADIAMgAGAGgBAQmAAQAh AAAAMTNDMTg3MzQxNTkwQzk0QzlERDVCRjAyQkE1NjQxOUEAHQcBA5AGAIQRAAA2AAAAAwA2AAAA AABAADkAnjQJx9FWwgEeAD0AAQAAAAUAAABSRTogAAAAAAIBRwABAAAALQAAAGM9QlI7YT0gO3A9 RVBVU1A7bD1BUEwwMy0wMjA5MDgwMDUwNTBaLTY3NjIzAAAAAB4ASQABAAAAEAAAAFJlOiBbb2Jt LWxdID8/PwBAAE4AAPgnrstWwgEeAFoAAQAAABYAAABBdWd1c3RvIEPpc2FyIE1vcmdhZG8AAAAC AVsAAQAAAEsAAAAAAAAAgSsfpL6jEBmdbgDdAQ9UAgAAAABBdWd1c3RvIEPpc2FyIE1vcmdhZG8A U01UUABtb3JnYWRvQGNlbnRyb2luLmNvbS5icgAAAgFcAAEAAAAdAAAAU01UUDpNT1JHQURPQENF TlRST0lOLkNPTS5CUgAAAAAeAF0AAQAAABYAAABBdWd1c3RvIEPpc2FyIE1vcmdhZG8AAAACAV4A AQAAAEsAAAAAAAAAgSsfpL6jEBmdbgDdAQ9UAgAAAABBdWd1c3RvIEPpc2FyIE1vcmdhZG8AU01U UABtb3JnYWRvQGNlbnRyb2luLmNvbS5icgAAAgFfAAEAAAAdAAAAU01UUDpNT1JHQURPQENFTlRS T0lOLkNPTS5CUgAAAAAeAGYAAQAAAAUAAABTTVRQAAAAAB4AZwABAAAAGAAAAG1vcmdhZG9AY2Vu dHJvaW4uY29tLmJyAB4AaAABAAAABQAAAFNNVFAAAAAAHgBpAAEAAAAYAAAAbW9yZ2Fkb0BjZW50 cm9pbi5jb20uYnIAHgBwAAEAAAAMAAAAW29ibS1sXSA/Pz8AAgFxAAEAAAAbAAAAAcJWz+7VfM4V 3uRDT9a76p7fjaYLfQAAUuUBAB4AdAABAAAAFQAAAG9ibS1sQG1hdC5wdWMtcmlvLmJyAAAAAB4A GgwBAAAAGAAAAEVkaWxvbiBSaWJlaXJvIGRhIFNpbHZhAB4AHQ4BAAAADAAAAFtvYm0tbF0gPz8/ AAIBCRABAAAADwsAAAsLAAAsLQAATFpGdSagel8DAAoAcmNwZzEyNYIyA0NodG1sMQMwPwEDAfcK gAKkA+MCAGNowQrAc2V0MCAHEwKA/xADAFAEVghVB7IR1Q5RAwHdENcyBgAGwxHVMwRGENm5Eu9m NBBvFx8CADUYX28RdhHjCO8J9zsdDw4wNd8eIx9BHVwR4QxgYwBQCwmZAWQzNhFgC6U0IBAC7Cpc DrIBkGcU8AqjEeOhI3cxOCA8DrI+JHwbJA8mgjMjECWAZWFkJyXdDvEm/2c0DvA8L1coXyavK/Q1 EWA8BuBkQHkgZGlyPSwQcp8rYCvTACEDMC5xZG8A4PMucQqxXHEc8C5xEPADMB8u1RFgK4sigSyP Zzk28SVwRElWLqkAADDnK6lMNjQ0HzE/MTQlYUYgT05UIGYA0GU9UiIZeSIgAJB6OSAyXzQLGDAD sgHQMMlNBbBnqStAbyw3fTUzoS84ov8uqS63NUsBwC63CqI/OAqA9SSMMCghLzPcPzkxfzKP/zOf NK81vzbPN9847zn/Ow8vSq89v0PvIyw4IxAmbphic3ACgC7IJ2EBQP9Ib0BvQX9Cj0OfUo9Fv0bP /0n/SO9dv0sPTB9NJwqjTb//Ts9gv1O/VM5mT2dfaG9pf/9qj2ufbK9tv27Pb99w73H//3MPdB91 L3Y/d094X3lven//e498n32vfr9/z4Dfge+C//+ED4Ufhi+HP4hPiV+Kb1qx5k+MwASQdmUjLGUh W+OecQpQYO9h9RyDPSMBITuSkV17YxPwZZ+MCDxTVFRSYiBHXXtik9lQACgzMikgPSAt+iAjADdc PBTwKwCVn1o/N1B/md+a4yCO4JeAMjGZl8IxNzzwHKBnb47/91vjl4SYAHCeRZgDl/GesD2X0jJf gDzwkFJU4GU5bS4hdgQAowFkjtADIHB/BbGXoJfyDvCX4J9PW/Ix/DEumz+cTz8/Vi9XP1hP/1lf mr9bf1yPX79er7Evmw//Yd9ND2RPZV+mr6e/uP+ML/+NP6kfqi+rP6xPuz+ub69//7CPsZ+yr7O/ tM+137bvt/8dyeZFLjAcoAOgUmli+mUuQG+mf7qPqJ+/j8Cf/8Gvwr/J38Tfxe/JH8gP2u//2A+9 L74/0z/UT9Vf1m/XfwPYj9mfQkxPQ0tRMFVPVEUuJszwdHkCbMwBTUFSR0lOAC1SSUdIVDog8DBw eCIuqgqxEAIv1d8wczAxuN/mvyuqNJ3h54/f2o/gn9yv7agvUGkiYspvr8t0zQ8B8TusLfmSTwUQ XmcLgAdABdAHkHMsMGVr+ZP1/THoglIuqwuAZbcKge2/lQdClf/9xkYDYRo6mF8v/1qdWkF1Z/Z1 6eCfIEOjAvqgYyClD4/vX98h8KE8hSBbbQtw2SwQbzoWEDyjQMvwD7ALDoAh0C4cgG0uYnJ+Xfs/ /E/9X/5v/38MlVMPCQEBPwJPnYdTYXQgUDkvNy87sDAU8DnkOjCh0FBNBO8F/wcFvwoPCx8MLw0/ Dk8ZFFQIUAcQDxEfnZZvYm0tbKJACAB0LnAhEC358P/QMAnACf8XfxiPGZ8aryLV/ENjHE8dX52H IE8hXyJvByN/JI8PVXViamVj5w/vJx+deFJl6xAT/xUP7RYUWx7zCeA/NNAozynf/yrvNT82TzdZ ME/hD+If4y//5D87P++dy1HRdfN/9I/1lfhIYSD6QAQACACR8aOAW0WQjrByCLBFkG6fIHDt0CBi 6hBF8TLQTTjvOf/4RW0gReAxzzLfBwWkID83AQEQH+CjQI7gCYBlZtxpY8xwCRD6gCD6IE8QbdAR c57Ql4BhoNKXgGI5l8BlaKNEo2Cj9WEt/5bAUACe0JbASz9MTxYUT1it+jB0zFD6QG1O8ilHz2dI 30nuCRBhb57Qobky/jMrr99P4FxOUI7Q+fBFkP+OoVDZUm9Tf01WBN1bn4qu/6Ropl9XT1hfZX9m j0n9TlD9zJB30CBPEC7saU9qX17//2APBwU8Tz1fPm/ofU7QTxAaPU4QZBOAE0BkMDEEYzLRMDk4 JDM3J9EwP3CSkCQ3y+AxZrBlYTlAa3Pp8HDMAf91wutv7lFun+7P790WT/XR4DxNRVRBkfFPAQkQ BeoxU+rwTUwgNS5ANTAuNDUyR7AxHjiSkMzg+jBV8D1HRcBORVJBVE9tUXsPj3wffS/wF5WBWUxF gg3/8GKGXkJShf+Cz4mPg5/wr3/xv/LPQ19EbHJQ9e/LW0GvXeHtgPd/+IE1zl9HT7DHixBd4k5R YWp1RsJPIO9PEm8vi4JHJnNsHdE/O2//cd9y7z6fP69Av4rfi++M//+OD48fkC+RP2G/Ys/gb51v /55/n4+gn6Gvor+jz6Tfpe//rG+oD6kfkl+Tb5R/lY+x4uwxKVlgfyJyl3AEMEbAq08wv2FzTcB1 BHE3rCD/vVBPIb8ACBBPME5RmG+zP4cWE7ShCYBzeCsow+IQKV41K8PhN3g9/jPQT5tfnG+tX65v r3+wj/+xn7Kvs7+0z7Xfx4+3/7kP/6o/q0+sX8h/yY/Kn8uvzL//zc/O38/v0P/Xf9Mf1C+6Ly+7 P7xPvV/c8EIJkCxlZnW/IEagbmZRkFXwaT9/EcQBxMNQoErgRyFkdf0EMCxOUFGAwOHBT94vw2bb RlAEcWTmUecwbgfB5xD8bXD1oE6x7xLANu1hRZB/8FJRoL/AVfN/EUrgw+Iu9y5xBCB10HX6IAfB w+JRcY/sX+1vw1fpkGNoZQQAX+phRZBLAU2zwEE0ThJiDetBIpgB6fFobyIu/kYJQOZQBHF2oAgg fxD3IMXxsnH3UCBtPWUA9F+T9W/DZj091R9ndNZp/j7ceMPi+zD+D/8f3IfFAPgya3DqcAE/1i/X Ou8hUjm/uWUz9DAogSAg/13BRsBOUQcT+edOUV3BT0B+c7/ARuL7b/x/w2YPwHXnvrBd8POgKSxR gQcSSuD/+CEIAr/ALoD7AcDSCrD64v/vcl5B7AReYAphDhDxUgoS+V4gIGj58grPC9/DVw8w/+nh v67FP8ZPx1/YT9lf2m//23/cj92f3q/fv+DPGM/i7//j/wRPBV/XTxm/Gs8b3xzv/x3/Hw8gHyEv Ij8ovyRfJW8f5R/mL+c/6E8uMDIpQ/9/MDhQa4GXcPe+Tl4Sjy9v18NmT1by8GH4sC0R8Q/xryZf J28oelFwKGTwKfsw2jcOEChkoESALVnSl3DVRNE3RIAzxSBQmcBRQP/60xHx8UM9fz6Pw2Y4IPdA ffjQTr8gUTHqQE+wRDFOXUSANWVQWWEH5E44kDb/Fn8XjxifKY8qnyuvLL8tz/8u3y/vMP8yD06v NC81P0Fv/0J/KI9Pn1CvUb9Sz1PfVO//Vf9XD1gfXp9aP1tPXF9db/9o71+PYJ9hr2K/Y89k32Xv /2b/aA9uj2ovaz82bzd/OI/ROZ8gT2KXIGcN0etg/GVsl0CXk0xfTW9Of29P/3BfcW9yf3OPdJ91 r3a/d8//g/9573r/bC9tP25PhO+F//+HD4gfiS+KP4tPjF+Nb5Pvf4+PkJ98H30vfj9/T5lgRf87 IYG/gs+D35TPld+W75f//5kPmh+bL5w/nU+of59voH//ka+Sv7LPqW+qf6uPrJ+tr/+uv6/PsN+x 77hvtA+1H7Yv/7c/wr+5X7pvu3+8j72fy8YAQkxPQ0tRVU98VEXMe77xvfDNP8uFMfIywZBCUqPp 0G/N787/X8zfvg+/H8Aj2Mw1olEv+mLrkHmj4Njc2G+/49JQvcB+MkVgzCDHUqPgfd+wAB4ANRAB AAAAPQAAADwyQjE4NERGRTk3NDU2NzQ0OTI0QUNGNTg5ODdEOTQxRDAxOTM1MkZFQGFwbDAzLnBv bGkudXNwLmJyPgAAAAAeAEcQAQAAAA8AAABtZXNzYWdlL3JmYzgyMgAACwDyEAEAAAAfAPMQAQAA ADgAAABSAEUAJQAzAEEAIABbAG8AYgBtAC0AbABdACAAJQAzAEYAJQAzAEYAJQAzAEYALgBFAE0A TAAAAAsA9hAAAAAAQAAHMJR1ZzrRVsIBQAAIMIqCF8fRVsIBAwDeP69vAAADAPE/CQQAAB4A+D8B AAAAGAAAAEVkaWxvbiBSaWJlaXJvIGRhIFNpbHZhAAIB+T8BAAAAUAAAAAAAAADcp0DIwEIQGrS5 CAArL+GCAQAAAAAAAAAvTz1FUFVTUC9PVT1ESVJFVE9SSUEvQ049UkVDSVBJRU5UUy9DTj1FRElM T04uU0lMVkEAHgD6PwEAAAAVAAAAU3lzdGVtIEFkbWluaXN0cmF0b3IAAAAAAgH7PwEAAAAeAAAA AAAAANynQMjAQhAatLkIACsv4YIBAAAAAAAAAC4AAAADAP0/5AQAAAMAGUAAAAAAAwAaQAAAAAAD AB1AAAAAAAMAHkAAAAAAHgAwQAEAAAANAAAARURJTE9OLlNJTFZBAAAAAB4AMUABAAAADQAAAEVE SUxPTi5TSUxWQQAAAAAeADJAAQAAABgAAABtb3JnYWRvQGNlbnRyb2luLmNvbS5icgAeADNAAQAA ABgAAABtb3JnYWRvQGNlbnRyb2luLmNvbS5icgAeADhAAQAAAA0AAABFRElMT04uU0lMVkEAAAAA HgA5QAEAAAACAAAALgAAAAsAKQAAAAAACwAjAAAAAAADAAYQpvCWSQMABxCoAwAAAwAQEAAAAAAD ABEQAAAAAB4ACBABAAAAZQAAAE1PUkdBRE8sT0JTRVJWRVFVRVAoMzIpPS0yNDdFUCgyMSk9MTcs TE9HT1AoMzIpLVAoMjEpPS0yNDctMTc9LTI2NCxRVUXpRElWSVPtVkVMUE9SMzItMjE9MTFFRElM T05SSUIAAAAAAgF/AAEAAAA9AAAAPDJCMTg0REZFOTc0NTY3NDQ5MjRBQ0Y1ODk4N0Q5NDFEMDE5 MzUyRkVAYXBsMDMucG9saS51c3AuYnI+AAAAADlq ------_=_NextPart_001_01C256D1.C709349E-- ========================================================================= Instruções para entrar na lista, sair da lista e usar a lista em http://www.mat.puc-rio.br/~nicolau/olimp/obm-l.html O administrador desta lista é ========================================================================= From owner-obm-l@sucuri.mat.puc-rio.br Sat Sep 7 21:54:49 2002 Return-Path: Received: (from majordom@localhost) by sucuri.mat.puc-rio.br (8.9.3/8.9.3) id VAA16665 for obm-l-MTTP; Sat, 7 Sep 2002 21:54:48 -0300 Received: from hotmail.com (f77.sea2.hotmail.com [207.68.165.77]) by sucuri.mat.puc-rio.br (8.9.3/8.9.3) with ESMTP id VAA16660 for ; Sat, 7 Sep 2002 21:54:44 -0300 Received: from mail pickup service by hotmail.com with Microsoft SMTPSVC; Sat, 7 Sep 2002 17:55:35 -0700 Received: from 200.220.36.213 by sea2fd.sea2.hotmail.msn.com with HTTP; Sun, 08 Sep 2002 00:55:34 GMT X-Originating-IP: [200.220.36.213] From: "leonardo mattos" To: obm-l@mat.puc-rio.br Subject: [obm-l] Geometria(Quadrilatero) Date: Sun, 08 Sep 2002 00:55:34 +0000 Mime-Version: 1.0 Content-Type: text/plain; charset=iso-8859-1; format=flowed Message-ID: X-OriginalArrivalTime: 08 Sep 2002 00:55:35.0124 (UTC) FILETIME=[708D8940:01C256D2] Sender: owner-obm-l@sucuri.mat.puc-rio.br Precedence: bulk Reply-To: obm-l@mat.puc-rio.br Ola pessoal, Dado um quadrilatero ABCD qualquer sao traçadas suas diagonais AC e BC. Pode afirmar que o angulo(ACD) é congruente ao angulo(ABD),assim como o angulo(BAC) é congruente ao angulo(BDC)?Se sim, porque? Um abraço,Leonardo _________________________________________________________________ MSN Photos é a maneira mais fácil e prática de editar e compartilhar sua fotos: http://photos.msn.com.br ========================================================================= Instruções para entrar na lista, sair da lista e usar a lista em http://www.mat.puc-rio.br/~nicolau/olimp/obm-l.html O administrador desta lista é ========================================================================= From owner-obm-l@sucuri.mat.puc-rio.br Sat Sep 7 22:11:38 2002 Return-Path: Received: (from majordom@localhost) by sucuri.mat.puc-rio.br (8.9.3/8.9.3) id WAA17526 for obm-l-MTTP; Sat, 7 Sep 2002 22:11:29 -0300 Received: from silva5.uol.com.br (silva5.uol.com.br [200.221.4.52]) by sucuri.mat.puc-rio.br (8.9.3/8.9.3) with ESMTP id WAA17521 for ; Sat, 7 Sep 2002 22:11:26 -0300 Received: from Eder ([200.211.154.146]) by silva5.uol.com.br (8.9.1/8.9.1) with SMTP id WAA14503 for ; Sat, 7 Sep 2002 22:14:34 -0300 (EST) Message-ID: <004401c256d4$2b55e460$3758fea9@Eder> From: "Eder" To: References: <002d01c25698$37561000$7ac1fea9@Eder> <3D7A94B0.3050802@centroin.com.br> Subject: Re: [obm-l] ??? Date: Sat, 7 Sep 2002 22:07:52 -0300 MIME-Version: 1.0 Content-Type: multipart/alternative; boundary="----=_NextPart_000_0041_01C256BB.02FCF300" X-Priority: 3 X-MSMail-Priority: Normal X-Mailer: Microsoft Outlook Express 5.00.2314.1300 X-MimeOLE: Produced By Microsoft MimeOLE V5.00.2314.1300 Sender: owner-obm-l@sucuri.mat.puc-rio.br Precedence: bulk Reply-To: obm-l@mat.puc-rio.br This is a multi-part message in MIME format. ------=_NextPart_000_0041_01C256BB.02FCF300 Content-Type: text/plain; charset="iso-8859-1" Content-Transfer-Encoding: quoted-printable Agora que percebi! N=E3o d=E1 para ver direito que p(32)=3D - 247.Assim, = -247-17=3D264 que =E9 divis=EDvel por 11.Continuo esperando alguma luz = nesse problema... ----- Original Message -----=20 From: Augusto C=E9sar Morgado=20 To: obm-l@mat.puc-rio.br=20 Sent: Saturday, September 07, 2002 9:07 PM Subject: Re: [obm-l] ??? Ha alguma coisa errada no problema 2. Em um polinomio de coeficientes inteiros, P(a) - P(b) eh divisivel por = a-b (a, b inteiros, naturalmente). Entao, 247 - 17 =3D 230 deveria ser divisivel por 32 - 21 =3D 11. Eder wrote: Gostaria de ajuda nestes problemas: 1)Encontre todas as solu=E7=F5es reais de cosx+(cosx)^5+cos7x=3D3. Bom,eu tranformei cosx+cos7x em produto,depois sa=ED fazendo = simplifica=E7=F5es para ficar somente com cosx.Substituindo cosx po = m,cheguei a um polin=F4mio bem "estranho".Foi f=E1cil checar que m=3D1 = =3D=3D>cosx=3D1=3D=3D>x=3D2kpi =E9 solu=E7=E3o (na verdade =E9 f=E1cil = de ver isso no enunciado),por=E9m n=E3o soube mais o que fazer depois = disso,para ver se h=E1 outras solu=E7=F5es. 2)Considere um polin=F4mio de coeficientes inteiros.Sabe-se que = p(21)=3D17,p(32)=3D-247 e p(37)=3D33.Prove que se para algum N tivermos = p(N)=3D51.Ent=E3o N=3D26. Obrigado pela ajuda. Eder ------=_NextPart_000_0041_01C256BB.02FCF300 Content-Type: text/html; charset="iso-8859-1" Content-Transfer-Encoding: quoted-printable
Agora que percebi! N=E3o d=E1 para ver = direito que=20 p(32)=3D - 247.Assim, -247-17=3D264 que =E9 divis=EDvel por 11.Continuo = esperando alguma=20 luz nesse problema...
----- Original Message -----
From:=20 Augusto=20 C=E9sar Morgado
Sent: Saturday, September 07, = 2002 9:07=20 PM
Subject: Re: [obm-l] ???

Ha alguma coisa errada no problema 2.
Em um = polinomio de=20 coeficientes inteiros, P(a) - P(b) eh divisivel por a-b (a, b = inteiros,=20 naturalmente).
Entao, 247 - 17 =3D 230  deveria ser divisivel = por=20  32 - 21 =3D 11.

Eder wrote:
Gostaria de ajuda nestes=20 problemas:
 
1)Encontre todas as solu=E7=F5es = reais de=20 cosx+(cosx)^5+cos7x=3D3.
 
Bom,eu tranformei cosx+cos7x em = produto,depois=20 sa=ED fazendo simplifica=E7=F5es para ficar somente com = cosx.Substituindo cosx po=20 m,cheguei a um polin=F4mio bem "estranho".Foi f=E1cil checar que = m=3D1=20 =3D=3D>cosx=3D1=3D=3D>x=3D2kpi  =E9 solu=E7=E3o (na = verdade =E9 f=E1cil de ver isso no=20 enunciado),por=E9m n=E3o soube mais o que fazer depois disso,para = ver se h=E1=20 outras solu=E7=F5es.
 
2)Considere um polin=F4mio de = coeficientes=20 inteiros.Sabe-se que  p(21)=3D17,p(32)=3D-247 e = p(37)=3D33.Prove que se para=20 algum N tivermos p(N)=3D51.Ent=E3o N=3D26.
 
 
Obrigado pela ajuda.
 
Eder
 
 

------=_NextPart_000_0041_01C256BB.02FCF300-- ========================================================================= Instruções para entrar na lista, sair da lista e usar a lista em http://www.mat.puc-rio.br/~nicolau/olimp/obm-l.html O administrador desta lista é ========================================================================= From owner-obm-l@sucuri.mat.puc-rio.br Sat Sep 7 22:13:00 2002 Return-Path: Received: (from majordom@localhost) by sucuri.mat.puc-rio.br (8.9.3/8.9.3) id WAA17603 for obm-l-MTTP; Sat, 7 Sep 2002 22:12:58 -0300 Received: from silva5.uol.com.br (silva5.uol.com.br [200.221.4.52]) by sucuri.mat.puc-rio.br (8.9.3/8.9.3) with ESMTP id WAA17599 for ; Sat, 7 Sep 2002 22:12:56 -0300 Received: from Eder ([200.211.154.146]) by silva5.uol.com.br (8.9.1/8.9.1) with SMTP id WAA15458 for ; Sat, 7 Sep 2002 22:16:07 -0300 (EST) Message-ID: <004f01c256d4$61bfa400$3758fea9@Eder> From: "Eder" To: Subject: Re: [obm-l] ??? Date: Sat, 7 Sep 2002 22:09:25 -0300 MIME-Version: 1.0 Content-Type: multipart/alternative; boundary="----=_NextPart_000_004C_01C256BB.39F49AC0" X-Priority: 3 X-MSMail-Priority: Normal X-Mailer: Microsoft Outlook Express 5.00.2314.1300 X-MimeOLE: Produced By Microsoft MimeOLE V5.00.2314.1300 Sender: owner-obm-l@sucuri.mat.puc-rio.br Precedence: bulk Reply-To: obm-l@mat.puc-rio.br This is a multi-part message in MIME format. ------=_NextPart_000_004C_01C256BB.39F49AC0 Content-Type: text/plain; charset="iso-8859-1" Content-Transfer-Encoding: quoted-printable -247-17 =3D -264 ----- Original Message -----=20 From: Eder=20 To: obm-l@mat.puc-rio.br=20 Sent: Saturday, September 07, 2002 10:07 PM Subject: Re: [obm-l] ??? Agora que percebi! N=E3o d=E1 para ver direito que p(32)=3D - = 247.Assim, -247-17=3D264 que =E9 divis=EDvel por 11.Continuo esperando = alguma luz nesse problema... ----- Original Message -----=20 From: Augusto C=E9sar Morgado=20 To: obm-l@mat.puc-rio.br=20 Sent: Saturday, September 07, 2002 9:07 PM Subject: Re: [obm-l] ??? Ha alguma coisa errada no problema 2. Em um polinomio de coeficientes inteiros, P(a) - P(b) eh divisivel = por a-b (a, b inteiros, naturalmente). Entao, 247 - 17 =3D 230 deveria ser divisivel por 32 - 21 =3D 11. Eder wrote: Gostaria de ajuda nestes problemas: 1)Encontre todas as solu=E7=F5es reais de cosx+(cosx)^5+cos7x=3D3. Bom,eu tranformei cosx+cos7x em produto,depois sa=ED fazendo = simplifica=E7=F5es para ficar somente com cosx.Substituindo cosx po = m,cheguei a um polin=F4mio bem "estranho".Foi f=E1cil checar que m=3D1 = =3D=3D>cosx=3D1=3D=3D>x=3D2kpi =E9 solu=E7=E3o (na verdade =E9 f=E1cil = de ver isso no enunciado),por=E9m n=E3o soube mais o que fazer depois = disso,para ver se h=E1 outras solu=E7=F5es. 2)Considere um polin=F4mio de coeficientes inteiros.Sabe-se que = p(21)=3D17,p(32)=3D-247 e p(37)=3D33.Prove que se para algum N tivermos = p(N)=3D51.Ent=E3o N=3D26. Obrigado pela ajuda. Eder ------=_NextPart_000_004C_01C256BB.39F49AC0 Content-Type: text/html; charset="iso-8859-1" Content-Transfer-Encoding: quoted-printable
-247-17 =3D -264
----- Original Message -----
From:=20 Eder
Sent: Saturday, September 07, = 2002 10:07=20 PM
Subject: Re: [obm-l] ???

Agora que percebi! N=E3o d=E1 para = ver direito que=20 p(32)=3D - 247.Assim, -247-17=3D264 que =E9 divis=EDvel por = 11.Continuo esperando=20 alguma luz nesse problema...
----- Original Message -----
From:=20 Augusto C=E9sar Morgado =
Sent: Saturday, September 07, = 2002 9:07=20 PM
Subject: Re: [obm-l] = ???

Ha alguma coisa errada no problema 2.
Em um = polinomio de=20 coeficientes inteiros, P(a) - P(b) eh divisivel por a-b (a, b = inteiros,=20 naturalmente).
Entao, 247 - 17 =3D 230  deveria ser = divisivel por=20  32 - 21 =3D 11.

Eder wrote:
Gostaria de ajuda nestes=20 problemas:
 
1)Encontre todas as solu=E7=F5es = reais de=20 cosx+(cosx)^5+cos7x=3D3.
 
Bom,eu tranformei cosx+cos7x em=20 produto,depois sa=ED fazendo simplifica=E7=F5es para ficar somente = com=20 cosx.Substituindo cosx po m,cheguei a um polin=F4mio bem = "estranho".Foi=20 f=E1cil checar que m=3D1 =3D=3D>cosx=3D1=3D=3D>x=3D2kpi =  =E9 solu=E7=E3o (na verdade=20 =E9 f=E1cil de ver isso no enunciado),por=E9m n=E3o soube mais o = que fazer depois=20 disso,para ver se h=E1 outras solu=E7=F5es.
 
2)Considere um polin=F4mio de = coeficientes=20 inteiros.Sabe-se que  p(21)=3D17,p(32)=3D-247 e = p(37)=3D33.Prove que se=20 para algum N tivermos p(N)=3D51.Ent=E3o N=3D26.
 
 
Obrigado pela ajuda.
 
Eder
 
=
 

------=_NextPart_000_004C_01C256BB.39F49AC0-- ========================================================================= Instruções para entrar na lista, sair da lista e usar a lista em http://www.mat.puc-rio.br/~nicolau/olimp/obm-l.html O administrador desta lista é ========================================================================= From owner-obm-l@sucuri.mat.puc-rio.br Sat Sep 7 22:15:49 2002 Return-Path: Received: (from majordom@localhost) by sucuri.mat.puc-rio.br (8.9.3/8.9.3) id WAA17724 for obm-l-MTTP; Sat, 7 Sep 2002 22:15:47 -0300 Received: from seki.bol.com.br (seki.bol.com.br [200.221.24.26]) by sucuri.mat.puc-rio.br (8.9.3/8.9.3) with ESMTP id WAA17720 for ; Sat, 7 Sep 2002 22:15:44 -0300 Received: from lba (200.221.24.192) by seki.bol.com.br (5.1.071) id 3D6143DE00569C49 for obm-l@mat.puc-rio.br; Sat, 7 Sep 2002 22:16:15 -0300 Message-ID: <002001c256d5$ddc51a20$1103c3c8@lba> From: "Leonardo Borges Avelino" To: Subject: [obm-l] Leonardo Mattos Date: Sat, 7 Sep 2002 22:19:58 -0300 MIME-Version: 1.0 Content-Type: multipart/alternative; boundary="----=_NextPart_000_001D_01C256BC.B41DFDE0" X-Priority: 3 X-MSMail-Priority: Normal X-Mailer: Microsoft Outlook Express 5.00.2615.200 X-MimeOLE: Produced By Microsoft MimeOLE V5.00.2615.200 X-Sender-IP: 200.195.3.17 Sender: owner-obm-l@sucuri.mat.puc-rio.br Precedence: bulk Reply-To: obm-l@mat.puc-rio.br This is a multi-part message in MIME format. ------=_NextPart_000_001D_01C256BC.B41DFDE0 Content-Type: text/plain; charset="iso-8859-1" Content-Transfer-Encoding: quoted-printable Ei Leonardo O resto dos arquivos eu tb naum achei. Baixei todas as provas e arquivos = do site e estava resolvendo umas provas do Cone-Sul Tenho alguns arquivos interessantes em ingl=EAs. Te mandarei a URL do = site http://madvax.maths.uwa.edu.au/~gregg/Olympiad/ Abra=E7os Leonardo Borges Avelino ------=_NextPart_000_001D_01C256BC.B41DFDE0 Content-Type: text/html; charset="iso-8859-1" Content-Transfer-Encoding: quoted-printable
Ei Leonardo
O resto dos arquivos eu tb naum achei. = Baixei todas=20 as provas e arquivos do site e estava resolvendo umas provas do=20 Cone-Sul
Tenho alguns arquivos interessantes em = ingl=EAs. Te=20 mandarei a URL do site
 
http://madvax.ma= ths.uwa.edu.au/~gregg/Olympiad/
 
Abra=E7os
Leonardo Borges = Avelino
------=_NextPart_000_001D_01C256BC.B41DFDE0-- ========================================================================= Instruções para entrar na lista, sair da lista e usar a lista em http://www.mat.puc-rio.br/~nicolau/olimp/obm-l.html O administrador desta lista é ========================================================================= From owner-obm-l@sucuri.mat.puc-rio.br Sat Sep 7 22:18:07 2002 Return-Path: Received: (from majordom@localhost) by sucuri.mat.puc-rio.br (8.9.3/8.9.3) id WAA17915 for obm-l-MTTP; Sat, 7 Sep 2002 22:18:06 -0300 Received: from gorgo.centroin.com.br (gorgo.centroin.com.br [200.225.63.128]) by sucuri.mat.puc-rio.br (8.9.3/8.9.3) with ESMTP id WAA17911 for ; Sat, 7 Sep 2002 22:18:03 -0300 Received: from centroin.com.br (du178c.rjo.centroin.com.br [200.225.58.178]) (authenticated bits=0) by gorgo.centroin.com.br (8.12.2/8.12.1) with ESMTP id g881JHnH024466 for ; Sat, 7 Sep 2002 22:19:18 -0300 (BRT) Message-ID: <3D7AA62E.8060200@centroin.com.br> Date: Sat, 07 Sep 2002 22:21:50 -0300 From: Augusto =?ISO-8859-1?Q?C=E9sar?= Morgado User-Agent: Mozilla/5.0 (Windows; U; Win98; en-US; rv:0.9.4.1) Gecko/20020508 Netscape6/6.2.3 X-Accept-Language: en-us MIME-Version: 1.0 To: obm-l@mat.puc-rio.br Subject: Re: [obm-l] ??? References: <002d01c25698$37561000$7ac1fea9@Eder> <3D7A94B0.3050802@centroin.com.br> <003701c256cf$da426ac0$3758fea9@Eder> Content-Type: multipart/alternative; boundary="------------080700090100040504070805" Sender: owner-obm-l@sucuri.mat.puc-rio.br Precedence: bulk Reply-To: obm-l@mat.puc-rio.br --------------080700090100040504070805 Content-Type: text/plain; charset=ISO-8859-1; format=flowed Content-Transfer-Encoding: 8bit OK, o sinal de - estava ilegivel. Obrigado ao Edilon que chamou a atençao. Morgado Eder wrote: > Vou checar se não distorci o problema.Ele caiu na olimpíada interna do > meu colégio. > > ----- Original Message ----- > > From: Augusto César Morgado > > To: obm-l@mat.puc-rio.br > > Sent: Saturday, September 07, 2002 9:07 PM > > Subject: Re: [obm-l] ??? > > > Ha alguma coisa errada no problema 2. > Em um polinomio de coeficientes inteiros, P(a) - P(b) eh divisivel > por a-b (a, b inteiros, naturalmente). > Entao, 247 - 17 = 230 deveria ser divisivel por 32 - 21 = 11. > > Eder wrote: > >> Gostaria de ajuda nestes problemas: >> >> >> >> 1)Encontre todas as soluções reais de cosx+(cosx)^5+cos7x=3. >> >> >> >> Bom,eu tranformei cosx+cos7x em produto,depois saí fazendo >> simplificações para ficar somente com cosx.Substituindo cosx po >> m,cheguei a um polinômio bem "estranho".Foi fácil checar que m=1 >> ==>cosx=1==>x=2kpi é solução (na verdade é fácil de ver isso no >> enunciado),porém não soube mais o que fazer depois disso,para ver >> se há outras soluções. >> >> >> >> 2)Considere um polinômio de coeficientes inteiros.Sabe-se que >> p(21)=17,p(32)=-247 e p(37)=33.Prove que se para algum N tivermos >> p(N)=51.Então N=26. >> >> >> >> >> >> Obrigado pela ajuda. >> >> >> >> Eder >> >> >> >> >> > --------------080700090100040504070805 Content-Type: text/html; charset=us-ascii Content-Transfer-Encoding: 7bit OK, o sinal de - estava ilegivel. Obrigado ao Edilon que chamou a atençao.
Morgado

Eder wrote:
Vou checar se não distorci o problema.Ele caiu na olimpíada interna do meu colégio.
----- Original Message -----
Sent: Saturday, September 07, 2002 9:07 PM
Subject: Re: [obm-l] ???

Ha alguma coisa errada no problema 2.
Em um polinomio de coeficientes inteiros, P(a) - P(b) eh divisivel por a-b (a, b inteiros, naturalmente).
Entao, 247 - 17 = 230  deveria ser divisivel por  32 - 21 = 11.

Eder wrote:
Gostaria de ajuda nestes problemas:
 
1)Encontre todas as soluções reais de cosx+(cosx)^5+cos7x=3.
 
Bom,eu tranformei cosx+cos7x em produto,depois saí fazendo simplificações para ficar somente com cosx.Substituindo cosx po m,cheguei a um polinômio bem "estranho".Foi fácil checar que m=1 ==>cosx=1==>x=2kpi  é solução (na verdade é fácil de ver isso no enunciado),porém não soube mais o que fazer depois disso,para ver se há outras soluções.
 
2)Considere um polinômio de coeficientes inteiros.Sabe-se que  p(21)=17,p(32)=-247 e p(37)=33.Prove que se para algum N tivermos p(N)=51.Então N=26.
 
 
Obrigado pela ajuda.
 
Eder
 
 


--------------080700090100040504070805-- ========================================================================= Instruções para entrar na lista, sair da lista e usar a lista em http://www.mat.puc-rio.br/~nicolau/olimp/obm-l.html O administrador desta lista é ========================================================================= From owner-obm-l@sucuri.mat.puc-rio.br Sat Sep 7 22:26:21 2002 Return-Path: Received: (from majordom@localhost) by sucuri.mat.puc-rio.br (8.9.3/8.9.3) id WAA18599 for obm-l-MTTP; Sat, 7 Sep 2002 22:26:12 -0300 Received: from pina.terra.com.br (pina.terra.com.br [200.176.3.17]) by sucuri.mat.puc-rio.br (8.9.3/8.9.3) with ESMTP id WAA18595 for ; Sat, 7 Sep 2002 22:26:09 -0300 Received: from pavuna.terra.com.br (pavuna.terra.com.br [200.176.3.41]) by pina.terra.com.br (Postfix) with ESMTP id 50F2852F82 for ; Sat, 7 Sep 2002 22:27:01 -0300 (EST) Received: from stabel (unknown [200.203.38.123]) (authenticated user dudasta) by pavuna.terra.com.br (Postfix) with ESMTP id 045916853A for ; Sat, 7 Sep 2002 22:27:01 -0300 (EST) Message-ID: <008f01c256d6$d7711e20$0301a8c0@stabel> From: "Eduardo Casagrande Stabel" To: References: Subject: Re: [obm-l] Geometria(Quadrilatero) Date: Sat, 7 Sep 2002 22:27:05 -0300 MIME-Version: 1.0 Content-Type: text/plain; charset="iso-8859-1" Content-Transfer-Encoding: 8bit X-Priority: 3 X-MSMail-Priority: Normal X-Mailer: Microsoft Outlook Express 6.00.2600.0000 X-MimeOLE: Produced By Microsoft MimeOLE V6.00.2600.0000 Sender: owner-obm-l@sucuri.mat.puc-rio.br Precedence: bulk Reply-To: obm-l@mat.puc-rio.br From: "leonardo mattos" > Ola pessoal, > > Dado um quadrilatero ABCD qualquer sao traçadas suas diagonais AC e BC. > Pode afirmar que o angulo(ACD) é congruente ao angulo(ABD),assim como o > angulo(BAC) é congruente ao angulo(BDC)?Se sim, porque? > Um abraço,Leonardo Olá Leonardo Mattos, uma pequena correção: as diagonais são AC e BD. Use o seguinte Lema. Trace um círculo com centro O e marque dois pontos A e B na circunferência. Demonstre que dado um ponto C no arco maior AB então 2*angulo(ACB) = angulo(AOB). Esse lema é muito conhecido, você encontra ele em qq. livro de geometria elementar. Aí você pode afirmar que se ABCD (no sentido horário ou anti-horário) é um quadrilátero INSCRITÍVEL entao o 2*angulo(ACD) = angulo(AOD) = 2*angulo(ABD), sendo O o centro da circunferência onde ele está inscrito. Vale também a recíproca: angulo(ACD) = angulo(ABD) (C e B no mesmo semiplano determinado por AB) implica que o quadilátero ABCD é inscritível. De forma que o que você disse não vale para qualquer quadrilátero. Só para os inscritíveis. Eduardo. ========================================================================= Instruções para entrar na lista, sair da lista e usar a lista em http://www.mat.puc-rio.br/~nicolau/olimp/obm-l.html O administrador desta lista é ========================================================================= From owner-obm-l@sucuri.mat.puc-rio.br Sat Sep 7 22:27:52 2002 Return-Path: Received: (from majordom@localhost) by sucuri.mat.puc-rio.br (8.9.3/8.9.3) id WAA18944 for obm-l-MTTP; Sat, 7 Sep 2002 22:27:46 -0300 Received: from shen.bol.com.br (shen.bol.com.br [200.221.24.14]) by sucuri.mat.puc-rio.br (8.9.3/8.9.3) with ESMTP id WAA18939 for ; Sat, 7 Sep 2002 22:27:44 -0300 Received: from bol.com.br (200.221.24.138) by shen.bol.com.br (5.1.071) id 3D63D22F005C3512 for obm-l@mat.puc-rio.br; Sat, 7 Sep 2002 22:27:54 -0300 Date: Sat, 7 Sep 2002 22:26:50 -0300 Message-Id: Subject: Re: [obm-l] funcao MIME-Version: 1.0 Content-Type: text/plain;charset="iso-8859-1" From: "adr.scr.m" To: obm-l@mat.puc-rio.br X-XaM3-API-Version: 2.4.3.4.4 X-SenderIP: 200.151.54.156 Content-Transfer-Encoding: 8bit X-MIME-Autoconverted: from quoted-printable to 8bit by sucuri.mat.puc-rio.br id WAA18940 Sender: owner-obm-l@sucuri.mat.puc-rio.br Precedence: bulk Reply-To: obm-l@mat.puc-rio.br agora eu entendi muito bem. muito obrigado Morgado. []'s. Adriano. __________________________________________________________________________ AcessoBOL, só R$ 9,90! O menor preço do mercado! Assine já! http://www.bol.com.br/acessobol ========================================================================= Instruções para entrar na lista, sair da lista e usar a lista em http://www.mat.puc-rio.br/~nicolau/olimp/obm-l.html O administrador desta lista é ========================================================================= From owner-obm-l@sucuri.mat.puc-rio.br Sat Sep 7 22:45:36 2002 Return-Path: Received: (from majordom@localhost) by sucuri.mat.puc-rio.br (8.9.3/8.9.3) id WAA20719 for obm-l-MTTP; Sat, 7 Sep 2002 22:45:24 -0300 Received: from shannon.bol.com.br (shannon.bol.com.br [200.221.24.13]) by sucuri.mat.puc-rio.br (8.9.3/8.9.3) with ESMTP id WAA20708 for ; Sat, 7 Sep 2002 22:45:20 -0300 Received: from bol.com.br (200.221.24.118) by shannon.bol.com.br (5.1.071) id 3D60E4DC0056A60D for obm-l@mat.puc-rio.br; Sat, 7 Sep 2002 22:45:54 -0300 Date: Sat, 7 Sep 2002 22:44:16 -0300 Message-Id: Subject: [obm-l] =?iso-8859-1?q?Re=3A=5Bobm=2Dl=5D_Equa=E7=E3o_Polinomial?= MIME-Version: 1.0 Content-Type: multipart/mixed; boundary="_=__=_XaM3_Boundary.1031449456.2A.649879.42.22948.52.42.101010.235728456" From: "rafaelc.l" To: obm-l@mat.puc-rio.br X-XaM3-API-Version: 2.4.3.4.4 X-SenderIP: 200.176.166.145 Sender: owner-obm-l@sucuri.mat.puc-rio.br Precedence: bulk Reply-To: obm-l@mat.puc-rio.br --_=__=_XaM3_Boundary.1031449456.2A.649879.42.22948.52.42.101010.235728456 Content-Type: text/plain;charset="iso-8859-1" Content-Transfer-Encoding: quoted-printable Esse m=E9todo usando o termo independente =E9 s=F3 para ra=EDzes racionais. Este polin=F4mio deve ter obrigatoriamente uma ra=EDz irracional. __________________________________________________________________________ AcessoBOL, s=F3 R$ 9,90! O menor pre=E7o do mercado! Assine j=E1! http://www.bol.com.br/acessobol --_=__=_XaM3_Boundary.1031449456.2A.649879.42.22948.52.42.101010.235728456 Content-Type: text/plain; name="00000H14" Content-Transfer-Encoding: base64 RWkgcGVzc29hbA0KRXUgYXByZW5kaSBhIHJlc29sdmVyIGVxdWHn9WVzIHBvbGlub21pYWlz IGF0cmF26XMgZG8gbel0b2RvIGRlIGNvbnN1bHRhIGRlIHJh7Xplcy4gQXF1ZWxlIHF1ZSB2 YyBwZWdhIG9zIGRpdmlzb3JlcyBkbyB0ZXJtbyBpbmRlcGVuZGVudGUgZSBkaXZpZGUgcGVs b3MgLi4uICwgZW5maW0uIFVtYSBk+nZpZGE6IFF1YW5kbyBvIHRlcm1vIGluZGVwZW5kZW50 ZSDpIHByaW1vIGUgdXRpbGl6YXJtb3MgZGVzdGUgbel0b2RvLCBjb21vIHNhYmVyZW1vcyBx dWFpcyBz428gYXMgcmHtemVzIHNlIG5lbmh1bWEgZGFzIHBvc3PtdmVpcyByYe16ZXMgc2Vy dmlyIGNvbW8gcmFpei4NClBhcmEgZmljYXIgY2xhcmEgbWluaGEgcGVyZ3VudGEgdmFpIHVt IGV4ZW1wbG8uDQoNCnheMyArIDV4XjIgLSA0eCArIDc9MA0KDQpDb25zdWx04XLtYW1vcyAr IG91IC0gMSAgICwgZSArIG91IC0gNy4gTWFzIG7jbyBjb25zZWd1aXLtYW1vcyBhY2hhciBy YWl6IGFsZ3VtYS4NCg0KRGVzY3VscGVtLW1lIHNlIGZhbG8gYmVzdGVpcmEsIG1hcyBmb2kg YWxnbyBxdWUgbmF1bSBkZXNjb2JyaSBhaW5kYS4NCg0KVmFsZXUhIQ0KTGVvbmFyZG8gQm9y Z2VzIEF2ZWxpbm8NCg== --_=__=_XaM3_Boundary.1031449456.2A.649879.42.22948.52.42.101010.235728456-- ========================================================================= Instruções para entrar na lista, sair da lista e usar a lista em http://www.mat.puc-rio.br/~nicolau/olimp/obm-l.html O administrador desta lista é ========================================================================= From owner-obm-l@sucuri.mat.puc-rio.br Sat Sep 7 22:49:55 2002 Return-Path: Received: (from majordom@localhost) by sucuri.mat.puc-rio.br (8.9.3/8.9.3) id WAA20912 for obm-l-MTTP; Sat, 7 Sep 2002 22:49:50 -0300 Received: from gorgo.centroin.com.br (gorgo.centroin.com.br [200.225.63.128]) by sucuri.mat.puc-rio.br (8.9.3/8.9.3) with ESMTP id WAA20907 for ; Sat, 7 Sep 2002 22:49:48 -0300 Received: from centroin.com.br (du143c.rjo.centroin.com.br [200.225.58.143]) (authenticated bits=0) by gorgo.centroin.com.br (8.12.2/8.12.1) with ESMTP id g881p3nH025754 for ; Sat, 7 Sep 2002 22:51:03 -0300 (BRT) Message-ID: <3D7AAD9D.2020602@centroin.com.br> Date: Sat, 07 Sep 2002 22:53:33 -0300 From: Augusto =?ISO-8859-1?Q?C=E9sar?= Morgado User-Agent: Mozilla/5.0 (Windows; U; Win98; en-US; rv:0.9.4.1) Gecko/20020508 Netscape6/6.2.3 X-Accept-Language: en-us MIME-Version: 1.0 To: obm-l@mat.puc-rio.br Subject: [Fwd: Re: [obm-l] ???] Content-Type: multipart/alternative; boundary="------------040107010105020200090808" Sender: owner-obm-l@sucuri.mat.puc-rio.br Precedence: bulk Reply-To: obm-l@mat.puc-rio.br --------------040107010105020200090808 Content-Type: text/plain; charset=ISO-8859-1; format=flowed Content-Transfer-Encoding: 8bit Eder wrote: > Gostaria de ajuda nestes problemas: > > > > 2)Considere um polinômio de coeficientes inteiros.Sabe-se que > p(21)=17,p(32)=-247 e p(37)=33.Prove que se para algum N tivermos > p(N)=51.Então N=26. > > > > > > Em um polinomio de coeficientes inteiros, P(a) - P(b) eh > divisivel por a - b (a, b inteiros, naturalmente). > Logo, N - 21 divide 51 - 17 = 34 N - 32 divide 51 - (-247) = 298 N - 37 divide 51 - 33 = 18 A ultima condiçao diz que N - 37 so pode ser 1, 2, 3, 6, 9, 18 ou os seus simetricos. Isso significa que N so pode ser 38, 39, 40, 43, 46, 55, 36, 35, 34, 31, 28, 19. Levando em conta a primeira condiçao, sobram como possiveis valores de N 38, 55, 19. Levando em conta a segunda condiçao resta apenas NINGUEM Nao pode haver um N inteiro tal que P(N) = 51 ou eu errei contas! > > --------------040107010105020200090808 Content-Type: text/html; charset=us-ascii Content-Transfer-Encoding: 7bit
Eder wrote:
Gostaria de ajuda nestes problemas:
 
2)Considere um polinômio de coeficientes inteiros.Sabe-se que  p(21)=17,p(32)=-247 e p(37)=33.Prove que se para algum N tivermos p(N)=51.Então N=26.
 
 
Em um polinomio de coeficientes inteiros, P(a) - P(b)  eh divisivel por  a - b (a, b inteiros, naturalmente).
Logo, N - 21 divide 51 - 17 = 34
N - 32 divide 51 - (-247) = 298
N - 37 divide 51 - 33 = 18
A ultima condiçao diz que  N - 37 so pode ser  1, 2, 3, 6, 9, 18 ou os seus simetricos.
Isso significa que N so pode ser 38, 39, 40, 43, 46, 55, 36, 35, 34, 31, 28, 19.
Levando em conta a primeira condiçao, sobram como possiveis valores de N
38, 55, 19.
Levando em conta a segunda condiçao resta apenas    NINGUEM

Nao pode haver um N inteiro tal que P(N) = 51    ou  eu errei contas!

 

--------------040107010105020200090808-- ========================================================================= Instruções para entrar na lista, sair da lista e usar a lista em http://www.mat.puc-rio.br/~nicolau/olimp/obm-l.html O administrador desta lista é ========================================================================= From owner-obm-l@sucuri.mat.puc-rio.br Sat Sep 7 22:59:29 2002 Return-Path: Received: (from majordom@localhost) by sucuri.mat.puc-rio.br (8.9.3/8.9.3) id WAA21595 for obm-l-MTTP; Sat, 7 Sep 2002 22:59:07 -0300 Received: from shannon.bol.com.br (shannon.bol.com.br [200.221.24.13]) by sucuri.mat.puc-rio.br (8.9.3/8.9.3) with ESMTP id WAA21590 for ; Sat, 7 Sep 2002 22:59:03 -0300 Received: from bol.com.br (200.221.24.118) by shannon.bol.com.br (5.1.071) id 3D60E4DC0056ADBA for obm-l@mat.puc-rio.br; Sat, 7 Sep 2002 22:59:37 -0300 Date: Sat, 7 Sep 2002 22:58:00 -0300 Message-Id: Subject: Re:[obm-l] circuito IME MIME-Version: 1.0 Content-Type: multipart/mixed; boundary="_=__=_XaM3_Boundary.1031450280.2A.305263.42.32709.52.42.101010.1504760947" From: "rafaelc.l" To: obm-l@mat.puc-rio.br X-XaM3-API-Version: 2.4.3.4.4 X-SenderIP: 200.176.166.145 Sender: owner-obm-l@sucuri.mat.puc-rio.br Precedence: bulk Reply-To: obm-l@mat.puc-rio.br --_=__=_XaM3_Boundary.1031450280.2A.305263.42.32709.52.42.101010.1504760947 Content-Type: text/plain;charset="iso-8859-1" Content-Transfer-Encoding: quoted-printable Adriano, n=E3o entendi o q vc disse. Como assim a quest=E3o 10 n=E3o estar no programa? Pelo menos eles poder=EDam ter dado a f=F3rmula da tens=E3o em fun=E7=E3o do tempo ou mesmo a f=F3rmula da carga em func=E3o do tempo(que s=F3 podem ser obtidas por integral e n=E3o s=E3o comentadas em livros de ensino m=E9dio), a=ED caber=EDa ao canditado interpret=E1-la e resolver a quest=E3o. __________________________________________________________________________ AcessoBOL, s=F3 R$ 9,90! O menor pre=E7o do mercado! Assine j=E1! http://www.bol.com.br/acessobol --_=__=_XaM3_Boundary.1031450280.2A.305263.42.32709.52.42.101010.1504760947 Content-Type: text/plain; name="00000NLG" Content-Transfer-Encoding: base64 dW0gdGVtcG8gYXRyYXMgdW0gZ2Fyb3RvIHBlcmd1bnRvdSxlIGV1IA0KcmVzcG9uZGksdmVq YSBlbSBsaXN0YXMgYW50ZXJpb3Jlcy4NCnNvIHF1ZSBlbGUgcGVyZ3VudG91IGNvbW8gIHF1 ZXN0YW8gMy4NCg0KDQpvYnMuIG1lc21vIHF1ZSBzbyBwdWRlc3NlIHNlciBmZWl0YSBjb20g IA0KaW50ZWdyYWwsbmFvIGlyaWEgaW50ZXJlc3NhciBzZSBlc3RhdmEgbm8gDQpwcm9ncmFt YSxqYSBxdWUgYSBxdWVzdGFvIDEwIG5hbyBlc3RhdmEgbm8gDQpwcm9ncmFtYSxlIG5lbSBm b2kgYW51bGFkYS4NCg0KDQpbXSdzLg0KQWRyaWFuby4NCg0KIA0KX19fX19fX19fX19fX19f X19fX19fX19fX19fX19fX19fX19fX19fX19fX19fX19fX19fX19fX19fX19fX19fX19fX19f X19fX18NCkFjZXNzb0JPTCwgc/MgUiQgOSw5MCEgTyBtZW5vciBwcmXnbyBkbyBtZXJjYWRv IQ0KQXNzaW5lIGrhISBodHRwOi8vd3d3LmJvbC5jb20uYnIvYWNlc3NvYm9sDQoNCg0KPT09 PT09PT09PT09PT09PT09PT09PT09PT09PT09PT09PT09PT09PT09PT09PT09PT09PT09PT09 PT09PT09PT09PT09PT09PQ0KSW5zdHJ15/VlcyBwYXJhIGVudHJhciBuYSBsaXN0YSwgc2Fp ciBkYSBsaXN0YSBlIHVzYXIgYSBsaXN0YSBlbQ0KaHR0cDovL3d3dy5tYXQucHVjLXJpby5i ci9+bmljb2xhdS9vbGltcC9vYm0tbC5odG1sDQpPIGFkbWluaXN0cmFkb3IgZGVzdGEgbGlz dGEg6SA8bmljb2xhdUBtYXQucHVjLXJpby5icj4NCj09PT09PT09PT09PT09PT09PT09PT09 PT09PT09PT09PT09PT09PT09PT09PT09PT09PT09PT09PT09PT09PT09PT09PT09PT0NCg== --_=__=_XaM3_Boundary.1031450280.2A.305263.42.32709.52.42.101010.1504760947-- ========================================================================= Instruções para entrar na lista, sair da lista e usar a lista em http://www.mat.puc-rio.br/~nicolau/olimp/obm-l.html O administrador desta lista é ========================================================================= From owner-obm-l@sucuri.mat.puc-rio.br Sat Sep 7 23:32:53 2002 Return-Path: Received: (from majordom@localhost) by sucuri.mat.puc-rio.br (8.9.3/8.9.3) id XAA23214 for obm-l-MTTP; Sat, 7 Sep 2002 23:32:41 -0300 Received: from hotmail.com (f87.law4.hotmail.com [216.33.149.87]) by sucuri.mat.puc-rio.br (8.9.3/8.9.3) with ESMTP id XAA23210 for ; Sat, 7 Sep 2002 23:32:37 -0300 Received: from mail pickup service by hotmail.com with Microsoft SMTPSVC; Sat, 7 Sep 2002 19:33:28 -0700 Received: from 200.163.7.40 by lw4fd.law4.hotmail.msn.com with HTTP; Sun, 08 Sep 2002 02:33:27 GMT X-Originating-IP: [200.163.7.40] From: "rafael dowsley" To: obm-l@mat.puc-rio.br Subject: Re: [obm-l] Re-probabilidade Date: Sun, 08 Sep 2002 02:33:27 +0000 Mime-Version: 1.0 Content-Type: text/plain; charset=iso-8859-1; format=flowed Message-ID: X-OriginalArrivalTime: 08 Sep 2002 02:33:28.0210 (UTC) FILETIME=[1D2F5720:01C256E0] Sender: owner-obm-l@sucuri.mat.puc-rio.br Precedence: bulk Reply-To: obm-l@mat.puc-rio.br Sejam as alturas 1,2,3,4,5(em ordem crescente) Se (1 e 2) ou (1 e 3) ou (1 e 4) ou (1 e 5) sairem a probabilidade de prender a pessoa certa é 0 Se (2 e 3) ou (2 e 4) ou (2 e 5) sairem a probabilidade de prender a pessoa certa é 1 Se (3 e 4) ou (3 e 5) sairem a probabilidade de prender a pessoa certa é 1/2 Se (4 e 5) sairem a probabilidade de prender a pessoa certa é 1/3 Ai e so multiplicar a probabilidade de ocorrer cada situação pela probabilidade de prender a pessoa certa em cada situação e somar todos resultados (4/10 X 0) + (3/10 X 1) + (2/10 X 1/2) + (1/10 X 1/3) = 13/30 Rafael Baião ========================================================================= Instruções para entrar na lista, sair da lista e usar a lista em http://www.mat.puc-rio.br/~nicolau/olimp/obm-l.html O administrador desta lista é ========================================================================= From owner-obm-l@sucuri.mat.puc-rio.br Sat Sep 7 23:44:04 2002 Return-Path: Received: (from majordom@localhost) by sucuri.mat.puc-rio.br (8.9.3/8.9.3) id XAA23485 for obm-l-MTTP; Sat, 7 Sep 2002 23:43:56 -0300 Received: from sr1.terra.com.br (sr1.terra.com.br [200.176.3.16]) by sucuri.mat.puc-rio.br (8.9.3/8.9.3) with ESMTP id XAA23481 for ; Sat, 7 Sep 2002 23:43:53 -0300 Received: from pavuna.terra.com.br (pavuna.terra.com.br [200.176.3.41]) by sr1.terra.com.br (Postfix) with ESMTP id 9895B6EF42 for ; Sat, 7 Sep 2002 23:44:45 -0300 (EST) Received: from xt (200-171-249-185.customer.telesp.net.br [200.171.249.185]) (authenticated user macwad) by pavuna.terra.com.br (Postfix) with ESMTP id CFB8868252 for ; Sat, 7 Sep 2002 23:44:44 -0300 (EST) Message-ID: <001001c256e1$b97e07b0$0401010a@xt> From: =?iso-8859-1?Q?Vinicius_Jos=E9_Fortuna?= To: References: Subject: Re: [obm-l] violencia Date: Sat, 7 Sep 2002 23:44:58 -0300 MIME-Version: 1.0 Content-Type: text/plain; charset="iso-8859-1" Content-Transfer-Encoding: 8bit X-Priority: 3 X-MSMail-Priority: Normal X-Mailer: Microsoft Outlook Express 6.00.2600.0000 X-MimeOLE: Produced By Microsoft MimeOLE V6.00.2600.0000 Sender: owner-obm-l@sucuri.mat.puc-rio.br Precedence: bulk Reply-To: obm-l@mat.puc-rio.br ----- Original Message ----- From: "Fernanda Medeiros" To: Sent: Saturday, September 07, 2002 8:45 PM Subject: [obm-l] violencia > Olá, > alguém pode dar uma ajuda nestas questões? > 1.a)uma "gang" tem infinitos bandidos e cada um dos meliantes tem um único > inimigo no interior da "gang",que ele quer matar.Prove q é possivel reunir > uma quantidade infinita de bandidos desta "gang", semq haja o risco de q > um bandido mate outro durante a reunião. Pense no seguinte algoritmo: Temos o conjunto C de candidatos à reunião que inicialmente contém todos os infinitos bandidos da gangue. Temos o conjunto R de bandidos selecionados para a reunião que inicialmente está vazio. A cada passo do algoritmo procuramos em C alguém que não que matar ninguém de R e ninguém em R quer matá-lo. Seja M o subconjunto de C de bandidos que pelo menos um de R quer matar. Como cada bandido de R só quer matar um, |M|<=|R| Então, como R é finito, M será finito e V=C-M será infinito, pois C é infinito. V será o subconjunto de C dos bandidos que ninguém de R quer matar. Em V procuramos um bandido que não quer matar ninguém de R, retiramos ele de C, o inserimos em R e repete-se o processo. Se sempre for possível encontrar tal bandido, o processo se repetirá indefinidamente e com R sempre crescendo. Assim teremos infnitos bandidos na reunião sem derramamento de sangue. Se em algum momento não for possível encontrar um bandido em V, é porque todos os bandidos de V querem matar alguém de R. Ou seja, ninguém de V quer matar outro de V. Pegamos, então, V como o conjunto de bandidos para a reunião. Como V é infinito, teremos infinitos participantes na reunião. > b)Se cada bandido tiver um nº finito mas indefinido de inimigos(um bandido > pode ter 2 inimigos, outro somente 1, um terceiro pode ter 20 e assim por > diante).Será sempre possivel promover uma reunião com infinitos bandidos sem > risco de derramamento de sangue? Não é possível. Existe um contra-exemplo: Ordene os bandidos formando uma sequência. Imagine que cada bandido quer matar todos que vêm antes dele na sequência. Nunca poderemos ter dois bandidos 'a' e 'b' na reunião, pois ou a vem antes de b, ou b vem antes de, assim haverá um que vai querer matar o outro. Então só poderemos ter um bandido na reunião. Até mais Vinicius Fortuna IC-Unicamp ========================================================================= Instruções para entrar na lista, sair da lista e usar a lista em http://www.mat.puc-rio.br/~nicolau/olimp/obm-l.html O administrador desta lista é ========================================================================= From owner-obm-l@sucuri.mat.puc-rio.br Sun Sep 8 00:29:59 2002 Return-Path: Received: (from majordom@localhost) by sucuri.mat.puc-rio.br (8.9.3/8.9.3) id AAA24722 for obm-l-MTTP; Sun, 8 Sep 2002 00:29:23 -0300 Received: from puma.unisys.com.br (smtp.unisys.com.br [200.220.64.7]) by sucuri.mat.puc-rio.br (8.9.3/8.9.3) with ESMTP id AAA24718 for ; Sun, 8 Sep 2002 00:29:21 -0300 Received: from jf (riohiper01p125.uninet.com.br [200.220.2.125]) by puma.unisys.com.br (8.12.3/8.12.3) with SMTP id g883UBjR016361 for ; Sun, 8 Sep 2002 00:30:12 -0300 (EST) X-Spam-Filter: check_local@puma.unisys.com.br by digitalanswers.org Message-ID: <014101c256e8$b2659b80$5710dcc8@jf> From: "Jose Francisco Guimaraes Costa" To: "obm-l" References: <20020907040635.23024.qmail@web40307.mail.yahoo.com> Subject: Re: [obm-l] Geodesias Date: Sun, 8 Sep 2002 00:34:53 -0300 MIME-Version: 1.0 Content-Type: multipart/alternative; boundary="----=_NextPart_000_013C_01C256CF.8C368F00" X-Priority: 3 X-MSMail-Priority: Normal X-Mailer: Microsoft Outlook Express 6.00.2600.0000 X-MIMEOLE: Produced By Microsoft MimeOLE V6.00.2600.0000 Sender: owner-obm-l@sucuri.mat.puc-rio.br Precedence: bulk Reply-To: obm-l@mat.puc-rio.br This is a multi-part message in MIME format. ------=_NextPart_000_013C_01C256CF.8C368F00 Content-Type: text/plain; charset="iso-8859-1" Content-Transfer-Encoding: quoted-printable V=E1 at=E9 http://www.nima.mil/GandG/pubs.html e selecione o documento = (1) Geodesy for the Layman. Entre os outros documentos, o (11) tamb=E9m = =E9 interessante. JF ----- Original Message -----=20 From: Sharon Guedes=20 To: obm-l@mat.puc-rio.br=20 Sent: Saturday, September 07, 2002 1:06 AM Subject: [obm-l] Geodesias Ol=E1 pessoal, algu=E9m poderia me ajudar, com algum material sobre = geodesia? =C9 para um trabalho que eu tenho que apresentar.=20 At. Sharon. -------------------------------------------------------------------------= ----- Yahoo! PageBuilder - O super editor para cria=E7=E3o de sites: =E9 = gr=E1tis, f=E1cil e r=E1pido. ------=_NextPart_000_013C_01C256CF.8C368F00 Content-Type: text/html; charset="iso-8859-1" Content-Transfer-Encoding: quoted-printable
V=E1 at=E9 http://www.nima.mil/GandG/pu= bs.html e=20 selecione o documento (1) Geodesy for the Layman. Entre os outros = documentos, o=20 (11) tamb=E9m =E9 interessante.
 
JF
 
----- Original Message -----
From:=20 Sharon=20 Guedes
Sent: Saturday, September 07, = 2002 1:06=20 AM
Subject: [obm-l] = Geodesias

 Ol=E1 pessoal, algu=E9m poderia me ajudar, com algum material = sobre=20 geodesia?  =C9 para um trabalho que eu tenho que apresentar.=20

At. Sharon.


Yahoo!=20 PageBuilder - O super editor para cria=E7=E3o de sites: =E9 = gr=E1tis, f=E1cil e=20 r=E1pido. ------=_NextPart_000_013C_01C256CF.8C368F00-- ========================================================================= Instruções para entrar na lista, sair da lista e usar a lista em http://www.mat.puc-rio.br/~nicolau/olimp/obm-l.html O administrador desta lista é ========================================================================= From owner-obm-l@sucuri.mat.puc-rio.br Sun Sep 8 03:23:39 2002 Return-Path: Received: (from majordom@localhost) by sucuri.mat.puc-rio.br (8.9.3/8.9.3) id DAA26088 for obm-l-MTTP; Sun, 8 Sep 2002 03:23:04 -0300 Received: from hotmail.com (oe33.law12.hotmail.com [64.4.18.90]) by sucuri.mat.puc-rio.br (8.9.3/8.9.3) with ESMTP id DAA26084 for ; Sun, 8 Sep 2002 03:23:01 -0300 Received: from mail pickup service by hotmail.com with Microsoft SMTPSVC; Sat, 7 Sep 2002 23:23:52 -0700 X-Originating-IP: [200.151.27.130] From: "Danilo Artigas" To: References: <5.1.0.14.2.20020907135312.017450d8@pop3.uol.com.br> <5.1.0.14.2.20020907172144.0258a8a8@pop3.uol.com.br> Subject: Re: [obm-l] Re-probabilidade Date: Sun, 8 Sep 2002 03:32:35 -0300 MIME-Version: 1.0 Content-Type: text/plain; charset="iso-8859-1" Content-Transfer-Encoding: 8bit X-Priority: 3 X-MSMail-Priority: Normal X-Mailer: Microsoft Outlook Express 5.00.2615.200 X-MimeOLE: Produced By Microsoft MimeOLE V5.00.2615.200 Message-ID: X-OriginalArrivalTime: 08 Sep 2002 06:23:52.0188 (UTC) FILETIME=[4CEAFBC0:01C25700] Sender: owner-obm-l@sucuri.mat.puc-rio.br Precedence: bulk Reply-To: obm-l@mat.puc-rio.br ----- Original Message ----- From: Nick To: ; Sent: Saturday, September 07, 2002 5:46 PM Subject: Re: [obm-l] Re-probabilidade > Inicialmente fiz o seguinte :1) na situação em que saem os dois primeiros > (1,2)ou(1,3)ou(1,4)ou(1,5) teremos quatro possibilidades em que o inspetor > não prenderá alguém , independente da ordem que saem os outros . 2)na > situação em que os dois primeiros a sairem são (2,3)ou (2,4)ou (2,5) > teremos tres possibilidades em que o inspetor prenderá o mais baixo , > independente de qual ordem saem os outros do edifício.3)Nas situações em > que os dois primeiros a sairem são : (3,4)ou (3,5) teremos um total de > 8 ( 4 para cada) possibilidades em o inspetor prenderá alguém , 1 ou 2. > 4)na situação (4,5) teremos tres casos em o inspetor prenderá alguém . > Como há entre todos esses casos 8 possibilidades em em ele prenderá o > mais baixo ,teremos como probabildade 8/18 . Ou será que na situação > (1) deverei contar 24 casos ? Estopu errado ? > > []´s Nick > > > Olá Nick, eu acho que sua etapa 1 esta correta, vc começou a se complicar a partir do número 3 (ou talvez eu não tenha entendido) e outro problema foi que vc procurou encontrar as diversas possibilidades e esqueceu de multiplicar por suas probabilidades, consulte a resposta do Rafael Baião que esta dentro dessa linha de raciocínio e é bonita, bom eu procurei fazer por arranjo, a solução fica feia mas ai vai: Bom P(p/n) = probabilidade do chefe ser preso dado que ele estava na posição n. P(p/1) = 0 P(p/2) = 0 P(p/3) = 1 P(p/4) = 1 - 3!/4! (P(2 preso sendo q 1 esta na 4º)) - 2/4! (P(3 ser preso com 1 na 4º)) = 2/3 P(p/5) = 2. 3!/4! (1 só vai preso estando na 5º posição se 2 tiver passado entre os dois primeiros.) P(p) = P(1).P(p/1) + ... + P(5).P(p/5) = 13/30. Danilo Artigas. > > At 14:59 7/9/2002 -0300, Afemano wrote: > >Acho que o começo está errado por que se sair o 4 e o 5 por exemplo ele > >poderá prender o 1 , o 2 ou o 3... > > > >----- Original Message ----- > >From: "Pacini" > >To: > >Sent: Saturday, September 07, 2002 2:11 PM > >Subject: [obm-l] Re-probabilidade > > > > > > > > > > Olá , > > > Tenho uma dúvida com relação a este problema . Enumerando > > > as alturas por 1,2,3,4 e5 ; o número de maneiras de sair os dois > > > primeiros é C5,2 = 10 . Como o inspetor irá prender o mais baixo que > > > saiu até o momento , o Espaço Amostral será 10 X 3! = 60 ? ; pois > > > se dentre os dois primeiros que sairem estiver o mais baixo , o > > > inspetor não irá prender alguém ou , seja , a minha dúvida é a seguinte > > > : a sequencia 12345 ,12453 por exemplo não deverá entrar duas vezes > > > para o Espaço Amostral . O que vocês acham ? > > > Pacini > > > > > > > > > > > > > > > > > > > > > > > > To: > > > obm-l@mat.puc-rio.br Subject: [obm-l] probabilidade > > > Olá , > > > Poderiam me ajudar no seguinte problema ? > > > Um inspetor sabe que o chefe de 5 bandidos é o mais baixo de todos e que > > > todas as alturas são diferentes . Sabe -se também que eles estarão > > > presentes numa reunião em um edifício . Depois da reunião , os bandidos > >por > > > medida de precaução deixam o edifício em um intervalo de 15 minutos .Como > >o > > > inspetor não sabe qual deles é o mais baixo , decide deixar sair os dois > > > primeiros bandidos , e prender o primeiro dos seguintes que seja mais > >baixo > > > do que os que até esse momento sairam .Qual a probabilidade do inspetor > > > prender a pessoa certa ? > > > []´s Nick > > > > > > > > > ========================================================================= > > > Instruções para entrar na lista, sair da lista e usar a lista em > > > http://www.mat.puc-rio.br/~nicolau/olimp/obm-l.html > > > O administrador desta lista é > > > ========================================================================= > > > > > > >========================================================================= > >Instruções para entrar na lista, sair da lista e usar a lista em > >http://www.mat.puc-rio.br/~nicolau/olimp/obm-l.html > >O administrador desta lista é > >========================================================================= > > > ========================================================================= > Instruções para entrar na lista, sair da lista e usar a lista em > http://www.mat.puc-rio.br/~nicolau/olimp/obm-l.html > O administrador desta lista é > ========================================================================= > ========================================================================= Instruções para entrar na lista, sair da lista e usar a lista em http://www.mat.puc-rio.br/~nicolau/olimp/obm-l.html O administrador desta lista é ========================================================================= From owner-obm-l@sucuri.mat.puc-rio.br Sun Sep 8 04:31:05 2002 Return-Path: Received: (from majordom@localhost) by sucuri.mat.puc-rio.br (8.9.3/8.9.3) id EAA30164 for obm-l-MTTP; Sun, 8 Sep 2002 04:30:52 -0300 Received: from traven.uol.com.br (traven.uol.com.br [200.221.4.39]) by sucuri.mat.puc-rio.br (8.9.3/8.9.3) with ESMTP id EAA30160 for ; Sun, 8 Sep 2002 04:30:49 -0300 Received: from ui.uol.com.br ([200.160.246.241]) by traven.uol.com.br (8.9.1/8.9.1) with ESMTP id EAA04203 for ; Sun, 8 Sep 2002 04:19:00 -0300 (BRT) Message-Id: <5.1.0.14.2.20020908042615.01dafc58@pop3.uol.com.br> X-Sender: paciniv@uol.com.br@pop3.uol.com.br (Unverified) X-Mailer: QUALCOMM Windows Eudora Version 5.1 Date: Sun, 08 Sep 2002 04:30:15 -0300 To: obm-l@mat.puc-rio.br From: Pacini Subject: [obm-l] Re-probabilidade Mime-Version: 1.0 Content-Type: multipart/alternative; boundary="=====================_3818110==_.ALT" Sender: owner-obm-l@sucuri.mat.puc-rio.br Precedence: bulk Reply-To: obm-l@mat.puc-rio.br --=====================_3818110==_.ALT Content-Type: text/plain; charset="iso-8859-1"; format=flowed Content-Transfer-Encoding: quoted-printable O interessante =E9 que se considerarmos o Espa=E7o Amostral como=20 sendo 10X3! =3D60 e contarmos os casos em que ele prender=E1 o=20 mais baixo (repetidos ou n=E3o) , teremos um total de 26 casos = favor=E1veis=20 ; ou seja p =3D26/60 =3D 13/30 ; a mesma resposta que o Rafael = encontrou=20 . Podemos sempre fazer isto ? []=B4s Pacini At 14:11 7/9/2002 -0300, Pacini wrote: Ol=E1 , Tenho uma d=FAvida com rela=E7=E3o a este problema . Enumerando=20 as alturas por 1,2,3,4 e5 ; o n=FAmero de maneiras de sair os dois=20 primeiros =E9 C5,2 =3D 10 . Como o inspetor ir=E1 prender o mais baixo = que=20 saiu at=E9 o momento , o Espa=E7o Amostral ser=E1 10 X 3! =3D 60 ? ; pois= =20 se dentre os dois primeiros que sairem estiver o mais baixo , o=20 inspetor n=E3o ir=E1 prender algu=E9m ou , seja , a minha d=FAvida =E9 a= seguinte=20 : a sequencia 12345 ,12453 por exemplo n=E3o dever=E1 entrar duas vezes= =20 para o Espa=E7o Amostral . O que voc=EAs acham ? Pacini --=====================_3818110==_.ALT Content-Type: text/html; charset="iso-8859-1" Content-Transfer-Encoding: quoted-printable O  interessante  =E9  que  se considerarmos  o Espa=E7o  Amostral  como sendo  10X3! =3D60   e contarmos  os  casos  em que  ele  prender=E1  o mais  baixo (repetidos  ou n=E3o) , teremos  um total de 26 casos  favor=E1veis ; ou  seja  p =3D26/60 =3D 13/30 ;  a mesma  resposta  que o Rafael  encontrou . Podemos sempre  fazer isto ? 



[]=B4s  Pacini




At 14:11 7/9/2002 -0300, Pacini wrote:

Ol=E1 ,
Tenho  uma d=FAvida  com rela=E7=E3o  a este problema . Enumerando as  alturas  por 1,2,3,4 e5 ; o n=FAmero  de maneiras de sair os dois primeiros  =E9 C5,2 =3D 10 . Como  o inspetor ir=E1   prender o mais  baixo  que saiu  at=E9 o momento , o Espa=E7o  Amostral ser=E1  10 X 3! =3D 60 ? ; = pois se  dentre os  dois primeiros que sairem estiver  o mais  baixo , o inspetor n=E3o ir=E1  prender algu=E9m ou , seja ,= a minha d=FAvida  =E9  a seguinte : a sequencia  12345 ,12453 por  exemplo n=E3o  dever=E1  entrar  duas vezes para o Espa=E7o  Amostral . O que  voc=EAs  acham  ?
Pacini

--=====================_3818110==_.ALT-- ========================================================================= Instruções para entrar na lista, sair da lista e usar a lista em http://www.mat.puc-rio.br/~nicolau/olimp/obm-l.html O administrador desta lista é ========================================================================= From owner-obm-l@sucuri.mat.puc-rio.br Sun Sep 8 07:51:07 2002 Return-Path: Received: (from majordom@localhost) by sucuri.mat.puc-rio.br (8.9.3/8.9.3) id HAA31427 for obm-l-MTTP; Sun, 8 Sep 2002 07:50:15 -0300 Received: from hotmail.com (oe70.pav0.hotmail.com [64.4.33.212]) by sucuri.mat.puc-rio.br (8.9.3/8.9.3) with ESMTP id HAA31423 for ; Sun, 8 Sep 2002 07:50:11 -0300 Received: from mail pickup service by hotmail.com with Microsoft SMTPSVC; Sun, 8 Sep 2002 03:51:03 -0700 X-Originating-IP: [200.164.79.64] From: "e isso mesmo" To: Subject: [obm-l] =?iso-8859-1?Q?N=FAmeros_rand=F4micos?= Date: Sun, 8 Sep 2002 07:51:00 -0300 MIME-Version: 1.0 X-Mailer: MSN Explorer 7.00.0021.1900 Content-Type: multipart/alternative; boundary="----=_NextPart_001_0000_01C2570C.79213A40" Message-ID: X-OriginalArrivalTime: 08 Sep 2002 10:51:03.0679 (UTC) FILETIME=[A06E60F0:01C25725] Sender: owner-obm-l@sucuri.mat.puc-rio.br Precedence: bulk Reply-To: obm-l@mat.puc-rio.br ------=_NextPart_001_0000_01C2570C.79213A40 Content-Type: text/plain; charset="iso-8859-1" Content-Transfer-Encoding: quoted-printable Amigos, algu=E9m poderia me explicar detalhadamente o que s=E3o n=FAmeros= rand=F4micos e com se constr=F3i esses n=FAmeros? Li isso em algum lugar= sobre loterias. Obrigado V=EDctorAproveite melhor a Web. Fa=E7a o download GR=C1TIS do MSN Explore= r : http://explorer.msn.com.br/intl.asp#po ------=_NextPart_001_0000_01C2570C.79213A40 Content-Type: text/html; charset="iso-8859-1" Content-Transfer-Encoding: quoted-printable
Amigos, algu=E9= m poderia me explicar detalhadamente o que s=E3o n=FAmeros rand=F4micos e= com se constr=F3i esses n=FAmeros? Li isso em algum lugar sobre loterias= .
Obrigado
V=EDctor


Aproveite melhor a Web. Fa=E7a o download GR=C1TIS do MSN Explore= r : http://explorer.ms= n.com.br/intl.asp#po

------=_NextPart_001_0000_01C2570C.79213A40-- ========================================================================= Instruções para entrar na lista, sair da lista e usar a lista em http://www.mat.puc-rio.br/~nicolau/olimp/obm-l.html O administrador desta lista é ========================================================================= From owner-obm-l@sucuri.mat.puc-rio.br Sun Sep 8 10:40:46 2002 Return-Path: Received: (from majordom@localhost) by sucuri.mat.puc-rio.br (8.9.3/8.9.3) id KAA32578 for obm-l-MTTP; Sun, 8 Sep 2002 10:40:37 -0300 Received: from artemis.opendf.com.br (artemis.opengate.com.br [200.181.71.15]) by sucuri.mat.puc-rio.br (8.9.3/8.9.3) with ESMTP id KAA32564 for ; Sun, 8 Sep 2002 10:40:32 -0300 Received: from localhost (localhost [127.0.0.1]) by artemis.opendf.com.br (Postfix) with ESMTP id 3AAAA1CAB9 for ; Sun, 8 Sep 2002 10:40:53 -0300 (BRT) Received: from computer (200-181-89-056-bsace7001.dsl.telebrasilia.net.br [200.181.89.56]) by artemis.opendf.com.br (Postfix) with ESMTP id 9DADA1BF90 for ; Sun, 8 Sep 2002 10:40:48 -0300 (BRT) From: "Artur Costa Steiner" To: Subject: [obm-l] =?iso-8859-1?Q?problemas_interessantes_de_c=E1lculo?= Date: Sun, 8 Sep 2002 10:41:29 -0300 Organization: Steiner Consultoria LTDA Message-ID: <000601c2573d$71c62db0$9865fea9@computer> MIME-Version: 1.0 Content-Type: multipart/alternative; boundary="----=_NextPart_000_0007_01C25724.4C78F5B0" X-Priority: 3 (Normal) X-MSMail-Priority: Normal X-Mailer: Microsoft Outlook, Build 10.0.2627 Importance: Normal In-Reply-To: X-MimeOLE: Produced By Microsoft MimeOLE V6.00.2600.0000 X-Virus-Scanned: by AMaViS new-20020517 Sender: owner-obm-l@sucuri.mat.puc-rio.br Precedence: bulk Reply-To: obm-l@mat.puc-rio.br This is a multi-part message in MIME format. ------=_NextPart_000_0007_01C25724.4C78F5B0 Content-Type: text/plain; charset="iso-8859-1" Content-Transfer-Encoding: quoted-printable =20 =20 Para os que curtem C=E1lculo, estes problemas s=E3o, na minha = opini=E3o, interessantes:=20 =20 1) Seja f uma fun=E7=E3o real, diferenci=E1vel em (0, oo), e seja = a<>0. Suponhamos que lim x=3D> oo a f(x) + f=92(x) =3D L =20 Se a>0, ent=E3o lim x=3D> f(x) =3D L/a e lim x=3D> f=92(x) =3D0 =20 Se a<0 ent=E3o lim x=3D> f(x) =3D L/a e lim x=3D> f=92(x) =3D0 se, e = somente se lim x =3D> e^(a*x) f(x) =3D 0 =20 2) Mostre que se f for diferenci=E1vel em um intervalo I e se f=92 = for monot=F4nica e I, ent=E3o f=92 =E9 cont=EDnua em I =20 Artur =20 =20 ------=_NextPart_000_0007_01C25724.4C78F5B0 Content-Type: text/html; charset="iso-8859-1" Content-Transfer-Encoding: quoted-printable

 

 

Para os que curtem=A0 = C=E1lculo, estes problemas s=E3o, na minha opini=E3o, interessantes: =

 

1)       Seja= f uma fun=E7=E3o real, diferenci=E1vel em (0, oo), e seja a<>0. = Suponhamos que lim x=3D> oo a f(x) + f’(x) =3D L

 

Se a>0, ent=E3o lim x=3D> f(x) =3D L/a e lim x=3D> f’(x) = =3D0

 

Se a<0 ent=E3o lim x=3D> f(x) =3D L/a e lim x=3D> f’(x) =3D0 = se, e somente se lim x=A0 =3D>=A0 e^(a*x) f(x) =3D = 0

 

2)       Most= re que se f for diferenci=E1vel em um intervalo I e se f’ for = monot=F4nica e I, ent=E3o f’ =E9 cont=EDnua em I

 

Artu= r

 

 

------=_NextPart_000_0007_01C25724.4C78F5B0-- ========================================================================= Instruções para entrar na lista, sair da lista e usar a lista em http://www.mat.puc-rio.br/~nicolau/olimp/obm-l.html O administrador desta lista é ========================================================================= From owner-obm-l@sucuri.mat.puc-rio.br Sun Sep 8 10:50:33 2002 Return-Path: Received: (from majordom@localhost) by sucuri.mat.puc-rio.br (8.9.3/8.9.3) id KAA32759 for obm-l-MTTP; Sun, 8 Sep 2002 10:50:32 -0300 Received: from artemis.opendf.com.br (artemis.opengate.com.br [200.181.71.15]) by sucuri.mat.puc-rio.br (8.9.3/8.9.3) with ESMTP id KAA32755 for ; Sun, 8 Sep 2002 10:50:29 -0300 Received: from localhost (localhost [127.0.0.1]) by artemis.opendf.com.br (Postfix) with ESMTP id 8BB7A1CAB9 for ; Sun, 8 Sep 2002 10:50:49 -0300 (BRT) Received: from artur (200-181-89-056-bsace7001.dsl.telebrasilia.net.br [200.181.89.56]) by artemis.opendf.com.br (Postfix) with ESMTP id 52BFE1BF90 for ; Sun, 8 Sep 2002 10:50:41 -0300 (BRT) From: "Artur Costa Steiner" To: , Subject: [obm-l] =?iso-8859-1?Q?RE:_=5Bobm-l=5D_Re:_=5Bobm-l=5D_Uma_prova_simples_para_a_s?= =?iso-8859-1?Q?eguinte_afirma=E7=E3o?= Date: Sun, 8 Sep 2002 10:52:16 -0700 Organization: Steiner Consultoria LTDA Message-ID: <001001c25760$78ca5c80$0b01a8c0@mshome.net> MIME-Version: 1.0 Content-Type: multipart/alternative; boundary="----=_NextPart_000_0011_01C25725.CC6B8480" X-Priority: 3 (Normal) X-MSMail-Priority: Normal X-Mailer: Microsoft Outlook, Build 10.0.2627 X-MimeOLE: Produced By Microsoft MimeOLE V6.00.2600.0000 Importance: Normal In-Reply-To: X-Virus-Scanned: by AMaViS new-20020517 Sender: owner-obm-l@sucuri.mat.puc-rio.br Precedence: bulk Reply-To: obm-l@mat.puc-rio.br This is a multi-part message in MIME format. ------=_NextPart_000_0011_01C25725.CC6B8480 Content-Type: text/plain; charset="iso-8859-1" Content-Transfer-Encoding: quoted-printable -----Original Message----- From: owner-obm-l@sucuri.mat.puc-rio.br [mailto:owner-obm-l@sucuri.mat.puc-rio.br] On Behalf Of Angelo Barone Netto Sent: Friday, September 06, 2002 1:42 PM To: obm-l@mat.puc-rio.br Subject: [obm-l] Re: [obm-l] Uma prova simples para a seguinte = afirma=E7=E3o =20 Obrigado. Uma Linda e simples prova! =20 =20 A demonstra=E7=E3o que eu tinha na cabe=E7a, um tanto mais complicada = que a sua, era a seguinte: escolha uma base numer=E1vel {Bk} de Rn (sabemos = que esta base certamente existe) e defina W como a uni=E3o de todos os Bk = que contenham um n=FAmero apenas finito de elementos de A. Temos ent=E3o que = W inter A =E9 numer=E1vel. Podemos facilmente mostrar que W =E9 o = complementar de D, sendo D o conjunto dos pontos de acumula=E7=E3o de A. Logo, para qualquer subconjunto A de Rn, o conjunto dos elementos de A que n=E3o = s=E3o pontos de acumula=E7=E3o do mesmo =E9 numer=E1vel. Segue-se que, se A = n=E3o contiver pontos de acumula=E7=E3o, ent=E3o A =E9 numer=E1vel.=20 =20 Um aspecto interessante =E9 que uma prova similar vale para pontos de condensa=E7=E3o. Se P =E9 o conjunto dos pontos de condensa=E7=E3o de A, = ent=E3o A inter (complementar de P) =E9 numer=E1vel. Se A n=E3o tem pontos de condensa=E7=E3o, ent=E3o=20 A =E9 numer=E1vel. =20 No caso de pontos de condensa=E7=E3o, a prova vale em qualquer espa=E7o topol=F3gico que possua uma base numer=E1vel (caso dos espa=E7os = m=E9tricos separ=E1veis). No caso de pontos de acumula=E7=E3o, creio que s=F3 vale = se, al=E9m de separ=E1vel, o espa=E7o for de Hausdorff, pois, caso contr=E1rio, vizinhan=E7as de pontos de acumula=E7=E3o de um conjunto podem conter um = n=FAmro finito de elementos do conjunto.=20 =20 Um abra=E7o! Artur=20 =20 Caro Artur. Para cada ponto de A tome um aberto que so encontra A nesse ponto. Em cada um dos abertos tome um ponto com todas as coordenadas racionais. Pronto. Ja de enumeravel. =20 Angelo Barone{\ --\ }Netto Universidade de Sao Paulo Departamento de Matematica Aplicada Instituto de Matematica e Estatistica Rua do Matao, 1010 Butanta - Cidade Universitaria Caixa Postal 66 281 phone +55-11-3091-6162/6224/6136 05311-970 - Sao Paulo - SP fax +55-11-3091-6131 Agencia Cidade de Sao Paulo . =20 =20 =20 =20 =20 =20 =20 =20 =20 =20 =3D=3D=3D=3D=3D=3D=3D=3D=3D=3D=3D=3D=3D=3D=3D=3D=3D=3D=3D=3D=3D=3D=3D=3D=3D= =3D=3D=3D=3D=3D=3D=3D=3D=3D=3D=3D=3D=3D=3D=3D=3D=3D=3D=3D=3D=3D=3D=3D=3D=3D= =3D=3D=3D=3D=3D=3D=3D=3D=3D=3D=3D=3D=3D=3D=3D=3D=3D=3D=3D=3D=3D=3D =3D Instru=E7=F5es para entrar na lista, sair da lista e usar a lista em http://www.mat.puc-rio.br/~nicolau/olimp/obm-l.html O administrador desta lista =E9 =3D=3D=3D=3D=3D=3D=3D=3D=3D=3D=3D=3D=3D=3D=3D=3D=3D=3D=3D=3D=3D=3D=3D=3D=3D= =3D=3D=3D=3D=3D=3D=3D=3D=3D=3D=3D=3D=3D=3D=3D=3D=3D=3D=3D=3D=3D=3D=3D=3D=3D= =3D=3D=3D=3D=3D=3D=3D=3D=3D=3D=3D=3D=3D=3D=3D=3D=3D=3D=3D=3D=3D=3D =3D ------=_NextPart_000_0011_01C25725.CC6B8480 Content-Type: text/html; charset="iso-8859-1" Content-Transfer-Encoding: quoted-printable

-----Original Message-----

From: owner-obm-l@sucuri.mat.puc-rio.br [mailto:owner-obm-l@sucuri.mat.puc-rio.br] On Behalf Of Angelo Barone = Netto

Sent: Friday, September 06, 2002 1:42 = PM

To: obm-l@mat.puc-rio.br

Subject: [obm-l] Re: [obm-l] Uma prova simples para a seguinte afirma=E7=E3o

 

Obrigado. Uma Linda e = simples prova!

 

 

A demonstra=E7=E3o que eu tinha na cabe=E7a, um tanto mais complicada que a sua, era a = seguinte: escolha uma base numer=E1vel {Bk} de Rn (sabemos que esta base certamente = existe) e defina W como a uni=E3o de todos os Bk que contenham um n=FAmero apenas = finito de elementos de A. Temos ent=E3o que W inter A =E9 numer=E1vel. Podemos = facilmente mostrar que W =E9 o complementar de D, sendo D o conjunto dos pontos de acumula=E7=E3o de A. Logo, para qualquer subconjunto A de Rn, o conjunto = dos elementos de A que n=E3o s=E3o pontos de acumula=E7=E3o do mesmo =E9 = numer=E1vel. Segue-se que, se A n=E3o contiver pontos de acumula=E7=E3o, ent=E3o A =E9 = numer=E1vel.

 

Um aspecto interessante =E9 que uma prova similar vale para = pontos de condensa=E7=E3o. Se P =E9 o conjunto dos pontos de condensa=E7=E3o de A, = ent=E3o A inter (complementar de P) =E9 numer=E1vel. Se A n=E3o tem pontos de = condensa=E7=E3o, ent=E3o

A =E9 numer=E1vel.

 

No caso de pontos de condensa=E7=E3o, a prova vale em qualquer = espa=E7o topol=F3gico que possua uma base numer=E1vel (caso dos espa=E7os = m=E9tricos separ=E1veis). No caso de pontos de acumula=E7=E3o, creio que s=F3 vale = se, al=E9m de separ=E1vel, o espa=E7o for de Hausdorff, pois, caso contr=E1rio, = vizinhan=E7as de pontos de acumula=E7=E3o de um conjunto podem conter um n=FAmro finito = de elementos do conjunto.

 

Um abra=E7o!

Artur

 

Caro Artur.

Para cada ponto de A tome um aberto que so encontra A nesse = ponto.

Em cada um dos abertos tome um ponto com todas as coordenadas racionais.

Pronto. Ja de enumeravel.

 

Angelo Barone{\ --\ }Netto=A0=A0=A0=A0=A0=A0=A0=A0=A0=A0 Universidade de Sao Paulo

Departamento de Matematica Aplicada=A0 Instituto de Matematica e = Estatistica

Rua do Matao, 1010=A0=A0=A0=A0=A0=A0=A0=A0=A0=A0=A0=A0=A0=A0=A0=A0= =A0=A0 Butanta - Cidade Universitaria

Caixa Postal 66 281=A0=A0=A0=A0=A0=A0=A0=A0=A0=A0=A0=A0=A0=A0=A0=A0= =A0 phone +55-11-3091-6162/6224/6136

05311-970 - Sao Paulo - SP=A0=A0=A0=A0=A0=A0=A0=A0=A0=A0 fax +55-11-3091-6131

Agencia Cidade de Sao Paulo

.

 

 

 

 

 

 

 

 

 

 

=3D=3D=3D=3D=3D=3D=3D=3D=3D=3D=3D=3D=3D=3D=3D=3D=3D=3D=3D=3D=3D=3D= =3D=3D=3D=3D=3D=3D=3D=3D=3D=3D=3D=3D=3D=3D=3D=3D=3D=3D=3D=3D=3D=3D=3D=3D=3D= =3D=3D=3D=3D=3D=3D=3D=3D=3D=3D=3D=3D=3D=3D=3D=3D=3D=3D=3D=3D=3D=3D=3D=3D=3D= =3D

Instru=E7=F5es para entrar na lista, sair da lista e usar a = lista em

http://www.mat.puc-rio.br/~nicolau/olimp/obm-l.html

O administrador desta lista =E9 = <nicolau@mat.puc-rio.br>

=3D=3D=3D=3D=3D=3D=3D=3D=3D=3D=3D=3D=3D=3D=3D=3D=3D=3D=3D=3D=3D=3D= =3D=3D=3D=3D=3D=3D=3D=3D=3D=3D=3D=3D=3D=3D=3D=3D=3D=3D=3D=3D=3D=3D=3D=3D=3D= =3D=3D=3D=3D=3D=3D=3D=3D=3D=3D=3D=3D=3D=3D=3D=3D=3D=3D=3D=3D=3D=3D=3D=3D=3D= =3D

------=_NextPart_000_0011_01C25725.CC6B8480-- ========================================================================= Instruções para entrar na lista, sair da lista e usar a lista em http://www.mat.puc-rio.br/~nicolau/olimp/obm-l.html O administrador desta lista é ========================================================================= From owner-obm-l@sucuri.mat.puc-rio.br Sun Sep 8 11:08:38 2002 Return-Path: Received: (from majordom@localhost) by sucuri.mat.puc-rio.br (8.9.3/8.9.3) id LAA01051 for obm-l-MTTP; Sun, 8 Sep 2002 11:08:19 -0300 Received: from puma.unisys.com.br (smtp.unisys.com.br [200.220.64.7]) by sucuri.mat.puc-rio.br (8.9.3/8.9.3) with ESMTP id LAA01041 for ; Sun, 8 Sep 2002 11:08:15 -0300 Received: from jf (riopm18p128.uninet.com.br [200.220.16.128]) by puma.unisys.com.br (8.12.3/8.12.3) with SMTP id g88E97jR025738 for ; Sun, 8 Sep 2002 11:09:08 -0300 (EST) X-Spam-Filter: check_local@puma.unisys.com.br by digitalanswers.org Message-ID: <00b601c25741$f5756f00$8010dcc8@jf> From: "Jose Francisco Guimaraes Costa" To: "obm-l" Subject: [obm-l] alguem conhece este livro? Date: Sun, 8 Sep 2002 11:13:50 -0300 MIME-Version: 1.0 Content-Type: multipart/alternative; boundary="----=_NextPart_000_00B3_01C25728.CF466280" X-Priority: 3 X-MSMail-Priority: Normal X-Mailer: Microsoft Outlook Express 6.00.2600.0000 X-MimeOLE: Produced By Microsoft MimeOLE V6.00.2600.0000 Sender: owner-obm-l@sucuri.mat.puc-rio.br Precedence: bulk Reply-To: obm-l@mat.puc-rio.br This is a multi-part message in MIME format. ------=_NextPart_000_00B3_01C25728.CF466280 Content-Type: text/plain; charset="iso-8859-1" Content-Transfer-Encoding: quoted-printable Algum de VV conhece o livro Famous Problems of Geometry and How to Solve = Them (Benjamin Bold, Dover, 1982)? Eu tenho, e acho =F3timo, o 100 Great Problems of Elementary Mathematics = (D=F6rrie, Dover, 1965). H=E1 como compar=E1-los? JF ------=_NextPart_000_00B3_01C25728.CF466280 Content-Type: text/html; charset="iso-8859-1" Content-Transfer-Encoding: quoted-printable
Algum de VV conhece o livro = Famous Problems of Geometry and How to Solve = Them=20 (Benjamin Bold, Dover, 1982)?
 
Eu tenho, e acho =F3timo, o 100 = Great Problems=20 of Elementary Mathematics (D=F6rrie, Dover, 1965). H=E1 como=20 compar=E1-los?
 
JF


------=_NextPart_000_00B3_01C25728.CF466280-- ========================================================================= Instruções para entrar na lista, sair da lista e usar a lista em http://www.mat.puc-rio.br/~nicolau/olimp/obm-l.html O administrador desta lista é ========================================================================= From owner-obm-l@sucuri.mat.puc-rio.br Sun Sep 8 11:13:07 2002 Return-Path: Received: (from majordom@localhost) by sucuri.mat.puc-rio.br (8.9.3/8.9.3) id LAA01206 for obm-l-MTTP; Sun, 8 Sep 2002 11:13:05 -0300 Received: from artemis.opendf.com.br (artemis.opengate.com.br [200.181.71.15]) by sucuri.mat.puc-rio.br (8.9.3/8.9.3) with ESMTP id LAA01201 for ; Sun, 8 Sep 2002 11:13:02 -0300 Received: from localhost (localhost [127.0.0.1]) by artemis.opendf.com.br (Postfix) with ESMTP id 69CF31CAB9 for ; Sun, 8 Sep 2002 11:13:22 -0300 (BRT) Received: from artur (200-181-89-056-bsace7001.dsl.telebrasilia.net.br [200.181.89.56]) by artemis.opendf.com.br (Postfix) with ESMTP id 1DBD71BF90 for ; Sun, 8 Sep 2002 11:13:17 -0300 (BRT) From: "Artur Costa Steiner" To: Subject: [obm-l] =?iso-8859-1?Q?RE:_=5Bobm-l=5D_Re:_=5Bobm-l=5D_Uma_prova_simples_para_a_s?= =?iso-8859-1?Q?eguinte_afirma=E7=E3o?= Date: Sun, 8 Sep 2002 11:14:53 -0700 Organization: Steiner Consultoria LTDA Message-ID: <002a01c25763$a110b240$0b01a8c0@mshome.net> MIME-Version: 1.0 Content-Type: multipart/alternative; boundary="----=_NextPart_000_002B_01C25728.F4B1DA40" X-Priority: 3 (Normal) X-MSMail-Priority: Normal X-Mailer: Microsoft Outlook, Build 10.0.2627 X-MimeOLE: Produced By Microsoft MimeOLE V6.00.2600.0000 Importance: Normal In-Reply-To: X-Virus-Scanned: by AMaViS new-20020517 Sender: owner-obm-l@sucuri.mat.puc-rio.br Precedence: bulk Reply-To: obm-l@mat.puc-rio.br This is a multi-part message in MIME format. ------=_NextPart_000_002B_01C25728.F4B1DA40 Content-Type: text/plain; charset="iso-8859-1" Content-Transfer-Encoding: quoted-printable =20 =20 Obrigado. Uma Linda e simples prova! =20 =20 A demonstra=E7=E3o que eu tinha na cabe=E7a, um tanto mais complicada = que a sua, era a seguinte: escolha uma base numer=E1vel {Bk} de Rn (sabemos = que esta base certamente existe) e defina W como a uni=E3o de todos os Bk = que contenham um n=FAmero apenas finito de elementos de A. Temos ent=E3o que = W inter A =E9 numer=E1vel. Podemos facilmente mostrar que W =E9 o = complementar de D, sendo D o conjunto dos pontos de acumula=E7=E3o de A. Logo, para qualquer subconjunto A de Rn, o conjunto dos elementos de A que n=E3o = s=E3o pontos de acumula=E7=E3o do mesmo =E9 numer=E1vel. Segue-se que, se A = n=E3o contiver pontos de acumula=E7=E3o, ent=E3o A =E9 numer=E1vel.=20 =20 Um aspecto interessante =E9 que uma prova similar vale para pontos de condensa=E7=E3o. Se P =E9 o conjunto dos pontos de condensa=E7=E3o de A, = ent=E3o A inter (complementar de P) =E9 numer=E1vel. Se A n=E3o tem pontos de condensa=E7=E3o, ent=E3o=20 A =E9 numer=E1vel. =20 No caso de pontos de condensa=E7=E3o, a prova vale em qualquer espa=E7o topol=F3gico que possua uma base numer=E1vel (caso dos espa=E7os = m=E9tricos separ=E1veis). No caso de pontos de acumula=E7=E3o, creio que s=F3 vale = se, al=E9m de separ=E1vel, o espa=E7o for de Hausdorff, pois, caso contr=E1rio, vizinhan=E7as de pontos de acumula=E7=E3o de um conjunto podem conter um = n=FAmro finito de elementos do conjunto.=20 =20 Um abra=E7o! Artur=20 =20 Caro Artur. Para cada ponto de A tome um aberto que so encontra A nesse ponto. Em cada um dos abertos tome um ponto com todas as coordenadas racionais. Pronto. Ja de enumeravel. =20 Angelo Barone{\ --\ }Netto Universidade de Sao Paulo Departamento de Matematica Aplicada Instituto de Matematica e Estatistica Rua do Matao, 1010 Butanta - Cidade Universitaria Caixa Postal 66 281 phone +55-11-3091-6162/6224/6136 05311-970 - Sao Paulo - SP fax +55-11-3091-6131 Agencia Cidade de Sao Paulo . =20 =20 =20 =20 =20 =20 =20 =20 =20 =20 =3D=3D=3D=3D=3D=3D=3D=3D=3D=3D=3D=3D=3D=3D=3D=3D=3D=3D=3D=3D=3D=3D=3D=3D=3D= =3D=3D=3D=3D=3D=3D=3D=3D=3D=3D=3D=3D=3D=3D=3D=3D=3D=3D=3D=3D=3D=3D=3D=3D=3D= =3D=3D=3D=3D=3D=3D=3D=3D=3D=3D=3D=3D=3D=3D=3D=3D=3D=3D=3D=3D=3D=3D =3D Instru=E7=F5es para entrar na lista, sair da lista e usar a lista em http://www.mat.puc-rio.br/~nicolau/olimp/obm-l.html O administrador desta lista =E9 =3D=3D=3D=3D=3D=3D=3D=3D=3D=3D=3D=3D=3D=3D=3D=3D=3D=3D=3D=3D=3D=3D=3D=3D=3D= =3D=3D=3D=3D=3D=3D=3D=3D=3D=3D=3D=3D=3D=3D=3D=3D=3D=3D=3D=3D=3D=3D=3D=3D=3D= =3D=3D=3D=3D=3D=3D=3D=3D=3D=3D=3D=3D=3D=3D=3D=3D=3D=3D=3D=3D=3D=3D =3D ------=_NextPart_000_002B_01C25728.F4B1DA40 Content-Type: text/html; charset="iso-8859-1" Content-Transfer-Encoding: quoted-printable

 

 

Obrigado. Uma Linda e simples prova!

 

 

A demonstra=E7=E3o que eu tinha na cabe=E7a, = um tanto mais complicada que a sua, era a seguinte: escolha uma base numer=E1vel {Bk} = de Rn (sabemos que esta base certamente existe) e defina W como a uni=E3o de = todos os Bk que contenham um n=FAmero apenas finito de elementos de A. Temos = ent=E3o que W inter A =E9 numer=E1vel. Podemos facilmente mostrar que W =E9 o = complementar de D, sendo D o conjunto dos pontos de acumula=E7=E3o de A. Logo, para = qualquer subconjunto A de Rn, o conjunto dos elementos de A que n=E3o s=E3o = pontos de acumula=E7=E3o do mesmo =E9 numer=E1vel. Segue-se que, se A n=E3o = contiver pontos de acumula=E7=E3o, ent=E3o A =E9 numer=E1vel.

 

Um aspecto interessante =E9 que uma prova = similar vale para pontos de condensa=E7=E3o. Se P =E9 o conjunto dos pontos de = condensa=E7=E3o de A, ent=E3o A inter (complementar de P) =E9 numer=E1vel. Se A n=E3o tem = pontos de condensa=E7=E3o, ent=E3o

A =E9 = numer=E1vel.

 

No caso de pontos de condensa=E7=E3o, a prova = vale em qualquer espa=E7o topol=F3gico que possua uma base numer=E1vel (caso dos = espa=E7os m=E9tricos separ=E1veis). No caso de pontos de acumula=E7=E3o, creio que = s=F3 vale se, al=E9m de separ=E1vel, o espa=E7o for de Hausdorff, pois, caso = contr=E1rio, vizinhan=E7as de pontos de acumula=E7=E3o de um conjunto podem conter um n=FAmro = finito de elementos do conjunto.

 

Um abra=E7o!

Artur

 

Caro Artur.

Para cada ponto de A tome um aberto que so encontra A nesse = ponto.

Em cada um dos abertos tome um ponto com = todas as coordenadas racionais.

Pronto. Ja de = enumeravel.

 

Angelo Barone{\ --\ }Netto=A0=A0=A0=A0=A0=A0=A0=A0=A0=A0 = Universidade de Sao Paulo

Departamento de Matematica Aplicada=A0 Instituto de Matematica e = Estatistica

Rua do Matao, 1010=A0=A0=A0=A0=A0=A0=A0=A0=A0=A0=A0=A0=A0=A0=A0=A0= =A0=A0 Butanta - Cidade Universitaria

Caixa Postal 66 281=A0=A0=A0=A0=A0=A0=A0=A0=A0=A0=A0=A0=A0=A0=A0=A0= =A0 phone +55-11-3091-6162/6224/6136

05311-970 - = Sao Paulo - SP=A0=A0=A0=A0=A0=A0=A0=A0=A0=A0 = fax +55-11-3091-6131

Agencia Cidade de Sao = Paulo

.

 

 

 

 

 

 

 

 

 

 

=3D=3D=3D=3D=3D=3D=3D=3D=3D=3D=3D=3D=3D=3D=3D=3D= =3D=3D=3D=3D=3D=3D=3D=3D=3D=3D=3D=3D=3D=3D=3D=3D=3D=3D=3D=3D=3D=3D=3D=3D=3D= =3D=3D=3D=3D=3D=3D=3D=3D=3D=3D=3D=3D=3D=3D=3D=3D=3D=3D=3D=3D=3D=3D=3D=3D=3D= =3D=3D=3D=3D=3D=3D=3D

Instru=E7=F5es para entrar na lista, sair da = lista e usar a lista em

http://www.mat.puc-rio.br/~nicolau/olimp/obm-l= .html

O administrador desta lista =E9 <nicolau@mat.puc-rio.br>

=3D=3D=3D=3D=3D=3D=3D=3D=3D=3D=3D=3D=3D=3D=3D=3D= =3D=3D=3D=3D=3D=3D=3D=3D=3D=3D=3D=3D=3D=3D=3D=3D=3D=3D=3D=3D=3D=3D=3D=3D=3D= =3D=3D=3D=3D=3D=3D=3D=3D=3D=3D=3D=3D=3D=3D=3D=3D=3D=3D=3D=3D=3D=3D=3D=3D=3D= =3D=3D=3D=3D=3D=3D=3D

------=_NextPart_000_002B_01C25728.F4B1DA40-- ========================================================================= Instruções para entrar na lista, sair da lista e usar a lista em http://www.mat.puc-rio.br/~nicolau/olimp/obm-l.html O administrador desta lista é ========================================================================= From owner-obm-l@sucuri.mat.puc-rio.br Sun Sep 8 11:24:12 2002 Return-Path: Received: (from majordom@localhost) by sucuri.mat.puc-rio.br (8.9.3/8.9.3) id LAA01870 for obm-l-MTTP; Sun, 8 Sep 2002 11:24:05 -0300 Received: from artemis.opendf.com.br (artemis.opengate.com.br [200.181.71.15]) by sucuri.mat.puc-rio.br (8.9.3/8.9.3) with ESMTP id LAA01860 for ; Sun, 8 Sep 2002 11:24:00 -0300 Received: from localhost (localhost [127.0.0.1]) by artemis.opendf.com.br (Postfix) with ESMTP id 3F7571CAB9 for ; Sun, 8 Sep 2002 11:24:21 -0300 (BRT) Received: from computer (200-181-89-056-bsace7001.dsl.telebrasilia.net.br [200.181.89.56]) by artemis.opendf.com.br (Postfix) with ESMTP id CC8321BF90 for ; Sun, 8 Sep 2002 11:24:15 -0300 (BRT) From: "Artur Costa Steiner" To: Subject: RE: [obm-l] violencia Date: Sun, 8 Sep 2002 11:24:56 -0300 Organization: Steiner Consultoria LTDA Message-ID: <000001c25743$83c31220$9865fea9@computer> MIME-Version: 1.0 Content-Type: multipart/alternative; boundary="----=_NextPart_000_0001_01C2572A.5E75DA20" X-Priority: 3 (Normal) X-MSMail-Priority: Normal X-Mailer: Microsoft Outlook, Build 10.0.2627 Importance: Normal In-Reply-To: X-MimeOLE: Produced By Microsoft MimeOLE V6.00.2600.0000 X-Virus-Scanned: by AMaViS new-20020517 Sender: owner-obm-l@sucuri.mat.puc-rio.br Precedence: bulk Reply-To: obm-l@mat.puc-rio.br This is a multi-part message in MIME format. ------=_NextPart_000_0001_01C2572A.5E75DA20 Content-Type: text/plain; charset="iso-8859-1" Content-Transfer-Encoding: quoted-printable Bom, com rela=E7=E3o =E0 primeira quest=E3o. Comecemos pela segunda = parte e suponhamos, conforme vc disse, que cada bandido tenha um n=FAmero finito de inimigos. Vou supor que, embora variando com o bandido, este n=FAmero = =E9 conhecido para cada bandido. Escolha um bandido. Isto =E9 poss=EDvel pois h=E1 infinitos (Meu Deus!!) = Dado que este bandido tem um n=FAmero finito de inimigos e existem infinitos bandidos, podemos escolher um bandido que n=E3o seja inimigo dele. Logo, = a base de nosso processo indutivo est=E1 formada. Suponhamos agora que, para algum natural n, tenhamos escolhido n bandidos tais que ningu=E9m seja inimigo de ningu=E9m. Suponhamos. Ora, = cada um destes n bandidos tem um n=FAmero apenas finito de inimigos. Logo, o n=FAmero total de bandidos que s=E3o inimigos de um destes n bandidos escolhidos =E9 finito - pois =E9 a soma de n parcelas finitas..=20 Mas, temos infinitos bandidos, de modo que podemos escolher um que n=E3o seja inimigo de nenhum dos n que j=E1 escolhemos (e, =E9 claro, que = n=E3o seja um membro deste conjunto de n que escolhemos). Com isto, obtemos n +1 bandidos distintos tais que, neste conjunto, ningu=E9m vai matar ningu=E9m.=20 A "ponte" de nosso processo indutivo est=E1 portanto formada, e mostra = que nosso processo de escolha pode prosseguir indefinidamente. OK? Podemos dizer que geramos um seq=FC=EAncia B_n de bandidos tal que, dados = quaisquer naturais m e n, com m<>n, ent=E3o B_n n=E3o quer matar B_m. Estou = assumindo, implicitamente, que ningu=E9m quer cometer suic=EDdio.=20 A primeira parte de sua 1a quest=E3o =E9 um caso particular da segunda, obtida quando cada bandido tem apenas um inimigo. Espero ter ajudado Artur Artur Costa Steiner SHCGN 705 Bloco P Ap 506 Bras=EDlia - DF Cep 70730-776 61 340-9788 61 913-3745 61 9987-0709 -----Original Message----- From: owner-obm-l@sucuri.mat.puc-rio.br [mailto:owner-obm-l@sucuri.mat.puc-rio.br] On Behalf Of Fernanda Medeiros Sent: Saturday, September 07, 2002 8:46 PM To: obm-l@mat.puc-rio.br Subject: [obm-l] violencia=20 Ol=E1, algu=E9m pode dar uma ajuda nestas quest=F5es? 1.a)uma "gang" tem infinitos bandidos e cada um dos meliantes tem um =FAnico=20 inimigo no interior da "gang",que ele quer matar.Prove q =E9 possivel reunir=20 uma quantidade infinita de bandidos desta "gang", semq haja o risco de q=20 um bandido mate outro durante a reuni=E3o. b)Se cada bandido tiver um n=BA finito mas indefinido de inimigos(um bandido=20 pode ter 2 inimigos, outro somente 1, um terceiro pode ter 20 e assim por=20 diante).Ser=E1 sempre possivel promover uma reuni=E3o com infinitos = bandidos sem=20 risco de derramamento de sangue? 2.Encontre todas as solu=E7=F5es em inteiros n=E3o negativos para: 2^x +3^y =3Dz^2 3.Encontre todos os inteiros positivos (x,y) tais q 7^x - 3^y =3D4 Valeu!! F=CA _________________________________________________________________ Converse com seus amigos online, fa=E7a o download gr=E1tis do MSN Messenger:=20 http://messenger.msn.com.br =3D=3D=3D=3D=3D=3D=3D=3D=3D=3D=3D=3D=3D=3D=3D=3D=3D=3D=3D=3D=3D=3D=3D=3D=3D= =3D=3D=3D=3D=3D=3D=3D=3D=3D=3D=3D=3D=3D=3D=3D=3D=3D=3D=3D=3D=3D=3D=3D=3D=3D= =3D=3D=3D=3D=3D=3D=3D=3D=3D=3D=3D=3D=3D=3D=3D=3D=3D=3D=3D=3D=3D=3D =3D Instru=E7=F5es para entrar na lista, sair da lista e usar a lista em http://www.mat.puc-rio.br/~nicolau/olimp/obm-l.html O administrador desta lista =E9 =3D=3D=3D=3D=3D=3D=3D=3D=3D=3D=3D=3D=3D=3D=3D=3D=3D=3D=3D=3D=3D=3D=3D=3D=3D= =3D=3D=3D=3D=3D=3D=3D=3D=3D=3D=3D=3D=3D=3D=3D=3D=3D=3D=3D=3D=3D=3D=3D=3D=3D= =3D=3D=3D=3D=3D=3D=3D=3D=3D=3D=3D=3D=3D=3D=3D=3D=3D=3D=3D=3D=3D=3D =3D ------=_NextPart_000_0001_01C2572A.5E75DA20 Content-Type: text/html; charset="iso-8859-1" Content-Transfer-Encoding: quoted-printable RE: [obm-l] violencia

Bom, com rela=E7=E3o =E0 primeira quest=E3o. Comecemos pela segunda = parte e suponhamos, conforme vc disse, que cada bandido tenha um = n=FAmero finito de inimigos. Vou supor que, embora variando com o = bandido, este n=FAmero =E9 conhecido para cada = bandido.

Escolha um bandido. Isto =E9 poss=EDvel pois h=E1 infinitos (Meu = Deus!!) Dado que este bandido tem um n=FAmero finito de inimigos e = existem infinitos bandidos, podemos escolher um bandido que n=E3o seja = inimigo dele. Logo, a base de nosso processo indutivo est=E1 = formada.

Suponhamos agora que, para algum natural n, tenhamos escolhido n = bandidos tais que ningu=E9m seja inimigo de ningu=E9m. Suponhamos. Ora, = cada um destes n bandidos tem um n=FAmero apenas finito de inimigos. = Logo, o n=FAmero total de bandidos que s=E3o inimigos de um destes n = bandidos escolhidos =E9 finito - pois =E9 a soma de n parcelas finitas.. =

Mas, temos infinitos bandidos, de modo que podemos escolher um que = n=E3o seja inimigo de nenhum dos n que j=E1 = escolhemos (e, =E9 claro, = que  n=E3o seja um = membro = deste conjunto de n que escolhemos).  Com isto, = obtemos n +1 bandidos distintos = tais que, neste conjunto, ningu=E9m = vai matar ningu=E9m.

A "ponte" de nosso processo indutivo est=E1 portanto = formada, e mostra que nosso processo de escolha pode prosseguir = indefinidamente. OK? Podemos dizer que geramos um seq=FC=EAncia B_n de = bandidos tal que, dados quaisquer naturais m e n, com m<>n, = ent=E3o B_n n=E3o quer matar B_m. Estou assumindo, implicitamente, que = ningu=E9m quer cometer suic=EDdio.

A primeira parte de sua 1a quest=E3o =E9 um caso particular da = segunda, obtida quando cada bandido tem apenas um = inimigo.

Espero ter ajudado

Artur



Artur Costa = Steiner

SHCGN 705 Bloco P Ap 506

Bras=EDlia - DF

Cep 70730-776

61 340-9788

61 913-3745

61 9987-0709

-----Original Message-----
From: owner-obm-l@sucuri.mat.puc-rio.br [mailto:owner-obm-l@sucu= ri.mat.puc-rio.br] On Behalf Of Fernanda Medeiros
Sent: Saturday, September 07, 2002 8:46 PM
To: obm-l@mat.puc-rio.br
Subject: [obm-l] violencia
=



Ol=E1,

algu=E9m pode dar uma ajuda nestas quest=F5es?

1.a)uma "gang" tem infinitos bandidos e cada um dos = meliantes tem um =FAnico

inimigo no interior da "gang",que ele quer matar.Prove q = =E9 possivel reunir

uma quantidade infinita de bandidos desta "gang", = semq  haja  o risco de q

um bandido mate outro durante a reuni=E3o.

b)Se cada bandido tiver um n=BA finito mas indefinido de = inimigos(um bandido

pode ter 2 inimigos, outro somente 1, um terceiro pode ter 20 e = assim por

diante).Ser=E1 sempre possivel promover uma reuni=E3o com infinitos = bandidos sem

risco de derramamento de sangue?

2.Encontre todas as solu=E7=F5es em inteiros n=E3o negativos = para:

2^x +3^y =3Dz^2

3.Encontre todos os inteiros positivos (x,y) tais q 7^x - 3^y = =3D4

Valeu!!

F=CA

_________________________________________________________________

Converse com seus amigos online, fa=E7a o download gr=E1tis do MSN = Messenger:

http://messenger.msn.com.br

=3D=3D=3D=3D=3D=3D=3D=3D=3D=3D=3D=3D=3D=3D=3D=3D=3D=3D=3D=3D=3D=3D=3D= =3D=3D=3D=3D=3D=3D=3D=3D=3D=3D=3D=3D=3D=3D=3D=3D=3D=3D=3D=3D=3D=3D=3D=3D=3D= =3D=3D=3D=3D=3D=3D=3D=3D=3D=3D=3D=3D=3D=3D=3D=3D=3D=3D=3D=3D=3D=3D=3D=3D=3D=

Instru=E7=F5es para entrar na lista, sair da lista e usar a lista = em

http://www.m= at.puc-rio.br/~nicolau/olimp/obm-l.html

O administrador desta lista =E9 = <nicolau@mat.puc-rio.br>

=3D=3D=3D=3D=3D=3D=3D=3D=3D=3D=3D=3D=3D=3D=3D=3D=3D=3D=3D=3D=3D=3D=3D= =3D=3D=3D=3D=3D=3D=3D=3D=3D=3D=3D=3D=3D=3D=3D=3D=3D=3D=3D=3D=3D=3D=3D=3D=3D= =3D=3D=3D=3D=3D=3D=3D=3D=3D=3D=3D=3D=3D=3D=3D=3D=3D=3D=3D=3D=3D=3D=3D=3D=3D=

------=_NextPart_000_0001_01C2572A.5E75DA20-- ========================================================================= Instruções para entrar na lista, sair da lista e usar a lista em http://www.mat.puc-rio.br/~nicolau/olimp/obm-l.html O administrador desta lista é ========================================================================= From owner-obm-l@sucuri.mat.puc-rio.br Sun Sep 8 13:11:30 2002 Return-Path: Received: (from majordom@localhost) by sucuri.mat.puc-rio.br (8.9.3/8.9.3) id NAA03697 for obm-l-MTTP; Sun, 8 Sep 2002 13:10:35 -0300 Received: from pina.terra.com.br (pina.terra.com.br [200.176.3.17]) by sucuri.mat.puc-rio.br (8.9.3/8.9.3) with ESMTP id NAA03693 for ; Sun, 8 Sep 2002 13:10:31 -0300 Received: from taipe.terra.com.br (taipe.terra.com.br [200.176.3.34]) by pina.terra.com.br (Postfix) with ESMTP id B1F2C52EE3 for ; Sun, 8 Sep 2002 13:11:25 -0300 (EST) Received: from xt (200-171-249-185.customer.telesp.net.br [200.171.249.185]) (authenticated user macwad) by taipe.terra.com.br (Postfix) with ESMTP id 9BB7E1B40A9 for ; Sun, 8 Sep 2002 13:11:24 -0300 (EST) Message-ID: <001301c25752$6a6a4780$0401010a@xt> From: =?iso-8859-1?Q?Vinicius_Jos=E9_Fortuna?= To: References: <000001c25743$83c31220$9865fea9@computer> Subject: Re: [obm-l] violencia Date: Sun, 8 Sep 2002 13:11:39 -0300 MIME-Version: 1.0 Content-Type: multipart/alternative; boundary="----=_NextPart_000_0010_01C25739.44BFD450" X-Priority: 3 X-MSMail-Priority: Normal X-Mailer: Microsoft Outlook Express 6.00.2600.0000 X-MimeOLE: Produced By Microsoft MimeOLE V6.00.2600.0000 Sender: owner-obm-l@sucuri.mat.puc-rio.br Precedence: bulk Reply-To: obm-l@mat.puc-rio.br This is a multi-part message in MIME format. ------=_NextPart_000_0010_01C25739.44BFD450 Content-Type: text/plain; charset="iso-8859-1" Content-Transfer-Encoding: quoted-printable RE: [obm-l] violenciaExiste uma passagem que, ao meu ver, est=E1 falsa. = Observe abaixo. ----- Original Message -----=20 From: Artur Costa Steiner=20 To: obm-l@mat.puc-rio.br=20 Sent: Sunday, September 08, 2002 11:24 AM Subject: RE: [obm-l] violencia=20 Bom, com rela=E7=E3o =E0 primeira quest=E3o. Comecemos pela segunda = parte e suponhamos, conforme vc disse, que cada bandido tenha um = n=FAmero finito de inimigos. Vou supor que, embora variando com o = bandido, este n=FAmero =E9 conhecido para cada bandido. Escolha um bandido. Isto =E9 poss=EDvel pois h=E1 infinitos (Meu = Deus!!) Dado que este bandido tem um n=FAmero finito de inimigos e = existem infinitos bandidos, podemos escolher um bandido que n=E3o seja = inimigo dele. Logo, a base de nosso processo indutivo est=E1 formada. Suponhamos agora que, para algum natural n, tenhamos escolhido n = bandidos tais que ningu=E9m seja inimigo de ningu=E9m. Suponhamos. Ora, = cada um destes n bandidos tem um n=FAmero apenas finito de inimigos. = Logo, o n=FAmero total de bandidos que s=E3o inimigos de um destes n = bandidos escolhidos =E9 finito - pois =E9 a soma de n parcelas finitas.. = Mas, temos infinitos bandidos, de modo que podemos escolher um que = n=E3o seja inimigo de nenhum dos n que j=E1 escolhemos (e, =E9 claro, = que n=E3o seja um membro deste conjunto de n que escolhemos). Com = isto, obtemos n +1 bandidos distintos tais que, neste conjunto, = ningu=E9m vai matar ningu=E9m.=20 N=E3o basta que ele n=E3o seja inimigo de nenhum que j=E1 foi escolhido, = mas tb nenhum dos que j=E1 foram escolhidos pode ser inimigo dele. Pode = haver o caso em que todos os infinitos bandidos restantes querem matar = algu=E9m do grupo que j=E1 foi escolhido. Neste caso o grupo da = reuni=E3o n=E3o poderia ser aumentado. Observe a mensagem que mandei anteriormente. At=E9 mais Vinicius Fortuna A "ponte" de nosso processo indutivo est=E1 portanto formada, e mostra = que nosso processo de escolha pode prosseguir indefinidamente. OK? = Podemos dizer que geramos um seq=FC=EAncia B_n de bandidos tal que, = dados quaisquer naturais m e n, com m<>n, ent=E3o B_n n=E3o quer matar = B_m. Estou assumindo, implicitamente, que ningu=E9m quer cometer = suic=EDdio.=20 A primeira parte de sua 1a quest=E3o =E9 um caso particular da = segunda, obtida quando cada bandido tem apenas um inimigo. Espero ter ajudado Artur ------=_NextPart_000_0010_01C25739.44BFD450 Content-Type: text/html; charset="iso-8859-1" Content-Transfer-Encoding: quoted-printable RE: [obm-l] violencia
Existe uma passagem que, ao meu ver, = est=E1 falsa.=20 Observe abaixo.
 
----- Original Message -----
From:=20 Artur Costa=20 Steiner
Sent: Sunday, September 08, = 2002 11:24=20 AM
Subject: RE: [obm-l] violencia =

Bom, com=20 rela=E7=E3o =E0 primeira quest=E3o. Comecemos pela segunda parte e = suponhamos,=20 conforme vc disse, que cada bandido tenha um n=FAmero finito de = inimigos. Vou=20 supor que, embora variando com o bandido, este n=FAmero =E9 conhecido = para cada=20 bandido.

Escolha um=20 bandido. Isto =E9 poss=EDvel pois h=E1 infinitos (Meu Deus!!) Dado que = este bandido=20 tem um n=FAmero finito de inimigos e existem infinitos bandidos, = podemos=20 escolher um bandido que n=E3o seja inimigo dele. Logo, a base de nosso = processo=20 indutivo est=E1 formada.

Suponhamos=20 agora que, para algum natural n, tenhamos escolhido n bandidos tais = que=20 ningu=E9m seja inimigo de ningu=E9m. Suponhamos. Ora, cada um destes n = bandidos=20 tem um n=FAmero apenas finito de inimigos. Logo, o n=FAmero total de = bandidos que=20 s=E3o inimigos de um destes n bandidos escolhidos =E9 finito - pois = =E9 a soma de n=20 parcelas finitas..

Mas, temos=20 infinitos bandidos, de modo que podemos escolher um que n=E3o seja = inimigo de=20 nenhum dos n que j=E1 escolhemos (e, =E9=20 claro, = que  n=E3o seja um = membro deste conjunto de n que = escolhemos).  Com isto, obtemos n +1 = bandidos = distintos tais que,=20 neste conjunto, ningu=E9m vai matar ningu=E9m. =

N=E3o basta=20 que ele n=E3o seja inimigo de nenhum que j=E1 foi escolhido, mas = tb nenhum=20 dos que j=E1 foram escolhidos pode ser inimigo dele. Pode = haver o=20 caso em que todos os infinitos bandidos restantes querem matar algu=E9m = do grupo=20 que j=E1 foi escolhido. Neste caso o grupo da reuni=E3o n=E3o poderia = ser=20 aumentado.

Observe a=20 mensagem que mandei anteriormente.

At=E9=20 mais

Vinicius=20 Fortuna

 

A "ponte" de=20 nosso processo indutivo est=E1 portanto formada, e mostra que nosso = processo de=20 escolha pode prosseguir indefinidamente. OK? Podemos dizer que geramos = um=20 seq=FC=EAncia B_n de bandidos tal que, dados quaisquer naturais m e n, = com=20 m<>n, ent=E3o B_n n=E3o quer matar B_m. Estou assumindo, = implicitamente, que=20 ningu=E9m quer cometer suic=EDdio.

A primeira=20 parte de sua 1a quest=E3o =E9 um caso particular da segunda, obtida = quando cada=20 bandido tem apenas um inimigo.

Espero ter=20 ajudado

Artur

------=_NextPart_000_0010_01C25739.44BFD450-- ========================================================================= Instruções para entrar na lista, sair da lista e usar a lista em http://www.mat.puc-rio.br/~nicolau/olimp/obm-l.html O administrador desta lista é ========================================================================= From owner-obm-l@sucuri.mat.puc-rio.br Sun Sep 8 13:18:18 2002 Return-Path: Received: (from majordom@localhost) by sucuri.mat.puc-rio.br (8.9.3/8.9.3) id NAA03794 for obm-l-MTTP; Sun, 8 Sep 2002 13:18:17 -0300 Received: from hotmail.com (f83.sea2.hotmail.com [207.68.165.83]) by sucuri.mat.puc-rio.br (8.9.3/8.9.3) with ESMTP id NAA03790 for ; Sun, 8 Sep 2002 13:18:13 -0300 Received: from mail pickup service by hotmail.com with Microsoft SMTPSVC; Sun, 8 Sep 2002 09:19:06 -0700 Received: from 200.222.206.188 by sea2fd.sea2.hotmail.msn.com with HTTP; Sun, 08 Sep 2002 16:19:06 GMT X-Originating-IP: [200.222.206.188] From: "Paulo Santa Rita" To: obm-l@mat.puc-rio.br Subject: Re: [obm-l] Pergunta para Paulo Santa Rita Date: Sun, 08 Sep 2002 16:19:06 +0000 Mime-Version: 1.0 Content-Type: text/plain; charset=iso-8859-1; format=flowed Message-ID: X-OriginalArrivalTime: 08 Sep 2002 16:19:06.0316 (UTC) FILETIME=[743270C0:01C25753] Sender: owner-obm-l@sucuri.mat.puc-rio.br Precedence: bulk Reply-To: obm-l@mat.puc-rio.br Ola Wagner e demais colegas desta lista ... OBM-L, A relacao que eu usei e muito conhecida e foi descoberta por Euler. Ela afirma que : e^(Ti)=cos(T) + i*sen(T), onde "i" e a UNIDADE IMAGINARIA e "e" a BASE DOS LOGARITMOS NEPERIANOS. Desta relacao podemos tirar muitos resultados interessantes e, em particular : e^(pi*i)=cos(pi)+i*sen(pi) = -1. Procure detalhar mais a prova de existencia que voce apresentou, PARECE-ME QUE ESTA MUITO CONFUSA E PASSIVEL DE SOFRER DIVERSAS CRITICAS... O ponto crucial e a passagem do expoente racional para o irracional. Se voce aceita uma sugestao, faca Y=X^irr, "irr" irracional, e considere particularmente e previamente esta equacao para um Y complexo dado ... Fica com Deus Paulo Santa Rita 1,1317,080902 >From: "Wagner" >Reply-To: obm-l@mat.puc-rio.br >To: >Subject: [obm-l] Pergunta para Paulo Santa Rita >Date: Sat, 7 Sep 2002 11:39:10 -0300 > >Bom dia pra todos! > >-Notação log n (a) = logaritmo natural de a >-(a,b) = a + bi > >Caro Paulo, na sua resposta para o meu problema (x^(PI)-5x^(PI-1)+3=0), >você diz que : > >- e^Pi.i = -1 => (estou considerando que o e da resposta seja o nº >neperiano) >e^Pi.i = (i.sen(Pi) + cos(Pi)), isso implicaria que: e^i(i.sen1 + cos1), >certo? Então a^i = e^log n (a).i = (i.sen(log n (a)) + cos(log n (a))). >Então : a^(x,y) = a^x.(i.sen(y.log n (a)) + cos(y.log n (a))) ? Ou seja um >nº real pode ser elevado a um expoente imaginário ? >Então quanto seria (a,b)^(c,d) ? E também qual a dedução de que e^Pi.i = -1 >? >- Também queria saber porque x = a.e^T.i e consequentemente x^Pi = a(-1)T. > >André T. _________________________________________________________________ MSN Photos é a maneira mais fácil e prática de editar e compartilhar sua fotos: http://photos.msn.com.br ========================================================================= Instruções para entrar na lista, sair da lista e usar a lista em http://www.mat.puc-rio.br/~nicolau/olimp/obm-l.html O administrador desta lista é ========================================================================= From owner-obm-l@sucuri.mat.puc-rio.br Sun Sep 8 14:03:31 2002 Return-Path: Received: (from majordom@localhost) by sucuri.mat.puc-rio.br (8.9.3/8.9.3) id OAA05062 for obm-l-MTTP; Sun, 8 Sep 2002 14:03:22 -0300 Received: from artemis.opendf.com.br (artemis.opengate.com.br [200.181.71.15]) by sucuri.mat.puc-rio.br (8.9.3/8.9.3) with ESMTP id OAA05058 for ; Sun, 8 Sep 2002 14:03:19 -0300 Received: from localhost (localhost [127.0.0.1]) by artemis.opendf.com.br (Postfix) with ESMTP id E532B1CAB9 for ; Sun, 8 Sep 2002 14:03:39 -0300 (BRT) Received: from artur (200-181-89-056-bsace7001.dsl.telebrasilia.net.br [200.181.89.56]) by artemis.opendf.com.br (Postfix) with ESMTP id 885C11C833 for ; Sun, 8 Sep 2002 14:03:34 -0300 (BRT) From: "Artur Costa Steiner" To: Subject: RE: [obm-l] violencia Date: Sun, 8 Sep 2002 14:05:08 -0700 Organization: Steiner Consultoria LTDA Message-ID: <000401c2577b$6b8a12c0$0b01a8c0@mshome.net> MIME-Version: 1.0 Content-Type: multipart/alternative; boundary="----=_NextPart_000_0005_01C25740.BF2B3AC0" X-Priority: 3 (Normal) X-MSMail-Priority: Normal X-Mailer: Microsoft Outlook, Build 10.0.2627 X-MimeOLE: Produced By Microsoft MimeOLE V6.00.2600.0000 Importance: Normal In-Reply-To: <001301c25752$6a6a4780$0401010a@xt> X-Virus-Scanned: by AMaViS new-20020517 Sender: owner-obm-l@sucuri.mat.puc-rio.br Precedence: bulk Reply-To: obm-l@mat.puc-rio.br This is a multi-part message in MIME format. ------=_NextPart_000_0005_01C25740.BF2B3AC0 Content-Type: text/plain; charset="iso-8859-1" Content-Transfer-Encoding: quoted-printable =20 =20 -----Original Message----- From: owner-obm-l@sucuri.mat.puc-rio.br [mailto:owner-obm-l@sucuri.mat.puc-rio.br] On Behalf Of Vinicius Jos=E9 Fortuna Sent: Sunday, September 08, 2002 8:12 AM To: obm-l@mat.puc-rio.br Subject: Re: [obm-l] violencia=20 =20 Existe uma passagem que, ao meu ver, est=E1 falsa. Observe abaixo. =20 ----- Original Message -----=20 From: Artur Costa Steiner=20 To: obm-l@mat.puc-rio.br=20 Sent: Sunday, September 08, 2002 11:24 AM Subject: RE: [obm-l] violencia=20 =20 Bom, com rela=E7=E3o =E0 primeira quest=E3o. Comecemos pela segunda = parte e suponhamos, conforme vc disse, que cada bandido tenha um n=FAmero finito de inimigos. Vou supor que, embora variando com o bandido, este n=FAmero = =E9 conhecido para cada bandido. Escolha um bandido. Isto =E9 poss=EDvel pois h=E1 infinitos (Meu Deus!!) = Dado que este bandido tem um n=FAmero finito de inimigos e existem infinitos bandidos, podemos escolher um bandido que n=E3o seja inimigo dele. Logo, = a base de nosso processo indutivo est=E1 formada. Suponhamos agora que, para algum natural n, tenhamos escolhido n bandidos tais que ningu=E9m seja inimigo de ningu=E9m. Suponhamos. Ora, = cada um destes n bandidos tem um n=FAmero apenas finito de inimigos. Logo, o n=FAmero total de bandidos que s=E3o inimigos de um destes n bandidos escolhidos =E9 finito - pois =E9 a soma de n parcelas finitas..=20 Mas, temos infinitos bandidos, de modo que podemos escolher um que n=E3o seja inimigo de nenhum dos n que j=E1 escolhemos (e, =E9 claro, que = n=E3o seja um membro deste conjunto de n que escolhemos). Com isto, obtemos n +1 bandidos distintos tais que, neste conjunto, ningu=E9m vai matar ningu=E9m.=20 N=E3o basta que ele n=E3o seja inimigo de nenhum que j=E1 foi escolhido, = mas tb nenhum dos que j=E1 foram escolhidos pode ser inimigo dele. Pode = haver o caso em que todos os infinitos bandidos restantes querem matar = algu=E9m do grupo que j=E1 foi escolhido. Neste caso o grupo da reuni=E3o n=E3o = poderia ser aumentado. Observe a mensagem que mandei anteriormente. =20 =C9, tem raz=E3o, neste caso, o processo que citei n=E3o vigora. Eu = coloquei uma outra mensagem sobre isso. Abra=E7os Artur At=E9 mais Vinicius Fortuna =20 A "ponte" de nosso processo indutivo est=E1 portanto formada, e mostra = que nosso processo de escolha pode prosseguir indefinidamente. OK? Podemos dizer que geramos um seq=FC=EAncia B_n de bandidos tal que, dados = quaisquer naturais m e n, com m<>n, ent=E3o B_n n=E3o quer matar B_m. Estou = assumindo, implicitamente, que ningu=E9m quer cometer suic=EDdio.=20 A primeira parte de sua 1a quest=E3o =E9 um caso particular da segunda, obtida quando cada bandido tem apenas um inimigo. Espero ter ajudado Artur ------=_NextPart_000_0005_01C25740.BF2B3AC0 Content-Type: text/html; charset="iso-8859-1" Content-Transfer-Encoding: quoted-printable RE: [obm-l] violencia

 

 

-----Original = Message-----
From: owner-obm-l@sucuri.mat.puc-rio.br = [mailto:owner-obm-l@sucuri.mat.puc-rio.br] On Behalf Of Vinicius Jos=E9 = Fortuna
Sent: =
Sunday, September 08, = 2002 8:12 AM
To: = obm-l@mat.puc-rio.br
Subject: Re: [obm-l] = violencia

 

Existe uma passagem que, ao = meu ver, est=E1 falsa. Observe abaixo.

 

----- Original Message = -----

To:<= /font> obm-l@mat.puc-rio.br

Sent: Sunday, September 08, = 2002 11:24 AM

Subject: RE: [obm-l] violencia

 

Bom, com rela=E7=E3o =E0 primeira quest=E3o. Comecemos pela segunda parte e = suponhamos, conforme vc disse, que cada bandido tenha um n=FAmero finito de = inimigos. Vou supor que, embora variando com o bandido, este n=FAmero =E9 conhecido = para cada bandido.

Escolha um bandido. Isto =E9 poss=EDvel pois h=E1 infinitos (Meu Deus!!) Dado = que este bandido tem um n=FAmero finito de inimigos e existem infinitos bandidos, = podemos escolher um bandido que n=E3o seja inimigo dele. Logo, a base de nosso = processo indutivo est=E1 formada.

Suponhamos agora que, para algum natural n, tenhamos escolhido n bandidos tais que = ningu=E9m seja inimigo de ningu=E9m. Suponhamos. Ora, cada um destes n bandidos = tem um n=FAmero apenas finito de inimigos. Logo, o n=FAmero total de bandidos = que s=E3o inimigos de um destes n bandidos escolhidos =E9 finito - pois =E9 a soma = de n parcelas finitas..

Mas, temos infinitos bandidos, de modo que podemos escolher um que n=E3o seja = inimigo de nenhum dos n que j=E1 escolhemos (e, =E9 claro, = que  n=E3o seja um membro deste conjunto de n que = escolhemos).  Com isto, obtemos n +1 = bandidos distintos tais que, neste conjunto, ningu=E9m vai matar ningu=E9m. =

N=E3o basta que ele n=E3o seja inimigo de nenhum que j=E1 foi escolhido, mas tb nenhum dos que j=E1 foram escolhidos pode ser inimigo dele. Pode haver o caso em que todos os infinitos bandidos restantes querem matar algu=E9m do grupo que j=E1 foi escolhido. Neste = caso o grupo da reuni=E3o n=E3o poderia ser aumentado.

Observe a mensagem que mandei anteriormente.

 

=C9, tem raz=E3o, = neste caso, o processo que citei n=E3o vigora. Eu coloquei uma outra mensagem sobre = isso.

Abra=E7os

Artur

At=E9 mais

Vinicius Fortuna

 

A "ponte" de nosso processo indutivo est=E1 portanto formada, e = mostra que nosso processo de escolha pode prosseguir indefinidamente. OK? = Podemos dizer que geramos um seq=FC=EAncia B_n de bandidos tal que, dados = quaisquer naturais m e n, com m<>n, ent=E3o B_n n=E3o quer matar B_m. Estou = assumindo, implicitamente, que ningu=E9m quer cometer suic=EDdio. =

A primeira parte de sua 1a quest=E3o =E9 um caso particular da segunda, = obtida quando cada bandido tem apenas um inimigo.

Espero ter ajudado

Artur

------=_NextPart_000_0005_01C25740.BF2B3AC0-- ========================================================================= Instruções para entrar na lista, sair da lista e usar a lista em http://www.mat.puc-rio.br/~nicolau/olimp/obm-l.html O administrador desta lista é ========================================================================= From owner-obm-l@sucuri.mat.puc-rio.br Sun Sep 8 14:16:48 2002 Return-Path: Received: (from majordom@localhost) by sucuri.mat.puc-rio.br (8.9.3/8.9.3) id OAA05327 for obm-l-MTTP; Sun, 8 Sep 2002 14:16:32 -0300 Received: from hotmail.com (f101.law9.hotmail.com [64.4.9.101]) by sucuri.mat.puc-rio.br (8.9.3/8.9.3) with ESMTP id OAA05323 for ; Sun, 8 Sep 2002 14:16:29 -0300 Received: from mail pickup service by hotmail.com with Microsoft SMTPSVC; Sun, 8 Sep 2002 10:17:22 -0700 Received: from 200.190.8.44 by lw9fd.law9.hotmail.msn.com with HTTP; Sun, 08 Sep 2002 17:17:22 GMT X-Originating-IP: [200.190.8.44] From: "Rogerio Fajardo" To: obm-l@mat.puc-rio.br Subject: Re: [obm-l] violencia Date: Sun, 08 Sep 2002 17:17:22 +0000 Mime-Version: 1.0 Content-Type: text/plain; format=flowed Message-ID: X-OriginalArrivalTime: 08 Sep 2002 17:17:22.0386 (UTC) FILETIME=[98046320:01C2575B] Sender: owner-obm-l@sucuri.mat.puc-rio.br Precedence: bulk Reply-To: obm-l@mat.puc-rio.br É bom notar que essa solução usa o axioma da escolha (de infinitos conjuntos não-vazios, escolhemos um elemento de cada). É essencial o axioma da escolha para resolvê-lo? >From: Vinicius José Fortuna >Reply-To: obm-l@mat.puc-rio.br >To: >Subject: Re: [obm-l] violencia Date: Sat, 7 Sep 2002 23:44:58 -0300 > >----- Original Message ----- >From: "Fernanda Medeiros" >To: >Sent: Saturday, September 07, 2002 8:45 PM >Subject: [obm-l] violencia > > > > Olá, > > alguém pode dar uma ajuda nestas questões? > > 1.a)uma "gang" tem infinitos bandidos e cada um dos meliantes tem um >único > > inimigo no interior da "gang",que ele quer matar.Prove q é possivel >reunir > > uma quantidade infinita de bandidos desta "gang", semq haja o risco de >q > > um bandido mate outro durante a reunião. > >Pense no seguinte algoritmo: >Temos o conjunto C de candidatos à reunião que inicialmente contém todos os >infinitos bandidos da gangue. >Temos o conjunto R de bandidos selecionados para a reunião que inicialmente >está vazio. > >A cada passo do algoritmo procuramos em C alguém que não que matar ninguém >de R e ninguém em R quer matá-lo. >Seja M o subconjunto de C de bandidos que pelo menos um de R quer matar. >Como cada bandido de R só quer matar um, |M|<=|R| >Então, como R é finito, M será finito e V=C-M será infinito, pois C é >infinito. >V será o subconjunto de C dos bandidos que ninguém de R quer matar. >Em V procuramos um bandido que não quer matar ninguém de R, retiramos ele >de >C, o inserimos em R e repete-se o processo. > >Se sempre for possível encontrar tal bandido, o processo se repetirá >indefinidamente e com R sempre crescendo. Assim teremos infnitos bandidos >na >reunião sem derramamento de sangue. > >Se em algum momento não for possível encontrar um bandido em V, é porque >todos os bandidos de V querem matar alguém de R. Ou seja, ninguém de V quer >matar outro de V. Pegamos, então, V como o conjunto de bandidos para a >reunião. Como V é infinito, teremos infinitos participantes na reunião. > > > b)Se cada bandido tiver um nº finito mas indefinido de inimigos(um >bandido > > pode ter 2 inimigos, outro somente 1, um terceiro pode ter 20 e assim >por > > diante).Será sempre possivel promover uma reunião com infinitos bandidos >sem > > risco de derramamento de sangue? >Não é possível. Existe um contra-exemplo: >Ordene os bandidos formando uma sequência. Imagine que cada bandido quer >matar todos que vêm antes dele na sequência. Nunca poderemos ter dois >bandidos 'a' e 'b' na reunião, pois ou a vem antes de b, ou b vem antes de, >assim haverá um que vai querer matar o outro. Então só poderemos ter um >bandido na reunião. > >Até mais > >Vinicius Fortuna >IC-Unicamp > > >========================================================================= >Instruções para entrar na lista, sair da lista e usar a lista em >http://www.mat.puc-rio.br/~nicolau/olimp/obm-l.html >O administrador desta lista é >========================================================================= s _________________________________________________________________ Join the world’s largest e-mail service with MSN Hotmail. http://www.hotmail.com ========================================================================= Instruções para entrar na lista, sair da lista e usar a lista em http://www.mat.puc-rio.br/~nicolau/olimp/obm-l.html O administrador desta lista é ========================================================================= From owner-obm-l@sucuri.mat.puc-rio.br Sun Sep 8 16:13:59 2002 Return-Path: Received: (from majordom@localhost) by sucuri.mat.puc-rio.br (8.9.3/8.9.3) id QAA06756 for obm-l-MTTP; Sun, 8 Sep 2002 16:13:12 -0300 Received: from ginsberg.uol.com.br (ginsberg.uol.com.br [200.221.4.48]) by sucuri.mat.puc-rio.br (8.9.3/8.9.3) with ESMTP id QAA06748 for ; Sun, 8 Sep 2002 16:13:08 -0300 Received: from u2z7z2 ([200.158.144.204]) by ginsberg.uol.com.br (8.9.1/8.9.1) with ESMTP id QAA13877 for ; Sun, 8 Sep 2002 16:12:18 -0300 (BRT) Message-ID: <000201c2576c$080c3f20$cc909ec8@u2z7z2> From: "Wagner" To: References: <008401c25684$5c609320$6898cfc8@slap> Subject: [obm-l] =?iso-8859-1?Q?Re:_=5Bobm-l=5D_Teste_dif=EDcil...?= Date: Sun, 8 Sep 2002 13:51:04 -0300 Organization: Wagner MIME-Version: 1.0 Content-Type: multipart/alternative; boundary="----=_NextPart_000_0014_01C2573E.C6715460" X-Priority: 3 X-MSMail-Priority: Normal X-Mailer: Microsoft Outlook Express 5.50.4133.2400 X-MimeOLE: Produced By Microsoft MimeOLE V5.50.4133.2400 Sender: owner-obm-l@sucuri.mat.puc-rio.br Precedence: bulk Reply-To: obm-l@mat.puc-rio.br This is a multi-part message in MIME format. ------=_NextPart_000_0014_01C2573E.C6715460 Content-Type: text/plain; charset="iso-8859-1" Content-Transfer-Encoding: quoted-printable Caro Afemano a^b=3D a elevado a b log a (b)=3D logaritmo de b na base a Por defini=E7=E3o , o n=BA de casas de a^b em um sistema de base c =E9 = igual a : b/log a (c), arredondado para o inteiro imediatamente = superior, Logo B =3D 4444/log 4444 (10). Andr=E9 T. ----- Original Message -----=20 From: Afemano=20 To: obm-l@mat.puc-rio.br=20 Sent: Saturday, September 07, 2002 12:36 PM Subject: [obm-l] Teste dif=EDcil... Oi, sou novo por aqui e n=E3o sei se este teste j=E1 foi = respondido.... falou ... "A forma decimal do n=FAmero 4444 elevado a 4444 possui um certo = n=FAmero de algarismos. A soma desses algarismos =E1 A. A soma dos = algarismos de A =E9 B. Qual a soma dos algarismos de B ???" Isso est=E1 num caderno de exerc=EDcios de um amigo meu do cursinho. = Se algu=E9m puder ajudar ae valeu !! ------=_NextPart_000_0014_01C2573E.C6715460 Content-Type: text/html; charset="iso-8859-1" Content-Transfer-Encoding: quoted-printable
Caro Afemano
 
a^b=3D a elevado a b
log a (b)=3D logaritmo de b na base = a
 
Por defini=E7=E3o , o n=BA de casas de = a^b em um sistema=20 de base c =E9 igual a : b/log a (c), arredondado para o inteiro = imediatamente=20 superior,
Logo B =3D 4444/log 4444 = (10).
 
Andr=E9 T.
 
 
----- Original Message -----
From:=20 Afemano=20
Sent: Saturday, September 07, = 2002 12:36=20 PM
Subject: [obm-l] Teste = dif=EDcil...

Oi, sou novo por aqui e n=E3o sei se = este teste j=E1=20 foi respondido.... falou ...
 
"A forma decimal do n=FAmero 4444 = elevado a 4444=20 possui um certo n=FAmero de algarismos. A soma desses algarismos =E1 = A. A soma dos=20 algarismos de A =E9 B. Qual a soma dos algarismos de B = ???"
 
Isso est=E1 num caderno de = exerc=EDcios de um amigo=20 meu do cursinho. Se algu=E9m puder ajudar ae valeu=20 !!
------=_NextPart_000_0014_01C2573E.C6715460-- ========================================================================= Instruções para entrar na lista, sair da lista e usar a lista em http://www.mat.puc-rio.br/~nicolau/olimp/obm-l.html O administrador desta lista é ========================================================================= From owner-obm-l@sucuri.mat.puc-rio.br Sun Sep 8 16:30:46 2002 Return-Path: Received: (from majordom@localhost) by sucuri.mat.puc-rio.br (8.9.3/8.9.3) id QAA07081 for obm-l-MTTP; Sun, 8 Sep 2002 16:30:34 -0300 Received: from silva5.uol.com.br (silva5.uol.com.br [200.221.4.52]) by sucuri.mat.puc-rio.br (8.9.3/8.9.3) with ESMTP id QAA07076 for ; Sun, 8 Sep 2002 16:30:32 -0300 Received: from u2z7z2 ([200.158.144.204]) by silva5.uol.com.br (8.9.1/8.9.1) with ESMTP id QAA20641 for ; Sun, 8 Sep 2002 16:33:47 -0300 (EST) Message-ID: <001401c2576e$7aa8d820$cc909ec8@u2z7z2> From: "Wagner" To: Subject: [obm-l] A volta do futebol arte! Date: Sun, 8 Sep 2002 16:32:33 -0300 Organization: Wagner MIME-Version: 1.0 Content-Type: multipart/alternative; boundary="----=_NextPart_000_0011_01C25755.5530E6A0" X-Priority: 3 X-MSMail-Priority: Normal X-Mailer: Microsoft Outlook Express 5.50.4133.2400 X-MimeOLE: Produced By Microsoft MimeOLE V5.50.4133.2400 Sender: owner-obm-l@sucuri.mat.puc-rio.br Precedence: bulk Reply-To: obm-l@mat.puc-rio.br This is a multi-part message in MIME format. ------=_NextPart_000_0011_01C25755.5530E6A0 Content-Type: text/plain; charset="iso-8859-1" Content-Transfer-Encoding: quoted-printable Oi pessoas!!! =20 Como hoje n=E3o estou muito inspirado hoje, vou propor um problema = simples ( para n=E3o dizer rid=EDculo ): Se voc=EA pegar uma bola de futebol e achatar todas as suas faces de = modo que elas fiquem retas, voc=EA ter=E1 um poliedro com 60 v=E9rtices (em uma bola de futebol de qualidade e que n=E3o tenha = sido comprada na 25 de mar=E7o, =E9 claro) . Como a=20 maioria sabe, a costura da bola de futebol forma pent=E1gonos e = hex=E1gonos regulares, arranjados de forma que em volta de cada = pent=E1gono existem 5 hex=E1gonos e em volta de cada hex=E1gono existem 3 = pent=E1gonos e 3 hex=E1gonos. Logo quantas faces de uma bola de futebol s=E3o pentagonais e quantas s=E3o hex=E1gonais ? Andr=E9 T. ------=_NextPart_000_0011_01C25755.5530E6A0 Content-Type: text/html; charset="iso-8859-1" Content-Transfer-Encoding: quoted-printable
Oi pessoas!!!  
 
    Como hoje n=E3o = estou muito=20 inspirado hoje, vou propor um problema simples ( para n=E3o dizer = rid=EDculo=20 ):
 
    Se voc=EA pegar uma = bola de=20 futebol e achatar todas as suas faces de modo que elas fiquem retas, = voc=EA ter=E1=20 um poliedro
com 60 v=E9rtices (em uma bola de = futebol de=20 qualidade e que n=E3o tenha sido comprada na 25 de mar=E7o, =E9 = claro) . Como a=20
maioria sabe, a costura da bola de = futebol forma=20 pent=E1gonos e hex=E1gonos regulares, arranjados de forma que em volta = de cada=20 pent=E1gono
existem 5 hex=E1gonos e em volta de = cada hex=E1gono=20 existem 3 pent=E1gonos e 3 hex=E1gonos. Logo quantas faces de uma bola = de=20 futebol
s=E3o pentagonais e quantas s=E3o = hex=E1gonais=20 ?
 
    Andr=E9=20 T.
------=_NextPart_000_0011_01C25755.5530E6A0-- ========================================================================= Instruções para entrar na lista, sair da lista e usar a lista em http://www.mat.puc-rio.br/~nicolau/olimp/obm-l.html O administrador desta lista é ========================================================================= From owner-obm-l@sucuri.mat.puc-rio.br Sun Sep 8 16:35:03 2002 Return-Path: Received: (from majordom@localhost) by sucuri.mat.puc-rio.br (8.9.3/8.9.3) id QAA07252 for obm-l-MTTP; Sun, 8 Sep 2002 16:35:01 -0300 Received: from silva5.uol.com.br (silva5.uol.com.br [200.221.4.52]) by sucuri.mat.puc-rio.br (8.9.3/8.9.3) with ESMTP id QAA07248 for ; Sun, 8 Sep 2002 16:34:59 -0300 Received: from u2z7z2 ([200.158.144.204]) by silva5.uol.com.br (8.9.1/8.9.1) with ESMTP id QAA02296 for ; Sun, 8 Sep 2002 16:38:14 -0300 (EST) Message-ID: <001a01c2576f$1990c6a0$cc909ec8@u2z7z2> From: "Wagner" To: References: Subject: Re: [obm-l] Pergunta para Paulo Santa Rita Date: Sun, 8 Sep 2002 16:36:58 -0300 Organization: Wagner MIME-Version: 1.0 Content-Type: text/plain; charset="iso-8859-1" Content-Transfer-Encoding: 8bit X-Priority: 3 X-MSMail-Priority: Normal X-Mailer: Microsoft Outlook Express 5.50.4133.2400 X-MimeOLE: Produced By Microsoft MimeOLE V5.50.4133.2400 Sender: owner-obm-l@sucuri.mat.puc-rio.br Precedence: bulk Reply-To: obm-l@mat.puc-rio.br Oi para todos A dedução fica melhor assim: e^(Ti)=cos(T) + i*sen(T), em que T é o logaritmo natural de a. Portanto: a^i=cos(log n (a))+i*sen(log n (a)) André T. ----- Original Message ----- From: "Paulo Santa Rita" To: Sent: Sunday, September 08, 2002 1:19 PM Subject: Re: [obm-l] Pergunta para Paulo Santa Rita > Ola Wagner e demais colegas > desta lista ... OBM-L, > > A relacao que eu usei e muito conhecida e foi descoberta por Euler. Ela > afirma que : > > e^(Ti)=cos(T) + i*sen(T), > > onde "i" e a UNIDADE IMAGINARIA e "e" a BASE DOS LOGARITMOS NEPERIANOS. > Desta relacao podemos tirar muitos resultados interessantes e, em particular > : > > e^(pi*i)=cos(pi)+i*sen(pi) = -1. > > Procure detalhar mais a prova de existencia que voce apresentou, PARECE-ME > QUE ESTA MUITO CONFUSA E PASSIVEL DE SOFRER DIVERSAS CRITICAS... O ponto > crucial e a passagem do expoente racional para o irracional. Se voce aceita > uma sugestao, faca Y=X^irr, "irr" irracional, e considere particularmente e > previamente esta equacao para um Y complexo dado ... > > Fica com Deus > Paulo Santa Rita > 1,1317,080902 > > > > > >From: "Wagner" > >Reply-To: obm-l@mat.puc-rio.br > >To: > >Subject: [obm-l] Pergunta para Paulo Santa Rita > >Date: Sat, 7 Sep 2002 11:39:10 -0300 > > > >Bom dia pra todos! > > > >-Notação log n (a) = logaritmo natural de a > >-(a,b) = a + bi > > > >Caro Paulo, na sua resposta para o meu problema (x^(PI)-5x^(PI-1)+3=0), > >você diz que : > > > >- e^Pi.i = -1 => (estou considerando que o e da resposta seja o nº > >neperiano) > >e^Pi.i = (i.sen(Pi) + cos(Pi)), isso implicaria que: e^i(i.sen1 + cos1), > >certo? Então a^i = e^log n (a).i = (i.sen(log n (a)) + cos(log n (a))). > >Então : a^(x,y) = a^x.(i.sen(y.log n (a)) + cos(y.log n (a))) ? Ou seja um > >nº real pode ser elevado a um expoente imaginário ? > >Então quanto seria (a,b)^(c,d) ? E também qual a dedução de que e^Pi.i = -1 > >? > >- Também queria saber porque x = a.e^T.i e consequentemente x^Pi = a(-1)T. > > > >André T. > > > > > _________________________________________________________________ > MSN Photos é a maneira mais fácil e prática de editar e compartilhar sua > fotos: http://photos.msn.com.br > > ========================================================================= > Instruções para entrar na lista, sair da lista e usar a lista em > http://www.mat.puc-rio.br/~nicolau/olimp/obm-l.html > O administrador desta lista é > ========================================================================= > ========================================================================= Instruções para entrar na lista, sair da lista e usar a lista em http://www.mat.puc-rio.br/~nicolau/olimp/obm-l.html O administrador desta lista é ========================================================================= From owner-obm-l@sucuri.mat.puc-rio.br Sun Sep 8 17:01:23 2002 Return-Path: Received: (from majordom@localhost) by sucuri.mat.puc-rio.br (8.9.3/8.9.3) id RAA08389 for obm-l-MTTP; Sun, 8 Sep 2002 17:01:14 -0300 Received: from silva5.uol.com.br (silva5.uol.com.br [200.221.4.52]) by sucuri.mat.puc-rio.br (8.9.3/8.9.3) with ESMTP id RAA08385 for ; Sun, 8 Sep 2002 17:01:12 -0300 Received: from u2z7z2 ([200.158.144.204]) by silva5.uol.com.br (8.9.1/8.9.1) with ESMTP id RAA04274 for ; Sun, 8 Sep 2002 17:04:27 -0300 (EST) Message-ID: <002f01c25772$c1fa74a0$cc909ec8@u2z7z2> From: "Wagner" To: Subject: [obm-l] =?iso-8859-1?Q?Peteca_Rob=F3tica?= Date: Sun, 8 Sep 2002 17:03:10 -0300 Organization: Wagner MIME-Version: 1.0 Content-Type: multipart/alternative; boundary="----=_NextPart_000_002C_01C25759.9C7AE200" X-Priority: 3 X-MSMail-Priority: Normal X-Mailer: Microsoft Outlook Express 5.50.4133.2400 X-MimeOLE: Produced By Microsoft MimeOLE V5.50.4133.2400 Sender: owner-obm-l@sucuri.mat.puc-rio.br Precedence: bulk Reply-To: obm-l@mat.puc-rio.br This is a multi-part message in MIME format. ------=_NextPart_000_002C_01C25759.9C7AE200 Content-Type: text/plain; charset="iso-8859-1" Content-Transfer-Encoding: quoted-printable Ol=E1 companheiros de lista De=EAm uma olhada nesse problema de l=F3gica. -Em um laborat=F3rio de rob=F3tica foi feito um jogo de peteca entre = rob=F4s. 6 rob=F4s formaram uma roda e foram numerados de 1 a 6. O rob=F4 que ficasse com o n=FAmero 1 iria come=E7ar o jogo com a = peteca, marcando assim 1 ponto, todos os demais come=E7ariam o jogo sem = pontos. Os rob=F4s foram numerados ou no sentido hor=E1rio ou no sentido = anti-hor=E1rio. Os rob=F4s foram programados de 3 modos diferntes, de = modo que somente 2 rob=F4s possuem a mesma configura=E7=E3o e rob=F4s = com a mesma configura=E7=E3o se encontram diametralmente opostos na roda = formada. -As programa=E7=F5es poss=EDveis s=E3o: -P1=3Dpasse a peteca sempre para o rob=F4 imediatamente =E0 direita. -P2=3Dpasse a peteca para o rob=F4 imediatamente =E0 esquerda, a menos = que ele tenha uma pontua=E7=E3o MENOR que a sua. Nesse caso passe a = peteca para o outro rob=F4 com configura=E7=E3o P2 -P3=3DSendo n o n=FAmero do rob=F4 que lhe passou a peteca, passe a = peteca para o rob=F4 de n=FAmero n-2, considere que o antecessor de 1 = =E9 6. Caso voc=EA comece com a peteca, siga a regra A. 1=AA parte-Sabendo que toda vez que um rob=F4 recebe a peteca ele ganha = um ponto e que no final do jogo (um jogo s=E3o 30 passes) o produto das = pontua=E7=F5es dos 6 rob=F4s foi maior que zero. Descreva os 10 = primeiros passes e diga qual =E9 a regra A. Andr=E9 T. ------=_NextPart_000_002C_01C25759.9C7AE200 Content-Type: text/html; charset="iso-8859-1" Content-Transfer-Encoding: quoted-printable
Ol=E1 companheiros de = lista
 
De=EAm uma olhada nesse problema de=20 l=F3gica.
 
-Em um laborat=F3rio de rob=F3tica foi = feito um jogo de=20 peteca entre rob=F4s. 6 rob=F4s formaram uma roda e foram numerados de 1 = a=20 6.
O rob=F4 que ficasse com o n=FAmero 1 = iria come=E7ar o=20 jogo com a peteca, marcando assim 1 ponto, todos os demais  = come=E7ariam=20 o jogo sem pontos. Os rob=F4s foram = numerados ou no=20 sentido hor=E1rio ou no sentido anti-hor=E1rio. Os rob=F4s foram = programados de 3=20 modos diferntes, de modo que somente 2 rob=F4s possuem a mesma = configura=E7=E3o e=20 rob=F4s com a mesma configura=E7=E3o se encontram diametralmente opostos = na roda=20 formada.
-As programa=E7=F5es poss=EDveis = s=E3o:
-P1=3Dpasse a peteca sempre para o = rob=F4 imediatamente=20 =E0 direita.
-P2=3Dpasse a peteca para o rob=F4 = imediatamente =E0=20 esquerda, a menos que ele tenha uma pontua=E7=E3o MENOR que a sua. Nesse = caso passe=20 a peteca para o outro rob=F4 com configura=E7=E3o P2
-P3=3DSendo n o n=FAmero do rob=F4 que = lhe passou a=20 peteca, passe a peteca para o rob=F4 de n=FAmero n-2, considere que o = antecessor de=20 1 =E9 6.
Caso voc=EA comece com a peteca, siga a = regra=20 A.
 
1=AA parte-Sabendo que toda vez que um = rob=F4 recebe a=20 peteca ele ganha um ponto e que no final do jogo (um jogo s=E3o 30 = passes) o=20 produto das pontua=E7=F5es dos 6 rob=F4s foi maior que zero. Descreva os = 10 primeiros=20 passes e diga qual =E9 a regra A.
 
 
Andr=E9 T.
------=_NextPart_000_002C_01C25759.9C7AE200-- ========================================================================= Instruções para entrar na lista, sair da lista e usar a lista em http://www.mat.puc-rio.br/~nicolau/olimp/obm-l.html O administrador desta lista é ========================================================================= From owner-obm-l@sucuri.mat.puc-rio.br Sun Sep 8 18:41:00 2002 Return-Path: Received: (from majordom@localhost) by sucuri.mat.puc-rio.br (8.9.3/8.9.3) id SAA09762 for obm-l-MTTP; Sun, 8 Sep 2002 18:40:39 -0300 Received: from videira.terra.com.br (videira.terra.com.br [200.176.3.5]) by sucuri.mat.puc-rio.br (8.9.3/8.9.3) with ESMTP id SAA09758 for ; Sun, 8 Sep 2002 18:40:37 -0300 Received: from taipe.terra.com.br (taipe.terra.com.br [200.176.3.34]) by videira.terra.com.br (Postfix) with ESMTP id 5DAD4E117D for ; Sun, 8 Sep 2002 18:41:31 -0300 (EST) Received: from xt (200-171-249-185.customer.telesp.net.br [200.171.249.185]) (authenticated user macwad) by taipe.terra.com.br (Postfix) with ESMTP id AF89E1B40AE for ; Sun, 8 Sep 2002 18:41:30 -0300 (EST) Message-ID: <007201c25780$87e57ef0$0401010a@xt> From: =?iso-8859-1?Q?Vinicius_Jos=E9_Fortuna?= To: References: Subject: Re: [obm-l] violencia Date: Sun, 8 Sep 2002 18:41:45 -0300 MIME-Version: 1.0 Content-Type: text/plain; charset="iso-8859-1" Content-Transfer-Encoding: 8bit X-Priority: 3 X-MSMail-Priority: Normal X-Mailer: Microsoft Outlook Express 6.00.2600.0000 X-MimeOLE: Produced By Microsoft MimeOLE V6.00.2600.0000 Sender: owner-obm-l@sucuri.mat.puc-rio.br Precedence: bulk Reply-To: obm-l@mat.puc-rio.br Oi Rogério Acho que não saquei. Em que momento foi utilizado o axioma da escolha? Eu nem tinha infinitos conjuntos! Apenas conjuntos infinitos. Até mais Vinicius ----- Original Message ----- From: "Rogerio Fajardo" To: Sent: Sunday, September 08, 2002 2:17 PM Subject: Re: [obm-l] violencia > É bom notar que essa solução usa o axioma da escolha (de infinitos conjuntos > não-vazios, escolhemos um elemento de cada). É essencial o axioma da escolha > para resolvê-lo? > > > >From: Vinicius José Fortuna > >Reply-To: obm-l@mat.puc-rio.br > >To: > >Subject: Re: [obm-l] violencia Date: Sat, 7 Sep 2002 23:44:58 -0300 > > > >----- Original Message ----- > >From: "Fernanda Medeiros" > >To: > >Sent: Saturday, September 07, 2002 8:45 PM > >Subject: [obm-l] violencia > > > > > > > Olá, > > > alguém pode dar uma ajuda nestas questões? > > > 1.a)uma "gang" tem infinitos bandidos e cada um dos meliantes tem um > >único > > > inimigo no interior da "gang",que ele quer matar.Prove q é possivel > >reunir > > > uma quantidade infinita de bandidos desta "gang", semq haja o risco de > >q > > > um bandido mate outro durante a reunião. > > > >Pense no seguinte algoritmo: > >Temos o conjunto C de candidatos à reunião que inicialmente contém todos os > >infinitos bandidos da gangue. > >Temos o conjunto R de bandidos selecionados para a reunião que inicialmente > >está vazio. > > > >A cada passo do algoritmo procuramos em C alguém que não que matar ninguém > >de R e ninguém em R quer matá-lo. > >Seja M o subconjunto de C de bandidos que pelo menos um de R quer matar. > >Como cada bandido de R só quer matar um, |M|<=|R| > >Então, como R é finito, M será finito e V=C-M será infinito, pois C é > >infinito. > >V será o subconjunto de C dos bandidos que ninguém de R quer matar. > >Em V procuramos um bandido que não quer matar ninguém de R, retiramos ele > >de > >C, o inserimos em R e repete-se o processo. > > > >Se sempre for possível encontrar tal bandido, o processo se repetirá > >indefinidamente e com R sempre crescendo. Assim teremos infnitos bandidos > >na > >reunião sem derramamento de sangue. > > > >Se em algum momento não for possível encontrar um bandido em V, é porque > >todos os bandidos de V querem matar alguém de R. Ou seja, ninguém de V quer > >matar outro de V. Pegamos, então, V como o conjunto de bandidos para a > >reunião. Como V é infinito, teremos infinitos participantes na reunião. > > > > > b)Se cada bandido tiver um nº finito mas indefinido de inimigos(um > >bandido > > > pode ter 2 inimigos, outro somente 1, um terceiro pode ter 20 e assim > >por > > > diante).Será sempre possivel promover uma reunião com infinitos bandidos > >sem > > > risco de derramamento de sangue? > >Não é possível. Existe um contra-exemplo: > >Ordene os bandidos formando uma sequência. Imagine que cada bandido quer > >matar todos que vêm antes dele na sequência. Nunca poderemos ter dois > >bandidos 'a' e 'b' na reunião, pois ou a vem antes de b, ou b vem antes de, > >assim haverá um que vai querer matar o outro. Então só poderemos ter um > >bandido na reunião. > > > >Até mais > > > >Vinicius Fortuna > >IC-Unicamp ========================================================================= Instruções para entrar na lista, sair da lista e usar a lista em http://www.mat.puc-rio.br/~nicolau/olimp/obm-l.html O administrador desta lista é ========================================================================= From owner-obm-l@sucuri.mat.puc-rio.br Sun Sep 8 19:27:32 2002 Return-Path: Received: (from majordom@localhost) by sucuri.mat.puc-rio.br (8.9.3/8.9.3) id TAA10561 for obm-l-MTTP; Sun, 8 Sep 2002 19:27:13 -0300 Received: from studer.bol.com.br (studer.bol.com.br [200.221.24.21]) by sucuri.mat.puc-rio.br (8.9.3/8.9.3) with ESMTP id TAA10557 for ; Sun, 8 Sep 2002 19:27:10 -0300 Received: from lba (200.221.24.99) by studer.bol.com.br (5.1.071) id 3D5D1D430079A99A for obm-l@mat.puc-rio.br; Sun, 8 Sep 2002 19:27:36 -0300 Message-ID: <001101c25787$84434dc0$9f06c3c8@lba> From: "Leonardo Borges Avelino" To: Subject: [obm-l] =?iso-8859-1?Q?En:_Quest=E3o_de_Geometria?= Date: Sun, 8 Sep 2002 19:31:44 -0300 MIME-Version: 1.0 Content-Type: multipart/alternative; boundary="----=_NextPart_000_000E_01C2576E.5D9EC320" X-Priority: 3 X-MSMail-Priority: Normal X-Mailer: Microsoft Outlook Express 5.00.2615.200 X-MimeOLE: Produced By Microsoft MimeOLE V5.00.2615.200 X-Sender-IP: 200.195.6.159 Sender: owner-obm-l@sucuri.mat.puc-rio.br Precedence: bulk Reply-To: obm-l@mat.puc-rio.br This is a multi-part message in MIME format. ------=_NextPart_000_000E_01C2576E.5D9EC320 Content-Type: text/plain; charset="iso-8859-1" Content-Transfer-Encoding: quoted-printable ----- Original Message -----=20 From: Leonardo Borges Avelino=20 To: obm-l@mat.puc-rio.br=20 Sent: Friday, September 06, 2002 10:56 AM Subject: Quest=E3o de Geometria Ei pessoal Acho que todos conhecem a quest=E3o de geometria que existe uma = circunfer=EAncia e duas tangentes. As tangentes se encontram num ponto = P. Ligamos os pontos de tang=EAncia que determinam os pontos A e B. Do = segmento AB levantamos uma perpendicular que toca a circunfer=EAncia no = ponto C. Dados os valores das dist=E2ncias C at=E9 a reta AP; e de C = at=E9 BP. Calcule o valor da perpendicular. (N=E3o est=E1 muito claro de entender o enunciado pois eu escrevi com = pressa.) Essa quest=E3o jah caiu no Colegio Naval/2001 e me disseram que ela caiu = no IME, e tamb=E9m ela est=E1 num livrinho preto do prof. Eduardo = Wagner. Para todos que conhecem tal quest=E3o e principalmente ao Prof. Eduardo = Wagner, pergunto: * Essa quest=E3o eh n=EDvel IMO? * Quem eh o autor dessa quest=E3o? Valeu!! Leonardo Borges Avelino ------=_NextPart_000_000E_01C2576E.5D9EC320 Content-Type: text/html; charset="iso-8859-1" Content-Transfer-Encoding: quoted-printable
 
----- Original Message -----=20
From: Leonardo=20 Borges Avelino
Sent: Friday, September 06, 2002 10:56 AM
Subject: Quest=E3o de Geometria

Ei pessoal
Acho que todos conhecem a quest=E3o de = geometria que=20 existe uma circunfer=EAncia e duas tangentes. As tangentes se encontram = num ponto=20 P. Ligamos os pontos de tang=EAncia que determinam os pontos A e B. = Do=20 segmento AB levantamos uma  perpendicular que toca a = circunfer=EAncia no=20 ponto C. Dados os valores das dist=E2ncias C at=E9 a reta AP; e de C = at=E9 BP. Calcule=20 o valor da perpendicular.
(N=E3o est=E1 muito claro de entender o = enunciado pois=20 eu escrevi com pressa.)
 
Essa quest=E3o jah caiu no Colegio = Naval/2001 e me=20 disseram que ela caiu no IME, e tamb=E9m ela est=E1 num livrinho preto = do prof.=20 Eduardo Wagner.
Para todos que conhecem tal quest=E3o e = principalmente ao Prof. Eduardo Wagner, pergunto:
* Essa quest=E3o eh n=EDvel = IMO?
* Quem eh o autor dessa = quest=E3o?
 
Valeu!!
Leonardo Borges = Avelino
------=_NextPart_000_000E_01C2576E.5D9EC320-- ========================================================================= Instruções para entrar na lista, sair da lista e usar a lista em http://www.mat.puc-rio.br/~nicolau/olimp/obm-l.html O administrador desta lista é ========================================================================= From owner-obm-l@sucuri.mat.puc-rio.br Sun Sep 8 20:23:06 2002 Return-Path: Received: (from majordom@localhost) by sucuri.mat.puc-rio.br (8.9.3/8.9.3) id UAA11512 for obm-l-MTTP; Sun, 8 Sep 2002 20:22:20 -0300 Received: from artemis.opendf.com.br (artemis.opengate.com.br [200.181.71.15]) by sucuri.mat.puc-rio.br (8.9.3/8.9.3) with ESMTP id UAA11508 for ; Sun, 8 Sep 2002 20:22:17 -0300 Received: from localhost (localhost [127.0.0.1]) by artemis.opendf.com.br (Postfix) with ESMTP id 5F36E1C833 for ; Sun, 8 Sep 2002 20:22:36 -0300 (BRT) Received: from computer (200-181-90-211-bsace7001.dsl.telebrasilia.net.br [200.181.90.211]) by artemis.opendf.com.br (Postfix) with ESMTP id 2787C1BF90 for ; Sun, 8 Sep 2002 20:22:31 -0300 (BRT) From: "Artur Costa Steiner" To: Subject: RE: [obm-l] violencia Date: Sun, 8 Sep 2002 20:23:02 -0300 Organization: Steiner Consultoria LTDA Message-ID: <000c01c2578e$afdf8870$9865fea9@computer> MIME-Version: 1.0 Content-Type: multipart/alternative; boundary="----=_NextPart_000_000D_01C25775.8A925070" X-Priority: 3 (Normal) X-MSMail-Priority: Normal X-Mailer: Microsoft Outlook, Build 10.0.2627 X-MimeOLE: Produced By Microsoft MimeOLE V6.00.2600.0000 Importance: Normal In-Reply-To: <007201c25780$87e57ef0$0401010a@xt> X-Virus-Scanned: by AMaViS new-20020517 Sender: owner-obm-l@sucuri.mat.puc-rio.br Precedence: bulk Reply-To: obm-l@mat.puc-rio.br This is a multi-part message in MIME format. ------=_NextPart_000_000D_01C25775.8A925070 Content-Type: text/plain; charset="iso-8859-1" Content-Transfer-Encoding: quoted-printable =20 =20 =20 =20 =20 =20 > Oi Rog=E9rio > Acho que n=E3o saquei. Em que momento foi utilizado o axioma da = escolha? Eu > nem tinha infinitos conjuntos! Apenas conjuntos infinitos. =20 Eu acho que voc=EA est=E1 certo. O axioma da escolha (a menos que eu esteja com um conceito equivocado) diz que, dada uma cole=E7=E3o infinita de conjuntos disjuntos, podemos formar um conjunto E escolhendo-se precisamente um elemento de cada conjunto da cole=E7=E3o. Isto n=E3o foi usado na sua prova. =20 Acho ainda interessante fazer o seguinte coment=E1rio: No contra-exemplo que vc deu para a parte 2 da quest=E3o, observamos que sendo B_n o = n-=E9simo bandido, ent=E3o ele quer matar n-1 outros. Logo, sendo (M_n), a = sequencia dos n=FAmeros de bandidos que cada um quer matar, esta sequencia =E9 ilimitada. Se admitirmos que tal sequencia seja limitada, isto =E9, se considerarmos esta condi=E7=E3o adicional, ent=E3o me parece que sua = prova permanece v=E1lida. Pois para cada passo de seu processo indutivo, o conjunto dos elementos de C que os elementos de R querem matar continua sendo finito. Acho que neste caso o seu processo acaba sendo equivalente ao que eu havia sugerido.=20 =20 Agora, se a sequencia M_n for ilimitada, et=E3o pode n=E3o ser = poss=EDvel achar uma reuni=E3o sem sangue, conforme mostra seu contra exemplo. = N=E3o estou por=E9m certo se, neste caso, =E9 sempre imposs=EDvel achar a tal reuni=E3o.=20 =20 Um abra=E7o Artur ------=_NextPart_000_000D_01C25775.8A925070 Content-Type: text/html; charset="iso-8859-1" Content-Transfer-Encoding: quoted-printable

 

 

 

 


=A0

 

> =A0Oi = Rog=E9rio

> Acho que n=E3o = saquei. Em que momento foi utilizado o axioma da escolha? Eu

> nem tinha infinitos conjuntos! Apenas conjuntos = infinitos.

 

Eu acho que voc=EA = est=E1 certo.

O axioma da escolha (a menos que eu esteja com um conceito equivocado) diz que, = dada uma cole=E7=E3o infinita de conjuntos disjuntos, podemos formar um conjunto = E escolhendo-se precisamente =A0um = elemento de cada conjunto da cole=E7=E3o. =A0Isto n=E3o foi usado na sua prova.

 = ;

Acho ainda interessante fazer o seguinte coment=E1rio: No contra-exemplo que = vc deu para a parte 2 da quest=E3o, observamos que sendo B_n o n-=E9simo = bandido, ent=E3o ele quer matar n-1 outros. Logo, sendo (M_n), a sequencia dos n=FAmeros = de bandidos que cada um quer matar, esta sequencia =E9 ilimitada. Se admitirmos que = tal sequencia seja limitada, isto =E9, se considerarmos esta condi=E7=E3o = adicional, ent=E3o me parece que sua prova permanece v=E1lida. Pois para cada passo de seu = processo indutivo, o conjunto dos elementos de C que os elementos de R querem = matar continua sendo finito. Acho que neste caso o seu processo acaba sendo equivalente ao que eu havia sugerido.

 = ;

Agora, se a sequencia M_n for ilimitada, et=E3o pode n=E3o ser poss=EDvel achar = uma reuni=E3o sem sangue, conforme mostra seu contra exemplo. N=E3o estou por=E9m = certo se, neste caso, =E9 sempre imposs=EDvel achar a tal reuni=E3o. =

 = ;

Um = abra=E7o

Artur=

------=_NextPart_000_000D_01C25775.8A925070-- ========================================================================= Instruções para entrar na lista, sair da lista e usar a lista em http://www.mat.puc-rio.br/~nicolau/olimp/obm-l.html O administrador desta lista é ========================================================================= From owner-obm-l@sucuri.mat.puc-rio.br Sun Sep 8 21:16:58 2002 Return-Path: Received: (from majordom@localhost) by sucuri.mat.puc-rio.br (8.9.3/8.9.3) id VAA12256 for obm-l-MTTP; Sun, 8 Sep 2002 21:16:48 -0300 Received: from riemann ([200.222.249.58]) by sucuri.mat.puc-rio.br (8.9.3/8.9.3) with ESMTP id VAA12252 for ; Sun, 8 Sep 2002 21:16:41 -0300 Received: from fabiodias by riemann with local (Exim 3.35 #1 (Debian)) id 17oCFC-0000AC-00 for ; Sun, 08 Sep 2002 21:17:34 -0300 Date: Sun, 8 Sep 2002 21:17:34 -0300 To: obm-l@mat.puc-rio.br Subject: Re: [obm-l] violencia Message-ID: <20020909001733.GB579@ieg.com.br> Mail-Followup-To: obm-l@mat.puc-rio.br References: <007201c25780$87e57ef0$0401010a@xt> <000c01c2578e$afdf8870$9865fea9@computer> Mime-Version: 1.0 Content-Type: multipart/signed; micalg=pgp-sha1; protocol="application/pgp-signature"; boundary="oLBj+sq0vYjzfsbl" Content-Disposition: inline In-Reply-To: <000c01c2578e$afdf8870$9865fea9@computer> User-Agent: Mutt/1.3.28i From: Fabio Dias Moreira Sender: owner-obm-l@sucuri.mat.puc-rio.br Precedence: bulk Reply-To: obm-l@mat.puc-rio.br --oLBj+sq0vYjzfsbl Content-Type: text/plain; charset=unknown-8bit Content-Disposition: inline Content-Transfer-Encoding: quoted-printable On Sun, Sep 08, 2002 at 08:23:02PM -0300, Artur Costa Steiner wrote: >=20 > [...] >=20 > Agora, se a sequencia M_n for ilimitada, et?o pode n?o ser poss?vel > achar uma reuni?o sem sangue, conforme mostra seu contra exemplo. N?o > estou por?m certo se, neste caso, ? sempre imposs?vel achar a tal > reuni?o.=20 > =20 > [...] > EMmalguns casos =E9 poss=EDvel. Crie duas sequ=EAncias de bandidos, a_n e b= _n. a_i odeia a_m, m ========================================================================= From owner-obm-l@sucuri.mat.puc-rio.br Sun Sep 8 21:21:06 2002 Return-Path: Received: (from majordom@localhost) by sucuri.mat.puc-rio.br (8.9.3/8.9.3) id VAA12364 for obm-l-MTTP; Sun, 8 Sep 2002 21:21:03 -0300 Received: from ginsberg.uol.com.br (ginsberg.uol.com.br [200.221.4.48]) by sucuri.mat.puc-rio.br (8.9.3/8.9.3) with ESMTP id VAA12359 for ; Sun, 8 Sep 2002 21:21:00 -0300 Received: from u2z7z2 ([200.158.144.156]) by ginsberg.uol.com.br (8.9.1/8.9.1) with ESMTP id VAA29594 for ; Sun, 8 Sep 2002 21:20:15 -0300 (BRT) Message-ID: <000901c25797$0b3201a0$9c909ec8@u2z7z2> From: "Wagner" To: References: <001101c25787$84434dc0$9f06c3c8@lba> Subject: [obm-l] =?iso-8859-1?Q?Re:_=5Bobm-l=5D_En:_Quest=E3o_de_Geometria?= Date: Sun, 8 Sep 2002 21:22:54 -0300 Organization: Wagner MIME-Version: 1.0 Content-Type: multipart/alternative; boundary="----=_NextPart_000_0006_01C2577D.E556E180" X-Priority: 3 X-MSMail-Priority: Normal X-Mailer: Microsoft Outlook Express 5.50.4133.2400 X-MimeOLE: Produced By Microsoft MimeOLE V5.50.4133.2400 Sender: owner-obm-l@sucuri.mat.puc-rio.br Precedence: bulk Reply-To: obm-l@mat.puc-rio.br This is a multi-part message in MIME format. ------=_NextPart_000_0006_01C2577D.E556E180 Content-Type: text/plain; charset="iso-8859-1" Content-Transfer-Encoding: quoted-printable Caro Leonardo Aparentemente faltam dados para a resolu=E7=E3o, dada uma = circunfer=EAncia qualquer =E9 poss=EDvel obter uma dist=E2ncia C pelo = menos menor que o raio simplesmente variando o =E2ngulo com que as = tangentes se encontram.O tri=E2ngulo ABP =E9 is=F3celes e C =E9 um ponto = qualquer da altura desse tri=E2ngulo, logo variando B=C2P ou qualquer = outro =E2ngulo, a dist=E2ncia de C ao lado muda. ex: B=C2P=3D0 = (tangentes coincidentes) e=20 B=C2P > 0. Andr=E9 T. ----- Original Message -----=20 From: Leonardo Borges Avelino=20 To: obm-l@mat.puc-rio.br=20 Sent: Sunday, September 08, 2002 7:31 PM Subject: [obm-l] En: Quest=E3o de Geometria ----- Original Message -----=20 From: Leonardo Borges Avelino=20 To: obm-l@mat.puc-rio.br=20 Sent: Friday, September 06, 2002 10:56 AM Subject: Quest=E3o de Geometria Ei pessoal Acho que todos conhecem a quest=E3o de geometria que existe uma = circunfer=EAncia e duas tangentes. As tangentes se encontram num ponto = P. Ligamos os pontos de tang=EAncia que determinam os pontos A e B. Do = segmento AB levantamos uma perpendicular que toca a circunfer=EAncia no = ponto C. Dados os valores das dist=E2ncias C at=E9 a reta AP; e de C = at=E9 BP. Calcule o valor da perpendicular. (N=E3o est=E1 muito claro de entender o enunciado pois eu escrevi com = pressa.) Essa quest=E3o jah caiu no Colegio Naval/2001 e me disseram que ela = caiu no IME, e tamb=E9m ela est=E1 num livrinho preto do prof. Eduardo = Wagner. Para todos que conhecem tal quest=E3o e principalmente ao Prof. = Eduardo Wagner, pergunto: * Essa quest=E3o eh n=EDvel IMO? * Quem eh o autor dessa quest=E3o? Valeu!! Leonardo Borges Avelino ------=_NextPart_000_0006_01C2577D.E556E180 Content-Type: text/html; charset="iso-8859-1" Content-Transfer-Encoding: quoted-printable
Caro Leonardo
 
Aparentemente faltam dados para a = resolu=E7=E3o, dada=20 uma circunfer=EAncia qualquer =E9 poss=EDvel obter uma dist=E2ncia = C pelo=20 menos menor que o raio simplesmente variando o =E2ngulo com = que as=20 tangentes se encontram.O tri=E2ngulo ABP =E9 is=F3celes e C =E9 um ponto = qualquer da=20 altura desse tri=E2ngulo, logo variando B=C2P ou qualquer outro = =E2ngulo, a dist=E2ncia=20 de C ao lado muda. ex: B=C2P=3D0 (tangentes coincidentes) e =
B=C2P > 0.
 
Andr=E9 T.
 
 
----- Original Message -----
From:=20 Leonardo=20 Borges Avelino
Sent: Sunday, September 08, = 2002 7:31=20 PM
Subject: [obm-l] En: Quest=E3o = de=20 Geometria

 
----- Original Message -----=20
From: Leonardo=20 Borges Avelino
Sent: Friday, September 06, 2002 10:56 AM
Subject: Quest=E3o de Geometria

Ei pessoal
Acho que todos conhecem a quest=E3o = de geometria=20 que existe uma circunfer=EAncia e duas tangentes. As tangentes se = encontram num=20 ponto P. Ligamos os pontos de tang=EAncia que determinam os = pontos A e B.=20 Do segmento AB levantamos uma  perpendicular que toca a = circunfer=EAncia no=20 ponto C. Dados os valores das dist=E2ncias C at=E9 a reta AP; e de C = at=E9 BP.=20 Calcule o valor da perpendicular.
(N=E3o est=E1 muito claro de entender = o enunciado=20 pois eu escrevi com pressa.)
 
Essa quest=E3o jah caiu no Colegio = Naval/2001 e me=20 disseram que ela caiu no IME, e tamb=E9m ela est=E1 num livrinho preto = do prof.=20 Eduardo Wagner.
Para todos que conhecem tal quest=E3o = e=20 principalmente ao Prof. Eduardo Wagner, pergunto:
* Essa quest=E3o eh n=EDvel = IMO?
* Quem eh o autor dessa = quest=E3o?
 
Valeu!!
Leonardo Borges=20 Avelino
------=_NextPart_000_0006_01C2577D.E556E180-- ========================================================================= Instruções para entrar na lista, sair da lista e usar a lista em http://www.mat.puc-rio.br/~nicolau/olimp/obm-l.html O administrador desta lista é ========================================================================= From owner-obm-l@sucuri.mat.puc-rio.br Sun Sep 8 21:27:37 2002 Return-Path: Received: (from majordom@localhost) by sucuri.mat.puc-rio.br (8.9.3/8.9.3) id VAA12541 for obm-l-MTTP; Sun, 8 Sep 2002 21:27:21 -0300 Received: from smtp.ieg.com.br (stone.protocoloweb.com.br [200.226.139.11]) by sucuri.mat.puc-rio.br (8.9.3/8.9.3) with ESMTP id VAA12537 for ; Sun, 8 Sep 2002 21:27:18 -0300 Received: from dois (200-161-154-154.dsl.telesp.net.br [200.161.154.154]) by smtp.ieg.com.br (IeG relay/8.9.3) with SMTP id g890O2O3043628 for ; Sun, 8 Sep 2002 21:24:03 -0300 (BRT) Message-ID: <000201c25798$4d8061e0$0200a8c0@dois> From: =?iso-8859-1?Q?Gabriel_P=E9rgola?= To: References: <002401c255a5$a48a7f60$0201a8c0@x> Subject: Re: [obm-l] um sistema Date: Sun, 8 Sep 2002 19:47:18 -0300 MIME-Version: 1.0 Content-Type: text/plain; charset="iso-8859-1" Content-Transfer-Encoding: 8bit X-Priority: 3 X-MSMail-Priority: Normal X-Mailer: Microsoft Outlook Express 5.50.4807.1700 X-MimeOLE: Produced By Microsoft MimeOLE V5.50.4807.1700 Sender: owner-obm-l@sucuri.mat.puc-rio.br Precedence: bulk Reply-To: obm-l@mat.puc-rio.br Sabendo que ax-by=1 e que ay+bx=0, prove que x= a/a^2 +b^2 e y = -b/a^2+b^2 Resolução pelo teorema de Cramer: D = | a -b | = a² + b² | b a | Dx = | -b 1 | = - a | a 0 | Dy = | a 1 | = -b | b 0 | logo: x = Dx/D = -a/a² + b² y = Dy/D= -b/a² + b² a resposta do x deu diferente do enunciado. Minha resoluçao ou o enunciado que está incorreto? Gabriel ========================================================================= Instruções para entrar na lista, sair da lista e usar a lista em http://www.mat.puc-rio.br/~nicolau/olimp/obm-l.html O administrador desta lista é ========================================================================= From owner-obm-l@sucuri.mat.puc-rio.br Sun Sep 8 21:58:43 2002 Return-Path: Received: (from majordom@localhost) by sucuri.mat.puc-rio.br (8.9.3/8.9.3) id VAA13978 for obm-l-MTTP; Sun, 8 Sep 2002 21:58:33 -0300 Received: from shen.bol.com.br (shen.bol.com.br [200.221.24.14]) by sucuri.mat.puc-rio.br (8.9.3/8.9.3) with ESMTP id VAA13974 for ; Sun, 8 Sep 2002 21:58:30 -0300 Received: from bol.com.br (200.221.24.138) by shen.bol.com.br (5.1.071) id 3D63D22F005F833D for obm-l@mat.puc-rio.br; Sun, 8 Sep 2002 21:58:43 -0300 Date: Sun, 8 Sep 2002 21:57:37 -0300 Message-Id: Subject: Re:[obm-l] circuito IME MIME-Version: 1.0 Content-Type: text/plain;charset="iso-8859-1" From: "adr.scr.m" To: obm-l@mat.puc-rio.br X-XaM3-API-Version: 2.4.3.4.4 X-SenderIP: 200.151.58.237 Content-Transfer-Encoding: 8bit X-MIME-Autoconverted: from quoted-printable to 8bit by sucuri.mat.puc-rio.br id VAA13975 Sender: owner-obm-l@sucuri.mat.puc-rio.br Precedence: bulk Reply-To: obm-l@mat.puc-rio.br Na questao 10 foi necessario o emprego das equacoes do circuito RC no regime transitorio,equacoes que nao pentencem ao programa. []'s. Adriano. __________________________________________________________________________ AcessoBOL, só R$ 9,90! O menor preço do mercado! Assine já! http://www.bol.com.br/acessobol ========================================================================= Instruções para entrar na lista, sair da lista e usar a lista em http://www.mat.puc-rio.br/~nicolau/olimp/obm-l.html O administrador desta lista é ========================================================================= From owner-obm-l@sucuri.mat.puc-rio.br Sun Sep 8 22:07:27 2002 Return-Path: Received: (from majordom@localhost) by sucuri.mat.puc-rio.br (8.9.3/8.9.3) id WAA14212 for obm-l-MTTP; Sun, 8 Sep 2002 22:07:22 -0300 Received: from smtp.ieg.com.br (sharon.protocoloweb.com.br [200.226.139.12]) by sucuri.mat.puc-rio.br (8.9.3/8.9.3) with ESMTP id WAA14208 for ; Sun, 8 Sep 2002 22:07:19 -0300 Received: from localhost (200-158-118-125.dsl.telesp.net.br [200.158.118.125]) by smtp.ieg.com.br (IeG relay/8.9.3) with SMTP id g8919Zfx026970 for ; Sun, 8 Sep 2002 22:09:36 -0300 (BRT) From: Tonik To: obm-l@mat.puc-rio.br Date: Sun, 08 Sep 2002 22:07:25 -0300 X-Priority: 3 (Normal) Organization: Tonik In-Reply-To: <000201c25798$4d8061e0$0200a8c0@dois> Message-Id: <62HC531WHFFE5ZIF98WTNJHC05FDVQ0.3d7bf44d@localhost> Subject: Re: [obm-l] um sistema MIME-Version: 1.0 Content-Type: text/plain; charset="iso-8859-1" X-Mailer: Opera 6.04 build 1135 Sender: owner-obm-l@sucuri.mat.puc-rio.br Precedence: bulk Reply-To: obm-l@mat.puc-rio.br Quando um problema pede as solucoes complexas, ele pede tambem as reais ou somente as solucoes pertencentes aos Complexos nao pertencentes aos reais? >Sabendo que ax-by=1 e que ay+bx=0, prove que x= a/a^2 +b^2 e y = -b/a^2+b^2 xa-yb=1 xb+ya=0 [a -b 1] [b a 0] D=a^2+b^2 Dx=a Dy=-b x=a/(a^2+b^2) y=-b/(a^2+b^2) ========================================================================= Instruções para entrar na lista, sair da lista e usar a lista em http://www.mat.puc-rio.br/~nicolau/olimp/obm-l.html O administrador desta lista é ========================================================================= From owner-obm-l@sucuri.mat.puc-rio.br Sun Sep 8 22:14:26 2002 Return-Path: Received: (from majordom@localhost) by sucuri.mat.puc-rio.br (8.9.3/8.9.3) id WAA14431 for obm-l-MTTP; Sun, 8 Sep 2002 22:14:18 -0300 Received: from shen.bol.com.br (shen.bol.com.br [200.221.24.14]) by sucuri.mat.puc-rio.br (8.9.3/8.9.3) with ESMTP id WAA14426 for ; Sun, 8 Sep 2002 22:14:15 -0300 Received: from bol.com.br (200.221.24.138) by shen.bol.com.br (5.1.071) id 3D63D22F005F91C7 for obm-l@mat.puc-rio.br; Sun, 8 Sep 2002 22:14:28 -0300 Date: Sun, 8 Sep 2002 22:13:22 -0300 Message-Id: Subject: [obm-l] geometria MIME-Version: 1.0 Content-Type: text/plain;charset="iso-8859-1" From: "adr.scr.m" To: obm-l@mat.puc-rio.br X-XaM3-API-Version: 2.4.3.4.4 X-SenderIP: 200.151.58.237 Content-Transfer-Encoding: 8bit X-MIME-Autoconverted: from quoted-printable to 8bit by sucuri.mat.puc-rio.br id WAA14427 Sender: owner-obm-l@sucuri.mat.puc-rio.br Precedence: bulk Reply-To: obm-l@mat.puc-rio.br alguem poderia fazer essas questoes,por favor, 1-Sejam A,B e C nesta ordem sobre uma reta tais que AB=12 e BC=3.Seja D conjugado harmonico de B em relacao ao segmento AO. Entao BD mede : 2-Os pontos A,M,B e N de reta formam uma divisao harmonica de razao MA/MB = NA/NB = k .Se J eh o ponto medio de MN, a razao JA/JB,vale : 3-Em um triangulo ABC de lados AB=12,AC=8 e BC=10,a bissetriz interna de B encontra a bissetriz AN externa de A no ponto F. A razao FN/FA vale : []'s. Adriano. __________________________________________________________________________ AcessoBOL, só R$ 9,90! O menor preço do mercado! Assine já! http://www.bol.com.br/acessobol ========================================================================= Instruções para entrar na lista, sair da lista e usar a lista em http://www.mat.puc-rio.br/~nicolau/olimp/obm-l.html O administrador desta lista é ========================================================================= From owner-obm-l@sucuri.mat.puc-rio.br Mon Sep 9 08:04:21 2002 Return-Path: Received: (from majordom@localhost) by sucuri.mat.puc-rio.br (8.9.3/8.9.3) id HAA16045 for obm-l-MTTP; Mon, 9 Sep 2002 07:46:51 -0300 Received: from hotmail.com (f212.pav2.hotmail.com [64.4.37.212]) by sucuri.mat.puc-rio.br (8.9.3/8.9.3) with ESMTP id HAA16041 for ; Mon, 9 Sep 2002 07:46:47 -0300 Received: from mail pickup service by hotmail.com with Microsoft SMTPSVC; Sun, 8 Sep 2002 19:17:31 -0700 Received: from 200.199.193.135 by pv2fd.pav2.hotmail.msn.com with HTTP; Mon, 09 Sep 2002 02:17:30 GMT X-Originating-IP: [200.199.193.135] From: "ricardo matos" To: obm-l@mat.puc-rio.br Subject: [obm-l] =?iso-8859-1?B?UmU6IFtvYm0tbF0gUmU6IFtvYm0tbF0gRW46IFF1ZXN0428gZGUgR2Vv?= =?iso-8859-1?B?bWV0cmlh?= Date: Sun, 08 Sep 2002 23:17:30 -0300 Mime-Version: 1.0 Content-Type: text/plain; charset=iso-8859-1; format=flowed Message-ID: X-OriginalArrivalTime: 09 Sep 2002 02:17:31.0113 (UTC) FILETIME=[0D1FAD90:01C257A7] Sender: owner-obm-l@sucuri.mat.puc-rio.br Precedence: bulk Reply-To: obm-l@mat.puc-rio.br >From: "Wagner" >Reply-To: obm-l@mat.puc-rio.br >To: >Subject: [obm-l] Re: [obm-l] En: Questão de Geometria >Date: Sun, 8 Sep 2002 21:22:54 -0300 > >Caro Leonardo > >Aparentemente faltam dados para a resolução, dada uma circunferência >qualquer é possível obter uma distância C pelo menos menor que o raio >simplesmente variando o ângulo com que as tangentes se encontram.O >triângulo ABP é isóceles e C é um ponto qualquer da altura desse triângulo, >logo variando BÂP ou qualquer outro ângulo, a distância de C ao lado muda. >ex: BÂP=0 (tangentes coincidentes) e >BÂP > 0. > >André T. > > > ----- Original Message ----- > From: Leonardo Borges Avelino > To: obm-l@mat.puc-rio.br > Sent: Sunday, September 08, 2002 7:31 PM > Subject: [obm-l] En: Questão de Geometria > > > > ----- Original Message ----- > From: Leonardo Borges Avelino > To: obm-l@mat.puc-rio.br > Sent: Friday, September 06, 2002 10:56 AM > Subject: Questão de Geometria > > > Ei pessoal > Acho que todos conhecem a questão de geometria que existe uma >circunferência e duas tangentes. As tangentes se encontram num ponto P. >Ligamos os pontos de tangência que determinam os pontos A e B. Do segmento >AB levantamos uma perpendicular que toca a circunferência no ponto C. >Dados os valores das distâncias C até a reta AP; e de C até BP. Calcule o >valor da perpendicular. > (Não está muito claro de entender o enunciado pois eu escrevi com >pressa.) > > Essa questão jah caiu no Colegio Naval/2001 e me disseram que ela caiu >no IME, e também ela está num livrinho preto do prof. Eduardo Wagner. > Para todos que conhecem tal questão e principalmente ao Prof. Eduardo >Wagner, pergunto: > * Essa questão eh nível IMO? > * Quem eh o autor dessa questão? > > Valeu!! > Leonardo Borges Avelino Não entendi o que vc falou direito, C não precisa estar na altura, já vi esta questão uma vez e o problema é de fato resolúvel apesar da circunferência poder variar. Aqui vai a resolução sejam: M o pé da perpendicular à AP por C N o pé da perpendicular à PB por C Q o pé da perpendicular à AB por C (desculpa mas meu computador não coloca circunflexo em cima de consoante e prefiro manter a notação) Claro que PBA=PAB, agora CBP é o ângulo secante do arco BC então CBP=CAB da mesma forma CBA=PAC. Isto que implica que os triângulos BNC é semelhante à AQC e AMC é semelhante à BQC(Pelo caso AA) logo: BC/CA=NC/QC e CA/BC=MC/QC então multiplicando: 1=NC*MC/(QC*QC) e QC^2=NC*MC e acabou! prefiro pensar que quando mostramos a semelhança dos triângulos retângulos estamos mostrando que os quadiláteros BQCN e AMCQ são semelhantes(nesta ordem) e então CN/CQ=CQ/CM mais diretamente. Genial não?, Na verdade tem um pequeno detalhe faltando. É que na resolução supus que o ponto C está dentro de ABP se colocarmos C no círculo só que externamente temos outro problema(no enunciado original não fica claro pois depende de como se desenhou a figura na parte do levanta-se um perpendicular). Este outro problema tem solução com a mesma relação e solução similar. ========================================================================= Instruções para entrar na lista, sair da lista e usar a lista em http://www.mat.puc-rio.br/~nicolau/olimp/obm-l.html O administrador desta lista é ========================================================================= From owner-obm-l@sucuri.mat.puc-rio.br Mon Sep 9 08:13:03 2002 Return-Path: Received: (from majordom@localhost) by sucuri.mat.puc-rio.br (8.9.3/8.9.3) id IAA16531 for obm-l-MTTP; Mon, 9 Sep 2002 08:09:07 -0300 Received: from mediterraneo.rjnet.com.br (mediterraneo.rjnet.com.br [200.222.31.30]) by sucuri.mat.puc-rio.br (8.9.3/8.9.3) with ESMTP id IAA16525 for ; Mon, 9 Sep 2002 08:09:04 -0300 Received: from locutus.rjnet.com.br (root@locutus.rjnet.com.br [200.222.31.10]) by mediterraneo.rjnet.com.br (8.11.4/8.11.4) with ESMTP id g8933PC11693 for ; Mon, 9 Sep 2002 00:03:25 -0300 Received: from felipe ([200.222.39.86]) by locutus.rjnet.com.br (8.11.2/8.11.2) with SMTP id g892hb326626 for ; Sun, 8 Sep 2002 23:43:37 -0300 Message-ID: <000901c257ad$55d709b0$158c000a@felipe> From: "Felipe Villela Dias" To: References: <002401c255a5$a48a7f60$0201a8c0@x> <000201c25798$4d8061e0$0200a8c0@dois> Subject: Re: [obm-l] um sistema Date: Mon, 9 Sep 2002 00:02:29 -0300 MIME-Version: 1.0 Content-Type: text/plain; charset="iso-8859-1" Content-Transfer-Encoding: 8bit X-Priority: 3 X-MSMail-Priority: Normal X-Mailer: Microsoft Outlook Express 6.00.2600.0000 X-MimeOLE: Produced By Microsoft MimeOLE V6.00.2600.0000 Sender: owner-obm-l@sucuri.mat.puc-rio.br Precedence: bulk Reply-To: obm-l@mat.puc-rio.br Gabriel, não sou um bom matemático e pode ser que eu esteja falando besteira, mas pelo teorema de Cramer você deveria substituir os valores a direita da igualdade na coluna onde estão os coeficientes que multiplicam a viaravel desejada não? Assim: Dx = | 1 b | , assim Dx/D = a / a² + b² , como diz o enunciado do problema. | 0 a | Um abraço e desculpas a todos se tiver dito besteira. ----- Original Message ----- From: "Gabriel Pérgola" To: Sent: Sunday, September 08, 2002 7:47 PM Subject: Re: [obm-l] um sistema > Sabendo que ax-by=1 e que ay+bx=0, prove que x= a/a^2 +b^2 e y = -b/a^2+b^2 > > > Resolução pelo teorema de Cramer: > > D = | a -b | = a² + b² > | b a | > > Dx = | -b 1 | = - a > | a 0 | > > Dy = | a 1 | = -b > | b 0 | > > > logo: > > x = Dx/D = -a/a² + b² > y = Dy/D= -b/a² + b² > > a resposta do x deu diferente do enunciado. Minha resoluçao ou o enunciado > que está incorreto? > > Gabriel > > ========================================================================= > Instruções para entrar na lista, sair da lista e usar a lista em > http://www.mat.puc-rio.br/~nicolau/olimp/obm-l.html > O administrador desta lista é > ========================================================================= > --- Outgoing mail is certified Virus Free. Checked by AVG anti-virus system (http://www.grisoft.com). Version: 6.0.385 / Virus Database: 217 - Release Date: 5/9/2002 ========================================================================= Instruções para entrar na lista, sair da lista e usar a lista em http://www.mat.puc-rio.br/~nicolau/olimp/obm-l.html O administrador desta lista é ========================================================================= From owner-obm-l@sucuri.mat.puc-rio.br Mon Sep 9 09:48:41 2002 Return-Path: Received: (from majordom@localhost) by sucuri.mat.puc-rio.br (8.9.3/8.9.3) id JAA20646 for obm-l-MTTP; Mon, 9 Sep 2002 09:36:19 -0300 Received: from smtp-5.ig.com.br (smtp-5.ig.com.br [200.226.132.154]) by sucuri.mat.puc-rio.br (8.9.3/8.9.3) with SMTP id JAA20642 for ; Mon, 9 Sep 2002 09:36:16 -0300 Received: (qmail 7602 invoked from network); 9 Sep 2002 12:36:32 -0000 Received: from shasta009167.ig.com.br (HELO ig) (200.151.9.167) by smtp-5.ig.com.br with SMTP; 9 Sep 2002 12:36:32 -0000 Message-ID: <002b01c25734$bcc2e3c0$a70997c8@ig> From: =?iso-8859-1?Q?Andr=E9_Silva?= To: Subject: En: [obm-l] Circulo de 9 pontos e reta de Simson Date: Sun, 8 Sep 2002 09:39:12 -0300 MIME-Version: 1.0 Content-Type: text/plain; charset="iso-8859-1" Content-Transfer-Encoding: 8bit X-Priority: 3 X-MSMail-Priority: Normal X-Mailer: Microsoft Outlook Express 5.00.2615.200 X-MIMEOLE: Produced By Microsoft MimeOLE V5.00.2615.200 Sender: owner-obm-l@sucuri.mat.puc-rio.br Precedence: bulk Reply-To: obm-l@mat.puc-rio.br ----- Original Message ----- From: André Silva To: Sent: Sunday, September 08, 2002 9:38 AM Subject: En: [obm-l] Circulo de 9 pontos e reta de Simson > > Leonardo, a alguns dias te respondi sobre inversão e números > complexos. Esqueci de mencionar o livro Complex Numbers & Geometry de Hahn > Ling-shin (Mathematical Association of America - encontrei no IM-UFRJ). Este > talvez te ajude, tanto para aquele assunto, como para o circulo de 9 pontos > e a reta de Simson. Outra boa fonte é a RPM que não lembro o número, mas tem > uma demonstração do círculo de 9 pontos! Gosto muito desses assuntos. Se > estiver escrevendo sobre algo, mantenha contato gostaria de trocar idéias. > > Abraço, André. > > > ----- Original Message ----- > > From: leonardo mattos > > To: > > Sent: Friday, September 06, 2002 10:51 PM > > Subject: [obm-l] Circulo de 9 pontos e reta de Simson > > > > > > > Ola pessoal, > > > > > > Gostaria muito de saber quais seriam as propriedades > > > Um abraço,Leonardo > > > > > > > > > > > > > > > > > > _________________________________________________________________ > > > MSN Photos é a maneira mais fácil e prática de editar e compartilhar sua > > > fotos: http://photos.msn.com.br > > > > > > > ========================================================================= > > > Instruções para entrar na lista, sair da lista e usar a lista em > > > http://www.mat.puc-rio.br/~nicolau/olimp/obm-l.html > > > O administrador desta lista é > > > > ========================================================================= > > > > > > ========================================================================= Instruções para entrar na lista, sair da lista e usar a lista em http://www.mat.puc-rio.br/~nicolau/olimp/obm-l.html O administrador desta lista é ========================================================================= From owner-obm-l@sucuri.mat.puc-rio.br Mon Sep 9 10:28:34 2002 Return-Path: Received: (from majordom@localhost) by sucuri.mat.puc-rio.br (8.9.3/8.9.3) id KAA21744 for obm-l-MTTP; Mon, 9 Sep 2002 10:28:22 -0300 Received: from smtp-33.ig.com.br (smtp-33.ig.com.br [200.226.132.183]) by sucuri.mat.puc-rio.br (8.9.3/8.9.3) with SMTP id KAA21736 for ; Mon, 9 Sep 2002 10:28:18 -0300 Received: (qmail 6650 invoked from network); 9 Sep 2002 13:28:54 -0000 Received: from shasta085047.ig.com.br (HELO ig) (200.151.85.47) by smtp-33.ig.com.br with SMTP; 9 Sep 2002 13:28:54 -0000 Message-ID: <009301c2573c$0d8b9160$a70997c8@ig> From: =?iso-8859-1?Q?Andr=E9_Silva?= To: References: Subject: Re: [obm-l] Geometria(Quadrilatero) Date: Sun, 8 Sep 2002 10:31:30 -0300 MIME-Version: 1.0 Content-Type: text/plain; charset="iso-8859-1" Content-Transfer-Encoding: 8bit X-Priority: 3 X-MSMail-Priority: Normal X-Mailer: Microsoft Outlook Express 5.00.2615.200 X-MIMEOLE: Produced By Microsoft MimeOLE V5.00.2615.200 Sender: owner-obm-l@sucuri.mat.puc-rio.br Precedence: bulk Reply-To: obm-l@mat.puc-rio.br ----- Original Message ----- From: leonardo mattos To: Sent: Saturday, September 07, 2002 9:55 PM Subject: [obm-l] Geometria(Quadrilatero) > Ola pessoal, > > Dado um quadrilatero ABCD qualquer sao traçadas suas diagonais AC e BC. > Pode afirmar que o angulo(ACD) é congruente ao angulo(ABD),assim como o > angulo(BAC) é congruente ao angulo(BDC)?Se sim, porque? > Um abraço,Leonardo > > Leonardo, não tenho certeza, mas acho que sua dúvida se originou na leitura do retasimson.doc sugerido na lista: Note que o triângulo XNC é congruente ao tri XLC, pois são retângulos de mesma hipotenusa => no quadrilátero NLXC, o ângulo NLC é congruente ao ângulo NXC. O mesmo ocorre para o quadrilátero ALXN. Se não for isso, acho que o Eduardo Casagrande te respondeu bem! Abraço, André. ========================================================================= Instruções para entrar na lista, sair da lista e usar a lista em http://www.mat.puc-rio.br/~nicolau/olimp/obm-l.html O administrador desta lista é ========================================================================= From owner-obm-l@sucuri.mat.puc-rio.br Mon Sep 9 12:08:09 2002 Return-Path: Received: (from majordom@localhost) by sucuri.mat.puc-rio.br (8.9.3/8.9.3) id MAA24883 for obm-l-MTTP; Mon, 9 Sep 2002 12:07:29 -0300 Received: from hotmail.com (f55.law9.hotmail.com [64.4.9.55]) by sucuri.mat.puc-rio.br (8.9.3/8.9.3) with ESMTP id MAA24879 for ; Mon, 9 Sep 2002 12:07:25 -0300 Received: from mail pickup service by hotmail.com with Microsoft SMTPSVC; Mon, 9 Sep 2002 08:08:21 -0700 Received: from 143.107.45.30 by lw9fd.law9.hotmail.msn.com with HTTP; Mon, 09 Sep 2002 15:08:20 GMT X-Originating-IP: [143.107.45.30] From: "Rogerio Fajardo" To: obm-l@mat.puc-rio.br Subject: Re: [obm-l] violencia Date: Mon, 09 Sep 2002 15:08:20 +0000 Mime-Version: 1.0 Content-Type: text/plain; format=flowed Message-ID: X-OriginalArrivalTime: 09 Sep 2002 15:08:21.0587 (UTC) FILETIME=[BC8DF230:01C25812] Sender: owner-obm-l@sucuri.mat.puc-rio.br Precedence: bulk Reply-To: obm-l@mat.puc-rio.br Olá, Vinicius Cada vez que voce "retira um elemento de C" e coloca em R, na verdade voce mudou o conjunto C. Ou seja, cada escolha que voce fez, no processo indutivo, foi sobre um conjunto diferente. É semelhante a demonstração de que todo conjunto infinto possui um subconjunto enumerável, em que, dado um conjunto V, construímos indutivamente um conjunto S colocando nele, a cada passo, um elemento de V que não está em S, usando o Axioma da Escolha >From: Vinicius José Fortuna >Reply-To: obm-l@mat.puc-rio.br >To: >Subject: Re: [obm-l] violencia >Date: Sun, 8 Sep 2002 18:41:45 -0300 > >Oi Rogério >Acho que não saquei. Em que momento foi utilizado o axioma da escolha? Eu >nem tinha infinitos conjuntos! Apenas conjuntos infinitos. > >Até mais > >Vinicius > >----- Original Message ----- >From: "Rogerio Fajardo" >To: >Sent: Sunday, September 08, 2002 2:17 PM >Subject: Re: [obm-l] violencia > > > > É bom notar que essa solução usa o axioma da escolha (de infinitos >conjuntos > > não-vazios, escolhemos um elemento de cada). É essencial o axioma da >escolha > > para resolvê-lo? > > > > > > >From: Vinicius José Fortuna > > >Reply-To: obm-l@mat.puc-rio.br > > >To: > > >Subject: Re: [obm-l] violencia Date: Sat, 7 Sep 2002 23:44:58 -0300 > > > > > >----- Original Message ----- > > >From: "Fernanda Medeiros" > > >To: > > >Sent: Saturday, September 07, 2002 8:45 PM > > >Subject: [obm-l] violencia > > > > > > > > > > Olá, > > > > alguém pode dar uma ajuda nestas questões? > > > > 1.a)uma "gang" tem infinitos bandidos e cada um dos meliantes tem um > > >único > > > > inimigo no interior da "gang",que ele quer matar.Prove q é possivel > > >reunir > > > > uma quantidade infinita de bandidos desta "gang", semq haja o >risco >de > > >q > > > > um bandido mate outro durante a reunião. > > > > > >Pense no seguinte algoritmo: > > >Temos o conjunto C de candidatos à reunião que inicialmente contém >todos >os > > >infinitos bandidos da gangue. > > >Temos o conjunto R de bandidos selecionados para a reunião que >inicialmente > > >está vazio. > > > > > >A cada passo do algoritmo procuramos em C alguém que não que matar >ninguém > > >de R e ninguém em R quer matá-lo. > > >Seja M o subconjunto de C de bandidos que pelo menos um de R quer >matar. > > >Como cada bandido de R só quer matar um, |M|<=|R| > > >Então, como R é finito, M será finito e V=C-M será infinito, pois C é > > >infinito. > > >V será o subconjunto de C dos bandidos que ninguém de R quer matar. > > >Em V procuramos um bandido que não quer matar ninguém de R, retiramos >ele > > >de > > >C, o inserimos em R e repete-se o processo. > > > > > >Se sempre for possível encontrar tal bandido, o processo se repetirá > > >indefinidamente e com R sempre crescendo. Assim teremos infnitos >bandidos > > >na > > >reunião sem derramamento de sangue. > > > > > >Se em algum momento não for possível encontrar um bandido em V, é >porque > > >todos os bandidos de V querem matar alguém de R. Ou seja, ninguém de V >quer > > >matar outro de V. Pegamos, então, V como o conjunto de bandidos para a > > >reunião. Como V é infinito, teremos infinitos participantes na reunião. > > > > > > > b)Se cada bandido tiver um nº finito mas indefinido de inimigos(um > > >bandido > > > > pode ter 2 inimigos, outro somente 1, um terceiro pode ter 20 e >assim > > >por > > > > diante).Será sempre possivel promover uma reunião com infinitos >bandidos > > >sem > > > > risco de derramamento de sangue? > > >Não é possível. Existe um contra-exemplo: > > >Ordene os bandidos formando uma sequência. Imagine que cada bandido >quer > > >matar todos que vêm antes dele na sequência. Nunca poderemos ter dois > > >bandidos 'a' e 'b' na reunião, pois ou a vem antes de b, ou b vem antes >de, > > >assim haverá um que vai querer matar o outro. Então só poderemos ter um > > >bandido na reunião. > > > > > >Até mais > > > > > >Vinicius Fortuna > > >IC-Unicamp > > >========================================================================= >Instruções para entrar na lista, sair da lista e usar a lista em >http://www.mat.puc-rio.br/~nicolau/olimp/obm-l.html >O administrador desta lista é >========================================================================= _________________________________________________________________ MSN Photos is the easiest way to share and print your photos: http://photos.msn.com/support/worldwide.aspx ========================================================================= Instruções para entrar na lista, sair da lista e usar a lista em http://www.mat.puc-rio.br/~nicolau/olimp/obm-l.html O administrador desta lista é ========================================================================= From owner-obm-l@sucuri.mat.puc-rio.br Mon Sep 9 13:26:36 2002 Return-Path: Received: (from majordom@localhost) by sucuri.mat.puc-rio.br (8.9.3/8.9.3) id NAA26677 for obm-l-MTTP; Mon, 9 Sep 2002 13:26:07 -0300 Received: from zeus.opendf.com.br (zeus.opengate.com.br [200.181.71.10]) by sucuri.mat.puc-rio.br (8.9.3/8.9.3) with ESMTP id NAA26673 for ; Mon, 9 Sep 2002 13:26:04 -0300 Received: from localhost (localhost.opengate.com.br [127.0.0.1]) by zeus.opendf.com.br (Postfix) with ESMTP id 2D4B73EAE4 for ; Mon, 9 Sep 2002 13:02:24 -0300 (BRT) Received: by zeus.opendf.com.br (Postfix, from userid 48) id B5FCA3EB0E; Mon, 9 Sep 2002 13:02:23 -0300 (BRT) From: "498 - Artur Costa Steiner" To: obm-l@mat.puc-rio.br Subject: Re: [obm-l] violencia X-Mailer: NeoMail 1.25 X-IPAddress: 200.252.155.2 MIME-Version: 1.0 Content-Type: text/plain; charset=iso-8859-1 Message-Id: <20020909160223.B5FCA3EB0E@zeus.opendf.com.br> Date: Mon, 9 Sep 2002 13:02:23 -0300 (BRT) X-Virus-Scanned: by AMaViS new-20020517 Sender: owner-obm-l@sucuri.mat.puc-rio.br Precedence: bulk Reply-To: obm-l@mat.puc-rio.br Você acha que a utilização do axioma da escolha invalida uma prova? Por exemplo, a prova de que todo conjunto aberto de R é dado por uma união numerável de intervalos abertos disjuntos também utiliza o axioma da escolha. Artur ========================================================================= Instruções para entrar na lista, sair da lista e usar a lista em http://www.mat.puc-rio.br/~nicolau/olimp/obm-l.html O administrador desta lista é ========================================================================= From owner-obm-l@sucuri.mat.puc-rio.br Mon Sep 9 13:33:12 2002 Return-Path: Received: (from majordom@localhost) by sucuri.mat.puc-rio.br (8.9.3/8.9.3) id NAA26857 for obm-l-MTTP; Mon, 9 Sep 2002 13:32:50 -0300 Received: from smtp.ieg.com.br (stone.protocoloweb.com.br [200.226.139.11]) by sucuri.mat.puc-rio.br (8.9.3/8.9.3) with ESMTP id NAA26852 for ; Mon, 9 Sep 2002 13:32:47 -0300 Received: from dois (200-161-156-172.dsl.telesp.net.br [200.161.156.172]) by smtp.ieg.com.br (IeG relay/8.9.3) with SMTP id g89GTVO3039525 for ; Mon, 9 Sep 2002 13:29:32 -0300 (BRT) Message-ID: <001501c2581f$2fb4be40$0200a8c0@dois> From: =?iso-8859-1?Q?Gabriel_P=E9rgola?= To: References: <002401c255a5$a48a7f60$0201a8c0@x> <000201c25798$4d8061e0$0200a8c0@dois> <000901c257ad$55d709b0$158c000a@felipe> Subject: Re: [obm-l] um sistema Date: Mon, 9 Sep 2002 13:37:15 -0300 MIME-Version: 1.0 Content-Type: text/plain; charset="iso-8859-1" Content-Transfer-Encoding: 8bit X-Priority: 3 X-MSMail-Priority: Normal X-Mailer: Microsoft Outlook Express 5.50.4807.1700 X-MimeOLE: Produced By Microsoft MimeOLE V5.50.4807.1700 Sender: owner-obm-l@sucuri.mat.puc-rio.br Precedence: bulk Reply-To: obm-l@mat.puc-rio.br Você está correto Felipe. Errei na ordem dos coeficientes, valeu pelo toque! Gabriel > Gabriel, não sou um bom matemático e pode ser que eu esteja falando > besteira, mas pelo teorema de Cramer você deveria substituir os valores a > direita da igualdade na coluna onde estão os coeficientes que multiplicam a > viaravel desejada não? Assim: > > Dx = | 1 b | , assim Dx/D = a / a² + b² , como diz o enunciado do > problema. > | 0 a | > > > Um abraço e desculpas a todos se tiver dito besteira. > ----- Original Message ----- > > From: "Gabriel Pérgola" > To: > Sent: Sunday, September 08, 2002 7:47 PM > Subject: Re: [obm-l] um sistema > > > > Sabendo que ax-by=1 e que ay+bx=0, prove que x= a/a^2 +b^2 e y > = -b/a^2+b^2 > > > > > > Resolução pelo teorema de Cramer: > > > > D = | a -b | = a² + b² > > | b a | > > > > Dx = | -b 1 | = - a > > | a 0 | > > > > Dy = | a 1 | = -b > > | b 0 | > > > > > > logo: > > > > x = Dx/D = -a/a² + b² > > y = Dy/D= -b/a² + b² > > > > a resposta do x deu diferente do enunciado. Minha resoluçao ou o enunciado > > que está incorreto? > > > > Gabriel > > > > ========================================================================= > > Instruções para entrar na lista, sair da lista e usar a lista em > > http://www.mat.puc-rio.br/~nicolau/olimp/obm-l.html > > O administrador desta lista é > > ========================================================================= > > > > > --- > Outgoing mail is certified Virus Free. > Checked by AVG anti-virus system (http://www.grisoft.com). > Version: 6.0.385 / Virus Database: 217 - Release Date: 5/9/2002 > > ========================================================================= > Instruções para entrar na lista, sair da lista e usar a lista em > http://www.mat.puc-rio.br/~nicolau/olimp/obm-l.html > O administrador desta lista é > ========================================================================= > ========================================================================= Instruções para entrar na lista, sair da lista e usar a lista em http://www.mat.puc-rio.br/~nicolau/olimp/obm-l.html O administrador desta lista é ========================================================================= From owner-obm-l@sucuri.mat.puc-rio.br Mon Sep 9 14:16:40 2002 Return-Path: Received: (from majordom@localhost) by sucuri.mat.puc-rio.br (8.9.3/8.9.3) id OAA28451 for obm-l-MTTP; Mon, 9 Sep 2002 14:16:23 -0300 Received: from mat.puc-rio.br (IDENT:root@perere.mat.puc-rio.br [139.82.27.60]) by sucuri.mat.puc-rio.br (8.9.3/8.9.3) with ESMTP id OAA28447 for ; Mon, 9 Sep 2002 14:16:21 -0300 Received: from localhost (fredpalm@localhost) by mat.puc-rio.br (8.9.3/8.9.3) with ESMTP id GAA31270 for ; Mon, 9 Sep 2002 06:34:03 -0300 Date: Mon, 9 Sep 2002 06:34:02 -0300 (BRT) From: Carlos Frederico Borges Palmeira To: obm-l@mat.puc-rio.br Subject: [obm-l] questao da olimpiada estadual do Rio de janeiro In-Reply-To: <3D7939C600001DAD@www.zipmail.com.br> Message-ID: MIME-Version: 1.0 Content-Type: TEXT/PLAIN; charset=US-ASCII Sender: owner-obm-l@sucuri.mat.puc-rio.br Precedence: bulk Reply-To: obm-l@mat.puc-rio.br uma das questoes da olimpiada estadual do rio de janeiro ,nivel 2, saiu com o enunciado errado, e a gente so percebeu depois da prova. A questao era para ser: duas formigas caminham em um circulo, indo do ponto A ao ponto B. Existe um outro circulo concentrico e INTERIOR a ele. Se as formigas andarem RADIALMENTE ate' o circulo interno, andarem pelo circulo interno, e voltarem radialmente ao ponto B, o trajeto e' mais curto ou mais longo? Saiu EXTERIOR e sem o radialmente, o que deixa a questao praticamente sem sentido. A comissao organizadora lamenta e pede desculpas aos candidatos do nivel 2. Fred Palmeira ========================================================================= Instruções para entrar na lista, sair da lista e usar a lista em http://www.mat.puc-rio.br/~nicolau/olimp/obm-l.html O administrador desta lista é ========================================================================= From owner-obm-l@sucuri.mat.puc-rio.br Mon Sep 9 16:30:01 2002 Return-Path: Received: (from majordom@localhost) by sucuri.mat.puc-rio.br (8.9.3/8.9.3) id QAA31250 for obm-l-MTTP; Mon, 9 Sep 2002 16:27:49 -0300 Received: from web12905.mail.yahoo.com (web12905.mail.yahoo.com [216.136.174.72]) by sucuri.mat.puc-rio.br (8.9.3/8.9.3) with SMTP id QAA31246 for ; Mon, 9 Sep 2002 16:27:46 -0300 Message-ID: <20020909192842.95532.qmail@web12905.mail.yahoo.com> Received: from [200.206.103.3] by web12905.mail.yahoo.com via HTTP; Mon, 09 Sep 2002 16:28:42 ART Date: Mon, 9 Sep 2002 16:28:42 -0300 (ART) From: =?iso-8859-1?q?Johann=20Peter=20Gustav=20Lejeune=20Dirichlet?= Subject: Re: [obm-l] O caráter não enumerável de R To: obm-l@mat.puc-rio.br In-Reply-To: <20020905204856.191213EB26@zeus.opendf.com.br> MIME-Version: 1.0 Content-Type: multipart/alternative; boundary="0-1780226937-1031599722=:95241" Content-Transfer-Encoding: 8bit Sender: owner-obm-l@sucuri.mat.puc-rio.br Precedence: bulk Reply-To: obm-l@mat.puc-rio.br --0-1780226937-1031599722=:95241 Content-Type: text/plain; charset=iso-8859-1 Content-Transfer-Encoding: 8bit Eu acho que voce ta viajando demais.Enumeravel e o conjunto com uma bijecao nos naturais. Os reais nao sao enumeraveis pelo fato de que N*N*N************nao e enumeravel 498 - Artur Costa Steiner escreveu: Um abraço a todos os amigos deste grupo no qual acabei de me inscrever! O assunto que mencionei sempre me intriga um pouco. Há uma clássica demonstração de que R (o conjunto dos reais)não é numerável e que pode ser encontrada na maioria dos livros sobre Análise. Estas provas baseiam-se no fato de que, nos espaços euclidianos, conjuntos perfeitos não são numeráveis. Logo, um ponto chave em tais provas é que os elementos do espaço são pontos de acumulação do mesmo. Sabemos que todo elemento de R é ponto de acumulação. Mas, e este é o ponto que me intriga, tal conclusão depende da métrica definida em R. Na métrica euclidiana usual tal fato é demonstrado (admitindo-se que R seja completo). Mas, se tomarmos, por exemplo, a chamada métrica discreta (d(x,y)=1, se x<>y e d(x,y)=0 se x=y))então nenhum elemento de R (ou do espaço métrico em questão) é ponto de acumulação. A provas que conheço sobre a não enumerabilidade de R (que consistem em se construir uma seqüência de intervalos fechados aninhados) não mais se aplicam na métrica discreta. Não me parece plausível que um espaço métrico seja enumerável numa métrica (ou topologia) e não numerável em outra, mas será que existe uma prova de que R (ou um espaço métrico qualquer) não é numerável a qual seja independente da forma segundo a qual definamos seus conjuntos abertos? Artur ========================================================================= Instruções para entrar na lista, sair da lista e usar a lista em http://www.mat.puc-rio.br/~nicolau/olimp/obm-l.html O administrador desta lista é ========================================================================= --------------------------------- Yahoo! PageBuilder - O super editor para criação de sites: é grátis, fácil e rápido. --0-1780226937-1031599722=:95241 Content-Type: text/html; charset=iso-8859-1 Content-Transfer-Encoding: 8bit

Eu acho que voce ta viajando demais.Enumeravel e o conjunto com uma bijecao nos naturais.

Os reais nao sao enumeraveis pelo fato de que N*N*N************nao e enumeravel

  498 - Artur Costa Steiner <artur@opendf.com.br> escreveu:

Um abraço a todos os amigos deste grupo no qual acabei de me inscrever!

O assunto que mencionei sempre me intriga um pouco. Há uma clássica
demonstração de que R (o conjunto dos reais)não é numerável e que pode
ser encontrada na maioria dos livros sobre Análise. Estas provas
baseiam-se no fato de que, nos espaços euclidianos, conjuntos perfeitos
não são numeráveis. Logo, um ponto chave em tais provas é que os
elementos do espaço são pontos de acumulação do mesmo.

Sabemos que todo elemento de R é ponto de acumulação. Mas, e este é o
ponto que me intriga, tal conclusão depende da métrica definida em R.
Na métrica euclidiana usual tal fato é demonstrado (admitindo-se que R
seja completo). Mas, se tomarmos, por exemplo, a chamada métrica
discreta (d(x,y)=1, se x<>y e d(x,y)=0 se x=y))então nenhum elemento de
R (ou do espaço métrico em questão) é ponto de acumulação. A provas que
conheço sobre a não enumerabilidade de R (que consistem em se construir
uma seqüência de intervalos fechados aninhados) não mais se aplicam na
métrica discreta.

Não me parece plausível que um espaço métrico seja enumerável numa
métrica (ou topologia) e não numerável em outra, mas será que existe
uma prova de que R (ou um espaço métrico qualquer) não é numerável a
qual seja independente da forma segundo a qual definamos seus conjuntos
abertos?

Artur
=========================================================================
Instruções para entrar na lista, sair da lista e usar a lista em
http://www.mat.puc-rio.br/~nicolau/olimp/obm-l.html
O administrador desta lista é
=========================================================================


Yahoo! PageBuilder - O super editor para criação de sites: é grátis, fácil e rápido. --0-1780226937-1031599722=:95241-- ========================================================================= Instruções para entrar na lista, sair da lista e usar a lista em http://www.mat.puc-rio.br/~nicolau/olimp/obm-l.html O administrador desta lista é ========================================================================= From owner-obm-l@sucuri.mat.puc-rio.br Mon Sep 9 17:29:48 2002 Return-Path: Received: (from majordom@localhost) by sucuri.mat.puc-rio.br (8.9.3/8.9.3) id RAA32190 for obm-l-MTTP; Mon, 9 Sep 2002 17:29:31 -0300 Received: from videira.terra.com.br (videira.terra.com.br [200.176.3.5]) by sucuri.mat.puc-rio.br (8.9.3/8.9.3) with ESMTP id RAA32186 for ; Mon, 9 Sep 2002 17:29:27 -0300 Received: from penha.terra.com.br (penha.terra.com.br [200.176.3.43]) by videira.terra.com.br (Postfix) with ESMTP id C8356E23DE for ; Mon, 9 Sep 2002 17:16:11 -0300 (EST) Received: from xt (200-171-249-237.customer.telesp.net.br [200.171.249.237]) (authenticated user macwad) by penha.terra.com.br (Postfix) with ESMTP id 29461683DD for ; Mon, 9 Sep 2002 17:16:10 -0300 (EST) Message-ID: <009d01c2583d$c7d59d70$0401010a@xt> From: =?iso-8859-1?Q?Vinicius_Jos=E9_Fortuna?= To: References: Subject: Re: [obm-l] violencia e axioma da escolha Date: Mon, 9 Sep 2002 17:16:26 -0300 MIME-Version: 1.0 Content-Type: text/plain; charset="iso-8859-1" Content-Transfer-Encoding: 8bit X-Priority: 3 X-MSMail-Priority: Normal X-Mailer: Microsoft Outlook Express 6.00.2600.0000 X-MimeOLE: Produced By Microsoft MimeOLE V6.00.2600.0000 Sender: owner-obm-l@sucuri.mat.puc-rio.br Precedence: bulk Reply-To: obm-l@mat.puc-rio.br Não sei se entendi direito, mas, ao meu ver, não teríamos conjuntos dois a dois disjuntos e tal propriedade é necessária para aplicar o axioma da escolha (ou não?). De qualquer forma, não poderíamos mapear os bandidos nos números inteiros? Assim teríamos uma função de escolha que pegaria em C o bandido mapeado no menor inteiro tal que satisfaça aquelas condições para entrar em R. Se temos uma função de escolha então podemos escolhê-los independentemente do axioma da escolha. Agradeço esclarecimentos Vinicius Fortuna ----- Original Message ----- From: "Rogerio Fajardo" To: Sent: Monday, September 09, 2002 12:08 PM Subject: Re: [obm-l] violencia > Olá, Vinicius > > Cada vez que voce "retira um elemento de C" e coloca em R, na verdade voce > mudou o conjunto C. Ou seja, cada escolha que voce fez, no processo > indutivo, foi sobre um conjunto diferente. É semelhante a demonstração de > que todo conjunto infinto possui um subconjunto enumerável, em que, dado um > conjunto V, construímos indutivamente um conjunto S colocando nele, a cada > passo, um elemento de V que não está em S, usando o Axioma da Escolha > > > >From: Vinicius José Fortuna > >Reply-To: obm-l@mat.puc-rio.br > >To: > >Subject: Re: [obm-l] violencia > >Date: Sun, 8 Sep 2002 18:41:45 -0300 > > > >Oi Rogério > >Acho que não saquei. Em que momento foi utilizado o axioma da escolha? Eu > >nem tinha infinitos conjuntos! Apenas conjuntos infinitos. > > > >Até mais > > > >Vinicius > > > >----- Original Message ----- > >From: "Rogerio Fajardo" > >To: > >Sent: Sunday, September 08, 2002 2:17 PM > >Subject: Re: [obm-l] violencia > > > > > > > É bom notar que essa solução usa o axioma da escolha (de infinitos > >conjuntos > > > não-vazios, escolhemos um elemento de cada). É essencial o axioma da > >escolha > > > para resolvê-lo? > > > > > > > > > >From: Vinicius José Fortuna > > > >Reply-To: obm-l@mat.puc-rio.br > > > >To: > > > >Subject: Re: [obm-l] violencia Date: Sat, 7 Sep 2002 23:44:58 -0300 > > > > > > > >----- Original Message ----- > > > >From: "Fernanda Medeiros" > > > >To: > > > >Sent: Saturday, September 07, 2002 8:45 PM > > > >Subject: [obm-l] violencia > > > > > > > > > > > > > Olá, > > > > > alguém pode dar uma ajuda nestas questões? > > > > > 1.a)uma "gang" tem infinitos bandidos e cada um dos meliantes tem um > > > >único > > > > > inimigo no interior da "gang",que ele quer matar.Prove q é possivel > > > >reunir > > > > > uma quantidade infinita de bandidos desta "gang", semq haja o > >risco > >de > > > >q > > > > > um bandido mate outro durante a reunião. > > > > > > > >Pense no seguinte algoritmo: > > > >Temos o conjunto C de candidatos à reunião que inicialmente contém > >todos > >os > > > >infinitos bandidos da gangue. > > > >Temos o conjunto R de bandidos selecionados para a reunião que > >inicialmente > > > >está vazio. > > > > > > > >A cada passo do algoritmo procuramos em C alguém que não que matar > >ninguém > > > >de R e ninguém em R quer matá-lo. > > > >Seja M o subconjunto de C de bandidos que pelo menos um de R quer > >matar. > > > >Como cada bandido de R só quer matar um, |M|<=|R| > > > >Então, como R é finito, M será finito e V=C-M será infinito, pois C é > > > >infinito. > > > >V será o subconjunto de C dos bandidos que ninguém de R quer matar. > > > >Em V procuramos um bandido que não quer matar ninguém de R, retiramos > >ele > > > >de > > > >C, o inserimos em R e repete-se o processo. > > > > > > > >Se sempre for possível encontrar tal bandido, o processo se repetirá > > > >indefinidamente e com R sempre crescendo. Assim teremos infnitos > >bandidos > > > >na > > > >reunião sem derramamento de sangue. > > > > > > > >Se em algum momento não for possível encontrar um bandido em V, é > >porque > > > >todos os bandidos de V querem matar alguém de R. Ou seja, ninguém de V > >quer > > > >matar outro de V. Pegamos, então, V como o conjunto de bandidos para a > > > >reunião. Como V é infinito, teremos infinitos participantes na reunião. > > > > > > > > > b)Se cada bandido tiver um nº finito mas indefinido de inimigos(um > > > >bandido > > > > > pode ter 2 inimigos, outro somente 1, um terceiro pode ter 20 e > >assim > > > >por > > > > > diante).Será sempre possivel promover uma reunião com infinitos > >bandidos > > > >sem > > > > > risco de derramamento de sangue? > > > >Não é possível. Existe um contra-exemplo: > > > >Ordene os bandidos formando uma sequência. Imagine que cada bandido > >quer > > > >matar todos que vêm antes dele na sequência. Nunca poderemos ter dois > > > >bandidos 'a' e 'b' na reunião, pois ou a vem antes de b, ou b vem antes > >de, > > > >assim haverá um que vai querer matar o outro. Então só poderemos ter um > > > >bandido na reunião. > > > > > > > >Até mais > > > > > > > >Vinicius Fortuna > > > >IC-Unicamp ========================================================================= Instruções para entrar na lista, sair da lista e usar a lista em http://www.mat.puc-rio.br/~nicolau/olimp/obm-l.html O administrador desta lista é ========================================================================= From owner-obm-l@sucuri.mat.puc-rio.br Mon Sep 9 17:46:01 2002 Return-Path: Received: (from majordom@localhost) by sucuri.mat.puc-rio.br (8.9.3/8.9.3) id RAA32749 for obm-l-MTTP; Mon, 9 Sep 2002 17:45:31 -0300 Received: from sr1.terra.com.br (sr1.terra.com.br [200.176.3.16]) by sucuri.mat.puc-rio.br (8.9.3/8.9.3) with ESMTP id RAA32745 for ; Mon, 9 Sep 2002 17:45:27 -0300 Received: from penha.terra.com.br (penha.terra.com.br [200.176.3.43]) by sr1.terra.com.br (Postfix) with ESMTP id BA0DF6F59E for ; Mon, 9 Sep 2002 17:46:23 -0300 (EST) Received: from Itautec.terra.com.br (200-158-61-138.dsl.telesp.net.br [200.158.61.138]) (authenticated user bruleite) by penha.terra.com.br (Postfix) with ESMTP id EFDB968268 for ; Mon, 9 Sep 2002 17:46:21 -0300 (EST) Message-Id: <5.1.0.14.0.20020909174206.00a10240@pop.sao.terra.com.br> X-Sender: bruleite@pop.sao.terra.com.br X-Mailer: QUALCOMM Windows Eudora Version 5.1 Date: Mon, 09 Sep 2002 17:42:37 -0300 To: obm-l@mat.puc-rio.br From: "Bruno F. C. Leite" Subject: Re: [obm-l] violencia e axioma da escolha In-Reply-To: <009d01c2583d$c7d59d70$0401010a@xt> References: Mime-Version: 1.0 Content-Type: text/plain; charset="iso-8859-1"; format=flowed Content-Transfer-Encoding: 8bit X-MIME-Autoconverted: from quoted-printable to 8bit by sucuri.mat.puc-rio.br id RAA32746 Sender: owner-obm-l@sucuri.mat.puc-rio.br Precedence: bulk Reply-To: obm-l@mat.puc-rio.br At 17:16 09/09/02 -0300, you wrote: >Não sei se entendi direito, mas, ao meu ver, não teríamos conjuntos dois a >dois disjuntos e tal propriedade é necessária para aplicar o axioma da >escolha (ou não?). > >De qualquer forma, não poderíamos mapear os bandidos nos números inteiros? >Assim teríamos uma função de escolha que pegaria em C o bandido mapeado no >menor inteiro tal que satisfaça aquelas condições para entrar em R. Se temos >uma função de escolha então podemos escolhê-los independentemente do axioma >da escolha. Acho que o conjunto dos bandidos não precisa ser enumerável. Bruno Leite http://www.ime.usp.br/~brleite >Agradeço esclarecimentos > >Vinicius Fortuna > >----- Original Message ----- >From: "Rogerio Fajardo" >To: >Sent: Monday, September 09, 2002 12:08 PM >Subject: Re: [obm-l] violencia > > > > Olá, Vinicius > > > > Cada vez que voce "retira um elemento de C" e coloca em R, na verdade >voce > > mudou o conjunto C. Ou seja, cada escolha que voce fez, no processo > > indutivo, foi sobre um conjunto diferente. É semelhante a demonstração de > > que todo conjunto infinto possui um subconjunto enumerável, em que, dado >um > > conjunto V, construímos indutivamente um conjunto S colocando nele, a cada > > passo, um elemento de V que não está em S, usando o Axioma da Escolha > > > > > > >From: Vinicius José Fortuna > > >Reply-To: obm-l@mat.puc-rio.br > > >To: > > >Subject: Re: [obm-l] violencia > > >Date: Sun, 8 Sep 2002 18:41:45 -0300 > > > > > >Oi Rogério > > >Acho que não saquei. Em que momento foi utilizado o axioma da escolha? Eu > > >nem tinha infinitos conjuntos! Apenas conjuntos infinitos. > > > > > >Até mais > > > > > >Vinicius > > > > > >----- Original Message ----- > > >From: "Rogerio Fajardo" > > >To: > > >Sent: Sunday, September 08, 2002 2:17 PM > > >Subject: Re: [obm-l] violencia > > > > > > > > > > É bom notar que essa solução usa o axioma da escolha (de infinitos > > >conjuntos > > > > não-vazios, escolhemos um elemento de cada). É essencial o axioma da > > >escolha > > > > para resolvê-lo? > > > > > > > > > > > > >From: Vinicius José Fortuna > > > > >Reply-To: obm-l@mat.puc-rio.br > > > > >To: > > > > >Subject: Re: [obm-l] violencia Date: Sat, 7 Sep 2002 23:44:58 -0300 > > > > > > > > > >----- Original Message ----- > > > > >From: "Fernanda Medeiros" > > > > >To: > > > > >Sent: Saturday, September 07, 2002 8:45 PM > > > > >Subject: [obm-l] violencia > > > > > > > > > > > > > > > > Olá, > > > > > > alguém pode dar uma ajuda nestas questões? > > > > > > 1.a)uma "gang" tem infinitos bandidos e cada um dos meliantes tem >um > > > > >único > > > > > > inimigo no interior da "gang",que ele quer matar.Prove q é >possivel > > > > >reunir > > > > > > uma quantidade infinita de bandidos desta "gang", semq haja o > > >risco > > >de > > > > >q > > > > > > um bandido mate outro durante a reunião. > > > > > > > > > >Pense no seguinte algoritmo: > > > > >Temos o conjunto C de candidatos à reunião que inicialmente contém > > >todos > > >os > > > > >infinitos bandidos da gangue. > > > > >Temos o conjunto R de bandidos selecionados para a reunião que > > >inicialmente > > > > >está vazio. > > > > > > > > > >A cada passo do algoritmo procuramos em C alguém que não que matar > > >ninguém > > > > >de R e ninguém em R quer matá-lo. > > > > >Seja M o subconjunto de C de bandidos que pelo menos um de R quer > > >matar. > > > > >Como cada bandido de R só quer matar um, |M|<=|R| > > > > >Então, como R é finito, M será finito e V=C-M será infinito, pois C é > > > > >infinito. > > > > >V será o subconjunto de C dos bandidos que ninguém de R quer matar. > > > > >Em V procuramos um bandido que não quer matar ninguém de R, retiramos > > >ele > > > > >de > > > > >C, o inserimos em R e repete-se o processo. > > > > > > > > > >Se sempre for possível encontrar tal bandido, o processo se repetirá > > > > >indefinidamente e com R sempre crescendo. Assim teremos infnitos > > >bandidos > > > > >na > > > > >reunião sem derramamento de sangue. > > > > > > > > > >Se em algum momento não for possível encontrar um bandido em V, é > > >porque > > > > >todos os bandidos de V querem matar alguém de R. Ou seja, ninguém de >V > > >quer > > > > >matar outro de V. Pegamos, então, V como o conjunto de bandidos para >a > > > > >reunião. Como V é infinito, teremos infinitos participantes na >reunião. > > > > > > > > > > > b)Se cada bandido tiver um nº finito mas indefinido de inimigos(um > > > > >bandido > > > > > > pode ter 2 inimigos, outro somente 1, um terceiro pode ter 20 e > > >assim > > > > >por > > > > > > diante).Será sempre possivel promover uma reunião com infinitos > > >bandidos > > > > >sem > > > > > > risco de derramamento de sangue? > > > > >Não é possível. Existe um contra-exemplo: > > > > >Ordene os bandidos formando uma sequência. Imagine que cada bandido > > >quer > > > > >matar todos que vêm antes dele na sequência. Nunca poderemos ter dois > > > > >bandidos 'a' e 'b' na reunião, pois ou a vem antes de b, ou b vem >antes > > >de, > > > > >assim haverá um que vai querer matar o outro. Então só poderemos ter >um > > > > >bandido na reunião. > > > > > > > > > >Até mais > > > > > > > > > >Vinicius Fortuna > > > > >IC-Unicamp > > >========================================================================= >Instruções para entrar na lista, sair da lista e usar a lista em >http://www.mat.puc-rio.br/~nicolau/olimp/obm-l.html >O administrador desta lista é >========================================================================= ========================================================================= Instruções para entrar na lista, sair da lista e usar a lista em http://www.mat.puc-rio.br/~nicolau/olimp/obm-l.html O administrador desta lista é ========================================================================= From owner-obm-l@sucuri.mat.puc-rio.br Mon Sep 9 18:18:58 2002 Return-Path: Received: (from majordom@localhost) by sucuri.mat.puc-rio.br (8.9.3/8.9.3) id SAA01706 for obm-l-MTTP; Mon, 9 Sep 2002 18:18:31 -0300 Received: from hotmail.com (f135.sea2.hotmail.com [207.68.165.135]) by sucuri.mat.puc-rio.br (8.9.3/8.9.3) with ESMTP id SAA01702 for ; Mon, 9 Sep 2002 18:18:27 -0300 Received: from mail pickup service by hotmail.com with Microsoft SMTPSVC; Mon, 9 Sep 2002 14:19:23 -0700 Received: from 32.94.119.253 by sea2fd.sea2.hotmail.msn.com with HTTP; Mon, 09 Sep 2002 21:19:23 GMT X-Originating-IP: [32.94.119.253] From: "Paulo Santa Rita" To: obm-l@mat.puc-rio.br Subject: [obm-l] Um Estranho Sentimento ... Date: Mon, 09 Sep 2002 21:19:23 +0000 Mime-Version: 1.0 Content-Type: text/plain; charset=iso-8859-1; format=flowed Message-ID: X-OriginalArrivalTime: 09 Sep 2002 21:19:23.0355 (UTC) FILETIME=[919A2AB0:01C25846] Sender: owner-obm-l@sucuri.mat.puc-rio.br Precedence: bulk Reply-To: obm-l@mat.puc-rio.br Ola Pessoal e demais colegas desta lista ... OBM-L, Eu tenho pensado continua e longamente em um conjunto de questoes relacionadas que me levam, invariavelmente, a uma mesma direcao que nao estou conseguindo acreditar ... ate parece que depois de tanto refletir cheguei a alguma constatacao insana ou simploria demais ... Se algum colega puder falar algo esclarecedor e/ou interessante eu ficarei muito grato ! Desde a infancia somos instados a pensar que os numeros reais estao dispostos ao longo de uma linha reta. Nos dizemos : 3 < 5 ! E imediatamente visualizamos o 3 a esquerda do 5, ambos em uma linha reta ! Por que nos pensamos assim ? E verdade que em cursos de analise os livros definem R como um corpo ordenado completo e derivam as propriedades dos numeros reais dos axiomas que definem esta estrutura, sem recorrer a qualquer propriedade geometrica derivada de uma eventual visualizacao dos numeros sobre uma reta ... Mas se, por um lado, os axiomas de um corpo ordenado completo nao implicam ou requerem explicitamente uma estrutura geometrica conhecida, e inegavel que a visualizacao "informal" que fazemos facilita muitos raciocinio ... SERIA REALMENTE IMPOSSIVEL ASSOCIAR A UM CORPO ORDENADO COMPLETO UMA GEOMETRIA, ATRAVES DE AXIOMAS OU OUTROS RECURSOS, DE FORMA QUE PUDESSEMOS TER OUTRAS VISUALIZACOES, MESMO QUE ESTRANHAS, POREM, CONMSISTENTES ? NAO PODERIAM ALGUMAS PROPRIEDADES NUMERICAS DEPENDEREM INEXORAVELMENTE DE UMA TAL GEOMETRIA ? E bem provavel que eu esteja errado, mas nao consigo perceber o meu erro ... EU ACHO QUE CERTOS MAPEAMENTOS NUMERICOS REQUEREM OU IMPLICAM QUE OS NUMEROS NATURAIS TEM UMA GEOMETRIA OU ESTRUTURA INTRINSECA, SEM A QUAL NAO DA PRA COMPREENDER CERTAS COISAS ... E NECESSARIO OU POSTULAR UMA DISPOSICAO ESTRATIGRAFICA OU SUPOR QUE CERTOS MAPEAMENTOS INDUZEM UMA TAL ESTRATIFICACAO ... Bom, se alguem puder falar alguma coisa interessante sobre este tema eu fico muito grato, pois este e realmente um SENTIMENTO ESTRANHO que me tem ocorrido com alguma frequencia. Pode ser uma burrice momentanea que esta me levando a estas perguntas aparentemente idiotas e sem sentido, mas eu nao iria ocupar o tempo de voces, meus amigos, se nao tivesse razoes seria pra fazer isso ... Um abraco a Todos Paulo Santa Rita 2,1818,090902 _________________________________________________________________ Converse com seus amigos online, faça o download grátis do MSN Messenger: http://messenger.msn.com.br ========================================================================= Instruções para entrar na lista, sair da lista e usar a lista em http://www.mat.puc-rio.br/~nicolau/olimp/obm-l.html O administrador desta lista é ========================================================================= From owner-obm-l@sucuri.mat.puc-rio.br Mon Sep 9 18:19:00 2002 Return-Path: Received: (from majordom@localhost) by sucuri.mat.puc-rio.br (8.9.3/8.9.3) id SAA01726 for obm-l-MTTP; Mon, 9 Sep 2002 18:18:54 -0300 Received: from shannon.bol.com.br (shannon.bol.com.br [200.221.24.13]) by sucuri.mat.puc-rio.br (8.9.3/8.9.3) with ESMTP id SAA01722 for ; Mon, 9 Sep 2002 18:18:51 -0300 Received: from bol.com.br (200.221.24.128) by shannon.bol.com.br (5.1.071) id 3D60E4DC005CC05A for obm-l@mat.puc-rio.br; Mon, 9 Sep 2002 18:19:31 -0300 Date: Mon, 9 Sep 2002 18:17:37 -0300 Message-Id: Subject: Re:[obm-l] circuito IME MIME-Version: 1.0 Content-Type: multipart/mixed; boundary="_=__=_XaM3_Boundary.1031606257.2A.600010.42.25133.52.42.101010.995297547" From: "rafaelc.l" To: obm-l@mat.puc-rio.br X-XaM3-API-Version: 2.4.3.4.4 X-SenderIP: 200.176.166.74 Sender: owner-obm-l@sucuri.mat.puc-rio.br Precedence: bulk Reply-To: obm-l@mat.puc-rio.br --_=__=_XaM3_Boundary.1031606257.2A.600010.42.25133.52.42.101010.995297547 Content-Type: text/plain;charset="iso-8859-1" Content-Transfer-Encoding: quoted-printable meu caro Adriano: vc n=E3o acha que deve ter alguma solu=E7=E3o para essa quest=E3o sem o uso dessa equa=E7=F5es? (pense bem nisso),analise o problema. Se n=E3o tivesse alguma solu=E7=E3o s=F3 com a abordagem do 2 grau, a quest=E3o teria sido anulada. Poucos candidatos devem ter resolvido ela intergralmente. __________________________________________________________________________ AcessoBOL, s=F3 R$ 9,90! O menor pre=E7o do mercado! Assine j=E1! http://www.bol.com.br/acessobol --_=__=_XaM3_Boundary.1031606257.2A.600010.42.25133.52.42.101010.995297547 Content-Type: text/plain; name="00000DRE" Content-Transfer-Encoding: base64 TmEgcXVlc3RhbyAxMCBmb2kgbmVjZXNzYXJpbyBvIGVtcHJlZ28gZGFzIA0KZXF1YWNvZXMg ZG8gY2lyY3VpdG8gUkMgbm8gcmVnaW1lIA0KdHJhbnNpdG9yaW8sZXF1YWNvZXMgcXVlIG5h byBwZW50ZW5jZW0gYW8gDQpwcm9ncmFtYS4NCltdJ3MuDQpBZHJpYW5vLg0KDQogDQpfX19f X19fX19fX19fX19fX19fX19fX19fX19fX19fX19fX19fX19fX19fX19fX19fX19fX19fX19f X19fX19fX19fX19fX19fXw0KQWNlc3NvQk9MLCBz8yBSJCA5LDkwISBPIG1lbm9yIHByZedv IGRvIG1lcmNhZG8hDQpBc3NpbmUgauEhIGh0dHA6Ly93d3cuYm9sLmNvbS5ici9hY2Vzc29i b2wNCg0KDQo9PT09PT09PT09PT09PT09PT09PT09PT09PT09PT09PT09PT09PT09PT09PT09 PT09PT09PT09PT09PT09PT09PT09PT09PT09DQpJbnN0cnXn9WVzIHBhcmEgZW50cmFyIG5h IGxpc3RhLCBzYWlyIGRhIGxpc3RhIGUgdXNhciBhIGxpc3RhIGVtDQpodHRwOi8vd3d3Lm1h dC5wdWMtcmlvLmJyL35uaWNvbGF1L29saW1wL29ibS1sLmh0bWwNCk8gYWRtaW5pc3RyYWRv ciBkZXN0YSBsaXN0YSDpIDxuaWNvbGF1QG1hdC5wdWMtcmlvLmJyPg0KPT09PT09PT09PT09 PT09PT09PT09PT09PT09PT09PT09PT09PT09PT09PT09PT09PT09PT09PT09PT09PT09PT09 PT09PT09PQ0K --_=__=_XaM3_Boundary.1031606257.2A.600010.42.25133.52.42.101010.995297547-- ========================================================================= Instruções para entrar na lista, sair da lista e usar a lista em http://www.mat.puc-rio.br/~nicolau/olimp/obm-l.html O administrador desta lista é ========================================================================= From owner-obm-l@sucuri.mat.puc-rio.br Mon Sep 9 19:31:31 2002 Return-Path: Received: (from majordom@localhost) by sucuri.mat.puc-rio.br (8.9.3/8.9.3) id TAA03497 for obm-l-MTTP; Mon, 9 Sep 2002 19:30:15 -0300 Received: from hotmail.com (f136.law4.hotmail.com [216.33.149.136]) by sucuri.mat.puc-rio.br (8.9.3/8.9.3) with ESMTP id TAA03493 for ; Mon, 9 Sep 2002 19:30:12 -0300 Received: from mail pickup service by hotmail.com with Microsoft SMTPSVC; Mon, 9 Sep 2002 15:31:08 -0700 Received: from 200.165.5.140 by lw4fd.law4.hotmail.msn.com with HTTP; Mon, 09 Sep 2002 22:31:07 GMT X-Originating-IP: [200.165.5.140] From: "Laurito Alves" To: obm-l@mat.puc-rio.br Subject: [obm-l] =?iso-8859-1?B?UmU6IFtvYm0tbF0gTyBjYXLhdGVyIG7jbyBlbnVtZXLhdmVsIGRlIFI=?= Date: Mon, 09 Sep 2002 22:31:07 +0000 Mime-Version: 1.0 Content-Type: text/plain; charset=iso-8859-1; format=flowed Message-ID: X-OriginalArrivalTime: 09 Sep 2002 22:31:08.0131 (UTC) FILETIME=[9772DB30:01C25850] Sender: owner-obm-l@sucuri.mat.puc-rio.br Precedence: bulk Reply-To: obm-l@mat.puc-rio.br O que você chama de N*N*N************ ??? Se for um produto cartesiano de N uma quantidade enumerável de vezes, ele é enumerável. Laurito >From: Johann Peter Gustav Lejeune Dirichlet > >Reply-To: obm-l@mat.puc-rio.br >To: obm-l@mat.puc-rio.br >Subject: Re: [obm-l] O caráter não enumerável de R >Date: Mon, 9 Sep 2002 16:28:42 -0300 (ART) > > > Eu acho que voce ta viajando demais.Enumeravel e o conjunto com uma >bijecao nos naturais. >Os reais nao sao enumeraveis pelo fato de que N*N*N************nao e >enumeravel > 498 - Artur Costa Steiner escreveu: Um abraço a >todos os amigos deste grupo no qual acabei de me inscrever! > >O assunto que mencionei sempre me intriga um pouco. Há uma clássica >demonstração de que R (o conjunto dos reais)não é numerável e que pode >ser encontrada na maioria dos livros sobre Análise. Estas provas >baseiam-se no fato de que, nos espaços euclidianos, conjuntos perfeitos >não são numeráveis. Logo, um ponto chave em tais provas é que os >elementos do espaço são pontos de acumulação do mesmo. > >Sabemos que todo elemento de R é ponto de acumulação. Mas, e este é o >ponto que me intriga, tal conclusão depende da métrica definida em R. >Na métrica euclidiana usual tal fato é demonstrado (admitindo-se que R >seja completo). Mas, se tomarmos, por exemplo, a chamada métrica >discreta (d(x,y)=1, se x<>y e d(x,y)=0 se x=y))então nenhum elemento de >R (ou do espaço métrico em questão) é ponto de acumulação. A provas que >conheço sobre a não enumerabilidade de R (que consistem em se construir >uma seqüência de intervalos fechados aninhados) não mais se aplicam na >métrica discreta. > >Não me parece plausível que um espaço métrico seja enumerável numa >métrica (ou topologia) e não numerável em outra, mas será que existe >uma prova de que R (ou um espaço métrico qualquer) não é numerável a >qual seja independente da forma segundo a qual definamos seus conjuntos >abertos? > >Artur >========================================================================= >Instruções para entrar na lista, sair da lista e usar a lista em >http://www.mat.puc-rio.br/~nicolau/olimp/obm-l.html >O administrador desta lista é >========================================================================= > >--------------------------------- >Yahoo! PageBuilder - O super editor para criação de sites: é grátis, fácil >e rápido. _________________________________________________________________ Tenha você também um MSN Hotmail, o maior webmail do mundo: http://www.hotmail.com/br ========================================================================= Instruções para entrar na lista, sair da lista e usar a lista em http://www.mat.puc-rio.br/~nicolau/olimp/obm-l.html O administrador desta lista é ========================================================================= From owner-obm-l@sucuri.mat.puc-rio.br Mon Sep 9 19:50:02 2002 Return-Path: Received: (from majordom@localhost) by sucuri.mat.puc-rio.br (8.9.3/8.9.3) id TAA04007 for obm-l-MTTP; Mon, 9 Sep 2002 19:49:50 -0300 Received: from frigg.h8.ita.cta.br (frigg.h8.ita.cta.br [161.24.72.4]) by sucuri.mat.puc-rio.br (8.9.3/8.9.3) with ESMTP id TAA04003 for ; Mon, 9 Sep 2002 19:49:46 -0300 Received: from jackson ([161.24.73.41]) by frigg.h8.ita.cta.br with smtp (Exim 3.35 #1 (Debian)) id 17oXN4-0003EX-00 for ; Mon, 09 Sep 2002 19:51:06 -0300 Message-ID: <000b01c25853$50e17d40$294918a1@jackson> From: "Jackson Graziano" To: Subject: [obm-l] Problema com inteiros gaussianos Date: Mon, 9 Sep 2002 19:50:18 -0300 MIME-Version: 1.0 Content-Type: text/plain; charset="iso-8859-1" Content-Transfer-Encoding: 8bit X-Priority: 3 X-MSMail-Priority: Normal X-Mailer: Microsoft Outlook Express 6.00.2600.0000 X-MimeOLE: Produced By Microsoft MimeOLE V6.00.2600.0000 Sender: owner-obm-l@sucuri.mat.puc-rio.br Precedence: bulk Reply-To: obm-l@mat.puc-rio.br Caros amigos, Como devo interpretar o enunciado abaixo? Nao consegui entender como são as regras dele pra andar na reta real... One cannot walk to infinity on the real line if one uses steps of bounded length and steps on the prime numbers. This is simply a restatement of the classic result that there are arbitrarily large gaps in the primes. The proof is simple: a gap of size k is given by (k + 1)! + 2, (k + 1)! + 3,..., (k + 1)! + (k,+1). But the same problem in the complex realm is unsolved. More precisely, an analogous question asks whether one can walk to infinity in Z[i], the Gaussian integers, using the Gaussian primes as stepping stones, and taking steps of bounded length Obrigado, Jackson Graziano ========================================================================= Instruções para entrar na lista, sair da lista e usar a lista em http://www.mat.puc-rio.br/~nicolau/olimp/obm-l.html O administrador desta lista é ========================================================================= From owner-obm-l@sucuri.mat.puc-rio.br Mon Sep 9 20:54:47 2002 Return-Path: Received: (from majordom@localhost) by sucuri.mat.puc-rio.br (8.9.3/8.9.3) id UAA05760 for obm-l-MTTP; Mon, 9 Sep 2002 20:54:13 -0300 Received: from videira.terra.com.br (videira.terra.com.br [200.176.3.5]) by sucuri.mat.puc-rio.br (8.9.3/8.9.3) with ESMTP id UAA05756 for ; Mon, 9 Sep 2002 20:54:11 -0300 Received: from pavuna.terra.com.br (pavuna.terra.com.br [200.176.3.41]) by videira.terra.com.br (Postfix) with ESMTP id 3025CE1084 for ; Mon, 9 Sep 2002 20:55:09 -0300 (EST) Received: from stabel (unknown [200.203.39.236]) (authenticated user dudasta) by pavuna.terra.com.br (Postfix) with ESMTP id C0081683CD for ; Mon, 9 Sep 2002 20:55:08 -0300 (EST) Message-ID: <007e01c2585c$56b62b90$0301a8c0@stabel> From: "Eduardo Casagrande Stabel" To: References: Subject: [obm-l] =?iso-8859-1?Q?Re:_=5Bobm-l=5D_Re:_=5Bobm-l=5D_O_car=E1ter_n=E3o_en?= =?iso-8859-1?Q?umer=E1vel_de_R?= Date: Mon, 9 Sep 2002 20:55:12 -0300 MIME-Version: 1.0 Content-Type: text/plain; charset="iso-8859-1" Content-Transfer-Encoding: 8bit X-Priority: 3 X-MSMail-Priority: Normal X-Mailer: Microsoft Outlook Express 6.00.2600.0000 X-MimeOLE: Produced By Microsoft MimeOLE V6.00.2600.0000 Sender: owner-obm-l@sucuri.mat.puc-rio.br Precedence: bulk Reply-To: obm-l@mat.puc-rio.br From: "Laurito Alves" > O que você chama de N*N*N************ ??? > > Se for um produto cartesiano de N uma quantidade enumerável de vezes, ele é > enumerável. > > Laurito Caro Laurito Alves, certamente que ou você está usando uma definição de o que é "produto cartesiano enumerável" diferente da usual ou você cometeu um engano. O produto cartesiano X = N * N * N * N * ... é o conjunto das seqüências (a, b, c, d, e, ....) onde cada a,b,c,d... é um elemento de N. Com esta definição X é um conjunto não enumerável. Para ver isso, perceba que a função f que leva (a, b, c, d, e, ...) |-> a + b/2 + c/4 + d/8 + e/16 + ... é sobretetiva nos reais positivos. Pegue a representação em base 2 de cada número y = INTEIRO + 0.10110101101..., daí tome a=INTEIRO e b=1, c=0, d=1, e=1, ... e assim por diante. Uma definição para produto cartesiano pode ser a seguinte. Seja I um conjunto de índices e a cada índice i de I está associado um conjunto X_i. Defina PRODUTO CARTESIANO(X_i, i em I) como o conjunto das funções f:I->UNIÃO{X_i, i em I} com a restrição adicional: para cada i em I, f(i) pertence a X_i. Talvez a definição usual não seja em torno de função, mas deve ser algo com a mesma cardinalidade desse conjunto. Alguém esclarece qual a definição mais usada? Eduardo. Porto Alegre, RS. ========================================================================= Instruções para entrar na lista, sair da lista e usar a lista em http://www.mat.puc-rio.br/~nicolau/olimp/obm-l.html O administrador desta lista é ========================================================================= From owner-obm-l@sucuri.mat.puc-rio.br Mon Sep 9 21:16:29 2002 Return-Path: Received: (from majordom@localhost) by sucuri.mat.puc-rio.br (8.9.3/8.9.3) id VAA06234 for obm-l-MTTP; Mon, 9 Sep 2002 21:16:18 -0300 Received: from gorgo.centroin.com.br (gorgo.centroin.com.br [200.225.63.128]) by sucuri.mat.puc-rio.br (8.9.3/8.9.3) with ESMTP id VAA06230 for ; Mon, 9 Sep 2002 21:16:15 -0300 Received: from centroin.com.br (du48c.rjo.centroin.com.br [200.225.58.48]) (authenticated bits=0) by gorgo.centroin.com.br (8.12.2/8.12.1) with ESMTP id g8A0HcnH023964 for ; Mon, 9 Sep 2002 21:17:38 -0300 (BRT) Message-ID: <3D7D3AB6.8070306@centroin.com.br> Date: Mon, 09 Sep 2002 21:20:06 -0300 From: Augusto =?ISO-8859-1?Q?C=E9sar?= Morgado User-Agent: Mozilla/5.0 (Windows; U; Win98; en-US; rv:0.9.4.1) Gecko/20020508 Netscape6/6.2.3 X-Accept-Language: en-us MIME-Version: 1.0 To: obm-l@mat.puc-rio.br Subject: Re: [obm-l] Problema com inteiros gaussianos References: <000b01c25853$50e17d40$294918a1@jackson> Content-Type: text/plain; charset=ISO-8859-1; format=flowed Content-Transfer-Encoding: 8bit Sender: owner-obm-l@sucuri.mat.puc-rio.br Precedence: bulk Reply-To: obm-l@mat.puc-rio.br Nao podemos chegar ao infinito na reta real se os nossos passos tiverem comprimento limitado (isto eh, existe um numero M tal que nossos passosnao podem ser maiores que M) e andarmos apenas sobre os numeros primos. (por exemplo, suponha que voce nao possa dar passos maiores que 5. Andando nosprimos, voce pode começar no 2, passar para o 7, depois para o 11, depois para o 13, o 17, o 19, o 23. Agora voce empaca. Do 23 para o primo seguinte o salto eh maior que 5. A essencia do que o autor quer dizer eh que, por maior que seja a limitaçao dos seus passos (5, no meu exemplo), chega uma hora que voce empaca. ) Isso eh simplesmente uma outra maneira de dizer que ha vazios arbitrariamente grandes entre os primos. (ou seja, tomando o 5, encontramos um vazio de tamanho maior que 5 nos primos: entre o 23 e o 29. Tomando o 7, encontrariamos um vazio de tamanho maior que 7; nao eh dificil perceber que esse vazio se dah entre o 31 e o 39. Tomando o 20, encontrariamos um vazio maior que 20 etc). A prova eh simples. Um vazio de tamanho k ocorre entre (k+1)!+2 e (k+1)!+k+1. Com efeito, nenhum desse numeros eh primo, pois (k+1)!+2 eh divisivel por 2(e eh maior que 2) , (k+1)!+3 eh divisivel por 3 (e eh maior que 3) etc Jackson Graziano wrote: >Caros amigos, > >Como devo interpretar o enunciado abaixo? Nao consegui entender como são as >regras dele pra andar na reta real... > >One cannot walk to infinity on the real line if one uses steps of bounded >length and steps on the prime numbers. This is simply a restatement of the >classic result that there are arbitrarily large gaps in the primes. The >proof is simple: a gap of size k is given by (k + 1)! + 2, (k + 1)! + 3,..., >(k + 1)! + (k,+1). >But the same problem in the complex realm is unsolved. More precisely, an >analogous question asks whether one can walk to infinity in Z[i], the >Gaussian integers, using the Gaussian primes as stepping stones, and taking >steps of bounded length > >Obrigado, > >Jackson Graziano > >========================================================================= >Instruções para entrar na lista, sair da lista e usar a lista em >http://www.mat.puc-rio.br/~nicolau/olimp/obm-l.html >O administrador desta lista é >========================================================================= > > ========================================================================= Instruções para entrar na lista, sair da lista e usar a lista em http://www.mat.puc-rio.br/~nicolau/olimp/obm-l.html O administrador desta lista é ========================================================================= From owner-obm-l@sucuri.mat.puc-rio.br Mon Sep 9 21:58:13 2002 Return-Path: Received: (from majordom@localhost) by sucuri.mat.puc-rio.br (8.9.3/8.9.3) id VAA07155 for obm-l-MTTP; Mon, 9 Sep 2002 21:58:01 -0300 Received: from gorgo.centroin.com.br (gorgo.centroin.com.br [200.225.63.128]) by sucuri.mat.puc-rio.br (8.9.3/8.9.3) with ESMTP id VAA07151 for ; Mon, 9 Sep 2002 21:57:59 -0300 Received: from centroin.com.br (du110c.rjo.centroin.com.br [200.225.58.110]) (authenticated bits=0) by gorgo.centroin.com.br (8.12.2/8.12.1) with ESMTP id g8A0xKnH025902 for ; Mon, 9 Sep 2002 21:59:22 -0300 (BRT) Message-ID: <3D7D447C.2060109@centroin.com.br> Date: Mon, 09 Sep 2002 22:01:48 -0300 From: Augusto =?ISO-8859-1?Q?C=E9sar?= Morgado User-Agent: Mozilla/5.0 (Windows; U; Win98; en-US; rv:0.9.4.1) Gecko/20020508 Netscape6/6.2.3 X-Accept-Language: en-us MIME-Version: 1.0 To: obm-l@mat.puc-rio.br Subject: Re: [obm-l] probabilidade References: <5.1.0.14.2.20020906174140.018336b0@pop3.uol.com.br> Content-Type: text/plain; charset=ISO-8859-1; format=flowed Content-Transfer-Encoding: 8bit Sender: owner-obm-l@sucuri.mat.puc-rio.br Precedence: bulk Reply-To: obm-l@mat.puc-rio.br Numeremos os bandidos, por ordem de altura, de 1(mais baixo) a 5(mais alto). Ha tres casos em que o bandido 1 eh preso. caso 1: o bandido 1 sai em terceiro lugar. A probabilidade eh 1/5. caso 2: o bandido 1 sai em quarto lugar e o bandido que sai em terceiro eh mais alto que pelo menos um dos que sairam em primeiro e e em segundo. Vou escrever somente os 4 primeiros a sair: Isso corresponde as ordens 2341 2351 2431 2451 2531 2541 3241 3251 3451 3541 4251 4351 5231 5241 5341 A probabilidade eh 15/120 = 1/8. caso 3: o bandido 1 sai em quinto lugar e os bandidos que sairam em terceiro e quarto lugares sao ambos mais altos que pelo menos um dos que sairam em primeiro e e em segundo. Isso corresponde as ordens 23451 23541 24351 24531 25341 25431 32451 32541 35241 35421 42351 42531 52341 52431 A probabilidade eh 14/120 = 7/60 Portanto, a resposta eh 1/5 + 1/15 + 1/30 = 9/30 = 3/10. Nicks wrote: > Olá , > Poderiam me ajudar no seguinte problema ? > > Um inspetor sabe que o chefe de 5 bandidos é o mais baixo de todos > e que todas as alturas são diferentes . Sabe -se também que eles > estarão presentes numa reunião em um edifício . Depois da reunião > , os bandidos por medida de precaução deixam o edifício em um > intervalo de 15 minutos .Como o inspetor não sabe qual deles é o > mais baixo , decide deixar sair os dois primeiros bandidos , e > prender o primeiro dos seguintes que seja mais baixo do que os > que até esse momento sairam .Qual a probabilidade do inspetor > prender a pessoa certa ? > > []´s Nick > > > ========================================================================= > Instruções para entrar na lista, sair da lista e usar a lista em > http://www.mat.puc-rio.br/~nicolau/olimp/obm-l.html > O administrador desta lista é > ========================================================================= > > ========================================================================= Instruções para entrar na lista, sair da lista e usar a lista em http://www.mat.puc-rio.br/~nicolau/olimp/obm-l.html O administrador desta lista é ========================================================================= From owner-obm-l@sucuri.mat.puc-rio.br Mon Sep 9 22:02:11 2002 Return-Path: Received: (from majordom@localhost) by sucuri.mat.puc-rio.br (8.9.3/8.9.3) id WAA07302 for obm-l-MTTP; Mon, 9 Sep 2002 22:02:10 -0300 Received: from gorgo.centroin.com.br (gorgo.centroin.com.br [200.225.63.128]) by sucuri.mat.puc-rio.br (8.9.3/8.9.3) with ESMTP id WAA07277 for ; Mon, 9 Sep 2002 22:02:07 -0300 Received: from centroin.com.br (du110c.rjo.centroin.com.br [200.225.58.110]) (authenticated bits=0) by gorgo.centroin.com.br (8.12.2/8.12.1) with ESMTP id g8A13TnH026218 for ; Mon, 9 Sep 2002 22:03:29 -0300 (BRT) Message-ID: <3D7D4573.9090902@centroin.com.br> Date: Mon, 09 Sep 2002 22:05:55 -0300 From: Augusto =?ISO-8859-1?Q?C=E9sar?= Morgado User-Agent: Mozilla/5.0 (Windows; U; Win98; en-US; rv:0.9.4.1) Gecko/20020508 Netscape6/6.2.3 X-Accept-Language: en-us MIME-Version: 1.0 To: obm-l@mat.puc-rio.br Subject: Re: [obm-l] probabilidade References: <5.1.0.14.2.20020906174140.018336b0@pop3.uol.com.br> Content-Type: text/plain; charset=ISO-8859-1; format=flowed Content-Transfer-Encoding: 8bit Sender: owner-obm-l@sucuri.mat.puc-rio.br Precedence: bulk Reply-To: obm-l@mat.puc-rio.br Numeremos os bandidos, por ordem de altura, de 1(mais baixo) a 5(mais alto). Primeiro raciocinio: Ha tres casos em que o bandido 1 eh preso. caso 1: o bandido 1 sai em terceiro lugar. A probabilidade eh 1/5. caso 2: o bandido 1 sai em quarto lugar e o bandido que sai em terceiro eh mais alto que pelo menos um dos que sairam em primeiro e e em segundo. Vou escrever somente os 4 primeiros a sair: Isso corresponde as ordens 2341 2351 2431 2451 2531 2541 3241 3251 3451 3541 4231 4251 4351 5231 5241 5341 A probabilidade eh 16/120 = 2/15. caso 3: o bandido 1 sai em quinto lugar e os bandidos que sairam em terceiro e quarto lugares sao ambos mais altos que pelo menos um dos que sairam em primeiro e e em segundo. Isso corresponde as ordens 23451 23541 24351 24531 25341 25431 32451 32541 42351 42531 52341 52431 A probabilidade eh 12/120 = 1/10 Portanto, a resposta eh 1/5 + 2/15 + 1/10 = 13/30 . Segundo raciocinio: O bandido 1 nao eh preso nos casos seguintes: caso 1: o bandido 1 sai em primeiro lugar. A probabilidade eh 1/5. caso 2: o bandido 1 sai em segundo lugar. A probabilidade eh 1/5. caso 3: o bandido 1 sai em quarto lugar e o bandido que sai em terceiro eh menor que os dois que sairam primeiro. Isso corresponde a 3421 ou 4321 ou 3521 ou 5321 ou 4521 ou 5421 ou 4531 ou 5431 A probabilidade eh 8/120 = 1/15 caso 4: o bandido 1 sai em quinto lugar e o bandido que sai em terceiro eh menor que os dois que sairam em primeiro e segundo. Isso corresponde a 45 (ou 54) 32(ou23)1 ou 35(ou53)241 ou 34(ou43)251. Quatro+dois+dois possibilidades. A probabilidade eh 8/120 = 1/15 caso 5: o bandido 1 sai em quinto lugar e o bandido que sai em terceiro nao eh menor que os dois que sairam em primeiro e segundo e o que sai em quarto eh. Isso corresponde a 34(ou43)521 ou 35(ou53)421 A probab. eh 4/120=1/30. Portanto, a prob. de 1 NAO ser preso eh 1/5 +1/5 + 1/15 + 1/15 +1/30 = 17/30 e a de 1 ser preso eh 1 - 17/30 = 13/30. Nicks wrote: > Olá , > Poderiam me ajudar no seguinte problema ? > > Um inspetor sabe que o chefe de 5 bandidos é o mais baixo de todos > e que todas as alturas são diferentes . Sabe -se também que eles > estarão presentes numa reunião em um edifício . Depois da reunião > , os bandidos por medida de precaução deixam o edifício em um > intervalo de 15 minutos .Como o inspetor não sabe qual deles é o > mais baixo , decide deixar sair os dois primeiros bandidos , e > prender o primeiro dos seguintes que seja mais baixo do que os > que até esse momento sairam .Qual a probabilidade do inspetor > prender a pessoa certa ? > > []´s Nick > > > ========================================================================= > Instruções para entrar na lista, sair da lista e usar a lista em > http://www.mat.puc-rio.br/~nicolau/olimp/obm-l.html > O administrador desta lista é > ========================================================================= > > ========================================================================= Instruções para entrar na lista, sair da lista e usar a lista em http://www.mat.puc-rio.br/~nicolau/olimp/obm-l.html O administrador desta lista é ========================================================================= From owner-obm-l@sucuri.mat.puc-rio.br Tue Sep 10 01:00:28 2002 Return-Path: Received: (from majordom@localhost) by sucuri.mat.puc-rio.br (8.9.3/8.9.3) id BAA09727 for obm-l-MTTP; Tue, 10 Sep 2002 01:00:02 -0300 Received: from artemis.opendf.com.br (artemis.opengate.com.br [200.181.71.15]) by sucuri.mat.puc-rio.br (8.9.3/8.9.3) with ESMTP id AAA09720 for ; Tue, 10 Sep 2002 00:59:59 -0300 Received: from localhost (localhost [127.0.0.1]) by artemis.opendf.com.br (Postfix) with ESMTP id 1715B1C833 for ; Tue, 10 Sep 2002 01:00:52 -0300 (BRT) Received: from artur (200-181-88-010-bsace7001.dsl.telebrasilia.net.br [200.181.88.10]) by artemis.opendf.com.br (Postfix) with ESMTP id 2E5BE1BF90 for ; Tue, 10 Sep 2002 01:00:47 -0300 (BRT) From: "Artur Costa Steiner" To: Subject: [obm-l] =?iso-8859-1?Q?RE:_=5Bobm-l=5D_O_car=E1ter_n=E3o_enumer=E1vel_de_R?= Date: Tue, 10 Sep 2002 01:01:53 -0700 Organization: Steiner Consultoria LTDA Message-ID: <002101c258a0$554fcce0$0b01a8c0@mshome.net> MIME-Version: 1.0 Content-Type: multipart/alternative; boundary="----=_NextPart_000_0022_01C25865.A8F0F4E0" X-Priority: 3 (Normal) X-MSMail-Priority: Normal X-Mailer: Microsoft Outlook, Build 10.0.2627 X-MimeOLE: Produced By Microsoft MimeOLE V6.00.2600.0000 In-Reply-To: <20020909192842.95532.qmail@web12905.mail.yahoo.com> Importance: Normal X-Virus-Scanned: by AMaViS new-20020517 Sender: owner-obm-l@sucuri.mat.puc-rio.br Precedence: bulk Reply-To: obm-l@mat.puc-rio.br This is a multi-part message in MIME format. ------=_NextPart_000_0022_01C25865.A8F0F4E0 Content-Type: text/plain; charset="iso-8859-1" Content-Transfer-Encoding: quoted-printable =20 =20 -----Original Message----- From: owner-obm-l@sucuri.mat.puc-rio.br [mailto:owner-obm-l@sucuri.mat.puc-rio.br] On Behalf Of Johann Peter Gustav Lejeune Dirichlet Sent: Monday, September 09, 2002 11:29 AM To: obm-l@mat.puc-rio.br Subject: Re: [obm-l] O car=E1ter n=E3o enumer=E1vel de R =20 Eu acho que voce ta viajando demais.Enumeravel e o conjunto com uma bijecao nos naturais.=20 Sim, mas, porque vc acha que eu estou viajando? Uma das provas de que R n=E3o =E9 enumer=E1vel baseia-se no fato de que ele cont=E9m conjuntos = perfeitos e =E9 um espa=E7o m=E9trico completo. N=E3o h=E1 nenhuma viagem nisso. Os reais nao sao enumeraveis pelo fato de que N*N*N************nao e enumeravel=20 Vc se refere =E0 expans=E3o deciuma dos elementos de R.? 498 - Artur Costa Steiner escreveu:=20 Um abra=E7o a todos os amigos deste grupo no qual acabei de me = inscrever! O assunto que mencionei sempre me intriga um pouco. H=E1 uma cl=E1ssica=20 demonstra=E7=E3o de que R (o conjunto dos reais)n=E3o =E9 numer=E1vel e = que pode=20 ser encontrada na maioria dos livros sobre An=E1lise. Estas provas=20 baseiam-se no fato de que, nos espa=E7os euclidianos, conjuntos = perfeitos n=E3o s=E3o numer=E1veis. Logo, um ponto chave em tais provas =E9 que os = elementos do espa=E7o s=E3o pontos de acumula=E7=E3o do mesmo. Sabemos que todo elemento de R =E9 ponto de acumula=E7=E3o. Mas, e este = =E9 o=20 ponto que me intriga, tal conclus=E3o depende da m=E9trica definida em = R.=20 Na m=E9trica euclidiana usual tal fato =E9 demonstrado (admitindo-se que = R=20 seja completo). Mas, se tomarmos, por exemplo, a chamada m=E9trica=20 discreta (d(x,y)=3D1, se x<>y e d(x,y)=3D0 se x=3Dy))ent=E3o nenhum = elemento de=20 R (ou do espa=E7o m=E9trico em ! quest=E3o) =E9 ponto de acumula=E7=E3o. = A provas que=20 conhe=E7o sobre a n=E3o enumerabilidade de R (que consistem em se = construir=20 uma seq=FC=EAncia de intervalos fechados aninhados) n=E3o mais se = aplicam na=20 m=E9trica discreta. N=E3o me parece plaus=EDvel que um espa=E7o m=E9trico seja enumer=E1vel = numa=20 m=E9trica (ou topologia) e n=E3o numer=E1vel em outra, mas ser=E1 que = existe=20 uma prova de que R (ou um espa=E7o m=E9trico qualquer) n=E3o =E9 = numer=E1vel a=20 qual seja independente da forma segundo a qual definamos seus conjuntos=20 abertos? Artur =3D=3D=3D=3D=3D=3D=3D=3D=3D=3D=3D=3D=3D=3D=3D=3D=3D=3D=3D=3D=3D=3D=3D=3D=3D= =3D=3D=3D=3D=3D=3D=3D=3D=3D=3D=3D=3D=3D=3D=3D=3D=3D=3D=3D=3D=3D=3D=3D=3D=3D= =3D=3D=3D=3D=3D=3D=3D=3D=3D=3D=3D=3D=3D=3D=3D=3D=3D=3D=3D=3D=3D=3D =3D Instru=E7=F5es para entrar na lista, sair da lista e usar a lista em http://www.mat.puc-rio.br/~nicolau/olimp/obm-l.html O administrador desta lista =E9=20 =3D=3D=3D=3D=3D=3D=3D=3D=3D=3D=3D=3D=3D=3D=3D=3D=3D=3D=3D=3D=3D=3D=3D=3D=3D= =3D=3D=3D=3D=3D=3D=3D=3D=3D=3D=3D=3D=3D=3D=3D=3D=3D=3D=3D=3D=3D=3D=3D=3D=3D= =3D=3D=3D=3D=3D=3D=3D=3D=3D=3D=3D=3D=3D=3D=3D=3D=3D=3D=3D=3D=3D=3D =3D _____ =20 Yahoo! PageBuilder - O super editor para cria=E7=E3o de sites: =E9 gr=E1tis, f=E1cil e r=E1pido. ------=_NextPart_000_0022_01C25865.A8F0F4E0 Content-Type: text/html; charset="iso-8859-1" Content-Transfer-Encoding: quoted-printable

 

 

-----Original = Message-----
From: owner-obm-l@sucuri.mat.puc-rio.br = [mailto:owner-obm-l@sucuri.mat.puc-rio.br] On Behalf Of Johann Peter Gustav = Lejeune Dirichlet
Sent: =
Monday, September 09, = 2002 11:29 AM
To: = obm-l@mat.puc-rio.br
Subject: Re: [obm-l] O = car=E1ter n=E3o enumer=E1vel de R

 

Eu acho que voce ta viajando = demais.Enumeravel e o conjunto com uma bijecao nos naturais.

Sim, mas, porque vc acha que eu estou viajando? Uma das provas de = que R n=E3o =E9 enumer=E1vel baseia-se no fato de que ele cont=E9m conjuntos = perfeitos e =E9 um espa=E7o m=E9trico completo. N=E3o h=E1 nenhuma viagem = nisso.

Os reais = nao sao enumeraveis pelo fato de que N*N*N************nao e enumeravel =

=A0=A0=A0=A0=A0=A0=A0=A0=A0=A0=A0 Vc = se refere =E0 expans=E3o deciuma dos elementos de R.?

  = 498 - Artur Costa Steiner = <artur@opendf.com.br> = escreveu:

Um abra=E7o a todos os amigos deste grupo no = qual acabei de me inscrever!

O assunto que mencionei sempre me intriga um pouco. H=E1 uma cl=E1ssica =
demonstra=E7=E3o de que R (o conjunto dos reais)n=E3o =E9 numer=E1vel e = que pode
ser encontrada na maioria dos livros sobre An=E1lise. =
Estas provas
baseiam-se no fato de que, nos espa=E7os euclidianos, conjuntos = perfeitos n=E3o s=E3o numer=E1veis.
Logo, um ponto chave em tais provas =E9 que os
elementos do espa=E7o s=E3o pontos de acumula=E7=E3o do mesmo.

Sabemos que todo elemento de R =E9 ponto de acumula=E7=E3o. Mas, e este = =E9 o
ponto que me intriga, tal conclus=E3o depende da m=E9trica definida em = R.
Na m=E9trica euclidiana usual tal fato =E9 demonstrado (admitindo-se que = R
seja completo). Mas, se tomarmos, por exemplo, a chamada m=E9trica
discreta (d(x,y)=3D1, se x<>y e d(x,y)=3D0 se x=3Dy))ent=E3o = nenhum elemento de
R (ou do espa=E7o m=E9trico em ! quest=E3o) =E9 ponto de acumula=E7=E3o. = A provas que
conhe=E7o sobre a n=E3o enumerabilidade de R (que consistem em se = construir
uma seq=FC=EAncia de intervalos fechados aninhados) n=E3o mais se = aplicam na
m=E9trica discreta.

N=E3o me parece plaus=EDvel que um espa=E7o m=E9trico seja enumer=E1vel = numa
m=E9trica (ou topologia) e n=E3o numer=E1vel em outra, mas ser=E1 que = existe
uma prova de que R (ou um espa=E7o m=E9trico qualquer) n=E3o =E9 = numer=E1vel a
qual seja independente da forma segundo a qual definamos seus conjuntos =
abertos?

Artur
=3D=3D=3D=3D=3D=3D=3D=3D=3D=3D=3D=3D=3D=3D=3D=3D=3D=3D=3D=3D=3D=3D=3D=3D=3D= =3D=3D=3D=3D=3D=3D=3D=3D=3D=3D=3D=3D=3D=3D=3D=3D=3D=3D=3D=3D=3D=3D=3D=3D=3D= =3D=3D=3D=3D=3D=3D=3D=3D=3D=3D=3D=3D=3D=3D=3D=3D=3D=3D=3D=3D=3D=3D=3D
= Instru=E7=F5es para entrar na lista, sair da lista e usar a lista em
http://www.mat.puc-rio.br/~nicolau/olimp/obm-l.html
O administrador desta lista =E9
=3D=3D=3D=3D=3D=3D=3D=3D=3D=3D=3D=3D=3D=3D=3D=3D=3D= =3D=3D=3D=3D=3D=3D=3D=3D=3D=3D=3D=3D=3D=3D=3D=3D=3D=3D=3D=3D=3D=3D=3D=3D=3D= =3D=3D=3D=3D=3D=3D=3D=3D=3D=3D=3D=3D=3D=3D=3D=3D=3D=3D=3D=3D=3D=3D=3D=3D=3D= =3D=3D=3D=3D=3D=3D


Yahoo! PageBuilder - O super = editor para cria=E7=E3o de sites: =E9 gr=E1tis, f=E1cil e = r=E1pido.

------=_NextPart_000_0022_01C25865.A8F0F4E0-- ========================================================================= Instruções para entrar na lista, sair da lista e usar a lista em http://www.mat.puc-rio.br/~nicolau/olimp/obm-l.html O administrador desta lista é ========================================================================= From owner-obm-l@sucuri.mat.puc-rio.br Tue Sep 10 01:25:33 2002 Return-Path: Received: (from majordom@localhost) by sucuri.mat.puc-rio.br (8.9.3/8.9.3) id BAA10359 for obm-l-MTTP; Tue, 10 Sep 2002 01:25:12 -0300 Received: from hotmail.com (f229.pav2.hotmail.com [64.4.37.229]) by sucuri.mat.puc-rio.br (8.9.3/8.9.3) with ESMTP id BAA10355 for ; Tue, 10 Sep 2002 01:25:09 -0300 Received: from mail pickup service by hotmail.com with Microsoft SMTPSVC; Mon, 9 Sep 2002 21:26:07 -0700 Received: from 200.199.193.135 by pv2fd.pav2.hotmail.msn.com with HTTP; Tue, 10 Sep 2002 04:26:06 GMT X-Originating-IP: [200.199.193.135] From: "ricardo matos" To: obm-l@mat.puc-rio.br Subject: Re: [obm-l] Problema com inteiros gaussianos Date: Tue, 10 Sep 2002 01:26:06 -0300 Mime-Version: 1.0 Content-Type: text/plain; charset=iso-8859-1; format=flowed Message-ID: X-OriginalArrivalTime: 10 Sep 2002 04:26:07.0111 (UTC) FILETIME=[2EA14970:01C25882] Sender: owner-obm-l@sucuri.mat.puc-rio.br Precedence: bulk Reply-To: obm-l@mat.puc-rio.br O que ele tá querendo dizer é que se você só pisar nos números primos e se você deseja ir para o infinito não pode dar passos de tamanho limitado. A série de números compostos dada por ele faz com que para cada n existam espaços sem primos arbitrariamente grandes. E ele diz que o problema em números gaussianos não foi resolvido(andando no plano só pisando em primos gaussianos). >From: "Jackson Graziano" >Reply-To: obm-l@mat.puc-rio.br >To: >Subject: [obm-l] Problema com inteiros gaussianos >Date: Mon, 9 Sep 2002 19:50:18 -0300 > >Caros amigos, > >Como devo interpretar o enunciado abaixo? Nao consegui entender como são as >regras dele pra andar na reta real... > >One cannot walk to infinity on the real line if one uses steps of bounded >length and steps on the prime numbers. This is simply a restatement of the >classic result that there are arbitrarily large gaps in the primes. The >proof is simple: a gap of size k is given by (k + 1)! + 2, (k + 1)! + >3,..., >(k + 1)! + (k,+1). >But the same problem in the complex realm is unsolved. More precisely, an >analogous question asks whether one can walk to infinity in Z[i], the >Gaussian integers, using the Gaussian primes as stepping stones, and taking >steps of bounded length > >Obrigado, > >Jackson Graziano > >========================================================================= >Instruções para entrar na lista, sair da lista e usar a lista em >http://www.mat.puc-rio.br/~nicolau/olimp/obm-l.html >O administrador desta lista é >========================================================================= ========================================================================= Instruções para entrar na lista, sair da lista e usar a lista em http://www.mat.puc-rio.br/~nicolau/olimp/obm-l.html O administrador desta lista é ========================================================================= From owner-obm-l@sucuri.mat.puc-rio.br Tue Sep 10 01:38:57 2002 Return-Path: Received: (from majordom@localhost) by sucuri.mat.puc-rio.br (8.9.3/8.9.3) id BAA10836 for obm-l-MTTP; Tue, 10 Sep 2002 01:38:46 -0300 Received: from ginsberg.uol.com.br (ginsberg.uol.com.br [200.221.4.48]) by sucuri.mat.puc-rio.br (8.9.3/8.9.3) with ESMTP id BAA10831 for ; Tue, 10 Sep 2002 01:38:43 -0300 Received: from Standard ([200.182.237.20]) by ginsberg.uol.com.br (8.9.1/8.9.1) with SMTP id WAA27047 for ; Sun, 8 Sep 2002 22:54:15 -0300 (BRT) Message-ID: <001301c257a4$0f2f3400$14edb6c8@virtua.com.br> From: "Bruno Furlan" To: References: <002401c255a5$a48a7f60$0201a8c0@x> <000201c25798$4d8061e0$0200a8c0@dois> Subject: Re: [obm-l] um sistema Date: Sun, 8 Sep 2002 22:56:05 -0300 MIME-Version: 1.0 Content-Type: text/plain; charset="iso-8859-1" Content-Transfer-Encoding: 8bit X-Priority: 3 X-MSMail-Priority: Normal X-Mailer: Microsoft Outlook Express 5.50.4133.2400 X-MimeOLE: Produced By Microsoft MimeOLE V5.50.4133.2400 Sender: owner-obm-l@sucuri.mat.puc-rio.br Precedence: bulk Reply-To: obm-l@mat.puc-rio.br ax - by = 1 => a^2.x - aby = a bx + ay = 0 => b^2.x + aby = 0 (a^2 + b^2)x = a x = a/a^2 +b^2 bx + ay = 0 => ay = -ab/a^2+b^2 y = -b/a^2+b^2 O enunciado está certo. O determinante Dx é | 1 -b| e não | -b 1 |. | 0 a | | a 0 | Bruno F. > Sabendo que ax-by=1 e que ay+bx=0, prove que x= a/a^2 +b^2 e y = -b/a^2+b^2 > > > Resolução pelo teorema de Cramer: > > D = | a -b | = a² + b² > | b a | > > Dx = | -b 1 | = - a > | a 0 | > > Dy = | a 1 | = -b > | b 0 | > > > logo: > > x = Dx/D = -a/a² + b² > y = Dy/D= -b/a² + b² > > a resposta do x deu diferente do enunciado. Minha resoluçao ou o enunciado > que está incorreto? > > Gabriel ========================================================================= Instruções para entrar na lista, sair da lista e usar a lista em http://www.mat.puc-rio.br/~nicolau/olimp/obm-l.html O administrador desta lista é ========================================================================= From owner-obm-l@sucuri.mat.puc-rio.br Tue Sep 10 08:18:14 2002 Return-Path: Received: (from majordom@localhost) by sucuri.mat.puc-rio.br (8.9.3/8.9.3) id IAA14500 for obm-l-MTTP; Tue, 10 Sep 2002 08:16:19 -0300 Received: from shrek1.uol.com.br (shrek1.uol.com.br [200.221.4.16]) by sucuri.mat.puc-rio.br (8.9.3/8.9.3) with ESMTP id IAA14492 for ; Tue, 10 Sep 2002 08:16:01 -0300 Received: from mtauol11.mail.sys.intranet (172.26.5.67) by shrek1.uol.com.br (5.1.071) id 3D73A7C9000EE098 for obm-l@mat.puc-rio.br; Tue, 10 Sep 2002 08:14:28 -0300 Received: from uol.com.br (172.26.5.46) by mtauol11.mail.sys.intranet (5.1.071) id 3D7CF8260004232F for obm-l@mat.puc-rio.br; Tue, 10 Sep 2002 08:16:30 -0300 Date: Tue, 10 Sep 2002 08:19:38 -0300 Message-Id: Subject: Re:[obm-l] Um Estranho Sentimento ... MIME-Version: 1.0 Content-Type: multipart/mixed; boundary="_=__=_XaM3_Boundary.1031656778.2A.200625.42.701.52.42.101010.8458" From: "alexkirsten" To: obm-l@mat.puc-rio.br X-XaM3-API-Version: 2.4.3.4.4 X-SenderIP: 200.250.137.91 Sender: owner-obm-l@sucuri.mat.puc-rio.br Precedence: bulk Reply-To: obm-l@mat.puc-rio.br --_=__=_XaM3_Boundary.1031656778.2A.200625.42.701.52.42.101010.8458 Content-Type: text/plain;charset="iso-8859-1" Content-Transfer-Encoding: quoted-printable Imaginamos isso por causa das professoras do ensino prim=E1rio, que se utilizam deste meio para nos explicar como um n=FAmero =E9 maior que outro (5>3). Imagine como seria complicado para uma crian=E7a de 6 anos aprender que 5>3 simplesmente imaginando que 5 =E9 uma numero mais significativo que 3. Todos somos livres para contestar as professoras prim=E1rias, mas como, com 6 anos mal sabemos mascar chiclete e subir escadas ao mesmo tempo, engolimos o conhecimento que nos =E9 passado e achamos que aquela maneira =E9 a correta. Eu costumo pensar nos numeros (nos caso de 5>3) como sendo pontos agrupados em um conjunto, sem nenhum tipo de mensura=E7=E3o volum=E9trica ou linear. Voc=EA n=E3o esta ficando louco. Alex --- UOL Elei=E7=F5es 2002 - Todos os lances da disputa pol=EDtica http://eleicoes.uol.com.br/ --_=__=_XaM3_Boundary.1031656778.2A.200625.42.701.52.42.101010.8458 Content-Type: text/plain; name="00000JX0" Content-Transfer-Encoding: base64 T2xhIFBlc3NvYWwgZSBkZW1haXMNCmNvbGVnYXMgZGVzdGEgbGlzdGEgLi4uIE9CTS1MLA0K DQpFdSB0ZW5obyBwZW5zYWRvIGNvbnRpbnVhIGUgbG9uZ2FtZW50ZSBlbSB1bSBjb25qdW50 byBkZSBxdWVzdG9lcyANCnJlbGFjaW9uYWRhcyBxdWUgbWUgbGV2YW0sIGludmFyaWF2ZWxt ZW50ZSwgYSB1bWEgbWVzbWEgZGlyZWNhbyBxdWUgbmFvIA0KZXN0b3UgY29uc2VndWluZG8g YWNyZWRpdGFyIC4uLiBhdGUgcGFyZWNlIHF1ZSBkZXBvaXMgZGUgdGFudG8gcmVmbGV0aXIg DQpjaGVndWVpIGEgYWxndW1hIGNvbnN0YXRhY2FvIGluc2FuYSBvdSBzaW1wbG9yaWEgZGVt YWlzIC4uLiBTZSBhbGd1bSBjb2xlZ2EgDQpwdWRlciBmYWxhciBhbGdvIGVzY2xhcmVjZWRv ciBlL291IGludGVyZXNzYW50ZSAgZXUgZmljYXJlaSBtdWl0byBncmF0byAhDQoNCkRlc2Rl IGEgaW5mYW5jaWEgc29tb3MgaW5zdGFkb3MgYSBwZW5zYXIgcXVlIG9zIG51bWVyb3MgcmVh aXMgZXN0YW8gDQpkaXNwb3N0b3MgYW8gbG9uZ28gZGUgdW1hIGxpbmhhIHJldGEuIE5vcyBk aXplbW9zIDogMyA8IDUgISBFIGltZWRpYXRhbWVudGUgDQp2aXN1YWxpemFtb3MgbyAzIGEg ZXNxdWVyZGEgZG8gNSwgYW1ib3MgZW0gdW1hIGxpbmhhIHJldGEgISBQb3IgcXVlIG5vcyAN CnBlbnNhbW9zIGFzc2ltID8NCg0KRSB2ZXJkYWRlIHF1ZSBlbSBjdXJzb3MgZGUgYW5hbGlz ZSBvcyBsaXZyb3MgZGVmaW5lbSBSIGNvbW8gdW0gY29ycG8gDQpvcmRlbmFkbyBjb21wbGV0 byBlIGRlcml2YW0gYXMgcHJvcHJpZWRhZGVzIGRvcyBudW1lcm9zIHJlYWlzIGRvcyBheGlv bWFzIA0KcXVlIGRlZmluZW0gZXN0YSBlc3RydXR1cmEsIHNlbSByZWNvcnJlciBhIHF1YWxx dWVyIHByb3ByaWVkYWRlIGdlb21ldHJpY2EgDQpkZXJpdmFkYSBkZSB1bWEgZXZlbnR1YWwg dmlzdWFsaXphY2FvIGRvcyBudW1lcm9zIHNvYnJlIHVtYSByZXRhIC4uLg0KDQpNYXMgc2Us IHBvciB1bSBsYWRvLCBvcyBheGlvbWFzIGRlIHVtIGNvcnBvIG9yZGVuYWRvIGNvbXBsZXRv IG5hbyBpbXBsaWNhbSANCm91IHJlcXVlcmVtIGV4cGxpY2l0YW1lbnRlIHVtYSBlc3RydXR1 cmEgZ2VvbWV0cmljYSBjb25oZWNpZGEsIGUgaW5lZ2F2ZWwgDQpxdWUgYSB2aXN1YWxpemFj YW8gImluZm9ybWFsIiBxdWUgZmF6ZW1vcyBmYWNpbGl0YSBtdWl0b3MgcmFjaW9jaW5pbyAu Li4gDQpTRVJJQSBSRUFMTUVOVEUgSU1QT1NTSVZFTCBBU1NPQ0lBUiBBIFVNIENPUlBPIE9S REVOQURPIENPTVBMRVRPIFVNQSANCkdFT01FVFJJQSwgQVRSQVZFUyBERSBBWElPTUFTIE9V IE9VVFJPUyBSRUNVUlNPUywgREUgRk9STUEgUVVFIFBVREVTU0VNT1MgDQpURVIgT1VUUkFT IFZJU1VBTElaQUNPRVMsIE1FU01PIFFVRSBFU1RSQU5IQVMsIFBPUkVNLCBDT05NU0lTVEVO VEVTID8gTkFPIA0KUE9ERVJJQU0gQUxHVU1BUyBQUk9QUklFREFERVMgTlVNRVJJQ0FTIERF UEVOREVSRU0gSU5FWE9SQVZFTE1FTlRFIERFIFVNQSANClRBTCBHRU9NRVRSSUEgPw0KDQpF IGJlbSBwcm92YXZlbCBxdWUgZXUgZXN0ZWphIGVycmFkbywgbWFzIG5hbyBjb25zaWdvIHBl cmNlYmVyIG8gbWV1IGVycm8gLi4uIA0KRVUgQUNITyBRVUUgQ0VSVE9TIE1BUEVBTUVOVE9T IE5VTUVSSUNPUyBSRVFVRVJFTSBPVSAgSU1QTElDQU0gUVVFIE9TIA0KTlVNRVJPUyBOQVRV UkFJUyBURU0gVU1BIEdFT01FVFJJQSBPVSBFU1RSVVRVUkEgSU5UUklOU0VDQSwgU0VNIEEg UVVBTCBOQU8gDQpEQSBQUkEgQ09NUFJFRU5ERVIgQ0VSVEFTIENPSVNBUyAuLi4gRSBORUNF U1NBUklPIE9VIFBPU1RVTEFSIFVNQSBESVNQT1NJQ0FPIA0KRVNUUkFUSUdSQUZJQ0EgT1Ug U1VQT1IgUVVFIENFUlRPUyBNQVBFQU1FTlRPUyBJTkRVWkVNIFVNQSBUQUwgDQpFU1RSQVRJ RklDQUNBTyAuLi4NCg0KQm9tLCBzZSBhbGd1ZW0gcHVkZXIgZmFsYXIgYWxndW1hIGNvaXNh IGludGVyZXNzYW50ZSBzb2JyZSBlc3RlIHRlbWEgZXUgZmljbyANCm11aXRvIGdyYXRvLCBw b2lzIGVzdGUgZSByZWFsbWVudGUgdW0gU0VOVElNRU5UTyBFU1RSQU5ITyBxdWUgbWUgdGVt IA0Kb2NvcnJpZG8gY29tIGFsZ3VtYSBmcmVxdWVuY2lhLiBQb2RlIHNlciB1bWEgYnVycmlj ZSBtb21lbnRhbmVhIHF1ZSBlc3RhIG1lIA0KbGV2YW5kbyBhIGVzdGFzIHBlcmd1bnRhcyBh cGFyZW50ZW1lbnRlIGlkaW90YXMgZSBzZW0gc2VudGlkbywgbWFzIGV1IG5hbyANCmlyaWEg b2N1cGFyIG8gdGVtcG8gZGUgdm9jZXMsIG1ldXMgYW1pZ29zLCBzZSBuYW8gdGl2ZXNzZSBy YXpvZXMgc2VyaWEgcHJhIA0KZmF6ZXIgaXNzbyAuLi4NCg0KVW0gYWJyYWNvIGEgIFRvZG9z DQpQYXVsbyBTYW50YSBSaXRhDQoyLDE4MTgsMDkwOTAyDQoNCg0KX19fX19fX19fX19fX19f X19fX19fX19fX19fX19fX19fX19fX19fX19fX19fX19fX19fX19fX19fX19fX19fX18NCkNv bnZlcnNlIGNvbSBzZXVzIGFtaWdvcyBvbmxpbmUsIGZh52EgbyBkb3dubG9hZCBncuF0aXMg ZG8gTVNOIE1lc3NlbmdlcjogDQpodHRwOi8vbWVzc2VuZ2VyLm1zbi5jb20uYnINCg0KPT09 PT09PT09PT09PT09PT09PT09PT09PT09PT09PT09PT09PT09PT09PT09PT09PT09PT09PT09 PT09PT09PT09PT09PT09PQ0KSW5zdHJ15/VlcyBwYXJhIGVudHJhciBuYSBsaXN0YSwgc2Fp ciBkYSBsaXN0YSBlIHVzYXIgYSBsaXN0YSBlbQ0KaHR0cDovL3d3dy5tYXQucHVjLXJpby5i ci9+bmljb2xhdS9vbGltcC9vYm0tbC5odG1sDQpPIGFkbWluaXN0cmFkb3IgZGVzdGEgbGlz dGEg6SA8bmljb2xhdUBtYXQucHVjLXJpby5icj4NCj09PT09PT09PT09PT09PT09PT09PT09 PT09PT09PT09PT09PT09PT09PT09PT09PT09PT09PT09PT09PT09PT09PT09PT09PT0NCg== --_=__=_XaM3_Boundary.1031656778.2A.200625.42.701.52.42.101010.8458-- ========================================================================= Instruções para entrar na lista, sair da lista e usar a lista em http://www.mat.puc-rio.br/~nicolau/olimp/obm-l.html O administrador desta lista é ========================================================================= From owner-obm-l@sucuri.mat.puc-rio.br Tue Sep 10 08:49:02 2002 Return-Path: Received: (from majordom@localhost) by sucuri.mat.puc-rio.br (8.9.3/8.9.3) id IAA15009 for obm-l-MTTP; Tue, 10 Sep 2002 08:48:38 -0300 Received: (from nicolau@localhost) by sucuri.mat.puc-rio.br (8.9.3/8.9.3) id IAA15004 for obm-l@mat.puc-rio.br; Tue, 10 Sep 2002 08:48:38 -0300 Date: Tue, 10 Sep 2002 08:48:38 -0300 From: "Nicolau C. Saldanha" To: obm-l@mat.puc-rio.br Subject: [obm-l] Re: =?iso-8859-1?Q?=5Bobm-l=5D_Re:_=5Bobm-l=5D_O_car=E1ter_n=E3o_enumer=E1ve?= =?iso-8859-1?Q?l_de_R?= Message-ID: <20020910084838.B14851@sucuri.mat.puc-rio.br> References: Mime-Version: 1.0 Content-Type: text/plain; charset=iso-8859-1 Content-Disposition: inline Content-Transfer-Encoding: 8bit User-Agent: Mutt/1.2.5i In-Reply-To: ; from lauritoalves@hotmail.com on Mon, Sep 09, 2002 at 10:31:07PM +0000 Sender: owner-obm-l@sucuri.mat.puc-rio.br Precedence: bulk Reply-To: obm-l@mat.puc-rio.br On Mon, Sep 09, 2002 at 10:31:07PM +0000, Laurito Alves wrote: > O que você chama de N*N*N************ ??? > > Se for um produto cartesiano de N uma quantidade enumerável de vezes, ele é > enumerável. O limite direto das inclusões (usando o sinal < no lugar de está contido) N < N^2 < N^3 < ... é o conjunto das seqüências de naturais que são zero a partir de certo ponto; este conjunto é enumerável. Este conjunto às vezes é chamado de N^(N) ou N^infty. Mas há outra interpretação: o conjunto N^N de todas as seqüências de naturais (com qualquer comportamento no infinito) é não enumerável, tem o cardinal de R. Na verdade, via frações contínuas, é fácil construir uma bijeção natural entre este conjunto e o dos números irracionais. []s, N. ========================================================================= Instruções para entrar na lista, sair da lista e usar a lista em http://www.mat.puc-rio.br/~nicolau/olimp/obm-l.html O administrador desta lista é ========================================================================= From owner-obm-l@sucuri.mat.puc-rio.br Tue Sep 10 09:06:42 2002 Return-Path: Received: (from majordom@localhost) by sucuri.mat.puc-rio.br (8.9.3/8.9.3) id JAA15497 for obm-l-MTTP; Tue, 10 Sep 2002 09:04:17 -0300 Received: (from nicolau@localhost) by sucuri.mat.puc-rio.br (8.9.3/8.9.3) id JAA15492 for obm-l@mat.puc-rio.br; Tue, 10 Sep 2002 09:04:16 -0300 Date: Tue, 10 Sep 2002 09:04:16 -0300 From: "Nicolau C. Saldanha" To: obm-l@mat.puc-rio.br Subject: Re: [obm-l] violencia Message-ID: <20020910090416.A15169@sucuri.mat.puc-rio.br> References: Mime-Version: 1.0 Content-Type: text/plain; charset=iso-8859-1 Content-Disposition: inline Content-Transfer-Encoding: 8bit User-Agent: Mutt/1.2.5i In-Reply-To: ; from femedeiros2001@hotmail.com on Sat, Sep 07, 2002 at 11:45:37PM +0000 Sender: owner-obm-l@sucuri.mat.puc-rio.br Precedence: bulk Reply-To: obm-l@mat.puc-rio.br On Sat, Sep 07, 2002 at 11:45:37PM +0000, Fernanda Medeiros wrote: > > > Olá, > alguém pode dar uma ajuda nestas questões? > 1.a)uma "gang" tem infinitos bandidos e cada um dos meliantes tem um único > inimigo no interior da "gang",que ele quer matar.Prove q é possivel reunir > uma quantidade infinita de bandidos desta "gang", semq haja o risco de q > um bandido mate outro durante a reunião. > b)Se cada bandido tiver um nº finito mas indefinido de inimigos(um bandido > pode ter 2 inimigos, outro somente 1, um terceiro pode ter 20 e assim por > diante).Será sempre possivel promover uma reunião com infinitos bandidos sem > risco de derramamento de sangue? Sei que um monte de gente já respondeu mas gostei da questão e quero dar a minha solução. Antes de mais nada devemos entender que no enunciado estão implicitamente excluídos os bandidos suicidas (inimigos deles próprios)... (a) Se existe algum bandido x que é odiado por uma infinidade de outros bandidos, escolhemos todos os bandidos que odeiam x. Supomos a partir de agora que qualquer bandido é odiado apenas por um número finito de outros bandidos: assim a presença de um bandido na reunião só exclui um número finito de outros (o que ele odeia e os que o odeiam). É bem fácil montar um conjunto infinito: pegue um bandido qualquer, um que não tenha sido excluido pelo primeiro, um que não tenha sido excluido pelos dois primeiros,... A qualquer momento apenas um número finito de bandidos foi excluido logo podemos continuar. (b) Se os bandidos se chamam 0, 1, 2, 3, ..., n, ... e cada bandido odeia todos os que têm um número menor do que o dele é impossível reunir sequer dois bandidos. Mas o item que me parece mais interessante é: (c) se cada bandido odeia k outros bandidos (k fixo), ainda é possível reunir uma infinidade de bandidos? A resposta ainda é sim e provamos isso por indução em k. Imagine que cada bandido ordena seus inimigos de 1 a k. Por indução, podemos formar um subconjunto em que ninguém odeia ninguém nas (k-1) primeiras posições (mas talvez odeie na posição k). Acabamos de reduzir o problema ao item (a)... []s, N. ========================================================================= Instruções para entrar na lista, sair da lista e usar a lista em http://www.mat.puc-rio.br/~nicolau/olimp/obm-l.html O administrador desta lista é ========================================================================= From owner-obm-l@sucuri.mat.puc-rio.br Tue Sep 10 13:39:44 2002 Return-Path: Received: (from majordom@localhost) by sucuri.mat.puc-rio.br (8.9.3/8.9.3) id NAA20756 for obm-l-MTTP; Tue, 10 Sep 2002 13:37:56 -0300 Received: from traven.uol.com.br (traven.uol.com.br [200.221.4.39]) by sucuri.mat.puc-rio.br (8.9.3/8.9.3) with ESMTP id NAA20752 for ; Tue, 10 Sep 2002 13:37:39 -0300 Received: from u2z7z2 ([200.158.145.202]) by traven.uol.com.br (8.9.1/8.9.1) with ESMTP id NAA24901 for ; Tue, 10 Sep 2002 13:25:51 -0300 (BRT) Message-ID: <007e01c258e8$a7a41300$ca919ec8@u2z7z2> From: "Wagner" To: References: Subject: Re: [obm-l] Um Estranho Sentimento ... Date: Tue, 10 Sep 2002 13:39:38 -0300 Organization: Wagner MIME-Version: 1.0 Content-Type: text/plain; charset="iso-8859-1" Content-Transfer-Encoding: 8bit X-Priority: 3 X-MSMail-Priority: Normal X-Mailer: Microsoft Outlook Express 5.50.4133.2400 X-MimeOLE: Produced By Microsoft MimeOLE V5.50.4133.2400 Sender: owner-obm-l@sucuri.mat.puc-rio.br Precedence: bulk Reply-To: obm-l@mat.puc-rio.br Caro Paulo, a representação dos números como sendo pontos em uma retavem da idéia intuitiva de que números são quantidades. Mas na verdade números são símbolos que representam quantidades. Como os números são símbolos eles podem ser agrupados em conjuntos sem nenhum problema. André T. ----- Original Message ----- From: "Paulo Santa Rita" To: Sent: Monday, September 09, 2002 6:19 PM Subject: [obm-l] Um Estranho Sentimento ... > Ola Pessoal e demais > colegas desta lista ... OBM-L, > > Eu tenho pensado continua e longamente em um conjunto de questoes > relacionadas que me levam, invariavelmente, a uma mesma direcao que nao > estou conseguindo acreditar ... ate parece que depois de tanto refletir > cheguei a alguma constatacao insana ou simploria demais ... Se algum colega > puder falar algo esclarecedor e/ou interessante eu ficarei muito grato ! > > Desde a infancia somos instados a pensar que os numeros reais estao > dispostos ao longo de uma linha reta. Nos dizemos : 3 < 5 ! E imediatamente > visualizamos o 3 a esquerda do 5, ambos em uma linha reta ! Por que nos > pensamos assim ? > > E verdade que em cursos de analise os livros definem R como um corpo > ordenado completo e derivam as propriedades dos numeros reais dos axiomas > que definem esta estrutura, sem recorrer a qualquer propriedade geometrica > derivada de uma eventual visualizacao dos numeros sobre uma reta ... > > Mas se, por um lado, os axiomas de um corpo ordenado completo nao implicam > ou requerem explicitamente uma estrutura geometrica conhecida, e inegavel > que a visualizacao "informal" que fazemos facilita muitos raciocinio ... > SERIA REALMENTE IMPOSSIVEL ASSOCIAR A UM CORPO ORDENADO COMPLETO UMA > GEOMETRIA, ATRAVES DE AXIOMAS OU OUTROS RECURSOS, DE FORMA QUE PUDESSEMOS > TER OUTRAS VISUALIZACOES, MESMO QUE ESTRANHAS, POREM, CONMSISTENTES ? NAO > PODERIAM ALGUMAS PROPRIEDADES NUMERICAS DEPENDEREM INEXORAVELMENTE DE UMA > TAL GEOMETRIA ? > > E bem provavel que eu esteja errado, mas nao consigo perceber o meu erro ... > EU ACHO QUE CERTOS MAPEAMENTOS NUMERICOS REQUEREM OU IMPLICAM QUE OS > NUMEROS NATURAIS TEM UMA GEOMETRIA OU ESTRUTURA INTRINSECA, SEM A QUAL NAO > DA PRA COMPREENDER CERTAS COISAS ... E NECESSARIO OU POSTULAR UMA DISPOSICAO > ESTRATIGRAFICA OU SUPOR QUE CERTOS MAPEAMENTOS INDUZEM UMA TAL > ESTRATIFICACAO ... > > Bom, se alguem puder falar alguma coisa interessante sobre este tema eu fico > muito grato, pois este e realmente um SENTIMENTO ESTRANHO que me tem > ocorrido com alguma frequencia. Pode ser uma burrice momentanea que esta me > levando a estas perguntas aparentemente idiotas e sem sentido, mas eu nao > iria ocupar o tempo de voces, meus amigos, se nao tivesse razoes seria pra > fazer isso ... > > Um abraco a Todos > Paulo Santa Rita > 2,1818,090902 > > > _________________________________________________________________ > Converse com seus amigos online, faça o download grátis do MSN Messenger: > http://messenger.msn.com.br > > ========================================================================= > Instruções para entrar na lista, sair da lista e usar a lista em > http://www.mat.puc-rio.br/~nicolau/olimp/obm-l.html > O administrador desta lista é > ========================================================================= ========================================================================= Instruções para entrar na lista, sair da lista e usar a lista em http://www.mat.puc-rio.br/~nicolau/olimp/obm-l.html O administrador desta lista é ========================================================================= From owner-obm-l@sucuri.mat.puc-rio.br Tue Sep 10 13:51:22 2002 Return-Path: Received: (from majordom@localhost) by sucuri.mat.puc-rio.br (8.9.3/8.9.3) id NAA21013 for obm-l-MTTP; Tue, 10 Sep 2002 13:50:57 -0300 Received: from hotmail.com (f55.law4.hotmail.com [216.33.149.55]) by sucuri.mat.puc-rio.br (8.9.3/8.9.3) with ESMTP id NAA21009 for ; Tue, 10 Sep 2002 13:50:54 -0300 Received: from mail pickup service by hotmail.com with Microsoft SMTPSVC; Tue, 10 Sep 2002 09:51:53 -0700 Received: from 200.194.208.222 by lw4fd.law4.hotmail.msn.com with HTTP; Tue, 10 Sep 2002 16:51:53 GMT X-Originating-IP: [200.194.208.222] From: "Laurito Alves" To: obm-l@mat.puc-rio.br Subject: [obm-l] =?iso-8859-1?B?UmU6IFtvYm0tbF0gTyBjYXLhdGVyIG7jbyBlbnVtZXLhdmVsIGRlIFI=?= Date: Tue, 10 Sep 2002 16:51:53 +0000 Mime-Version: 1.0 Content-Type: text/plain; charset=iso-8859-1; format=flowed Message-ID: X-OriginalArrivalTime: 10 Sep 2002 16:51:53.0846 (UTC) FILETIME=[5DC33160:01C258EA] Sender: owner-obm-l@sucuri.mat.puc-rio.br Precedence: bulk Reply-To: obm-l@mat.puc-rio.br Colegas, Ou estou viajando muito ou é certo que a união enumerável de conjuntos enumeráveis é enumerável ? Se me lembro bem, provamos este fato com um argumento semelhante ao que Cantor usou para provar a enumerabilidade de Q. Onde está meu erro ? Laurito >From: "Nicolau C. Saldanha" >Reply-To: obm-l@mat.puc-rio.br >To: obm-l@mat.puc-rio.br >Subject: [obm-l] Re: [obm-l] Re: [obm-l] O caráter não enumerável de R >Date: Tue, 10 Sep 2002 08:48:38 -0300 > >On Mon, Sep 09, 2002 at 10:31:07PM +0000, Laurito Alves wrote: > > O que você chama de N*N*N************ ??? > > > > Se for um produto cartesiano de N uma quantidade enumerável de vezes, >ele é > > enumerável. > >O limite direto das inclusões (usando o sinal < no lugar de está contido) > >N < N^2 < N^3 < ... > >é o conjunto das seqüências de naturais que são zero a partir de certo >ponto; este conjunto é enumerável. Este conjunto às vezes é chamado >de N^(N) ou N^infty. Mas há outra interpretação: o conjunto N^N >de todas as seqüências de naturais (com qualquer comportamento no infinito) >é não enumerável, tem o cardinal de R. Na verdade, via frações contínuas, >é fácil construir uma bijeção natural entre este conjunto e o dos números >irracionais. > >[]s, N. >========================================================================= >Instruções para entrar na lista, sair da lista e usar a lista em >http://www.mat.puc-rio.br/~nicolau/olimp/obm-l.html >O administrador desta lista é >========================================================================= _________________________________________________________________ MSN Photos é a maneira mais fácil e prática de editar e compartilhar sua fotos: http://photos.msn.com.br ========================================================================= Instruções para entrar na lista, sair da lista e usar a lista em http://www.mat.puc-rio.br/~nicolau/olimp/obm-l.html O administrador desta lista é ========================================================================= From owner-obm-l@sucuri.mat.puc-rio.br Tue Sep 10 14:43:58 2002 Return-Path: Received: (from majordom@localhost) by sucuri.mat.puc-rio.br (8.9.3/8.9.3) id OAA22544 for obm-l-MTTP; Tue, 10 Sep 2002 14:42:24 -0300 Received: from silva5.uol.com.br (silva5.uol.com.br [200.221.4.52]) by sucuri.mat.puc-rio.br (8.9.3/8.9.3) with ESMTP id OAA22522 for ; Tue, 10 Sep 2002 14:42:13 -0300 Received: from u2z7z2 ([200.158.145.202]) by silva5.uol.com.br (8.9.1/8.9.1) with ESMTP id OAA21445 for ; Tue, 10 Sep 2002 14:45:35 -0300 (EST) Message-ID: <007101c258f1$b1bcd3a0$ca919ec8@u2z7z2> From: "Wagner" To: Subject: [obm-l] =?iso-8859-1?Q?Algumas_d=FAvidas?= Date: Tue, 10 Sep 2002 14:44:20 -0300 Organization: Wagner MIME-Version: 1.0 Content-Type: multipart/alternative; boundary="----=_NextPart_000_006E_01C258D8.8C01BEA0" X-Priority: 3 X-MSMail-Priority: Normal X-Mailer: Microsoft Outlook Express 5.50.4133.2400 X-MimeOLE: Produced By Microsoft MimeOLE V5.50.4133.2400 Sender: owner-obm-l@sucuri.mat.puc-rio.br Precedence: bulk Reply-To: obm-l@mat.puc-rio.br This is a multi-part message in MIME format. ------=_NextPart_000_006E_01C258D8.8C01BEA0 Content-Type: text/plain; charset="iso-8859-1" Content-Transfer-Encoding: quoted-printable Ol=E1 pessoal Ser=E1 que alguem poderia me ajudar com as seguintes d=FAvidas: -O que =E9 uma prova por indu=E7=E3o? -Qualquer n=FAmero pode ser descrito como o limite de uma sequ=EAncia = convergente? -Por que lim x--> 0 de 1/x existe, mas 1/x existe? Por favor merespondam essas perguntas Andr=E9 T. ------=_NextPart_000_006E_01C258D8.8C01BEA0 Content-Type: text/html; charset="iso-8859-1" Content-Transfer-Encoding: quoted-printable
Ol=E1 pessoal
 
Ser=E1 que alguem poderia me ajudar com = as seguintes=20 d=FAvidas:
 
-O que =E9 uma prova por = indu=E7=E3o?
-Qualquer n=FAmero pode ser descrito = como o limite de=20 uma sequ=EAncia convergente?
-Por que lim x--> 0 de 1/x existe, = mas 1/x=20 existe?
 
Por favor merespondam essas = perguntas
Andr=E9 T.
------=_NextPart_000_006E_01C258D8.8C01BEA0-- ========================================================================= Instruções para entrar na lista, sair da lista e usar a lista em http://www.mat.puc-rio.br/~nicolau/olimp/obm-l.html O administrador desta lista é ========================================================================= From owner-obm-l@sucuri.mat.puc-rio.br Tue Sep 10 14:54:04 2002 Return-Path: Received: (from majordom@localhost) by sucuri.mat.puc-rio.br (8.9.3/8.9.3) id OAA22840 for obm-l-MTTP; Tue, 10 Sep 2002 14:53:42 -0300 Received: from hotmail.com (oe92.pav0.hotmail.com [64.4.33.234]) by sucuri.mat.puc-rio.br (8.9.3/8.9.3) with ESMTP id OAA22836 for ; Tue, 10 Sep 2002 14:53:38 -0300 Received: from mail pickup service by hotmail.com with Microsoft SMTPSVC; Tue, 10 Sep 2002 10:54:37 -0700 X-Originating-IP: [200.217.188.1] From: "e isso mesmo" To: Subject: [obm-l] Continuo Perguntando... Date: Tue, 10 Sep 2002 14:57:34 -0300 MIME-Version: 1.0 X-Mailer: MSN Explorer 6.10.0016.1624 Content-Type: multipart/alternative; boundary="----=_NextPart_001_0000_01C258DA.651B54C0" Message-ID: X-OriginalArrivalTime: 10 Sep 2002 17:54:37.0980 (UTC) FILETIME=[215C75C0:01C258F3] Sender: owner-obm-l@sucuri.mat.puc-rio.br Precedence: bulk Reply-To: obm-l@mat.puc-rio.br ------=_NextPart_001_0000_01C258DA.651B54C0 Content-Type: text/plain; charset="iso-8859-1" Content-Transfer-Encoding: quoted-printable Amigos, algu=E9m poderia me explicar detalhadamente o que s=E3o n=FAmeros= rand=F4micos e como se constr=F3i esses n=FAmeros? Li isso em algum luga= r sobre loterias. Obrigado V=EDctorAproveite melhor a Web. Fa=E7a o download GR=C1TIS do MSN Explore= r : http://explorer.msn.com.br/intl.asp#po ------=_NextPart_001_0000_01C258DA.651B54C0 Content-Type: text/html; charset="iso-8859-1" Content-Transfer-Encoding: quoted-printable
 
<= DIV> 
=
 
Amigos, algu=E9m poderia me explicar detalhadamen= te o que s=E3o n=FAmeros rand=F4micos e como se constr=F3i esses n=FAmero= s? Li isso em algum lugar sobre loterias.
Obrigado
= V=EDctor


=
Aproveite melhor a Web. Fa=E7a o download GR=C1TIS do MSN Explorer : = http://explorer.msn.co= m.br/intl.asp#po

------=_NextPart_001_0000_01C258DA.651B54C0-- ========================================================================= Instruções para entrar na lista, sair da lista e usar a lista em http://www.mat.puc-rio.br/~nicolau/olimp/obm-l.html O administrador desta lista é ========================================================================= From owner-obm-l@sucuri.mat.puc-rio.br Tue Sep 10 15:15:13 2002 Return-Path: Received: (from majordom@localhost) by sucuri.mat.puc-rio.br (8.9.3/8.9.3) id PAA23689 for obm-l-MTTP; Tue, 10 Sep 2002 15:14:35 -0300 Received: from hotmail.com (f250.law8.hotmail.com [216.33.240.75]) by sucuri.mat.puc-rio.br (8.9.3/8.9.3) with ESMTP id PAA23685 for ; Tue, 10 Sep 2002 15:14:31 -0300 Received: from mail pickup service by hotmail.com with Microsoft SMTPSVC; Tue, 10 Sep 2002 11:15:31 -0700 Received: from 200.222.105.178 by lw8fd.law8.hotmail.msn.com with HTTP; Tue, 10 Sep 2002 18:15:30 GMT X-Originating-IP: [200.222.105.178] From: "Antonio Neto" To: obm-l@mat.puc-rio.br Subject: Re: [obm-l] Continuo Perguntando... Date: Tue, 10 Sep 2002 18:15:30 +0000 Mime-Version: 1.0 Content-Type: text/plain; format=flowed Message-ID: X-OriginalArrivalTime: 10 Sep 2002 18:15:31.0233 (UTC) FILETIME=[0C5BB110:01C258F6] Sender: owner-obm-l@sucuri.mat.puc-rio.br Precedence: bulk Reply-To: obm-l@mat.puc-rio.br >From: "e isso mesmo" >Reply-To: obm-l@mat.puc-rio.br >To: >Subject: [obm-l] Continuo Perguntando... >Date: Tue, 10 Sep 2002 14:57:34 -0300 > > > > >Amigos, alguém poderia me explicar detalhadamente o que são números >randômicos e como se constrói esses números? Li isso em algum lugar sobre >loterias. >Obrigado > Randomico eh ingles de Praça Mauah (sou do Rio de Janeiro), onde hah senhoritas que servem de guias aos marinheiros americanos, de onde tambem veio *rango*, de *hungry*. Em ingles, *at random* quer dizer ao acaso, e numeros randomicos sao numeros aleatorios. Abracos, olavo. _________________________________________________________________ Chat with friends online, try MSN Messenger: http://messenger.msn.com ========================================================================= Instruções para entrar na lista, sair da lista e usar a lista em http://www.mat.puc-rio.br/~nicolau/olimp/obm-l.html O administrador desta lista é ========================================================================= From owner-obm-l@sucuri.mat.puc-rio.br Tue Sep 10 15:41:02 2002 Return-Path: Received: (from majordom@localhost) by sucuri.mat.puc-rio.br (8.9.3/8.9.3) id PAA24743 for obm-l-MTTP; Tue, 10 Sep 2002 15:40:47 -0300 Received: from ginsberg.uol.com.br (ginsberg.uol.com.br [200.221.4.48]) by sucuri.mat.puc-rio.br (8.9.3/8.9.3) with ESMTP id PAA24736 for ; Tue, 10 Sep 2002 15:40:27 -0300 Received: from u2z7z2 ([200.158.145.202]) by ginsberg.uol.com.br (8.9.1/8.9.1) with ESMTP id PAA19140 for ; Tue, 10 Sep 2002 15:39:40 -0300 (BRT) Message-ID: <001901c258f9$c9bb6e00$ca919ec8@u2z7z2> From: "Wagner" To: Subject: [obm-l] Problema da OBM - 2001 Date: Tue, 10 Sep 2002 15:42:14 -0300 Organization: Wagner MIME-Version: 1.0 Content-Type: multipart/alternative; boundary="----=_NextPart_000_0016_01C258E0.A3061A80" X-Priority: 3 X-MSMail-Priority: Normal X-Mailer: Microsoft Outlook Express 5.50.4133.2400 X-MimeOLE: Produced By Microsoft MimeOLE V5.50.4133.2400 Sender: owner-obm-l@sucuri.mat.puc-rio.br Precedence: bulk Reply-To: obm-l@mat.puc-rio.br This is a multi-part message in MIME format. ------=_NextPart_000_0016_01C258E0.A3061A80 Content-Type: text/plain; charset="iso-8859-1" Content-Transfer-Encoding: quoted-printable Alo colegas de lista Essa mensagem =E9 sobre o 6=BA problema do n=EDvel 3 da 3=AA fase da OBM = do ano passado. A resolu=E7=E3o de Humberto Silva Navaes pode ser vista = na Eureka! n=BA13, 2002. Na solu=E7=E3o da segunda parte esta escrita abaixo: -Nota=E7=E3o: S(x) A: Somat=F3ria dos termos do tipo A para todos os = valores de x pertencentes ao evento. Sabemos que dada uma configura=E7=E3o inicial, independente das escolhas = dos movimentos sempre chegamos a uma configura=E7=E3o onde =E9 = imposs=EDvel mover (configura=E7=E3o parada). Suponha por absurdo que a = partir de uma configura=E7=E3o inicial se chegue a duas = configura=E7=F5es paradas distintas A e B.=20 Seja k' a posi=E7=E3o da pedra mais =E0 direita das configura=E7=F5es A = e B e k =3D k' + 2. Considere o seguinte invariante: (n=E3o varia a cada movimento)=20 I=3D S(x) F (k-pos(x)), onde Fn =E9 o n-=E9simo n=FAmero = de Fibonacci. Lembramos que F1 =3D 1, F2 =3D 1 e Fn + 2 =3D Fn + 1 + Fn, para todo n = =B3 1) Sabemos que Ia=3DIb ,pois I =E9 invariante, isto =E9, permanece o mesmo = depois de cada movimento. De fato, Fk - (p - 1) =3D Fk - p + 1 =3D Fk - p + Fk - p - 1 =3D Fk - p = + Fk - (p + 1), donde I n=E3o muda ap=F3s um movimento do tipo A, e F = (k-(p-1)) + F (k-(p+2)) =3D F (k-p) + F (k-p-1) + F (k-p-2)=3D 2 F = (k-p), donde I n=E3o muda ap=F3s um movimento do tipo B. Algoritmo: 1.. Seja x a pedra mais =E0 esquerda de A e y a pedra mais =E0 = esquerda de B. Devemos ter pos(x) =3D pos(y), pois se fosse pos(x) =B9 pos(y) = (assumimos sem perda de generalidade que pos(x) > pos(y)), ter=EDamos: Ia=3DS(t \in A) F (k-pos(t)), menor ou igual =E0 F (k-pos(x)) + F = (k-pos(x)-2) + F (k-pos(x)-4) + ... + F (2) se k-pos(x) for par e Ia=3D S(t \in A) F (k-pos(x)), menor ou igual =E0 F (k-pos(x)) + F = (k-pos(x)-2) + ... + F (3) caso contr=E1rio: Mas F (2) + ... + F (2k)=3DF(2k+1)-1, menor que F (2k+1) e F(3) + ... = + F (2k+1)=3D F (2k+2)-1, menor que F (2k+2). (como se prova facilmente = por indu=E7=E3o). logo Ia =E9 menor ou igual que F (k-pos(x)+1)-1, que =E9 menor que F = (k-pos(x)+1), menor ou igual =E0 F(k-pos(y),menor ou igual que Ib, um absurdo! 2.. Seja A: =3D A - {x} e B: =3D {y}. 3.. V=E1 para o 1. Pronto! Demonstramos que A e B s=E3o a mesma configura=E7=E3o, o que =E9 = um absurdo! A configura=E7=E3o final independe da escolha dos = movimentos. Queria uma explica=E7=E3o mais detalhada de cada passo e se isso implica = que as duas configura=E7=F5es s=E3o iguais por qu=EA se pode deduzir um = invariante. Andr=E9 T. =20 ------=_NextPart_000_0016_01C258E0.A3061A80 Content-Type: text/html; charset="iso-8859-1" Content-Transfer-Encoding: quoted-printable
Alo colegas de lista
 
Essa mensagem =E9 sobre o 6=BA problema = do n=EDvel 3 da=20 3=AA fase da OBM do ano passado. A resolu=E7=E3o de Humberto Silva = Navaes pode ser=20 vista na Eureka! n=BA13, 2002.
 
Na solu=E7=E3o da segunda = parte esta escrita=20 abaixo:
 
-Nota=E7=E3o: S(x) A: Somat=F3ria dos = termos do tipo A=20 para todos os valores de x pertencentes ao evento.
 

Sabemos que dada uma configura=E7=E3o inicial, = independente das=20 escolhas dos movimentos sempre chegamos a uma configura=E7=E3o onde =E9 = imposs=EDvel=20 mover (configura=E7=E3o parada). Suponha por absurdo que a partir de uma = configura=E7=E3o inicial se chegue a duas configura=E7=F5es paradas = distintas A e=20 B.

Seja k' a posi=E7=E3o da pedra mais =E0 = direita das=20 configura=E7=F5es A e B e k =3D k' + 2.

Considere o seguinte invariante: (n=E3o varia a cada = movimento)=20

         &nb= sp;    =20 I=3D S(x) F (k-pos(x)), onde = Fn =E9 o=20 n-=E9simo n=FAmero de Fibonacci.

Lembramos que F1 =3D 1, F2 =3D 1 = e=20 Fn + 2 =3D Fn + 1 + = Fn, para=20 todo n =B3 1)

Sabemos que Ia=3DIb ,pois I =E9 = invariante, isto =E9,=20 permanece o mesmo depois de cada movimento.

De fato, Fk – (p = – 1) =3D=20 Fk – p + 1 =3D Fk – p + = Fk –=20 p – 1 =3D Fk – p + = Fk – (p +=20 1), donde I n=E3o muda ap=F3s um movimento do tipo = A, e F=20 (k-(p-1)) + F (k-(p+2)) =3D F (k-p) + F (k-p-1) + F (k-p-2)=3D 2 F = (k-p), donde=20 I n=E3o muda ap=F3s um movimento do tipo B.

Algoritmo:

  1. Seja x a pedra mais =E0 esquerda de A e y a = pedra mais=20 =E0 esquerda de B.
  2. Devemos ter pos(x) =3D pos(y), pois = se fosse=20 pos(x) =B9 pos(y) (assumimos = sem perda de=20 generalidade que pos(x) > pos(y)), ter=EDamos:

    Ia=3DS(t \in A) F (k-pos(t)), menor ou igual = =E0 F (k-pos(x))=20 + F (k-pos(x)-2) + F (k-pos(x)-4) + ... + F (2)

    se k-pos(x) for par e

    Ia=3D S(t \in A) F (k-pos(x)), menor ou igual = =E0 F=20 (k-pos(x)) + F (k-pos(x)-2) + ... + F (3) caso = contr=E1rio:

    Mas F (2) + ... + F (2k)=3DF(2k+1)-1, = menor que F=20 (2k+1) e F(3) + ... + F (2k+1)=3D F (2k+2)-1, menor que=20 F (2k+2). (como se prova facilmente por = indu=E7=E3o).

    logo Ia =E9 menor ou igual que F = (k-pos(x)+1)-1,=20 que =E9 menor que F (k-pos(x)+1), menor ou igual =E0

    F(k-pos(y),menor ou igual que Ib, um=20 absurdo!

  3. Seja A: =3D A – {x} e B: =3D = {y}.
  4. V=E1 para o 1.

Pronto! Demonstramos que A e B s=E3o a = mesma=20 configura=E7=E3o, o que =E9 um absurdo! A configura=E7=E3o final = independe da escolha dos=20 movimentos.

Queria uma explica=E7=E3o mais = detalhada de cada passo=20 e se isso implica que as duas configura=E7=F5es s=E3o iguais por qu=EA = se pode deduzir=20 um invariante.

 

Andr=E9 T.

 

 

------=_NextPart_000_0016_01C258E0.A3061A80-- ========================================================================= Instruções para entrar na lista, sair da lista e usar a lista em http://www.mat.puc-rio.br/~nicolau/olimp/obm-l.html O administrador desta lista é ========================================================================= From owner-obm-l@sucuri.mat.puc-rio.br Tue Sep 10 16:03:40 2002 Return-Path: Received: (from majordom@localhost) by sucuri.mat.puc-rio.br (8.9.3/8.9.3) id QAA25466 for obm-l-MTTP; Tue, 10 Sep 2002 16:02:21 -0300 Received: from zeus.opendf.com.br (zeus.opengate.com.br [200.181.71.10]) by sucuri.mat.puc-rio.br (8.9.3/8.9.3) with ESMTP id QAA25452 for ; Tue, 10 Sep 2002 16:02:16 -0300 Received: from localhost (localhost.opengate.com.br [127.0.0.1]) by zeus.opendf.com.br (Postfix) with ESMTP id C6D653EB0E for ; Tue, 10 Sep 2002 16:03:12 -0300 (BRT) Received: by zeus.opendf.com.br (Postfix, from userid 48) id 12ABE3EB26; Tue, 10 Sep 2002 16:03:12 -0300 (BRT) From: "498 - Artur Costa Steiner" To: obm-l@mat.puc-rio.br Subject: Re: [obm-l] Continuo Perguntando... X-Mailer: NeoMail 1.25 X-IPAddress: 200.252.155.2 MIME-Version: 1.0 Content-Type: text/plain; charset=iso-8859-1 Message-Id: <20020910190312.12ABE3EB26@zeus.opendf.com.br> Date: Tue, 10 Sep 2002 16:03:12 -0300 (BRT) X-Virus-Scanned: by AMaViS new-20020517 Sender: owner-obm-l@sucuri.mat.puc-rio.br Precedence: bulk Reply-To: obm-l@mat.puc-rio.br > > > > > > Amigos, alguém poderia me explicar detalhadamente o que são números randômicos e como se constrói esses números? Li isso em algum lugar sobre loterias. > Obrigado > Víctor Oi Victor, Números randômicos, mais conhecidos por números aleatórios, são, conforme o próprio nome indica, números gerados aleatoriamente através de um algoritmo, geralmente implementado em um computador, segundo uma distribuição de probabilidade definida pelo usário. Na planilha Excel, por exemplo, há um gerador de números aleatórios, que permite ao usuário gerar tais números de acordo com distribuições uniformes, normais, poisson e outras. O usuário fornece um número básico, conhecido por semente, e o algoritmo gera os números aleatórios. Na verdade, números gerados assim são ditos pseudo-aleatórios, pois para uma mesma semente sempre correspondem os mesmos números alatórios. Um abraço Artur ========================================================================= Instruções para entrar na lista, sair da lista e usar a lista em http://www.mat.puc-rio.br/~nicolau/olimp/obm-l.html O administrador desta lista é ========================================================================= From owner-obm-l@sucuri.mat.puc-rio.br Tue Sep 10 16:19:06 2002 Return-Path: Received: (from majordom@localhost) by sucuri.mat.puc-rio.br (8.9.3/8.9.3) id QAA26147 for obm-l-MTTP; Tue, 10 Sep 2002 16:18:45 -0300 Received: from zeus.opendf.com.br (zeus.opengate.com.br [200.181.71.10]) by sucuri.mat.puc-rio.br (8.9.3/8.9.3) with ESMTP id QAA26140 for ; Tue, 10 Sep 2002 16:18:41 -0300 Received: from localhost (localhost.opengate.com.br [127.0.0.1]) by zeus.opendf.com.br (Postfix) with ESMTP id AF2513EB14 for ; Tue, 10 Sep 2002 16:19:39 -0300 (BRT) Received: by zeus.opendf.com.br (Postfix, from userid 48) id 9D3F53EB26; Tue, 10 Sep 2002 16:19:38 -0300 (BRT) From: "498 - Artur Costa Steiner" To: obm-l@mat.puc-rio.br Subject: Re: [obm-l] Algumas dúvidas X-Mailer: NeoMail 1.25 X-IPAddress: 200.252.155.2 MIME-Version: 1.0 Content-Type: text/plain; charset=iso-8859-1 Message-Id: <20020910191938.9D3F53EB26@zeus.opendf.com.br> Date: Tue, 10 Sep 2002 16:19:38 -0300 (BRT) X-Virus-Scanned: by AMaViS new-20020517 Sender: owner-obm-l@sucuri.mat.puc-rio.br Precedence: bulk Reply-To: obm-l@mat.puc-rio.br > > Olá pessoal > > Será que alguem poderia me ajudar com as seguintes dúvidas: > > -O que é uma prova por indução? É uma técnica utilizada para demosntrar que uma condição verifica-se para todo natural n. Inicialmente, vc prova que a condição vale para n=1; em seguida, supondo que valha para algum natural n, prova que vale para o natural n+1. Segundo o chamada Princípio da Indução Finita, isto acarreta que a condição valha para qualquer natural n. Um exemplo muito simples. Vamos demonstrar que a soma dos n primeiros naturais é dada por S_n = n(n+1)/2 . Para n=1, obtemos 1, logo a expressão é valida. Supondo-se que valha para algum natural n, temos que S_n+1 = S_n + n = n(n+1)/2 + n+1 = (n+1)(n+2)/2 , que é a expressão anterior quando se substitui n por n+1. Logo, a expressão é válida para o natural n+1, o que acarreta que seja válida para todo n (neste caso, poderíamos chegar à mesma conclusão utilizando-se progressão aritmética). Para vc aplicar este tipo de prova, vc tem que desconfiar previamente de que a expressão que quer provar é válida. > -Qualquer número pode ser descrito como o limite de uma sequência convergente? Sim, isto, aliás é uma das formas de se completar o conjunto dos números racionais e obter o conjunto dos reais, que é completo. Tais sequências são ditas de Cauchy (ou fundamentais) . > -Por que lim x--> 0 de 1/x existe, ????? não existe não! qaundo x tende a zero pela direita (respectivamente, esquerda), 1/x tende a infinito (respectiv, - infinito) observe que 1/0 não existe. mas 1/x existe? Existe para x<> 0. Nenhum real (ou mesmo complexo) não nulo pode ser divido por zero . E 0/0 não é definida. Abraços Artur > > Por favor merespondam essas perguntas > André T. > > -- OPEN Internet - o 1º Provedor do DF com anit-virus no servidor de e- mail! ========================================================================= Instruções para entrar na lista, sair da lista e usar a lista em http://www.mat.puc-rio.br/~nicolau/olimp/obm-l.html O administrador desta lista é ========================================================================= From owner-obm-l@sucuri.mat.puc-rio.br Tue Sep 10 16:22:09 2002 Return-Path: Received: (from majordom@localhost) by sucuri.mat.puc-rio.br (8.9.3/8.9.3) id QAA26286 for obm-l-MTTP; Tue, 10 Sep 2002 16:22:04 -0300 Received: from zeus.opendf.com.br (zeus.opengate.com.br [200.181.71.10]) by sucuri.mat.puc-rio.br (8.9.3/8.9.3) with ESMTP id QAA26280 for ; Tue, 10 Sep 2002 16:22:00 -0300 Received: from localhost (localhost.opengate.com.br [127.0.0.1]) by zeus.opendf.com.br (Postfix) with ESMTP id 743FB3EB43 for ; Tue, 10 Sep 2002 16:22:57 -0300 (BRT) Received: by zeus.opendf.com.br (Postfix, from userid 48) id 621243EB26; Tue, 10 Sep 2002 16:22:54 -0300 (BRT) From: "498 - Artur Costa Steiner" To: obm-l@mat.puc-rio.br, obm-l@mat.puc-rio.br Subject: Re: [obm-l] Re: [obm-l] O caráter não enumerável de R X-Mailer: NeoMail 1.25 X-IPAddress: 200.252.155.2 MIME-Version: 1.0 Content-Type: text/plain; charset=iso-8859-1 Message-Id: <20020910192254.621243EB26@zeus.opendf.com.br> Date: Tue, 10 Sep 2002 16:22:54 -0300 (BRT) X-Virus-Scanned: by AMaViS new-20020517 Sender: owner-obm-l@sucuri.mat.puc-rio.br Precedence: bulk Reply-To: obm-l@mat.puc-rio.br > Colegas, > > Ou estou viajando muito ou é certo que a união enumerável de conjuntos > enumeráveis é enumerável ? > > Se me lembro bem, provamos este fato com um argumento semelhante ao que > Cantor usou para provar a enumerabilidade de Q. > > Onde está meu erro ? Não há erro, isto é verdade. Artur ========================================================================= Instruções para entrar na lista, sair da lista e usar a lista em http://www.mat.puc-rio.br/~nicolau/olimp/obm-l.html O administrador desta lista é ========================================================================= From owner-obm-l@sucuri.mat.puc-rio.br Tue Sep 10 16:38:33 2002 Return-Path: Received: (from majordom@localhost) by sucuri.mat.puc-rio.br (8.9.3/8.9.3) id QAA27312 for obm-l-MTTP; Tue, 10 Sep 2002 16:36:21 -0300 Received: (from nicolau@localhost) by sucuri.mat.puc-rio.br (8.9.3/8.9.3) id QAA27307 for obm-l@mat.puc-rio.br; Tue, 10 Sep 2002 16:36:21 -0300 Date: Tue, 10 Sep 2002 16:36:20 -0300 From: "Nicolau C. Saldanha" To: obm-l@mat.puc-rio.br Subject: [obm-l] Re: =?iso-8859-1?Q?=5Bobm-l=5D_Re:_=5Bobm-l=5D_O_car=E1ter_n=E3o_enumer=E1ve?= =?iso-8859-1?Q?l_de_R?= Message-ID: <20020910163620.A27048@sucuri.mat.puc-rio.br> References: Mime-Version: 1.0 Content-Type: text/plain; charset=iso-8859-1 Content-Disposition: inline Content-Transfer-Encoding: 8bit User-Agent: Mutt/1.2.5i In-Reply-To: ; from lauritoalves@hotmail.com on Tue, Sep 10, 2002 at 04:51:53PM +0000 Sender: owner-obm-l@sucuri.mat.puc-rio.br Precedence: bulk Reply-To: obm-l@mat.puc-rio.br On Tue, Sep 10, 2002 at 04:51:53PM +0000, Laurito Alves wrote: > Colegas, > > Ou estou viajando muito ou é certo que a união enumerável de conjuntos > enumeráveis é enumerável ? Sim, a união enumerável de conjunto enumeráveis é enumerável. > > Se me lembro bem, provamos este fato com um argumento semelhante ao que > Cantor usou para provar a enumerabilidade de Q. > > Onde está meu erro ? Você deveria repetir o raciocínio, assim eu não entendo a pergunta... []s, N. ========================================================================= Instruções para entrar na lista, sair da lista e usar a lista em http://www.mat.puc-rio.br/~nicolau/olimp/obm-l.html O administrador desta lista é ========================================================================= From owner-obm-l@sucuri.mat.puc-rio.br Tue Sep 10 16:42:27 2002 Return-Path: Received: (from majordom@localhost) by sucuri.mat.puc-rio.br (8.9.3/8.9.3) id QAA27673 for obm-l-MTTP; Tue, 10 Sep 2002 16:41:05 -0300 Received: (from nicolau@localhost) by sucuri.mat.puc-rio.br (8.9.3/8.9.3) id QAA27667 for obm-l@mat.puc-rio.br; Tue, 10 Sep 2002 16:41:04 -0300 Date: Tue, 10 Sep 2002 16:41:04 -0300 From: "Nicolau C. Saldanha" To: obm-l@mat.puc-rio.br Subject: Re: [obm-l] Continuo Perguntando... Message-ID: <20020910164104.B27048@sucuri.mat.puc-rio.br> References: Mime-Version: 1.0 Content-Type: text/plain; charset=iso-8859-1 Content-Disposition: inline Content-Transfer-Encoding: 8bit User-Agent: Mutt/1.2.5i In-Reply-To: ; from rdrql@msn.com on Tue, Sep 10, 2002 at 02:57:34PM -0300 Sender: owner-obm-l@sucuri.mat.puc-rio.br Precedence: bulk Reply-To: obm-l@mat.puc-rio.br On Tue, Sep 10, 2002 at 02:57:34PM -0300, e isso mesmo wrote: > Amigos, alguém poderia me explicar detalhadamente o que são números > randômicos e como se constrói esses números? Li isso em algum lugar sobre > loterias. Obrigado Se você está interessado em saber como fazer um algoritmo para gerar número pseudo-aleatórios, veja o clássico 'The Art of Computer Programming', de D. Knuth. O assunto é não trivial e muitos softwares que supostamente geram números aleatórios fazem um péssimo serviço. Pelo que eu sei loterias não usam computadores, usam aquelas gaiolonas esféricas cheias de bolinhas que você vê na televisão. []s, N. ========================================================================= Instruções para entrar na lista, sair da lista e usar a lista em http://www.mat.puc-rio.br/~nicolau/olimp/obm-l.html O administrador desta lista é ========================================================================= From owner-obm-l@sucuri.mat.puc-rio.br Tue Sep 10 16:47:00 2002 Return-Path: Received: (from majordom@localhost) by sucuri.mat.puc-rio.br (8.9.3/8.9.3) id QAA27961 for obm-l-MTTP; Tue, 10 Sep 2002 16:46:23 -0300 Received: (from nicolau@localhost) by sucuri.mat.puc-rio.br (8.9.3/8.9.3) id QAA27933 for obm-l@mat.puc-rio.br; Tue, 10 Sep 2002 16:46:18 -0300 Date: Tue, 10 Sep 2002 16:46:16 -0300 From: "Nicolau C. Saldanha" To: obm-l@mat.puc-rio.br Subject: Re: [obm-l] violencia Message-ID: <20020910164616.C27048@sucuri.mat.puc-rio.br> References: <20020910090416.A15169@sucuri.mat.puc-rio.br> Mime-Version: 1.0 Content-Type: text/plain; charset=iso-8859-1 Content-Disposition: inline Content-Transfer-Encoding: 8bit User-Agent: Mutt/1.2.5i In-Reply-To: <20020910090416.A15169@sucuri.mat.puc-rio.br>; from nicolau@sucuri.mat.puc-rio.br on Tue, Sep 10, 2002 at 09:04:16AM -0300 Sender: owner-obm-l@sucuri.mat.puc-rio.br Precedence: bulk Reply-To: obm-l@mat.puc-rio.br On Tue, Sep 10, 2002 at 09:04:16AM -0300, Nicolau C. Saldanha wrote: > On Sat, Sep 07, 2002 at 11:45:37PM +0000, Fernanda Medeiros wrote: > > > > > > Olá, > > alguém pode dar uma ajuda nestas questões? > > 1.a)uma "gang" tem infinitos bandidos e cada um dos meliantes tem um único > > inimigo no interior da "gang",que ele quer matar.Prove q é possivel reunir > > uma quantidade infinita de bandidos desta "gang", semq haja o risco de q > > um bandido mate outro durante a reunião. > > (a) Se existe algum bandido x que é odiado por uma infinidade de outros > bandidos, escolhemos todos os bandidos que odeiam x. Supomos a partir > de agora que qualquer bandido é odiado apenas por um número finito > de outros bandidos: assim a presença de um bandido na reunião só exclui > um número finito de outros (o que ele odeia e os que o odeiam). > É bem fácil montar um conjunto infinito: pegue um bandido qualquer, > um que não tenha sido excluido pelo primeiro, um que não tenha sido > excluido pelos dois primeiros,... A qualquer momento apenas um número > finito de bandidos foi excluido logo podemos continuar. Alguém andou perguntando se esta demonstração precisa do axioma da escolha. Precisa. Mesmo no caso particular em que os bandidos se odeiam aos pares (se x odeia y então y odeia x) e que basta pegar um bandido de cada par estamos usando o axioma da escolha (*qual* bandido de cada par você vai pegar?). Isto não deveria incomodar ninguém. O axioma da escolha é usado implicitamente o tempo todo. []s, N. ========================================================================= Instruções para entrar na lista, sair da lista e usar a lista em http://www.mat.puc-rio.br/~nicolau/olimp/obm-l.html O administrador desta lista é ========================================================================= From owner-obm-l@sucuri.mat.puc-rio.br Tue Sep 10 17:14:20 2002 Return-Path: Received: (from majordom@localhost) by sucuri.mat.puc-rio.br (8.9.3/8.9.3) id RAA29580 for obm-l-MTTP; Tue, 10 Sep 2002 17:12:56 -0300 Received: from sr1.terra.com.br (sr1.terra.com.br [200.176.3.16]) by sucuri.mat.puc-rio.br (8.9.3/8.9.3) with ESMTP id RAA29576 for ; Tue, 10 Sep 2002 17:12:53 -0300 Received: from pavuna.terra.com.br (pavuna.terra.com.br [200.176.3.41]) by sr1.terra.com.br (Postfix) with ESMTP id B978971B8D for ; Tue, 10 Sep 2002 17:03:03 -0300 (EST) Received: from terra.com.br (webmail3.terra.com.br [200.176.3.178]) (authenticated user mgmelo) by pavuna.terra.com.br (Postfix) with ESMTP id 6D91F689EB; Tue, 10 Sep 2002 17:03:02 -0300 (EST) Date: Tue, 10 Sep 2002 18:03:02 -0200 Message-Id: Subject: [obm-l] Randomico x anglicismos MIME-Version: 1.0 Content-Type: text/plain;charset="iso-8859-1" From: "Marcos Melo" To: "obm-l" Cc: "obm-l" X-XaM3-API-Version: 2.4 R5 B5 JSC SMTP AUTH X-SenderIP: 200.228.200.3 Content-Transfer-Encoding: 8bit X-MIME-Autoconverted: from quoted-printable to 8bit by sucuri.mat.puc-rio.br id RAA29577 Sender: owner-obm-l@sucuri.mat.puc-rio.br Precedence: bulk Reply-To: obm-l@mat.puc-rio.br Randomico como tantos outros anglicismos já está no HOUAISS. Lá também está deletar. Forró, cuja origem é "for all" não consta lá como anglicismo. De forma similar "cut a bit" que era a expressão utilizada pelos engenheiros americanos que construiram estradas para aeroportos no nordeste, virou, na terra do Elon Lages, o termo "catabi" (que eu sempre ouvi como "catabil" ou "catabiu") e consta no HOUAISS como, da mesma forma que "catabïl", com o significado: "1. irregularidade de terreno que provoca solavanco em veículos; costela de vaca, costela 2. O solavanco produzido por essa irregularidade. Etim: sugere origem onomotopaica" Ou seja, "catabiu" é um anglicismo (apesar do HOUAISS achar que a origem é onomatopaica) usado no interior de Alagoas que significa "lombada concava ou convexa que provoca solavancos" A língua é viva. SDS, Marcos Melo. > ---------- Mensagem original ----------- > > De : owner-obm-l@sucuri.mat.puc-rio.br > Para : obm-l@mat.puc-rio.br > Cc : > Data : Tue, 10 Sep 2002 18:15:30 +0000 > Assunto : Re: [obm-l] Continuo Perguntando... > > > > > >From: "e isso mesmo" > >Reply-To: obm-l@mat.puc-rio.br > >To: > >Subject: [obm-l] Continuo Perguntando... > >Date: Tue, 10 Sep 2002 14:57:34 -0300 > > > > > > > > > >Amigos, alguém poderia me explicar detalhadamente o que são números > >randômicos e como se constrói esses números? Li isso em algum lugar sobre > >loterias. > >Obrigado > > > > Randomico eh ingles de Praça Mauah (sou do Rio de Janeiro), onde hah > senhoritas que servem de guias aos marinheiros americanos, de onde t ambem > veio *rango*, de *hungry*. Em ingles, *at random* quer dizer ao acas o, e > numeros randomicos sao numeros aleatorios. Abracos, olavo. > > _________________________________________________________________ > Chat with friends online, try MSN Messenger: http://messenger.msn.co m > > ==================================================================== ===== > Instruções para entrar na lista, sair da lista e usar a lista em > http://www.mat.puc-rio.br/~nicolau/olimp/obm-l.html > O administrador desta lista é > ==================================================================== ===== > > ========================================================================= Instruções para entrar na lista, sair da lista e usar a lista em http://www.mat.puc-rio.br/~nicolau/olimp/obm-l.html O administrador desta lista é ========================================================================= From owner-obm-l@sucuri.mat.puc-rio.br Tue Sep 10 17:21:06 2002 Return-Path: Received: (from majordom@localhost) by sucuri.mat.puc-rio.br (8.9.3/8.9.3) id RAA30083 for obm-l-MTTP; Tue, 10 Sep 2002 17:20:52 -0300 Received: from mediterraneo.rjnet.com.br (mediterraneo.rjnet.com.br [200.222.31.30]) by sucuri.mat.puc-rio.br (8.9.3/8.9.3) with ESMTP id RAA30058 for ; Tue, 10 Sep 2002 17:20:44 -0300 Received: from locutus.rjnet.com.br (root@locutus.rjnet.com.br [200.222.31.10]) by mediterraneo.rjnet.com.br (8.11.4/8.11.4) with ESMTP id g8AKMLC29146 for ; Tue, 10 Sep 2002 17:22:21 -0300 Received: from felipe ([200.222.39.86]) by locutus.rjnet.com.br (8.11.2/8.11.2) with SMTP id g8AK2O228099 for ; Tue, 10 Sep 2002 17:02:26 -0300 Message-ID: <000901c25907$aca0d5e0$158c000a@felipe> From: "Felipe Villela Dias" To: References: <20020910190312.12ABE3EB26@zeus.opendf.com.br> Subject: Re: [obm-l] Continuo Perguntando... Date: Tue, 10 Sep 2002 17:21:30 -0300 MIME-Version: 1.0 Content-Type: text/plain; charset="iso-8859-1" Content-Transfer-Encoding: 8bit X-Priority: 3 X-MSMail-Priority: Normal X-Mailer: Microsoft Outlook Express 6.00.2600.0000 X-MimeOLE: Produced By Microsoft MimeOLE V6.00.2600.0000 Sender: owner-obm-l@sucuri.mat.puc-rio.br Precedence: bulk Reply-To: obm-l@mat.puc-rio.br Eu li algum tempo atrás que alguns programas de computador geram números aleatórios realizando operações matemáticas com os numeros da hora do computador. Por exemplo, pegar os milésimos de segundo da hora registrada no sistema multiplicar pelos segundos e elevar tudo ao quadrado... assim os números gerados dependerão apenas da hora de seu computador... ----- Original Message ----- From: "498 - Artur Costa Steiner" To: Sent: Tuesday, September 10, 2002 4:03 PM Subject: Re: [obm-l] Continuo Perguntando... > > > > > > > > > > > > > > Amigos, alguém poderia me explicar detalhadamente o que são números > randômicos e como se constrói esses números? Li isso em algum lugar > sobre loterias. > > Obrigado > > Víctor > > Oi Victor, > Números randômicos, mais conhecidos por números aleatórios, são, > conforme o próprio nome indica, números gerados aleatoriamente através > de um algoritmo, geralmente implementado em um computador, segundo uma > distribuição de probabilidade definida pelo usário. Na planilha Excel, > por exemplo, há um gerador de números aleatórios, que permite ao > usuário gerar tais números de acordo com distribuições uniformes, > normais, poisson e outras. O usuário fornece um número básico, > conhecido por semente, e o algoritmo gera os números aleatórios. Na > verdade, números gerados assim são ditos pseudo-aleatórios, pois para > uma mesma semente sempre correspondem os mesmos números alatórios. > > Um abraço > Artur > ========================================================================= > Instruções para entrar na lista, sair da lista e usar a lista em > http://www.mat.puc-rio.br/~nicolau/olimp/obm-l.html > O administrador desta lista é > ========================================================================= > ========================================================================= Instruções para entrar na lista, sair da lista e usar a lista em http://www.mat.puc-rio.br/~nicolau/olimp/obm-l.html O administrador desta lista é ========================================================================= From owner-obm-l@sucuri.mat.puc-rio.br Tue Sep 10 19:32:06 2002 Return-Path: Received: (from majordom@localhost) by sucuri.mat.puc-rio.br (8.9.3/8.9.3) id TAA00563 for obm-l-MTTP; Tue, 10 Sep 2002 19:31:38 -0300 Received: from ginsberg.uol.com.br (ginsberg.uol.com.br [200.221.4.48]) by sucuri.mat.puc-rio.br (8.9.3/8.9.3) with ESMTP id TAA00559 for ; Tue, 10 Sep 2002 19:31:36 -0300 Received: from u2z7z2 ([200.158.145.229]) by ginsberg.uol.com.br (8.9.1/8.9.1) with ESMTP id TAA26449 for ; Tue, 10 Sep 2002 19:30:55 -0300 (BRT) Message-ID: <000b01c2591a$1d1da8e0$2101a8c0@u2z7z2> From: "Wagner" To: References: Subject: Re: [obm-l] Continuo Perguntando... Date: Tue, 10 Sep 2002 19:33:40 -0300 Organization: Wagner MIME-Version: 1.0 Content-Type: multipart/alternative; boundary="----=_NextPart_000_0008_01C25900.F720F700" X-Priority: 3 X-MSMail-Priority: Normal X-Mailer: Microsoft Outlook Express 5.50.4133.2400 X-MimeOLE: Produced By Microsoft MimeOLE V5.50.4133.2400 Sender: owner-obm-l@sucuri.mat.puc-rio.br Precedence: bulk Reply-To: obm-l@mat.puc-rio.br This is a multi-part message in MIME format. ------=_NextPart_000_0008_01C25900.F720F700 Content-Type: text/plain; charset="iso-8859-1" Content-Transfer-Encoding: quoted-printable Oi pessoal N=FAmeros rand=F4micos s=E3o n=FAmeros construidos a partir de um = algoritmo e um n=FAmero inicial (semente). Exemplo: Considere o algoritmo A abaixo em que o n=BA inicial entra como = x(0). 1- y=3D2x(n) 2- z =E9 o resto da divis=E3o de y por 3 (yz>0, ou yz=3D0) 3- w=3Dxyz-2 4- a =E9 o quadrado do resto da divis=E3o de w por 7 5- b=3Dw-a+x(n) 6- b =E9 o (n+1)-=E9simo termo da sequ=EAncia A. 7- b-1=3Dx(n+1) A sequ=EAncia A para semente 1 =E9 A: 1,-1,-43,-7477,... para semente 0 A: 0,-6,-12,-18,-24 (A apar=EAncia aleat=F3ria da sequ=EAncia depende da semente) Aqui vai uma boa pergunta sobre esses n=FAmeros, =E9 poss=EDvel provar = que uma sequ=EAncia aleat=F3ria qualquer segue um algoritmo em que o = valor seguinte depende dos anteriores? Andr=E9 T. ----- Original Message -----=20 From: e isso mesmo=20 To: obm-l@mat.puc-rio.br=20 Sent: Tuesday, September 10, 2002 2:57 PM Subject: [obm-l] Continuo Perguntando... Amigos, algu=E9m poderia me explicar detalhadamente o que s=E3o = n=FAmeros rand=F4micos e como se constr=F3i esses n=FAmeros? Li isso em = algum lugar sobre loterias. Obrigado V=EDctor -------------------------------------------------------------------------= ----- Aproveite melhor a Web. Fa=E7a o download GR=C1TIS do MSN Explorer : = http://explorer.msn.com.br/intl.asp#po ------=_NextPart_000_0008_01C25900.F720F700 Content-Type: text/html; charset="iso-8859-1" Content-Transfer-Encoding: quoted-printable
Oi pessoal
 
N=FAmeros rand=F4micos s=E3o n=FAmeros = construidos a partir de um=20 algoritmo e um n=FAmero inicial (semente).
Exemplo: Considere o algoritmo A abaixo em que o = n=BA=20 inicial entra como x(0).
 
1-   y=3D2x(n)
2-   z =E9 o resto da divis=E3o de y = por 3 (yz>0,=20 ou yz=3D0)
3-   w=3Dxyz-2
4-   a =E9 o quadrado = do resto da=20 divis=E3o de w por 7
5-   b=3Dw-a+x(n)
6-   b =E9 o (n+1)-=E9simo termo = da sequ=EAncia=20 A.
7-   b-1=3Dx(n+1)
 
A sequ=EAncia A para semente 1 =E9 A:=20 1,-1,-43,-7477,...
para semente 0 A: 0,-6,-12,-18,-24
(A apar=EAncia aleat=F3ria da sequ=EAncia = depende da=20 semente)
 
Aqui vai uma boa pergunta sobre esses n=FAmeros, = =E9 poss=EDvel=20 provar que uma sequ=EAncia aleat=F3ria qualquer segue um algoritmo em = que o valor=20 seguinte depende dos anteriores?
 
Andr=E9 T.
 
 
----- Original Message -----
From:=20 e isso = mesmo
Sent: Tuesday, September 10, = 2002 2:57=20 PM
Subject: [obm-l] Continuo=20 Perguntando...

 
 
 
Amigos, algu=E9m poderia me explicar detalhadamente o que s=E3o = n=FAmeros=20 rand=F4micos e como se constr=F3i esses n=FAmeros? Li isso em algum = lugar sobre=20 loterias.
Obrigado
V=EDctor



Aproveite melhor a Web. Fa=E7a o download GR=C1TIS do MSN Explorer : = http://explorer.msn.com.b= r/intl.asp#po

------=_NextPart_000_0008_01C25900.F720F700-- ========================================================================= Instruções para entrar na lista, sair da lista e usar a lista em http://www.mat.puc-rio.br/~nicolau/olimp/obm-l.html O administrador desta lista é ========================================================================= From owner-obm-l@sucuri.mat.puc-rio.br Tue Sep 10 19:37:21 2002 Return-Path: Received: (from majordom@localhost) by sucuri.mat.puc-rio.br (8.9.3/8.9.3) id TAA00729 for obm-l-MTTP; Tue, 10 Sep 2002 19:37:12 -0300 Received: from seki.bol.com.br (seki.bol.com.br [200.221.24.26]) by sucuri.mat.puc-rio.br (8.9.3/8.9.3) with ESMTP id TAA00725 for ; Tue, 10 Sep 2002 19:37:09 -0300 Received: from lba (200.221.24.192) by seki.bol.com.br (5.1.071) id 3D6143DE00625428 for obm-l@mat.puc-rio.br; Tue, 10 Sep 2002 19:37:52 -0300 Message-ID: <003901c2591b$4ba6b200$9506c3c8@lba> From: "Leonardo Borges Avelino" To: Subject: [obm-l] =?iso-8859-1?Q?Quest=E3o_da_OBM-2001?= Date: Tue, 10 Sep 2002 19:42:06 -0300 MIME-Version: 1.0 Content-Type: multipart/alternative; boundary="----=_NextPart_000_0036_01C25902.24D76DE0" X-Priority: 3 X-MSMail-Priority: Normal X-Mailer: Microsoft Outlook Express 5.00.2615.200 X-MimeOLE: Produced By Microsoft MimeOLE V5.00.2615.200 X-Sender-IP: 200.195.6.149 Sender: owner-obm-l@sucuri.mat.puc-rio.br Precedence: bulk Reply-To: obm-l@mat.puc-rio.br This is a multi-part message in MIME format. ------=_NextPart_000_0036_01C25902.24D76DE0 Content-Type: text/plain; charset="iso-8859-1" Content-Transfer-Encoding: quoted-printable Prove que (a+b)(a+c)>=3D2sqrt[abc(a+b+c)], para a,b e c inteiros positivos Vejam se este in=EDcio da minha solu=E7=E3o tem futuro, e se tiver = complementem pra mim a+b>=3D2sqrt[ab] a+c>=3D2sqrt[ac] (a+c)(a+b)>=3D4sqrt[a^2*b*c] ou seja (a+c)(a+b)>=3D2sqrt[abc]*(2sqrt[a]) pergunta: Tem como continuar essa express=E3o para provarmos a primeira = de um modo f=E1cil? ------=_NextPart_000_0036_01C25902.24D76DE0 Content-Type: text/html; charset="iso-8859-1" Content-Transfer-Encoding: quoted-printable
Prove que
(a+b)(a+c)>=3D2sqrt[abc(a+b+c)], = para a,b e c=20 inteiros positivos
 
Vejam se este in=EDcio da minha = solu=E7=E3o tem futuro, e=20 se tiver complementem pra mim
a+b>=3D2sqrt[ab]
a+c>=3D2sqrt[ac]
(a+c)(a+b)>=3D4sqrt[a^2*b*c]
ou seja
(a+c)(a+b)>=3D2sqrt[abc]*(2sqrt[a])
 
pergunta: Tem como continuar essa express=E3o para provarmos a = primeira de um=20 modo f=E1cil?
------=_NextPart_000_0036_01C25902.24D76DE0-- ========================================================================= Instruções para entrar na lista, sair da lista e usar a lista em http://www.mat.puc-rio.br/~nicolau/olimp/obm-l.html O administrador desta lista é ========================================================================= From owner-obm-l@sucuri.mat.puc-rio.br Tue Sep 10 21:17:04 2002 Return-Path: Received: (from majordom@localhost) by sucuri.mat.puc-rio.br (8.9.3/8.9.3) id VAA02702 for obm-l-MTTP; Tue, 10 Sep 2002 21:16:35 -0300 Received: from hotmail.com (f121.law9.hotmail.com [64.4.9.121]) by sucuri.mat.puc-rio.br (8.9.3/8.9.3) with ESMTP id VAA02698 for ; Tue, 10 Sep 2002 21:16:32 -0300 Received: from mail pickup service by hotmail.com with Microsoft SMTPSVC; Tue, 10 Sep 2002 17:17:32 -0700 Received: from 200.190.90.208 by lw9fd.law9.hotmail.msn.com with HTTP; Wed, 11 Sep 2002 00:17:31 GMT X-Originating-IP: [200.190.90.208] From: "Rogerio Fajardo" To: obm-l@mat.puc-rio.br Subject: Re: [obm-l] violencia e axioma da escolha Date: Wed, 11 Sep 2002 00:17:31 +0000 Mime-Version: 1.0 Content-Type: text/plain; format=flowed Message-ID: X-OriginalArrivalTime: 11 Sep 2002 00:17:32.0396 (UTC) FILETIME=[9F2E22C0:01C25928] Sender: owner-obm-l@sucuri.mat.puc-rio.br Precedence: bulk Reply-To: obm-l@mat.puc-rio.br O axioma da escolha pode ser enunciado com ou sem a hipótese dos conjuntos serem disjuntos. As duas formas são equivalentes, pois, se não forem disjuntos, voce pode disjuntá-los (usando produto cartesiano). Para usar a função escolha que voce falou, precisa que o conjunto de bandidos seja enumerável. De fato, basta que esse conjunto possa ser bem ordenado (todo subconjunto tem menor elemento) para voce usar essa função escolha. Mas o axioma da boa ordem, que diz que todo conjunto pode ser bem ordenado, é equivalente ao axioma da escolha (uma das implicações é exatamente o que voce fez). >From: Vinicius José Fortuna >Reply-To: obm-l@mat.puc-rio.br >To: >Subject: Re: [obm-l] violencia e axioma da escolha >Date: Mon, 9 Sep 2002 17:16:26 -0300 > >Não sei se entendi direito, mas, ao meu ver, não teríamos conjuntos dois a >dois disjuntos e tal propriedade é necessária para aplicar o axioma da >escolha (ou não?). > >De qualquer forma, não poderíamos mapear os bandidos nos números inteiros? >Assim teríamos uma função de escolha que pegaria em C o bandido mapeado no >menor inteiro tal que satisfaça aquelas condições para entrar em R. Se >temos >uma função de escolha então podemos escolhê-los independentemente do axioma >da escolha. > >Agradeço esclarecimentos > >Vinicius Fortuna > >----- Original Message ----- >From: "Rogerio Fajardo" >To: >Sent: Monday, September 09, 2002 12:08 PM >Subject: Re: [obm-l] violencia > > > > Olá, Vinicius > > > > Cada vez que voce "retira um elemento de C" e coloca em R, na verdade >voce > > mudou o conjunto C. Ou seja, cada escolha que voce fez, no processo > > indutivo, foi sobre um conjunto diferente. É semelhante a demonstração >de > > que todo conjunto infinto possui um subconjunto enumerável, em que, dado >um > > conjunto V, construímos indutivamente um conjunto S colocando nele, a >cada > > passo, um elemento de V que não está em S, usando o Axioma da Escolha > > > > > > >From: Vinicius José Fortuna > > >Reply-To: obm-l@mat.puc-rio.br > > >To: > > >Subject: Re: [obm-l] violencia > > >Date: Sun, 8 Sep 2002 18:41:45 -0300 > > > > > >Oi Rogério > > >Acho que não saquei. Em que momento foi utilizado o axioma da escolha? >Eu > > >nem tinha infinitos conjuntos! Apenas conjuntos infinitos. > > > > > >Até mais > > > > > >Vinicius > > > > > >----- Original Message ----- > > >From: "Rogerio Fajardo" > > >To: > > >Sent: Sunday, September 08, 2002 2:17 PM > > >Subject: Re: [obm-l] violencia > > > > > > > > > > É bom notar que essa solução usa o axioma da escolha (de infinitos > > >conjuntos > > > > não-vazios, escolhemos um elemento de cada). É essencial o axioma da > > >escolha > > > > para resolvê-lo? > > > > > > > > > > > > >From: Vinicius José Fortuna > > > > >Reply-To: obm-l@mat.puc-rio.br > > > > >To: > > > > >Subject: Re: [obm-l] violencia Date: Sat, 7 Sep 2002 23:44:58 -0300 > > > > > > > > > >----- Original Message ----- > > > > >From: "Fernanda Medeiros" > > > > >To: > > > > >Sent: Saturday, September 07, 2002 8:45 PM > > > > >Subject: [obm-l] violencia > > > > > > > > > > > > > > > > Olá, > > > > > > alguém pode dar uma ajuda nestas questões? > > > > > > 1.a)uma "gang" tem infinitos bandidos e cada um dos meliantes >tem >um > > > > >único > > > > > > inimigo no interior da "gang",que ele quer matar.Prove q é >possivel > > > > >reunir > > > > > > uma quantidade infinita de bandidos desta "gang", semq haja o > > >risco > > >de > > > > >q > > > > > > um bandido mate outro durante a reunião. > > > > > > > > > >Pense no seguinte algoritmo: > > > > >Temos o conjunto C de candidatos à reunião que inicialmente contém > > >todos > > >os > > > > >infinitos bandidos da gangue. > > > > >Temos o conjunto R de bandidos selecionados para a reunião que > > >inicialmente > > > > >está vazio. > > > > > > > > > >A cada passo do algoritmo procuramos em C alguém que não que matar > > >ninguém > > > > >de R e ninguém em R quer matá-lo. > > > > >Seja M o subconjunto de C de bandidos que pelo menos um de R quer > > >matar. > > > > >Como cada bandido de R só quer matar um, |M|<=|R| > > > > >Então, como R é finito, M será finito e V=C-M será infinito, pois C >é > > > > >infinito. > > > > >V será o subconjunto de C dos bandidos que ninguém de R quer matar. > > > > >Em V procuramos um bandido que não quer matar ninguém de R, >retiramos > > >ele > > > > >de > > > > >C, o inserimos em R e repete-se o processo. > > > > > > > > > >Se sempre for possível encontrar tal bandido, o processo se >repetirá > > > > >indefinidamente e com R sempre crescendo. Assim teremos infnitos > > >bandidos > > > > >na > > > > >reunião sem derramamento de sangue. > > > > > > > > > >Se em algum momento não for possível encontrar um bandido em V, é > > >porque > > > > >todos os bandidos de V querem matar alguém de R. Ou seja, ninguém >de >V > > >quer > > > > >matar outro de V. Pegamos, então, V como o conjunto de bandidos >para >a > > > > >reunião. Como V é infinito, teremos infinitos participantes na >reunião. > > > > > > > > > > > b)Se cada bandido tiver um nº finito mas indefinido de >inimigos(um > > > > >bandido > > > > > > pode ter 2 inimigos, outro somente 1, um terceiro pode ter 20 e > > >assim > > > > >por > > > > > > diante).Será sempre possivel promover uma reunião com infinitos > > >bandidos > > > > >sem > > > > > > risco de derramamento de sangue? > > > > >Não é possível. Existe um contra-exemplo: > > > > >Ordene os bandidos formando uma sequência. Imagine que cada bandido > > >quer > > > > >matar todos que vêm antes dele na sequência. Nunca poderemos ter >dois > > > > >bandidos 'a' e 'b' na reunião, pois ou a vem antes de b, ou b vem >antes > > >de, > > > > >assim haverá um que vai querer matar o outro. Então só poderemos >ter >um > > > > >bandido na reunião. > > > > > > > > > >Até mais > > > > > > > > > >Vinicius Fortuna > > > > >IC-Unicamp > > >========================================================================= >Instruções para entrar na lista, sair da lista e usar a lista em >http://www.mat.puc-rio.br/~nicolau/olimp/obm-l.html >O administrador desta lista é >========================================================================= _________________________________________________________________ Chat with friends online, try MSN Messenger: http://messenger.msn.com ========================================================================= Instruções para entrar na lista, sair da lista e usar a lista em http://www.mat.puc-rio.br/~nicolau/olimp/obm-l.html O administrador desta lista é ========================================================================= From owner-obm-l@sucuri.mat.puc-rio.br Tue Sep 10 21:31:45 2002 Return-Path: Received: (from majordom@localhost) by sucuri.mat.puc-rio.br (8.9.3/8.9.3) id VAA03049 for obm-l-MTTP; Tue, 10 Sep 2002 21:31:36 -0300 Received: from hotmail.com (f153.law9.hotmail.com [64.4.9.153]) by sucuri.mat.puc-rio.br (8.9.3/8.9.3) with ESMTP id VAA03045 for ; Tue, 10 Sep 2002 21:31:33 -0300 Received: from mail pickup service by hotmail.com with Microsoft SMTPSVC; Tue, 10 Sep 2002 17:32:34 -0700 Received: from 200.190.90.208 by lw9fd.law9.hotmail.msn.com with HTTP; Wed, 11 Sep 2002 00:32:33 GMT X-Originating-IP: [200.190.90.208] From: "Rogerio Fajardo" To: obm-l@mat.puc-rio.br Subject: Re: [obm-l] Um Estranho Sentimento ... Date: Wed, 11 Sep 2002 00:32:33 +0000 Mime-Version: 1.0 Content-Type: text/plain; format=flowed Message-ID: X-OriginalArrivalTime: 11 Sep 2002 00:32:34.0353 (UTC) FILETIME=[B8C9DA10:01C2592A] Sender: owner-obm-l@sucuri.mat.puc-rio.br Precedence: bulk Reply-To: obm-l@mat.puc-rio.br Não compreendi bem o que voce quer dizer, mas me interessei por seu comentário. Pelo que entendi, voce quer saber se existe outra forma de visualizar, intuitivamente, os números, de forma a enxergar propriedades que são difíceis de enxergar com a visualização com as quais estamos acostumados. É isso ou nada a ver? O que eu percebo é que não é bem a geometria que serve para nos dar uma intuição dos números reais, mas os números reais surgiram para descrever a geometria de forma precisa. Não sei se isso tem algo a ver com seu e-mail. Detalhe-me mais o seu pensamento. >From: "Paulo Santa Rita" >Reply-To: obm-l@mat.puc-rio.br >To: obm-l@mat.puc-rio.br >Subject: [obm-l] Um Estranho Sentimento ... >Date: Mon, 09 Sep 2002 21:19:23 +0000 > >Ola Pessoal e demais >colegas desta lista ... OBM-L, > >Eu tenho pensado continua e longamente em um conjunto de questoes >relacionadas que me levam, invariavelmente, a uma mesma direcao que nao >estou conseguindo acreditar ... ate parece que depois de tanto refletir >cheguei a alguma constatacao insana ou simploria demais ... Se algum colega >puder falar algo esclarecedor e/ou interessante eu ficarei muito grato ! > >Desde a infancia somos instados a pensar que os numeros reais estao >dispostos ao longo de uma linha reta. Nos dizemos : 3 < 5 ! E imediatamente >visualizamos o 3 a esquerda do 5, ambos em uma linha reta ! Por que nos >pensamos assim ? > >E verdade que em cursos de analise os livros definem R como um corpo >ordenado completo e derivam as propriedades dos numeros reais dos axiomas >que definem esta estrutura, sem recorrer a qualquer propriedade geometrica >derivada de uma eventual visualizacao dos numeros sobre uma reta ... > >Mas se, por um lado, os axiomas de um corpo ordenado completo nao implicam >ou requerem explicitamente uma estrutura geometrica conhecida, e inegavel >que a visualizacao "informal" que fazemos facilita muitos raciocinio ... >SERIA REALMENTE IMPOSSIVEL ASSOCIAR A UM CORPO ORDENADO COMPLETO UMA >GEOMETRIA, ATRAVES DE AXIOMAS OU OUTROS RECURSOS, DE FORMA QUE PUDESSEMOS >TER OUTRAS VISUALIZACOES, MESMO QUE ESTRANHAS, POREM, CONMSISTENTES ? NAO >PODERIAM ALGUMAS PROPRIEDADES NUMERICAS DEPENDEREM INEXORAVELMENTE DE UMA >TAL GEOMETRIA ? > >E bem provavel que eu esteja errado, mas nao consigo perceber o meu erro >... EU ACHO QUE CERTOS MAPEAMENTOS NUMERICOS REQUEREM OU IMPLICAM QUE OS >NUMEROS NATURAIS TEM UMA GEOMETRIA OU ESTRUTURA INTRINSECA, SEM A QUAL NAO >DA PRA COMPREENDER CERTAS COISAS ... E NECESSARIO OU POSTULAR UMA >DISPOSICAO ESTRATIGRAFICA OU SUPOR QUE CERTOS MAPEAMENTOS INDUZEM UMA TAL >ESTRATIFICACAO ... > >Bom, se alguem puder falar alguma coisa interessante sobre este tema eu >fico muito grato, pois este e realmente um SENTIMENTO ESTRANHO que me tem >ocorrido com alguma frequencia. Pode ser uma burrice momentanea que esta me >levando a estas perguntas aparentemente idiotas e sem sentido, mas eu nao >iria ocupar o tempo de voces, meus amigos, se nao tivesse razoes seria pra >fazer isso ... > >Um abraco a Todos >Paulo Santa Rita >2,1818,090902 > > >_________________________________________________________________ >Converse com seus amigos online, faça o download grátis do MSN Messenger: >http://messenger.msn.com.br > >========================================================================= >Instruções para entrar na lista, sair da lista e usar a lista em >http://www.mat.puc-rio.br/~nicolau/olimp/obm-l.html >O administrador desta lista é >========================================================================= _________________________________________________________________ Send and receive Hotmail on your mobile device: http://mobile.msn.com ========================================================================= Instruções para entrar na lista, sair da lista e usar a lista em http://www.mat.puc-rio.br/~nicolau/olimp/obm-l.html O administrador desta lista é ========================================================================= From owner-obm-l@sucuri.mat.puc-rio.br Tue Sep 10 22:13:49 2002 Return-Path: Received: (from majordom@localhost) by sucuri.mat.puc-rio.br (8.9.3/8.9.3) id WAA04225 for obm-l-MTTP; Tue, 10 Sep 2002 22:13:35 -0300 Received: from hotmail.com (f30.pav2.hotmail.com [64.4.37.30]) by sucuri.mat.puc-rio.br (8.9.3/8.9.3) with ESMTP id WAA04220 for ; Tue, 10 Sep 2002 22:13:32 -0300 Received: from mail pickup service by hotmail.com with Microsoft SMTPSVC; Tue, 10 Sep 2002 18:14:33 -0700 Received: from 200.193.250.13 by pv2fd.pav2.hotmail.msn.com with HTTP; Wed, 11 Sep 2002 01:14:32 GMT X-Originating-IP: [200.193.250.13] From: "ricardo matos" To: obm-l@mat.puc-rio.br Subject: [obm-l] =?iso-8859-1?B?UmU6IFtvYm0tbF0gUmU6IFtvYm0tbF0gTyBjYXLhdGVyIG7jbyBlbnVt?= =?iso-8859-1?B?ZXLhdmVsIGRlIFI=?= Date: Tue, 10 Sep 2002 22:14:32 -0300 Mime-Version: 1.0 Content-Type: text/plain; charset=iso-8859-1; format=flowed Message-ID: X-OriginalArrivalTime: 11 Sep 2002 01:14:33.0229 (UTC) FILETIME=[9627D3D0:01C25930] Sender: owner-obm-l@sucuri.mat.puc-rio.br Precedence: bulk Reply-To: obm-l@mat.puc-rio.br Se eu tô entendendo direito o que se tá perguntando, o Nicolau Saldanha comentou do seguinte conjunto N^(infinito) que não é(esse é o problema) o conjunto que você tava comentando N*N*N*N*... produto cartesiano enumeráveis vezes dos naturais. O conjunto que o Saldanha comentou é como ele esplicou o limite direto por inclusões de N < N^2 < N^3 < ... o que baixando a bola é como a "união" de todos eles, um jeito mais claro de definir isto é pegando a união de todos os conjuntos (N,0,0,0,0,... ) equivalente à N (N,N,0,0,0,0... ) equivalente à N*N (N,N,N,0,0,0,0... ) equivalente à N*N*N (. . ) (. . ) (. . ) que é enumerável pelo seu raciocínio(união enumerável de conjuntos enumeráveis é enumerável pelo o que é chamado argumento diagonal do Cantor). MAS, repare que (1,1,1,1,1,1,1,...) não está em nenhum desses conjuntos logo não está na união, da mesma forma todos aqueles elementos que não terminam em infinitos zeros, ou seja os únicos elementos que pertencem ao nosso conjunto são aqueles que tem todas as coordenadas zero a menos de finitas coordenadas ou "quase-nulos". Isto lucraria quase imediatamente que todos os números decimais são enumeráveis(o que já era claro), mas muito mais interessante dele resulta o fato de que os polinômios com coeficientes naturais ou inteiros são enumeráveis(pois os polinômios são justamente sequências "quase-nulas" de inteiros), o que implica que os números algébricos são enumeráveis(o que é bem mais massa) pela definição números algébricos são aqueles que são raízes de polinômios de coeficientes inteiros mas acaba que os algébricos é o conjunto formado pelos números inteiros com finitas operações de soma, multiplicação, divisão e radiciação que tipo 2/(sqrt(sqrt(5) + (2)^(1/3))+5^(1/5), o que de certa forma é natural(alguém sabe uma demonstração simples??). Enquanto o conjunto N*N*N*N*... tem uma bijeção com os números da forma a+1/(b+1/(c...)...) {frações contínuas} a,b,c,... que acabam por um argumento simples serem todos os reais positivos. Se falarmos que A^N=A*A*A*A*...(produto catesiano enumeráveis vezes) teremos que N^(N)=reais=2^N (pela notação binária)=10^N(pela notação decimal). Acho que o comentário a seguir é meio técnico(formal) mas coloquei ele pra dizer pelo menos o que eu entendo como "limite direto" a que o Nicolau Saldanha se referiu e para quem quiser saber taí(se conhecerem outra definição agradeceria). A definição que conheço é por propriedade universal que para conjuntos fica: Um conjunto A é limite direto de uma coleção de conjuntos A_i junto com as funções injetoras f_i_j (como se um dentro do outro) se existem funções injetivas g_i de cada um deles em A tal que (g_j(f_i_j))=g_i qualquer i,j em I(ou compatível). Bem abstrato não!!? Quando essas funções injetoras são inclusões(função dele para ele mesmo com imagem maior) o limite direto pode ser definido como a união desses conjuntos um exemplo: os intervalos [1,2] < [1/2,2] < [1/3,2] < [1/4,2]<...< [1/n,2],... tem como limite direto (0,2] Não definimos como união deles imediatamente por que a princípio eles podem ser separados ( não um dentro do outro) como era no nosso problema original N,N*N,N*N*N,... logo "mergulhamos" todos eles num conjunto grande que possua eles naturalmente e temos as injeções naturais desejadas(N*N*N* vai em N*N com (a,b) indo em (a,b,0) como inclusões e tomamos a união deles depois de "mergulhados". >From: "Laurito Alves" >Reply-To: obm-l@mat.puc-rio.br >To: obm-l@mat.puc-rio.br >Subject: [obm-l] Re: [obm-l] O caráter não enumerável de R >Date: Tue, 10 Sep 2002 16:51:53 +0000 > >Colegas, > >Ou estou viajando muito ou é certo que a união enumerável de conjuntos >enumeráveis é enumerável ? > >Se me lembro bem, provamos este fato com um argumento semelhante ao que >Cantor usou para provar a enumerabilidade de Q. > >Onde está meu erro ? > >Laurito > > >>From: "Nicolau C. Saldanha" >>Reply-To: obm-l@mat.puc-rio.br >>To: obm-l@mat.puc-rio.br >>Subject: [obm-l] Re: [obm-l] Re: [obm-l] O caráter não enumerável de R >>Date: Tue, 10 Sep 2002 08:48:38 -0300 >> >>On Mon, Sep 09, 2002 at 10:31:07PM +0000, Laurito Alves wrote: >> > O que você chama de N*N*N************ ??? >> > >> > Se for um produto cartesiano de N uma quantidade enumerável de vezes, >>ele é >> > enumerável. >> >>O limite direto das inclusões (usando o sinal < no lugar de está contido) >> >>N < N^2 < N^3 < ... >> >>é o conjunto das seqüências de naturais que são zero a partir de certo >>ponto; este conjunto é enumerável. Este conjunto às vezes é chamado >>de N^(N) ou N^infty. Mas há outra interpretação: o conjunto N^N >>de todas as seqüências de naturais (com qualquer comportamento no >>infinito) >>é não enumerável, tem o cardinal de R. Na verdade, via frações contínuas, >>é fácil construir uma bijeção natural entre este conjunto e o dos números >>irracionais. >> >>[]s, N. >>========================================================================= >>Instruções para entrar na lista, sair da lista e usar a lista em >>http://www.mat.puc-rio.br/~nicolau/olimp/obm-l.html >>O administrador desta lista é >>========================================================================= > > > > >_________________________________________________________________ >MSN Photos é a maneira mais fácil e prática de editar e compartilhar sua >fotos: http://photos.msn.com.br > >========================================================================= >Instruções para entrar na lista, sair da lista e usar a lista em >http://www.mat.puc-rio.br/~nicolau/olimp/obm-l.html >O administrador desta lista é >========================================================================= _________________________________________________________________ Tenha você também um MSN Hotmail, o maior webmail do mundo: http://www.hotmail.com/br ========================================================================= Instruções para entrar na lista, sair da lista e usar a lista em http://www.mat.puc-rio.br/~nicolau/olimp/obm-l.html O administrador desta lista é ========================================================================= From owner-obm-l@sucuri.mat.puc-rio.br Wed Sep 11 00:14:29 2002 Return-Path: Received: (from majordom@localhost) by sucuri.mat.puc-rio.br (8.9.3/8.9.3) id AAA05652 for obm-l-MTTP; Wed, 11 Sep 2002 00:14:13 -0300 Received: from shen.bol.com.br (shen.bol.com.br [200.221.24.14]) by sucuri.mat.puc-rio.br (8.9.3/8.9.3) with ESMTP id AAA05648 for ; Wed, 11 Sep 2002 00:14:11 -0300 Received: from bol.com.br (200.221.24.130) by shen.bol.com.br (5.1.071) id 3D63D22F006AD04B for obm-l@mat.puc-rio.br; Wed, 11 Sep 2002 00:14:30 -0300 Date: Wed, 11 Sep 2002 00:13:09 -0300 Message-Id: Subject: Re:[obm-l] circuito IME MIME-Version: 1.0 Content-Type: text/plain;charset="iso-8859-1" From: "adr.scr.m" To: obm-l@mat.puc-rio.br X-XaM3-API-Version: 2.4.3.4.4 X-SenderIP: 200.151.53.51 Content-Transfer-Encoding: 8bit X-MIME-Autoconverted: from quoted-printable to 8bit by sucuri.mat.puc-rio.br id AAA05649 Sender: owner-obm-l@sucuri.mat.puc-rio.br Precedence: bulk Reply-To: obm-l@mat.puc-rio.br claro que nao meu caro Rafael,nao eh so porque nao esta sendo ensinada no 2ºgrau que deveria ser anulada,era porque nao estava no programa. Houveram muitas reclama. por causa disso,mas a grande maioria das pessoas que acertaram eh porque fizeram 2º grau tecnico. []'s. Adriano. __________________________________________________________________________ AcessoBOL, só R$ 9,90! O menor preço do mercado! Assine já! http://www.bol.com.br/acessobol ========================================================================= Instruções para entrar na lista, sair da lista e usar a lista em http://www.mat.puc-rio.br/~nicolau/olimp/obm-l.html O administrador desta lista é ========================================================================= From owner-obm-l@sucuri.mat.puc-rio.br Wed Sep 11 00:18:33 2002 Return-Path: Received: (from majordom@localhost) by sucuri.mat.puc-rio.br (8.9.3/8.9.3) id AAA05734 for obm-l-MTTP; Wed, 11 Sep 2002 00:18:26 -0300 Received: from smtp-6.ig.com.br (smtp-6.ig.com.br [200.226.132.155]) by sucuri.mat.puc-rio.br (8.9.3/8.9.3) with SMTP id AAA05727 for ; Wed, 11 Sep 2002 00:18:23 -0300 Received: (qmail 5921 invoked from network); 11 Sep 2002 03:19:04 -0000 Received: from unknown (HELO nome) (200.190.242.12) by smtp-6.ig.com.br with SMTP; 11 Sep 2002 03:19:04 -0000 Message-ID: <015e01c25942$006926c0$0cf2bec8@nome> From: "Silvio" To: Subject: [obm-l] Ajuda Algebra linear (Off Topic) Date: Wed, 11 Sep 2002 00:19:04 -0300 MIME-Version: 1.0 Content-Type: text/plain; charset="iso-8859-1" Content-Transfer-Encoding: 8bit X-Priority: 3 X-MSMail-Priority: Normal X-Mailer: Microsoft Outlook Express 5.50.4133.2400 X-MIMEOLE: Produced By Microsoft MimeOLE V5.50.4133.2400 Sender: owner-obm-l@sucuri.mat.puc-rio.br Precedence: bulk Reply-To: obm-l@mat.puc-rio.br Caros amigos, Preciso apresentar um trabalho sobre algebra linear aplicada, gostaria de sugestões de sites e/ou livros e até mesmo quais das aplicações de alg. linear seria interessante de apresentar,"E ,claro, seja fácil de encontrar documentação". Agradeço desde já. Silvio srtb@bol.com.br ========================================================================= Instruções para entrar na lista, sair da lista e usar a lista em http://www.mat.puc-rio.br/~nicolau/olimp/obm-l.html O administrador desta lista é ========================================================================= From owner-obm-l@sucuri.mat.puc-rio.br Wed Sep 11 00:26:11 2002 Return-Path: Received: (from majordom@localhost) by sucuri.mat.puc-rio.br (8.9.3/8.9.3) id AAA06029 for obm-l-MTTP; Wed, 11 Sep 2002 00:26:01 -0300 Received: from shen.bol.com.br (shen.bol.com.br [200.221.24.14]) by sucuri.mat.puc-rio.br (8.9.3/8.9.3) with ESMTP id AAA06021 for ; Wed, 11 Sep 2002 00:25:58 -0300 Received: from bol.com.br (200.221.24.130) by shen.bol.com.br (5.1.071) id 3D63D22F006ADBF8 for obm-l@mat.puc-rio.br; Wed, 11 Sep 2002 00:26:17 -0300 Date: Wed, 11 Sep 2002 00:24:56 -0300 Message-Id: Subject: [obm-l] IME-geometria MIME-Version: 1.0 Content-Type: text/plain;charset="iso-8859-1" From: "adr.scr.m" To: obm-l@mat.puc-rio.br X-XaM3-API-Version: 2.4.3.4.4 X-SenderIP: 200.151.53.51 Content-Transfer-Encoding: 8bit X-MIME-Autoconverted: from quoted-printable to 8bit by sucuri.mat.puc-rio.br id AAA06022 Sender: owner-obm-l@sucuri.mat.puc-rio.br Precedence: bulk Reply-To: obm-l@mat.puc-rio.br alguem poderia fazer essa questao do IME, por favor: Uma piramide de vertice V e base ABCD constitui a metade de um octaedro regular de aresta a.Marcam-se sobre VA e VB segmentos VA'=VB'=x,marcam-se sobre VC e VD os segmentos VC'=VD'=y.Supoe-se que x e y variam sob a condicao x+y=a.Determine x e y em funcao de a,de forma que a area de quadrilatero A'B'C'D' seja igual a a(elevado ao quadrado) /4. obrigado. []'s. Adriano __________________________________________________________________________ AcessoBOL, só R$ 9,90! O menor preço do mercado! Assine já! http://www.bol.com.br/acessobol ========================================================================= Instruções para entrar na lista, sair da lista e usar a lista em http://www.mat.puc-rio.br/~nicolau/olimp/obm-l.html O administrador desta lista é ========================================================================= From owner-obm-l@sucuri.mat.puc-rio.br Wed Sep 11 09:35:28 2002 Return-Path: Received: (from majordom@localhost) by sucuri.mat.puc-rio.br (8.9.3/8.9.3) id JAA11406 for obm-l-MTTP; Wed, 11 Sep 2002 09:34:31 -0300 Received: from mediterraneo.rjnet.com.br (mediterraneo.rjnet.com.br [200.222.31.30]) by sucuri.mat.puc-rio.br (8.9.3/8.9.3) with ESMTP id JAA11401 for ; Wed, 11 Sep 2002 09:34:27 -0300 Received: from locutus.rjnet.com.br (root@locutus.rjnet.com.br [200.222.31.10]) by mediterraneo.rjnet.com.br (8.11.4/8.11.4) with ESMTP id g8BCa6C15356 for ; Wed, 11 Sep 2002 09:36:06 -0300 Received: from felipe ([200.222.39.86]) by locutus.rjnet.com.br (8.11.2/8.11.2) with SMTP id g8BCG7212319 for ; Wed, 11 Sep 2002 09:16:07 -0300 Message-ID: <001301c2598f$b4d3f3e0$158c000a@felipe> From: "Felipe Villela Dias" To: References: <015e01c25942$006926c0$0cf2bec8@nome> Subject: Re: [obm-l] Ajuda Algebra linear (Off Topic) Date: Wed, 11 Sep 2002 09:35:26 -0300 MIME-Version: 1.0 Content-Type: text/plain; charset="iso-8859-1" Content-Transfer-Encoding: 8bit X-Priority: 3 X-MSMail-Priority: Normal X-Mailer: Microsoft Outlook Express 6.00.2600.0000 X-MIMEOLE: Produced By Microsoft MimeOLE V6.00.2600.0000 Sender: owner-obm-l@sucuri.mat.puc-rio.br Precedence: bulk Reply-To: obm-l@mat.puc-rio.br Caro Silvio, Não sei o nível de aprofundamento do seu trabalho, mas existem diversos temas que se aplicam Álgebra Linear. Quando eu cursei Álgebra Linear II eu apresentei um trabalho sobre fractais. Fractais podem ser gerados através de sucessivas transformações lineares em figuras ou até mesmo pontos. ----- Original Message ----- From: "Silvio" To: Sent: Wednesday, September 11, 2002 12:19 AM Subject: [obm-l] Ajuda Algebra linear (Off Topic) > Caros amigos, > > Preciso apresentar um trabalho sobre algebra linear aplicada, gostaria > de sugestões de sites e/ou livros e até mesmo quais das aplicações de alg. > linear seria interessante de apresentar,"E ,claro, seja fácil de encontrar > documentação". > > Agradeço desde já. > > > Silvio > srtb@bol.com.br > > > ========================================================================= > Instruções para entrar na lista, sair da lista e usar a lista em > http://www.mat.puc-rio.br/~nicolau/olimp/obm-l.html > O administrador desta lista é > ========================================================================= > --- Outgoing mail is certified Virus Free. Checked by AVG anti-virus system (http://www.grisoft.com). Version: 6.0.385 / Virus Database: 217 - Release Date: 4/9/2002 ========================================================================= Instruções para entrar na lista, sair da lista e usar a lista em http://www.mat.puc-rio.br/~nicolau/olimp/obm-l.html O administrador desta lista é ========================================================================= From owner-obm-l@sucuri.mat.puc-rio.br Wed Sep 11 10:40:22 2002 Return-Path: Received: (from majordom@localhost) by sucuri.mat.puc-rio.br (8.9.3/8.9.3) id KAA12885 for obm-l-MTTP; Wed, 11 Sep 2002 10:39:57 -0300 Received: from hotmail.com (oe70.law10.hotmail.com [64.4.14.205]) by sucuri.mat.puc-rio.br (8.9.3/8.9.3) with ESMTP id KAA12881 for ; Wed, 11 Sep 2002 10:39:54 -0300 Received: from mail pickup service by hotmail.com with Microsoft SMTPSVC; Wed, 11 Sep 2002 06:40:56 -0700 X-Originating-IP: [200.211.146.56] From: "Rubens Vilhena" To: Subject: Re: [obm-l] Ajuda Algebra linear (Off Topic) Date: Wed, 11 Sep 2002 10:40:46 -0300 MIME-Version: 1.0 X-Mailer: MSN Explorer 7.00.0021.1900 Content-Type: multipart/alternative; boundary="----=_NextPart_001_0000_01C2597F.AFB134A0" Message-ID: X-OriginalArrivalTime: 11 Sep 2002 13:40:56.0266 (UTC) FILETIME=[DAECFAA0:01C25998] Sender: owner-obm-l@sucuri.mat.puc-rio.br Precedence: bulk Reply-To: obm-l@mat.puc-rio.br ------=_NextPart_001_0000_01C2597F.AFB134A0 Content-Type: text/plain; charset="iso-8859-1" Content-Transfer-Encoding: quoted-printable Entre no site http://www.math.bcit.ca/examples/index.shtml Acho que vai lhe ajudar muito. Rubens -----Mensagem Original----- De: Silvio Enviado: quarta-feira, 11 de setembro de 2002 00:45 Para: obm-l@mat.puc-rio.br Assunto: [obm-l] Ajuda Algebra linear (Off Topic) Caros amigos, Preciso apresentar um trabalho sobre algebra linear aplicada, gostari= a de sugest=F5es de sites e/ou livros e at=E9 mesmo quais das aplica=E7=F5e= s de alg. linear seria interessante de apresentar,"E ,claro, seja f=E1cil de encont= rar documenta=E7=E3o". Agrade=E7o desde j=E1. Silvio srtb@bol.com.br =3D=3D=3D=3D=3D=3D=3D=3D=3D=3D=3D=3D=3D=3D=3D=3D=3D=3D=3D=3D=3D=3D=3D=3D=3D= =3D=3D=3D=3D=3D=3D=3D=3D=3D=3D=3D=3D=3D=3D=3D=3D=3D=3D=3D=3D=3D=3D=3D=3D=3D= =3D=3D=3D=3D=3D=3D=3D=3D=3D=3D=3D=3D=3D=3D=3D=3D=3D=3D=3D=3D=3D=3D=3D Instru=E7=F5es para entrar na lista, sair da lista e usar a lista em http://www.mat.puc-rio.br/~nicolau/olimp/obm-l.html O administrador desta lista =E9 =3D=3D=3D=3D=3D=3D=3D=3D=3D=3D=3D=3D=3D=3D=3D=3D=3D=3D=3D=3D=3D=3D=3D=3D=3D= =3D=3D=3D=3D=3D=3D=3D=3D=3D=3D=3D=3D=3D=3D=3D=3D=3D=3D=3D=3D=3D=3D=3D=3D=3D= =3D=3D=3D=3D=3D=3D=3D=3D=3D=3D=3D=3D=3D=3D=3D=3D=3D=3D=3D=3D=3D=3D=3DApro= veite melhor a Web. Fa=E7a o download GR=C1TIS do MSN Explorer : http://e= xplorer.msn.com.br/intl.asp#po ------=_NextPart_001_0000_01C2597F.AFB134A0 Content-Type: text/html; charset="iso-8859-1" Content-Transfer-Encoding: quoted-printable
Acho que vai lhe ajudar muito= .
Rubens
 


Aproveite melhor= a Web. Fa=E7a o download GR=C1TIS do MSN Explorer : http://explorer.msn.com.br/intl.asp#po=

------=_NextPart_001_0000_01C2597F.AFB134A0-- ========================================================================= Instruções para entrar na lista, sair da lista e usar a lista em http://www.mat.puc-rio.br/~nicolau/olimp/obm-l.html O administrador desta lista é ========================================================================= From owner-obm-l@sucuri.mat.puc-rio.br Wed Sep 11 12:10:51 2002 Return-Path: Received: (from majordom@localhost) by sucuri.mat.puc-rio.br (8.9.3/8.9.3) id MAA14613 for obm-l-MTTP; Wed, 11 Sep 2002 12:10:27 -0300 Received: from mtasjc.directnet.com.br (mtasjc.directnet.com.br [200.152.0.15]) by sucuri.mat.puc-rio.br (8.9.3/8.9.3) with ESMTP id MAA14609 for ; Wed, 11 Sep 2002 12:10:24 -0300 Received: from meu ([200.152.17.12]) by pop.directnet.com.br (iPlanet Messaging Server 5.1 HotFix 1.3 (built Jul 16 2002)) with SMTP id <0H2A007BH4Q8A3@pop.directnet.com.br> for obm-l@mat.puc-rio.br; Wed, 11 Sep 2002 12:08:38 -0300 (EST) Date: Wed, 11 Sep 2002 12:27:03 -0300 From: Daniel Subject: Re: Re:[obm-l] circuito IME To: obm-l@mat.puc-rio.br Message-id: <003a01c259a7$ae6106c0$0c1198c8@directnet.com.br> MIME-version: 1.0 X-MIMEOLE: Produced By Microsoft MimeOLE V5.00.2615.200 X-Mailer: Microsoft Outlook Express 5.00.2615.200 Content-type: text/plain; charset=iso-8859-1 Content-transfer-encoding: 8BIT X-Priority: 3 X-MSMail-priority: Normal References: Sender: owner-obm-l@sucuri.mat.puc-rio.br Precedence: bulk Reply-To: obm-l@mat.puc-rio.br Rafael, o IME não divulga gabarito oficial e não deve explicações sobre suas provas, assim como o ITA, a questão do circuito só e somente pode ser resolvida empregando-se o teorema de thevenin (para simplificar) e as equações do regime rc transitório, não era preciso realizar nenhuma integral, apenas ter os resultados prontos para usá-los com os dados fornecidos. Se vc vai prestar IME é bom que tenha esses dados em mente, algumas questões não serão resolvidas com conhecimento do ensino médio. Daniel O. Costa ----- Original Message ----- From: rafaelc.l To: Sent: Monday, September 09, 2002 6:17 PM Subject: Re:[obm-l] circuito IME > > meu caro Adriano: vc não acha que deve ter alguma > solução para essa questão sem o uso dessa equações? > (pense bem nisso),analise o problema. Se não tivesse > alguma solução só com a abordagem do 2 grau, a questão > teria sido anulada. Poucos candidatos devem ter resolvido > ela intergralmente. > > > __________________________________________________________________________ > AcessoBOL, só R$ 9,90! O menor preço do mercado! > Assine já! http://www.bol.com.br/acessobol > ========================================================================= Instruções para entrar na lista, sair da lista e usar a lista em http://www.mat.puc-rio.br/~nicolau/olimp/obm-l.html O administrador desta lista é ========================================================================= From owner-obm-l@sucuri.mat.puc-rio.br Wed Sep 11 12:23:32 2002 Return-Path: Received: (from majordom@localhost) by sucuri.mat.puc-rio.br (8.9.3/8.9.3) id MAA14917 for obm-l-MTTP; Wed, 11 Sep 2002 12:23:23 -0300 Received: from hotmail.com (f90.sea2.hotmail.com [207.68.165.90]) by sucuri.mat.puc-rio.br (8.9.3/8.9.3) with ESMTP id MAA14913 for ; Wed, 11 Sep 2002 12:23:20 -0300 Received: from mail pickup service by hotmail.com with Microsoft SMTPSVC; Wed, 11 Sep 2002 08:24:22 -0700 Received: from 32.94.119.254 by sea2fd.sea2.hotmail.msn.com with HTTP; Wed, 11 Sep 2002 15:24:22 GMT X-Originating-IP: [32.94.119.254] From: "Paulo Santa Rita" To: obm-l@mat.puc-rio.br Subject: Re: [obm-l] Um Estranho Sentimento ... Date: Wed, 11 Sep 2002 15:24:22 +0000 Mime-Version: 1.0 Content-Type: text/plain; charset=iso-8859-1; format=flowed Message-ID: X-OriginalArrivalTime: 11 Sep 2002 15:24:22.0577 (UTC) FILETIME=[4E2CE210:01C259A7] Sender: owner-obm-l@sucuri.mat.puc-rio.br Precedence: bulk Reply-To: obm-l@mat.puc-rio.br Ola ROGERIO FAJARDO e demais colegas desta lista ... OBM-L, E entao Fajardo, tudo legal ? Conseguiu o Livro de Logica-Matematica ? Eu sei que foram fatos geometricos e outros fenomenos cotidianos (divida -> numero negativo, divisao de um objeto -> fracao, etc etc ) que nos levaram a descoberta das diversas classes de numeros, "construidos" posteriormente e hoje apresentados com o auxilio da abstracao matematica ... Colocar estes numeros em uma reta, porem, e uma construcao humana gratuita ... nao ha nenhum razao forte para tanto e os axiomas de um corpo ordenado completo nao induzem, a priori, a nenhuma topologia particular ... O que estes axiomas podem falar sobre disposicao ou configuracao ? Isso : Nada ! Nos poderiamos pensar sobre eles com igual correcao se os visualizassemos sobre um ramo de parabola, por exemplo. ME PARECE, salvo melhor juizo, que a unica exigencia que podemos fazer sobre uma possivel representacao e a de continuidade ... E a continuidade, conforme todos nos sabemos, nao e uma propriedade metrica. Bom, sendo assim, respeitados os axiomas de um corpo ordenado completo, nos podemos pensar nos numeros reais como estando disposto de outra forma, desde que esta estratificacao preserve a continuidade ... A questao e : e vantajoso fazer isso ? e util ? Com esta imagem nos conseguiremos resolver ou esclarecer algum fato que ainda nao foi resolvido ou esclarecido ? So assim um mudanca ou inovacao e justificavel ... Nao sei se consegui ser claro, mas percebi que voce pensou seriamente sobre a minha mensagem e nao supos que eu seja tao simplorio que nao perceba sobre a gravidade e implicacoes do que estou falando ... Um abraco Paulo Santa Rita 4,1223,110902 >From: "Rogerio Fajardo" >Reply-To: obm-l@mat.puc-rio.br >To: obm-l@mat.puc-rio.br >Subject: Re: [obm-l] Um Estranho Sentimento ... >Date: Wed, 11 Sep 2002 00:32:33 +0000 > >Não compreendi bem o que voce quer dizer, mas me interessei por seu >comentário. Pelo que entendi, voce quer saber se existe outra forma de >visualizar, intuitivamente, os números, de forma a enxergar propriedades >que são difíceis de enxergar com a visualização com as quais estamos >acostumados. É isso ou nada a ver? > >O que eu percebo é que não é bem a geometria que serve para nos dar uma >intuição dos números reais, mas os números reais surgiram para descrever a >geometria de forma precisa. Não sei se isso tem algo a ver com seu e-mail. >Detalhe-me mais o seu pensamento. > > >>From: "Paulo Santa Rita" >>Reply-To: obm-l@mat.puc-rio.br >>To: obm-l@mat.puc-rio.br >>Subject: [obm-l] Um Estranho Sentimento ... >>Date: Mon, 09 Sep 2002 21:19:23 +0000 >> >>Ola Pessoal e demais >>colegas desta lista ... OBM-L, >> >>Eu tenho pensado continua e longamente em um conjunto de questoes >>relacionadas que me levam, invariavelmente, a uma mesma direcao que nao >>estou conseguindo acreditar ... ate parece que depois de tanto refletir >>cheguei a alguma constatacao insana ou simploria demais ... Se algum >>colega puder falar algo esclarecedor e/ou interessante eu ficarei muito >>grato ! >> >>Desde a infancia somos instados a pensar que os numeros reais estao >>dispostos ao longo de uma linha reta. Nos dizemos : 3 < 5 ! E >>imediatamente visualizamos o 3 a esquerda do 5, ambos em uma linha reta ! >>Por que nos pensamos assim ? >> >>E verdade que em cursos de analise os livros definem R como um corpo >>ordenado completo e derivam as propriedades dos numeros reais dos axiomas >>que definem esta estrutura, sem recorrer a qualquer propriedade geometrica >>derivada de uma eventual visualizacao dos numeros sobre uma reta ... >> >>Mas se, por um lado, os axiomas de um corpo ordenado completo nao implicam >>ou requerem explicitamente uma estrutura geometrica conhecida, e inegavel >>que a visualizacao "informal" que fazemos facilita muitos raciocinio ... >>SERIA REALMENTE IMPOSSIVEL ASSOCIAR A UM CORPO ORDENADO COMPLETO UMA >>GEOMETRIA, ATRAVES DE AXIOMAS OU OUTROS RECURSOS, DE FORMA QUE PUDESSEMOS >>TER OUTRAS VISUALIZACOES, MESMO QUE ESTRANHAS, POREM, CONMSISTENTES ? NAO >>PODERIAM ALGUMAS PROPRIEDADES NUMERICAS DEPENDEREM INEXORAVELMENTE DE UMA >>TAL GEOMETRIA ? >> >>E bem provavel que eu esteja errado, mas nao consigo perceber o meu erro >>... EU ACHO QUE CERTOS MAPEAMENTOS NUMERICOS REQUEREM OU IMPLICAM QUE OS >>NUMEROS NATURAIS TEM UMA GEOMETRIA OU ESTRUTURA INTRINSECA, SEM A QUAL NAO >>DA PRA COMPREENDER CERTAS COISAS ... E NECESSARIO OU POSTULAR UMA >>DISPOSICAO ESTRATIGRAFICA OU SUPOR QUE CERTOS MAPEAMENTOS INDUZEM UMA TAL >>ESTRATIFICACAO ... >> >>Bom, se alguem puder falar alguma coisa interessante sobre este tema eu >>fico muito grato, pois este e realmente um SENTIMENTO ESTRANHO que me tem >>ocorrido com alguma frequencia. Pode ser uma burrice momentanea que esta >>me levando a estas perguntas aparentemente idiotas e sem sentido, mas eu >>nao iria ocupar o tempo de voces, meus amigos, se nao tivesse razoes seria >>pra fazer isso ... >> >>Um abraco a Todos >>Paulo Santa Rita >>2,1818,090902 >> >> >>_________________________________________________________________ >>Converse com seus amigos online, faça o download grátis do MSN Messenger: >>http://messenger.msn.com.br >> >>========================================================================= >>Instruções para entrar na lista, sair da lista e usar a lista em >>http://www.mat.puc-rio.br/~nicolau/olimp/obm-l.html >>O administrador desta lista é >>========================================================================= > > > > >_________________________________________________________________ >Send and receive Hotmail on your mobile device: http://mobile.msn.com > >========================================================================= >Instruções para entrar na lista, sair da lista e usar a lista em >http://www.mat.puc-rio.br/~nicolau/olimp/obm-l.html >O administrador desta lista é >========================================================================= _________________________________________________________________ Converse com seus amigos online, faça o download grátis do MSN Messenger: http://messenger.msn.com.br ========================================================================= Instruções para entrar na lista, sair da lista e usar a lista em http://www.mat.puc-rio.br/~nicolau/olimp/obm-l.html O administrador desta lista é ========================================================================= From owner-obm-l@sucuri.mat.puc-rio.br Wed Sep 11 13:24:42 2002 Return-Path: Received: (from majordom@localhost) by sucuri.mat.puc-rio.br (8.9.3/8.9.3) id NAA16500 for obm-l-MTTP; Wed, 11 Sep 2002 13:23:43 -0300 Received: from hotmail.com (f18.law4.hotmail.com [216.33.149.18]) by sucuri.mat.puc-rio.br (8.9.3/8.9.3) with ESMTP id NAA16496 for ; Wed, 11 Sep 2002 13:23:40 -0300 Received: from mail pickup service by hotmail.com with Microsoft SMTPSVC; Wed, 11 Sep 2002 09:24:43 -0700 Received: from 200.194.208.222 by lw4fd.law4.hotmail.msn.com with HTTP; Wed, 11 Sep 2002 16:24:43 GMT X-Originating-IP: [200.194.208.222] From: "Laurito Alves" To: obm-l@mat.puc-rio.br Subject: Re: [obm-l] Um Estranho Sentimento ... Date: Wed, 11 Sep 2002 16:24:43 +0000 Mime-Version: 1.0 Content-Type: text/plain; charset=iso-8859-1; format=flowed Message-ID: X-OriginalArrivalTime: 11 Sep 2002 16:24:43.0378 (UTC) FILETIME=[BC574520:01C259AF] Sender: owner-obm-l@sucuri.mat.puc-rio.br Precedence: bulk Reply-To: obm-l@mat.puc-rio.br Colegas da lista, Os gregos representavam números como segmentos. Penso que a questão não é "por que se pensou na reta para representar os números ?" mas "por que se representam como números os segmentos da reta ?" Laurito >>Eu sei que foram fatos geometricos e outros fenomenos cotidianos >(divida -> numero negativo, divisao de um objeto -> fracao, etc etc ) que >nos levaram a descoberta das diversas classes de numeros, "construidos" >posteriormente e hoje apresentados com o auxilio da abstracao matematica >... > >Colocar estes numeros em uma reta, porem, e uma construcao humana gratuita >... nao ha nenhum razao forte para tanto e os axiomas de um corpo ordenado >completo nao induzem, a priori, a nenhuma topologia particular ... O que >estes axiomas podem falar sobre disposicao ou configuracao ? Isso : Nada ! >Nos poderiamos pensar sobre eles com igual correcao se os visualizassemos >sobre um ramo de parabola, por exemplo. ME PARECE, salvo melhor juizo, que >a unica exigencia que podemos fazer sobre uma possivel representacao e a de >continuidade ... E a continuidade, conforme todos nos sabemos, nao e uma >propriedade metrica. > >Bom, sendo assim, respeitados os axiomas de um corpo ordenado completo, nos >podemos pensar nos numeros reais como estando disposto de outra forma, >desde que esta estratificacao preserve a continuidade ... A questao e : e >vantajoso fazer isso ? e util ? Com esta imagem nos conseguiremos resolver >ou esclarecer algum fato que ainda nao foi resolvido ou esclarecido ? So >assim um mudanca ou inovacao e justificavel ... > >Nao sei se consegui ser claro, mas percebi que voce pensou seriamente sobre >a minha mensagem e nao supos que eu seja tao simplorio que nao perceba >sobre a gravidade e implicacoes do que estou falando ... > >Um abraco >Paulo Santa Rita >4,1223,110902 > > > > >>From: "Rogerio Fajardo" >>Reply-To: obm-l@mat.puc-rio.br >>To: obm-l@mat.puc-rio.br >>Subject: Re: [obm-l] Um Estranho Sentimento ... >>Date: Wed, 11 Sep 2002 00:32:33 +0000 >> >>Não compreendi bem o que voce quer dizer, mas me interessei por seu >>comentário. Pelo que entendi, voce quer saber se existe outra forma de >>visualizar, intuitivamente, os números, de forma a enxergar propriedades >>que são difíceis de enxergar com a visualização com as quais estamos >>acostumados. É isso ou nada a ver? >> >>O que eu percebo é que não é bem a geometria que serve para nos dar uma >>intuição dos números reais, mas os números reais surgiram para descrever a >>geometria de forma precisa. Não sei se isso tem algo a ver com seu e-mail. >>Detalhe-me mais o seu pensamento. >> >> >>>From: "Paulo Santa Rita" >>>Reply-To: obm-l@mat.puc-rio.br >>>To: obm-l@mat.puc-rio.br >>>Subject: [obm-l] Um Estranho Sentimento ... >>>Date: Mon, 09 Sep 2002 21:19:23 +0000 >>> >>>Ola Pessoal e demais >>>colegas desta lista ... OBM-L, >>> >>>Eu tenho pensado continua e longamente em um conjunto de questoes >>>relacionadas que me levam, invariavelmente, a uma mesma direcao que nao >>>estou conseguindo acreditar ... ate parece que depois de tanto refletir >>>cheguei a alguma constatacao insana ou simploria demais ... Se algum >>>colega puder falar algo esclarecedor e/ou interessante eu ficarei muito >>>grato ! >>> >>>Desde a infancia somos instados a pensar que os numeros reais estao >>>dispostos ao longo de uma linha reta. Nos dizemos : 3 < 5 ! E >>>imediatamente visualizamos o 3 a esquerda do 5, ambos em uma linha reta ! >>>Por que nos pensamos assim ? >>> >>>E verdade que em cursos de analise os livros definem R como um corpo >>>ordenado completo e derivam as propriedades dos numeros reais dos axiomas >>>que definem esta estrutura, sem recorrer a qualquer propriedade >>>geometrica derivada de uma eventual visualizacao dos numeros sobre uma >>>reta ... >>> >>>Mas se, por um lado, os axiomas de um corpo ordenado completo nao >>>implicam ou requerem explicitamente uma estrutura geometrica conhecida, e >>>inegavel que a visualizacao "informal" que fazemos facilita muitos >>>raciocinio ... SERIA REALMENTE IMPOSSIVEL ASSOCIAR A UM CORPO ORDENADO >>>COMPLETO UMA GEOMETRIA, ATRAVES DE AXIOMAS OU OUTROS RECURSOS, DE FORMA >>>QUE PUDESSEMOS TER OUTRAS VISUALIZACOES, MESMO QUE ESTRANHAS, POREM, >>>CONMSISTENTES ? NAO PODERIAM ALGUMAS PROPRIEDADES NUMERICAS DEPENDEREM >>>INEXORAVELMENTE DE UMA TAL GEOMETRIA ? >>> >>>E bem provavel que eu esteja errado, mas nao consigo perceber o meu erro >>>... EU ACHO QUE CERTOS MAPEAMENTOS NUMERICOS REQUEREM OU IMPLICAM QUE OS >>>NUMEROS NATURAIS TEM UMA GEOMETRIA OU ESTRUTURA INTRINSECA, SEM A QUAL >>>NAO DA PRA COMPREENDER CERTAS COISAS ... E NECESSARIO OU POSTULAR UMA >>>DISPOSICAO ESTRATIGRAFICA OU SUPOR QUE CERTOS MAPEAMENTOS INDUZEM UMA TAL >>>ESTRATIFICACAO ... >>> >>>Bom, se alguem puder falar alguma coisa interessante sobre este tema eu >>>fico muito grato, pois este e realmente um SENTIMENTO ESTRANHO que me tem >>>ocorrido com alguma frequencia. Pode ser uma burrice momentanea que esta >>>me levando a estas perguntas aparentemente idiotas e sem sentido, mas eu >>>nao iria ocupar o tempo de voces, meus amigos, se nao tivesse razoes >>>seria pra fazer isso ... >>> >>>Um abraco a Todos >>>Paulo Santa Rita >>>2,1818,090902 >>> >>> >>>_________________________________________________________________ >>>Converse com seus amigos online, faça o download grátis do MSN Messenger: >>>http://messenger.msn.com.br >>> >>>========================================================================= >>>Instruções para entrar na lista, sair da lista e usar a lista em >>>http://www.mat.puc-rio.br/~nicolau/olimp/obm-l.html >>>O administrador desta lista é >>>========================================================================= >> >> >> >> >>_________________________________________________________________ >>Send and receive Hotmail on your mobile device: http://mobile.msn.com >> >>========================================================================= >>Instruções para entrar na lista, sair da lista e usar a lista em >>http://www.mat.puc-rio.br/~nicolau/olimp/obm-l.html >>O administrador desta lista é >>========================================================================= > > > > >_________________________________________________________________ >Converse com seus amigos online, faça o download grátis do MSN Messenger: >http://messenger.msn.com.br > >========================================================================= >Instruções para entrar na lista, sair da lista e usar a lista em >http://www.mat.puc-rio.br/~nicolau/olimp/obm-l.html >O administrador desta lista é >========================================================================= ========================================================================= Instruções para entrar na lista, sair da lista e usar a lista em http://www.mat.puc-rio.br/~nicolau/olimp/obm-l.html O administrador desta lista é ========================================================================= From owner-obm-l@sucuri.mat.puc-rio.br Wed Sep 11 13:55:53 2002 Return-Path: Received: (from majordom@localhost) by sucuri.mat.puc-rio.br (8.9.3/8.9.3) id NAA17363 for obm-l-MTTP; Wed, 11 Sep 2002 13:55:00 -0300 Received: from web12908.mail.yahoo.com (web12908.mail.yahoo.com [216.136.174.75]) by sucuri.mat.puc-rio.br (8.9.3/8.9.3) with SMTP id NAA17359 for ; Wed, 11 Sep 2002 13:54:57 -0300 Message-ID: <20020911165600.20895.qmail@web12908.mail.yahoo.com> Received: from [200.206.103.3] by web12908.mail.yahoo.com via HTTP; Wed, 11 Sep 2002 13:56:00 ART Date: Wed, 11 Sep 2002 13:56:00 -0300 (ART) From: =?iso-8859-1?q?Johann=20Peter=20Gustav=20Lejeune=20Dirichlet?= Subject: Re: [obm-l] Re: [obm-l] O caráter não enumerável de R To: obm-l@mat.puc-rio.br In-Reply-To: MIME-Version: 1.0 Content-Type: multipart/alternative; boundary="0-1597605345-1031763360=:20736" Content-Transfer-Encoding: 8bit Sender: owner-obm-l@sucuri.mat.puc-rio.br Precedence: bulk Reply-To: obm-l@mat.puc-rio.br --0-1597605345-1031763360=:20736 Content-Type: text/plain; charset=iso-8859-1 Content-Transfer-Encoding: 8bit Deve ter uma confusao ai....O que quero dizer e F(N,N),e essa notaçao eu vi no livro de analise do Elon. Essa demonstraçao,em linhas gerais,seria... 1.Cada irracional tem uma unica representaçao decimal;Cada racional e representado por noves infinitos no fim;e 0=0.0000000000...... 2.Suponha que no [0,1) de pra enumerar.Veja um exemplo: r_1=0,22109... r_2=0,26353... r_3=0.94553... r_4=0,93442... r_5=0.83429... Seja r=0,26549... o real obtido pegando as diagonais(o 1° de r_1,o 2° de r_2,o 3° de r_3....).Onde tem 2,poe 9,e onde nao tem 2 poe 2:r'=0,92222... Veja que r' sempre sera diferente de r_i(o i° algarismo apos a virgula nao e igual nos numeros).Logo r' nao esta na lista.E pronto! Laurito Alves wrote:O que você chama de N*N*N************ ??? Se for um produto cartesiano de N uma quantidade enumerável de vezes, ele é enumerável. Laurito >From: Johann Peter Gustav Lejeune Dirichlet > >Reply-To: obm-l@mat.puc-rio.br >To: obm-l@mat.puc-rio.br >Subject: Re: [obm-l] O caráter não enumerável de R >Date: Mon, 9 Sep 2002 16:28:42 -0300 (ART) > > > Eu acho que voce ta viajando demais.Enumeravel e o conjunto com uma >bijecao nos naturais. >Os reais nao sao enumeraveis pelo fato de que N*N*N************nao e >enumeravel > 498 - Artur Costa Steiner escreveu: Um abraço a >todos os amigos deste grupo no qual acabei de me inscrever! > >O assunto que mencionei sempre me intriga um pouco. Há uma clássica >demonstração de que R (o conjunto dos reais)não é numerável e que pode >ser encontrada na maioria dos livros sobre Análise. Estas provas >baseiam-se no fato de que, nos espaços euclidianos, conjuntos perfeitos >não são numeráveis. Logo, um ponto chave em tais provas é que os >elementos do espaço são pontos de acumulação do mesmo. > >Sabemos que todo elemento de R é ponto de acumulação. Mas, e este é o >ponto que me intriga, tal conclusão depende da métrica definida em R. >Na métrica euclidiana usual tal fato é demonstrado (admitindo-se que R >seja completo). Mas, se tomarmos, por exemplo, a chamada métrica >discreta (d(x,y)=1, se x<>y e d(x,y)=0 se x=y))então nenhum elemento de >R (ou do espaço métrico em questão) é ponto de acumulação. A provas que >conheço sobre a não enumerabilidade de R (que consistem em se construir >uma seqüência de intervalos fechados aninhados) não mais se aplicam na >métrica discreta. > >Não me parece plausível que um espaço métrico seja enumerável numa >métrica (ou topologia) e não numerável em outra, mas será que existe >uma prova de que R (ou um espaço métrico qualquer) não é numerável a >qual seja independente da forma segundo a qual definamos seus conjuntos >abertos? > >Artur >========================================================================= >Instruções para entrar na lista, sair da lista e usar a lista em >http://www.mat.puc-rio.br/~nicolau/olimp/obm-l.html >O administrador desta lista é >========================================================================= > >--------------------------------- >Yahoo! PageBuilder - O super editor para criação de sites: é grátis, fácil >e rápido. _________________________________________________________________ Tenha você também um MSN Hotmail, o maior webmail do mundo: http://www.hotmail.com/br ========================================================================= Instruções para entrar na lista, sair da lista e usar a lista em http://www.mat.puc-rio.br/~nicolau/olimp/obm-l.html O administrador desta lista é ========================================================================= --------------------------------- Yahoo! PageBuilder - O super editor para criação de sites: é grátis, fácil e rápido. --0-1597605345-1031763360=:20736 Content-Type: text/html; charset=iso-8859-1 Content-Transfer-Encoding: 8bit

Deve ter uma confusao ai....O que quero dizer e F(N,N),e essa notaçao eu vi no livro de analise do Elon.

Essa demonstraçao,em linhas gerais,seria...

1.Cada irracional tem uma unica representaçao decimal;Cada racional e representado por noves infinitos no fim;e 0=0.0000000000......

2.Suponha que no [0,1) de pra enumerar.Veja um exemplo:   

r_1=0,22109...

r_2=0,26353...

r_3=0.94553...

r_4=0,93442...

r_5=0.83429...

Seja r=0,26549... o real obtido pegando as diagonais(o 1° de r_1,o 2° de r_2,o 3° de r_3....).Onde tem 2,poe 9,e onde nao tem 2 poe 2:r'=0,92222...

Veja que r' sempre sera diferente de r_i(o i° algarismo apos a virgula nao e igual nos numeros).Logo r' nao esta na lista.E pronto!

 

 

 Laurito Alves wrote:

O que você chama de N*N*N************ ???

Se for um produto cartesiano de N uma quantidade enumerável de vezes, ele é
enumerável.

Laurito


>From: Johann Peter Gustav Lejeune Dirichlet
>
>Reply-To: obm-l@mat.puc-rio.br
>To: obm-l@mat.puc-rio.br
>Subject: Re: [obm-l] O caráter não enumerável de R
>Date: Mon, 9 Sep 2002 16:28:42 -0300 (ART)
>
>
> Eu acho que voce ta viajando demais.Enumeravel e o conjunto com uma
>bijecao nos naturais.
>Os reais nao sao enumeraveis pelo fato de que N*N*N************nao e
>enumeravel
> 498 - Artur Costa Steiner escreveu: Um abraço a
>todos os amigos deste grupo no qual acabei de me inscrever!
>
>O assunto que mencionei sempre me intriga um pouco. Há uma clássica
>demonstração de que R (o conjunto dos reais)não é numerável e que pode
>ser encontrada na maioria dos livros sobre Análise. Estas provas
>baseiam-se no fato de que, nos espaços euclidianos, conjuntos perfeitos
>não são numeráveis. Logo, um ponto chave em tais provas é que os
>elementos do espaço são pontos de acumulação do mesmo.
>
>Sabemos que todo elemento de R é ponto de acumulação. Mas, e este é o
>ponto que me intriga, tal conclusão depende da métrica definida em R.
>Na métrica euclidiana usual tal fato é demonstrado (admitindo-se que R
>seja completo). Mas, se tomarmos, por exemplo, a chamada métrica
>discreta (d(x,y)=1, se x<>y e d(x,y)=0 se x=y))então nenhum elemento de
>R (ou do espaço métrico em questão) é ponto de acumulação. A provas que
>conheço sobre a não enumerabilidade de R (que consistem em se construir
>uma seqüência de intervalos fechados aninhados) não mais se aplicam na
>métrica discreta.
>
>Não me parece plausível que um espaço métrico seja enumerável numa
>métrica (ou topologia) e não numerável em outra, mas será que existe
>uma prova de que R (ou um espaço métrico qualquer) não é numerável a
>qual seja independente da forma segundo a qual definamos seus conjuntos
>abertos?
>
>Artur
>=========================================================================
>Instruções para entrar na lista, sair da lista e usar a lista em
>http://www.mat.puc-rio.br/~nicolau/olimp/obm-l.html
>O administrador desta lista é
>=========================================================================
>
>---------------------------------
>Yahoo! PageBuilder - O super editor para criação de sites: é grátis, fácil
>e rápido.




_________________________________________________________________
Tenha você também um MSN Hotmail, o maior webmail do mundo:
http://www.hotmail.com/br

=========================================================================
Instruções para entrar na lista, sair da lista e usar a lista em
http://www.mat.puc-rio.br/~nicolau/olimp/obm-l.html
O administrador desta lista é
=========================================================================


Yahoo! PageBuilder - O super editor para criação de sites: é grátis, fácil e rápido. --0-1597605345-1031763360=:20736-- ========================================================================= Instruções para entrar na lista, sair da lista e usar a lista em http://www.mat.puc-rio.br/~nicolau/olimp/obm-l.html O administrador desta lista é ========================================================================= From owner-obm-l@sucuri.mat.puc-rio.br Wed Sep 11 14:30:06 2002 Return-Path: Received: (from majordom@localhost) by sucuri.mat.puc-rio.br (8.9.3/8.9.3) id OAA18261 for obm-l-MTTP; Wed, 11 Sep 2002 14:27:53 -0300 Received: from zeus.opendf.com.br (zeus.opengate.com.br [200.181.71.10]) by sucuri.mat.puc-rio.br (8.9.3/8.9.3) with ESMTP id OAA18257 for ; Wed, 11 Sep 2002 14:27:50 -0300 Received: from localhost (localhost.opengate.com.br [127.0.0.1]) by zeus.opendf.com.br (Postfix) with ESMTP id CF3E23EB58 for ; Wed, 11 Sep 2002 14:28:48 -0300 (BRT) Received: by zeus.opendf.com.br (Postfix, from userid 48) id DB2023EB56; Wed, 11 Sep 2002 14:28:34 -0300 (BRT) From: "498 - Artur Costa Steiner" To: obm-l@mat.puc-rio.br Subject: Re: [obm-l] Um Estranho Sentimento ... X-Mailer: NeoMail 1.25 X-IPAddress: 200.252.155.2 MIME-Version: 1.0 Content-Type: text/plain; charset=iso-8859-1 Message-Id: <20020911172834.DB2023EB56@zeus.opendf.com.br> Date: Wed, 11 Sep 2002 14:28:34 -0300 (BRT) X-Virus-Scanned: by AMaViS new-20020517 Sender: owner-obm-l@sucuri.mat.puc-rio.br Precedence: bulk Reply-To: obm-l@mat.puc-rio.br > Colegas da lista, > > Os gregos representavam números como segmentos. Penso que a questão não é > "por que se pensou na reta para representar os números ?" mas "por que se > representam como números os segmentos da reta ?" Existe uma relação biunívuca entre números reais e pontos de uma reta. Disto decorre que um segmento de reta poosa ser visto como um intervalo no conjunto dos reais. Artur ========================================================================= Instruções para entrar na lista, sair da lista e usar a lista em http://www.mat.puc-rio.br/~nicolau/olimp/obm-l.html O administrador desta lista é ========================================================================= From owner-obm-l@sucuri.mat.puc-rio.br Wed Sep 11 14:39:44 2002 Return-Path: Received: (from majordom@localhost) by sucuri.mat.puc-rio.br (8.9.3/8.9.3) id OAA18693 for obm-l-MTTP; Wed, 11 Sep 2002 14:39:25 -0300 Received: from web21307.mail.yahoo.com (web21307.mail.yahoo.com [216.136.128.232]) by sucuri.mat.puc-rio.br (8.9.3/8.9.3) with SMTP id OAA18689 for ; Wed, 11 Sep 2002 14:39:22 -0300 Message-ID: <20020911174020.9679.qmail@web21307.mail.yahoo.com> Received: from [200.247.197.140] by web21307.mail.yahoo.com via HTTP; Wed, 11 Sep 2002 14:40:20 ART Date: Wed, 11 Sep 2002 14:40:20 -0300 (ART) From: =?iso-8859-1?q?Jorge=20Paulino?= Subject: [obm-l] IRRACIONALIDADE DE PI To: obm-l@mat.puc-rio.br MIME-Version: 1.0 Content-Type: text/plain; charset=iso-8859-1 Content-Transfer-Encoding: 8bit Sender: owner-obm-l@sucuri.mat.puc-rio.br Precedence: bulk Reply-To: obm-l@mat.puc-rio.br Galera, alguém conhece alguma demonstração, acessível a alunos de segundo grau, da irracionalidade do número PI? Um Abraço, Jorge _______________________________________________________________________ Yahoo! PageBuilder O super editor para criação de sites: é grátis, fácil e rápido. http://br.geocities.yahoo.com/v/pb.html ========================================================================= Instruções para entrar na lista, sair da lista e usar a lista em http://www.mat.puc-rio.br/~nicolau/olimp/obm-l.html O administrador desta lista é ========================================================================= From owner-obm-l@sucuri.mat.puc-rio.br Wed Sep 11 14:44:06 2002 Return-Path: Received: (from majordom@localhost) by sucuri.mat.puc-rio.br (8.9.3/8.9.3) id OAA18890 for obm-l-MTTP; Wed, 11 Sep 2002 14:43:50 -0300 Received: from zeus.opendf.com.br (zeus.opengate.com.br [200.181.71.10]) by sucuri.mat.puc-rio.br (8.9.3/8.9.3) with ESMTP id OAA18886 for ; Wed, 11 Sep 2002 14:43:45 -0300 Received: from localhost (localhost.opengate.com.br [127.0.0.1]) by zeus.opendf.com.br (Postfix) with ESMTP id 2F1A13EB48 for ; Wed, 11 Sep 2002 14:44:46 -0300 (BRT) Received: by zeus.opendf.com.br (Postfix, from userid 48) id 132933EB52; Wed, 11 Sep 2002 14:44:44 -0300 (BRT) From: "498 - Artur Costa Steiner" To: obm-l@mat.puc-rio.br Subject: Re: [obm-l] Ajuda Algebra linear (Off Topic) X-Mailer: NeoMail 1.25 X-IPAddress: 200.252.155.2 MIME-Version: 1.0 Content-Type: text/plain; charset=iso-8859-1 Message-Id: <20020911174444.132933EB52@zeus.opendf.com.br> Date: Wed, 11 Sep 2002 14:44:44 -0300 (BRT) X-Virus-Scanned: by AMaViS new-20020517 Sender: owner-obm-l@sucuri.mat.puc-rio.br Precedence: bulk Reply-To: obm-l@mat.puc-rio.br Se você quiser uma aplicação prática de álgebra linear, eu lhe dou uma, pois trabalho com isso: otimização da operação do sistema elétrico brasileiro, isto é, o processo de decidir quanto você deve gerar em cada usina hidrelétrica e em cada usina termelétrica, bem como quando cadaegião do país deve enviar para outra, de modo a minimizar o custo total da operação. Isto é resolvido através de um algoritmo de programação dinâmica estocástica. Há modelos de curto prazo, utilizados pelo ONS e outros de longo prazo, utilizados para horizontes de mais de 5 anos. O problema não é linear, mas algumas de suas partes podem ser aproximadas muito bem por funcões objetivo e restrições lineares. Esta parte do problema utiliza Programação Linear, a qual é uma aplicação da Álgebra Linear, baseia-se em matrizes, bases de um espaço vetorial, dependência linear, etc. No caso que estou citando, usamos o Simplex, desenvolvido na década de 60 por George Dantzig, nos EUA. Embora hoje existam outros algoritmos para resolução de problemas lineares, baseados em pontos interiores, o Simplex continua sendo uma boa opção. Já utilizei também Programação Linear em uma planilha Excel para otimizar a expansão de um sistema térmico no Norte do Brasil, o qual ajudava a decidir que tipos de unidades deveriam ser implantadas. Artur ========================================================================= Instruções para entrar na lista, sair da lista e usar a lista em http://www.mat.puc-rio.br/~nicolau/olimp/obm-l.html O administrador desta lista é ========================================================================= From owner-obm-l@sucuri.mat.puc-rio.br Wed Sep 11 14:57:58 2002 Return-Path: Received: (from majordom@localhost) by sucuri.mat.puc-rio.br (8.9.3/8.9.3) id OAA19568 for obm-l-MTTP; Wed, 11 Sep 2002 14:57:32 -0300 Received: from studer.bol.com.br (studer.bol.com.br [200.221.24.21]) by sucuri.mat.puc-rio.br (8.9.3/8.9.3) with ESMTP id OAA19532 for ; Wed, 11 Sep 2002 14:57:25 -0300 Received: from proxyvr (200.221.24.99) by studer.bol.com.br (5.1.071) id 3D5D1D43008A0E26 for obm-l@mat.puc-rio.br; Wed, 11 Sep 2002 14:57:38 -0300 Message-ID: <000e01c259bd$4e76e160$2932a8c0@redeconora.com.br> From: "Hely Jr." To: Subject: [obm-l] TESTE Date: Wed, 11 Sep 2002 15:01:50 -0300 MIME-Version: 1.0 Content-Type: text/plain; charset="iso-8859-1" Content-Transfer-Encoding: 8bit X-Priority: 3 X-MSMail-Priority: Normal X-Mailer: Microsoft Outlook Express 6.00.2600.0000 X-MimeOLE: Produced By Microsoft MimeOLE V6.00.2600.0000 X-Sender-IP: 200.222.64.218 Sender: owner-obm-l@sucuri.mat.puc-rio.br Precedence: bulk Reply-To: obm-l@mat.puc-rio.br Observem a sequência abaixo: 2 - 10 - 12 - 16 - 17 - 18 - 19 ...... E agora, respondam: qual o próximo número da sequência??? a) 20 b) 22 c) 175 d) 200 Existe alguma pegadinha neste exercício? ========================================================================= Instruções para entrar na lista, sair da lista e usar a lista em http://www.mat.puc-rio.br/~nicolau/olimp/obm-l.html O administrador desta lista é ========================================================================= From owner-obm-l@sucuri.mat.puc-rio.br Wed Sep 11 15:03:10 2002 Return-Path: Received: (from majordom@localhost) by sucuri.mat.puc-rio.br (8.9.3/8.9.3) id PAA19966 for obm-l-MTTP; Wed, 11 Sep 2002 15:03:02 -0300 Received: (from nicolau@localhost) by sucuri.mat.puc-rio.br (8.9.3/8.9.3) id PAA19954 for obm-l@mat.puc-rio.br; Wed, 11 Sep 2002 15:03:01 -0300 Date: Wed, 11 Sep 2002 15:03:01 -0300 From: "Nicolau C. Saldanha" To: obm-l@mat.puc-rio.br Subject: Re: [obm-l] IRRACIONALIDADE DE PI Message-ID: <20020911150301.B17567@sucuri.mat.puc-rio.br> References: <20020911174020.9679.qmail@web21307.mail.yahoo.com> Mime-Version: 1.0 Content-Type: text/plain; charset=iso-8859-1 Content-Disposition: inline Content-Transfer-Encoding: 8bit User-Agent: Mutt/1.2.5i In-Reply-To: <20020911174020.9679.qmail@web21307.mail.yahoo.com>; from jorgepsf@yahoo.com.br on Wed, Sep 11, 2002 at 02:40:20PM -0300 Sender: owner-obm-l@sucuri.mat.puc-rio.br Precedence: bulk Reply-To: obm-l@mat.puc-rio.br On Wed, Sep 11, 2002 at 02:40:20PM -0300, Jorge Paulino wrote: > Galera, alguém conhece alguma demonstração, > acessível a alunos de segundo grau, da irracionalidade > do número PI? > Um Abraço, > Jorge Não acho que *exista* uma demonstração da irracionalidade de pi que seja acessível a alunos (típicos) de segundo grau. Mas há uma demonstração curta usando cálculo em 'Proofs from the book' de Aigner e Ziegler (há uma tradução bem recente). Outra referência é 'Irrational numbers' de Ivan Niven. []s, N. ========================================================================= Instruções para entrar na lista, sair da lista e usar a lista em http://www.mat.puc-rio.br/~nicolau/olimp/obm-l.html O administrador desta lista é ========================================================================= From owner-obm-l@sucuri.mat.puc-rio.br Wed Sep 11 15:05:30 2002 Return-Path: Received: (from majordom@localhost) by sucuri.mat.puc-rio.br (8.9.3/8.9.3) id PAA20088 for obm-l-MTTP; Wed, 11 Sep 2002 15:05:19 -0300 Received: (from nicolau@localhost) by sucuri.mat.puc-rio.br (8.9.3/8.9.3) id PAA20083 for obm-l@mat.puc-rio.br; Wed, 11 Sep 2002 15:05:19 -0300 Date: Wed, 11 Sep 2002 15:05:19 -0300 From: "Nicolau C. Saldanha" To: obm-l@mat.puc-rio.br Subject: Re: [obm-l] TESTE Message-ID: <20020911150519.C17567@sucuri.mat.puc-rio.br> References: <000e01c259bd$4e76e160$2932a8c0@redeconora.com.br> Mime-Version: 1.0 Content-Type: text/plain; charset=iso-8859-1 Content-Disposition: inline Content-Transfer-Encoding: 8bit User-Agent: Mutt/1.2.5i In-Reply-To: <000e01c259bd$4e76e160$2932a8c0@redeconora.com.br>; from helynatal@bol.com.br on Wed, Sep 11, 2002 at 03:01:50PM -0300 Sender: owner-obm-l@sucuri.mat.puc-rio.br Precedence: bulk Reply-To: obm-l@mat.puc-rio.br On Wed, Sep 11, 2002 at 03:01:50PM -0300, Hely Jr. wrote: > Observem a sequência abaixo: > > 2 - 10 - 12 - 16 - 17 - 18 - 19 ...... > > E agora, respondam: qual o próximo número da sequência??? > > a) 20 > b) 22 > c) 175 > d) 200 > > Existe alguma pegadinha neste exercício? Existe. Este problema é fácil para turmas de alfabetização (no Brasil!) mas difícil para matemáticos... ;-) []s, N. ========================================================================= Instruções para entrar na lista, sair da lista e usar a lista em http://www.mat.puc-rio.br/~nicolau/olimp/obm-l.html O administrador desta lista é ========================================================================= From owner-obm-l@sucuri.mat.puc-rio.br Wed Sep 11 16:01:59 2002 Return-Path: Received: (from majordom@localhost) by sucuri.mat.puc-rio.br (8.9.3/8.9.3) id QAA22977 for obm-l-MTTP; Wed, 11 Sep 2002 16:00:45 -0300 Received: from zeus.opendf.com.br (zeus.opengate.com.br [200.181.71.10]) by sucuri.mat.puc-rio.br (8.9.3/8.9.3) with ESMTP id QAA22973 for ; Wed, 11 Sep 2002 16:00:41 -0300 Received: from localhost (localhost.opengate.com.br [127.0.0.1]) by zeus.opendf.com.br (Postfix) with ESMTP id DFACF3EB0E for ; Wed, 11 Sep 2002 16:01:42 -0300 (BRT) Received: by zeus.opendf.com.br (Postfix, from userid 48) id 605DB3EB20; Wed, 11 Sep 2002 16:01:42 -0300 (BRT) From: "498 - Artur Costa Steiner" To: obm-l@mat.puc-rio.br Subject: [obm-l] Axioma da Escolha X-Mailer: NeoMail 1.25 X-IPAddress: 200.252.155.2 MIME-Version: 1.0 Content-Type: text/plain; charset=iso-8859-1 Message-Id: <20020911190142.605DB3EB20@zeus.opendf.com.br> Date: Wed, 11 Sep 2002 16:01:42 -0300 (BRT) X-Virus-Scanned: by AMaViS new-20020517 Sender: owner-obm-l@sucuri.mat.puc-rio.br Precedence: bulk Reply-To: obm-l@mat.puc-rio.br Nos últimos dias o Axioma da Escolha foi bastante mencionado nesta lista, motivado por um interessante problema (violência), sugerido por uma das participantes, e que involve este axioma. Eu não estou certo, mas, no meio matemático, ainda existem hoje restrições a este axioma, no sentido de que alguma prova nele baseada possa ser considerada questionável ou mesmo inválida? Parece-me que a grande maioria dos matemáticos o usam sem qualquer problema. Em livros de autores consagrados, como Rudin, Folland e Munkres, há provas de teoremas que, ao que me parece, involvem implicitamente este axioma. Por exemplo, o conhecido teorema de que um espaço métrico é totalmente limitado se, e somente se, qualquer sequencia do mesmo contiver uma subsequencia de Cauchy, envolve, ao que me parece, o Axioma da Escolha. Na parte "se", geralmente se usa um argumento contrapositivo e mostra- se que, se o espaço não for totalmente limitado, então há nele uma sequencia que não pode ser de Cauchy; com este objetivo gera-se uma sequencia escolhendo-se elementos de subconjuntos do espaço que distem dos anteriores um valor maior ou igual que um dado r>0. Na parte "somente", geralmente cobre-se sequencialmente o espaço por coleções finitas de bolas aberta de raio 1/n e escolhem-se termos da sequencia contidos nas intersecções das bolas selecionada em cada passo. Isto não é o axioma da escolha? Vários teoremas sobre conjuntos compactos também envolvem escolha. Por exemplo, o que prova que se S (espaço métrico) é compacto se, e somente se, for sequencialmente compacto. Não estou certo se isto é exatamente o axioma da escolha, mas a prova que conheço do teorema que diz que todo conjunto aberto de R é dado por uma união numerável e disjunta de intervalos abertos também envolve escolha de números racionais na coleção de intervalos para mostra que a mesma é numerável. Não sei exatamente porque, mas como há uma prova de que o conjunto dos racionais é numerável, parece que um processo de escolha de racionais não envolve o axioma da escolha. Estou um tanto confuso a respeito deste axioma e qualquer contribuição é bem vinda. Um abraço Artur ========================================================================= Instruções para entrar na lista, sair da lista e usar a lista em http://www.mat.puc-rio.br/~nicolau/olimp/obm-l.html O administrador desta lista é ========================================================================= From owner-obm-l@sucuri.mat.puc-rio.br Wed Sep 11 16:04:29 2002 Return-Path: Received: (from majordom@localhost) by sucuri.mat.puc-rio.br (8.9.3/8.9.3) id QAA23068 for obm-l-MTTP; Wed, 11 Sep 2002 16:04:10 -0300 Received: from hotmail.com (f71.sea2.hotmail.com [207.68.165.71]) by sucuri.mat.puc-rio.br (8.9.3/8.9.3) with ESMTP id QAA23064 for ; Wed, 11 Sep 2002 16:04:07 -0300 Received: from mail pickup service by hotmail.com with Microsoft SMTPSVC; Wed, 11 Sep 2002 12:04:57 -0700 Received: from 32.94.119.254 by sea2fd.sea2.hotmail.msn.com with HTTP; Wed, 11 Sep 2002 19:04:57 GMT X-Originating-IP: [32.94.119.254] From: "Paulo Santa Rita" To: obm-l@mat.puc-rio.br Subject: Re: [obm-l] TESTE Date: Wed, 11 Sep 2002 19:04:57 +0000 Mime-Version: 1.0 Content-Type: text/plain; charset=iso-8859-1; format=flowed Message-ID: X-OriginalArrivalTime: 11 Sep 2002 19:04:57.0571 (UTC) FILETIME=[1ED8B330:01C259C6] Sender: owner-obm-l@sucuri.mat.puc-rio.br Precedence: bulk Reply-To: obm-l@mat.puc-rio.br Ola HELY e demais colegas desta lista ... OBM-L, De imediato, adianto que NAO VEJO como alguem possa descobrir o proximo numero e acho dificil alguem apresentar argumentos apoditicos numa circunstancia como essa. Como lenitivo, observo que os nomes dos numeros da sequencia comecam com "D" e 200 e a unica resposta tambem com esta propriedade ... Mas se alguem me falasse isso eu nao aceitaria uma tal argumentacao ! Existem Testes de Inteligencia ( avaliacao de QI )que lancam mao desta monstruosidade, isto e, dao uma sequencia e pedem que se adivinhe o proximo numero : Burro e o avaliador que supoe que ha uma unica resposta aceitavel ! Num teste assim, sem duvida eu teria uma inteligencia proxima de algum chipanze ou orangotango, mas nao acredito que eu seja tao burro assim ... Mesmo nos testes sigma, onde nao se estipula tempo para encontrar solucoes criativas e onde os avaliadores ( uma banca ) nao estipulam uma resposta correta, havendo para toda solucao elementos indicativos de criatividade e esperteza, a avaliacao ( do que eles chamam pensamento paralelo ) e suscetivel de receber muitas criticas, de forma que ninguem pode se sentir desprestigiado se nestas avaliacoes muquiranas que rotineiramente aparecem nao tiver um bom desempenho. Em minha modesta opiniao - pelo que tenho visto em pessoas que realizam tarefas "sui generis" que sugerem inteligencia - a inteligencia nao tem nada a ver com velocidade de raciocinio, aprendizagem rapida ou logica. A inteligencia me parece estar ligada a sensibilidade ... Me parece que a Mente Humana tem uma ante-sala onde ficam a IMAGINACAO, a MEMORIA e a CAPACIDADE DE CONCENTRACAO sao recursos da SENSIBILIDADE. A sensibilidade e a secretaria da INTUICAO : Depois que este pessoal ( imaginacao, memoria, cpacidade de concentracao e sensibilidade ) trabalha e faz a parte criativa eles atribuem aos servos inferiores, a logica e o raciocinio, a tarefa de justificar e convencer. Assim, a INTELIGENCIA e uma medida da maior ou menor SENSIBILIDADE com que uma pessoa percebe os objetos pelos quais se interessa, sejam eles cientificos ou nao. Beethoven foi genialissimo ( em minha opiniao o maior de todos os genios ! ), nao obstante nao ter sido um cientista, nao ter sido precoce e nao ter tido uma predizagem brilhante. Einstein foi um cara genial, nao obstante nao ter sido precoce, ter sido reprovado no primeiro vestibular que prestou e ter se formado a duras penas ( colando do colega Marcel Groissman ) e sem nenhuma distincao academica. De Newton pode-se dizer o mesmo que se disse de Einstein. Como se ve, estes testes de QI nao passam, em sua imensa maioria, de um imenso engodo. Um Abraco Paulo Santa Rita 4,1604,110902 >From: "Hely Jr." >Reply-To: obm-l@mat.puc-rio.br >To: >Subject: [obm-l] TESTE >Date: Wed, 11 Sep 2002 15:01:50 -0300 > >Observem a sequência abaixo: > >2 - 10 - 12 - 16 - 17 - 18 - 19 ...... > >E agora, respondam: qual o próximo número da sequência??? > >a) 20 >b) 22 >c) 175 >d) 200 > >Existe alguma pegadinha neste exercício? > > >========================================================================= >Instruções para entrar na lista, sair da lista e usar a lista em >http://www.mat.puc-rio.br/~nicolau/olimp/obm-l.html >O administrador desta lista é >========================================================================= _________________________________________________________________ Converse com seus amigos online, faça o download grátis do MSN Messenger: http://messenger.msn.com.br ========================================================================= Instruções para entrar na lista, sair da lista e usar a lista em http://www.mat.puc-rio.br/~nicolau/olimp/obm-l.html O administrador desta lista é ========================================================================= From owner-obm-l@sucuri.mat.puc-rio.br Wed Sep 11 16:38:44 2002 Return-Path: Received: (from majordom@localhost) by sucuri.mat.puc-rio.br (8.9.3/8.9.3) id QAA25054 for obm-l-MTTP; Wed, 11 Sep 2002 16:38:01 -0300 Received: from puma.unisys.com.br (smtp.unisys.com.br [200.220.64.7]) by sucuri.mat.puc-rio.br (8.9.3/8.9.3) with ESMTP id QAA25050 for ; Wed, 11 Sep 2002 16:37:57 -0300 Received: from josefran (riohiper01p221.uninet.com.br [200.220.2.221]) by puma.unisys.com.br (8.12.3/8.12.3) with SMTP id g8BJcujR025185 for ; Wed, 11 Sep 2002 16:39:00 -0300 (EST) X-Spam-Filter: check_local@puma.unisys.com.br by digitalanswers.org Message-ID: <00bd01c259cb$693f8980$dd02dcc8@josefran> From: "Jose Francisco Guimaraes Costa" To: References: <000e01c259bd$4e76e160$2932a8c0@redeconora.com.br> <20020911150519.C17567@sucuri.mat.puc-rio.br> Subject: Re: [obm-l] TESTE Date: Wed, 11 Sep 2002 16:42:24 -0300 MIME-Version: 1.0 Content-Type: text/plain; charset="iso-8859-1" Content-Transfer-Encoding: 8bit X-Priority: 3 X-MSMail-Priority: Normal X-Mailer: Microsoft Outlook Express 5.00.2314.1300 X-MIMEOLE: Produced By Microsoft MimeOLE V5.00.2314.1300 Sender: owner-obm-l@sucuri.mat.puc-rio.br Precedence: bulk Reply-To: obm-l@mat.puc-rio.br Passei o problema para minha neta, que está na turma de alfabetização (no Brasil, embora ela tenha nascido em Lyon, cidade onde o N já trabalhou), mas ela não conseguiu resolver. O problema não é difícil apenas para matemáticos. Também é para engenheiros, como eu. Resumindo - qual a solução? JF -----Mensagem Original----- De: Nicolau C. Saldanha Para: Enviada em: Quarta-feira, 11 de Setembro de 2002 15:05 Assunto: Re: [obm-l] TESTE > On Wed, Sep 11, 2002 at 03:01:50PM -0300, Hely Jr. wrote: > > Observem a sequência abaixo: > > > > 2 - 10 - 12 - 16 - 17 - 18 - 19 ...... > > > > E agora, respondam: qual o próximo número da sequência??? > > > > a) 20 > > b) 22 > > c) 175 > > d) 200 > > > > Existe alguma pegadinha neste exercício? > > Existe. Este problema é fácil para turmas de alfabetização (no Brasil!) > mas difícil para matemáticos... ;-) > > []s, N. > ========================================================================= Instruções para entrar na lista, sair da lista e usar a lista em http://www.mat.puc-rio.br/~nicolau/olimp/obm-l.html O administrador desta lista é ========================================================================= From owner-obm-l@sucuri.mat.puc-rio.br Wed Sep 11 16:52:37 2002 Return-Path: Received: (from majordom@localhost) by sucuri.mat.puc-rio.br (8.9.3/8.9.3) id QAA25429 for obm-l-MTTP; Wed, 11 Sep 2002 16:51:43 -0300 Received: from traven9 (traven9.uol.com.br [200.221.4.35]) by sucuri.mat.puc-rio.br (8.9.3/8.9.3) with ESMTP id QAA25425 for ; Wed, 11 Sep 2002 16:51:41 -0300 Received: from u2z7z2 ([200.158.145.96]) by traven9 (8.9.1/8.9.1) with ESMTP id QAA21364 for ; Wed, 11 Sep 2002 16:54:22 -0300 (BRT) Message-ID: <004401c259cc$f465ade0$2101a8c0@u2z7z2> From: "Wagner" To: References: <20020911174020.9679.qmail@web21307.mail.yahoo.com> Subject: Re: [obm-l] IRRACIONALIDADE DE PI Date: Wed, 11 Sep 2002 16:53:51 -0300 Organization: Wagner MIME-Version: 1.0 Content-Type: text/plain; charset="iso-8859-1" Content-Transfer-Encoding: 8bit X-Priority: 3 X-MSMail-Priority: Normal X-Mailer: Microsoft Outlook Express 5.50.4133.2400 X-MimeOLE: Produced By Microsoft MimeOLE V5.50.4133.2400 Sender: owner-obm-l@sucuri.mat.puc-rio.br Precedence: bulk Reply-To: obm-l@mat.puc-rio.br Não há uma demonstração para esse nível, mas através do calculo integral é possível deduzir que: Pi = 4( 1 - 1/3 + 1/5 - 1/7 + 1/9 -1/11 + ...) ----- Original Message ----- From: "Jorge Paulino" To: Sent: Wednesday, September 11, 2002 2:40 PM Subject: [obm-l] IRRACIONALIDADE DE PI > Galera, alguém conhece alguma demonstração, > acessível a alunos de segundo grau, da irracionalidade > do número PI? > Um Abraço, > Jorge > > _______________________________________________________________________ > Yahoo! PageBuilder > O super editor para criação de sites: é grátis, fácil e rápido. > http://br.geocities.yahoo.com/v/pb.html > ========================================================================= > Instruções para entrar na lista, sair da lista e usar a lista em > http://www.mat.puc-rio.br/~nicolau/olimp/obm-l.html > O administrador desta lista é > ========================================================================= ========================================================================= Instruções para entrar na lista, sair da lista e usar a lista em http://www.mat.puc-rio.br/~nicolau/olimp/obm-l.html O administrador desta lista é ========================================================================= From owner-obm-l@sucuri.mat.puc-rio.br Wed Sep 11 19:27:52 2002 Return-Path: Received: (from majordom@localhost) by sucuri.mat.puc-rio.br (8.9.3/8.9.3) id TAA28530 for obm-l-MTTP; Wed, 11 Sep 2002 19:27:15 -0300 Received: from shannon.bol.com.br (shannon.bol.com.br [200.221.24.13]) by sucuri.mat.puc-rio.br (8.9.3/8.9.3) with ESMTP id TAA28526 for ; Wed, 11 Sep 2002 19:27:13 -0300 Received: from bol.com.br (200.221.24.118) by shannon.bol.com.br (5.1.071) id 3D60E4DC00670CDA for obm-l@mat.puc-rio.br; Wed, 11 Sep 2002 19:28:03 -0300 Date: Wed, 11 Sep 2002 19:26:10 -0300 Message-Id: Subject: Re:[obm-l] circuito IME MIME-Version: 1.0 Content-Type: multipart/mixed; boundary="_=__=_XaM3_Boundary.1031783170.2A.86150.42.8806.52.42.101010.808428507" From: "rafaelc.l" To: obm-l@mat.puc-rio.br X-XaM3-API-Version: 2.4.3.4.4 X-SenderIP: 200.176.166.181 Sender: owner-obm-l@sucuri.mat.puc-rio.br Precedence: bulk Reply-To: obm-l@mat.puc-rio.br --_=__=_XaM3_Boundary.1031783170.2A.86150.42.8806.52.42.101010.808428507 Content-Type: text/plain;charset="iso-8859-1" Content-Transfer-Encoding: quoted-printable Me expressei mal adriano, claro que n=E3o devia ser anulada s=F3 pq n=E3o deram no ensino m=E9dio tradicional. Mas pq ser=E1 que n=E3o anularam? reclamaram mesmo dela? e o q a banca disse? __________________________________________________________________________ AcessoBOL, s=F3 R$ 9,90! O menor pre=E7o do mercado! Assine j=E1! http://www.bol.com.br/acessobol --_=__=_XaM3_Boundary.1031783170.2A.86150.42.8806.52.42.101010.808428507 Content-Type: text/plain; name="00000557" Content-Transfer-Encoding: base64 Y2xhcm8gcXVlIG5hbyBtZXUgY2FybyBSYWZhZWwsbmFvIGVoIHNvIA0KcG9ycXVlIG5hbyBl c3RhIHNlbmRvIGVuc2luYWRhIG5vIDK6Z3JhdSBxdWUgDQpkZXZlcmlhIHNlciBhbnVsYWRh LGVyYSBwb3JxdWUgbmFvIGVzdGF2YSBubyANCnByb2dyYW1hLg0KSG91dmVyYW0gbXVpdGFz IHJlY2xhbWEuIHBvciBjYXVzYSBkaXNzbyxtYXMgDQphIGdyYW5kZSBtYWlvcmlhIGRhcyBw ZXNzb2FzIHF1ZSBhY2VydGFyYW0gDQplaCBwb3JxdWUgZml6ZXJhbSAyuiBncmF1IHRlY25p Y28uDQpbXSdzLg0KQWRyaWFuby4NCg0KIA0KX19fX19fX19fX19fX19fX19fX19fX19fX19f X19fX19fX19fX19fX19fX19fX19fX19fX19fX19fX19fX19fX19fX19fX19fX18NCkFjZXNz b0JPTCwgc/MgUiQgOSw5MCEgTyBtZW5vciBwcmXnbyBkbyBtZXJjYWRvIQ0KQXNzaW5lIGrh ISBodHRwOi8vd3d3LmJvbC5jb20uYnIvYWNlc3NvYm9sDQoNCg0KPT09PT09PT09PT09PT09 PT09PT09PT09PT09PT09PT09PT09PT09PT09PT09PT09PT09PT09PT09PT09PT09PT09PT09 PT09PQ0KSW5zdHJ15/VlcyBwYXJhIGVudHJhciBuYSBsaXN0YSwgc2FpciBkYSBsaXN0YSBl IHVzYXIgYSBsaXN0YSBlbQ0KaHR0cDovL3d3dy5tYXQucHVjLXJpby5ici9+bmljb2xhdS9v bGltcC9vYm0tbC5odG1sDQpPIGFkbWluaXN0cmFkb3IgZGVzdGEgbGlzdGEg6SA8bmljb2xh dUBtYXQucHVjLXJpby5icj4NCj09PT09PT09PT09PT09PT09PT09PT09PT09PT09PT09PT09 PT09PT09PT09PT09PT09PT09PT09PT09PT09PT09PT09PT09PT0NCg== --_=__=_XaM3_Boundary.1031783170.2A.86150.42.8806.52.42.101010.808428507-- ========================================================================= Instruções para entrar na lista, sair da lista e usar a lista em http://www.mat.puc-rio.br/~nicolau/olimp/obm-l.html O administrador desta lista é ========================================================================= From owner-obm-l@sucuri.mat.puc-rio.br Wed Sep 11 19:57:09 2002 Return-Path: Received: (from majordom@localhost) by sucuri.mat.puc-rio.br (8.9.3/8.9.3) id TAA29077 for obm-l-MTTP; Wed, 11 Sep 2002 19:56:59 -0300 Received: from shannon.bol.com.br (shannon.bol.com.br [200.221.24.13]) by sucuri.mat.puc-rio.br (8.9.3/8.9.3) with ESMTP id TAA29072 for ; Wed, 11 Sep 2002 19:56:57 -0300 Received: from bol.com.br (200.221.24.118) by shannon.bol.com.br (5.1.071) id 3D60E4DC006727F4 for obm-l@mat.puc-rio.br; Wed, 11 Sep 2002 19:57:46 -0300 Date: Wed, 11 Sep 2002 19:55:53 -0300 Message-Id: Subject: Re: Re:[obm-l] circuito IME MIME-Version: 1.0 Content-Type: multipart/mixed; boundary="_=__=_XaM3_Boundary.1031784953.2A.505099.42.7587.52.42.101010.1849727196" From: "rafaelc.l" To: obm-l@mat.puc-rio.br X-XaM3-API-Version: 2.4.3.4.4 X-SenderIP: 200.176.166.181 Sender: owner-obm-l@sucuri.mat.puc-rio.br Precedence: bulk Reply-To: obm-l@mat.puc-rio.br --_=__=_XaM3_Boundary.1031784953.2A.505099.42.7587.52.42.101010.1849727196 Content-Type: text/plain;charset="iso-8859-1" Content-Transfer-Encoding: quoted-printable Daniel, tudo bem, mas eu acho errado eles n=E3o falarem do teorema de thevenin no programa e pedirem isto na prova. Para um cara que j=E1 fez f=EDsica 3 na faculdade, a quest=E3o seria simples. Se pelo que vc est=E1 dizendo, =E9 melhor eu estudar tipler para o IME. o ITA n=E3o divulga nem o gabarito das quest=F5es dissertativas? Voltando a quest=E3o do circuito,poder=EDamos tbm usar a f=F3rmula: q(t)=3D q.e^-t/RC ?? qual seria a resist=EAncia? __________________________________________________________________________ AcessoBOL, s=F3 R$ 9,90! O menor pre=E7o do mercado! Assine j=E1! http://www.bol.com.br/acessobol --_=__=_XaM3_Boundary.1031784953.2A.505099.42.7587.52.42.101010.1849727196 Content-Type: text/plain; name="00000I40" Content-Transfer-Encoding: base64 ICAgICAgICAgICAgICAgIFJhZmFlbCwgbyBJTUUgbuNvIGRpdnVsZ2EgZ2FiYXJpdG8gb2Zp Y2lhbCBlIG7jbyBkZXZlDQpleHBsaWNh5/VlcyBzb2JyZSBzdWFzIHByb3ZhcywgYXNzaW0g Y29tbyBvIElUQSwgYSBxdWVzdONvIGRvIGNpcmN1aXRvIHPzIGUNCnNvbWVudGUgcG9kZSBz ZXIgcmVzb2x2aWRhIGVtcHJlZ2FuZG8tc2UgbyB0ZW9yZW1hIGRlIHRoZXZlbmluIChwYXJh DQpzaW1wbGlmaWNhcikgZSBhcyBlcXVh5/VlcyBkbyByZWdpbWUgcmMgdHJhbnNpdPNyaW8s IG7jbyBlcmEgcHJlY2lzbw0KcmVhbGl6YXIgbmVuaHVtYSBpbnRlZ3JhbCwgYXBlbmFzIHRl ciBvcyByZXN1bHRhZG9zIHByb250b3MgcGFyYSB1c+EtbG9zIGNvbQ0Kb3MgZGFkb3MgZm9y bmVjaWRvcy4gU2UgdmMgdmFpIHByZXN0YXIgSU1FIOkgYm9tIHF1ZSB0ZW5oYSBlc3NlcyBk YWRvcyBlbQ0KbWVudGUsIGFsZ3VtYXMgcXVlc3T1ZXMgbuNvIHNlcuNvIHJlc29sdmlkYXMg Y29tIGNvbmhlY2ltZW50byBkbyBlbnNpbm8NCm3pZGlvLg0KDQogICAgICAgICAgICBEYW5p ZWwgTy4gQ29zdGENCg0KDQotLS0tLSBPcmlnaW5hbCBNZXNzYWdlIC0tLS0tDQpGcm9tOiBy YWZhZWxjLmwgPHJhZmFlbGMubEBib2wuY29tLmJyPg0KVG86IDxvYm0tbEBtYXQucHVjLXJp by5icj4NClNlbnQ6IE1vbmRheSwgU2VwdGVtYmVyIDA5LCAyMDAyIDY6MTcgUE0NClN1Ympl Y3Q6IFJlOltvYm0tbF0gY2lyY3VpdG8gSU1FDQoNCg0KPg0KPiAgIG1ldSBjYXJvIEFkcmlh bm86IHZjIG7jbyBhY2hhIHF1ZSBkZXZlIHRlciBhbGd1bWENCj4gc29sdefjbyBwYXJhIGVz c2EgcXVlc3TjbyBzZW0gbyB1c28gZGVzc2EgZXF1Yef1ZXM/DQo+IChwZW5zZSBiZW0gbmlz c28pLGFuYWxpc2UgbyBwcm9ibGVtYS4gU2UgbuNvIHRpdmVzc2UNCj4gYWxndW1hIHNvbHXn 428gc/MgY29tIGEgYWJvcmRhZ2VtIGRvIDIgZ3JhdSwgYSBxdWVzdONvDQo+IHRlcmlhIHNp ZG8gYW51bGFkYS4gUG91Y29zIGNhbmRpZGF0b3MgZGV2ZW0gdGVyIHJlc29sdmlkbw0KPiBl bGEgaW50ZXJncmFsbWVudGUuDQo+DQo+DQo+IF9fX19fX19fX19fX19fX19fX19fX19fX19f X19fX19fX19fX19fX19fX19fX19fX19fX19fX19fX19fX19fX19fX19fX19fX19fDQo+IEFj ZXNzb0JPTCwgc/MgUiQgOSw5MCEgTyBtZW5vciBwcmXnbyBkbyBtZXJjYWRvIQ0KPiBBc3Np bmUgauEhIGh0dHA6Ly93d3cuYm9sLmNvbS5ici9hY2Vzc29ib2wNCj4NCg0KDQo9PT09PT09 PT09PT09PT09PT09PT09PT09PT09PT09PT09PT09PT09PT09PT09PT09PT09PT09PT09PT09 PT09PT09PT09PT09DQpJbnN0cnXn9WVzIHBhcmEgZW50cmFyIG5hIGxpc3RhLCBzYWlyIGRh IGxpc3RhIGUgdXNhciBhIGxpc3RhIGVtDQpodHRwOi8vd3d3Lm1hdC5wdWMtcmlvLmJyL35u aWNvbGF1L29saW1wL29ibS1sLmh0bWwNCk8gYWRtaW5pc3RyYWRvciBkZXN0YSBsaXN0YSDp IDxuaWNvbGF1QG1hdC5wdWMtcmlvLmJyPg0KPT09PT09PT09PT09PT09PT09PT09PT09PT09 PT09PT09PT09PT09PT09PT09PT09PT09PT09PT09PT09PT09PT09PT09PT09PQ0K --_=__=_XaM3_Boundary.1031784953.2A.505099.42.7587.52.42.101010.1849727196-- ========================================================================= Instruções para entrar na lista, sair da lista e usar a lista em http://www.mat.puc-rio.br/~nicolau/olimp/obm-l.html O administrador desta lista é ========================================================================= From owner-obm-l@sucuri.mat.puc-rio.br Wed Sep 11 19:59:56 2002 Return-Path: Received: (from majordom@localhost) by sucuri.mat.puc-rio.br (8.9.3/8.9.3) id TAA29251 for obm-l-MTTP; Wed, 11 Sep 2002 19:59:53 -0300 Received: from silva5.uol.com.br (silva5.uol.com.br [200.221.4.52]) by sucuri.mat.puc-rio.br (8.9.3/8.9.3) with ESMTP id TAA29247 for ; Wed, 11 Sep 2002 19:59:51 -0300 Received: from slap ([200.207.152.104]) by silva5.uol.com.br (8.9.1/8.9.1) with SMTP id UAA19598 for ; Wed, 11 Sep 2002 20:03:20 -0300 (EST) Message-ID: <001701c259e7$dc356740$6898cfc8@slap> From: "Afemano" To: References: <000e01c259bd$4e76e160$2932a8c0@redeconora.com.br> Subject: Re: [obm-l] TESTE Date: Wed, 11 Sep 2002 20:06:26 -0300 MIME-Version: 1.0 Content-Type: text/plain; charset="iso-8859-1" Content-Transfer-Encoding: 8bit X-Priority: 3 X-MSMail-Priority: Normal X-Mailer: Microsoft Outlook Express 6.00.2600.0000 X-MimeOLE: Produced By Microsoft MimeOLE V6.00.2600.0000 Sender: owner-obm-l@sucuri.mat.puc-rio.br Precedence: bulk Reply-To: obm-l@mat.puc-rio.br Resposta : 200 Todos os números começam com a letra "D"... Falou !!! ----- Original Message ----- From: "Hely Jr." To: Sent: Wednesday, September 11, 2002 3:01 PM Subject: [obm-l] TESTE > Observem a sequência abaixo: > > 2 - 10 - 12 - 16 - 17 - 18 - 19 ...... > > E agora, respondam: qual o próximo número da sequência??? > > a) 20 > b) 22 > c) 175 > d) 200 > > Existe alguma pegadinha neste exercício? > > > ========================================================================= > Instruções para entrar na lista, sair da lista e usar a lista em > http://www.mat.puc-rio.br/~nicolau/olimp/obm-l.html > O administrador desta lista é > ========================================================================= ========================================================================= Instruções para entrar na lista, sair da lista e usar a lista em http://www.mat.puc-rio.br/~nicolau/olimp/obm-l.html O administrador desta lista é ========================================================================= From owner-obm-l@sucuri.mat.puc-rio.br Wed Sep 11 20:07:43 2002 Return-Path: Received: (from majordom@localhost) by sucuri.mat.puc-rio.br (8.9.3/8.9.3) id UAA29520 for obm-l-MTTP; Wed, 11 Sep 2002 20:07:26 -0300 Received: from silva5.uol.com.br (silva5.uol.com.br [200.221.4.52]) by sucuri.mat.puc-rio.br (8.9.3/8.9.3) with ESMTP id UAA29515 for ; Wed, 11 Sep 2002 20:07:23 -0300 Received: from slap ([200.207.152.104]) by silva5.uol.com.br (8.9.1/8.9.1) with SMTP id UAA25749 for ; Wed, 11 Sep 2002 20:10:53 -0300 (EST) Message-ID: <003c01c259e8$e9d79660$6898cfc8@slap> From: "Afemano" To: Subject: [obm-l] Geometria PLZ !!! Date: Wed, 11 Sep 2002 20:14:00 -0300 MIME-Version: 1.0 Content-Type: multipart/alternative; boundary="----=_NextPart_000_0039_01C259CF.C425A920" X-Priority: 3 X-MSMail-Priority: Normal X-Mailer: Microsoft Outlook Express 6.00.2600.0000 X-MimeOLE: Produced By Microsoft MimeOLE V6.00.2600.0000 Sender: owner-obm-l@sucuri.mat.puc-rio.br Precedence: bulk Reply-To: obm-l@mat.puc-rio.br This is a multi-part message in MIME format. ------=_NextPart_000_0039_01C259CF.C425A920 Content-Type: text/plain; charset="iso-8859-1" Content-Transfer-Encoding: quoted-printable Ol=E1 galera.. algu=E9m resolve esse exerc=EDcio pra mim plz ? ( FUVEST ) A, B e P s=E3o tr=EAs pontos de uma circunfer=EAncia de = centro O e raio unit=E1rio, base de um cone reto de v=E9rtice C e altura = igual a 2. A e B s=E3o diametralmente opostos e t =E9 a reta tangente = =E0 circunfer=EAncia pelo ponto B. Determine : a ) "Eu consequi,, deixa" b )a dist=E2ncia entre a reta t e a geratriz PC, em fun=E7=E3o do = =E2ngulo B=D4P. Obs : eu fiz v=E1rias vezes o desenho, e pelo que eu enxerguei ser=E3o = necess=E1rios tr=EAs tri=E2ngulos pra achar isso.. acho que a base = est=E1 na lei dos cossenos mas n=E3o tenho certeza. Entre a reta t e a = geratriz PC formar=E1 uma outra reta perpendiuclar as duas ( =E9 isso = que ele est=E1 pedindo n=E3o ? ), chame de ponto D onde essa reta = encontra a reta t e de ponto E onde essa reta encontra a geratriz... = s=F3 pra ficar mais f=E1cil. Nesse caso queremos achar o valor de DE, = obrigad=E3o !!!!!!!!!! ------=_NextPart_000_0039_01C259CF.C425A920 Content-Type: text/html; charset="iso-8859-1" Content-Transfer-Encoding: quoted-printable
Ol=E1 galera.. algu=E9m resolve esse = exerc=EDcio pra mim=20 plz ?
 
( FUVEST ) A, B e=20 P s=E3o tr=EAs pontos de uma circunfer=EAncia de centro = O e raio unit=E1rio, base de um cone reto de v=E9rtice=20 C e altura igual a 2. A e = B=20 s=E3o diametralmente opostos e t =E9 a reta tangente =E0 = circunfer=EAncia pelo=20 ponto B. Determine :
a ) "Eu consequi,, = deixa"
b )a dist=E2ncia entre a reta = t e a geratriz=20 PC, em fun=E7=E3o do =E2ngulo = B=D4P.
 
Obs : eu fiz v=E1rias vezes o desenho, = e pelo que eu=20 enxerguei ser=E3o necess=E1rios tr=EAs tri=E2ngulos pra achar isso.. = acho que a base=20 est=E1 na lei dos cossenos mas n=E3o tenho certeza. Entre a reta = t e a=20 geratriz PC formar=E1 uma outra reta perpendiuclar as = duas ( =E9=20 isso que ele est=E1 pedindo n=E3o ? ), chame de ponto D = onde essa=20 reta encontra a reta t e de ponto E onde essa = reta=20 encontra a geratriz... s=F3 pra ficar mais f=E1cil. Nesse caso queremos = achar o=20 valor de DE, obrigad=E3o = !!!!!!!!!!
------=_NextPart_000_0039_01C259CF.C425A920-- ========================================================================= Instruções para entrar na lista, sair da lista e usar a lista em http://www.mat.puc-rio.br/~nicolau/olimp/obm-l.html O administrador desta lista é ========================================================================= From owner-obm-l@sucuri.mat.puc-rio.br Wed Sep 11 22:24:30 2002 Return-Path: Received: (from majordom@localhost) by sucuri.mat.puc-rio.br (8.9.3/8.9.3) id WAA31958 for obm-l-MTTP; Wed, 11 Sep 2002 22:23:49 -0300 Received: from puma.unisys.com.br (ns2.unisys.com.br [200.220.64.7]) by sucuri.mat.puc-rio.br (8.9.3/8.9.3) with ESMTP id WAA31954 for ; Wed, 11 Sep 2002 22:23:46 -0300 Received: from jf (riopm18p160.uninet.com.br [200.220.16.160]) by puma.unisys.com.br (8.12.3/8.12.3) with SMTP id g8C1OojR016276 for ; Wed, 11 Sep 2002 22:24:51 -0300 (EST) X-Spam-Filter: check_local@puma.unisys.com.br by digitalanswers.org Message-ID: <001401c259fb$e4d62ce0$a010dcc8@jf> From: "Jose Francisco Guimaraes Costa" To: References: <000e01c259bd$4e76e160$2932a8c0@redeconora.com.br> <20020911150519.C17567@sucuri.mat.puc-rio.br> <00bd01c259cb$693f8980$dd02dcc8@josefran> Subject: Re: [obm-l] TESTE Date: Wed, 11 Sep 2002 22:27:28 -0300 MIME-Version: 1.0 Content-Type: text/plain; charset="iso-8859-1" Content-Transfer-Encoding: 8bit X-Priority: 3 X-MSMail-Priority: Normal X-Mailer: Microsoft Outlook Express 6.00.2600.0000 X-MimeOLE: Produced By Microsoft MimeOLE V6.00.2600.0000 Sender: owner-obm-l@sucuri.mat.puc-rio.br Precedence: bulk Reply-To: obm-l@mat.puc-rio.br Descobri porque minha neta, que está na classe de alfabetização aqui no Brasil e que portanto, segundo o N, deveria resolver o problema com facilidade, não o resolveu. É fato conhecido por educadores e psicólogos - e ambos têm lindas teorias para isso - que não interessa quão distante alguém fica de sua lingua nativa, isto é, a lingua em que aprendeu a falar, para contar e fazer contas só ela é utilizada. É como se a unidade aritmética do cérebro não pudesse ser reprogramada. Para minha neta, 16 é "seize", e seize não começa com "D". JF ----- Original Message ----- From: "Afemano" To: Sent: Wednesday, September 11, 2002 8:06 PM Subject: Re: [obm-l] TESTE > Resposta : > 200 > Todos os números começam com a letra "D"... > > Falou !!! > ----- Original Message ----- From: "Jose Francisco Guimaraes Costa" To: Sent: Wednesday, September 11, 2002 4:42 PM Subject: Re: [obm-l] TESTE > Passei o problema para minha neta, que está na turma de alfabetização (no > Brasil, embora ela tenha nascido em Lyon, cidade onde o N já trabalhou), mas > ela não conseguiu resolver. > > O problema não é difícil apenas para matemáticos. Também é para engenheiros, > como eu. > > Resumindo - qual a solução? > > JF > > -----Mensagem Original----- > De: Nicolau C. Saldanha > Para: > Enviada em: Quarta-feira, 11 de Setembro de 2002 15:05 > Assunto: Re: [obm-l] TESTE > > > > On Wed, Sep 11, 2002 at 03:01:50PM -0300, Hely Jr. wrote: > > > Observem a sequência abaixo: > > > > > > 2 - 10 - 12 - 16 - 17 - 18 - 19 ...... > > > > > > E agora, respondam: qual o próximo número da sequência??? > > > > > > a) 20 > > > b) 22 > > > c) 175 > > > d) 200 > > > > > > Existe alguma pegadinha neste exercício? > > > > Existe. Este problema é fácil para turmas de alfabetização (no Brasil!) > > mas difícil para matemáticos... ;-) > > > > []s, N. > > ========================================================================= Instruções para entrar na lista, sair da lista e usar a lista em http://www.mat.puc-rio.br/~nicolau/olimp/obm-l.html O administrador desta lista é ========================================================================= From owner-obm-l@sucuri.mat.puc-rio.br Wed Sep 11 23:00:58 2002 Return-Path: Received: (from majordom@localhost) by sucuri.mat.puc-rio.br (8.9.3/8.9.3) id XAA32755 for obm-l-MTTP; Wed, 11 Sep 2002 23:00:43 -0300 Received: from mediterraneo.rjnet.com.br (mediterraneo.rjnet.com.br [200.222.31.30]) by sucuri.mat.puc-rio.br (8.9.3/8.9.3) with ESMTP id XAA32751 for ; Wed, 11 Sep 2002 23:00:40 -0300 Received: from locutus.rjnet.com.br (root@locutus.rjnet.com.br [200.222.31.10]) by mediterraneo.rjnet.com.br (8.11.4/8.11.4) with ESMTP id g8C22JC19976 for ; Wed, 11 Sep 2002 23:02:19 -0300 Received: from felipe ([200.222.39.86]) by locutus.rjnet.com.br (8.11.2/8.11.2) with SMTP id g8C1gI204391 for ; Wed, 11 Sep 2002 22:42:18 -0300 Message-ID: <000e01c25a00$542388f0$158c000a@felipe> From: "Felipe Villela Dias" To: References: <000e01c259bd$4e76e160$2932a8c0@redeconora.com.br> <20020911150519.C17567@sucuri.mat.puc-rio.br> <00bd01c259cb$693f8980$dd02dcc8@josefran> <001401c259fb$e4d62ce0$a010dcc8@jf> Subject: Re: [obm-l] TESTE Date: Wed, 11 Sep 2002 23:01:37 -0300 MIME-Version: 1.0 Content-Type: text/plain; charset="iso-8859-1" Content-Transfer-Encoding: 8bit X-Priority: 3 X-MSMail-Priority: Normal X-Mailer: Microsoft Outlook Express 6.00.2600.0000 X-MimeOLE: Produced By Microsoft MimeOLE V6.00.2600.0000 Sender: owner-obm-l@sucuri.mat.puc-rio.br Precedence: bulk Reply-To: obm-l@mat.puc-rio.br Acho que toda essa discussão é inútil... já tinha essa questão não passa de uma brincadeira que as pessoas repassam via -e-mail. Há muito tempo recebi esse e-mail e tinha como resposta correta 200, pq começa com D. É uma brincadeira, não uma questão matemática. ----- Original Message ----- From: "Jose Francisco Guimaraes Costa" To: Sent: Wednesday, September 11, 2002 10:27 PM Subject: Re: [obm-l] TESTE > Descobri porque minha neta, que está na classe de alfabetização aqui no > Brasil e que portanto, segundo o N, deveria resolver o problema com > facilidade, não o resolveu. > > É fato conhecido por educadores e psicólogos - e ambos têm lindas teorias > para isso - que não interessa quão distante alguém fica de sua lingua > nativa, isto é, a lingua em que aprendeu a falar, para contar e fazer contas > só ela é utilizada. É como se a unidade aritmética do cérebro não pudesse > ser reprogramada. > > Para minha neta, 16 é "seize", e seize não começa com "D". > > JF > > ----- Original Message ----- > From: "Afemano" > To: > Sent: Wednesday, September 11, 2002 8:06 PM > Subject: Re: [obm-l] TESTE > > > > Resposta : > > 200 > > Todos os números começam com a letra "D"... > > > > Falou !!! > > > > ----- Original Message ----- > From: "Jose Francisco Guimaraes Costa" > To: > Sent: Wednesday, September 11, 2002 4:42 PM > Subject: Re: [obm-l] TESTE > > > > Passei o problema para minha neta, que está na turma de alfabetização (no > > Brasil, embora ela tenha nascido em Lyon, cidade onde o N já trabalhou), > mas > > ela não conseguiu resolver. > > > > O problema não é difícil apenas para matemáticos. Também é para > engenheiros, > > como eu. > > > > Resumindo - qual a solução? > > > > JF > > > > -----Mensagem Original----- > > De: Nicolau C. Saldanha > > Para: > > Enviada em: Quarta-feira, 11 de Setembro de 2002 15:05 > > Assunto: Re: [obm-l] TESTE > > > > > > > On Wed, Sep 11, 2002 at 03:01:50PM -0300, Hely Jr. wrote: > > > > Observem a sequência abaixo: > > > > > > > > 2 - 10 - 12 - 16 - 17 - 18 - 19 ...... > > > > > > > > E agora, respondam: qual o próximo número da sequência??? > > > > > > > > a) 20 > > > > b) 22 > > > > c) 175 > > > > d) 200 > > > > > > > > Existe alguma pegadinha neste exercício? > > > > > > Existe. Este problema é fácil para turmas de alfabetização (no Brasil!) > > > mas difícil para matemáticos... ;-) > > > > > > []s, N. > > > > > > ========================================================================= > Instruções para entrar na lista, sair da lista e usar a lista em > http://www.mat.puc-rio.br/~nicolau/olimp/obm-l.html > O administrador desta lista é > ========================================================================= > --- Outgoing mail is certified Virus Free. Checked by AVG anti-virus system (http://www.grisoft.com). Version: 6.0.386 / Virus Database: 218 - Release Date: 9/9/2002 ========================================================================= Instruções para entrar na lista, sair da lista e usar a lista em http://www.mat.puc-rio.br/~nicolau/olimp/obm-l.html O administrador desta lista é ========================================================================= From owner-obm-l@sucuri.mat.puc-rio.br Wed Sep 11 23:20:20 2002 Return-Path: Received: (from majordom@localhost) by sucuri.mat.puc-rio.br (8.9.3/8.9.3) id XAA00857 for obm-l-MTTP; Wed, 11 Sep 2002 23:20:09 -0300 Received: from mtasjc.directnet.com.br (mtasjc.directnet.com.br [200.152.0.15]) by sucuri.mat.puc-rio.br (8.9.3/8.9.3) with ESMTP id XAA00853 for ; Wed, 11 Sep 2002 23:20:06 -0300 Received: from meu ([200.152.17.12]) by pop.directnet.com.br (iPlanet Messaging Server 5.1 HotFix 1.3 (built Jul 16 2002)) with SMTP id <0H2A009QYZQJNF@pop.directnet.com.br> for obm-l@mat.puc-rio.br; Wed, 11 Sep 2002 23:18:23 -0300 (EST) Date: Wed, 11 Sep 2002 23:36:53 -0300 From: Daniel Subject: Re: Re:[obm-l] circuito IME To: obm-l@mat.puc-rio.br Message-id: <005c01c25a05$41a0f8c0$0c1198c8@directnet.com.br> MIME-version: 1.0 X-MIMEOLE: Produced By Microsoft MimeOLE V5.00.2615.200 X-Mailer: Microsoft Outlook Express 5.00.2615.200 Content-type: text/plain; charset=iso-8859-1 Content-transfer-encoding: 8BIT X-Priority: 3 X-MSMail-priority: Normal References: Sender: owner-obm-l@sucuri.mat.puc-rio.br Precedence: bulk Reply-To: obm-l@mat.puc-rio.br Use a resistência thevenin equivalente, pra vc ver acho que para este ano deveremos estudar circuitos RLC para o IME, em todo caso boa sorte para nós!!! Daniel O. Costa OBS: O ITA não divulga gabarito nenhum!!!! ----- Original Message ----- From: rafaelc.l To: Sent: Wednesday, September 11, 2002 7:55 PM Subject: Re: Re:[obm-l] circuito IME > > > Daniel, tudo bem, mas eu acho errado eles não falarem do > teorema de thevenin no programa e pedirem isto na prova. > Para um cara que já fez física 3 na faculdade, a questão > seria simples. Se pelo que vc está dizendo, é melhor eu > estudar tipler para o IME. > o ITA não divulga nem o gabarito das questões > dissertativas? > Voltando a questão do circuito,poderíamos tbm usar a > fórmula: q(t)= q.e^-t/RC ?? qual seria a resistência? > > > __________________________________________________________________________ > AcessoBOL, só R$ 9,90! O menor preço do mercado! > Assine já! http://www.bol.com.br/acessobol > ========================================================================= Instruções para entrar na lista, sair da lista e usar a lista em http://www.mat.puc-rio.br/~nicolau/olimp/obm-l.html O administrador desta lista é ========================================================================= From owner-obm-l@sucuri.mat.puc-rio.br Thu Sep 12 00:13:17 2002 Return-Path: Received: (from majordom@localhost) by sucuri.mat.puc-rio.br (8.9.3/8.9.3) id AAA02120 for obm-l-MTTP; Thu, 12 Sep 2002 00:12:57 -0300 Received: from smtp-33.ig.com.br (smtp-33.ig.com.br [200.226.132.183]) by sucuri.mat.puc-rio.br (8.9.3/8.9.3) with SMTP id AAA02116 for ; Thu, 12 Sep 2002 00:12:55 -0300 From: ezer@ig.com.br Received: (qmail 12133 invoked from network); 12 Sep 2002 03:13:29 -0000 Received: from shasta060254.ig.com.br (HELO almirfam) (200.151.60.254) by smtp-33.ig.com.br with SMTP; 12 Sep 2002 03:13:29 -0000 To: obm-l@mat.puc-rio.br Date: Wed, 1 Jan 1997 02:20:31 -0200 MIME-Version: 1.0 Subject: =?ISO-8859-1?Q?Re:_[obm-l]_N=FAmeros_rand=F4micos?= Message-ID: <32C9C9EF.24726.8051B4@localhost> In-reply-to: X-mailer: Pegasus Mail for Windows (v4.01) Content-type: text/plain; charset=ISO-8859-1 Content-description: Mail message body Content-Transfer-Encoding: 8bit X-MIME-Autoconverted: from Quoted-printable to 8bit by sucuri.mat.puc-rio.br id AAA02117 Sender: owner-obm-l@sucuri.mat.puc-rio.br Precedence: bulk Reply-To: obm-l@mat.puc-rio.br Ola Victor e colegas da lista, Estava pensando na questao dos numeros aleatorios, e acho que cheguei a uma conclusao satisfatoria. Em primeiro lugar, temos que definir aleatoriedade. A definicao de dicionario é a de "algo que eh sujeito ao acaso". Nao achei essa definicao interessante, por que aih teriamos de definir acaso, e a discussao entraria por um caminho um tanto "mistico". Entao pensei nessa definicao: Aleatoriedade: Eh a caracteristica de algo que nao pode ter sua condicao determinada por um observador ideal. Observador Ideal: Eh um observador hipotetico, que 1) teria a capacidade de saber todas as condições que influenciam um evento, mesmo que tais condições sejam infinitas, e 2) sempre soubesse como essas condições afetam o evento e determinam a sua condição. Seguindo esta definicao, veremos que nada eh "realmente" aleatorio, pois mesmo que existam infinitos eventos e condições que modifiquem o resultado de algum evento, o nosso observador pode inferir corretamente o resultado. Mas como nao existe tal observador ideal, podemos alterar a definicao e dizer que algo eh aleatorio quando nao pode ter sua condicao determinada por nada nem ninguem sobre a face da Terra (ou alem da Terra ;c)). Um exemplo, disso: Pegue um copo com agua. Coloque um pouquinho de oleo (digamos, uma colher de cha). Agora, de uma sacudida no copo. Quantas "bolinhas" de oleo se formarao? Como determinar isso? Isso depende de infinitas configuracoes: da intensidade da sacudida, da geometria do movimento feito para sacudir, da quantidade de agua e oleo, da temperatura... enfim, apesar de algumas condicoes serem mais significativas (por exemplo, quanto maior a intensidade da sacudidela, maior o numero de "bolinhas" de oleo), todas modificam o resultado, e pior, de forma que a interacao das condicoes nao se dah de forma linear, importando sim a ordem de modificacoes ao se tentar descobrir o resultado. Quem conseguirah determinar o resultado? Estah obvio que um homem nao. Declaro que um computador mil vezes mais rapido que o mais rapido existente passaria longe de determinar esse resultado, mesmo em um trilhao de anos. Entao podemos dizer que esse eh um evento aleatorio. O mundo real estah cheio de eventos aleatorios (por exemplo, o sorteio feito para escolher os numeros da loteria). Mas sao eventos que dependem de infinitas variacoes. Agora, como fazer isso no computador? Com certeza nao podemos depender de infinitas variacoes, nem ao menos de um numero satisfatorio de condicoes. Tal coisa exigiria muito mais tempo do que temos. Logo, nao existem resultados aleatorios gerados por um computador. O que os computadores fazem eh "simular" resultados aleatorios, buscando solucoes que sejam satisfatorias em tempo habil. Nesse caso, o problema se reduz a encontrar "fontes" de dados altamente variaveis e que nao mostrem nenhum vicio nem tendencia a apresentar certos resultados. Por exemplo, eles podem "pegar" os milesimos de segundos do momento e trabalhar com isso para apresentar um resultado satisfatorio, ou pegar uma parte qualquer da memoria e usar os bits para determinar um resultado conveniente. Nao estou informado quanto a isto, mas pelo que sei o uso do relogio do computador eh uma das formas mais eficientes para essa simulacao, por apresentar uma chance "igual" de escolha entre os numeros e por ter um ciclo muito rapido (no caso dos milesimos..) Esse eh um assunto deveras interessante, e espero ter contribuido para a discussao Um abraco a todos, Ezer Fernandes On 8 Sep 2002 at 7:51, e isso mesmo wrote: > Amigos, alguém poderia me explicar detalhadamente o que são números randômicos e com se > constrói esses números? Li isso em algum lugar sobre loterias. > Obrigado > Víctor > ========================================================================= Instruções para entrar na lista, sair da lista e usar a lista em http://www.mat.puc-rio.br/~nicolau/olimp/obm-l.html O administrador desta lista é ========================================================================= From owner-obm-l@sucuri.mat.puc-rio.br Thu Sep 12 01:52:27 2002 Return-Path: Received: (from majordom@localhost) by sucuri.mat.puc-rio.br (8.9.3/8.9.3) id BAA03343 for obm-l-MTTP; Thu, 12 Sep 2002 01:50:31 -0300 Received: from shannon.bol.com.br (shannon.bol.com.br [200.221.24.13]) by sucuri.mat.puc-rio.br (8.9.3/8.9.3) with ESMTP id BAA03339 for ; Thu, 12 Sep 2002 01:50:29 -0300 Received: from bol.com.br (200.221.24.127) by shannon.bol.com.br (5.1.071) id 3D60E4DC00684FA2 for obm-l@mat.puc-rio.br; Thu, 12 Sep 2002 01:51:19 -0300 Date: Thu, 12 Sep 2002 01:49:39 -0300 Message-Id: Subject: Re: Re:[obm-l] circuito IME MIME-Version: 1.0 Content-Type: multipart/mixed; boundary="_=__=_XaM3_Boundary.1031806179.2A.464124.42.29169.52.42.101010.1882322412" From: "rafaelc.l" To: obm-l@mat.puc-rio.br X-XaM3-API-Version: 2.4.3.4.4 X-SenderIP: 172.187.87.125 Sender: owner-obm-l@sucuri.mat.puc-rio.br Precedence: bulk Reply-To: obm-l@mat.puc-rio.br --_=__=_XaM3_Boundary.1031806179.2A.464124.42.29169.52.42.101010.1882322412 Content-Type: text/plain;charset="iso-8859-1" Content-Transfer-Encoding: quoted-printable O que seriam circuitos RLC? __________________________________________________________________________ AcessoBOL, s=F3 R$ 9,90! O menor pre=E7o do mercado! Assine j=E1! http://www.bol.com.br/acessobol --_=__=_XaM3_Boundary.1031806179.2A.464124.42.29169.52.42.101010.1882322412 Content-Type: text/plain; name="00000MFC" Content-Transfer-Encoding: base64 ICAgICAgICAgICAgICAgICAgICBVc2UgYSByZXNpc3TqbmNpYSB0aGV2ZW5pbiBlcXVpdmFs ZW50ZSwgcHJhIHZjIHZlciBhY2hvDQpxdWUgcGFyYSBlc3RlIGFubyBkZXZlcmVtb3MgZXN0 dWRhciBjaXJjdWl0b3MgUkxDIHBhcmEgbyBJTUUsIGVtIHRvZG8gY2Fzbw0KYm9hIHNvcnRl IHBhcmEgbvNzISEhDQoNCiAgICAgICAgICAgICAgICBEYW5pZWwgTy4gQ29zdGENCk9CUzog TyBJVEEgbuNvIGRpdnVsZ2EgZ2FiYXJpdG8gbmVuaHVtISEhIQ0KLS0tLS0gT3JpZ2luYWwg TWVzc2FnZSAtLS0tLQ0KRnJvbTogcmFmYWVsYy5sIDxyYWZhZWxjLmxAYm9sLmNvbS5icj4N ClRvOiA8b2JtLWxAbWF0LnB1Yy1yaW8uYnI+DQpTZW50OiBXZWRuZXNkYXksIFNlcHRlbWJl ciAxMSwgMjAwMiA3OjU1IFBNDQpTdWJqZWN0OiBSZTogUmU6W29ibS1sXSBjaXJjdWl0byBJ TUUNCg0KDQo+DQo+DQo+ICBEYW5pZWwsIHR1ZG8gYmVtLCBtYXMgZXUgYWNobyBlcnJhZG8g ZWxlcyBu428gZmFsYXJlbSBkbw0KPiB0ZW9yZW1hIGRlIHRoZXZlbmluIG5vIHByb2dyYW1h IGUgcGVkaXJlbSBpc3RvIG5hIHByb3ZhLg0KPiBQYXJhIHVtIGNhcmEgcXVlIGrhIGZleiBm 7XNpY2EgMyBuYSBmYWN1bGRhZGUsIGEgcXVlc3Tjbw0KPiBzZXJpYSBzaW1wbGVzLiBTZSBw ZWxvIHF1ZSB2YyBlc3ThIGRpemVuZG8sIOkgbWVsaG9yIGV1DQo+IGVzdHVkYXIgdGlwbGVy IHBhcmEgbyBJTUUuDQo+ICBvIElUQSBu428gZGl2dWxnYSBuZW0gbyBnYWJhcml0byBkYXMg cXVlc3T1ZXMNCj4gZGlzc2VydGF0aXZhcz8NCj4gIFZvbHRhbmRvIGEgcXVlc3TjbyBkbyBj aXJjdWl0byxwb2Rlcu1hbW9zIHRibSB1c2FyIGENCj4gZvNybXVsYTogcSh0KT0gcS5lXi10 L1JDID8/IHF1YWwgc2VyaWEgYSByZXNpc3TqbmNpYT8NCj4NCj4NCj4gX19fX19fX19fX19f X19fX19fX19fX19fX19fX19fX19fX19fX19fX19fX19fX19fX19fX19fX19fX19fX19fX19f X19fX19fX18NCj4gQWNlc3NvQk9MLCBz8yBSJCA5LDkwISBPIG1lbm9yIHByZedvIGRvIG1l cmNhZG8hDQo+IEFzc2luZSBq4SEgaHR0cDovL3d3dy5ib2wuY29tLmJyL2FjZXNzb2JvbA0K Pg0KDQoNCj09PT09PT09PT09PT09PT09PT09PT09PT09PT09PT09PT09PT09PT09PT09PT09 PT09PT09PT09PT09PT09PT09PT09PT09PT0NCkluc3Rydef1ZXMgcGFyYSBlbnRyYXIgbmEg bGlzdGEsIHNhaXIgZGEgbGlzdGEgZSB1c2FyIGEgbGlzdGEgZW0NCmh0dHA6Ly93d3cubWF0 LnB1Yy1yaW8uYnIvfm5pY29sYXUvb2xpbXAvb2JtLWwuaHRtbA0KTyBhZG1pbmlzdHJhZG9y IGRlc3RhIGxpc3RhIOkgPG5pY29sYXVAbWF0LnB1Yy1yaW8uYnI+DQo9PT09PT09PT09PT09 PT09PT09PT09PT09PT09PT09PT09PT09PT09PT09PT09PT09PT09PT09PT09PT09PT09PT09 PT09PT09DQo= --_=__=_XaM3_Boundary.1031806179.2A.464124.42.29169.52.42.101010.1882322412-- ========================================================================= Instruções para entrar na lista, sair da lista e usar a lista em http://www.mat.puc-rio.br/~nicolau/olimp/obm-l.html O administrador desta lista é ========================================================================= From owner-obm-l@sucuri.mat.puc-rio.br Thu Sep 12 07:44:43 2002 Return-Path: Received: (from majordom@localhost) by sucuri.mat.puc-rio.br (8.9.3/8.9.3) id HAA06195 for obm-l-MTTP; Thu, 12 Sep 2002 07:43:12 -0300 Received: from hotmail.com ([216.33.149.58]) by sucuri.mat.puc-rio.br (8.9.3/8.9.3) with ESMTP id HAA06191 for ; Thu, 12 Sep 2002 07:43:08 -0300 Received: from mail pickup service by hotmail.com with Microsoft SMTPSVC; Thu, 12 Sep 2002 03:43:39 -0700 Received: from 200.194.208.222 by lw4fd.law4.hotmail.msn.com with HTTP; Thu, 12 Sep 2002 10:43:39 GMT X-Originating-IP: [200.194.208.222] From: "Laurito Alves" To: obm-l@mat.puc-rio.br Subject: Re: [obm-l] TESTE Date: Thu, 12 Sep 2002 10:43:39 +0000 Mime-Version: 1.0 Content-Type: text/plain; charset=iso-8859-1; format=flowed Message-ID: X-OriginalArrivalTime: 12 Sep 2002 10:43:39.0957 (UTC) FILETIME=[419AC650:01C25A49] Sender: owner-obm-l@sucuri.mat.puc-rio.br Precedence: bulk Reply-To: obm-l@mat.puc-rio.br Felipe, Discordo de vocÊ. Essa é uma questão matemática cuja resposta você aceita que é 200 pois lhe é apresentada uma lei de formação da sequencia que você supõe simples e plausível. Matematicamente falando qualquer alternativa apresentada, na realidade qualquer outro número, poderia ser o próximo termo da sequencia e é possível apresentar a lei de formação que comprove isso. Laurito >From: "Felipe Villela Dias" >Reply-To: obm-l@mat.puc-rio.br >To: >Subject: Re: [obm-l] TESTE >Date: Wed, 11 Sep 2002 23:01:37 -0300 > >Acho que toda essa discussão é inútil... já tinha essa questão não passa de >uma brincadeira que as pessoas repassam via -e-mail. Há muito tempo recebi >esse e-mail e tinha como resposta correta 200, pq começa com D. É uma >brincadeira, não uma questão matemática. >----- Original Message ----- >From: "Jose Francisco Guimaraes Costa" >To: >Sent: Wednesday, September 11, 2002 10:27 PM >Subject: Re: [obm-l] TESTE > > > > Descobri porque minha neta, que está na classe de alfabetização aqui no > > Brasil e que portanto, segundo o N, deveria resolver o problema com > > facilidade, não o resolveu. > > > > É fato conhecido por educadores e psicólogos - e ambos têm lindas >teorias > > para isso - que não interessa quão distante alguém fica de sua lingua > > nativa, isto é, a lingua em que aprendeu a falar, para contar e fazer >contas > > só ela é utilizada. É como se a unidade aritmética do cérebro não >pudesse > > ser reprogramada. > > > > Para minha neta, 16 é "seize", e seize não começa com "D". > > > > JF > > > > ----- Original Message ----- > > From: "Afemano" > > To: > > Sent: Wednesday, September 11, 2002 8:06 PM > > Subject: Re: [obm-l] TESTE > > > > > > > Resposta : > > > 200 > > > Todos os números começam com a letra "D"... > > > > > > Falou !!! > > > > > > > ----- Original Message ----- > > From: "Jose Francisco Guimaraes Costa" > > To: > > Sent: Wednesday, September 11, 2002 4:42 PM > > Subject: Re: [obm-l] TESTE > > > > > > > Passei o problema para minha neta, que está na turma de alfabetização >(no > > > Brasil, embora ela tenha nascido em Lyon, cidade onde o N já >trabalhou), > > mas > > > ela não conseguiu resolver. > > > > > > O problema não é difícil apenas para matemáticos. Também é para > > engenheiros, > > > como eu. > > > > > > Resumindo - qual a solução? > > > > > > JF > > > > > > -----Mensagem Original----- > > > De: Nicolau C. Saldanha > > > Para: > > > Enviada em: Quarta-feira, 11 de Setembro de 2002 15:05 > > > Assunto: Re: [obm-l] TESTE > > > > > > > > > > On Wed, Sep 11, 2002 at 03:01:50PM -0300, Hely Jr. wrote: > > > > > Observem a sequência abaixo: > > > > > > > > > > 2 - 10 - 12 - 16 - 17 - 18 - 19 ...... > > > > > > > > > > E agora, respondam: qual o próximo número da sequência??? > > > > > > > > > > a) 20 > > > > > b) 22 > > > > > c) 175 > > > > > d) 200 > > > > > > > > > > Existe alguma pegadinha neste exercício? > > > > > > > > Existe. Este problema é fácil para turmas de alfabetização (no >Brasil!) > > > > mas difícil para matemáticos... ;-) > > > > > > > > []s, N. > > > > > > > > > > >========================================================================= > > Instruções para entrar na lista, sair da lista e usar a lista em > > http://www.mat.puc-rio.br/~nicolau/olimp/obm-l.html > > O administrador desta lista é > > >========================================================================= > > > > >--- >Outgoing mail is certified Virus Free. >Checked by AVG anti-virus system (http://www.grisoft.com). >Version: 6.0.386 / Virus Database: 218 - Release Date: 9/9/2002 > >========================================================================= >Instruções para entrar na lista, sair da lista e usar a lista em >http://www.mat.puc-rio.br/~nicolau/olimp/obm-l.html >O administrador desta lista é >========================================================================= _________________________________________________________________ Converse com seus amigos online, faça o download grátis do MSN Messenger: http://messenger.msn.com.br ========================================================================= Instruções para entrar na lista, sair da lista e usar a lista em http://www.mat.puc-rio.br/~nicolau/olimp/obm-l.html O administrador desta lista é ========================================================================= From owner-obm-l@sucuri.mat.puc-rio.br Thu Sep 12 08:56:41 2002 Return-Path: Received: (from majordom@localhost) by sucuri.mat.puc-rio.br (8.9.3/8.9.3) id IAA07423 for obm-l-MTTP; Thu, 12 Sep 2002 08:56:20 -0300 Received: from mta2sjc.directnet.com.br (mta2sjc.directnet.com.br [200.152.0.13]) by sucuri.mat.puc-rio.br (8.9.3/8.9.3) with ESMTP id IAA07419 for ; Thu, 12 Sep 2002 08:56:15 -0300 Received: from meu ([200.152.17.12]) by pop.directnet.com.br (iPlanet Messaging Server 5.1 HotFix 1.3 (built Jul 16 2002)) with SMTP id <0H2B00M1LQILXG@pop.directnet.com.br> for obm-l@mat.puc-rio.br; Thu, 12 Sep 2002 08:56:51 -0300 (EST) Date: Thu, 12 Sep 2002 09:13:04 -0300 From: Daniel Subject: Re: Re:[obm-l] circuito IME To: obm-l@mat.puc-rio.br Message-id: <002601c25a55$c0659d00$0c1198c8@directnet.com.br> MIME-version: 1.0 X-MIMEOLE: Produced By Microsoft MimeOLE V5.00.2615.200 X-Mailer: Microsoft Outlook Express 5.00.2615.200 Content-type: text/plain; charset=iso-8859-1 Content-transfer-encoding: 8BIT X-Priority: 3 X-MSMail-priority: Normal References: Sender: owner-obm-l@sucuri.mat.puc-rio.br Precedence: bulk Reply-To: obm-l@mat.puc-rio.br Resistor, capacitor, indutor, ma é um tiro no escruo, isso só é visto na faculdade, ou em cursinhos especializados, se vc tiver um professor bom e com paciência peça pra ele te explicar, mas a chance de cair é bem remota. Se tiver mais dúvidas é melhor usar meu e-mail pessoal e não a lista. Daniel O. Costa costafdaniel@ig.com.br ----- Original Message ----- From: rafaelc.l To: Sent: Thursday, September 12, 2002 1:49 AM Subject: Re: Re:[obm-l] circuito IME > > > O que seriam circuitos RLC? > > > __________________________________________________________________________ > AcessoBOL, só R$ 9,90! O menor preço do mercado! > Assine já! http://www.bol.com.br/acessobol > ========================================================================= Instruções para entrar na lista, sair da lista e usar a lista em http://www.mat.puc-rio.br/~nicolau/olimp/obm-l.html O administrador desta lista é ========================================================================= From owner-obm-l@sucuri.mat.puc-rio.br Thu Sep 12 10:13:13 2002 Return-Path: Received: (from majordom@localhost) by sucuri.mat.puc-rio.br (8.9.3/8.9.3) id KAA08887 for obm-l-MTTP; Thu, 12 Sep 2002 10:12:13 -0300 Received: from mediterraneo.rjnet.com.br (mediterraneo.rjnet.com.br [200.222.31.30]) by sucuri.mat.puc-rio.br (8.9.3/8.9.3) with ESMTP id KAA08883 for ; Thu, 12 Sep 2002 10:12:08 -0300 Received: from locutus.rjnet.com.br (root@locutus.rjnet.com.br [200.222.31.10]) by mediterraneo.rjnet.com.br (8.11.4/8.11.4) with ESMTP id g8CDDlC06933 for ; Thu, 12 Sep 2002 10:13:47 -0300 Received: from felipe ([200.222.39.86]) by locutus.rjnet.com.br (8.11.2/8.11.2) with SMTP id g8CCri218382 for ; Thu, 12 Sep 2002 09:53:44 -0300 Message-ID: <000a01c25a5e$22d2c820$158c000a@felipe> From: "Felipe Villela Dias" To: References: Subject: Re: [obm-l] TESTE Date: Thu, 12 Sep 2002 10:13:07 -0300 MIME-Version: 1.0 Content-Type: text/plain; charset="iso-8859-1" Content-Transfer-Encoding: 8bit X-Priority: 3 X-MSMail-Priority: Normal X-Mailer: Microsoft Outlook Express 6.00.2600.0000 X-MimeOLE: Produced By Microsoft MimeOLE V6.00.2600.0000 Sender: owner-obm-l@sucuri.mat.puc-rio.br Precedence: bulk Reply-To: obm-l@mat.puc-rio.br Laurito, acredito que você tenha toda razão. Mas o que eu quero dizer é que aparentemente, a pessoa que formulou essa questão não tinha outra intenção que não o deboche. Era pra ser uma piada... o e-mail que recebi continha um texto antes perguntando algo do tipo "você é bom de matemática?", desafiando as pessoas a quebrarem a cabeça quanto a isso. No final a resposta vinha em tom de deboche, com gargalhadas ao lado. Abraços ----- Original Message ----- From: "Laurito Alves" To: Sent: Thursday, September 12, 2002 7:43 AM Subject: Re: [obm-l] TESTE > Felipe, > > Discordo de vocÊ. Essa é uma questão matemática cuja resposta você aceita > que é 200 pois lhe é apresentada uma lei de formação da sequencia que você > supõe simples e plausível. > > Matematicamente falando qualquer alternativa apresentada, na realidade > qualquer outro número, poderia ser o próximo termo da sequencia e é possível > apresentar a lei de formação que comprove isso. > > Laurito > > > >From: "Felipe Villela Dias" > >Reply-To: obm-l@mat.puc-rio.br > >To: > >Subject: Re: [obm-l] TESTE > >Date: Wed, 11 Sep 2002 23:01:37 -0300 > > > >Acho que toda essa discussão é inútil... já tinha essa questão não passa de > >uma brincadeira que as pessoas repassam via -e-mail. Há muito tempo recebi > >esse e-mail e tinha como resposta correta 200, pq começa com D. É uma > >brincadeira, não uma questão matemática. > >----- Original Message ----- > >From: "Jose Francisco Guimaraes Costa" > >To: > >Sent: Wednesday, September 11, 2002 10:27 PM > >Subject: Re: [obm-l] TESTE > > > > > > > Descobri porque minha neta, que está na classe de alfabetização aqui no > > > Brasil e que portanto, segundo o N, deveria resolver o problema com > > > facilidade, não o resolveu. > > > > > > É fato conhecido por educadores e psicólogos - e ambos têm lindas > >teorias > > > para isso - que não interessa quão distante alguém fica de sua lingua > > > nativa, isto é, a lingua em que aprendeu a falar, para contar e fazer > >contas > > > só ela é utilizada. É como se a unidade aritmética do cérebro não > >pudesse > > > ser reprogramada. > > > > > > Para minha neta, 16 é "seize", e seize não começa com "D". > > > > > > JF > > > > > > ----- Original Message ----- > > > From: "Afemano" > > > To: > > > Sent: Wednesday, September 11, 2002 8:06 PM > > > Subject: Re: [obm-l] TESTE > > > > > > > > > > Resposta : > > > > 200 > > > > Todos os números começam com a letra "D"... > > > > > > > > Falou !!! > > > > > > > > > > ----- Original Message ----- > > > From: "Jose Francisco Guimaraes Costa" > > > To: > > > Sent: Wednesday, September 11, 2002 4:42 PM > > > Subject: Re: [obm-l] TESTE > > > > > > > > > > Passei o problema para minha neta, que está na turma de alfabetização > >(no > > > > Brasil, embora ela tenha nascido em Lyon, cidade onde o N já > >trabalhou), > > > mas > > > > ela não conseguiu resolver. > > > > > > > > O problema não é difícil apenas para matemáticos. Também é para > > > engenheiros, > > > > como eu. > > > > > > > > Resumindo - qual a solução? > > > > > > > > JF > > > > > > > > -----Mensagem Original----- > > > > De: Nicolau C. Saldanha > > > > Para: > > > > Enviada em: Quarta-feira, 11 de Setembro de 2002 15:05 > > > > Assunto: Re: [obm-l] TESTE > > > > > > > > > > > > > On Wed, Sep 11, 2002 at 03:01:50PM -0300, Hely Jr. wrote: > > > > > > Observem a sequência abaixo: > > > > > > > > > > > > 2 - 10 - 12 - 16 - 17 - 18 - 19 ...... > > > > > > > > > > > > E agora, respondam: qual o próximo número da sequência??? > > > > > > > > > > > > a) 20 > > > > > > b) 22 > > > > > > c) 175 > > > > > > d) 200 > > > > > > > > > > > > Existe alguma pegadinha neste exercício? > > > > > > > > > > Existe. Este problema é fácil para turmas de alfabetização (no > >Brasil!) > > > > > mas difícil para matemáticos... ;-) > > > > > > > > > > []s, N. > > > > > > > > > > > > > > > >========================================================================= > > > Instruções para entrar na lista, sair da lista e usar a lista em > > > http://www.mat.puc-rio.br/~nicolau/olimp/obm-l.html > > > O administrador desta lista é > > > > >========================================================================= > > > > > > > > >--- > >Outgoing mail is certified Virus Free. > >Checked by AVG anti-virus system (http://www.grisoft.com). > >Version: 6.0.386 / Virus Database: 218 - Release Date: 9/9/2002 > > > >========================================================================= > >Instruções para entrar na lista, sair da lista e usar a lista em > >http://www.mat.puc-rio.br/~nicolau/olimp/obm-l.html > >O administrador desta lista é > >========================================================================= > > > > > _________________________________________________________________ > Converse com seus amigos online, faça o download grátis do MSN Messenger: > http://messenger.msn.com.br > > ========================================================================= > Instruções para entrar na lista, sair da lista e usar a lista em > http://www.mat.puc-rio.br/~nicolau/olimp/obm-l.html > O administrador desta lista é > ========================================================================= > --- Outgoing mail is certified Virus Free. Checked by AVG anti-virus system (http://www.grisoft.com). Version: 6.0.386 / Virus Database: 218 - Release Date: 9/9/2002 ========================================================================= Instruções para entrar na lista, sair da lista e usar a lista em http://www.mat.puc-rio.br/~nicolau/olimp/obm-l.html O administrador desta lista é ========================================================================= From owner-obm-l@sucuri.mat.puc-rio.br Thu Sep 12 12:10:13 2002 Return-Path: Received: (from majordom@localhost) by sucuri.mat.puc-rio.br (8.9.3/8.9.3) id MAA10449 for obm-l-MTTP; Thu, 12 Sep 2002 12:09:21 -0300 Received: from puma.unisys.com.br (ns2.unisys.com.br [200.220.64.7]) by sucuri.mat.puc-rio.br (8.9.3/8.9.3) with ESMTP id MAA10446 for ; Thu, 12 Sep 2002 12:09:16 -0300 Received: from josefran (riopm18p162.uninet.com.br [200.220.16.162]) by puma.unisys.com.br (8.12.3/8.12.3) with SMTP id g8CFAKjR029967 for ; Thu, 12 Sep 2002 12:10:21 -0300 (EST) X-Spam-Filter: check_local@puma.unisys.com.br by digitalanswers.org Message-ID: <004901c25a6f$248d39a0$a210dcc8@josefran> From: "Jose Francisco Guimaraes Costa" To: References: <000a01c25a5e$22d2c820$158c000a@felipe> Subject: Re: [obm-l] TESTE Date: Thu, 12 Sep 2002 12:14:45 -0300 MIME-Version: 1.0 Content-Type: text/plain; charset="iso-8859-1" Content-Transfer-Encoding: 8bit X-Priority: 3 X-MSMail-Priority: Normal X-Mailer: Microsoft Outlook Express 5.00.2314.1300 X-MIMEOLE: Produced By Microsoft MimeOLE V5.00.2314.1300 Sender: owner-obm-l@sucuri.mat.puc-rio.br Precedence: bulk Reply-To: obm-l@mat.puc-rio.br Não se deve levar para esse lado. Deboche certamente não é. Uma brincadeira, pode ser; mas o que não é, dependendo da forma como se olha? JF -----Mensagem Original----- De: Felipe Villela Dias Para: Enviada em: Quinta-feira, 12 de Setembro de 2002 10:13 Assunto: Re: [obm-l] TESTE > Laurito, > acredito que você tenha toda razão. Mas o que eu quero dizer é que > aparentemente, a pessoa que formulou essa questão não tinha outra intenção > que não o deboche. Era pra ser uma piada... o e-mail que recebi continha um > texto antes perguntando algo do tipo "você é bom de matemática?", desafiando > as pessoas a quebrarem a cabeça quanto a isso. No final a resposta vinha em > tom de deboche, com gargalhadas ao lado. > > Abraços > > ----- Original Message ----- > From: "Laurito Alves" > To: > Sent: Thursday, September 12, 2002 7:43 AM > Subject: Re: [obm-l] TESTE > > > > Felipe, > > > > Discordo de vocÊ. Essa é uma questão matemática cuja resposta você aceita > > que é 200 pois lhe é apresentada uma lei de formação da sequencia que você > > supõe simples e plausível. > > > > Matematicamente falando qualquer alternativa apresentada, na realidade > > qualquer outro número, poderia ser o próximo termo da sequencia e é > possível > > apresentar a lei de formação que comprove isso. > > > > Laurito > > > > > > >From: "Felipe Villela Dias" > > >Reply-To: obm-l@mat.puc-rio.br > > >To: > > >Subject: Re: [obm-l] TESTE > > >Date: Wed, 11 Sep 2002 23:01:37 -0300 > > > > > >Acho que toda essa discussão é inútil... já tinha essa questão não passa > de > > >uma brincadeira que as pessoas repassam via -e-mail. Há muito tempo > recebi > > >esse e-mail e tinha como resposta correta 200, pq começa com D. É uma > > >brincadeira, não uma questão matemática. > > >----- Original Message ----- > > >From: "Jose Francisco Guimaraes Costa" > > >To: > > >Sent: Wednesday, September 11, 2002 10:27 PM > > >Subject: Re: [obm-l] TESTE > > > > > > > > > > Descobri porque minha neta, que está na classe de alfabetização aqui > no > > > > Brasil e que portanto, segundo o N, deveria resolver o problema com > > > > facilidade, não o resolveu. > > > > > > > > É fato conhecido por educadores e psicólogos - e ambos têm lindas > > >teorias > > > > para isso - que não interessa quão distante alguém fica de sua lingua > > > > nativa, isto é, a lingua em que aprendeu a falar, para contar e fazer > > >contas > > > > só ela é utilizada. É como se a unidade aritmética do cérebro não > > >pudesse > > > > ser reprogramada. > > > > > > > > Para minha neta, 16 é "seize", e seize não começa com "D". > > > > > > > > JF > > > > > > > > ----- Original Message ----- > > > > From: "Afemano" > > > > To: > > > > Sent: Wednesday, September 11, 2002 8:06 PM > > > > Subject: Re: [obm-l] TESTE > > > > > > > > > > > > > Resposta : > > > > > 200 > > > > > Todos os números começam com a letra "D"... > > > > > > > > > > Falou !!! > > > > > > > > > > > > > ----- Original Message ----- > > > > From: "Jose Francisco Guimaraes Costa" > > > > To: > > > > Sent: Wednesday, September 11, 2002 4:42 PM > > > > Subject: Re: [obm-l] TESTE > > > > > > > > > > > > > Passei o problema para minha neta, que está na turma de > alfabetização > > >(no > > > > > Brasil, embora ela tenha nascido em Lyon, cidade onde o N já > > >trabalhou), > > > > mas > > > > > ela não conseguiu resolver. > > > > > > > > > > O problema não é difícil apenas para matemáticos. Também é para > > > > engenheiros, > > > > > como eu. > > > > > > > > > > Resumindo - qual a solução? > > > > > > > > > > JF > > > > > > > > > > -----Mensagem Original----- > > > > > De: Nicolau C. Saldanha > > > > > Para: > > > > > Enviada em: Quarta-feira, 11 de Setembro de 2002 15:05 > > > > > Assunto: Re: [obm-l] TESTE > > > > > > > > > > > > > > > > On Wed, Sep 11, 2002 at 03:01:50PM -0300, Hely Jr. wrote: > > > > > > > Observem a sequência abaixo: > > > > > > > > > > > > > > 2 - 10 - 12 - 16 - 17 - 18 - 19 ...... > > > > > > > > > > > > > > E agora, respondam: qual o próximo número da sequência??? > > > > > > > > > > > > > > a) 20 > > > > > > > b) 22 > > > > > > > c) 175 > > > > > > > d) 200 > > > > > > > > > > > > > > Existe alguma pegadinha neste exercício? > > > > > > > > > > > > Existe. Este problema é fácil para turmas de alfabetização (no > > >Brasil!) > > > > > > mas difícil para matemáticos... ;-) > > > > > > > > > > > > []s, N. > > > > > > > > > > > > > > > > > > > > >========================================================================= > > > > Instruções para entrar na lista, sair da lista e usar a lista em > > > > http://www.mat.puc-rio.br/~nicolau/olimp/obm-l.html > > > > O administrador desta lista é > > > > > > >========================================================================= > > > > > > > > > > > > >--- > > >Outgoing mail is certified Virus Free. > > >Checked by AVG anti-virus system (http://www.grisoft.com). > > >Version: 6.0.386 / Virus Database: 218 - Release Date: 9/9/2002 > > > > > >========================================================================= > > >Instruções para entrar na lista, sair da lista e usar a lista em > > >http://www.mat.puc-rio.br/~nicolau/olimp/obm-l.html > > >O administrador desta lista é > > >========================================================================= > > > > > > > > > > _________________________________________________________________ > > Converse com seus amigos online, faça o download grátis do MSN Messenger: > > http://messenger.msn.com.br > > > > ========================================================================= > > Instruções para entrar na lista, sair da lista e usar a lista em > > http://www.mat.puc-rio.br/~nicolau/olimp/obm-l.html > > O administrador desta lista é > > ========================================================================= > > > > > --- > Outgoing mail is certified Virus Free. > Checked by AVG anti-virus system (http://www.grisoft.com). > Version: 6.0.386 / Virus Database: 218 - Release Date: 9/9/2002 > > ========================================================================= > Instruções para entrar na lista, sair da lista e usar a lista em > http://www.mat.puc-rio.br/~nicolau/olimp/obm-l.html > O administrador desta lista é > ========================================================================= > ========================================================================= Instruções para entrar na lista, sair da lista e usar a lista em http://www.mat.puc-rio.br/~nicolau/olimp/obm-l.html O administrador desta lista é ========================================================================= From owner-obm-l@sucuri.mat.puc-rio.br Thu Sep 12 13:37:34 2002 Return-Path: Received: (from majordom@localhost) by sucuri.mat.puc-rio.br (8.9.3/8.9.3) id NAA12087 for obm-l-MTTP; Thu, 12 Sep 2002 13:35:55 -0300 Received: from web12905.mail.yahoo.com (web12905.mail.yahoo.com [216.136.174.72]) by sucuri.mat.puc-rio.br (8.9.3/8.9.3) with SMTP id NAA12084 for ; Thu, 12 Sep 2002 13:35:52 -0300 Message-ID: <20020912163657.16530.qmail@web12905.mail.yahoo.com> Received: from [200.206.103.3] by web12905.mail.yahoo.com via HTTP; Thu, 12 Sep 2002 13:36:57 ART Date: Thu, 12 Sep 2002 13:36:57 -0300 (ART) From: =?iso-8859-1?q?Johann=20Peter=20Gustav=20Lejeune=20Dirichlet?= Subject: Re: [obm-l] IRRACIONALIDADE DE PI To: obm-l@mat.puc-rio.br In-Reply-To: <20020911174020.9679.qmail@web21307.mail.yahoo.com> MIME-Version: 1.0 Content-Type: multipart/alternative; boundary="0-1815814972-1031848617=:15858" Content-Transfer-Encoding: 8bit Sender: owner-obm-l@sucuri.mat.puc-rio.br Precedence: bulk Reply-To: obm-l@mat.puc-rio.br --0-1815814972-1031848617=:15858 Content-Type: text/plain; charset=iso-8859-1 Content-Transfer-Encoding: 8bit Meu,tem uma demonstraçao no site da OBM.Va ate a Semana Olimpica e entre no link de teoria dos numeros.Tem uma prova la que so usa integral e e acessivel a quem conhece os rudimentos de Diferencial. Jorge Paulino wrote: Galera, alguém conhece alguma demonstração, acessível a alunos de segundo grau, da irracionalidade do número PI? Um Abraço, Jorge _______________________________________________________________________ Yahoo! PageBuilder O super editor para criação de sites: é grátis, fácil e rápido. http://br.geocities.yahoo.com/v/pb.html ========================================================================= Instruções para entrar na lista, sair da lista e usar a lista em http://www.mat.puc-rio.br/~nicolau/olimp/obm-l.html O administrador desta lista é ========================================================================= --------------------------------- Yahoo! PageBuilder - O super editor para criação de sites: é grátis, fácil e rápido. --0-1815814972-1031848617=:15858 Content-Type: text/html; charset=iso-8859-1 Content-Transfer-Encoding: 8bit

Meu,tem uma demonstraçao no site da OBM.Va ate a Semana Olimpica e entre no link de teoria dos numeros.Tem uma prova la que so usa integral e e acessivel a quem conhece os rudimentos de Diferencial.

 Jorge Paulino wrote:

Galera, alguém conhece alguma demonstração,
acessível a alunos de segundo grau, da irracionalidade
do número PI?
Um Abraço,
Jorge

_______________________________________________________________________
Yahoo! PageBuilder
O super editor para criação de sites: é grátis, fácil e rápido.
http://br.geocities.yahoo.com/v/pb.html
=========================================================================
Instruções para entrar na lista, sair da lista e usar a lista em
http://www.mat.puc-rio.br/~nicolau/olimp/obm-l.html
O administrador desta lista é
=========================================================================


Yahoo! PageBuilder - O super editor para criação de sites: é grátis, fácil e rápido. --0-1815814972-1031848617=:15858-- ========================================================================= Instruções para entrar na lista, sair da lista e usar a lista em http://www.mat.puc-rio.br/~nicolau/olimp/obm-l.html O administrador desta lista é ========================================================================= From owner-obm-l@sucuri.mat.puc-rio.br Thu Sep 12 15:37:29 2002 Return-Path: Received: (from majordom@localhost) by sucuri.mat.puc-rio.br (8.9.3/8.9.3) id PAA14047 for obm-l-MTTP; Thu, 12 Sep 2002 15:36:24 -0300 Received: from c0mailgw13.prontomail.com ([209.185.149.10]) by sucuri.mat.puc-rio.br (8.9.3/8.9.3) with ESMTP id PAA14044 for ; Thu, 12 Sep 2002 15:36:21 -0300 Received: from c2web17 (209.185.149.10) by c0mailgw13.prontomail.com (NPlex 6.0.045) id 3D80C7A7000040A6 for obm-l@mat.puc-rio.br; Thu, 12 Sep 2002 11:37:40 -0700 X-Version: Mailcentro 1.0 X-SenderIP: 143.107.26.3 X-SenderID: 6757770 From: "kenji yoshitaki" Message-Id: Date: Thu, 12 Sep 2002 15:32:36 -0300 X-Priority: Normal Content-Type: text/plain; charset=iso-8859-1 To: obm-l@mat.puc-rio.br Subject: Re:[obm-l] circuito IME X-Mailer: Web Based Pronto Mime-Version: 1.0 Content-Transfer-Encoding: 8bit X-MIME-Autoconverted: from quoted-printable to 8bit by sucuri.mat.puc-rio.br id PAA14045 Sender: owner-obm-l@sucuri.mat.puc-rio.br Precedence: bulk Reply-To: obm-l@mat.puc-rio.br Olha , realmente essa historia de discitir o nivel do ime no seu vestibular parece que tá longe de acabar. Cicuito RLC , eu só comecei a ver em lab. de fisica III e muita coisa que cai lá no exame deles nem mesmo está no pragrama , qdo prestei estudei pra caramba e chegando lá , nao passei..Isso porque pedir coisas que nao estao no programa parece ser normal pra eles, ou coisas de nivel superior. A eq. de Van der Waals de gases caiu uma vez e isso eu tb só vi no segundo ano de faculdade. A verdade que só passa quem faz aqueles cursinhos especializados nesse tipo de exame. Isso se é que vale a pena se matar de estudar pra um exame que nao mede quem realmente é melhor, só mede o conhecimento de coisas extremamente especificas e que o cara vai ver de novo na facu. Conheço dois caras que estudam comigo que passaram , fizeram alguns meses e viram que nao valeu o esforço, principalmente por causa da estrutura do curso que encontraram. Mas é isso quem for prestar , boa sorte ! "A matemática é a mais alta das ciências, o dom mais alto que os deuses deram aos homens" Arquimedes(287a.C.-212a.C.) Kenji Yoshitaki Engenharia/Poli-USP Email grátis em português no http://www.econofinance.com/zapmail.html ___________________________________________________________ Get your own Web-Based E-mail Service at http://www.zzn.com ========================================================================= Instruções para entrar na lista, sair da lista e usar a lista em http://www.mat.puc-rio.br/~nicolau/olimp/obm-l.html O administrador desta lista é ========================================================================= From owner-obm-l@sucuri.mat.puc-rio.br Thu Sep 12 15:37:29 2002 Return-Path: Received: (from majordom@localhost) by sucuri.mat.puc-rio.br (8.9.3/8.9.3) id PAA14055 for obm-l-MTTP; Thu, 12 Sep 2002 15:37:07 -0300 Received: from c0mailgw06.prontomail.com ([209.185.149.10]) by sucuri.mat.puc-rio.br (8.9.3/8.9.3) with ESMTP id PAA14052 for ; Thu, 12 Sep 2002 15:37:04 -0300 Received: from c2web17 (209.185.149.10) by c0mailgw06.prontomail.com (NPlex 6.0.045) id 3D80DE9A0000011F for obm-l@mat.puc-rio.br; Thu, 12 Sep 2002 11:38:23 -0700 X-Version: Mailcentro 1.0 X-SenderIP: 143.107.26.3 X-SenderID: 6757770 From: "kenji yoshitaki" Message-Id: Date: Thu, 12 Sep 2002 15:33:22 -0300 X-Priority: Normal Content-Type: text/plain; charset=iso-8859-1 To: obm-l@mat.puc-rio.br Subject: Re:[obm-l] circuito IME X-Mailer: Web Based Pronto Mime-Version: 1.0 Content-Transfer-Encoding: 8bit X-MIME-Autoconverted: from quoted-printable to 8bit by sucuri.mat.puc-rio.br id PAA14053 Sender: owner-obm-l@sucuri.mat.puc-rio.br Precedence: bulk Reply-To: obm-l@mat.puc-rio.br Olha , realmente essa historia de discitir o nivel do ime no seu vestibular parece que tá longe de acabar. Cicuito RLC , eu só comecei a ver em lab. de fisica III e muita coisa que cai lá no exame deles nem mesmo está no pragrama , qdo prestei estudei pra caramba e chegando lá , nao passei..Isso porque pedir coisas que nao estao no programa parece ser normal pra eles, ou coisas de nivel superior. A eq. de Van der Waals de gases caiu uma vez e isso eu tb só vi no segundo ano de faculdade. A verdade que só passa quem faz aqueles cursinhos especializados nesse tipo de exame. Isso se é que vale a pena se matar de estudar pra um exame que nao mede quem realmente é melhor, só mede o conhecimento de coisas extremamente especificas e que o cara vai ver de novo na facu. Conheço dois caras que estudam comigo que passaram , fizeram alguns meses e viram que nao valeu o esforço, principalmente por causa da estrutura do curso que encontraram. Mas é isso quem for prestar , boa sorte ! "A matemática é a mais alta das ciências, o dom mais alto que os deuses deram aos homens" Arquimedes(287a.C.-212a.C.) Kenji Yoshitaki Engenharia/Poli-USP Email grátis em português no http://www.econofinance.com/zapmail.html ___________________________________________________________ Get your own Web-Based E-mail Service at http://www.zzn.com ========================================================================= Instruções para entrar na lista, sair da lista e usar a lista em http://www.mat.puc-rio.br/~nicolau/olimp/obm-l.html O administrador desta lista é ========================================================================= From owner-obm-l@sucuri.mat.puc-rio.br Thu Sep 12 17:33:41 2002 Return-Path: Received: (from majordom@localhost) by sucuri.mat.puc-rio.br (8.9.3/8.9.3) id RAA16517 for obm-l-MTTP; Thu, 12 Sep 2002 17:32:18 -0300 Received: (from nicolau@localhost) by sucuri.mat.puc-rio.br (8.9.3/8.9.3) id RAA16513 for obm-l@mat.puc-rio.br; Thu, 12 Sep 2002 17:32:17 -0300 Date: Thu, 12 Sep 2002 17:32:17 -0300 From: "Nicolau C. Saldanha" To: obm-l@mat.puc-rio.br Subject: Re: [obm-l] Axioma da Escolha Message-ID: <20020912173217.A16414@sucuri.mat.puc-rio.br> References: <20020911190142.605DB3EB20@zeus.opendf.com.br> Mime-Version: 1.0 Content-Type: text/plain; charset=iso-8859-1 Content-Disposition: inline Content-Transfer-Encoding: 8bit User-Agent: Mutt/1.2.5i In-Reply-To: <20020911190142.605DB3EB20@zeus.opendf.com.br>; from artur@opendf.com.br on Wed, Sep 11, 2002 at 04:01:42PM -0300 Sender: owner-obm-l@sucuri.mat.puc-rio.br Precedence: bulk Reply-To: obm-l@mat.puc-rio.br On Wed, Sep 11, 2002 at 04:01:42PM -0300, 498 - Artur Costa Steiner wrote: > Nos últimos dias o Axioma da Escolha foi bastante mencionado nesta > lista, motivado por um interessante problema (violência), sugerido por > uma das participantes, e que involve este axioma. > > Eu não estou certo, mas, no meio matemático, ainda existem hoje > restrições a este axioma, no sentido de que alguma prova nele baseada > possa ser considerada questionável ou mesmo inválida? Este assunto mereceria uma resposta mais longa, mas o axioma da escolha é 'aceito' no sentido seguinte: a maioria dos matemáticos usa este axioma sem parar para pensar no assunto. Aliás sem nem saber direito quando está realmente usando o axioma. Alguns matemáticos, entretanto, especialmente especialistas em lógica ou teoria dos conjuntos, acham interessante notar exatamente quando o tal axioma é utilizado. []s, N. ========================================================================= Instruções para entrar na lista, sair da lista e usar a lista em http://www.mat.puc-rio.br/~nicolau/olimp/obm-l.html O administrador desta lista é ========================================================================= From owner-obm-l@sucuri.mat.puc-rio.br Thu Sep 12 17:56:18 2002 Return-Path: Received: (from majordom@localhost) by sucuri.mat.puc-rio.br (8.9.3/8.9.3) id RAA16870 for obm-l-MTTP; Thu, 12 Sep 2002 17:56:09 -0300 Received: from traven10 (traven10.uol.com.br [200.221.4.45]) by sucuri.mat.puc-rio.br (8.9.3/8.9.3) with ESMTP id RAA16867 for ; Thu, 12 Sep 2002 17:56:07 -0300 Received: from franklin ([200.214.41.206]) by traven10 (8.9.1/8.9.1) with SMTP id RAA29690 for ; Thu, 12 Sep 2002 17:56:35 -0300 (BRT) From: "Franklin de Lima Marquezino" To: Subject: [obm-l] =?iso-8859-1?Q?Re:_=5Bobm-l=5D_N=FAmeros_rand=F4micos?= Date: Thu, 12 Sep 2002 17:57:24 -0300 Message-ID: <01c25a9e$fec5be60$ce29d6c8@franklin> MIME-Version: 1.0 Content-Type: multipart/alternative; boundary="----=_NextPart_000_0069_01C25A85.D9788660" X-Priority: 3 X-MSMail-Priority: Normal X-Mailer: Microsoft Outlook Express 4.71.1712.3 X-MIMEOLE: Produced By Microsoft MimeOLE V4.71.1712.3 Sender: owner-obm-l@sucuri.mat.puc-rio.br Precedence: bulk Reply-To: obm-l@mat.puc-rio.br This is a multi-part message in MIME format. ------=_NextPart_000_0069_01C25A85.D9788660 Content-Type: text/plain; charset="iso-8859-1" Content-Transfer-Encoding: quoted-printable N=E3o sou nenhum especialista nesse assunto, mas como venho estudando = Computa=E7=E3o Qu=E2ntica h=E1 algum tempo (inicia=E7=E3o cient=EDfica) = acabei estudando algo sobre gera=E7=E3o de n=FAmeros aleat=F3rios. >O que os computadores fazem eh "simular" resultados aleatorios, = buscando >solucoes que sejam satisfatorias em tempo habil. > >Nesse caso, o problema se reduz a encontrar "fontes" de dados altamente = variaveis >e que nao mostrem nenhum vicio nem tendencia a apresentar certos = resultados. > Segundo o livro "Explorations in Quantum Computing", de C.P.Willianms e = S.H.Clearwater, para passar no teste de aleatoriedade, uma dada = sequ=EAncia n=FAmerica precisa ter uma certa distribui=E7=E3o e n=E3o = deve haver nenhuma correla=E7=E3o percept=EDvel entre os n=FAmeros = gerados. Por exemplo, imagine que eu jogue uma moeda e marque os = resultados C, K, C, K, C, K, C, K, C, K, C, K, C, K, C, K,... Essa = sequ=EAncia passa no teste da distribui=E7=E3o (os resultados est=E3o = uniformemente distribu=EDdos entre K e C), mas n=E3o passam no teste da = correla=E7=E3o (por que a sequ=EAncia se repete, e por isso, =E9 = facilmente previs=EDvel qual ser=E1 o pr=F3ximo n=FAmero).=20 >Por exemplo, eles podem "pegar" os milesimos de segundos do momento e = trabalhar=20 >com isso para apresentar um resultado satisfatorio, ou pegar uma parte = qualquer da=20 >memoria e usar os bits para determinar um resultado conveniente. > >Nao estou informado quanto a isto, mas pelo que sei o uso do relogio do = >computador eh uma das formas mais eficientes para essa simulacao, por >apresentar uma chance "igual" de escolha entre os numeros e por ter um >ciclo muito rapido (no caso dos milesimos..) Sim, estas s=E3o algumas das melhores alternativas, mas n=E3o s=E3o = muito eficientes, por que se a condi=E7=E3o inicial (os mil=E9simos de = segundo ou os bits de uma parte da mem=F3ria) se repetir, toda a = sequ=EAncia num=E9rica tamb=E9m ser=E1 repetida. J=E1 ouvi dizer que algumas pessoas ultimamente estavam utilizando = informa=E7=F5es da rede (n=FAmero de pessoas conectadas em um instante, = por exemplo) para gerar as sequ=EAncias. Mas de qualquer forma os = computadores cl=E1ssicos somente geram n=FAmeros pseudo-aleat=F3rios. = Por outro lado, um computador qu=E2ntico ser=E1 bem eficiente em gerar = sequ=EAncias de n=FAmeros rand=F4micos. Espero ter contribu=EDdo. Caso eu tenha dito alguma besteira, = sintam-se =E0 vontade para me corrigir. Abra=E7os, Franklin -------------------------------------------------------------------------= -------------------------------------------------------------------- CBPF - Centro Brasileiro de Pesquisas F=EDsicas (Bolsista PIBIC/CNPq) LNCC - Laborat=F3rio Nacional de Computa=E7=E3o Cient=EDfica (Grupo de = Computa=E7=E3o Qu=E2ntica)=20 GFT - Grupo de F=EDsica Te=F3rica Jos=E9 Leite Lopes http://virtual01.lncc.br/dcs/links/dcs_quantum_comp.html http://gft.ucp.br/staff/franklin -------------------------------------------------------------------------= --------------------------------------------------------------------=20 ------=_NextPart_000_0069_01C25A85.D9788660 Content-Type: text/html; charset="iso-8859-1" Content-Transfer-Encoding: quoted-printable
Não sou nenhum especialista nesse assunto, mas como venho = estudando=20 Computação Quântica há algum tempo=20 (iniciação científica) acabei estudando algo sobre=20 geração de números aleatórios. 

>O que os computadores fazem eh "simular" = resultados=20 aleatorios, buscando
>solucoes que sejam satisfatorias em tempo=20 habil.
>
>Nesse caso, o problema se reduz a encontrar=20 "fontes" de dados altamente variaveis
>e que nao mostrem = nenhum=20 vicio nem tendencia a apresentar certos resultados.
 
Segundo o livro "Explorations in Quantum Computing", de=20 C.P.Willianms e S.H.Clearwater, para passar no teste de aleatoriedade, = uma dada=20 sequência númerica precisa ter uma certa = distribuição=20 e não deve haver nenhuma correlação = perceptível=20 entre os números gerados. Por exemplo, imagine que eu jogue uma = moeda e=20 marque os resultados C, K, C, K, C, K, C, K, C, K, C, K, C, K, C, K,... = Essa=20 sequência passa no teste da distribuição (os = resultados=20 estão uniformemente distribuídos entre K e C), mas = não=20 passam no teste da correlação (por que a sequência = se=20 repete, e por isso, é facilmente previsível qual = será o=20 próximo número).
 

>Por exemplo, eles podem "pegar" os milesimos de = segundos=20 do momento e trabalhar
>com isso para apresentar um resultado=20 satisfatorio, ou pegar uma parte qualquer da
>memoria e usar os = bits para=20 determinar um resultado conveniente.
>Nao estou informado quanto a isto, mas pelo que sei o uso do = relogio do=20
>computador eh uma das formas mais eficientes para essa = simulacao,=20 por
>apresentar uma chance "igual" de escolha entre os = numeros e=20 por ter um
>ciclo muito rapido (no caso dos = milesimos..) 

 
   Sim, estas são algumas das melhores = alternativas, mas=20 não são muito eficientes, por que se a = condição=20 inicial (os milésimos de segundo ou os bits de uma parte da=20 memória) se repetir, toda a sequência numérica = também=20 será repetida.
    Já ouvi dizer que algumas pessoas = ultimamente=20 estavam utilizando informações da rede (número de = pessoas=20 conectadas em um instante, por exemplo) para gerar as sequências. = Mas de=20 qualquer forma os computadores clássicos somente geram = números=20 pseudo-aleatórios. Por outro lado, um computador quântico=20 será bem eficiente em gerar sequências de números=20 randômicos.
 
    Espero ter contribuído. Caso eu tenha = dito alguma=20 besteira, sintam-se à vontade para me corrigir.
 
        Abraços,
         Franklin
 
 
 
 
 
 
----------------------------------------------------------------= -------------------------------------------------------------------------= ----
CBPF=20 - Centro Brasileiro de Pesquisas Físicas (Bolsista = PIBIC/CNPq)
LNCC -=20 Laboratório Nacional de Computação = Científica (Grupo=20 de Computação Quântica) 
GFT - Grupo de Física Teórica = José Leite=20 Lopes
http://gft.ucp.br/staff/frankli= n
----------------------------------------------------------------= -------------------------------------------------------------------------= ---- 
------=_NextPart_000_0069_01C25A85.D9788660-- ========================================================================= Instruções para entrar na lista, sair da lista e usar a lista em http://www.mat.puc-rio.br/~nicolau/olimp/obm-l.html O administrador desta lista é ========================================================================= From owner-obm-l@sucuri.mat.puc-rio.br Thu Sep 12 18:55:42 2002 Return-Path: Received: (from majordom@localhost) by sucuri.mat.puc-rio.br (8.9.3/8.9.3) id SAA17897 for obm-l-MTTP; Thu, 12 Sep 2002 18:55:09 -0300 Received: from hotmail.com (f38.sea2.hotmail.com [207.68.165.38]) by sucuri.mat.puc-rio.br (8.9.3/8.9.3) with ESMTP id SAA17885 for ; Thu, 12 Sep 2002 18:55:05 -0300 Received: from mail pickup service by hotmail.com with Microsoft SMTPSVC; Thu, 12 Sep 2002 14:56:08 -0700 Received: from 32.94.119.254 by sea2fd.sea2.hotmail.msn.com with HTTP; Thu, 12 Sep 2002 21:56:07 GMT X-Originating-IP: [32.94.119.254] From: "Paulo Santa Rita" To: obm-l@mat.puc-rio.br Subject: Re: [obm-l] Axioma da Escolha Date: Thu, 12 Sep 2002 21:56:07 +0000 Mime-Version: 1.0 Content-Type: text/plain; charset=iso-8859-1; format=flowed Message-ID: X-OriginalArrivalTime: 12 Sep 2002 21:56:08.0302 (UTC) FILETIME=[3317A0E0:01C25AA7] Sender: owner-obm-l@sucuri.mat.puc-rio.br Precedence: bulk Reply-To: obm-l@mat.puc-rio.br Ola PROF NICOLAU e demais colegas desta lista ... OBM-L, Vou fazer um comentario a mensagem abaixo que talvez ajude alguns membros desta lista entenderem porque muitos Matematicos - em especial aqueles que conhecem logica e teoria dos conjuntos - acham relevante destacar que esta ou aquela prova ou argumentacao matematica usa o AXIOMA DA ESCOLHA. Tarski, entre outros, mostrou que o uso do AXIOMA DA ESCOLHA com os demais axiomas da teoria dos conjuntos levam-nos inevitavelmente a conclusoes pouco verossimeis. Em particular ele mostrou que poderiamos dividir uma esfera dada em ao menos cinco partes e, unindo novamente as partes de outra forma, derivar nao uma, mas duas esferas identicas a primitiva. O fato acima, por paradoxal que parece, e logicamente inatacavel e uma das consequencias bizarras que este axioma implica. Muitos Matematicos supunham que tais resultados eram derivados da liberalidade dado a FUNCAO DE ESCOLHA, pois pode-se usar a que quisermos conquanto respeitemos o aspecto formal do axioma. A partir deste resultado do Tarski ( e de outros tambem ) os Matematicos comecaram a suspeitar que o AXIOMA DA ESCOLHA era um principio nefasto, sendo provavelmente o responsavel por possiveis e potenciais inconsistencias que a teoria do conjuntos tivesse ou viesse a ter. Dai surgiu a desconfianca com as demonstracoes com este axioma. Por prudencia, toda prova que usava este teorema era rotulada "USA O AXIOMA DA ESCOLHA", como que insinuando : "PROCURE UMA OUTRA MANEIRA DE PROVAR ISSO ... " Aqui entra o Magistral GODEL ... Godel classificou as teorias dos conjuntos em : 1) Teoria Cantoriana A e aquela que usa o AXIOMA DA ESCOLHA. 2) Teoria Cantoriana B e aquela em que nao usa o AXIOMA DA ESCOLHA. E provou o seguinte : SE A TEORIA CANTORIANA "A" CONTIVER OU GERAR INCONSISTENCIAS, A TEORIA CANTORIA "B" TAMBEM CONTERA E GERARA INCONSISTENCIAS. Isto e, o AXIOMA DA ESCOLHA nao e o responsavel por possiveis inconsistencias ou paradoxas que porventura derivem da teoria dos conjuntos. Ele pode ser um CATALISADOR destas inconsistencias, evidenciando de forma mais direta e clara possiveis absurdos ... Agora, uma observacao sobre o Paradoxo de Tarski. O que ha de absurdo nele ? A criacao de Massa ( duplicacao de uma esfera ) sem a necessaria absorcao de uma fabulosa quantidade de Energia ? Mas ... Nao e isso que rotineiramente ocorre no mundo das particulas elementares, quando, do nada, surge uma massa que posteriormente desapare num par de particulas antipodas ? E mais provavel que este fato ou operacao seja paradoxal para o nosso cotidiano, nao para o Autor da Natureza que, hoje sabemos, continuamente faz isso ... Um abraco Paulo Santa Rita 5,1854,120902 >From: "Nicolau C. Saldanha" >Reply-To: obm-l@mat.puc-rio.br >To: obm-l@mat.puc-rio.br >Subject: Re: [obm-l] Axioma da Escolha >Date: Thu, 12 Sep 2002 17:32:17 -0300 > >On Wed, Sep 11, 2002 at 04:01:42PM -0300, 498 - Artur Costa Steiner wrote: > > Nos últimos dias o Axioma da Escolha foi bastante mencionado nesta > > lista, motivado por um interessante problema (violência), sugerido por > > uma das participantes, e que involve este axioma. > > > > Eu não estou certo, mas, no meio matemático, ainda existem hoje > > restrições a este axioma, no sentido de que alguma prova nele baseada > > possa ser considerada questionável ou mesmo inválida? > >Este assunto mereceria uma resposta mais longa, mas o axioma da escolha >é 'aceito' no sentido seguinte: a maioria dos matemáticos usa este >axioma sem parar para pensar no assunto. Aliás sem nem saber direito >quando está realmente usando o axioma. Alguns matemáticos, entretanto, >especialmente especialistas em lógica ou teoria dos conjuntos, acham >interessante notar exatamente quando o tal axioma é utilizado. > >[]s, N. >========================================================================= >Instruções para entrar na lista, sair da lista e usar a lista em >http://www.mat.puc-rio.br/~nicolau/olimp/obm-l.html >O administrador desta lista é >========================================================================= _________________________________________________________________ Tenha você também um MSN Hotmail, o maior webmail do mundo: http://www.hotmail.com/br ========================================================================= Instruções para entrar na lista, sair da lista e usar a lista em http://www.mat.puc-rio.br/~nicolau/olimp/obm-l.html O administrador desta lista é ========================================================================= From owner-obm-l@sucuri.mat.puc-rio.br Thu Sep 12 20:50:06 2002 Return-Path: Received: (from majordom@localhost) by sucuri.mat.puc-rio.br (8.9.3/8.9.3) id UAA19037 for obm-l-MTTP; Thu, 12 Sep 2002 20:49:23 -0300 Received: from ginsberg.pub1 (ginsberg.uol.com.br [200.221.4.48]) by sucuri.mat.puc-rio.br (8.9.3/8.9.3) with ESMTP id UAA19033 for ; Thu, 12 Sep 2002 20:49:20 -0300 Received: from u2z7z2 ([200.158.144.83]) by ginsberg.pub1 (8.9.1/8.9.1) with ESMTP id UAA29515 for ; Thu, 12 Sep 2002 20:48:46 -0300 (BRT) Message-ID: <003201c25ab7$546ade00$53909ec8@u2z7z2> From: "Wagner" To: Subject: [obm-l] =?iso-8859-1?Q?D=FAvida_sobre_sequ=EAncias_rand=F4micas_=28problema_do_ti?= =?iso-8859-1?Q?po_NP=29?= Date: Thu, 12 Sep 2002 20:51:35 -0300 Organization: Wagner MIME-Version: 1.0 Content-Type: multipart/alternative; boundary="----=_NextPart_000_002F_01C25A9E.2E8FBDE0" X-Priority: 3 X-MSMail-Priority: Normal X-Mailer: Microsoft Outlook Express 5.50.4133.2400 X-MimeOLE: Produced By Microsoft MimeOLE V5.50.4133.2400 Sender: owner-obm-l@sucuri.mat.puc-rio.br Precedence: bulk Reply-To: obm-l@mat.puc-rio.br This is a multi-part message in MIME format. ------=_NextPart_000_002F_01C25A9E.2E8FBDE0 Content-Type: text/plain; charset="iso-8859-1" Content-Transfer-Encoding: quoted-printable Oi pessoal Queria perguntar ao Nicolau ou a quem conseguir me resolver essa = pergunta: Se um algoritmo pode construir uma sequ=EAncia rand=F4mica, uma = sequ=EAncia qualquer desse tipo com um n=FAmero finito n de termos = poderia ent=E3o ser descrita por uma infinidade de algoritmos = diferentes, e esses algoritmos podem ser todos construidos a partir de = um algoritmo.(todos os algoritmos iriam variar de problema para = problema). Ou seja uma sequ=EAncia pode ser construida por infinitos = algoritmos diferentes, mas todos esses algoritmos podem ser construidos = a partir de um mesmo algoritmo. A problema =E9 o seguinte: Dada uma sequ=EAncia qualquer, qual =E9 o = algoritmo que gera todos os outros? A minha d=FAvida =E9 que pelo o que posso ver esse problema =E9 um = problema do tipo NP (polinominal n=E3o-determin=EDstico), ou seja a sua = resposta existe mas =E9 imposs=EDvel de ser dada na pr=E1tica. O meu racioc=EDnio para chegar a essa conclus=E3o foi o seguinte: Primeiro =E9 preciso provar que o problema tem solu=E7=E3o: Sendo a sequ=EAncia A=3Da,b,c,d,e,f,g,...,n (em que a =E9 o 1=BA termo, = b =E9 o 2=BA, at=E9 n que =E9 o n-=E9simo termo). Sendo B o conjunto de algoritmos que geram sequ=EAncias em que quando o = primeiro termo =E9 a, o segundo =E9 b e sendo C o conjunto dos = algoritmos para os quais se o 1=BA termo =E9 b, o 2=BA =E9 C. O algoritmo que descreve a sequ=EAncia A ent=E3o pertence =E0 B = intersec=E7=E3o com C. B e C possuem infinitos elementos. Como a,b,c = s=E3o 3 n=FAmeros aleat=F3rios, B intersec=E7=E3o com C =E9 aleat=F3rio. = A intersec=E7=E3o aleat=F3ria de 2 conjuntos infinitos =E9 um conjunto = infinito. Expandindo esse racioc=EDnio, existem infinitos algoritmos que = perfazem A. Logo existem infinitas alternativas a serem analisadas, ou = seja o problema =E9 do tipo NP. Esse pensamento esta certo? J=E1 vou agradecendo Andr=E9 T. ------=_NextPart_000_002F_01C25A9E.2E8FBDE0 Content-Type: text/html; charset="iso-8859-1" Content-Transfer-Encoding: quoted-printable
Oi pessoal
 
Queria perguntar ao Nicolau ou a quem = conseguir me=20 resolver essa pergunta:
 
Se um algoritmo pode construir uma = sequ=EAncia=20 rand=F4mica, uma sequ=EAncia qualquer desse tipo com um n=FAmero finito = n de=20 termos poderia ent=E3o ser descrita por uma infinidade de = algoritmos=20 diferentes, e esses algoritmos podem ser todos construidos a partir de = um=20 algoritmo.(todos os algoritmos iriam variar de problema para = problema). Ou=20 seja uma sequ=EAncia pode ser construida por infinitos algoritmos = diferentes, mas=20 todos esses algoritmos podem ser construidos a partir de um mesmo=20 algoritmo.
 
A problema =E9 o seguinte: Dada uma = sequ=EAncia=20 qualquer, qual =E9 o algoritmo que gera todos os outros?
 
A minha d=FAvida =E9 que pelo o que = posso ver esse=20 problema =E9 um problema do tipo NP (polinominal = n=E3o-determin=EDstico), ou seja a=20 sua resposta existe mas =E9 imposs=EDvel de ser dada na = pr=E1tica.
 
O meu racioc=EDnio para chegar a essa = conclus=E3o foi o=20 seguinte:
 
Primeiro =E9 preciso provar que o = problema tem=20 solu=E7=E3o:
Sendo a sequ=EAncia = A=3Da,b,c,d,e,f,g,...,n (em que a =E9=20 o 1=BA termo, b =E9 o 2=BA, at=E9 n que =E9 o n-=E9simo = termo).
Sendo B o conjunto de algoritmos que = geram=20 sequ=EAncias em que quando o primeiro termo =E9 a, o segundo =E9 b e = sendo C o=20 conjunto dos algoritmos para os quais se o 1=BA termo =E9 b, o 2=BA =E9 = C.
 
O algoritmo que descreve a sequ=EAncia = A ent=E3o=20 pertence =E0 B intersec=E7=E3o com C. B e C possuem infinitos = elementos.=20 Como a,b,c s=E3o 3 n=FAmeros aleat=F3rios, B intersec=E7=E3o com C =E9 = aleat=F3rio. A=20 intersec=E7=E3o aleat=F3ria de 2 conjuntos infinitos =E9 um conjunto = infinito.=20 Expandindo esse racioc=EDnio, existem infinitos algoritmos que perfazem = A. Logo=20 existem infinitas alternativas a serem analisadas, ou seja o problema = =E9 do tipo=20 NP.
 
Esse pensamento esta = certo?
 
J=E1 vou agradecendo
 
Andr=E9 T.
 
 
 
------=_NextPart_000_002F_01C25A9E.2E8FBDE0-- ========================================================================= Instruções para entrar na lista, sair da lista e usar a lista em http://www.mat.puc-rio.br/~nicolau/olimp/obm-l.html O administrador desta lista é ========================================================================= From owner-obm-l@sucuri.mat.puc-rio.br Thu Sep 12 21:20:02 2002 Return-Path: Received: (from majordom@localhost) by sucuri.mat.puc-rio.br (8.9.3/8.9.3) id VAA19572 for obm-l-MTTP; Thu, 12 Sep 2002 21:19:45 -0300 Received: from toole.pub1 (toole.uol.com.br [200.221.4.26]) by sucuri.mat.puc-rio.br (8.9.3/8.9.3) with ESMTP id VAA19563 for ; Thu, 12 Sep 2002 21:19:40 -0300 Received: from u2z7z2 ([200.158.144.83]) by toole.pub1 (8.9.1/8.9.1) with ESMTP id VAA19701 for ; Thu, 12 Sep 2002 21:15:19 -0300 (BRT) Message-ID: <000f01c25abb$9257b180$53909ec8@u2z7z2> From: "Wagner" To: References: <001401c2576e$7aa8d820$cc909ec8@u2z7z2> Subject: Re: [obm-l] A volta do futebol arte! (resposta do problema) Date: Thu, 12 Sep 2002 21:21:57 -0300 Organization: Wagner MIME-Version: 1.0 Content-Type: multipart/alternative; boundary="----=_NextPart_000_000C_01C25AA2.6CB68D20" X-Priority: 3 X-MSMail-Priority: Normal X-Mailer: Microsoft Outlook Express 5.50.4133.2400 X-MimeOLE: Produced By Microsoft MimeOLE V5.50.4133.2400 Sender: owner-obm-l@sucuri.mat.puc-rio.br Precedence: bulk Reply-To: obm-l@mat.puc-rio.br This is a multi-part message in MIME format. ------=_NextPart_000_000C_01C25AA2.6CB68D20 Content-Type: text/plain; charset="iso-8859-1" Content-Transfer-Encoding: quoted-printable Oi para todos Como ningu=E9m quis discutir esse problema aqui vai a resposta do = problema. Existem 2 formas principais de se resolver esse problema: 1=AA forma - Como n=E3o existem pent=E1gonos adjacentes e todos os = v=E9rtices pertencem a um pent=E1gono, portanto 5P=3D60, em que P =E9 o = n=BA de pent=E1gonos =3D> P=3D12. Como para cada hex=E1gono existem 3 = pent=E1gonos e 3 outros hex=E1gonos, 3H=3D5P =3D> 3H=3D60 =3D> H=3D20 2=AA forma - Cada aresta possui dois v=E9rtices distintos e cada = v=E9rtice possui 3 arestas distintas, logo : A=3D3V/2 =3D> A=3D90. Da = rela=E7=E3o de Euler : A+2=3DF+V =3D> F=3D32. Como F=3DH+P, H+P=3D32 ( I ). Como 3H=3D5P( II ) = ( mesma dedu=E7=E3o da 1=AA resolu=E7=E3o ). Resolvendo o sistema ( I ) = e ( II ), tem-se: H=3D20 e P=3D12. Outra forma de resolver o problema, sem usar a dedu=E7=E3o 3H=3D5P, =E9 = deduzir P como na 1=AA forma e deduzir F como na 2=AA forma, como = F=3DH+P e P=3D12, H=3D20 Andr=E9 T. ----- Original Message -----=20 From: Wagner=20 To: obm-l@mat.puc-rio.br=20 Sent: Sunday, September 08, 2002 4:32 PM Subject: [obm-l] A volta do futebol arte! Oi pessoas!!! =20 Como hoje n=E3o estou muito inspirado hoje, vou propor um problema = simples ( para n=E3o dizer rid=EDculo ): Se voc=EA pegar uma bola de futebol e achatar todas as suas faces = de modo que elas fiquem retas, voc=EA ter=E1 um poliedro com 60 v=E9rtices (em uma bola de futebol de qualidade e que n=E3o = tenha sido comprada na 25 de mar=E7o, =E9 claro) . Como a=20 maioria sabe, a costura da bola de futebol forma pent=E1gonos e = hex=E1gonos regulares, arranjados de forma que em volta de cada = pent=E1gono existem 5 hex=E1gonos e em volta de cada hex=E1gono existem 3 = pent=E1gonos e 3 hex=E1gonos. Logo quantas faces de uma bola de futebol s=E3o pentagonais e quantas s=E3o hex=E1gonais ? Andr=E9 T. ------=_NextPart_000_000C_01C25AA2.6CB68D20 Content-Type: text/html; charset="iso-8859-1" Content-Transfer-Encoding: quoted-printable
Oi para todos
 
Como ningu=E9m quis discutir esse = problema aqui vai a=20 resposta do problema.
 
Existem 2 formas principais de se = resolver=20 esse problema:
 
1=AA forma - Como n=E3o existem = pent=E1gonos adjacentes e=20 todos os v=E9rtices pertencem a um pent=E1gono, portanto 5P=3D60, em que = P =E9 o n=BA de=20 pent=E1gonos =3D> P=3D12. Como para cada hex=E1gono existem 3 = pent=E1gonos e 3 outros=20 hex=E1gonos, 3H=3D5P =3D> 3H=3D60 =3D> H=3D20
 
2=AA forma - Cada aresta possui dois = v=E9rtices=20 distintos e cada v=E9rtice possui 3 arestas distintas, logo : A=3D3V/2 = =3D> A=3D90.=20 Da rela=E7=E3o de Euler :
A+2=3DF+V =3D> F=3D32. Como F=3DH+P, = H+P=3D32 ( I ). Como=20 3H=3D5P( II ) ( mesma dedu=E7=E3o da 1=AA resolu=E7=E3o ). = Resolvendo o sistema ( I ) e=20 ( II ),   tem-se: H=3D20 e P=3D12.
 
Outra forma de resolver o problema, sem = usar a=20 dedu=E7=E3o 3H=3D5P, =E9 deduzir P como na 1=AA forma e deduzir F = como na 2=AA forma,=20 como F=3DH+P e P=3D12, H=3D20
 
 
 
Andr=E9 T.
----- Original Message -----
From:=20 Wagner
Sent: Sunday, September 08, = 2002 4:32=20 PM
Subject: [obm-l] A volta do = futebol=20 arte!

Oi pessoas!!!  
 
    Como hoje n=E3o = estou muito=20 inspirado hoje, vou propor um problema simples ( para n=E3o dizer = rid=EDculo=20 ):
 
    Se voc=EA pegar = uma bola de=20 futebol e achatar todas as suas faces de modo que elas fiquem retas, = voc=EA ter=E1=20 um poliedro
com 60 v=E9rtices (em uma bola de = futebol de=20 qualidade e que n=E3o tenha sido comprada na 25 de mar=E7o, =E9 = claro) . Como a=20
maioria sabe, a costura da bola de = futebol forma=20 pent=E1gonos e hex=E1gonos regulares, arranjados de forma que em volta = de cada=20 pent=E1gono
existem 5 hex=E1gonos e em volta de = cada hex=E1gono=20 existem 3 pent=E1gonos e 3 hex=E1gonos. Logo quantas faces de uma bola = de=20 futebol
s=E3o pentagonais e quantas s=E3o = hex=E1gonais=20 ?
 
    Andr=E9=20 T.
------=_NextPart_000_000C_01C25AA2.6CB68D20-- ========================================================================= Instruções para entrar na lista, sair da lista e usar a lista em http://www.mat.puc-rio.br/~nicolau/olimp/obm-l.html O administrador desta lista é ========================================================================= From owner-obm-l@sucuri.mat.puc-rio.br Thu Sep 12 21:43:11 2002 Return-Path: Received: (from majordom@localhost) by sucuri.mat.puc-rio.br (8.9.3/8.9.3) id VAA19935 for obm-l-MTTP; Thu, 12 Sep 2002 21:42:49 -0300 Received: from ginsberg.pub1 (ginsberg.uol.com.br [200.221.4.48]) by sucuri.mat.puc-rio.br (8.9.3/8.9.3) with ESMTP id VAA19932 for ; Thu, 12 Sep 2002 21:42:47 -0300 Received: from slap ([200.207.152.104]) by ginsberg.pub1 (8.9.1/8.9.1) with SMTP id VAA24185 for ; Thu, 12 Sep 2002 21:42:13 -0300 (BRT) Message-ID: <000d01c25abf$6f013a40$6898cfc8@slap> From: "Afemano" To: References: <003c01c259e8$e9d79660$6898cfc8@slap> Subject: Re: [obm-l] Geometria PLZ !!! Date: Thu, 12 Sep 2002 21:49:36 -0300 MIME-Version: 1.0 Content-Type: multipart/alternative; boundary="----=_NextPart_000_000A_01C25AA6.496015E0" X-Priority: 3 X-MSMail-Priority: Normal X-Mailer: Microsoft Outlook Express 6.00.2600.0000 X-MimeOLE: Produced By Microsoft MimeOLE V6.00.2600.0000 Sender: owner-obm-l@sucuri.mat.puc-rio.br Precedence: bulk Reply-To: obm-l@mat.puc-rio.br This is a multi-part message in MIME format. ------=_NextPart_000_000A_01C25AA6.496015E0 Content-Type: text/plain; charset="iso-8859-1" Content-Transfer-Encoding: quoted-printable Hehehe ningu=E9m teve saco ? Tentem ae plz !!! ----- Original Message -----=20 From: Afemano=20 To: obm-l@mat.puc-rio.br=20 Sent: Wednesday, September 11, 2002 8:14 PM Subject: [obm-l] Geometria PLZ !!! Ol=E1 galera.. algu=E9m resolve esse exerc=EDcio pra mim plz ? ( FUVEST ) A, B e P s=E3o tr=EAs pontos de uma circunfer=EAncia de = centro O e raio unit=E1rio, base de um cone reto de v=E9rtice C e altura = igual a 2. A e B s=E3o diametralmente opostos e t =E9 a reta tangente = =E0 circunfer=EAncia pelo ponto B. Determine : a ) "Eu consequi,, deixa" b )a dist=E2ncia entre a reta t e a geratriz PC, em fun=E7=E3o do = =E2ngulo B=D4P. Obs : eu fiz v=E1rias vezes o desenho, e pelo que eu enxerguei ser=E3o = necess=E1rios tr=EAs tri=E2ngulos pra achar isso.. acho que a base = est=E1 na lei dos cossenos mas n=E3o tenho certeza. Entre a reta t e a = geratriz PC formar=E1 uma outra reta perpendiuclar as duas ( =E9 isso = que ele est=E1 pedindo n=E3o ? ), chame de ponto D onde essa reta = encontra a reta t e de ponto E onde essa reta encontra a geratriz... = s=F3 pra ficar mais f=E1cil. Nesse caso queremos achar o valor de DE, = obrigad=E3o !!!!!!!!!! ------=_NextPart_000_000A_01C25AA6.496015E0 Content-Type: text/html; charset="iso-8859-1" Content-Transfer-Encoding: quoted-printable
Hehehe ningu=E9m teve saco ? Tentem ae = plz=20 !!!
----- Original Message -----
From:=20 Afemano=20
Sent: Wednesday, September 11, = 2002 8:14=20 PM
Subject: [obm-l] Geometria PLZ = !!!

Ol=E1 galera.. algu=E9m resolve esse = exerc=EDcio pra=20 mim plz ?
 
( FUVEST ) A, B e=20 P s=E3o tr=EAs pontos de uma circunfer=EAncia de = centro=20 O e raio unit=E1rio, base de um cone reto de = v=E9rtice=20 C e altura igual a 2. A e = B=20 s=E3o diametralmente opostos e t =E9 a reta tangente =E0 = circunfer=EAncia=20 pelo ponto B. Determine :
a ) "Eu consequi,, = deixa"
b )a dist=E2ncia entre a reta = t e a=20 geratriz PC, em fun=E7=E3o do =E2ngulo=20 B=D4P.
 
Obs : eu fiz v=E1rias vezes o = desenho, e pelo que=20 eu enxerguei ser=E3o necess=E1rios tr=EAs tri=E2ngulos pra achar = isso.. acho que a=20 base est=E1 na lei dos cossenos mas n=E3o tenho certeza. Entre a reta = t e=20 a geratriz PC formar=E1 uma outra reta perpendiuclar = as duas (=20 =E9 isso que ele est=E1 pedindo n=E3o ? ), chame de ponto = D onde=20 essa reta encontra a reta t e de ponto E = onde essa=20 reta encontra a geratriz... s=F3 pra ficar mais f=E1cil. Nesse caso = queremos achar=20 o valor de DE, obrigad=E3o=20 !!!!!!!!!!
------=_NextPart_000_000A_01C25AA6.496015E0-- ========================================================================= Instruções para entrar na lista, sair da lista e usar a lista em http://www.mat.puc-rio.br/~nicolau/olimp/obm-l.html O administrador desta lista é ========================================================================= From owner-obm-l@sucuri.mat.puc-rio.br Thu Sep 12 22:34:36 2002 Return-Path: Received: (from majordom@localhost) by sucuri.mat.puc-rio.br (8.9.3/8.9.3) id WAA20855 for obm-l-MTTP; Thu, 12 Sep 2002 22:33:10 -0300 Received: from home.iis.com.br (mail.iis.com.br [200.202.96.2]) by sucuri.mat.puc-rio.br (8.9.3/8.9.3) with ESMTP id WAA20851 for ; Thu, 12 Sep 2002 22:33:07 -0300 Received: from Marcio (rio-tc0-tty68.iis.com.br [200.202.98.68]) by home.iis.com.br (8.11.6/8.11.6/1.1.1.16) with SMTP id g8D1YC405450 for ; Thu, 12 Sep 2002 22:34:13 -0300 Message-ID: <001701c25ac5$ce4ff940$4462cac8@epq.ime.eb.br> From: "Marcio" To: References: <003c01c259e8$e9d79660$6898cfc8@slap> <000d01c25abf$6f013a40$6898cfc8@slap> Subject: Re: [obm-l] Geometria PLZ !!! Date: Thu, 12 Sep 2002 22:35:10 -0300 MIME-Version: 1.0 Content-Type: multipart/alternative; boundary="----=_NextPart_000_0014_01C25AAC.A7578240" X-Priority: 3 X-MSMail-Priority: Normal X-Mailer: Microsoft Outlook Express 5.50.4133.2400 X-MimeOLE: Produced By Microsoft MimeOLE V5.50.4133.2400 X-AntiVirus: Antivirus for sendmail by Petr Rehor Sender: owner-obm-l@sucuri.mat.puc-rio.br Precedence: bulk Reply-To: obm-l@mat.puc-rio.br This is a multi-part message in MIME format. ------=_NextPart_000_0014_01C25AAC.A7578240 Content-Type: text/plain; charset="iso-8859-1" Content-Transfer-Encoding: quoted-printable Se bem me lembro, essa questao tmb ja caiu num vestibular do IME, com = exatamente os mesmos dados.=20 Na minha opiniao, a maneira mais simples de achar distancia entre = retas reversas eh usando vetores (bom, com certeza eh a maneira mais = simples de resolver esse tipo de problema usando apenas ascii.. :) Considere o cone num sistema de eixos Oxyz tq C =3D (0,0,2); A =3D = (-1, 0,0); B=3D(1,0,0), P =3D (cosx, senx, 0). A reta t tem direcao a=3D(0,1,0), enqto que a geratriz PC tem direcao = b=3D(cosx,senx,-2). Note que o vetor axb (produto vetorial) eh = perpendicular a a e a b, e portanto da a direcao da perpendicular comum. = Pegando um ponto X qualquer de PC e outro Y qq de t, temos um vetor = cuja projecao na perpendicular comum da exatamente a distancia entre as = duas retas reversas (afinal, as projecoes de X e Y em axb sao exatamente = os extremos do segmento limitado pelas retas reversas na perpendicular = comum!) Agora, as contas: axb=3D(2, 0, -cosx). Tomando X=3DC=3D(0,0,2) e = Y=3DB=3D(1,0,0), temos XY=3D(1,0,-2) e=20 d =3D |proj| =3D |XY . axb| / /|axb| =3D |2 + 2cosx| / = sqrt[4+cos^2(x)] ObS: usei q proj u,v =3D [(u.v)/(v.v)]v, e seu modulo eh |u.v| / |v| Abracos, Marcio ----- Original Message -----=20 From: Afemano=20 To: obm-l@mat.puc-rio.br=20 Sent: Wednesday, September 11, 2002 8:14 PM Subject: [obm-l] Geometria PLZ !!! Ol=E1 galera.. algu=E9m resolve esse exerc=EDcio pra mim plz ? ( FUVEST ) A, B e P s=E3o tr=EAs pontos de uma circunfer=EAncia de = centro O e raio unit=E1rio, base de um cone reto de v=E9rtice C e altura = igual a 2. A e B s=E3o diametralmente opostos e t =E9 a reta tangente = =E0 circunfer=EAncia pelo ponto B. Determine : a ) "Eu consequi,, deixa" b )a dist=E2ncia entre a reta t e a geratriz PC, em fun=E7=E3o do = =E2ngulo B=D4P. Obs : eu fiz v=E1rias vezes o desenho, e pelo que eu enxerguei = ser=E3o necess=E1rios tr=EAs tri=E2ngulos pra achar isso.. acho que a = base est=E1 na lei dos cossenos mas n=E3o tenho certeza. Entre a reta t = e a geratriz PC formar=E1 uma outra reta perpendiuclar as duas ( =E9 = isso que ele est=E1 pedindo n=E3o ? ), chame de ponto D onde essa reta = encontra a reta t e de ponto E onde essa reta encontra a geratriz... = s=F3 pra ficar mais f=E1cil. Nesse caso queremos achar o valor de DE, = obrigad=E3o !!!!!!!!!! ------=_NextPart_000_0014_01C25AAC.A7578240 Content-Type: text/html; charset="iso-8859-1" Content-Transfer-Encoding: quoted-printable
   Se bem me lembro, essa = questao tmb ja=20 caiu num vestibular do IME, com exatamente os mesmos dados. =
   Na minha opiniao, a = maneira mais=20 simples de achar distancia entre retas reversas eh usando vetores (bom, = com=20 certeza eh a maneira mais simples de resolver esse tipo de problema = usando=20 apenas ascii.. :)
   Considere o cone num = sistema de=20 eixos Oxyz tq C =3D (0,0,2); A =3D (-1, 0,0); B=3D(1,0,0), P =3D = (cosx, senx,=20 0).
   A reta t tem direcao = a=3D(0,1,0), enqto=20 que a geratriz PC tem direcao b=3D(cosx,senx,-2). Note que o vetor axb = (produto=20 vetorial) eh perpendicular a a e a b, e portanto da a direcao da = perpendicular=20 comum.
   Pegando um ponto X = qualquer de PC e=20 outro Y qq de t, temos um vetor cuja projecao na perpendicular comum da=20 exatamente a distancia entre as duas retas reversas (afinal, as = projecoes de X e=20 Y em axb sao exatamente os extremos do segmento limitado pelas retas = reversas na=20 perpendicular comum!)
   Agora, as contas: = axb=3D(2, 0, -cosx).=20 Tomando X=3DC=3D(0,0,2) e Y=3DB=3D(1,0,0), temos XY=3D(1,0,-2) e =
d =3D |proj<XY, axb>| =3D = |XY . axb| /=20 /|axb| =3D |2 + 2cosx| / sqrt[4+cos^2(x)]
ObS: usei q proj u,v =3D = [(u.v)/(v.v)]v, e seu modulo=20 eh |u.v| / |v|
 
Abracos,
Marcio
 
----- Original Message -----
From:=20 Afemano=20
Sent: Wednesday, September = 11, 2002=20 8:14 PM
Subject: [obm-l] Geometria = PLZ=20 !!!

Ol=E1 galera.. algu=E9m resolve = esse exerc=EDcio pra=20 mim plz ?
 
( FUVEST ) A, B e=20 P s=E3o tr=EAs pontos de uma circunfer=EAncia de = centro=20 O e raio unit=E1rio, base de um cone reto de = v=E9rtice=20 C e altura igual a 2. A e=20 B s=E3o diametralmente opostos e t =E9 a = reta tangente=20 =E0 circunfer=EAncia pelo ponto B. Determine = :
a ) "Eu consequi,, = deixa"
b )a dist=E2ncia entre a reta = t e a=20 geratriz PC, em fun=E7=E3o do =E2ngulo=20 B=D4P.
 
Obs : eu fiz v=E1rias vezes o = desenho, e pelo que=20 eu enxerguei ser=E3o necess=E1rios tr=EAs tri=E2ngulos pra achar = isso.. acho que a=20 base est=E1 na lei dos cossenos mas n=E3o tenho certeza. Entre a = reta t=20 e a geratriz PC formar=E1 uma outra reta = perpendiuclar as=20 duas ( =E9 isso que ele est=E1 pedindo n=E3o ? ), chame de ponto=20 D onde essa reta encontra a reta t e de = ponto=20 E onde essa reta encontra a geratriz... s=F3 pra = ficar mais=20 f=E1cil. Nesse caso queremos achar o valor de DE, = obrigad=E3o=20 !!!!!!!!!!
------=_NextPart_000_0014_01C25AAC.A7578240-- ========================================================================= Instruções para entrar na lista, sair da lista e usar a lista em http://www.mat.puc-rio.br/~nicolau/olimp/obm-l.html O administrador desta lista é ========================================================================= From owner-obm-l@sucuri.mat.puc-rio.br Fri Sep 13 00:14:18 2002 Return-Path: Received: (from majordom@localhost) by sucuri.mat.puc-rio.br (8.9.3/8.9.3) id AAA22085 for obm-l-MTTP; Fri, 13 Sep 2002 00:13:50 -0300 Received: from shannon.bol.com.br (shannon.bol.com.br [200.221.24.13]) by sucuri.mat.puc-rio.br (8.9.3/8.9.3) with ESMTP id AAA22081 for ; Fri, 13 Sep 2002 00:13:48 -0300 Received: from bol.com.br (200.221.24.140) by shannon.bol.com.br (5.1.071) id 3D60E4DC006CC9AC for obm-l@mat.puc-rio.br; Fri, 13 Sep 2002 00:14:41 -0300 Date: Fri, 13 Sep 2002 00:12:44 -0300 Message-Id: Subject: Re:[obm-l] circuito IME MIME-Version: 1.0 Content-Type: multipart/mixed; boundary="_=__=_XaM3_Boundary.1031886764.2A.698094.42.13647.52.42.101010.1369173307" From: "rafaelc.l" To: obm-l@mat.puc-rio.br X-XaM3-API-Version: 2.4.3.4.4 X-SenderIP: 172.187.60.110 Sender: owner-obm-l@sucuri.mat.puc-rio.br Precedence: bulk Reply-To: obm-l@mat.puc-rio.br --_=__=_XaM3_Boundary.1031886764.2A.698094.42.13647.52.42.101010.1369173307 Content-Type: text/plain;charset="iso-8859-1" Content-Transfer-Encoding: quoted-printable Pelo menos a equa=E7=E3o de van der valls est=E1 no programa. Concordo com o q vc falou, o vestibular deles mede muito mais t=E9cnica do que reflex=E3o e racioc=EDcio pr=E1tico. Estes seus colegas falaram que n=E3o valeu a pena o curso pelo esfor=E7o que fizeram. porque? o q tem de errado no curso do IME? Reconhe=E7o que seu curso da USP =E9 muito bom (n=E3o fa=E7o a=ED pq cai hist=F3ria e geografia, que sou p=E9ssimo) mas IME e ITA s=E3o mais puxados, pelo que ouvi falar. Mas talvez n=E3o sejam melhores, devido ao seu rigor t=E9cnico e por ser muito direcionado. O que vc acha disso? __________________________________________________________________________ AcessoBOL, s=F3 R$ 9,90! O menor pre=E7o do mercado! Assine j=E1! http://www.bol.com.br/acessobol --_=__=_XaM3_Boundary.1031886764.2A.698094.42.13647.52.42.101010.1369173307 Content-Type: text/plain; name="000006FW" Content-Transfer-Encoding: base64 T2xoYSAsIHJlYWxtZW50ZSBlc3NhIGhpc3RvcmlhIGRlIGRpc2NpdGlyIG8gbml2ZWwgZG8g aW1lIG5vIHNldSB2ZXN0aWJ1bGFyIHBhcmVjZSBxdWUgdOEgbG9uZ2UgZGUgYWNhYmFyLg0K Q2ljdWl0byBSTEMgLCBldSBz8yBjb21lY2VpIGEgdmVyIGVtIGxhYi4gZGUgZmlzaWNhIElJ SSBlIG11aXRhIGNvaXNhIHF1ZSBjYWkgbOEgbm8gZXhhbWUgZGVsZXMgbmVtIG1lc21vIGVz dOEgbm8gcHJhZ3JhbWEgLCBxZG8gcHJlc3RlaSBlc3R1ZGVpIHByYSBjYXJhbWJhIGUgY2hl Z2FuZG8gbOEgLCBuYW8gcGFzc2VpLi5Jc3NvIHBvcnF1ZSBwZWRpciBjb2lzYXMgcXVlIG5h byBlc3RhbyBubyBwcm9ncmFtYSBwYXJlY2Ugc2VyIG5vcm1hbCBwcmEgZWxlcywgb3UgY29p c2FzIGRlIG5pdmVsIHN1cGVyaW9yLg0KQSBlcS4gZGUgVmFuIGRlciBXYWFscyBkZSBnYXNl cyBjYWl1IHVtYSB2ZXogZSBpc3NvIGV1IHRiIHPzIHZpIG5vIHNlZ3VuZG8gYW5vIGRlIGZh Y3VsZGFkZS4gIEEgdmVyZGFkZSBxdWUgc/MgcGFzc2EgcXVlbSBmYXogYXF1ZWxlcyBjdXJz aW5ob3MgZXNwZWNpYWxpemFkb3MgbmVzc2UgdGlwbyBkZSBleGFtZS4gDQpJc3NvIHNlIOkg cXVlIHZhbGUgYSBwZW5hIHNlIG1hdGFyIGRlIGVzdHVkYXIgcHJhIHVtIGV4YW1lIHF1ZSBu YW8gbWVkZSBxdWVtIHJlYWxtZW50ZSDpIG1lbGhvciwgc/MgbWVkZSBvIGNvbmhlY2ltZW50 byBkZSBjb2lzYXMgZXh0cmVtYW1lbnRlIGVzcGVjaWZpY2FzIGUgcXVlIG8gY2FyYSB2YWkg dmVyIGRlIG5vdm8gbmEgZmFjdS4NCkNvbmhl528gZG9pcyBjYXJhcyBxdWUgZXN0dWRhbSBj b21pZ28gcXVlIHBhc3NhcmFtICwgZml6ZXJhbSBhbGd1bnMgbWVzZXMgZSB2aXJhbSBxdWUg bmFvIHZhbGV1IG8gZXNmb3LnbywgcHJpbmNpcGFsbWVudGUgcG9yIGNhdXNhIGRhIGVzdHJ1 dHVyYSBkbyBjdXJzbyBxdWUgZW5jb250cmFyYW0uDQpNYXMg6SBpc3NvIHF1ZW0gZm9yIHBy ZXN0YXIgLCBib2Egc29ydGUgIQ0KDQoNCiJBIG1hdGVt4XRpY2Eg6SBhIG1haXMgYWx0YSBk YXMgY2nqbmNpYXMsIG8gZG9tIG1haXMgYWx0byBxdWUgb3MgZGV1c2VzIGRlcmFtIGFvcyBo b21lbnMiDQogICAgICAgICAgICAgICAgICBBcnF1aW1lZGVzKDI4N2EuQy4tMjEyYS5DLikN Cg0KS2VuamkgWW9zaGl0YWtpICANCkVuZ2VuaGFyaWEvUG9saS1VU1ANCg0KDQoNCg0KRW1h aWwgZ3LhdGlzIGVtIHBvcnR1Z3XqcyBubyBodHRwOi8vd3d3LmVjb25vZmluYW5jZS5jb20v emFwbWFpbC5odG1sDQpfX19fX19fX19fX19fX19fX19fX19fX19fX19fX19fX19fX19fX19f X19fX19fX19fX19fX19fX19fXw0KR2V0IHlvdXIgb3duIFdlYi1CYXNlZCBFLW1haWwgU2Vy dmljZSBhdCBodHRwOi8vd3d3Lnp6bi5jb20NCj09PT09PT09PT09PT09PT09PT09PT09PT09 PT09PT09PT09PT09PT09PT09PT09PT09PT09PT09PT09PT09PT09PT09PT09PT0NCkluc3Ry def1ZXMgcGFyYSBlbnRyYXIgbmEgbGlzdGEsIHNhaXIgZGEgbGlzdGEgZSB1c2FyIGEgbGlz dGEgZW0NCmh0dHA6Ly93d3cubWF0LnB1Yy1yaW8uYnIvfm5pY29sYXUvb2xpbXAvb2JtLWwu aHRtbA0KTyBhZG1pbmlzdHJhZG9yIGRlc3RhIGxpc3RhIOkgPG5pY29sYXVAbWF0LnB1Yy1y aW8uYnI+DQo9PT09PT09PT09PT09PT09PT09PT09PT09PT09PT09PT09PT09PT09PT09PT09 PT09PT09PT09PT09PT09PT09PT09PT09PT09DQo= --_=__=_XaM3_Boundary.1031886764.2A.698094.42.13647.52.42.101010.1369173307-- ========================================================================= Instruções para entrar na lista, sair da lista e usar a lista em http://www.mat.puc-rio.br/~nicolau/olimp/obm-l.html O administrador desta lista é ========================================================================= From owner-obm-l@sucuri.mat.puc-rio.br Fri Sep 13 10:09:53 2002 Return-Path: Received: (from majordom@localhost) by sucuri.mat.puc-rio.br (8.9.3/8.9.3) id KAA26781 for obm-l-MTTP; Fri, 13 Sep 2002 10:08:54 -0300 Received: from c0mailgw04.prontomail.com (mailgw.prontomail.com [209.185.149.10]) by sucuri.mat.puc-rio.br (8.9.3/8.9.3) with ESMTP id KAA26777 for ; Fri, 13 Sep 2002 10:08:51 -0300 Received: from c2web29 (209.185.149.10) by c0mailgw04.prontomail.com (NPlex 6.0.045) id 3D81DDD8000011E2 for obm-l@mat.puc-rio.br; Fri, 13 Sep 2002 06:10:36 -0700 X-Version: Mailcentro 1.0 X-SenderIP: 143.107.26.3 X-SenderID: 6757770 From: "kenji yoshitaki" Message-Id: <0B41A5C7A2828144F881B2D6751B818C@kenji.economics.zzn.com> Date: Fri, 13 Sep 2002 10:05:33 -0300 X-Priority: Normal Content-Type: text/plain; charset=iso-8859-1 To: obm-l@mat.puc-rio.br Subject: Re:[obm-l] circuito IME X-Mailer: Web Based Pronto Mime-Version: 1.0 Content-Transfer-Encoding: 8bit X-MIME-Autoconverted: from quoted-printable to 8bit by sucuri.mat.puc-rio.br id KAA26778 Sender: owner-obm-l@sucuri.mat.puc-rio.br Precedence: bulk Reply-To: obm-l@mat.puc-rio.br Rafel, Bom,não quis de maneira alguma subestimar a competencia dos cursos do ime e do ita, mas o fato de serem instituiçoes militares restringe muito o relacionamento( e convenios) com as várias empresas do ramo da engenharia e serviços em geral. Isso sem contar as restriçoes academicas com áreas de humanas. Em relaçao aos meus colegas daqui da poli, um fez mais um semestre de curso lá , e outro só fez alguns meses, ambos decidiram fazer cursinho de novo pra entrar aqui, pq segundo eles nao compensava ficar mais dois anos naquela historia de trabalhar pro exercito depois da formatura. E tb por causa da crise que o exercito , aeronautica passam.. nao possibilatava grandes investimentos na estrutura dos cursos tanto do ime como do ita. Tenho um tio que é da embraer e da aula no ita, ele confirma tudo, na época dele o ita nao tinha dinheiro pra nada... Nao sei pra que área vc vai , mas cursos como engenharia eletrica e computaçao , vc deve saber que a Unicamp , por exemplo, é a melhor da america latina.. Isso por que os investimentos nao vem apenas do governo , assim como aqui na poli, as empresas fazem parcerias que elevam a estrutura dos cursos por meio de investimentos. E sobre o nível do curso , eu tb pensava que era maior no ita e ime, mas depois vi o quão dificil é fazer ( bem feito) um curso de engenharia, pq na verdade quem faz o nível é o próprio aluno. Depende apenas dele mesmo buscar conhecimento e investir na sua formaçao.Isso vale tambem, buscar o "saber" até mesmo em assuntos como história e geografia, porque nessa geraçao que passamos, a sabedoria é o maior valor que alguem pode ter. Quem dessa lista é engenheiro, matemático,etc..., deve saber o que estou dizendo. Bom é isso , qualquer outra opinião , pode mandar. Falow, um abraço! "A matemática é a mais alta das ciências, o dom mais alto que os deuses deram aos homens" Arquimedes(287a.C.-212a.C.) Kenji Yoshitaki Engenharia/Poli-USP Email grátis em português no http://www.econofinance.com/zapmail.html ___________________________________________________________ Get your own Web-Based E-mail Service at http://www.zzn.com ========================================================================= Instruções para entrar na lista, sair da lista e usar a lista em http://www.mat.puc-rio.br/~nicolau/olimp/obm-l.html O administrador desta lista é ========================================================================= From owner-obm-l@sucuri.mat.puc-rio.br Fri Sep 13 10:54:10 2002 Return-Path: Received: (from majordom@localhost) by sucuri.mat.puc-rio.br (8.9.3/8.9.3) id KAA28025 for obm-l-MTTP; Fri, 13 Sep 2002 10:53:47 -0300 Received: (from nicolau@localhost) by sucuri.mat.puc-rio.br (8.9.3/8.9.3) id KAA28020 for obm-l@mat.puc-rio.br; Fri, 13 Sep 2002 10:53:46 -0300 Date: Fri, 13 Sep 2002 10:53:45 -0300 From: "Nicolau C. Saldanha" To: obm-l@mat.puc-rio.br Subject: [obm-l] Re: =?iso-8859-1?Q?=5Bobm-l=5D_D=FAvida_sobre_sequ=EAncias_rand=F4micas_=28p?= =?iso-8859-1?Q?roblema_do_tipo_NP=29?= Message-ID: <20020913105345.A27659@sucuri.mat.puc-rio.br> References: <003201c25ab7$546ade00$53909ec8@u2z7z2> Mime-Version: 1.0 Content-Type: text/plain; charset=iso-8859-1 Content-Disposition: inline Content-Transfer-Encoding: 8bit User-Agent: Mutt/1.2.5i In-Reply-To: <003201c25ab7$546ade00$53909ec8@u2z7z2>; from timpa@uol.com.br on Thu, Sep 12, 2002 at 08:51:35PM -0300 Sender: owner-obm-l@sucuri.mat.puc-rio.br Precedence: bulk Reply-To: obm-l@mat.puc-rio.br On Thu, Sep 12, 2002 at 08:51:35PM -0300, Wagner wrote: > Oi pessoal > > Queria perguntar ao Nicolau ou a quem conseguir me resolver essa pergunta: > > Se um algoritmo pode construir uma sequência randômica, ... Confesso que não entendi bem a pergunta, mas um algoritmo *não* pode gerar uma verdadeira seqüência aleatória (randômica). O máximo que um algorimo pode fazer é gerar uma imitação aceitável (seqüência pseudo-aleatória) para alguns fins. Para que a seqüência seja aleatória deve ser impossível prever termos futuros da seqüência e o algoritmo é uma forma perfeita de prever seu próprio resultado. :-] Alguém disse que tentar gerar seqüências aleatórias por um algoritmo é "viver em pecado"... []s, N. ========================================================================= Instruções para entrar na lista, sair da lista e usar a lista em http://www.mat.puc-rio.br/~nicolau/olimp/obm-l.html O administrador desta lista é ========================================================================= From owner-obm-l@sucuri.mat.puc-rio.br Fri Sep 13 11:29:05 2002 Return-Path: Received: (from majordom@localhost) by sucuri.mat.puc-rio.br (8.9.3/8.9.3) id LAA28770 for obm-l-MTTP; Fri, 13 Sep 2002 11:27:46 -0300 Received: from ginsberg.pub1 (ginsberg.uol.com.br [200.221.4.48]) by sucuri.mat.puc-rio.br (8.9.3/8.9.3) with ESMTP id LAA28761 for ; Fri, 13 Sep 2002 11:27:42 -0300 Received: from franklin ([200.214.41.210]) by ginsberg.pub1 (8.9.1/8.9.1) with SMTP id LAA25941 for ; Fri, 13 Sep 2002 11:27:09 -0300 (BRT) From: "Franklin de Lima Marquezino" To: Subject: [obm-l] =?iso-8859-1?Q?Re:_=5Bobm-l=5D_D=FAvida_sobre_sequ=EAncias_rand=F4m?= =?iso-8859-1?Q?icas_=28problema_do_tipo_NP=29?= Date: Fri, 13 Sep 2002 11:29:04 -0300 Message-ID: <01c25b31$e913a8a0$d229d6c8@franklin> MIME-Version: 1.0 Content-Type: multipart/alternative; boundary="----=_NextPart_000_0032_01C25B18.C3C670A0" X-Priority: 3 X-MSMail-Priority: Normal X-Mailer: Microsoft Outlook Express 4.71.1712.3 X-MimeOLE: Produced By Microsoft MimeOLE V4.71.1712.3 Sender: owner-obm-l@sucuri.mat.puc-rio.br Precedence: bulk Reply-To: obm-l@mat.puc-rio.br This is a multi-part message in MIME format. ------=_NextPart_000_0032_01C25B18.C3C670A0 Content-Type: text/plain; charset="iso-8859-1" Content-Transfer-Encoding: quoted-printable >Confesso que n=E3o entendi bem a pergunta, mas um algoritmo *n=E3o* = pode >gerar uma verdadeira seq=FC=EAncia aleat=F3ria (rand=F4mica). O = m=E1ximo que >um algorimo pode fazer =E9 gerar uma imita=E7=E3o aceit=E1vel = (seq=FC=EAncia >pseudo-aleat=F3ria) para alguns fins. Para que a seq=FC=EAncia seja = aleat=F3ria >deve ser imposs=EDvel prever termos futuros da seq=FC=EAncia e o = algoritmo >=E9 uma forma perfeita de prever seu pr=F3prio resultado. :-] Por=E9m se utilizarmos as propriedades da Mec=E2nica Qu=E2ntica para = escrevermos nosso algoritmo, isso se torna poss=EDvel. Podemos, por = exemplo, colocar n =E1tomos (ou f=F3tons, ... qualquer sistema = qu=E2ntico) em estado superposto, com iguais amplitudes (fazendo uma = transformada de Hadamard), e depois simplesmente medir. Quando efetuamos = a medida, o sistema colapsa para um dos estados da superposi=E7=E3o, sem = que n=F3s possamos prever qual ser=E1 esse estado. O resultado ent=E3o, = ser=E1 um n=FAmero entre 0 e (2^n)-1. O algoritmo qu=E2ntico =E9 previs=EDvel (determin=EDstico) at=E9 o = momento da medida, depois passa a ser probabil=EDstico. Abra=E7os, Franklin -------------------------------------------------------------------------= -------------------------------------------------------------------- CBPF - Centro Brasileiro de Pesquisas F=EDsicas (Bolsista PIBIC/CNPq) LNCC - Laborat=F3rio Nacional de Computa=E7=E3o Cient=EDfica (Grupo de = Computa=E7=E3o Qu=E2ntica) =20 GFT - Grupo de F=EDsica Te=F3rica Jos=E9 Leite Lopes http://virtual01.lncc.br/dcs/links/dcs_quantum_comp.html http://gft.ucp.br/staff/franklin -------------------------------------------------------------------------= --------------------------------------------------------------------=20 ------=_NextPart_000_0032_01C25B18.C3C670A0 Content-Type: text/html; charset="iso-8859-1" Content-Transfer-Encoding: quoted-printable
>Confesso que não entendi bem a pergunta, mas um = algoritmo=20 *não* pode
>gerar uma verdadeira seqüência=20 aleatória (randômica). O máximo que
>um = algorimo pode=20 fazer é gerar uma imitação aceitável=20 (seqüência
>pseudo-aleatória) para alguns fins. = Para que=20 a seqüência seja aleatória
>deve ser = impossível=20 prever termos futuros da seqüência e o = algoritmo
>é uma=20 forma perfeita de prever seu próprio resultado. :-] 

 
    = Porém se=20 utilizarmos as propriedades da Mecânica Quântica para = escrevermos=20 nosso algoritmo, isso se torna possível. Podemos, por exemplo, = colocar n=20 átomos (ou fótons, ... qualquer sistema quântico) em = estado=20 superposto, com iguais amplitudes (fazendo uma transformada de = Hadamard), e=20 depois simplesmente medir. Quando efetuamos a medida, o sistema colapsa = para um=20 dos estados da superposição, sem que nós possamos = prever=20 qual será esse estado. O resultado então, será um=20 número entre 0 e (2^n)-1.
    O = algoritmo=20 quântico é previsível (determinístico) = até o=20 momento da medida, depois passa a ser = probabilístico.
 
  = Abraços,
       = Franklin
 
 
 
 
 
 
----------------------------------------------------------------= -------------------------------------------------------------------------= ----
CBPF=20 - Centro Brasileiro de Pesquisas Físicas (Bolsista = PIBIC/CNPq)
LNCC -=20 Laboratório Nacional de Computação = Científica (Grupo=20 de Computação Quântica) 
GFT - Grupo de=20 Física Teórica José Leite Lopes
http://= virtual01.lncc.br/dcs/links/dcs_quantum_comp.html
http://gft.ucp.br/staff/frankli= n
----------------------------------------------------------------= -------------------------------------------------------------------------= ---- 
 
------=_NextPart_000_0032_01C25B18.C3C670A0-- ========================================================================= Instruções para entrar na lista, sair da lista e usar a lista em http://www.mat.puc-rio.br/~nicolau/olimp/obm-l.html O administrador desta lista é ========================================================================= From owner-obm-l@sucuri.mat.puc-rio.br Fri Sep 13 12:26:34 2002 Return-Path: Received: (from majordom@localhost) by sucuri.mat.puc-rio.br (8.9.3/8.9.3) id MAA30081 for obm-l-MTTP; Fri, 13 Sep 2002 12:25:38 -0300 Received: from pina.terra.com.br (pina.terra.com.br [200.176.3.17]) by sucuri.mat.puc-rio.br (8.9.3/8.9.3) with ESMTP id MAA30077 for ; Fri, 13 Sep 2002 12:25:34 -0300 Received: from penha.terra.com.br (penha.terra.com.br [200.176.3.43]) by pina.terra.com.br (Postfix) with ESMTP id A1826533E4 for ; Fri, 13 Sep 2002 12:26:43 -0300 (EST) Received: from xt (200-171-248-2.customer.telesp.net.br [200.171.248.2]) (authenticated user macwad) by penha.terra.com.br (Postfix) with ESMTP id 391156827D for ; Fri, 13 Sep 2002 12:26:42 -0300 (EST) Message-ID: <00a801c25b39$fb5f5e70$0401010a@xt> From: =?iso-8859-1?Q?Vinicius_Jos=E9_Fortuna?= To: References: <003201c25ab7$546ade00$53909ec8@u2z7z2> Subject: [obm-l] =?iso-8859-1?Q?Re:_=5Bobm-l=5D_D=FAvida_sobre_sequ=EAncias_rand=F4m?= =?iso-8859-1?Q?icas_=28problema_do_tipo_NP=29?= Date: Fri, 13 Sep 2002 12:26:49 -0300 MIME-Version: 1.0 Content-Type: multipart/alternative; boundary="----=_NextPart_000_00A5_01C25B20.D58D3F00" X-Priority: 3 X-MSMail-Priority: Normal X-Mailer: Microsoft Outlook Express 6.00.2600.0000 X-MimeOLE: Produced By Microsoft MimeOLE V6.00.2600.0000 Sender: owner-obm-l@sucuri.mat.puc-rio.br Precedence: bulk Reply-To: obm-l@mat.puc-rio.br This is a multi-part message in MIME format. ------=_NextPart_000_00A5_01C25B20.D58D3F00 Content-Type: text/plain; charset="iso-8859-1" Content-Transfer-Encoding: quoted-printable No meu ponto de vista, existem infinitos algoritmos para fazer alguma = coisa. At=E9 para n=E3o fazer nada. Por exemplo, um algoritmo para calcular a+b. Pode-se fazer da maneira = tradicional, ou ent=E3o pode-se fazer um loop percorrendo os n=FAmeros, = subtraindo b e verificando se =E9 igual a 'a'. Ou ent=E3o pode-se somar 6 a 'a' somar 'b' ao resultado e depois = subtrair '6'. Agora, em vez de fazer as opera=E7=E3o acima com 6, fa=E7a com todas os = outros naturais. Como s=E3o constantes, haver=E1 infinitos algoritmos = que fazem a soma. Enfim, o meu racioc=EDnio =E9 por a=ED. Esse =E9 mais um motivo que as = classes P e NP, por exemplo, s=E3o conjuntos de problemas e n=E3o de = algoritmos. Estudar algoritmos pode ser muito mais complicado. Formalmente, em teoria da computa=E7=E3o, podemos definir um problema de = decis=E3o como uma linguagem. Um problema de decis=E3o =E9 um cuja = resposta =E9 sim ou n=E3o. Uma linguagem pode ser considerada como um = conjuntos de seq=FC=EAncias de caracteres de um alfabeto. Podemos sempre = escrever uma inst=E2ncia de um problema como uma string em um alfabeto. = Assim o problema pode ser visto como um mapeamento de tais = seq=FC=EAncias em {0,1}. A linguagem associada ao problema cont=E9m = todas as seq=FC=EAncias (e somente elas) que s=E3o mapeadas em 1. Assim todo problema de decis=E3o fica reduzido ao problema: Essa = seq=FC=EAncia pertence a essa linguagem? Isso facilita bastante a teoria. Problemas de otimiza=E7=E3o podem ser reduzidos a problemas de decis=E3o = utilizando busca bin=E1ria com perguntas do tipo: O melhor valor =E9 = melhor que k? Maiores informa=E7=F5es podem ser encontradas no texto de Cook sobre o = problema "P=3DNP?" no site www.claymath.org/prizeproblems Uma outra observa=E7=E3o =E9 que nem todo problema pertence = necessariamente a NP. Pode ser que vc n=E3o consiga um algoritmo = polinomial em uma m=E1quina n=E3o-determin=EDstica para ele. Eu n=E3o entendi a conclus=E3o: "Logo existem infinitas alternativas a = serem analisadas, ou seja o problema =E9 do tipo NP." Poderia tb me explicar o que =E9 "intersec=E7=E3o aleat=F3ria"? At=E9 mais Vinicius ----- Original Message -----=20 From: Wagner=20 To: obm-l@mat.puc-rio.br=20 Sent: Thursday, September 12, 2002 8:51 PM Subject: [obm-l] D=FAvida sobre sequ=EAncias rand=F4micas (problema do = tipo NP) Oi pessoal Queria perguntar ao Nicolau ou a quem conseguir me resolver essa = pergunta: Se um algoritmo pode construir uma sequ=EAncia rand=F4mica, uma = sequ=EAncia qualquer desse tipo com um n=FAmero finito n de termos = poderia ent=E3o ser descrita por uma infinidade de algoritmos = diferentes, e esses algoritmos podem ser todos construidos a partir de = um algoritmo.(todos os algoritmos iriam variar de problema para = problema). Ou seja uma sequ=EAncia pode ser construida por infinitos = algoritmos diferentes, mas todos esses algoritmos podem ser construidos = a partir de um mesmo algoritmo. A problema =E9 o seguinte: Dada uma sequ=EAncia qualquer, qual =E9 o = algoritmo que gera todos os outros? A minha d=FAvida =E9 que pelo o que posso ver esse problema =E9 um = problema do tipo NP (polinominal n=E3o-determin=EDstico), ou seja a sua = resposta existe mas =E9 imposs=EDvel de ser dada na pr=E1tica. O meu racioc=EDnio para chegar a essa conclus=E3o foi o seguinte: Primeiro =E9 preciso provar que o problema tem solu=E7=E3o: Sendo a sequ=EAncia A=3Da,b,c,d,e,f,g,...,n (em que a =E9 o 1=BA = termo, b =E9 o 2=BA, at=E9 n que =E9 o n-=E9simo termo). Sendo B o conjunto de algoritmos que geram sequ=EAncias em que quando = o primeiro termo =E9 a, o segundo =E9 b e sendo C o conjunto dos = algoritmos para os quais se o 1=BA termo =E9 b, o 2=BA =E9 C. O algoritmo que descreve a sequ=EAncia A ent=E3o pertence =E0 B = intersec=E7=E3o com C. B e C possuem infinitos elementos. Como a,b,c = s=E3o 3 n=FAmeros aleat=F3rios, B intersec=E7=E3o com C =E9 aleat=F3rio. = A intersec=E7=E3o aleat=F3ria de 2 conjuntos infinitos =E9 um conjunto = infinito. Expandindo esse racioc=EDnio, existem infinitos algoritmos que = perfazem A. Logo existem infinitas alternativas a serem analisadas, ou = seja o problema =E9 do tipo NP. Esse pensamento esta certo? J=E1 vou agradecendo Andr=E9 T. ------=_NextPart_000_00A5_01C25B20.D58D3F00 Content-Type: text/html; charset="iso-8859-1" Content-Transfer-Encoding: quoted-printable
No meu ponto de vista, existem = infinitos algoritmos=20 para fazer alguma coisa. At=E9 para n=E3o fazer nada.
 
Por exemplo, um algoritmo = para calcular a+b.=20 Pode-se fazer da maneira tradicional, ou ent=E3o pode-se fazer um loop = percorrendo=20 os n=FAmeros, subtraindo b e verificando se =E9 igual a = 'a'.
Ou ent=E3o pode-se somar 6 a 'a' somar = 'b' ao=20 resultado e depois subtrair '6'.
Agora, em vez de fazer as opera=E7=E3o = acima com 6,=20 fa=E7a com todas os outros naturais. Como s=E3o constantes, haver=E1 = infinitos=20 algoritmos que fazem a soma.
 
Enfim, o meu racioc=EDnio =E9 por a=ED. = Esse =E9 mais um=20 motivo que as classes P e NP, por exemplo, s=E3o conjuntos de = problemas e n=E3o=20 de algoritmos. Estudar algoritmos pode ser muito mais = complicado.
 
Formalmente, em teoria da = computa=E7=E3o, podemos=20 definir um problema de decis=E3o como uma linguagem. Um problema de = decis=E3o =E9 um=20 cuja resposta =E9 sim ou n=E3o. Uma linguagem pode ser considerada como = um conjuntos=20 de seq=FC=EAncias de caracteres de um alfabeto. Podemos sempre escrever = uma=20 inst=E2ncia de um problema como uma string em um alfabeto. Assim = o problema=20 pode ser visto como um mapeamento de tais seq=FC=EAncias em {0,1}. A = linguagem=20 associada ao problema cont=E9m todas as seq=FC=EAncias (e somente elas) = que s=E3o=20 mapeadas em 1.
 
Assim todo problema de decis=E3o fica = reduzido ao=20 problema: Essa seq=FC=EAncia pertence a essa = linguagem?
Isso facilita bastante a = teoria.
 
Problemas de otimiza=E7=E3o podem ser = reduzidos a=20 problemas de decis=E3o utilizando busca bin=E1ria com perguntas do tipo: = O melhor=20 valor =E9 melhor que k?
Maiores informa=E7=F5es podem ser = encontradas no texto=20 de Cook sobre o problema "P=3DNP?" no site www.claymath.org/prizeprob= lems
 
Uma outra observa=E7=E3o =E9 que nem = todo problema=20 pertence necessariamente a NP. Pode ser que vc n=E3o consiga um = algoritmo=20 polinomial em uma m=E1quina n=E3o-determin=EDstica para = ele.
 
Eu n=E3o entendi a conclus=E3o: "Logo = existem infinitas=20 alternativas a serem analisadas, ou seja o problema =E9 do tipo = NP."
 
Poderia tb me explicar o que =E9 = "intersec=E7=E3o=20 aleat=F3ria"?
 
At=E9 mais
 
Vinicius
----- Original Message -----
From:=20 Wagner
Sent: Thursday, September 12, = 2002 8:51=20 PM
Subject: [obm-l] D=FAvida sobre = sequ=EAncias=20 rand=F4micas (problema do tipo NP)

Oi pessoal
 
Queria perguntar ao Nicolau ou a quem = conseguir=20 me resolver essa pergunta:
 
Se um algoritmo pode construir uma = sequ=EAncia=20 rand=F4mica, uma sequ=EAncia qualquer desse tipo com um n=FAmero = finito n de=20 termos poderia ent=E3o ser descrita por uma infinidade de = algoritmos=20 diferentes, e esses algoritmos podem ser todos construidos a partir de = um=20 algoritmo.(todos os algoritmos iriam variar de problema para = problema).=20 Ou seja uma sequ=EAncia pode ser construida por infinitos algoritmos = diferentes,=20 mas todos esses algoritmos podem ser construidos a partir de um mesmo=20 algoritmo.
 
A problema =E9 o seguinte: Dada uma = sequ=EAncia=20 qualquer, qual =E9 o algoritmo que gera todos os outros?
 
A minha d=FAvida =E9 que pelo o que = posso ver esse=20 problema =E9 um problema do tipo NP (polinominal = n=E3o-determin=EDstico), ou seja a=20 sua resposta existe mas =E9 imposs=EDvel de ser dada na = pr=E1tica.
 
O meu racioc=EDnio para chegar a essa = conclus=E3o foi=20 o seguinte:
 
Primeiro =E9 preciso provar que o = problema tem=20 solu=E7=E3o:
Sendo a sequ=EAncia = A=3Da,b,c,d,e,f,g,...,n (em que a=20 =E9 o 1=BA termo, b =E9 o 2=BA, at=E9 n que =E9 o n-=E9simo = termo).
Sendo B o conjunto de algoritmos que = geram=20 sequ=EAncias em que quando o primeiro termo =E9 a, o segundo =E9 b e = sendo C o=20 conjunto dos algoritmos para os quais se o 1=BA termo =E9 b, o 2=BA = =E9=20 C.
 
O algoritmo que descreve a = sequ=EAncia A ent=E3o=20 pertence =E0 B intersec=E7=E3o com C. B e C possuem = infinitos elementos.=20 Como a,b,c s=E3o 3 n=FAmeros aleat=F3rios, B intersec=E7=E3o com C =E9 = aleat=F3rio. A=20 intersec=E7=E3o aleat=F3ria de 2 conjuntos infinitos =E9 um conjunto = infinito.=20 Expandindo esse racioc=EDnio, existem infinitos algoritmos que = perfazem A. Logo=20 existem infinitas alternativas a serem analisadas, ou seja o problema = =E9 do=20 tipo NP.
 
Esse pensamento esta = certo?
 
J=E1 vou agradecendo
 
Andr=E9 = T.
------=_NextPart_000_00A5_01C25B20.D58D3F00-- ========================================================================= Instruções para entrar na lista, sair da lista e usar a lista em http://www.mat.puc-rio.br/~nicolau/olimp/obm-l.html O administrador desta lista é ========================================================================= From owner-obm-l@sucuri.mat.puc-rio.br Fri Sep 13 13:36:28 2002 Return-Path: Received: (from majordom@localhost) by sucuri.mat.puc-rio.br (8.9.3/8.9.3) id NAA31334 for obm-l-MTTP; Fri, 13 Sep 2002 13:34:41 -0300 Received: from web12906.mail.yahoo.com (web12906.mail.yahoo.com [216.136.174.73]) by sucuri.mat.puc-rio.br (8.9.3/8.9.3) with SMTP id NAA31330 for ; Fri, 13 Sep 2002 13:34:37 -0300 Message-ID: <20020913163544.95341.qmail@web12906.mail.yahoo.com> Received: from [200.206.103.3] by web12906.mail.yahoo.com via HTTP; Fri, 13 Sep 2002 13:35:44 ART Date: Fri, 13 Sep 2002 13:35:44 -0300 (ART) From: =?iso-8859-1?q?Johann=20Peter=20Gustav=20Lejeune=20Dirichlet?= Subject: Re: [obm-l] Ajuda Algebra linear (Off Topic) To: obm-l@mat.puc-rio.br In-Reply-To: <20020911174444.132933EB52@zeus.opendf.com.br> MIME-Version: 1.0 Content-Type: multipart/alternative; boundary="0-1244131820-1031934944=:94939" Content-Transfer-Encoding: 8bit Sender: owner-obm-l@sucuri.mat.puc-rio.br Precedence: bulk Reply-To: obm-l@mat.puc-rio.br --0-1244131820-1031934944=:94939 Content-Type: text/plain; charset=iso-8859-1 Content-Transfer-Encoding: 8bit Tem um artigo do Carlos Yuzo Shine(meu idolo!)no site da OBM semana olimpica.Vai querer? 498 - Artur Costa Steiner wrote:Se você quiser uma aplicação prática de álgebra linear, eu lhe dou uma, pois trabalho com isso: otimização da operação do sistema elétrico brasileiro, isto é, o processo de decidir quanto você deve gerar em cada usina hidrelétrica e em cada usina termelétrica, bem como quando cadaegião do país deve enviar para outra, de modo a minimizar o custo total da operação. Isto é resolvido através de um algoritmo de programação dinâmica estocástica. Há modelos de curto prazo, utilizados pelo ONS e outros de longo prazo, utilizados para horizontes de mais de 5 anos. O problema não é linear, mas algumas de suas partes podem ser aproximadas muito bem por funcões objetivo e restrições lineares. Esta parte do problema utiliza Programação Linear, a qual é uma aplicação da Álgebra Linear, baseia-se em matrizes, bases de um espaço vetorial, dependência linear, etc. No caso que estou citando, usamos o Simplex, desenvolvido na década de 60 por George Dantzig, nos EUA. Embora hoje existam outros algoritmos para resolução de problemas lineares, baseados em pontos interiores, o Simplex continua sendo uma boa opção. Já utilizei também Programação Linear em uma planilha Excel para otimizar a expansão de um sistema térmico no Norte do Brasil, o qual ajudava a decidir que tipos de unidades deveriam ser implantadas. Artur ========================================================================= Instruções para entrar na lista, sair da lista e usar a lista em http://www.mat.puc-rio.br/~nicolau/olimp/obm-l.html O administrador desta lista é =========================================================================m --------------------------------- Yahoo! PageBuilder - O super editor para criação de sites: é grátis, fácil e rápido. --0-1244131820-1031934944=:94939 Content-Type: text/html; charset=iso-8859-1 Content-Transfer-Encoding: 8bit

Tem um artigo do Carlos Yuzo Shine(meu idolo!)no site da OBM semana olimpica.Vai querer?

 498 - Artur Costa Steiner wrote:

Se você quiser uma aplicação prática de álgebra linear, eu lhe dou uma,
pois trabalho com isso: otimização da operação do sistema elétrico
brasileiro, isto é, o processo de decidir quanto você deve gerar em
cada usina hidrelétrica e em cada usina termelétrica, bem como quando
cadaegião do país deve enviar para outra, de modo a minimizar o custo
total da operação. Isto é resolvido através de um algoritmo de
programação dinâmica estocástica. Há modelos de curto prazo, utilizados
pelo ONS e outros de longo prazo, utilizados para horizontes de mais de
5 anos.

O problema não é linear, mas algumas de suas partes podem ser
aproximadas muito bem por funcões objetivo e restrições lineares. Esta
parte do problema utiliza Programação Linear, a qual é uma aplicação
da Álgebra Linear, baseia-se em matrizes, bases de um espaço vetorial,
dependência linear, etc. No caso que estou citando, usamos o Simplex,
desenvolvido na década de 60 por George Dantzig, nos EUA. Embora hoje
existam outros algoritmos para resolução de problemas lineares,
baseados em pontos interiores, o Simplex continua sendo uma boa opção.

Já utilizei também Programação Linear em uma planilha Excel para
otimizar a expansão de um sistema térmico no Norte do Brasil, o qual
ajudava a decidir que tipos de unidades deveriam ser implantadas.

Artur
=========================================================================
Instruções para entrar na lista, sair da lista e usar a lista em
http://www.mat.puc-rio.br/~nicolau/olimp/obm-l.html
O administrador desta lista é
=========================================================================
m


Yahoo! PageBuilder - O super editor para criação de sites: é grátis, fácil e rápido. --0-1244131820-1031934944=:94939-- ========================================================================= Instruções para entrar na lista, sair da lista e usar a lista em http://www.mat.puc-rio.br/~nicolau/olimp/obm-l.html O administrador desta lista é ========================================================================= From owner-obm-l@sucuri.mat.puc-rio.br Fri Sep 13 14:30:30 2002 Return-Path: Received: (from majordom@localhost) by sucuri.mat.puc-rio.br (8.9.3/8.9.3) id OAA32561 for obm-l-MTTP; Fri, 13 Sep 2002 14:29:25 -0300 Received: from zeus.opendf.com.br (zeus.opengate.com.br [200.181.71.10]) by sucuri.mat.puc-rio.br (8.9.3/8.9.3) with ESMTP id OAA32555 for ; Fri, 13 Sep 2002 14:29:23 -0300 Received: from localhost (localhost.opengate.com.br [127.0.0.1]) by zeus.opendf.com.br (Postfix) with ESMTP id 33DF43EA7C for ; Fri, 13 Sep 2002 14:30:27 -0300 (BRT) Received: by zeus.opendf.com.br (Postfix, from userid 48) id 82F833EA9C; Fri, 13 Sep 2002 14:30:26 -0300 (BRT) From: "498 - Artur Costa Steiner" To: obm-l@mat.puc-rio.br Subject: Re: [obm-l] Ajuda Algebra linear (Off Topic) X-Mailer: NeoMail 1.25 X-IPAddress: 200.252.155.2 MIME-Version: 1.0 Content-Type: text/plain; charset=iso-8859-1 Message-Id: <20020913173026.82F833EA9C@zeus.opendf.com.br> Date: Fri, 13 Sep 2002 14:30:26 -0300 (BRT) X-Virus-Scanned: by AMaViS new-20020517 Sender: owner-obm-l@sucuri.mat.puc-rio.br Precedence: bulk Reply-To: obm-l@mat.puc-rio.br > > > Tem um artigo do Carlos Yuzo Shine(meu idolo!)no site da OBM semana olimpica.Vai querer? Quero sim! Artur ========================================================================= Instruções para entrar na lista, sair da lista e usar a lista em http://www.mat.puc-rio.br/~nicolau/olimp/obm-l.html O administrador desta lista é ========================================================================= From owner-obm-l@sucuri.mat.puc-rio.br Fri Sep 13 14:53:34 2002 Return-Path: Received: (from majordom@localhost) by sucuri.mat.puc-rio.br (8.9.3/8.9.3) id OAA00728 for obm-l-MTTP; Fri, 13 Sep 2002 14:52:27 -0300 Received: from web12904.mail.yahoo.com (web12904.mail.yahoo.com [216.136.174.71]) by sucuri.mat.puc-rio.br (8.9.3/8.9.3) with SMTP id OAA00724 for ; Fri, 13 Sep 2002 14:52:23 -0300 Message-ID: <20020913175333.64271.qmail@web12904.mail.yahoo.com> Received: from [200.206.103.3] by web12904.mail.yahoo.com via HTTP; Fri, 13 Sep 2002 14:53:33 ART Date: Fri, 13 Sep 2002 14:53:33 -0300 (ART) From: =?iso-8859-1?q?Johann=20Peter=20Gustav=20Lejeune=20Dirichlet?= Subject: Re: [obm-l] Circulo de 9 pontos e reta de Simson To: obm-l@mat.puc-rio.br In-Reply-To: MIME-Version: 1.0 Content-Type: multipart/alternative; boundary="0-1838736391-1031939613=:63561" Content-Transfer-Encoding: 8bit Sender: owner-obm-l@sucuri.mat.puc-rio.br Precedence: bulk Reply-To: obm-l@mat.puc-rio.br --0-1838736391-1031939613=:63561 Content-Type: text/plain; charset=iso-8859-1 Content-Transfer-Encoding: 8bit Milhares.Nao da pra dizer todas.Sobre Simson pegue no site da obm. leonardo mattos wrote:Ola pessoal, Gostaria muito de saber quais seriam as propriedades do circulo de 9 pontos e da reta de Simson. Um abraço,Leonardo _________________________________________________________________ MSN Photos é a maneira mais fácil e prática de editar e compartilhar sua fotos: http://photos.msn.com.br ========================================================================= Instruções para entrar na lista, sair da lista e usar a lista em http://www.mat.puc-rio.br/~nicolau/olimp/obm-l.html O administrador desta lista é ========================================================================= --------------------------------- Yahoo! PageBuilder - O super editor para criação de sites: é grátis, fácil e rápido. --0-1838736391-1031939613=:63561 Content-Type: text/html; charset=iso-8859-1 Content-Transfer-Encoding: 8bit

Milhares.Nao da pra dizer todas.Sobre Simson pegue no site da obm.

 leonardo mattos wrote:

Ola pessoal,

Gostaria muito de saber quais seriam as propriedades do circulo de 9 pontos
e da reta de Simson.
Um abraço,Leonardo





_________________________________________________________________
MSN Photos é a maneira mais fácil e prática de editar e compartilhar sua
fotos: http://photos.msn.com.br

=========================================================================
Instruções para entrar na lista, sair da lista e usar a lista em
http://www.mat.puc-rio.br/~nicolau/olimp/obm-l.html
O administrador desta lista é
=========================================================================


Yahoo! PageBuilder - O super editor para criação de sites: é grátis, fácil e rápido. --0-1838736391-1031939613=:63561-- ========================================================================= Instruções para entrar na lista, sair da lista e usar a lista em http://www.mat.puc-rio.br/~nicolau/olimp/obm-l.html O administrador desta lista é ========================================================================= From owner-obm-l@sucuri.mat.puc-rio.br Fri Sep 13 15:33:30 2002 Return-Path: Received: (from majordom@localhost) by sucuri.mat.puc-rio.br (8.9.3/8.9.3) id PAA01729 for obm-l-MTTP; Fri, 13 Sep 2002 15:32:58 -0300 Received: from puma.unisys.com.br (smtp.unisys.com.br [200.220.64.7]) by sucuri.mat.puc-rio.br (8.9.3/8.9.3) with ESMTP id PAA01723 for ; Fri, 13 Sep 2002 15:32:51 -0300 Received: from josefran (riohiper01p223.uninet.com.br [200.220.2.223]) by puma.unisys.com.br (8.12.3/8.12.3) with SMTP id g8DIXq4u023124 for ; Fri, 13 Sep 2002 15:33:54 -0300 (EST) X-Spam-Filter: check_local@puma.unisys.com.br by digitalanswers.org Message-ID: <00cd01c25b54$bcc1e780$df02dcc8@josefran> From: "Jose Francisco Guimaraes Costa" To: "obm-l" Subject: [obm-l] =?iso-8859-1?Q?En:_=5Bobm-l=5D_Re:_=5Bobm-l=5D_D=FAvida_sobre_sequ=EAncia?= =?iso-8859-1?Q?s_rand=F4micas_=28problema_do_tipo_NP=29?= Date: Fri, 13 Sep 2002 15:38:13 -0300 MIME-Version: 1.0 Content-Type: text/plain; charset="iso-8859-1" Content-Transfer-Encoding: 8bit X-Priority: 3 X-MSMail-Priority: Normal X-Mailer: Microsoft Outlook Express 5.00.2314.1300 X-MimeOLE: Produced By Microsoft MimeOLE V5.00.2314.1300 Sender: owner-obm-l@sucuri.mat.puc-rio.br Precedence: bulk Reply-To: obm-l@mat.puc-rio.br Uma seqüência realmente randômica é a seqüência de dígitos que compõem números como pi e a base dos logaritmos naturais. JF -----Mensagem Original----- De: Nicolau C. Saldanha Para: Enviada em: Sexta-feira, 13 de Setembro de 2002 10:53 Assunto: [obm-l] Re: [obm-l] Dúvida sobre sequências randômicas (problema do tipo NP) > On Thu, Sep 12, 2002 at 08:51:35PM -0300, Wagner wrote: > > Oi pessoal > > > > Queria perguntar ao Nicolau ou a quem conseguir me resolver essa pergunta: > > > > Se um algoritmo pode construir uma sequência randômica, ... > > Confesso que não entendi bem a pergunta, mas um algoritmo *não* pode > gerar uma verdadeira seqüência aleatória (randômica). O máximo que > um algorimo pode fazer é gerar uma imitação aceitável (seqüência > pseudo-aleatória) para alguns fins. Para que a seqüência seja aleatória > deve ser impossível prever termos futuros da seqüência e o algoritmo > é uma forma perfeita de prever seu próprio resultado. :-] > > Alguém disse que tentar gerar seqüências aleatórias por um algoritmo > é "viver em pecado"... > > []s, N. > ========================================================================= > Instruções para entrar na lista, sair da lista e usar a lista em > http://www.mat.puc-rio.br/~nicolau/olimp/obm-l.html > O administrador desta lista é > ========================================================================= > ========================================================================= Instruções para entrar na lista, sair da lista e usar a lista em http://www.mat.puc-rio.br/~nicolau/olimp/obm-l.html O administrador desta lista é ========================================================================= From owner-obm-l@sucuri.mat.puc-rio.br Fri Sep 13 16:00:04 2002 Return-Path: Received: (from majordom@localhost) by sucuri.mat.puc-rio.br (8.9.3/8.9.3) id PAA02428 for obm-l-MTTP; Fri, 13 Sep 2002 15:58:29 -0300 Received: from smtp-6.ig.com.br (smtp-6.ig.com.br [200.226.132.155]) by sucuri.mat.puc-rio.br (8.9.3/8.9.3) with SMTP id PAA02421 for ; Fri, 13 Sep 2002 15:58:25 -0300 From: ezer@ig.com.br Received: (qmail 32358 invoked from network); 13 Sep 2002 18:59:07 -0000 Received: from shasta022003.ig.com.br (HELO almirfam) (200.151.22.3) by smtp-6.ig.com.br with SMTP; 13 Sep 2002 18:59:07 -0000 To: obm-l@mat.puc-rio.br Date: Wed, 1 Jan 1997 00:48:15 -0200 MIME-Version: 1.0 Subject: =?ISO-8859-1?Q?Re:_[obm-l]_Re:_[obm-l]_N=FAmeros_rand=F4micos?= Message-ID: <32C9B44F.25016.18020A@localhost> In-reply-to: <01c25a9e$fec5be60$ce29d6c8@franklin> X-mailer: Pegasus Mail for Windows (v4.01) Content-type: text/plain; charset=ISO-8859-1 Content-description: Mail message body Content-Transfer-Encoding: 8bit X-MIME-Autoconverted: from Quoted-printable to 8bit by sucuri.mat.puc-rio.br id PAA02422 Sender: owner-obm-l@sucuri.mat.puc-rio.br Precedence: bulk Reply-To: obm-l@mat.puc-rio.br On 12 Sep 2002 at 17:57, Franklin de Lima Marquezino wrote: > Já ouvi dizer que algumas pessoas ultimamente estavam utilizando informações da rede > (número de pessoas conectadas em um instante, por exemplo) para gerar as sequências. Mas de > qualquer forma os computadores clássicos somente geram números pseudo-aleatórios. Por outro > lado, um computador quântico será bem eficiente em gerar sequências de números randômicos. > > Espero ter contribuído. Caso eu tenha dito alguma besteira, sintam-se à vontade para me > corrigir. > > Abraços, > Franklin Sim, suas observacoes tiveram bastante propriedade e acrescentaram à discussao. Ja que voce tocou no assunto, e dada suas experiencias com Computacao Quantica, gostaria de saber como seria um Computador Quantico? Jah existe algum projeto de tentativa de implementacao de algum? Como eh que funcionaria? Quais sao as maiores barreiras que se interpoe a criacao de um computador quantico? Desde ja agradeco qualquer informacao, Ezer Fernandes "A estrada vai sempre em frente" (Bilbo Baggins) ========================================================================= Instruções para entrar na lista, sair da lista e usar a lista em http://www.mat.puc-rio.br/~nicolau/olimp/obm-l.html O administrador desta lista é ========================================================================= From owner-obm-l@sucuri.mat.puc-rio.br Fri Sep 13 17:05:25 2002 Return-Path: Received: (from majordom@localhost) by sucuri.mat.puc-rio.br (8.9.3/8.9.3) id RAA03806 for obm-l-MTTP; Fri, 13 Sep 2002 17:02:52 -0300 Received: from traven9.pub1 (traven9.uol.com.br [200.221.4.35]) by sucuri.mat.puc-rio.br (8.9.3/8.9.3) with ESMTP id RAA03802 for ; Fri, 13 Sep 2002 17:02:50 -0300 Received: from franklin ([200.214.74.85]) by traven9.pub1 (8.9.1/8.9.1) with SMTP id RAA11632 for ; Fri, 13 Sep 2002 17:05:39 -0300 (BRT) From: "Franklin de Lima Marquezino" To: Subject: [obm-l] =?iso-8859-1?Q?Re:_=5Bobm-l=5D_Re:_=5Bobm-l=5D_N=FAmeros_rand=F4micos?= Date: Fri, 13 Sep 2002 17:04:10 -0300 Message-ID: <01c25b60$b93bfea0$554ad6c8@franklin> MIME-Version: 1.0 Content-Type: multipart/alternative; boundary="----=_NextPart_000_001E_01C25B47.93EEC6A0" X-Priority: 3 X-MSMail-Priority: Normal X-Mailer: Microsoft Outlook Express 4.71.1712.3 X-MimeOLE: Produced By Microsoft MimeOLE V4.71.1712.3 Sender: owner-obm-l@sucuri.mat.puc-rio.br Precedence: bulk Reply-To: obm-l@mat.puc-rio.br This is a multi-part message in MIME format. ------=_NextPart_000_001E_01C25B47.93EEC6A0 Content-Type: text/plain; charset="iso-8859-1" Content-Transfer-Encoding: quoted-printable >Sim, suas observacoes tiveram bastante propriedade e acrescentaram =E0 = discussao. > > >Ja que voce tocou no assunto, e dada suas experiencias com Computacao >Quantica, gostaria de saber como seria um Computador Quantico? >Jah existe algum projeto de tentativa de implementacao de algum? >Como eh que funcionaria? Quais sao as maiores barreiras que se >interpoe a criacao de um computador quantico? > >Desde ja agradeco qualquer informacao, > >Ezer Fernandes Um computador qu=E2ntico utilizaria as propriedades da Mec=E2nica = Qu=E2ntica e vez da Mec=E2nica Newtoniana. Assim sendo, os bits = qu=E2nticos (chamados "qubits", de quantum bits) poderiam ser 0, 1 ou = uma mistura estat=EDtica destes dois. Justamente essa superposi=E7=E3o = de estados =E9 um dos maiores respons=E1veis pelas vantagens do = computador qu=E2ntico. Ela permite que se se possa, por exemplo, = calcular o valor de uma fun=E7=E3o em mais de um ponto ao mesmo tempo. = As aplica=E7=F5es seriam muito interessantes, como por exemplo, simular = sistemas qu=E2nticos (o que n=E3o se consegue fazer de forma eficiente = em um computador cl=E1ssico, cf. R.Feynman), criptografia completamente = segura, gera=E7=E3o de sequ=EAncias num=E9ricas aleat=F3rias, = fatora=E7=E3o de inteiros e busca de dados em listas n=E3o-ordenada. Existem sim projetos de realiza=E7=E3o f=EDsica, por=E9m estes = ainda n=E3o est=E3o muito avan=E7ados. No Rio de Janeiro mesmo (no CBPF) = tem um grupo de faz algumas experi=EAncias com resson=E2ncia magn=E9tica = nuclear. As maiores barreiras s=E3o na parte pr=E1tica, na = realiza=E7=E3o experimental, j=E1 que =E9 muito dif=EDcil controlar um = n=FAmero grande qubits, e tamb=E9m por que estes precisam ficar muito = bem isolados do meio, caso contr=E1rio podem interagir com o ambiente = provocando o que se chama "descoer=EAncia" (decoherence). Mas isso = est=E1 come=E7ando a mudar. J=E1 ouvi dizer que recentemente tem surgido = algumas novas t=E9cnicas que permitem o controle de um n=FAmero = razo=E1vel de qubits. Na parte te=F3rica (onde eu estou) o mais = dif=EDcil =E9 mudar a forma de pensar. Fazer um algoritmo qu=E2ntico =E9 = uma tarefa ainda MUITO mais dif=EDcil que criar um algoritmo cl=E1ssico. =C9 claro que o assunto =E9 muito mais do que isso, mas n=E3o quero = ficar me extendendo muito, j=E1 que talvez nem todos nessa lista estejam = interessados. Qualquer coisa, pode mandar perguntas em off. At=E9 mais, Franklin. -------------------------------------------------------------------------= -------------------------------------------------------------------- CBPF - Centro Brasileiro de Pesquisas F=EDsicas (Bolsista PIBIC/CNPq) LNCC - Laborat=F3rio Nacional de Computa=E7=E3o Cient=EDfica (Grupo de = Computa=E7=E3o Qu=E2ntica) =20 GFT - Grupo de F=EDsica Te=F3rica Jos=E9 Leite Lopes http://virtual01.lncc.br/dcs/links/dcs_quantum_comp.html http://gft.ucp.br/staff/franklin -------------------------------------------------------------------------= -------------------------------------------------------------------- =20 ------=_NextPart_000_001E_01C25B47.93EEC6A0 Content-Type: text/html; charset="iso-8859-1" Content-Transfer-Encoding: quoted-printable
>Sim, suas observacoes tiveram bastante propriedade e = acrescentaram=20 à discussao.
>
>
>Ja que voce tocou no assunto, = e dada=20 suas experiencias com Computacao
>Quantica, gostaria de saber como = seria=20 um Computador Quantico?
>Jah existe algum projeto de tentativa de=20 implementacao de algum?
>Como eh que funcionaria? Quais sao as = maiores=20 barreiras que se
>interpoe a criacao de um computador=20 quantico?
>
>Desde ja agradeco qualquer=20 informacao,
>
>Ezer Fernandes
 
 
     Um computador quântico utilizaria as = propriedades da Mecânica Quântica e vez da Mecânica=20 Newtoniana. Assim sendo, os bits quânticos (chamados = "qubits",=20 de quantum bits) poderiam ser 0, 1 ou uma mistura estatítica = destes dois.=20 Justamente essa superposição de estados é um dos = maiores=20 responsáveis pelas vantagens do computador quântico. Ela = permite=20 que se se possa, por exemplo, calcular o valor de uma = função em=20 mais de um ponto ao mesmo tempo. As aplicações seriam = muito=20 interessantes, como por exemplo, simular sistemas quânticos (o que = não se consegue fazer de forma eficiente em um computador=20 clássico, cf. R.Feynman), criptografia completamente segura,=20 geração de sequências numéricas = aleatórias,=20 fatoração de inteiros e busca de dados em listas=20 não-ordenada.
     Existem sim projetos de = realização=20 física, porém estes ainda não estão muito=20 avançados. No Rio de Janeiro mesmo (no CBPF) tem um grupo de faz = algumas=20 experiências com ressonância magnética nuclear. As = maiores=20 barreiras são na parte prática, na = realização=20 experimental, já que é muito difícil controlar um=20 número grande qubits, e também por que estes precisam = ficar muito=20 bem isolados do meio, caso contrário podem interagir com o = ambiente=20 provocando o que se chama "descoerência" (decoherence). = Mas isso=20 está começando a mudar. Já ouvi dizer que = recentemente tem=20 surgido algumas novas técnicas que permitem o controle de um=20 número razoável de qubits. Na parte teórica (onde = eu estou)=20 o mais difícil é mudar a forma de pensar. Fazer um = algoritmo=20 quântico é uma tarefa ainda MUITO mais difícil que = criar um=20 algoritmo clássico.
    É claro que o assunto é muito mais = do que=20 isso, mas não quero ficar me extendendo muito, já que = talvez nem=20 todos nessa lista estejam interessados. Qualquer coisa, pode mandar = perguntas em=20 off.
 
     Até mais,
 
          =20 Franklin.
 
 
 

------------------------------------------------------------= -------------------------------------------------------------------------= --------
CBPF=20 - Centro Brasileiro de Pesquisas Físicas (Bolsista = PIBIC/CNPq)
LNCC -=20 Laboratório Nacional de Computação = Científica (Grupo=20 de Computação Quântica) 
GFT - Grupo de=20 Física Teórica José Leite Lopes
http://= virtual01.lncc.br/dcs/links/dcs_quantum_comp.html
http://gft.ucp.br/staff/frankli= n
----------------------------------------------------------------= -------------------------------------------------------------------------= ----   
------=_NextPart_000_001E_01C25B47.93EEC6A0-- ========================================================================= Instruções para entrar na lista, sair da lista e usar a lista em http://www.mat.puc-rio.br/~nicolau/olimp/obm-l.html O administrador desta lista é ========================================================================= From owner-obm-l@sucuri.mat.puc-rio.br Fri Sep 13 17:07:50 2002 Return-Path: Received: (from majordom@localhost) by sucuri.mat.puc-rio.br (8.9.3/8.9.3) id RAA03857 for obm-l-MTTP; Fri, 13 Sep 2002 17:06:33 -0300 Received: from hotmail.com (f155.sea2.hotmail.com [207.68.165.155]) by sucuri.mat.puc-rio.br (8.9.3/8.9.3) with ESMTP id RAA03853 for ; Fri, 13 Sep 2002 17:06:29 -0300 Received: from mail pickup service by hotmail.com with Microsoft SMTPSVC; Fri, 13 Sep 2002 13:07:35 -0700 Received: from 32.94.119.254 by sea2fd.sea2.hotmail.msn.com with HTTP; Fri, 13 Sep 2002 20:07:34 GMT X-Originating-IP: [32.94.119.254] From: "Paulo Santa Rita" To: obm-l@mat.puc-rio.br Subject: [obm-l] =?iso-8859-1?B?UmU6IFtvYm0tbF0gUmU6IFtvYm0tbF0gTvptZXJvcyByYW5k9G1pY29z?= Date: Fri, 13 Sep 2002 20:07:34 +0000 Mime-Version: 1.0 Content-Type: text/plain; charset=iso-8859-1; format=flowed Message-ID: X-OriginalArrivalTime: 13 Sep 2002 20:07:35.0012 (UTC) FILETIME=[33483240:01C25B61] Sender: owner-obm-l@sucuri.mat.puc-rio.br Precedence: bulk Reply-To: obm-l@mat.puc-rio.br Ola Ezer e demais colegas desta lista ... OBM-L, Eu sei que ja se falou muito sobre numeros randomicos ( aleatorios ) e computacao quantica. Todso sabem que esta computacao e, em ultima analise, o uso das propriedades quanticas na computacao, tal como usar os spin dos eletron para representar os bit ( para cima = 0, para baixo=1 ). Mas tudo nao passa de projetos e existem diversos deles em producao. Eu so li alguma coisa e portanto nao entendo do assunto a ponto de me arriscar a falar sobre ele. Mas selecionei o texto abaixo pra voce, de um ganhador de premio nobel que trabalha nesta area : Ganhador do Nobel diz como será 2010 A humanidade está ingressando numa década incrível. Quando ela terminar, todas as formas de comunicação do mundo - sejam de voz, dados ou imagens - poderão ser transmitidas através de uma única fibra óptica, em apenas um segundo. Um transistor poderá ser feito de apenas um átomo. Os microprocessadores mais avançados não serão maiores do que uma molécula. Antes dessa miniaturização extrema, teremos transistores de plástico, produzindo aparelhos e computadores da espessura do tecido de nossa roupa. Curvaremos raios de luz dentro de pastilhas de silício. Faremos a seleção de comprimentos de onda por meio de cristais fotônicos. A banda larga e as transmissões em alta velocidade - inclusive sem fio - triunfarão sobre todas as formas de comunicações atuais. A nanotecnologia permitirá a produção de minúsculos robôs, com apenas alguns bilionésimos de milímetro (ou nanômetros) de altura. Para quê? Para combater doenças, limpar nossas artérias, despoluir o ambiente ou realizar tarefas totalmente impossíveis neste fim de milênio. Os pesquisadores aprenderão muito com os processos físicos, químicos e biológicos, para aplicá-los ou imitá-los em novas formas de computação. Em menos de dez anos, talvez possamos chegar ao computador quântico, capaz de superar tudo que a imaginação humana já concebeu. Eis aí uma síntese das previsões feitas por Horst Störmer - cientista dos Laboratórios Bell e um dos três laureados do Prêmio Nobel de Física de 1998, em entrevista exclusiva ao Estado. Limite é o átomo - Nos anos 70, Gordon Moore, um dos cientistas fundadores da Intel, previu que o número de componentes microeletrônicos por chip iria dobrar a cada 18 meses. Cumprida ao longo dos últimos 20 anos, sua previsão ficou conhecida com o nome de lei de Moore. Mas essa tendência à miniaturização talvez não prevaleça muito além de 2010. Para Störmer, não há dúvida de que o limite desse processo é o átomo: "Talvez possamos ir além e chegar ao núcleo atômico, mas aí os problemas são muito maiores. A questão central é saber quando iremos atingir o limite do átomo, o que, a meu ver, é mais uma questão econômica do que tecnológica. Alcançaremos esse limite quando uma nova geração de chips se tornar mais cara do que sua antecessora." Tecnologicamente, não se pode prever quando será atingido aquele limite atômico. Mas algumas projeções mais confiáveis estimam que esses transistores de dimensões moleculares já possam ser produzidos por volta de 2010. Computador quântico - Para Horst Störmer, temos de pensar hoje em múltiplas formas de computação, desde a computação analógica até a óptica, quântica e a biológica - sem falar, é claro, da velha computação digital: "Para muitos, a computação quântica é muito parecida com a computação analógica. O importante, em última instância, é buscarmos a solução mais adequada a cada setor ou grupo de problemas. Por outras palavras, é preciso escolher entre a melhor solução digital ou o melhor modelo." Störmer não acha adequada a expressão computação óptica. Prefere falar em optical switching ou comutação óptica: "Hoje já não pensamos num computador óptico. Isso porque a transmissão se tornou tão mais importante, que nós preferimos comutar (ligar) de modo muito mais eficiente um sinal óptico, desviando-o de um roteador para outro." Para o Prêmio Nobel, a computação quântica é algo totalmente novo: "TENHO DITO EM MINHAS PALESTRAS QUE POUCAS PESSOAS NO MUNDO REALMENTE ENTENDEM O QUE SEJA COMPUTACAO QUANTICA. Pense no que significa manipular elétrons, um a um ou em grupos, juntando-os ou separando-os. Isso ainda é algo impensável. A rigor, o que temos hoje são algumas idéias em discussão, alguns caminhos a seguir. Mas é, sem dúvida, um tema fascinante." Outra tendência é a do computador biológico. "Não estamos falando de nada rigorosamente biológico, vivo ou orgânico. Mas, sim, de moléculas de DNA e proteínas. Não estou afirmando também que teremos redes neurais apoiadas sobre estruturas biológicas, até porque estas são lentas. A biologia, no entanto, pode nos fornecer modelos, materiais básicos e componentes." Um abraco Paulo Santa Rita 6,1800,130902 >From: ezer@ig.com.br >Reply-To: obm-l@mat.puc-rio.br >To: obm-l@mat.puc-rio.br >Subject: Re: [obm-l] Re: [obm-l] Números randômicos >Date: Wed, 1 Jan 1997 00:48:15 -0200 > >On 12 Sep 2002 at 17:57, Franklin de Lima Marquezino wrote: > > Já ouvi dizer que algumas pessoas ultimamente estavam utilizando >informações da rede > > (número de pessoas conectadas em um instante, por exemplo) para gerar as >sequências. Mas de > > qualquer forma os computadores clássicos somente geram números >pseudo-aleatórios. Por outro > > lado, um computador quântico será bem eficiente em gerar sequências de >números randômicos. > > > > Espero ter contribuído. Caso eu tenha dito alguma besteira, sintam-se à >vontade para me > > corrigir. > > > > Abraços, > > Franklin > >Sim, suas observacoes tiveram bastante propriedade e acrescentaram à >discussao. > > >Ja que voce tocou no assunto, e dada suas experiencias com Computacao >Quantica, gostaria de saber como seria um Computador Quantico? >Jah existe algum projeto de tentativa de implementacao de algum? >Como eh que funcionaria? Quais sao as maiores barreiras que se >interpoe a criacao de um computador quantico? > >Desde ja agradeco qualquer informacao, > >Ezer Fernandes > > >"A estrada vai sempre em frente" > (Bilbo Baggins) >========================================================================= >Instruções para entrar na lista, sair da lista e usar a lista em >http://www.mat.puc-rio.br/~nicolau/olimp/obm-l.html >O administrador desta lista é >========================================================================= ========================================================================= Instruções para entrar na lista, sair da lista e usar a lista em http://www.mat.puc-rio.br/~nicolau/olimp/obm-l.html O administrador desta lista é ========================================================================= From owner-obm-l@sucuri.mat.puc-rio.br Fri Sep 13 18:04:05 2002 Return-Path: Received: (from majordom@localhost) by sucuri.mat.puc-rio.br (8.9.3/8.9.3) id SAA05434 for obm-l-MTTP; Fri, 13 Sep 2002 18:02:35 -0300 Received: from smtp.brturbo.com (smtp2.brturbo.com [200.199.201.30]) by sucuri.mat.puc-rio.br (8.9.3/8.9.3) with ESMTP id SAA05430 for ; Fri, 13 Sep 2002 18:02:32 -0300 Received: from x (200-181-95-057-bsace7011.dsl.telebrasilia.net.br [200.181.95.57]) by smtp.brturbo.com (Postfix) with SMTP id 297B813C51B for ; Fri, 13 Sep 2002 18:03:24 -0300 (BRT) Message-ID: <000a01c25b69$0a083940$0201a8c0@x> From: "Jeremias de Paula Eduardo" To: Subject: [obm-l] =?iso-8859-1?Q?quest=E3o_de_polinomios?= Date: Fri, 13 Sep 2002 18:03:40 -0300 MIME-Version: 1.0 Content-Type: multipart/alternative; boundary="----=_NextPart_000_0007_01C25B4F.E3D92CC0" X-Priority: 3 X-MSMail-Priority: Normal X-Mailer: Microsoft Outlook Express 5.00.2919.6600 X-MimeOLE: Produced By Microsoft MimeOLE V5.00.2919.6600 Sender: owner-obm-l@sucuri.mat.puc-rio.br Precedence: bulk Reply-To: obm-l@mat.puc-rio.br This is a multi-part message in MIME format. ------=_NextPart_000_0007_01C25B4F.E3D92CC0 Content-Type: text/plain; charset="iso-8859-1" Content-Transfer-Encoding: quoted-printable Quem puder me ajudar a resolver essa a=ED eu agrade=E7o. (ITA-62) Se x^3+px+q =E9 divis=EDvel por x^2+ax+b e x^2+rx+s, = demonstrar que b=3D -r(a+r) Jeremias de Paula Eduardo ------=_NextPart_000_0007_01C25B4F.E3D92CC0 Content-Type: text/html; charset="iso-8859-1" Content-Transfer-Encoding: quoted-printable
Quem puder me ajudar a resolver essa = a=ED eu=20 agrade=E7o.
 
(ITA-62) Se x^3+px+q =E9 divis=EDvel = por x^2+ax+b e =20 x^2+rx+s, demonstrar que b=3D -r(a+r)
 
Jeremias de Paula=20 Eduardo
------=_NextPart_000_0007_01C25B4F.E3D92CC0-- ========================================================================= Instruções para entrar na lista, sair da lista e usar a lista em http://www.mat.puc-rio.br/~nicolau/olimp/obm-l.html O administrador desta lista é ========================================================================= From owner-obm-l@sucuri.mat.puc-rio.br Fri Sep 13 18:04:05 2002 Return-Path: Received: (from majordom@localhost) by sucuri.mat.puc-rio.br (8.9.3/8.9.3) id SAA05407 for obm-l-MTTP; Fri, 13 Sep 2002 18:02:17 -0300 Received: (from nicolau@localhost) by sucuri.mat.puc-rio.br (8.9.3/8.9.3) id SAA05402 for obm-l@mat.puc-rio.br; Fri, 13 Sep 2002 18:02:17 -0300 Date: Fri, 13 Sep 2002 18:02:17 -0300 From: "Nicolau C. Saldanha" To: obm-l@mat.puc-rio.br Subject: [obm-l] Re: =?iso-8859-1?Q?=5Bobm-l=5D_Re:_=5Bobm-l=5D_D=FAvida_sobre_sequ=EAncias_r?= =?iso-8859-1?Q?and=F4micas_=28problema_do_tipo_NP=29?= Message-ID: <20020913180217.A5248@sucuri.mat.puc-rio.br> References: <01c25b31$e913a8a0$d229d6c8@franklin> Mime-Version: 1.0 Content-Type: text/plain; charset=iso-8859-1 Content-Disposition: inline Content-Transfer-Encoding: 8bit User-Agent: Mutt/1.2.5i In-Reply-To: <01c25b31$e913a8a0$d229d6c8@franklin>; from fmarquezino@uol.com.br on Fri, Sep 13, 2002 at 11:29:04AM -0300 Sender: owner-obm-l@sucuri.mat.puc-rio.br Precedence: bulk Reply-To: obm-l@mat.puc-rio.br On Fri, Sep 13, 2002 at 11:29:04AM -0300, Franklin de Lima Marquezino wrote: > >Confesso que não entendi bem a pergunta, mas um algoritmo *não* pode > >gerar uma verdadeira seqüência aleatória (randômica). O máximo que > >um algorimo pode fazer é gerar uma imitação aceitável (seqüência > >pseudo-aleatória) para alguns fins. Para que a seqüência seja aleatória > >deve ser impossível prever termos futuros da seqüência e o algoritmo > >é uma forma perfeita de prever seu próprio resultado. :-] > > > Porém se utilizarmos as propriedades da Mecânica Quântica para > escrevermos nosso algoritmo, isso se torna possível. Podemos, por > exemplo, colocar n átomos (ou fótons, ... qualquer sistema quântico) em > estado superposto, com iguais amplitudes (fazendo uma transformada de > Hadamard), e depois simplesmente medir. Quando efetuamos a medida, o > sistema colapsa para um dos estados da superposição, sem que nós possamos > prever qual será esse estado. O resultado então, será um número entre 0 e > (2^n)-1. O algoritmo quântico é previsível (determinístico) até o > momento da medida, depois passa a ser probabilístico. Para mim isto não se chama 'algoritmo'; algoritmo é algo que pode ser feito por uma máquina de Turing. []s, N. ========================================================================= Instruções para entrar na lista, sair da lista e usar a lista em http://www.mat.puc-rio.br/~nicolau/olimp/obm-l.html O administrador desta lista é ========================================================================= From owner-obm-l@sucuri.mat.puc-rio.br Fri Sep 13 18:10:16 2002 Return-Path: Received: (from majordom@localhost) by sucuri.mat.puc-rio.br (8.9.3/8.9.3) id SAA05606 for obm-l-MTTP; Fri, 13 Sep 2002 18:08:42 -0300 Received: from traven9.pub1 (traven9.uol.com.br [200.221.4.35]) by sucuri.mat.puc-rio.br (8.9.3/8.9.3) with ESMTP id SAA05601 for ; Fri, 13 Sep 2002 18:08:39 -0300 Received: from slap ([200.207.152.104]) by traven9.pub1 (8.9.1/8.9.1) with SMTP id SAA08201 for ; Fri, 13 Sep 2002 18:11:30 -0300 (BRT) Message-ID: <001001c25b6a$b511d160$6898cfc8@slap> From: "Afemano" To: References: <003c01c259e8$e9d79660$6898cfc8@slap> <000d01c25abf$6f013a40$6898cfc8@slap> <001701c25ac5$ce4ff940$4462cac8@epq.ime.eb.br> Subject: Re: [obm-l] Geometria PLZ !!! Date: Fri, 13 Sep 2002 18:15:37 -0300 MIME-Version: 1.0 Content-Type: multipart/alternative; boundary="----=_NextPart_000_000D_01C25B51.8F5FE420" X-Priority: 3 X-MSMail-Priority: Normal X-Mailer: Microsoft Outlook Express 6.00.2600.0000 X-MimeOLE: Produced By Microsoft MimeOLE V6.00.2600.0000 Sender: owner-obm-l@sucuri.mat.puc-rio.br Precedence: bulk Reply-To: obm-l@mat.puc-rio.br This is a multi-part message in MIME format. ------=_NextPart_000_000D_01C25B51.8F5FE420 Content-Type: text/plain; charset="iso-8859-1" Content-Transfer-Encoding: quoted-printable Nossa cara valeu mesmo..=20 mas o que =E9 "proj" e "sqrt" ? hehehe desculpe mas sou meio novo por = aqui :D Abra=E7os... ----- Original Message -----=20 From: Marcio=20 To: obm-l@mat.puc-rio.br=20 Sent: Thursday, September 12, 2002 10:35 PM Subject: Re: [obm-l] Geometria PLZ !!! Se bem me lembro, essa questao tmb ja caiu num vestibular do IME, = com exatamente os mesmos dados.=20 Na minha opiniao, a maneira mais simples de achar distancia entre = retas reversas eh usando vetores (bom, com certeza eh a maneira mais = simples de resolver esse tipo de problema usando apenas ascii.. :) Considere o cone num sistema de eixos Oxyz tq C =3D (0,0,2); A =3D = (-1, 0,0); B=3D(1,0,0), P =3D (cosx, senx, 0). A reta t tem direcao a=3D(0,1,0), enqto que a geratriz PC tem = direcao b=3D(cosx,senx,-2). Note que o vetor axb (produto vetorial) eh = perpendicular a a e a b, e portanto da a direcao da perpendicular comum. = Pegando um ponto X qualquer de PC e outro Y qq de t, temos um vetor = cuja projecao na perpendicular comum da exatamente a distancia entre as = duas retas reversas (afinal, as projecoes de X e Y em axb sao exatamente = os extremos do segmento limitado pelas retas reversas na perpendicular = comum!) Agora, as contas: axb=3D(2, 0, -cosx). Tomando X=3DC=3D(0,0,2) e = Y=3DB=3D(1,0,0), temos XY=3D(1,0,-2) e=20 d =3D |proj| =3D |XY . axb| / /|axb| =3D |2 + 2cosx| / = sqrt[4+cos^2(x)] ObS: usei q proj u,v =3D [(u.v)/(v.v)]v, e seu modulo eh |u.v| / |v| Abracos, Marcio ----- Original Message -----=20 From: Afemano=20 To: obm-l@mat.puc-rio.br=20 Sent: Wednesday, September 11, 2002 8:14 PM Subject: [obm-l] Geometria PLZ !!! Ol=E1 galera.. algu=E9m resolve esse exerc=EDcio pra mim plz ? ( FUVEST ) A, B e P s=E3o tr=EAs pontos de uma circunfer=EAncia de = centro O e raio unit=E1rio, base de um cone reto de v=E9rtice C e altura = igual a 2. A e B s=E3o diametralmente opostos e t =E9 a reta tangente = =E0 circunfer=EAncia pelo ponto B. Determine : a ) "Eu consequi,, deixa" b )a dist=E2ncia entre a reta t e a geratriz PC, em fun=E7=E3o do = =E2ngulo B=D4P. Obs : eu fiz v=E1rias vezes o desenho, e pelo que eu enxerguei = ser=E3o necess=E1rios tr=EAs tri=E2ngulos pra achar isso.. acho que a = base est=E1 na lei dos cossenos mas n=E3o tenho certeza. Entre a reta t = e a geratriz PC formar=E1 uma outra reta perpendiuclar as duas ( =E9 = isso que ele est=E1 pedindo n=E3o ? ), chame de ponto D onde essa reta = encontra a reta t e de ponto E onde essa reta encontra a geratriz... = s=F3 pra ficar mais f=E1cil. Nesse caso queremos achar o valor de DE, = obrigad=E3o !!!!!!!!!! ------=_NextPart_000_000D_01C25B51.8F5FE420 Content-Type: text/html; charset="iso-8859-1" Content-Transfer-Encoding: quoted-printable
Nossa cara valeu mesmo..
mas o que =E9 "proj" e "sqrt" ? hehehe = desculpe mas=20 sou meio novo por aqui :D
 
Abra=E7os...
----- Original Message -----
From:=20 Marcio
Sent: Thursday, September 12, = 2002 10:35=20 PM
Subject: Re: [obm-l] Geometria = PLZ=20 !!!

   Se bem me lembro, essa = questao tmb=20 ja caiu num vestibular do IME, com exatamente os mesmos dados. =
   Na minha opiniao, a = maneira mais=20 simples de achar distancia entre retas reversas eh usando vetores = (bom, com=20 certeza eh a maneira mais simples de resolver esse tipo = de problema=20 usando apenas ascii.. :)
   Considere o cone = num sistema de=20 eixos Oxyz tq C =3D (0,0,2); A =3D (-1, 0,0); B=3D(1,0,0), P =3D = (cosx, senx,=20 0).
   A reta t tem direcao = a=3D(0,1,0),=20 enqto que a geratriz PC tem direcao b=3D(cosx,senx,-2). Note que o = vetor axb=20 (produto vetorial) eh perpendicular a a e a b, e portanto da a direcao = da=20 perpendicular comum.
   Pegando um ponto X = qualquer de PC e=20 outro Y qq de t, temos um vetor cuja projecao na perpendicular comum = da=20 exatamente a distancia entre as duas retas reversas (afinal, as = projecoes de X=20 e Y em axb sao exatamente os extremos do segmento limitado pelas retas = reversas na perpendicular comum!)
   Agora, as contas: = axb=3D(2, 0, -cosx).=20 Tomando X=3DC=3D(0,0,2) e Y=3DB=3D(1,0,0), temos XY=3D(1,0,-2) e =
d =3D |proj<XY, axb>| =3D = |XY . axb|=20 / /|axb| =3D |2 + 2cosx| / sqrt[4+cos^2(x)]
ObS: usei q proj u,v =3D = [(u.v)/(v.v)]v, e seu=20 modulo eh |u.v| / |v|
 
Abracos,
Marcio
 
----- Original Message -----
From:=20 Afemano=20
Sent: Wednesday, September = 11, 2002=20 8:14 PM
Subject: [obm-l] Geometria = PLZ=20 !!!

Ol=E1 galera.. algu=E9m resolve = esse exerc=EDcio=20 pra mim plz ?
 
( FUVEST ) A, B = e=20 P s=E3o tr=EAs pontos de uma circunfer=EAncia de = centro=20 O e raio unit=E1rio, base de um cone reto de = v=E9rtice=20 C e altura igual a 2. A e=20 B s=E3o diametralmente opostos e t =E9 a = reta=20 tangente =E0 circunfer=EAncia pelo ponto B. = Determine=20 :
a ) "Eu consequi,, = deixa"
b )a dist=E2ncia entre a reta = t e a=20 geratriz PC, em fun=E7=E3o do =E2ngulo=20 B=D4P.
 
Obs : eu fiz v=E1rias vezes o = desenho, e pelo=20 que eu enxerguei ser=E3o necess=E1rios tr=EAs tri=E2ngulos pra = achar isso.. acho=20 que a base est=E1 na lei dos cossenos mas n=E3o tenho certeza. = Entre a reta=20 t e a geratriz PC formar=E1 uma outra = reta=20 perpendiuclar as duas ( =E9 isso que ele est=E1 pedindo n=E3o ? ), = chame de=20 ponto D onde essa reta encontra a reta t = e de=20 ponto E onde essa reta encontra a geratriz... = s=F3 pra=20 ficar mais f=E1cil. Nesse caso queremos achar o valor de=20 DE, obrigad=E3o=20 = !!!!!!!!!!
------=_NextPart_000_000D_01C25B51.8F5FE420-- ========================================================================= Instruções para entrar na lista, sair da lista e usar a lista em http://www.mat.puc-rio.br/~nicolau/olimp/obm-l.html O administrador desta lista é ========================================================================= From owner-obm-l@sucuri.mat.puc-rio.br Fri Sep 13 18:12:57 2002 Return-Path: Received: (from majordom@localhost) by sucuri.mat.puc-rio.br (8.9.3/8.9.3) id SAA05661 for obm-l-MTTP; Fri, 13 Sep 2002 18:11:35 -0300 Received: from hotmail.com (f25.law9.hotmail.com [64.4.9.25]) by sucuri.mat.puc-rio.br (8.9.3/8.9.3) with ESMTP id SAA05657 for ; Fri, 13 Sep 2002 18:11:31 -0300 Received: from mail pickup service by hotmail.com with Microsoft SMTPSVC; Fri, 13 Sep 2002 14:12:41 -0700 Received: from 200.190.10.185 by lw9fd.law9.hotmail.msn.com with HTTP; Fri, 13 Sep 2002 21:12:40 GMT X-Originating-IP: [200.190.10.185] From: "Rogerio Fajardo" To: obm-l@mat.puc-rio.br Subject: Re: [obm-l] Um Estranho Sentimento ... Date: Fri, 13 Sep 2002 21:12:40 +0000 Mime-Version: 1.0 Content-Type: text/plain; format=flowed Message-ID: X-OriginalArrivalTime: 13 Sep 2002 21:12:41.0447 (UTC) FILETIME=[4BB2DF70:01C25B6A] Sender: owner-obm-l@sucuri.mat.puc-rio.br Precedence: bulk Reply-To: obm-l@mat.puc-rio.br Olá, Paulo Quanto ao livro de lógica, ainda não tive tempo de estudá-lo, mas tenho participado de alguns seminários sobre forcing usando o livro de Kunen. Parece que agora ficou mais claro para mim o que voce pensou. A respeito disso proponho a seguinte pergunta: se alguém com uma brilhante aptidão matemática, mas que nunca tinha ouvido falar de números reais nem de geometria, aprende os axiomas do corpo ordenado completo, sem nenhuma interpretação geométrica, que visão ele teria dos números reais? Será que, com o tempo, ele ia perceber por si só essa interpretação geométrica? Será que ia obter uma outra visualização? Será que, nessa visualização, iria enxergar coisas que nós não enxergamos? E por aí vai... Mas o fato de atribuirmos a certas estruturas matemáticas novas visualizações que nos permitem enxergar coisas que não enxergávamos, e que ajude a associarmos coisas aparentemente totalmente distintas, já ocorre com frequência na matemática e é bem interessante. Por exemplo, o fato de pensarmos em funções como elementos de espaço vetorial e utilizarmos produto interno dado pela integral (como fazemos na Análise Funcional), nos ajudou a descobrirmos um monte de resultados importantíssimos em equações diferenciais. No fundo, será que isso não é o que voce falou sobre achar uma nova visualização de uma teoria? Quem imaginava, antes de se criar a Álgebra Linear abstrata, que as funções podiam ser vistas como vetores, inclusive com a noção de ortogonalidade, e que isso fosse tão útil? O que fizemos não foi usar a geometria para visualizarmos algo que até então ninguém pensava haver qualquer relação com geometria? Acho isso um dos fenômenos mais maravilhosos da matemática. Voce cria definições abstratas que generalizam conceitos mais concretos e, apesar das dificuldades iniciais geradas pelo excesso de abstração, isso acaba facilitando nosso raciocínio, a posteriori, fazendo-nos associarmos fatos aparentemente independentes e usarmos resultados obtidos para um fim, em outro fim totalmente diferente, além de que, muitas vezes, é mais fácil enxergarmos coisas particulares quando estamos no geral (algo parecido com o que Polya chama de "paradoxo da invenção"). A topologia generaliza conceito de vizinhanças que herdamos da reta real, para que seja aplicada para diversos fins. Mas depois que a aprendemos, fica mais fácil descobrirmos coisas relativas à própria reta real e "limpar" o nosso raciocínio de informações irrelevantes para um determinado problema (por exemplo, ao invés de considerarmos aquela definição por epsilons e deltas de continuidade de função, usarmos que a imagem inversa de abertos é aberta pode ser mais útil, claro e "limpo"). Com relação a essas coisas tenho pensado bastante, e creio que está relacionado com seu "estranho sentimento". De qualquer forma, essa é uma das coisas que mais me admira na Matemática. >From: "Paulo Santa Rita" >Reply-To: obm-l@mat.puc-rio.br >To: obm-l@mat.puc-rio.br >Subject: Re: [obm-l] Um Estranho Sentimento ... >Date: Wed, 11 Sep 2002 15:24:22 +0000 > >Ola ROGERIO FAJARDO e demais >colegas desta lista ... OBM-L, > >E entao Fajardo, tudo legal ? >Conseguiu o Livro de Logica-Matematica ? > >Eu sei que foram fatos geometricos e outros fenomenos cotidianos (divida -> >numero negativo, divisao de um objeto -> fracao, etc etc ) que nos levaram >a descoberta das diversas classes de numeros, "construidos" posteriormente >e hoje apresentados com o auxilio da abstracao matematica ... > >Colocar estes numeros em uma reta, porem, e uma construcao humana gratuita >... nao ha nenhum razao forte para tanto e os axiomas de um corpo ordenado >completo nao induzem, a priori, a nenhuma topologia particular ... O que >estes axiomas podem falar sobre disposicao ou configuracao ? Isso : Nada ! >Nos poderiamos pensar sobre eles com igual correcao se os visualizassemos >sobre um ramo de parabola, por exemplo. ME PARECE, salvo melhor juizo, que >a unica exigencia que podemos fazer sobre uma possivel representacao e a de >continuidade ... E a continuidade, conforme todos nos sabemos, nao e uma >propriedade metrica. > >Bom, sendo assim, respeitados os axiomas de um corpo ordenado completo, nos >podemos pensar nos numeros reais como estando disposto de outra forma, >desde que esta estratificacao preserve a continuidade ... A questao e : e >vantajoso fazer isso ? e util ? Com esta imagem nos conseguiremos resolver >ou esclarecer algum fato que ainda nao foi resolvido ou esclarecido ? So >assim um mudanca ou inovacao e justificavel ... > >Nao sei se consegui ser claro, mas percebi que voce pensou seriamente sobre >a minha mensagem e nao supos que eu seja tao simplorio que nao perceba >sobre a gravidade e implicacoes do que estou falando ... > >Um abraco >Paulo Santa Rita >4,1223,110902 > > > > >>From: "Rogerio Fajardo" >>Reply-To: obm-l@mat.puc-rio.br >>To: obm-l@mat.puc-rio.br >>Subject: Re: [obm-l] Um Estranho Sentimento ... >>Date: Wed, 11 Sep 2002 00:32:33 +0000 >> >>Não compreendi bem o que voce quer dizer, mas me interessei por seu >>comentário. Pelo que entendi, voce quer saber se existe outra forma de >>visualizar, intuitivamente, os números, de forma a enxergar propriedades >>que são difíceis de enxergar com a visualização com as quais estamos >>acostumados. É isso ou nada a ver? >> >>O que eu percebo é que não é bem a geometria que serve para nos dar uma >>intuição dos números reais, mas os números reais surgiram para descrever a >>geometria de forma precisa. Não sei se isso tem algo a ver com seu e-mail. >>Detalhe-me mais o seu pensamento. >> >> >>>From: "Paulo Santa Rita" >>>Reply-To: obm-l@mat.puc-rio.br >>>To: obm-l@mat.puc-rio.br >>>Subject: [obm-l] Um Estranho Sentimento ... >>>Date: Mon, 09 Sep 2002 21:19:23 +0000 >>> >>>Ola Pessoal e demais >>>colegas desta lista ... OBM-L, >>> >>>Eu tenho pensado continua e longamente em um conjunto de questoes >>>relacionadas que me levam, invariavelmente, a uma mesma direcao que nao >>>estou conseguindo acreditar ... ate parece que depois de tanto refletir >>>cheguei a alguma constatacao insana ou simploria demais ... Se algum >>>colega puder falar algo esclarecedor e/ou interessante eu ficarei muito >>>grato ! >>> >>>Desde a infancia somos instados a pensar que os numeros reais estao >>>dispostos ao longo de uma linha reta. Nos dizemos : 3 < 5 ! E >>>imediatamente visualizamos o 3 a esquerda do 5, ambos em uma linha reta ! >>>Por que nos pensamos assim ? >>> >>>E verdade que em cursos de analise os livros definem R como um corpo >>>ordenado completo e derivam as propriedades dos numeros reais dos axiomas >>>que definem esta estrutura, sem recorrer a qualquer propriedade >>>geometrica derivada de uma eventual visualizacao dos numeros sobre uma >>>reta ... >>> >>>Mas se, por um lado, os axiomas de um corpo ordenado completo nao >>>implicam ou requerem explicitamente uma estrutura geometrica conhecida, e >>>inegavel que a visualizacao "informal" que fazemos facilita muitos >>>raciocinio ... SERIA REALMENTE IMPOSSIVEL ASSOCIAR A UM CORPO ORDENADO >>>COMPLETO UMA GEOMETRIA, ATRAVES DE AXIOMAS OU OUTROS RECURSOS, DE FORMA >>>QUE PUDESSEMOS TER OUTRAS VISUALIZACOES, MESMO QUE ESTRANHAS, POREM, >>>CONMSISTENTES ? NAO PODERIAM ALGUMAS PROPRIEDADES NUMERICAS DEPENDEREM >>>INEXORAVELMENTE DE UMA TAL GEOMETRIA ? >>> >>>E bem provavel que eu esteja errado, mas nao consigo perceber o meu erro >>>... EU ACHO QUE CERTOS MAPEAMENTOS NUMERICOS REQUEREM OU IMPLICAM QUE OS >>>NUMEROS NATURAIS TEM UMA GEOMETRIA OU ESTRUTURA INTRINSECA, SEM A QUAL >>>NAO DA PRA COMPREENDER CERTAS COISAS ... E NECESSARIO OU POSTULAR UMA >>>DISPOSICAO ESTRATIGRAFICA OU SUPOR QUE CERTOS MAPEAMENTOS INDUZEM UMA TAL >>>ESTRATIFICACAO ... >>> >>>Bom, se alguem puder falar alguma coisa interessante sobre este tema eu >>>fico muito grato, pois este e realmente um SENTIMENTO ESTRANHO que me tem >>>ocorrido com alguma frequencia. Pode ser uma burrice momentanea que esta >>>me levando a estas perguntas aparentemente idiotas e sem sentido, mas eu >>>nao iria ocupar o tempo de voces, meus amigos, se nao tivesse razoes >>>seria pra fazer isso ... >>> >>>Um abraco a Todos >>>Paulo Santa Rita >>>2,1818,090902 >>> >>> >>>_________________________________________________________________ >>>Converse com seus amigos online, faça o download grátis do MSN Messenger: >>>http://messenger.msn.com.br >>> >>>========================================================================= >>>Instruções para entrar na lista, sair da lista e usar a lista em >>>http://www.mat.puc-rio.br/~nicolau/olimp/obm-l.html >>>O administrador desta lista é >>>========================================================================= >> >> >> >> >>_________________________________________________________________ >>Send and receive Hotmail on your mobile device: http://mobile.msn.com >> >>========================================================================= >>Instruções para entrar na lista, sair da lista e usar a lista em >>http://www.mat.puc-rio.br/~nicolau/olimp/obm-l.html >>O administrador desta lista é >>========================================================================= > > > > >_________________________________________________________________ >Converse com seus amigos online, faça o download grátis do MSN Messenger: >http://messenger.msn.com.br > >========================================================================= >Instruções para entrar na lista, sair da lista e usar a lista em >http://www.mat.puc-rio.br/~nicolau/olimp/obm-l.html >O administrador desta lista é >========================================================================= _________________________________________________________________ Chat with friends online, try MSN Messenger: http://messenger.msn.com ========================================================================= Instruções para entrar na lista, sair da lista e usar a lista em http://www.mat.puc-rio.br/~nicolau/olimp/obm-l.html O administrador desta lista é ========================================================================= From owner-obm-l@sucuri.mat.puc-rio.br Fri Sep 13 18:14:14 2002 Return-Path: Received: (from majordom@localhost) by sucuri.mat.puc-rio.br (8.9.3/8.9.3) id SAA05727 for obm-l-MTTP; Fri, 13 Sep 2002 18:12:53 -0300 Received: (from nicolau@localhost) by sucuri.mat.puc-rio.br (8.9.3/8.9.3) id SAA05722 for obm-l@mat.puc-rio.br; Fri, 13 Sep 2002 18:12:52 -0300 Date: Fri, 13 Sep 2002 18:12:52 -0300 From: "Nicolau C. Saldanha" To: obm-l@mat.puc-rio.br Subject: [obm-l] Re: =?iso-8859-1?Q?=5Bobm-l=5D_En:_=5Bobm-l=5D_Re:_=5Bobm-l=5D_D=FAvida_sobr?= =?iso-8859-1?Q?e_sequ=EAncias_rand=F4micas_=28problema_do_tipo_NP=29?= Message-ID: <20020913181252.B5248@sucuri.mat.puc-rio.br> References: <00cd01c25b54$bcc1e780$df02dcc8@josefran> Mime-Version: 1.0 Content-Type: text/plain; charset=iso-8859-1 Content-Disposition: inline Content-Transfer-Encoding: 8bit User-Agent: Mutt/1.2.5i In-Reply-To: <00cd01c25b54$bcc1e780$df02dcc8@josefran>; from jfgcosta@unisys.com.br on Fri, Sep 13, 2002 at 03:38:13PM -0300 Sender: owner-obm-l@sucuri.mat.puc-rio.br Precedence: bulk Reply-To: obm-l@mat.puc-rio.br On Fri, Sep 13, 2002 at 03:38:13PM -0300, Jose Francisco Guimaraes Costa wrote: > Uma seqüência realmente randômica é a seqüência de dígitos que compõem > números como pi e a base dos logaritmos naturais. A seqüência dos algarismos de pi ou de e pode ser obtida por um algoritmo e portanto não é realmente randômica. Conjectura-se (mas isto não é um teorema) que estas duas seqüências de algarismos sejam pseudo-aleatórias (por exemplo, que a freqüência dos 10 algarismos seja a mesma). []s, N. ========================================================================= Instruções para entrar na lista, sair da lista e usar a lista em http://www.mat.puc-rio.br/~nicolau/olimp/obm-l.html O administrador desta lista é ========================================================================= From owner-obm-l@sucuri.mat.puc-rio.br Fri Sep 13 18:38:35 2002 Return-Path: Received: (from majordom@localhost) by sucuri.mat.puc-rio.br (8.9.3/8.9.3) id SAA07591 for obm-l-MTTP; Fri, 13 Sep 2002 18:37:01 -0300 Received: from sv1sintese.sintese.com (mail.sintese.com [200.176.113.35]) by sucuri.mat.puc-rio.br (8.9.3/8.9.3) with ESMTP id SAA07586 for ; Fri, 13 Sep 2002 18:36:58 -0300 Received: from wsjujuba (unknown [10.0.0.195]) by sv1sintese.sintese.com (Postfix) with SMTP id 5211327AD09 for ; Fri, 13 Sep 2002 18:40:53 -0300 (BRT) Message-ID: <001d01c25b6e$87323560$c300000a@wsjujuba> From: =?iso-8859-1?Q?Juliana_L=F6ff?= To: Subject: [obm-l] =?iso-8859-1?B?c2Vx/OpuY2lh?= Date: Fri, 13 Sep 2002 18:42:57 -0300 MIME-Version: 1.0 Content-Type: multipart/alternative; boundary="----=_NextPart_000_001A_01C25B55.61261A30" X-Priority: 3 X-MSMail-Priority: Normal X-Mailer: Microsoft Outlook Express 6.00.2600.0000 X-MimeOLE: Produced By Microsoft MimeOLE V6.00.2600.0000 Sender: owner-obm-l@sucuri.mat.puc-rio.br Precedence: bulk Reply-To: obm-l@mat.puc-rio.br This is a multi-part message in MIME format. ------=_NextPart_000_001A_01C25B55.61261A30 Content-Type: text/plain; charset="iso-8859-1" Content-Transfer-Encoding: quoted-printable E a=EA, pessoal! Me ajudem com essa (mas para resolver em n=EDvel secund=E1rio!!) Muito obrigada! Ju ------------------------------- (UFRGS - 2002) Se n =E9 um natural =EDmpar, o n=FAmero de elementos da = seq=FC=EAncia 1,2,2,3,3,3,4,4,4,4,...,n,n,(n vezes),n que s=E3o = n=FAmeros pares =E9: (a) n=B2 - 1=20 4=20 (b) n=B2 - 1=20 2=20 (c) n(n + 1)=20 4=20 (d) n(n + 1)=20 2=20 (e) (n + 1)=B2=20 4=20 ------=_NextPart_000_001A_01C25B55.61261A30 Content-Type: text/html; charset="iso-8859-1" Content-Transfer-Encoding: quoted-printable
E a=EA, pessoal!
Me ajudem com essa (mas para resolver = em n=EDvel=20 secund=E1rio!!)
Muito obrigada!
Ju
-------------------------------
(UFRGS - 2002) Se n =E9 um natural = =EDmpar, o n=FAmero de=20 elementos da seq=FC=EAncia 1,2,2,3,3,3,4,4,4,4,...,n,n,(n vezes),n que = s=E3o n=FAmeros=20 pares =E9:
 
(a) n=B2 - 1
4
 
(b) n=B2 - 1
2
 
(c) n(n + 1)
4
 
(d) n(n + 1)
2
 
(e) (n + 1)=B2
4
 
------=_NextPart_000_001A_01C25B55.61261A30-- ========================================================================= Instruções para entrar na lista, sair da lista e usar a lista em http://www.mat.puc-rio.br/~nicolau/olimp/obm-l.html O administrador desta lista é ========================================================================= From owner-obm-l@sucuri.mat.puc-rio.br Fri Sep 13 18:41:40 2002 Return-Path: Received: (from majordom@localhost) by sucuri.mat.puc-rio.br (8.9.3/8.9.3) id SAA07768 for obm-l-MTTP; Fri, 13 Sep 2002 18:40:23 -0300 Received: from hotmail.com (f41.law9.hotmail.com [64.4.9.41]) by sucuri.mat.puc-rio.br (8.9.3/8.9.3) with ESMTP id SAA07763 for ; Fri, 13 Sep 2002 18:40:20 -0300 Received: from mail pickup service by hotmail.com with Microsoft SMTPSVC; Fri, 13 Sep 2002 14:41:30 -0700 Received: from 200.190.10.185 by lw9fd.law9.hotmail.msn.com with HTTP; Fri, 13 Sep 2002 21:41:30 GMT X-Originating-IP: [200.190.10.185] From: "Rogerio Fajardo" To: obm-l@mat.puc-rio.br Subject: Re: [obm-l] Axioma da Escolha Date: Fri, 13 Sep 2002 21:41:30 +0000 Mime-Version: 1.0 Content-Type: text/plain; format=flowed Message-ID: X-OriginalArrivalTime: 13 Sep 2002 21:41:30.0285 (UTC) FILETIME=[522A9DD0:01C25B6E] Sender: owner-obm-l@sucuri.mat.puc-rio.br Precedence: bulk Reply-To: obm-l@mat.puc-rio.br Apenas lembrando, porque costuma-se realçar quando se usa o axioma da escolha, há uma corrente filosófica de matemáticos que não aceitam o axioma da escolha: os construtivistas (ou, mais geralmente, os intuicionistas). O axioma da escolha nos garante a existência de objetos que não podemos determinar quem, exatamente, ele é. Esse tipo de coisa os construtivistas não aceitam, pois de que serve saber que existe alguma coisa que nunca saberemos quem é, ou onde está? O teorema de Weierstrass, que diz que toda função real contínua sobre um intervalo fechado assume máximo, não é aceita pelos construtivistas, pois não podemos exibir esse ponto de máximo. Por outro lado não podemos dizer que não existe ponto de máximo, pois isso seria garantir que todos os pontos não são de máximo, o que não devemos assegurar. Por isso na lógica intuicionista "A ou não A" pode ser falso, e A não é equivalente a "não não A". Todas essas complicações geradas pelo construtivismo fizeram que esse caísse um pouco no esquecimento. Hoje parece que há poucos matemáticos construtivistas. Mas devemos nos lembrar que o argumento central que gerou o construtivismo faz sentido. Realmente, podemos pensar o que fazemos com coisas obtidas não construtivamente. Enfim, há sempre uma fagulha de construtivista em nós. É certo que os mais radicais não admitem nem prova por absurdo, mas o axioma da escolha já seria o maior crime que se poderia cometer contra o construtivismo. Por isso, nas demonstrações, é sempre bom ressaltar o que é construtivo e o que não é. Por exemplo, o Paradoxo de Banach-Tarski, sobre a duplicação da esfera, citada pelo Paulo, é não-construtiva. Sobre o problema da violência, resta um consolo: se o conjunto dos bandidos, dado pelo problema, já estiver bem ordenado (por exemplo, se é enumerável), não precisamos do axioma da escolha, e não cometeremos uma "violência" contra os intuicionistas. O difícil vai ser achar bandidos bem ordenados... >From: "Paulo Santa Rita" >Reply-To: obm-l@mat.puc-rio.br >To: obm-l@mat.puc-rio.br >Subject: Re: [obm-l] Axioma da Escolha >Date: Thu, 12 Sep 2002 21:56:07 +0000 > >Ola PROF NICOLAU e demais >colegas desta lista ... OBM-L, > >Vou fazer um comentario a mensagem abaixo que talvez ajude alguns membros >desta lista entenderem porque muitos Matematicos - em especial aqueles que >conhecem logica e teoria dos conjuntos - acham relevante destacar que esta >ou aquela prova ou argumentacao matematica usa o AXIOMA DA ESCOLHA. > >Tarski, entre outros, mostrou que o uso do AXIOMA DA ESCOLHA com os demais >axiomas da teoria dos conjuntos levam-nos inevitavelmente a conclusoes >pouco verossimeis. Em particular ele mostrou que poderiamos dividir uma >esfera dada em ao menos cinco partes e, unindo novamente as partes de outra >forma, derivar nao uma, mas duas esferas identicas a primitiva. > >O fato acima, por paradoxal que parece, e logicamente inatacavel e uma das >consequencias bizarras que este axioma implica. Muitos Matematicos supunham >que tais resultados eram derivados da liberalidade dado a FUNCAO DE >ESCOLHA, pois pode-se usar a que quisermos conquanto respeitemos o aspecto >formal do axioma. > >A partir deste resultado do Tarski ( e de outros tambem ) os Matematicos >comecaram a suspeitar que o AXIOMA DA ESCOLHA era um principio nefasto, >sendo provavelmente o responsavel por possiveis e potenciais >inconsistencias que a teoria do conjuntos tivesse ou viesse a ter. Dai >surgiu a desconfianca com as demonstracoes com este axioma. Por prudencia, >toda prova que usava este teorema era rotulada "USA O AXIOMA DA ESCOLHA", >como que insinuando : "PROCURE UMA OUTRA MANEIRA DE PROVAR ISSO ... " > >Aqui entra o Magistral GODEL ... > >Godel classificou as teorias dos conjuntos em : > >1) Teoria Cantoriana A e aquela que usa o AXIOMA DA ESCOLHA. >2) Teoria Cantoriana B e aquela em que nao usa o AXIOMA DA ESCOLHA. > >E provou o seguinte : > >SE A TEORIA CANTORIANA "A" CONTIVER OU GERAR INCONSISTENCIAS, A TEORIA >CANTORIA "B" TAMBEM CONTERA E GERARA INCONSISTENCIAS. > >Isto e, o AXIOMA DA ESCOLHA nao e o responsavel por possiveis >inconsistencias ou paradoxas que porventura derivem da teoria dos >conjuntos. Ele pode ser um CATALISADOR destas inconsistencias, evidenciando >de forma mais direta e clara possiveis absurdos ... > >Agora, uma observacao sobre o Paradoxo de Tarski. > >O que ha de absurdo nele ? A criacao de Massa ( duplicacao de uma esfera ) >sem a necessaria absorcao de uma fabulosa quantidade de Energia ? Mas ... >Nao e isso que rotineiramente ocorre no mundo das particulas elementares, >quando, do nada, surge uma massa que posteriormente desapare num par de >particulas antipodas ? E mais provavel que este fato ou operacao seja >paradoxal para o nosso cotidiano, nao para o Autor da Natureza que, hoje >sabemos, continuamente faz isso ... > >Um abraco >Paulo Santa Rita >5,1854,120902 > >>From: "Nicolau C. Saldanha" >>Reply-To: obm-l@mat.puc-rio.br >>To: obm-l@mat.puc-rio.br >>Subject: Re: [obm-l] Axioma da Escolha >>Date: Thu, 12 Sep 2002 17:32:17 -0300 >> >>On Wed, Sep 11, 2002 at 04:01:42PM -0300, 498 - Artur Costa Steiner wrote: >> > Nos últimos dias o Axioma da Escolha foi bastante mencionado nesta >> > lista, motivado por um interessante problema (violência), sugerido por >> > uma das participantes, e que involve este axioma. >> > >> > Eu não estou certo, mas, no meio matemático, ainda existem hoje >> > restrições a este axioma, no sentido de que alguma prova nele baseada >> > possa ser considerada questionável ou mesmo inválida? >> >>Este assunto mereceria uma resposta mais longa, mas o axioma da escolha >>é 'aceito' no sentido seguinte: a maioria dos matemáticos usa este >>axioma sem parar para pensar no assunto. Aliás sem nem saber direito >>quando está realmente usando o axioma. Alguns matemáticos, entretanto, >>especialmente especialistas em lógica ou teoria dos conjuntos, acham >>interessante notar exatamente quando o tal axioma é utilizado. >> >>[]s, N. >>========================================================================= >>Instruções para entrar na lista, sair da lista e usar a lista em >>http://www.mat.puc-rio.br/~nicolau/olimp/obm-l.html >>O administrador desta lista é >>========================================================================= > > >_________________________________________________________________ >Tenha você também um MSN Hotmail, o maior webmail do mundo: >http://www.hotmail.com/br > >========================================================================= >Instruções para entrar na lista, sair da lista e usar a lista em >http://www.mat.puc-rio.br/~nicolau/olimp/obm-l.html >O administrador desta lista é >========================================================================= _________________________________________________________________ MSN Photos is the easiest way to share and print your photos: http://photos.msn.com/support/worldwide.aspx ========================================================================= Instruções para entrar na lista, sair da lista e usar a lista em http://www.mat.puc-rio.br/~nicolau/olimp/obm-l.html O administrador desta lista é ========================================================================= From owner-obm-l@sucuri.mat.puc-rio.br Fri Sep 13 18:45:10 2002 Return-Path: Received: (from majordom@localhost) by sucuri.mat.puc-rio.br (8.9.3/8.9.3) id SAA07920 for obm-l-MTTP; Fri, 13 Sep 2002 18:43:48 -0300 Received: from sv1sintese.sintese.com (mail.sintese.com [200.176.113.35]) by sucuri.mat.puc-rio.br (8.9.3/8.9.3) with ESMTP id SAA07916 for ; Fri, 13 Sep 2002 18:43:45 -0300 Received: from wsjujuba (unknown [10.0.0.195]) by sv1sintese.sintese.com (Postfix) with SMTP id 2337427AD09 for ; Fri, 13 Sep 2002 18:47:41 -0300 (BRT) Message-ID: <003301c25b6f$7a3cfd30$c300000a@wsjujuba> From: =?iso-8859-1?Q?Juliana_L=F6ff?= To: Subject: [obm-l] =?iso-8859-1?B?ZnVu5/Vlcw==?= Date: Fri, 13 Sep 2002 18:49:44 -0300 MIME-Version: 1.0 Content-Type: multipart/alternative; boundary="----=_NextPart_000_0030_01C25B56.53934270" X-Priority: 3 X-MSMail-Priority: Normal X-Mailer: Microsoft Outlook Express 6.00.2600.0000 X-MimeOLE: Produced By Microsoft MimeOLE V6.00.2600.0000 Sender: owner-obm-l@sucuri.mat.puc-rio.br Precedence: bulk Reply-To: obm-l@mat.puc-rio.br This is a multi-part message in MIME format. ------=_NextPart_000_0030_01C25B56.53934270 Content-Type: text/plain; charset="iso-8859-1" Content-Transfer-Encoding: quoted-printable E a=EA, pessoal! Mais uma que eu gostaria de ajuda (em n=EDvel secund=E1rio!!) Muito obrigada! Ju ------------------------------- (UFRGS - 2002) Esbo=E7ando os gr=E1ficos das fun=E7=F5es definidas por = f(a) =3D 5=AA e g(a)=3D2 + x - x=B2 num mesmo plano cartesiano, = verifica-se que todas as ra=EDzes da equa=E7=E3o f(a)=3Dg(a) pertencem = ao intervalo: a) (-2,-1) b) (-1,0) c) (-1,1) d) (0,1) e) (0,2) ------=_NextPart_000_0030_01C25B56.53934270 Content-Type: text/html; charset="iso-8859-1" Content-Transfer-Encoding: quoted-printable
E a=EA, pessoal!
Mais uma que eu gostaria de ajuda (em = n=EDvel=20 secund=E1rio!!)
Muito obrigada!
Ju
-------------------------------
(UFRGS - 2002) Esbo=E7ando os = gr=E1ficos das fun=E7=F5es=20 definidas por f(a) =3D 5=AA e g(a)=3D2 + x - x=B2 num mesmo plano = cartesiano,=20 verifica-se que todas as ra=EDzes da equa=E7=E3o f(a)=3Dg(a) pertencem = ao=20 intervalo:
 
a) = (-2,-1)
b) = (-1,0)
c) = (-1,1)
d) = (0,1)
e)=20 (0,2)
=
------=_NextPart_000_0030_01C25B56.53934270-- ========================================================================= Instruções para entrar na lista, sair da lista e usar a lista em http://www.mat.puc-rio.br/~nicolau/olimp/obm-l.html O administrador desta lista é ========================================================================= From owner-obm-l@sucuri.mat.puc-rio.br Fri Sep 13 18:56:21 2002 Return-Path: Received: (from majordom@localhost) by sucuri.mat.puc-rio.br (8.9.3/8.9.3) id SAA08992 for obm-l-MTTP; Fri, 13 Sep 2002 18:54:46 -0300 Received: from pina.terra.com.br (pina.terra.com.br [200.176.3.17]) by sucuri.mat.puc-rio.br (8.9.3/8.9.3) with ESMTP id SAA08988 for ; Fri, 13 Sep 2002 18:54:43 -0300 Received: from engenho.terra.com.br (engenho.terra.com.br [200.176.3.42]) by pina.terra.com.br (Postfix) with ESMTP id 3DDD252F59 for ; Fri, 13 Sep 2002 18:55:55 -0300 (EST) Received: from stabel (unknown [200.203.38.171]) (authenticated user dudasta) by engenho.terra.com.br (Postfix) with ESMTP id 5647F680A1 for ; Fri, 13 Sep 2002 18:55:54 -0300 (EST) Message-ID: <002501c25b70$55d4a820$0301a8c0@stabel> From: "Eduardo Casagrande Stabel" To: References: <001d01c25b6e$87323560$c300000a@wsjujuba> Subject: [obm-l] =?iso-8859-1?Q?Re:_=5Bobm-l=5D_seq=FC=EAncia?= Date: Fri, 13 Sep 2002 18:55:54 -0300 MIME-Version: 1.0 Content-Type: multipart/alternative; boundary="----=_NextPart_000_0022_01C25B57.3031FD20" X-Priority: 3 X-MSMail-Priority: Normal X-Mailer: Microsoft Outlook Express 6.00.2600.0000 X-MimeOLE: Produced By Microsoft MimeOLE V6.00.2600.0000 Sender: owner-obm-l@sucuri.mat.puc-rio.br Precedence: bulk Reply-To: obm-l@mat.puc-rio.br This is a multi-part message in MIME format. ------=_NextPart_000_0022_01C25B57.3031FD20 Content-Type: text/plain; charset="iso-8859-1" Content-Transfer-Encoding: quoted-printable Considere n =3D 2k + 1. Temos 2, 2, 4, 4, 4, 4, 6, 6, 6, 6, 6, 6, ..., 2k, 2k, 2k, ...., 2k, 2k, = 2k. 2 + 4 + 6 + ... + 2k =3D 2(1 + 2 + 3 + ... + k) =3D k(k + 1) =3D (n - = 1)(n + 1)/4 =3D (n^2 - 1)/4. Eduardo. ----- Original Message -----=20 From: Juliana L=F6ff=20 To: obm-l@mat.puc-rio.br=20 Sent: Friday, September 13, 2002 6:42 PM Subject: [obm-l] seq=FC=EAncia E a=EA, pessoal! Me ajudem com essa (mas para resolver em n=EDvel secund=E1rio!!) Muito obrigada! Ju ------------------------------- (UFRGS - 2002) Se n =E9 um natural =EDmpar, o n=FAmero de elementos da = seq=FC=EAncia 1,2,2,3,3,3,4,4,4,4,...,n,n,(n vezes),n que s=E3o = n=FAmeros pares =E9: (a) n=B2 - 1=20 4=20 (b) n=B2 - 1=20 2=20 (c) n(n + 1)=20 4=20 (d) n(n + 1)=20 2=20 (e) (n + 1)=B2=20 4=20 ------=_NextPart_000_0022_01C25B57.3031FD20 Content-Type: text/html; charset="iso-8859-1" Content-Transfer-Encoding: quoted-printable
Considere n =3D 2k + 1.
Temos 2, 2, 4, 4, 4, 4, 6, 6, 6, 6, 6, = 6, ..., 2k,=20 2k, 2k, ...., 2k, 2k, 2k.
2 + 4 + 6 + ... + 2k =3D 2(1 + 2 + 3 + = ... + k) =3D k(k=20 + 1) =3D (n - 1)(n + 1)/4 =3D (n^2 - 1)/4.
 
Eduardo.
----- Original Message -----
From:=20 Juliana L=F6ff
Sent: Friday, September 13, = 2002 6:42=20 PM
Subject: [obm-l] = seq=FC=EAncia

E a=EA, pessoal!
Me ajudem com essa (mas para resolver = em n=EDvel=20 secund=E1rio!!)
Muito obrigada!
Ju
-------------------------------
(UFRGS - 2002) Se n =E9 um natural = =EDmpar, o n=FAmero=20 de elementos da seq=FC=EAncia 1,2,2,3,3,3,4,4,4,4,...,n,n,(n vezes),n = que s=E3o=20 n=FAmeros pares =E9:
 
(a) n=B2 - 1
4
 
(b) n=B2 - 1
2
 
(c) n(n + 1)
4
 
(d) n(n + 1)
2
 
(e) (n + 1)=B2
4
 
------=_NextPart_000_0022_01C25B57.3031FD20-- ========================================================================= Instruções para entrar na lista, sair da lista e usar a lista em http://www.mat.puc-rio.br/~nicolau/olimp/obm-l.html O administrador desta lista é ========================================================================= From owner-obm-l@sucuri.mat.puc-rio.br Fri Sep 13 19:02:51 2002 Return-Path: Received: (from majordom@localhost) by sucuri.mat.puc-rio.br (8.9.3/8.9.3) id TAA09538 for obm-l-MTTP; Fri, 13 Sep 2002 19:01:25 -0300 Received: from smtp-33.ig.com.br (smtp-33.ig.com.br [200.226.132.183]) by sucuri.mat.puc-rio.br (8.9.3/8.9.3) with SMTP id TAA09534 for ; Fri, 13 Sep 2002 19:01:22 -0300 Received: (qmail 11332 invoked from network); 13 Sep 2002 22:02:13 -0000 Received: from rj230148.user.veloxzone.com.br (HELO jigglypuff) (200.165.230.148) by smtp-33.ig.com.br with SMTP; 13 Sep 2002 22:02:13 -0000 Message-ID: <003a01c25b72$6737ba60$019da8c0@dummy.net> From: "Juliana Freire" To: References: <000a01c25b69$0a083940$0201a8c0@x> Subject: [obm-l] =?iso-8859-1?Q?Re:_=5Bobm-l=5D_quest=E3o_de_polinomios?= Date: Fri, 13 Sep 2002 19:10:43 -0300 MIME-Version: 1.0 Content-Type: text/plain; charset="iso-8859-1" Content-Transfer-Encoding: 8bit X-Priority: 3 X-MSMail-Priority: Normal X-Mailer: Microsoft Outlook Express 5.50.4522.1200 X-MimeOLE: Produced By Microsoft MimeOLE V5.50.4522.1200 Sender: owner-obm-l@sucuri.mat.puc-rio.br Precedence: bulk Reply-To: obm-l@mat.puc-rio.br Dividindo x^3+px+q por x^2+ax+b dá x-a e tem resto (p-b+a^2)x + q + ab, que tem que ser zero para x^3+px+q ser divisível por x^2+ax+b. Da mesma forma (p-s+r^2)x + q + rs = 0. p-b+a^2 = p-s+r^2 = 0 -b+a^2 = -s+r^2 q + rs = q + ab = 0 rs = ab Se r diferente de zero, s = ab/r -b+a^2 = -ab/r + r^2 b - ab/r = a^2 - r^2 b(1-a/r) = a^2 - r^2 b = (a^2 - r^2)/(1-a/r) b = (a^2 - r^2)/((r-a)/r) b = r(a^2-r^2)/(r-a) b = r(a-r)(a+r)/(r-a) b = -r(a+r) - Juliana ----- Original Message ----- From: "Jeremias de Paula Eduardo" To: Sent: Friday, September 13, 2002 6:03 PM Subject: [obm-l] questão de polinomios Quem puder me ajudar a resolver essa aí eu agradeço. (ITA-62) Se x^3+px+q é divisível por x^2+ax+b e x^2+rx+s, demonstrar que b= -r(a+r) Jeremias de Paula Eduardo ========================================================================= Instruções para entrar na lista, sair da lista e usar a lista em http://www.mat.puc-rio.br/~nicolau/olimp/obm-l.html O administrador desta lista é ========================================================================= From owner-obm-l@sucuri.mat.puc-rio.br Fri Sep 13 19:23:46 2002 Return-Path: Received: (from majordom@localhost) by sucuri.mat.puc-rio.br (8.9.3/8.9.3) id TAA11098 for obm-l-MTTP; Fri, 13 Sep 2002 19:21:46 -0300 Received: from sv1sintese.sintese.com (mail.sintese.com [200.176.113.35]) by sucuri.mat.puc-rio.br (8.9.3/8.9.3) with ESMTP id TAA11093 for ; Fri, 13 Sep 2002 19:21:43 -0300 Received: from wsjujuba (unknown [10.0.0.195]) by sv1sintese.sintese.com (Postfix) with SMTP id 82CBE27AD09 for ; Fri, 13 Sep 2002 19:25:38 -0300 (BRT) Message-ID: <005801c25b74$c77ae850$c300000a@wsjujuba> From: =?iso-8859-1?Q?Juliana_L=F6ff?= To: References: Subject: [obm-l] =?iso-8859-1?Q?infinitas_progress=F5es?= Date: Fri, 13 Sep 2002 19:27:43 -0300 MIME-Version: 1.0 Content-Type: multipart/alternative; boundary="----=_NextPart_000_0055_01C25B5B.A1E5D1E0" X-Priority: 3 X-MSMail-Priority: Normal X-Mailer: Microsoft Outlook Express 6.00.2600.0000 X-MimeOLE: Produced By Microsoft MimeOLE V6.00.2600.0000 Sender: owner-obm-l@sucuri.mat.puc-rio.br Precedence: bulk Reply-To: obm-l@mat.puc-rio.br This is a multi-part message in MIME format. ------=_NextPart_000_0055_01C25B5B.A1E5D1E0 Content-Type: text/plain; charset="iso-8859-1" Content-Transfer-Encoding: quoted-printable (UFRGS - 2002) A disposi=E7=E3o de n=FAmeros abaixo representa infinitas = progress=F5es. 1/2 =20 1/4 1/4 1/4 =20 1/8 1/8 1/8 1/8 1/8 =20 1/16 1/16 1/16 1/16 1/16 1/16 1/16 =20 . . . . . . . . . =20 . . . . . . . . . . .=20 Considere as afirma=E7=F5es referentes =E0 disposi=E7=E3o dada. (1) A d=E9cima linha =E9 formada por 19 elementos. (2) Chamando-se de a1 o primeiro elemento de uma coluna qualquer, a soma = dos termos desta coluna =E9 2a1. (3) A soma dos infinitos elementos da disposi=E7=E3o =E9 3. Quais s=E3o verdadeiras? (a) Apenas 1. (b) Apenas 1 e 2. (c) Apenas 1 e 3. (d) Apenas 2 e 3. (e) 1, 2, e 3. ------=_NextPart_000_0055_01C25B5B.A1E5D1E0 Content-Type: text/html; charset="iso-8859-1" Content-Transfer-Encoding: quoted-printable
(UFRGS - 2002) A disposi=E7=E3o de n=FAmeros abaixo representa = infinitas=20 progress=F5es.
1/2
1/4 1/4 1/4
1/8 1/8 1/8 1/8 1/8
1/16 1/16 1/16 1/16 1/16 1/16 1/16
=95 =95 =95 =95 =95 =95 =95 =95 =95
=95 =95 =95 =95 =95 =95 =95 =95 =95 =95 =95
 
Considere as afirma=E7=F5es referentes =E0 disposi=E7=E3o = dada.
(1) A d=E9cima linha =E9 formada por 19 elementos.
(2) Chamando-se de a1 o primeiro elemento de uma coluna qualquer, a = soma=20 dos termos desta coluna =E9 2a1.
(3) A soma dos infinitos elementos da disposi=E7=E3o =E9 3.
 
Quais s=E3o verdadeiras?
(a) Apenas 1.
(b) Apenas 1 e 2.
(c) Apenas 1 e 3.
(d) Apenas 2 e 3.
(e) 1, 2, e 3.
 
 
------=_NextPart_000_0055_01C25B5B.A1E5D1E0-- ========================================================================= Instruções para entrar na lista, sair da lista e usar a lista em http://www.mat.puc-rio.br/~nicolau/olimp/obm-l.html O administrador desta lista é ========================================================================= From owner-obm-l@sucuri.mat.puc-rio.br Fri Sep 13 20:00:39 2002 Return-Path: Received: (from majordom@localhost) by sucuri.mat.puc-rio.br (8.9.3/8.9.3) id TAA12386 for obm-l-MTTP; Fri, 13 Sep 2002 19:59:06 -0300 Received: from smtp.brturbo.com (smtp2.brturbo.com [200.199.201.30]) by sucuri.mat.puc-rio.br (8.9.3/8.9.3) with ESMTP id TAA12382 for ; Fri, 13 Sep 2002 19:59:01 -0300 Received: from x (200-181-95-057-bsace7011.dsl.telebrasilia.net.br [200.181.95.57]) by smtp.brturbo.com (Postfix) with SMTP id 5AAF213BD99 for ; Fri, 13 Sep 2002 20:00:07 -0300 (BRT) Message-ID: <001601c25b79$589315c0$0201a8c0@x> From: "Jeremias de Paula Eduardo" To: References: <000a01c25b69$0a083940$0201a8c0@x> <003a01c25b72$6737ba60$019da8c0@dummy.net> Subject: [obm-l] =?iso-8859-1?Q?Re:_=5Bobm-l=5D_quest=E3o_de_polinomios?= Date: Fri, 13 Sep 2002 20:00:19 -0300 MIME-Version: 1.0 Content-Type: text/plain; charset="iso-8859-1" Content-Transfer-Encoding: 8bit X-Priority: 3 X-MSMail-Priority: Normal X-Mailer: Microsoft Outlook Express 5.00.2919.6600 X-MimeOLE: Produced By Microsoft MimeOLE V5.00.2919.6600 Sender: owner-obm-l@sucuri.mat.puc-rio.br Precedence: bulk Reply-To: obm-l@mat.puc-rio.br Eu entendi o raciocínio, até fiz a primeira parte aqui em casa , mas eu empaquei na hora que achei os restos. Eu sei que os restos são iguais zero, mas não entendi porque você pode igualar os coeficientes à zero independentemente. Se soma dos termos do polinômio "resto" é igual a zero isso, como isso pode implicar em que os coeficientes também o sejam? > Dividindo x^3+px+q por x^2+ax+b dá x-a e tem resto (p-b+a^2)x + q + ab, > que tem que ser zero para x^3+px+q ser divisível por x^2+ax+b. > Da mesma forma (p-s+r^2)x + q + rs = 0. > > p-b+a^2 = p-s+r^2 = 0 > -b+a^2 = -s+r^2 > > q + rs = q + ab = 0 > rs = ab > > Se r diferente de zero, > s = ab/r > > -b+a^2 = -ab/r + r^2 > b - ab/r = a^2 - r^2 > b(1-a/r) = a^2 - r^2 > b = (a^2 - r^2)/(1-a/r) > b = (a^2 - r^2)/((r-a)/r) > b = r(a^2-r^2)/(r-a) > b = r(a-r)(a+r)/(r-a) > b = -r(a+r) > > - Juliana >> Quem puder me ajudar a resolver essa aí eu agradeço. >> >> (ITA-62) Se x^3+px+q é divisível por x^2+ax+b e x^2+rx+s, >demonstrar que b= -r(a+r) >> >> Jeremias de Paula Eduardo ========================================================================= Instruções para entrar na lista, sair da lista e usar a lista em http://www.mat.puc-rio.br/~nicolau/olimp/obm-l.html O administrador desta lista é ========================================================================= From owner-obm-l@sucuri.mat.puc-rio.br Fri Sep 13 20:09:09 2002 Return-Path: Received: (from majordom@localhost) by sucuri.mat.puc-rio.br (8.9.3/8.9.3) id UAA12534 for obm-l-MTTP; Fri, 13 Sep 2002 20:07:49 -0300 Received: from suter.bol.com.br (suter.bol.com.br [200.221.24.19]) by sucuri.mat.puc-rio.br (8.9.3/8.9.3) with ESMTP id UAA12530 for ; Fri, 13 Sep 2002 20:07:47 -0300 Received: from bol.com.br (200.221.24.118) by suter.bol.com.br (5.1.071) id 3D7E534E001493F4 for obm-l@mat.puc-rio.br; Fri, 13 Sep 2002 20:08:44 -0300 Date: Fri, 13 Sep 2002 20:06:43 -0300 Message-Id: Subject: [obm-l] =?iso-8859-1?q?2_Problemas_Cl=E1ssicos_de_DG?= MIME-Version: 1.0 Content-Type: text/plain;charset="iso-8859-1" From: "=?iso-8859-1?q?Ant=C4=A7nio_Lacerda_J=C5=93nior?=" To: obm-l@mat.puc-rio.br X-XaM3-API-Version: 2.4.3.4.4 X-SenderIP: 200.207.14.157 Content-Transfer-Encoding: 8bit X-MIME-Autoconverted: from quoted-printable to 8bit by sucuri.mat.puc-rio.br id UAA12531 Sender: owner-obm-l@sucuri.mat.puc-rio.br Precedence: bulk Reply-To: obm-l@mat.puc-rio.br Olá, todos. Estou procurando a solução destes 2 problemas clássicos de Desenho Geométrico: 1) Dadas as três bissetrizes de um triângulo, construa esse triângulo. 2) Determinar o centro de uma circunferência apenas com compasso. Eu as perdi, por isso eu as procuro. Obrigado. Antônio Lacerda Júnior __________________________________________________________________________ AcessoBOL, só R$ 9,90! O menor preço do mercado! Assine já! http://www.bol.com.br/acessobol ========================================================================= Instruções para entrar na lista, sair da lista e usar a lista em http://www.mat.puc-rio.br/~nicolau/olimp/obm-l.html O administrador desta lista é ========================================================================= From owner-obm-l@sucuri.mat.puc-rio.br Fri Sep 13 20:19:01 2002 Return-Path: Received: (from majordom@localhost) by sucuri.mat.puc-rio.br (8.9.3/8.9.3) id UAA12928 for obm-l-MTTP; Fri, 13 Sep 2002 20:17:39 -0300 Received: from home.iis.com.br (mail.iis.com.br [200.202.96.2]) by sucuri.mat.puc-rio.br (8.9.3/8.9.3) with ESMTP id UAA12924 for ; Fri, 13 Sep 2002 20:17:35 -0300 Received: from Marcio (rio-tc0-tty155.iis.com.br [200.202.98.155]) by home.iis.com.br (8.11.6/8.11.6/1.1.1.16) with SMTP id g8DNIT404214 for ; Fri, 13 Sep 2002 20:18:30 -0300 Message-ID: <003101c25b7c$0424fb40$9b62cac8@epq.ime.eb.br> From: "Marcio" To: References: <003c01c259e8$e9d79660$6898cfc8@slap> <000d01c25abf$6f013a40$6898cfc8@slap> <001701c25ac5$ce4ff940$4462cac8@epq.ime.eb.br> <001001c25b6a$b511d160$6898cfc8@slap> Subject: Re: [obm-l] Geometria PLZ !!! Date: Fri, 13 Sep 2002 20:19:29 -0300 MIME-Version: 1.0 Content-Type: multipart/alternative; boundary="----=_NextPart_000_002E_01C25B62.DCE1BFA0" X-Priority: 3 X-MSMail-Priority: Normal X-Mailer: Microsoft Outlook Express 5.50.4133.2400 X-MimeOLE: Produced By Microsoft MimeOLE V5.50.4133.2400 X-AntiVirus: Antivirus for sendmail by Petr Rehor Sender: owner-obm-l@sucuri.mat.puc-rio.br Precedence: bulk Reply-To: obm-l@mat.puc-rio.br This is a multi-part message in MIME format. ------=_NextPart_000_002E_01C25B62.DCE1BFA0 Content-Type: text/plain; charset="iso-8859-1" Content-Transfer-Encoding: quoted-printable sqrt eh raiz quadrada. proj u,v eu estou usando para representar o vetor que se obtem qdo se = projeta u em v. (ele tem direcao v e modulo =3D |u|cos(angulo entre = u,v).. []'s ----- Original Message -----=20 From: Afemano=20 To: obm-l@mat.puc-rio.br=20 Sent: Friday, September 13, 2002 6:15 PM Subject: Re: [obm-l] Geometria PLZ !!! Nossa cara valeu mesmo..=20 mas o que =E9 "proj" e "sqrt" ? hehehe desculpe mas sou meio novo por = aqui :D Abra=E7os... ----- Original Message -----=20 From: Marcio=20 To: obm-l@mat.puc-rio.br=20 Sent: Thursday, September 12, 2002 10:35 PM Subject: Re: [obm-l] Geometria PLZ !!! Se bem me lembro, essa questao tmb ja caiu num vestibular do IME, = com exatamente os mesmos dados.=20 Na minha opiniao, a maneira mais simples de achar distancia entre = retas reversas eh usando vetores (bom, com certeza eh a maneira mais = simples de resolver esse tipo de problema usando apenas ascii.. :) Considere o cone num sistema de eixos Oxyz tq C =3D (0,0,2); A = =3D (-1, 0,0); B=3D(1,0,0), P =3D (cosx, senx, 0). A reta t tem direcao a=3D(0,1,0), enqto que a geratriz PC tem = direcao b=3D(cosx,senx,-2). Note que o vetor axb (produto vetorial) eh = perpendicular a a e a b, e portanto da a direcao da perpendicular comum. = Pegando um ponto X qualquer de PC e outro Y qq de t, temos um = vetor cuja projecao na perpendicular comum da exatamente a distancia = entre as duas retas reversas (afinal, as projecoes de X e Y em axb sao = exatamente os extremos do segmento limitado pelas retas reversas na = perpendicular comum!) Agora, as contas: axb=3D(2, 0, -cosx). Tomando X=3DC=3D(0,0,2) e = Y=3DB=3D(1,0,0), temos XY=3D(1,0,-2) e=20 d =3D |proj| =3D |XY . axb| / /|axb| =3D |2 + 2cosx| / = sqrt[4+cos^2(x)] ObS: usei q proj u,v =3D [(u.v)/(v.v)]v, e seu modulo eh |u.v| / |v| Abracos, Marcio ----- Original Message -----=20 From: Afemano=20 To: obm-l@mat.puc-rio.br=20 Sent: Wednesday, September 11, 2002 8:14 PM Subject: [obm-l] Geometria PLZ !!! Ol=E1 galera.. algu=E9m resolve esse exerc=EDcio pra mim plz ? ( FUVEST ) A, B e P s=E3o tr=EAs pontos de uma circunfer=EAncia = de centro O e raio unit=E1rio, base de um cone reto de v=E9rtice C e = altura igual a 2. A e B s=E3o diametralmente opostos e t =E9 a reta = tangente =E0 circunfer=EAncia pelo ponto B. Determine : a ) "Eu consequi,, deixa" b )a dist=E2ncia entre a reta t e a geratriz PC, em fun=E7=E3o = do =E2ngulo B=D4P. Obs : eu fiz v=E1rias vezes o desenho, e pelo que eu enxerguei = ser=E3o necess=E1rios tr=EAs tri=E2ngulos pra achar isso.. acho que a = base est=E1 na lei dos cossenos mas n=E3o tenho certeza. Entre a reta t = e a geratriz PC formar=E1 uma outra reta perpendiuclar as duas ( =E9 = isso que ele est=E1 pedindo n=E3o ? ), chame de ponto D onde essa reta = encontra a reta t e de ponto E onde essa reta encontra a geratriz... = s=F3 pra ficar mais f=E1cil. Nesse caso queremos achar o valor de DE, = obrigad=E3o !!!!!!!!!! ------=_NextPart_000_002E_01C25B62.DCE1BFA0 Content-Type: text/html; charset="iso-8859-1" Content-Transfer-Encoding: quoted-printable
sqrt eh raiz quadrada.
proj u,v eu estou usando para = representar o vetor=20 que se obtem qdo se projeta u em v. (ele tem direcao v e modulo =3D = |u|cos(angulo=20 entre u,v)..
[]'s
----- Original Message -----
From:=20 Afemano=20
Sent: Friday, September 13, = 2002 6:15=20 PM
Subject: Re: [obm-l] Geometria = PLZ=20 !!!

Nossa cara valeu mesmo.. =
mas o que =E9 "proj" e "sqrt" ? = hehehe desculpe mas=20 sou meio novo por aqui :D
 
Abra=E7os...
----- Original Message -----
From:=20 Marcio
Sent: Thursday, September 12, = 2002=20 10:35 PM
Subject: Re: [obm-l] = Geometria PLZ=20 !!!

   Se bem me lembro, essa = questao tmb=20 ja caiu num vestibular do IME, com exatamente os mesmos dados. =
   Na minha opiniao, a = maneira mais=20 simples de achar distancia entre retas reversas eh usando vetores = (bom, com=20 certeza eh a maneira mais simples de resolver esse tipo = de problema=20 usando apenas ascii.. :)
   Considere o cone = num sistema=20 de eixos Oxyz tq C =3D (0,0,2); A =3D (-1, 0,0); B=3D(1,0,0), P = =3D (cosx,=20 senx, 0).
   A reta t tem direcao = a=3D(0,1,0),=20 enqto que a geratriz PC tem direcao b=3D(cosx,senx,-2). Note que o = vetor axb=20 (produto vetorial) eh perpendicular a a e a b, e portanto da a = direcao da=20 perpendicular comum.
   Pegando um ponto X = qualquer de PC=20 e outro Y qq de t, temos um vetor cuja projecao na perpendicular = comum da=20 exatamente a distancia entre as duas retas reversas (afinal, as = projecoes de=20 X e Y em axb sao exatamente os extremos do segmento limitado pelas = retas=20 reversas na perpendicular comum!)
   Agora, as contas: = axb=3D(2, 0,=20 -cosx). Tomando X=3DC=3D(0,0,2) e Y=3DB=3D(1,0,0), temos = XY=3D(1,0,-2) e
d =3D |proj<XY, axb>| =3D=20 |XY . axb| / /|axb| =3D |2 + 2cosx| /=20 sqrt[4+cos^2(x)]
ObS: usei q proj u,v =3D = [(u.v)/(v.v)]v, e seu=20 modulo eh |u.v| / |v|
 
Abracos,
Marcio
 
----- Original Message -----
From:=20 Afemano=20
Sent: Wednesday, = September 11, 2002=20 8:14 PM
Subject: [obm-l] = Geometria PLZ=20 !!!

Ol=E1 galera.. algu=E9m resolve = esse exerc=EDcio=20 pra mim plz ?
 
( FUVEST ) A, = B e=20 P s=E3o tr=EAs pontos de uma circunfer=EAncia = de centro=20 O e raio unit=E1rio, base de um cone reto de = v=E9rtice=20 C e altura igual a 2. A e=20 B s=E3o diametralmente opostos e t =E9 = a reta=20 tangente =E0 circunfer=EAncia pelo ponto B. = Determine=20 :
a ) "Eu consequi,, = deixa"
b )a dist=E2ncia entre a reta = t e a=20 geratriz PC, em fun=E7=E3o do =E2ngulo=20 B=D4P.
 
Obs : eu fiz v=E1rias vezes o = desenho, e pelo=20 que eu enxerguei ser=E3o necess=E1rios tr=EAs tri=E2ngulos pra = achar isso.. acho=20 que a base est=E1 na lei dos cossenos mas n=E3o tenho certeza. = Entre a reta=20 t e a geratriz PC formar=E1 uma outra = reta=20 perpendiuclar as duas ( =E9 isso que ele est=E1 pedindo n=E3o ? = ), chame de=20 ponto D onde essa reta encontra a reta t = e de=20 ponto E onde essa reta encontra a geratriz... = s=F3 pra=20 ficar mais f=E1cil. Nesse caso queremos achar o valor de=20 DE, obrigad=E3o=20 = !!!!!!!!!!
------=_NextPart_000_002E_01C25B62.DCE1BFA0-- ========================================================================= Instruções para entrar na lista, sair da lista e usar a lista em http://www.mat.puc-rio.br/~nicolau/olimp/obm-l.html O administrador desta lista é ========================================================================= From owner-obm-l@sucuri.mat.puc-rio.br Fri Sep 13 21:37:48 2002 Return-Path: Received: (from majordom@localhost) by sucuri.mat.puc-rio.br (8.9.3/8.9.3) id VAA14782 for obm-l-MTTP; Fri, 13 Sep 2002 21:36:48 -0300 Received: from siegel.bol.com.br (siegel.bol.com.br [200.221.24.20]) by sucuri.mat.puc-rio.br (8.9.3/8.9.3) with ESMTP id VAA14778 for ; Fri, 13 Sep 2002 21:36:46 -0300 Received: from comp1 (200.221.24.191) by siegel.bol.com.br (5.1.071) id 3D5D1C9F009ADB27 for obm-l@mat.puc-rio.br; Fri, 13 Sep 2002 21:37:47 -0300 Message-ID: <001001c25bb3$ad7e3bc0$d15ab7c8@comp1> From: "Caio Augusto" To: Subject: [obm-l] Nivel Universitario Urgente Date: Fri, 13 Sep 2002 21:57:57 -0800 MIME-Version: 1.0 Content-Type: text/plain; charset="iso-8859-1" Content-Transfer-Encoding: 8bit X-Priority: 3 X-MSMail-Priority: Normal X-Mailer: Microsoft Outlook Express 5.50.4522.1200 X-MimeOLE: Produced By Microsoft MimeOLE V5.50.4522.1200 X-Sender-IP: 200.183.90.209 Sender: owner-obm-l@sucuri.mat.puc-rio.br Precedence: bulk Reply-To: obm-l@mat.puc-rio.br Ola pessoal da lista, Eu sou estudante Universitário e não me inscrevi na minha faculdade. Gostaria de saber se posso fazer a prova da OBM dia 14/09 no ETAPA. Alguém que seja de São Paulo e saiba me avise por favor urgentemente. Obrigado, Caio Augusto ========================================================================= Instruções para entrar na lista, sair da lista e usar a lista em http://www.mat.puc-rio.br/~nicolau/olimp/obm-l.html O administrador desta lista é ========================================================================= From owner-obm-l@sucuri.mat.puc-rio.br Fri Sep 13 22:21:38 2002 Return-Path: Received: (from majordom@localhost) by sucuri.mat.puc-rio.br (8.9.3/8.9.3) id WAA15651 for obm-l-MTTP; Fri, 13 Sep 2002 22:21:28 -0300 Received: from smtp.brturbo.com (smtp2.brturbo.com [200.199.201.30]) by sucuri.mat.puc-rio.br (8.9.3/8.9.3) with ESMTP id WAA15647 for ; Fri, 13 Sep 2002 22:21:24 -0300 Received: from x (200-181-95-057-bsace7011.dsl.telebrasilia.net.br [200.181.95.57]) by smtp.brturbo.com (Postfix) with SMTP id 07B0813CDD6 for ; Fri, 13 Sep 2002 22:22:36 -0300 (BRT) Message-ID: <001301c25b8d$3f407360$0201a8c0@x> From: "Jeremias de Paula Eduardo" To: References: <003301c25b6f$7a3cfd30$c300000a@wsjujuba> Subject: [obm-l] =?iso-8859-1?Q?Re:_=5Bobm-l=5D_fun=E7=F5es?= Date: Fri, 13 Sep 2002 22:22:52 -0300 MIME-Version: 1.0 Content-Type: multipart/alternative; boundary="----=_NextPart_000_0010_01C25B74.19B7B900" X-Priority: 3 X-MSMail-Priority: Normal X-Mailer: Microsoft Outlook Express 5.00.2919.6600 X-MimeOLE: Produced By Microsoft MimeOLE V5.00.2919.6600 Sender: owner-obm-l@sucuri.mat.puc-rio.br Precedence: bulk Reply-To: obm-l@mat.puc-rio.br This is a multi-part message in MIME format. ------=_NextPart_000_0010_01C25B74.19B7B900 Content-Type: text/plain; charset="iso-8859-1" Content-Transfer-Encoding: quoted-printable Se vc esbo=E7ar o gr=E1fico como pede, colocando valores para os pontos = das multiplas escolhas perceber=E1 primeiro que a parte positiva da = fun=E7=E3o quadr=E1tca fica entre as ra=EDzes -1 e 2. E que essa = fun=E7=E3o corta a fun=E7=E3o exponencial dada entre -1 e 0 e corta = denovo entre 0 e 1.=20 Se alguem a=ED tiver uma sugest=E3o para para resolver algebricamente... Como vc est=E1 estudando uma prova de vestibular, n=E3o deixe de = perceber que a op=E7=E3o "c" descarta "b" e a "d". E a "a" fica fora do = intervalo positivo da fun=E7=E3o quadr=E1tica, restando a "c" e a "e", = sendo que a a "e" vc pode descartar imaginando que toda fun=E7=E3o = exponencial passa pelo eixo y sempre no ponto 1, e a fun=E7=E3o = quadr=E1tica pelo ponto c(de ax^2+bx+c) que =E9 2, ou seja, nesse ponto, = com x=3D0, ela j=E1 ultrapassou(na altura do grafico) a fun=E7=E3o = exponencial, logo o primeiro encontro entra as fun=E7=F5es foi anterior. = Pode ser tosco responder uma quest=E3o dessa forma, mas vestibular n=E3o = precisa ser bonito, =E9 s=F3 passar, e a avalia=E7=E3o das alternativas = e uma boa imagina=E7=E3o s=E3o bons recursos. Jeremias de Paula Eduardo E a=EA, pessoal! Mais uma que eu gostaria de ajuda (em n=EDvel secund=E1rio!!) Muito obrigada! Ju ------------------------------- (UFRGS - 2002) Esbo=E7ando os gr=E1ficos das fun=E7=F5es definidas por = f(a) =3D 5=AA e g(a)=3D2 + x - x=B2 num mesmo plano cartesiano, = verifica-se que todas as ra=EDzes da equa=E7=E3o f(a)=3Dg(a) pertencem = ao intervalo: =20 a) (-2,-1) b) (-1,0) c) (-1,1) d) (0,1) e) (0,2) ------=_NextPart_000_0010_01C25B74.19B7B900 Content-Type: text/html; charset="iso-8859-1" Content-Transfer-Encoding: quoted-printable
Se vc esbo=E7ar o gr=E1fico como pede, = colocando=20 valores para os pontos das multiplas escolhas perceber=E1 primeiro = que a=20 parte positiva da fun=E7=E3o quadr=E1tca fica entre as ra=EDzes -1 e 2. = E que essa=20 fun=E7=E3o corta a fun=E7=E3o exponencial dada entre -1 e 0 e corta = denovo entre 0 e 1.=20
Se alguem a=ED tiver uma sugest=E3o = para para resolver=20 algebricamente...
 
Como vc est=E1 estudando uma prova de = vestibular, n=E3o=20 deixe de perceber que a op=E7=E3o "c" descarta "b" e a "d". E a "a" = fica fora=20 do intervalo positivo da fun=E7=E3o quadr=E1tica, restando a "c" e a = "e", sendo que a=20 a "e" vc pode descartar imaginando que toda fun=E7=E3o exponencial = passa pelo=20 eixo y sempre no ponto 1, e a fun=E7=E3o quadr=E1tica pelo ponto c(de = ax^2+bx+c) que =E9=20 2, ou seja, nesse ponto, com x=3D0, ela j=E1 ultrapassou(na altura = do grafico)=20 a fun=E7=E3o exponencial, logo o primeiro encontro entra as fun=E7=F5es = foi=20 anterior. 
 
Pode ser tosco responder uma quest=E3o = dessa forma,=20 mas vestibular n=E3o precisa ser bonito, =E9 s=F3 passar, e a = avalia=E7=E3o das=20 alternativas e uma boa imagina=E7=E3o s=E3o bons = recursos.
 
Jeremias de Paula Eduardo
E a=EA, pessoal!
Mais uma que eu gostaria de ajuda (em = n=EDvel=20 secund=E1rio!!)
Muito obrigada!
Ju
-------------------------------
(UFRGS - 2002) Esbo=E7ando os = gr=E1ficos das fun=E7=F5es=20 definidas por f(a) =3D 5=AA e g(a)=3D2 + x - x=B2 num mesmo plano = cartesiano,=20 verifica-se que todas as ra=EDzes da equa=E7=E3o f(a)=3Dg(a) pertencem = ao=20 intervalo:
 
a) = (-2,-1)
b) = (-1,0)
c) = (-1,1)
d) = (0,1)
e)=20 = (0,2)
=
------=_NextPart_000_0010_01C25B74.19B7B900-- ========================================================================= Instruções para entrar na lista, sair da lista e usar a lista em http://www.mat.puc-rio.br/~nicolau/olimp/obm-l.html O administrador desta lista é ========================================================================= From owner-obm-l@sucuri.mat.puc-rio.br Fri Sep 13 22:51:10 2002 Return-Path: Received: (from majordom@localhost) by sucuri.mat.puc-rio.br (8.9.3/8.9.3) id WAA16167 for obm-l-MTTP; Fri, 13 Sep 2002 22:48:57 -0300 Received: from shannon.bol.com.br (shannon.bol.com.br [200.221.24.13]) by sucuri.mat.puc-rio.br (8.9.3/8.9.3) with ESMTP id WAA16162 for ; Fri, 13 Sep 2002 22:48:55 -0300 Received: from bol.com.br (200.221.24.133) by shannon.bol.com.br (5.1.071) id 3D60E4DC0071808F for obm-l@mat.puc-rio.br; Fri, 13 Sep 2002 22:49:52 -0300 Date: Fri, 13 Sep 2002 22:47:40 -0300 Message-Id: Subject: [obm-l] =?iso-8859-1?q?Re=3A=5Bobm=2Dl=5D_infinitas_progress=F5es?= MIME-Version: 1.0 Content-Type: multipart/mixed; boundary="_=__=_XaM3_Boundary.1031968060.2A.234424.42.22235.52.42.101010.811526410" From: "rafaelc.l" To: obm-l@mat.puc-rio.br X-XaM3-API-Version: 2.4.3.4.4 X-SenderIP: 200.176.166.168 Sender: owner-obm-l@sucuri.mat.puc-rio.br Precedence: bulk Reply-To: obm-l@mat.puc-rio.br --_=__=_XaM3_Boundary.1031968060.2A.234424.42.22235.52.42.101010.811526410 Content-Type: text/plain;charset="iso-8859-1" Content-Transfer-Encoding: quoted-printable (UFRGS - 2002) A disposi=E7=E3o de n=FAmeros abaixo representa infinitas progress=F5es. 1/2 1/4 1/4 1/4 1/8 1/8 1/8 1/8 1/8 1/16 1/16 1/16 1/16 1/16 1/16 1/16 =95 =95 =95 =95 =95 =95 =95 =95 =95 =95 =95 =95 =95 =95 =95 =95 =95 =95 =95 =95 Considere as afirma=E7=F5es referentes =E0 disposi=E7=E3o dada. (1) A d=E9cima linha =E9 formada por 19 elementos. (2) Chamando-se de a1 o primeiro elemento de uma coluna qualquer, a soma dos termos desta coluna =E9 2a1. (3) A soma dos infinitos elementos da disposi=E7=E3o =E9 3. Quais s=E3o verdadeiras? (a) Apenas 1. (b) Apenas 1 e 2. (c) Apenas 1 e 3. (d) Apenas 2 e 3. (e) 1, 2, e 3. 1- verdadeira, pois a cada linha aumenta dois elementos. A linha 1 tem 1 elemento e a d=E9cima tem 1+ 9.2=3D19 elementos. 2- sendo s=3D a1/1-q a soma numa p.g infinita, e sendo a raz=E3o=3D1/2, para qualquer coluna, S=3D a1/1-1/2=3D 2a1. 3- Como a soma de cada coluna =E9 2a1, temos que a soma total =E9 a soma da coluna do meio com o dobro das outras colunas(uma de cada lado). S=3D 2.1/2 + 2(2.1/4 + 2.1/8 + 2.1/16 +...) =3D 1 + 4( 1/4 + 1/8 + 1/16...)=3D 1 + 4.2.1/4=3D 1+2=3D3. verdadeira __________________________________________________________________________ AcessoBOL, s=F3 R$ 9,90! O menor pre=E7o do mercado! Assine j=E1! http://www.bol.com.br/acessobol --_=__=_XaM3_Boundary.1031968060.2A.234424.42.22235.52.42.101010.811526410 Content-Type: text/plain; name="0000020O" Content-Transfer-Encoding: base64 KFVGUkdTIC0gMjAwMikgQSBkaXNwb3Np5+NvIGRlIG76bWVyb3MgYWJhaXhvIHJlcHJlc2Vu dGEgaW5maW5pdGFzIHByb2dyZXNz9WVzLg0KICAgICAgICAgMS8yICAgICAgDQogICAgICAg IDEvNCAxLzQgMS80ICAgICANCiAgICAgICAxLzggMS84IDEvOCAxLzggMS84ICAgIA0KICAg ICAgMS8xNiAxLzE2IDEvMTYgMS8xNiAxLzE2IDEvMTYgMS8xNiAgIA0KICAgICAuIC4gLiAu IC4gLiAuIC4gLiAgDQogICAgICAuIC4gLiAuIC4gLiAuIC4gLiAuIC4gDQoNCg0KQ29uc2lk ZXJlIGFzIGFmaXJtYef1ZXMgcmVmZXJlbnRlcyDgIGRpc3Bvc2nn428gZGFkYS4NCigxKSBB IGTpY2ltYSBsaW5oYSDpIGZvcm1hZGEgcG9yIDE5IGVsZW1lbnRvcy4NCigyKSBDaGFtYW5k by1zZSBkZSBhMSBvIHByaW1laXJvIGVsZW1lbnRvIGRlIHVtYSBjb2x1bmEgcXVhbHF1ZXIs IGEgc29tYSBkb3MgdGVybW9zIGRlc3RhIGNvbHVuYSDpIDJhMS4NCigzKSBBIHNvbWEgZG9z IGluZmluaXRvcyBlbGVtZW50b3MgZGEgZGlzcG9zaefjbyDpIDMuDQoNClF1YWlzIHPjbyB2 ZXJkYWRlaXJhcz8NCihhKSBBcGVuYXMgMS4NCihiKSBBcGVuYXMgMSBlIDIuDQooYykgQXBl bmFzIDEgZSAzLg0KKGQpIEFwZW5hcyAyIGUgMy4NCihlKSAxLCAyLCBlIDMuDQoNCg0K --_=__=_XaM3_Boundary.1031968060.2A.234424.42.22235.52.42.101010.811526410-- ========================================================================= Instruções para entrar na lista, sair da lista e usar a lista em http://www.mat.puc-rio.br/~nicolau/olimp/obm-l.html O administrador desta lista é ========================================================================= From owner-obm-l@sucuri.mat.puc-rio.br Fri Sep 13 23:44:35 2002 Return-Path: Received: (from majordom@localhost) by sucuri.mat.puc-rio.br (8.9.3/8.9.3) id XAA17286 for obm-l-MTTP; Fri, 13 Sep 2002 23:43:31 -0300 Received: from artemis.opendf.com.br (artemis.opengate.com.br [200.181.71.15]) by sucuri.mat.puc-rio.br (8.9.3/8.9.3) with ESMTP id XAA17282 for ; Fri, 13 Sep 2002 23:43:28 -0300 Received: from localhost (localhost [127.0.0.1]) by artemis.opendf.com.br (Postfix) with ESMTP id 219AE1CAB9 for ; Fri, 13 Sep 2002 23:44:15 -0300 (BRT) Received: from artur (200-181-88-186-bsace7001.dsl.telebrasilia.net.br [200.181.88.186]) by artemis.opendf.com.br (Postfix) with ESMTP id A75B71BF90 for ; Fri, 13 Sep 2002 23:44:11 -0300 (BRT) From: "Artur Costa Steiner" To: Subject: RE: [obm-l] Axioma da Escolha Date: Fri, 13 Sep 2002 23:45:41 -0700 Organization: Steiner Consultoria LTDA Message-ID: <002c01c25bba$5959d7a0$0b01a8c0@mshome.net> MIME-Version: 1.0 Content-Type: text/plain; charset="iso-8859-1" X-Priority: 3 (Normal) X-MSMail-Priority: Normal X-Mailer: Microsoft Outlook, Build 10.0.2627 Importance: Normal In-Reply-To: X-MimeOLE: Produced By Microsoft MimeOLE V6.00.2600.0000 X-Virus-Scanned: by AMaViS new-20020517 Content-Transfer-Encoding: 8bit X-MIME-Autoconverted: from quoted-printable to 8bit by sucuri.mat.puc-rio.br id XAA17283 Sender: owner-obm-l@sucuri.mat.puc-rio.br Precedence: bulk Reply-To: obm-l@mat.puc-rio.br > Apenas lembrando, porque costuma-se realçar quando se usa o axioma da > escolha, há uma corrente filosófica de matemáticos que não aceitam o > axioma > da escolha: os construtivistas (ou, mais geralmente, os intuicionistas). O > axioma da escolha nos garante a existência de objetos que não podemos > determinar quem, exatamente, ele é. Esse tipo de coisa os construtivistas > não aceitam, pois de que serve saber que existe alguma coisa que nunca > saberemos quem é, ou onde está? O teorema de Weierstrass, que diz que toda > função real contínua sobre um intervalo fechado assume máximo, não é > aceita > pelos construtivistas, pois não podemos exibir esse ponto de máximo. [Artur Costa Steiner] Ms este teorema é um dos mais importantes da matemática Por > outro lado não podemos dizer que não existe ponto de máximo, pois isso > seria > garantir que todos os pontos não são de máximo, o que não devemos > assegurar. > Por isso na lógica intuicionista "A ou não A" pode ser falso, e A não é > equivalente a "não não A". [Artur Costa Steiner] Acho que é também importante lembrar que muitas provas na matemática basiam-se em infinitas escolhas. Por exemplo, várias das provas dos teoremas ligados à compaticidade de espaços métricos enquadram-se nesta categoria, como o que afirma que S é compacto <===> S é sequencialmente compacto. Parece-me que estas provam usam o axioma da escolha. E ninguém as questiona. , > Todas essas complicações geradas pelo construtivismo fizeram que esse > caísse um pouco no esquecimento. Hoje parece que há poucos matemáticos > construtivistas. Mas devemos nos lembrar que o argumento central que gerou > o > construtivismo faz sentido. Realmente, podemos pensar o que fazemos com > coisas obtidas não construtivamente. Enfim, há sempre uma fagulha de > construtivista em nós. É certo que os mais radicais não admitem nem prova > por absurdo, mas o axioma da escolha já seria o maior crime que se poderia > cometer contra o construtivismo. Por isso, nas demonstrações, é sempre bom > ressaltar o que é construtivo e o que não é. Por exemplo, o Paradoxo de > Banach-Tarski, sobre a duplicação da esfera, citada pelo Paulo, é > não-construtiva. > Sobre o problema da violência, resta um consolo: se o conjunto dos > bandidos, dado pelo problema, já estiver bem ordenado (por exemplo, se é > enumerável), não precisamos do axioma da escolha, e não cometeremos uma > "violência" contra os intuicionistas. O difícil vai ser achar bandidos bem > ordenados... > [Artur Costa Steiner] Quando podemos fazer infinitas escolhas sem usar o axioma da escolha? Sempre que tivermos conjuntos bem ordenados? Por exemplo, se fizermos infinitas escolhas em infinitos subconjuntos dos racionais, então não precisamos do axioma? Artur > ========================================================================= Instruções para entrar na lista, sair da lista e usar a lista em http://www.mat.puc-rio.br/~nicolau/olimp/obm-l.html O administrador desta lista é ========================================================================= From owner-obm-l@sucuri.mat.puc-rio.br Sat Sep 14 00:03:10 2002 Return-Path: Received: (from majordom@localhost) by sucuri.mat.puc-rio.br (8.9.3/8.9.3) id AAA17712 for obm-l-MTTP; Sat, 14 Sep 2002 00:02:40 -0300 Received: from sr1.terra.com.br (sr1.terra.com.br [200.176.3.16]) by sucuri.mat.puc-rio.br (8.9.3/8.9.3) with ESMTP id AAA17708 for ; Sat, 14 Sep 2002 00:02:38 -0300 Received: from penha.terra.com.br (penha.terra.com.br [200.176.3.43]) by sr1.terra.com.br (Postfix) with ESMTP id 444B37015B for ; Sat, 14 Sep 2002 00:03:50 -0300 (EST) Received: from Itautec.terra.com.br (200-158-60-172.dsl.telesp.net.br [200.158.60.172]) (authenticated user bruleite) by penha.terra.com.br (Postfix) with ESMTP id C13DE681F7 for ; Sat, 14 Sep 2002 00:03:48 -0300 (EST) Message-Id: <5.1.0.14.0.20020913235842.00a0acf0@pop.sao.terra.com.br> X-Sender: bruleite@pop.sao.terra.com.br X-Mailer: QUALCOMM Windows Eudora Version 5.1 Date: Fri, 13 Sep 2002 23:59:37 -0300 To: obm-l@mat.puc-rio.br From: "Bruno F. C. Leite" Subject: Re: [obm-l] Nivel Universitario Urgente In-Reply-To: <001001c25bb3$ad7e3bc0$d15ab7c8@comp1> Mime-Version: 1.0 Content-Type: text/plain; charset="iso-8859-1"; format=flowed Content-Transfer-Encoding: 8bit X-MIME-Autoconverted: from quoted-printable to 8bit by sucuri.mat.puc-rio.br id AAA17709 Sender: owner-obm-l@sucuri.mat.puc-rio.br Precedence: bulk Reply-To: obm-l@mat.puc-rio.br At 21:57 13/09/02 -0800, you wrote: >Ola pessoal da lista, > >Eu sou estudante Universitário e não me inscrevi na minha faculdade. Eu não sei bem, mas acho que vc deveria tentar sim. Aliás eu queria saber a que horas é a prova... Bruno >Gostaria de saber se posso fazer a prova da OBM dia 14/09 no ETAPA. Alguém >que seja de São Paulo e saiba me avise por favor urgentemente. > >Obrigado, >Caio Augusto > >========================================================================= >Instruções para entrar na lista, sair da lista e usar a lista em >http://www.mat.puc-rio.br/~nicolau/olimp/obm-l.html >O administrador desta lista é >========================================================================= ========================================================================= Instruções para entrar na lista, sair da lista e usar a lista em http://www.mat.puc-rio.br/~nicolau/olimp/obm-l.html O administrador desta lista é ========================================================================= From owner-obm-l@sucuri.mat.puc-rio.br Sat Sep 14 08:00:06 2002 Return-Path: Received: (from majordom@localhost) by sucuri.mat.puc-rio.br (8.9.3/8.9.3) id HAA21340 for obm-l-MTTP; Sat, 14 Sep 2002 07:59:09 -0300 Received: from web13106.mail.yahoo.com (web13106.mail.yahoo.com [216.136.174.151]) by sucuri.mat.puc-rio.br (8.9.3/8.9.3) with SMTP id HAA21336 for ; Sat, 14 Sep 2002 07:59:06 -0300 Message-ID: <20020914110018.57138.qmail@web13106.mail.yahoo.com> Received: from [200.207.158.20] by web13106.mail.yahoo.com via HTTP; Sat, 14 Sep 2002 04:00:18 PDT Date: Sat, 14 Sep 2002 04:00:18 -0700 (PDT) From: edmilson motta Subject: Re: [obm-l] Nivel Universitario Urgente To: obm-l@mat.puc-rio.br In-Reply-To: <5.1.0.14.0.20020913235842.00a0acf0@pop.sao.terra.com.br> MIME-Version: 1.0 Content-Type: text/plain; charset=us-ascii Sender: owner-obm-l@sucuri.mat.puc-rio.br Precedence: bulk Reply-To: obm-l@mat.puc-rio.br A prova será realizada no Etapa. O horário é das 14h às 18h30min. Abraços, Ed. --- "Bruno F. C. Leite" wrote: > At 21:57 13/09/02 -0800, you wrote: > >Ola pessoal da lista, > > > >Eu sou estudante Universitário e não me inscrevi na > minha faculdade. > > Eu não sei bem, mas acho que vc deveria tentar sim. > Aliás eu queria saber a > que horas é a prova... > > Bruno > > >Gostaria de saber se posso fazer a prova da OBM dia > 14/09 no ETAPA. Alguém > >que seja de São Paulo e saiba me avise por favor > urgentemente. > > > >Obrigado, > >Caio Augusto > > > >========================================================================= > >Instruções para entrar na lista, sair da lista e > usar a lista em > >http://www.mat.puc-rio.br/~nicolau/olimp/obm-l.html > >O administrador desta lista é > > >========================================================================= > > ========================================================================= > Instruções para entrar na lista, sair da lista e > usar a lista em > http://www.mat.puc-rio.br/~nicolau/olimp/obm-l.html > O administrador desta lista é > > ========================================================================= __________________________________________________ Do you Yahoo!? Yahoo! News - Today's headlines http://news.yahoo.com ========================================================================= Instruções para entrar na lista, sair da lista e usar a lista em http://www.mat.puc-rio.br/~nicolau/olimp/obm-l.html O administrador desta lista é ========================================================================= From owner-obm-l@sucuri.mat.puc-rio.br Sat Sep 14 13:16:22 2002 Return-Path: Received: (from majordom@localhost) by sucuri.mat.puc-rio.br (8.9.3/8.9.3) id NAA23637 for obm-l-MTTP; Sat, 14 Sep 2002 13:13:36 -0300 Received: from traven.pub1 (traven.uol.com.br [200.221.4.39]) by sucuri.mat.puc-rio.br (8.9.3/8.9.3) with ESMTP id NAA23633 for ; Sat, 14 Sep 2002 13:13:33 -0300 Received: from slap ([200.207.152.104]) by traven.pub1 (8.9.1/8.9.1) with SMTP id NAA05765 for ; Sat, 14 Sep 2002 13:02:00 -0300 (BRT) Message-ID: <002501c25c0a$ab7b2d40$6898cfc8@slap> From: "Afemano" To: References: <20020914110018.57138.qmail@web13106.mail.yahoo.com> Subject: Re: [obm-l] Nivel Universitario Urgente Date: Sat, 14 Sep 2002 13:20:41 -0300 MIME-Version: 1.0 Content-Type: text/plain; charset="iso-8859-1" Content-Transfer-Encoding: 8bit X-Priority: 3 X-MSMail-Priority: Normal X-Mailer: Microsoft Outlook Express 6.00.2600.0000 X-MimeOLE: Produced By Microsoft MimeOLE V6.00.2600.0000 Sender: owner-obm-l@sucuri.mat.puc-rio.br Precedence: bulk Reply-To: obm-l@mat.puc-rio.br ÉÉÉÉÉDDDDDD professor do ETAPA ? HEHEHE sou seu aluno Ed.. heheheh nem sabia que você participava dessa lista huhuhuhuh !! Vai ser meu plantonista agora hauhauhauhauh zuera.. falo ae []´s !!!! ----- Original Message ----- From: "edmilson motta" To: Sent: Saturday, September 14, 2002 8:00 AM Subject: Re: [obm-l] Nivel Universitario Urgente > A prova será realizada no Etapa. O horário é das 14h > às 18h30min. > > Abraços, Ed. > > > --- "Bruno F. C. Leite" wrote: > > At 21:57 13/09/02 -0800, you wrote: > > >Ola pessoal da lista, > > > > > >Eu sou estudante Universitário e não me inscrevi na > > minha faculdade. > > > > Eu não sei bem, mas acho que vc deveria tentar sim. > > Aliás eu queria saber a > > que horas é a prova... > > > > Bruno > > > > >Gostaria de saber se posso fazer a prova da OBM dia > > 14/09 no ETAPA. Alguém > > >que seja de São Paulo e saiba me avise por favor > > urgentemente. > > > > > >Obrigado, > > >Caio Augusto > > > > > > >========================================================================= > > >Instruções para entrar na lista, sair da lista e > > usar a lista em > > >http://www.mat.puc-rio.br/~nicolau/olimp/obm-l.html > > >O administrador desta lista é > > > > > >========================================================================= > > > > > ========================================================================= > > Instruções para entrar na lista, sair da lista e > > usar a lista em > > http://www.mat.puc-rio.br/~nicolau/olimp/obm-l.html > > O administrador desta lista é > > > > > ========================================================================= > > > __________________________________________________ > Do you Yahoo!? > Yahoo! News - Today's headlines > http://news.yahoo.com > ========================================================================= > Instruções para entrar na lista, sair da lista e usar a lista em > http://www.mat.puc-rio.br/~nicolau/olimp/obm-l.html > O administrador desta lista é > ========================================================================= ========================================================================= Instruções para entrar na lista, sair da lista e usar a lista em http://www.mat.puc-rio.br/~nicolau/olimp/obm-l.html O administrador desta lista é ========================================================================= From owner-obm-l@sucuri.mat.puc-rio.br Sat Sep 14 13:26:55 2002 Return-Path: Received: (from majordom@localhost) by sucuri.mat.puc-rio.br (8.9.3/8.9.3) id NAA23769 for obm-l-MTTP; Sat, 14 Sep 2002 13:24:07 -0300 Received: from puma.unisys.com.br (ns2.unisys.com.br [200.220.64.7]) by sucuri.mat.puc-rio.br (8.9.3/8.9.3) with ESMTP id NAA23765 for ; Sat, 14 Sep 2002 13:24:04 -0300 Received: from jf (riohiper01p99.uninet.com.br [200.220.2.99]) by puma.unisys.com.br (8.12.3/8.12.3) with SMTP id g8EGPB4w005200 for ; Sat, 14 Sep 2002 13:25:14 -0300 (EST) X-Spam-Filter: check_local@puma.unisys.com.br by digitalanswers.org Message-ID: <010201c25c0c$0c880940$6302dcc8@jf> From: "Jose Francisco Guimaraes Costa" To: References: <00cd01c25b54$bcc1e780$df02dcc8@josefran> <20020913181252.B5248@sucuri.mat.puc-rio.br> Subject: [obm-l] =?iso-8859-1?Q?Re:_=5Bobm-l=5D_Re:_=5Bobm-l=5D_En:_=5Bobm-l=5D_Re:_=5Bobm?= =?iso-8859-1?Q?-l=5D_D=FAvida_sobre_sequ=EAncias_rand=F4micas_=28prob?= =?iso-8859-1?Q?lema_do_tipo_NP=29?= Date: Sat, 14 Sep 2002 13:07:29 -0300 MIME-Version: 1.0 Content-Type: text/plain; charset="iso-8859-1" Content-Transfer-Encoding: 8bit X-Priority: 3 X-MSMail-Priority: Normal X-Mailer: Microsoft Outlook Express 6.00.2600.0000 X-MimeOLE: Produced By Microsoft MimeOLE V6.00.2600.0000 Sender: owner-obm-l@sucuri.mat.puc-rio.br Precedence: bulk Reply-To: obm-l@mat.puc-rio.br Imediatamente após ter clicado SEND vi que havia escrito bobagem. Arranquei o fio do telefone, mas já era tarde; deu apenas para ver o último bit sumindo parede adentro. Mas já que estamos falando nisso. N diz "conjectura-se (mas isto não é um teorema) que estas duas seqüências de algarismos sejam pseudo-aleatórias...". Assumindo que uma conjectura seja alguma coisa que tudo indica que é verdadeira, mas que não se conseguiu provar, e quando se prova vira um teorema, existiria alguma forma de se provar a conjectura acima? Tal prova não exigiria, antes, definir de forma rigorosa o que vem a ser uma "seqüência de algarismos aleatórios"? JF ----- Original Message ----- From: "Nicolau C. Saldanha" To: Sent: Friday, September 13, 2002 6:12 PM Subject: [obm-l] Re: [obm-l] En: [obm-l] Re: [obm-l] Dúvida sobre sequências randômicas (problema do tipo NP) > On Fri, Sep 13, 2002 at 03:38:13PM -0300, Jose Francisco Guimaraes Costa wrote: > > Uma seqüência realmente randômica é a seqüência de dígitos que compõem > > números como pi e a base dos logaritmos naturais. > > A seqüência dos algarismos de pi ou de e pode ser obtida por um algoritmo > e portanto não é realmente randômica. Conjectura-se (mas isto não é um teorema) > que estas duas seqüências de algarismos sejam pseudo-aleatórias (por exemplo, > que a freqüência dos 10 algarismos seja a mesma). > > []s, N. > ========================================================================= > Instruções para entrar na lista, sair da lista e usar a lista em > http://www.mat.puc-rio.br/~nicolau/olimp/obm-l.html > O administrador desta lista é > ========================================================================= > ========================================================================= Instruções para entrar na lista, sair da lista e usar a lista em http://www.mat.puc-rio.br/~nicolau/olimp/obm-l.html O administrador desta lista é ========================================================================= From owner-obm-l@sucuri.mat.puc-rio.br Sat Sep 14 13:26:56 2002 Return-Path: Received: (from majordom@localhost) by sucuri.mat.puc-rio.br (8.9.3/8.9.3) id NAA23778 for obm-l-MTTP; Sat, 14 Sep 2002 13:24:11 -0300 Received: from puma.unisys.com.br (ns2.unisys.com.br [200.220.64.7]) by sucuri.mat.puc-rio.br (8.9.3/8.9.3) with ESMTP id NAA23774 for ; Sat, 14 Sep 2002 13:24:08 -0300 Received: from jf (riohiper01p99.uninet.com.br [200.220.2.99]) by puma.unisys.com.br (8.12.3/8.12.3) with SMTP id g8EGPB52005200 for ; Sat, 14 Sep 2002 13:25:20 -0300 (EST) X-Spam-Filter: check_local@puma.unisys.com.br by digitalanswers.org Message-ID: <010401c25c0c$0df024c0$6302dcc8@jf> From: "Jose Francisco Guimaraes Costa" To: "obm-l" Subject: [obm-l] circuito IME/ITA Date: Sat, 14 Sep 2002 13:29:24 -0300 MIME-Version: 1.0 Content-Type: multipart/alternative; boundary="----=_NextPart_000_00F6_01C25BF2.BDC7DB00" X-Priority: 3 X-MSMail-Priority: Normal X-Mailer: Microsoft Outlook Express 6.00.2600.0000 X-MimeOLE: Produced By Microsoft MimeOLE V6.00.2600.0000 Sender: owner-obm-l@sucuri.mat.puc-rio.br Precedence: bulk Reply-To: obm-l@mat.puc-rio.br This is a multi-part message in MIME format. ------=_NextPart_000_00F6_01C25BF2.BDC7DB00 Content-Type: text/plain; charset="iso-8859-1" Content-Transfer-Encoding: quoted-printable Eu sou formado pelo ITA, e nesta qualidade gostaria de tecer alguns = coment=E1rios sobre o que foi dito na mensagem abaixo. O ITA n=E3o deve ser considerado uma institui=E7=E3o militar. Embora = subordinado ao Comando da Aeron=E1utica, seu reitor =E9 um cargo civil - = nos seus 53 anos de exist=EAncia teve apenas um reitor militar, o = ent=E3o Brigadeiro Engenheiro Tercio Pacitti, formado pelo ITA, muito = mais educador do que militar, autor do Fortran Monitor (livro que = ensinou Fortran a milhares), e que, enquanto reitor, n=E3o usava farda = -, seus professores s=E3o civis, e atualmente forma civis e militares, = dependendo da op=E7=E3o que =E9 feita quando o candidato se inscreve no = vestibular. Quem opta pela carreira civil est=E1 livre de qualquer = obriga=E7=E3o com a Aeron=E1utica quando se forma. Pode ir trabalhar = onde bem quiser. A =FAnica obriga=E7=E3o militar de todo aluno (e desde = 95, aluna) do ITA =E9 cursar o Centro de Prepara=E7=E3o de Oficiais da = Reserva da Aeron=E1utica no primeiro ano, o que toma um dia por semana, = =FAnica ocasi=E3o em que ele usa farda. =C9 excelente o relacionamento entre o ITA e as empresas de uma maneira = geral, n=E3o s=F3 do ramo de engenharia. H=E1 uns dez anos foi criada a = Funda=E7=E3o Casimiro Montenegro Filho (nome do idealizador e "fundador" = do ITA), cujo principal objetivo =E9 desenvolver projetos de = coopera=E7=E3o entre a "escola" (como n=F3s chamamos o ITA) e o mundo = empresarial. Os resultados t=EAm sido excelentes. No ITA n=E3o h=E1 absolutamente nenhuma restri=E7=E3o acad=EAmica quanto = =E0 =E1rea de ci=EAncias humanas. Durante o curso fundamental (dois = primeiros anos) os alunos s=E3o obrigados a frequentar mat=E9rias do = Departamento de Ci=EAncias Humanas, que tamb=E9m oferece mat=E9rias = opcionais para quem quiser durante os cinco anos do curso. Existe um = centro acad=EAmico (Centro Acad=EAmico Santos Dumont) ativo, que promove atividades culturais as mais variadas - de = teatro a concursos liter=E1rios a shows a... O curso do ITA =E9 extremamente puxado. Isso, aliado ao isolamento do = aluno do ITA, que estuda, mora, come, pratica esportes, se diverte, = enfim, vive dentro do campus do CTA (Centro T=E9cnico Aeroespacial) faz = com que muitos desistam nos primeiros meses, ou sejam desligados por = problemas de aproveitamento ao fim do primeiro per=EDodo. E uma vez aqui = fora, de volta para sua cidade natal, ningu=E9m vai dizer que n=E3o = aguentou o ritmo, ou que ficou com saudades de casa, ou que foi = desligado por nota baixa. O ITA "n=E3o tem dinheiro para nada" da mesma forma que, hoje em dia e = desde sempre, ningu=E9m - nem nenhuma institui=E7=E3o - no Brasil tem = dinheiro para nada. Eu sou suspeito para falar sobre o nivel dos cursos do ITA. Para quem = quiser ter uma id=E9ia, sugiro conferir os resultados do prov=E3o. JF -----Mensagem Original----- De: kenji yoshitaki Para: Enviada em: Sexta-feira, 13 de Setembro de 2002 10:05 Assunto: Re:[obm-l] circuito IME > Rafel, > Bom,n=E3o quis de maneira alguma subestimar a competencia dos cursos = do ime e do ita, mas o fato de serem institui=E7oes militares restringe muito o relacionamento( e convenios) com as v=E1rias empresas do ramo da = engenharia e servi=E7os em geral. Isso sem contar as restri=E7oes academicas com = =E1reas de humanas. > Em rela=E7ao aos meus colegas daqui da poli, um fez mais um semestre = de curso l=E1 , e outro s=F3 fez alguns meses, ambos decidiram fazer = cursinho de novo pra entrar aqui, pq segundo eles nao compensava ficar mais dois = anos naquela historia de trabalhar pro exercito depois da formatura. E tb por causa da crise que o exercito , aeronautica passam.. nao possibilatava grandes investimentos na estrutura dos cursos tanto do ime como do ita. Tenho um tio que =E9 da embraer e da aula no ita, ele confirma tudo, na = =E9poca dele o ita nao tinha dinheiro pra nada... > Nao sei pra que =E1rea vc vai , mas cursos como engenharia eletrica e computa=E7ao , vc deve saber que a Unicamp , por exemplo, =E9 a melhor = da america latina.. Isso por que os investimentos nao vem apenas do governo = , assim como aqui na poli, as empresas fazem parcerias que elevam a = estrutura dos cursos por meio de investimentos. > > E sobre o n=EDvel do curso , eu tb pensava que era maior no ita e ime, = mas depois vi o qu=E3o dificil =E9 fazer ( bem feito) um curso de = engenharia, pq na verdade quem faz o n=EDvel =E9 o pr=F3prio aluno. Depende apenas dele = mesmo buscar conhecimento e investir na sua forma=E7ao.Isso vale tambem, buscar o = "saber" at=E9 mesmo em assuntos como hist=F3ria e geografia, porque nessa = gera=E7ao que passamos, a sabedoria =E9 o maior valor que alguem pode ter. Quem dessa = lista =E9 engenheiro, matem=E1tico,etc..., deve saber o que estou dizendo. > > Bom =E9 isso , qualquer outra opini=E3o , pode mandar. > Falow, um abra=E7o! > > > "A matem=E1tica =E9 a mais alta das ci=EAncias, o dom mais alto que os = deuses deram aos homens" > Arquimedes(287a.C.-212a.C.) > > Kenji Yoshitaki > Engenharia/Poli-USP ------=_NextPart_000_00F6_01C25BF2.BDC7DB00 Content-Type: text/html; charset="iso-8859-1" Content-Transfer-Encoding: quoted-printable
Eu sou formado pelo ITA, e nesta = qualidade=20 gostaria de tecer alguns coment=E1rios sobre o que foi dito na mensagem=20 abaixo.

O ITA n=E3o deve ser considerado uma institui=E7=E3o = militar. Embora=20 subordinado ao Comando da Aeron=E1utica, seu reitor =E9 um cargo civil - = nos seus 53=20 anos de exist=EAncia teve apenas um reitor militar, o ent=E3o Brigadeiro = Engenheiro=20 Tercio Pacitti, formado pelo ITA, muito mais educador do que militar, = autor do=20 Fortran Monitor (livro que ensinou Fortran a milhares), e que, enquanto = reitor,=20 n=E3o usava farda -, seus professores s=E3o civis, e atualmente forma = civis e=20 militares, dependendo da op=E7=E3o que =E9 feita quando o candidato se = inscreve no=20 vestibular. Quem opta pela carreira civil est=E1 livre de qualquer = obriga=E7=E3o com a=20 Aeron=E1utica quando se forma. Pode ir trabalhar onde bem quiser. A = =FAnica=20 obriga=E7=E3o militar de todo aluno (e desde 95, aluna) do ITA =E9 = cursar o Centro de=20 Prepara=E7=E3o de Oficiais da Reserva da Aeron=E1utica no primeiro ano, = o que toma um=20 dia por semana, =FAnica ocasi=E3o em que ele usa = farda.

=C9 excelente=20 o relacionamento entre o ITA e as empresas de uma maneira geral, n=E3o = s=F3 do ramo=20 de engenharia. H=E1 uns dez anos foi criada a Funda=E7=E3o Casimiro = Montenegro Filho=20 (nome do idealizador e "fundador" do ITA), cujo principal objetivo =E9 = desenvolver=20 projetos de coopera=E7=E3o entre a "escola" (como n=F3s chamamos o ITA) = e o mundo=20 empresarial. Os resultados t=EAm sido excelentes.

No ITA n=E3o = h=E1=20 absolutamente nenhuma restri=E7=E3o acad=EAmica quanto =E0 =E1rea de = ci=EAncias humanas.=20 Durante o curso fundamental (dois primeiros anos) os alunos s=E3o = obrigados a=20 frequentar mat=E9rias do Departamento de Ci=EAncias Humanas, que = tamb=E9m oferece=20 mat=E9rias opcionais para quem quiser durante os cinco anos do curso. = Existe um=20 centro acad=EAmico (Centro Acad=EAmico Santos
Dumont) ativo, que = promove=20 atividades culturais as mais variadas - de teatro a concursos = liter=E1rios=20 a shows a...

O curso do ITA =E9 extremamente puxado. Isso, = aliado ao=20 isolamento do aluno do ITA, que estuda, mora, come, pratica esportes, se = diverte, enfim, vive dentro do campus do CTA (Centro T=E9cnico = Aeroespacial) faz=20 com que muitos desistam nos primeiros meses, ou sejam desligados por = problemas=20 de aproveitamento ao fim do primeiro per=EDodo. E uma vez aqui fora, de = volta para=20 sua cidade natal, ningu=E9m vai dizer que n=E3o aguentou o ritmo, ou que = ficou com=20 saudades de casa, ou que foi desligado por nota baixa.

O ITA = "n=E3o tem=20 dinheiro para nada" da mesma forma que, hoje em dia e desde sempre, = ningu=E9m -=20 nem nenhuma institui=E7=E3o - no Brasil tem dinheiro para = nada.

Eu sou=20 suspeito para falar sobre o nivel dos cursos do ITA. Para quem quiser = ter uma=20 id=E9ia, sugiro conferir os resultados do = prov=E3o.

JF

-----Mensagem=20 Original-----
De: kenji yoshitaki
"><kenji@economics.zzn.com>

Para:=20 "><obm-l@mat.puc-rio.br>
Enviada em:=20 Sexta-feira, 13 de Setembro de 2002 10:05
Assunto: Re:[obm-l] = circuito=20 IME


> Rafel,
> Bom,n=E3o quis de maneira alguma = subestimar a=20 competencia dos cursos do ime
e do ita, mas o fato de serem = institui=E7oes=20 militares restringe muito o
relacionamento( e convenios) com as = v=E1rias=20 empresas do ramo da engenharia e
servi=E7os em geral. Isso sem contar = as=20 restri=E7oes academicas com =E1reas de
humanas.
> Em rela=E7ao = aos meus=20 colegas daqui da poli, um fez mais um semestre de
curso l=E1 , e = outro s=F3 fez=20 alguns meses, ambos decidiram fazer cursinho de
novo pra entrar aqui, = pq=20 segundo eles nao compensava ficar mais dois anos
naquela historia de=20 trabalhar pro exercito depois da formatura. E tb por
causa da crise = que o=20 exercito , aeronautica passam.. nao possibilatava
grandes = investimentos na=20 estrutura dos cursos tanto do ime como do ita.
Tenho um tio que =E9 = da embraer=20 e da aula no ita, ele confirma tudo, na =E9poca
dele o ita nao tinha = dinheiro=20 pra nada...
>  Nao sei pra que =E1rea vc vai , mas cursos = como=20 engenharia eletrica e
computa=E7ao , vc deve saber que a Unicamp , = por exemplo,=20 =E9 a melhor da
america latina.. Isso por que os investimentos nao = vem apenas=20 do governo ,
assim como aqui na poli, as empresas fazem parcerias que = elevam=20 a estrutura
dos cursos por meio de investimentos.
>
> E = sobre o=20 n=EDvel do curso , eu tb pensava que era maior no ita e ime, = mas
depois vi o=20 qu=E3o dificil =E9 fazer ( bem feito) um curso de engenharia, pq = na
verdade quem=20 faz o n=EDvel =E9 o pr=F3prio aluno. Depende apenas dele mesmo = buscar
conhecimento=20 e investir na sua forma=E7ao.Isso vale tambem, buscar o "saber"
at=E9 = mesmo em=20 assuntos como hist=F3ria e geografia, porque nessa gera=E7ao = que
passamos, a=20 sabedoria =E9 o  maior valor que alguem pode ter. Quem dessa = lista
=E9=20 engenheiro, matem=E1tico,etc..., deve saber o que estou = dizendo.
>
>=20 Bom =E9 isso , qualquer outra opini=E3o , pode mandar.
> Falow, um = abra=E7o!
>
>
> "A matem=E1tica =E9 a mais alta das = ci=EAncias, o dom=20 mais alto que os deuses
deram aos=20 homens"
>         &nbs= p;        =20 Arquimedes(287a.C.-212a.C.)
>
> Kenji Yoshitaki
>=20 Engenharia/Poli-USP
------=_NextPart_000_00F6_01C25BF2.BDC7DB00-- ========================================================================= Instruções para entrar na lista, sair da lista e usar a lista em http://www.mat.puc-rio.br/~nicolau/olimp/obm-l.html O administrador desta lista é ========================================================================= From owner-obm-l@sucuri.mat.puc-rio.br Sat Sep 14 14:30:41 2002 Return-Path: Received: (from majordom@localhost) by sucuri.mat.puc-rio.br (8.9.3/8.9.3) id OAA25574 for obm-l-MTTP; Sat, 14 Sep 2002 14:27:50 -0300 Received: from traven9.pub1 (traven9.uol.com.br [200.221.4.35]) by sucuri.mat.puc-rio.br (8.9.3/8.9.3) with ESMTP id OAA25570 for ; Sat, 14 Sep 2002 14:27:48 -0300 Received: from Eder ([200.211.161.4]) by traven9.pub1 (8.9.1/8.9.1) with SMTP id OAA21976 for ; Sat, 14 Sep 2002 14:30:39 -0300 (BRT) Message-ID: <038201c25c13$c6aa9fc0$6c52fea9@Eder> From: "Eder" To: References: <010401c25c0c$0df024c0$6302dcc8@jf> Subject: Re: [obm-l] circuito IME/ITA Date: Sat, 14 Sep 2002 14:25:50 -0300 MIME-Version: 1.0 Content-Type: multipart/alternative; boundary="----=_NextPart_000_037F_01C25BFA.9FFCED60" X-Priority: 3 X-MSMail-Priority: Normal X-Mailer: Microsoft Outlook Express 5.00.2314.1300 X-MimeOLE: Produced By Microsoft MimeOLE V5.00.2314.1300 Sender: owner-obm-l@sucuri.mat.puc-rio.br Precedence: bulk Reply-To: obm-l@mat.puc-rio.br This is a multi-part message in MIME format. ------=_NextPart_000_037F_01C25BFA.9FFCED60 Content-Type: text/plain; charset="iso-8859-1" Content-Transfer-Encoding: quoted-printable Desculpem um pouco pelo off,apenas entrando nessa discuss=E3o sobre o = ITA,gostaria de dizer... =D3timo email.O ITA =E9 muitas vezes (em outras lista como = uol.vestibular) v=EDtima de preconceitos,id=E9ias equivocadas.Falam cada = absurdo, que =E9 revoltante.Na minha opini=E3o,essas pessoas n=E3o se = julgam capazes ou n=E3o tem a disposi=E7=E3o necess=E1ria para entrar = l=E1 e ficam querendo desvalorizar a institui=E7=E3o ou =E9 simplesmente = falta de informa=E7=E3o (o que n=E3o autoriza a inventar coisas).J=E1 = quem n=E3o se interessa por engenharia ou pelo ITA n=E3o deveria tecer = tais coment=E1rios.N=E3o tem nada a ver!N=E3o tem que ficar = falando!Deixem os outros em paz! Foi um email bem oportuno para os pr=E9-vestibulandos da lista,como eu = (se bem que eu j=E1 sabia muito sobre o ITA,tenho amigos l=E1 e = prestarei seu vestibular neste ano). Mais uma vez,valeu Francisco! Abaixo aos mitos sobre o ITA! Eder =20 ----- Original Message -----=20 From: Jose Francisco Guimaraes Costa=20 To: obm-l=20 Sent: Saturday, September 14, 2002 1:29 PM Subject: [obm-l] circuito IME/ITA Eu sou formado pelo ITA, e nesta qualidade gostaria de tecer alguns = coment=E1rios sobre o que foi dito na mensagem abaixo. O ITA n=E3o deve ser considerado uma institui=E7=E3o militar. Embora = subordinado ao Comando da Aeron=E1utica, seu reitor =E9 um cargo civil - = nos seus 53 anos de exist=EAncia teve apenas um reitor militar, o = ent=E3o Brigadeiro Engenheiro Tercio Pacitti, formado pelo ITA, muito = mais educador do que militar, autor do Fortran Monitor (livro que = ensinou Fortran a milhares), e que, enquanto reitor, n=E3o usava farda = -, seus professores s=E3o civis, e atualmente forma civis e militares, = dependendo da op=E7=E3o que =E9 feita quando o candidato se inscreve no = vestibular. Quem opta pela carreira civil est=E1 livre de qualquer = obriga=E7=E3o com a Aeron=E1utica quando se forma. Pode ir trabalhar = onde bem quiser. A =FAnica obriga=E7=E3o militar de todo aluno (e desde = 95, aluna) do ITA =E9 cursar o Centro de Prepara=E7=E3o de Oficiais da = Reserva da Aeron=E1utica no primeiro ano, o que toma um dia por semana, = =FAnica ocasi=E3o em que ele usa farda. =C9 excelente o relacionamento entre o ITA e as empresas de uma = maneira geral, n=E3o s=F3 do ramo de engenharia. H=E1 uns dez anos foi = criada a Funda=E7=E3o Casimiro Montenegro Filho (nome do idealizador e = "fundador" do ITA), cujo principal objetivo =E9 desenvolver projetos de = coopera=E7=E3o entre a "escola" (como n=F3s chamamos o ITA) e o mundo = empresarial. Os resultados t=EAm sido excelentes. No ITA n=E3o h=E1 absolutamente nenhuma restri=E7=E3o acad=EAmica = quanto =E0 =E1rea de ci=EAncias humanas. Durante o curso fundamental = (dois primeiros anos) os alunos s=E3o obrigados a frequentar mat=E9rias = do Departamento de Ci=EAncias Humanas, que tamb=E9m oferece mat=E9rias = opcionais para quem quiser durante os cinco anos do curso. Existe um = centro acad=EAmico (Centro Acad=EAmico Santos Dumont) ativo, que promove atividades culturais as mais variadas - de = teatro a concursos liter=E1rios a shows a... O curso do ITA =E9 extremamente puxado. Isso, aliado ao isolamento do = aluno do ITA, que estuda, mora, come, pratica esportes, se diverte, = enfim, vive dentro do campus do CTA (Centro T=E9cnico Aeroespacial) faz = com que muitos desistam nos primeiros meses, ou sejam desligados por = problemas de aproveitamento ao fim do primeiro per=EDodo. E uma vez aqui = fora, de volta para sua cidade natal, ningu=E9m vai dizer que n=E3o = aguentou o ritmo, ou que ficou com saudades de casa, ou que foi = desligado por nota baixa. O ITA "n=E3o tem dinheiro para nada" da mesma forma que, hoje em dia e = desde sempre, ningu=E9m - nem nenhuma institui=E7=E3o - no Brasil tem = dinheiro para nada. Eu sou suspeito para falar sobre o nivel dos cursos do ITA. Para quem = quiser ter uma id=E9ia, sugiro conferir os resultados do prov=E3o. JF -----Mensagem Original----- De: kenji yoshitaki Para: Enviada em: Sexta-feira, 13 de Setembro de 2002 10:05 Assunto: Re:[obm-l] circuito IME > Rafel, > Bom,n=E3o quis de maneira alguma subestimar a competencia dos cursos = do ime e do ita, mas o fato de serem institui=E7oes militares restringe muito = o relacionamento( e convenios) com as v=E1rias empresas do ramo da = engenharia e servi=E7os em geral. Isso sem contar as restri=E7oes academicas com = =E1reas de humanas. > Em rela=E7ao aos meus colegas daqui da poli, um fez mais um semestre = de curso l=E1 , e outro s=F3 fez alguns meses, ambos decidiram fazer = cursinho de novo pra entrar aqui, pq segundo eles nao compensava ficar mais dois = anos naquela historia de trabalhar pro exercito depois da formatura. E tb = por causa da crise que o exercito , aeronautica passam.. nao possibilatava grandes investimentos na estrutura dos cursos tanto do ime como do = ita. Tenho um tio que =E9 da embraer e da aula no ita, ele confirma tudo, = na =E9poca dele o ita nao tinha dinheiro pra nada... > Nao sei pra que =E1rea vc vai , mas cursos como engenharia eletrica = e computa=E7ao , vc deve saber que a Unicamp , por exemplo, =E9 a melhor = da america latina.. Isso por que os investimentos nao vem apenas do = governo , assim como aqui na poli, as empresas fazem parcerias que elevam a = estrutura dos cursos por meio de investimentos. > > E sobre o n=EDvel do curso , eu tb pensava que era maior no ita e = ime, mas depois vi o qu=E3o dificil =E9 fazer ( bem feito) um curso de = engenharia, pq na verdade quem faz o n=EDvel =E9 o pr=F3prio aluno. Depende apenas dele = mesmo buscar conhecimento e investir na sua forma=E7ao.Isso vale tambem, buscar o = "saber" at=E9 mesmo em assuntos como hist=F3ria e geografia, porque nessa = gera=E7ao que passamos, a sabedoria =E9 o maior valor que alguem pode ter. Quem = dessa lista =E9 engenheiro, matem=E1tico,etc..., deve saber o que estou dizendo. > > Bom =E9 isso , qualquer outra opini=E3o , pode mandar. > Falow, um abra=E7o! > > > "A matem=E1tica =E9 a mais alta das ci=EAncias, o dom mais alto que = os deuses deram aos homens" > Arquimedes(287a.C.-212a.C.) > > Kenji Yoshitaki > Engenharia/Poli-USP ------=_NextPart_000_037F_01C25BFA.9FFCED60 Content-Type: text/html; charset="iso-8859-1" Content-Transfer-Encoding: quoted-printable
Desculpem um pouco pelo off,apenas = entrando nessa=20 discuss=E3o sobre o ITA,gostaria de dizer...
 
 
=D3timo email.O ITA =E9 muitas vezes = (em outras=20 lista como  uol.vestibular) v=EDtima de preconceitos,id=E9ias=20 equivocadas.Falam cada absurdo, que =E9 revoltante.Na minha = opini=E3o,essas pessoas=20 n=E3o se julgam capazes ou n=E3o tem a disposi=E7=E3o necess=E1ria para = entrar l=E1 e ficam=20 querendo desvalorizar a institui=E7=E3o ou =E9 simplesmente falta de = informa=E7=E3o (o que=20 n=E3o autoriza a inventar coisas).J=E1 quem n=E3o se interessa por = engenharia ou pelo=20 ITA n=E3o deveria tecer tais coment=E1rios.N=E3o tem nada a ver!N=E3o = tem que ficar=20 falando!Deixem os outros em paz!
 
Foi um email bem oportuno para os = pr=E9-vestibulandos=20 da lista,como eu (se bem que eu j=E1 sabia muito sobre o ITA,tenho = amigos l=E1 e=20 prestarei  seu vestibular  neste ano).
 
Mais uma vez,valeu Francisco! Abaixo = aos mitos=20 sobre o ITA!
 
 
Eder
        =
 
----- Original Message -----
From:=20 Jose=20 Francisco Guimaraes Costa
To: obm-l
Sent: Saturday, September 14, = 2002 1:29=20 PM
Subject: [obm-l] circuito = IME/ITA

Eu sou formado pelo ITA, e = nesta qualidade=20 gostaria de tecer alguns coment=E1rios sobre o que foi dito na = mensagem=20 abaixo.

O ITA n=E3o deve ser considerado uma institui=E7=E3o = militar. Embora=20 subordinado ao Comando da Aeron=E1utica, seu reitor =E9 um cargo civil = - nos seus=20 53 anos de exist=EAncia teve apenas um reitor militar, o ent=E3o = Brigadeiro=20 Engenheiro Tercio Pacitti, formado pelo ITA, muito mais educador do = que=20 militar, autor do Fortran Monitor (livro que ensinou Fortran a = milhares), e=20 que, enquanto reitor, n=E3o usava farda -, seus professores s=E3o = civis, e=20 atualmente forma civis e militares, dependendo da op=E7=E3o que =E9 = feita quando o=20 candidato se inscreve no vestibular. Quem opta pela carreira civil = est=E1 livre=20 de qualquer obriga=E7=E3o com a Aeron=E1utica quando se forma. Pode ir = trabalhar=20 onde bem quiser. A =FAnica obriga=E7=E3o militar de todo aluno (e = desde 95, aluna)=20 do ITA =E9 cursar o Centro de Prepara=E7=E3o de Oficiais da Reserva da = Aeron=E1utica=20 no primeiro ano, o que toma um dia por semana, =FAnica ocasi=E3o em=20 que ele usa farda.

=C9 excelente o relacionamento = entre o ITA e=20 as empresas de uma maneira geral, n=E3o s=F3 do ramo de engenharia. = H=E1 uns dez=20 anos foi criada a Funda=E7=E3o Casimiro Montenegro Filho (nome do = idealizador e=20 "fundador" do ITA), cujo principal objetivo =E9 desenvolver projetos = de=20 coopera=E7=E3o entre a "escola" (como n=F3s chamamos o ITA) e o mundo = empresarial.=20 Os resultados t=EAm sido excelentes.

No ITA n=E3o h=E1 = absolutamente nenhuma=20 restri=E7=E3o acad=EAmica quanto =E0 =E1rea de ci=EAncias humanas. = Durante o curso=20 fundamental (dois primeiros anos) os alunos s=E3o obrigados a = frequentar=20 mat=E9rias do Departamento de Ci=EAncias Humanas, que tamb=E9m oferece = mat=E9rias=20 opcionais para quem quiser durante os cinco anos do curso. Existe um = centro=20 acad=EAmico (Centro Acad=EAmico Santos
Dumont) ativo, que promove = atividades=20 culturais as mais variadas - de teatro a concursos liter=E1rios = a shows=20 a...

O curso do ITA =E9 extremamente puxado. Isso, aliado ao = isolamento=20 do aluno do ITA, que estuda, mora, come, pratica esportes, se diverte, = enfim,=20 vive dentro do campus do CTA (Centro T=E9cnico Aeroespacial) faz com = que muitos=20 desistam nos primeiros meses, ou sejam desligados por problemas de=20 aproveitamento ao fim do primeiro per=EDodo. E uma vez aqui fora, de = volta para=20 sua cidade natal, ningu=E9m vai dizer que n=E3o aguentou o ritmo, ou = que ficou com=20 saudades de casa, ou que foi desligado por nota baixa.

O ITA = "n=E3o tem=20 dinheiro para nada" da mesma forma que, hoje em dia e desde sempre, = ningu=E9m -=20 nem nenhuma institui=E7=E3o - no Brasil tem dinheiro para = nada.

Eu sou=20 suspeito para falar sobre o nivel dos cursos do ITA. Para quem quiser = ter uma=20 id=E9ia, sugiro conferir os resultados do = prov=E3o.

JF

-----Mensagem=20 Original-----
De: kenji yoshitaki
"><kenji@economics.zzn.com>

Para:=20 "><obm-l@mat.puc-rio.br>
Enviada em:=20 Sexta-feira, 13 de Setembro de 2002 10:05
Assunto: Re:[obm-l] = circuito=20 IME


> Rafel,
> Bom,n=E3o quis de maneira alguma = subestimar a=20 competencia dos cursos do ime
e do ita, mas o fato de serem = institui=E7oes=20 militares restringe muito o
relacionamento( e convenios) com as = v=E1rias=20 empresas do ramo da engenharia e
servi=E7os em geral. Isso sem = contar as=20 restri=E7oes academicas com =E1reas de
humanas.
> Em = rela=E7ao aos meus=20 colegas daqui da poli, um fez mais um semestre de
curso l=E1 , e = outro s=F3 fez=20 alguns meses, ambos decidiram fazer cursinho de
novo pra entrar = aqui, pq=20 segundo eles nao compensava ficar mais dois anos
naquela historia = de=20 trabalhar pro exercito depois da formatura. E tb por
causa da crise = que o=20 exercito , aeronautica passam.. nao possibilatava
grandes = investimentos na=20 estrutura dos cursos tanto do ime como do ita.
Tenho um tio que =E9 = da=20 embraer e da aula no ita, ele confirma tudo, na =E9poca
dele o ita = nao tinha=20 dinheiro pra nada...
>  Nao sei pra que =E1rea vc vai , mas = cursos=20 como engenharia eletrica e
computa=E7ao , vc deve saber que a = Unicamp , por=20 exemplo, =E9 a melhor da
america latina.. Isso por que os = investimentos nao=20 vem apenas do governo ,
assim como aqui na poli, as empresas fazem=20 parcerias que elevam a estrutura
dos cursos por meio de=20 investimentos.
>
> E sobre o n=EDvel do curso , eu tb = pensava que=20 era maior no ita e ime, mas
depois vi o qu=E3o dificil =E9 fazer ( = bem feito)=20 um curso de engenharia, pq na
verdade quem faz o n=EDvel =E9 o = pr=F3prio aluno.=20 Depende apenas dele mesmo buscar
conhecimento e investir na sua=20 forma=E7ao.Isso vale tambem, buscar o "saber"
at=E9 mesmo em = assuntos como=20 hist=F3ria e geografia, porque nessa gera=E7ao que
passamos, a = sabedoria =E9=20 o  maior valor que alguem pode ter. Quem dessa lista
=E9 = engenheiro,=20 matem=E1tico,etc..., deve saber o que estou dizendo.
>
> = Bom =E9 isso=20 , qualquer outra opini=E3o , pode mandar.
> Falow, um=20 abra=E7o!
>
>
> "A matem=E1tica =E9 a mais alta das = ci=EAncias, o=20 dom mais alto que os deuses
deram aos=20 = homens"
>         &nbs= p;        =20 Arquimedes(287a.C.-212a.C.)
>
> Kenji Yoshitaki
>=20 Engenharia/Poli-USP
------=_NextPart_000_037F_01C25BFA.9FFCED60-- ========================================================================= Instruções para entrar na lista, sair da lista e usar a lista em http://www.mat.puc-rio.br/~nicolau/olimp/obm-l.html O administrador desta lista é ========================================================================= From owner-obm-l@sucuri.mat.puc-rio.br Sat Sep 14 17:47:17 2002 Return-Path: Received: (from majordom@localhost) by sucuri.mat.puc-rio.br (8.9.3/8.9.3) id RAA27170 for obm-l-MTTP; Sat, 14 Sep 2002 17:43:52 -0300 Received: from trex.centroin.com.br (trex.centroin.com.br [200.225.63.134]) by sucuri.mat.puc-rio.br (8.9.3/8.9.3) with ESMTP id RAA27166 for ; Sat, 14 Sep 2002 17:43:50 -0300 Received: from trex.centroin.com.br (localhost [127.0.0.1]) by trex.centroin.com.br (8.12.5/8.12.1) with ESMTP id g8EKjRT0007835 for ; Sat, 14 Sep 2002 17:45:27 -0300 (BRT) Received: by trex.centroin.com.br (8.12.5/8.12.5/Submit) id g8EKjR9J007834; Sat, 14 Sep 2002 17:45:27 -0300 (BRT) Message-Id: <200209142045.g8EKjR9J007834@trex.centroin.com.br> Received: from 200.225.58.61 by trex.centroin.com.br (CIPWM versao 1.4C1) with HTTPS for ; Sat, 14 Sep 2002 17:45:27 -0300 (BRT) Date: Sat, 14 Sep 2002 17:45:27 -0300 (BRT) From: Augusto Cesar de Oliveira Morgado To: obm-l@mat.puc-rio.br Subject: =?iso-8859-1?q?Re: [obm-l] seq=FC=EAncia?= MIME-Version: 1.0 X-Mailer: CentroIn Internet Provider WebMail v. 1.4C1 (http://www.centroin.com.br/) Content-Type: text/plain; charset="iso-8859-1" Content-Transfer-Encoding: 8bit X-MIME-Autoconverted: from quoted-printable to 8bit by sucuri.mat.puc-rio.br id RAA27167 Sender: owner-obm-l@sucuri.mat.puc-rio.br Precedence: bulk Reply-To: obm-l@mat.puc-rio.br Em Fri, 13 Sep 2002 18:42:57 -0300, Juliana_Löff disse: > E aê, pessoal! > Me ajudem com essa (mas para resolver em nível secundário!!) > Muito obrigada! > Ju > ------------------------------- > (UFRGS - 2002) Se n é um natural ímpar, o número de elementos da seqüência 1,2,2,3,3,3,4,4,4,4,...,n,n,(n vezes),n que são números pares é: > > (a) n² - 1 > 4 > > > (b) n² - 1 > 2 > > > (c) n(n + 1) > 4 > > > (d) n(n + 1) > 2 > > > (e) (n + 1)²/4 >Os pares sao 2 (2 vezes), 4 (4 vezes),..., n-1(n-1 vezes). A quantidade de pares eh 2+4+6+...+(n-1) = [2+(n-1)].[(n-1)/2] /2 = (n^2-1)/4. (A) ========================================================================= Instruções para entrar na lista, sair da lista e usar a lista em http://www.mat.puc-rio.br/~nicolau/olimp/obm-l.html O administrador desta lista é ========================================================================= From owner-obm-l@sucuri.mat.puc-rio.br Sat Sep 14 19:07:51 2002 Return-Path: Received: (from majordom@localhost) by sucuri.mat.puc-rio.br (8.9.3/8.9.3) id TAA28110 for obm-l-MTTP; Sat, 14 Sep 2002 19:04:31 -0300 Received: from hotmail.com (f35.pav1.hotmail.com [64.4.31.35]) by sucuri.mat.puc-rio.br (8.9.3/8.9.3) with ESMTP id TAA28106 for ; Sat, 14 Sep 2002 19:04:28 -0300 Received: from mail pickup service by hotmail.com with Microsoft SMTPSVC; Sat, 14 Sep 2002 15:05:41 -0700 Received: from 200.199.179.214 by pv1fd.pav1.hotmail.msn.com with HTTP; Sat, 14 Sep 2002 22:05:41 GMT X-Originating-IP: [200.199.179.214] From: "Adherbal Rocha Filho" To: obm-l@mat.puc-rio.br Subject: [obm-l] =?iso-8859-1?B?MqogZmFzZSBuaXZlbCAzLi4u?= Date: Sat, 14 Sep 2002 22:05:41 +0000 Mime-Version: 1.0 Content-Type: text/plain; charset=iso-8859-1; format=flowed Message-ID: X-OriginalArrivalTime: 14 Sep 2002 22:05:41.0806 (UTC) FILETIME=[DDC0E4E0:01C25C3A] Sender: owner-obm-l@sucuri.mat.puc-rio.br Precedence: bulk Reply-To: obm-l@mat.puc-rio.br Putz, essa prova tava muito,muito,muito,muito dificil!!!!!!!!!! Espero q pelo menos a nota de corte pra 3ª fase seja menor q 50 pnts... a do ano passado tava tao facilzinha...mas acho q jah sei, ano passado vcs fizeram uma 1ª fase dificil e uma segunda bem facil, esse ano foi o contrario...soh q a 2ª vale muito mais pontos q a 1ª !!!! pô, assim eh dificil... falou _________________________________________________________________ MSN Photos é a maneira mais fácil e prática de editar e compartilhar sua fotos: http://photos.msn.com.br ========================================================================= Instruções para entrar na lista, sair da lista e usar a lista em http://www.mat.puc-rio.br/~nicolau/olimp/obm-l.html O administrador desta lista é ========================================================================= From owner-obm-l@sucuri.mat.puc-rio.br Sat Sep 14 19:38:57 2002 Return-Path: Received: (from majordom@localhost) by sucuri.mat.puc-rio.br (8.9.3/8.9.3) id TAA28718 for obm-l-MTTP; Sat, 14 Sep 2002 19:35:30 -0300 Received: from hotmail.com (oe62.law10.hotmail.com [64.4.14.197]) by sucuri.mat.puc-rio.br (8.9.3/8.9.3) with ESMTP id TAA28713 for ; Sat, 14 Sep 2002 19:35:27 -0300 Received: from mail pickup service by hotmail.com with Microsoft SMTPSVC; Sat, 14 Sep 2002 15:36:40 -0700 X-Originating-IP: [200.211.146.28] From: "Rubens Vilhena" To: Subject: [obm-l] =?iso-8859-1?Q?T=E1_pegando...?= Date: Sat, 14 Sep 2002 19:36:22 -0300 MIME-Version: 1.0 X-Mailer: MSN Explorer 7.00.0021.1900 Content-Type: multipart/alternative; boundary="----=_NextPart_001_0001_01C25C26.01BFB9D0" Message-ID: X-OriginalArrivalTime: 14 Sep 2002 22:36:40.0054 (UTC) FILETIME=[315B0960:01C25C3F] Sender: owner-obm-l@sucuri.mat.puc-rio.br Precedence: bulk Reply-To: obm-l@mat.puc-rio.br ------=_NextPart_001_0001_01C25C26.01BFB9D0 Content-Type: text/plain; charset="iso-8859-1" Content-Transfer-Encoding: quoted-printable Al=F4 amigos, algu=E9m me ajuda nesta quest=E3o? "Prove que n =E9 primo se, e somente se, (n-1)!=3D-1 (mod n). Agrade=E7oAproveite melhor a Web. Fa=E7a o download GR=C1TIS do MSN Explo= rer : http://explorer.msn.com.br/intl.asp#po ------=_NextPart_001_0001_01C25C26.01BFB9D0 Content-Type: text/html; charset="iso-8859-1" Content-Transfer-Encoding: quoted-printable
 
<= DIV>Al=F4 amigos, algu=E9m me ajuda nesta quest=E3o?
 
"Prove que n =E9 primo se, e somente se, (n-1)!=3D-1 (mod n).
 
Agrade=E7o


Aproveite melhor a Web. Fa=E7a o download GR=C1TIS do MSN E= xplorer : http://explo= rer.msn.com.br/intl.asp#po

------=_NextPart_001_0001_01C25C26.01BFB9D0-- ========================================================================= Instruções para entrar na lista, sair da lista e usar a lista em http://www.mat.puc-rio.br/~nicolau/olimp/obm-l.html O administrador desta lista é ========================================================================= From owner-obm-l@sucuri.mat.puc-rio.br Sat Sep 14 19:57:24 2002 Return-Path: Received: (from majordom@localhost) by sucuri.mat.puc-rio.br (8.9.3/8.9.3) id TAA29216 for obm-l-MTTP; Sat, 14 Sep 2002 19:54:27 -0300 Received: from trex.centroin.com.br (trex.centroin.com.br [200.225.63.134]) by sucuri.mat.puc-rio.br (8.9.3/8.9.3) with ESMTP id TAA29212 for ; Sat, 14 Sep 2002 19:54:25 -0300 Received: from trex.centroin.com.br (localhost [127.0.0.1]) by trex.centroin.com.br (8.12.5/8.12.1) with ESMTP id g8EMu2T0018191 for ; Sat, 14 Sep 2002 19:56:02 -0300 (BRT) Received: by trex.centroin.com.br (8.12.5/8.12.5/Submit) id g8EMu2XW018190; Sat, 14 Sep 2002 19:56:02 -0300 (BRT) Message-Id: <200209142256.g8EMu2XW018190@trex.centroin.com.br> Received: from 200.225.58.131 by trex.centroin.com.br (CIPWM versao 1.4C1) with HTTPS for ; Sat, 14 Sep 2002 19:56:02 -0300 (BRT) Date: Sat, 14 Sep 2002 19:56:02 -0300 (BRT) From: Augusto Cesar de Oliveira Morgado To: obm-l@mat.puc-rio.br Subject: =?iso-8859-1?q?Re: [obm-l] T=E1_pegando...?= MIME-Version: 1.0 X-Mailer: CentroIn Internet Provider WebMail v. 1.4C1 (http://www.centroin.com.br/) Content-Type: text/plain; charset="iso-8859-1" Content-Transfer-Encoding: 8bit X-MIME-Autoconverted: from quoted-printable to 8bit by sucuri.mat.puc-rio.br id TAA29213 Sender: owner-obm-l@sucuri.mat.puc-rio.br Precedence: bulk Reply-To: obm-l@mat.puc-rio.br Em Sat, 14 Sep 2002 19:36:22 -0300, Rubens Vilhena disse: > > Alô amigos, alguém me ajuda nesta questão? > > "Prove que n é primo se, e somente se, (n-1)!=-1 (mod n). > > AgradeçoAproveite melhor a Web. Faça o download GRÁTIS do MSN Explorer : http://explorer.msn.com.br/intl.asp#po Este eh o Teorema de Wilson. Em qualquer livro de Teoria dos Numeros voce o encontra.Ou na internet. ========================================================================= Instruções para entrar na lista, sair da lista e usar a lista em http://www.mat.puc-rio.br/~nicolau/olimp/obm-l.html O administrador desta lista é ========================================================================= From owner-obm-l@sucuri.mat.puc-rio.br Sat Sep 14 20:19:30 2002 Return-Path: Received: (from majordom@localhost) by sucuri.mat.puc-rio.br (8.9.3/8.9.3) id UAA29832 for obm-l-MTTP; Sat, 14 Sep 2002 20:15:43 -0300 Received: from smtp.estaminas.com.br (smtp.estaminas.com.br [200.188.191.51]) by sucuri.mat.puc-rio.br (8.9.3/8.9.3) with ESMTP id UAA29828 for ; Sat, 14 Sep 2002 20:15:40 -0300 Received: from uai.com.br (pop.uai.com.br [200.188.191.2]) by smtp.estaminas.com.br (8.11.0/8.11.0) with ESMTP id g8ENOoQ04990 for ; Sat, 14 Sep 2002 20:24:51 -0300 Received: from mesantos (dial-tm-mp-200-165-30-197.estaminas.com.br [200.165.30.197]) by uai.com.br (8.12.4/8.12.4) with SMTP id g8ENOm9N002857 for ; Sat, 14 Sep 2002 20:24:49 -0300 (BRT) Message-ID: <004801c86f60$20745ea0$427995c8@mesantos> From: "Marcos Eike Tinen dos Santos" To: References: <010401c25c0c$0df024c0$6302dcc8@jf> <038201c25c13$c6aa9fc0$6c52fea9@Eder> Subject: Re: [obm-l] circuito IME/ITA Date: Thu, 14 Feb 2008 20:20:01 -0300 MIME-Version: 1.0 Content-Type: text/plain; charset="iso-8859-1" Content-Transfer-Encoding: 8bit X-Priority: 3 X-MSMail-Priority: Normal X-Mailer: Microsoft Outlook Express 5.50.4133.2400 X-MimeOLE: Produced By Microsoft MimeOLE V5.50.4133.2400 Sender: owner-obm-l@sucuri.mat.puc-rio.br Precedence: bulk Reply-To: obm-l@mat.puc-rio.br Concerteza o ITA tem um dos melhores cursos de Engenharia do Brasil. Assim, como a Unicamp e a USP. O fato de as vezes ocorrerem algumas revoltas, até mesmo entre professores, é a prova de vestibular que sempre foge de assuntos ditados em seu manual. Isso vale também para o IME. Bom, eu tentei o ITA ano passado, e infelizmente, devido a química, que não tenho consideração, não fui feliz. Mas, eu defendo o ITA e o IME em suas provas difíceis já que eles não têm as provas normais como: História, Geografia e Biologia para eliminar os candidatos. Portanto, se a prova fosse do nível da Unicamp ou mesmo da Usp, vários canditados tirariam total, o que é um absurdo, numa prova em que se quer eliminar canditados. Ats, Marcos Eike ----- Original Message ----- From: Eder To: obm-l@mat.puc-rio.br Sent: Saturday, September 14, 2002 2:25 PM Subject: Re: [obm-l] circuito IME/ITA Desculpem um pouco pelo off,apenas entrando nessa discussão sobre o ITA,gostaria de dizer... Ótimo email.O ITA é muitas vezes (em outras lista como uol.vestibular) vítima de preconceitos,idéias equivocadas.Falam cada absurdo, que é revoltante.Na minha opinião,essas pessoas não se julgam capazes ou não tem a disposição necessária para entrar lá e ficam querendo desvalorizar a instituição ou é simplesmente falta de informação (o que não autoriza a inventar coisas).Já quem não se interessa por engenharia ou pelo ITA não deveria tecer tais comentários.Não tem nada a ver!Não tem que ficar falando!Deixem os outros em paz! Foi um email bem oportuno para os pré-vestibulandos da lista,como eu (se bem que eu já sabia muito sobre o ITA,tenho amigos lá e prestarei seu vestibular neste ano). Mais uma vez,valeu Francisco! Abaixo aos mitos sobre o ITA! Eder ----- Original Message ----- From: Jose Francisco Guimaraes Costa To: obm-l Sent: Saturday, September 14, 2002 1:29 PM Subject: [obm-l] circuito IME/ITA Eu sou formado pelo ITA, e nesta qualidade gostaria de tecer alguns comentários sobre o que foi dito na mensagem abaixo. O ITA não deve ser considerado uma instituição militar. Embora subordinado ao Comando da Aeronáutica, seu reitor é um cargo civil - nos seus 53 anos de existência teve apenas um reitor militar, o então Brigadeiro Engenheiro Tercio Pacitti, formado pelo ITA, muito mais educador do que militar, autor do Fortran Monitor (livro que ensinou Fortran a milhares), e que, enquanto reitor, não usava farda -, seus professores são civis, e atualmente forma civis e militares, dependendo da opção que é feita quando o candidato se inscreve no vestibular. Quem opta pela carreira civil está livre de qualquer obrigação com a Aeronáutica quando se forma. Pode ir trabalhar onde bem quiser. A única obrigação militar de todo aluno (e desde 95, aluna) do ITA é cursar o Centro de Preparação de Oficiais da Reserva da Aeronáutica no primeiro ano, o que toma um dia por semana, única ocasião em que ele usa farda. É excelente o relacionamento entre o ITA e as empresas de uma maneira geral, não só do ramo de engenharia. Há uns dez anos foi criada a Fundação Casimiro Montenegro Filho (nome do idealizador e "fundador" do ITA), cujo principal objetivo é desenvolver projetos de cooperação entre a "escola" (como nós chamamos o ITA) e o mundo empresarial. Os resultados têm sido excelentes. No ITA não há absolutamente nenhuma restrição acadêmica quanto à área de ciências humanas. Durante o curso fundamental (dois primeiros anos) os alunos são obrigados a frequentar matérias do Departamento de Ciências Humanas, que também oferece matérias opcionais para quem quiser durante os cinco anos do curso. Existe um centro acadêmico (Centro Acadêmico Santos Dumont) ativo, que promove atividades culturais as mais variadas - de teatro a concursos literários a shows a... O curso do ITA é extremamente puxado. Isso, aliado ao isolamento do aluno do ITA, que estuda, mora, come, pratica esportes, se diverte, enfim, vive dentro do campus do CTA (Centro Técnico Aeroespacial) faz com que muitos desistam nos primeiros meses, ou sejam desligados por problemas de aproveitamento ao fim do primeiro período. E uma vez aqui fora, de volta para sua cidade natal, ninguém vai dizer que não aguentou o ritmo, ou que ficou com saudades de casa, ou que foi desligado por nota baixa. O ITA "não tem dinheiro para nada" da mesma forma que, hoje em dia e desde sempre, ninguém - nem nenhuma instituição - no Brasil tem dinheiro para nada. Eu sou suspeito para falar sobre o nivel dos cursos do ITA. Para quem quiser ter uma idéia, sugiro conferir os resultados do provão. JF -----Mensagem Original----- De: kenji yoshitaki Para: Enviada em: Sexta-feira, 13 de Setembro de 2002 10:05 Assunto: Re:[obm-l] circuito IME > Rafel, > Bom,não quis de maneira alguma subestimar a competencia dos cursos do ime e do ita, mas o fato de serem instituiçoes militares restringe muito o relacionamento( e convenios) com as várias empresas do ramo da engenharia e serviços em geral. Isso sem contar as restriçoes academicas com áreas de humanas. > Em relaçao aos meus colegas daqui da poli, um fez mais um semestre de curso lá , e outro só fez alguns meses, ambos decidiram fazer cursinho de novo pra entrar aqui, pq segundo eles nao compensava ficar mais dois anos naquela historia de trabalhar pro exercito depois da formatura. E tb por causa da crise que o exercito , aeronautica passam.. nao possibilatava grandes investimentos na estrutura dos cursos tanto do ime como do ita. Tenho um tio que é da embraer e da aula no ita, ele confirma tudo, na época dele o ita nao tinha dinheiro pra nada... > Nao sei pra que área vc vai , mas cursos como engenharia eletrica e computaçao , vc deve saber que a Unicamp , por exemplo, é a melhor da america latina.. Isso por que os investimentos nao vem apenas do governo , assim como aqui na poli, as empresas fazem parcerias que elevam a estrutura dos cursos por meio de investimentos. > > E sobre o nível do curso , eu tb pensava que era maior no ita e ime, mas depois vi o quão dificil é fazer ( bem feito) um curso de engenharia, pq na verdade quem faz o nível é o próprio aluno. Depende apenas dele mesmo buscar conhecimento e investir na sua formaçao.Isso vale tambem, buscar o "saber" até mesmo em assuntos como história e geografia, porque nessa geraçao que passamos, a sabedoria é o maior valor que alguem pode ter. Quem dessa lista é engenheiro, matemático,etc..., deve saber o que estou dizendo. > > Bom é isso , qualquer outra opinião , pode mandar. > Falow, um abraço! > > > "A matemática é a mais alta das ciências, o dom mais alto que os deuses deram aos homens" > Arquimedes(287a.C.-212a.C.) > > Kenji Yoshitaki > Engenharia/Poli-USP ========================================================================= Instruções para entrar na lista, sair da lista e usar a lista em http://www.mat.puc-rio.br/~nicolau/olimp/obm-l.html O administrador desta lista é ========================================================================= From owner-obm-l@sucuri.mat.puc-rio.br Sat Sep 14 20:24:17 2002 Return-Path: Received: (from majordom@localhost) by sucuri.mat.puc-rio.br (8.9.3/8.9.3) id UAA30011 for obm-l-MTTP; Sat, 14 Sep 2002 20:21:56 -0300 Received: from hotmail.com (f183.pav1.hotmail.com [64.4.31.183]) by sucuri.mat.puc-rio.br (8.9.3/8.9.3) with ESMTP id UAA30006 for ; Sat, 14 Sep 2002 20:21:53 -0300 Received: from mail pickup service by hotmail.com with Microsoft SMTPSVC; Sat, 14 Sep 2002 16:23:06 -0700 Received: from 200.199.179.214 by pv1fd.pav1.hotmail.msn.com with HTTP; Sat, 14 Sep 2002 23:23:06 GMT X-Originating-IP: [200.199.179.214] From: "Adherbal Rocha Filho" To: obm-l@mat.puc-rio.br Subject: [obm-l] =?iso-8859-1?B?UmU6IFtvYm0tbF0gVOEgcGVnYW5kby4uLg==?= Date: Sat, 14 Sep 2002 23:23:06 +0000 Mime-Version: 1.0 Content-Type: text/plain; charset=iso-8859-1; format=flowed Message-ID: X-OriginalArrivalTime: 14 Sep 2002 23:23:06.0614 (UTC) FILETIME=[AE465960:01C25C45] Sender: owner-obm-l@sucuri.mat.puc-rio.br Precedence: bulk Reply-To: obm-l@mat.puc-rio.br >Alô amigos, alguém me ajuda nesta questão? > >"Prove que n é primo se, e somente se, (n-1)!=-1 (mod n). > eh facil provar q se n eh composto e >4 (n-1)!==0(modn) (basta fazer n=ab e n=a^2)...mas isso q tu falou eh o teorema de wilson, note q se n eh composto, digamos, n=ab 1b|(n-1)! e como n|(n-1)!+1 , b tbem o faz, logo b|1 => b=1, absurdo! n eh primo. falou _________________________________________________________________ Tenha você também um MSN Hotmail, o maior webmail do mundo: http://www.hotmail.com/br ========================================================================= Instruções para entrar na lista, sair da lista e usar a lista em http://www.mat.puc-rio.br/~nicolau/olimp/obm-l.html O administrador desta lista é ========================================================================= From owner-obm-l@sucuri.mat.puc-rio.br Sat Sep 14 22:00:33 2002 Return-Path: Received: (from majordom@localhost) by sucuri.mat.puc-rio.br (8.9.3/8.9.3) id VAA31540 for obm-l-MTTP; Sat, 14 Sep 2002 21:55:06 -0300 Received: from shen.bol.com.br (shen.bol.com.br [200.221.24.14]) by sucuri.mat.puc-rio.br (8.9.3/8.9.3) with ESMTP id VAA31536 for ; Sat, 14 Sep 2002 21:55:04 -0300 Received: from bol.com.br (200.221.24.137) by shen.bol.com.br (5.1.071) id 3D63D22F007FAC62 for obm-l@mat.puc-rio.br; Sat, 14 Sep 2002 21:55:33 -0300 Date: Sat, 14 Sep 2002 21:53:58 -0300 Message-Id: Subject: Re: Re:[obm-l] circuito IME MIME-Version: 1.0 Content-Type: text/plain;charset="iso-8859-1" From: "adr.scr.m" To: obm-l@mat.puc-rio.br X-XaM3-API-Version: 2.4.3.4.4 X-SenderIP: 200.151.4.203 Content-Transfer-Encoding: 8bit X-MIME-Autoconverted: from quoted-printable to 8bit by sucuri.mat.puc-rio.br id VAA31537 Sender: owner-obm-l@sucuri.mat.puc-rio.br Precedence: bulk Reply-To: obm-l@mat.puc-rio.br outro obs: o ITA tambem nao divulga classificacao, somente o IME,que tambem nao divulga gabarito. Por exemplo no IME voce pode ser aprovado mas nao entrar porque esta na outra lista, por causa de sua nota(as vezes tem muita gente com notas maiores que a sua),mas no ITA nunca se sabe,so se voce realmente estiver dentro. []'s. Adriano. __________________________________________________________________________ AcessoBOL, só R$ 9,90! O menor preço do mercado! Assine já! http://www.bol.com.br/acessobol ========================================================================= Instruções para entrar na lista, sair da lista e usar a lista em http://www.mat.puc-rio.br/~nicolau/olimp/obm-l.html O administrador desta lista é ========================================================================= From owner-obm-l@sucuri.mat.puc-rio.br Sat Sep 14 22:05:12 2002 Return-Path: Received: (from majordom@localhost) by sucuri.mat.puc-rio.br (8.9.3/8.9.3) id WAA31603 for obm-l-MTTP; Sat, 14 Sep 2002 22:02:13 -0300 Received: from shen.bol.com.br (shen.bol.com.br [200.221.24.14]) by sucuri.mat.puc-rio.br (8.9.3/8.9.3) with ESMTP id WAA31599 for ; Sat, 14 Sep 2002 22:02:11 -0300 Received: from bol.com.br (200.221.24.137) by shen.bol.com.br (5.1.071) id 3D63D22F007FB289 for obm-l@mat.puc-rio.br; Sat, 14 Sep 2002 22:02:41 -0300 Date: Sat, 14 Sep 2002 22:01:06 -0300 Message-Id: Subject: Re:[obm-l] circuito IME MIME-Version: 1.0 Content-Type: text/plain;charset="iso-8859-1" From: "adr.scr.m" To: obm-l@mat.puc-rio.br X-XaM3-API-Version: 2.4.3.4.4 X-SenderIP: 200.151.4.203 Content-Transfer-Encoding: 8bit X-MIME-Autoconverted: from quoted-printable to 8bit by sucuri.mat.puc-rio.br id WAA31600 Sender: owner-obm-l@sucuri.mat.puc-rio.br Precedence: bulk Reply-To: obm-l@mat.puc-rio.br Rafael eu realmente nao sei porque eles nao anularam a questao,mas meu professor de Fisica falou que tinha(ele e outros professores) reclamado e os professores da banca falaram que nao iriam fazer nada a respeito,ou seja a questao iria continuar valendo. []'s. Adriano. __________________________________________________________________________ AcessoBOL, só R$ 9,90! O menor preço do mercado! Assine já! http://www.bol.com.br/acessobol ========================================================================= Instruções para entrar na lista, sair da lista e usar a lista em http://www.mat.puc-rio.br/~nicolau/olimp/obm-l.html O administrador desta lista é ========================================================================= From owner-obm-l@sucuri.mat.puc-rio.br Sat Sep 14 22:20:19 2002 Return-Path: Received: (from majordom@localhost) by sucuri.mat.puc-rio.br (8.9.3/8.9.3) id WAA32296 for obm-l-MTTP; Sat, 14 Sep 2002 22:19:39 -0300 Received: from smtp-33.ig.com.br (smtp-33.ig.com.br [200.226.132.183]) by sucuri.mat.puc-rio.br (8.9.3/8.9.3) with SMTP id WAA32292 for ; Sat, 14 Sep 2002 22:19:37 -0300 Received: (qmail 19796 invoked from network); 15 Sep 2002 01:20:31 -0000 Received: from unknown (HELO windows9) (200.222.191.46) by smtp-33.ig.com.br with SMTP; 15 Sep 2002 01:20:31 -0000 Message-ID: <001801c25c56$956719a0$2ebfdec8@windows9> From: "Eric Campos Bastos Guedes" To: References: <200209142256.g8EMu2XW018190@trex.centroin.com.br> Subject: [obm-l] =?iso-8859-1?Q?Re:_=5Bobm-l=5D_T=E1_pegando...?= Date: Sat, 14 Sep 2002 22:23:54 -0300 MIME-Version: 1.0 Content-Type: text/plain; charset="iso-8859-1" Content-Transfer-Encoding: 8bit X-Priority: 3 X-MSMail-Priority: Normal X-Mailer: Microsoft Outlook Express 5.00.2615.200 X-MIMEOLE: Produced By Microsoft MimeOLE V5.00.2615.200 Sender: owner-obm-l@sucuri.mat.puc-rio.br Precedence: bulk Reply-To: obm-l@mat.puc-rio.br > "Prove que n é primo se, e somente se, (n-1)!=-1 (mod n). > > Este eh o Teorema de Wilson. Em qualquer livro de Teoria dos Numeros voce o encontra.Ou na internet. Uma prova que conheço é assim: Inicialmente lembremos o pequeno teorema de Fermat: "Se p eh primo e p nao divide a entao a^(p-1) == 1 mod p" agora suponha n=p primo. Considere [1] X^(n-1) == 1 mod n conforme o pequeno teorema de Fermat, a equação [1] acima tem as soluções X=1,2,3,...,n-1, isto eh se [2] f(X) = X^(n-1) -1 entao f(X) tem as raizes 1,2,3,...,n-1 modulo n, e portanto podemos fatorar f(X) = X^(n-1)-1 assim: f(X) = X^(n-1) - 1 = (X-1)(X-2)(X-3)...(X-(n-1)) mod n igualando os termos independentes de X^(n-1) - 1 e de (X-1)(X-2)(X-3)...(X-(n-1)) obtemos -1 == (-1)(-2)(-3)...(-(n-1)) mod n isto eh [3] -1 == (n-1)!(-1)^(n-1) mod n se n eh primo impar tem-se (-1)^(n-1) = 1 e portanto [3] fornece (n-1)! == -1 mod n se n=2 tem-se (n-1)!=(2-1)!=1== 1 == -1 mod 2, isto eh (n-1)! == -1 mod n de qualquer forma (n-1)! == -1 mod n Abrac,os, --------------------------------------------- Eric Campos Bastos Guedes mathfire@ig.com.br Confira o livro: "Formulas que geram numeros primos" no site www.primeformulas.hpg.com.br ========================================================================= Instruções para entrar na lista, sair da lista e usar a lista em http://www.mat.puc-rio.br/~nicolau/olimp/obm-l.html O administrador desta lista é ========================================================================= From owner-obm-l@sucuri.mat.puc-rio.br Sat Sep 14 22:20:20 2002 Return-Path: Received: (from majordom@localhost) by sucuri.mat.puc-rio.br (8.9.3/8.9.3) id WAA32313 for obm-l-MTTP; Sat, 14 Sep 2002 22:20:08 -0300 Received: from traven10.pub1 (traven10.uol.com.br [200.221.4.45]) by sucuri.mat.puc-rio.br (8.9.3/8.9.3) with ESMTP id WAA32309 for ; Sat, 14 Sep 2002 22:20:05 -0300 Received: from u2z7z2 ([200.158.144.191]) by traven10.pub1 (8.9.1/8.9.1) with ESMTP id WAA05711 for ; Sat, 14 Sep 2002 22:20:40 -0300 (BRT) Message-ID: <004301c25c56$599e88e0$2101a8c0@u2z7z2> From: "Wagner" To: References: <003201c25ab7$546ade00$53909ec8@u2z7z2> <00a801c25b39$fb5f5e70$0401010a@xt> Subject: [obm-l] =?iso-8859-1?Q?Re:_=5Bobm-l=5D_Re:_=5Bobm-l=5D_D=FAvida_sobre_sequ=EAncia?= =?iso-8859-1?Q?s_rand=F4micas_=28problema_do_tipo_NP=29=28to_vinicius=29?= Date: Sat, 14 Sep 2002 22:22:25 -0300 Organization: Wagner MIME-Version: 1.0 Content-Type: multipart/alternative; boundary="----=_NextPart_000_003F_01C25C3D.33E4FA80" X-Priority: 3 X-MSMail-Priority: Normal X-Mailer: Microsoft Outlook Express 5.50.4133.2400 X-MimeOLE: Produced By Microsoft MimeOLE V5.50.4133.2400 Sender: owner-obm-l@sucuri.mat.puc-rio.br Precedence: bulk Reply-To: obm-l@mat.puc-rio.br This is a multi-part message in MIME format. ------=_NextPart_000_003F_01C25C3D.33E4FA80 Content-Type: text/plain; charset="iso-8859-1" Content-Transfer-Encoding: quoted-printable Oi para todos Vinicius perguntou o que =E9 uma interssec=E7=E3o aleat=F3ria entre 2 = conjuntos Imagine que A e B s=E3o conjuntos com infinitos elementos. E C =E9 uma = interssec=E7=E3o aleat=F3ria de A e B. Isso quer dizer que todos os = termos de C pertencem a A e B, mas foram escolhidos de forma = aleat=F3ria, de modo que at=E9 o n=FAmero de elementos de C =E9 = aleat=F3rio e todas as quantidades de elementos tem a mesma = probabiliadde de ocorrer. Como P(x)=3Dn(x)/(E.A). Como E.A =E9 infinito, = pois A e B s=E3o infinitos, a chance de C ter finitos termos =E9 0. Por = isso se C =E9 uma interssec=E7=E3o aleat=F3ria de A e B infinitos, tal = que C n=E3o =E9 vazio, C possui infinitos elementos. Quanto a 2=AA pergunta, se existem infinitos algoritmos capazes de criar = a sequ=EAncia do problema. O algoritmo gen=E9rico que criou todos esses = algoritmos, s=F3 pode ser determinado ap=F3s a an=E1lise de todos os = algoritmos (pelo que eu entendi), isso caracteriza um problema do tipo = NP. Andr=E9 T. ----- Original Message -----=20 From: Vinicius Jos=E9 Fortuna=20 To: obm-l@mat.puc-rio.br=20 Sent: Friday, September 13, 2002 12:26 PM Subject: [obm-l] Re: [obm-l] D=FAvida sobre sequ=EAncias rand=F4micas = (problema do tipo NP) No meu ponto de vista, existem infinitos algoritmos para fazer alguma = coisa. At=E9 para n=E3o fazer nada. Por exemplo, um algoritmo para calcular a+b. Pode-se fazer da maneira = tradicional, ou ent=E3o pode-se fazer um loop percorrendo os n=FAmeros, = subtraindo b e verificando se =E9 igual a 'a'. Ou ent=E3o pode-se somar 6 a 'a' somar 'b' ao resultado e depois = subtrair '6'. Agora, em vez de fazer as opera=E7=E3o acima com 6, fa=E7a com todas = os outros naturais. Como s=E3o constantes, haver=E1 infinitos algoritmos = que fazem a soma. Enfim, o meu racioc=EDnio =E9 por a=ED. Esse =E9 mais um motivo que as = classes P e NP, por exemplo, s=E3o conjuntos de problemas e n=E3o de = algoritmos. Estudar algoritmos pode ser muito mais complicado. Formalmente, em teoria da computa=E7=E3o, podemos definir um problema = de decis=E3o como uma linguagem. Um problema de decis=E3o =E9 um cuja = resposta =E9 sim ou n=E3o. Uma linguagem pode ser considerada como um = conjuntos de seq=FC=EAncias de caracteres de um alfabeto. Podemos sempre = escrever uma inst=E2ncia de um problema como uma string em um alfabeto. = Assim o problema pode ser visto como um mapeamento de tais = seq=FC=EAncias em {0,1}. A linguagem associada ao problema cont=E9m = todas as seq=FC=EAncias (e somente elas) que s=E3o mapeadas em 1. Assim todo problema de decis=E3o fica reduzido ao problema: Essa = seq=FC=EAncia pertence a essa linguagem? Isso facilita bastante a teoria. Problemas de otimiza=E7=E3o podem ser reduzidos a problemas de = decis=E3o utilizando busca bin=E1ria com perguntas do tipo: O melhor = valor =E9 melhor que k? Maiores informa=E7=F5es podem ser encontradas no texto de Cook sobre o = problema "P=3DNP?" no site www.claymath.org/prizeproblems Uma outra observa=E7=E3o =E9 que nem todo problema pertence = necessariamente a NP. Pode ser que vc n=E3o consiga um algoritmo = polinomial em uma m=E1quina n=E3o-determin=EDstica para ele. Eu n=E3o entendi a conclus=E3o: "Logo existem infinitas alternativas a = serem analisadas, ou seja o problema =E9 do tipo NP." Poderia tb me explicar o que =E9 "intersec=E7=E3o aleat=F3ria"? At=E9 mais Vinicius ----- Original Message -----=20 From: Wagner=20 To: obm-l@mat.puc-rio.br=20 Sent: Thursday, September 12, 2002 8:51 PM Subject: [obm-l] D=FAvida sobre sequ=EAncias rand=F4micas (problema = do tipo NP) Oi pessoal Queria perguntar ao Nicolau ou a quem conseguir me resolver essa = pergunta: Se um algoritmo pode construir uma sequ=EAncia rand=F4mica, uma = sequ=EAncia qualquer desse tipo com um n=FAmero finito n de termos = poderia ent=E3o ser descrita por uma infinidade de algoritmos = diferentes, e esses algoritmos podem ser todos construidos a partir de = um algoritmo.(todos os algoritmos iriam variar de problema para = problema). Ou seja uma sequ=EAncia pode ser construida por infinitos = algoritmos diferentes, mas todos esses algoritmos podem ser construidos = a partir de um mesmo algoritmo. A problema =E9 o seguinte: Dada uma sequ=EAncia qualquer, qual =E9 o = algoritmo que gera todos os outros? A minha d=FAvida =E9 que pelo o que posso ver esse problema =E9 um = problema do tipo NP (polinominal n=E3o-determin=EDstico), ou seja a sua = resposta existe mas =E9 imposs=EDvel de ser dada na pr=E1tica. O meu racioc=EDnio para chegar a essa conclus=E3o foi o seguinte: Primeiro =E9 preciso provar que o problema tem solu=E7=E3o: Sendo a sequ=EAncia A=3Da,b,c,d,e,f,g,...,n (em que a =E9 o 1=BA = termo, b =E9 o 2=BA, at=E9 n que =E9 o n-=E9simo termo). Sendo B o conjunto de algoritmos que geram sequ=EAncias em que = quando o primeiro termo =E9 a, o segundo =E9 b e sendo C o conjunto dos = algoritmos para os quais se o 1=BA termo =E9 b, o 2=BA =E9 C. O algoritmo que descreve a sequ=EAncia A ent=E3o pertence =E0 B = intersec=E7=E3o com C. B e C possuem infinitos elementos. Como a,b,c = s=E3o 3 n=FAmeros aleat=F3rios, B intersec=E7=E3o com C =E9 aleat=F3rio. = A intersec=E7=E3o aleat=F3ria de 2 conjuntos infinitos =E9 um conjunto = infinito. Expandindo esse racioc=EDnio, existem infinitos algoritmos que = perfazem A. Logo existem infinitas alternativas a serem analisadas, ou = seja o problema =E9 do tipo NP. Esse pensamento esta certo? J=E1 vou agradecendo Andr=E9 T. ------=_NextPart_000_003F_01C25C3D.33E4FA80 Content-Type: text/html; charset="iso-8859-1" Content-Transfer-Encoding: quoted-printable
Oi para todos
 
Vinicius perguntou o que =E9 uma = interssec=E7=E3o=20 aleat=F3ria entre 2 conjuntos
 
Imagine que A e B s=E3o conjuntos com = infinitos=20 elementos. E C =E9 uma interssec=E7=E3o aleat=F3ria de A e B. Isso quer = dizer que todos=20 os termos de C pertencem a A e B, mas foram escolhidos de forma = aleat=F3ria, de=20 modo que at=E9 o n=FAmero de elementos de C =E9 aleat=F3rio e todas as = quantidades de=20 elementos tem a mesma probabiliadde de ocorrer. Como P(x)=3Dn(x)/(E.A). = Como E.A =E9=20 infinito, pois A e B s=E3o infinitos, a chance de C ter finitos termos = =E9 0. Por=20 isso se C =E9 uma interssec=E7=E3o aleat=F3ria de A e B infinitos, tal = que C n=E3o =E9=20 vazio, C possui infinitos elementos.
 
Quanto a 2=AA pergunta, se existem = infinitos=20 algoritmos capazes de criar a sequ=EAncia do problema. O algoritmo = gen=E9rico que=20 criou todos esses algoritmos, s=F3 pode ser determinado ap=F3s a = an=E1lise de todos os=20 algoritmos (pelo que eu entendi), isso caracteriza um problema do tipo=20 NP.
 
Andr=E9 T.
 
 
 
----- Original Message -----
From:=20 Vinicius Jos=E9 = Fortuna
Sent: Friday, September 13, = 2002 12:26=20 PM
Subject: [obm-l] Re: [obm-l] = D=FAvida sobre=20 sequ=EAncias rand=F4micas (problema do tipo NP)

No meu ponto de vista, existem = infinitos=20 algoritmos para fazer alguma coisa. At=E9 para n=E3o fazer = nada.
 
Por exemplo, um algoritmo = para calcular a+b.=20 Pode-se fazer da maneira tradicional, ou ent=E3o pode-se fazer um loop = percorrendo os n=FAmeros, subtraindo b e verificando se =E9 igual a=20 'a'.
Ou ent=E3o pode-se somar 6 a 'a' = somar 'b' ao=20 resultado e depois subtrair '6'.
Agora, em vez de fazer as = opera=E7=E3o acima com 6,=20 fa=E7a com todas os outros naturais. Como s=E3o constantes, haver=E1 = infinitos=20 algoritmos que fazem a soma.
 
Enfim, o meu racioc=EDnio =E9 por = a=ED. Esse =E9 mais um=20 motivo que as classes P e NP, por exemplo, s=E3o conjuntos de = problemas e=20 n=E3o de algoritmos. Estudar algoritmos pode ser muito mais=20 complicado.
 
Formalmente, em teoria da = computa=E7=E3o, podemos=20 definir um problema de decis=E3o como uma linguagem. Um problema de = decis=E3o =E9 um=20 cuja resposta =E9 sim ou n=E3o. Uma linguagem pode ser considerada = como um=20 conjuntos de seq=FC=EAncias de caracteres de um alfabeto. Podemos = sempre escrever=20 uma inst=E2ncia de um problema como uma string em um alfabeto. Assim=20 o problema pode ser visto como um mapeamento de tais = seq=FC=EAncias em {0,1}.=20 A linguagem associada ao problema cont=E9m todas as seq=FC=EAncias (e = somente elas)=20 que s=E3o mapeadas em 1.
 
Assim todo problema de decis=E3o fica = reduzido ao=20 problema: Essa seq=FC=EAncia pertence a essa = linguagem?
Isso facilita bastante a = teoria.
 
Problemas de otimiza=E7=E3o podem ser = reduzidos a=20 problemas de decis=E3o utilizando busca bin=E1ria com perguntas do = tipo: O melhor=20 valor =E9 melhor que k?
Maiores informa=E7=F5es podem ser = encontradas no=20 texto de Cook sobre o problema "P=3DNP?" no site www.claymath.org/prizeprob= lems
 
Uma outra observa=E7=E3o =E9 que nem = todo problema=20 pertence necessariamente a NP. Pode ser que vc n=E3o consiga um = algoritmo=20 polinomial em uma m=E1quina n=E3o-determin=EDstica para = ele.
 
Eu n=E3o entendi a conclus=E3o: "Logo = existem=20 infinitas alternativas a serem analisadas, ou seja o problema =E9 do = tipo=20 NP."
 
Poderia tb me explicar o que =E9 = "intersec=E7=E3o=20 aleat=F3ria"?
 
At=E9 mais
 
Vinicius
----- Original Message -----
From:=20 Wagner
Sent: Thursday, September 12, = 2002 8:51=20 PM
Subject: [obm-l] D=FAvida = sobre=20 sequ=EAncias rand=F4micas (problema do tipo NP)

Oi pessoal
 
Queria perguntar ao Nicolau ou a = quem conseguir=20 me resolver essa pergunta:
 
Se um algoritmo pode construir uma = sequ=EAncia=20 rand=F4mica, uma sequ=EAncia qualquer desse tipo com um n=FAmero = finito n de=20 termos poderia ent=E3o ser descrita por uma infinidade de = algoritmos=20 diferentes, e esses algoritmos podem ser todos construidos a partir = de um=20 algoritmo.(todos os algoritmos iriam variar de problema para = problema).=20 Ou seja uma sequ=EAncia pode ser construida por infinitos algoritmos = diferentes, mas todos esses algoritmos podem ser construidos a = partir de um=20 mesmo algoritmo.
 
A problema =E9 o seguinte: Dada uma = sequ=EAncia=20 qualquer, qual =E9 o algoritmo que gera todos os = outros?
 
A minha d=FAvida =E9 que pelo o que = posso ver esse=20 problema =E9 um problema do tipo NP (polinominal = n=E3o-determin=EDstico), ou seja=20 a sua resposta existe mas =E9 imposs=EDvel de ser dada na = pr=E1tica.
 
O meu racioc=EDnio para chegar a = essa conclus=E3o=20 foi o seguinte:
 
Primeiro =E9 preciso provar que o = problema tem=20 solu=E7=E3o:
Sendo a sequ=EAncia = A=3Da,b,c,d,e,f,g,...,n (em que=20 a =E9 o 1=BA termo, b =E9 o 2=BA, at=E9 n que =E9 o n-=E9simo = termo).
Sendo B o conjunto de algoritmos = que geram=20 sequ=EAncias em que quando o primeiro termo =E9 a, o segundo =E9 b e = sendo C o=20 conjunto dos algoritmos para os quais se o 1=BA termo =E9 b, o 2=BA = =E9=20 C.
 
O algoritmo que descreve a = sequ=EAncia A ent=E3o=20 pertence =E0 B intersec=E7=E3o com C. B e C possuem = infinitos elementos.=20 Como a,b,c s=E3o 3 n=FAmeros aleat=F3rios, B intersec=E7=E3o com C = =E9 aleat=F3rio. A=20 intersec=E7=E3o aleat=F3ria de 2 conjuntos infinitos =E9 um conjunto = infinito.=20 Expandindo esse racioc=EDnio, existem infinitos algoritmos que = perfazem A.=20 Logo existem infinitas alternativas a serem analisadas, ou seja o = problema =E9=20 do tipo NP.
 
Esse pensamento esta = certo?
 
J=E1 vou agradecendo
 
Andr=E9=20 T.
------=_NextPart_000_003F_01C25C3D.33E4FA80-- ========================================================================= Instruções para entrar na lista, sair da lista e usar a lista em http://www.mat.puc-rio.br/~nicolau/olimp/obm-l.html O administrador desta lista é ========================================================================= From owner-obm-l@sucuri.mat.puc-rio.br Sat Sep 14 22:45:08 2002 Return-Path: Received: (from majordom@localhost) by sucuri.mat.puc-rio.br (8.9.3/8.9.3) id WAA01156 for obm-l-MTTP; Sat, 14 Sep 2002 22:44:35 -0300 Received: from mail.gmx.net (mail.gmx.net [213.165.64.20]) by sucuri.mat.puc-rio.br (8.9.3/8.9.3) with SMTP id WAA01152 for ; Sat, 14 Sep 2002 22:44:31 -0300 Received: (qmail 14834 invoked by uid 0); 15 Sep 2002 01:39:03 -0000 Received: from unknown (HELO gomes) (200.216.104.68) by mail.gmx.net (mp006-rz3) with SMTP; 15 Sep 2002 01:39:03 -0000 Date: Sat, 14 Sep 2002 22:39:31 -0300 From: Igor GomeZZ X-Mailer: The Bat! (v1.61) Organization: -- X-Priority: 3 (Normal) Message-ID: <16860203007.20020914223931@gmx.net> To: Marcelo Souza Subject: Re: [obm-l] fatorial In-Reply-To: References: MIME-Version: 1.0 Content-Type: text/plain; charset=ISO-8859-15 Content-Transfer-Encoding: 8bit Sender: owner-obm-l@sucuri.mat.puc-rio.br Precedence: bulk Reply-To: obm-l@mat.puc-rio.br Em 24/8/2002, 21:57, Marcelo (marcelo_souza7@hotmail.com) disse: > A resposta eh 249. [..] > inf > e= sum(parte inteira de)[n/p^k] > k=1 > bom, espero que tenha sido claro, somatorio do piso de n/p^k..fazendo n=1000 > e p=5, dah 249 =) Tem como vc explicar um pouco mais esse teorema, parace ter muitos usos :) e = sum(1000/(5^k)) e = 250 (exato), num deu os 259 Fui! ####### Igor GomeZZ ######## UIN: 29249895 Vitória, Espírito Santo, Brasil Criação: 14/9/2002 (21:15) #################################### Pare para pensar: A verdade é filha do tempo, não da autoridade. (Francis Bacon) #################################### ========================================================================= Instruções para entrar na lista, sair da lista e usar a lista em http://www.mat.puc-rio.br/~nicolau/olimp/obm-l.html O administrador desta lista é ========================================================================= From owner-obm-l@sucuri.mat.puc-rio.br Sat Sep 14 23:29:46 2002 Return-Path: Received: (from majordom@localhost) by sucuri.mat.puc-rio.br (8.9.3/8.9.3) id XAA02142 for obm-l-MTTP; Sat, 14 Sep 2002 23:28:27 -0300 Received: from mediterraneo.rjnet.com.br (mediterraneo.rjnet.com.br [200.222.31.30]) by sucuri.mat.puc-rio.br (8.9.3/8.9.3) with ESMTP id XAA02138 for ; Sat, 14 Sep 2002 23:28:24 -0300 Received: from locutus.rjnet.com.br (root@locutus.rjnet.com.br [200.222.31.10]) by mediterraneo.rjnet.com.br (8.11.4/8.11.4) with ESMTP id g8F2U2C05618 for ; Sat, 14 Sep 2002 23:30:02 -0300 Received: from felipe ([200.222.39.86]) by locutus.rjnet.com.br (8.11.2/8.11.2) with SMTP id g8F29m216346 for ; Sat, 14 Sep 2002 23:09:48 -0300 Message-ID: <003c01c25c5f$bacb4f50$158c000a@felipe> From: "Felipe Villela Dias" To: Subject: [obm-l] =?iso-8859-1?Q?=5Boff=5D_uma_d=FAvida_de_F=EDsica.?= Date: Sat, 14 Sep 2002 23:29:32 -0300 MIME-Version: 1.0 Content-Type: multipart/alternative; boundary="----=_NextPart_000_0039_01C25C46.94528FA0" X-Priority: 3 X-MSMail-Priority: Normal X-Mailer: Microsoft Outlook Express 6.00.2600.0000 X-MimeOLE: Produced By Microsoft MimeOLE V6.00.2600.0000 Sender: owner-obm-l@sucuri.mat.puc-rio.br Precedence: bulk Reply-To: obm-l@mat.puc-rio.br This is a multi-part message in MIME format. ------=_NextPart_000_0039_01C25C46.94528FA0 Content-Type: text/plain; charset="iso-8859-1" Content-Transfer-Encoding: quoted-printable Eu sei que pela Teoria da Relatividade a velocidade da luz =E9 constante = em todas as dire=E7=F5es e sentidos e n=E3o existe um tempo universal. A minha d=FAvida =E9: esta afirmativa =E9 um axioma ou existe uma forma = de se provar isso? Abra=E7os. P.S. Galera, se eu tiver fugido muito dos prop=F3sitos desta lista, por = favor me avisem, para que n=E3o volte a ocorrer. --- Outgoing mail is certified Virus Free. Checked by AVG anti-virus system (http://www.grisoft.com). Version: 6.0.386 / Virus Database: 218 - Release Date: 9/9/2002 ------=_NextPart_000_0039_01C25C46.94528FA0 Content-Type: text/html; charset="iso-8859-1" Content-Transfer-Encoding: quoted-printable
Eu sei que pela Teoria da Relatividade a velocidade da luz =E9 = constante em=20 todas as dire=E7=F5es e sentidos e n=E3o existe um tempo = universal.
A minha d=FAvida =E9: esta afirmativa =E9 um axioma ou existe = uma forma de=20 se provar isso?
Abra=E7os.
 
P.S. Galera, se eu tiver fugido muito dos prop=F3sitos desta lista, = por favor=20 me avisem, para que n=E3o volte a ocorrer.
 
 
 

---
Outgoing mail is certified Virus Free.
Checked by AVG = anti-virus system (http://www.grisoft.com).
Version: = 6.0.386 /=20 Virus Database: 218 - Release Date: 9/9/2002
------=_NextPart_000_0039_01C25C46.94528FA0-- ========================================================================= Instruções para entrar na lista, sair da lista e usar a lista em http://www.mat.puc-rio.br/~nicolau/olimp/obm-l.html O administrador desta lista é ========================================================================= From owner-obm-l@sucuri.mat.puc-rio.br Sun Sep 15 01:04:21 2002 Return-Path: Received: (from majordom@localhost) by sucuri.mat.puc-rio.br (8.9.3/8.9.3) id BAA03274 for obm-l-MTTP; Sun, 15 Sep 2002 01:02:57 -0300 Received: from smtp-33.ig.com.br (smtp-33.ig.com.br [200.226.132.183]) by sucuri.mat.puc-rio.br (8.9.3/8.9.3) with SMTP id BAA03270 for ; Sun, 15 Sep 2002 01:02:54 -0300 Received: (qmail 14379 invoked from network); 15 Sep 2002 04:03:44 -0000 Received: from unknown (HELO henrique) (200.203.126.118) by smtp-33.ig.com.br with SMTP; 15 Sep 2002 04:03:44 -0000 Message-ID: <000f01c25c6c$f2736f70$019da8c0@henrique> From: "Henrique Branco" To: References: <003c01c25c5f$bacb4f50$158c000a@felipe> Subject: [obm-l] =?iso-8859-1?Q?Re:_=5Bobm-l=5D_=5Boff=5D_uma_d=FAvida_de_F=EDsica.?= Date: Sun, 15 Sep 2002 01:04:10 -0300 MIME-Version: 1.0 Content-Type: multipart/alternative; boundary="----=_NextPart_000_000C_01C25C53.CCC46860" X-Priority: 3 X-MSMail-Priority: Normal X-Mailer: Microsoft Outlook Express 6.00.2600.0000 X-MimeOLE: Produced By Microsoft MimeOLE V6.00.2600.0000 Sender: owner-obm-l@sucuri.mat.puc-rio.br Precedence: bulk Reply-To: obm-l@mat.puc-rio.br This is a multi-part message in MIME format. ------=_NextPart_000_000C_01C25C53.CCC46860 Content-Type: text/plain; charset="iso-8859-1" Content-Transfer-Encoding: quoted-printable Olha, n=E3o conhe=E7o a forma matem=E1tica pra tal postulado (sim, =E9 = um postulado f=EDsico). Mas acho que ele foi baseado unicamente no experimento Michelson-Morley. Se quiser mais detalhes (caso voc=EA n=E3o conhe=E7a o experimento), = escreva de volta que eu explico direitinho. Abra=E7os, Henrique. ------=_NextPart_000_000C_01C25C53.CCC46860 Content-Type: text/html; charset="iso-8859-1" Content-Transfer-Encoding: quoted-printable
Olha, n=E3o conhe=E7o a forma = matem=E1tica pra tal=20 postulado (sim, =E9 um postulado f=EDsico).
Mas acho que ele foi baseado unicamente = no=20 experimento Michelson-Morley.
Se quiser mais detalhes (caso voc=EA = n=E3o conhe=E7a o=20 experimento), escreva de volta que eu explico direitinho.
 
Abra=E7os,
Henrique.
------=_NextPart_000_000C_01C25C53.CCC46860-- ========================================================================= Instruções para entrar na lista, sair da lista e usar a lista em http://www.mat.puc-rio.br/~nicolau/olimp/obm-l.html O administrador desta lista é ========================================================================= From owner-obm-l@sucuri.mat.puc-rio.br Sun Sep 15 06:46:59 2002 Return-Path: Received: (from majordom@localhost) by sucuri.mat.puc-rio.br (8.9.3/8.9.3) id GAA08249 for obm-l-MTTP; Sun, 15 Sep 2002 06:45:45 -0300 Received: from hotmail.com (oe85.pav0.hotmail.com [64.4.33.227]) by sucuri.mat.puc-rio.br (8.9.3/8.9.3) with ESMTP id GAA08245 for ; Sun, 15 Sep 2002 06:45:42 -0300 Received: from mail pickup service by hotmail.com with Microsoft SMTPSVC; Sun, 15 Sep 2002 02:46:50 -0700 X-Originating-IP: [200.227.231.71] From: "e isso mesmo" To: Subject: [obm-l] =?iso-8859-1?Q?Como_anda_seu_cora=E7=E3o...=3F?= Date: Sun, 15 Sep 2002 06:46:46 -0300 MIME-Version: 1.0 X-Mailer: MSN Explorer 7.00.0021.1900 Content-Type: multipart/alternative; boundary="----=_NextPart_001_0000_01C25C83.A91A27D0" Message-ID: X-OriginalArrivalTime: 15 Sep 2002 09:46:50.0084 (UTC) FILETIME=[D066B240:01C25C9C] Sender: owner-obm-l@sucuri.mat.puc-rio.br Precedence: bulk Reply-To: obm-l@mat.puc-rio.br ------=_NextPart_001_0000_01C25C83.A91A27D0 Content-Type: text/plain; charset="iso-8859-1" Content-Transfer-Encoding: quoted-printable Companheiros, me auxiliem: "A teoria do Biorritmo diz que os estados f=EDsico, mental e emocional de= uma pessoa oscilam periodicamente, a partir do dia do nascimento, em cic= los de 23 dias, 29 dias e 33 dias, respectivamente. Dado que os dias mais= positivos dos ciclos f=EDsico, mental e emocional s=E3o, respectivamente= , o sexto, o s=E9timo e o oitavo de cada ciclo, nos primeiros dez anos de= vida de uma pessoa, quantas vezes os tr=EAs ciclos est=E3o simultaneamen= te no ponto m=E1ximo?" Tentei o Teorema Chin=EAs do resto, mas n=E3o consegui montar o problema. ObrigadoAproveite melhor a Web. Fa=E7a o download GR=C1TIS do MSN Explore= r : http://explorer.msn.com.br/intl.asp#po ------=_NextPart_001_0000_01C25C83.A91A27D0 Content-Type: text/html; charset="iso-8859-1" Content-Transfer-Encoding: quoted-printable
Companheiros, me auxiliem:
&nb= sp;
"A= teoria do Biorritmo diz que os estados f=EDsico, mental e emocional de u= ma pessoa oscilam periodicamente, a partir do dia do nascimento, em ciclo= s de 23 dias, 29 dias e 33 dias, respectivamente. Dado que os dias mais p= ositivos dos ciclos f=EDsico, mental e emocional s=E3o, respectivamente, = o sexto, o s=E9timo e o oitavo de cada ciclo, nos primeiros dez anos de v= ida de uma pessoa, quantas vezes os tr=EAs ciclos est=E3o simultaneamente= no ponto m=E1ximo?"
 
Tentei o Teorema Chin=EAs do resto, mas n=E3o consegui montar o problem= a.
 
Obrigado

Aproveite melhor a Web. Fa=E7a o download GR=C1TIS do MSN Expl= orer : http://explorer= .msn.com.br/intl.asp#po

------=_NextPart_001_0000_01C25C83.A91A27D0-- ========================================================================= Instruções para entrar na lista, sair da lista e usar a lista em http://www.mat.puc-rio.br/~nicolau/olimp/obm-l.html O administrador desta lista é ========================================================================= From owner-obm-l@sucuri.mat.puc-rio.br Sun Sep 15 06:51:42 2002 Return-Path: Received: (from majordom@localhost) by sucuri.mat.puc-rio.br (8.9.3/8.9.3) id GAA08396 for obm-l-MTTP; Sun, 15 Sep 2002 06:51:25 -0300 Received: from hotmail.com (oe95.pav0.hotmail.com [64.4.33.237]) by sucuri.mat.puc-rio.br (8.9.3/8.9.3) with ESMTP id GAA08392 for ; Sun, 15 Sep 2002 06:51:22 -0300 Received: from mail pickup service by hotmail.com with Microsoft SMTPSVC; Sun, 15 Sep 2002 02:52:38 -0700 X-Originating-IP: [200.227.231.71] From: "e isso mesmo" To: Subject: [obm-l] Pescaria Date: Sun, 15 Sep 2002 06:52:35 -0300 MIME-Version: 1.0 X-Mailer: MSN Explorer 7.00.0021.1900 Content-Type: multipart/alternative; boundary="----=_NextPart_001_0001_01C25C84.78DC57E0" Message-ID: X-OriginalArrivalTime: 15 Sep 2002 09:52:38.0080 (UTC) FILETIME=[9FD2AC00:01C25C9D] Sender: owner-obm-l@sucuri.mat.puc-rio.br Precedence: bulk Reply-To: obm-l@mat.puc-rio.br ------=_NextPart_001_0001_01C25C84.78DC57E0 Content-Type: text/plain; charset="iso-8859-1" Content-Transfer-Encoding: quoted-printable Colegas me ajudem a verificar se minha equa=E7=E3o est=E1 correta. "Um pescador tenta pescar um cardume jogando diversas redes na =E1gua. Se= cair exatamente um peixe em cada rede, salvam-se ainda n peixes. Se cair= em n peixes em cada rede, sobram n redes vazias. Quantas s=E3o as redes? = Quantos s=E3o os peixes? Obrigado!Aproveite melhor a Web. Fa=E7a o download GR=C1TIS do MSN Explor= er : http://explorer.msn.com.br/intl.asp#po ------=_NextPart_001_0001_01C25C84.78DC57E0 Content-Type: text/html; charset="iso-8859-1" Content-Transfer-Encoding: quoted-printable
 
Colegas me ajudem a ver= ificar se minha equa=E7=E3o est=E1 correta.
 
= "Um pescador tenta pescar um cardume jogando diversas rede= s na =E1gua. Se cair exatamente um peixe em cada rede, salvam-se ainda n = peixes. Se cairem n peixes em cada rede, sobram n redes vazias. Quantas s= =E3o as redes? Quantos s=E3o os peixes?
=  
Obrigado!


Aproveite melhor a Web. Fa=E7a o download GR=C1TIS= do MSN Explorer : htt= p://explorer.msn.com.br/intl.asp#po

------=_NextPart_001_0001_01C25C84.78DC57E0-- ========================================================================= Instruções para entrar na lista, sair da lista e usar a lista em http://www.mat.puc-rio.br/~nicolau/olimp/obm-l.html O administrador desta lista é ========================================================================= From owner-obm-l@sucuri.mat.puc-rio.br Sun Sep 15 09:10:00 2002 Return-Path: Received: (from majordom@localhost) by sucuri.mat.puc-rio.br (8.9.3/8.9.3) id JAA09986 for obm-l-MTTP; Sun, 15 Sep 2002 09:09:00 -0300 Received: from web21308.mail.yahoo.com (web21308.mail.yahoo.com [216.136.128.174]) by sucuri.mat.puc-rio.br (8.9.3/8.9.3) with SMTP id JAA09982 for ; Sun, 15 Sep 2002 09:08:57 -0300 Message-ID: <20020915121010.97329.qmail@web21308.mail.yahoo.com> Received: from [200.180.84.126] by web21308.mail.yahoo.com via HTTP; Sun, 15 Sep 2002 09:10:10 ART Date: Sun, 15 Sep 2002 09:10:10 -0300 (ART) From: =?iso-8859-1?q?Jorge=20Paulino?= Subject: [obm-l] IME To: obm-l@mat.puc-rio.br MIME-Version: 1.0 Content-Type: text/plain; charset=iso-8859-1 Content-Transfer-Encoding: 8bit Sender: owner-obm-l@sucuri.mat.puc-rio.br Precedence: bulk Reply-To: obm-l@mat.puc-rio.br Galera, alguém sabe onde posso encontrar a prova do IME/2001 RESOLVIDA??? Um abraço Jorge _______________________________________________________________________ Yahoo! PageBuilder O super editor para criação de sites: é grátis, fácil e rápido. http://br.geocities.yahoo.com/v/pb.html ========================================================================= Instruções para entrar na lista, sair da lista e usar a lista em http://www.mat.puc-rio.br/~nicolau/olimp/obm-l.html O administrador desta lista é ========================================================================= From owner-obm-l@sucuri.mat.puc-rio.br Sun Sep 15 09:56:48 2002 Return-Path: Received: (from majordom@localhost) by sucuri.mat.puc-rio.br (8.9.3/8.9.3) id JAA10681 for obm-l-MTTP; Sun, 15 Sep 2002 09:56:07 -0300 Received: from traven9.pub1 (traven9.uol.com.br [200.221.4.35]) by sucuri.mat.puc-rio.br (8.9.3/8.9.3) with ESMTP id JAA10677 for ; Sun, 15 Sep 2002 09:56:05 -0300 Received: from ui.uol.com.br ([200.222.181.197]) by traven9.pub1 (8.9.1/8.9.1) with ESMTP id JAA26835; Sun, 15 Sep 2002 09:59:01 -0300 (BRT) Message-Id: <5.1.0.14.2.20020915095357.0202dbf0@pop3.uol.com.br> X-Sender: cavictor@pop3.uol.com.br X-Mailer: QUALCOMM Windows Eudora Version 5.1 Date: Sun, 15 Sep 2002 09:56:58 -0300 To: obm-l@mat.puc-rio.br, obm-l@mat.puc-rio.br From: Carlos Victor Subject: Re: [obm-l] IME In-Reply-To: <20020915121010.97329.qmail@web21308.mail.yahoo.com> Mime-Version: 1.0 Content-Type: text/plain; charset="iso-8859-1"; format=flowed Content-Transfer-Encoding: 8bit X-MIME-Autoconverted: from quoted-printable to 8bit by sucuri.mat.puc-rio.br id JAA10678 Sender: owner-obm-l@sucuri.mat.puc-rio.br Precedence: bulk Reply-To: obm-l@mat.puc-rio.br Olá , Em www.estudemais.com.br acredito que você encontre . []´s Carlos Victor At 09:10 15/9/2002 -0300, Jorge Paulino wrote: >Galera, alguém sabe onde posso encontrar >a prova do IME/2001 RESOLVIDA??? >Um abraço >Jorge > >_______________________________________________________________________ >Yahoo! PageBuilder >O super editor para criação de sites: é grátis, fácil e rápido. >http://br.geocities.yahoo.com/v/pb.html >========================================================================= >Instruções para entrar na lista, sair da lista e usar a lista em >http://www.mat.puc-rio.br/~nicolau/olimp/obm-l.html >O administrador desta lista é >========================================================================= ========================================================================= Instruções para entrar na lista, sair da lista e usar a lista em http://www.mat.puc-rio.br/~nicolau/olimp/obm-l.html O administrador desta lista é ========================================================================= From owner-obm-l@sucuri.mat.puc-rio.br Sun Sep 15 10:51:02 2002 Return-Path: Received: (from majordom@localhost) by sucuri.mat.puc-rio.br (8.9.3/8.9.3) id KAA11451 for obm-l-MTTP; Sun, 15 Sep 2002 10:48:40 -0300 Received: from imo-d03.mx.aol.com (imo-d03.mx.aol.com [205.188.157.35]) by sucuri.mat.puc-rio.br (8.9.3/8.9.3) with ESMTP id KAA11447 for ; Sun, 15 Sep 2002 10:48:37 -0300 From: Lltmdrtm@aol.com Received: from Lltmdrtm@aol.com by imo-d03.mx.aol.com (mail_out_v34.10.) id z.ca.1142e28d (4539) for ; Sun, 15 Sep 2002 09:49:44 -0400 (EDT) Message-ID: Date: Sun, 15 Sep 2002 09:49:44 EDT Subject: [obm-l] =?ISO-8859-1?Q?Contagem=20pol=EAmica-=20ajuda=20de=20novo?= To: obm-l@mat.puc-rio.br MIME-Version: 1.0 Content-Type: multipart/alternative; boundary="part1_ca.1142e28d.2ab5e9f8_boundary" X-Mailer: AOL 7.0 for Windows BR sub 10501 Sender: owner-obm-l@sucuri.mat.puc-rio.br Precedence: bulk Reply-To: obm-l@mat.puc-rio.br --part1_ca.1142e28d.2ab5e9f8_boundary Content-Type: text/plain; charset="US-ASCII" Content-Transfer-Encoding: 7bit De quantas maneiras 24 pessoas podem subir numa roda gigante de 12 assentos, sabendo que cada assento comporta duas pessoas e, a ordem das pessoas no assento importa? --part1_ca.1142e28d.2ab5e9f8_boundary Content-Type: text/html; charset="US-ASCII" Content-Transfer-Encoding: 7bit De quantas maneiras 24 pessoas podem subir numa roda gigante de 12 assentos, sabendo que cada assento comporta duas pessoas e, a ordem das pessoas no assento importa?

--part1_ca.1142e28d.2ab5e9f8_boundary-- ========================================================================= Instruções para entrar na lista, sair da lista e usar a lista em http://www.mat.puc-rio.br/~nicolau/olimp/obm-l.html O administrador desta lista é ========================================================================= From owner-obm-l@sucuri.mat.puc-rio.br Sun Sep 15 10:58:54 2002 Return-Path: Received: (from majordom@localhost) by sucuri.mat.puc-rio.br (8.9.3/8.9.3) id KAA11582 for obm-l-MTTP; Sun, 15 Sep 2002 10:58:37 -0300 Received: from riemann ([200.222.249.68]) by sucuri.mat.puc-rio.br (8.9.3/8.9.3) with ESMTP id KAA11577 for ; Sun, 15 Sep 2002 10:58:33 -0300 Received: from fabiodias by riemann with local (Exim 3.35 #1 (Debian)) id 17qZw8-0000GS-00 for ; Sun, 15 Sep 2002 10:59:44 -0300 Date: Sun, 15 Sep 2002 10:59:44 -0300 To: obm-l@mat.puc-rio.br Subject: Re: [obm-l] Pescaria Message-ID: <20020915135943.GA996@ieg.com.br> Mail-Followup-To: obm-l@mat.puc-rio.br References: Mime-Version: 1.0 Content-Type: multipart/signed; micalg=pgp-sha1; protocol="application/pgp-signature"; boundary="fdj2RfSjLxBAspz7" Content-Disposition: inline In-Reply-To: User-Agent: Mutt/1.3.28i From: Fabio Dias Moreira Sender: owner-obm-l@sucuri.mat.puc-rio.br Precedence: bulk Reply-To: obm-l@mat.puc-rio.br --fdj2RfSjLxBAspz7 Content-Type: text/plain; charset=unknown-8bit Content-Disposition: inline Content-Transfer-Encoding: quoted-printable On Sun, Sep 15, 2002 at 06:52:35AM -0300, e isso mesmo wrote: >=20 > Colegas me ajudem a verificar se minha equa??o est? correta. >=20 > "Um pescador tenta pescar um cardume jogando diversas redes na ?gua. Se c= air exatamente um peixe em cada rede, salvam-se ainda n peixes. Se cairem n= peixes em cada rede, sobram n redes vazias. Quantas s?o as redes? Quantos = s?o os peixes? >=20 Seja p o n=FAmero de peixes, r o n=FAmero de redes. Temos que r + n =3D p (1) p/n + n =3D r (2) Substituindo (2) em (1): p/n + n + n =3D p 2n =3D [(n-1)p]/n p =3D 2n^2/[n-1] (3) Como p =E9 inteiro (a n=E3o ser que um tubar=E3o tenha aparecido e comido p= arte de um peixe...), n-1 divide 2n^2. Mas n-1 e n s=E3o primos entre si, l= ogo n-1 e n^2 s=E3o primos entre si. Logo n-1 deve dividir 2. Por isso, n = =E9 2 ou 3, p =E9 8 ou 9 (por (3)), respectivamente, e r =E9 6 nos dois cas= os, por (1). []s, --=20 F=E1bio Dias Moreira (fabiodias@ieg.com.br) GPG fingerprint: 72F8 289F 1118 D225 700E 28D9 6A53 9016 BBF3 190A --fdj2RfSjLxBAspz7 Content-Type: application/pgp-signature Content-Disposition: inline -----BEGIN PGP SIGNATURE----- Version: GnuPG v1.0.6 (GNU/Linux) Comment: For info see http://www.gnupg.org iEYEARECAAYFAj2Ekk8ACgkQalOQFrvzGQrpawCghKe4oFQM3ytWk5j0Z+MUjueL kZIAnjpuhF3Cf2EHV5/wW68tWguI20ne =p7V1 -----END PGP SIGNATURE----- --fdj2RfSjLxBAspz7-- ========================================================================= Instruções para entrar na lista, sair da lista e usar a lista em http://www.mat.puc-rio.br/~nicolau/olimp/obm-l.html O administrador desta lista é ========================================================================= From owner-obm-l@sucuri.mat.puc-rio.br Sun Sep 15 10:59:02 2002 Return-Path: Received: (from majordom@localhost) by sucuri.mat.puc-rio.br (8.9.3/8.9.3) id KAA11571 for obm-l-MTTP; Sun, 15 Sep 2002 10:58:23 -0300 Received: from data2.poli.usp.br (data2.poli.usp.br [143.107.106.108]) by sucuri.mat.puc-rio.br (8.9.3/8.9.3) with ESMTP id KAA11566 for ; Sun, 15 Sep 2002 10:58:20 -0300 Received: from web2.poli.usp.br ([143.107.106.102]) by data2.poli.usp.br with Microsoft SMTPSVC(5.0.2195.4453); Sun, 15 Sep 2002 10:59:37 -0300 Received: from apl03.poli.usp.br ([143.107.106.15]) by web2.poli.usp.br with Microsoft SMTPSVC(5.0.2195.5329); Sun, 15 Sep 2002 10:59:16 -0300 X-MimeOLE: Produced By Microsoft Exchange V6.0.5762.3 content-class: urn:content-classes:message MIME-Version: 1.0 Content-Type: multipart/mixed; boundary="----_=_NextPart_001_01C25CC0.1407E16A" Subject: RES: [obm-l] Pescaria Date: Sun, 15 Sep 2002 10:59:15 -0300 Message-ID: <2B184DFE97456744924ACF58987D941D01935302@apl03.poli.usp.br> X-MS-Has-Attach: X-MS-TNEF-Correlator: <2B184DFE97456744924ACF58987D941D01935302@apl03.poli.usp.br> Thread-Topic: [obm-l] Pescaria Thread-Index: AcJcrP5cWDfwar46SliQtmFynTZYiAAElbNc From: "Edilon Ribeiro da Silva" To: X-OriginalArrivalTime: 15 Sep 2002 13:59:16.0304 (UTC) FILETIME=[14407500:01C25CC0] Sender: owner-obm-l@sucuri.mat.puc-rio.br Precedence: bulk Reply-To: obm-l@mat.puc-rio.br This is a multi-part message in MIME format. ------_=_NextPart_001_01C25CC0.1407E16A Content-Type: text/plain; charset="iso-8859-1" Content-Transfer-Encoding: quoted-printable Como podem cair n peixes em cada rede e sobrarem redes vazias? Se = ca=EDram n peixes em cada rede, =E9 porque todas as redes est=E3o com n = peixes. Concorda? =20 Edilon Ribeiro. -----Mensagem original----- De: e isso mesmo [mailto:rdrql@msn.com]=20 Enviada: dom 15/9/2002 06:52=20 Para: obm-l@mat.puc-rio.br=20 Cc:=20 Assunto: [obm-l] Pescaria =09 =09 =20 Colegas me ajudem a verificar se minha equa=E7=E3o est=E1 correta. =20 "Um pescador tenta pescar um cardume jogando diversas redes na =E1gua. = Se cair exatamente um peixe em cada rede, salvam-se ainda n peixes. Se = cairem n peixes em cada rede, sobram n redes vazias. Quantas s=E3o as = redes? Quantos s=E3o os peixes? =20 Obrigado! _____ =20 Aproveite melhor a Web. Fa=E7a o download GR=C1TIS do MSN Explorer : = http://explorer.msn.com.br/intl.asp#po ------_=_NextPart_001_01C25CC0.1407E16A Content-Type: application/ms-tnef; name="winmail.dat" Content-Transfer-Encoding: base64 eJ8+IhENAQaQCAAEAAAAAAABAAEAAQeQBgAIAAAA5AQAAAAAAADoAAEIgAcAGAAAAElQTS5NaWNy b3NvZnQgTWFpbC5Ob3RlADEIAQ2ABAACAAAAAgACAAEEgAEAFgAAAFJFUzogW29ibS1sXSBQZXNj YXJpYQAbBwEFgAMADgAAANIHCQAPAAoAOwAPAAAARQEBIIADAA4AAADSBwkADwAKADsADwAAAEUB AQmAAQAhAAAARUIyQTExN0YzQ0FFN0M0MUFEODQyRDI1MzU5QjFGRTUAVgcBA5AGAAwOAAA2AAAA AwA2AAAAAABAADkAauEHFMBcwgEeAD0AAQAAAAYAAABSRVM6IAAAAAIBRwABAAAALQAAAGM9QlI7 YT0gO3A9RVBVU1A7bD1BUEwwMy0wMjA5MTUxMzU5MTVaLTc2NTMxAAAAAB4ASQABAAAAEQAAAFtv Ym0tbF0gUGVzY2FyaWEAAAAAQABOAICz/J2dXMIBHgBaAAEAAAANAAAAZSBpc3NvIG1lc21vAAAA AAIBWwABAAAAOAAAAAAAAACBKx+kvqMQGZ1uAN0BD1QCAAAAAGUgaXNzbyBtZXNtbwBTTVRQAHJk cnFsQG1zbi5jb20AAgFcAAEAAAATAAAAU01UUDpSRFJRTEBNU04uQ09NAAAeAF0AAQAAAA0AAABl IGlzc28gbWVzbW8AAAAAAgFeAAEAAAA4AAAAAAAAAIErH6S+oxAZnW4A3QEPVAIAAAAAZSBpc3Nv IG1lc21vAFNNVFAAcmRycWxAbXNuLmNvbQACAV8AAQAAABMAAABTTVRQOlJEUlFMQE1TTi5DT00A AB4AZgABAAAABQAAAFNNVFAAAAAAHgBnAAEAAAAOAAAAcmRycWxAbXNuLmNvbQAAAB4AaAABAAAA BQAAAFNNVFAAAAAAHgBpAAEAAAAOAAAAcmRycWxAbXNuLmNvbQAAAB4AcAABAAAAEQAAAFtvYm0t bF0gUGVzY2FyaWEAAAAAAgFxAAEAAAAbAAAAAcJcrP5cWDfwar46SliQtmFynTZYiAAElbNcAB4A dAABAAAAFQAAAG9ibS1sQG1hdC5wdWMtcmlvLmJyAAAAAB4AGgwBAAAAGAAAAEVkaWxvbiBSaWJl aXJvIGRhIFNpbHZhAB4AHQ4BAAAAEQAAAFtvYm0tbF0gUGVzY2FyaWEAAAAAAgEJEAEAAADyBwAA 7gcAAEIbAABMWkZ1uGMmvAMACgByY3BnMTI1gjIDQ2h0bWwxAzA/AQMB9wqAAqQD4wIAY2jBCsBz ZXQwIAcTAoD/EAMAUARWCFUHshHVDlEDAd0Q1zIGAAbDEdUzBEYQ2fkS72Y0EG8XHBHjCO8J9/Y7 Gx8OMDUcMx1RG28bT+8cXx1hEeEMYGMAUAsJAWRMMzYRYAulNCAQAir2XA6yAZBnFPAKoxHjJAcA MTggPEhUTUwKPiUMNRFgPEJPRABZIEJHQ09MT7BSPSIjASAogiIGADBUWUxFKFAnkFJEQEVSLVJJ RyYgOgIgB4BkaXVtIG4ZAiBlOyfQKYRUT1CRKj1GT04qITEwBTFqdgSQZABwYSsHKTBGVyovKYQn kFQrkE0urkEAQ0tHUk9VTkSeLSgDLqAody7gcj0kUF5yJmAkEwAhAzB2CJB3UmsLgGQ1I8RiANBr FwnACGA0kCAAAHNocGcj4jXxC4BzdDXRNhFzdQOgZgMQbAhQGrECkXMSdi0ANjc4cDIxNfsaMDbY RjdxCYA39RowOmFyXDNBZG8A4DNBCrFc/nEbADNBEPADMDOlEWAjyyMjESTPZzk2JgBESZ5WM3kA ADxHI+k2ND9/eTyfMTQl8SyyN1AA0GX7KFAXKSI/axgwPCkIUARg7CBwBHEqoGMLcAXAA6DwcGVp eAeRQu9D9AqjtRqTPShxMErxP2tjE/BrPC8kJTgjoDwpADGgTrpHP2tiTDlHky2QIB7xymUjvDkU 8DwvTf8zh/VCzjU/AS8sslI/U09DqfdKZUrySx8zTzojvEZRPd//Ta9Ov1ZPV19YYAEgSz1GWvBz b2JyCsBHkVBiBCDodmF6BzBzU59Ur1W/N1EfUi9mdD8GUUexXCf/CYBi4CqhWm9bc0g/X29Kr/9L v0zPXL9O70/5Zt9n72Yft2RfZW92tCwDMGowOUdR5HJxClAgdARwY+Bqv69bc3rxY0QHkHRqITNH QNcFoGqCbBQuFxFuBaEtkP4/dx94L3Zvf2+Af0B+AcBfM4cKooEoCoAlHDBxUS//PzyBLz0/Pk8/ X4RPQX+Jb39Dn0SvRb+Pj4KfiM9wjSacbmI28AKAM5gnYQFA/40vhS+GP4dPiF+Wf4p/i49/jr+N r6Gvj8+Q35Hvkv9Fxy7gGrADoFJpYmwgA2D+LpQPlR+D/5ofmy+cP51PN6Rvn2+gekIoIDFwUVXN MBBFMvZioHR5OcAoUMBNQVJHSU4p1i0g/ninagqiEAI7NTvTO5GTX/uyXyPqNBFgvDCzP6EvrE/3 o0+5VzqwaSLypO+l9ACQunqmgDKnfAvxuTktxTI6TQnwcyRwR5EFsGlnvwuAB0DFMyUcOFAncVIz et/BQCrgCoG5X3D3QnHvyWboRGU6dD8vyvqxSVpB/wQBR0AHgUcxey+6/3FBvEFuWwDAAxB6wDoL IHpwbNRAbTcwLn2RXcGdx8/vyN/J78r/qIhuNDBQIcy/H83Pztc68CqgOLAvOS/FAdAwFPAwNjoO QNAv/9E/0kXUL9U/1k/XX9hv41Q+UArA2j/bT87IYsBtLaHTcWF0LnAhoC0FEP+p4GLQ1B/hv+LP 49/k70am/mPmr+e/ztfqr+u/7M/t3/vu7/XEQQQQNZDTAfCP8Z/f8q/ez9Hd6VPUAFAHkEfA/wch 8y/0P/VP/68AvwHJ+q//rK+tv67Pr98Fr7s9JfGrJf+57/0PvA+9H74vvz/AT8Ff/5efmK8GvwfP CN8J7wr/DZ//Dq8PvxDPEd8S7xP/RvK1sIZnevEwgCBhanVHgntQQC1haTdg/yFioFCQbbXGUGhQ QGV6gGoSN300tX0EMW3hcnPwHcAuGA//GR8aLxs/HE8dXx5vH38gj/8hnyKvI78VDxYfFy8pbyp/ /yuPLJ8try6/L88w3zHvOG9vNA81H8G/wsgzw36oWiK+Vd0QbBD/EdzwpxB0xZD3PrBJlKcQdU/y fsBLAFCQ9GpvJSBu3PC08SYhxbA/fIb7/z8v4FTGYHnyMWffJ3B+UGnTtRAnQHjpwGpwf0pBJXBL AWwTc2t54MWwbP9joOlwJsHS0EvwJgB92E0v/04/4FRQZcXhfdZSHmLCaoK7Y0p+UFEncEpRfOBz fTS/fHZpsFSvVb/gVFqjb1sH/18BbBR+/6sPBf85TzpfO2//PH89jz6fP69Av0HPYq9D7/9E/0YP Rx9ILwJPYS9nXzZP/zdfbN9jn2SvZb9mz3X/aO//af9rD24/bS+CP29PcF9xb91yf0/qcMYwSeEh c490n/9ib3mfeq97v3zPfd+P+LRYL5DLt9ORj4AUNQzCQk9kRFkNTTI3izG2kE3eTA1NA0D6Mepx Y7Wwt+A6PcZwbLcflR+BWGhyFad8MbkAY7kAalx1xGxk+BF+IF+fUp8RnwRjjmObioOPhJ9BcKnQ zSYgaVFhz9BsaEoBJgDoV2ViWoBGJ4MmAEwRGG93bqlAUmAgR1JBJ9BjMVRJU9zhIMhNU06pAHhw qUBSoB+kQLaw9i8mAJ1QZWY9gid30HRwOi8vUOAtpzQu05XqYS/GUHRsgi5bkHAjcG8nw3zqaaQA ZH/CZp7gxlC1gIB7SFlQRVJMtjAeS0lQqV+qb6tyIn19z8QhnuBMcIAwXGOeQZ7Q/3LZrl+vb6ty kO8FT5vv+gX+YZifgcHqcAO/oI+Nb459r5q/tv+8NIsxcLoAfcEQAAAeADUQAQAAAD0AAAA8MkIx ODRERkU5NzQ1Njc0NDkyNEFDRjU4OTg3RDk0MUQwMTkzNTMwMkBhcGwwMy5wb2xpLnVzcC5icj4A AAAAHgBHEAEAAAAPAAAAbWVzc2FnZS9yZmM4MjIAAAsA8hABAAAAHwDzEAEAAAA4AAAAUgBFAFMA JQAzAEEAIABbAG8AYgBtAC0AbABdACAAUABlAHMAYwBhAHIAaQBhAC4ARQBNAEwAAAALAPYQAAAA AEAABzD4uSVVv1zCAUAACDCyahEUwFzCAQMA3j+vbwAAAwDxPxYEAAAeAPg/AQAAABgAAABFZGls b24gUmliZWlybyBkYSBTaWx2YQACAfk/AQAAAFAAAAAAAAAA3KdAyMBCEBq0uQgAKy/hggEAAAAA AAAAL089RVBVU1AvT1U9RElSRVRPUklBL0NOPVJFQ0lQSUVOVFMvQ049RURJTE9OLlNJTFZBAB4A +j8BAAAAFQAAAFN5c3RlbSBBZG1pbmlzdHJhdG9yAAAAAAIB+z8BAAAAHgAAAAAAAADcp0DIwEIQ GrS5CAArL+GCAQAAAAAAAAAuAAAAAwD9P+QEAAADABlAAAAAAAMAGkAAAAAAAwAdQAAAAAADAB5A AAAAAB4AMEABAAAADQAAAEVESUxPTi5TSUxWQQAAAAAeADFAAQAAAA0AAABFRElMT04uU0lMVkEA AAAAHgAyQAEAAAAOAAAAcmRycWxAbXNuLmNvbQAAAB4AM0ABAAAADgAAAHJkcnFsQG1zbi5jb20A AAAeADhAAQAAAA0AAABFRElMT04uU0lMVkEAAAAAHgA5QAEAAAACAAAALgAAAAsAKQAAAAAACwAj AAAAAAADAAYQYn4GewMABxBYAgAAAwAQEAAAAAADABEQAAAAAB4ACBABAAAAZQAAAENPTU9QT0RF TUNBSVJOUEVJWEVTRU1DQURBUkVERUVTT0JSQVJFTVJFREVTVkFaSUFTP1NFQ0HtUkFNTlBFSVhF U0VNQ0FEQVJFREUs6VBPUlFVRVRPREFTQVNSRURFU0VTVOMAAAAAAgF/AAEAAAA9AAAAPDJCMTg0 REZFOTc0NTY3NDQ5MjRBQ0Y1ODk4N0Q5NDFEMDE5MzUzMDJAYXBsMDMucG9saS51c3AuYnI+AAAA AMyM ------_=_NextPart_001_01C25CC0.1407E16A-- ========================================================================= Instruções para entrar na lista, sair da lista e usar a lista em http://www.mat.puc-rio.br/~nicolau/olimp/obm-l.html O administrador desta lista é ========================================================================= From owner-obm-l@sucuri.mat.puc-rio.br Sun Sep 15 12:06:03 2002 Return-Path: Received: (from majordom@localhost) by sucuri.mat.puc-rio.br (8.9.3/8.9.3) id MAA13298 for obm-l-MTTP; Sun, 15 Sep 2002 12:04:49 -0300 Received: from hotmail.com (f64.sea2.hotmail.com [207.68.165.64]) by sucuri.mat.puc-rio.br (8.9.3/8.9.3) with ESMTP id MAA13294 for ; Sun, 15 Sep 2002 12:04:45 -0300 Received: from mail pickup service by hotmail.com with Microsoft SMTPSVC; Sun, 15 Sep 2002 08:06:01 -0700 Received: from 200.214.93.205 by sea2fd.sea2.hotmail.msn.com with HTTP; Sun, 15 Sep 2002 15:06:01 GMT X-Originating-IP: [200.214.93.205] From: "Paulo Santa Rita" To: obm-l@mat.puc-rio.br Subject: Re: [obm-l] Um Estranho Sentimento ... Date: Sun, 15 Sep 2002 15:06:01 +0000 Mime-Version: 1.0 Content-Type: text/plain; charset=iso-8859-1; format=flowed Message-ID: X-OriginalArrivalTime: 15 Sep 2002 15:06:01.0674 (UTC) FILETIME=[67A36AA0:01C25CC9] Sender: owner-obm-l@sucuri.mat.puc-rio.br Precedence: bulk Reply-To: obm-l@mat.puc-rio.br Ola FAJARDO e demais colegas desta lista ... OBM-L, Eu nao conheco o "Livro do Kunen", mas na compilacao a que lhe remeti quem fala DETALHADAMENTE do conceito de forcar ( Forcing ) e um dos desbravadores desta ideia, Paul Cohen. Muito provavelmente, um curso atual de divulgacao nao tera toda a riqueza de ideias contida em uma exposicao inicial ... >From: "Rogerio Fajardo" >Reply-To: obm-l@mat.puc-rio.br >To: obm-l@mat.puc-rio.br >Subject: Re: [obm-l] Um Estranho Sentimento ... >Date: Fri, 13 Sep 2002 21:12:40 +0000 > >Olá, Paulo > > Quanto ao livro de lógica, ainda não tive tempo de estudá-lo, mas tenho >participado de alguns seminários sobre forcing usando o livro de Kunen. Em minha modesta investigacao pessoal eu me fiz estas perguntas abaixo, inspirado sobretudo pela memoria original do Gauss sobre a Geometria intrinseca das Superficies ... Parece que os Axiomas do Corpo Ordenado Completo surgirao MUITO MAIS COMO UMA CONSTRUCAO para justificar os fatos numericos que ja sabiamos, assim como a Geometria Euclidiana surgiu como uma formalizacao dos fatos geometricos que os povos de entao ja usavam ... E portanto so novos fatos e fenomenos, talvez de coerencia interna, nos permitam olhar os axiomas do corpo como UMA FORMALIZACAO possivel, nao como A FORMALIZACAO POSSIVEL ... Me parece que os fatos ... 1) Dois Corpos Ordenados Completos sao necessariamente isomorfos 2) Em todo Corpo Ordenado Completo existe uma "copia" do Naturais e de sua Aritmetica 3) Existe mais de uma "construcao" dos numeros reais Sao suficientes para suspeitarmos de sua incompletude ... Estas Ideias Estranhas chegaram em virtude da reflexao que venho fazendo sobre uma abordagem diferente do problema 3N+1, depois de ler um artigo do Coway sobre o assunto. > Parece que agora ficou mais claro para mim o que voce pensou. A respeito >disso proponho a seguinte pergunta: se alguém com uma brilhante aptidão >matemática, mas que nunca tinha ouvido falar de números reais nem de >geometria, aprende os axiomas do corpo ordenado completo, sem nenhuma >interpretação geométrica, que visão ele teria dos números reais? Será que, >com o tempo, ele ia perceber por si só essa interpretação geométrica? Será >que ia obter uma outra visualização? >Será que, nessa visualização, iria enxergar coisas que nós não enxergamos? >E por aí vai... Concordo plenamente com o fragmento abaixo e considero-o uma observacao ( e exposicao ) muito boa, tipicamente do estudante que ja atingiu um grau avancado de compreensao e maturidade : salvo melhor juizo, me parece ser um passo necessario para se viver e praticar a Alta Matematica ... > Mas o fato de atribuirmos a certas estruturas matemáticas novas >visualizações que nos permitem enxergar coisas que não enxergávamos, e que >ajude a associarmos coisas aparentemente totalmente distintas, já ocorre >com frequência na matemática e é bem interessante. Por exemplo, o fato de >pensarmos em funções como elementos de espaço vetorial e utilizarmos >produto interno dado pela integral (como fazemos na Análise Funcional), nos >ajudou a descobrirmos um monte de resultados importantíssimos em equações >diferenciais. No fundo, será que isso não é o que voce falou sobre achar >uma nova visualização de uma teoria? Quem imaginava, antes de se criar a >Álgebra Linear abstrata, que as funções podiam ser vistas como vetores, >inclusive com a noção de ortogonalidade, e que isso fosse tão útil? O que >fizemos não foi usar a geometria para visualizarmos algo que até então >ninguém pensava haver qualquer relação com geometria? Acho isso um dos >fenômenos mais maravilhosos da matemática. Voce cria definições abstratas >que generalizam conceitos mais concretos e, apesar das dificuldades >iniciais geradas pelo excesso de abstração, isso acaba facilitando nosso >raciocínio, a posteriori, fazendo-nos associarmos fatos aparentemente >independentes e usarmos resultados obtidos para um fim, em outro fim >totalmente diferente, além de que, muitas vezes, é mais fácil enxergarmos >coisas particulares quando estamos no geral (algo parecido com o que Polya >chama de "paradoxo da invenção"). A topologia generaliza conceito de >vizinhanças que herdamos da reta real, para que seja aplicada para diversos >fins. Mas depois que a aprendemos, fica mais fácil descobrirmos coisas >relativas à própria reta real e "limpar" o nosso raciocínio de informações >irrelevantes para um determinado problema (por exemplo, ao invés de >considerarmos aquela definição por epsilons e deltas de continuidade de >função, usarmos que a imagem inversa de abertos é aberta pode ser mais >útil, claro e "limpo"). Incrivel, nao ? Pessoas distintas, em locais distintos, comecarem a perceber a mesma ideia ! E como se ela estivesse no ar ... E so fechar os olhos e olhar serenamente para o Universo que se nos apresenta ... Ei-la ! Ai, valeu maluco ! Foi muito estimulante para mim a sua mensagem. Obrigado ! > Com relação a essas coisas tenho pensado bastante, e creio que está >relacionado com seu "estranho sentimento". De qualquer forma, essa é uma >das coisas que mais me admira na Matemática. Com os melhores votos de Paz Profunda, sou Paulo Santa Rita 1,1202,150902 >>From: "Paulo Santa Rita" >>Reply-To: obm-l@mat.puc-rio.br >>To: obm-l@mat.puc-rio.br >>Subject: Re: [obm-l] Um Estranho Sentimento ... >>Date: Wed, 11 Sep 2002 15:24:22 +0000 >> >>Ola ROGERIO FAJARDO e demais >>colegas desta lista ... OBM-L, >> >>E entao Fajardo, tudo legal ? >>Conseguiu o Livro de Logica-Matematica ? >> >>Eu sei que foram fatos geometricos e outros fenomenos cotidianos (divida >>-> numero negativo, divisao de um objeto -> fracao, etc etc ) que nos >>levaram a descoberta das diversas classes de numeros, "construidos" >>posteriormente e hoje apresentados com o auxilio da abstracao matematica >>... >> >>Colocar estes numeros em uma reta, porem, e uma construcao humana gratuita >>... nao ha nenhum razao forte para tanto e os axiomas de um corpo ordenado >>completo nao induzem, a priori, a nenhuma topologia particular ... O que >>estes axiomas podem falar sobre disposicao ou configuracao ? Isso : Nada ! >>Nos poderiamos pensar sobre eles com igual correcao se os visualizassemos >>sobre um ramo de parabola, por exemplo. ME PARECE, salvo melhor juizo, que >>a unica exigencia que podemos fazer sobre uma possivel representacao e a >>de continuidade ... E a continuidade, conforme todos nos sabemos, nao e >>uma propriedade metrica. >> >>Bom, sendo assim, respeitados os axiomas de um corpo ordenado completo, >>nos podemos pensar nos numeros reais como estando disposto de outra forma, >>desde que esta estratificacao preserve a continuidade ... A questao e : e >>vantajoso fazer isso ? e util ? Com esta imagem nos conseguiremos resolver >>ou esclarecer algum fato que ainda nao foi resolvido ou esclarecido ? So >>assim um mudanca ou inovacao e justificavel ... >> >>Nao sei se consegui ser claro, mas percebi que voce pensou seriamente >>sobre a minha mensagem e nao supos que eu seja tao simplorio que nao >>perceba sobre a gravidade e implicacoes do que estou falando ... >> >>Um abraco >>Paulo Santa Rita >>4,1223,110902 >> >> >> >> >>>From: "Rogerio Fajardo" >>>Reply-To: obm-l@mat.puc-rio.br >>>To: obm-l@mat.puc-rio.br >>>Subject: Re: [obm-l] Um Estranho Sentimento ... >>>Date: Wed, 11 Sep 2002 00:32:33 +0000 >>> >>>Não compreendi bem o que voce quer dizer, mas me interessei por seu >>>comentário. Pelo que entendi, voce quer saber se existe outra forma de >>>visualizar, intuitivamente, os números, de forma a enxergar propriedades >>>que são difíceis de enxergar com a visualização com as quais estamos >>>acostumados. É isso ou nada a ver? >>> >>>O que eu percebo é que não é bem a geometria que serve para nos dar uma >>>intuição dos números reais, mas os números reais surgiram para descrever >>>a geometria de forma precisa. Não sei se isso tem algo a ver com seu >>>e-mail. Detalhe-me mais o seu pensamento. >>> >>> >>>>From: "Paulo Santa Rita" >>>>Reply-To: obm-l@mat.puc-rio.br >>>>To: obm-l@mat.puc-rio.br >>>>Subject: [obm-l] Um Estranho Sentimento ... >>>>Date: Mon, 09 Sep 2002 21:19:23 +0000 >>>> >>>>Ola Pessoal e demais >>>>colegas desta lista ... OBM-L, >>>> >>>>Eu tenho pensado continua e longamente em um conjunto de questoes >>>>relacionadas que me levam, invariavelmente, a uma mesma direcao que nao >>>>estou conseguindo acreditar ... ate parece que depois de tanto refletir >>>>cheguei a alguma constatacao insana ou simploria demais ... Se algum >>>>colega puder falar algo esclarecedor e/ou interessante eu ficarei muito >>>>grato ! >>>> >>>>Desde a infancia somos instados a pensar que os numeros reais estao >>>>dispostos ao longo de uma linha reta. Nos dizemos : 3 < 5 ! E >>>>imediatamente visualizamos o 3 a esquerda do 5, ambos em uma linha reta >>>>! Por que nos pensamos assim ? >>>> >>>>E verdade que em cursos de analise os livros definem R como um corpo >>>>ordenado completo e derivam as propriedades dos numeros reais dos >>>>axiomas que definem esta estrutura, sem recorrer a qualquer propriedade >>>>geometrica derivada de uma eventual visualizacao dos numeros sobre uma >>>>reta ... >>>> >>>>Mas se, por um lado, os axiomas de um corpo ordenado completo nao >>>>implicam ou requerem explicitamente uma estrutura geometrica conhecida, >>>>e inegavel que a visualizacao "informal" que fazemos facilita muitos >>>>raciocinio ... SERIA REALMENTE IMPOSSIVEL ASSOCIAR A UM CORPO ORDENADO >>>>COMPLETO UMA GEOMETRIA, ATRAVES DE AXIOMAS OU OUTROS RECURSOS, DE FORMA >>>>QUE PUDESSEMOS TER OUTRAS VISUALIZACOES, MESMO QUE ESTRANHAS, POREM, >>>>CONMSISTENTES ? NAO PODERIAM ALGUMAS PROPRIEDADES NUMERICAS DEPENDEREM >>>>INEXORAVELMENTE DE UMA TAL GEOMETRIA ? >>>> >>>>E bem provavel que eu esteja errado, mas nao consigo perceber o meu erro >>>>... EU ACHO QUE CERTOS MAPEAMENTOS NUMERICOS REQUEREM OU IMPLICAM QUE >>>>OS NUMEROS NATURAIS TEM UMA GEOMETRIA OU ESTRUTURA INTRINSECA, SEM A >>>>QUAL NAO DA PRA COMPREENDER CERTAS COISAS ... E NECESSARIO OU POSTULAR >>>>UMA DISPOSICAO ESTRATIGRAFICA OU SUPOR QUE CERTOS MAPEAMENTOS INDUZEM >>>>UMA TAL ESTRATIFICACAO ... >>>> >>>>Bom, se alguem puder falar alguma coisa interessante sobre este tema eu >>>>fico muito grato, pois este e realmente um SENTIMENTO ESTRANHO que me >>>>tem ocorrido com alguma frequencia. Pode ser uma burrice momentanea que >>>>esta me levando a estas perguntas aparentemente idiotas e sem sentido, >>>>mas eu nao iria ocupar o tempo de voces, meus amigos, se nao tivesse >>>>razoes seria pra fazer isso ... >>>> >>>>Um abraco a Todos >>>>Paulo Santa Rita >>>>2,1818,090902 >>>> >>>> >>>>_________________________________________________________________ >>>>Converse com seus amigos online, faça o download grátis do MSN >>>>Messenger: http://messenger.msn.com.br >>>> >>>>========================================================================= >>>>Instruções para entrar na lista, sair da lista e usar a lista em >>>>http://www.mat.puc-rio.br/~nicolau/olimp/obm-l.html >>>>O administrador desta lista é >>>>========================================================================= >>> >>> >>> >>> >>>_________________________________________________________________ >>>Send and receive Hotmail on your mobile device: http://mobile.msn.com >>> >>>========================================================================= >>>Instruções para entrar na lista, sair da lista e usar a lista em >>>http://www.mat.puc-rio.br/~nicolau/olimp/obm-l.html >>>O administrador desta lista é >>>========================================================================= >> >> >> >> >>_________________________________________________________________ >>Converse com seus amigos online, faça o download grátis do MSN Messenger: >>http://messenger.msn.com.br >> >>========================================================================= >>Instruções para entrar na lista, sair da lista e usar a lista em >>http://www.mat.puc-rio.br/~nicolau/olimp/obm-l.html >>O administrador desta lista é >>========================================================================= > > > > >_________________________________________________________________ >Chat with friends online, try MSN Messenger: http://messenger.msn.com > >========================================================================= >Instruções para entrar na lista, sair da lista e usar a lista em >http://www.mat.puc-rio.br/~nicolau/olimp/obm-l.html >O administrador desta lista é >========================================================================= ========================================================================= Instruções para entrar na lista, sair da lista e usar a lista em http://www.mat.puc-rio.br/~nicolau/olimp/obm-l.html O administrador desta lista é ========================================================================= From owner-obm-l@sucuri.mat.puc-rio.br Sun Sep 15 12:35:08 2002 Return-Path: Received: (from majordom@localhost) by sucuri.mat.puc-rio.br (8.9.3/8.9.3) id MAA13568 for obm-l-MTTP; Sun, 15 Sep 2002 12:34:46 -0300 Received: from mediterraneo.rjnet.com.br (mediterraneo.rjnet.com.br [200.222.31.30]) by sucuri.mat.puc-rio.br (8.9.3/8.9.3) with ESMTP id MAA13564 for ; Sun, 15 Sep 2002 12:34:43 -0300 Received: from locutus.rjnet.com.br (root@locutus.rjnet.com.br [200.222.31.10]) by mediterraneo.rjnet.com.br (8.11.4/8.11.4) with ESMTP id g8FFaMC07548 for ; Sun, 15 Sep 2002 12:36:22 -0300 Received: from felipe ([200.222.39.86]) by locutus.rjnet.com.br (8.11.2/8.11.2) with SMTP id g8FFG8216547 for ; Sun, 15 Sep 2002 12:16:08 -0300 Message-ID: <001901c25ccd$94f68470$158c000a@felipe> From: "Felipe Villela Dias" To: References: <003c01c25c5f$bacb4f50$158c000a@felipe> <000f01c25c6c$f2736f70$019da8c0@henrique> Subject: [obm-l] =?iso-8859-1?Q?Re:_=5Bobm-l=5D_Re:_=5Bobm-l=5D_=5Boff=5D_uma_d=FAvida_de_?= =?iso-8859-1?Q?F=EDsica.?= Date: Sun, 15 Sep 2002 12:35:53 -0300 MIME-Version: 1.0 Content-Type: multipart/alternative; boundary="----=_NextPart_000_0011_01C25CB4.6E60C6E0" X-Priority: 3 X-MSMail-Priority: Normal X-Mailer: Microsoft Outlook Express 6.00.2600.0000 X-MimeOLE: Produced By Microsoft MimeOLE V6.00.2600.0000 Sender: owner-obm-l@sucuri.mat.puc-rio.br Precedence: bulk Reply-To: obm-l@mat.puc-rio.br This is a multi-part message in MIME format. ------=_NextPart_000_0011_01C25CB4.6E60C6E0 Content-Type: text/plain; charset="iso-8859-1" Content-Transfer-Encoding: quoted-printable =C9 exatamente por isso que eu estou perguntando. Eu conhe=E7o o = experimento. A minha d=FAvida =E9 exatamente essa, Eistein conhecendo o = experimento, assumiu que a velocidade da luz =E9 constante e deduziu = todo o resto? Ou ele, conhecendo o experimento conseguiu demonstrar que = a velocidade da luz =E9 constante? ----- Original Message -----=20 From: Henrique Branco=20 To: obm-l@mat.puc-rio.br=20 Sent: Sunday, September 15, 2002 1:04 AM Subject: [obm-l] Re: [obm-l] [off] uma d=FAvida de F=EDsica. Olha, n=E3o conhe=E7o a forma matem=E1tica pra tal postulado (sim, =E9 = um postulado f=EDsico). Mas acho que ele foi baseado unicamente no experimento = Michelson-Morley. Se quiser mais detalhes (caso voc=EA n=E3o conhe=E7a o experimento), = escreva de volta que eu explico direitinho. Abra=E7os, Henrique. --- Outgoing mail is certified Virus Free. Checked by AVG anti-virus system (http://www.grisoft.com). Version: 6.0.386 / Virus Database: 218 - Release Date: 9/9/2002 ------=_NextPart_000_0011_01C25CB4.6E60C6E0 Content-Type: text/html; charset="iso-8859-1" Content-Transfer-Encoding: quoted-printable
=C9 exatamente por isso que eu estou perguntando. Eu conhe=E7o o = experimento. A=20 minha d=FAvida =E9 exatamente essa, Eistein conhecendo o experimento, = assumiu que a=20 velocidade da luz =E9 constante e deduziu todo o resto? Ou ele, = conhecendo o=20 experimento conseguiu demonstrar que a velocidade da luz =E9 = constante?
----- Original Message -----
From:=20 Henrique Branco
Sent: Sunday, September 15, = 2002 1:04=20 AM
Subject: [obm-l] Re: [obm-l] = [off] uma=20 d=FAvida de F=EDsica.

Olha, n=E3o conhe=E7o a forma = matem=E1tica pra tal=20 postulado (sim, =E9 um postulado f=EDsico).
Mas acho que ele foi baseado = unicamente no=20 experimento Michelson-Morley.
Se quiser mais detalhes (caso voc=EA = n=E3o conhe=E7a o=20 experimento), escreva de volta que eu explico direitinho.
 
Abra=E7os,
Henrique.
 

---
Outgoing mail is certified = Virus=20 Free.
Checked by AVG anti-virus system (http://www.grisoft.com).
Version: = 6.0.386=20 / Virus Database: 218 - Release Date:=20 9/9/2002
------=_NextPart_000_0011_01C25CB4.6E60C6E0-- ========================================================================= Instruções para entrar na lista, sair da lista e usar a lista em http://www.mat.puc-rio.br/~nicolau/olimp/obm-l.html O administrador desta lista é ========================================================================= From owner-obm-l@sucuri.mat.puc-rio.br Sun Sep 15 13:33:09 2002 Return-Path: Received: (from majordom@localhost) by sucuri.mat.puc-rio.br (8.9.3/8.9.3) id NAA14718 for obm-l-MTTP; Sun, 15 Sep 2002 13:30:59 -0300 Received: from traven.pub1 (traven.uol.com.br [200.221.4.39]) by sucuri.mat.puc-rio.br (8.9.3/8.9.3) with ESMTP id NAA14714 for ; Sun, 15 Sep 2002 13:30:57 -0300 Received: from u2z7z2 ([200.158.145.78]) by traven.pub1 (8.9.1/8.9.1) with ESMTP id NAA21851 for ; Sun, 15 Sep 2002 13:19:27 -0300 (BRT) Message-ID: <003901c25cd5$9d79f480$2101a8c0@u2z7z2> From: "Wagner" To: Subject: [obm-l] O problema do ano Date: Sun, 15 Sep 2002 13:33:25 -0300 Organization: Wagner MIME-Version: 1.0 Content-Type: multipart/alternative; boundary="----=_NextPart_000_0036_01C25CBC.77C98DE0" X-Priority: 3 X-MSMail-Priority: Normal X-Mailer: Microsoft Outlook Express 5.50.4133.2400 X-MIMEOLE: Produced By Microsoft MimeOLE V5.50.4133.2400 Sender: owner-obm-l@sucuri.mat.puc-rio.br Precedence: bulk Reply-To: obm-l@mat.puc-rio.br This is a multi-part message in MIME format. ------=_NextPart_000_0036_01C25CBC.77C98DE0 Content-Type: text/plain; charset="iso-8859-1" Content-Transfer-Encoding: quoted-printable Ola pessoal! Quem conseguir resolver esse problema realmente merece o t=EDtulo de = g=EAnio do ano: -Nota=E7=E3o: a^b=3D a elevado a b Seja o polinomio p(x) =3D x^(2n) + x^(2n-2) + x^(2n-4) + ... + x^2 + 1 - = (2x^n)(x^(n-1) - x^(n-2) + x^(n-3) - x^(n-4) + ... + x - 1) - = (2x^(n-1))(x^(n-2) - x^(n-3) + ... + 1) - (2x^(n-2))(x^(n-3) - x^(n-4) + = ... - 1) - (2x^(n-3))(...) + ... - 2x( 1 ). Prove que se ((n-1)/2) =E9 um n=FAmero natural, p(x) possui exatamente 2 = ra=EDzes reais e pelo menos 2 ra=EDzes coincidentes. Prove tamb=E9m que = todos os coeficientes do polin=F4mio seguem um padr=E3o para esses = valores de n. Andr=E9 T. ------=_NextPart_000_0036_01C25CBC.77C98DE0 Content-Type: text/html; charset="iso-8859-1" Content-Transfer-Encoding: quoted-printable
Ola pessoal!
 
Quem conseguir resolver esse problema = realmente=20 merece o t=EDtulo de g=EAnio do ano:
 
-Nota=E7=E3o: a^b=3D a elevado a = b
 
Seja o polinomio p(x) =3D x^(2n) + = x^(2n-2) +=20 x^(2n-4) + ... + x^2 + 1 - (2x^n)(x^(n-1) - x^(n-2) + x^(n-3) - x^(n-4) = + ... +=20 x - 1) - (2x^(n-1))(x^(n-2) - x^(n-3) + ... + 1) - (2x^(n-2))(x^(n-3) - = x^(n-4)=20 + ... - 1) - (2x^(n-3))(...) + ... - 2x( 1 ).
 
Prove que se ((n-1)/2) =E9 um n=FAmero = natural, p(x)=20 possui exatamente 2 ra=EDzes reais e pelo menos 2 ra=EDzes coincidentes. = Prove=20 tamb=E9m que todos os coeficientes do polin=F4mio seguem um padr=E3o = para esses=20 valores de n.
 
 
Andr=E9 T.
------=_NextPart_000_0036_01C25CBC.77C98DE0-- ========================================================================= Instruções para entrar na lista, sair da lista e usar a lista em http://www.mat.puc-rio.br/~nicolau/olimp/obm-l.html O administrador desta lista é ========================================================================= From owner-obm-l@sucuri.mat.puc-rio.br Sun Sep 15 15:28:18 2002 Return-Path: Received: (from majordom@localhost) by sucuri.mat.puc-rio.br (8.9.3/8.9.3) id PAA15730 for obm-l-MTTP; Sun, 15 Sep 2002 15:26:25 -0300 Received: from traven.pub1 (traven.uol.com.br [200.221.4.39]) by sucuri.mat.puc-rio.br (8.9.3/8.9.3) with ESMTP id PAA15726 for ; Sun, 15 Sep 2002 15:26:22 -0300 Received: from u2z7z2 ([200.158.144.68]) by traven.pub1 (8.9.1/8.9.1) with ESMTP id PAA04168 for ; Sun, 15 Sep 2002 15:14:52 -0300 (BRT) Message-ID: <001a01c25ce5$bde40a20$44909ec8@u2z7z2> From: "Wagner" To: Subject: [obm-l] Incompletudes de Boltman Date: Sun, 15 Sep 2002 15:28:51 -0300 Organization: Wagner MIME-Version: 1.0 Content-Type: multipart/alternative; boundary="----=_NextPart_000_0017_01C25CCC.98647780" X-Priority: 3 X-MSMail-Priority: Normal X-Mailer: Microsoft Outlook Express 5.50.4133.2400 X-MimeOLE: Produced By Microsoft MimeOLE V5.50.4133.2400 Sender: owner-obm-l@sucuri.mat.puc-rio.br Precedence: bulk Reply-To: obm-l@mat.puc-rio.br This is a multi-part message in MIME format. ------=_NextPart_000_0017_01C25CCC.98647780 Content-Type: text/plain; charset="iso-8859-1" Content-Transfer-Encoding: quoted-printable Oi para todos! Eu queria saber porque o Teorema das Incompletudes de Boltman =E9 = verdadeiro O teorema =E9 o seguinte: Em um conjunto de axiomas sempre persistem = quest=F5es que n=E3o podem ser negadas, nem comprovadas por esses mesmos = axiomas. Andr=E9 T. ------=_NextPart_000_0017_01C25CCC.98647780 Content-Type: text/html; charset="iso-8859-1" Content-Transfer-Encoding: quoted-printable
Oi para todos!
 
Eu queria saber porque o = Teorema das=20 Incompletudes de Boltman =E9 verdadeiro
 
O teorema =E9 o seguinte: Em um = conjunto de axiomas=20 sempre persistem quest=F5es que n=E3o podem ser negadas, nem comprovadas = por esses=20 mesmos axiomas.
 
Andr=E9 T.
------=_NextPart_000_0017_01C25CCC.98647780-- ========================================================================= Instruções para entrar na lista, sair da lista e usar a lista em http://www.mat.puc-rio.br/~nicolau/olimp/obm-l.html O administrador desta lista é ========================================================================= From owner-obm-l@sucuri.mat.puc-rio.br Sun Sep 15 19:17:23 2002 Return-Path: Received: (from majordom@localhost) by sucuri.mat.puc-rio.br (8.9.3/8.9.3) id TAA17294 for obm-l-MTTP; Sun, 15 Sep 2002 19:15:12 -0300 Received: from hotmail.com (f135.sea2.hotmail.com [207.68.165.135]) by sucuri.mat.puc-rio.br (8.9.3/8.9.3) with ESMTP id TAA17290 for ; Sun, 15 Sep 2002 19:15:09 -0300 Received: from mail pickup service by hotmail.com with Microsoft SMTPSVC; Sun, 15 Sep 2002 15:16:26 -0700 Received: from 200.214.83.138 by sea2fd.sea2.hotmail.msn.com with HTTP; Sun, 15 Sep 2002 22:16:26 GMT X-Originating-IP: [200.214.83.138] From: "Paulo Santa Rita" To: obm-l@mat.puc-rio.br Subject: [obm-l] =?iso-8859-1?B?IFJlOiBbb2JtLWxdIHVtYSBk+nZpZGEgZGUgRu1zaWNhLg==?= Date: Sun, 15 Sep 2002 22:16:26 +0000 Mime-Version: 1.0 Content-Type: text/plain; charset=iso-8859-1; format=flowed Message-ID: X-OriginalArrivalTime: 15 Sep 2002 22:16:26.0686 (UTC) FILETIME=[888B85E0:01C25D05] Sender: owner-obm-l@sucuri.mat.puc-rio.br Precedence: bulk Reply-To: obm-l@mat.puc-rio.br Ola FELIPE e demais colegas desta lista ... OBM-L, Nem uma coisa e nem outra ... A velocidade da Luz ser constante, independente do movimento da fonte, e um postulado na Teoria da Relatividade, mas nao foi deste postulado que Einstein DEDUZIU TUDO. Esse resultado foi considerado um RESULTADO NEGATIVO pois esperava-se que o interferometro evidenciasse diferencas nas velocidades dos dois raios de luz, o que seria uma prova indireta da existencia do eter e, com isso, ficava facil explicar a propagacao das ondas eletromagneticas como uma pertubacao deste eter. Este RESULTADO NEGATIVO da experiencia de Michelson ficou sem uma explicacao plausivel, mas, por esta epoca, Lorentz deduziu suas equacoes para o fenomeno mas nao apresentou uma explicacao consistente, pois ele implicava em hipoteses consideradas entao pouco plausiveis e artificiais. O Genio de Einstein se evidenciou aqui. Ele aceitou o RESULTADO APARENTEMENTE NEGATIVO da experiencia de Michelson-Morley e mostrou que isto supunha um tempo nao absoluto, um tempo de cada abservador e que os conceitos de simultaneidade e correlatos deveriam ser modificados. Dai ele deduziu as equacoes de Lorentz como consequencias destes postulados. Foi como na gravitacao. Kepler e Galileu ja haviam descoberto muitas leis importantes e interessantes, mas eles nao atinaram COM A IDEIA CERTA. Newton fez isso, e mostrou que a sua hipotese nao so explicava ou deduzia as leis particulares de Galileu e Kepler como permitia explicar muitas outras coisas mais. As ideias de Newton, como a de Einstein, eram mais simples e mais gerais e permitiam deduzir novos fatos que poderiam ser submetidos ao crivo da experiemntacao, vale dizer, elas preservavam todas as conquistas anteriores, eram mais simples e fundamentais e alargavam a compreensao humana. E sempre assim com toda boa ideia ... Por ser sempre muito simples, e dificil atinar com ela mas, quando a descobrimos, a claridade e coerencia que ela da torna-a quase irrefutavel. Voce diz : tem que ser assim ... nao pode ser de outra forma ... por que eu nao vi isso antes ? A ideia genial e sempre simples e o poder de compreensao que ela fornece e um misto de beleza e clareza que quase se impoe a nossa inteligencia. Voltando a relatividade, as equacoes de Lorentz estao para a relatividade assim como as leis de Kepler estao para a gravitacao Newtoniana. A constancia da luz SE TORNOU um postulado porque foi verificado experimentalmente mas tambem e sobretudo porque de sua adocao foi possivel explicar todos so fatos controversos de entao. A relatividade restrita nao e matematicamente complicada. Existem muitos bons livros. Eu indico dois : Introducao a relatividade restrita David Resnick, LTC Introduction to special relativity Wolfgang Rindler Oxford university Press O segundo e mais completo que o primeiro . Ambos exigem uma Matematica elementar. Um abraco Paulo Santa Rita 1,1911,150902 >From: "Felipe Villela Dias" >Reply-To: obm-l@mat.puc-rio.br >To: >Subject: [obm-l] Re: [obm-l] Re: [obm-l] [off] uma dúvida de Física. >Date: Sun, 15 Sep 2002 12:35:53 -0300 > >É exatamente por isso que eu estou perguntando. Eu conheço o experimento. A >minha dúvida é exatamente essa, Eistein conhecendo o experimento, assumiu >que a velocidade da luz é constante e deduziu todo o resto? Ou ele, >conhecendo o experimento conseguiu demonstrar que a velocidade da luz é >constante? > ----- Original Message ----- > From: Henrique Branco > To: obm-l@mat.puc-rio.br > Sent: Sunday, September 15, 2002 1:04 AM > Subject: [obm-l] Re: [obm-l] [off] uma dúvida de Física. > > > Olha, não conheço a forma matemática pra tal postulado (sim, é um >postulado físico). > Mas acho que ele foi baseado unicamente no experimento Michelson-Morley. > Se quiser mais detalhes (caso você não conheça o experimento), escreva >de volta que eu explico direitinho. > > Abraços, > Henrique. > > > --- > Outgoing mail is certified Virus Free. > Checked by AVG anti-virus system (http://www.grisoft.com). > Version: 6.0.386 / Virus Database: 218 - Release Date: 9/9/2002 _________________________________________________________________ MSN Photos é a maneira mais fácil e prática de editar e compartilhar sua fotos: http://photos.msn.com.br ========================================================================= Instruções para entrar na lista, sair da lista e usar a lista em http://www.mat.puc-rio.br/~nicolau/olimp/obm-l.html O administrador desta lista é ========================================================================= From owner-obm-l@sucuri.mat.puc-rio.br Sun Sep 15 20:37:23 2002 Return-Path: Received: (from majordom@localhost) by sucuri.mat.puc-rio.br (8.9.3/8.9.3) id UAA18277 for obm-l-MTTP; Sun, 15 Sep 2002 20:36:35 -0300 Received: from siegel.bol.com.br (siegel.bol.com.br [200.221.24.20]) by sucuri.mat.puc-rio.br (8.9.3/8.9.3) with ESMTP id UAA18273 for ; Sun, 15 Sep 2002 20:36:33 -0300 Received: from comp1 (200.221.24.191) by siegel.bol.com.br (5.1.071) id 3D5D1C9F00A31665 for obm-l@mat.puc-rio.br; Sun, 15 Sep 2002 20:37:43 -0300 Message-ID: <000701c25d3d$a330ae60$efced5c8@comp1> From: "Caio Augusto" To: References: Subject: [obm-l] =?iso-8859-1?Q?Re:_=5Bobm-l=5D__Re:_=5Bobm-l=5D_uma_d=FAvida_de_F=EDsica.?= Date: Sun, 15 Sep 2002 20:58:02 -0800 MIME-Version: 1.0 Content-Type: text/plain; charset="iso-8859-1" Content-Transfer-Encoding: 8bit X-Priority: 3 X-MSMail-Priority: Normal X-Mailer: Microsoft Outlook Express 5.50.4522.1200 X-MimeOLE: Produced By Microsoft MimeOLE V5.50.4522.1200 X-Sender-IP: 200.213.206.239 Sender: owner-obm-l@sucuri.mat.puc-rio.br Precedence: bulk Reply-To: obm-l@mat.puc-rio.br Ainda sobre esta questão, como ainda não entendo tão bem de eletromagnetismo, gostaria de saber se não é verdade que se pode deduzir a constancia da velocidade da luz no vacuo através das Equações de Maxwell, porque eu já ouvi dizer isso. Caio Augusto ========================================================================= Instruções para entrar na lista, sair da lista e usar a lista em http://www.mat.puc-rio.br/~nicolau/olimp/obm-l.html O administrador desta lista é ========================================================================= From owner-obm-l@sucuri.mat.puc-rio.br Sun Sep 15 22:06:19 2002 Return-Path: Received: (from majordom@localhost) by sucuri.mat.puc-rio.br (8.9.3/8.9.3) id WAA19158 for obm-l-MTTP; Sun, 15 Sep 2002 22:03:50 -0300 Received: from mediterraneo.rjnet.com.br (mediterraneo.rjnet.com.br [200.222.31.30]) by sucuri.mat.puc-rio.br (8.9.3/8.9.3) with ESMTP id WAA19154 for ; Sun, 15 Sep 2002 22:03:47 -0300 Received: from locutus.rjnet.com.br (root@locutus.rjnet.com.br [200.222.31.10]) by mediterraneo.rjnet.com.br (8.11.4/8.11.4) with ESMTP id g8G15QC22762 for ; Sun, 15 Sep 2002 22:05:26 -0300 Received: from felipe ([200.222.39.86]) by locutus.rjnet.com.br (8.11.2/8.11.2) with SMTP id g8G0jA217103 for ; Sun, 15 Sep 2002 21:45:10 -0300 Message-ID: <000701c25d1d$14771b20$158c000a@felipe> From: "Felipe Villela Dias" To: References: <000701c25d3d$a330ae60$efced5c8@comp1> Subject: [obm-l] =?iso-8859-1?Q?Re:_=5Bobm-l=5D_Re:_=5Bobm-l=5D__Re:_=5Bobm-l=5D_uma?= =?iso-8859-1?Q?_d=FAvida_de_F=EDsica.?= Date: Sun, 15 Sep 2002 22:04:58 -0300 MIME-Version: 1.0 Content-Type: text/plain; charset="iso-8859-1" Content-Transfer-Encoding: 8bit X-Priority: 3 X-MSMail-Priority: Normal X-Mailer: Microsoft Outlook Express 6.00.2600.0000 X-MimeOLE: Produced By Microsoft MimeOLE V6.00.2600.0000 Sender: owner-obm-l@sucuri.mat.puc-rio.br Precedence: bulk Reply-To: obm-l@mat.puc-rio.br Podem sim, a demonstração que eu tenho é um pouco grande, mas procura em qualquer livro de Física IV que você encontra. ( O Halliday e Resnick com certeza tem). Foi mais ou menos assim que descobriram que a luz era uma onda eletromagnética. Já se conhecia aproximadamente a velocidade da luz, e viram que era a mesma de uma onda eletromagnética. ----- Original Message ----- From: "Caio Augusto" To: Sent: Monday, September 16, 2002 1:58 AM Subject: [obm-l] Re: [obm-l] Re: [obm-l] uma dúvida de Física. > Ainda sobre esta questão, como ainda não entendo tão bem de > eletromagnetismo, gostaria de saber se não é verdade que se pode deduzir a > constancia da velocidade da luz no vacuo através das Equações de Maxwell, > porque eu já ouvi dizer isso. > > Caio Augusto > > ========================================================================= > Instruções para entrar na lista, sair da lista e usar a lista em > http://www.mat.puc-rio.br/~nicolau/olimp/obm-l.html > O administrador desta lista é > ========================================================================= > http://www.derrubarm2a.cjb.net/ --- Outgoing mail is certified Virus Free. Checked by AVG anti-virus system (http://www.grisoft.com). Version: 6.0.386 / Virus Database: 218 - Release Date: 9/9/2002 ========================================================================= Instruções para entrar na lista, sair da lista e usar a lista em http://www.mat.puc-rio.br/~nicolau/olimp/obm-l.html O administrador desta lista é ========================================================================= From owner-obm-l@sucuri.mat.puc-rio.br Sun Sep 15 23:40:21 2002 Return-Path: Received: (from majordom@localhost) by sucuri.mat.puc-rio.br (8.9.3/8.9.3) id XAA20161 for obm-l-MTTP; Sun, 15 Sep 2002 23:39:27 -0300 Received: from smtp-4.ig.com.br (smtp-4.ig.com.br [200.226.132.153]) by sucuri.mat.puc-rio.br (8.9.3/8.9.3) with SMTP id XAA20157 for ; Sun, 15 Sep 2002 23:39:25 -0300 From: ezer@ig.com.br Received: (qmail 8872 invoked from network); 16 Sep 2002 02:40:22 -0000 Received: from shasta064199.ig.com.br (HELO almirfam) (200.151.64.199) by smtp-4.ig.com.br with SMTP; 16 Sep 2002 02:40:22 -0000 To: obm-l@mat.puc-rio.br Date: Wed, 1 Jan 1997 00:48:12 -0200 MIME-Version: 1.0 Subject: =?ISO-8859-1?Q?Re:_[obm-l]__Re:_[obm-l]_uma_d=FAvida_de_F=EDsica.?= Message-ID: <32C9B44C.10876.2B584E@localhost> In-reply-to: X-mailer: Pegasus Mail for Windows (v4.01) Content-type: text/plain; charset=ISO-8859-1 Content-description: Mail message body Content-Transfer-Encoding: 8bit X-MIME-Autoconverted: from Quoted-printable to 8bit by sucuri.mat.puc-rio.br id XAA20158 Sender: owner-obm-l@sucuri.mat.puc-rio.br Precedence: bulk Reply-To: obm-l@mat.puc-rio.br Sim, mas como pode se afirmar com tal veracidade que a velocidade da luz é constante? O fato de ter sido uma ideia aceita mesmo sendo um resultado negativo (ou nao-esperado) nao comprova nada senao que a ciencia feita por Einstein tinha um grau de objetividade bastante saudavel. Se estamos numa busca pela realidade, devemos nos abster de classificacoes que tendam para o ambito da opiniao. Eh certo que toda a ciencia estah permeada de crencas e opinioes, mas o esforco por um julgamento racional eh sempre coroado por uma analise que, se nao eh a mais bonita ou extensa, estah pelo menos mais 'liberta' das crencas de sua epoca (ou seja, serah mais 'duravel'). Abracos, Ezer Fernandes On 15 Sep 2002 at 22:16, Paulo Santa Rita wrote: > Ola FELIPE e demais > colegas desta lista ... OBM-L, > > Nem uma coisa e nem outra ... A velocidade da Luz ser constante, > independente do movimento da fonte, e um postulado na Teoria da > Relatividade, mas nao foi deste postulado que Einstein DEDUZIU TUDO. > > Esse resultado foi considerado um RESULTADO NEGATIVO pois esperava-se que o > interferometro evidenciasse diferencas nas velocidades dos dois raios de > luz, o que seria uma prova indireta da existencia do eter e, com isso, > ficava facil explicar a propagacao das ondas eletromagneticas como uma > pertubacao deste eter. > > Este RESULTADO NEGATIVO da experiencia de Michelson ficou sem uma explicacao > plausivel, mas, por esta epoca, Lorentz deduziu suas equacoes para o > fenomeno mas nao apresentou uma explicacao consistente, pois ele implicava > em hipoteses consideradas entao pouco plausiveis e artificiais. > > O Genio de Einstein se evidenciou aqui. Ele aceitou o RESULTADO > APARENTEMENTE NEGATIVO da experiencia de Michelson-Morley e mostrou que isto > supunha um tempo nao absoluto, um tempo de cada abservador e que os > conceitos de simultaneidade e correlatos deveriam ser modificados. Dai ele > deduziu as equacoes de Lorentz como consequencias destes postulados. > > Foi como na gravitacao. Kepler e Galileu ja haviam descoberto muitas leis > importantes e interessantes, mas eles nao atinaram COM A IDEIA CERTA. > Newton fez isso, e mostrou que a sua hipotese nao so explicava ou deduzia as > leis particulares de Galileu e Kepler como permitia explicar muitas outras > coisas mais. As ideias de Newton, como a de Einstein, eram mais simples e > mais gerais e permitiam deduzir novos fatos que poderiam ser submetidos ao > crivo da experiemntacao, vale dizer, elas preservavam todas as conquistas > anteriores, eram mais simples e fundamentais e alargavam a compreensao > humana. ========================================================================= Instruções para entrar na lista, sair da lista e usar a lista em http://www.mat.puc-rio.br/~nicolau/olimp/obm-l.html O administrador desta lista é ========================================================================= From owner-obm-l@sucuri.mat.puc-rio.br Mon Sep 16 00:37:54 2002 Return-Path: Received: (from majordom@localhost) by sucuri.mat.puc-rio.br (8.9.3/8.9.3) id AAA21033 for obm-l-MTTP; Mon, 16 Sep 2002 00:37:15 -0300 Received: from videira.terra.com.br (videira.terra.com.br [200.176.3.5]) by sucuri.mat.puc-rio.br (8.9.3/8.9.3) with ESMTP id AAA21029 for ; Mon, 16 Sep 2002 00:37:13 -0300 Received: from smtp4-poa.terra.com.br (smtp4-poa.terra.com.br [200.176.3.35]) by videira.terra.com.br (Postfix) with ESMTP id 74DFBE1151 for ; Mon, 16 Sep 2002 00:38:31 -0300 (EST) Received: from xt (200-171-248-2.customer.telesp.net.br [200.171.248.2]) (authenticated user macwad) by smtp4-poa.terra.com.br (Postfix) with ESMTP id 04E7CAC5A0 for ; Mon, 16 Sep 2002 00:38:31 -0300 (EST) Message-ID: <004401c25d32$9233c710$0401010a@xt> From: =?iso-8859-1?Q?Vinicius_Jos=E9_Fortuna?= To: References: <32C9B44C.10876.2B584E@localhost> Subject: [obm-l] =?iso-8859-1?Q?Re:_=5Bobm-l=5D__Re:_=5Bobm-l=5D_uma_d=FAvida_de_F=EDsica.?= Date: Mon, 16 Sep 2002 00:38:49 -0300 MIME-Version: 1.0 Content-Type: text/plain; charset="iso-8859-1" Content-Transfer-Encoding: 8bit X-Priority: 3 X-MSMail-Priority: Normal X-Mailer: Microsoft Outlook Express 6.00.2600.0000 X-MimeOLE: Produced By Microsoft MimeOLE V6.00.2600.0000 Sender: owner-obm-l@sucuri.mat.puc-rio.br Precedence: bulk Reply-To: obm-l@mat.puc-rio.br ----- Original Message ----- From: To: Sent: Tuesday, December 31, 1996 11:48 PM Subject: Re: [obm-l] Re: [obm-l] uma dúvida de Física. > > Sim, mas como pode se afirmar com tal > veracidade que a velocidade da luz é constante? > O fato de ter sido uma ideia aceita mesmo sendo > um resultado negativo (ou nao-esperado) nao comprova > nada senao que a ciencia feita por Einstein tinha um > grau de objetividade bastante saudavel. > > Se estamos numa busca pela realidade, devemos nos > abster de classificacoes que tendam para o ambito > da opiniao. Eh certo que toda a ciencia estah permeada > de crencas e opinioes, mas o esforco por um julgamento > racional eh sempre coroado por uma analise que, se nao > eh a mais bonita ou extensa, estah pelo menos mais 'liberta' > das crencas de sua epoca (ou seja, serah mais 'duravel'). O perfeccionismo exagerado, refletido em extremo rigor, impõe barreiras de difícil transposição ao desenvolvimento do conhecimento humano. Deve-se deixar as idéias fluirem soltas para então aperfeiçoá-las. Assim sendo, não há nada de grave em "classificações que tendam para o âmbito da opinião", pois elas também nos aproximam da realidade. O próprio fato de terem aceito a velocidade da luz como constante levou ao entendimento melhor do fenômeno, a ponto de comprová-lo um tempo depois. Até mais Vinicius > Abracos, > > Ezer Fernandes ========================================================================= Instruções para entrar na lista, sair da lista e usar a lista em http://www.mat.puc-rio.br/~nicolau/olimp/obm-l.html O administrador desta lista é ========================================================================= From owner-obm-l@sucuri.mat.puc-rio.br Mon Sep 16 04:04:46 2002 Return-Path: Received: (from majordom@localhost) by sucuri.mat.puc-rio.br (8.9.3/8.9.3) id EAA22814 for obm-l-MTTP; Mon, 16 Sep 2002 04:04:07 -0300 Received: from smtp-3.ig.com.br (smtp-3.ig.com.br [200.226.132.152]) by sucuri.mat.puc-rio.br (8.9.3/8.9.3) with SMTP id EAA22810 for ; Mon, 16 Sep 2002 04:04:02 -0300 From: ezer@ig.com.br Received: (qmail 16257 invoked from network); 16 Sep 2002 07:05:02 -0000 Received: from shasta040254.ig.com.br (HELO almirfam) (200.151.40.254) by smtp-3.ig.com.br with SMTP; 16 Sep 2002 07:05:02 -0000 To: obm-l@mat.puc-rio.br Date: Wed, 1 Jan 1997 03:25:16 -0200 MIME-Version: 1.0 Subject: =?ISO-8859-1?Q?Re:_[obm-l]_Re:_[obm-l]__Re:_[obm-l]_uma_d=FAvida_de_F=EDsica.?= Message-ID: <32C9D91C.32348.213CF1@localhost> In-reply-to: <004401c25d32$9233c710$0401010a@xt> X-mailer: Pegasus Mail for Windows (v4.01) Content-type: text/plain; charset=ISO-8859-1 Content-description: Mail message body Content-Transfer-Encoding: 8bit X-MIME-Autoconverted: from Quoted-printable to 8bit by sucuri.mat.puc-rio.br id EAA22811 Sender: owner-obm-l@sucuri.mat.puc-rio.br Precedence: bulk Reply-To: obm-l@mat.puc-rio.br > O perfeccionismo exagerado, refletido em extremo rigor, impõe barreiras de > difícil transposição ao desenvolvimento do conhecimento humano. Deve-se > deixar as idéias fluirem soltas para então aperfeiçoá-las. Assim sendo, não > há nada de grave em "classificações que tendam para o âmbito da opinião", > pois elas também nos aproximam da realidade. > O próprio fato de terem aceito a velocidade da luz como constante levou ao > entendimento melhor do fenômeno, a ponto de comprová-lo um tempo depois. A questao do "criar" e do "classificar" ideias cientificas destoam um tanto nessa questao. Como voce disse, realmente devemos deixar que as ideias fluam soltas, sem impor-lhes as barreiras de uma critica exarcebada, para entao aperfeicoa-las. Porem, ademais o desenvolvimento de teorias, que necessitam muita criatividade do seu criador, a necessidade de uma capacidade critica 'livre' de nossos pre-conceitos, para uma busca real, uma busca para alem de nosso 'mundinho' conhecido, eh latente. Mas, diria-se: "Isso nao restringiria o nosso modo de pensar?" Pelo contrario! Quando nós nos arriscamos a nos aventurarmos para alem de nossas opinioes pessoais, estamos na verdade exercitando nossa capacidade criativa(!), pois existe uma relacao intensa entre a criatividade e a nossa capacidade de tolerar e trabalhar com ideias que nao sao nossas, e as vezes mesmo contrarias às nossas Abracos, Ezer > Até mais > > Vinicius ========================================================================= Instruções para entrar na lista, sair da lista e usar a lista em http://www.mat.puc-rio.br/~nicolau/olimp/obm-l.html O administrador desta lista é ========================================================================= From owner-obm-l@sucuri.mat.puc-rio.br Mon Sep 16 04:21:05 2002 Return-Path: Received: (from majordom@localhost) by sucuri.mat.puc-rio.br (8.9.3/8.9.3) id EAA22966 for obm-l-MTTP; Mon, 16 Sep 2002 04:20:42 -0300 Received: from smtp-3.ig.com.br (smtp-3.ig.com.br [200.226.132.152]) by sucuri.mat.puc-rio.br (8.9.3/8.9.3) with SMTP id EAA22962 for ; Mon, 16 Sep 2002 04:20:39 -0300 From: ezer@ig.com.br Received: (qmail 11497 invoked from network); 16 Sep 2002 07:21:41 -0000 Received: from shasta040254.ig.com.br (HELO almirfam) (200.151.40.254) by smtp-3.ig.com.br with SMTP; 16 Sep 2002 07:21:41 -0000 To: obm-l@mat.puc-rio.br Date: Wed, 1 Jan 1997 03:41:56 -0200 MIME-Version: 1.0 Subject: [obm-l] Piadinhas Infames.. Message-ID: <32C9DD04.555.307F4E@localhost> X-mailer: Pegasus Mail for Windows (v4.01) Content-type: text/plain; charset=ISO-8859-1 Content-description: Mail message body Content-Transfer-Encoding: 8bit X-MIME-Autoconverted: from Quoted-printable to 8bit by sucuri.mat.puc-rio.br id EAA22963 Sender: owner-obm-l@sucuri.mat.puc-rio.br Precedence: bulk Reply-To: obm-l@mat.puc-rio.br Ola Colegas da Lista, Estou lhes enviando algumas frasezinhas grudentas encontradas num site... "A vida eh complexa. Composta de partes imaginárias e reais." (fonte: Tpotter@voyager.cris.com (Tom_Potter) ) "Recentemente foi noticiado na CNN que encontraram um número primo 4 vezes maior que o anterior." "Está provado que fazer aniversário é saudável. Estatísticas mostram que pessoas que fazem mais aniversários vivem mais." -- S. den Hartog, Ph D. Thesis Universtity of Groningen "O número que você discou é imaginário, por favor gire 90 graus o seu telefone e tente novamente." "Um homem com um relógio sabe a hora certa. Um homem com dois relógios só sabe a média." P: Qual é o resultado de (x-a)(x-b)(x-c)...(x-z) R: Dica, analise as igualdades! (fonte: mlc@iberia.cca.rockwell.com (Michael Cook)) P: Quanto é 8 dividido em duas partes? R: Na vertical é 3 e na horizontal é 0. (fonte: maelmill@EUnet.at (Elisabeth)) Bem, eh isso aih... :c) Ezer Fernandes ========================================================================= Instruções para entrar na lista, sair da lista e usar a lista em http://www.mat.puc-rio.br/~nicolau/olimp/obm-l.html O administrador desta lista é ========================================================================= From owner-obm-l@sucuri.mat.puc-rio.br Mon Sep 16 08:13:54 2002 Return-Path: Received: (from majordom@localhost) by sucuri.mat.puc-rio.br (8.9.3/8.9.3) id IAA25277 for obm-l-MTTP; Mon, 16 Sep 2002 08:12:42 -0300 Received: from seki.bol.com.br (seki.bol.com.br [200.221.24.26]) by sucuri.mat.puc-rio.br (8.9.3/8.9.3) with ESMTP id IAA25273 for ; Mon, 16 Sep 2002 08:12:40 -0300 Received: from proxyvr (200.221.24.192) by seki.bol.com.br (5.1.071) id 3D6143DE007A31BF for obm-l@mat.puc-rio.br; Mon, 16 Sep 2002 08:13:42 -0300 Message-ID: <000a01c25d72$b173b3c0$2932a8c0@redeconora.com.br> From: "Hely Jr." To: Subject: [obm-l] Teorema binomial! Date: Mon, 16 Sep 2002 08:17:47 -0300 MIME-Version: 1.0 Content-Type: multipart/related; type="multipart/alternative"; boundary="----=_NextPart_000_0006_01C25D59.8ACDA280" X-Priority: 3 X-MSMail-Priority: Normal X-Mailer: Microsoft Outlook Express 6.00.2600.0000 X-MimeOLE: Produced By Microsoft MimeOLE V6.00.2600.0000 X-Sender-IP: 200.222.64.218 Sender: owner-obm-l@sucuri.mat.puc-rio.br Precedence: bulk Reply-To: obm-l@mat.puc-rio.br This is a multi-part message in MIME format. ------=_NextPart_000_0006_01C25D59.8ACDA280 Content-Type: multipart/alternative; boundary="----=_NextPart_001_0007_01C25D59.8AD6CA40" ------=_NextPart_001_0007_01C25D59.8AD6CA40 Content-Type: text/plain; charset="iso-8859-1" Content-Transfer-Encoding: quoted-printable O teorema binomial pode ser definido como: ------=_NextPart_001_0007_01C25D59.8AD6CA40 Content-Type: text/html; charset="iso-8859-1" Content-Transfer-Encoding: quoted-printable
O teorema binomial pode ser definido=20 como:
 
3D""
------=_NextPart_001_0007_01C25D59.8AD6CA40-- ------=_NextPart_000_0006_01C25D59.8ACDA280 Content-Type: image/jpeg; name="imagem.jpg" Content-Transfer-Encoding: base64 Content-ID: <000501c25d72$b0133960$2932a8c0@redeconora.com.br> /9j/4AAQSkZJRgABAQEBLAEsAAD/2wBDAAgGBgcGBQgHBwcJCQgKDBQNDAsLDBkSEw8UHRofHh0a HBwgJC4nICIsIxwcKDcpLDAxNDQ0Hyc5PTgyPC4zNDL/2wBDAQkJCQwLDBgNDRgyIRwhMjIyMjIy MjIyMjIyMjIyMjIyMjIyMjIyMjIyMjIyMjIyMjIyMjIyMjIyMjIyMjIyMjL/wAARCAEsAk8DASIA AhEBAxEB/8QAHwAAAQUBAQEBAQEAAAAAAAAAAAECAwQFBgcICQoL/8QAtRAAAgEDAwIEAwUFBAQA AAF9AQIDAAQRBRIhMUEGE1FhByJxFDKBkaEII0KxwRVS0fAkM2JyggkKFhcYGRolJicoKSo0NTY3 ODk6Q0RFRkdISUpTVFVWV1hZWmNkZWZnaGlqc3R1dnd4eXqDhIWGh4iJipKTlJWWl5iZmqKjpKWm p6ipqrKztLW2t7i5usLDxMXGx8jJytLT1NXW19jZ2uHi4+Tl5ufo6erx8vP09fb3+Pn6/8QAHwEA AwEBAQEBAQEBAQAAAAAAAAECAwQFBgcICQoL/8QAtREAAgECBAQDBAcFBAQAAQJ3AAECAxEEBSEx BhJBUQdhcRMiMoEIFEKRobHBCSMzUvAVYnLRChYkNOEl8RcYGRomJygpKjU2Nzg5OkNERUZHSElK U1RVVldYWVpjZGVmZ2hpanN0dXZ3eHl6goOEhYaHiImKkpOUlZaXmJmaoqOkpaanqKmqsrO0tba3 uLm6wsPExcbHyMnK0tPU1dbX2Nna4uPk5ebn6Onq8vP09fb3+Pn6/9oADAMBAAIRAxEAPwD1/wD4 Tvwf/wBDXof/AIMYf/iqkg8aeFbq4it7fxLo008rhI447+JmdicAABskk8YrD1zw7ceHry98V+Gb v7NMN1zf6XPcCOxvBjMjnPEUxCqfN6ZX5hhmNdBcWFn4p0fTZ7y0ng2y22oQxzxhZoJEZZFBBztb gqwHOCwzzQBny+O9OhvL+FrDVfJsLtbO6uRZtsSRgmwKv35NzSKo2Kx5BICsrNsaTqyarFP/AKNP a3FtL5NxbT7d8T7VcAlGZTlHRvlY/ewcEEDn7zwnrFz/AGr5etWMf27VbbUk3ac7eX5Pl7UP74bs +RFk8fx8fMNuxo+k3mnanrV1c3sFxHqN2LmOOO2MZhxGke0sXbd8sac4XncehAABsVXvr+z0yzkv L+7gtLWPG+aeQRouSAMseBkkD8asV5/4Q/4qjxp4j8R3/wC8bStQl0fTYH+YWixgCWRTwN0pYZON wA27iOKAOssvEug6kkj2Gt6bdJG8aO0F0jhWdtqA4PBZuAO54FWNS1Wx0i3We/uY4EdxHGG5aVyC QiKOXc4OFUEnsDWP4m8F6b4lvdK1CVI4NQ029t7qK6WFS7LE+7ymbrsOW4B4ODzjBy/DV1/wlfjT XdXubeBrXQ7t9K0zen72GQAfaXPUfOSgBznauMLltwB3FFcHa+PtY1HW9f0ew8G3bXmlJC2y4voI wxkQuFcqWCkgYXbvBJG4oMkUx8VZX8CQ+Ml8NzppC7PtLTXaB+ZRE3kKu7zNrbvv+VnAx1OAD0ii iuH07xvrOs+I9a0Sx8LeRcaX5BkbUNQSMYkQsAfKWXDegG4EZJKkBSAdZqWq2Oj263Oo3Mdrblwh nl+WNCQSN79EBxgFiASQOpANyub8K+JLHx14alnewkhId7PUNPu48mGUACSJgRhhhvTkHkA5Az/A mpTRX+v+ErhY8+H7iOO1MQO0WcibrdCScl1UbScdhyxySAdpRRUc5mW3la3jjknCExpI5RWbHALA EgZ74OPQ0ASVh6r4nh0zWbfSE07Ur6/uLd7iKO1gG0qjKrAyOVQH5s8sOgBwWQNz8fj7UkuNW0u9 0O0tvEFq6Cx0w6gzNqKsT88beSBswDyAdu1vM8sAmthtK1681mx1mS502xnt7K4tTarE90u6RlIf zN0ZIBiiO3YP4xnkMACxo/iqx1u4hjtYbtIrq3N1ZzzReWt1EpUM6A/OADIg+dV3bgV3DmtyuT0T wpqWk3Hh5pNWtJ4NI0x9OKLYsjTKxTDbvNIUgQxdjnD9Nw29ZQAVHPMttbyzuJCkaF2EcbOxAGeF UEsfYAk9q5Pxn4v1TwtqOjQ22hQX1rql3HYpcPf+T5U7nChl8tjtI53DPQ5A4zoanq2sL4hi0bSb Kxk8y0NxJdz3L/6N84UbolQ7s5JUF03+XIMrtzQBTsvHttqNvZS2mi6zI9/bi5sYjAitcoAu8jc4 CBN6gmQoGyNhfIz0lhfW+p6dbX9nJ5lrdRJNC+0jcjAFTg8jII61y+l+E9Y0v+w/L1qxf+ydKl01 N2nP+83bNrn99xjyYsjvh+RuG3Q0fwpZ2eh6JY6pBY6ndaPEkVtdvZgFNmArKGLFGwqZIPJGeOAA CPSfGljrFxAkNlqUENxcTWkNxdW/lK88RfdGFY787Y3bJXaMFSQ4KDpK4PwhpdzqBS+uLi7itLTW L+8gtLrT3t5VeSScKQzhSYjFPuxtJ3sfnwNg7ygAooooAKKKKACiiigAooooAKKKKACiiigAoooo AKKKKACiiigAooooAKKKKACiiigAooooAKKKKACiiigAooooAKKKKACiiigAooooAKKKKACiiigA ooooAKKKKACiiigDh9U8WeG9R1y70bUPEGlW2m2W1LuKW9jQ3UxzuhYMQfLQBdwAIctsJwkiNc1X 4jeGbHS7i4sdZ03U7xUxbWNpdrLLcynhI1VNzZZiBkA4zk8Crn/CCeD/APoVND/8F0P/AMTUkHgv wra3EVxb+GtGhnicPHJHYRKyMDkEELkEHnNAG5RRRQAVw9lYXngrxRrVzHaT3XhzV5ftzfZIzLJZ XRAWTMYzJIshwcpnaQRtVctXcUUAYcGt3OrXEUel6fdx24cGa8v7V4FVQcsixvtkZyMANtCDJO5i uw4fgS2/sfxH400Z1naQ6r/aqzPFsSSO5QEBeTu2tHIpPT5fXIHcVl6roq6hcW9/BPJa6naI62tw pYqocqWV48gSI2xcg88ZUq2GABxfhO6ZfiP471aTT9VisbyKzmt5ZtMuI/NWGErIFDICWBIAXG49 ga4/+ztT/wCGYf7C/sfVf7W83yfsf9nzeZu+1ednbt+7s53fdzxnPFe8UUAc3e+M7TT9Z02wm0vW fIv0iKagLFxbxNIxVEkY4ZHLbRgrwXXOOccnourxaT8XviA1xa30sMn9nDzLS1e42MLc4BWMM/Iz zt2/KQSCVB9AutJS+1GC5ubmeSCHayWR2+T5qnKykbdzMOwLFQQrBdyhhz+geE9Y0jxfq2v3OtWN z/a3k/a4I9OeLHlRlE2MZm29cnIbOOMUAHw60TUdI0fU7nVrf7JfatqtzqUlnvWT7N5jABN6kh+F ByMfexjiqfgiCa68b+OvEAikis7y9hsoBKpVna1QxSOOxQtkAg/wnIBGK6zWdPuNV057K31GfT1l yss9sAJghB4jY5CNnHzYOBnGCQy2LCxt9M062sLOPy7W1iSGFNxO1FACjJ5OAB1oAr6rqv8AZf2L /QL68+1Xcdr/AKJD5nk7s/vJORtjGOW7ZFXJ5ltreWdxIUjQuwjjZ2IAzwqglj7AEntUlFAHkevx arr1k3j/AEnT7uw1zR7gLp9ne6XJ58kGzbJFIiqWYu0j4ZSyqoXBjYyMPTNG1eLW9OS8itb61zgN DfWr28iNgEgq4GcZxlcrkHBOK0KKACq9/dfYdOubz7PPceRE8vk26b5JNoJ2ovdjjAHc1YooA5/W 9Jt/HPgafT7qGe0j1O0VglxGVkt3IDoXQEfMjBSVz1XB4rP+Hn9o6hoaeINa51W+ijhkI24EcOVU qU+VldzLMGUYImABKqpqx4k8PeIta1GM6d4vn0jTGiEVxa29nG0knJ3Mkx+aNipABAO0jNdBYWNv pmnW1hZx+Xa2sSQwpuJ2ooAUZPJwAOtAFiiiigAooooAKKKKACiiigAooooAKKKKACiiigAooooA KKKKACiiigAooooAKKKKACiiigAooooAKKKKACiiigAooooAKKKKACiiigAooooAKKKKACiiigAo oooAKKKKACiiigAooooAKKKKAPM/F19NZeJdfX+3dSiuF0eC40izSUpE15m4CIoUDzHZkXETFvMy 2VcIoT0ysuDRRB4lvNb+33bvdW8du1swj8pFjLFSuED5y8h5Y/fPouNSgAooooAKKKKACiiigAoo ooAKKKKACiiigAooooAKKKKACiiigAooooAKKKKACiiigAooooAKKKKACiiigAooooAKKKKACiii gAooooAKKKKACiiigAooooAKKKKACiiigAooooAKKKKACiiigAooooAKKKKACiiigAooooAKKKKA CiiigAooooAKKKKACiiigAooooAKKKKACiiigAooooAKKKKACiiigAooooAKKKKACiiigAooooAK KKKACiiigAooooAKKKKACiiigAooooAKKKKACiiigAooooAKKKKACiiigAooooAKKKKACiiigAoo ooAKKKKACiiigAooooAKKKKACiiigAooooAKKKKACiiigAooooAKKKKACiiigAooooAKKKKACiii gAooooAKKKKACiiigAooooAKKKKACiiigAooooAKKKKACiiigAooooAKKKKACiiigAooooAKKKKA CiiigAooooAKKKKACiiigAooooAKKKKACiiigAooooAKKKKACiiigAooooAKKKKACiiigAooooAK KKKACiiigAooooAKKKKACiiigAooooAx77xRpOnXkltcTThosefLHaSyQwZAP72VVKR4UhjvYYUh jgEGtCwvrfU9Otr+zk8y1uokmhfaRuRgCpweRkEda5fR9c0vw7bXWmavfwWmof2heTJaSP8Avpkl uZJIzEg+aXcrrjYG+bK/eBAx/E979h+1RLNPpE1vokUmgWCXPkbrz99mFY422TspW3Xy/nUZAAw/ zAHpFFeX+LdWSz1zxfL/AMJTPFNpmiQXlpZC9WNIbn98Fyi4LcmHKOWDecoYMPLC3P7Za2fxVp+g 65Jqd5FoUeoWY+0LdSvcOsxLovIwQLchFAQb12qA/IB6JWfq+sW+iwW0tyk7rcXcNonkxF8PK4RS x6KuTySR6DJIBw/Cl1aS6zqkOj6nJqOipb27Rzm+e8UXBaYSqJWZjkKsJKbsDIOBuJPN+LdRii1a 8N/qF8l1b63pv2Wyh3yCWzElq5byFByvmmQCULuLqIwx+4QD1Co4J4bq3iuLeWOaCVA8ckbBldSM ggjggjnNed6P9qv/ABbdJfeJ9txJd3kM2kReeJ1tg0ixFgJtsC7BE6yrEhOUG8mTL8/pk0J8G+EY LbxJ/ZelPp5e7vL26uRCbtY7cLEkoniIwGmxGr7AUkG3ch2gHtFFeb2UqXnii/ttS8aXxay0Syu3 AmWz8mUCXzJmhwCmBsdkkyv70B1ICBcvw9qNjc2/heC/8Y3bJqOhSX2ol9W2NI8QiCjcCDEF/f5M ZQt5TeYX2vkA9YMjC4SIQyFGRmMoK7VIIwp5zk5JGAR8pyRxmSvM9H8VRahceGJ7zxZHE994Xnmu Qs8KrHOphLTbSMbxif7wKr5T4Aw+afhvxCltp2hY8Q32oXmo+Gp7u4jS6W5uZJ4hEEEMbZUSD/SF AC/OYyX3spNAHrFRiRjcPEYZAiorCUldrEk5Uc5yMAnIA+YYJ5x4vN4rhiF5YR+L7TToluNLliaz 8QDUC2+5ZJ/31wu7AQIzIPlAUE8O4bsLa8h0nxL4o0q21jUrv7Lo8FxDZpci6uoWzcFxEsm4s5zE fn3cugPy7VAB3lFef+AtdiuvEOq6VHqdjcQx2lvcQxW2tvqfzF5VkbzZAHHAiBXlVyp4LnJqeoOn ii7Q6jPHq6arZxafZC6ZfNsmFv5zC3B2yqN11mQqxXa3I8sbQD0Ciub8XRGFNM1f7RdxxadexvcR w3MkKPC7BHMm1gNke5ZSWBGImBwGY1ycrNpXhfRv7Rur6Br/AM29kGq69cWMcDOVZYHuPmk8xFbY sfyq4SV2G4DAB6Bbaxb3WuX+kRpOLixihllZ4iqMJd+3aT97/VnJHHbOQQNCvJ9D8VS6vp2sX2m6 nBe+IT4PspgtsUkkN0ouSw8tc/MHdMrjgsoxyBRrGr2drbXX9j+L57fRVu9LEd8mpC5Bme5cXEYm mL9IhG5TOABkjazBgD1is+21i3utcv8ASI0nFxYxQyys8RVGEu/btJ+9/qzkjjtnIIFPV4bnRvBW qJopu5Ly3sp3tN8j3Upl2sy8yFmc7sYBz2HTiuLttRih1HxifC2oX2pTJ4fhlsJvnvF8xDdYWOZg 3m/OwwCzktvUcIVUA9Qorj/AuB9vEHieDXLX93tNt50scT/Nu/fSzTFmI25QP8oCnaN+W5fUdZgt rzxJdt40njmtfEFrZwZvoglrHKIFlAjxsbC+eB5isF8lmwGDsQD1io55ltreWdxIUjQuwjjZ2IAz wqglj7AEntXN+EdQt5b7xFpVvqP2uPTNQEMaPdGeSJDDGxDOxLn96Zh8xOCpXouBzf8Awka2vxDs YDqMcBudTmtJ4bvV2M7J5cvlj7GVEcSF0i2SKdzqY85MjZAPQNJ1KHWdGsdUt1kWC9t47iNZAAwV 1DAHBIzg+pqxPI0NvLKkMk7ohZYoyoZyB90biBk9OSB6kV538O5PI/sLT7bUp7to9E26tA8/mfY7 hPIWKJkHEDAGddoClthLbmUmuX1Pxd9ms9Wij8Rf2dJcaJeT/vNb868F0hiMaSROoW1mO6RTFF3L BcGNSAD3CivJ7/WfDs2sXjReLp/7Ek1C2N3dxa1IPs8zK5WGIiYYhf8AiKqQuWAyFLW2xqeoOnii 7Q6jPHq6arZxafZC6ZfNsmFv5zC3B2yqN11mQqxXa3I8sbQD0Ciq9mbcwMbWfzo/NkBbzjLh97b1 yScYbcu3+HG0AAYFigAooooAKKKKACiiigAooooAKKKKACiiigAooooAKKKKACiiigAooooAKKKK ACiiigAooooAKKKKACiiigAooooAKKKKACiiigAooooAKKKKACiiigAooooAKKKKACiiigAooooA KKKKACiiigAooooAKKKKACiiigAooooAKKKKACiiigAooooAKKKKACiiigAooooAKKKKACiiigAo oooAKKKKACiiigAooooAKKKKACiiigAooooAKKKKACiiigAooooAKKKKACiiigAooooAKKKKACii igAooooAKKKKACiiigAooooAKK8r1X4cWXhrVLjXdJ8P6bq+ku/m3ehyadFJKufvyW0jDORgEQn5 TlwuCVA6A+B/CGuWGkaro2labZlLi11K0u7WyWJmVXWQAgBThlyMN03A4yBQBqP448NR3l1bPq8A azlMN1JhvKtnABxLJjZHknaNxGWDKMspA1NN1S01a3aa0eQhHKSJLE8UkbYBw6OAynBBAIGQwPQg 1yd9oPiSb+2vItdKP2vW7TUIN99Iv7uDychsQnDH7MvAyB5h5+QbtzQrDUrPVvEE97FaJBfXq3Fs YZ2dtohjiw4KKFOIg3Bb7xH8OSAblFFef6R/xXnijxBcan++0DSbttLttNk/1csyBTNLPHysnzbf LySABnaG5oA9AqOeeG1t5bi4ljhgiQvJJIwVUUDJJJ4AA5zXF6z8N9OkutOu/D0MGjzQahaXF1Fb boYbuGGXfseNCELAncGKk5UDIByDT5rfxv40v57i183TPDN2LeyYyHZLegZldozj5ovlVCRgbnYE 5G0A7iiuLj+JemXN/q1hZaR4gu7vTUR5IY9MkVpAyF+A+3aQoPEmwt0TcSAYx8VNCfw5D4hitNVf SDs+0XZtCiW29wnzbiPMwxIPleZjac9sgHcUUVx9p8Q7PU9Y1HStL0TXLy70/wAozJ9kFv8AK67g f37JjsMNhjnKhlBIAOsknhheFJZY0eZ9kSswBdtpbC+p2qxwOwJ7VJXN6bqXhz4leDmkjWO+0u9Q xz28ow0bcEo4ByrqcEEH0ZT0NV/BWtzXL6t4dv8AzP7Q0G4FszyyGR57dlzBM7dC7oPm5zkEkLkA AHWUUUUAFZer+I9I0F7SPU76OCW8dktosFnmYLuIRVBLHoAAOSyqMllB878V/DTSV8Jat4mvxOPF sNpJqD6jbXkuY7hFMgEeTgRqVCKMZCAc5Ga2NZv9RtLL4fx6jd2MXiF7tDLHeSKqyzfY5lkX5OmX cJuUMAzrhW4UgHcWN9b6jZx3VrJ5kL5AJUqQQSGVlOCrAggqQCCCCARVisvw/ps2laT5Fw0ZnkuL i6kEZJVGmmeUoCQCwUvt3YGcZwM4GpQAUVl6lqt5Y3CxW/h/UtQQoGMtrJbqoOT8p8yVDnjPTHI5 64p/8JDqn/Qma5/3+sv/AJIoA6Cio4JGmt4pXhkgd0DNFIVLISPunaSMjpwSPQms/wAR6Q2v+H7z SFvJLRLxBDLLGis3lEjzFG7gFk3LnBxuzg4oA1KK8rvvhN4HtPEGhWdhoMYnluGuZt88smLeEZY7 XYoQZGgQqQSVkbA43L0mt/D7SfF2sT3nii2+2wxbYtPhS5lRIo9oLsVUj94zlgTyNqR9CDkA7Civ G4NV1/wP8GPE08FzJL/ZepzWOkTXe2VltlnWBScdSreZgMMcDjbgVseNLPRfCPhiLxz4et40u7B4 ZfPtJR/xMYJZI1kSaQhvNDqd29tzBgGBznIB6ZRRRQAUUUUAFFFFABRRRQAUUUUAFFFFABRRRQAU UUUAFFFFABRRRQAUUUUAFFFFABRRRQAUUUUAFFFFABRRRQAUUUUAFFFFABRRRQAUUUUAFFFFABRR RQAUUUUAc/qn/FQ3l34eXiwSJV1OUcl1kB/0dSPusV5c8MqOm0ZkDpoa5rNn4e0O91e/fZa2kTSv ggFsdFXJALE4AGeSQK5Of4M+Arq4luLjRJJp5XLySSX9yzOxOSSTJkknnNSWHwg8CaZqNtf2eheX dWsqTQv9rnO11IKnBfBwQOtAHcUUUUAFef6R/wAUH4o8QW+p/udA1a7bVLbUpP8AVxTOFE0U8nCx /Nt8vIAIONxbiu01LVtN0a3W41TULSxgZwiyXUyxKWwTgFiBnAJx7GrlAGHB4osdUuIoNCkj1Xc4 82e2fdbwpn5i0wBTeBjEYJYllyApLjn/AIdxrp2t+ONHlmje8TXZL50QNhYrhEeM5IAzgMCB0Knt gnvKx9T0m4k1GLV9MufJv4YjE0L4EN2mQQkpClhj5tjD7hdjhgWVgDh/Buu6PP8AFL4h3kWq2Mlq 0VjOsyXCFDHHARI4bONqkgE9BnmuI+32f/DJPkfa4PO83yPL8wbvM+2eZsx/e2fNjrt56V9D0UAc /P448NW2uado0urwLfalEk1moDFJkfIQrIBs+YqQOeeMdRnj9F1zS9F+L3xAOq38FjHL/ZwSa5fy 4yRbn5d5+UMc5Ck5IDEAhTjvNR0qbU723Fxcx/2ZC8c5tViIeSaN96FpN33AwVtoUElRlipKnm/D eg+JNO8d6/rl/a6UtrrX2fekF9JI8HkxFBgGFQ+4kd1x70AHwwsLy30fWtRvLSez/tfW7vUIba5j Mc0UbsFUSKfut8hOBkYI5qv4I/0z4j/EHVrf57GS7tbJJemZoISsq4PPylgM4wc8E12Gsw6pcac9 vpF1BZ3UuU+1zR+Z5AwfnWPgO2cAAkAZyc42tJpWmw6RpdvYQNI6QpgySkF5W6s7kAbnZiWY9ySe 9AEeq63p2ifYv7RuPJ+3Xcdlb/Izb5nztXgHGcHk4HvWhRRQB5PrHxT8CapriWt/r/lafpd2kyiG OdheTpyp3RqVMKNgjk73UHhVHmekaHqya9odlq0VtPbw3kSzRxz7d+xuVJ2sw5GD1784ORWhRQAU UUUAZepaBZ6rcLPcTakjqgQC11K4t1xknlY3UE89cZ6egqn/AMIbpf8Az9a5/wCD29/+PV0FFAEc EK21vFAhkKRoEUySM7EAY5ZiSx9yST3qSiigDn9J/wCJj4o1jUz80Nts022P3l+Qb5nRu2XcRsB/ FbDJJGF5/wAdfEXw7ot4fDd3rf2C6uIj9puYlkd7SNh/D5YJEzA5XOAv3znCq/oFFAHF+HPFfhDV vCt2liY4/C+momnm5v8AalvIuxR5f7w7iAGVTvAyW43c1z8XgrQ9WuNH0zQdLu/+EXtbj7Tcz3V1 LJbSbDuSOCKcsGDswYyooUpvUOd7CvVKKACiiigAooooAKKKKACiiigAooooAKKKKACiiigAoooo AKKKKACiiigAooooAKKKKACiiigAooooAKKKKACiiigAooooAKKKKACiiigAooooAKKKKACiiigA ooooAKKKKAPP/Euuazp+ueIxDrcEMOmaImrWtitum+Yr5wYOzEloyUUNtCn5kAZCCX0Nb/t7+2J/ sX/CV/Z/l2fYP7L8n7ozt8/951znd3zjjFaDeGvtHi241i/axu7Vordbe2kssvBJCzskgkLkbszS chQRlcHg7ugoA4/T/tX/AAlfh/7b9u+0f2VqO/7f5Hnf6+1xu8j930xjb2xnnNaHgT/knnhr/sFW v/opa1NS0nTdZt1t9U0+0voFcOsd1CsqhsEZAYEZwSM+5q5QAUUUUAFFFFABRRRQAUUUUAFFFFAB RRRQAUUUUAFFFFABRRRQAUUUUAFFFFABRRRQAUUUUAFFFFABRRRQAUUUUAFFFFABRRRQAUUUUAFF FFABRRRQAUUUUAFFFFABRRRQAUUUUAFFFFABRRRQAUUUUAFFFFABRRRQAUUUUAFFFFABRRRQAUUU UAFFFFABRRRQAUUUUAFFFFABRRRQAUUUUAFFFFABRRRQAUUUUAFFFFABRRRQAUUUUAFFFFABRRRQ AUUUUAFFFFABRRRQAUUUUAFFFFABRRRQAUUUUAFFFFABRRRQAUUUUAFFFFABRRRQAUUUUAFFFFAB RRRQAUUUUAFFFFABRRRQAUUUUAFFFFABRRRQAUUUUAFFFFABRRRQAUUUUAFFFFABRRRQAUUUUAFF FFABRRRQAUUUUAFFFFABRRRQAUUUUAFFFFABRRRQAUUUUAFFFFABRRRQAUUUUAFFFFABRRRQAUUU UAFFFFABRRRQAUUUUAFFFFABRRRQAUUUUAFFFFABRRRQAUUUUAFFFFABRRRQAUUUUAFFFFABRRRQ AUUUUAFFFFABRRRQAUUUUAFFFFABRRRQAUUUUAFFFFABRRRQAUUUUAFFFFABRRRQAUUUUAFFFFAB RRRQAUUUUAFFFFABRRRQAUUUUAFFFFABRRRQAUUUUAFFFFABRRRQAUUUUAFFFFABRRRQAUUUUAFF FFABRRRQAUUUUAFFFFABRRRQAUUUUAFFFFABRRRQAUUUUAFFFFABRRRQAUUUUAFFFFABRRRQAUUU UAFFFFABRRRQAUUUUAFFFFABRRRQAUUUUAFFFFABRRRQAUUUUAFFFFABRRRQAUUUUAFFFFABRRRQ AUUUUAFFFFABRRRQAUUUUAFFFFABRRRQAUUUUAFFFFABRRRQAUUUUAFFFFABRRRQAUUUUAFFFFAB RRRQAUUUUAFFFFABRRRQAUUUUAFFFFABRRRQAUUUUAFFFFABRRRQAUUUUAFFFFABRRRQAUUUUAFF FFABRRRQAUUUUAFFFFABRRRQAUUUUAFFFFABRRRQAUUUUAFFFFABRRRQAUUUUAFFFFABRRRQAUUU UAFFFFABRRRQAUUUUAFFFFABRRRQAUUUUAFFFFABRRRQAUUUUAFFFFABRRRQAUUUUAFFFFABRRRQ AUUUUAFFFFABRRRQAUUUUAFFFFABRRRQAUUUUAFFFFABRRRQAUUUUAFFFFABRRRQAUUUUAFFFFAB RRRQAUUUUAFFFFABRRRQAUUUUAFFFFABRRRQAUUUUAFFFFABRRRQAUUUUAFFFFABRRRQAUUUUAFF FFABRRRQAUUUUAFFFFABRRRQAUUUUAFFFFABRRRQAUUUUAFFFFABRRRQAUUUUAFFFFABRRRQAUUU UAFFFFABRRRQAUUUUAFFFFABRRRQAUUUUAFFFFABRRRQAUUUUAFFFFABRRRQAUUUUAFFFFABRRRQ AUUUUAFFFFABRRRQAUUUUAFFFFABRRRQAUUUUAFFFFABRRRQAUUUUAFFFFABRRRQAUUUUAFFFFAB RRRQAUUUUAFFFFABRRRQAUUUUAFFFFABRRRQAUUUUAFFFFABRRRQAUUUUAFFFFABRRRQAUUUUAFF FFABRRRQAUUUUAFFFFABRRRQAUUUUAFFFFABRRRQAUUUUAFFFFABRRRQAUUUUAFFFFABRRRQAUUU UAFFFFABRRRQAUUUUAFFFFABRRRQAUUUUAFFFFABRRRQAUUUUAFFFFABRRRQAUUUUAFFFFABRRRQ AUUUUAFFFFABRRRQAUUUUAFFFFABRRRQAUUUUAFFFFABRRRQAUUUUAFFFFABRRRQAUUUUAFFFFAB RRRQAUUUUAFFFFABRRRQAUUUUAFFFFABRRRQAUUUUAFFFFABRRRQAUUUUAFFFFABRRRQAUUUUAFF FFABRRRQAUUUUAFFFFABRRRQAUUUUAf/2Q== ------=_NextPart_000_0006_01C25D59.8ACDA280-- ========================================================================= Instruções para entrar na lista, sair da lista e usar a lista em http://www.mat.puc-rio.br/~nicolau/olimp/obm-l.html O administrador desta lista é ========================================================================= From owner-obm-l@sucuri.mat.puc-rio.br Mon Sep 16 08:46:45 2002 Return-Path: Received: (from majordom@localhost) by sucuri.mat.puc-rio.br (8.9.3/8.9.3) id IAA25792 for obm-l-MTTP; Mon, 16 Sep 2002 08:46:33 -0300 Received: from gorgo.centroin.com.br (gorgo.centroin.com.br [200.225.63.128]) by sucuri.mat.puc-rio.br (8.9.3/8.9.3) with ESMTP id IAA25784 for ; Mon, 16 Sep 2002 08:46:29 -0300 Received: from centroin.com.br (du162c.rjo.centroin.com.br [200.225.58.162]) (authenticated bits=0) by gorgo.centroin.com.br (8.12.2/8.12.1) with ESMTP id g8GBm7I8007125 for ; Mon, 16 Sep 2002 08:48:07 -0300 (BRT) Message-ID: <3D85C59A.6070305@centroin.com.br> Date: Mon, 16 Sep 2002 08:50:50 -0300 From: Augusto =?ISO-8859-1?Q?C=E9sar?= Morgado User-Agent: Mozilla/5.0 (Windows; U; Win98; en-US; rv:0.9.4.1) Gecko/20020508 Netscape6/6.2.3 X-Accept-Language: en-us MIME-Version: 1.0 To: obm-l@mat.puc-rio.br Subject: Re: [obm-l] Teorema binomial! References: <000a01c25d72$b173b3c0$2932a8c0@redeconora.com.br> Content-Type: multipart/alternative; boundary="------------010507090903070205060203" Sender: owner-obm-l@sucuri.mat.puc-rio.br Precedence: bulk Reply-To: obm-l@mat.puc-rio.br --------------010507090903070205060203 Content-Type: text/plain; charset=us-ascii; format=flowed Content-Transfer-Encoding: 7bit A sua formula do teorema binomial estah errada. Eh x^p e nao x^m. Os limites sao p=0 e p=m. A soma pedida eh o valor para x=2 e y = 1. Vale, portanto, 3^m. Logo, m=6. Hely Jr. wrote: > O teorema binomial pode ser definido como: > > > --------------010507090903070205060203 Content-Type: multipart/related; boundary="------------000508080704040904000101" --------------000508080704040904000101 Content-Type: text/html; charset=us-ascii Content-Transfer-Encoding: 7bit A sua formula do teorema binomial estah errada. Eh  x^p  e nao  x^m. Os limites sao  p=0  e p=m.

A soma pedida eh o valor para x=2  e  y = 1. Vale, portanto, 3^m. Logo, m=6.


Hely Jr. wrote:
O teorema binomial pode ser definido como:
 

--------------000508080704040904000101 Content-Type: image/jpeg; name="Inbox" Content-Transfer-Encoding: base64 Content-ID: Content-Disposition: inline; filename="Inbox" /9j/4AAQSkZJRgABAQEBLAEsAAD/2wBDAAgGBgcGBQgHBwcJCQgKDBQNDAsLDBkSEw8UHRof Hh0aHBwgJC4nICIsIxwcKDcpLDAxNDQ0Hyc5PTgyPC4zNDL/2wBDAQkJCQwLDBgNDRgyIRwh MjIyMjIyMjIyMjIyMjIyMjIyMjIyMjIyMjIyMjIyMjIyMjIyMjIyMjIyMjIyMjIyMjL/wAAR CAEsAk8DASIAAhEBAxEB/8QAHwAAAQUBAQEBAQEAAAAAAAAAAAECAwQFBgcICQoL/8QAtRAA AgEDAwIEAwUFBAQAAAF9AQIDAAQRBRIhMUEGE1FhByJxFDKBkaEII0KxwRVS0fAkM2JyggkK FhcYGRolJicoKSo0NTY3ODk6Q0RFRkdISUpTVFVWV1hZWmNkZWZnaGlqc3R1dnd4eXqDhIWG h4iJipKTlJWWl5iZmqKjpKWmp6ipqrKztLW2t7i5usLDxMXGx8jJytLT1NXW19jZ2uHi4+Tl 5ufo6erx8vP09fb3+Pn6/8QAHwEAAwEBAQEBAQEBAQAAAAAAAAECAwQFBgcICQoL/8QAtREA AgECBAQDBAcFBAQAAQJ3AAECAxEEBSExBhJBUQdhcRMiMoEIFEKRobHBCSMzUvAVYnLRChYk NOEl8RcYGRomJygpKjU2Nzg5OkNERUZHSElKU1RVVldYWVpjZGVmZ2hpanN0dXZ3eHl6goOE hYaHiImKkpOUlZaXmJmaoqOkpaanqKmqsrO0tba3uLm6wsPExcbHyMnK0tPU1dbX2Nna4uPk 5ebn6Onq8vP09fb3+Pn6/9oADAMBAAIRAxEAPwD1/wD4Tvwf/wBDXof/AIMYf/iqkg8aeFbq 4it7fxLo008rhI447+JmdicAABskk8YrD1zw7ceHry98V+Gbv7NMN1zf6XPcCOxvBjMjnPEU xCqfN6ZX5hhmNdBcWFn4p0fTZ7y0ng2y22oQxzxhZoJEZZFBBztbgqwHOCwzzQBny+O9OhvL +FrDVfJsLtbO6uRZtsSRgmwKv35NzSKo2Kx5BICsrNsaTqyarFP/AKNPa3FtL5NxbT7d8T7V cAlGZTlHRvlY/ewcEEDn7zwnrFz/AGr5etWMf27VbbUk3ac7eX5Pl7UP74bs+RFk8fx8fMNu xo+k3mnanrV1c3sFxHqN2LmOOO2MZhxGke0sXbd8sac4XncehAABsVXvr+z0yzkvL+7gtLWP G+aeQRouSAMseBkkD8asV5/4Q/4qjxp4j8R3/wC8bStQl0fTYH+YWixgCWRTwN0pYZONwA27 iOKAOssvEug6kkj2Gt6bdJG8aO0F0jhWdtqA4PBZuAO54FWNS1Wx0i3We/uY4EdxHGG5aVyC QiKOXc4OFUEnsDWP4m8F6b4lvdK1CVI4NQ029t7qK6WFS7LE+7ymbrsOW4B4ODzjBy/DV1/w lfjTXdXubeBrXQ7t9K0zen72GQAfaXPUfOSgBznauMLltwB3FFcHa+PtY1HW9f0ew8G3bXml JC2y4voIwxkQuFcqWCkgYXbvBJG4oMkUx8VZX8CQ+Ml8NzppC7PtLTXaB+ZRE3kKu7zNrbvv +VnAx1OAD0iiiuH07xvrOs+I9a0Sx8LeRcaX5BkbUNQSMYkQsAfKWXDegG4EZJKkBSAdZqWq 2Oj263Oo3Mdrblwhnl+WNCQSN79EBxgFiASQOpANyub8K+JLHx14alnewkhId7PUNPu48mGU ACSJgRhhhvTkHkA5Az/AmpTRX+v+ErhY8+H7iOO1MQO0WcibrdCScl1UbScdhyxySAdpRRUc 5mW3la3jjknCExpI5RWbHALAEgZ74OPQ0ASVh6r4nh0zWbfSE07Ur6/uLd7iKO1gG0qjKrAy OVQH5s8sOgBwWQNz8fj7UkuNW0u90O0tvEFq6Cx0w6gzNqKsT88beSBswDyAdu1vM8sAmtht K1681mx1mS502xnt7K4tTarE90u6RlIfzN0ZIBiiO3YP4xnkMACxo/iqx1u4hjtYbtIrq3N1 ZzzReWt1EpUM6A/OADIg+dV3bgV3DmtyuT0TwpqWk3Hh5pNWtJ4NI0x9OKLYsjTKxTDbvNIU gQxdjnD9Nw29ZQAVHPMttbyzuJCkaF2EcbOxAGeFUEsfYAk9q5Pxn4v1TwtqOjQ22hQX1rql 3HYpcPf+T5U7nChl8tjtI53DPQ5A4zoanq2sL4hi0bSbKxk8y0NxJdz3L/6N84UbolQ7s5JU F03+XIMrtzQBTsvHttqNvZS2mi6zI9/bi5sYjAitcoAu8jc4CBN6gmQoGyNhfIz0lhfW+p6d bX9nJ5lrdRJNC+0jcjAFTg8jII61y+l+E9Y0v+w/L1qxf+ydKl01N2nP+83bNrn99xjyYsjv h+RuG3Q0fwpZ2eh6JY6pBY6ndaPEkVtdvZgFNmArKGLFGwqZIPJGeOAACPSfGljrFxAkNlqU ENxcTWkNxdW/lK88RfdGFY787Y3bJXaMFSQ4KDpK4PwhpdzqBS+uLi7itLTWL+8gtLrT3t5V eSScKQzhSYjFPuxtJ3sfnwNg7ygAooooAKKKKACiiigAooooAKKKKACiiigAooooAKKKKACi iigAooooAKKKKACiiigAooooAKKKKACiiigAooooAKKKKACiiigAooooAKKKKACiiigAoooo AKKKKACiiigDh9U8WeG9R1y70bUPEGlW2m2W1LuKW9jQ3UxzuhYMQfLQBdwAIctsJwkiNc1X 4jeGbHS7i4sdZ03U7xUxbWNpdrLLcynhI1VNzZZiBkA4zk8Crn/CCeD/APoVND/8F0P/AMTU kHgvwra3EVxb+GtGhnicPHJHYRKyMDkEELkEHnNAG5RRRQAVw9lYXngrxRrVzHaT3XhzV5ft zfZIzLJZXRAWTMYzJIshwcpnaQRtVctXcUUAYcGt3OrXEUel6fdx24cGa8v7V4FVQcsixvtk ZyMANtCDJO5iuw4fgS2/sfxH400Z1naQ6r/aqzPFsSSO5QEBeTu2tHIpPT5fXIHcVl6roq6h cW9/BPJa6naI62twpYqocqWV48gSI2xcg88ZUq2GABxfhO6ZfiP471aTT9VisbyKzmt5ZtMu I/NWGErIFDICWBIAXG49ga4/+ztT/wCGYf7C/sfVf7W83yfsf9nzeZu+1ednbt+7s53fdzxn PFe8UUAc3e+M7TT9Z02wm0vWfIv0iKagLFxbxNIxVEkY4ZHLbRgrwXXOOccnourxaT8XviA1 xa30sMn9nDzLS1e42MLc4BWMM/Izzt2/KQSCVB9AutJS+1GC5ubmeSCHayWR2+T5qnKykbdz MOwLFQQrBdyhhz+geE9Y0jxfq2v3OtWNz/a3k/a4I9OeLHlRlE2MZm29cnIbOOMUAHw60TUd I0fU7nVrf7JfatqtzqUlnvWT7N5jABN6kh+FByMfexjiqfgiCa68b+OvEAikis7y9hsoBKpV na1QxSOOxQtkAg/wnIBGK6zWdPuNV057K31GfT1lyss9sAJghB4jY5CNnHzYOBnGCQy2LCxt 9M062sLOPy7W1iSGFNxO1FACjJ5OAB1oAr6rqv8AZf2L/QL68+1Xcdr/AKJD5nk7s/vJORtj GOW7ZFXJ5ltreWdxIUjQuwjjZ2IAzwqglj7AEntUlFAHkevxarr1k3j/AEnT7uw1zR7gLp9n e6XJ58kGzbJFIiqWYu0j4ZSyqoXBjYyMPTNG1eLW9OS8itb61zgNDfWr28iNgEgq4GcZxlcr kHBOK0KKACq9/dfYdOubz7PPceRE8vk26b5JNoJ2ovdjjAHc1YooA5/W9Jt/HPgafT7qGe0j 1O0VglxGVkt3IDoXQEfMjBSVz1XB4rP+Hn9o6hoaeINa51W+ijhkI24EcOVUqU+VldzLMGUY ImABKqpqx4k8PeIta1GM6d4vn0jTGiEVxa29nG0knJ3Mkx+aNipABAO0jNdBYWNvpmnW1hZx +Xa2sSQwpuJ2ooAUZPJwAOtAFiiiigAooooAKKKKACiiigAooooAKKKKACiiigAooooAKKKK ACiiigAooooAKKKKACiiigAooooAKKKKACiiigAooooAKKKKACiiigAooooAKKKKACiiigAo oooAKKKKACiiigAooooAKKKKAPM/F19NZeJdfX+3dSiuF0eC40izSUpE15m4CIoUDzHZkXET FvMy2VcIoT0ysuDRRB4lvNb+33bvdW8du1swj8pFjLFSuED5y8h5Y/fPouNSgAooooAKKKKA CiiigAooooAKKKKACiiigAooooAKKKKACiiigAooooAKKKKACiiigAooooAKKKKACiiigAoo ooAKKKKACiiigAooooAKKKKACiiigAooooAKKKKACiiigAooooAKKKKACiiigAooooAKKKKA CiiigAooooAKKKKACiiigAooooAKKKKACiiigAooooAKKKKACiiigAooooAKKKKACiiigAoo ooAKKKKACiiigAooooAKKKKACiiigAooooAKKKKACiiigAooooAKKKKACiiigAooooAKKKKA CiiigAooooAKKKKACiiigAooooAKKKKACiiigAooooAKKKKACiiigAooooAKKKKACiiigAoo ooAKKKKACiiigAooooAKKKKACiiigAooooAKKKKACiiigAooooAKKKKACiiigAooooAKKKKA CiiigAooooAKKKKACiiigAooooAKKKKACiiigAooooAKKKKACiiigAooooAKKKKACiiigAoo ooAKKKKACiiigAooooAKKKKACiiigAooooAKKKKACiiigAooooAKKKKACiiigAooooAx77xR pOnXkltcTThosefLHaSyQwZAP72VVKR4UhjvYYUhjgEGtCwvrfU9Otr+zk8y1uokmhfaRuRg CpweRkEda5fR9c0vw7bXWmavfwWmof2heTJaSP8AvpkluZJIzEg+aXcrrjYG+bK/eBAx/E97 9h+1RLNPpE1vokUmgWCXPkbrz99mFY422TspW3Xy/nUZAAw/zAHpFFeX+LdWSz1zxfL/AMJT PFNpmiQXlpZC9WNIbn98Fyi4LcmHKOWDecoYMPLC3P7Za2fxVp+g65Jqd5FoUeoWY+0LdSvc OsxLovIwQLchFAQb12qA/IB6JWfq+sW+iwW0tyk7rcXcNonkxF8PK4RSx6KuTySR6DJIBw/C l1aS6zqkOj6nJqOipb27Rzm+e8UXBaYSqJWZjkKsJKbsDIOBuJPN+LdRii1a8N/qF8l1b63p v2Wyh3yCWzElq5byFByvmmQCULuLqIwx+4QD1Co4J4bq3iuLeWOaCVA8ckbBldSMggjggjnN ed6P9qv/ABbdJfeJ9txJd3kM2kReeJ1tg0ixFgJtsC7BE6yrEhOUG8mTL8/pk0J8G+EYLbxJ /ZelPp5e7vL26uRCbtY7cLEkoniIwGmxGr7AUkG3ch2gHtFFeb2UqXnii/ttS8aXxay0Syu3 AmWz8mUCXzJmhwCmBsdkkyv70B1ICBcvw9qNjc2/heC/8Y3bJqOhSX2ol9W2NI8QiCjcCDEF /f5MZQt5TeYX2vkA9YMjC4SIQyFGRmMoK7VIIwp5zk5JGAR8pyRxmSvM9H8VRahceGJ7zxZH E994XnmuQs8KrHOphLTbSMbxif7wKr5T4Aw+afhvxCltp2hY8Q32oXmo+Gp7u4jS6W5uZJ4h EEEMbZUSD/SFAC/OYyX3spNAHrFRiRjcPEYZAiorCUldrEk5Uc5yMAnIA+YYJ5x4vN4rhiF5 YR+L7TToluNLliaz8QDUC2+5ZJ/31wu7AQIzIPlAUE8O4bsLa8h0nxL4o0q21jUrv7Lo8FxD Zpci6uoWzcFxEsm4s5zEfn3cugPy7VAB3lFef+AtdiuvEOq6VHqdjcQx2lvcQxW2tvqfzF5V kbzZAHHAiBXlVyp4LnJqeoOnii7Q6jPHq6arZxafZC6ZfNsmFv5zC3B2yqN11mQqxXa3I8sb QD0Ciub8XRGFNM1f7RdxxadexvcRw3MkKPC7BHMm1gNke5ZSWBGImBwGY1ycrNpXhfRv7Rur 6Br/AM29kGq69cWMcDOVZYHuPmk8xFbYsfyq4SV2G4DAB6Bbaxb3WuX+kRpOLixihllZ4iqM Jd+3aT97/VnJHHbOQQNCvJ9D8VS6vp2sX2m6nBe+IT4PspgtsUkkN0ouSw8tc/MHdMrjgsox yBRrGr2drbXX9j+L57fRVu9LEd8mpC5Bme5cXEYmmL9IhG5TOABkjazBgD1is+21i3utcv8A SI0nFxYxQyys8RVGEu/btJ+9/qzkjjtnIIFPV4bnRvBWqJopu5Ly3sp3tN8j3Upl2sy8yFmc 7sYBz2HTiuLttRih1HxifC2oX2pTJ4fhlsJvnvF8xDdYWOZg3m/OwwCzktvUcIVUA9Qorj/A uB9vEHieDXLX93tNt50scT/Nu/fSzTFmI25QP8oCnaN+W5fUdZgtrzxJdt40njmtfEFrZwZv oglrHKIFlAjxsbC+eB5isF8lmwGDsQD1io55ltreWdxIUjQuwjjZ2IAzwqglj7AEntXN+EdQ t5b7xFpVvqP2uPTNQEMaPdGeSJDDGxDOxLn96Zh8xOCpXouBzf8Awka2vxDsYDqMcBudTmtJ 4bvV2M7J5cvlj7GVEcSF0i2SKdzqY85MjZAPQNJ1KHWdGsdUt1kWC9t47iNZAAwV1DAHBIzg +pqxPI0NvLKkMk7ohZYoyoZyB90biBk9OSB6kV538O5PI/sLT7bUp7to9E26tA8/mfY7hPIW KJkHEDAGddoClthLbmUmuX1Pxd9ms9Wij8Rf2dJcaJeT/vNb868F0hiMaSROoW1mO6RTFF3L BcGNSAD3CivJ7/WfDs2sXjReLp/7Ek1C2N3dxa1IPs8zK5WGIiYYhf8AiKqQuWAyFLW2xqeo Onii7Q6jPHq6arZxafZC6ZfNsmFv5zC3B2yqN11mQqxXa3I8sbQD0Ciq9mbcwMbWfzo/NkBb zjLh97b1yScYbcu3+HG0AAYFigAooooAKKKKACiiigAooooAKKKKACiiigAooooAKKKKACii igAooooAKKKKACiiigAooooAKKKKACiiigAooooAKKKKACiiigAooooAKKKKACiiigAooooA KKKKACiiigAooooAKKKKACiiigAooooAKKKKACiiigAooooAKKKKACiiigAooooAKKKKACii igAooooAKKKKACiiigAooooAKKKKACiiigAooooAKKKKACiiigAooooAKKKKACiiigAooooA KKKKACiiigAooooAKKKKACiiigAooooAKKKKACiiigAooooAKK8r1X4cWXhrVLjXdJ8P6bq+ ku/m3ehyadFJKufvyW0jDORgEQn5TlwuCVA6A+B/CGuWGkaro2labZlLi11K0u7WyWJmVXWQ AgBThlyMN03A4yBQBqP448NR3l1bPq8AazlMN1JhvKtnABxLJjZHknaNxGWDKMspA1NN1S01 a3aa0eQhHKSJLE8UkbYBw6OAynBBAIGQwPQg1yd9oPiSb+2vItdKP2vW7TUIN99Iv7uDychs QnDH7MvAyB5h5+QbtzQrDUrPVvEE97FaJBfXq3FsYZ2dtohjiw4KKFOIg3Bb7xH8OSAblFFe f6R/xXnijxBcan++0DSbttLttNk/1csyBTNLPHysnzbfLySABnaG5oA9AqOeeG1t5bi4ljhg iQvJJIwVUUDJJJ4AA5zXF6z8N9OkutOu/D0MGjzQahaXF1FbboYbuGGXfseNCELAncGKk5UD IByDT5rfxv40v57i183TPDN2LeyYyHZLegZldozj5ovlVCRgbnYE5G0A7iiuLj+JemXN/q1h ZaR4gu7vTUR5IY9MkVpAyF+A+3aQoPEmwt0TcSAYx8VNCfw5D4hitNVfSDs+0XZtCiW29wnz biPMwxIPleZjac9sgHcUUVx9p8Q7PU9Y1HStL0TXLy70/wAozJ9kFv8AK67gf37JjsMNhjnK hlBIAOsknhheFJZY0eZ9kSswBdtpbC+p2qxwOwJ7VJXN6bqXhz4leDmkjWO+0u9Qxz28ow0b cEo4ByrqcEEH0ZT0NV/BWtzXL6t4dv8AzP7Q0G4FszyyGR57dlzBM7dC7oPm5zkEkLkAAHWU UUUAFZer+I9I0F7SPU76OCW8dktosFnmYLuIRVBLHoAAOSyqMllB878V/DTSV8Jat4mvxOPF sNpJqD6jbXkuY7hFMgEeTgRqVCKMZCAc5Ga2NZv9RtLL4fx6jd2MXiF7tDLHeSKqyzfY5lkX 5OmXcJuUMAzrhW4UgHcWN9b6jZx3VrJ5kL5AJUqQQSGVlOCrAggqQCCCCARVisvw/ps2laT5 Fw0ZnkuLi6kEZJVGmmeUoCQCwUvt3YGcZwM4GpQAUVl6lqt5Y3CxW/h/UtQQoGMtrJbqoOT8 p8yVDnjPTHI564p/8JDqn/Qma5/3+sv/AJIoA6Cio4JGmt4pXhkgd0DNFIVLISPunaSMjpwS PQms/wAR6Q2v+H7zSFvJLRLxBDLLGis3lEjzFG7gFk3LnBxuzg4oA1KK8rvvhN4HtPEGhWdh oMYnluGuZt88smLeEZY7XYoQZGgQqQSVkbA43L0mt/D7SfF2sT3nii2+2wxbYtPhS5lRIo9o LsVUj94zlgTyNqR9CDkA7CivG4NV1/wP8GPE08FzJL/ZepzWOkTXe2VltlnWBScdSreZgMMc DjbgVseNLPRfCPhiLxz4et40u7B4ZfPtJR/xMYJZI1kSaQhvNDqd29tzBgGBznIB6ZRRRQAU UUUAFFFFABRRRQAUUUUAFFFFABRRRQAUUUUAFFFFABRRRQAUUUUAFFFFABRRRQAUUUUAFFFF ABRRRQAUUUUAFFFFABRRRQAUUUUAFFFFABRRRQAUUUUAc/qn/FQ3l34eXiwSJV1OUcl1kB/0 dSPusV5c8MqOm0ZkDpoa5rNn4e0O91e/fZa2kTSvggFsdFXJALE4AGeSQK5Of4M+Arq4luLj RJJp5XLySSX9yzOxOSSTJkknnNSWHwg8CaZqNtf2eheXdWsqTQv9rnO11IKnBfBwQOtAHcUU UUAFef6R/wAUH4o8QW+p/udA1a7bVLbUpP8AVxTOFE0U8nCx/Nt8vIAIONxbiu01LVtN0a3W 41TULSxgZwiyXUyxKWwTgFiBnAJx7GrlAGHB4osdUuIoNCkj1Xc482e2fdbwpn5i0wBTeBjE YJYllyApLjn/AIdxrp2t+ONHlmje8TXZL50QNhYrhEeM5IAzgMCB0KntgnvKx9T0m4k1GLV9 MufJv4YjE0L4EN2mQQkpClhj5tjD7hdjhgWVgDh/Buu6PP8AFL4h3kWq2Mlq0VjOsyXCFDHH ARI4bONqkgE9BnmuI+32f/DJPkfa4PO83yPL8wbvM+2eZsx/e2fNjrt56V9D0UAc/P448NW2 uado0urwLfalEk1moDFJkfIQrIBs+YqQOeeMdRnj9F1zS9F+L3xAOq38FjHL/ZwSa5fy4yRb n5d5+UMc5Ck5IDEAhTjvNR0qbU723Fxcx/2ZC8c5tViIeSaN96FpN33AwVtoUElRlipKnm/D eg+JNO8d6/rl/a6UtrrX2fekF9JI8HkxFBgGFQ+4kd1x70AHwwsLy30fWtRvLSez/tfW7vUI ba5jMc0UbsFUSKfut8hOBkYI5qv4I/0z4j/EHVrf57GS7tbJJemZoISsq4PPylgM4wc8E12G sw6pcac9vpF1BZ3UuU+1zR+Z5AwfnWPgO2cAAkAZyc42tJpWmw6RpdvYQNI6QpgySkF5W6s7 kAbnZiWY9ySe9AEeq63p2ifYv7RuPJ+3Xcdlb/Izb5nztXgHGcHk4HvWhRRQB5PrHxT8Capr iWt/r/lafpd2kyiGOdheTpyp3RqVMKNgjk73UHhVHmekaHqya9odlq0VtPbw3kSzRxz7d+xu VJ2sw5GD1784ORWhRQAUUUUAZepaBZ6rcLPcTakjqgQC11K4t1xknlY3UE89cZ6egqn/AMIb pf8Az9a5/wCD29/+PV0FFAEcEK21vFAhkKRoEUySM7EAY5ZiSx9yST3qSiigDn9J/wCJj4o1 jUz80Nts022P3l+Qb5nRu2XcRsB/FbDJJGF5/wAdfEXw7ot4fDd3rf2C6uIj9puYlkd7SNh/ D5YJEzA5XOAv3znCq/oFFAHF+HPFfhDVvCt2liY4/C+momnm5v8AalvIuxR5f7w7iAGVTvAy W43c1z8XgrQ9WuNH0zQdLu/+EXtbj7Tcz3V1LJbSbDuSOCKcsGDswYyooUpvUOd7CvVKKACi iigAooooAKKKKACiiigAooooAKKKKACiiigAooooAKKKKACiiigAooooAKKKKACiiigAoooo AKKKKACiiigAooooAKKKKACiiigAooooAKKKKACiiigAooooAKKKKAPP/Euuazp+ueIxDrcE MOmaImrWtitum+Yr5wYOzEloyUUNtCn5kAZCCX0Nb/t7+2J/sX/CV/Z/l2fYP7L8n7ozt8/9 51znd3zjjFaDeGvtHi241i/axu7Vordbe2kssvBJCzskgkLkbszSchQRlcHg7ugoA4/T/tX/ AAlfh/7b9u+0f2VqO/7f5Hnf6+1xu8j930xjb2xnnNaHgT/knnhr/sFWv/opa1NS0nTdZt1t 9U0+0voFcOsd1CsqhsEZAYEZwSM+5q5QAUUUUAFFFFABRRRQAUUUUAFFFFABRRRQAUUUUAFF FFABRRRQAUUUUAFFFFABRRRQAUUUUAFFFFABRRRQAUUUUAFFFFABRRRQAUUUUAFFFFABRRRQ AUUUUAFFFFABRRRQAUUUUAFFFFABRRRQAUUUUAFFFFABRRRQAUUUUAFFFFABRRRQAUUUUAFF FFABRRRQAUUUUAFFFFABRRRQAUUUUAFFFFABRRRQAUUUUAFFFFABRRRQAUUUUAFFFFABRRRQ AUUUUAFFFFABRRRQAUUUUAFFFFABRRRQAUUUUAFFFFABRRRQAUUUUAFFFFABRRRQAUUUUAFF FFABRRRQAUUUUAFFFFABRRRQAUUUUAFFFFABRRRQAUUUUAFFFFABRRRQAUUUUAFFFFABRRRQ AUUUUAFFFFABRRRQAUUUUAFFFFABRRRQAUUUUAFFFFABRRRQAUUUUAFFFFABRRRQAUUUUAFF FFABRRRQAUUUUAFFFFABRRRQAUUUUAFFFFABRRRQAUUUUAFFFFABRRRQAUUUUAFFFFABRRRQ AUUUUAFFFFABRRRQAUUUUAFFFFABRRRQAUUUUAFFFFABRRRQAUUUUAFFFFABRRRQAUUUUAFF FFABRRRQAUUUUAFFFFABRRRQAUUUUAFFFFABRRRQAUUUUAFFFFABRRRQAUUUUAFFFFABRRRQ AUUUUAFFFFABRRRQAUUUUAFFFFABRRRQAUUUUAFFFFABRRRQAUUUUAFFFFABRRRQAUUUUAFF FFABRRRQAUUUUAFFFFABRRRQAUUUUAFFFFABRRRQAUUUUAFFFFABRRRQAUUUUAFFFFABRRRQ AUUUUAFFFFABRRRQAUUUUAFFFFABRRRQAUUUUAFFFFABRRRQAUUUUAFFFFABRRRQAUUUUAFF FFABRRRQAUUUUAFFFFABRRRQAUUUUAFFFFABRRRQAUUUUAFFFFABRRRQAUUUUAFFFFABRRRQ AUUUUAFFFFABRRRQAUUUUAFFFFABRRRQAUUUUAFFFFABRRRQAUUUUAFFFFABRRRQAUUUUAFF FFABRRRQAUUUUAFFFFABRRRQAUUUUAFFFFABRRRQAUUUUAFFFFABRRRQAUUUUAFFFFABRRRQ AUUUUAFFFFABRRRQAUUUUAFFFFABRRRQAUUUUAFFFFABRRRQAUUUUAFFFFABRRRQAUUUUAFF FFABRRRQAUUUUAFFFFABRRRQAUUUUAFFFFABRRRQAUUUUAFFFFABRRRQAUUUUAFFFFABRRRQ AUUUUAFFFFABRRRQAUUUUAFFFFABRRRQAUUUUAFFFFABRRRQAUUUUAFFFFABRRRQAUUUUAFF FFABRRRQAUUUUAFFFFABRRRQAUUUUAFFFFABRRRQAUUUUAFFFFABRRRQAUUUUAFFFFABRRRQ AUUUUAFFFFABRRRQAUUUUAFFFFABRRRQAUUUUAFFFFABRRRQAUUUUAFFFFABRRRQAUUUUAFF FFABRRRQAUUUUAFFFFABRRRQAUUUUAFFFFABRRRQAUUUUAFFFFABRRRQAUUUUAFFFFABRRRQ AUUUUAFFFFABRRRQAUUUUAFFFFABRRRQAUUUUAFFFFABRRRQAUUUUAFFFFABRRRQAUUUUAFF FFABRRRQAUUUUAFFFFABRRRQAUUUUAFFFFABRRRQAUUUUAFFFFABRRRQAUUUUAFFFFABRRRQ AUUUUAFFFFABRRRQAUUUUAFFFFABRRRQAUUUUAFFFFABRRRQAUUUUAFFFFABRRRQAUUUUAFF FFABRRRQAUUUUAFFFFABRRRQAUUUUAFFFFABRRRQAUUUUAFFFFABRRRQAUUUUAFFFFABRRRQ AUUUUAFFFFABRRRQAUUUUAFFFFABRRRQAUUUUAFFFFABRRRQAUUUUAFFFFABRRRQAUUUUAFF FFABRRRQAUUUUAFFFFABRRRQAUUUUAFFFFABRRRQAUUUUAf/2Q== --------------000508080704040904000101-- --------------010507090903070205060203-- ========================================================================= Instruções para entrar na lista, sair da lista e usar a lista em http://www.mat.puc-rio.br/~nicolau/olimp/obm-l.html O administrador desta lista é ========================================================================= From owner-obm-l@sucuri.mat.puc-rio.br Mon Sep 16 08:46:47 2002 Return-Path: Received: (from majordom@localhost) by sucuri.mat.puc-rio.br (8.9.3/8.9.3) id IAA25779 for obm-l-MTTP; Mon, 16 Sep 2002 08:46:24 -0300 Received: from gorgo.centroin.com.br (gorgo.centroin.com.br [200.225.63.128]) by sucuri.mat.puc-rio.br (8.9.3/8.9.3) with ESMTP id IAA25775 for ; Mon, 16 Sep 2002 08:46:19 -0300 Received: from centroin.com.br (du162c.rjo.centroin.com.br [200.225.58.162]) (authenticated bits=0) by gorgo.centroin.com.br (8.12.2/8.12.1) with ESMTP id g8GBltI8007116 for ; Mon, 16 Sep 2002 08:47:56 -0300 (BRT) Message-ID: <3D85C58E.8070804@centroin.com.br> Date: Mon, 16 Sep 2002 08:50:38 -0300 From: Augusto =?ISO-8859-1?Q?C=E9sar?= Morgado User-Agent: Mozilla/5.0 (Windows; U; Win98; en-US; rv:0.9.4.1) Gecko/20020508 Netscape6/6.2.3 X-Accept-Language: en-us MIME-Version: 1.0 To: obm-l@mat.puc-rio.br Subject: Re: [obm-l] Teorema binomial! References: <000a01c25d72$b173b3c0$2932a8c0@redeconora.com.br> Content-Type: multipart/alternative; boundary="------------040809030205000309000408" Sender: owner-obm-l@sucuri.mat.puc-rio.br Precedence: bulk Reply-To: obm-l@mat.puc-rio.br --------------040809030205000309000408 Content-Type: text/plain; charset=us-ascii; format=flowed Content-Transfer-Encoding: 7bit A sua formula do teorema binomial estah errada. Eh x^p e nao x^m. Os limites sao p=0 e p=m. A soma pedida eh o valor para x=2 e y = 1. Vale, portantp, 3^m. Logo, m=6. Hely Jr. wrote: > O teorema binomial pode ser definido como: > > > --------------040809030205000309000408 Content-Type: multipart/related; boundary="------------080903000307030102040602" --------------080903000307030102040602 Content-Type: text/html; charset=us-ascii Content-Transfer-Encoding: 7bit A sua formula do teorema binomial estah errada. Eh  x^p  e nao  x^m. Os limites sao  p=0  e p=m.

A soma pedida eh o valor para x=2  e  y = 1. Vale, portantp, 3^m. Logo, m=6.


Hely Jr. wrote:
O teorema binomial pode ser definido como:
 

--------------080903000307030102040602 Content-Type: image/jpeg; name="Inbox" Content-Transfer-Encoding: base64 Content-ID: Content-Disposition: inline; filename="Inbox" /9j/4AAQSkZJRgABAQEBLAEsAAD/2wBDAAgGBgcGBQgHBwcJCQgKDBQNDAsLDBkSEw8UHRof Hh0aHBwgJC4nICIsIxwcKDcpLDAxNDQ0Hyc5PTgyPC4zNDL/2wBDAQkJCQwLDBgNDRgyIRwh MjIyMjIyMjIyMjIyMjIyMjIyMjIyMjIyMjIyMjIyMjIyMjIyMjIyMjIyMjIyMjIyMjL/wAAR CAEsAk8DASIAAhEBAxEB/8QAHwAAAQUBAQEBAQEAAAAAAAAAAAECAwQFBgcICQoL/8QAtRAA AgEDAwIEAwUFBAQAAAF9AQIDAAQRBRIhMUEGE1FhByJxFDKBkaEII0KxwRVS0fAkM2JyggkK FhcYGRolJicoKSo0NTY3ODk6Q0RFRkdISUpTVFVWV1hZWmNkZWZnaGlqc3R1dnd4eXqDhIWG h4iJipKTlJWWl5iZmqKjpKWmp6ipqrKztLW2t7i5usLDxMXGx8jJytLT1NXW19jZ2uHi4+Tl 5ufo6erx8vP09fb3+Pn6/8QAHwEAAwEBAQEBAQEBAQAAAAAAAAECAwQFBgcICQoL/8QAtREA AgECBAQDBAcFBAQAAQJ3AAECAxEEBSExBhJBUQdhcRMiMoEIFEKRobHBCSMzUvAVYnLRChYk NOEl8RcYGRomJygpKjU2Nzg5OkNERUZHSElKU1RVVldYWVpjZGVmZ2hpanN0dXZ3eHl6goOE hYaHiImKkpOUlZaXmJmaoqOkpaanqKmqsrO0tba3uLm6wsPExcbHyMnK0tPU1dbX2Nna4uPk 5ebn6Onq8vP09fb3+Pn6/9oADAMBAAIRAxEAPwD1/wD4Tvwf/wBDXof/AIMYf/iqkg8aeFbq 4it7fxLo008rhI447+JmdicAABskk8YrD1zw7ceHry98V+Gbv7NMN1zf6XPcCOxvBjMjnPEU xCqfN6ZX5hhmNdBcWFn4p0fTZ7y0ng2y22oQxzxhZoJEZZFBBztbgqwHOCwzzQBny+O9OhvL +FrDVfJsLtbO6uRZtsSRgmwKv35NzSKo2Kx5BICsrNsaTqyarFP/AKNPa3FtL5NxbT7d8T7V cAlGZTlHRvlY/ewcEEDn7zwnrFz/AGr5etWMf27VbbUk3ac7eX5Pl7UP74bs+RFk8fx8fMNu xo+k3mnanrV1c3sFxHqN2LmOOO2MZhxGke0sXbd8sac4XncehAABsVXvr+z0yzkvL+7gtLWP G+aeQRouSAMseBkkD8asV5/4Q/4qjxp4j8R3/wC8bStQl0fTYH+YWixgCWRTwN0pYZONwA27 iOKAOssvEug6kkj2Gt6bdJG8aO0F0jhWdtqA4PBZuAO54FWNS1Wx0i3We/uY4EdxHGG5aVyC QiKOXc4OFUEnsDWP4m8F6b4lvdK1CVI4NQ029t7qK6WFS7LE+7ymbrsOW4B4ODzjBy/DV1/w lfjTXdXubeBrXQ7t9K0zen72GQAfaXPUfOSgBznauMLltwB3FFcHa+PtY1HW9f0ew8G3bXml JC2y4voIwxkQuFcqWCkgYXbvBJG4oMkUx8VZX8CQ+Ml8NzppC7PtLTXaB+ZRE3kKu7zNrbvv +VnAx1OAD0iiiuH07xvrOs+I9a0Sx8LeRcaX5BkbUNQSMYkQsAfKWXDegG4EZJKkBSAdZqWq 2Oj263Oo3Mdrblwhnl+WNCQSN79EBxgFiASQOpANyub8K+JLHx14alnewkhId7PUNPu48mGU ACSJgRhhhvTkHkA5Az/AmpTRX+v+ErhY8+H7iOO1MQO0WcibrdCScl1UbScdhyxySAdpRRUc 5mW3la3jjknCExpI5RWbHALAEgZ74OPQ0ASVh6r4nh0zWbfSE07Ur6/uLd7iKO1gG0qjKrAy OVQH5s8sOgBwWQNz8fj7UkuNW0u90O0tvEFq6Cx0w6gzNqKsT88beSBswDyAdu1vM8sAmtht K1681mx1mS502xnt7K4tTarE90u6RlIfzN0ZIBiiO3YP4xnkMACxo/iqx1u4hjtYbtIrq3N1 ZzzReWt1EpUM6A/OADIg+dV3bgV3DmtyuT0TwpqWk3Hh5pNWtJ4NI0x9OKLYsjTKxTDbvNIU gQxdjnD9Nw29ZQAVHPMttbyzuJCkaF2EcbOxAGeFUEsfYAk9q5Pxn4v1TwtqOjQ22hQX1rql 3HYpcPf+T5U7nChl8tjtI53DPQ5A4zoanq2sL4hi0bSbKxk8y0NxJdz3L/6N84UbolQ7s5JU F03+XIMrtzQBTsvHttqNvZS2mi6zI9/bi5sYjAitcoAu8jc4CBN6gmQoGyNhfIz0lhfW+p6d bX9nJ5lrdRJNC+0jcjAFTg8jII61y+l+E9Y0v+w/L1qxf+ydKl01N2nP+83bNrn99xjyYsjv h+RuG3Q0fwpZ2eh6JY6pBY6ndaPEkVtdvZgFNmArKGLFGwqZIPJGeOAACPSfGljrFxAkNlqU ENxcTWkNxdW/lK88RfdGFY787Y3bJXaMFSQ4KDpK4PwhpdzqBS+uLi7itLTWL+8gtLrT3t5V eSScKQzhSYjFPuxtJ3sfnwNg7ygAooooAKKKKACiiigAooooAKKKKACiiigAooooAKKKKACi iigAooooAKKKKACiiigAooooAKKKKACiiigAooooAKKKKACiiigAooooAKKKKACiiigAoooo AKKKKACiiigDh9U8WeG9R1y70bUPEGlW2m2W1LuKW9jQ3UxzuhYMQfLQBdwAIctsJwkiNc1X 4jeGbHS7i4sdZ03U7xUxbWNpdrLLcynhI1VNzZZiBkA4zk8Crn/CCeD/APoVND/8F0P/AMTU kHgvwra3EVxb+GtGhnicPHJHYRKyMDkEELkEHnNAG5RRRQAVw9lYXngrxRrVzHaT3XhzV5ft zfZIzLJZXRAWTMYzJIshwcpnaQRtVctXcUUAYcGt3OrXEUel6fdx24cGa8v7V4FVQcsixvtk ZyMANtCDJO5iuw4fgS2/sfxH400Z1naQ6r/aqzPFsSSO5QEBeTu2tHIpPT5fXIHcVl6roq6h cW9/BPJa6naI62twpYqocqWV48gSI2xcg88ZUq2GABxfhO6ZfiP471aTT9VisbyKzmt5ZtMu I/NWGErIFDICWBIAXG49ga4/+ztT/wCGYf7C/sfVf7W83yfsf9nzeZu+1ednbt+7s53fdzxn PFe8UUAc3e+M7TT9Z02wm0vWfIv0iKagLFxbxNIxVEkY4ZHLbRgrwXXOOccnourxaT8XviA1 xa30sMn9nDzLS1e42MLc4BWMM/Izzt2/KQSCVB9AutJS+1GC5ubmeSCHayWR2+T5qnKykbdz MOwLFQQrBdyhhz+geE9Y0jxfq2v3OtWNz/a3k/a4I9OeLHlRlE2MZm29cnIbOOMUAHw60TUd I0fU7nVrf7JfatqtzqUlnvWT7N5jABN6kh+FByMfexjiqfgiCa68b+OvEAikis7y9hsoBKpV na1QxSOOxQtkAg/wnIBGK6zWdPuNV057K31GfT1lyss9sAJghB4jY5CNnHzYOBnGCQy2LCxt 9M062sLOPy7W1iSGFNxO1FACjJ5OAB1oAr6rqv8AZf2L/QL68+1Xcdr/AKJD5nk7s/vJORtj GOW7ZFXJ5ltreWdxIUjQuwjjZ2IAzwqglj7AEntUlFAHkevxarr1k3j/AEnT7uw1zR7gLp9n e6XJ58kGzbJFIiqWYu0j4ZSyqoXBjYyMPTNG1eLW9OS8itb61zgNDfWr28iNgEgq4GcZxlcr kHBOK0KKACq9/dfYdOubz7PPceRE8vk26b5JNoJ2ovdjjAHc1YooA5/W9Jt/HPgafT7qGe0j 1O0VglxGVkt3IDoXQEfMjBSVz1XB4rP+Hn9o6hoaeINa51W+ijhkI24EcOVUqU+VldzLMGUY ImABKqpqx4k8PeIta1GM6d4vn0jTGiEVxa29nG0knJ3Mkx+aNipABAO0jNdBYWNvpmnW1hZx +Xa2sSQwpuJ2ooAUZPJwAOtAFiiiigAooooAKKKKACiiigAooooAKKKKACiiigAooooAKKKK ACiiigAooooAKKKKACiiigAooooAKKKKACiiigAooooAKKKKACiiigAooooAKKKKACiiigAo oooAKKKKACiiigAooooAKKKKAPM/F19NZeJdfX+3dSiuF0eC40izSUpE15m4CIoUDzHZkXET FvMy2VcIoT0ysuDRRB4lvNb+33bvdW8du1swj8pFjLFSuED5y8h5Y/fPouNSgAooooAKKKKA CiiigAooooAKKKKACiiigAooooAKKKKACiiigAooooAKKKKACiiigAooooAKKKKACiiigAoo ooAKKKKACiiigAooooAKKKKACiiigAooooAKKKKACiiigAooooAKKKKACiiigAooooAKKKKA CiiigAooooAKKKKACiiigAooooAKKKKACiiigAooooAKKKKACiiigAooooAKKKKACiiigAoo ooAKKKKACiiigAooooAKKKKACiiigAooooAKKKKACiiigAooooAKKKKACiiigAooooAKKKKA CiiigAooooAKKKKACiiigAooooAKKKKACiiigAooooAKKKKACiiigAooooAKKKKACiiigAoo ooAKKKKACiiigAooooAKKKKACiiigAooooAKKKKACiiigAooooAKKKKACiiigAooooAKKKKA CiiigAooooAKKKKACiiigAooooAKKKKACiiigAooooAKKKKACiiigAooooAKKKKACiiigAoo ooAKKKKACiiigAooooAKKKKACiiigAooooAKKKKACiiigAooooAKKKKACiiigAooooAx77xR pOnXkltcTThosefLHaSyQwZAP72VVKR4UhjvYYUhjgEGtCwvrfU9Otr+zk8y1uokmhfaRuRg CpweRkEda5fR9c0vw7bXWmavfwWmof2heTJaSP8AvpkluZJIzEg+aXcrrjYG+bK/eBAx/E97 9h+1RLNPpE1vokUmgWCXPkbrz99mFY422TspW3Xy/nUZAAw/zAHpFFeX+LdWSz1zxfL/AMJT PFNpmiQXlpZC9WNIbn98Fyi4LcmHKOWDecoYMPLC3P7Za2fxVp+g65Jqd5FoUeoWY+0LdSvc OsxLovIwQLchFAQb12qA/IB6JWfq+sW+iwW0tyk7rcXcNonkxF8PK4RSx6KuTySR6DJIBw/C l1aS6zqkOj6nJqOipb27Rzm+e8UXBaYSqJWZjkKsJKbsDIOBuJPN+LdRii1a8N/qF8l1b63p v2Wyh3yCWzElq5byFByvmmQCULuLqIwx+4QD1Co4J4bq3iuLeWOaCVA8ckbBldSMggjggjnN ed6P9qv/ABbdJfeJ9txJd3kM2kReeJ1tg0ixFgJtsC7BE6yrEhOUG8mTL8/pk0J8G+EYLbxJ /ZelPp5e7vL26uRCbtY7cLEkoniIwGmxGr7AUkG3ch2gHtFFeb2UqXnii/ttS8aXxay0Syu3 AmWz8mUCXzJmhwCmBsdkkyv70B1ICBcvw9qNjc2/heC/8Y3bJqOhSX2ol9W2NI8QiCjcCDEF /f5MZQt5TeYX2vkA9YMjC4SIQyFGRmMoK7VIIwp5zk5JGAR8pyRxmSvM9H8VRahceGJ7zxZH E994XnmuQs8KrHOphLTbSMbxif7wKr5T4Aw+afhvxCltp2hY8Q32oXmo+Gp7u4jS6W5uZJ4h EEEMbZUSD/SFAC/OYyX3spNAHrFRiRjcPEYZAiorCUldrEk5Uc5yMAnIA+YYJ5x4vN4rhiF5 YR+L7TToluNLliaz8QDUC2+5ZJ/31wu7AQIzIPlAUE8O4bsLa8h0nxL4o0q21jUrv7Lo8FxD Zpci6uoWzcFxEsm4s5zEfn3cugPy7VAB3lFef+AtdiuvEOq6VHqdjcQx2lvcQxW2tvqfzF5V kbzZAHHAiBXlVyp4LnJqeoOnii7Q6jPHq6arZxafZC6ZfNsmFv5zC3B2yqN11mQqxXa3I8sb QD0Ciub8XRGFNM1f7RdxxadexvcRw3MkKPC7BHMm1gNke5ZSWBGImBwGY1ycrNpXhfRv7Rur 6Br/AM29kGq69cWMcDOVZYHuPmk8xFbYsfyq4SV2G4DAB6Bbaxb3WuX+kRpOLixihllZ4iqM Jd+3aT97/VnJHHbOQQNCvJ9D8VS6vp2sX2m6nBe+IT4PspgtsUkkN0ouSw8tc/MHdMrjgsox yBRrGr2drbXX9j+L57fRVu9LEd8mpC5Bme5cXEYmmL9IhG5TOABkjazBgD1is+21i3utcv8A SI0nFxYxQyys8RVGEu/btJ+9/qzkjjtnIIFPV4bnRvBWqJopu5Ly3sp3tN8j3Upl2sy8yFmc 7sYBz2HTiuLttRih1HxifC2oX2pTJ4fhlsJvnvF8xDdYWOZg3m/OwwCzktvUcIVUA9Qorj/A uB9vEHieDXLX93tNt50scT/Nu/fSzTFmI25QP8oCnaN+W5fUdZgtrzxJdt40njmtfEFrZwZv oglrHKIFlAjxsbC+eB5isF8lmwGDsQD1io55ltreWdxIUjQuwjjZ2IAzwqglj7AEntXN+EdQ t5b7xFpVvqP2uPTNQEMaPdGeSJDDGxDOxLn96Zh8xOCpXouBzf8Awka2vxDsYDqMcBudTmtJ 4bvV2M7J5cvlj7GVEcSF0i2SKdzqY85MjZAPQNJ1KHWdGsdUt1kWC9t47iNZAAwV1DAHBIzg +pqxPI0NvLKkMk7ohZYoyoZyB90biBk9OSB6kV538O5PI/sLT7bUp7to9E26tA8/mfY7hPIW KJkHEDAGddoClthLbmUmuX1Pxd9ms9Wij8Rf2dJcaJeT/vNb868F0hiMaSROoW1mO6RTFF3L BcGNSAD3CivJ7/WfDs2sXjReLp/7Ek1C2N3dxa1IPs8zK5WGIiYYhf8AiKqQuWAyFLW2xqeo Onii7Q6jPHq6arZxafZC6ZfNsmFv5zC3B2yqN11mQqxXa3I8sbQD0Ciq9mbcwMbWfzo/NkBb zjLh97b1yScYbcu3+HG0AAYFigAooooAKKKKACiiigAooooAKKKKACiiigAooooAKKKKACii igAooooAKKKKACiiigAooooAKKKKACiiigAooooAKKKKACiiigAooooAKKKKACiiigAooooA KKKKACiiigAooooAKKKKACiiigAooooAKKKKACiiigAooooAKKKKACiiigAooooAKKKKACii igAooooAKKKKACiiigAooooAKKKKACiiigAooooAKKKKACiiigAooooAKKKKACiiigAooooA KKKKACiiigAooooAKKKKACiiigAooooAKKKKACiiigAooooAKK8r1X4cWXhrVLjXdJ8P6bq+ ku/m3ehyadFJKufvyW0jDORgEQn5TlwuCVA6A+B/CGuWGkaro2labZlLi11K0u7WyWJmVXWQ AgBThlyMN03A4yBQBqP448NR3l1bPq8AazlMN1JhvKtnABxLJjZHknaNxGWDKMspA1NN1S01 a3aa0eQhHKSJLE8UkbYBw6OAynBBAIGQwPQg1yd9oPiSb+2vItdKP2vW7TUIN99Iv7uDychs QnDH7MvAyB5h5+QbtzQrDUrPVvEE97FaJBfXq3FsYZ2dtohjiw4KKFOIg3Bb7xH8OSAblFFe f6R/xXnijxBcan++0DSbttLttNk/1csyBTNLPHysnzbfLySABnaG5oA9AqOeeG1t5bi4ljhg iQvJJIwVUUDJJJ4AA5zXF6z8N9OkutOu/D0MGjzQahaXF1FbboYbuGGXfseNCELAncGKk5UD IByDT5rfxv40v57i183TPDN2LeyYyHZLegZldozj5ovlVCRgbnYE5G0A7iiuLj+JemXN/q1h ZaR4gu7vTUR5IY9MkVpAyF+A+3aQoPEmwt0TcSAYx8VNCfw5D4hitNVfSDs+0XZtCiW29wnz biPMwxIPleZjac9sgHcUUVx9p8Q7PU9Y1HStL0TXLy70/wAozJ9kFv8AK67gf37JjsMNhjnK hlBIAOsknhheFJZY0eZ9kSswBdtpbC+p2qxwOwJ7VJXN6bqXhz4leDmkjWO+0u9Qxz28ow0b cEo4ByrqcEEH0ZT0NV/BWtzXL6t4dv8AzP7Q0G4FszyyGR57dlzBM7dC7oPm5zkEkLkAAHWU UUUAFZer+I9I0F7SPU76OCW8dktosFnmYLuIRVBLHoAAOSyqMllB878V/DTSV8Jat4mvxOPF sNpJqD6jbXkuY7hFMgEeTgRqVCKMZCAc5Ga2NZv9RtLL4fx6jd2MXiF7tDLHeSKqyzfY5lkX 5OmXcJuUMAzrhW4UgHcWN9b6jZx3VrJ5kL5AJUqQQSGVlOCrAggqQCCCCARVisvw/ps2laT5 Fw0ZnkuLi6kEZJVGmmeUoCQCwUvt3YGcZwM4GpQAUVl6lqt5Y3CxW/h/UtQQoGMtrJbqoOT8 p8yVDnjPTHI564p/8JDqn/Qma5/3+sv/AJIoA6Cio4JGmt4pXhkgd0DNFIVLISPunaSMjpwS PQms/wAR6Q2v+H7zSFvJLRLxBDLLGis3lEjzFG7gFk3LnBxuzg4oA1KK8rvvhN4HtPEGhWdh oMYnluGuZt88smLeEZY7XYoQZGgQqQSVkbA43L0mt/D7SfF2sT3nii2+2wxbYtPhS5lRIo9o LsVUj94zlgTyNqR9CDkA7CivG4NV1/wP8GPE08FzJL/ZepzWOkTXe2VltlnWBScdSreZgMMc DjbgVseNLPRfCPhiLxz4et40u7B4ZfPtJR/xMYJZI1kSaQhvNDqd29tzBgGBznIB6ZRRRQAU UUUAFFFFABRRRQAUUUUAFFFFABRRRQAUUUUAFFFFABRRRQAUUUUAFFFFABRRRQAUUUUAFFFF ABRRRQAUUUUAFFFFABRRRQAUUUUAFFFFABRRRQAUUUUAc/qn/FQ3l34eXiwSJV1OUcl1kB/0 dSPusV5c8MqOm0ZkDpoa5rNn4e0O91e/fZa2kTSvggFsdFXJALE4AGeSQK5Of4M+Arq4luLj RJJp5XLySSX9yzOxOSSTJkknnNSWHwg8CaZqNtf2eheXdWsqTQv9rnO11IKnBfBwQOtAHcUU UUAFef6R/wAUH4o8QW+p/udA1a7bVLbUpP8AVxTOFE0U8nCx/Nt8vIAIONxbiu01LVtN0a3W 41TULSxgZwiyXUyxKWwTgFiBnAJx7GrlAGHB4osdUuIoNCkj1Xc482e2fdbwpn5i0wBTeBjE YJYllyApLjn/AIdxrp2t+ONHlmje8TXZL50QNhYrhEeM5IAzgMCB0KntgnvKx9T0m4k1GLV9 MufJv4YjE0L4EN2mQQkpClhj5tjD7hdjhgWVgDh/Buu6PP8AFL4h3kWq2Mlq0VjOsyXCFDHH ARI4bONqkgE9BnmuI+32f/DJPkfa4PO83yPL8wbvM+2eZsx/e2fNjrt56V9D0UAc/P448NW2 uado0urwLfalEk1moDFJkfIQrIBs+YqQOeeMdRnj9F1zS9F+L3xAOq38FjHL/ZwSa5fy4yRb n5d5+UMc5Ck5IDEAhTjvNR0qbU723Fxcx/2ZC8c5tViIeSaN96FpN33AwVtoUElRlipKnm/D eg+JNO8d6/rl/a6UtrrX2fekF9JI8HkxFBgGFQ+4kd1x70AHwwsLy30fWtRvLSez/tfW7vUI ba5jMc0UbsFUSKfut8hOBkYI5qv4I/0z4j/EHVrf57GS7tbJJemZoISsq4PPylgM4wc8E12G sw6pcac9vpF1BZ3UuU+1zR+Z5AwfnWPgO2cAAkAZyc42tJpWmw6RpdvYQNI6QpgySkF5W6s7 kAbnZiWY9ySe9AEeq63p2ifYv7RuPJ+3Xcdlb/Izb5nztXgHGcHk4HvWhRRQB5PrHxT8Capr iWt/r/lafpd2kyiGOdheTpyp3RqVMKNgjk73UHhVHmekaHqya9odlq0VtPbw3kSzRxz7d+xu VJ2sw5GD1784ORWhRQAUUUUAZepaBZ6rcLPcTakjqgQC11K4t1xknlY3UE89cZ6egqn/AMIb pf8Az9a5/wCD29/+PV0FFAEcEK21vFAhkKRoEUySM7EAY5ZiSx9yST3qSiigDn9J/wCJj4o1 jUz80Nts022P3l+Qb5nRu2XcRsB/FbDJJGF5/wAdfEXw7ot4fDd3rf2C6uIj9puYlkd7SNh/ D5YJEzA5XOAv3znCq/oFFAHF+HPFfhDVvCt2liY4/C+momnm5v8AalvIuxR5f7w7iAGVTvAy W43c1z8XgrQ9WuNH0zQdLu/+EXtbj7Tcz3V1LJbSbDuSOCKcsGDswYyooUpvUOd7CvVKKACi iigAooooAKKKKACiiigAooooAKKKKACiiigAooooAKKKKACiiigAooooAKKKKACiiigAoooo AKKKKACiiigAooooAKKKKACiiigAooooAKKKKACiiigAooooAKKKKAPP/Euuazp+ueIxDrcE MOmaImrWtitum+Yr5wYOzEloyUUNtCn5kAZCCX0Nb/t7+2J/sX/CV/Z/l2fYP7L8n7ozt8/9 51znd3zjjFaDeGvtHi241i/axu7Vordbe2kssvBJCzskgkLkbszSchQRlcHg7ugoA4/T/tX/ AAlfh/7b9u+0f2VqO/7f5Hnf6+1xu8j930xjb2xnnNaHgT/knnhr/sFWv/opa1NS0nTdZt1t 9U0+0voFcOsd1CsqhsEZAYEZwSM+5q5QAUUUUAFFFFABRRRQAUUUUAFFFFABRRRQAUUUUAFF FFABRRRQAUUUUAFFFFABRRRQAUUUUAFFFFABRRRQAUUUUAFFFFABRRRQAUUUUAFFFFABRRRQ AUUUUAFFFFABRRRQAUUUUAFFFFABRRRQAUUUUAFFFFABRRRQAUUUUAFFFFABRRRQAUUUUAFF FFABRRRQAUUUUAFFFFABRRRQAUUUUAFFFFABRRRQAUUUUAFFFFABRRRQAUUUUAFFFFABRRRQ AUUUUAFFFFABRRRQAUUUUAFFFFABRRRQAUUUUAFFFFABRRRQAUUUUAFFFFABRRRQAUUUUAFF FFABRRRQAUUUUAFFFFABRRRQAUUUUAFFFFABRRRQAUUUUAFFFFABRRRQAUUUUAFFFFABRRRQ AUUUUAFFFFABRRRQAUUUUAFFFFABRRRQAUUUUAFFFFABRRRQAUUUUAFFFFABRRRQAUUUUAFF FFABRRRQAUUUUAFFFFABRRRQAUUUUAFFFFABRRRQAUUUUAFFFFABRRRQAUUUUAFFFFABRRRQ AUUUUAFFFFABRRRQAUUUUAFFFFABRRRQAUUUUAFFFFABRRRQAUUUUAFFFFABRRRQAUUUUAFF FFABRRRQAUUUUAFFFFABRRRQAUUUUAFFFFABRRRQAUUUUAFFFFABRRRQAUUUUAFFFFABRRRQ AUUUUAFFFFABRRRQAUUUUAFFFFABRRRQAUUUUAFFFFABRRRQAUUUUAFFFFABRRRQAUUUUAFF FFABRRRQAUUUUAFFFFABRRRQAUUUUAFFFFABRRRQAUUUUAFFFFABRRRQAUUUUAFFFFABRRRQ AUUUUAFFFFABRRRQAUUUUAFFFFABRRRQAUUUUAFFFFABRRRQAUUUUAFFFFABRRRQAUUUUAFF FFABRRRQAUUUUAFFFFABRRRQAUUUUAFFFFABRRRQAUUUUAFFFFABRRRQAUUUUAFFFFABRRRQ AUUUUAFFFFABRRRQAUUUUAFFFFABRRRQAUUUUAFFFFABRRRQAUUUUAFFFFABRRRQAUUUUAFF FFABRRRQAUUUUAFFFFABRRRQAUUUUAFFFFABRRRQAUUUUAFFFFABRRRQAUUUUAFFFFABRRRQ AUUUUAFFFFABRRRQAUUUUAFFFFABRRRQAUUUUAFFFFABRRRQAUUUUAFFFFABRRRQAUUUUAFF FFABRRRQAUUUUAFFFFABRRRQAUUUUAFFFFABRRRQAUUUUAFFFFABRRRQAUUUUAFFFFABRRRQ AUUUUAFFFFABRRRQAUUUUAFFFFABRRRQAUUUUAFFFFABRRRQAUUUUAFFFFABRRRQAUUUUAFF FFABRRRQAUUUUAFFFFABRRRQAUUUUAFFFFABRRRQAUUUUAFFFFABRRRQAUUUUAFFFFABRRRQ AUUUUAFFFFABRRRQAUUUUAFFFFABRRRQAUUUUAFFFFABRRRQAUUUUAFFFFABRRRQAUUUUAFF FFABRRRQAUUUUAFFFFABRRRQAUUUUAFFFFABRRRQAUUUUAFFFFABRRRQAUUUUAFFFFABRRRQ AUUUUAFFFFABRRRQAUUUUAFFFFABRRRQAUUUUAFFFFABRRRQAUUUUAFFFFABRRRQAUUUUAFF FFABRRRQAUUUUAFFFFABRRRQAUUUUAFFFFABRRRQAUUUUAFFFFABRRRQAUUUUAFFFFABRRRQ AUUUUAFFFFABRRRQAUUUUAFFFFABRRRQAUUUUAFFFFABRRRQAUUUUAFFFFABRRRQAUUUUAFF FFABRRRQAUUUUAFFFFABRRRQAUUUUAFFFFABRRRQAUUUUAFFFFABRRRQAUUUUAFFFFABRRRQ AUUUUAFFFFABRRRQAUUUUAFFFFABRRRQAUUUUAFFFFABRRRQAUUUUAFFFFABRRRQAUUUUAFF FFABRRRQAUUUUAFFFFABRRRQAUUUUAFFFFABRRRQAUUUUAf/2Q== --------------080903000307030102040602-- --------------040809030205000309000408-- ========================================================================= Instruções para entrar na lista, sair da lista e usar a lista em http://www.mat.puc-rio.br/~nicolau/olimp/obm-l.html O administrador desta lista é ========================================================================= From owner-obm-l@sucuri.mat.puc-rio.br Mon Sep 16 10:11:16 2002 Return-Path: Received: (from majordom@localhost) by sucuri.mat.puc-rio.br (8.9.3/8.9.3) id KAA28208 for obm-l-MTTP; Mon, 16 Sep 2002 10:08:45 -0300 Received: (from nicolau@localhost) by sucuri.mat.puc-rio.br (8.9.3/8.9.3) id KAA28203 for obm-l@mat.puc-rio.br; Mon, 16 Sep 2002 10:08:44 -0300 Date: Mon, 16 Sep 2002 10:08:44 -0300 From: "Nicolau C. Saldanha" To: obm-l@mat.puc-rio.br Subject: Re: [obm-l] Um Estranho Sentimento ... Message-ID: <20020916100844.B27807@sucuri.mat.puc-rio.br> References: Mime-Version: 1.0 Content-Type: text/plain; charset=iso-8859-1 Content-Disposition: inline Content-Transfer-Encoding: 8bit User-Agent: Mutt/1.2.5i In-Reply-To: ; from p_ssr@hotmail.com on Sun, Sep 15, 2002 at 03:06:01PM +0000 Sender: owner-obm-l@sucuri.mat.puc-rio.br Precedence: bulk Reply-To: obm-l@mat.puc-rio.br On Sun, Sep 15, 2002 at 03:06:01PM +0000, Paulo Santa Rita wrote: > Em minha modesta investigacao pessoal eu me fiz estas perguntas abaixo, > inspirado sobretudo pela memoria original do Gauss sobre a Geometria > intrinseca das Superficies ... Parece que os Axiomas do Corpo Ordenado > Completo surgirao MUITO MAIS COMO UMA CONSTRUCAO para justificar os fatos > numericos que ja sabiamos, assim como a Geometria Euclidiana surgiu como uma > formalizacao dos fatos geometricos que os povos de entao ja usavam ... E > portanto so novos fatos e fenomenos, talvez de coerencia interna, nos > permitam olhar os axiomas do corpo como UMA FORMALIZACAO possivel, nao como > A FORMALIZACAO POSSIVEL ... Me parece que os fatos ... Só tem vagamente a ver, mas acho que pode interessar. Os axiomas de corpo ordenado completo (conforme apresentados, por exemplo, no livro do Elon do projeto Euclides) são muito desiguais. Os axiomas de corpo ordenado são axiomas no sentido de lógica de primeira ordem e dispensam outra teoria anterior; dispensam até os números naturais ou inteiros. O axioma da completude é algo totalmente diferente: ele diz que todo *conjunto* de números reais satisfaz uma certa propriedade. Este último axioma portanto pressupõe algum tipo de teoria de conjuntos. Ora, se você tem uma teoria de conjuntos você pode construir o conjunto dos números reais (e muito mais) e portanto não precisa de axiomas, precisa de uma construção ou uma definição. Os axiomas a meu ver são mais corretamente pensados como um teorema de caracterização e uma proposta pedagógica. A proposta pedagógica é nunca usar em análise os detalhes da construção dos reais (via cortes de Dedekind, seqüências de Cauchy ou outro recurso similar qualquer) e sim apenas os "axiomas" ou suas conseqüências. Vale aliás o mesmo para os naturais: o quinto axioma de Peano fala de conjuntos de naturais. []s, N. ========================================================================= Instruções para entrar na lista, sair da lista e usar a lista em http://www.mat.puc-rio.br/~nicolau/olimp/obm-l.html O administrador desta lista é ========================================================================= From owner-obm-l@sucuri.mat.puc-rio.br Mon Sep 16 10:45:59 2002 Return-Path: Received: (from majordom@localhost) by sucuri.mat.puc-rio.br (8.9.3/8.9.3) id KAA29011 for obm-l-MTTP; Mon, 16 Sep 2002 10:44:51 -0300 Received: from hotmail.com (f125.law9.hotmail.com [64.4.9.125]) by sucuri.mat.puc-rio.br (8.9.3/8.9.3) with ESMTP id KAA29007 for ; Mon, 16 Sep 2002 10:44:48 -0300 Received: from mail pickup service by hotmail.com with Microsoft SMTPSVC; Mon, 16 Sep 2002 06:45:44 -0700 Received: from 143.107.45.30 by lw9fd.law9.hotmail.msn.com with HTTP; Mon, 16 Sep 2002 13:45:43 GMT X-Originating-IP: [143.107.45.30] From: "Rogerio Fajardo" To: obm-l@mat.puc-rio.br Subject: Re: [obm-l] Incompletudes de Boltman Date: Mon, 16 Sep 2002 13:45:43 +0000 Mime-Version: 1.0 Content-Type: text/plain; format=flowed Message-ID: X-OriginalArrivalTime: 16 Sep 2002 13:45:44.0305 (UTC) FILETIME=[5AACFE10:01C25D87] Sender: owner-obm-l@sucuri.mat.puc-rio.br Precedence: bulk Reply-To: obm-l@mat.puc-rio.br >From: "Wagner" >Reply-To: obm-l@mat.puc-rio.br >To: >Subject: [obm-l] Incompletudes de Boltman Isso me parece o Teorema de Godel, mas falta detalhes no enunciado. Na forma como está enunciado o teorema é falso. Precisamos supor que o conjunto é consistente (senão prova-se tudo), é recursivo (existe um algoritmo que permite-nos saber se uma sentença está ou não no conjunto), a linguagem e axiomática são finitárias (não tem fórmulas ou demonstrações infinitas), e a teoria é capaz de expressar a aritmética. Talvez Boltman tenha feito alguma verão um pouco mais geral do Teorema de Godel. Godel supôs que a teoria fosse w-consistente (omega-consistente), isto é, se for verdade que "existe um natural n tal que P(n)", então não pode ocorrer que, para cada n fixado, seja verdade "não P(n)". Parece-me que houve um matemático que eliminou essa hipótese. Não sei se foi ele (se alguém souber quem é, gostaria de saber). >Date: Sun, 15 Sep 2002 15:28:51 -0300 > >Oi para todos! > >Eu queria saber porque o Teorema das Incompletudes de Boltman é verdadeiro > >O teorema é o seguinte: Em um conjunto de axiomas sempre persistem questões >que não podem ser negadas, nem comprovadas por esses mesmos axiomas. > >André T. _________________________________________________________________ Send and receive Hotmail on your mobile device: http://mobile.msn.com ========================================================================= Instruções para entrar na lista, sair da lista e usar a lista em http://www.mat.puc-rio.br/~nicolau/olimp/obm-l.html O administrador desta lista é ========================================================================= From owner-obm-l@sucuri.mat.puc-rio.br Mon Sep 16 11:24:34 2002 Return-Path: Received: (from majordom@localhost) by sucuri.mat.puc-rio.br (8.9.3/8.9.3) id LAA30039 for obm-l-MTTP; Mon, 16 Sep 2002 11:22:42 -0300 Received: from salmon.bol.com.br (salmon.bol.com.br [200.221.24.15]) by sucuri.mat.puc-rio.br (8.9.3/8.9.3) with ESMTP id LAA30012 for ; Mon, 16 Sep 2002 11:22:32 -0300 Received: from bol.com.br (200.221.24.137) by salmon.bol.com.br (5.1.071) id 3D63EEF600835C4D for obm-l@mat.puc-rio.br; Mon, 16 Sep 2002 11:23:00 -0300 Date: Mon, 16 Sep 2002 11:21:22 -0300 Message-Id: Subject: [obm-l] =?iso-8859-1?q?Problema_4_OBM_universit=E1ria?= MIME-Version: 1.0 Content-Type: text/plain;charset="iso-8859-1" From: "fredericogomes" To: obm-l@mat.puc-rio.br X-XaM3-API-Version: 2.4.3.4.4 X-SenderIP: 150.161.4.32 Content-Transfer-Encoding: 8bit X-MIME-Autoconverted: from quoted-printable to 8bit by sucuri.mat.puc-rio.br id LAA30028 Sender: owner-obm-l@sucuri.mat.puc-rio.br Precedence: bulk Reply-To: obm-l@mat.puc-rio.br Pessoal, consegui encontrar usando ajuda de um software que para m = 4 + 6k o polinômio é divisível, se alguém souber mostrar isso, gostaria de uma ajuda! PROBLEMA 4 Determine todos os valores inteiros positivos de m para os quais o polinômio (x+1)^m + x^m +1 é divisível por (x^2 + x + 1)^2. __________________________________________________________________________ AcessoBOL, só R$ 9,90! O menor preço do mercado! Assine já! http://www.bol.com.br/acessobol ========================================================================= Instruções para entrar na lista, sair da lista e usar a lista em http://www.mat.puc-rio.br/~nicolau/olimp/obm-l.html O administrador desta lista é ========================================================================= From owner-obm-l@sucuri.mat.puc-rio.br Mon Sep 16 12:11:48 2002 Return-Path: Received: (from majordom@localhost) by sucuri.mat.puc-rio.br (8.9.3/8.9.3) id MAA31287 for obm-l-MTTP; Mon, 16 Sep 2002 12:10:46 -0300 Received: from smtp.ieg.com.br (stone.protocoloweb.com.br [200.226.139.11]) by sucuri.mat.puc-rio.br (8.9.3/8.9.3) with ESMTP id MAA31283 for ; Mon, 16 Sep 2002 12:10:42 -0300 Received: from localhost (200-158-118-125.dsl.telesp.net.br [200.158.118.125]) by smtp.ieg.com.br (IeG relay/8.9.3) with SMTP id g8GF7cO3053936 for ; Mon, 16 Sep 2002 12:07:39 -0300 (BRT) From: Tonik To: obm-l@mat.puc-rio.br Date: Mon, 16 Sep 2002 12:11:10 -0300 X-Priority: 3 (Normal) Organization: Tonik In-Reply-To: Message-Id: Subject: =?windows-1252?Q?Re:=20[obm-l]=202=AA=20fase=20nivel=203...?= MIME-Version: 1.0 Content-Type: text/plain; charset="iso-8859-1" X-Mailer: Opera 6.04 build 1135 Sender: owner-obm-l@sucuri.mat.puc-rio.br Precedence: bulk Reply-To: obm-l@mat.puc-rio.br Eu consegui fazer 4 problemas, sendo q um eu usei derivadas, acho q aceitam neh? ... Quanto foi a nota de corte ano passado? Quando costumam soltar a prova com as respostas no site? valew > 14/09/02 19:05:41, "Adherbal Rocha Filho" wrote: >Putz, essa prova tava muito,muito,muito,muito dificil!!!!!!!!!! >Espero q pelo menos a nota de corte pra 3ª fase seja menor q 50 pnts... >a do ano passado tava tao facilzinha...mas acho q jah sei, ano passado vcs >fizeram uma 1ª fase dificil e uma segunda bem facil, esse ano foi o >contrario...soh q a 2ª vale muito mais pontos q a 1ª !!!! pô, assim eh >dificil... >falou > > ========================================================================= Instruções para entrar na lista, sair da lista e usar a lista em http://www.mat.puc-rio.br/~nicolau/olimp/obm-l.html O administrador desta lista é ========================================================================= From owner-obm-l@sucuri.mat.puc-rio.br Mon Sep 16 12:21:21 2002 Return-Path: Received: (from majordom@localhost) by sucuri.mat.puc-rio.br (8.9.3/8.9.3) id MAA31526 for obm-l-MTTP; Mon, 16 Sep 2002 12:20:42 -0300 Received: from mta2sjc.directnet.com.br (mta2sjc.directnet.com.br [200.152.0.13]) by sucuri.mat.puc-rio.br (8.9.3/8.9.3) with ESMTP id MAA31521 for ; Mon, 16 Sep 2002 12:20:38 -0300 Received: from meu ([200.152.17.12]) by pop.directnet.com.br (iPlanet Messaging Server 5.1 HotFix 1.3 (built Jul 16 2002)) with SMTP id <0H2J00435ENK29@pop.directnet.com.br> for obm-l@mat.puc-rio.br; Mon, 16 Sep 2002 12:21:25 -0300 (EST) Date: Mon, 16 Sep 2002 12:38:15 -0300 From: Daniel Subject: [obm-l] Livros To: Lista OBM Message-id: <002c01c25d97$140aeb60$0c1198c8@directnet.com.br> MIME-version: 1.0 X-MIMEOLE: Produced By Microsoft MimeOLE V5.00.2615.200 X-Mailer: Microsoft Outlook Express 5.00.2615.200 Content-type: multipart/alternative; boundary="Boundary_(ID_v8EuBC3j/sxW7URwfSl1DA)" X-Priority: 3 X-MSMail-priority: Normal Sender: owner-obm-l@sucuri.mat.puc-rio.br Precedence: bulk Reply-To: obm-l@mat.puc-rio.br This is a multi-part message in MIME format. --Boundary_(ID_v8EuBC3j/sxW7URwfSl1DA) Content-type: text/plain; charset=iso-8859-1 Content-transfer-encoding: quoted-printable Ol=E1 a todos: Gostaria de saber onde posso encontrar os livros = completos: Princ=EDpios Matem=E1ticos da Filosofia Natural = - Isaac Newton Os Elementos - Euclides De prefer=EAncia em Portugu=EAs, mas pode ser em = Franc=EAs ou Ingl=EAs Daniel O. Costa - S=E3o Jos=E9 dos Campos SP --Boundary_(ID_v8EuBC3j/sxW7URwfSl1DA) Content-type: text/html; charset=iso-8859-1 Content-transfer-encoding: quoted-printable
       =20     Ol=E1 a todos:
 
       =20             Gostaria de = saber onde=20 posso encontrar os livros completos:
 
       =20             =    =20 Princ=EDpios Matem=E1ticos da Filosofia Natural - Isaac = Newton
       =20             =     Os=20 Elementos - Euclides
 
          &nbs= p;    =20 De prefer=EAncia em Portugu=EAs, mas pode ser em Franc=EAs ou = Ingl=EAs
 
       =20     Daniel O. Costa - S=E3o Jos=E9 dos Campos=20 SP
--Boundary_(ID_v8EuBC3j/sxW7URwfSl1DA)-- ========================================================================= Instruções para entrar na lista, sair da lista e usar a lista em http://www.mat.puc-rio.br/~nicolau/olimp/obm-l.html O administrador desta lista é ========================================================================= From owner-obm-l@sucuri.mat.puc-rio.br Mon Sep 16 12:38:27 2002 Return-Path: Received: (from majordom@localhost) by sucuri.mat.puc-rio.br (8.9.3/8.9.3) id MAA32034 for obm-l-MTTP; Mon, 16 Sep 2002 12:37:17 -0300 Received: from web21510.mail.yahoo.com (web21510.mail.yahoo.com [66.163.169.59]) by sucuri.mat.puc-rio.br (8.9.3/8.9.3) with SMTP id MAA32028 for ; Mon, 16 Sep 2002 12:37:13 -0300 Message-ID: <20020916153832.35295.qmail@web21510.mail.yahoo.com> Received: from [200.137.198.82] by web21510.mail.yahoo.com via HTTP; Mon, 16 Sep 2002 12:38:32 ART Date: Mon, 16 Sep 2002 12:38:32 -0300 (ART) From: =?iso-8859-1?q?bruno=20lima?= Subject: [obm-l] EDO To: OBM lISTA MIME-Version: 1.0 Content-Type: text/plain; charset=iso-8859-1 Content-Transfer-Encoding: 8bit Sender: owner-obm-l@sucuri.mat.puc-rio.br Precedence: bulk Reply-To: obm-l@mat.puc-rio.br Numa dessas provas universitarias por ai apareceu: y''=y*exp(x) y: Vai de [a,b]em R , e y(a)=0 e y(0)=0. Estou errado ou a unica solucáo é a identicamente nula?? Fazendo uma mudança de variaveis vc cai numa de Ricati, mas isso nao resolve muito! _______________________________________________________________________ Yahoo! PageBuilder O super editor para criação de sites: é grátis, fácil e rápido. http://br.geocities.yahoo.com/v/pb.html ========================================================================= Instruções para entrar na lista, sair da lista e usar a lista em http://www.mat.puc-rio.br/~nicolau/olimp/obm-l.html O administrador desta lista é ========================================================================= From owner-obm-l@sucuri.mat.puc-rio.br Mon Sep 16 13:03:02 2002 Return-Path: Received: (from majordom@localhost) by sucuri.mat.puc-rio.br (8.9.3/8.9.3) id NAA00309 for obm-l-MTTP; Mon, 16 Sep 2002 13:02:07 -0300 Received: from hotmail.com (f107.sea2.hotmail.com [207.68.165.107]) by sucuri.mat.puc-rio.br (8.9.3/8.9.3) with ESMTP id NAA00305 for ; Mon, 16 Sep 2002 13:02:04 -0300 Received: from mail pickup service by hotmail.com with Microsoft SMTPSVC; Mon, 16 Sep 2002 09:03:18 -0700 Received: from 32.94.119.253 by sea2fd.sea2.hotmail.msn.com with HTTP; Mon, 16 Sep 2002 16:03:18 GMT X-Originating-IP: [32.94.119.253] From: "Paulo Santa Rita" To: obm-l@mat.puc-rio.br Subject: [obm-l] =?iso-8859-1?B?UmU6IFtvYm0tbF0gUmU6IFtvYm0tbF0gdW1hIGT6dmlkYSBkZSBG7XNp?= =?iso-8859-1?B?Y2Eu?= Date: Mon, 16 Sep 2002 16:03:18 +0000 Mime-Version: 1.0 Content-Type: text/plain; charset=iso-8859-1; format=flowed Message-ID: X-OriginalArrivalTime: 16 Sep 2002 16:03:18.0990 (UTC) FILETIME=[92D9F6E0:01C25D9A] Sender: owner-obm-l@sucuri.mat.puc-rio.br Precedence: bulk Reply-To: obm-l@mat.puc-rio.br Ola EZER e demais colegas desta lista ... OBM-L, Nao entendi direito o primeiro paragrafo. Se voce quer saber porque o resultado inesperado foi aceito, entao e suficiente saber que a veracidade de uma lei Fisica e aceita ate que algum fato bem observado a desminta. Assim, a constancia da velocidade da luz e aceita porque : 1) As experiencias confirmam tal constancia. 2) As previsoes teoricas que decorrem de sua aceitacao como postulado realmente sao verificadas experimentamente. Nao entendi o segundo paragrafo. E uma pergunta ou um discurso ? Um abraco Paulo Santa Rita 2,1302,160902 >From: ezer@ig.com.br >Reply-To: obm-l@mat.puc-rio.br >To: obm-l@mat.puc-rio.br >Subject: Re: [obm-l] Re: [obm-l] uma dúvida de Física. >Date: Wed, 1 Jan 1997 00:48:12 -0200 > > >Sim, mas como pode se afirmar com tal >veracidade que a velocidade da luz é constante? >O fato de ter sido uma ideia aceita mesmo sendo >um resultado negativo (ou nao-esperado) nao comprova >nada senao que a ciencia feita por Einstein tinha um >grau de objetividade bastante saudavel. > >Se estamos numa busca pela realidade, devemos nos >abster de classificacoes que tendam para o ambito >da opiniao. Eh certo que toda a ciencia estah permeada >de crencas e opinioes, mas o esforco por um julgamento >racional eh sempre coroado por uma analise que, se nao >eh a mais bonita ou extensa, estah pelo menos mais 'liberta' >das crencas de sua epoca (ou seja, serah mais 'duravel'). > >Abracos, > >Ezer Fernandes > >On 15 Sep 2002 at 22:16, Paulo Santa Rita wrote: > > Ola FELIPE e demais > > colegas desta lista ... OBM-L, > > > > Nem uma coisa e nem outra ... A velocidade da Luz ser constante, > > independente do movimento da fonte, e um postulado na Teoria da > > Relatividade, mas nao foi deste postulado que Einstein DEDUZIU TUDO. > > > > Esse resultado foi considerado um RESULTADO NEGATIVO pois esperava-se >que o > > interferometro evidenciasse diferencas nas velocidades dos dois raios de > > luz, o que seria uma prova indireta da existencia do eter e, com isso, > > ficava facil explicar a propagacao das ondas eletromagneticas como uma > > pertubacao deste eter. > > > > Este RESULTADO NEGATIVO da experiencia de Michelson ficou sem uma >explicacao > > plausivel, mas, por esta epoca, Lorentz deduziu suas equacoes para o > > fenomeno mas nao apresentou uma explicacao consistente, pois ele >implicava > > em hipoteses consideradas entao pouco plausiveis e artificiais. > > > > O Genio de Einstein se evidenciou aqui. Ele aceitou o RESULTADO > > APARENTEMENTE NEGATIVO da experiencia de Michelson-Morley e mostrou que >isto > > supunha um tempo nao absoluto, um tempo de cada abservador e que os > > conceitos de simultaneidade e correlatos deveriam ser modificados. Dai >ele > > deduziu as equacoes de Lorentz como consequencias destes postulados. > > > > Foi como na gravitacao. Kepler e Galileu ja haviam descoberto muitas >leis > > importantes e interessantes, mas eles nao atinaram COM A IDEIA CERTA. > > Newton fez isso, e mostrou que a sua hipotese nao so explicava ou >deduzia as > > leis particulares de Galileu e Kepler como permitia explicar muitas >outras > > coisas mais. As ideias de Newton, como a de Einstein, eram mais simples >e > > mais gerais e permitiam deduzir novos fatos que poderiam ser submetidos >ao > > crivo da experiemntacao, vale dizer, elas preservavam todas as >conquistas > > anteriores, eram mais simples e fundamentais e alargavam a compreensao > > humana. > >========================================================================= >Instruções para entrar na lista, sair da lista e usar a lista em >http://www.mat.puc-rio.br/~nicolau/olimp/obm-l.html >O administrador desta lista é >========================================================================= _________________________________________________________________ Tenha você também um MSN Hotmail, o maior webmail do mundo: http://www.hotmail.com/br ========================================================================= Instruções para entrar na lista, sair da lista e usar a lista em http://www.mat.puc-rio.br/~nicolau/olimp/obm-l.html O administrador desta lista é ========================================================================= From owner-obm-l@sucuri.mat.puc-rio.br Mon Sep 16 13:57:55 2002 Return-Path: Received: (from majordom@localhost) by sucuri.mat.puc-rio.br (8.9.3/8.9.3) id NAA02229 for obm-l-MTTP; Mon, 16 Sep 2002 13:56:39 -0300 Received: from web12902.mail.yahoo.com (web12902.mail.yahoo.com [216.136.174.69]) by sucuri.mat.puc-rio.br (8.9.3/8.9.3) with SMTP id NAA02225 for ; Mon, 16 Sep 2002 13:56:35 -0300 Message-ID: <20020916165754.81751.qmail@web12902.mail.yahoo.com> Received: from [200.206.103.3] by web12902.mail.yahoo.com via HTTP; Mon, 16 Sep 2002 13:57:54 ART Date: Mon, 16 Sep 2002 13:57:54 -0300 (ART) From: =?iso-8859-1?q?Johann=20Peter=20Gustav=20Lejeune=20Dirichlet?= Subject: RE de re:[obm-l] Ajuda Algebra linear (Off Topic) To: obm-l@mat.puc-rio.br In-Reply-To: <20020913173026.82F833EA9C@zeus.opendf.com.br> MIME-Version: 1.0 Content-Type: multipart/alternative; boundary="0-1097315872-1032195474=:81726" Content-Transfer-Encoding: 8bit Sender: owner-obm-l@sucuri.mat.puc-rio.br Precedence: bulk Reply-To: obm-l@mat.puc-rio.br --0-1097315872-1032195474=:81726 Content-Type: text/plain; charset=iso-8859-1 Content-Transfer-Encoding: 8bit Beleza!La ele da uma aplicaçao bem interessante:como se dar bem blefando em um jogo de truco(parece jogo de truco mas mudam algumas regras).So que para ler o artigo voce precisa do Ghostscript e do GSview(va ate o site www.teorema.mat.br no link "Softwares" se voce nao tiver).Ai voce consegue ler as besteiras do olimpico mais hilario de toda face da Grande Sao Paulo!!!!!!!!E so ir ao site www.obm.org.br/semana.htm e divirta-se! 498 - Artur Costa Steiner wrote: > > > Tem um artigo do Carlos Yuzo Shine(meu idolo!)no site da OBM semana olimpica.Vai querer? Quero sim! Artur ========================================================================= Instruções para entrar na lista, sair da lista e usar a lista em http://www.mat.puc-rio.br/~nicolau/olimp/obm-l.html O administrador desta lista é ========================================================================= --------------------------------- Yahoo! PageBuilder - O super editor para criação de sites: é grátis, fácil e rápido. --0-1097315872-1032195474=:81726 Content-Type: text/html; charset=iso-8859-1 Content-Transfer-Encoding: 8bit

Beleza!La ele da uma aplicaçao bem interessante:como se dar bem blefando em um jogo de truco(parece jogo de truco mas mudam algumas regras).So que para  ler o artigo voce precisa do Ghostscript e do GSview(va ate o site www.teorema.mat.br no link "Softwares" se voce nao tiver).Ai voce consegue ler as besteiras do olimpico mais hilario de toda face da Grande Sao Paulo!!!!!!!!E so ir ao site www.obm.org.br/semana.htm e divirta-se!

 498 - Artur Costa Steiner wrote:



>
>
> Tem um artigo do Carlos Yuzo Shine(meu idolo!)no site da OBM semana
olimpica.Vai querer?

Quero sim!
Artur
=========================================================================
Instruções para entrar na lista, sair da lista e usar a lista em
http://www.mat.puc-rio.br/~nicolau/olimp/obm-l.html
O administrador desta lista é
=========================================================================


Yahoo! PageBuilder - O super editor para criação de sites: é grátis, fácil e rápido. --0-1097315872-1032195474=:81726-- ========================================================================= Instruções para entrar na lista, sair da lista e usar a lista em http://www.mat.puc-rio.br/~nicolau/olimp/obm-l.html O administrador desta lista é ========================================================================= From owner-obm-l@sucuri.mat.puc-rio.br Mon Sep 16 14:14:34 2002 Return-Path: Received: (from majordom@localhost) by sucuri.mat.puc-rio.br (8.9.3/8.9.3) id OAA02566 for obm-l-MTTP; Mon, 16 Sep 2002 14:14:03 -0300 Received: from www.zipmail.com.br (smtp.zipmail.com.br [200.187.242.10]) by sucuri.mat.puc-rio.br (8.9.3/8.9.3) with ESMTP id OAA02560 for ; Mon, 16 Sep 2002 14:13:38 -0300 From: yurigomes@zipmail.com.br Received: from [200.151.189.206] by www.zipmail.com.br with HTTP; Mon, 16 Sep 2002 14:14:50 -0300 Message-ID: <3D80940900000E24@www.zipmail.com.br> Date: Mon, 16 Sep 2002 14:14:50 -0300 In-Reply-To: Subject: [obm-l] =?iso-8859-1?Q?Re=3A=20=5Bobm=2Dl=5D=20Problema=204=20OBM=20universit=E1ria?= To: obm-l@mat.puc-rio.br MIME-Version: 1.0 Content-Type: text/plain; charset="iso-8859-1" Content-Transfer-Encoding: 8bit X-MIME-Autoconverted: from quoted-printable to 8bit by sucuri.mat.puc-rio.br id OAA02563 Sender: owner-obm-l@sucuri.mat.puc-rio.br Precedence: bulk Reply-To: obm-l@mat.puc-rio.br Sabemos que p(x)|q(x) sss toda raiz de p é tbm raiz de q. Logo, sendo p(x)=(x^2+x+1)^2 e q(x)= (x+1)^m + x^m + 1, temos q(w)=0, onde w=cis(2pi/3). Supondo m impar nao divisivel por 3, teremos (w+1)^m + w^m + 1 = 0, donde -w^(2m) + w^m + 1=0 . Mas isso não pode ocorrer, visto que w^(2m) + w^m + 1 já é igual a zero! Se m for ímpar, divisível por 3, teremos -w^(2m) + w^m + 1 igual a 1, absurdo. Logo m é par. Se m é múltiplo de 3, teremos agora q(w)= w^(2m) + w^m + 1 = 3, absurdo. Daí 3 náo divide m. Sendo assim, w^(2m) + w^m + 1 = (w^(3m) - 1)/(w^m - 1) = 0. Logo, q(w)=0, isto é, w já é raiz de q. Mas nós queremos que w seja raiz dupla. Para isso devemos ter tbm q´(w)=0. Derivando, queremos m.(x + 1)^(m-1) + m.w^(m-1) = 0 sss (w + 1)^(m-1) + w^(m-1)= 0 sss -w^[2(m-1)] + w^(m-1) = 0 ( pois m-1 é ímpar ) sss w^(m-1)=1, i.e., 3|(m-1) sss m = 6k-2, k natural. -- Mensagem original -- >Pessoal, consegui encontrar usando ajuda de um software >que para m = 4 + 6k o polinômio é divisível, se alguém >souber mostrar isso, gostaria de uma ajuda! > > PROBLEMA 4 >Determine todos os valores inteiros positivos de m para >os quais o polinômio (x+1)^m + x^m +1 é divisível por >(x^2 + x + 1)^2. > > >__________________________________________________________________________ >AcessoBOL, só R$ 9,90! O menor preço do mercado! >Assine já! http://www.bol.com.br/acessobol > > >========================================================================= >Instruções para entrar na lista, sair da lista e usar a lista em >http://www.mat.puc-rio.br/~nicolau/olimp/obm-l.html >O administrador desta lista é >========================================================================= > []'s, Yuri ICQ: 64992515 ------------------------------------------ Use o melhor sistema de busca da Internet Radar UOL - http://www.radaruol.com.br ========================================================================= Instruções para entrar na lista, sair da lista e usar a lista em http://www.mat.puc-rio.br/~nicolau/olimp/obm-l.html O administrador desta lista é ========================================================================= From owner-obm-l@sucuri.mat.puc-rio.br Mon Sep 16 14:22:15 2002 Return-Path: Received: (from majordom@localhost) by sucuri.mat.puc-rio.br (8.9.3/8.9.3) id OAA02733 for obm-l-MTTP; Mon, 16 Sep 2002 14:22:01 -0300 Received: from web12905.mail.yahoo.com (web12905.mail.yahoo.com [216.136.174.72]) by sucuri.mat.puc-rio.br (8.9.3/8.9.3) with SMTP id OAA02729 for ; Mon, 16 Sep 2002 14:21:58 -0300 Message-ID: <20020916172318.55059.qmail@web12905.mail.yahoo.com> Received: from [200.206.103.3] by web12905.mail.yahoo.com via HTTP; Mon, 16 Sep 2002 14:23:18 ART Date: Mon, 16 Sep 2002 14:23:18 -0300 (ART) From: =?iso-8859-1?q?Johann=20Peter=20Gustav=20Lejeune=20Dirichlet?= Subject: Re: [obm-l] Axioma da Escolha To: obm-l@mat.puc-rio.br In-Reply-To: MIME-Version: 1.0 Content-Type: multipart/alternative; boundary="0-1042005992-1032196998=:54602" Content-Transfer-Encoding: 8bit Sender: owner-obm-l@sucuri.mat.puc-rio.br Precedence: bulk Reply-To: obm-l@mat.puc-rio.br --0-1042005992-1032196998=:54602 Content-Type: text/plain; charset=iso-8859-1 Content-Transfer-Encoding: 8bit Ah,meu,o Gödel e o Gödel e nao ha discussao!O cara foi na minha opiniao o melhor logico que ja existiu na face da Terra.O cara conseguiu de maneira magistral resolver o teorema de Hilbert do Teorema Indecidivel.So o cara poderia provar esse treco de Algebra Cantoriana e Axioma da Escolha.Mas tem uma fruta verde e de gosto acido que e equivalente ao axioma da escolha,o Limao de Zorn(o piada imbecil!!!!!!),que e algo intuitivamente obvio. So quero saber que historia e essa de constutivismo.Qual o problema de "nao nao" significar "sim"?O Principio da Casa dos Pombos e provado por absurdo e e um treco bem obvio na nossa intuicao:se temos 14 bolas para por em 15 caixas uma delas fica vazia,e pronto,cabou!!!!!! Bem,aqui eu encerro minhas opinioes.Ate mais!!!!!Ass.:Johann Rogerio Fajardo wrote:Apenas lembrando, porque costuma-se realçar quando se usa o axioma da escolha, há uma corrente filosófica de matemáticos que não aceitam o axioma da escolha: os construtivistas (ou, mais geralmente, os intuicionistas). O axioma da escolha nos garante a existência de objetos que não podemos determinar quem, exatamente, ele é. Esse tipo de coisa os construtivistas não aceitam, pois de que serve saber que existe alguma coisa que nunca saberemos quem é, ou onde está? O teorema de Weierstrass, que diz que toda função real contínua sobre um intervalo fechado assume máximo, não é aceita pelos construtivistas, pois não podemos exibir esse ponto de máximo. Por outro lado não podemos dizer que não existe ponto de máximo, pois isso seria garantir que todos os pontos não são de máximo, o que não devemos assegurar. Por isso na lógica intuicionista "A ou não A" pode ser falso, e A não é equivalente a "não não A". Todas essas complicações geradas pelo construtivismo fizeram que esse caísse um pouco no esquecimento. Hoje parece que há poucos matemáticos construtivistas. Mas devemos nos lembrar que o argumento central que gerou o construtivismo faz sentido. Realmente, podemos pensar o que fazemos com coisas obtidas não construtivamente. Enfim, há sempre uma fagulha de construtivista em nós. É certo que os mais radicais não admitem nem prova por absurdo, mas o axioma da escolha já seria o maior crime que se poderia cometer contra o construtivismo. Por isso, nas demonstrações, é sempre bom ressaltar o que é construtivo e o que não é. Por exemplo, o Paradoxo de Banach-Tarski, sobre a duplicação da esfera, citada pelo Paulo, é não-construtiva. Sobre o problema da violência, resta um consolo: se o conjunto dos bandidos, dado pelo problema, já estiver bem ordenado (por exemplo, se é enumerável), não precisamos do axioma da escolha, e não cometeremos uma "violência" contra os intuicionistas. O difícil vai ser achar bandidos bem ordenados... >From: "Paulo Santa Rita" >Reply-To: obm-l@mat.puc-rio.br >To: obm-l@mat.puc-rio.br >Subject: Re: [obm-l] Axioma da Escolha >Date: Thu, 12 Sep 2002 21:56:07 +0000 > >Ola PROF NICOLAU e demais >colegas desta lista ... OBM-L, > >Vou fazer um comentario a mensagem abaixo que talvez ajude alguns membros >desta lista entenderem porque muitos Matematicos - em especial aqueles que >conhecem logica e teoria dos conjuntos - acham relevante destacar que esta >ou aquela prova ou argumentacao matematica usa o AXIOMA DA ESCOLHA. > >Tarski, entre outros, mostrou que o uso do AXIOMA DA ESCOLHA com os demais >axiomas da teoria dos conjuntos levam-nos inevitavelmente a conclusoes >pouco verossimeis. Em particular ele mostrou que poderiamos dividir uma >esfera dada em ao menos cinco partes e, unindo novamente as partes de outra >forma, derivar nao uma, mas duas esferas identicas a primitiva. > >O fato acima, por paradoxal que parece, e logicamente inatacavel e uma das >consequencias bizarras que este axioma implica. Muitos Matematicos supunham >que tais resultados eram derivados da liberalidade dado a FUNCAO DE >ESCOLHA, pois pode-se usar a que quisermos conquanto respeitemos o aspecto >formal do axioma. > >A partir deste resultado do Tarski ( e de outros tambem ) os Matematicos >comecaram a suspeitar que o AXIOMA DA ESCOLHA era um principio nefasto, >sendo provavelmente o responsavel por possiveis e potenciais >inconsistencias que a teoria do conjuntos tivesse ou viesse a ter. Dai >surgiu a desconfianca com as demonstracoes com este axioma. Por prudencia, >toda prova que usava este teorema era rotulada "USA O AXIOMA DA ESCOLHA", >como que insinuando : "PROCURE UMA OUTRA MANEIRA DE PROVAR ISSO ... " > >Aqui entra o Magistral GODEL ... > >Godel classificou as teorias dos conjuntos em : > >1) Teoria Cantoriana A e aquela que usa o AXIOMA DA ESCOLHA. >2) Teoria Cantoriana B e aquela em que nao usa o AXIOMA DA ESCOLHA. > >E provou o seguinte : > >SE A TEORIA CANTORIANA "A" CONTIVER OU GERAR INCONSISTENCIAS, A TEORIA >CANTORIA "B" TAMBEM CONTERA E GERARA INCONSISTENCIAS. > >Isto e, o AXIOMA DA ESCOLHA nao e o responsavel por possiveis >inconsistencias ou paradoxas que porventura derivem da teoria dos >conjuntos. Ele pode ser um CATALISADOR destas inconsistencias, evidenciando >de forma mais direta e clara possiveis absurdos ... > >Agora, uma observacao sobre o Paradoxo de Tarski. > >O que ha de absurdo nele ? A criacao de Massa ( duplicacao de uma esfera ) >sem a necessaria absorcao de uma fabulosa quantidade de Energia ? Mas ... >Nao e isso que rotineiramente ocorre no mundo das particulas elementares, >quando, do nada, surge uma massa que posteriormente desapare num par de >particulas antipodas ? E mais provavel que este fato ou operacao seja >paradoxal para o nosso cotidiano, nao para o Autor da Natureza que, hoje >sabemos, continuamente faz isso ... > >Um abraco >Paulo Santa Rita >5,1854,120902 > >>From: "Nicolau C. Saldanha" >>Reply-To: obm-l@mat.puc-rio.br >>To: obm-l@mat.puc-rio.br >>Subject: Re: [obm-l] Axioma da Escolha >>Date: Thu, 12 Sep 2002 17:32:17 -0300 >> >>On Wed, Sep 11, 2002 at 04:01:42PM -0300, 498 - Artur Costa Steiner wrote: >> > Nos últimos dias o Axioma da Escolha foi bastante mencionado nesta >> > lista, motivado por um interessante problema (violência), sugerido por >> > uma das participantes, e que involve este axioma. >> > >> > Eu não estou certo, mas, no meio matemático, ainda existem hoje >> > restrições a este axioma, no sentido de que alguma prova nele baseada >> > possa ser considerada questionável ou mesmo inválida? >> >>Este assunto mereceria uma resposta mais longa, mas o axioma da escolha >>é 'aceito' no sentido seguinte: a maioria dos matemáticos usa este >>axioma sem parar para pensar no assunto. Aliás sem nem saber direito >>quando está realmente usando o axioma. Alguns matemáticos, entretanto, >>especialmente especialistas em lógica ou teoria dos conjuntos, acham >>interessante notar exatamente quando o tal axioma é utilizado. >> >>[]s, N. >>========================================================================= >>Instruções para entrar na lista, sair da lista e usar a lista em >>http://www.mat.puc-rio.br/~nicolau/olimp/obm-l.html >>O administrador desta lista é >>========================================================================= > > >_________________________________________________________________ >Tenha você também um MSN Hotmail, o maior webmail do mundo: >http://www.hotmail.com/br > >========================================================================= >Instruções para entrar na lista, sair da lista e usar a lista em >http://www.mat.puc-rio.br/~nicolau/olimp/obm-l.html >O administrador desta lista é >========================================================================= _________________________________________________________________ MSN Photos is the easiest way to share and print your photos: http://photos.msn.com/support/worldwide.aspx ========================================================================= Instruções para entrar na lista, sair da lista e usar a lista em http://www.mat.puc-rio.br/~nicolau/olimp/obm-l.html O administrador desta lista é ========================================================================= TRANSIRE SVVM PECTVS MVNDOQUE POTIRE CONGREGATI EX TOTO ORBE MATHEMATICI OB SCRIPTA INSIGNIA TRIBVERE Fields Medal(John Charles Fields) --------------------------------- Yahoo! PageBuilder - O super editor para criação de sites: é grátis, fácil e rápido. --0-1042005992-1032196998=:54602 Content-Type: text/html; charset=iso-8859-1 Content-Transfer-Encoding: 8bit

Ah,meu,o Gödel e o Gödel e nao ha discussao!O cara foi na minha opiniao o melhor logico que ja existiu na face da Terra.O cara conseguiu de maneira magistral resolver o teorema de Hilbert do Teorema Indecidivel.So o cara poderia provar esse treco de Algebra Cantoriana e Axioma da Escolha.Mas tem uma fruta verde e de gosto acido que e equivalente ao axioma da escolha,o Limao de Zorn(o piada imbecil!!!!!!),que e algo intuitivamente obvio.

So quero saber que historia e essa de constutivismo.Qual o problema de "nao nao" significar "sim"?O Principio da Casa dos Pombos e provado por absurdo e e um treco bem obvio na nossa intuicao:se temos 14 bolas para por em 15 caixas uma delas fica vazia,e pronto,cabou!!!!!!  

Bem,aqui eu encerro minhas opinioes.Ate mais!!!!!Ass.:Johann

 Rogerio Fajardo wrote:

Apenas lembrando, porque costuma-se realçar quando se usa o axioma da
escolha, há uma corrente filosófica de matemáticos que não aceitam o axioma
da escolha: os construtivistas (ou, mais geralmente, os intuicionistas). O
axioma da escolha nos garante a existência de objetos que não podemos
determinar quem, exatamente, ele é. Esse tipo de coisa os construtivistas
não aceitam, pois de que serve saber que existe alguma coisa que nunca
saberemos quem é, ou onde está? O teorema de Weierstrass, que diz que toda
função real contínua sobre um intervalo fechado assume máximo, não é aceita
pelos construtivistas, pois não podemos exibir esse ponto de máximo. Por
outro lado não podemos dizer que não existe ponto de máximo, pois isso seria
garantir que todos os pontos não são de máximo, o que não devemos assegurar.
Por isso na lógica intuicionista "A ou não A" pode ser falso, e A não é
equivalente a "não não A".
Todas essas complicações geradas pelo construtivismo fizeram que esse
caísse um pouco no esquecimento. Hoje parece que há poucos matemáticos
construtivistas. Mas devemos nos lembrar que o argumento central que gerou o
construtivismo faz sentido. Realmente, podemos pensar o que fazemos com
coisas obtidas não construtivamente. Enfim, há sempre uma fagulha de
construtivista em nós. É certo que os mais radicais não admitem nem prova
por absurdo, mas o axioma da escolha já seria o maior crime que se poderia
cometer contra o construtivismo. Por isso, nas demonstrações, é sempre bom
ressaltar o que é construtivo e o que não é. Por exemplo, o Paradoxo de
Banach-Tarski, sobre a duplicação da esfera, citada pelo Paulo, é
não-construtiva.
Sobre o problema da violência, resta um consolo: se o conjunto dos
bandidos, dado pelo problema, já estiver bem ordenado (por exemplo, se é
enumerável), não precisamos do axioma da escolha, e não cometeremos uma
"violência" contra os intuicionistas. O difícil vai ser achar bandidos bem
ordenados...


>From: "Paulo Santa Rita"
>Reply-To: obm-l@mat.puc-rio.br
>To: obm-l@mat.puc-rio.br
>Subject: Re: [obm-l] Axioma da Escolha
>Date: Thu, 12 Sep 2002 21:56:07 +0000
>
>Ola PROF NICOLAU e demais
>colegas desta lista ... OBM-L,
>
>Vou fazer um comentario a mensagem abaixo que talvez ajude alguns membros
>desta lista entenderem porque muitos Matematicos - em especial aqueles que
>conhecem logica e teoria dos conjuntos - acham relevante destacar que esta
>ou aquela prova ou argumentacao matematica usa o AXIOMA DA ESCOLHA.
>
>Tarski, entre outros, mostrou que o uso do AXIOMA DA ESCOLHA com os demais
>axiomas da teoria dos conjuntos levam-nos inevitavelmente a conclusoes
>pouco verossimeis. Em particular ele mostrou que poderiamos dividir uma
>esfera dada em ao menos cinco partes e, unindo novamente as partes de outra
>forma, derivar nao uma, mas duas esferas identicas a primitiva.
>
>O fato acima, por paradoxal que parece, e logicamente inatacavel e uma das
>consequencias bizarras que este axioma implica. Muitos Matematicos supunham
>que tais resultados eram derivados da liberalidade dado a FUNCAO DE
>ESCOLHA, pois pode-se usar a que quisermos conquanto respeitemos o aspecto
>formal do axioma.
>
>A partir deste resultado do Tarski ( e de outros tambem ) os Matematicos
>comecaram a suspeitar que o AXIOMA DA ESCOLHA era um principio nefasto,
>sendo provavelmente o responsavel por possiveis e potenciais
>inconsistencias que a teoria do conjuntos tivesse ou viesse a ter. Dai
>surgiu a desconfianca com as demonstracoes com este axioma. Por prudencia,
>toda prova que usava este teorema era rotulada "USA O AXIOMA DA ESCOLHA",
>como que insinuando : "PROCURE UMA OUTRA MANEIRA DE PROVAR ISSO ... "
>
>Aqui entra o Magistral GODEL ...
>
>Godel classificou as teorias dos conjuntos em :
>
>1) Teoria Cantoriana A e aquela que usa o AXIOMA DA ESCOLHA.
>2) Teoria Cantoriana B e aquela em que nao usa o AXIOMA DA ESCOLHA.
>
>E provou o seguinte :
>
>SE A TEORIA CANTORIANA "A" CONTIVER OU GERAR INCONSISTENCIAS, A TEORIA
>CANTORIA "B" TAMBEM CONTERA E GERARA INCONSISTENCIAS.
>
>Isto e, o AXIOMA DA ESCOLHA nao e o responsavel por possiveis
>inconsistencias ou paradoxas que porventura derivem da teoria dos
>conjuntos. Ele pode ser um CATALISADOR destas inconsistencias, evidenciando
>de forma mais direta e clara possiveis absurdos ...
>
>Agora, uma observacao sobre o Paradoxo de Tarski.
>
>O que ha de absurdo nele ? A criacao de Massa ( duplicacao de uma esfera )
>sem a necessaria absorcao de uma fabulosa quantidade de Energia ? Mas ...
>Nao e isso que rotineiramente ocorre no mundo das particulas elementares,
>quando, do nada, surge uma massa que posteriormente desapare num par de
>particulas antipodas ? E mais provavel que este fato ou operacao seja
>paradoxal para o nosso cotidiano, nao para o Autor da Natureza que, hoje
>sabemos, continuamente faz isso ...
>
>Um abraco
>Paulo Santa Rita
>5,1854,120902
>
>>From: "Nicolau C. Saldanha"
>>Reply-To: obm-l@mat.puc-rio.br
>>To: obm-l@mat.puc-rio.br
>>Subject: Re: [obm-l] Axioma da Escolha
>>Date: Thu, 12 Sep 2002 17:32:17 -0300
>>
>>On Wed, Sep 11, 2002 at 04:01:42PM -0300, 498 - Artur Costa Steiner wrote:
>> > Nos últimos dias o Axioma da Escolha foi bastante mencionado nesta
>> > lista, motivado por um interessante problema (violência), sugerido por
>> > uma das participantes, e que involve este axioma.
>> >
>> > Eu não estou certo, mas, no meio matemático, ainda existem hoje
>> > restrições a este axioma, no sentido de que alguma prova nele baseada
>> > possa ser considerada questionável ou mesmo inválida?
>>
>>Este assunto mereceria uma resposta mais longa, mas o axioma da escolha
>>é 'aceito' no sentido seguinte: a maioria dos matemáticos usa este
>>axioma sem parar para pensar no assunto. Aliás sem nem saber direito
>>quando está realmente usando o axioma. Alguns matemáticos, entretanto,
>>especialmente especialistas em lógica ou teoria dos conjuntos, acham
>>interessante notar exatamente quando o tal axioma é utilizado.
>>
>>[]s, N.
>>=========================================================================
>>Instruções para entrar na lista, sair da lista e usar a lista em
>>http://www.mat.puc-rio.br/~nicolau/olimp/obm-l.html
>>O administrador desta lista é
>>=========================================================================
>
>
>_________________________________________________________________
>Tenha você também um MSN Hotmail, o maior webmail do mundo:
>http://www.hotmail.com/br
>
>=========================================================================
>Instruções para entrar na lista, sair da lista e usar a lista em
>http://www.mat.puc-rio.br/~nicolau/olimp/obm-l.html
>O administrador desta lista é
>=========================================================================




_________________________________________________________________
MSN Photos is the easiest way to share and print your photos:
http://photos.msn.com/support/worldwide.aspx

=========================================================================
Instruções para entrar na lista, sair da lista e usar a lista em
http://www.mat.puc-rio.br/~nicolau/olimp/obm-l.html
O administrador desta lista é
=========================================================================


TRANSIRE SVVM PECTVS MVNDOQUE POTIRE

CONGREGATI EX TOTO ORBE MATHEMATICI OB SCRIPTA INSIGNIA TRIBVERE

Fields Medal(John Charles Fields)


Yahoo! PageBuilder - O super editor para criação de sites: é grátis, fácil e rápido. --0-1042005992-1032196998=:54602-- ========================================================================= Instruções para entrar na lista, sair da lista e usar a lista em http://www.mat.puc-rio.br/~nicolau/olimp/obm-l.html O administrador desta lista é ========================================================================= From owner-obm-l@sucuri.mat.puc-rio.br Mon Sep 16 14:47:49 2002 Return-Path: Received: (from majordom@localhost) by sucuri.mat.puc-rio.br (8.9.3/8.9.3) id OAA03680 for obm-l-MTTP; Mon, 16 Sep 2002 14:47:36 -0300 Received: from smtp.brturbo.com (smtp1.brturbo.com [200.199.201.21]) by sucuri.mat.puc-rio.br (8.9.3/8.9.3) with ESMTP id OAA03676 for ; Mon, 16 Sep 2002 14:47:33 -0300 Received: from x (unknown [200.181.95.119]) by smtp.brturbo.com (Postfix) with SMTP id 1583517DF67 for ; Mon, 16 Sep 2002 14:48:51 -0300 (BRT) Message-ID: <001d01c25da9$5b808240$0201a8c0@x> From: "Jeremias de Paula Eduardo" To: References: <002c01c25d97$140aeb60$0c1198c8@directnet.com.br> Subject: Re: [obm-l] Livros Date: Mon, 16 Sep 2002 14:47:17 -0300 MIME-Version: 1.0 Content-Type: multipart/alternative; boundary="----=_NextPart_000_0016_01C25D8F.F3B495C0" X-Priority: 3 X-MSMail-Priority: Normal X-Mailer: Microsoft Outlook Express 5.00.2919.6600 X-MIMEOLE: Produced By Microsoft MimeOLE V5.00.2919.6600 Sender: owner-obm-l@sucuri.mat.puc-rio.br Precedence: bulk Reply-To: obm-l@mat.puc-rio.br This is a multi-part message in MIME format. ------=_NextPart_000_0016_01C25D8F.F3B495C0 Content-Type: text/plain; charset="iso-8859-1" Content-Transfer-Encoding: quoted-printable na biblioteca de uma boa universidade Ol=E1 a todos: Gostaria de saber onde posso encontrar os livros = completos: Princ=EDpios Matem=E1ticos da Filosofia = Natural - Isaac Newton Os Elementos - Euclides De prefer=EAncia em Portugu=EAs, mas pode ser em = Franc=EAs ou Ingl=EAs Daniel O. Costa - S=E3o Jos=E9 dos Campos SP ------=_NextPart_000_0016_01C25D8F.F3B495C0 Content-Type: text/html; charset="iso-8859-1" Content-Transfer-Encoding: quoted-printable
na biblioteca de uma boa = universidade
 
        =     Ol=E1 a todos:
 
        =             Gostaria de = saber=20 onde posso encontrar os livros completos:
 
        =             =    =20 Princ=EDpios Matem=E1ticos da Filosofia Natural - Isaac = Newton
        =             =     Os=20 Elementos - Euclides
 
          &nbs= p;    =20 De prefer=EAncia em Portugu=EAs, mas pode ser em Franc=EAs ou = Ingl=EAs
 
        =     Daniel O. Costa - S=E3o Jos=E9 dos Campos=20 SP
------=_NextPart_000_0016_01C25D8F.F3B495C0-- ========================================================================= Instruções para entrar na lista, sair da lista e usar a lista em http://www.mat.puc-rio.br/~nicolau/olimp/obm-l.html O administrador desta lista é ========================================================================= From owner-obm-l@sucuri.mat.puc-rio.br Mon Sep 16 14:57:03 2002 Return-Path: Received: (from majordom@localhost) by sucuri.mat.puc-rio.br (8.9.3/8.9.3) id OAA04051 for obm-l-MTTP; Mon, 16 Sep 2002 14:56:42 -0300 Received: from gorgo.centroin.com.br (gorgo.centroin.com.br [200.225.63.128]) by sucuri.mat.puc-rio.br (8.9.3/8.9.3) with ESMTP id OAA04045 for ; Mon, 16 Sep 2002 14:56:39 -0300 Received: from centroin.com.br (du33b.nit.centroin.com.br [200.225.56.33]) (authenticated bits=0) by gorgo.centroin.com.br (8.12.2/8.12.1) with ESMTP id g8GHwII8028497 for ; Mon, 16 Sep 2002 14:58:20 -0300 (BRT) Message-ID: <3D861C5C.1090900@centroin.com.br> Date: Mon, 16 Sep 2002 15:01:00 -0300 From: Augusto =?ISO-8859-1?Q?C=E9sar?= Morgado User-Agent: Mozilla/5.0 (Windows; U; Win98; en-US; rv:0.9.4.1) Gecko/20020508 Netscape6/6.2.3 X-Accept-Language: en-us MIME-Version: 1.0 To: obm-l@mat.puc-rio.br Subject: Re: [obm-l] Livros References: <002c01c25d97$140aeb60$0c1198c8@directnet.com.br> Content-Type: multipart/alternative; boundary="------------000409050306090008010909" Sender: owner-obm-l@sucuri.mat.puc-rio.br Precedence: bulk Reply-To: obm-l@mat.puc-rio.br --------------000409050306090008010909 Content-Type: text/plain; charset=ISO-8859-1; format=flowed Content-Transfer-Encoding: 8bit Creio que haja ediçoes da Dover. Procure na internet. Daniel wrote: > Olá a todos: > > > > Gostaria de saber onde posso encontrar os livros > completos: > > > > Princípios Matemáticos da Filosofia Natural - > Isaac Newton > > Os Elementos - Euclides > > > > De preferência em Português, mas pode ser em Francês > ou Inglês > > > > Daniel O. Costa - São José dos Campos SP > --------------000409050306090008010909 Content-Type: text/html; charset=us-ascii Content-Transfer-Encoding: 7bit Creio que haja ediçoes da Dover.
Procure na internet.

Daniel wrote:
            Olá a todos:
 
                    Gostaria de saber onde posso encontrar os livros completos:
 
                        Princípios Matemáticos da Filosofia Natural - Isaac Newton
                        Os Elementos - Euclides
 
                De preferência em Português, mas pode ser em Francês ou Inglês
 
            Daniel O. Costa - São José dos Campos SP

--------------000409050306090008010909-- ========================================================================= Instruções para entrar na lista, sair da lista e usar a lista em http://www.mat.puc-rio.br/~nicolau/olimp/obm-l.html O administrador desta lista é ========================================================================= From owner-obm-l@sucuri.mat.puc-rio.br Mon Sep 16 15:00:48 2002 Return-Path: Received: (from majordom@localhost) by sucuri.mat.puc-rio.br (8.9.3/8.9.3) id OAA04167 for obm-l-MTTP; Mon, 16 Sep 2002 14:59:06 -0300 Received: from traven10.pub1 (traven10.uol.com.br [200.221.4.45]) by sucuri.mat.puc-rio.br (8.9.3/8.9.3) with ESMTP id OAA04156 for ; Mon, 16 Sep 2002 14:58:53 -0300 Received: from u2z7z2 ([200.158.144.91]) by traven10.pub1 (8.9.1/8.9.1) with ESMTP id OAA01203 for ; Mon, 16 Sep 2002 14:59:33 -0300 (BRT) Message-ID: <002601c25dab$1366f320$5b909ec8@u2z7z2> From: "Wagner" To: References: Subject: [obm-l] =?iso-8859-1?Q?Re:_=5Bobm-l=5D_2=AA_fase_nivel_3...?= Date: Mon, 16 Sep 2002 15:01:26 -0300 Organization: Wagner MIME-Version: 1.0 Content-Type: text/plain; charset="iso-8859-1" Content-Transfer-Encoding: 8bit X-Priority: 3 X-MSMail-Priority: Normal X-Mailer: Microsoft Outlook Express 5.50.4133.2400 X-MimeOLE: Produced By Microsoft MimeOLE V5.50.4133.2400 Sender: owner-obm-l@sucuri.mat.puc-rio.br Precedence: bulk Reply-To: obm-l@mat.puc-rio.br Oi para todos Você usou derivadas no problema 4? Cheguei a pensar nisso na hora mas achei um problema ( não sei se estava fazendo errado ) O problema foi esse: P(n) (a probabilidade com n dados) = n.5^(n-1)/(6^n) (a^b= a elevado a b). Logo P'(n) = n.5^(n-1).(ln5 - ln6)/6^n . 0 é raiz de P'(n) , logo se n é diferente de 0 . n.5^(n-1).(ln5 - ln6)/6^n = 0. Multiplicando ambos os lados por 6^n/(n.5^(n-1)) que é real porque 5^(n-1) é diferente de 0. => ln5 - ln6 = 0 => ln5 = ln6 => 5 = 6. O que implica que 0 é a única raiz de P'(n). A única explicação lógica para isso é que P'(n) não esta definida em nenhuma outra "raiz", ou seja a função apenas tende a zero no limite. André T. ----- Original Message ----- From: "Tonik" To: Sent: Monday, September 16, 2002 12:11 PM Subject: Re: [obm-l] 2ª fase nivel 3... > > Eu consegui fazer 4 problemas, sendo q um eu usei derivadas, acho q aceitam neh? ... > Quanto foi a nota de corte ano passado? > Quando costumam soltar a prova com as respostas no site? > valew > > > > 14/09/02 19:05:41, "Adherbal Rocha Filho" wrote: > >Putz, essa prova tava muito,muito,muito,muito dificil!!!!!!!!!! > >Espero q pelo menos a nota de corte pra 3ª fase seja menor q 50 pnts... > >a do ano passado tava tao facilzinha...mas acho q jah sei, ano passado vcs > >fizeram uma 1ª fase dificil e uma segunda bem facil, esse ano foi o > >contrario...soh q a 2ª vale muito mais pontos q a 1ª !!!! pô, assim eh > >dificil... > >falou > > > > > > > > ========================================================================= > Instruções para entrar na lista, sair da lista e usar a lista em > http://www.mat.puc-rio.br/~nicolau/olimp/obm-l.html > O administrador desta lista é > ========================================================================= ========================================================================= Instruções para entrar na lista, sair da lista e usar a lista em http://www.mat.puc-rio.br/~nicolau/olimp/obm-l.html O administrador desta lista é ========================================================================= From owner-obm-l@sucuri.mat.puc-rio.br Mon Sep 16 15:14:40 2002 Return-Path: Received: (from majordom@localhost) by sucuri.mat.puc-rio.br (8.9.3/8.9.3) id PAA05301 for obm-l-MTTP; Mon, 16 Sep 2002 15:14:09 -0300 Received: from sv1sintese.sintese.com (mail.sintese.com [200.176.113.35]) by sucuri.mat.puc-rio.br (8.9.3/8.9.3) with ESMTP id PAA05290 for ; Mon, 16 Sep 2002 15:14:03 -0300 Received: from wsjujuba (unknown [10.0.0.195]) by sv1sintese.sintese.com (Postfix) with SMTP id 9394327AD14 for ; Mon, 16 Sep 2002 14:54:09 -0300 (BRT) Message-ID: <001c01c25daa$44730f90$c300000a@wsjujuba> From: =?iso-8859-1?Q?Juliana_L=F6ff?= To: Subject: [obm-l] =?iso-8859-1?Q?d=FAvida_de_pir=E2mide?= Date: Mon, 16 Sep 2002 14:55:38 -0300 MIME-Version: 1.0 Content-Type: multipart/related; type="multipart/alternative"; boundary="----=_NextPart_000_0018_01C25D91.1EEEE910" X-Priority: 3 X-MSMail-Priority: Normal X-Mailer: Microsoft Outlook Express 6.00.2600.0000 X-MimeOLE: Produced By Microsoft MimeOLE V6.00.2600.0000 Sender: owner-obm-l@sucuri.mat.puc-rio.br Precedence: bulk Reply-To: obm-l@mat.puc-rio.br This is a multi-part message in MIME format. ------=_NextPart_000_0018_01C25D91.1EEEE910 Content-Type: multipart/alternative; boundary="----=_NextPart_001_0019_01C25D91.1EEEE910" ------=_NextPart_001_0019_01C25D91.1EEEE910 Content-Type: text/plain; charset="iso-8859-1" Content-Transfer-Encoding: quoted-printable Imagine que na figura abaixo, voc=EA una os pontos A, B,C e D = transformando esse "plano de papel" em um objeto. Qual o volume desse = objeto sendo que os lados dos tri=E2ngulos menores (na figura) medem = raiz de 2? =20 ------=_NextPart_001_0019_01C25D91.1EEEE910 Content-Type: text/html; charset="iso-8859-1" Content-Transfer-Encoding: quoted-printable
Imagine que na figura abaixo, voc=EA = una os pontos A,=20 B,C e D transformando esse "plano de papel" em um objeto. Qual o = volume=20 desse objeto sendo que os lados dos tri=E2ngulos menores (na figura) = medem raiz de=20 2?
 
=20
 
------=_NextPart_001_0019_01C25D91.1EEEE910-- ------=_NextPart_000_0018_01C25D91.1EEEE910 Content-Type: image/jpeg; name="tri.jpg" Content-Transfer-Encoding: base64 Content-ID: <001701c25daa$4432f950$c300000a@wsjujuba> /9j/4AAQSkZJRgABAQEBLAEsAAD/2wBDAAgGBgcGBQgHBwcJCQgKDBQNDAsLDBkSEw8UHRofHh0a HBwgJC4nICIsIxwcKDcpLDAxNDQ0Hyc5PTgyPC4zNDL/2wBDAQkJCQwLDBgNDRgyIRwhMjIyMjIy MjIyMjIyMjIyMjIyMjIyMjIyMjIyMjIyMjIyMjIyMjIyMjIyMjIyMjIyMjL/wAARCACUAJ8DASIA AhEBAxEB/8QAHwAAAQUBAQEBAQEAAAAAAAAAAAECAwQFBgcICQoL/8QAtRAAAgEDAwIEAwUFBAQA AAF9AQIDAAQRBRIhMUEGE1FhByJxFDKBkaEII0KxwRVS0fAkM2JyggkKFhcYGRolJicoKSo0NTY3 ODk6Q0RFRkdISUpTVFVWV1hZWmNkZWZnaGlqc3R1dnd4eXqDhIWGh4iJipKTlJWWl5iZmqKjpKWm p6ipqrKztLW2t7i5usLDxMXGx8jJytLT1NXW19jZ2uHi4+Tl5ufo6erx8vP09fb3+Pn6/8QAHwEA AwEBAQEBAQEBAQAAAAAAAAECAwQFBgcICQoL/8QAtREAAgECBAQDBAcFBAQAAQJ3AAECAxEEBSEx BhJBUQdhcRMiMoEIFEKRobHBCSMzUvAVYnLRChYkNOEl8RcYGRomJygpKjU2Nzg5OkNERUZHSElK U1RVVldYWVpjZGVmZ2hpanN0dXZ3eHl6goOEhYaHiImKkpOUlZaXmJmaoqOkpaanqKmqsrO0tba3 uLm6wsPExcbHyMnK0tPU1dbX2Nna4uPk5ebn6Onq8vP09fb3+Pn6/9oADAMBAAIRAxEAPwD3PVtS h0bRr7VLhZGgsreS4kWMAsVRSxAyQM4HqKj0XU/7X0wXRh8mRZZbeWMNuCyRSNG+1sDK7kbBIBIw SAeBHrmirrtvbWk88kdolws08cZZGlCAlAHUhkIkEb7lOfkx0JqPRvD8Wh3l89pcTta3flyNFPK8 z+cAVaQyyMzNuQRLtJwPL46mgDYorl9WTUpfFsFr/bt9ptjdWmLUWkVuQ06MxkV2lif5mRkKqp5E UpIG3mx/wj2qf9Dnrn/fmy/+R6AOgorn/wDhHtU/6HPXP+/Nl/8AI9H/AAj2qf8AQ565/wB+bL/5 HoA6Ciuf/wCEe1T/AKHPXP8AvzZf/I9H/CPap/0Oeuf9+bL/AOR6AOgorn/+Ee1T/oc9c/782X/y PWPr2neILf7HYaV401X+072XbEZ7WzkjijX5pZHVYFbaF+UEEDe8YJG7NAHcUVx+iWOqaxo8F63i 3XIJm3RzweXZN5MyMUkj3fZhu2urLuHBxkcEVof8I9qn/Q565/35sv8A5HoA6Ciuf/4R7VP+hz1z /vzZf/I9H/CPap/0Oeuf9+bL/wCR6AOgorn/APhHtU/6HPXP+/Nl/wDI9H/CPap/0Oeuf9+bL/5H oA6Ciuf/AOEe1T/oc9c/782X/wAj1X0NNSHijUIW12+1CwsolhlF3FbjNy4WTA8uJCNkZQ/xBvOH QocgFy58XaPZ39zZzSXYe1cJcyrYztBASiv88wQxqArKxJYAA5OK3K8z8Q+F77UrzxXbLpOsu+rP ttLyDVPJtFDWsUQaWITqWAdW3fu2JUdG4FemUAFFFFAGXr+mzalpbCzaOPULdxcWUkhIVZk5UMQM hG5R8clHcd6saVqUOr6Xb38CyIkyZMcoAeJujI4BO11YFWHYgjtVyuf0z/iV+KNS0xuIb/OpWYHC jARJ0CjphykhPG5rhuCVYkA6CiiigAooooAK5/w3/wATXzPEk3z/AG3P9n7ufKszjZt9PM2iU5Ab 51Vs+WuDXv8AibajZ+Hk+aGX/StQxyBboeIz1/1r4Xaww8azjqK6CgDn9P8A+JZ4tv8ATB8lrfRD ULZO3mBtlwFA4VcmB8EZLzSHJ529BXP+L/8ARNJi1wctokp1AoejxrG6TDHdvKeTaMgb9uTjNdBQ AUUUUAFFFFAGXrupTafZJHZLG+p3jm3sI5QdjTbGYFyCMIqozNznCkLliAbGlabDpGl29hA0jpCm DJKQXlbqzuQBudmJZj3JJ71l6B/xN7ybxK3MF1EkemA9VtcBt+D91pGO48jKJDuAZSB0FABRRRQA UUUUAFY/iSxuLrTo7qwj8zUtPlF3aJuA8x1BDR5PA8xGkj3HO3fuxlRWxRQBXsL631PTra/s5PMt bqJJoX2kbkYAqcHkZBHWrFc/oP8AxKdRvPDz/LDF/pWn54Bt3PMY6f6p8rtUYSNoB1NdBQAVXv76 30zTrm/vJPLtbWJ5pn2k7UUEscDk4APSrFc/df8AE68UQWI+ax0rbdXPdZLgj9zGeoOwZlKnBVjb sKALHh6xuIbM3+pR7dWvv3tyCwYwjJKQBhwVjDbQVwGO58Zds7FFFABXP+FP9Bs7jQZPlk0qUwxJ 2+yklrcrnllEeI9x6vFIMsQSegrn9T/4lfijTdTXiG/xpt4TwowHeByx6YcvGBxua4XklVBAOgoo ooAK5/X/APib3kPhpeYLqJ5NTI6ra4K7Mj7rSMdo4GUSbaQygjU1XUodI0u4v51kdIUyI4gC8rdF RASNzsxCqO5IHeq+habNp9k8l60b6neOLi/kiJ2NNsVSEBAwiqiqvGcKC2WJJANSiiigAooooAKK KKACiiigDn/Fn+g6cviGPibRt10+OsluB+/j98oCyrkAyRxE8LXQUVz/AIY/4l32rw2/H9l7Psvv Zvnyf++drxcksfJ3H74oA0NZ1P8AsrTnmjh+03b5jtLQNta5mwSsYODjOCS2MKoZjhVJBommf2Po 8Fk03nzLuknn27fOmdi8km3J27nZm2jgZwOAKz7X/ideKJ74/NY6VutbbuslwR++kHUHYMRBhgqx uFNdBQAUUUUAFZ+uaZ/bOh3unCbyJJ4mWKcLuMEnVJFGR8yMFYEEEFQQQea0KKAM/RNT/tjR4L1o fImbdHPBu3eTMjFJI92Bu2urLuHBxkcEVoVz+n/8Szxbf6YPktb6IahbJ28wNsuAoHCrkwPgjJea Q5PO2xr99cRxQ6Zp0nl6nqG+K3l2hhbAKS07Kc5VOOMYLtGpK78gArxf8T7xGt0P+PDRpZI4j1E9 0UCs6kfwxq8kfU5dpAQDGCegqvY2Nvp1nHa2sflwpkgFixJJJZmY5LMSSSxJJJJJJNWKACiiigAo oooAKKKKACiiigAri/iHfLotvpmqw3slnqD3H9mW8kds1wzC4GCAgByVZEmxjLeRsBG812lcnd6b D4x1S/Fw0g0+wRrS1kjI3C6ODJcRkgjfF8qI4wyOJwR0oAksPEmh6Zp1tYWdjrkdraxJDCn9h3x2 ooAUZMWTgAdasf8ACZaX/wA+uuf+CK9/+M1c0LUptQsnjvVjTU7Nxb38cQOxZtisShJOUZXVl5zh gGwwIGpQBz//AAmWl/8APrrn/givf/jNH/CZaX/z665/4Ir3/wCM10FFAHP/APCZaX/z665/4Ir3 /wCM0f8ACZaX/wA+uuf+CK9/+M10FFAHB+KPFmnLZW2qx22s+ZpFwL4LLot4iMgR0lBLRAA+VJJt JZQGCliFzXQaHBNd3Fzrd9FIk9w7JaRzKQ1vaggKMHBUyFRKwIDDcqNny1xHrX/Ey1zSdHT5445R qF6vYRx5MQJHKsZ/LdRwGEMgyQCpPDf/ABKvM8NzfJ9iz/Z+7jzbMY2bfXy9wiOSW+RWbHmLkA6C iiigAooooAKKKKACiiigAooooAz9b1P+x9HnvVh8+ZdscEG7b50zsEjj3YO3c7Ku48DOTwDRommf 2Po8Fk03nzLuknn27fOmdi8km3J27nZm2jgZwOAKz7r/AInXiiCxHzWOlbbq57rJcEfuYz1B2DMp U4KsbdhXQUAc/q3/ABJ9cg177tnJF9k1Jz0jjXc8UzHnCozOpwAMTF2IWPjoKjnghureW3uIo5oJ UKSRyKGV1IwQQeCCOMVj+HZ5oEl0K+lklvtORf3sjF2nt2Z1hlZu7lYyHzg71cgbSpIBuUUUUAFF FYfiueYaMdOs5ZIr/VX+w2skbFWiZ1YtKDxzHGskmMjPl7QQSKAI/DX/ABMJdQ8QPy19KYbcjoLW FnWIqRwyuTJMGxyJgMkKpqTxLBNHbwazZxSS3mluZhHEpZ54SMTQgDliy/Mq5AMiRE8CtiCCG1t4 re3ijhgiQJHHGoVUUDAAA4AA4xUlAEcE8N1bxXFvLHNBKgeOSNgyupGQQRwQRzmpK5/Tf+JLrl3p cnyWd7K13YOehkfc88IPdgwaUZOSJWCjbEcdBQAUUUUAFFFFABRRRQAVXv7630zTrm/vJPLtbWJ5 pn2k7UUEscDk4APSrFc/rv8AxNNW07Q4+VWWPULtx1jjhkDxjPOGeVUwCMMkc2CCooAseG7G4tdO kur+Py9S1CU3d2m4Hy3YALHkcHy0WOPcMbtm7GWNbFFFABWH4ggmt3ttetIpJrjTUl8y3iUs9zbs oMkSDn59yRuvGSYwuVDk1uUUARwTw3VvFcW8sc0EqB45I2DK6kZBBHBBHOakrn9H/wCJLqz+HT/x 6tE93pzHqI/MxJDgcBYi8YX7o2SKoB8tmPQUAFc/D/xNfGlzI3z2ujxLDEOwupRukJB/iWIw7WGM CeQZOSF1NW1KHRtGvtUuFkaCyt5LiRYwCxVFLEDJAzgeoqv4e02bStEht7po2u3eS4uTESU86V2l kCZAOze7Bc84xnJ5oA1KKKKAMvX9Nm1LS2Fm0ceoW7i4spJCQqzJyoYgZCNyj45KO471Y0vUodWs Fu4VkQF3jeOQANHIjlHQ4JGVZWXIJBxkEjBq5XPy/wDEh8RtdH/jw1mWOOU9BBdBCquxP8MipHH1 GHWMAEyEgA6CiiigAooooAKKKKACuf8AC3+nRXviA8rq0omtieStqqhYQD12sA0wUgbTOwIzkk8W f6dpy+Ho+ZtZ3Wr46x25H7+T2whKq2CBJJEDw1dBQAUUUUAFFFFAGXrmmzX9vbS2bRpf2Vwt1atI SFLAFWQ8HAeNpI92G279wBKirGlalDq+l29/AsiJMmTHKAHiboyOATtdWBVh2II7Vcrn4v8AiQ+I 1tR/x4azLJJEOgguggZkUD+GRUkk6DDrISSZAAAGtf8AEy1zSdHT5445RqF6vYRx5MQJHKsZ/LdR wGEMgyQCp6Cuf8Nf8TCXUPED8tfSmG3I6C1hZ1iKkcMrkyTBsciYDJCqa6CgAooooAKp6rpsOr6X cWE7SIkyYEkRAeJuquhIO11YBlPYgHtVyigDL0LUptQsnjvVjTU7Nxb38cQOxZtisShJOUZXVl5z hgGwwIGpXP6x/wASXVk8RD/j1aJLTUVHUR+ZmObJ4CxF5C33RskZiT5aqegoAKKKKAKeqalDpNg1 3MsjgOkaRxgFpJHcIiDJAyzMq5JAGckgZNR6Zc6pN5qarp0FpIuCjW1158bg54yURgwxyCuMFcEn IWTVNNh1awa0maRAXSRJIyA0ciOHRxkEZVlVsEEHGCCMio9MttUh819V1GC7kbARba18iNAM84Lu xY55JbGAuADksAZd5omvP4ln1ax1jTY0a3S3hhutNeYwqCWcqwnXl2I3YAyI4wfu5Mn2Pxh/0HdD /wDBNN/8lV0FFAHP/Y/GH/Qd0P8A8E03/wAlUfY/GH/Qd0P/AME03/yVXQUUAc/9j8Yf9B3Q/wDw TTf/ACVR9j8Yf9B3Q/8AwTTf/JVdBRQBz/2Pxh/0HdD/APBNN/8AJVZ+t+G/E+vaPPpt14g0qOOX aRLb6XNHJE6sGR0YXXDKyqw9xyCOK7CigDm4NM8VWtvFb2+r+H4YIkCRxx6JKqooGAABdYAA4xUn 2Pxh/wBB3Q//AATTf/JVdBRQBz/2Pxh/0HdD/wDBNN/8lUfY/GH/AEHdD/8ABNN/8lV0FFAHP/Y/ GH/Qd0P/AME03/yVR9j8Yf8AQd0P/wAE03/yVXQUUAc3Ppniq6t5be41fw/NBKhSSOTRJWV1IwQQ brBBHGKkW5uPCXhCzGpzT6xdW/kWnmQIEkuXeRYkJEkmNxLLuJfk5PHSugrH8T6feanoogsBA11H d2tyizyGNG8qeOUqWCsRkIRnB60AWNM1ZNS82J7aezvIcGa0udvmRhs7W+VmVlbBwykjIYZ3KwGh WPpOn3i6jd6vqQgjvrqKK3MFtIZI4o4jIV+dlUsxMrknaowVGPlLNsUAFFFFABRRRQAUUUUAFFFF ABRRRQAUUUUAFFFFABRRRQAUUUUAFFFFAH//2Q== ------=_NextPart_000_0018_01C25D91.1EEEE910-- ========================================================================= Instruções para entrar na lista, sair da lista e usar a lista em http://www.mat.puc-rio.br/~nicolau/olimp/obm-l.html O administrador desta lista é ========================================================================= From owner-obm-l@sucuri.mat.puc-rio.br Mon Sep 16 15:24:10 2002 Return-Path: Received: (from majordom@localhost) by sucuri.mat.puc-rio.br (8.9.3/8.9.3) id PAA05511 for obm-l-MTTP; Mon, 16 Sep 2002 15:16:59 -0300 Received: from traven10.pub1 (traven10.uol.com.br [200.221.4.45]) by sucuri.mat.puc-rio.br (8.9.3/8.9.3) with ESMTP id PAA05503 for ; Mon, 16 Sep 2002 15:16:37 -0300 Received: from u2z7z2 ([200.158.144.91]) by traven10.pub1 (8.9.1/8.9.1) with ESMTP id PAA27308 for ; Mon, 16 Sep 2002 15:17:16 -0300 (BRT) Message-ID: <003a01c25dad$8d79f2a0$5b909ec8@u2z7z2> From: "Wagner" To: Subject: [obm-l] =?iso-8859-1?Q?Problema_3_do_n=EDvel_3=282=AA_fase_de_2002=29?= Date: Mon, 16 Sep 2002 15:19:09 -0300 Organization: Wagner MIME-Version: 1.0 Content-Type: multipart/alternative; boundary="----=_NextPart_000_0037_01C25D94.6784E1E0" X-Priority: 3 X-MSMail-Priority: Normal X-Mailer: Microsoft Outlook Express 5.50.4133.2400 X-MimeOLE: Produced By Microsoft MimeOLE V5.50.4133.2400 Sender: owner-obm-l@sucuri.mat.puc-rio.br Precedence: bulk Reply-To: obm-l@mat.puc-rio.br This is a multi-part message in MIME format. ------=_NextPart_000_0037_01C25D94.6784E1E0 Content-Type: text/plain; charset="iso-8859-1" Content-Transfer-Encoding: quoted-printable Oi pessoal ! Eu queria que algu=E9m me dissesse se a solu=E7=E3o que eu dei no = problema 3 do n=EDvel 3 =E9 v=E1lida: Se r(a/b) =E9 o resto da divis=E3o de a por b. Logo = r(a/b).r(c/b)=3Dr(ac/b) e r(a/b)+r(c/b)=3Dr(r((a+c)/b)/b) e sendo = a^b=3Da elevado a b. r((n^3-3n^2+22)/3^k) =3D 0 =3D> para k>ou=3D3, como r(22/3^k) =3D 22 = =3D> r((n^3 - 3n^2)/3^k) + 22 =3D 3^k =3D> r(n^2/3^k).r((n-3)/3^k) =3D = 3^k - 22 . Donde deduz-se que n nunca ser=E1 inteiro (porqu=EA = r(n^2/3^k).r((n-3)/3^k) ser=E1 sempre par e 3^k sempre impar), logo k<3 = .Portanto o valor m=E1ximo de k =E9 2.=20 Andr=E9 T. ------=_NextPart_000_0037_01C25D94.6784E1E0 Content-Type: text/html; charset="iso-8859-1" Content-Transfer-Encoding: quoted-printable
Oi pessoal !
 
Eu queria que algu=E9m me dissesse se a = solu=E7=E3o que=20 eu dei no problema 3 do n=EDvel 3 =E9 v=E1lida:
 
Se r(a/b) =E9 o resto da divis=E3o de a = por b. Logo=20 r(a/b).r(c/b)=3Dr(ac/b)   e  =20 r(a/b)+r(c/b)=3Dr(r((a+c)/b)/b)   e sendo a^b=3Da elevado a=20 b.
 
r((n^3-3n^2+22)/3^k) =3D 0 =3D> para = k>ou=3D3, como=20 r(22/3^k) =3D 22  =3D> r((n^3 - 3n^2)/3^k) + 22 =3D 3^k=20 =3D> r(n^2/3^k).r((n-3)/3^k) =3D 3^k - 22 .
Donde deduz-se que n nunca ser=E1 = inteiro (porqu=EA=20 r(n^2/3^k).r((n-3)/3^k) ser=E1 sempre par e 3^k sempre impar), logo = k<3=20 .Portanto o valor m=E1ximo de k =E9 2.
 
Andr=E9 T.
------=_NextPart_000_0037_01C25D94.6784E1E0-- ========================================================================= Instruções para entrar na lista, sair da lista e usar a lista em http://www.mat.puc-rio.br/~nicolau/olimp/obm-l.html O administrador desta lista é ========================================================================= From owner-obm-l@sucuri.mat.puc-rio.br Mon Sep 16 15:51:11 2002 Return-Path: Received: (from majordom@localhost) by sucuri.mat.puc-rio.br (8.9.3/8.9.3) id PAA06855 for obm-l-MTTP; Mon, 16 Sep 2002 15:44:39 -0300 Received: from web12908.mail.yahoo.com (web12908.mail.yahoo.com [216.136.174.75]) by sucuri.mat.puc-rio.br (8.9.3/8.9.3) with SMTP id PAA06850 for ; Mon, 16 Sep 2002 15:44:35 -0300 Message-ID: <20020916184556.68958.qmail@web12908.mail.yahoo.com> Received: from [200.206.103.3] by web12908.mail.yahoo.com via HTTP; Mon, 16 Sep 2002 15:45:56 ART Date: Mon, 16 Sep 2002 15:45:56 -0300 (ART) From: =?iso-8859-1?q?Johann=20Peter=20Gustav=20Lejeune=20Dirichlet?= Subject: Re: [obm-l] 2ª fase nivel 3... To: obm-l@mat.puc-rio.br In-Reply-To: MIME-Version: 1.0 Content-Type: multipart/alternative; boundary="0-1386338502-1032201956=:68416" Content-Transfer-Encoding: 8bit Sender: owner-obm-l@sucuri.mat.puc-rio.br Precedence: bulk Reply-To: obm-l@mat.puc-rio.br --0-1386338502-1032201956=:68416 Content-Type: text/plain; charset=iso-8859-1 Content-Transfer-Encoding: 8bit MY GOD OF SKY!!!!!!!!!! Consegui fazer cinco problemas,deixei o 6 pra tras.De resto so preciso mesmo e torcer pra ter ido bem.Parece que em 2 semanas eles soltam os gabaritos. Tonik wrote: Eu consegui fazer 4 problemas, sendo q um eu usei derivadas, acho q aceitam neh? ... Quanto foi a nota de corte ano passado? Quando costumam soltar a prova com as respostas no site? valew > 14/09/02 19:05:41, "Adherbal Rocha Filho" wrote: >Putz, essa prova tava muito,muito,muito,muito dificil!!!!!!!!!! >Espero q pelo menos a nota de corte pra 3ª fase seja menor q 50 pnts... >a do ano passado tava tao facilzinha...mas acho q jah sei, ano passado vcs >fizeram uma 1ª fase dificil e uma segunda bem facil, esse ano foi o >contrario...soh q a 2ª vale muito mais pontos q a 1ª !!!! pô, assim eh >dificil... >falou > > ========================================================================= Instruções para entrar na lista, sair da lista e usar a lista em http://www.mat.puc-rio.br/~nicolau/olimp/obm-l.html O administrador desta lista é ========================================================================= --------------------------------- Yahoo! PageBuilder - O super editor para criação de sites: é grátis, fácil e rápido. --0-1386338502-1032201956=:68416 Content-Type: text/html; charset=iso-8859-1 Content-Transfer-Encoding: 8bit

MY GOD OF SKY!!!!!!!!!! Consegui fazer cinco problemas,deixei o 6 pra tras.De resto so preciso mesmo e torcer pra ter ido  bem.Parece que em 2 semanas eles soltam os gabaritos.

 Tonik wrote:


Eu consegui fazer 4 problemas, sendo q um eu usei derivadas, acho q aceitam neh? ...
Quanto foi a nota de corte ano passado?
Quando costumam soltar a prova com as respostas no site?
valew

>
14/09/02 19:05:41, "Adherbal Rocha Filho" wrote:
>Putz, essa prova tava muito,muito,muito,muito dificil!!!!!!!!!!
>Espero q pelo menos a nota de corte pra 3ª fase seja menor q 50 pnts...
>a do ano passado tava tao facilzinha...mas acho q jah sei, ano passado vcs
>fizeram uma 1ª fase dificil e uma segunda bem facil, esse ano foi o
>contrario...soh q a 2ª vale muito mais pontos q a 1ª !!!! pô, assim eh
>dificil...
>falou
>
>



=========================================================================
Instruções para entrar na lista, sair da lista e usar a lista em
http://www.mat.puc-rio.br/~nicolau/olimp/obm-l.html
O administrador desta lista é
=========================================================================


Yahoo! PageBuilder - O super editor para criação de sites: é grátis, fácil e rápido. --0-1386338502-1032201956=:68416-- ========================================================================= Instruções para entrar na lista, sair da lista e usar a lista em http://www.mat.puc-rio.br/~nicolau/olimp/obm-l.html O administrador desta lista é ========================================================================= From owner-obm-l@sucuri.mat.puc-rio.br Mon Sep 16 15:58:55 2002 Return-Path: Received: (from majordom@localhost) by sucuri.mat.puc-rio.br (8.9.3/8.9.3) id PAA07466 for obm-l-MTTP; Mon, 16 Sep 2002 15:58:03 -0300 Received: from hotmail.com (f147.sea2.hotmail.com [207.68.165.147]) by sucuri.mat.puc-rio.br (8.9.3/8.9.3) with ESMTP id PAA07460 for ; Mon, 16 Sep 2002 15:58:00 -0300 Received: from mail pickup service by hotmail.com with Microsoft SMTPSVC; Mon, 16 Sep 2002 11:59:19 -0700 Received: from 32.94.119.254 by sea2fd.sea2.hotmail.msn.com with HTTP; Mon, 16 Sep 2002 18:59:19 GMT X-Originating-IP: [32.94.119.254] From: "Paulo Santa Rita" To: obm-l@mat.puc-rio.br Subject: [obm-l] =?iso-8859-1?B?UmU6IFtvYm0tbF0gUHJvYmxlbWEgNCBPQk0gdW5pdmVyc2l04XJpYQ==?= Date: Mon, 16 Sep 2002 18:59:19 +0000 Mime-Version: 1.0 Content-Type: text/plain; charset=iso-8859-1; format=flowed Message-ID: X-OriginalArrivalTime: 16 Sep 2002 18:59:19.0606 (UTC) FILETIME=[29781560:01C25DB3] Sender: owner-obm-l@sucuri.mat.puc-rio.br Precedence: bulk Reply-To: obm-l@mat.puc-rio.br Ola FREDERICO e demais colegas desta lista ... OBM-L, Uma ideia obvia seria : observando que as raizes de X^2 + X + 1 = 0 sao X1=-(1/2)+(Raiz(3)/2)i e X2=-(1/2)-(Raiz(3)/2)i e que esses complexos tem modulo unitario e um argumento facil de encontrar,isto e : X1=cos(2pi/3)+sen(2pi/3)i e X2=cos(4pi/3)+sen(4pi/3)i. agora, X+1=(1/2)+(Raiz(3)/2)i se X=X1 e X+1=(1/2)-(Raiz(3)/2)i se X=X2. Segue que (X+1)^M e X^M sao faceis de calcular aplicando a 1 formula de Moivre. De resto e so igualar a zero e resolver a equacao. Um abraco Paulo Santa Rita 2,1557,160902 >From: "fredericogomes" >Reply-To: obm-l@mat.puc-rio.br >To: obm-l@mat.puc-rio.br >Subject: [obm-l] Problema 4 OBM universitária >Date: Mon, 16 Sep 2002 11:21:22 -0300 > >Pessoal, consegui encontrar usando ajuda de um software >que para m = 4 + 6k o polinômio é divisível, se alguém >souber mostrar isso, gostaria de uma ajuda! > > PROBLEMA 4 >Determine todos os valores inteiros positivos de m para >os quais o polinômio (x+1)^m + x^m +1 é divisível por >(x^2 + x + 1)^2. > > >__________________________________________________________________________ >AcessoBOL, só R$ 9,90! O menor preço do mercado! >Assine já! http://www.bol.com.br/acessobol > > >========================================================================= >Instruções para entrar na lista, sair da lista e usar a lista em >http://www.mat.puc-rio.br/~nicolau/olimp/obm-l.html >O administrador desta lista é >========================================================================= _________________________________________________________________ MSN Photos é a maneira mais fácil e prática de editar e compartilhar sua fotos: http://photos.msn.com.br ========================================================================= Instruções para entrar na lista, sair da lista e usar a lista em http://www.mat.puc-rio.br/~nicolau/olimp/obm-l.html O administrador desta lista é ========================================================================= From owner-obm-l@sucuri.mat.puc-rio.br Mon Sep 16 15:59:53 2002 Return-Path: Received: (from majordom@localhost) by sucuri.mat.puc-rio.br (8.9.3/8.9.3) id PAA07629 for obm-l-MTTP; Mon, 16 Sep 2002 15:59:49 -0300 Received: from smtp-32.ig.com.br (smtp-32.ig.com.br [200.226.132.182]) by sucuri.mat.puc-rio.br (8.9.3/8.9.3) with SMTP id PAA07625 for ; Mon, 16 Sep 2002 15:59:45 -0300 Message-Id: <200209161859.PAA07625@sucuri.mat.puc-rio.br> Received: (qmail 32062 invoked from network); 16 Sep 2002 19:06:00 -0000 Received: from webmail-4.ig.com.br (HELO localhost) (200.226.133.91) by smtp-32.ig.com.br with SMTP; 16 Sep 2002 19:06:00 -0000 Date: Mon, 16 Sep 2002 16:00:51 -0300 To: obm-l@mat.puc-rio.br From: Leonardo Zão Cc: X-Originating-IP: [200.226.133.110]200.20.123.165 X-Mailer: InMail by Insite - www.insite.com.br X-user: leoaugzao@ig.com.br MIME-Version: 1.0 Content-type: text/plain Subject: Re: [obm-l] Problema_4_OBM_universitária Sender: owner-obm-l@sucuri.mat.puc-rio.br Precedence: bulk Reply-To: obm-l@mat.puc-rio.br Em 16 Sep 2002, obm-l@mat.puc-rio.br escreveu: >Pessoal, consegui encontrar usando ajuda de um software >que para m = 4 + 6k o polinômio é divisível, se alguém >souber mostrar isso, gostaria de uma ajuda! > > PROBLEMA 4 >Determine todos os valores inteiros positivos de m para >os quais o polinômio (x+1)^m + x^m +1 é divisível por >(x^2 + x + 1)^2. > Eu fiz assim: Seja w uma raiz de x^2+x+1. Provemos que ela é raiz de P(x)=(x+1)^m + x^m + 1 com m=6k+4. como w^2+w+1=0, w+1=-w^2 Substituindo no dividendo:P(w)=(-w^2)^(6k+4)+w^(6k+4)+1= (w^(6k+4))^2 + w^(6k+4) + 1. Repare que w é raiz cúbica da unidade, ou seja, w^3=1. Assim, w^(6k+4)=w e P(w)=w^2+w+1=0. Agora temos que provar que w também é raiz da derivada de P(x) P´(x)=m((x+1)^(m-1)) + m(x^(m-1)) m=6k+4 => P´(w)=m[(w+1)^6k+3 + w^(6k+3)]=m[-w^(6k+3)^2 + w^(6k+3)]=m[-1^2 + 1]=0. Assim, as raízes de x^2+x+1 são raízes duoplas de P(x), sendo P(x), portanto, divisível por (x^2+x+1)^2. Espero q tenha ajudado e q esteja certo (foi assim que eu fiz). _________________________________________________________ Oi! Você quer um iG-mail gratuito? Então clique aqui: http://registro.ig.com.br/ ========================================================================= Instruções para entrar na lista, sair da lista e usar a lista em http://www.mat.puc-rio.br/~nicolau/olimp/obm-l.html O administrador desta lista é ========================================================================= From owner-obm-l@sucuri.mat.puc-rio.br Mon Sep 16 16:12:21 2002 Return-Path: Received: (from majordom@localhost) by sucuri.mat.puc-rio.br (8.9.3/8.9.3) id QAA08491 for obm-l-MTTP; Mon, 16 Sep 2002 16:11:51 -0300 Received: from puma.unisys.com.br (smtp.unisys.com.br [200.220.64.7]) by sucuri.mat.puc-rio.br (8.9.3/8.9.3) with ESMTP id QAA08467 for ; Mon, 16 Sep 2002 16:11:43 -0300 Received: from josefran (riohiper01p177.uninet.com.br [200.220.2.177]) by puma.unisys.com.br (8.12.3/8.12.3) with SMTP id g8GJCoF6010483 for ; Mon, 16 Sep 2002 16:12:56 -0300 (EST) X-Spam-Filter: check_local@puma.unisys.com.br by digitalanswers.org Message-ID: <00f801c25db5$d2d91120$ae10dcc8@josefran> From: "Jose Francisco Guimaraes Costa" To: "obm-l" Subject: En: [obm-l] Livros Date: Mon, 16 Sep 2002 16:15:55 -0300 MIME-Version: 1.0 Content-Type: multipart/alternative; boundary="----=_NextPart_000_00E3_01C25D9C.5619ADC0" X-Priority: 3 X-MSMail-Priority: Normal X-Mailer: Microsoft Outlook Express 5.00.2314.1300 X-MimeOLE: Produced By Microsoft MimeOLE V5.00.2314.1300 Sender: owner-obm-l@sucuri.mat.puc-rio.br Precedence: bulk Reply-To: obm-l@mat.puc-rio.br This is a multi-part message in MIME format. ------=_NextPart_000_00E3_01C25D9C.5619ADC0 Content-Type: text/plain; charset="iso-8859-1" Content-Transfer-Encoding: quoted-printable A Editora da USP publicou a tradu=E7=E3o em portugu=EAs do primeiro = volume do Principia. V pode compr=E1-lo na Livraria da F=EDsica = (www.livrariadafisica.com.br). Em ingl=EAs, a edi=E7=E3o mais barata =E9 = a da Prometheus. Tanto quanto eu saiba n=E3o existe tradu=E7=E3o do Elementos de = Euclides. Uma edi=E7=E3o em ingl=EAs, comentada por Thomas L Heath, =E9 = o "The Thirteen Books of Euclid's Elements", acho que da Dover, ISBN's = 0486600882 (livros I e II), 486600890 (livros III a IX) e 0486600882 = (livros X a XIII).=20 As edi=E7=F5es acima tanto do Principia como do Elementos est=E3o = dispon=EDveis na Amazon (www.amazon.com). JF -----Mensagem Original-----=20 De: Daniel=20 Para: Lista OBM=20 Enviada em: Segunda-feira, 16 de Setembro de 2002 12:38 Assunto: [obm-l] Livros Ol=E1 a todos: Gostaria de saber onde posso encontrar os livros = completos: Princ=EDpios Matem=E1ticos da Filosofia Natural = - Isaac Newton Os Elementos - Euclides De prefer=EAncia em Portugu=EAs, mas pode ser em = Franc=EAs ou Ingl=EAs Daniel O. Costa - S=E3o Jos=E9 dos Campos SP ------=_NextPart_000_00E3_01C25D9C.5619ADC0 Content-Type: text/html; charset="iso-8859-1" Content-Transfer-Encoding: quoted-printable
A Editora da USP publicou a tradu=E7=E3o em portugu=EAs do primeiro = volume do=20 Principia. V pode compr=E1-lo na Livraria da F=EDsica (www.livrariadafisica.com.br). Em=20 ingl=EAs, a edi=E7=E3o mais barata =E9 a da Prometheus.
 
Tanto quanto eu saiba n=E3o existe tradu=E7=E3o do Elementos de = Euclides. Uma=20 edi=E7=E3o em ingl=EAs, comentada por Thomas L Heath, =E9 o "The = Thirteen Books of=20 Euclid's Elements", acho que da Dover, ISBN's 0486600882 (livros I e = II),=20 486600890 (livros III a IX) e 0486600882 (livros X a XIII).
 
As edi=E7=F5es acima tanto do Principia como do Elementos est=E3o = dispon=EDveis na=20 Amazon (www.amazon.com).
 
JF
 
-----Mensagem Original-----
De: Daniel=20
Para: Lista OBM
Enviada em: Segunda-feira, 16 de Setembro de 2002 = 12:38
Assunto: [obm-l] Livros

       =20     Ol=E1 a todos:
 
       =20             Gostaria de = saber onde=20 posso encontrar os livros completos:
 
       =20             =    =20 Princ=EDpios Matem=E1ticos da Filosofia Natural - Isaac = Newton
       =20             =     Os=20 Elementos - Euclides
 
          &nbs= p;    =20 De prefer=EAncia em Portugu=EAs, mas pode ser em Franc=EAs ou = Ingl=EAs
 
       =20     Daniel O. Costa - S=E3o Jos=E9 dos Campos=20 SP
------=_NextPart_000_00E3_01C25D9C.5619ADC0-- ========================================================================= Instruções para entrar na lista, sair da lista e usar a lista em http://www.mat.puc-rio.br/~nicolau/olimp/obm-l.html O administrador desta lista é ========================================================================= From owner-obm-l@sucuri.mat.puc-rio.br Mon Sep 16 16:55:06 2002 Return-Path: Received: (from majordom@localhost) by sucuri.mat.puc-rio.br (8.9.3/8.9.3) id QAA10878 for obm-l-MTTP; Mon, 16 Sep 2002 16:54:26 -0300 Received: from Euler.impa.br (euler.impa.br [147.65.1.3]) by sucuri.mat.puc-rio.br (8.9.3/8.9.3) with ESMTP id QAA10874 for ; Mon, 16 Sep 2002 16:54:22 -0300 Received: from obm-01 (obm-01.impa.br [147.65.2.170]) by Euler.impa.br (8.11.6/8.11.6) with SMTP id g8GJth727822 for ; Mon, 16 Sep 2002 16:55:43 -0300 (EST) Message-Id: <3.0.5.32.20020916165618.007cfe50@pop.impa.br> X-Sender: obm@pop.impa.br X-Mailer: QUALCOMM Windows Eudora Light Version 3.0.5 (32) Date: Mon, 16 Sep 2002 16:56:18 -0300 To: obm-l@mat.puc-rio.br From: Olimpiada Brasileira de Matematica Subject: [obm-l] OBM-2002 In-Reply-To: <00f801c25db5$d2d91120$ae10dcc8@josefran> Mime-Version: 1.0 Content-Type: text/plain; charset="us-ascii" Sender: owner-obm-l@sucuri.mat.puc-rio.br Precedence: bulk Reply-To: obm-l@mat.puc-rio.br Oi Gente, Provas e solucoes da Segunda Fase da OBM Niveis 1, 2, 3 e da Primeira Fase Nivel U ja estao no site. Confiram! :) Alias daqui a alguns minutos a Eureka No 14 tambem no site. :) Abracos, Nelly. ========================================================================= Instruções para entrar na lista, sair da lista e usar a lista em http://www.mat.puc-rio.br/~nicolau/olimp/obm-l.html O administrador desta lista é ========================================================================= From owner-obm-l@sucuri.mat.puc-rio.br Mon Sep 16 17:18:05 2002 Return-Path: Received: (from majordom@localhost) by sucuri.mat.puc-rio.br (8.9.3/8.9.3) id RAA11722 for obm-l-MTTP; Mon, 16 Sep 2002 17:16:28 -0300 Received: from data2.poli.usp.br (data2.poli.usp.br [143.107.106.108]) by sucuri.mat.puc-rio.br (8.9.3/8.9.3) with ESMTP id RAA11718 for ; Mon, 16 Sep 2002 17:16:26 -0300 Received: from web2.poli.usp.br ([143.107.106.102]) by data2.poli.usp.br with Microsoft SMTPSVC(5.0.2195.4453); Mon, 16 Sep 2002 17:17:46 -0300 Received: from apl03.poli.usp.br ([143.107.106.15]) by web2.poli.usp.br with Microsoft SMTPSVC(5.0.2195.5329); Mon, 16 Sep 2002 17:17:24 -0300 X-MimeOLE: Produced By Microsoft Exchange V6.0.5762.3 content-class: urn:content-classes:message MIME-Version: 1.0 Content-Type: multipart/mixed; boundary="----_=_NextPart_001_01C25DBE.11B3D7CC" Subject: RES: [obm-l] Livros Date: Mon, 16 Sep 2002 17:17:24 -0300 Message-ID: X-MS-Has-Attach: X-MS-TNEF-Correlator: Thread-Topic: [obm-l] Livros Thread-Index: AcJduwTkQGSWjYzqSXOe1bp7V60dRAAAmh/6 From: "Andre Wulff Hirano" To: X-OriginalArrivalTime: 16 Sep 2002 20:17:24.0897 (UTC) FILETIME=[121EC110:01C25DBE] Sender: owner-obm-l@sucuri.mat.puc-rio.br Precedence: bulk Reply-To: obm-l@mat.puc-rio.br This is a multi-part message in MIME format. ------_=_NextPart_001_01C25DBE.11B3D7CC Content-Type: text/plain; charset="iso-8859-1" Content-Transfer-Encoding: quoted-printable Pode cr=EA ! Aqui na fisica eu vi esse livro , e realmente vale a pena = ter na sua cole=E7ao. Assim como varios bem interessantes na area de = fisica moderna. Vi uns de equa=E7oes diferenciais e matemat. aplicada = que achei sensacionais.. Se vc tiver oportunidade de vir pra c=E1 ,venha = com o bolso preparado porque vc vai querer levar v=E1rios...=20 -----Mensagem original-----=20 De: Jose Francisco Guimaraes Costa [mailto:jfgcosta@unisys.com.br]=20 Enviada: seg 16/9/2002 16:15=20 Para: obm-l=20 Cc:=20 Assunto: En: [obm-l] Livros =09 =09 A Editora da USP publicou a tradu=E7=E3o em portugu=EAs do primeiro = volume do Principia. V pode compr=E1-lo na Livraria da F=EDsica = (www.livrariadafisica.com.br). Em ingl=EAs, a edi=E7=E3o mais barata =E9 = a da Prometheus. =20 Tanto quanto eu saiba n=E3o existe tradu=E7=E3o do Elementos de = Euclides. Uma edi=E7=E3o em ingl=EAs, comentada por Thomas L Heath, =E9 = o "The Thirteen Books of Euclid's Elements", acho que da Dover, ISBN's = 0486600882 (livros I e II), 486600890 (livros III a IX) e 0486600882 = (livros X a XIII).=20 =20 As edi=E7=F5es acima tanto do Principia como do Elementos est=E3o = dispon=EDveis na Amazon (www.amazon.com). =20 JF =20 -----Mensagem Original-----=20 De: Daniel =20 Para: Lista OBM =20 Enviada em: Segunda-feira, 16 de Setembro de 2002 12:38 Assunto: [obm-l] Livros Ol=E1 a todos: =20 Gostaria de saber onde posso encontrar os livros = completos: =20 Princ=EDpios Matem=E1ticos da Filosofia Natural = - Isaac Newton Os Elementos - Euclides =20 De prefer=EAncia em Portugu=EAs, mas pode ser em = Franc=EAs ou Ingl=EAs =20 Daniel O. Costa - S=E3o Jos=E9 dos Campos SP ------_=_NextPart_001_01C25DBE.11B3D7CC Content-Type: application/ms-tnef; name="winmail.dat" Content-Transfer-Encoding: base64 eJ8+IhkUAQaQCAAEAAAAAAABAAEAAQeQBgAIAAAA5AQAAAAAAADoAAEIgAcAGAAAAElQTS5NaWNy b3NvZnQgTWFpbC5Ob3RlADEIAQ2ABAACAAAAAgACAAEEgAEAFAAAAFJFUzogW29ibS1sXSBMaXZy b3MAcgYBBYADAA4AAADSBwkAEAARABEAGAABAC0BASCAAwAOAAAA0gcJABAAEQARABgAAQAtAQEJ gAEAIQAAADJGQjMxMDUxN0NBNzkxNDQ5MkQxNDUyM0Q3RTI1NzVDAPwGAQOQBgDoFQAANgAAAAMA NgAAAAAAQAA5AMzXsxG+XcIBHgA9AAEAAAAGAAAAUkVTOiAAAAACAUcAAQAAAC0AAABjPUJSO2E9 IDtwPUVQVVNQO2w9QVBMMDMtMDIwOTE2MjAxNzI0Wi02MDg0MQAAAAAeAEkAAQAAABMAAABFbjog W29ibS1sXSBMaXZyb3MAAEAATgCAJ8V6tV3CAR4AWgABAAAAHwAAAEpvc2UgRnJhbmNpc2NvIEd1 aW1hcmFlcyBDb3N0YQAAAgFbAAEAAABTAAAAAAAAAIErH6S+oxAZnW4A3QEPVAIAAAAASm9zZSBG cmFuY2lzY28gR3VpbWFyYWVzIENvc3RhAFNNVFAAamZnY29zdGFAdW5pc3lzLmNvbS5icgAAAgFc AAEAAAAcAAAAU01UUDpKRkdDT1NUQUBVTklTWVMuQ09NLkJSAB4AXQABAAAAHwAAAEpvc2UgRnJh bmNpc2NvIEd1aW1hcmFlcyBDb3N0YQAAAgFeAAEAAABTAAAAAAAAAIErH6S+oxAZnW4A3QEPVAIA AAAASm9zZSBGcmFuY2lzY28gR3VpbWFyYWVzIENvc3RhAFNNVFAAamZnY29zdGFAdW5pc3lzLmNv bS5icgAAAgFfAAEAAAAcAAAAU01UUDpKRkdDT1NUQUBVTklTWVMuQ09NLkJSAB4AZgABAAAABQAA AFNNVFAAAAAAHgBnAAEAAAAXAAAAamZnY29zdGFAdW5pc3lzLmNvbS5icgAAHgBoAAEAAAAFAAAA U01UUAAAAAAeAGkAAQAAABcAAABqZmdjb3N0YUB1bmlzeXMuY29tLmJyAAAeAHAAAQAAAA8AAABb b2JtLWxdIExpdnJvcwAAAgFxAAEAAAAbAAAAAcJduwTkQGSWjYzqSXOe1bp7V60dRAAAmh/6AB4A dAABAAAABgAAAG9ibS1sAAAAHgAaDAEAAAATAAAAQW5kcmUgV3VsZmYgSGlyYW5vAAAeAB0OAQAA AA8AAABbb2JtLWxdIExpdnJvcwAAAgEJEAEAAABxDwAAbQ8AAKo/AABMWkZ1Xx8EAwMACgByY3Bn MTI1gjIDQ2h0bWwxAzA/AQMB9wqAAqQD4wIAY2jBCsBzZXQwIAcTAoD/EAMAUARWCFUHshHVDlED Ad0Q1zIGAAbDEdUzBEYQ2fkS72Y0EG8RdhHjCO8J97Y7Gr8OMDUR0gxgYwBQMwsJAWQzNhFgC6U0 INkQAipcDrIBkGcU8AqjCxHjH9g0FPA8IURPAENUWVBFIEhUAE1MIFBVQkxJAEMgIi0vL1czIkMj cERURCKENC7JEWBUcgBydGkCIAdA4SNwRU4iPiDdIH8mgdwxOCHgIpIlzTMfcCgw+EVBRCXNDvEm 7yrfJ7QCNg7wPE1FVEEgxwWgAjAJ8HQ9IgXgIpOCNSSAMC4yMzEkcGIxL0AwIiAlQAeAPQJHJZBF UkFUT1IvKf8sXyGSLaFTIkBMRdclzjNRIN45IeAvM58xz7smRTNSLym/Jp86BDURYAA8Qk9EWSBi ZzMIUBpRPSMBIDzSIGT2aTygOiByOXA54wAhAzCSdgiQd2sLgGQ1OZS6YgDQawnACGA+0CAAADhz aHA5skAxC4BzdKtAEUBRcwOgZgMQbDxjUQKRc3YgLYA3QrAy7DE1GdBBGEZBsQmAQjXLGdBEoVw9 gWRvAOA9gfkKsVxxGqA9gRDwAzA95Y8RYDmbHuE6n2c5NiHgeERJVj25AABGhzm5NnY0Sb9G0lAE cS4QRcAnEGVhICERcHF1aY8wEUGRDdFNwGV1ID5wR07gBBBNUGxpdgNgIPosTuAgGsAHQAeALkFP EPcHQE1QTcBwCfBNwC5QBcBnTlE4HEf1c3VNwBoxZeFNkTdhby4RcAQQB3AvLhEEYFDRBRBvBCBi Zf9UoAuAUaFPUQBwLlAEIE5RXwrATbFNQU6FBGRhVEBWv04wUh9IIj/QBCBNQWVOEFphU+JvB5E9 QGZV8W7+YwcwBABQEQDALlBb8VRA9mELUE6xZE3AThBRERDwfmVOMBEwAIAA0CUiBAAud1RABmBP EGNYf0gTJRB22RMxb3AaYT/QaVzgTUH3TUE+cAXAcCTALhBNkQ7w+ixgcG4RAFSyYKA8MAbw9nNP 4GHwZQqxXNBjoQWw/10SXtFQ4FhvSARdERrABcAXRABVETmbOB9wJm5iZ0EwAoA92CdhAUBGh3b3 YkJVMl5wLmdea+A9yEZpfzm5AcA9xwqiPccKcSc8MP8pgSPQSYtvCEcPSB9JIyMAwSIgS1FVT1Qi cD1FGV2gdHlEAC6RQVJHQElOLVJJRyKQOvAgMHB4JbE92AqxEAL/RXVGE0XRRm9y7yELEWBr8b9z 30l/bO9Ln3n2RPBpHsKFJzw0KBFGT05UXaB0aXowUDJ+SwviecktNYXCTV3CZ1WRBbBpZ/8LgAdA hcOCLS2AO9Ew8T3Y+U+gbmUKgXnvZ9Y74H5LYmJ5yURlOn0MIdEvs4uacdkgSlVQTVBGJMG/W3AE 8E/gZU97j2vER04gvwDAJMAHkQhQQPBNwFsAwIEDEHRvOmpmZwWgZZQxQGEBc3leYFTBLvhicl2H f4iPiZ+Kr4u/+ZkFRW4+cFzRjV+Ob493PREwZ5Dvkf9rtS2ALznaLwHQMBTwLYA6QvCWb/+Xf5iP mZ+ar0y2ZAGcn52v4Y93b2JtLQMgow+kH8+lL6Y/p0+upENjqP+qD/+Pd6wPrR+uL68/sE+3JFRh /z/QlMGx77L/j4acIHdAn49boJ9rxFuro5ZQTE+yc3+0j7Wftq/BL8I/w08nljX9feEvg1K7720f bi9vP3BP/8ifel++j3x/fY+Av3+v0w97gcUuAEU9QJTAYgFc4VX0U1BRQHUCYA3gCGBRId89kFzQ DHBT8U2RM0/gVZH1YMNn2KJhWfFjogdxPVD7VPEG8HUHgNpCvZ/P/2vE/lAFEFthQMBYIGdfaG9p ev5WZFFNQgNwYfBiQqvQT+CvTlHAklUh1yNGTZFkTqNeKLePLgAKo2vgaBrAZqMukA6wdHA6I3B3 5wD+Lk+i42Jc4E6EleV3mgHxaQiQbGQ5smbpwEDTSOUiUVIjEE5LI0Dmn+evfiJDEuohESA6IBog AEF1/3m668/oGM2vgZLxH87Nu4PqQTlwKVRARVSg27/cz/trxAuAZxqg2gJQAE7RPUD/2LiUgVVh ZAGUQU2RNhDXI+/eMANwEUBdcHVeYMnPyt//y+/M//Kvzx/QL9E/0k/TX//Ub9V/t2/fz2lc+2/8 f/2P//6f/68AvwHPAt8D7wT/Bg//Bx8kkVZBT+BaURgiTvFWMO5pP2AwENj1eJWgULHYbb1kMUVE AFCCVVFNQUUdcP9PoB5gXmDXYJSA9S8RT2u1//iLVZP312MBLmFc0mDBJKDuaPrQ2iEkUEhNsPsA UAD7+hNP4CIiIE1QIiBhwFCwQVFgIEJvb2v5gG/fPSAdHx4voaQcNCf5gBtlfHMi+EFdYBhCYWFN wESGb2BxUABJU0JOJ+GJDmA4NoIAMDg4OmAWKE+j+YBJUBFJSSnfUAAlDyYfoaQqdTkQICsX4yvQ USFJWClQESpv+YD6WFEhWC/B9NELLww/DU//Dl8PbxB/EY8SnxOvFL8Vz/8W3wgvCT8KTzKfM680 vzXP/zbfN+84/zoPOx9Bnz0/1a/zW7H4lmY1WtFd8RzhR+D/GCJkMd43VLQbO1rQ7iAa5PeVoGDA GWJkYHBboSwvSF/roaTi8UGUgHpeIOR/5YL95j9hVwOV4ui/6c/q31mb/+2P7p5ZuEYP8h9G7/Q6 Qi//Qz9ET0VfYq9Hf0iPSZ9Kr/9Lv0zPTd8/L0A/QU9lL2Y//2dPaF9pb2p/a49sn22vdC/zb89w 3kpGdL91z3bfd+//eP96D3sffC99P35Pf1+Ab/9xv3LPc9+B74L/hA+FH4Yvf4c/iE+JX4pvkO+M j42eLa2eUk0hcBkAZ9lRT9qQ8mdRIGFsnlORb5J/k49/lJ+Vr5a/l8+Y35nmGPB0PnkbcFjgyCK9 cKNAcHT7L/DjcWxaXKSvpb+mz6ffAajoQkFDS0dST4hVTkS9cCNlNLESf6qPm2+cf43Pt8+436uj RP5luq+7v7zHtX/lrflRrdC8bzrYAFLQ59DjsG5bQf5An1DoRhow1uCpQb1v6GN/Wm9bf1yFvP++ C15fX2pE/73Tua9iH6uf9DhUP6z/HyX/oB+hL6I/o0/I76wvrT+uT3+Z/7Ivsz+0T7Vftm9feVA/ +bG4T7lfum9X/8M1b2KMbS2+EPlQdC5wHED6LVEQb76T5cW+5OIvwH/fwY/Cn+RfxN9fLkwZ8VbQ +E9CTd7fyF/Sb8p5za//zr/Pz9Df7e/S/9QP1R/WLz/XP9hP2V/ab9t/X3lFbn52qlAhsRuA3X/e j9+XU5BlZ3VuIbAtZlQAe91QLAAx+LAcAQMAJCBtN+mwUzEwYDIqsGRgMTL4OjM48C/xP/JP81/0 b//1f/aP95/4r/m/+s/73/zv9/3/X3lPAHMDMMNxAG8Bf/Xfplvok13roStCBb8Gz/8H3wjvCf8b 3xzvHf8LfwyPvw2fDq8g3x8gBWATgVITuf+8IiEgJb8e3x/vIP8iDyMfPyQvJT8P/zAfEh8p0zQ4 s2RwqYIgZlAQqVBBqkJpr8BpeqlQMuWcy/Fm/nMFEF95jp+Pr5C6OE85X/86bzt/PI/frz6fP689 30Hf/0LvQ/9FD0BPy4/Mn0a/R89/Rf9Mf02PTp9Pr1C/LkFPMccwJ2UxqiC+0G9k08OwFbwxNa9h L6mCKq//kS8aDxsfKc9XbyvvSp8uD/8vHzJPMT9h3zNfUo9Tn1h//1mPWp9br1y/Xc9e31/vYP// Yg9jH2QvZT80nzWvNr9s//9mD2cfeY96n3uvfL99z0j//3/fgO9/H4MfhC+FP4ZPgY//Sd9MH0v/ iQ+HP42/js+P3/+Q75H/ir+UD5Uflj+XT5hf/5lvmn+Vv5yPnZ+bz5/PoN93oe+i/546R8OyqkEE 0nN4YWJlvFCLH4wvzURvKwNABPBww7BzBMBlbt/DoBWA3VC8UMOwIHTQGQL6IMShcL8QFZBVr1a/ V8//aJ9pr2q/a8+kH23vbv9wD/9xH7Avcz90T6Tfpe+6X7Ef/7Ivsz+0T7Vftm+3f7iPuZ//uq+7 v7zPdY92n3eveL+9n/++r6QP0i/TP9RP1V/Wb57f/9h/1q/ar9u/3M/d397v2i//qY+M7+Cf4a/f 3+Zf52/of//pj+JP65/sr+rf7t/v7/D///IP7U/0H/Uv81/3X/hv+X//+o/7n/bf/a/73//fAO8B /98DDwQf/1/j3+TqUKfwrAARVPFkcGmskU1hdGRlbVTydGmtMKygZJFVQEZpbKewb2aoARpOC9B1 rFDPQC0gSaeoYM7ADXBldxWQbq3f/67vr//Az8Hfwu/D/wT/xh//xy/IP8lPEV/Lb8x/zY/On//P r9C/BX8GjwS/I28kfyWP/yafJ6//jym/J+8r7yz/Lg//Lx8wLytvCR/k7zHfMu8xH/83nzivOb86 zzOPPN897zwf/0AfQS9CP0NPRF9FX0ZvRJ//SJ9Jr0q/S89M30gfTu9NH79RH1IvUz9UT1VfUJVP rKDXNH81jzaURa1wbavwrZGxDfFFdWMeMKtgcw7v/w//EQ8SHxMvFD8VT1ZPF2//GH8ZjxqfYB8c vx3PVw9YH/9qT2EPYh9jL2Q/ZU9mX2dv/2h/a69qn3lPbL8fDyAYdkP/p4AgvyHPdR9uL1XPgh+D L/+EP4VPhl+Hb4h/iY+Kn4uv/4y/jc+O34/vkP+SD5MflC//lT+WT5dfmG+Zf5qPm5+cr/+dv57P n9+g76H/ow+kH6Uvf6Y/p0+oX1AfWg9bH3eiRABlIHByZWZlch0LMWELEA1RC/AgUG8RcWB1Z3Ww QnMsIPZtsaCv0G9dcH+QsCCwwk5GDcALE7Hxb3UOEG78Z2yxc12vXr9fz3Cfca//cr9zz6nPde92 /3gPeR+2z/97P3xPqo+rn8D/t7+4z7nf/7rvu/+9D74fvy/CX8FPz///w299jyAfgB+BL8Q/xU/X v//Yz9nf2u/b/6wv3g/fH91P/+FP4l/jb+R/5Y+tD8zvNm//5z/oT+Z/7P/uD+8f8C/o6hJEsHBp ZdXATy4gLkMNEMygDfFTCzEzb2wgSg0QCzE5DKBc0UOcYW2yIFnP6pRTULRf/7Vvtn/HT8hfyW/K f/Fv/sgAQkxPQ0tRVU98VEX/m+qhzAAAX83ENWPUsQIQT0RZ1kDNXDeh/zFIVE1M1kB9B5AAAAAe ADUQAQAAADsAAAA8RDAzOEZBMEY5NjdEQzk0N0E0NTk0OEE1N0M4QzEzM0E1NEMyOTdAYXBsMDMu cG9saS51c3AuYnI+AAAeAEcQAQAAAA8AAABtZXNzYWdlL3JmYzgyMgAACwDyEAEAAAAfAPMQAQAA ADQAAABSAEUAUwAlADMAQQAgAFsAbwBiAG0ALQBsAF0AIABMAGkAdgByAG8AcwAuAEUATQBMAAAA CwD2EAAAAABAAAcw4glhbb1dwgFAAAgwUhHOEb5dwgEDAN4/r28AAAMA8T8WBAAAHgD4PwEAAAAT AAAAQW5kcmUgV3VsZmYgSGlyYW5vAAACAfk/AQAAAFAAAAAAAAAA3KdAyMBCEBq0uQgAKy/hggEA AAAAAAAAL089RVBVU1AvT1U9RElSRVRPUklBL0NOPVJFQ0lQSUVOVFMvQ049QU5EUkUuSElSQU5P AB4A+j8BAAAAFQAAAFN5c3RlbSBBZG1pbmlzdHJhdG9yAAAAAAIB+z8BAAAAHgAAAAAAAADcp0DI wEIQGrS5CAArL+GCAQAAAAAAAAAuAAAAAwD9P+QEAAADABlAAAAAAAMAGkAAAAAAAwAdQAAAAAAD AB5AAAAAAB4AMEABAAAADQAAAEFORFJFLkhJUkFOTwAAAAAeADFAAQAAAA0AAABBTkRSRS5ISVJB Tk8AAAAAHgAyQAEAAAAXAAAAamZnY29zdGFAdW5pc3lzLmNvbS5icgAAHgAzQAEAAAAXAAAAamZn Y29zdGFAdW5pc3lzLmNvbS5icgAAHgA4QAEAAAANAAAAQU5EUkUuSElSQU5PAAAAAB4AOUABAAAA AgAAAC4AAAALACkAAAAAAAsAIwAAAAAAAwAGEAMXjXgDAAcQ9gQAAAMAEBAAAAAAAwAREAAAAAAe AAgQAQAAAGUAAABQT0RFQ1LqQVFVSU5BRklTSUNBRVVWSUVTU0VMSVZSTyxFUkVBTE1FTlRFVkFM RUFQRU5BVEVSTkFTVUFDT0xF50FPQVNTSU1DT01PVkFSSU9TQkVNSU5URVJFU1NBTlRFU05BAAAA AAIBfwABAAAAOwAAADxEMDM4RkEwRjk2N0RDOTQ3QTQ1OTQ4QTU3QzhDMTMzQTU0QzI5N0BhcGww My5wb2xpLnVzcC5icj4AAI5l ------_=_NextPart_001_01C25DBE.11B3D7CC-- ========================================================================= Instruções para entrar na lista, sair da lista e usar a lista em http://www.mat.puc-rio.br/~nicolau/olimp/obm-l.html O administrador desta lista é ========================================================================= From owner-obm-l@sucuri.mat.puc-rio.br Mon Sep 16 17:23:48 2002 Return-Path: Received: (from majordom@localhost) by sucuri.mat.puc-rio.br (8.9.3/8.9.3) id RAA11883 for obm-l-MTTP; Mon, 16 Sep 2002 17:23:33 -0300 Received: from smtp.ieg.com.br (stone.protocoloweb.com.br [200.226.139.11]) by sucuri.mat.puc-rio.br (8.9.3/8.9.3) with ESMTP id RAA11878 for ; Mon, 16 Sep 2002 17:23:30 -0300 Received: from localhost (wh1stler@200-158-118-125.dsl.telesp.net.br [200.158.118.125]) by smtp.ieg.com.br (IeG relay/8.9.3) with SMTP id g8GKKQO3002859 for ; Mon, 16 Sep 2002 17:20:26 -0300 (BRT) From: Tonik To: obm-l@mat.puc-rio.br Date: Mon, 16 Sep 2002 17:23:55 -0300 X-Priority: 3 (Normal) Organization: Tonik In-Reply-To: <002601c25dab$1366f320$5b909ec8@u2z7z2> Message-Id: Subject: =?windows-1252?Q?Re:=20[obm-l]=20Re:=20[obm-l]=202=AA=20fase=20nivel=203...?= MIME-Version: 1.0 Content-Type: text/plain; charset="iso-8859-1" X-Mailer: Opera 6.04 build 1135 Sender: owner-obm-l@sucuri.mat.puc-rio.br Precedence: bulk Reply-To: obm-l@mat.puc-rio.br Ola.. Eu sei muito pouco das derivadas, nunca tinha usado antes num exercicio sequer, entao dado P(n)=n5^(n-1)/6^n eu nao sabia ao certo como derivar, mas entao lembrei de um teorema, e fiz: chamei f(n)=n5^(n-1) e g(n)=6^-n entao P(n) = f(n)g(n) P'(n) = 0 P'(n) = f(n)g'(n) + f'(n)g(n) P'(n) = n*5^(n-1)*(-n)*6^(-n-1) + n*(n-1)*5^(n-2)*6^(-n) n*n = n*(n-1)*5^(n-2-n+1)*6^(-n+n+1) n ser 0 eh um absurdo, concorda? n = (n-1)*5^(-1)*6 n=6 Note que 0 tambem eh raiz de P'(n). Eu nao sei aonde envolver ln para derivar como vc fez.. mais tarde, depois da prova, comecei a testar alguns valores no P(n) e notei que para n=5 e para n=6 o valor de P(n) é maximo, logo acho q a resposta mais correta, com quantos dados no minimo jogaremos, é n=5, mas eu respondi n=6 na prova. Quanto aos outros problemas, o primeiro deu 11; o segundo, dos poligonos, eu coloquei n=4k, k pertence a N* o terceiro, do 3^k eu nao consegui mas escrevi q 22 = 3*7+1 (tentando ganhar algum ponto) o quinto, dos pontos medios, eu nao fiz o sexto das combinacoes pra mim deu 9*m*n*2^(m*n) falow 16/09/02 15:01:26, "Wagner" wrote: > >Oi para todos > >Você usou derivadas no problema 4? >Cheguei a pensar nisso na hora mas achei um problema ( não sei se estava >fazendo errado ) > >O problema foi esse: P(n) (a probabilidade com n dados) = n.5^(n-1)/(6^n) >(a^b= a elevado a b). >Logo P'(n) = n.5^(n-1).(ln5 - ln6)/6^n . 0 é raiz de P'(n) , logo se n é >diferente de 0 . >n.5^(n-1).(ln5 - ln6)/6^n = 0. Multiplicando ambos os lados por >6^n/(n.5^(n-1)) que é real porque 5^(n-1) é diferente de 0. => >ln5 - ln6 = 0 => ln5 = ln6 => 5 = 6. O que implica que 0 é a única raiz de >P'(n). A única explicação lógica para isso é que P'(n) não esta definida em >nenhuma outra "raiz", ou seja a função apenas tende a zero no limite. > >André T. > >----- Original Message ----- >From: "Tonik" >To: >Sent: Monday, September 16, 2002 12:11 PM >Subject: Re: [obm-l] 2ª fase nivel 3... > > >> >> Eu consegui fazer 4 problemas, sendo q um eu usei derivadas, acho q >aceitam neh? ... >> Quanto foi a nota de corte ano passado? >> Quando costumam soltar a prova com as respostas no site? >> valew >> >> > >> 14/09/02 19:05:41, "Adherbal Rocha Filho" wrote: >> >Putz, essa prova tava muito,muito,muito,muito dificil!!!!!!!!!! >> >Espero q pelo menos a nota de corte pra 3ª fase seja menor q 50 pnts... >> >a do ano passado tava tao facilzinha...mas acho q jah sei, ano passado >vcs >> >fizeram uma 1ª fase dificil e uma segunda bem facil, esse ano foi o >> >contrario...soh q a 2ª vale muito mais pontos q a 1ª !!!! pô, assim eh >> >dificil... >> >falou >> > >> > >> >> >> ========================================================================= Instruções para entrar na lista, sair da lista e usar a lista em http://www.mat.puc-rio.br/~nicolau/olimp/obm-l.html O administrador desta lista é ========================================================================= From owner-obm-l@sucuri.mat.puc-rio.br Mon Sep 16 18:45:17 2002 Return-Path: Received: (from majordom@localhost) by sucuri.mat.puc-rio.br (8.9.3/8.9.3) id SAA13875 for obm-l-MTTP; Mon, 16 Sep 2002 18:44:52 -0300 Received: from home.iis.com.br (mail.iis.com.br [200.202.96.2]) by sucuri.mat.puc-rio.br (8.9.3/8.9.3) with ESMTP id SAA13871 for ; Mon, 16 Sep 2002 18:44:48 -0300 Received: from Marcio (rio-tc0-tty56.iis.com.br [200.202.98.56]) by home.iis.com.br (8.11.6/8.11.6/1.1.1.16) with SMTP id g8GLk7410281 for ; Mon, 16 Sep 2002 18:46:07 -0300 Message-ID: <001f01c25dca$9a4e60c0$3862cac8@epq.ime.eb.br> From: "Marcio" To: References: Subject: [obm-l] =?iso-8859-1?Q?Re:_=5Bobm-l=5D_Problema_4_OBM_universit=E1ria?= Date: Mon, 16 Sep 2002 18:46:58 -0300 MIME-Version: 1.0 Content-Type: text/plain; charset="iso-8859-1" Content-Transfer-Encoding: 8bit X-Priority: 3 X-MSMail-Priority: Normal X-Mailer: Microsoft Outlook Express 5.50.4133.2400 X-MimeOLE: Produced By Microsoft MimeOLE V5.50.4133.2400 X-AntiVirus: Antivirus for sendmail by Petr Rehor Sender: owner-obm-l@sucuri.mat.puc-rio.br Precedence: bulk Reply-To: obm-l@mat.puc-rio.br Questoes muito parecidas com esse problema ja apareceram aqui nessa lista (por exemplo, num email discutindo sobre a fatoracao de x^5 + x +1)... Vc sabe que d(x) divide P(x) sse as raizes de d(x) sao raizes de P(x) (com multiplicidade igual ou maior). Como as raizes de d(x) sao w,w,w^2,w^2 (onde w eh raiz cubica da unidade), o problema pedia para vc caracterizar quais valores de m fazem com que o polinomio P(x)=(x+1)^m + x^m +1 tenha P(w)=P'(w)=P(w^2)=P'(w^2)=0. Lembrando que 1+w+w^2 = 0 e que P(w^2) = P(w)* (conjugado), vc concluia q m=4+6k.. e 4+6k claramente funciona (quase q o mesmo argumento). []'s Marcio ----- Original Message ----- From: "fredericogomes" To: Sent: Monday, September 16, 2002 11:21 AM Subject: [obm-l] Problema 4 OBM universitária > Pessoal, consegui encontrar usando ajuda de um software > que para m = 4 + 6k o polinômio é divisível, se alguém > souber mostrar isso, gostaria de uma ajuda! > > PROBLEMA 4 > Determine todos os valores inteiros positivos de m para > os quais o polinômio (x+1)^m + x^m +1 é divisível por > (x^2 + x + 1)^2. > > > __________________________________________________________________________ > AcessoBOL, só R$ 9,90! O menor preço do mercado! > Assine já! http://www.bol.com.br/acessobol > > > ========================================================================= > Instruções para entrar na lista, sair da lista e usar a lista em > http://www.mat.puc-rio.br/~nicolau/olimp/obm-l.html > O administrador desta lista é > ========================================================================= > ========================================================================= Instruções para entrar na lista, sair da lista e usar a lista em http://www.mat.puc-rio.br/~nicolau/olimp/obm-l.html O administrador desta lista é ========================================================================= From owner-obm-l@sucuri.mat.puc-rio.br Mon Sep 16 19:47:35 2002 Return-Path: Received: (from majordom@localhost) by sucuri.mat.puc-rio.br (8.9.3/8.9.3) id TAA14902 for obm-l-MTTP; Mon, 16 Sep 2002 19:46:21 -0300 Received: from gorgo.centroin.com.br (gorgo.centroin.com.br [200.225.63.128]) by sucuri.mat.puc-rio.br (8.9.3/8.9.3) with ESMTP id TAA14898 for ; Mon, 16 Sep 2002 19:46:19 -0300 Received: from centroin.com.br (du54b.nit.centroin.com.br [200.225.56.54]) (authenticated bits=0) by gorgo.centroin.com.br (8.12.2/8.12.1) with ESMTP id g8GMm17p017754 for ; Mon, 16 Sep 2002 19:48:02 -0300 (BRT) Message-ID: <3D866042.5000401@centroin.com.br> Date: Mon, 16 Sep 2002 19:50:42 -0300 From: Augusto =?ISO-8859-1?Q?C=E9sar?= Morgado User-Agent: Mozilla/5.0 (Windows; U; Win98; en-US; rv:0.9.4.1) Gecko/20020508 Netscape6/6.2.3 X-Accept-Language: en-us MIME-Version: 1.0 To: obm-l@mat.puc-rio.br Subject: Re: [obm-l] EDO References: <20020916153832.35295.qmail@web21510.mail.yahoo.com> Content-Type: text/plain; charset=ISO-8859-1; format=flowed Content-Transfer-Encoding: 8bit Sender: owner-obm-l@sucuri.mat.puc-rio.br Precedence: bulk Reply-To: obm-l@mat.puc-rio.br Eh isso mesmo? A pergunta se refer principalmente ao y(0) =0. bruno lima wrote: >Numa dessas provas universitarias por ai apareceu: > >y''=y*exp(x) > >y: Vai de [a,b]em R , e y(a)=0 e y(0)=0. > >Estou errado ou a unica solucáo é a identicamente >nula?? >Fazendo uma mudança de variaveis vc cai numa de >Ricati, mas isso nao resolve muito! > >_______________________________________________________________________ >Yahoo! PageBuilder >O super editor para criação de sites: é grátis, fácil e rápido. >http://br.geocities.yahoo.com/v/pb.html >========================================================================= >Instruções para entrar na lista, sair da lista e usar a lista em >http://www.mat.puc-rio.br/~nicolau/olimp/obm-l.html >O administrador desta lista é >========================================================================= > > ========================================================================= Instruções para entrar na lista, sair da lista e usar a lista em http://www.mat.puc-rio.br/~nicolau/olimp/obm-l.html O administrador desta lista é ========================================================================= From owner-obm-l@sucuri.mat.puc-rio.br Mon Sep 16 22:32:13 2002 Return-Path: Received: (from majordom@localhost) by sucuri.mat.puc-rio.br (8.9.3/8.9.3) id WAA16771 for obm-l-MTTP; Mon, 16 Sep 2002 22:30:47 -0300 Received: from traven9.pub1 (traven9.uol.com.br [200.221.4.35]) by sucuri.mat.puc-rio.br (8.9.3/8.9.3) with ESMTP id WAA16767 for ; Mon, 16 Sep 2002 22:30:44 -0300 Received: from u2z7z2 ([200.158.144.234]) by traven9.pub1 (8.9.1/8.9.1) with ESMTP id WAA17013 for ; Mon, 16 Sep 2002 22:33:47 -0300 (BRT) Message-ID: <001501c25dea$344283e0$ea909ec8@u2z7z2> From: "Wagner" To: References: Subject: [obm-l] =?iso-8859-1?Q?Re:_=5Bobm-l=5D_Re:_=5Bobm-l=5D_2=AA_fase_nivel_3...?= Date: Mon, 16 Sep 2002 22:33:19 -0300 Organization: Wagner MIME-Version: 1.0 Content-Type: text/plain; charset="iso-8859-1" Content-Transfer-Encoding: 8bit X-Priority: 3 X-MSMail-Priority: Normal X-Mailer: Microsoft Outlook Express 5.50.4133.2400 X-MimeOLE: Produced By Microsoft MimeOLE V5.50.4133.2400 Sender: owner-obm-l@sucuri.mat.puc-rio.br Precedence: bulk Reply-To: obm-l@mat.puc-rio.br Oi pessoal Primeiro não é tão absurdo n=0 (mas convenhamos que isso não ajudava em nada no problema). Se 0 é raiz de P'(n), então P(0) era ou um máximo ou um mínimo de P(n). Fica evidente que P(0) é um mínimo, já que a probabilidade do evento acontecer sem jogar nenhum dado é zero. Depois eu usei a identidade : f(x) = a^x => f '(x) = lna.a^x (ou seja a derivada de uma função exponencial é outra função exponencial, um caso interessante disso é f(x) = e^x => f '(x) = lne.e^x = 1.e^x => f(x) = f '(x) ) . O seu resultado foi uma feliz coincidência, como a função P(n) possui características exponenciais, ela não possui mais de um ponto de máximo então nem P(5) nem P(6) são as probabilidades máximas, mas são os maiores valores possíveis de P(n) em que n é inteiro, o ponto de máximo seria P(x) em que 5 To: Sent: Monday, September 16, 2002 5:23 PM Subject: Re: [obm-l] Re: [obm-l] 2ª fase nivel 3... > Ola.. > > Eu sei muito pouco das derivadas, nunca tinha usado antes num exercicio > sequer, entao dado P(n)=n5^(n-1)/6^n eu nao sabia ao certo como derivar, > mas entao lembrei de um teorema, e fiz: > chamei f(n)=n5^(n-1) e g(n)=6^-n entao > P(n) = f(n)g(n) > P'(n) = 0 > P'(n) = f(n)g'(n) + f'(n)g(n) > P'(n) = n*5^(n-1)*(-n)*6^(-n-1) + n*(n-1)*5^(n-2)*6^(-n) > n*n = n*(n-1)*5^(n-2-n+1)*6^(-n+n+1) > n ser 0 eh um absurdo, concorda? > n = (n-1)*5^(-1)*6 > n=6 > > Note que 0 tambem eh raiz de P'(n). Eu nao sei aonde envolver > ln para derivar como vc fez.. > > mais tarde, depois da prova, comecei a testar alguns valores > no P(n) e notei que para n=5 e para n=6 o valor de P(n) é > maximo, logo acho q a resposta mais correta, com quantos > dados no minimo jogaremos, é n=5, mas eu respondi n=6 na prova. > > Quanto aos outros problemas, > o primeiro deu 11; > o segundo, dos poligonos, eu coloquei n=4k, k pertence a N* > o terceiro, do 3^k eu nao consegui mas escrevi q 22 = 3*7+1 > (tentando ganhar algum ponto) > o quinto, dos pontos medios, eu nao fiz > o sexto das combinacoes pra mim deu 9*m*n*2^(m*n) > > falow > > 16/09/02 15:01:26, "Wagner" wrote: > > > >Oi para todos > > > >Você usou derivadas no problema 4? > >Cheguei a pensar nisso na hora mas achei um problema ( não sei se estava > >fazendo errado ) > > > >O problema foi esse: P(n) (a probabilidade com n dados) = n.5^(n-1)/(6^n) > >(a^b= a elevado a b). > >Logo P'(n) = n.5^(n-1).(ln5 - ln6)/6^n . 0 é raiz de P'(n) , logo se n é > >diferente de 0 . > >n.5^(n-1).(ln5 - ln6)/6^n = 0. Multiplicando ambos os lados por > >6^n/(n.5^(n-1)) que é real porque 5^(n-1) é diferente de 0. => > >ln5 - ln6 = 0 => ln5 = ln6 => 5 = 6. O que implica que 0 é a única raiz de > >P'(n). A única explicação lógica para isso é que P'(n) não esta definida em > >nenhuma outra "raiz", ou seja a função apenas tende a zero no limite. > > > >André T. > > > >----- Original Message ----- > >From: "Tonik" > >To: > >Sent: Monday, September 16, 2002 12:11 PM > >Subject: Re: [obm-l] 2ª fase nivel 3... > > > > > >> > >> Eu consegui fazer 4 problemas, sendo q um eu usei derivadas, acho q > >aceitam neh? ... > >> Quanto foi a nota de corte ano passado? > >> Quando costumam soltar a prova com as respostas no site? > >> valew > >> > >> > > >> 14/09/02 19:05:41, "Adherbal Rocha Filho" wrote: > >> >Putz, essa prova tava muito,muito,muito,muito dificil!!!!!!!!!! > >> >Espero q pelo menos a nota de corte pra 3ª fase seja menor q 50 pnts... > >> >a do ano passado tava tao facilzinha...mas acho q jah sei, ano passado > >vcs > >> >fizeram uma 1ª fase dificil e uma segunda bem facil, esse ano foi o > >> >contrario...soh q a 2ª vale muito mais pontos q a 1ª !!!! pô, assim eh > >> >dificil... > >> >falou > >> > > >> > > >> > >> > >> > > > > ========================================================================= > Instruções para entrar na lista, sair da lista e usar a lista em > http://www.mat.puc-rio.br/~nicolau/olimp/obm-l.html > O administrador desta lista é > ========================================================================= ========================================================================= Instruções para entrar na lista, sair da lista e usar a lista em http://www.mat.puc-rio.br/~nicolau/olimp/obm-l.html O administrador desta lista é ========================================================================= From owner-obm-l@sucuri.mat.puc-rio.br Mon Sep 16 22:35:37 2002 Return-Path: Received: (from majordom@localhost) by sucuri.mat.puc-rio.br (8.9.3/8.9.3) id WAA16834 for obm-l-MTTP; Mon, 16 Sep 2002 22:35:35 -0300 Received: from imo-d07.mx.aol.com (imo-d07.mx.aol.com [205.188.157.39]) by sucuri.mat.puc-rio.br (8.9.3/8.9.3) with ESMTP id WAA16830 for ; Mon, 16 Sep 2002 22:35:32 -0300 From: Lltmdrtm@aol.com Received: from Lltmdrtm@aol.com by imo-d07.mx.aol.com (mail_out_v34.10.) id z.173.ea8e513 (25508) for ; Mon, 16 Sep 2002 21:36:48 -0400 (EDT) Message-ID: <173.ea8e513.2ab7e130@aol.com> Date: Mon, 16 Sep 2002 21:36:48 EDT Subject: [obm-l] ajuda To: obm-l@mat.puc-rio.br MIME-Version: 1.0 Content-Type: multipart/alternative; boundary="part1_173.ea8e513.2ab7e130_boundary" X-Mailer: AOL 7.0 for Windows BR sub 10501 Sender: owner-obm-l@sucuri.mat.puc-rio.br Precedence: bulk Reply-To: obm-l@mat.puc-rio.br --part1_173.ea8e513.2ab7e130_boundary Content-Type: text/plain; charset="US-ASCII" Content-Transfer-Encoding: 7bit De quantas maneiras 24 pessoas podem subir numa roda gigante de 12 assentos, sabendo que cada assento comporta duas pessoas e, a ordem das pessoas no assento importa? --part1_173.ea8e513.2ab7e130_boundary Content-Type: text/html; charset="US-ASCII" Content-Transfer-Encoding: 7bit De quantas maneiras 24 pessoas podem subir numa roda gigante de 12 assentos, sabendo que cada assento comporta duas pessoas e, a ordem das pessoas no assento importa?

--part1_173.ea8e513.2ab7e130_boundary-- ========================================================================= Instruções para entrar na lista, sair da lista e usar a lista em http://www.mat.puc-rio.br/~nicolau/olimp/obm-l.html O administrador desta lista é ========================================================================= From owner-obm-l@sucuri.mat.puc-rio.br Mon Sep 16 23:06:35 2002 Return-Path: Received: (from majordom@localhost) by sucuri.mat.puc-rio.br (8.9.3/8.9.3) id XAA17841 for obm-l-MTTP; Mon, 16 Sep 2002 23:06:12 -0300 Received: from traven9.pub1 (traven9.uol.com.br [200.221.4.35]) by sucuri.mat.puc-rio.br (8.9.3/8.9.3) with ESMTP id XAA17837 for ; Mon, 16 Sep 2002 23:06:10 -0300 Received: from u2z7z2 ([200.158.144.234]) by traven9.pub1 (8.9.1/8.9.1) with ESMTP id XAA25982 for ; Mon, 16 Sep 2002 23:09:12 -0300 (BRT) Message-ID: <003b01c25def$278527c0$ea909ec8@u2z7z2> From: "Wagner" To: References: <003a01c25dad$8d79f2a0$5b909ec8@u2z7z2> Subject: [obm-l] =?iso-8859-1?Q?Re:_=5Bobm-l=5D_Problema_3_do_n=EDvel_3=282=AA_fase_de_2?= =?iso-8859-1?Q?002=29?= Date: Mon, 16 Sep 2002 23:08:45 -0300 Organization: Wagner MIME-Version: 1.0 Content-Type: multipart/alternative; boundary="----=_NextPart_000_0038_01C25DD6.02165E00" X-Priority: 3 X-MSMail-Priority: Normal X-Mailer: Microsoft Outlook Express 5.50.4133.2400 X-MimeOLE: Produced By Microsoft MimeOLE V5.50.4133.2400 Sender: owner-obm-l@sucuri.mat.puc-rio.br Precedence: bulk Reply-To: obm-l@mat.puc-rio.br This is a multi-part message in MIME format. ------=_NextPart_000_0038_01C25DD6.02165E00 Content-Type: text/plain; charset="iso-8859-1" Content-Transfer-Encoding: quoted-printable Oi para todos Na mensagem anterior eu escrevi o que eu coloquei na prova, essa foi uma = id=E9ia que s=F3 me ocorreu agora: n ser=E1 inteiro para r(n^2/3^k).r((n-3)/3^k) =3D 3^k - 22 e k > ou =3D = 3, se n =E9 inteiro para k=3D3. Porqu=EA se k=3D3 implica em n n=E3o ser = inteiro ent=E3o prova-se por indu=E7=E3o que k>3 implica em n n=E3o ser = inteiro. Para k=3D3 . r(n^2/27).r((n-3)/27) =3D 5 . 5 =E9 um n=FAmero = primo ent=E3o um dos 2 fatores vale 1 e o outro 5. Se r(n^2/27) =3D 1 =3D> n^2 =3D 27a + 1, em que a =E9 natural, ent=E3o = r((n-3)/27) =3D 5 =3D> n-3 =3D 27b + 5 =3D> n =3D 27b + 8. Portanto: 729b^2 + 432b + 64 =3D 27a + 1 =3D> 729b^2 + 432b + 63 =3D 27a =3D = 27(27b^2 + 16b) + 9.7 =3D 27a. Logo a e b n=E3o podem ser inteiros ao = mesmo tempo. Se r(n^2/27) =3D 5 =3D> n^2 =3D 27a + 5 e tamb=E9m que r((n-3)/27) =3D 1 = =3D> n-3=3D27b+1 =3D> n=3D27b+4 .Portanto: 27(27b^2 + 8b) +16 -5 =3D 27a. Logo a e b n=E3o podem ser inteiros ao = mesmo tempo. Isso implica que para k=3D3, n n=E3o =E9 inteiro, logo por = indu=E7=E3o: k=3Dm+3 implica em n n=E3o ser inteiro (m =E9 natural). Se = m =E9 natural e k =E9 diferente de m+3, logo k<3. Portanto o valor = m=E1ximo de k =E9 2. ex: Para n^3 - 3n^2 +22 ser divis=EDvel por 9, n pode ser igual a 5: 125 = - 75 + 22 =3D 72 =3D 8.9. Ainda quero saber se a solu=E7=E3o =E9 v=E1lida ou se s=F3 marquei um = ponto (por ter achado k=3D2 sem uma boa explica=E7=E3o) Andr=E9 T. ----- Original Message -----=20 From: Wagner=20 To: obm-l@mat.puc-rio.br=20 Sent: Monday, September 16, 2002 3:19 PM Subject: [obm-l] Problema 3 do n=EDvel 3(2=AA fase de 2002) Oi pessoal ! Eu queria que algu=E9m me dissesse se a solu=E7=E3o que eu dei no = problema 3 do n=EDvel 3 =E9 v=E1lida: Se r(a/b) =E9 o resto da divis=E3o de a por b. Logo = r(a/b).r(c/b)=3Dr(ac/b) e r(a/b)+r(c/b)=3Dr(r((a+c)/b)/b) e sendo = a^b=3Da elevado a b. r((n^3-3n^2+22)/3^k) =3D 0 =3D> para k>ou=3D3, como r(22/3^k) =3D 22 = =3D> r((n^3 - 3n^2)/3^k) + 22 =3D 3^k =3D> r(n^2/3^k).r((n-3)/3^k) =3D = 3^k - 22 . Donde deduz-se que n nunca ser=E1 inteiro (porqu=EA = r(n^2/3^k).r((n-3)/3^k) ser=E1 sempre par e 3^k sempre impar), logo k<3 = .Portanto o valor m=E1ximo de k =E9 2.=20 Andr=E9 T. ------=_NextPart_000_0038_01C25DD6.02165E00 Content-Type: text/html; charset="iso-8859-1" Content-Transfer-Encoding: quoted-printable
Oi para todos
 
Na mensagem anterior eu escrevi o que = eu coloquei=20 na prova, essa foi uma id=E9ia que s=F3 me ocorreu agora:
 
n ser=E1 inteiro para = r(n^2/3^k).r((n-3)/3^k) =3D 3^k -=20 22 e k > ou =3D 3, se n =E9 inteiro para k=3D3. Porqu=EA se k=3D3 = implica em n n=E3o ser=20 inteiro ent=E3o prova-se por indu=E7=E3o que k>3 implica em n n=E3o = ser inteiro. Para=20 k=3D3 . r(n^2/27).r((n-3)/27) =3D 5 .  5 =E9 um n=FAmero primo = ent=E3o um dos 2=20 fatores vale 1 e o outro 5.
Se r(n^2/27) =3D 1 =3D> n^2 =3D 27a = + 1, em que a =E9=20 natural, ent=E3o r((n-3)/27) =3D 5 =3D> n-3 =3D 27b + 5 =3D> = n =3D 27b + 8.=20 Portanto:
729b^2 + 432b + 64 =3D 27a + 1 =3D> = 729b^2 + 432b +=20 63 =3D 27a =3D 27(27b^2 + 16b) + 9.7 =3D 27a. Logo a e b n=E3o podem ser = inteiros ao=20 mesmo tempo.
Se r(n^2/27) =3D 5 =3D> n^2 =3D 27a = + 5 e tamb=E9m que=20 r((n-3)/27) =3D 1 =3D> n-3=3D27b+1 =3D> n=3D27b+4 = .Portanto:
27(27b^2 + 8b) +16 -5 =3D 27a. = Logo a e b n=E3o=20 podem ser inteiros ao mesmo tempo. Isso implica que para k=3D3, n n=E3o = =E9 inteiro,=20 logo por indu=E7=E3o: k=3Dm+3 implica em n n=E3o ser inteiro (m =E9 = natural). Se m =E9=20 natural e k =E9 diferente de m+3, logo k<3. Portanto o valor m=E1ximo = de k =E9=20 2.
ex: Para n^3 - 3n^2 +22 ser divis=EDvel = por 9, n pode=20 ser igual a 5: 125 - 75 + 22 =3D 72 =3D 8.9.
 
Ainda quero saber se a solu=E7=E3o =E9 = v=E1lida ou se s=F3=20 marquei um ponto (por ter achado k=3D2 sem uma boa = explica=E7=E3o)
 
Andr=E9 T.
 
----- Original Message -----
From:=20 Wagner
Sent: Monday, September 16, = 2002 3:19=20 PM
Subject: [obm-l] Problema 3 do = n=EDvel 3(2=AA=20 fase de 2002)

Oi pessoal !
 
Eu queria que algu=E9m me dissesse se = a solu=E7=E3o que=20 eu dei no problema 3 do n=EDvel 3 =E9 v=E1lida:
 
Se r(a/b) =E9 o resto da divis=E3o de = a por b. Logo=20 r(a/b).r(c/b)=3Dr(ac/b)   e  =20 r(a/b)+r(c/b)=3Dr(r((a+c)/b)/b)   e sendo a^b=3Da elevado a=20 b.
 
r((n^3-3n^2+22)/3^k) =3D 0 =3D> = para k>ou=3D3,=20 como r(22/3^k) =3D 22  =3D> r((n^3 - 3n^2)/3^k) + 22 =3D = 3^k=20 =3D> r(n^2/3^k).r((n-3)/3^k) =3D 3^k - 22 .
Donde deduz-se que n nunca ser=E1 = inteiro (porqu=EA=20 r(n^2/3^k).r((n-3)/3^k) ser=E1 sempre par e 3^k sempre impar), logo = k<3=20 .Portanto o valor m=E1ximo de k =E9 2.
 
Andr=E9 = T.
------=_NextPart_000_0038_01C25DD6.02165E00-- ========================================================================= Instruções para entrar na lista, sair da lista e usar a lista em http://www.mat.puc-rio.br/~nicolau/olimp/obm-l.html O administrador desta lista é ========================================================================= From owner-obm-l@sucuri.mat.puc-rio.br Mon Sep 16 23:43:11 2002 Return-Path: Received: (from majordom@localhost) by sucuri.mat.puc-rio.br (8.9.3/8.9.3) id XAA18655 for obm-l-MTTP; Mon, 16 Sep 2002 23:42:17 -0300 Received: from Servmail.abeu.com.br ([200.255.31.34]) by sucuri.mat.puc-rio.br (8.9.3/8.9.3) with ESMTP id XAA18651 for ; Mon, 16 Sep 2002 23:42:14 -0300 Received: from edmilson ([200.255.31.101]) by Servmail.abeu.com.br (8.11.4/8.11.4) with SMTP id g8H2hZn24814 for ; Mon, 16 Sep 2002 23:43:36 -0300 Message-ID: <002201c25df4$a845b820$651fffc8@edmilson> From: "Edmilson" To: Subject: [obm-l] Problema de um amigo Date: Mon, 16 Sep 2002 23:48:06 -0300 MIME-Version: 1.0 Content-Type: multipart/alternative; boundary="----=_NextPart_000_001F_01C25DDB.816FBD40" X-Priority: 3 X-MSMail-Priority: Normal X-Mailer: Microsoft Outlook Express 6.00.2600.0000 X-MimeOLE: Produced By Microsoft MimeOLE V6.00.2600.0000 Sender: owner-obm-l@sucuri.mat.puc-rio.br Precedence: bulk Reply-To: obm-l@mat.puc-rio.br This is a multi-part message in MIME format. ------=_NextPart_000_001F_01C25DDB.816FBD40 Content-Type: text/plain; charset="iso-8859-1" Content-Transfer-Encoding: quoted-printable Dividir uma circunfer=EAncia em 9 partes iguais, porprocesso = geom=E9trico Atenciosamente, Edmilson edmilson@abeunet.com.br ------=_NextPart_000_001F_01C25DDB.816FBD40 Content-Type: text/html; charset="iso-8859-1" Content-Transfer-Encoding: quoted-printable
Dividir uma circunfer=EAncia em 9 = partes iguais,=20 porprocesso geom=E9trico
 
Atenciosamente,
Edmilson
edmilson@abeunet.com.br
= ------=_NextPart_000_001F_01C25DDB.816FBD40-- ========================================================================= Instruções para entrar na lista, sair da lista e usar a lista em http://www.mat.puc-rio.br/~nicolau/olimp/obm-l.html O administrador desta lista é ========================================================================= From owner-obm-l@sucuri.mat.puc-rio.br Tue Sep 17 00:40:51 2002 Return-Path: Received: (from majordom@localhost) by sucuri.mat.puc-rio.br (8.9.3/8.9.3) id AAA19627 for obm-l-MTTP; Tue, 17 Sep 2002 00:39:22 -0300 Received: from mail.gmx.net (mail.gmx.net [213.165.64.20]) by sucuri.mat.puc-rio.br (8.9.3/8.9.3) with SMTP id AAA19623 for ; Tue, 17 Sep 2002 00:39:18 -0300 Received: (qmail 12357 invoked by uid 0); 17 Sep 2002 03:40:37 -0000 Received: from 49.2-254.9.217.200.telemar.net.br (HELO gomes) (200.217.9.49) by mail.gmx.net (mp015-rz3) with SMTP; 17 Sep 2002 03:40:37 -0000 Date: Tue, 17 Sep 2002 00:41:35 -0300 From: Igor GomeZZ X-Mailer: The Bat! (v1.61) Organization: -- X-Priority: 3 (Normal) Message-ID: <622411757.20020917004135@gmx.net> To: Olimpiada Brasileira de Matematica Subject: Re: [obm-l] OBM-2002 In-Reply-To: <3.0.5.32.20020916165618.007cfe50@pop.impa.br> References: <3.0.5.32.20020916165618.007cfe50@pop.impa.br> MIME-Version: 1.0 Content-Type: text/plain; charset=ISO-8859-1 Content-Transfer-Encoding: 8bit Sender: owner-obm-l@sucuri.mat.puc-rio.br Precedence: bulk Reply-To: obm-l@mat.puc-rio.br Em 16/9/2002, 16:56, Olimpiada (obm@impa.br) disse: > Alias daqui a alguns minutos a Eureka No 14 > tambem no site. Opa! Mas num vai ter versão .pdf (ou .ps) não? Fui! ####### Igor GomeZZ ######## UIN: 29249895 Vitória, Espírito Santo, Brasil Criação: 17/9/2002 (00:40) #################################### Pare para pensar: A pior covardia de uma mulher é despertar o amor de um homem sem ter a intenção de amá-lo. (Autor Desconhecido) #################################### ========================================================================= Instruções para entrar na lista, sair da lista e usar a lista em http://www.mat.puc-rio.br/~nicolau/olimp/obm-l.html O administrador desta lista é ========================================================================= From owner-obm-l@sucuri.mat.puc-rio.br Tue Sep 17 11:59:41 2002 Return-Path: Received: (from majordom@localhost) by sucuri.mat.puc-rio.br (8.9.3/8.9.3) id LAA25881 for obm-l-MTTP; Tue, 17 Sep 2002 11:56:57 -0300 Received: (from nicolau@localhost) by sucuri.mat.puc-rio.br (8.9.3/8.9.3) id LAA25876 for obm-l@mat.puc-rio.br; Tue, 17 Sep 2002 11:56:56 -0300 Date: Tue, 17 Sep 2002 11:56:56 -0300 From: "Nicolau C. Saldanha" To: obm-l@mat.puc-rio.br Subject: Re: [obm-l] Problema de um amigo Message-ID: <20020917115656.D25705@sucuri.mat.puc-rio.br> References: <002201c25df4$a845b820$651fffc8@edmilson> Mime-Version: 1.0 Content-Type: text/plain; charset=iso-8859-1 Content-Disposition: inline Content-Transfer-Encoding: 8bit User-Agent: Mutt/1.2.5i In-Reply-To: <002201c25df4$a845b820$651fffc8@edmilson>; from edmilson@abeunet.com.br on Mon, Sep 16, 2002 at 11:48:06PM -0300 Sender: owner-obm-l@sucuri.mat.puc-rio.br Precedence: bulk Reply-To: obm-l@mat.puc-rio.br On Mon, Sep 16, 2002 at 11:48:06PM -0300, Edmilson wrote: > Dividir uma circunferência em 9 partes iguais, porprocesso geométrico Se por 'processo geométrico' você quer dizer usando apenas régua e compasso então é impossível. É um resultado clássico de álgebra que o círculo pode ser dividido em n partes iguais com régua e compasso apenas se n é da forma n = 2^k * p_1 * p_2 * ... * p_m onde p_1 < p_2 < ... < p_m são primos da forma p = 1 + 2^(2^a). Os únicos primos conhecidos desta forma são 3, 5, 17, 257 e 65537; existem uns argumentos que tornam plausível que estes sejam *todos*. Quando eu estudei desenho geométrico no ginásio apredíamos métodos para dividir o círculo em 5, 6, 7 ou 9 partes iguais mas era na decoreba sem explicação nenhuma e sem que ninguém mencionasse que uns desses processos (5 e 6) eram exatos e outros (7 e 9) não. []s, N. ========================================================================= Instruções para entrar na lista, sair da lista e usar a lista em http://www.mat.puc-rio.br/~nicolau/olimp/obm-l.html O administrador desta lista é ========================================================================= From owner-obm-l@sucuri.mat.puc-rio.br Tue Sep 17 13:17:55 2002 Return-Path: Received: (from majordom@localhost) by sucuri.mat.puc-rio.br (8.9.3/8.9.3) id NAA27363 for obm-l-MTTP; Tue, 17 Sep 2002 13:16:40 -0300 Received: from web12903.mail.yahoo.com (web12903.mail.yahoo.com [216.136.174.70]) by sucuri.mat.puc-rio.br (8.9.3/8.9.3) with SMTP id NAA27358 for ; Tue, 17 Sep 2002 13:16:36 -0300 Message-ID: <20020917161756.67912.qmail@web12903.mail.yahoo.com> Received: from [200.206.103.3] by web12903.mail.yahoo.com via HTTP; Tue, 17 Sep 2002 13:17:56 ART Date: Tue, 17 Sep 2002 13:17:56 -0300 (ART) From: =?iso-8859-1?q?Johann=20Peter=20Gustav=20Lejeune=20Dirichlet?= Subject: Re: [obm-l] Problema de um amigo To: obm-l@mat.puc-rio.br In-Reply-To: <002201c25df4$a845b820$651fffc8@edmilson> MIME-Version: 1.0 Content-Type: multipart/alternative; boundary="0-1801107084-1032279476=:67644" Content-Transfer-Encoding: 8bit Sender: owner-obm-l@sucuri.mat.puc-rio.br Precedence: bulk Reply-To: obm-l@mat.puc-rio.br --0-1801107084-1032279476=:67644 Content-Type: text/plain; charset=iso-8859-1 Content-Transfer-Encoding: 8bit Cara,quando eu vejo "por processo geometrico" eu traduzo como "com regua lisa e compasso" .Para o 9 o unico jeito que eu saiba que e exato e o do origami(papel de dobradura) Acesse o site da Crux Edmilson wrote:Dividir uma circunferência em 9 partes iguais, porprocesso geométrico Atenciosamente, Edmilson edmilson@abeunet.com.br --------------------------------- Yahoo! GeoCities Tudo para criar o seu site: ferramentas fáceis de usar, espaço de sobra e acessórios. --0-1801107084-1032279476=:67644 Content-Type: text/html; charset=iso-8859-1 Content-Transfer-Encoding: 8bit

Cara,quando eu vejo "por processo geometrico" eu traduzo como "com regua lisa e compasso" .Para o 9 o unico jeito que eu saiba que e exato e o do origami(papel de dobradura) Acesse o site da Crux

 Edmilson wrote:

Dividir uma circunferência em 9 partes iguais, porprocesso geométrico
 
Atenciosamente,
Edmilson
edmilson@abeunet.com.br



Yahoo! GeoCities
Tudo para criar o seu site: ferramentas fáceis de usar, espaço de sobra e acessórios. --0-1801107084-1032279476=:67644-- ========================================================================= Instruções para entrar na lista, sair da lista e usar a lista em http://www.mat.puc-rio.br/~nicolau/olimp/obm-l.html O administrador desta lista é ========================================================================= From owner-obm-l@sucuri.mat.puc-rio.br Tue Sep 17 13:29:19 2002 Return-Path: Received: (from majordom@localhost) by sucuri.mat.puc-rio.br (8.9.3/8.9.3) id NAA27686 for obm-l-MTTP; Tue, 17 Sep 2002 13:29:02 -0300 Received: from hotmail.com (f105.law9.hotmail.com [64.4.9.105]) by sucuri.mat.puc-rio.br (8.9.3/8.9.3) with ESMTP id NAA27682 for ; Tue, 17 Sep 2002 13:28:59 -0300 Received: from mail pickup service by hotmail.com with Microsoft SMTPSVC; Tue, 17 Sep 2002 09:30:23 -0700 Received: from 143.107.45.30 by lw9fd.law9.hotmail.msn.com with HTTP; Tue, 17 Sep 2002 16:30:22 GMT X-Originating-IP: [143.107.45.30] From: "Rogerio Fajardo" To: obm-l@mat.puc-rio.br Subject: RE: [obm-l] Axioma da Escolha Date: Tue, 17 Sep 2002 16:30:22 +0000 Mime-Version: 1.0 Content-Type: text/plain; format=flowed Message-ID: X-OriginalArrivalTime: 17 Sep 2002 16:30:23.0077 (UTC) FILETIME=[854BB950:01C25E67] Sender: owner-obm-l@sucuri.mat.puc-rio.br Precedence: bulk Reply-To: obm-l@mat.puc-rio.br A maneira usual de fazer infinitas escolhas sem usar o axioma da escolha é conseguir uma propriedade que escolhe um elemento de cada conjunto. Por exemplo, os racionais são enumeráveis. Em particular, podem ser bem ordenados. De fato, podemos "escrever" essa boa ordem (a lexicográfica, por exemplo). Para escolhermos elementos de infinitos conjuntos de racionais, basta pegarmos o menor elemento de cada conjunto. Esse tipo de demonstração é construtiva, no sentido de que essa escolha não foi feita ao acaso, mas obedecendo uma regra explícita. É claro, como voce ressaltou, o construtivismo não aceita princípios e teoremas tidos como fundamentais na matemática. A fim de eliminar algumas coisas estranhas da matemática, criou outras mais estranhas ainda. Trata-se apenas de uma forma de matemática que não é a mais usual, mas em algum momento pode até ser útil. Hoje há quase um consenso em aceitar o axioma da escolha. Mas, para alguns, um teorema que não depende do axioma da escolha pode ter um "status" maior do que os outros. Existem muitos trabalhos relacionados a independência em Teoria dos Conjuntos que mostram o que seria possível sem o axioma da escolha. >From: "Artur Costa Steiner" >Reply-To: obm-l@mat.puc-rio.br >To: >Subject: RE: [obm-l] Axioma da Escolha >Date: Fri, 13 Sep 2002 23:45:41 -0700 > > > Apenas lembrando, porque costuma-se realçar quando se usa o axioma da > > escolha, há uma corrente filosófica de matemáticos que não aceitam o > > axioma > > da escolha: os construtivistas (ou, mais geralmente, os >intuicionistas). O > > axioma da escolha nos garante a existência de objetos que não podemos > > determinar quem, exatamente, ele é. Esse tipo de coisa os >construtivistas > > não aceitam, pois de que serve saber que existe alguma coisa que nunca > > saberemos quem é, ou onde está? O teorema de Weierstrass, que diz que >toda > > função real contínua sobre um intervalo fechado assume máximo, não é > > aceita > > pelos construtivistas, pois não podemos exibir esse ponto de máximo. >[Artur Costa Steiner] > >Ms este teorema é um dos mais importantes da matemática > Por > > outro lado não podemos dizer que não existe ponto de máximo, pois isso > > seria > > garantir que todos os pontos não são de máximo, o que não devemos > > assegurar. > > Por isso na lógica intuicionista "A ou não A" pode ser falso, e A não >é > > equivalente a "não não A". >[Artur Costa Steiner] > >Acho que é também importante lembrar que muitas provas na matemática >basiam-se em infinitas escolhas. Por exemplo, várias das provas dos >teoremas ligados à compaticidade de espaços métricos enquadram-se nesta >categoria, como o que afirma que S é compacto <===> S é sequencialmente >compacto. Parece-me que estas provam usam o axioma da escolha. E ninguém >as questiona. > > , > > Todas essas complicações geradas pelo construtivismo fizeram que >esse > > caísse um pouco no esquecimento. Hoje parece que há poucos matemáticos > > construtivistas. Mas devemos nos lembrar que o argumento central que >gerou > > o > > construtivismo faz sentido. Realmente, podemos pensar o que fazemos >com > > coisas obtidas não construtivamente. Enfim, há sempre uma fagulha de > > construtivista em nós. É certo que os mais radicais não admitem nem >prova > > por absurdo, mas o axioma da escolha já seria o maior crime que se >poderia > > cometer contra o construtivismo. Por isso, nas demonstrações, é sempre >bom > > ressaltar o que é construtivo e o que não é. Por exemplo, o Paradoxo >de > > Banach-Tarski, sobre a duplicação da esfera, citada pelo Paulo, é > > não-construtiva. > > Sobre o problema da violência, resta um consolo: se o conjunto dos > > bandidos, dado pelo problema, já estiver bem ordenado (por exemplo, se >é > > enumerável), não precisamos do axioma da escolha, e não cometeremos >uma > > "violência" contra os intuicionistas. O difícil vai ser achar bandidos >bem > > ordenados... > > >[Artur Costa Steiner] >Quando podemos fazer infinitas escolhas sem usar o axioma da escolha? >Sempre que tivermos conjuntos bem ordenados? Por exemplo, se fizermos >infinitas escolhas em infinitos subconjuntos dos racionais, então não >precisamos do axioma? > >Artur > > > >========================================================================= >Instruções para entrar na lista, sair da lista e usar a lista em >http://www.mat.puc-rio.br/~nicolau/olimp/obm-l.html >O administrador desta lista é >========================================================================= _________________________________________________________________ Join the world’s largest e-mail service with MSN Hotmail. http://www.hotmail.com ========================================================================= Instruções para entrar na lista, sair da lista e usar a lista em http://www.mat.puc-rio.br/~nicolau/olimp/obm-l.html O administrador desta lista é ========================================================================= From owner-obm-l@sucuri.mat.puc-rio.br Tue Sep 17 13:42:59 2002 Return-Path: Received: (from majordom@localhost) by sucuri.mat.puc-rio.br (8.9.3/8.9.3) id NAA28229 for obm-l-MTTP; Tue, 17 Sep 2002 13:42:40 -0300 Received: from web10208.mail.yahoo.com (web10208.mail.yahoo.com [216.136.130.72]) by sucuri.mat.puc-rio.br (8.9.3/8.9.3) with SMTP id NAA28218 for ; Tue, 17 Sep 2002 13:42:36 -0300 Message-ID: <20020917164359.77901.qmail@web10208.mail.yahoo.com> Received: from [161.24.74.32] by web10208.mail.yahoo.com via HTTP; Tue, 17 Sep 2002 13:43:59 ART Date: Tue, 17 Sep 2002 13:43:59 -0300 (ART) From: =?iso-8859-1?q?Humberto=20Naves?= Subject: [obm-l] Conjuntos Fechados To: obm-l@mat.puc-rio.br MIME-Version: 1.0 Content-Type: text/plain; charset=iso-8859-1 Content-Transfer-Encoding: 8bit Sender: owner-obm-l@sucuri.mat.puc-rio.br Precedence: bulk Reply-To: obm-l@mat.puc-rio.br Oi, Li num livro de análise, que o conjunto dos irracionais não pode ser escrito como uma união enumerável de fechados. Como demostrar esse fato? Obrigado, Humberto Silva Naves _______________________________________________________________________ Yahoo! GeoCities Tudo para criar o seu site: ferramentas fáceis de usar, espaço de sobra e acessórios. http://br.geocities.yahoo.com/ ========================================================================= Instruções para entrar na lista, sair da lista e usar a lista em http://www.mat.puc-rio.br/~nicolau/olimp/obm-l.html O administrador desta lista é ========================================================================= From owner-obm-l@sucuri.mat.puc-rio.br Tue Sep 17 15:15:32 2002 Return-Path: Received: (from majordom@localhost) by sucuri.mat.puc-rio.br (8.9.3/8.9.3) id PAA30453 for obm-l-MTTP; Tue, 17 Sep 2002 15:14:09 -0300 Received: from puma.unisys.com.br (smtp.unisys.com.br [200.220.64.7]) by sucuri.mat.puc-rio.br (8.9.3/8.9.3) with ESMTP id PAA30447 for ; Tue, 17 Sep 2002 15:14:01 -0300 Received: from josefran (riohiper01p128.uninet.com.br [200.220.2.128]) by puma.unisys.com.br (8.12.3/8.12.3) with SMTP id g8HIFIF8026489 for ; Tue, 17 Sep 2002 15:15:25 -0300 (EST) X-Spam-Filter: check_local@puma.unisys.com.br by digitalanswers.org Message-ID: <018901c25e76$de07cc00$d702dcc8@josefran> From: "Jose Francisco Guimaraes Costa" To: "obm-l" Subject: [obm-l] Axioma da Escolha Date: Tue, 17 Sep 2002 15:18:29 -0300 MIME-Version: 1.0 Content-Type: text/plain; charset="iso-8859-1" Content-Transfer-Encoding: 8bit X-Priority: 3 X-MSMail-Priority: Normal X-Mailer: Microsoft Outlook Express 5.00.2314.1300 X-MimeOLE: Produced By Microsoft MimeOLE V5.00.2314.1300 Sender: owner-obm-l@sucuri.mat.puc-rio.br Precedence: bulk Reply-To: obm-l@mat.puc-rio.br Nos últimos dias o assunto mais tratado aqui neste forum vem sendo o Axioma da Escolha. Alguém poderia fornecer o enunciado e um pequeno histórico dele? JF -----Mensagem Original----- De: Rogerio Fajardo Para: Enviada em: Terça-feira, 17 de Setembro de 2002 13:30 Assunto: RE: [obm-l] Axioma da Escolha > A maneira usual de fazer infinitas escolhas sem usar o axioma da escolha é (...) ========================================================================= Instruções para entrar na lista, sair da lista e usar a lista em http://www.mat.puc-rio.br/~nicolau/olimp/obm-l.html O administrador desta lista é ========================================================================= From owner-obm-l@sucuri.mat.puc-rio.br Tue Sep 17 15:15:33 2002 Return-Path: Received: (from majordom@localhost) by sucuri.mat.puc-rio.br (8.9.3/8.9.3) id PAA30464 for obm-l-MTTP; Tue, 17 Sep 2002 15:14:22 -0300 Received: from zeus.opendf.com.br (zeus.opengate.com.br [200.181.71.10]) by sucuri.mat.puc-rio.br (8.9.3/8.9.3) with ESMTP id PAA30460 for ; Tue, 17 Sep 2002 15:14:19 -0300 Received: from localhost (localhost.opengate.com.br [127.0.0.1]) by zeus.opendf.com.br (Postfix) with ESMTP id D52243EA39 for ; Tue, 17 Sep 2002 15:15:37 -0300 (BRT) Received: by zeus.opendf.com.br (Postfix, from userid 48) id B132B3EAA7; Tue, 17 Sep 2002 15:15:36 -0300 (BRT) From: "498 - Artur Costa Steiner" To: obm-l@mat.puc-rio.br Subject: RE: [obm-l] Axioma da Escolha X-Mailer: NeoMail 1.25 X-IPAddress: 200.252.155.2 MIME-Version: 1.0 Content-Type: text/plain; charset=iso-8859-1 Message-Id: <20020917181536.B132B3EAA7@zeus.opendf.com.br> Date: Tue, 17 Sep 2002 15:15:36 -0300 (BRT) X-Virus-Scanned: by AMaViS new-20020517 Sender: owner-obm-l@sucuri.mat.puc-rio.br Precedence: bulk Reply-To: obm-l@mat.puc-rio.br > A maneira usual de fazer infinitas escolhas sem usar o axioma da escolha é > conseguir uma propriedade que escolhe um elemento de cada conjunto. A escolha com base nesta propriedade corresponde àquilo que se denomina de "função canônica de ecolha" ? (vi este termo no newsgroup americano sci.math, "canonical choice function", não sei se traduzi da forma usual em nossa língua). Se não for, vc saberia dizer o que significa "função canônica de escolha"? ´ Consideremos, por exemplo, o teorema que diz que, sendo E um espaço métrico geral, então E totalmente limitado <===> toda sequencia de E contem uma subsequencia de Cauchy. Para provarmos a parte ===>, geralmente cobrimos E sequencialmente por coleções finitas de bolas abertas de raios 1, 1/2..1/n....e escolhemos bolas B1, B2...Bn.. de modo que B1/\B2 .../\Bn....contenha termos da seq. (x_n) para infinitos índices n. Nas intersecões de tais bolas escolhemos termos x_k1, x_k2...x_kn tais que k_1 ========================================================================= From owner-obm-l@sucuri.mat.puc-rio.br Tue Sep 17 16:32:55 2002 Return-Path: Received: (from majordom@localhost) by sucuri.mat.puc-rio.br (8.9.3/8.9.3) id QAA32449 for obm-l-MTTP; Tue, 17 Sep 2002 16:29:03 -0300 Received: from videira.terra.com.br (videira.terra.com.br [200.176.3.5]) by sucuri.mat.puc-rio.br (8.9.3/8.9.3) with ESMTP id QAA32445 for ; Tue, 17 Sep 2002 16:29:00 -0300 Received: from taipe.terra.com.br (taipe.terra.com.br [200.176.3.34]) by videira.terra.com.br (Postfix) with ESMTP id 91D73E2C82 for ; Tue, 17 Sep 2002 16:30:24 -0300 (EST) Received: from stabel (unknown [200.203.39.74]) (authenticated user dudasta) by taipe.terra.com.br (Postfix) with ESMTP id B7FCA1B40A9 for ; Tue, 17 Sep 2002 16:30:23 -0300 (EST) Message-ID: <001c01c25e80$aa3b6800$0301a8c0@stabel> From: "Eduardo Casagrande Stabel" To: References: <20020917164359.77901.qmail@web10208.mail.yahoo.com> Subject: Re: [obm-l] Conjuntos Fechados Date: Tue, 17 Sep 2002 16:30:21 -0300 MIME-Version: 1.0 Content-Type: text/plain; charset="iso-8859-1" Content-Transfer-Encoding: 8bit X-Priority: 3 X-MSMail-Priority: Normal X-Mailer: Microsoft Outlook Express 6.00.2600.0000 X-MimeOLE: Produced By Microsoft MimeOLE V6.00.2600.0000 Sender: owner-obm-l@sucuri.mat.puc-rio.br Precedence: bulk Reply-To: obm-l@mat.puc-rio.br Oi Humberto e demais colegas, Seja I é o conjuntos dos irracionais e Q dos racionais. Se F é um subconjunto fechado de I então F tem interior vazio. Com efeito, se um intervalo aberto (a , b) está contido em int(F) então o próprio F contém (a , b), como existem pontos racionais em (a , b), F não está contido em I, uma contradição. Considere uma enumeração dos racionais Q = {q_1, q_2, q_3, ..., q_n, ...} e Q_i = {q_i} Suponhamos que I seja a união de conjuntos fechados F_1, F_2, F_3, ..., F_n, ... Temos então que os reais R são a união de Q_1, F_1, Q_2, F_2, ..., Q_n, F_n, ... Os Reais (um Espaço Métrico Completo) seria a união enumerável de conjuntos Fechados e com Interior Vazio, ou seja, R seria magro. Contrário ao Teorema de Baire que diz que todo Espaço Métrico Completo não é magro. Eduardo. From: "Humberto Naves" > Oi, > > Li num livro de análise, que o conjunto dos irracionais não pode ser escrito > como uma união enumerável de fechados. Como demostrar esse fato? > > Obrigado, > Humberto Silva Naves > > _______________________________________________________________________ > Yahoo! GeoCities > Tudo para criar o seu site: ferramentas fáceis de usar, espaço de sobra e acessórios. > http://br.geocities.yahoo.com/ > ========================================================================= > Instruções para entrar na lista, sair da lista e usar a lista em > http://www.mat.puc-rio.br/~nicolau/olimp/obm-l.html > O administrador desta lista é > ========================================================================= > > ========================================================================= Instruções para entrar na lista, sair da lista e usar a lista em http://www.mat.puc-rio.br/~nicolau/olimp/obm-l.html O administrador desta lista é ========================================================================= From owner-obm-l@sucuri.mat.puc-rio.br Tue Sep 17 16:39:57 2002 Return-Path: Received: (from majordom@localhost) by sucuri.mat.puc-rio.br (8.9.3/8.9.3) id QAA32633 for obm-l-MTTP; Tue, 17 Sep 2002 16:38:22 -0300 Received: from zeus.opendf.com.br (zeus.opengate.com.br [200.181.71.10]) by sucuri.mat.puc-rio.br (8.9.3/8.9.3) with ESMTP id QAA32629 for ; Tue, 17 Sep 2002 16:38:19 -0300 Received: from localhost (localhost.opengate.com.br [127.0.0.1]) by zeus.opendf.com.br (Postfix) with ESMTP id 9C0783EB27 for ; Tue, 17 Sep 2002 16:39:36 -0300 (BRT) Received: by zeus.opendf.com.br (Postfix, from userid 48) id A28F73EB3F; Tue, 17 Sep 2002 16:36:59 -0300 (BRT) From: "498 - Artur Costa Steiner" To: obm-l@mat.puc-rio.br Subject: Re: [obm-l] Axioma da Escolha X-Mailer: NeoMail 1.25 X-IPAddress: 200.252.155.2 MIME-Version: 1.0 Content-Type: text/plain; charset=iso-8859-1 Message-Id: <20020917193659.A28F73EB3F@zeus.opendf.com.br> Date: Tue, 17 Sep 2002 16:36:59 -0300 (BRT) X-Virus-Scanned: by AMaViS new-20020517 Sender: owner-obm-l@sucuri.mat.puc-rio.br Precedence: bulk Reply-To: obm-l@mat.puc-rio.br > Nos últimos dias o assunto mais tratado aqui neste forum vem sendo o Axioma > da > Escolha. > > Alguém poderia fornecer o enunciado e um pequeno histórico dele? > > JF O enunciado mais usual é o seguinte: Dada uma coleção qualquer de conjuntos disjuntos {A_a} (finita ou infinita, numerável ou não), é possível formar um conjunto S tal que cada elemento de S pertença a um dos conjuntos A_a. Isto é, é possível formar S escolhendo-se um elemento de cada um dos conjuntos A_a, daí o nome Axioma da Escolha. Alguns autores definem o axioma sem requerer que a coleção {A_a} seja disjunta. Artur ========================================================================= Instruções para entrar na lista, sair da lista e usar a lista em http://www.mat.puc-rio.br/~nicolau/olimp/obm-l.html O administrador desta lista é ========================================================================= From owner-obm-l@sucuri.mat.puc-rio.br Tue Sep 17 17:19:32 2002 Return-Path: Received: (from majordom@localhost) by sucuri.mat.puc-rio.br (8.9.3/8.9.3) id RAA01711 for obm-l-MTTP; Tue, 17 Sep 2002 17:17:36 -0300 Received: from web21510.mail.yahoo.com (web21510.mail.yahoo.com [66.163.169.59]) by sucuri.mat.puc-rio.br (8.9.3/8.9.3) with SMTP id RAA01707 for ; Tue, 17 Sep 2002 17:17:32 -0300 Message-ID: <20020917201852.37314.qmail@web21510.mail.yahoo.com> Received: from [200.137.198.82] by web21510.mail.yahoo.com via HTTP; Tue, 17 Sep 2002 17:18:52 ART Date: Tue, 17 Sep 2002 17:18:52 -0300 (ART) From: =?iso-8859-1?q?bruno=20lima?= Subject: Re: [obm-l] EDO - ERRATA To: obm-l@mat.puc-rio.br In-Reply-To: <3D866042.5000401@centroin.com.br> MIME-Version: 1.0 Content-Type: text/plain; charset=iso-8859-1 Content-Transfer-Encoding: 8bit Sender: owner-obm-l@sucuri.mat.puc-rio.br Precedence: bulk Reply-To: obm-l@mat.puc-rio.br Por favor me desculpem, o correto é y(b)=0 --- Augusto César Morgado escreveu: > Eh isso mesmo? A pergunta se refer principalmente ao > y(0) =0. > > bruno lima wrote: > > >Numa dessas provas universitarias por ai apareceu: > > > >y''=y*exp(x) > > > >y: Vai de [a,b]em R , e y(a)=0 e y(0)=0. > > > >Estou errado ou a unica solucáo é a identicamente > >nula?? > >Fazendo uma mudança de variaveis vc cai numa de > >Ricati, mas isso nao resolve muito! > > > >_______________________________________________________________________ > >Yahoo! PageBuilder > >O super editor para criação de sites: é grátis, > fácil e rápido. > >http://br.geocities.yahoo.com/v/pb.html > >========================================================================= > >Instruções para entrar na lista, sair da lista e > usar a lista em > >http://www.mat.puc-rio.br/~nicolau/olimp/obm-l.html > >O administrador desta lista é > > >========================================================================= > > > > > > > ========================================================================= > Instruções para entrar na lista, sair da lista e > usar a lista em > http://www.mat.puc-rio.br/~nicolau/olimp/obm-l.html > O administrador desta lista é > > ========================================================================= _______________________________________________________________________ Yahoo! GeoCities Tudo para criar o seu site: ferramentas fáceis de usar, espaço de sobra e acessórios. http://br.geocities.yahoo.com/ ========================================================================= Instruções para entrar na lista, sair da lista e usar a lista em http://www.mat.puc-rio.br/~nicolau/olimp/obm-l.html O administrador desta lista é ========================================================================= From owner-obm-l@sucuri.mat.puc-rio.br Tue Sep 17 17:26:08 2002 Return-Path: Received: (from majordom@localhost) by sucuri.mat.puc-rio.br (8.9.3/8.9.3) id RAA01905 for obm-l-MTTP; Tue, 17 Sep 2002 17:25:11 -0300 Received: from server3.claretianas.com.br (server3.claretianas.com.br [200.245.46.67]) by sucuri.mat.puc-rio.br (8.9.3/8.9.3) with ESMTP id RAA01901 for ; Tue, 17 Sep 2002 17:25:07 -0300 Received: from local (modem50.claretianas.com.br [200.245.46.210]) by server3.claretianas.com.br (8.11.2/8.11.2) with SMTP id g8HKPIh20245 for ; Tue, 17 Sep 2002 17:25:18 -0300 Message-ID: <000801c25e89$6f1d0ae0$d22ef5c8@www.claretianas.com.br> From: "Carlos Roberto de Moraes" To: Subject: [obm-l] polinomios Date: Tue, 17 Sep 2002 17:33:06 -0300 MIME-Version: 1.0 Content-Type: multipart/alternative; boundary="----=_NextPart_000_0005_01C25E70.4881A800" X-Priority: 3 X-MSMail-Priority: Normal X-Mailer: Microsoft Outlook Express 6.00.2600.0000 X-MimeOLE: Produced By Microsoft MimeOLE V6.00.2600.0000 X-MailScanner: Found to be clean Sender: owner-obm-l@sucuri.mat.puc-rio.br Precedence: bulk Reply-To: obm-l@mat.puc-rio.br This is a multi-part message in MIME format. ------=_NextPart_000_0005_01C25E70.4881A800 Content-Type: text/plain; charset="iso-8859-1" Content-Transfer-Encoding: quoted-printable Alguem pode me ajudar com esses 2 exerc=EDcios? 1) Um polinomio f, dividido por x+2 e x^2+4 d=E1 restos 0 e x+1, = respectivamente. Qual =E9 o resto da divis=E3o de f por (x+2)(x^2+4)? 2) Sabe-se que os restos da divis=E3o de um polinomio p(x) por x^2+x+1 e = x^2-x+1 s=E3o, respectivamente, 3x+5 e -x+9. Determine o resto da = divis=E3o de p(x) por x^4+x^2+1. ------=_NextPart_000_0005_01C25E70.4881A800 Content-Type: text/html; charset="iso-8859-1" Content-Transfer-Encoding: quoted-printable
Alguem pode me ajudar com esses 2 = exerc=EDcios?
 
1) Um polinomio f, dividido por x+2 e x^2+4 d=E1 = restos 0 e x+1,=20 respectivamente. Qual =E9 o resto da divis=E3o de f por = (x+2)(x^2+4)?
 
2) Sabe-se que os restos da divis=E3o de um = polinomio p(x) por=20 x^2+x+1 e x^2-x+1 s=E3o, respectivamente, 3x+5 e -x+9. Determine o resto = da=20 divis=E3o de p(x) por x^4+x^2+1.
------=_NextPart_000_0005_01C25E70.4881A800-- ========================================================================= Instruções para entrar na lista, sair da lista e usar a lista em http://www.mat.puc-rio.br/~nicolau/olimp/obm-l.html O administrador desta lista é ========================================================================= From owner-obm-l@sucuri.mat.puc-rio.br Tue Sep 17 19:08:21 2002 Return-Path: Received: (from majordom@localhost) by sucuri.mat.puc-rio.br (8.9.3/8.9.3) id TAA03921 for obm-l-MTTP; Tue, 17 Sep 2002 19:07:01 -0300 Received: from sr1.terra.com.br (sr1.terra.com.br [200.176.3.16]) by sucuri.mat.puc-rio.br (8.9.3/8.9.3) with ESMTP id TAA03917 for ; Tue, 17 Sep 2002 19:06:59 -0300 Received: from taipe.terra.com.br (taipe.terra.com.br [200.176.3.34]) by sr1.terra.com.br (Postfix) with ESMTP id 519977156E for ; Tue, 17 Sep 2002 19:08:23 -0300 (EST) Received: from nt (RJ231188.user.veloxzone.com.br [200.165.231.188]) (authenticated user ensr) by taipe.terra.com.br (Postfix) with ESMTP id CC70A1B40B8 for ; Tue, 17 Sep 2002 19:08:22 -0300 (EST) Message-ID: <00c501c25e96$9cd05700$5400a8c0@ensrbr> From: "Luis Lopes" To: References: Subject: [obm-l] =?iso-8859-1?Q?Re:_=5Bobm-l=5D_2_Problemas_Cl=E1ssicos_de_DG?= Date: Tue, 17 Sep 2002 19:07:27 -0300 MIME-Version: 1.0 Content-Type: text/plain; charset="iso-8859-1" Content-Transfer-Encoding: 8bit X-Priority: 3 X-MSMail-Priority: Normal X-Mailer: Microsoft Outlook Express 5.50.4807.1700 X-MimeOLE: Produced By Microsoft MimeOLE V5.50.4807.1700 Sender: owner-obm-l@sucuri.mat.puc-rio.br Precedence: bulk Reply-To: obm-l@mat.puc-rio.br Sauda,c~oes, O problema 1 não tem solução com régua e compasso. Mas sempre tem uma solução para três qq segmentos (ver AMM 101, 1994, pp. 58--60). Substituindo bissetrizes por alturas ou medianas, aí a coisa muda: a construção é possível, mas nem sempre. []'s Luis -----Mensagem Original----- De: "Antħnio Lacerda JÅ"nior" Para: Enviada em: sexta-feira, 13 de setembro de 2002 20:06 Assunto: [obm-l] 2 Problemas Clássicos de DG > Olá, todos. > > Estou procurando a solução destes 2 problemas clássicos > de Desenho Geométrico: > > 1) Dadas as três bissetrizes de um triângulo, construa > esse triângulo. > > 2) Determinar o centro de uma circunferência apenas com > compasso. > > Eu as perdi, por isso eu as procuro. > > Obrigado. > > Antônio Lacerda Júnior > ========================================================================= Instruções para entrar na lista, sair da lista e usar a lista em http://www.mat.puc-rio.br/~nicolau/olimp/obm-l.html O administrador desta lista é ========================================================================= From owner-obm-l@sucuri.mat.puc-rio.br Tue Sep 17 22:20:42 2002 Return-Path: Received: (from majordom@localhost) by sucuri.mat.puc-rio.br (8.9.3/8.9.3) id WAA05865 for obm-l-MTTP; Tue, 17 Sep 2002 22:18:33 -0300 Received: from cassino.digi.com.br (cassino.digi.com.br [200.241.100.140]) by sucuri.mat.puc-rio.br (8.9.3/8.9.3) with ESMTP id WAA05861 for ; Tue, 17 Sep 2002 22:18:31 -0300 Received: from david (host240.e.digizap.com.br [200.249.8.240]) by cassino.digi.com.br (8.11.6/8.11.6) with SMTP id g8I1KsK17999 for ; Tue, 17 Sep 2002 22:20:55 -0300 Message-ID: <000d01c25eb2$42d5e960$f008f9c8@david> From: "David Ricardo" To: Subject: [obm-l] Grafos Date: Tue, 17 Sep 2002 22:25:20 -0300 MIME-Version: 1.0 Content-Type: text/plain; charset="iso-8859-1" Content-Transfer-Encoding: 8bit X-Priority: 3 X-MSMail-Priority: Normal X-Mailer: Microsoft Outlook Express 5.00.2615.200 X-MimeOLE: Produced By Microsoft MimeOLE V5.00.2615.200 X-MailScanner: Found to be clean Sender: owner-obm-l@sucuri.mat.puc-rio.br Precedence: bulk Reply-To: obm-l@mat.puc-rio.br Pessoal, Gostaria que vocês me indicassem algum bom material (em inglês ou português) sobre grafos e suas aplicações. Grato, David ========================================================================= Instruções para entrar na lista, sair da lista e usar a lista em http://www.mat.puc-rio.br/~nicolau/olimp/obm-l.html O administrador desta lista é ========================================================================= From owner-obm-l@sucuri.mat.puc-rio.br Wed Sep 18 00:03:21 2002 Return-Path: Received: (from majordom@localhost) by sucuri.mat.puc-rio.br (8.9.3/8.9.3) id AAA07028 for obm-l-MTTP; Wed, 18 Sep 2002 00:02:02 -0300 Received: from mail.gmx.net (mail.gmx.net [213.165.64.20]) by sucuri.mat.puc-rio.br (8.9.3/8.9.3) with SMTP id AAA07024 for ; Wed, 18 Sep 2002 00:01:59 -0300 Received: (qmail 17338 invoked by uid 0); 18 Sep 2002 03:03:23 -0000 Received: from unknown (HELO gomes) (200.216.104.28) by mail.gmx.net (mp016-rz3) with SMTP; 18 Sep 2002 03:03:23 -0000 Date: Tue, 17 Sep 2002 15:47:30 -0300 From: Igor GomeZZ X-Mailer: The Bat! (v1.61) Organization: -- X-Priority: 3 (Normal) Message-ID: <3613822695.20020917154730@gmx.net> To: Johann Peter Gustav Lejeune Dirichlet Subject: Re: RE de re:[obm-l] Ajuda Algebra linear (Off Topic) In-Reply-To: <20020916165754.81751.qmail@web12902.mail.yahoo.com> References: <20020916165754.81751.qmail@web12902.mail.yahoo.com> MIME-Version: 1.0 Content-Type: text/plain; charset=ISO-8859-1 Content-Transfer-Encoding: 8bit Sender: owner-obm-l@sucuri.mat.puc-rio.br Precedence: bulk Reply-To: obm-l@mat.puc-rio.br Em 16/9/2002, 13:57, Johann (peterdirichlet2002@yahoo.com.br) disse: > Beleza!La ele da uma aplicaçao bem interessante:como se dar bem > blefando em um jogo de truco(parece jogo de truco mas mudam algumas regras). Qual o tópico? Não conseguir encontrar... Fui! ####### Igor GomeZZ ######## UIN: 29249895 Vitória, Espírito Santo, Brasil Criação: 17/9/2002 (15:45) #################################### Pare para pensar: O rio atinge seus objetivos porque aprendeu a contornar obstáculos. (Lao- Tsé) #################################### ========================================================================= Instruções para entrar na lista, sair da lista e usar a lista em http://www.mat.puc-rio.br/~nicolau/olimp/obm-l.html O administrador desta lista é ========================================================================= From owner-obm-l@sucuri.mat.puc-rio.br Wed Sep 18 00:46:26 2002 Return-Path: Received: (from majordom@localhost) by sucuri.mat.puc-rio.br (8.9.3/8.9.3) id AAA07763 for obm-l-MTTP; Wed, 18 Sep 2002 00:45:20 -0300 Received: from suter.bol.com.br (suter.bol.com.br [200.221.24.19]) by sucuri.mat.puc-rio.br (8.9.3/8.9.3) with ESMTP id AAA07759 for ; Wed, 18 Sep 2002 00:45:18 -0300 Received: from bol.com.br (200.221.24.137) by suter.bol.com.br (5.1.071) id 3D875A4D00036DCA for obm-l@mat.puc-rio.br; Wed, 18 Sep 2002 00:46:32 -0300 Date: Wed, 18 Sep 2002 00:44:12 -0300 Message-Id: Subject: [obm-l] =?iso-8859-1?q?Re=3A_=5Bobm=2Dl=5D_2_Problemas_Cl=E1ssicos_de_DG?= MIME-Version: 1.0 Content-Type: text/plain;charset="iso-8859-1" From: "Antonio Lacerda Junior" To: obm-l@mat.puc-rio.br X-XaM3-API-Version: 2.4.3.4.4 X-SenderIP: 200.207.14.157 Content-Transfer-Encoding: 8bit X-MIME-Autoconverted: from quoted-printable to 8bit by sucuri.mat.puc-rio.br id AAA07760 Sender: owner-obm-l@sucuri.mat.puc-rio.br Precedence: bulk Reply-To: obm-l@mat.puc-rio.br >Sauda,c~oes, >O problema 1 não tem solução com régua >e compasso. Mas sempre tem uma solução >para três qq segmentos (ver AMM 101, 1994, >pp. 58--60). >Substituindo bissetrizes por alturas ou medianas, >aí a coisa muda: a construção é possível, mas >nem sempre. >[]'s >Luis Luis, obrigado pelo esclarecimento, mas tu poderias explicar-me o que significa AMM? Quanto ao problema 2, se a minha memória não me falha, já saiu até na Superinteressante (talvez em 1992). Se alguém souber de algum outro registro desse problema, por favor, avise-me. Eu agradecerei. Antônio Lacerda __________________________________________________________________________ AcessoBOL, só R$ 9,90! O menor preço do mercado! Assine já! http://www.bol.com.br/acessobol ========================================================================= Instruções para entrar na lista, sair da lista e usar a lista em http://www.mat.puc-rio.br/~nicolau/olimp/obm-l.html O administrador desta lista é ========================================================================= From owner-obm-l@sucuri.mat.puc-rio.br Wed Sep 18 02:06:07 2002 Return-Path: Received: (from majordom@localhost) by sucuri.mat.puc-rio.br (8.9.3/8.9.3) id CAA08772 for obm-l-MTTP; Wed, 18 Sep 2002 02:02:54 -0300 Received: from sr1.terra.com.br (sr1.terra.com.br [200.176.3.16]) by sucuri.mat.puc-rio.br (8.9.3/8.9.3) with ESMTP id CAA08768 for ; Wed, 18 Sep 2002 02:02:52 -0300 Received: from engenho.terra.com.br (engenho.terra.com.br [200.176.3.42]) by sr1.terra.com.br (Postfix) with ESMTP id 6F55571B0D for ; Wed, 18 Sep 2002 01:59:20 -0300 (EST) Received: from stabel (unknown [200.203.38.50]) (authenticated user dudasta) by engenho.terra.com.br (Postfix) with ESMTP id C9DFD680DF for ; Wed, 18 Sep 2002 01:59:19 -0300 (EST) Message-ID: <001801c25ed0$2460fe20$0301a8c0@stabel> From: "Eduardo Casagrande Stabel" To: References: <20020917164359.77901.qmail@web10208.mail.yahoo.com> <001c01c25e80$aa3b6800$0301a8c0@stabel> Subject: Re: [obm-l] Conjuntos Fechados Date: Wed, 18 Sep 2002 01:59:17 -0300 MIME-Version: 1.0 Content-Type: text/plain; charset="iso-8859-1" Content-Transfer-Encoding: 8bit X-Priority: 3 X-MSMail-Priority: Normal X-Mailer: Microsoft Outlook Express 6.00.2600.0000 X-MimeOLE: Produced By Microsoft MimeOLE V6.00.2600.0000 Sender: owner-obm-l@sucuri.mat.puc-rio.br Precedence: bulk Reply-To: obm-l@mat.puc-rio.br From: "Eduardo Casagrande Stabel" > Oi Humberto e demais colegas, > > Seja I é o conjuntos dos irracionais e Q dos racionais. > Se F é um subconjunto fechado de I então F tem interior vazio. > Com efeito, se um intervalo aberto (a , b) está contido em int(F) então o > próprio F contém (a , b), como existem pontos racionais em (a , b), F não > está contido em I, uma contradição. Eu me enganei na definição de interior vazio. Basta substituir int(F) por int(fecho(F)), mas como F é fechado F = fecho(F), dá no mesmo e a prova continua válida. Perdão pelo cochilo. Eduardo. > Considere uma enumeração dos racionais Q = {q_1, q_2, q_3, ..., q_n, ...} e > Q_i = {q_i} > Suponhamos que I seja a união de conjuntos fechados F_1, F_2, F_3, ..., F_n, > ... > Temos então que os reais R são a união de Q_1, F_1, Q_2, F_2, ..., Q_n, F_n, > ... > Os Reais (um Espaço Métrico Completo) seria a união enumerável de conjuntos > Fechados e com Interior Vazio, ou seja, R seria magro. > Contrário ao Teorema de Baire que diz que todo Espaço Métrico Completo não é > magro. > > Eduardo. > > > > > From: "Humberto Naves" > > Oi, > > > > Li num livro de análise, que o conjunto dos irracionais não pode ser > escrito > > como uma união enumerável de fechados. Como demostrar esse fato? > > > > Obrigado, > > Humberto Silva Naves > > > > _______________________________________________________________________ > > Yahoo! GeoCities > > Tudo para criar o seu site: ferramentas fáceis de usar, espaço de sobra e > acessórios. > > http://br.geocities.yahoo.com/ > > ========================================================================= > > Instruções para entrar na lista, sair da lista e usar a lista em > > http://www.mat.puc-rio.br/~nicolau/olimp/obm-l.html > > O administrador desta lista é > > ========================================================================= > > > > > > ========================================================================= > Instruções para entrar na lista, sair da lista e usar a lista em > http://www.mat.puc-rio.br/~nicolau/olimp/obm-l.html > O administrador desta lista é > ========================================================================= > > ========================================================================= Instruções para entrar na lista, sair da lista e usar a lista em http://www.mat.puc-rio.br/~nicolau/olimp/obm-l.html O administrador desta lista é ========================================================================= From owner-obm-l@sucuri.mat.puc-rio.br Wed Sep 18 02:33:41 2002 Return-Path: Received: (from majordom@localhost) by sucuri.mat.puc-rio.br (8.9.3/8.9.3) id CAA09351 for obm-l-MTTP; Wed, 18 Sep 2002 02:31:01 -0300 Received: from videira.terra.com.br (videira.terra.com.br [200.176.3.5]) by sucuri.mat.puc-rio.br (8.9.3/8.9.3) with ESMTP id CAA09347 for ; Wed, 18 Sep 2002 02:30:59 -0300 Received: from smtp4-poa.terra.com.br (smtp4-poa.terra.com.br [200.176.3.35]) by videira.terra.com.br (Postfix) with ESMTP id C75B8E1728 for ; Wed, 18 Sep 2002 02:32:23 -0300 (EST) Received: from stabel (unknown [200.203.38.50]) (authenticated user dudasta) by smtp4-poa.terra.com.br (Postfix) with ESMTP id 5969DAC5A5 for ; Wed, 18 Sep 2002 02:32:23 -0300 (EST) Message-ID: <004301c25ed4$c2757330$0301a8c0@stabel> From: "Eduardo Casagrande Stabel" To: References: Subject: [obm-l] =?iso-8859-1?Q?Re:_=5Bobm-l=5D_Re:_=5Bobm-l=5D_2_Problemas_Cl=E1ssicos_de?= =?iso-8859-1?Q?_DG?= Date: Wed, 18 Sep 2002 02:32:20 -0300 MIME-Version: 1.0 Content-Type: text/plain; charset="iso-8859-1" Content-Transfer-Encoding: 8bit X-Priority: 3 X-MSMail-Priority: Normal X-Mailer: Microsoft Outlook Express 6.00.2600.0000 X-MimeOLE: Produced By Microsoft MimeOLE V6.00.2600.0000 Sender: owner-obm-l@sucuri.mat.puc-rio.br Precedence: bulk Reply-To: obm-l@mat.puc-rio.br From: "Antonio Lacerda Junior" > >Sauda,c~oes, > > >O problema 1 não tem solução com régua > >e compasso. Mas sempre tem uma solução > >para três qq segmentos (ver AMM 101, 1994, > >pp. 58--60). > > >Substituindo bissetrizes por alturas ou medianas, > >aí a coisa muda: a construção é possível, mas > >nem sempre. > > > >[]'s > >Luis > > Luis, obrigado pelo esclarecimento, mas tu poderias > explicar-me o que significa AMM? American Mathmatical Monthly, eu acho. > > Quanto ao problema 2, se a minha memória não me falha, > já saiu até na Superinteressante (talvez em 1992). Se > alguém souber de algum outro registro desse problema, > por favor, avise-me. Eu agradecerei. > > Antônio Lacerda > > > > > __________________________________________________________________________ > AcessoBOL, só R$ 9,90! O menor preço do mercado! > Assine já! http://www.bol.com.br/acessobol > > > ========================================================================= > Instruções para entrar na lista, sair da lista e usar a lista em > http://www.mat.puc-rio.br/~nicolau/olimp/obm-l.html > O administrador desta lista é > ========================================================================= > > ========================================================================= Instruções para entrar na lista, sair da lista e usar a lista em http://www.mat.puc-rio.br/~nicolau/olimp/obm-l.html O administrador desta lista é ========================================================================= From owner-obm-l@sucuri.mat.puc-rio.br Wed Sep 18 06:43:03 2002 Return-Path: Received: (from majordom@localhost) by sucuri.mat.puc-rio.br (8.9.3/8.9.3) id GAA11487 for obm-l-MTTP; Wed, 18 Sep 2002 06:39:46 -0300 Received: from gorgo.centroin.com.br (gorgo.centroin.com.br [200.225.63.128]) by sucuri.mat.puc-rio.br (8.9.3/8.9.3) with ESMTP id GAA11483 for ; Wed, 18 Sep 2002 06:39:44 -0300 Received: from centroin.com.br (du3b.nit.centroin.com.br [200.225.56.3]) (authenticated bits=0) by gorgo.centroin.com.br (8.12.2/8.12.1) with ESMTP id g8I9fP7p017027 for ; Wed, 18 Sep 2002 06:41:27 -0300 (BRT) Message-ID: <3D884AD7.6010201@centroin.com.br> Date: Wed, 18 Sep 2002 06:43:51 -0300 From: Augusto =?ISO-8859-1?Q?C=E9sar?= Morgado User-Agent: Mozilla/5.0 (Windows; U; Win98; en-US; rv:0.9.4.1) Gecko/20020508 Netscape6/6.2.3 X-Accept-Language: en-us MIME-Version: 1.0 To: obm-l@mat.puc-rio.br Subject: Re: [obm-l] polinomios References: <000801c25e89$6f1d0ae0$d22ef5c8@www.claretianas.com.br> Content-Type: multipart/alternative; boundary="------------060701000605020202050308" Sender: owner-obm-l@sucuri.mat.puc-rio.br Precedence: bulk Reply-To: obm-l@mat.puc-rio.br --------------060701000605020202050308 Content-Type: text/plain; charset=ISO-8859-1; format=flowed Content-Transfer-Encoding: 8bit 1) Dados: f(x) = (x+2) Q(x) f(x) = (x^2+4) P(x) + (x+1) Queremos f(x) = (x+2)(x^2+4) S(x) + (Ax^2+Bx+C) Para calcular A, B e C, faça x igual a -2, 2i e -2i. Obtem-se o sistema f(-2) = 4A -2B +C f(2i) = -4A +2Bi +C f(-2i) = -4A -2Bi + C Os dados mostram que f(-2) = 0, f(2i) = 1+2i e f(-2i) = 1-2i. Resolvendo o sistema, B=1, C=3/2, A=1/8. O resto eh (1/8)(x^2) + (3/2)x + 1. 2)Observe que x^4+x^2+1 = ( x^2+x+1)( x^2-x+1) Proceda analogamente. Os valores inteligentes para x sao as raizes de x^2+x+1 e x^2-x+1. Carlos Roberto de Moraes wrote: > Alguem pode me ajudar com esses 2 exercícios? > > > > 1) Um polinomio f, dividido por x+2 e x^2+4 dá restos 0 e x+1, > respectivamente. Qual é o resto da divisão de f por (x+2)(x^2+4)? > > > > 2) Sabe-se que os restos da divisão de um polinomio p(x) por x^2+x+1 e > x^2-x+1 são, respectivamente, 3x+5 e -x+9. Determine o resto da > divisão de p(x) por x^4+x^2+1. > --------------060701000605020202050308 Content-Type: text/html; charset=us-ascii Content-Transfer-Encoding: 7bit 1) Dados:  f(x) = (x+2) Q(x)
f(x) = (x^2+4) P(x) + (x+1)
Queremos
f(x) = (x+2)(x^2+4) S(x) + (Ax^2+Bx+C)
Para calcular A, B e C, faça  x igual a  -2, 2i  e -2i.
Obtem-se o sistema
f(-2) = 4A -2B +C
f(2i) = -4A +2Bi +C
f(-2i) = -4A -2Bi + C
Os dados mostram que  f(-2) = 0, f(2i) = 1+2i  e  f(-2i) = 1-2i.
Resolvendo o sistema, B=1, C=3/2, A=1/8.
O resto eh  (1/8)(x^2) + (3/2)x + 1.
2)Observe que x^4+x^2+1 = ( x^2+x+1)( x^2-x+1)
Proceda analogamente. Os valores inteligentes para x sao as raizes de
x^2+x+1 e x^2-x+1.


Carlos Roberto de Moraes wrote:
Alguem pode me ajudar com esses 2 exercícios?
 
1) Um polinomio f, dividido por x+2 e x^2+4 dá restos 0 e x+1, respectivamente. Qual é o resto da divisão de f por (x+2)(x^2+4)?
 
2) Sabe-se que os restos da divisão de um polinomio p(x) por x^2+x+1 e x^2-x+1 são, respectivamente, 3x+5 e -x+9. Determine o resto da divisão de p(x) por x^4+x^2+1.

--------------060701000605020202050308-- ========================================================================= Instruções para entrar na lista, sair da lista e usar a lista em http://www.mat.puc-rio.br/~nicolau/olimp/obm-l.html O administrador desta lista é ========================================================================= From owner-obm-l@sucuri.mat.puc-rio.br Wed Sep 18 09:31:01 2002 Return-Path: Received: (from majordom@localhost) by sucuri.mat.puc-rio.br (8.9.3/8.9.3) id JAA12817 for obm-l-MTTP; Wed, 18 Sep 2002 09:28:19 -0300 Received: from hotmail.com (f73.sea2.hotmail.com [207.68.165.73]) by sucuri.mat.puc-rio.br (8.9.3/8.9.3) with ESMTP id JAA12812 for ; Wed, 18 Sep 2002 09:28:16 -0300 Received: from mail pickup service by hotmail.com with Microsoft SMTPSVC; Wed, 18 Sep 2002 05:29:42 -0700 Received: from 200.220.36.213 by sea2fd.sea2.hotmail.msn.com with HTTP; Wed, 18 Sep 2002 12:29:41 GMT X-Originating-IP: [200.220.36.213] From: "leonardo mattos" To: obm-l@mat.puc-rio.br Subject: [obm-l] Polinomio(IME) Date: Wed, 18 Sep 2002 12:29:41 +0000 Mime-Version: 1.0 Content-Type: text/plain; charset=iso-8859-1; format=flowed Message-ID: X-OriginalArrivalTime: 18 Sep 2002 12:29:42.0014 (UTC) FILETIME=[102A21E0:01C25F0F] Sender: owner-obm-l@sucuri.mat.puc-rio.br Precedence: bulk Reply-To: obm-l@mat.puc-rio.br Ola pessoal, Prove que x^999+x^888+x^777+...+x^111+1 é divisivel por x^9+x^8+x^7+...+x+1. Um abraço,Leonardo _________________________________________________________________ Converse com seus amigos online, faça o download grátis do MSN Messenger: http://messenger.msn.com.br ========================================================================= Instruções para entrar na lista, sair da lista e usar a lista em http://www.mat.puc-rio.br/~nicolau/olimp/obm-l.html O administrador desta lista é ========================================================================= From owner-obm-l@sucuri.mat.puc-rio.br Wed Sep 18 10:51:57 2002 Return-Path: Received: (from majordom@localhost) by sucuri.mat.puc-rio.br (8.9.3/8.9.3) id KAA13987 for obm-l-MTTP; Wed, 18 Sep 2002 10:48:15 -0300 Received: from traven9.pub1 (traven9.uol.com.br [200.221.4.35]) by sucuri.mat.puc-rio.br (8.9.3/8.9.3) with ESMTP id KAA13982 for ; Wed, 18 Sep 2002 10:48:12 -0300 Received: from emmanuela ([200.150.128.234]) by traven9.pub1 (8.9.1/8.9.1) with SMTP id KAA15329 for ; Wed, 18 Sep 2002 10:51:02 -0300 (BRT) Message-ID: <008001c25f19$fb463a60$ea8096c8@emmanuela> From: "Paulo Rodrigues" To: References: Subject: Re: [obm-l] Polinomio(IME) Date: Wed, 18 Sep 2002 10:47:32 -0300 MIME-Version: 1.0 Content-Type: text/plain; charset="iso-8859-1" Content-Transfer-Encoding: 8bit X-Priority: 3 X-MSMail-Priority: Normal X-Mailer: Microsoft Outlook Express 6.00.2600.0000 X-MimeOLE: Produced By Microsoft MimeOLE V6.00.2600.0000 Sender: owner-obm-l@sucuri.mat.puc-rio.br Precedence: bulk Reply-To: obm-l@mat.puc-rio.br ----- Original Message ----- From: "leonardo mattos" To: Sent: Wednesday, September 18, 2002 9:29 AM Subject: [obm-l] Polinomio(IME) > Ola pessoal, > Prove que x^999+x^888+x^777+...+x^111+1 é divisivel por x^9+x^8+x^7+...+x+1. > Um abraço,Leonardo > Esse problema apareceu na lista em agosto (http://www.mat.puc-rio.br/~nicolau/olimp/obm-l.html) Para provar que um polin^omio P 'e divis'ivel por um polin^omio Q basta mostrar que toda raiz de Q 'e raiz de P. No seu exemplo, Q = (x^10 - 1)/(x - 1), ou seja, as raizes de Q s~ao as ra'izes 10as de 1, exceto 1. Assim se z 'e uma raiz de Q temos z^111 = z, z^222 = z^2, ..., z^999 = z^9 e portanto P(z) = Q(z) = 0. []s, N. ========================================================================= Instruções para entrar na lista, sair da lista e usar a lista em http://www.mat.puc-rio.br/~nicolau/olimp/obm-l.html O administrador desta lista é ========================================================================= From owner-obm-l@sucuri.mat.puc-rio.br Wed Sep 18 12:07:55 2002 Return-Path: Received: (from majordom@localhost) by sucuri.mat.puc-rio.br (8.9.3/8.9.3) id MAA15305 for obm-l-MTTP; Wed, 18 Sep 2002 12:00:35 -0300 Received: from web12902.mail.yahoo.com (web12902.mail.yahoo.com [216.136.174.69]) by sucuri.mat.puc-rio.br (8.9.3/8.9.3) with SMTP id MAA15301 for ; Wed, 18 Sep 2002 12:00:32 -0300 Message-ID: <20020918150158.5298.qmail@web12902.mail.yahoo.com> Received: from [200.206.103.3] by web12902.mail.yahoo.com via HTTP; Wed, 18 Sep 2002 12:01:58 ART Date: Wed, 18 Sep 2002 12:01:58 -0300 (ART) From: =?iso-8859-1?q?Johann=20Peter=20Gustav=20Lejeune=20Dirichlet?= Subject: Re: [obm-l] Re:_[obm-l]_2_Problemas_Clássicos_de_DG To: obm-l@mat.puc-rio.br In-Reply-To: <00c501c25e96$9cd05700$5400a8c0@ensrbr> MIME-Version: 1.0 Content-Type: multipart/alternative; boundary="0-1509914896-1032361318=:4830" Content-Transfer-Encoding: 8bit Sender: owner-obm-l@sucuri.mat.puc-rio.br Precedence: bulk Reply-To: obm-l@mat.puc-rio.br --0-1509914896-1032361318=:4830 Content-Type: text/plain; charset=iso-8859-1 Content-Transfer-Encoding: 8bit Quanto a esse segundo,eu testei umas ideias de inversao:tente pegar um ponto da circunferencia e inverter tudo.Voce obtera uma reta.Isso pode ajudar... Luis Lopes wrote:Sauda,c~oes, O problema 1 não tem solução com régua e compasso. Mas sempre tem uma solução para três qq segmentos (ver AMM 101, 1994, pp. 58--60). Substituindo bissetrizes por alturas ou medianas, aí a coisa muda: a construção é possível, mas nem sempre. []'s Luis -----Mensagem Original----- De: "Antħnio Lacerda JÅ"nior" Para: Enviada em: sexta-feira, 13 de setembro de 2002 20:06 Assunto: [obm-l] 2 Problemas Clássicos de DG > Olá, todos. > > Estou procurando a solução destes 2 problemas clássicos > de Desenho Geométrico: > > 1) Dadas as três bissetrizes de um triângulo, construa > esse triângulo. > > 2) Determinar o centro de uma circunferência apenas com > compasso. > > Eu as perdi, por isso eu as procuro. > > Obrigado. > > Antônio Lacerda Júnior > ========================================================================= Instruções para entrar na lista, sair da lista e usar a lista em http://www.mat.puc-rio.br/~nicolau/olimp/obm-l.html O administrador desta lista é ========================================================================= --------------------------------- Yahoo! GeoCities Tudo para criar o seu site: ferramentas fáceis de usar, espaço de sobra e acessórios. --0-1509914896-1032361318=:4830 Content-Type: text/html; charset=iso-8859-1 Content-Transfer-Encoding: 8bit

Quanto a esse segundo,eu testei umas ideias de inversao:tente pegar um ponto da circunferencia e inverter tudo.Voce obtera uma reta.Isso pode ajudar...

 Luis Lopes wrote:

Sauda,c~oes,

O problema 1 não tem solução com régua
e compasso. Mas sempre tem uma solução
para três qq segmentos (ver AMM 101, 1994,
pp. 58--60).

Substituindo bissetrizes por alturas ou medianas,
aí a coisa muda: a construção é possível, mas
nem sempre.

[]'s
Luis

-----Mensagem Original-----
De: "Antħnio Lacerda JÅ"nior"
Para:
Enviada em: sexta-feira, 13 de setembro de 2002 20:06
Assunto: [obm-l] 2 Problemas Clássicos de DG


> Olá, todos.
>
> Estou procurando a solução destes 2 problemas clássicos
> de Desenho Geométrico:
>
> 1) Dadas as três bissetrizes de um triângulo, construa
> esse triângulo.
>
> 2) Determinar o centro de uma circunferência apenas com
> compasso.
>
> Eu as perdi, por isso eu as procuro.
>
> Obrigado.
>
> Antônio Lacerda Júnior
>


=========================================================================
Instruções para entrar na lista, sair da lista e usar a lista em
http://www.mat.puc-rio.br/~nicolau/olimp/obm-l.html
O administrador desta lista é
=========================================================================



Yahoo! GeoCities
Tudo para criar o seu site: ferramentas fáceis de usar, espaço de sobra e acessórios. --0-1509914896-1032361318=:4830-- ========================================================================= Instruções para entrar na lista, sair da lista e usar a lista em http://www.mat.puc-rio.br/~nicolau/olimp/obm-l.html O administrador desta lista é ========================================================================= From owner-obm-l@sucuri.mat.puc-rio.br Wed Sep 18 12:11:50 2002 Return-Path: Received: (from majordom@localhost) by sucuri.mat.puc-rio.br (8.9.3/8.9.3) id MAA15387 for obm-l-MTTP; Wed, 18 Sep 2002 12:05:13 -0300 Received: from web12906.mail.yahoo.com (web12906.mail.yahoo.com [216.136.174.73]) by sucuri.mat.puc-rio.br (8.9.3/8.9.3) with SMTP id MAA15383 for ; Wed, 18 Sep 2002 12:05:10 -0300 Message-ID: <20020918150637.61533.qmail@web12906.mail.yahoo.com> Received: from [200.206.103.3] by web12906.mail.yahoo.com via HTTP; Wed, 18 Sep 2002 12:06:37 ART Date: Wed, 18 Sep 2002 12:06:37 -0300 (ART) From: =?iso-8859-1?q?Johann=20Peter=20Gustav=20Lejeune=20Dirichlet?= Subject: Re: [obm-l] OBM-2002 To: obm-l@mat.puc-rio.br In-Reply-To: <622411757.20020917004135@gmx.net> MIME-Version: 1.0 Content-Type: multipart/alternative; boundary="0-311018154-1032361597=:61204" Content-Transfer-Encoding: 8bit Sender: owner-obm-l@sucuri.mat.puc-rio.br Precedence: bulk Reply-To: obm-l@mat.puc-rio.br --0-311018154-1032361597=:61204 Content-Type: text/plain; charset=iso-8859-1 Content-Transfer-Encoding: 8bit Calma,e claro que vai ter.Basta esperar(eu acho).Alias como e que a Nelly tem coragem de fazer isso tudo no Word??????? Igor GomeZZ wrote: Em 16/9/2002, 16:56, Olimpiada (obm@impa.br) disse: > Alias daqui a alguns minutos a Eureka No 14 > tambem no site. Opa! Mas num vai ter versão .pdf (ou .ps) não? Fui! ####### Igor GomeZZ ######## UIN: 29249895 Vitória, Espírito Santo, Brasil Criação: 17/9/2002 (00:40) #################################### Pare para pensar: A pior covardia de uma mulher é despertar o amor de um homem sem ter a intenção de amá-lo. (Autor Desconhecido) #################################### ========================================================================= Instruções para entrar na lista, sair da lista e usar a lista em http://www.mat.puc-rio.br/~nicolau/olimp/obm-l.html O administrador desta lista é ========================================================================= --------------------------------- Yahoo! GeoCities Tudo para criar o seu site: ferramentas fáceis de usar, espaço de sobra e acessórios. --0-311018154-1032361597=:61204 Content-Type: text/html; charset=iso-8859-1 Content-Transfer-Encoding: 8bit

Calma,e claro que vai ter.Basta esperar(eu acho).Alias como e que a Nelly tem coragem de fazer isso tudo no Word???????

 Igor GomeZZ wrote:


Em 16/9/2002, 16:56, Olimpiada (obm@impa.br) disse:


> Alias daqui a alguns minutos a Eureka No 14
> tambem no site.


Opa! Mas num vai ter versão .pdf (ou .ps) não?

Fui!


####### Igor GomeZZ ########
UIN: 29249895
Vitória, Espírito Santo, Brasil
Criação: 17/9/2002 (00:40)
####################################
Pare para pensar:

A pior covardia de uma mulher é
despertar o amor de um homem sem
ter a intenção de amá-lo.
(Autor Desconhecido)

####################################

=========================================================================
Instruções para entrar na lista, sair da lista e usar a lista em
http://www.mat.puc-rio.br/~nicolau/olimp/obm-l.html
O administrador desta lista é
=========================================================================



Yahoo! GeoCities
Tudo para criar o seu site: ferramentas fáceis de usar, espaço de sobra e acessórios. --0-311018154-1032361597=:61204-- ========================================================================= Instruções para entrar na lista, sair da lista e usar a lista em http://www.mat.puc-rio.br/~nicolau/olimp/obm-l.html O administrador desta lista é ========================================================================= From owner-obm-l@sucuri.mat.puc-rio.br Wed Sep 18 12:15:47 2002 Return-Path: Received: (from majordom@localhost) by sucuri.mat.puc-rio.br (8.9.3/8.9.3) id MAA15464 for obm-l-MTTP; Wed, 18 Sep 2002 12:09:29 -0300 Received: from web12903.mail.yahoo.com (web12903.mail.yahoo.com [216.136.174.70]) by sucuri.mat.puc-rio.br (8.9.3/8.9.3) with SMTP id MAA15460 for ; Wed, 18 Sep 2002 12:09:26 -0300 Message-ID: <20020918151049.71505.qmail@web12903.mail.yahoo.com> Received: from [200.206.103.3] by web12903.mail.yahoo.com via HTTP; Wed, 18 Sep 2002 12:10:49 ART Date: Wed, 18 Sep 2002 12:10:49 -0300 (ART) From: =?iso-8859-1?q?Johann=20Peter=20Gustav=20Lejeune=20Dirichlet?= Subject: Re: [obm-l] Re:_[obm-l]_2_Problemas_Clássicos_de_DG To: obm-l@mat.puc-rio.br In-Reply-To: MIME-Version: 1.0 Content-Type: multipart/alternative; boundary="0-1342140147-1032361849=:71092" Content-Transfer-Encoding: 8bit Sender: owner-obm-l@sucuri.mat.puc-rio.br Precedence: bulk Reply-To: obm-l@mat.puc-rio.br --0-1342140147-1032361849=:71092 Content-Type: text/plain; charset=iso-8859-1 Content-Transfer-Encoding: 8bit Essa eu respondo:AMM=American Mathematical Monthly,"Matematica Estadunidense Mensal",a Eureka dos States.La tem alguns dos melhores problemas,e so perde para a CRUX Mathematicorum Antonio Lacerda Junior wrote:>Sauda,c~oes, >O problema 1 não tem solução com régua >e compasso. Mas sempre tem uma solução >para três qq segmentos (ver AMM 101, 1994, >pp. 58--60). >Substituindo bissetrizes por alturas ou medianas, >aí a coisa muda: a construção é possível, mas >nem sempre. >[]'s >Luis Luis, obrigado pelo esclarecimento, mas tu poderias explicar-me o que significa AMM? Quanto ao problema 2, se a minha memória não me falha, já saiu até na Superinteressante (talvez em 1992). Se alguém souber de algum outro registro desse problema, por favor, avise-me. Eu agradecerei. Antônio Lacerda __________________________________________________________________________ AcessoBOL, só R$ 9,90! O menor preço do mercado! Assine já! http://www.bol.com.br/acessobol ========================================================================= Instruções para entrar na lista, sair da lista e usar a lista em http://www.mat.puc-rio.br/~nicolau/olimp/obm-l.html O administrador desta lista é ========================================================================= --------------------------------- Yahoo! GeoCities Tudo para criar o seu site: ferramentas fáceis de usar, espaço de sobra e acessórios. --0-1342140147-1032361849=:71092 Content-Type: text/html; charset=iso-8859-1 Content-Transfer-Encoding: 8bit

Essa eu respondo:AMM=American Mathematical Monthly,"Matematica Estadunidense Mensal",a Eureka dos States.La tem alguns dos melhores problemas,e so perde para a CRUX Mathematicorum

 Antonio Lacerda Junior wrote:

>Sauda,c~oes,

>O problema 1 não tem solução com régua
>e compasso. Mas sempre tem uma solução
>para três qq segmentos (ver AMM 101, 1994,
>pp. 58--60).

>Substituindo bissetrizes por alturas ou medianas,
>aí a coisa muda: a construção é possível, mas
>nem sempre.


>[]'s
>Luis

Luis, obrigado pelo esclarecimento, mas tu poderias
explicar-me o que significa AMM?

Quanto ao problema 2, se a minha memória não me falha,
já saiu até na Superinteressante (talvez em 1992). Se
alguém souber de algum outro registro desse problema,
por favor, avise-me. Eu agradecerei.

Antônio Lacerda




__________________________________________________________________________
AcessoBOL, só R$ 9,90! O menor preço do mercado!
Assine já! http://www.bol.com.br/acessobol


=========================================================================
Instruções para entrar na lista, sair da lista e usar a lista em
http://www.mat.puc-rio.br/~nicolau/olimp/obm-l.html
O administrador desta lista é
=========================================================================



Yahoo! GeoCities
Tudo para criar o seu site: ferramentas fáceis de usar, espaço de sobra e acessórios. --0-1342140147-1032361849=:71092-- ========================================================================= Instruções para entrar na lista, sair da lista e usar a lista em http://www.mat.puc-rio.br/~nicolau/olimp/obm-l.html O administrador desta lista é ========================================================================= From owner-obm-l@sucuri.mat.puc-rio.br Wed Sep 18 13:42:38 2002 Return-Path: Received: (from majordom@localhost) by sucuri.mat.puc-rio.br (8.9.3/8.9.3) id NAA17599 for obm-l-MTTP; Wed, 18 Sep 2002 13:35:14 -0300 Received: from hotmail.com (f12.law9.hotmail.com [64.4.9.12]) by sucuri.mat.puc-rio.br (8.9.3/8.9.3) with ESMTP id NAA17595 for ; Wed, 18 Sep 2002 13:35:12 -0300 Received: from mail pickup service by hotmail.com with Microsoft SMTPSVC; Wed, 18 Sep 2002 09:36:38 -0700 Received: from 143.107.45.30 by lw9fd.law9.hotmail.msn.com with HTTP; Wed, 18 Sep 2002 16:36:38 GMT X-Originating-IP: [143.107.45.30] From: "Rogerio Fajardo" To: obm-l@mat.puc-rio.br Subject: Re: [obm-l] Axioma da Escolha Date: Wed, 18 Sep 2002 16:36:38 +0000 Mime-Version: 1.0 Content-Type: text/plain; format=flowed Message-ID: X-OriginalArrivalTime: 18 Sep 2002 16:36:38.0727 (UTC) FILETIME=[8F9D1D70:01C25F31] Sender: owner-obm-l@sucuri.mat.puc-rio.br Precedence: bulk Reply-To: obm-l@mat.puc-rio.br Realmente, parece que eu gerei mais polêmica do que esperava. Vou indicar um site que explica muito bem o Axioma da Escolha - seu enuniado, aplicações e discussões filosóficas a respeito de seu uso. O site é: http://math.vanderbilt.edu/~schectex/ccc/choice.html >From: "Jose Francisco Guimaraes Costa" >Reply-To: obm-l@mat.puc-rio.br >To: "obm-l" >Subject: [obm-l] Axioma da Escolha >Date: Tue, 17 Sep 2002 15:18:29 -0300 > >Nos últimos dias o assunto mais tratado aqui neste forum vem sendo o Axioma >da >Escolha. > >Alguém poderia fornecer o enunciado e um pequeno histórico dele? > >JF > >-----Mensagem Original----- >De: Rogerio Fajardo >Para: >Enviada em: Terça-feira, 17 de Setembro de 2002 13:30 >Assunto: RE: [obm-l] Axioma da Escolha > > > > A maneira usual de fazer infinitas escolhas sem usar o axioma da escolha >é >(...) > > >========================================================================= >Instruções para entrar na lista, sair da lista e usar a lista em >http://www.mat.puc-rio.br/~nicolau/olimp/obm-l.html >O administrador desta lista é >========================================================================= _________________________________________________________________ MSN Photos is the easiest way to share and print your photos: http://photos.msn.com/support/worldwide.aspx ========================================================================= Instruções para entrar na lista, sair da lista e usar a lista em http://www.mat.puc-rio.br/~nicolau/olimp/obm-l.html O administrador desta lista é ========================================================================= From owner-obm-l@sucuri.mat.puc-rio.br Wed Sep 18 14:04:50 2002 Return-Path: Received: (from majordom@localhost) by sucuri.mat.puc-rio.br (8.9.3/8.9.3) id OAA18128 for obm-l-MTTP; Wed, 18 Sep 2002 14:02:28 -0300 Received: from web12906.mail.yahoo.com (web12906.mail.yahoo.com [216.136.174.73]) by sucuri.mat.puc-rio.br (8.9.3/8.9.3) with SMTP id OAA18124 for ; Wed, 18 Sep 2002 14:02:25 -0300 Message-ID: <20020918170033.6093.qmail@web12906.mail.yahoo.com> Received: from [200.206.103.3] by web12906.mail.yahoo.com via HTTP; Wed, 18 Sep 2002 14:00:33 ART Date: Wed, 18 Sep 2002 14:00:33 -0300 (ART) From: =?iso-8859-1?q?Johann=20Peter=20Gustav=20Lejeune=20Dirichlet?= Subject: Re: RE de re:[obm-l] Ajuda Algebra linear (Off Topic) To: obm-l@mat.puc-rio.br In-Reply-To: <3613822695.20020917154730@gmx.net> MIME-Version: 1.0 Content-Type: multipart/alternative; boundary="0-1248707563-1032368433=:5033" Content-Transfer-Encoding: 8bit Sender: owner-obm-l@sucuri.mat.puc-rio.br Precedence: bulk Reply-To: obm-l@mat.puc-rio.br --0-1248707563-1032368433=:5033 Content-Type: text/plain; charset=iso-8859-1 Content-Transfer-Encoding: 8bit Como eu estava de saida nao dei muitas informaçoes.Mas aqui vai:depois de entrar no arquivo da Semana Olimpica,entre no icone da quarta semana olimpica e pronto!! Igor GomeZZ wrote: Em 16/9/2002, 13:57, Johann (peterdirichlet2002@yahoo.com.br) disse: > Beleza!La ele da uma aplicaçao bem interessante:como se dar bem > blefando em um jogo de truco(parece jogo de truco mas mudam algumas regras). Qual o tópico? Não conseguir encontrar... Fui! ####### Igor GomeZZ ######## UIN: 29249895 Vitória, Espírito Santo, Brasil Criação: 17/9/2002 (15:45) #################################### Pare para pensar: O rio atinge seus objetivos porque aprendeu a contornar obstáculos. (Lao- Tsé) #################################### ========================================================================= Instruções para entrar na lista, sair da lista e usar a lista em http://www.mat.puc-rio.br/~nicolau/olimp/obm-l.html O administrador desta lista é ========================================================================= --------------------------------- Yahoo! GeoCities Tudo para criar o seu site: ferramentas fáceis de usar, espaço de sobra e acessórios. --0-1248707563-1032368433=:5033 Content-Type: text/html; charset=iso-8859-1 Content-Transfer-Encoding: 8bit

Como eu estava de saida nao dei muitas informaçoes.Mas aqui vai:depois de entrar no arquivo da Semana Olimpica,entre no icone da quarta semana olimpica e pronto!!

 Igor GomeZZ wrote:


Em 16/9/2002, 13:57, Johann (peterdirichlet2002@yahoo.com.br) disse:


> Beleza!La ele da uma aplicaçao bem interessante:como se dar bem
> blefando em um jogo de truco(parece jogo de truco mas mudam algumas regras).

Qual o tópico? Não conseguir encontrar...

Fui!


####### Igor GomeZZ ########
UIN: 29249895
Vitória, Espírito Santo, Brasil
Criação: 17/9/2002 (15:45)
####################################
Pare para pensar:

O rio atinge seus objetivos porque
aprendeu a contornar obstáculos.
(Lao- Tsé)

####################################

=========================================================================
Instruções para entrar na lista, sair da lista e usar a lista em
http://www.mat.puc-rio.br/~nicolau/olimp/obm-l.html
O administrador desta lista é
=========================================================================



Yahoo! GeoCities
Tudo para criar o seu site: ferramentas fáceis de usar, espaço de sobra e acessórios. --0-1248707563-1032368433=:5033-- ========================================================================= Instruções para entrar na lista, sair da lista e usar a lista em http://www.mat.puc-rio.br/~nicolau/olimp/obm-l.html O administrador desta lista é ========================================================================= From owner-obm-l@sucuri.mat.puc-rio.br Wed Sep 18 14:12:36 2002 Return-Path: Received: (from majordom@localhost) by sucuri.mat.puc-rio.br (8.9.3/8.9.3) id OAA18453 for obm-l-MTTP; Wed, 18 Sep 2002 14:10:08 -0300 Received: from web12903.mail.yahoo.com (web12903.mail.yahoo.com [216.136.174.70]) by sucuri.mat.puc-rio.br (8.9.3/8.9.3) with SMTP id OAA18449 for ; Wed, 18 Sep 2002 14:10:05 -0300 Message-ID: <20020918171132.17259.qmail@web12903.mail.yahoo.com> Received: from [200.206.103.3] by web12903.mail.yahoo.com via HTTP; Wed, 18 Sep 2002 14:11:32 ART Date: Wed, 18 Sep 2002 14:11:32 -0300 (ART) From: =?iso-8859-1?q?Johann=20Peter=20Gustav=20Lejeune=20Dirichlet?= Subject: Re: [obm-l] Axioma da Escolha To: obm-l@mat.puc-rio.br In-Reply-To: <20020917193659.A28F73EB3F@zeus.opendf.com.br> MIME-Version: 1.0 Content-Type: multipart/alternative; boundary="0-804094115-1032369092=:16585" Content-Transfer-Encoding: 8bit Sender: owner-obm-l@sucuri.mat.puc-rio.br Precedence: bulk Reply-To: obm-l@mat.puc-rio.br --0-804094115-1032369092=:16585 Content-Type: text/plain; charset=iso-8859-1 Content-Transfer-Encoding: 8bit Tem um teorema que e equivalente ao Axioma da Escolha,o Lema de Zorn:Se X e um conjunto nao vazio parcialmente ordenado(ordenado de qualquer jeito,sem a ideia de que um cara seja maior,menor ou igual a outro)tal que toda cadeia(conjunto totalmente ordenado)em X tenha um limite superior,entao X contem um elemento maximal. 498 - Artur Costa Steiner wrote: > Nos últimos dias o assunto mais tratado aqui neste forum vem sendo o Axioma > da > Escolha. > > Alguém poderia fornecer o enunciado e um pequeno histórico dele? > > JF O enunciado mais usual é o seguinte: Dada uma coleção qualquer de conjuntos disjuntos {A_a} (finita ou infinita, numerável ou não), é possível formar um conjunto S tal que cada elemento de S pertença a um dos conjuntos A_a. Isto é, é possível formar S escolhendo-se um elemento de cada um dos conjuntos A_a, daí o nome Axioma da Escolha. Alguns autores definem o axioma sem requerer que a coleção {A_a} seja disjunta. Artur ========================================================================= Instruções para entrar na lista, sair da lista e usar a lista em http://www.mat.puc-rio.br/~nicolau/olimp/obm-l.html O administrador desta lista é ========================================================================= --------------------------------- Yahoo! GeoCities Tudo para criar o seu site: ferramentas fáceis de usar, espaço de sobra e acessórios. --0-804094115-1032369092=:16585 Content-Type: text/html; charset=iso-8859-1 Content-Transfer-Encoding: 8bit

Tem um teorema que e equivalente ao Axioma da Escolha,o

Lema de Zorn:Se X e um conjunto nao vazio parcialmente ordenado(ordenado de qualquer jeito,sem a ideia de que um cara seja maior,menor ou igual a outro)tal que toda cadeia(conjunto totalmente ordenado)em X tenha um limite superior,entao X contem um elemento maximal.

 498 - Artur Costa Steiner wrote:



> Nos últimos dias o assunto mais tratado aqui neste forum vem sendo o
Axioma
> da
> Escolha.
>
> Alguém poderia fornecer o enunciado e um pequeno histórico dele?
>
> JF

O enunciado mais usual é o seguinte:

Dada uma coleção qualquer de conjuntos disjuntos {A_a} (finita ou
infinita, numerável ou não), é possível formar um conjunto S tal que
cada elemento de S pertença a um dos conjuntos A_a. Isto é, é possível
formar S escolhendo-se um elemento de cada um dos conjuntos A_a, daí o
nome Axioma da Escolha.

Alguns autores definem o axioma sem requerer que a coleção {A_a} seja
disjunta.
Artur
=========================================================================
Instruções para entrar na lista, sair da lista e usar a lista em
http://www.mat.puc-rio.br/~nicolau/olimp/obm-l.html
O administrador desta lista é
=========================================================================



Yahoo! GeoCities
Tudo para criar o seu site: ferramentas fáceis de usar, espaço de sobra e acessórios. --0-804094115-1032369092=:16585-- ========================================================================= Instruções para entrar na lista, sair da lista e usar a lista em http://www.mat.puc-rio.br/~nicolau/olimp/obm-l.html O administrador desta lista é ========================================================================= From owner-obm-l@sucuri.mat.puc-rio.br Wed Sep 18 14:21:56 2002 Return-Path: Received: (from majordom@localhost) by sucuri.mat.puc-rio.br (8.9.3/8.9.3) id OAA18719 for obm-l-MTTP; Wed, 18 Sep 2002 14:18:51 -0300 Received: from web12908.mail.yahoo.com (web12908.mail.yahoo.com [216.136.174.75]) by sucuri.mat.puc-rio.br (8.9.3/8.9.3) with SMTP id OAA18713 for ; Wed, 18 Sep 2002 14:18:47 -0300 Message-ID: <20020918172013.67542.qmail@web12908.mail.yahoo.com> Received: from [200.206.103.3] by web12908.mail.yahoo.com via HTTP; Wed, 18 Sep 2002 14:20:13 ART Date: Wed, 18 Sep 2002 14:20:13 -0300 (ART) From: =?iso-8859-1?q?Johann=20Peter=20Gustav=20Lejeune=20Dirichlet?= Subject: Re: [obm-l] Grafos To: obm-l@mat.puc-rio.br In-Reply-To: <000d01c25eb2$42d5e960$f008f9c8@david> MIME-Version: 1.0 Content-Type: multipart/alternative; boundary="0-1554978541-1032369613=:67209" Content-Transfer-Encoding: 8bit Sender: owner-obm-l@sucuri.mat.puc-rio.br Precedence: bulk Reply-To: obm-l@mat.puc-rio.br --0-1554978541-1032369613=:67209 Content-Type: text/plain; charset=iso-8859-1 Content-Transfer-Encoding: 8bit Ola David e turma da lista!!!!!! Em portugues tem na Internet:acesse o site da OBM (www.obm.org.br)e va ate a Semana Olimpica,la voce encontra dois artigos sobre grafos,um do Paulo Jose Bonfim Gomes Rodrigues e outro do (meu idolo!!!!!!) Carlos Yuzo Shine. Em ingles tem o livro(best-seller da Matematica Elementar e adjacencias) "Proofs from THE BOOK",que voce encontra na Amazon( www.amazon.com),que nao fala so de grafos. Esses tem mais a ver com olimpiadas David Ricardo wrote:Pessoal, Gostaria que vocês me indicassem algum bom material (em inglês ou português) sobre grafos e suas aplicações. Grato, David ========================================================================= Instruções para entrar na lista, sair da lista e usar a lista em http://www.mat.puc-rio.br/~nicolau/olimp/obm-l.html O administrador desta lista é ========================================================================= TRANSIRE SVVM PECTVS MVNDOQUE POTIRE CONGREGATI EX TOTO ORBE MATHEMATICI OB SCRIPTA INSIGNIA TRIBVERE Fields Medal(John Charles Fields) --------------------------------- Yahoo! GeoCities Tudo para criar o seu site: ferramentas fáceis de usar, espaço de sobra e acessórios. --0-1554978541-1032369613=:67209 Content-Type: text/html; charset=iso-8859-1 Content-Transfer-Encoding: 8bit

Ola David e turma da lista!!!!!!

Em portugues tem na Internet:acesse o site da OBM (www.obm.org.br)e va ate a Semana Olimpica,la voce encontra dois artigos sobre grafos,um do Paulo Jose Bonfim Gomes Rodrigues e outro do (meu idolo!!!!!!) Carlos Yuzo Shine.

Em ingles tem o livro(best-seller da Matematica Elementar e adjacencias) "Proofs from THE BOOK",que voce encontra na Amazon( www.amazon.com),que nao fala so de grafos. 

Esses tem mais a ver com olimpiadas

 David Ricardo wrote:

Pessoal,

Gostaria que vocês me indicassem algum bom material (em inglês ou português)
sobre grafos e suas aplicações.

Grato,
David

=========================================================================
Instruções para entrar na lista, sair da lista e usar a lista em
http://www.mat.puc-rio.br/~nicolau/olimp/obm-l.html
O administrador desta lista é
=========================================================================


TRANSIRE SVVM PECTVS MVNDOQUE POTIRE

CONGREGATI EX TOTO ORBE MATHEMATICI OB SCRIPTA INSIGNIA TRIBVERE

Fields Medal(John Charles Fields)



Yahoo! GeoCities
Tudo para criar o seu site: ferramentas fáceis de usar, espaço de sobra e acessórios. --0-1554978541-1032369613=:67209-- ========================================================================= Instruções para entrar na lista, sair da lista e usar a lista em http://www.mat.puc-rio.br/~nicolau/olimp/obm-l.html O administrador desta lista é ========================================================================= From owner-obm-l@sucuri.mat.puc-rio.br Wed Sep 18 15:36:31 2002 Return-Path: Received: (from majordom@localhost) by sucuri.mat.puc-rio.br (8.9.3/8.9.3) id PAA21228 for obm-l-MTTP; Wed, 18 Sep 2002 15:32:30 -0300 Received: from zeus.opendf.com.br (zeus.opengate.com.br [200.181.71.10]) by sucuri.mat.puc-rio.br (8.9.3/8.9.3) with ESMTP id PAA21224 for ; Wed, 18 Sep 2002 15:32:27 -0300 Received: from localhost (localhost.opengate.com.br [127.0.0.1]) by zeus.opendf.com.br (Postfix) with ESMTP id 274D33EA91 for ; Wed, 18 Sep 2002 15:33:45 -0300 (BRT) Received: by zeus.opendf.com.br (Postfix, from userid 48) id 0E6303EB57; Wed, 18 Sep 2002 15:32:31 -0300 (BRT) From: "498 - Artur Costa Steiner" To: obm-l@mat.puc-rio.br Subject: Re: [obm-l] Axioma da Escolha X-Mailer: NeoMail 1.25 X-IPAddress: 200.252.155.2 MIME-Version: 1.0 Content-Type: text/plain; charset=iso-8859-1 Message-Id: <20020918183231.0E6303EB57@zeus.opendf.com.br> Date: Wed, 18 Sep 2002 15:32:31 -0300 (BRT) X-Virus-Scanned: by AMaViS new-20020517 Sender: owner-obm-l@sucuri.mat.puc-rio.br Precedence: bulk Reply-To: obm-l@mat.puc-rio.br > Realmente, parece que eu gerei mais polêmica do que esperava. Vou indicar um > site que explica muito bem o Axioma da Escolha - seu enuniado, aplicações e > discussões filosóficas a respeito de seu uso. O site é: > http://math.vanderbilt.edu/~schectex/ccc/choice.html Não tem problema, esta discussão toda foi excelente. Antes dela eu não havia prestado muito atenção no axioma, parecia estar na "massa do meu sangue". O artigo do site é excelente. Há um ponto a respeito do qual estou ainda um tanto intrigado: realizar infinitas escolhas arbitrárias em infinitos conjuntos requer o axioma. Mas, realizar infinitas escolhas arbitrárias em UM ÚNICO conjunto não requer o axioma. Quando se trata de um único conjunto, é sempre possível encontrar um afunção de escolha bem definida? Obrigado Artur ========================================================================= Instruções para entrar na lista, sair da lista e usar a lista em http://www.mat.puc-rio.br/~nicolau/olimp/obm-l.html O administrador desta lista é ========================================================================= From owner-obm-l@sucuri.mat.puc-rio.br Wed Sep 18 16:03:23 2002 Return-Path: Received: (from majordom@localhost) by sucuri.mat.puc-rio.br (8.9.3/8.9.3) id QAA21880 for obm-l-MTTP; Wed, 18 Sep 2002 16:00:33 -0300 Received: from Euler.impa.br (euler.impa.br [147.65.1.3]) by sucuri.mat.puc-rio.br (8.9.3/8.9.3) with ESMTP id QAA21853 for ; Wed, 18 Sep 2002 16:00:29 -0300 Received: from obm-01 (obm-01.impa.br [147.65.2.170]) by Euler.impa.br (8.11.6/8.11.6) with SMTP id g8IJ1vM13924 for ; Wed, 18 Sep 2002 16:01:57 -0300 (EST) Message-Id: <3.0.5.32.20020918160233.007d6870@pop.impa.br> X-Sender: obm@pop.impa.br X-Mailer: QUALCOMM Windows Eudora Light Version 3.0.5 (32) Date: Wed, 18 Sep 2002 16:02:33 -0300 To: obm-l@mat.puc-rio.br From: Olimpiada Brasileira de Matematica Subject: Re: [obm-l] OBM-2002 In-Reply-To: <20020918150637.61533.qmail@web12906.mail.yahoo.com> References: <622411757.20020917004135@gmx.net> Mime-Version: 1.0 Content-Type: text/plain; charset="us-ascii" Sender: owner-obm-l@sucuri.mat.puc-rio.br Precedence: bulk Reply-To: obm-l@mat.puc-rio.br Caros(as) amigos(as)da lista, Ja esta no site a versao .ps da Eureka No. 14 Abracos, Nelly. ========================================================================= Instruções para entrar na lista, sair da lista e usar a lista em http://www.mat.puc-rio.br/~nicolau/olimp/obm-l.html O administrador desta lista é ========================================================================= From owner-obm-l@sucuri.mat.puc-rio.br Wed Sep 18 19:06:21 2002 Return-Path: Received: (from majordom@localhost) by sucuri.mat.puc-rio.br (8.9.3/8.9.3) id TAA24701 for obm-l-MTTP; Wed, 18 Sep 2002 19:03:11 -0300 Received: from hotmail.com (f93.sea2.hotmail.com [207.68.165.93]) by sucuri.mat.puc-rio.br (8.9.3/8.9.3) with ESMTP id TAA24679 for ; Wed, 18 Sep 2002 19:02:27 -0300 Received: from mail pickup service by hotmail.com with Microsoft SMTPSVC; Wed, 18 Sep 2002 15:03:28 -0700 Received: from 32.94.119.253 by sea2fd.sea2.hotmail.msn.com with HTTP; Wed, 18 Sep 2002 22:03:28 GMT X-Originating-IP: [32.94.119.253] From: "Paulo Santa Rita" To: obm-l@mat.puc-rio.br Subject: [obm-l] Revista Eureka! Date: Wed, 18 Sep 2002 22:03:28 +0000 Mime-Version: 1.0 Content-Type: text/plain; charset=iso-8859-1; format=flowed Message-ID: X-OriginalArrivalTime: 18 Sep 2002 22:03:28.0465 (UTC) FILETIME=[37EDAC10:01C25F5F] Sender: owner-obm-l@sucuri.mat.puc-rio.br Precedence: bulk Reply-To: obm-l@mat.puc-rio.br Ola Pessoal, A mensagem anterior deveria ter ido para um colega, mas foi para a lista por engano. Peco desculpas. Um abraco Paulo Santa Rita 4,1902,180902 _________________________________________________________________ Converse com seus amigos online, faça o download grátis do MSN Messenger: http://messenger.msn.com.br ========================================================================= Instruções para entrar na lista, sair da lista e usar a lista em http://www.mat.puc-rio.br/~nicolau/olimp/obm-l.html O administrador desta lista é ========================================================================= From owner-obm-l@sucuri.mat.puc-rio.br Wed Sep 18 19:34:15 2002 Return-Path: Received: (from majordom@localhost) by sucuri.mat.puc-rio.br (8.9.3/8.9.3) id TAA25313 for obm-l-MTTP; Wed, 18 Sep 2002 19:31:24 -0300 Received: from traven9.pub1 (traven9.uol.com.br [200.221.4.35]) by sucuri.mat.puc-rio.br (8.9.3/8.9.3) with ESMTP id TAA25309 for ; Wed, 18 Sep 2002 19:31:22 -0300 Received: from u2z7z2 ([200.158.144.232]) by traven9.pub1 (8.9.1/8.9.1) with ESMTP id TAA25083 for ; Wed, 18 Sep 2002 19:34:31 -0300 (BRT) Message-ID: <002001c25f63$7e02a9e0$2101a8c0@u2z7z2> From: "Wagner" To: Subject: [obm-l] =?iso-8859-1?Q?fatoriais_n=E3o_inteiros?= Date: Wed, 18 Sep 2002 19:34:02 -0300 Organization: Wagner MIME-Version: 1.0 Content-Type: multipart/alternative; boundary="----=_NextPart_000_001D_01C25F4A.58047160" X-Priority: 3 X-MSMail-Priority: Normal X-Mailer: Microsoft Outlook Express 5.50.4133.2400 X-MimeOLE: Produced By Microsoft MimeOLE V5.50.4133.2400 Sender: owner-obm-l@sucuri.mat.puc-rio.br Precedence: bulk Reply-To: obm-l@mat.puc-rio.br This is a multi-part message in MIME format. ------=_NextPart_000_001D_01C25F4A.58047160 Content-Type: text/plain; charset="iso-8859-1" Content-Transfer-Encoding: quoted-printable Oi pessoal ! Qual o sentido de n! se n n=E3o pertence aos inteiros, seja n positivo = ou negativo? (esses fatoriais podem ser calculados por uma calculadora = cient=EDfica como a que vem no computador). Como =E9 feito o c=E1lculo = de n! nessas condi=E7=F5es? Se algu=E9m puder me esclarecer eu agrade=E7o Andr=E9 T. ------=_NextPart_000_001D_01C25F4A.58047160 Content-Type: text/html; charset="iso-8859-1" Content-Transfer-Encoding: quoted-printable
Oi pessoal !
 
Qual o sentido de n! se n n=E3o = pertence aos=20 inteiros, seja n positivo ou negativo? (esses fatoriais podem ser = calculados por=20 uma calculadora cient=EDfica como a que vem no computador). Como =E9 = feito o c=E1lculo=20 de n! nessas condi=E7=F5es?
 
Se algu=E9m puder me esclarecer eu=20 agrade=E7o
 
Andr=E9 T.
------=_NextPart_000_001D_01C25F4A.58047160-- ========================================================================= Instruções para entrar na lista, sair da lista e usar a lista em http://www.mat.puc-rio.br/~nicolau/olimp/obm-l.html O administrador desta lista é ========================================================================= From owner-obm-l@sucuri.mat.puc-rio.br Wed Sep 18 22:57:36 2002 Return-Path: Received: (from majordom@localhost) by sucuri.mat.puc-rio.br (8.9.3/8.9.3) id WAA27482 for obm-l-MTTP; Wed, 18 Sep 2002 22:55:12 -0300 Received: from traven10.pub1 (traven10.uol.com.br [200.221.4.45]) by sucuri.mat.puc-rio.br (8.9.3/8.9.3) with ESMTP id WAA27478 for ; Wed, 18 Sep 2002 22:55:10 -0300 Received: from Eder ([200.211.154.130]) by traven10.pub1 (8.9.1/8.9.1) with SMTP id WAA12074 for ; Wed, 18 Sep 2002 22:55:57 -0300 (BRT) Message-ID: <002701c25f80$4a03a280$f561fea9@Eder> From: "Eder" To: Subject: [obm-l] ? Date: Wed, 18 Sep 2002 22:59:21 -0300 MIME-Version: 1.0 Content-Type: multipart/alternative; boundary="----=_NextPart_000_0024_01C25F67.06443A00" X-Priority: 3 X-MSMail-Priority: Normal X-Mailer: Microsoft Outlook Express 5.00.2314.1300 X-MimeOLE: Produced By Microsoft MimeOLE V5.00.2314.1300 Sender: owner-obm-l@sucuri.mat.puc-rio.br Precedence: bulk Reply-To: obm-l@mat.puc-rio.br This is a multi-part message in MIME format. ------=_NextPart_000_0024_01C25F67.06443A00 Content-Type: text/plain; charset="iso-8859-1" Content-Transfer-Encoding: quoted-printable Como eu mostro que n(n+1)(n+2)(n+3)+1 =E9 um quadrado para n=3D1,2,3,... = ? ------=_NextPart_000_0024_01C25F67.06443A00 Content-Type: text/html; charset="iso-8859-1" Content-Transfer-Encoding: quoted-printable
Como eu mostro que n(n+1)(n+2)(n+3)+1 = =E9 um quadrado=20 para n=3D1,2,3,...      = ?
------=_NextPart_000_0024_01C25F67.06443A00-- ========================================================================= Instruções para entrar na lista, sair da lista e usar a lista em http://www.mat.puc-rio.br/~nicolau/olimp/obm-l.html O administrador desta lista é ========================================================================= From owner-obm-l@sucuri.mat.puc-rio.br Wed Sep 18 23:46:31 2002 Return-Path: Received: (from majordom@localhost) by sucuri.mat.puc-rio.br (8.9.3/8.9.3) id XAA28217 for obm-l-MTTP; Wed, 18 Sep 2002 23:44:04 -0300 Received: from trex.centroin.com.br (trex.centroin.com.br [200.225.63.134]) by sucuri.mat.puc-rio.br (8.9.3/8.9.3) with ESMTP id XAA28212 for ; Wed, 18 Sep 2002 23:44:02 -0300 Received: from trex.centroin.com.br (localhost [127.0.0.1]) by trex.centroin.com.br (8.12.5/8.12.1) with ESMTP id g8J2jrNg012185 for ; Wed, 18 Sep 2002 23:45:53 -0300 (BRT) Received: by trex.centroin.com.br (8.12.5/8.12.5/Submit) id g8J2jrHd012184; Wed, 18 Sep 2002 23:45:53 -0300 (BRT) Message-Id: <200209190245.g8J2jrHd012184@trex.centroin.com.br> Received: from 200.225.57.95 by trex.centroin.com.br (CIPWM versao 1.4C1) with HTTPS for ; Wed, 18 Sep 2002 23:45:53 -0300 (BRT) Date: Wed, 18 Sep 2002 23:45:53 -0300 (BRT) From: Augusto Cesar de Oliveira Morgado To: obm-l@mat.puc-rio.br Subject: Re: [obm-l] ? MIME-Version: 1.0 X-Mailer: CentroIn Internet Provider WebMail v. 1.4C1 (http://www.centroin.com.br/) Content-Type: text/plain; charset="iso-8859-1" Content-Transfer-Encoding: 8bit X-MIME-Autoconverted: from quoted-printable to 8bit by sucuri.mat.puc-rio.br id XAA28213 Sender: owner-obm-l@sucuri.mat.puc-rio.br Precedence: bulk Reply-To: obm-l@mat.puc-rio.br Em Wed, 18 Sep 2002 22:59:21 -0300, Eder disse: > Como eu mostro que n(n+1)(n+2)(n+3)+1 é um quadrado para n=1,2,3,... ? Faça as contas. Dá n^4 +6n^3+11n^2+6n +1 = [n^2+3n+1]^2 ========================================================================= Instruções para entrar na lista, sair da lista e usar a lista em http://www.mat.puc-rio.br/~nicolau/olimp/obm-l.html O administrador desta lista é ========================================================================= From owner-obm-l@sucuri.mat.puc-rio.br Thu Sep 19 00:13:13 2002 Return-Path: Received: (from majordom@localhost) by sucuri.mat.puc-rio.br (8.9.3/8.9.3) id AAA28688 for obm-l-MTTP; Thu, 19 Sep 2002 00:09:18 -0300 Received: from web13304.mail.yahoo.com (web13304.mail.yahoo.com [216.136.175.40]) by sucuri.mat.puc-rio.br (8.9.3/8.9.3) with SMTP id AAA28684 for ; Thu, 19 Sep 2002 00:09:15 -0300 Message-ID: <20020919031043.55855.qmail@web13304.mail.yahoo.com> Received: from [200.190.0.76] by web13304.mail.yahoo.com via HTTP; Wed, 18 Sep 2002 20:10:43 PDT Date: Wed, 18 Sep 2002 20:10:43 -0700 (PDT) From: Carlos Yuzo Shine Subject: Re: [obm-l] Grafos To: obm-l@mat.puc-rio.br In-Reply-To: <20020918172013.67542.qmail@web12908.mail.yahoo.com> MIME-Version: 1.0 Content-Type: text/plain; charset=us-ascii Sender: owner-obm-l@sucuri.mat.puc-rio.br Precedence: bulk Reply-To: obm-l@mat.puc-rio.br Além dessas referências, tem os seguintes livros: Belá Bollobas - Graph Theory - An Introductory Course R. Diestel - Modern Graph Theory (ou Graph Theory, não lembro direito o nome). Em diversas revistas Eureka! também há artigos sobre grafos. Tem um meu bem introdutório na Eureka! 12 e tem outro do Gugu que fala um pouco sobre Teoria de Ramsey na Eureka! 6. Ah, e vi um do PC na Eureka! 1, acho. Confira em http://www.obm.org.br/eureka/abstrac.htm []'s Shine --- Johann Peter Gustav Lejeune Dirichlet wrote: > > Ola David e turma da lista!!!!!! > Em portugues tem na Internet:acesse o site da OBM (www.obm.org.br)e va ate a Semana Olimpica,la voce encontra dois artigos sobre grafos,um do Paulo Jose Bonfim Gomes Rodrigues e outro do (meu idolo!!!!!!) Carlos Yuzo Shine. > Em ingles tem o livro(best-seller da Matematica Elementar e adjacencias) "Proofs from THE BOOK",que voce encontra na Amazon( www.amazon.com),que nao fala so de grafos. > Esses tem mais a ver com olimpiadas > David Ricardo wrote:Pessoal, > > Gostaria que vocês me indicassem algum bom material (em inglês ou português) > sobre grafos e suas aplicações. > > Grato, > David __________________________________________________ Do you Yahoo!? New DSL Internet Access from SBC & Yahoo! http://sbc.yahoo.com ========================================================================= Instruções para entrar na lista, sair da lista e usar a lista em http://www.mat.puc-rio.br/~nicolau/olimp/obm-l.html O administrador desta lista é ========================================================================= From owner-obm-l@sucuri.mat.puc-rio.br Thu Sep 19 00:14:59 2002 Return-Path: Received: (from majordom@localhost) by sucuri.mat.puc-rio.br (8.9.3/8.9.3) id AAA28725 for obm-l-MTTP; Thu, 19 Sep 2002 00:12:26 -0300 Received: from mail.gmx.net (mail.gmx.net [213.165.64.20]) by sucuri.mat.puc-rio.br (8.9.3/8.9.3) with SMTP id AAA28721 for ; Thu, 19 Sep 2002 00:12:23 -0300 Received: (qmail 6631 invoked by uid 0); 19 Sep 2002 03:13:50 -0000 Received: from unknown (HELO gomes) (200.216.104.118) by mail.gmx.net (mp011-rz3) with SMTP; 19 Sep 2002 03:13:50 -0000 Date: Thu, 19 Sep 2002 00:14:55 -0300 From: Igor GomeZZ X-Mailer: The Bat! (v1.61) Organization: -- X-Priority: 3 (Normal) Message-ID: <1193220681.20020919001455@gmx.net> To: Johann Peter Gustav Lejeune Dirichlet Subject: Re[2]: [obm-l] OBM-2002 In-Reply-To: <20020918150637.61533.qmail@web12906.mail.yahoo.com> References: <20020918150637.61533.qmail@web12906.mail.yahoo.com> MIME-Version: 1.0 Content-Type: text/plain; charset=ISO-8859-1 Content-Transfer-Encoding: 8bit Sender: owner-obm-l@sucuri.mat.puc-rio.br Precedence: bulk Reply-To: obm-l@mat.puc-rio.br Em 18/9/2002, 12:06, Johann (peterdirichlet2002@yahoo.com.br) disse: > Calma,e claro que vai ter.Basta esperar(eu acho).Alias como e > que a Nelly tem coragem de fazer isso tudo no Word??????? Opa, jah online :) Deve ser maior trampo mexer com isso no word :P Fui! ####### Igor GomeZZ ######## UIN: 29249895 Vitória, Espírito Santo, Brasil Criação: 19/9/2002 (00:13) #################################### Pare para pensar: Olho por olho, e o mundo terminará cego. (Gandhi) #################################### ========================================================================= Instruções para entrar na lista, sair da lista e usar a lista em http://www.mat.puc-rio.br/~nicolau/olimp/obm-l.html O administrador desta lista é ========================================================================= From owner-obm-l@sucuri.mat.puc-rio.br Thu Sep 19 00:20:59 2002 Return-Path: Received: (from majordom@localhost) by sucuri.mat.puc-rio.br (8.9.3/8.9.3) id AAA29017 for obm-l-MTTP; Thu, 19 Sep 2002 00:18:50 -0300 Received: from cassino.digi.com.br (cassino.digi.com.br [200.241.100.140]) by sucuri.mat.puc-rio.br (8.9.3/8.9.3) with ESMTP id AAA29013 for ; Thu, 19 Sep 2002 00:18:48 -0300 Received: from david (host229.e.digizap.com.br [200.249.8.229]) by cassino.digi.com.br (8.11.6/8.11.6) with SMTP id g8J3KrK09013 for ; Thu, 19 Sep 2002 00:20:53 -0300 Message-ID: <001301c25f8c$2d211c40$e508f9c8@david> From: "David Ricardo" To: References: <002701c25f80$4a03a280$f561fea9@Eder> Subject: Re: [obm-l] ? Date: Thu, 19 Sep 2002 00:25:14 -0300 MIME-Version: 1.0 Content-Type: text/plain; charset="iso-8859-1" Content-Transfer-Encoding: 8bit X-Priority: 3 X-MSMail-Priority: Normal X-Mailer: Microsoft Outlook Express 5.00.2615.200 X-MimeOLE: Produced By Microsoft MimeOLE V5.00.2615.200 X-MailScanner: Found to be clean Sender: owner-obm-l@sucuri.mat.puc-rio.br Precedence: bulk Reply-To: obm-l@mat.puc-rio.br n(n+1)(n+2)(n+3) + 1 = (n^2 + n)(n+2)(n+3) + 1 = (n^3 + 3n^2 + 2n)(n+3) + 1 = n^4 + 6n^3 + 11n^2 + 6n + 1 Mas n^4 + 6n^3 + 11n^2 + 6n + 1 é igual a (n^2 + 3n + 1)^2. Assim, para n=1,2,3,..., n(n+1)(n+2)(n+3) + 1 é um quadrado perfeito! []s David ----- Original Message ----- From: Eder To: obm-l@mat.puc-rio.br Sent: Wednesday, September 18, 2002 10:59 PM Subject: [obm-l] ? Como eu mostro que n(n+1)(n+2)(n+3)+1 é um quadrado para n=1,2,3,... ? ========================================================================= Instruções para entrar na lista, sair da lista e usar a lista em http://www.mat.puc-rio.br/~nicolau/olimp/obm-l.html O administrador desta lista é ========================================================================= From owner-obm-l@sucuri.mat.puc-rio.br Thu Sep 19 00:23:42 2002 Return-Path: Received: (from majordom@localhost) by sucuri.mat.puc-rio.br (8.9.3/8.9.3) id AAA29055 for obm-l-MTTP; Thu, 19 Sep 2002 00:21:09 -0300 Received: from gorgo.centroin.com.br (gorgo.centroin.com.br [200.225.63.128]) by sucuri.mat.puc-rio.br (8.9.3/8.9.3) with ESMTP id AAA29051 for ; Thu, 19 Sep 2002 00:21:07 -0300 Received: from centroin.com.br (du95b.rjo.centroin.com.br [200.225.57.95]) (authenticated bits=0) by gorgo.centroin.com.br (8.12.2/8.12.1) with ESMTP id g8J3Mu7p020644 for ; Thu, 19 Sep 2002 00:22:57 -0300 (BRT) Message-ID: <3D8943AA.3090809@centroin.com.br> Date: Thu, 19 Sep 2002 00:25:30 -0300 From: Augusto =?ISO-8859-1?Q?C=E9sar?= Morgado User-Agent: Mozilla/5.0 (Windows; U; Win98; en-US; rv:0.9.4.1) Gecko/20020508 Netscape6/6.2.3 X-Accept-Language: en-us MIME-Version: 1.0 To: obm-l@mat.puc-rio.br Subject: Re: [obm-l] ajuda References: <173.ea8e513.2ab7e130@aol.com> Content-Type: multipart/alternative; boundary="------------040807070706040404070002" Sender: owner-obm-l@sucuri.mat.puc-rio.br Precedence: bulk Reply-To: obm-l@mat.puc-rio.br --------------040807070706040404070002 Content-Type: text/plain; charset=us-ascii; format=flowed Content-Transfer-Encoding: 7bit 24! Se voce considerar que os carros sao iguais e que so interessa a ordem relativa dos casais, seria 24!/12. Lltmdrtm@aol.com wrote: > De quantas maneiras 24 pessoas podem subir numa roda gigante de 12 > assentos, sabendo que cada assento comporta duas pessoas e, a ordem > das pessoas no assento importa? > --------------040807070706040404070002 Content-Type: text/html; charset=us-ascii Content-Transfer-Encoding: 7bit 24!
Se voce considerar que os carros sao iguais e que so interessa a ordem relativa dos casais, seria 24!/12.

Lltmdrtm@aol.com wrote:
De quantas maneiras 24 pessoas podem subir numa roda gigante de 12 assentos, sabendo que cada assento comporta duas pessoas e, a ordem das pessoas no assento importa?


--------------040807070706040404070002-- ========================================================================= Instruções para entrar na lista, sair da lista e usar a lista em http://www.mat.puc-rio.br/~nicolau/olimp/obm-l.html O administrador desta lista é ========================================================================= From owner-obm-l@sucuri.mat.puc-rio.br Thu Sep 19 00:54:40 2002 Return-Path: Received: (from majordom@localhost) by sucuri.mat.puc-rio.br (8.9.3/8.9.3) id AAA31020 for obm-l-MTTP; Thu, 19 Sep 2002 00:52:31 -0300 Received: from traven9.pub1 (traven9.uol.com.br [200.221.4.35]) by sucuri.mat.puc-rio.br (8.9.3/8.9.3) with ESMTP id AAA31016 for ; Thu, 19 Sep 2002 00:52:29 -0300 Received: from franklin ([200.214.41.223]) by traven9.pub1 (8.9.1/8.9.1) with SMTP id AAA28847 for ; Thu, 19 Sep 2002 00:55:39 -0300 (BRT) From: "Franklin de Lima Marquezino" To: Subject: [obm-l] =?iso-8859-1?Q?Re:_=5Bobm-l=5D_fatoriais_n=E3o_inteiros?= Date: Thu, 19 Sep 2002 00:54:08 -0300 Message-ID: <01c25f90$34bc9a20$df29d6c8@franklin> MIME-Version: 1.0 Content-Type: text/plain; charset="iso-8859-1" Content-Transfer-Encoding: 8bit X-Priority: 3 X-MSMail-Priority: Normal X-Mailer: Microsoft Outlook Express 4.71.1712.3 X-MimeOLE: Produced By Microsoft MimeOLE V4.71.1712.3 Sender: owner-obm-l@sucuri.mat.puc-rio.br Precedence: bulk Reply-To: obm-l@mat.puc-rio.br >Oi pessoal ! > >Qual o sentido de n! se n não pertence aos inteiros, seja n positivo ou negativo? (esses fatoriais >podem ser calculados por uma calculadora científica como a que vem no computador). Como é feito >o cálculo de n! nessas condições? > >Se alguém puder me esclarecer eu agradeço > >André T. Olá, Está meio tarde, posso estar fazendo confusão, mas acho que você está se referindo à função Gama, que pode ser interpretada como uma generalização do fatorial. Depois eu confiro a definição de função e gama e vejo se tem sentido o que estou falando. Se não me engano, gama(n+1)=n!, quando n é natural, mas a função gama também é definida para números reais. Alguém interfira se eu estiver falando besteira, por favor. Até mais, Franklin PS: O Nicolau fez uma observação interessante na última mensagem que eu enviei. Ele disse que um "algoritmo quântico" não pode ser considerado um algoritmo por que ele não pode ser executado em uma máquina de Turing. Isso está correto considerando a definição original de máquina de Turing, mas posteriormente surgiram os conceitos de Máquina de Turing Probabilística e Maquina de Turing Quântica. Assim, o conceito de algoritmo é extendido. É apenas uma questão semântica, mas a observação do Nicolau foi legal. As duas últimas máquinas (MTP e MTQ) resolveriam facilmente o problema da geração de sequencias randômicas. ========================================================================= Instruções para entrar na lista, sair da lista e usar a lista em http://www.mat.puc-rio.br/~nicolau/olimp/obm-l.html O administrador desta lista é ========================================================================= From owner-obm-l@sucuri.mat.puc-rio.br Thu Sep 19 08:01:37 2002 Return-Path: Received: (from majordom@localhost) by sucuri.mat.puc-rio.br (8.9.3/8.9.3) id HAA01723 for obm-l-MTTP; Thu, 19 Sep 2002 07:59:27 -0300 Received: from artemis.opendf.com.br (artemis.opengate.com.br [200.181.71.15]) by sucuri.mat.puc-rio.br (8.9.3/8.9.3) with ESMTP id HAA01719 for ; Thu, 19 Sep 2002 07:59:24 -0300 Received: from localhost (localhost [127.0.0.1]) by artemis.opendf.com.br (Postfix) with ESMTP id 6964C1C887 for ; Thu, 19 Sep 2002 08:00:41 -0300 (BRT) Received: from computer (200-181-88-253-bsace7001.dsl.telebrasilia.net.br [200.181.88.253]) by artemis.opendf.com.br (Postfix) with ESMTP id 32E271BDFA for ; Thu, 19 Sep 2002 08:00:38 -0300 (BRT) From: "Artur Costa Steiner" To: Subject: [obm-l] =?iso-8859-1?Q?RE:_=5Bobm-l=5D_Re:_=5Bobm-l=5D_fatoriais_n=E3o_inteiros?= Date: Thu, 19 Sep 2002 08:00:54 -0300 Organization: Steiner Consultoria LTDA Message-ID: <003201c25fcb$d3ae71e0$9865fea9@computer> MIME-Version: 1.0 Content-Type: text/plain; charset="iso-8859-1" X-Priority: 3 (Normal) X-MSMail-Priority: Normal X-Mailer: Microsoft Outlook, Build 10.0.2627 X-MimeOLE: Produced By Microsoft MimeOLE V6.00.2600.0000 In-Reply-To: <01c25f90$34bc9a20$df29d6c8@franklin> Importance: Normal X-Virus-Scanned: by AMaViS new-20020517 Content-Transfer-Encoding: 8bit X-MIME-Autoconverted: from quoted-printable to 8bit by sucuri.mat.puc-rio.br id HAA01720 Sender: owner-obm-l@sucuri.mat.puc-rio.br Precedence: bulk Reply-To: obm-l@mat.puc-rio.br >Oi pessoal ! > >Qual o sentido de n! se n não pertence aos inteiros, seja n positivo ou negativo? (esses fatoriais >podem ser calculados por uma calculadora científica como a que vem no computador). Como é feito >o cálculo de n! nessas condições? > >Se alguém puder me esclarecer eu agradeço > >André T. Olá, Está meio tarde, posso estar fazendo confusão, mas acho que você está se referindo à função Gama, que pode ser interpretada como uma generalização do fatorial. Depois eu confiro a definição de função e gama e vejo se tem sentido o que estou falando. Se não me engano, gama(n+1)=n!, quando n é natural, mas a função gama também é definida para números reais. Alguém interfira se eu estiver falando besteira, por favor. Até mais, Franklin É realmente a função gama (muito usada em probabilidades), a qual é definida para x>=1 pela seguinte integral Gama(x) = Int (0 a oo) e^(-t) t^(x-1) dx. Para qualquer x do domínio, podemos mostra que Gama(x+1) = (x+1)gama(x) Por indução, concluímos que, se n for natural, então Gama(n) = n! Daí se dizer que esta função é uma generalização do fatorial de um número inteiro. Um abraço Artur ========================================================================= Instruções para entrar na lista, sair da lista e usar a lista em http://www.mat.puc-rio.br/~nicolau/olimp/obm-l.html O administrador desta lista é ========================================================================= From owner-obm-l@sucuri.mat.puc-rio.br Thu Sep 19 08:16:46 2002 Return-Path: Received: (from majordom@localhost) by sucuri.mat.puc-rio.br (8.9.3/8.9.3) id IAA02132 for obm-l-MTTP; Thu, 19 Sep 2002 08:15:23 -0300 Received: from artemis.opendf.com.br (artemis.opengate.com.br [200.181.71.15]) by sucuri.mat.puc-rio.br (8.9.3/8.9.3) with ESMTP id IAA02128 for ; Thu, 19 Sep 2002 08:15:20 -0300 Received: from localhost (localhost [127.0.0.1]) by artemis.opendf.com.br (Postfix) with ESMTP id 77CAB1C986 for ; Thu, 19 Sep 2002 08:16:37 -0300 (BRT) Received: from computer (200-181-88-253-bsace7001.dsl.telebrasilia.net.br [200.181.88.253]) by artemis.opendf.com.br (Postfix) with ESMTP id 81C521BDFA for ; Thu, 19 Sep 2002 08:16:34 -0300 (BRT) From: "Artur Costa Steiner" To: Subject: [obm-l] =?iso-8859-1?Q?RE:_=5Bobm-l=5D_Re:_=5Bobm-l=5D_fatoriais_n=E3o_inteiros?= Date: Thu, 19 Sep 2002 08:16:51 -0300 Organization: Steiner Consultoria LTDA Message-ID: <000701c25fce$0da5da30$9865fea9@computer> MIME-Version: 1.0 Content-Type: text/plain; charset="iso-8859-1" X-Priority: 3 (Normal) X-MSMail-Priority: Normal X-Mailer: Microsoft Outlook, Build 10.0.2627 X-MimeOLE: Produced By Microsoft MimeOLE V6.00.2600.0000 Importance: Normal In-Reply-To: <01c25f90$34bc9a20$df29d6c8@franklin> X-Virus-Scanned: by AMaViS new-20020517 Content-Transfer-Encoding: 8bit X-MIME-Autoconverted: from quoted-printable to 8bit by sucuri.mat.puc-rio.br id IAA02129 Sender: owner-obm-l@sucuri.mat.puc-rio.br Precedence: bulk Reply-To: obm-l@mat.puc-rio.br Oi, retificando um erro de digitação no meu outro email, a função gama é dada por Gama(x) = Int (de 0 a oo) e^(-t) t^(x-1)dt (e não dx) A integral é em t e depende de x Artur ========================================================================= Instruções para entrar na lista, sair da lista e usar a lista em http://www.mat.puc-rio.br/~nicolau/olimp/obm-l.html O administrador desta lista é ========================================================================= From owner-obm-l@sucuri.mat.puc-rio.br Thu Sep 19 09:56:11 2002 Return-Path: Received: (from majordom@localhost) by sucuri.mat.puc-rio.br (8.9.3/8.9.3) id JAA03706 for obm-l-MTTP; Thu, 19 Sep 2002 09:54:00 -0300 Received: (from nicolau@localhost) by sucuri.mat.puc-rio.br (8.9.3/8.9.3) id JAA03701 for obm-l@mat.puc-rio.br; Thu, 19 Sep 2002 09:54:00 -0300 Date: Thu, 19 Sep 2002 09:53:59 -0300 From: "Nicolau C. Saldanha" To: obm-l@mat.puc-rio.br Subject: Re: [obm-l] Conjuntos Fechados Message-ID: <20020919095359.F2646@sucuri.mat.puc-rio.br> References: <20020917164359.77901.qmail@web10208.mail.yahoo.com> Mime-Version: 1.0 Content-Type: text/plain; charset=iso-8859-1 Content-Disposition: inline Content-Transfer-Encoding: 8bit User-Agent: Mutt/1.2.5i In-Reply-To: <20020917164359.77901.qmail@web10208.mail.yahoo.com>; from hnaves@yahoo.com on Tue, Sep 17, 2002 at 01:43:59PM -0300 Sender: owner-obm-l@sucuri.mat.puc-rio.br Precedence: bulk Reply-To: obm-l@mat.puc-rio.br On Tue, Sep 17, 2002 at 01:43:59PM -0300, Humberto Naves wrote: > Oi, > > Li num livro de análise, que o conjunto dos irracionais não pode ser escrito > como uma união enumerável de fechados. Como demostrar esse fato? Isto é essencialmente o teorema de Baire. Uma união enumerável de fechados de interior vazio ainda tem interior vazio. Para provar isso vá tomando intervalos compactos encaixados disjuntos de cada fechado da seqüência. []s, N. ========================================================================= Instruções para entrar na lista, sair da lista e usar a lista em http://www.mat.puc-rio.br/~nicolau/olimp/obm-l.html O administrador desta lista é ========================================================================= From owner-obm-l@sucuri.mat.puc-rio.br Thu Sep 19 10:33:48 2002 Return-Path: Received: (from majordom@localhost) by sucuri.mat.puc-rio.br (8.9.3/8.9.3) id KAA04392 for obm-l-MTTP; Thu, 19 Sep 2002 10:32:09 -0300 Received: from hotmail.com (oe11.pav0.hotmail.com [64.4.32.91]) by sucuri.mat.puc-rio.br (8.9.3/8.9.3) with ESMTP id KAA04386 for ; Thu, 19 Sep 2002 10:32:06 -0300 Received: from mail pickup service by hotmail.com with Microsoft SMTPSVC; Thu, 19 Sep 2002 06:32:29 -0700 X-Originating-IP: [200.217.179.195] From: "e isso mesmo" To: Subject: [obm-l] =?iso-8859-1?Q?Ser=E1_que_ningu=E9m_me_ajuda=3F=3F=3F?= Date: Thu, 19 Sep 2002 10:32:10 -0300 MIME-Version: 1.0 X-Mailer: MSN Explorer 7.00.0021.1900 Content-Type: multipart/alternative; boundary="----=_NextPart_001_0000_01C25FC7.CF88E0E0" Message-ID: X-OriginalArrivalTime: 19 Sep 2002 13:32:29.0834 (UTC) FILETIME=[005F96A0:01C25FE1] Sender: owner-obm-l@sucuri.mat.puc-rio.br Precedence: bulk Reply-To: obm-l@mat.puc-rio.br ------=_NextPart_001_0000_01C25FC7.CF88E0E0 Content-Type: text/plain; charset="iso-8859-1" Content-Transfer-Encoding: quoted-printable Companheiros continuo esperando ajuda de algu=E8m.... "A teoria do Biorritmo diz que os estados f=EDsico, mental e emocional de= uma pessoa oscilam periodicamente, a partir do dia do nascimento, em cic= los de 23 dias, 29 dias e 33 dias, respectivamente. Dado que os dias mais= positivos dos ciclos f=EDsico, mental e emocional s=E3o, respectivamente= , o sexto, o s=E9timo e o oitavo de cada ciclo, nos primeiros dez anos de= vida de uma pessoa, quantas vezes os tr=EAs ciclos est=E3o simultaneamen= te no ponto m=E1ximo?" Tchau! Aproveite melhor a Web. Fa=E7a o download GR=C1TIS do MSN Explorer : htt= p://explorer.msn.com.br/intl.asp#po ------=_NextPart_001_0000_01C25FC7.CF88E0E0 Content-Type: text/html; charset="iso-8859-1" Content-Transfer-Encoding: quoted-printable
 
<= DIV>Companheiros continuo esperando ajuda de algu=E8m....
&nbs= p;

"A teoria do Biorritmo= diz que os estados f=EDsico, mental e emocional de uma pessoa oscilam pe= riodicamente, a partir do dia do nascimento, em ciclos de 23 dias, 29 dia= s e 33 dias, respectivamente. Dado que os dias mais positivos dos ciclos = f=EDsico, mental e emocional s=E3o, respectivamente, o sexto, o s=E9timo = e o oitavo de cada ciclo, nos primeiros dez anos de vida de uma pessoa, q= uantas vezes os tr=EAs ciclos est=E3o simultaneamente no ponto m=E1ximo?"=

&nbs= p;

= Tchau!

=

 

=

Aproveite melhor a Web. Fa=E7a o download GR=C1= TIS do MSN Explorer : = http://explorer.msn.com.br/intl.asp#po

------=_NextPart_001_0000_01C25FC7.CF88E0E0-- ========================================================================= Instruções para entrar na lista, sair da lista e usar a lista em http://www.mat.puc-rio.br/~nicolau/olimp/obm-l.html O administrador desta lista é ========================================================================= From owner-obm-l@sucuri.mat.puc-rio.br Thu Sep 19 12:59:18 2002 Return-Path: Received: (from majordom@localhost) by sucuri.mat.puc-rio.br (8.9.3/8.9.3) id MAA06424 for obm-l-MTTP; Thu, 19 Sep 2002 12:55:10 -0300 Received: from hotmail.com (f145.law9.hotmail.com [64.4.9.145]) by sucuri.mat.puc-rio.br (8.9.3/8.9.3) with ESMTP id MAA06420 for ; Thu, 19 Sep 2002 12:55:07 -0300 Received: from mail pickup service by hotmail.com with Microsoft SMTPSVC; Thu, 19 Sep 2002 08:56:29 -0700 Received: from 143.107.45.30 by lw9fd.law9.hotmail.msn.com with HTTP; Thu, 19 Sep 2002 15:56:28 GMT X-Originating-IP: [143.107.45.30] From: "Rogerio Fajardo" To: obm-l@mat.puc-rio.br Subject: Re: [obm-l] Axioma da Escolha Date: Thu, 19 Sep 2002 15:56:28 +0000 Mime-Version: 1.0 Content-Type: text/plain; format=flowed Message-ID: X-OriginalArrivalTime: 19 Sep 2002 15:56:29.0388 (UTC) FILETIME=[1DF2E8C0:01C25FF5] Sender: owner-obm-l@sucuri.mat.puc-rio.br Precedence: bulk Reply-To: obm-l@mat.puc-rio.br Na verdade, realizar uma escolha arbitrária em um conjunto não requer o axioma da escolha. Se voce sabe que um conjunto é não-vazio, voce sabe que existe um elemento x nele. Pelo axioma do par, existe o conjunto {x,x}={x}. Agora, se voce tem um conjunto infinito, para provar que existe um subconjunto enumerável, voce precisa do axioma da escolha. Em geral, sempre que se faz infinitas escolhas arbitrárias usa-se o axioma da escolha. Mas para entender bem quando se usa o axioma da escolha, acho que é inevitável discutir os outros axiomas de Teoria dos Conjuntos. Mas a idéia é que escolhas finitas voce pode fazer "na mão" (o conjunto é não vazio, logo tem um elemento, logo eu posso "pegar" esse elemento e fazer o que quiser com ele). Para infinitas escolhas, eu preciso de uma ferramenta maior para agrupar esses infinitos elementos tirados de infinitos conjuntos não-vazios. Acredito que todas essas polêmicas causadas aqui na lista sobre o uso do axioma da escolha não é muito diferente do que aconteceu entre os grandes matemáticos quando Zermelo criou a axiomática da Teoria dos Conjuntos. >From: "498 - Artur Costa Steiner" >Reply-To: obm-l@mat.puc-rio.br >To: obm-l@mat.puc-rio.br >Subject: Re: [obm-l] Axioma da Escolha >Date: Wed, 18 Sep 2002 15:32:31 -0300 (BRT) > > > > > Realmente, parece que eu gerei mais polêmica do que esperava. Vou >indicar um > > site que explica muito bem o Axioma da Escolha - seu enuniado, >aplicações e > > discussões filosóficas a respeito de seu uso. O site é: > > http://math.vanderbilt.edu/~schectex/ccc/choice.html > >Não tem problema, esta discussão toda foi excelente. Antes dela eu não >havia prestado muito atenção no axioma, parecia estar na "massa do meu >sangue". O artigo do site é excelente. > >Há um ponto a respeito do qual estou ainda um tanto intrigado: realizar >infinitas escolhas arbitrárias em infinitos conjuntos requer o axioma. >Mas, realizar infinitas escolhas arbitrárias em UM ÚNICO conjunto não >requer o axioma. Quando se trata de um único conjunto, é sempre >possível encontrar um afunção de escolha bem definida? > >Obrigado >Artur >========================================================================= >Instruções para entrar na lista, sair da lista e usar a lista em >http://www.mat.puc-rio.br/~nicolau/olimp/obm-l.html >O administrador desta lista é >========================================================================= _________________________________________________________________ Send and receive Hotmail on your mobile device: http://mobile.msn.com ========================================================================= Instruções para entrar na lista, sair da lista e usar a lista em http://www.mat.puc-rio.br/~nicolau/olimp/obm-l.html O administrador desta lista é ========================================================================= From owner-obm-l@sucuri.mat.puc-rio.br Thu Sep 19 16:44:31 2002 Return-Path: Received: (from majordom@localhost) by sucuri.mat.puc-rio.br (8.9.3/8.9.3) id QAA09859 for obm-l-MTTP; Thu, 19 Sep 2002 16:42:21 -0300 Received: from traven10.pub1 (traven10.uol.com.br [200.221.4.45]) by sucuri.mat.puc-rio.br (8.9.3/8.9.3) with ESMTP id QAA09854 for ; Thu, 19 Sep 2002 16:42:18 -0300 Received: from u2z7z2 ([200.158.145.62]) by traven10.pub1 (8.9.1/8.9.1) with ESMTP id QAA14436 for ; Thu, 19 Sep 2002 16:42:57 -0300 (BRT) Message-ID: <001701c26015$03555960$2101a8c0@u2z7z2> From: "Wagner" To: References: Subject: [obm-l] =?iso-8859-1?Q?Re:_=5Bobm-l=5D_Ser=E1_que_ningu=E9m_me_ajuda=3F=3F=3F?= Date: Thu, 19 Sep 2002 16:44:47 -0300 Organization: Wagner MIME-Version: 1.0 Content-Type: multipart/alternative; boundary="----=_NextPart_000_0014_01C25FFB.DD7A3940" X-Priority: 3 X-MSMail-Priority: Normal X-Mailer: Microsoft Outlook Express 5.50.4133.2400 X-MimeOLE: Produced By Microsoft MimeOLE V5.50.4133.2400 Sender: owner-obm-l@sucuri.mat.puc-rio.br Precedence: bulk Reply-To: obm-l@mat.puc-rio.br This is a multi-part message in MIME format. ------=_NextPart_000_0014_01C25FFB.DD7A3940 Content-Type: text/plain; charset="iso-8859-1" Content-Transfer-Encoding: quoted-printable Oi pessoal Em 10 anos existem ou 3652 ou 3653 dias (dependendo do ano inicial) Chamando o ciclo f=EDsico (de 23 dias) de A, o mental (de 29 dias) de B = , o emocional (de 33 dias) de C e chamando a n-=E9sima vez que os ciclos = se encontram no m=E1ximo de m(n). Entre m(1) e m(2) existem 23.29.33 dias =3D 22011 dias (porqu=EA 23,29 e = 33 s=E3o primos entre si). Logo m(2) s=F3 pode acontecer depois dos 10 = anos. Ent=E3o basta verificar se m(1) acontece antes dos 10 anos. Isso = se torna complicado porqu=EA o n=FAmero de ciclos =E9 obrigatoriamente = natural, mas se voc=EA quer uma estimativa, considerando os encontros = como aleat=F3rios, a chance de m(1) estar antes dos 10 anos =E9 = 3652,25/22011 que =E9 uma chance de aproximadamente 16,5% Andr=E9 T. ----- Original Message -----=20 From: e isso mesmo=20 To: obm-l@mat.puc-rio.br=20 Sent: Thursday, September 19, 2002 10:32 AM Subject: [obm-l] Ser=E1 que ningu=E9m me ajuda??? Companheiros continuo esperando ajuda de algu=E8m.... "A teoria do Biorritmo diz que os estados f=EDsico, mental e emocional = de uma pessoa oscilam periodicamente, a partir do dia do nascimento, em = ciclos de 23 dias, 29 dias e 33 dias, respectivamente. Dado que os dias = mais positivos dos ciclos f=EDsico, mental e emocional s=E3o, = respectivamente, o sexto, o s=E9timo e o oitavo de cada ciclo, nos = primeiros dez anos de vida de uma pessoa, quantas vezes os tr=EAs ciclos = est=E3o simultaneamente no ponto m=E1ximo?" Tchau! =20 -------------------------------------------------------------------------= ----- Aproveite melhor a Web. Fa=E7a o download GR=C1TIS do MSN Explorer : = http://explorer.msn.com.br/intl.asp#po ------=_NextPart_000_0014_01C25FFB.DD7A3940 Content-Type: text/html; charset="iso-8859-1" Content-Transfer-Encoding: quoted-printable
Oi pessoal
 
Em 10 anos existem ou 3652 ou 3653 dias = (dependendo do ano=20 inicial)
Chamando o ciclo f=EDsico (de 23 dias) de = A, o=20 mental (de 29 dias) de B , o emocional (de 33 dias) de C=20 e chamando a n-=E9sima vez que os ciclos se encontram no m=E1ximo = de=20 m(n).
Entre m(1) e m(2) existem 23.29.33 dias =3D = 22011 dias=20 (porqu=EA 23,29 e 33 s=E3o primos entre si). Logo m(2) s=F3 pode = acontecer depois dos=20 10 anos. Ent=E3o basta verificar se m(1) acontece antes dos 10 anos. = Isso se torna=20 complicado porqu=EA o n=FAmero de ciclos =E9 obrigatoriamente natural, = mas se voc=EA=20 quer uma estimativa, considerando os encontros como aleat=F3rios, a = chance de m(1)=20 estar antes dos 10 anos =E9 3652,25/22011 que =E9 uma chance de = aproximadamente=20 16,5%
 
Andr=E9 T.
 
 
----- Original Message -----
From:=20 e isso = mesmo
Sent: Thursday, September 19, = 2002 10:32=20 AM
Subject: [obm-l] Ser=E1 que = ningu=E9m me=20 ajuda???

 
Companheiros continuo esperando ajuda de algu=E8m....
 

"A teoria do Biorritmo diz que os = estados=20 f=EDsico, mental e emocional de uma pessoa oscilam periodicamente, a = partir do=20 dia do nascimento, em ciclos de 23 dias, 29 dias e 33 dias, = respectivamente.=20 Dado que os dias mais positivos dos ciclos f=EDsico, mental e = emocional s=E3o,=20 respectivamente, o sexto, o s=E9timo e o oitavo de cada ciclo, nos = primeiros dez=20 anos de vida de uma pessoa, quantas vezes os tr=EAs ciclos est=E3o = simultaneamente=20 no ponto m=E1ximo?"

 

Tchau!

 



Aproveite melhor a Web. Fa=E7a o download GR=C1TIS do MSN Explorer : = http://explorer.msn.com.b= r/intl.asp#po

------=_NextPart_000_0014_01C25FFB.DD7A3940-- ========================================================================= Instruções para entrar na lista, sair da lista e usar a lista em http://www.mat.puc-rio.br/~nicolau/olimp/obm-l.html O administrador desta lista é ========================================================================= From owner-obm-l@sucuri.mat.puc-rio.br Thu Sep 19 16:58:20 2002 Return-Path: Received: (from majordom@localhost) by sucuri.mat.puc-rio.br (8.9.3/8.9.3) id QAA10105 for obm-l-MTTP; Thu, 19 Sep 2002 16:57:50 -0300 Received: from traven10.pub1 (traven10.uol.com.br [200.221.4.45]) by sucuri.mat.puc-rio.br (8.9.3/8.9.3) with ESMTP id QAA10082 for ; Thu, 19 Sep 2002 16:57:47 -0300 Received: from u2z7z2 ([200.158.145.62]) by traven10.pub1 (8.9.1/8.9.1) with ESMTP id QAA09155 for ; Thu, 19 Sep 2002 16:58:35 -0300 (BRT) Message-ID: <002201c26017$35833220$2101a8c0@u2z7z2> From: "Wagner" To: Subject: [obm-l] =?iso-8859-1?Q?Quest=E3o_de_geometria?= Date: Thu, 19 Sep 2002 17:00:31 -0300 Organization: Wagner MIME-Version: 1.0 Content-Type: multipart/alternative; boundary="----=_NextPart_000_001F_01C25FFE.100CC740" X-Priority: 3 X-MSMail-Priority: Normal X-Mailer: Microsoft Outlook Express 5.50.4133.2400 X-MimeOLE: Produced By Microsoft MimeOLE V5.50.4133.2400 Sender: owner-obm-l@sucuri.mat.puc-rio.br Precedence: bulk Reply-To: obm-l@mat.puc-rio.br This is a multi-part message in MIME format. ------=_NextPart_000_001F_01C25FFE.100CC740 Content-Type: text/plain; charset="iso-8859-1" Content-Transfer-Encoding: quoted-printable Oi pessoal ! Essa =E9 uma pergunta de geometria um pouco chatinha, de=EAm uma olhada: Em um c=EDrculo existe um tri=E2ngulo ABC ret=E2ngulo inscrito nele. = Esse tri=E2ngulo possui uma circunfer=EAncia circunscrita nele. Essa = circunfer=EAncia possui um tri=E2ngulo DEF inscrito nela ,tal que AB =E9 = paralelo =E0 DE, AC =E9 paralelo =E0 DF e BC =E9 paralelo =E0 EF. Esse = tri=E2ngulo tamb=E9m possui uma circ=FBnfer=EAncia circunscrita nele, = tal que o padr=E3o descrito acima se repete. Seja o tri=E2ngulo XYZ = limite desse padr=E3o, tal que XY =E9 paralelo =E0 DE, XZ =E9 paralelo = =E0 DF e YZ =E9 paralelo =E0 EF. Logo, quanto vale a soma das = dist=E2ncias AX, AY e AZ em func=E3o de B=C2C e do raio da = circunfer=EAncia maior? Dado: AB e BC s=E3o perpendiculares Se algu=E9m tiver no=E7=E3o de como resolver essa pergunta j=E1 = agrade=E7o. Andr=E9 T. ------=_NextPart_000_001F_01C25FFE.100CC740 Content-Type: text/html; charset="iso-8859-1" Content-Transfer-Encoding: quoted-printable
Oi pessoal !
 
Essa =E9 uma pergunta de geometria um = pouco chatinha,=20 de=EAm uma olhada:
 
    Em um c=EDrculo = existe um=20 tri=E2ngulo ABC ret=E2ngulo inscrito nele. Esse tri=E2ngulo possui uma = circunfer=EAncia=20 circunscrita nele. Essa circunfer=EAncia possui um tri=E2ngulo DEF = inscrito nela=20 ,tal que AB =E9 paralelo =E0 DE, AC =E9 paralelo =E0 DF e BC =E9 = paralelo =E0 EF. Esse=20 tri=E2ngulo  tamb=E9m possui uma circ=FBnfer=EAncia circunscrita = nele, tal que o=20 padr=E3o descrito acima se repete. Seja o tri=E2ngulo XYZ limite desse = padr=E3o, tal=20 que XY =E9 paralelo =E0 DE, XZ =E9 paralelo =E0 DF e YZ =E9 paralelo =E0 = EF. Logo, quanto=20 vale a soma das dist=E2ncias AX, AY e AZ em func=E3o de B=C2C e do raio = da=20 circunfer=EAncia maior?
 
Dado: AB e BC s=E3o = perpendiculares
 
Se algu=E9m tiver no=E7=E3o de como = resolver essa=20 pergunta j=E1 agrade=E7o.
 
Andr=E9 T.
 
------=_NextPart_000_001F_01C25FFE.100CC740-- ========================================================================= Instruções para entrar na lista, sair da lista e usar a lista em http://www.mat.puc-rio.br/~nicolau/olimp/obm-l.html O administrador desta lista é ========================================================================= From owner-obm-l@sucuri.mat.puc-rio.br Thu Sep 19 21:48:32 2002 Return-Path: Received: (from majordom@localhost) by sucuri.mat.puc-rio.br (8.9.3/8.9.3) id VAA13519 for obm-l-MTTP; Thu, 19 Sep 2002 21:47:34 -0300 Received: from hotmail.com (oe12.pav0.hotmail.com [64.4.32.92]) by sucuri.mat.puc-rio.br (8.9.3/8.9.3) with ESMTP id VAA13515 for ; Thu, 19 Sep 2002 21:47:31 -0300 Received: from mail pickup service by hotmail.com with Microsoft SMTPSVC; Thu, 19 Sep 2002 17:49:02 -0700 X-Originating-IP: [200.217.188.1] From: "e isso mesmo" To: Subject: [obm-l] =?iso-8859-1?Q?Re:_=5Bobm-l=5DRe:_=5Bobm-l=5D_Ser=E1_que_ningu=E9m_me_ajuda?= =?iso-8859-1?Q?=3F=3F=3F?= Date: Thu, 19 Sep 2002 21:48:51 -0300 MIME-Version: 1.0 X-Mailer: MSN Explorer 6.10.0016.1624 Content-Type: multipart/alternative; boundary="----=_NextPart_001_0000_01C26026.578430E0" Message-ID: X-OriginalArrivalTime: 20 Sep 2002 00:49:02.0055 (UTC) FILETIME=[8338F770:01C2603F] Sender: owner-obm-l@sucuri.mat.puc-rio.br Precedence: bulk Reply-To: obm-l@mat.puc-rio.br ------=_NextPart_001_0000_01C26026.578430E0 Content-Type: text/plain; charset="iso-8859-1" Content-Transfer-Encoding: quoted-printable Ser=E1 que n=E3o ficava melhor escrever as congru=EAncias. O problema =E9= que n=E3o sei como escrever os m=F3dulos. Tchau -----Mensagem Original----- De: Wagner Enviado: quinta-feira, 19 de setembro de 2002 17:55 Para: obm-l@mat.puc-rio.br Assunto: [obm-l]Re: [obm-l] Ser=E1 que ningu=E9m me ajuda??? Oi pessoal Em 10 anos existem ou 3652 ou 3653 dias (dependendo do ano inicial) Chamando o ciclo f=EDsico (de 23 dias) de A, o mental (de 29 dias) de B ,= o emocional (de 33 dias) de C e chamando a n-=E9sima vez que os ciclos s= e encontram no m=E1ximo de m(n). Entre m(1) e m(2) existem 23.29.33 dias =3D 22011 dias (porqu=EA 23,29 e = 33 s=E3o primos entre si). Logo m(2) s=F3 pode acontecer depois dos 10 an= os. Ent=E3o basta verificar se m(1) acontece antes dos 10 anos. Isso se t= orna complicado porqu=EA o n=FAmero de ciclos =E9 obrigatoriamente natura= l, mas se voc=EA quer uma estimativa, considerando os encontros como alea= t=F3rios, a chance de m(1) estar antes dos 10 anos =E9 3652,25/22011 que = =E9 uma chance de aproximadamente 16,5% Andr=E9 T. ----- Original Message ----- =20 From: e isso mesmo =20 To: obm-l@mat.puc-rio.br =20 Sent: Thursday, September 19, 2002 10:32 AM Subject: [obm-l] Ser=E1 que ningu=E9m me ajuda??? Companheiros continuo esperando ajuda de algu=E8m.... "A teoria do Biorritmo diz que os estados f=EDsico, mental e emocional de= uma pessoa oscilam periodicamente, a partir do dia do nascimento, em cic= los de 23 dias, 29 dias e 33 dias, respectivamente. Dado que os dias mais= positivos dos ciclos f=EDsico, mental e emocional s=E3o, respectivamente= , o sexto, o s=E9timo e o oitavo de cada ciclo, nos primeiros dez anos de= vida de uma pessoa, quantas vezes os tr=EAs ciclos est=E3o simultaneamen= te no ponto m=E1ximo?" Tchau! =20 Aproveite melhor a Web. Fa=E7a o download GR=C1TIS do MSN Explorer : http= ://explorer.msn.com.br/intl.asp#poAproveite melhor a Web. Fa=E7a o downlo= ad GR=C1TIS do MSN Explorer : http://explorer.msn.com.br/intl.asp#po ------=_NextPart_001_0000_01C26026.578430E0 Content-Type: text/html; charset="iso-8859-1" Content-Transfer-Encoding: quoted-printable
Ser=E1 que n=E3o ficava melhor escrever a= s congru=EAncias. O  problema =E9 que n=E3o sei como escrever os m=F3= dulos.
 
Tchau
 
-----Mensagem Original-----
De: Wagner
Enviado: quinta-feira, 19 de setembro de 2002 1= 7:55
Para: obm-l@mat.puc-rio= .br
Assunto: [obm-l]Re: [obm= -l] Ser=E1 que ningu=E9m me ajuda???
 
Oi pessoal
 = ;
Em 10 anos existem ou 3652 ou 3653 dias (= dependendo do ano inicial)
Chamando = o ciclo f=EDsico (de 23 dias) de A, o mental (de 29 dias) = de B , o emocional (de 33 dias) de C e chamando a n-=E9sima vez= que os ciclos se encontram no m=E1ximo de m(n).
Entre m(1) e m(2) existem 23.29.33 dias =3D 22011 dias (porq= u=EA 23,29 e 33 s=E3o primos entre si). Logo m(2) s=F3 pode acontecer dep= ois dos 10 anos. Ent=E3o basta verificar se m(1) acontece antes dos 10 an= os. Isso se torna complicado porqu=EA o n=FAmero de ciclos =E9 obrigatori= amente natural, mas se voc=EA quer uma estimativa, considerando os encont= ros como aleat=F3rios, a chance de m(1) estar antes dos 10 anos =E9 3652,= 25/22011 que =E9 uma chance de aproximadamente 16,5%
 
Andr=E9 T.
 
 
----- Original Messag= e -----
Sent: Thursday, September 19, 2002 10:32 AM
Subject: [obm-l] Ser=E1 que ningu=E9m me ajuda???

 
Companheiros continuo esperando ajuda = de algu=E8m....
 

"A teoria do Biorritmo diz que os estados f=EDsico, mental e emoci= onal de uma pessoa oscilam periodicamente, a partir do dia do nascimento,= em ciclos de 23 dias, 29 dias e 33 dias, respectivamente. Dado que os di= as mais positivos dos ciclos f=EDsico, mental e emocional s=E3o, respecti= vamente, o sexto, o s=E9timo e o oitavo de cada ciclo, nos primeiros dez = anos de vida de uma pessoa, quantas vezes os tr=EAs ciclos est=E3o simult= aneamente no ponto m=E1ximo?"

 

T= chau!=

 



Aproveite melhor a Web. Fa=E7= a o download GR=C1TIS do MSN Explorer : http://explorer.msn.com.br/intl.asp#po

<= /BLOCKQUOTE>


Aproveite melh= or a Web. Fa=E7a o download GR=C1TIS do MSN Explorer : http://explorer.msn.com.br/intl.asp#po

------=_NextPart_001_0000_01C26026.578430E0-- ========================================================================= Instruções para entrar na lista, sair da lista e usar a lista em http://www.mat.puc-rio.br/~nicolau/olimp/obm-l.html O administrador desta lista é ========================================================================= From owner-obm-l@sucuri.mat.puc-rio.br Thu Sep 19 22:48:15 2002 Return-Path: Received: (from majordom@localhost) by sucuri.mat.puc-rio.br (8.9.3/8.9.3) id WAA14390 for obm-l-MTTP; Thu, 19 Sep 2002 22:47:58 -0300 Received: from gorgo.centroin.com.br (gorgo.centroin.com.br [200.225.63.128]) by sucuri.mat.puc-rio.br (8.9.3/8.9.3) with ESMTP id WAA14385 for ; Thu, 19 Sep 2002 22:47:56 -0300 Received: from centroin.com.br (du160c.rjo.centroin.com.br [200.225.58.160]) (authenticated bits=0) by gorgo.centroin.com.br (8.12.2/8.12.1) with ESMTP id g8K1nm7p007284 for ; Thu, 19 Sep 2002 22:49:49 -0300 (BRT) Message-ID: <3D8A7F4F.9090307@centroin.com.br> Date: Thu, 19 Sep 2002 22:52:15 -0300 From: Augusto =?us-ascii?Q?C=E9sar?= Morgado User-Agent: Mozilla/5.0 (Windows; U; Win98; en-US; rv:0.9.4.1) Gecko/20020508 Netscape6/6.2.3 X-Accept-Language: en-us MIME-Version: 1.0 To: obm-l@mat.puc-rio.br Subject: Re: [obm-l] =?us-ascii?Q?Ser=E1?= que =?us-ascii?Q?ningu=E9m?= me ajuda??? References: Content-Type: multipart/alternative; boundary="------------020904010502000706060405" Sender: owner-obm-l@sucuri.mat.puc-rio.br Precedence: bulk Reply-To: obm-l@mat.puc-rio.br --------------020904010502000706060405 Content-Type: text/plain; charset=us-ascii; format=flowed Content-Transfer-Encoding: 7bit Contando o dia do nascimento como dia 0, os dias de coincide^ncia sa~o da forma 6+23r = 7+29s = 8+33t. Temos umas diofantinas para resolver. A primeira delas e' 23r-29s=1 Resolvendo-a obtemos r = -5 +29k s = 4 + 23k, k inteiro. Substituindo, 23*29k - 33t = 117 Resolvendo-a, k = 33n +12 t = 667n + 239. Portanto as soluc,o~es inteiras do sistema la' de cima sa~o r = 957n + 343 s = 759n + 280 t = 667n + 239 Em particular (so' calculei isso tudo para que possam conferir e detectar eventuais erros de conta) os ciclos coincidem nos dias da forma 6 + 23r = 22 011 * n + 7 895 A primeira coincide^ncia se da' no dia 7 895 (cerca de 22 anos apo's o nascimento). e isso mesmo wrote: > Companheiros continuo esperando ajuda de algue`m.... > > "A teoria do Biorritmo diz que os estados fi'sico, mental e emocional > de uma pessoa oscilam periodicamente, a partir do dia do nascimento, > em ciclos de 23 dias, 29 dias e 33 dias, respectivamente. Dado que os > dias mais positivos dos ciclos fi'sico, mental e emocional sa~o, > respectivamente, o sexto, o se'timo e o oitavo de cada ciclo, nos > primeiros dez anos de vida de uma pessoa, quantas vezes os tre^s > ciclos esta~o simultaneamente no ponto ma'ximo?" > > Tchau! > > > ------------------------------------------------------------------------ > Aproveite melhor a Web. Fac,a o download GRA'TIS do MSN Explorer : > http://explorer.msn.com.br/intl.asp#po --------------020904010502000706060405 Content-Type: text/html; charset=us-ascii Content-Transfer-Encoding: 7bit Contando o dia do nascimento como dia 0,  os dias de coincidência são da forma 6+23r = 7+29s = 8+33t.
Temos umas diofantinas para resolver.
A primeira delas é
23r-29s=1
Resolvendo-a obtemos
r = -5 +29k
s = 4 + 23k, k inteiro.
Substituindo,  23*29k - 33t = 117
Resolvendo-a,
k = 33n +12
t = 667n + 239.
Portanto as soluções inteiras do sistema lá de cima são
r = 957n + 343
s = 759n + 280
t = 667n + 239

Em particular (só calculei isso tudo para que possam conferir e detectar eventuais erros de conta) os ciclos coincidem nos dias da forma 6 + 23r = 22 011 * n + 7 895
A primeira coincidência se dá no dia  7 895 (cerca de 22 anos após o nascimento).






e isso mesmo wrote:
 
Companheiros continuo esperando ajuda de alguèm....
 

"A teoria do Biorritmo diz que os estados físico, mental e emocional de uma pessoa oscilam periodicamente, a partir do dia do nascimento, em ciclos de 23 dias, 29 dias e 33 dias, respectivamente. Dado que os dias mais positivos dos ciclos físico, mental e emocional são, respectivamente, o sexto, o sétimo e o oitavo de cada ciclo, nos primeiros dez anos de vida de uma pessoa, quantas vezes os três ciclos estão simultaneamente no ponto máximo?"

 

Tchau!

 



Aproveite melhor a Web. Faça o download GRÁTIS do MSN Explorer :
http://explorer.msn.com.br/intl.asp#po


--------------020904010502000706060405-- ========================================================================= Instruções para entrar na lista, sair da lista e usar a lista em http://www.mat.puc-rio.br/~nicolau/olimp/obm-l.html O administrador desta lista é ========================================================================= From owner-obm-l@sucuri.mat.puc-rio.br Thu Sep 19 22:53:40 2002 Return-Path: Received: (from majordom@localhost) by sucuri.mat.puc-rio.br (8.9.3/8.9.3) id WAA14505 for obm-l-MTTP; Thu, 19 Sep 2002 22:53:22 -0300 Received: from gorgo.centroin.com.br (gorgo.centroin.com.br [200.225.63.128]) by sucuri.mat.puc-rio.br (8.9.3/8.9.3) with ESMTP id WAA14501 for ; Thu, 19 Sep 2002 22:53:19 -0300 Received: from centroin.com.br (du160c.rjo.centroin.com.br [200.225.58.160]) (authenticated bits=0) by gorgo.centroin.com.br (8.12.2/8.12.1) with ESMTP id g8K1tD7p007662 for ; Thu, 19 Sep 2002 22:55:13 -0300 (BRT) Message-ID: <3D8A80A4.3030100@centroin.com.br> Date: Thu, 19 Sep 2002 22:57:56 -0300 From: Augusto =?ISO-8859-1?Q?C=E9sar?= Morgado User-Agent: Mozilla/5.0 (Windows; U; Win98; en-US; rv:0.9.4.1) Gecko/20020508 Netscape6/6.2.3 X-Accept-Language: en-us MIME-Version: 1.0 To: obm-l@mat.puc-rio.br Subject: Re: [obm-l] =?ISO-8859-1?Q?Quest=E3o?= de geometria References: <002201c26017$35833220$2101a8c0@u2z7z2> Content-Type: multipart/alternative; boundary="------------050005060502060504080702" Sender: owner-obm-l@sucuri.mat.puc-rio.br Precedence: bulk Reply-To: obm-l@mat.puc-rio.br --------------050005060502060504080702 Content-Type: text/plain; charset=ISO-8859-1; format=flowed Content-Transfer-Encoding: 8bit Wagner wrote: > Oi pessoal ! > > > > Essa é uma pergunta de geometria um pouco chatinha, deêm uma olhada: > > > > Em um círculo existe um triângulo ABC retângulo inscrito nele. > Esse triângulo possui uma circunferência circunscrita nele.OU SEJA, > ESTA CIRCUNFERENCIA EH A CIRCUNFERENCIA ORIGINAL Essa circunferência > possui um triângulo DEF inscrito nela ,tal que AB é paralelo à DE, AC > é paralelo à DF e BC é paralelo à EF.OU SEJA ESSE TRIANGULO EH IGUAL > AO ORIGINAL Esse triângulo também possui uma circûnferência > circunscrita nele, tal que o padrão descrito acima se repete.OU SEJA > EH A CIRCUNFERENCIA ORIGINAL ATACANDO NOVAMENTE.Seja o triângulo XYZ > limite desse padrão, tal que XY é paralelo à DE, XZ é paralelo à DF e > YZ é paralelo à EF.EH O TRIANGULO ORIGINAL DE NOVO. Logo, quanto vale > a soma das distâncias AX, AY e AZ em funcão de BÂC e do raio da > circunferência maior? > > > > Dado: AB e BC são perpendiculares > > > > Se alguém tiver noção de como resolver essa pergunta já agradeço. > > > > André T. > > > --------------050005060502060504080702 Content-Type: text/html; charset=us-ascii Content-Transfer-Encoding: 7bit

Wagner wrote:
Oi pessoal !
 
Essa é uma pergunta de geometria um pouco chatinha, deêm uma olhada:
 
    Em um círculo existe um triângulo ABC retângulo inscrito nele. Esse triângulo possui uma circunferência circunscrita nele.OU SEJA, ESTA CIRCUNFERENCIA EH A CIRCUNFERENCIA ORIGINAL Essa circunferência possui um triângulo DEF inscrito nela ,tal que AB é paralelo à DE, AC é paralelo à DF e BC é paralelo à EF.OU SEJA ESSE TRIANGULO EH IGUAL AO ORIGINAL Esse triângulo  também possui uma circûnferência circunscrita nele, tal que o padrão descrito acima se repete.OU SEJA EH A CIRCUNFERENCIA ORIGINAL ATACANDO NOVAMENTE.Seja o triângulo XYZ limite desse padrão, tal que XY é paralelo à DE, XZ é paralelo à DF e YZ é paralelo à EF.EH O TRIANGULO ORIGINAL DE NOVO. Logo, quanto vale a soma das distâncias AX, AY e AZ em funcão de BÂC e do raio da circunferência maior?
 
Dado: AB e BC são perpendiculares
 
Se alguém tiver noção de como resolver essa pergunta já agradeço.
 
André T.
 

--------------050005060502060504080702-- ========================================================================= Instruções para entrar na lista, sair da lista e usar a lista em http://www.mat.puc-rio.br/~nicolau/olimp/obm-l.html O administrador desta lista é ========================================================================= From owner-obm-l@sucuri.mat.puc-rio.br Thu Sep 19 22:55:24 2002 Return-Path: Received: (from majordom@localhost) by sucuri.mat.puc-rio.br (8.9.3/8.9.3) id WAA14555 for obm-l-MTTP; Thu, 19 Sep 2002 22:55:21 -0300 Received: from gorgo.centroin.com.br (gorgo.centroin.com.br [200.225.63.128]) by sucuri.mat.puc-rio.br (8.9.3/8.9.3) with ESMTP id WAA14551 for ; Thu, 19 Sep 2002 22:55:19 -0300 Received: from centroin.com.br (du160c.rjo.centroin.com.br [200.225.58.160]) (authenticated bits=0) by gorgo.centroin.com.br (8.12.2/8.12.1) with ESMTP id g8K1vC7p007930 for ; Thu, 19 Sep 2002 22:57:13 -0300 (BRT) Message-ID: <3D8A811E.9080706@centroin.com.br> Date: Thu, 19 Sep 2002 22:59:58 -0300 From: Augusto =?ISO-8859-1?Q?C=E9sar?= Morgado User-Agent: Mozilla/5.0 (Windows; U; Win98; en-US; rv:0.9.4.1) Gecko/20020508 Netscape6/6.2.3 X-Accept-Language: en-us MIME-Version: 1.0 To: obm-l@mat.puc-rio.br Subject: Re: [obm-l] Re: [obm-l] =?ISO-8859-1?Q?Ser=E1?= que =?ISO-8859-1?Q?ningu=E9m?= me ajuda??? References: <001701c26015$03555960$2101a8c0@u2z7z2> Content-Type: multipart/alternative; boundary="------------030703090307030906090208" Sender: owner-obm-l@sucuri.mat.puc-rio.br Precedence: bulk Reply-To: obm-l@mat.puc-rio.br --------------030703090307030906090208 Content-Type: text/plain; charset=ISO-8859-1; format=flowed Content-Transfer-Encoding: 8bit Os encontros NAO sao aleatorios. Wagner wrote: > Oi pessoal > > > > Em 10 anos existem ou 3652 ou 3653 dias (dependendo do ano inicial) > > Chamando o ciclo físico (de 23 dias) de A, o mental (de 29 dias) de > B , o emocional (de 33 dias) de C e chamando a n-ésima vez que os > ciclos se encontram no máximo de m(n). > > Entre m(1) e m(2) existem 23.29.33 dias = 22011 dias (porquê 23,29 e > 33 são primos entre si). Logo m(2) só pode acontecer depois dos 10 > anos. Então basta verificar se m(1) acontece antes dos 10 anos. Isso > se torna complicado porquê o número de ciclos é obrigatoriamente > natural, mas se você quer uma estimativa, considerando os encontros > como aleatórios, a chance de m(1) estar antes dos 10 anos é > 3652,25/22011 que é uma chance de aproximadamente 16,5% > > > > André T. > > > > > > ----- Original Message ----- > > From:e isso mesmo > > To: obm-l@mat.puc-rio.br > > Sent: Thursday, September 19, 2002 10:32 AM > > Subject: [obm-l] Será que ninguém me ajuda??? > > > > > Companheiros continuo esperando ajuda de alguèm.... > > > > "A teoria do Biorritmo diz que os estados físico, mental e > emocional de uma pessoa oscilam periodicamente, a partir do dia do > nascimento, em ciclos de 23 dias, 29 dias e 33 dias, > respectivamente. Dado que os dias mais positivos dos ciclos > físico, mental e emocional são, respectivamente, o sexto, o sétimo > e o oitavo de cada ciclo, nos primeiros dez anos de vida de uma > pessoa, quantas vezes os três ciclos estão simultaneamente no > ponto máximo?" > > > > Tchau! > > > > > ------------------------------------------------------------------------ > Aproveite melhor a Web. Faça o download GRÁTIS do MSN Explorer : > http://explorer.msn.com.br/intl.asp#po > --------------030703090307030906090208 Content-Type: text/html; charset=us-ascii Content-Transfer-Encoding: 7bit Os encontros NAO sao aleatorios.

Wagner wrote:
Oi pessoal
 
Em 10 anos existem ou 3652 ou 3653 dias (dependendo do ano inicial)
Chamando o ciclo físico (de 23 dias) de A, o mental (de 29 dias) de B , o emocional (de 33 dias) de C e chamando a n-ésima vez que os ciclos se encontram no máximo de m(n).
Entre m(1) e m(2) existem 23.29.33 dias = 22011 dias (porquê 23,29 e 33 são primos entre si). Logo m(2) só pode acontecer depois dos 10 anos. Então basta verificar se m(1) acontece antes dos 10 anos. Isso se torna complicado porquê o número de ciclos é obrigatoriamente natural, mas se você quer uma estimativa, considerando os encontros como aleatórios, a chance de m(1) estar antes dos 10 anos é 3652,25/22011 que é uma chance de aproximadamente 16,5%
 
André T.
 
 
----- Original Message -----
Sent: Thursday, September 19, 2002 10:32 AM
Subject: [obm-l] Será que ninguém me ajuda???

 
Companheiros continuo esperando ajuda de alguèm....
 

"A teoria do Biorritmo diz que os estados físico, mental e emocional de uma pessoa oscilam periodicamente, a partir do dia do nascimento, em ciclos de 23 dias, 29 dias e 33 dias, respectivamente. Dado que os dias mais positivos dos ciclos físico, mental e emocional são, respectivamente, o sexto, o sétimo e o oitavo de cada ciclo, nos primeiros dez anos de vida de uma pessoa, quantas vezes os três ciclos estão simultaneamente no ponto máximo?"

 

Tchau!

 



Aproveite melhor a Web. Faça o download GRÁTIS do MSN Explorer : http://explorer.msn.com.br/intl.asp#po

--------------030703090307030906090208-- ========================================================================= Instruções para entrar na lista, sair da lista e usar a lista em http://www.mat.puc-rio.br/~nicolau/olimp/obm-l.html O administrador desta lista é ========================================================================= From owner-obm-l@sucuri.mat.puc-rio.br Thu Sep 19 23:27:24 2002 Return-Path: Received: (from majordom@localhost) by sucuri.mat.puc-rio.br (8.9.3/8.9.3) id XAA16026 for obm-l-MTTP; Thu, 19 Sep 2002 23:26:36 -0300 Received: from gorgo.centroin.com.br (gorgo.centroin.com.br [200.225.63.128]) by sucuri.mat.puc-rio.br (8.9.3/8.9.3) with ESMTP id XAA16022 for ; Thu, 19 Sep 2002 23:26:34 -0300 Received: from centroin.com.br (du140c.rjo.centroin.com.br [200.225.58.140]) (authenticated bits=0) by gorgo.centroin.com.br (8.12.2/8.12.1) with ESMTP id g8K2SR7p009743 for ; Thu, 19 Sep 2002 23:28:28 -0300 (BRT) Message-ID: <3D8A886F.80708@centroin.com.br> Date: Thu, 19 Sep 2002 23:31:11 -0300 From: Augusto =?ISO-8859-1?Q?C=E9sar?= Morgado User-Agent: Mozilla/5.0 (Windows; U; Win98; en-US; rv:0.9.4.1) Gecko/20020508 Netscape6/6.2.3 X-Accept-Language: en-us MIME-Version: 1.0 To: obm-l@mat.puc-rio.br Subject: [obm-l] =?ISO-8859-1?Q?corre=E7ao?= de problema muito antigo Content-Type: text/plain; charset=ISO-8859-1; format=flowed Content-Transfer-Encoding: 8bit Sender: owner-obm-l@sucuri.mat.puc-rio.br Precedence: bulk Reply-To: obm-l@mat.puc-rio.br Consultando mensagens antigas, notei que ha muito tempo pichurin pichurin escreveu: 1)quantos anagramas da palavra CASACO apresentam as três vogais juntas? A soluçao que foi apresentada parece-me incorreta. Ha 3 modos de juntar as vogais: AA0, AOA, OAA. Escolhida a ordem das vogais, basta arrumar as letras C, C, S e o bloco das vogais, o que pode ser feito de 4!/2!= 12 modos. A resposta eh 3*12 = 36. ========================================================================= Instruções para entrar na lista, sair da lista e usar a lista em http://www.mat.puc-rio.br/~nicolau/olimp/obm-l.html O administrador desta lista é ========================================================================= From owner-obm-l@sucuri.mat.puc-rio.br Thu Sep 19 23:29:52 2002 Return-Path: Received: (from majordom@localhost) by sucuri.mat.puc-rio.br (8.9.3/8.9.3) id XAA16110 for obm-l-MTTP; Thu, 19 Sep 2002 23:29:48 -0300 Received: from gorgo.centroin.com.br (gorgo.centroin.com.br [200.225.63.128]) by sucuri.mat.puc-rio.br (8.9.3/8.9.3) with ESMTP id XAA16105 for ; Thu, 19 Sep 2002 23:29:45 -0300 Received: from centroin.com.br (du140c.rjo.centroin.com.br [200.225.58.140]) (authenticated bits=0) by gorgo.centroin.com.br (8.12.2/8.12.1) with ESMTP id g8K2Vb7p009860 for ; Thu, 19 Sep 2002 23:31:37 -0300 (BRT) Message-ID: <3D8A892F.7050103@centroin.com.br> Date: Thu, 19 Sep 2002 23:34:23 -0300 From: Augusto =?ISO-8859-1?Q?C=E9sar?= Morgado User-Agent: Mozilla/5.0 (Windows; U; Win98; en-US; rv:0.9.4.1) Gecko/20020508 Netscape6/6.2.3 X-Accept-Language: en-us MIME-Version: 1.0 To: obm-l@mat.puc-rio.br Subject: [obm-l] [Fwd: RE: =?ISO-8859-1?Q?Poli=F4mios=5D?= Content-Type: multipart/mixed; boundary="------------080208010002010002080207" Sender: owner-obm-l@sucuri.mat.puc-rio.br Precedence: bulk Reply-To: obm-l@mat.puc-rio.br This is a multi-part message in MIME format. --------------080208010002010002080207 Content-Type: multipart/alternative; boundary="------------000206000300020308010005" --------------000206000300020308010005 Content-Type: text/plain; charset=ISO-8859-1; format=flowed Content-Transfer-Encoding: 8bit Este problema foi proposto (novamente) ha pouco tempo. Estou reenviando-o para a lista como um auxilio para quem o propos recentemente. Morgado -------- Original Message -------- From: - Tue Oct 16 21:20:31 2001 X-UIDL: >~!#!EE:!!%EP!!@Z5"! X-Mozilla-Status: 0001 X-Mozilla-Status2: 00000000 Return-Path: Received: from triceratops.centroin.com.br (mail-gw2.centroin.com.br [200.225.50.252]) by trex.centroin.com.br (8.10.1/8.10.1) with ESMTP id f9GDLje03205 for ; Tue, 16 Oct 2001 11:21:45 -0200 (EDT) Received: from matinta.mat.puc-rio.br (matinta.mat.puc-rio.br [139.82.27.1]) by triceratops.centroin.com.br (8.10.1/8.10.1) with ESMTP id f9GDLZR29426 for ; Tue, 16 Oct 2001 11:21:37 -0200 (EDT) Received: (from majordom@localhost) by matinta.mat.puc-rio.br (8.9.3/8.9.3) id LAA15406 for obm-l-list; Tue, 16 Oct 2001 11:01:20 -0200 Received: from pegasus.prt15.gov.br ([200.245.30.130]) by matinta.mat.puc-rio.br (8.9.3/8.9.3) with ESMTP id LAA15399 for ; Tue, 16 Oct 2001 11:01:03 -0200 Received: (from daemon@localhost) by pegasus.prt15.gov.br (8.11.1/8.11.1) id f9GDBvW62656 for obm-l@mat.puc-rio.br.CLEAN; Tue, 16 Oct 2001 11:11:57 -0200 (BRST) (envelope-from grasser@prt15.gov.br) Received: from codinsec.prt15.gov.br (CODINSEC.prt15.gov.br [192.168.1.63]) by pegasus.prt15.gov.br (8.11.1/8.11.1) with SMTP id f9GDBvg62651 for ; Tue, 16 Oct 2001 11:11:57 -0200 (BRST) (envelope-from grasser@prt15.gov.br) Received: by codinsec.prt15.gov.br with Microsoft Mail id <01C15633.47697920@codinsec.prt15.gov.br>; Tue, 16 Oct 2001 11:11:18 -0300 Message-ID: <01C15633.47697920@codinsec.prt15.gov.br> From: Eduardo Grasser To: "'obm-l@mat.puc-rio.br'" Subject: RE: Poliômios Date: Tue, 16 Oct 2001 11:10:54 -0300 MIME-Version: 1.0 Content-Type: multipart/mixed; boundary="---- =_NextPart_000_01C15633.49ABC800" X-Sanitizer: Este EMail foi desinfectado! X-Sanitizer-URL: http://www.prt15.gov.br/ X-Sanitizer-Rev: $Id: sanitizer.pl,v 1.35 2001/02/01 00:10:46 bre Exp $ Sender: owner-obm-l@mat.puc-rio.br Precedence: bulk Reply-To: obm-l@mat.puc-rio.br X-UIDL: >~!#!EE:!!%EP!!@Z5"! Status: U é a pressa... a idéia é essa mesma. O resto é sentar e desenvolver a idéia... Quase nunca consigo respostas corretas se não pego uma folha de papel e rescunho um pouco :-D obrigado Eduardo ---------- De: Eric Campos Bastos Guedes[SMTP:mathfire@ig.com.br] Enviada em: Segunda-feira, 15 de Outubro de 2001 19:01 Para: obm-l@mat.puc-rio.br Assunto: RES: Poliômios Eu nao faria melhor... Mas tem uns erros nas contas que nao invalidam a solucao. O resto certo eh -2x^3-2x^2+x+5 Eric. -----MENSAGEM ORIGINAL ABAIXO------------ p(x) = q(x)(x^4 + x^2 + 1) + ax^3 + bx^2 + cx + d (quis com isso dizer que o resto é um polinômio de grau 3) Divido por x^2 + x + 1, e tenho que a primeira parte dá zero pois x^2 + x + 1 divide x^4 + x^2 + 1 e a segunda dá resto (a-c)x + d-b+a = 3x + 5 Divido por x^2 - x + 1, e tenho que a primeira parte dá zero pois x^2 - x + 1 divide x^4 + x^2 + 1 e a segunda dá resto (c-2a-b)x + d-a-b = -x + 9 Assim, é só resolver o sistema a-c = 3 a-b+d = 5 -2a-b+c = -1 -a-b+d = 9 e achar o polinômio -2x^3 - 5x + 7 como resto Acho que é isso salvo erros de conta, já que fiz correndo. Eduardo Grasser Campinas sp ---------- De: René Retz[SMTP:rene.retz@bol.com.br] Enviada em: Segunda-feira, 15 de Outubro de 2001 15:54 Para: obm-l@mat.puc-rio.br Assunto: Poliômios Sabe-se que os restos da divisão de um polinõmio p(x) por x^2 + x + 1 e x^2 - x + 1 são repsctivamente 3x + 5 e -x + 9. Determine o resto de p(x) por x^4 + x^2 + 1. --------------000206000300020308010005 Content-Type: text/html; charset=us-ascii Content-Transfer-Encoding: 7bit Este problema foi proposto (novamente) ha pouco tempo. Estou reenviando-o para a lista como um auxilio para quem o propos recentemente.
Morgado

-------- Original Message --------
From: - Tue Oct 16 21:20:31 2001
X-UIDL: >~!#!EE:!!%EP!!@Z5"!
X-Mozilla-Status: 0001
X-Mozilla-Status2: 00000000
Return-Path: <owner-obm-l@mat.puc-rio.br>
Received: from triceratops.centroin.com.br (mail-gw2.centroin.com.br [200.225.50.252]) by trex.centroin.com.br (8.10.1/8.10.1) with ESMTP id f9GDLje03205 for <morgado@centroin.com.br>; Tue, 16 Oct 2001 11:21:45 -0200 (EDT)
Received: from matinta.mat.puc-rio.br (matinta.mat.puc-rio.br [139.82.27.1]) by triceratops.centroin.com.br (8.10.1/8.10.1) with ESMTP id f9GDLZR29426 for <morgado@centroin.com.br>; Tue, 16 Oct 2001 11:21:37 -0200 (EDT)
Received: (from majordom@localhost) by matinta.mat.puc-rio.br (8.9.3/8.9.3) id LAA15406 for obm-l-list; Tue, 16 Oct 2001 11:01:20 -0200
Received: from pegasus.prt15.gov.br ([200.245.30.130]) by matinta.mat.puc-rio.br (8.9.3/8.9.3) with ESMTP id LAA15399 for <obm-l@mat.puc-rio.br>; Tue, 16 Oct 2001 11:01:03 -0200
Received: (from daemon@localhost) by pegasus.prt15.gov.br (8.11.1/8.11.1) id f9GDBvW62656 for obm-l@mat.puc-rio.br.CLEAN; Tue, 16 Oct 2001 11:11:57 -0200 (BRST) (envelope-from grasser@prt15.gov.br)
Received: from codinsec.prt15.gov.br (CODINSEC.prt15.gov.br [192.168.1.63]) by pegasus.prt15.gov.br (8.11.1/8.11.1) with SMTP id f9GDBvg62651 for <obm-l@mat.puc-rio.br>; Tue, 16 Oct 2001 11:11:57 -0200 (BRST) (envelope-from grasser@prt15.gov.br)
Received: by codinsec.prt15.gov.br with Microsoft Mail id <01C15633.47697920@codinsec.prt15.gov.br>; Tue, 16 Oct 2001 11:11:18 -0300
Message-ID: <01C15633.47697920@codinsec.prt15.gov.br>
From: Eduardo Grasser <grasser@prt15.gov.br>
To: "'obm-l@mat.puc-rio.br'" <obm-l@mat.puc-rio.br>
Subject: RE: Poliômios
Date: Tue, 16 Oct 2001 11:10:54 -0300
MIME-Version: 1.0
Content-Type: multipart/mixed; boundary="---- =_NextPart_000_01C15633.49ABC800"
X-Sanitizer: Este EMail foi desinfectado!
X-Sanitizer-URL: http://www.prt15.gov.br/
X-Sanitizer-Rev: $Id: sanitizer.pl,v 1.35 2001/02/01 00:10:46 bre Exp $
Sender: owner-obm-l@mat.puc-rio.br
Precedence: bulk
Reply-To: obm-l@mat.puc-rio.br
X-UIDL: >~!#!EE:!!%EP!!@Z5"!
Status: U


é a pressa... a idéia é essa mesma. O resto é sentar e desenvolver a idéia...
Quase nunca consigo respostas corretas se não pego uma folha de papel e rescunho um pouco :-D

obrigado

Eduardo

----------
De:	Eric Campos Bastos Guedes[SMTP:mathfire@ig.com.br]
Enviada em:	Segunda-feira, 15 de Outubro de 2001 19:01
Para:	obm-l@mat.puc-rio.br
Assunto:	RES: Poliômios

Eu nao faria melhor...
Mas tem uns erros nas contas que nao invalidam a solucao.
O resto certo eh

-2x^3-2x^2+x+5

Eric.


-----MENSAGEM ORIGINAL ABAIXO------------

p(x) = q(x)(x^4 + x^2 + 1) + ax^3 + bx^2 + cx + d
(quis com isso dizer que o resto é um polinômio de grau 3)

Divido por x^2 + x + 1, e tenho que a primeira parte dá zero pois x^2 + x +
1 divide x^4 + x^2 + 1 e a segunda dá r
esto (a-c)x + d-b+a = 3x + 5

Divido por x^2 - x + 1, e tenho que a primeira parte dá zero pois x^2 - x +
1 divide x^4 + x^2 + 1 e a segunda dá resto (c-2a-b)x + d-a-b = -x + 9

Assim, é só resolver o sistema
a-c = 3
a-b+d = 5
-2a-b+c = -1
-a-b+d = 9

e achar o polinômio -2x^3 - 5x + 7 como resto

Acho que é isso salvo erros de conta, já que fiz correndo.

Eduardo Grasser
Campinas sp

----------
De:	René Retz[SMTP:rene.retz@bol.com.br]
Enviada em:	Segunda-feira, 15 de Outubro de 2001 15:54
Para:	obm-l@mat.puc-rio.br
Assunto:	Poliômios

Sabe-se que os restos da divisão de um polinõmio p(x) por x^2 + x + 1 e
x^2 - x + 1 são repsctivamente  3x + 5  e  -x + 9. Determine o resto de p(x)
por x^4 + x^2 + 1.







--------------000206000300020308010005-- --------------080208010002010002080207 Content-Type: application/octet-stream; name="nsmail.tmp" Content-Transfer-Encoding: base64 Content-Disposition: attachment; filename="nsmail.tmp" eJ8+IhcOAQaQCAAEAAAAAAABAAEAAQeQBgAIAAAA5AQAAAAAAADoAAENgAQAAgAAAAIAAgAB BJAGACABAAABAAAADAAAAAMAADADAAAACwAPDgAAAAACAf8PAQAAAEcAAAAAAAAAgSsfpL6j EBmdbgDdAQ9UAgAAAABvYm0tbEBtYXQucHVjLXJpby5icgBTTVRQAG9ibS1sQG1hdC5wdWMt cmlvLmJyAAAeAAIwAQAAAAUAAABTTVRQAAAAAB4AAzABAAAAFQAAAG9ibS1sQG1hdC5wdWMt cmlvLmJyAAAAAAMAFQwBAAAAAwD+DwYAAAAeAAEwAQAAABcAAAAnb2JtLWxAbWF0LnB1Yy1y aW8uYnInAAACAQswAQAAABoAAABTTVRQOk9CTS1MQE1BVC5QVUMtUklPLkJSAAAAAwAAOQAA AAALAEA6AQAAAAIB9g8BAAAABAAAAAAAAANxMgEIgAcAGAAAAElQTS5NaWNyb3NvZnQgTWFp bC5Ob3RlADEIAQSAAQAOAAAAUkU6IFBvbGn0bWlvcwAxBQEFgAMADgAAANEHCgAQAAsACgA2 AAIAPwEBIIADAA4AAADRBwoAEAALAAgAOwACAEIBAQmAAQAhAAAAMDRCQTY5MDkyNUMyRDUx MThGRTEwMDgwQUQzMDJBMTQA8gYBA5AGAEgGAAASAAAACwAjAAAAAAADACYAAAAAAAsAKQAA AAAAAwA2AAAAAABAADkAAF55XkxWwQEeAHAAAQAAAA4AAABSRTogUG9safRtaW9zAAAAAgFx AAEAAAAWAAAAAcFWTF5oCWm6BcIlEdWP4QCArTAqFAAAHgAeDAEAAAAFAAAAU01UUAAAAAAe AB8MAQAAABUAAABncmFzc2VyQHBydDE1Lmdvdi5icgAAAAADAAYQpRoebAMABxBWBAAAHgAI EAEAAABlAAAA6UFQUkVTU0FBSUTpSUHpRVNTQU1FU01BT1JFU1RP6VNFTlRBUkVERVNFTlZP TFZFUkFJROlJQVFVQVNFTlVOQ0FDT05TSUdPUkVTUE9TVEFTQ09SUkVUQVNTRU7jT1BFR09V TQAAAAACAQkQAQAAANIEAADOBAAAtAkAAExaRnX8LPB3/wAKAQ8CFQKoBesCgwBQAvIJAgBj aArAc2V0MjcGAAbDAoMyA8UCAHByQnER4nN0ZW0CgzN3AuQHEwKAfQqACM8J2TvxFg8yNTUC gAqBDbELYMBuZzEwNDYK+xRRDQvyYwBAAzAnZTkgjGEgE1AHkHNhLhvAfRsxaQswGwEHMBrk G4IgJweBAMAb4E8gG3F0b88a5BGwAjAKwWUgDbAekXp2BvB2BJAb+RvBCoVRBHVhEbAgbnVu YzcbQAWgAIFnHiAbcXBv9xPAIUAh0XIWECLCIVIa8VIzHiBwZSIxdQDAIN0CEGwRgB8RG1Bh JDADIOsfABtxYyGQaCRiG1AIYGEFoCA6LUQKhQqFb0JiBRBnYWRvJyxFHmQhMAsgKH0K9Gxp MQQ4MALRaS0xNDTvDfAM0CuzC1kxGWEDYBPQ+mMFQC0t1wqHLIsMMC1W+ERlOixvLXQMgilQ BRAgYyBDYW0ikSBCNyFAHhAEIEcKUB8hW1OQTVRQOgDAdGgrgJUWEEAiIC4FoG0uKBC2XS5/ L41FH2AHMGQbQG8T4DCfMaoGYGchkDfQLQRmZTSgYSwgMTXhJRJPdXR1KBAeICUhxQHQMBrQ MTk6PBA1j3UvjVAKwGE4LzGqKABtSC1sQDRRLnAmsC3dBRBvNUE8ny+NQQQQIZCHHhA+3zGq UkVTOj6AQwbwAKAnZjRtQZBzcynfKuMzNixXGkU3B3X1IXBhHiBmCsAcgQeAJOD7BbAgiE0i 0RPRJHAGMQSQvwNgBCBKYCLiHrEEIHEKUHtKUwuAdgdAHCAywBsxc98G8CawSnAgph3GYwSQ HhEEZWgnLC0yeF4zwVHiMit4KzUojTJxByCmUQ0t0k1FTlNBCEdFTR2wUklHSQhOQUwUsEJB SVgOTy3YLlcKhXAoeCkcID1NwFjBWMBeNCBsKyBSUVmhMVjgWbBhS1IBWaFiWdRjeFmhZP0K hShN0AQAIdEmcAQBO6H8aXofsU3SIkMeFiZjKxAvI8FG0yUSCcBhSkAzKfknLERpN6AoYCaB BcBZ1P1bcjE6wB8AE9AmIk3SG0L/B3E6kSVBACAfARrxGtBdQf9hYlxhYccKhRrQXSBhMR8A v1l7HvFO8ToEZFUd5Cg6UMRjKVtzLWIrG0BZAO4zW3JSvWEdLWIfYy9kP39lRGyTZf9nD2gf aSJBYDJdOlBiaZV0IVjxLVtyOf8nLEPhB3A6wB5ERrFasBtx7x+EHiAAkBPCYQqFaWFqMrt4 p2oAZFjxUrZ0Ayt5Mv8roFF2efZ1fW3BEXJdsV7L61HjbIE1W3I3XIJdxXWN/xFwbYQa81zT G6AfkFDBTNN3JSFNYzrAam8DTdIrgHr/IvQ6ME93KPwzgDqgBBAEkO8KhTKyC4AjYnBUby4M MHLPReMJ8BrziuB0ejP0hSHkZS4jMXpABuE1DTeKhzm/Os871jU6NTQKhf8+lEBvQXxD5zIi Rn8ndwYQ+GJlLSFRXYMEIB3jg1G/JQFhIXchI/IlIV6ZNV9S/1izYY0c8AqFbFl3EiPyFhD6 cATwdGEgMsAeoR8AalW/neCd0XUkG+AwcBPQcpVw/4wgXbclIljBWEZhk1mZU++/ou9UTkgf SS8KhRUxAKgwAAADABAQAAAAAAMAERABAAAAQAAHMCAU4xlMVsEBQAAIMCAU4xlMVsEBHgA9 AAEAAAAFAAAAUkU6IAAAAAAw8Q== --------------080208010002010002080207-- ========================================================================= Instruções para entrar na lista, sair da lista e usar a lista em http://www.mat.puc-rio.br/~nicolau/olimp/obm-l.html O administrador desta lista é ========================================================================= From owner-obm-l@sucuri.mat.puc-rio.br Fri Sep 20 01:44:06 2002 Return-Path: Received: (from majordom@localhost) by sucuri.mat.puc-rio.br (8.9.3/8.9.3) id BAA18485 for obm-l-MTTP; Fri, 20 Sep 2002 01:42:50 -0300 Received: from smtp-33.ig.com.br (smtp-33.ig.com.br [200.226.132.183]) by sucuri.mat.puc-rio.br (8.9.3/8.9.3) with SMTP id BAA18481 for ; Fri, 20 Sep 2002 01:42:48 -0300 Received: (qmail 26214 invoked from network); 20 Sep 2002 04:44:04 -0000 Received: from 200-163-25-135-bsace7026.dsl.telebrasilia.net.br (HELO henrique) (200.163.25.135) by smtp-33.ig.com.br with SMTP; 20 Sep 2002 04:44:04 -0000 Message-ID: <014e01c26060$5b6e7e10$019da8c0@henrique> From: "Henrique Branco" To: References: Subject: [obm-l] =?iso-8859-1?Q?Re:_=5Bobm-l=5D_Como_anda_seu_cora=E7=E3o...=3F?= Date: Fri, 20 Sep 2002 01:44:07 -0300 MIME-Version: 1.0 Content-Type: text/plain; charset="iso-8859-1" Content-Transfer-Encoding: 8bit X-Priority: 3 X-MSMail-Priority: Normal X-Mailer: Microsoft Outlook Express 6.00.2600.0000 X-MimeOLE: Produced By Microsoft MimeOLE V6.00.2600.0000 Sender: owner-obm-l@sucuri.mat.puc-rio.br Precedence: bulk Reply-To: obm-l@mat.puc-rio.br >"A teoria do Biorritmo diz que os estados físico, mental e emocional de uma pessoa oscilam periodicamente, a partir do dia do nascimento, em ciclos de 23 dias, 29 dias e 33 dias, >respectivamente. Dado que os dias mais positivos dos ciclos físico, mental e emocional são, respectivamente, o sexto, o sétimo e o oitavo de cada ciclo, nos primeiros dez anos de >vida de uma pessoa, quantas vezes os três ciclos estão simultaneamente no ponto máximo?" Aí vai uma tentativa... Em 10 anos, vamos ter 3652 ou 3653 dias. Os pontos máximos de cada ciclo encontram-se no dia 6, 7 e 8 de cada ciclo, portanto os pontos comuns serão mmc(6,7,8)=168. Usando o termo geral de uma P.A. para An=3528 (o maior número que se encontra entre 168 e 3523, sendo múltiplo de 168) temos: 3528 = 168 + (n-1)*168 3528 = 168 + 168n - 168 n = 21 Logo, os três ciclos estiveram em seu ponto máximo 21 vezes em 10 anos. Foi só uma tentativa... Se estiver errado, caiam matando! :-) Abraços, Henrique. P.S. - Depois que fiz o problema com P.A., percebi que o método que usei para achar o número 3528 daria a resposta direto... Coisas da vida... ========================================================================= Instruções para entrar na lista, sair da lista e usar a lista em http://www.mat.puc-rio.br/~nicolau/olimp/obm-l.html O administrador desta lista é ========================================================================= From owner-obm-l@sucuri.mat.puc-rio.br Fri Sep 20 02:05:39 2002 Return-Path: Received: (from majordom@localhost) by sucuri.mat.puc-rio.br (8.9.3/8.9.3) id CAA18999 for obm-l-MTTP; Fri, 20 Sep 2002 02:05:30 -0300 Received: from smtp-5.ig.com.br (smtp-5.ig.com.br [200.226.132.154]) by sucuri.mat.puc-rio.br (8.9.3/8.9.3) with SMTP id CAA18995 for ; Fri, 20 Sep 2002 02:05:28 -0300 Received: (qmail 7111 invoked from network); 20 Sep 2002 05:06:39 -0000 Received: from 200-163-25-135-bsace7026.dsl.telebrasilia.net.br (HELO henrique) (200.163.25.135) by smtp-5.ig.com.br with SMTP; 20 Sep 2002 05:06:39 -0000 Message-ID: <001a01c26063$81e43280$019da8c0@henrique> From: "Henrique Branco" To: Subject: [obm-l] =?iso-8859-1?Q?Re:_=5Bobm-l=5D_Como_anda_seu_cora=E7=E3o...=3F?= Date: Fri, 20 Sep 2002 02:06:41 -0300 MIME-Version: 1.0 Content-Type: multipart/alternative; boundary="----=_NextPart_000_0017_01C2604A.5C3CF3A0" X-Priority: 3 X-MSMail-Priority: Normal X-Mailer: Microsoft Outlook Express 6.00.2600.0000 X-MimeOLE: Produced By Microsoft MimeOLE V6.00.2600.0000 Sender: owner-obm-l@sucuri.mat.puc-rio.br Precedence: bulk Reply-To: obm-l@mat.puc-rio.br This is a multi-part message in MIME format. ------=_NextPart_000_0017_01C2604A.5C3CF3A0 Content-Type: text/plain; charset="iso-8859-1" Content-Transfer-Encoding: quoted-printable Talvez n=E3o tenha sido muito claro na resolu=E7=E3o que propus para o = problema em minha mensagem anterior... > Em 10 anos, vamos ter 3652 ou 3653 dias. Os pontos m=E1ximos de cada = ciclo > encontram-se no dia 6, 7 e 8 de cada ciclo, portanto os pontos comuns ser=E3o > mmc(6,7,8)=3D168. Quis dizer que os pontos comuns ocorrer=E3o, inicialmente, no 168=BA e = depois de 168 em 168 dias, j=E1 que esse =E9 o mmc entre 6,7,8. Abra=E7o, Henrique. ------=_NextPart_000_0017_01C2604A.5C3CF3A0 Content-Type: text/html; charset="iso-8859-1" Content-Transfer-Encoding: quoted-printable
Talvez n=E3o tenha sido muito claro na resolu=E7=E3o que propus = para o problema=20 em
minha mensagem anterior...

> Em 10 anos, vamos ter 3652 = ou 3653=20 dias. Os pontos m=E1ximos de cada ciclo
> encontram-se no dia 6, 7 = e 8 de=20 cada ciclo, portanto os pontos comuns
ser=E3o
>=20 mmc(6,7,8)=3D168.

Quis dizer que os pontos comuns ocorrer=E3o, = inicialmente,=20 no 168=BA e depois de
168 em 168 dias, j=E1 que esse =E9 o mmc entre=20 6,7,8.
Abra=E7o,
Henrique.
------=_NextPart_000_0017_01C2604A.5C3CF3A0-- ========================================================================= Instruções para entrar na lista, sair da lista e usar a lista em http://www.mat.puc-rio.br/~nicolau/olimp/obm-l.html O administrador desta lista é ========================================================================= From owner-obm-l@sucuri.mat.puc-rio.br Fri Sep 20 02:46:12 2002 Return-Path: Received: (from majordom@localhost) by sucuri.mat.puc-rio.br (8.9.3/8.9.3) id CAA19936 for obm-l-MTTP; Fri, 20 Sep 2002 02:45:36 -0300 Received: from smtp-4.ig.com.br (smtp-4.ig.com.br [200.226.132.153]) by sucuri.mat.puc-rio.br (8.9.3/8.9.3) with SMTP id CAA19932 for ; Fri, 20 Sep 2002 02:45:33 -0300 Received: (qmail 20471 invoked from network); 20 Sep 2002 05:46:50 -0000 Received: from 200-163-25-135-bsace7026.dsl.telebrasilia.net.br (HELO henrique) (200.163.25.135) by smtp-4.ig.com.br with SMTP; 20 Sep 2002 05:46:50 -0000 Message-ID: <004401c26069$2071b2b0$019da8c0@henrique> From: "Henrique Branco" To: Subject: [obm-l] =?iso-8859-1?Q?Re:_=5Bobm-l=5D_Como_anda_seu_cora=E7=E3o...=3F?= Date: Fri, 20 Sep 2002 02:46:49 -0300 MIME-Version: 1.0 Content-Type: multipart/alternative; boundary="----=_NextPart_000_0041_01C2604F.F7E83B70" X-Priority: 3 X-MSMail-Priority: Normal X-Mailer: Microsoft Outlook Express 6.00.2600.0000 X-MimeOLE: Produced By Microsoft MimeOLE V6.00.2600.0000 Sender: owner-obm-l@sucuri.mat.puc-rio.br Precedence: bulk Reply-To: obm-l@mat.puc-rio.br This is a multi-part message in MIME format. ------=_NextPart_000_0041_01C2604F.F7E83B70 Content-Type: text/plain; charset="iso-8859-1" Content-Transfer-Encoding: quoted-printable Mais um erro... Sono =E9 dose... :-) >Usando o termo geral de uma P.A. para An=3D3528 (o >maior n=FAmero que = se >encontra entre 168 e 3523, sendo m=FAltiplo de 168) >temos: Onde tem "o maior n=FAmero ... entre 168 e 3523" leia-se "o maior = n=FAmero ... entre 168 e 3653". Agora s=F3 falta o problema todo estar errado! :-) Abra=E7os, Henrique. ------=_NextPart_000_0041_01C2604F.F7E83B70 Content-Type: text/html; charset="iso-8859-1" Content-Transfer-Encoding: quoted-printable
Mais um erro... Sono =E9 dose... = :-)
 
>Usando o termo geral de uma P.A. para An=3D3528 (o >maior = n=FAmero que=20 se
>encontra entre 168 e 3523, sendo m=FAltiplo de 168) = >temos:
 
Onde tem "o maior n=FAmero ... entre = 168 e 3523"=20 leia-se "o maior n=FAmero ... entre 168 e 3653".
Agora s=F3 falta o problema todo estar = errado!=20 :-)
Abra=E7os,
Henrique.
------=_NextPart_000_0041_01C2604F.F7E83B70-- ========================================================================= Instruções para entrar na lista, sair da lista e usar a lista em http://www.mat.puc-rio.br/~nicolau/olimp/obm-l.html O administrador desta lista é ========================================================================= From owner-obm-l@sucuri.mat.puc-rio.br Fri Sep 20 06:57:32 2002 Return-Path: Received: (from majordom@localhost) by sucuri.mat.puc-rio.br (8.9.3/8.9.3) id GAA22007 for obm-l-MTTP; Fri, 20 Sep 2002 06:57:15 -0300 Received: from trex.centroin.com.br (trex.centroin.com.br [200.225.63.134]) by sucuri.mat.puc-rio.br (8.9.3/8.9.3) with ESMTP id GAA22003 for ; Fri, 20 Sep 2002 06:57:13 -0300 Received: from trex.centroin.com.br (localhost [127.0.0.1]) by trex.centroin.com.br (8.12.5/8.12.1) with ESMTP id g8K9x1Ng007486 for ; Fri, 20 Sep 2002 06:59:01 -0300 (BRT) Received: by trex.centroin.com.br (8.12.5/8.12.5/Submit) id g8K9x1eD007485; Fri, 20 Sep 2002 06:59:01 -0300 (BRT) Message-Id: <200209200959.g8K9x1eD007485@trex.centroin.com.br> Received: from 200.165.196.107 by trex.centroin.com.br (CIPWM versao 1.4C1) with HTTPS for ; Fri, 20 Sep 2002 06:59:00 -0300 (BRT) Date: Fri, 20 Sep 2002 06:59:00 -0300 (BRT) From: Augusto Cesar de Oliveira Morgado To: obm-l@mat.puc-rio.br Subject: =?iso-8859-1?q?Re: [obm-l] Re:_[obm-l]_Como_anda_seu_cora=E7=E3o...??= MIME-Version: 1.0 X-Mailer: CentroIn Internet Provider WebMail v. 1.4C1 (http://www.centroin.com.br/) Content-Type: text/plain; charset="iso-8859-1" Content-Transfer-Encoding: 8bit X-MIME-Autoconverted: from quoted-printable to 8bit by sucuri.mat.puc-rio.br id GAA22004 Sender: owner-obm-l@sucuri.mat.puc-rio.br Precedence: bulk Reply-To: obm-l@mat.puc-rio.br Em Fri, 20 Sep 2002 02:46:49 -0300, Henrique Branco disse: > Mais um erro... Sono é dose... :-) > > >Usando o termo geral de uma P.A. para An=3528 (o >maior número que se > >encontra entre 168 e 3523, sendo múltiplo de 168) >temos: > > Onde tem "o maior número ... entre 168 e 3523" leia-se "o maior número ... entre 168 e 3653". > Agora só falta o problema todo estar errado! :-) > Abraços, > Henrique. Eh, mas infelizmente estah. Morgado ========================================================================= Instruções para entrar na lista, sair da lista e usar a lista em http://www.mat.puc-rio.br/~nicolau/olimp/obm-l.html O administrador desta lista é ========================================================================= From owner-obm-l@sucuri.mat.puc-rio.br Fri Sep 20 07:05:42 2002 Return-Path: Received: (from majordom@localhost) by sucuri.mat.puc-rio.br (8.9.3/8.9.3) id HAA22158 for obm-l-MTTP; Fri, 20 Sep 2002 07:05:37 -0300 Received: from trex.centroin.com.br (trex.centroin.com.br [200.225.63.134]) by sucuri.mat.puc-rio.br (8.9.3/8.9.3) with ESMTP id HAA22153 for ; Fri, 20 Sep 2002 07:05:35 -0300 Received: from trex.centroin.com.br (localhost [127.0.0.1]) by trex.centroin.com.br (8.12.5/8.12.1) with ESMTP id g8KA7VNg009132 for ; Fri, 20 Sep 2002 07:07:31 -0300 (BRT) Received: by trex.centroin.com.br (8.12.5/8.12.5/Submit) id g8KA7VDn009131; Fri, 20 Sep 2002 07:07:31 -0300 (BRT) Message-Id: <200209201007.g8KA7VDn009131@trex.centroin.com.br> Received: from 200.165.196.107 by trex.centroin.com.br (CIPWM versao 1.4C1) with HTTPS for ; Fri, 20 Sep 2002 07:07:31 -0300 (BRT) Date: Fri, 20 Sep 2002 07:07:31 -0300 (BRT) From: Augusto Cesar de Oliveira Morgado To: obm-l@mat.puc-rio.br Subject: =?iso-8859-1?q?Re: [obm-l] Re:_[obm-l]_Como_anda_seu_cora=E7=E3o...??= MIME-Version: 1.0 X-Mailer: CentroIn Internet Provider WebMail v. 1.4C1 (http://www.centroin.com.br/) Content-Type: text/plain; charset="iso-8859-1" Content-Transfer-Encoding: 8bit X-MIME-Autoconverted: from quoted-printable to 8bit by sucuri.mat.puc-rio.br id HAA22154 Sender: owner-obm-l@sucuri.mat.puc-rio.br Precedence: bulk Reply-To: obm-l@mat.puc-rio.br Em Fri, 20 Sep 2002 01:44:07 -0300, Henrique Branco disse: > >"A teoria do Biorritmo diz que os estados físico, mental e emocional de uma > pessoa oscilam periodicamente, a partir do dia do nascimento, em ciclos de > 23 dias, 29 dias e 33 dias, >respectivamente. Dado que os dias mais > positivos dos ciclos físico, mental e emocional são, respectivamente, o > sexto, o sétimo e o oitavo de cada ciclo, nos primeiros dez anos de >vida de > uma pessoa, quantas vezes os três ciclos estão simultaneamente no ponto > máximo?" > > Aí vai uma tentativa... > Em 10 anos, vamos ter 3652 ou 3653 dias. Os pontos máximos de cada ciclo > encontram-se no dia 6, 7 e 8 de cada ciclo, portanto os pontos comuns serão > mmc(6,7,8)=168. AQUI, NAO TEM NADA A VER. IMAGINE QUE FOSSEM DOIS CICLOS COM DURAÇOES 5 E 7, COM MAXIMOS NOS DIAS 2 E 3 DE CADA CICLO. TERIAMOS, USANDO M PARA O MAXIMO E N PARA DIAS NORMAIS (ISTO E, NAO DE MAXIMO) NMNNN NMNNN NMNNN NMNNN... NNMNNNN NNMNNNN NNMNNNN... COMO SE PODE OBSERVAR NAO HA MAXIMOS SIMULTANEOS NO DIA 6=MMDC(2, 3) > Usando o termo geral de uma P.A. para An=3528 (o maior número que se > encontra entre 168 e 3523, sendo múltiplo de 168) temos: > 3528 = 168 + (n-1)*168 > 3528 = 168 + 168n - 168 > n = 21 > Logo, os três ciclos estiveram em seu ponto máximo 21 vezes em 10 anos. > Foi só uma tentativa... Se estiver errado, caiam matando! :-) > Abraços, > Henrique. > > P.S. - Depois que fiz o problema com P.A., percebi que o método que usei > para achar o número 3528 daria a resposta direto... Coisas da vida... > > > ========================================================================= > Instruções para entrar na lista, sair da lista e usar a lista em > http://www.mat.puc-rio.br/~nicolau/olimp/obm-l.html > O administrador desta lista é > ========================================================================= > > ========================================================================= Instruções para entrar na lista, sair da lista e usar a lista em http://www.mat.puc-rio.br/~nicolau/olimp/obm-l.html O administrador desta lista é ========================================================================= From owner-obm-l@sucuri.mat.puc-rio.br Fri Sep 20 09:01:58 2002 Return-Path: Received: (from majordom@localhost) by sucuri.mat.puc-rio.br (8.9.3/8.9.3) id JAA23692 for obm-l-MTTP; Fri, 20 Sep 2002 09:00:43 -0300 Received: from web21305.mail.yahoo.com (web21305.mail.yahoo.com [216.136.129.141]) by sucuri.mat.puc-rio.br (8.9.3/8.9.3) with SMTP id JAA23688 for ; Fri, 20 Sep 2002 09:00:40 -0300 Message-ID: <20020920120213.36287.qmail@web21305.mail.yahoo.com> Received: from [200.193.20.168] by web21305.mail.yahoo.com via HTTP; Fri, 20 Sep 2002 09:02:13 ART Date: Fri, 20 Sep 2002 09:02:13 -0300 (ART) From: =?iso-8859-1?q?Jorge=20Paulino?= Subject: [obm-l] QUESTÃO IME To: obm-l@mat.puc-rio.br MIME-Version: 1.0 Content-Type: text/plain; charset=iso-8859-1 Content-Transfer-Encoding: 8bit Sender: owner-obm-l@sucuri.mat.puc-rio.br Precedence: bulk Reply-To: obm-l@mat.puc-rio.br Oi galera, poderiam me ajudar na seguinte questão do IME... sqrt(5-sqrt(5-x))=x, para x>0 Um abraço, Jorge _______________________________________________________________________ Yahoo! GeoCities Tudo para criar o seu site: ferramentas fáceis de usar, espaço de sobra e acessórios. http://br.geocities.yahoo.com/ ========================================================================= Instruções para entrar na lista, sair da lista e usar a lista em http://www.mat.puc-rio.br/~nicolau/olimp/obm-l.html O administrador desta lista é ========================================================================= From owner-obm-l@sucuri.mat.puc-rio.br Fri Sep 20 11:56:51 2002 Return-Path: Received: (from majordom@localhost) by sucuri.mat.puc-rio.br (8.9.3/8.9.3) id LAA25970 for obm-l-MTTP; Fri, 20 Sep 2002 11:56:13 -0300 Received: from mail.vetor.com.br (wool.vetor.com.br [200.160.244.7]) by sucuri.mat.puc-rio.br (8.9.3/8.9.3) with ESMTP id LAA25955 for ; Fri, 20 Sep 2002 11:56:08 -0300 Received: from rodrigo (dl-ip194.wb.com.br [200.160.244.194] (may be forged)) by mail.vetor.com.br (8.12.5) with SMTP id g8KEqjwR027381 for ; Fri, 20 Sep 2002 11:52:46 -0300 Message-ID: <000801c260b5$daa097e0$c2f4a0c8@rodrigo> From: "Rodrigo Villard Milet" To: Subject: [obm-l] =?iso-8859-1?Q?Re:_=5Bobm-l=5D_QUEST=C3O_IME?= Date: Fri, 20 Sep 2002 11:56:07 -0300 MIME-Version: 1.0 Content-Type: text/plain; charset="iso-8859-1" Content-Transfer-Encoding: 8bit X-Priority: 3 X-MSMail-Priority: Normal X-Mailer: Microsoft Outlook Express 4.72.3110.5 X-MimeOLE: Produced By Microsoft MimeOLE V4.72.3110.3 X-Virus-Scanned: by amavis (http://amavis.org/) Sender: owner-obm-l@sucuri.mat.puc-rio.br Precedence: bulk Reply-To: obm-l@mat.puc-rio.br Essa questão já foi discutida aqui mais de mil vezes... procure nos arquivos. Villard -----Mensagem original----- De: Jorge Paulino Para: obm-l@mat.puc-rio.br Data: Sexta-feira, 20 de Setembro de 2002 09:24 Assunto: [obm-l] QUESTÃO IME >Oi galera, >poderiam me ajudar na seguinte questão >do IME... > >sqrt(5-sqrt(5-x))=x, para x>0 > >Um abraço, >Jorge > >_______________________________________________________________________ >Yahoo! GeoCities >Tudo para criar o seu site: ferramentas fáceis de usar, espaço de sobra e acessórios. >http://br.geocities.yahoo.com/ >========================================================================= >Instruções para entrar na lista, sair da lista e usar a lista em >http://www.mat.puc-rio.br/~nicolau/olimp/obm-l.html >O administrador desta lista é >========================================================================= > ========================================================================= Instruções para entrar na lista, sair da lista e usar a lista em http://www.mat.puc-rio.br/~nicolau/olimp/obm-l.html O administrador desta lista é ========================================================================= From owner-obm-l@sucuri.mat.puc-rio.br Fri Sep 20 13:14:52 2002 Return-Path: Received: (from majordom@localhost) by sucuri.mat.puc-rio.br (8.9.3/8.9.3) id NAA27426 for obm-l-MTTP; Fri, 20 Sep 2002 13:11:53 -0300 Received: from web12908.mail.yahoo.com (web12908.mail.yahoo.com [216.136.174.75]) by sucuri.mat.puc-rio.br (8.9.3/8.9.3) with SMTP id NAA27422 for ; Fri, 20 Sep 2002 13:11:50 -0300 Message-ID: <20020920161323.31513.qmail@web12908.mail.yahoo.com> Received: from [200.206.103.3] by web12908.mail.yahoo.com via HTTP; Fri, 20 Sep 2002 13:13:23 ART Date: Fri, 20 Sep 2002 13:13:23 -0300 (ART) From: =?iso-8859-1?q?Johann=20Peter=20Gustav=20Lejeune=20Dirichlet?= Subject: Re: [obm-l] QUESTÃO IME To: obm-l@mat.puc-rio.br In-Reply-To: <20020920120213.36287.qmail@web21305.mail.yahoo.com> MIME-Version: 1.0 Content-Type: multipart/alternative; boundary="0-148994060-1032538403=:30579" Content-Transfer-Encoding: 8bit Sender: owner-obm-l@sucuri.mat.puc-rio.br Precedence: bulk Reply-To: obm-l@mat.puc-rio.br --0-148994060-1032538403=:30579 Content-Type: text/plain; charset=iso-8859-1 Content-Transfer-Encoding: 8bit Jorge Paulino wrote: Oi galera, poderiam me ajudar na seguinte questão do IME... sqrt(5-sqrt(5-x))=x, para x>0 RESOLUÇAO:este problema ja caiu na lista.Eu ja dei uma soluçao e depois apareceram varias.Esta aqui e uma adaptaçao.Veja que x>0 por natureza.E 5-sqrt(5-x)=x^2,logo (5-x)-x^2=sqrt(5-x)-x,ou entao, (sqrt(5-x)-x)((sqrt(5-x)+x)-1)=0.Logo ou x=sqrt(5-x) ou x+sqrt(5-x)=1.Agora a questao ta de presente,faça as contas em casa. Ass.:Johann Um abraço, Jorge _______________________________________________________________________ Yahoo! GeoCities Tudo para criar o seu site: ferramentas fáceis de usar, espaço de sobra e acessórios. http://br.geocities.yahoo.com/ ========================================================================= Instruções para entrar na lista, sair da lista e usar a lista em http://www.mat.puc-rio.br/~nicolau/olimp/obm-l.html O administrador desta lista é ========================================================================= TRANSIRE SVVM PECTVS MVNDOQUE POTIRE CONGREGATI EX TOTO ORBE MATHEMATICI OB SCRIPTA INSIGNIA TRIBVERE Fields Medal(John Charles Fields) --------------------------------- Yahoo! GeoCities Tudo para criar o seu site: ferramentas fáceis de usar, espaço de sobra e acessórios. --0-148994060-1032538403=:30579 Content-Type: text/html; charset=iso-8859-1 Content-Transfer-Encoding: 8bit

 

 Jorge Paulino wrote:

Oi galera,
poderiam me ajudar na seguinte questão
do IME...

sqrt(5-sqrt(5-x))=x, para x>0
RESOLUÇAO:este problema ja caiu na lista.Eu ja dei uma soluçao e depois apareceram varias.Esta aqui e uma adaptaçao.Veja que x>0 por natureza.E 5-sqrt(5-x)=x^2,logo (5-x)-x^2=sqrt(5-x)-x,ou entao,                 (sqrt(5-x)-x)((sqrt(5-x)+x)-1)=0.Logo ou x=sqrt(5-x) ou x+sqrt(5-x)=1.Agora a questao ta de presente,faça as contas em casa.

Ass.:Johann


Um abraço,
Jorge

_______________________________________________________________________
Yahoo! GeoCities
Tudo para criar o seu site: ferramentas fáceis de usar, espaço de sobra e acessórios.
http://br.geocities.yahoo.com/
=========================================================================
Instruções para entrar na lista, sair da lista e usar a lista em
http://www.mat.puc-rio.br/~nicolau/olimp/obm-l.html
O administrador desta lista é
=========================================================================



TRANSIRE SVVM PECTVS MVNDOQUE POTIRE

CONGREGATI EX TOTO ORBE MATHEMATICI OB SCRIPTA INSIGNIA TRIBVERE

Fields Medal(John Charles Fields)



Yahoo! GeoCities
Tudo para criar o seu site: ferramentas fáceis de usar, espaço de sobra e acessórios. --0-148994060-1032538403=:30579-- ========================================================================= Instruções para entrar na lista, sair da lista e usar a lista em http://www.mat.puc-rio.br/~nicolau/olimp/obm-l.html O administrador desta lista é ========================================================================= From owner-obm-l@sucuri.mat.puc-rio.br Fri Sep 20 13:25:06 2002 Return-Path: Received: (from majordom@localhost) by sucuri.mat.puc-rio.br (8.9.3/8.9.3) id NAA27668 for obm-l-MTTP; Fri, 20 Sep 2002 13:23:40 -0300 Received: from web12905.mail.yahoo.com (web12905.mail.yahoo.com [216.136.174.72]) by sucuri.mat.puc-rio.br (8.9.3/8.9.3) with SMTP id NAA27663 for ; Fri, 20 Sep 2002 13:23:37 -0300 Message-ID: <20020920162509.51518.qmail@web12905.mail.yahoo.com> Received: from [200.206.103.3] by web12905.mail.yahoo.com via HTTP; Fri, 20 Sep 2002 13:25:09 ART Date: Fri, 20 Sep 2002 13:25:09 -0300 (ART) From: =?iso-8859-1?q?Johann=20Peter=20Gustav=20Lejeune=20Dirichlet?= Subject: Re: [obm-l] Questão_de_geometria To: obm-l@mat.puc-rio.br In-Reply-To: <002201c26017$35833220$2101a8c0@u2z7z2> MIME-Version: 1.0 Content-Type: multipart/alternative; boundary="0-1442578837-1032539109=:51060" Content-Transfer-Encoding: 8bit Sender: owner-obm-l@sucuri.mat.puc-rio.br Precedence: bulk Reply-To: obm-l@mat.puc-rio.br --0-1442578837-1032539109=:51060 Content-Type: text/plain; charset=iso-8859-1 Content-Transfer-Encoding: 8bit Fala serio meu!Isso tudo e a mesma coisa.VOCE NAO PARA DE FALAR NO MESMO ASSUNTO MAIS DE INFINITAS VEZES.MOROU? Wagner wrote:Oi pessoal ! Essa é uma pergunta de geometria um pouco chatinha, deêm uma olhada: Em um círculo existe um triângulo ABC retângulo inscrito nele. Esse triângulo possui uma circunferência circunscrita nele. Essa circunferência possui um triângulo DEF inscrito nela ,tal que AB é paralelo à DE, AC é paralelo à DF e BC é paralelo à EF. Esse triângulo também possui uma circûnferência circunscrita nele, tal que o padrão descrito acima se repete. Seja o triângulo XYZ limite desse padrão, tal que XY é paralelo à DE, XZ é paralelo à DF e YZ é paralelo à EF. Logo, quanto vale a soma das distâncias AX, AY e AZ em funcão de BÂC e do raio da circunferência maior? Dado: AB e BC são perpendiculares Se alguém tiver noção de como resolver essa pergunta já agradeço. André T. --------------------------------- Yahoo! GeoCities Tudo para criar o seu site: ferramentas fáceis de usar, espaço de sobra e acessórios. --0-1442578837-1032539109=:51060 Content-Type: text/html; charset=iso-8859-1 Content-Transfer-Encoding: 8bit

Fala serio meu!Isso tudo e a mesma coisa.VOCE NAO PARA DE FALAR NO MESMO ASSUNTO MAIS DE INFINITAS VEZES.MOROU?

 Wagner wrote:

Oi pessoal !
 
Essa é uma pergunta de geometria um pouco chatinha, deêm uma olhada:
 
    Em um círculo existe um triângulo ABC retângulo inscrito nele. Esse triângulo possui uma circunferência circunscrita nele. Essa circunferência possui um triângulo DEF inscrito nela ,tal que AB é paralelo à DE, AC é paralelo à DF e BC é paralelo à EF. Esse triângulo  também possui uma circûnferência circunscrita nele, tal que o padrão descrito acima se repete. Seja o triângulo XYZ limite desse padrão, tal que XY é paralelo à DE, XZ é paralelo à DF e YZ é paralelo à EF. Logo, quanto vale a soma das distâncias AX, AY e AZ em funcão de BÂC e do raio da circunferência maior?
 
Dado: AB e BC são perpendiculares
 
Se alguém tiver noção de como resolver essa pergunta já agradeço.
 
André T.
 



Yahoo! GeoCities
Tudo para criar o seu site: ferramentas fáceis de usar, espaço de sobra e acessórios. --0-1442578837-1032539109=:51060-- ========================================================================= Instruções para entrar na lista, sair da lista e usar a lista em http://www.mat.puc-rio.br/~nicolau/olimp/obm-l.html O administrador desta lista é ========================================================================= From owner-obm-l@sucuri.mat.puc-rio.br Fri Sep 20 13:56:07 2002 Return-Path: Received: (from majordom@localhost) by sucuri.mat.puc-rio.br (8.9.3/8.9.3) id NAA28687 for obm-l-MTTP; Fri, 20 Sep 2002 13:54:18 -0300 Received: from smtp-33.ig.com.br (smtp-33.ig.com.br [200.226.132.183]) by sucuri.mat.puc-rio.br (8.9.3/8.9.3) with SMTP id NAA28683 for ; Fri, 20 Sep 2002 13:54:15 -0300 Received: (qmail 26460 invoked from network); 20 Sep 2002 16:55:25 -0000 Received: from 200-163-25-135-bsace7026.dsl.telebrasilia.net.br (HELO henrique) (200.163.25.135) by smtp-33.ig.com.br with SMTP; 20 Sep 2002 16:55:25 -0000 Message-ID: <001d01c260c6$8999bbe0$019da8c0@henrique> From: "Henrique Branco" To: References: <20020920120213.36287.qmail@web21305.mail.yahoo.com> Subject: [obm-l] =?iso-8859-1?Q?Re:_=5Bobm-l=5D_QUEST=C3O_IME?= Date: Fri, 20 Sep 2002 13:55:33 -0300 MIME-Version: 1.0 Content-Type: text/plain; charset="iso-8859-1" Content-Transfer-Encoding: 8bit X-Priority: 3 X-MSMail-Priority: Normal X-Mailer: Microsoft Outlook Express 6.00.2600.0000 X-MimeOLE: Produced By Microsoft MimeOLE V6.00.2600.0000 Sender: owner-obm-l@sucuri.mat.puc-rio.br Precedence: bulk Reply-To: obm-l@mat.puc-rio.br > Oi galera, > poderiam me ajudar na seguinte questão > do IME... > > sqrt(5-sqrt(5-x))=x, para x>0 Eleve ao quadrado dos dois lados. Fica 5-sqrt(5-x)=x^2. Arranjando convenientemente a equação, ficamos com sqrt(5-x)=5-x^2. Vemos que y=5-x^2 é a função inversa de y=sqrt(5-x) sendo, portanto, as duas simétricas à reta y=x e com seus os pontos de intersecção nessa reta. Aí é só resolver y=5-x^2 com y=x. Abraço, Henrique. ========================================================================= Instruções para entrar na lista, sair da lista e usar a lista em http://www.mat.puc-rio.br/~nicolau/olimp/obm-l.html O administrador desta lista é ========================================================================= From owner-obm-l@sucuri.mat.puc-rio.br Fri Sep 20 16:35:50 2002 Return-Path: Received: (from majordom@localhost) by sucuri.mat.puc-rio.br (8.9.3/8.9.3) id QAA32003 for obm-l-MTTP; Fri, 20 Sep 2002 16:34:50 -0300 Received: from web12904.mail.yahoo.com (web12904.mail.yahoo.com [216.136.174.71]) by sucuri.mat.puc-rio.br (8.9.3/8.9.3) with SMTP id QAA31999 for ; Fri, 20 Sep 2002 16:34:47 -0300 Message-ID: <20020920193621.78014.qmail@web12904.mail.yahoo.com> Received: from [200.206.103.3] by web12904.mail.yahoo.com via HTTP; Fri, 20 Sep 2002 16:36:21 ART Date: Fri, 20 Sep 2002 16:36:21 -0300 (ART) From: =?iso-8859-1?q?Johann=20Peter=20Gustav=20Lejeune=20Dirichlet?= Subject: Re: [obm-l] Re:_[obm-l]_QUESTÃO_IME To: obm-l@mat.puc-rio.br In-Reply-To: <001d01c260c6$8999bbe0$019da8c0@henrique> MIME-Version: 1.0 Content-Type: multipart/alternative; boundary="0-1867202534-1032550581=:77242" Content-Transfer-Encoding: 8bit Sender: owner-obm-l@sucuri.mat.puc-rio.br Precedence: bulk Reply-To: obm-l@mat.puc-rio.br --0-1867202534-1032550581=:77242 Content-Type: text/plain; charset=iso-8859-1 Content-Transfer-Encoding: 8bit No fim das contas e a minha soluçao com graficos. Henrique Branco wrote:> Oi galera, > poderiam me ajudar na seguinte questão > do IME... > > sqrt(5-sqrt(5-x))=x, para x>0 Eleve ao quadrado dos dois lados. Fica 5-sqrt(5-x)=x^2. Arranjando convenientemente a equação, ficamos com sqrt(5-x)=5-x^2. Vemos que y=5-x^2 é a função inversa de y=sqrt(5-x) sendo, portanto, as duas simétricas à reta y=x e com seus os pontos de intersecção nessa reta. Aí é só resolver y=5-x^2 com y=x. Abraço, Henrique. ========================================================================= Instruções para entrar na lista, sair da lista e usar a lista em http://www.mat.puc-rio.br/~nicolau/olimp/obm-l.html O administrador desta lista é ========================================================================= --------------------------------- Yahoo! GeoCities Tudo para criar o seu site: ferramentas fáceis de usar, espaço de sobra e acessórios. --0-1867202534-1032550581=:77242 Content-Type: text/html; charset=iso-8859-1 Content-Transfer-Encoding: 8bit

No fim das contas e a minha soluçao com graficos.

 Henrique Branco wrote:

> Oi galera,
> poderiam me ajudar na seguinte questão
> do IME...
>
> sqrt(5-sqrt(5-x))=x, para x>0

Eleve ao quadrado dos dois lados. Fica 5-sqrt(5-x)=x^2. Arranjando
convenientemente a equação, ficamos com sqrt(5-x)=5-x^2. Vemos que y=5-x^2 é
a função inversa de y=sqrt(5-x) sendo, portanto, as duas simétricas à reta
y=x e com seus os pontos de intersecção nessa reta. Aí é só resolver y=5-x^2
com y=x.
Abraço,
Henrique.


=========================================================================
Instruções para entrar na lista, sair da lista e usar a lista em
http://www.mat.puc-rio.br/~nicolau/olimp/obm-l.html
O administrador desta lista é
=========================================================================



Yahoo! GeoCities
Tudo para criar o seu site: ferramentas fáceis de usar, espaço de sobra e acessórios. --0-1867202534-1032550581=:77242-- ========================================================================= Instruções para entrar na lista, sair da lista e usar a lista em http://www.mat.puc-rio.br/~nicolau/olimp/obm-l.html O administrador desta lista é ========================================================================= From owner-obm-l@sucuri.mat.puc-rio.br Fri Sep 20 18:51:02 2002 Return-Path: Received: (from majordom@localhost) by sucuri.mat.puc-rio.br (8.9.3/8.9.3) id SAA01702 for obm-l-MTTP; Fri, 20 Sep 2002 18:50:11 -0300 Received: from traven.pub1 (traven.uol.com.br [200.221.4.39]) by sucuri.mat.puc-rio.br (8.9.3/8.9.3) with ESMTP id SAA01698 for ; Fri, 20 Sep 2002 18:50:08 -0300 Received: from u2z7z2 ([200.158.144.141]) by traven.pub1 (8.9.1/8.9.1) with ESMTP id SAA02275 for ; Fri, 20 Sep 2002 18:38:52 -0300 (BRT) Message-ID: <001c01c260f0$15304e20$2101a8c0@u2z7z2> From: "Wagner" To: References: <002201c26017$35833220$2101a8c0@u2z7z2> <3D8A80A4.3030100@centroin.com.br> Subject: [obm-l] =?iso-8859-1?Q?Re:_=5Bobm-l=5D_Quest=E3o_de_geometria_=28ERRATA=29?= Date: Fri, 20 Sep 2002 18:52:56 -0300 Organization: Wagner MIME-Version: 1.0 Content-Type: multipart/alternative; boundary="----=_NextPart_000_0019_01C260D6.EF19ABA0" X-Priority: 3 X-MSMail-Priority: Normal X-Mailer: Microsoft Outlook Express 5.50.4133.2400 X-MimeOLE: Produced By Microsoft MimeOLE V5.50.4133.2400 Sender: owner-obm-l@sucuri.mat.puc-rio.br Precedence: bulk Reply-To: obm-l@mat.puc-rio.br This is a multi-part message in MIME format. ------=_NextPart_000_0019_01C260D6.EF19ABA0 Content-Type: text/plain; charset="iso-8859-1" Content-Transfer-Encoding: quoted-printable Oi pessoal ! Me desculpem, foi um erro de digita=E7=E3o, o tri=E2ngulo DEF n=E3o = possui uma circunfer=EAncia circunscrita nele e sim uma circunfer=EAncia = tal que todos os lados DEF sejam tangentes =E0 ela (me desculpem pensava = que era isso que significava circunscrito). Vejam se conseguem resolver o problema Andr=E9 T. ----- Original Message -----=20 From: Augusto C=E9sar Morgado=20 To: obm-l@mat.puc-rio.br=20 Sent: Thursday, September 19, 2002 10:57 PM Subject: Re: [obm-l] Quest=E3o de geometria Wagner wrote: Oi pessoal ! Essa =E9 uma pergunta de geometria um pouco chatinha, de=EAm uma = olhada: Em um c=EDrculo existe um tri=E2ngulo ABC ret=E2ngulo inscrito = nele. Esse tri=E2ngulo possui uma circunfer=EAncia circunscrita nele.OU = SEJA, ESTA CIRCUNFERENCIA EH A CIRCUNFERENCIA ORIGINAL Essa = circunfer=EAncia possui um tri=E2ngulo DEF inscrito nela ,tal que AB =E9 = paralelo =E0 DE, AC =E9 paralelo =E0 DF e BC =E9 paralelo =E0 EF.OU SEJA = ESSE TRIANGULO EH IGUAL AO ORIGINAL Esse tri=E2ngulo tamb=E9m possui = uma circ=FBnfer=EAncia circunscrita nele, tal que o padr=E3o descrito = acima se repete.OU SEJA EH A CIRCUNFERENCIA ORIGINAL ATACANDO = NOVAMENTE.Seja o tri=E2ngulo XYZ limite desse padr=E3o, tal que XY =E9 = paralelo =E0 DE, XZ =E9 paralelo =E0 DF e YZ =E9 paralelo =E0 EF.EH O = TRIANGULO ORIGINAL DE NOVO. Logo, quanto vale a soma das dist=E2ncias = AX, AY e AZ em func=E3o de B=C2C e do raio da circunfer=EAncia maior? Dado: AB e BC s=E3o perpendiculares Se algu=E9m tiver no=E7=E3o de como resolver essa pergunta j=E1 = agrade=E7o. Andr=E9 T. ------=_NextPart_000_0019_01C260D6.EF19ABA0 Content-Type: text/html; charset="iso-8859-1" Content-Transfer-Encoding: quoted-printable
Oi pessoal !
 
Me desculpem, foi um erro de = digita=E7=E3o, o tri=E2ngulo=20 DEF n=E3o possui uma circunfer=EAncia circunscrita nele e sim uma=20 circunfer=EAncia tal que todos os lados DEF sejam tangentes =E0 ela (me = desculpem=20 pensava que era isso que significava circunscrito).
 
Vejam se conseguem resolver o = problema
 
Andr=E9 T.
 
 
----- Original Message -----
From:=20 Augusto=20 C=E9sar Morgado
Sent: Thursday, September 19, = 2002 10:57=20 PM
Subject: Re: [obm-l] Quest=E3o = de=20 geometria



Wagner wrote:
Oi pessoal !
 
Essa =E9 uma pergunta de geometria = um pouco=20 chatinha, de=EAm uma olhada:
 
    Em um c=EDrculo = existe um=20 tri=E2ngulo ABC ret=E2ngulo inscrito nele. Esse tri=E2ngulo possui = uma=20 circunfer=EAncia circunscrita nele.OU SEJA, ESTA CIRCUNFERENCIA EH A = CIRCUNFERENCIA ORIGINAL Essa circunfer=EAncia possui um tri=E2ngulo = DEF inscrito=20 nela ,tal que AB =E9 paralelo =E0 DE, AC =E9 paralelo =E0 DF e BC = =E9 paralelo =E0 EF.OU=20 SEJA ESSE TRIANGULO EH IGUAL AO ORIGINAL Esse tri=E2ngulo  = tamb=E9m possui=20 uma circ=FBnfer=EAncia circunscrita nele, tal que o padr=E3o = descrito acima se=20 repete.OU SEJA EH A CIRCUNFERENCIA ORIGINAL ATACANDO NOVAMENTE.Seja = o=20 tri=E2ngulo XYZ limite desse padr=E3o, tal que XY =E9 paralelo =E0 = DE, XZ =E9 paralelo=20 =E0 DF e YZ =E9 paralelo =E0 EF.EH O TRIANGULO ORIGINAL DE NOVO. = Logo, quanto vale=20 a soma das dist=E2ncias AX, AY e AZ em func=E3o de B=C2C e do raio = da=20 circunfer=EAncia maior?
 
Dado: AB e BC s=E3o = perpendiculares
 
Se algu=E9m tiver no=E7=E3o de como = resolver essa=20 pergunta j=E1 agrade=E7o.
 
Andr=E9 T.
 

------=_NextPart_000_0019_01C260D6.EF19ABA0-- ========================================================================= Instruções para entrar na lista, sair da lista e usar a lista em http://www.mat.puc-rio.br/~nicolau/olimp/obm-l.html O administrador desta lista é ========================================================================= From owner-obm-l@sucuri.mat.puc-rio.br Fri Sep 20 18:55:11 2002 Return-Path: Received: (from majordom@localhost) by sucuri.mat.puc-rio.br (8.9.3/8.9.3) id SAA01820 for obm-l-MTTP; Fri, 20 Sep 2002 18:55:08 -0300 Received: from traven.pub1 (traven.uol.com.br [200.221.4.39]) by sucuri.mat.puc-rio.br (8.9.3/8.9.3) with ESMTP id SAA01816 for ; Fri, 20 Sep 2002 18:55:05 -0300 Received: from u2z7z2 ([200.158.144.141]) by traven.pub1 (8.9.1/8.9.1) with ESMTP id SAA06711 for ; Fri, 20 Sep 2002 18:43:49 -0300 (BRT) Message-ID: <005301c260f0$c89f5d20$2101a8c0@u2z7z2> From: "Wagner" To: Subject: [obm-l] =?iso-8859-1?Q?Pergunta_de_polin=F4mios?= Date: Fri, 20 Sep 2002 18:57:59 -0300 Organization: Wagner MIME-Version: 1.0 Content-Type: multipart/alternative; boundary="----=_NextPart_000_0050_01C260D7.A3309360" X-Priority: 3 X-MSMail-Priority: Normal X-Mailer: Microsoft Outlook Express 5.50.4133.2400 X-MimeOLE: Produced By Microsoft MimeOLE V5.50.4133.2400 Sender: owner-obm-l@sucuri.mat.puc-rio.br Precedence: bulk Reply-To: obm-l@mat.puc-rio.br This is a multi-part message in MIME format. ------=_NextPart_000_0050_01C260D7.A3309360 Content-Type: text/plain; charset="iso-8859-1" Content-Transfer-Encoding: quoted-printable Ola pessoal! Quem conseguir resolver esse problema realmente merece o t=EDtulo de = g=EAnio do ano: -Nota=E7=E3o: a^b=3D a elevado a b Seja o polinomio p(x) =3D x^(2n) + x^(2n-2) + x^(2n-4) + ... + x^2 + 1 - = (2x^n)(x^(n-1) - x^(n-2) + x^(n-3) - x^(n-4) + ... + x - 1) - = (2x^(n-1))(x^(n-2) - x^(n-3) + ... + 1) - (2x^(n-2))(x^(n-3) - x^(n-4) + = ... - 1) - (2x^(n-3))(...) + ... - 2x( 1 ). Prove que se ((n-1)/2) =E9 um n=FAmero natural, p(x) possui exatamente 2 = ra=EDzes reais e pelo menos 2 ra=EDzes coincidentes. Prove tamb=E9m que = todos os coeficientes do polin=F4mio seguem um padr=E3o para esses = valores de n. Andr=E9 T. ------=_NextPart_000_0050_01C260D7.A3309360 Content-Type: text/html; charset="iso-8859-1" Content-Transfer-Encoding: quoted-printable
Ola pessoal!
 
Quem conseguir resolver esse problema = realmente=20 merece o t=EDtulo de g=EAnio do ano:
 
-Nota=E7=E3o: a^b=3D a elevado a = b
 
Seja o polinomio p(x) =3D x^(2n) + = x^(2n-2) +=20 x^(2n-4) + ... + x^2 + 1 - (2x^n)(x^(n-1) - x^(n-2) + x^(n-3) - x^(n-4) = + ... +=20 x - 1) - (2x^(n-1))(x^(n-2) - x^(n-3) + ... + 1) - (2x^(n-2))(x^(n-3) - = x^(n-4)=20 + ... - 1) - (2x^(n-3))(...) + ... - 2x( 1 ).
 
Prove que se ((n-1)/2) =E9 um n=FAmero = natural, p(x)=20 possui exatamente 2 ra=EDzes reais e pelo menos 2 ra=EDzes coincidentes. = Prove=20 tamb=E9m que todos os coeficientes do polin=F4mio seguem um padr=E3o = para esses=20 valores de n.
 
 
Andr=E9 = T.
------=_NextPart_000_0050_01C260D7.A3309360-- ========================================================================= Instruções para entrar na lista, sair da lista e usar a lista em http://www.mat.puc-rio.br/~nicolau/olimp/obm-l.html O administrador desta lista é ========================================================================= From owner-obm-l@sucuri.mat.puc-rio.br Fri Sep 20 19:05:13 2002 Return-Path: Received: (from majordom@localhost) by sucuri.mat.puc-rio.br (8.9.3/8.9.3) id TAA02135 for obm-l-MTTP; Fri, 20 Sep 2002 19:04:44 -0300 Received: from hotmail.com (f78.sea2.hotmail.com [207.68.165.78]) by sucuri.mat.puc-rio.br (8.9.3/8.9.3) with ESMTP id TAA02131 for ; Fri, 20 Sep 2002 19:04:41 -0300 Received: from mail pickup service by hotmail.com with Microsoft SMTPSVC; Fri, 20 Sep 2002 15:06:15 -0700 Received: from 32.94.119.254 by sea2fd.sea2.hotmail.msn.com with HTTP; Fri, 20 Sep 2002 22:06:15 GMT X-Originating-IP: [32.94.119.254] From: "Paulo Santa Rita" To: obm-l@mat.puc-rio.br Subject: [obm-l] =?iso-8859-1?B?UmU6IFtvYm0tbF0gU2Vy4SBxdWUgbmluZ3XpbSBtZSBhanVkYT8/Pw==?= Date: Fri, 20 Sep 2002 22:06:15 +0000 Mime-Version: 1.0 Content-Type: text/plain; charset=iso-8859-1; format=flowed Message-ID: X-OriginalArrivalTime: 20 Sep 2002 22:06:15.0492 (UTC) FILETIME=[F04F7840:01C260F1] Sender: owner-obm-l@sucuri.mat.puc-rio.br Precedence: bulk Reply-To: obm-l@mat.puc-rio.br Ola TITULAR DE RDRQL@MSN.COM e demais colegas desta lista ... OBM-L, O fato e que nao ha alguem que queira lhe ajudar simplesmente porque o problema que voce postou esta mal formulado ... Voce se refere a um periodo de 10 anos mas e absolutamente imprescindivel que voce informe o ano exato em que comeca este periodo, sob pena de nao haver uma resposta satisfatoria. O problema esta com os anos bissextos. Um ano e bissexto se ele e divisivel por 4 mas nao por 100, exceto nos casos em que tambem for divisivel por 400. Isto implica que num intervalo arbitrario de 10 anos pode ocorrer um ou dois anos bissextos, SENDO IMPOSSIVEL SABERMOS PREVIAMENTE se ocorrera um caso ou outro caso nao saibamos o ano em que comeca a contagem de 10 anos. Entendeu ? Esta ausencia de uma informacao crucial no seu problema torna inviavel uma resposta precisa. Se o construtor do programa de Biorritmo nao percebeu isso, o programa e muito pobre. Um abraco Paulo Santa Rita 6,1905,200902 >From: "e isso mesmo" >Reply-To: obm-l@mat.puc-rio.br >To: >Subject: [obm-l] Será que ninguém me ajuda??? >Date: Thu, 19 Sep 2002 10:32:10 -0300 > > >Companheiros continuo esperando ajuda de alguèm.... > >"A teoria do Biorritmo diz que os estados físico, mental e emocional de uma >pessoa oscilam periodicamente, a partir do dia do nascimento, em ciclos de >23 dias, 29 dias e 33 dias, respectivamente. Dado que os dias mais >positivos dos ciclos físico, mental e emocional são, respectivamente, o >sexto, o sétimo e o oitavo de cada ciclo, nos primeiros dez anos de vida de >uma pessoa, quantas vezes os três ciclos estão simultaneamente no ponto >máximo?" > >Tchau! > Aproveite melhor a Web. Faça o download GRÁTIS do MSN Explorer : >http://explorer.msn.com.br/intl.asp#po ========================================================================= Instruções para entrar na lista, sair da lista e usar a lista em http://www.mat.puc-rio.br/~nicolau/olimp/obm-l.html O administrador desta lista é ========================================================================= From owner-obm-l@sucuri.mat.puc-rio.br Sat Sep 21 14:30:07 2002 Return-Path: Received: (from majordom@localhost) by sucuri.mat.puc-rio.br (8.9.3/8.9.3) id OAA09359 for obm-l-MTTP; Sat, 21 Sep 2002 14:29:12 -0300 Received: from shen.bol.com.br (shen.bol.com.br [200.221.24.14]) by sucuri.mat.puc-rio.br (8.9.3/8.9.3) with ESMTP id OAA09355 for ; Sat, 21 Sep 2002 14:29:10 -0300 Received: from bol.com.br (200.221.24.138) by shen.bol.com.br (5.1.071) id 3D8BAD7300031875 for obm-l@mat.puc-rio.br; Sat, 21 Sep 2002 14:30:00 -0300 Date: Sat, 21 Sep 2002 14:28:22 -0300 Message-Id: Subject: [obm-l] combinatoria MIME-Version: 1.0 Content-Type: text/plain;charset="iso-8859-1" From: "adr.scr.m" To: obm-l@mat.puc-rio.br X-XaM3-API-Version: 2.4.3.4.4 X-SenderIP: 200.151.62.104 Content-Transfer-Encoding: 8bit X-MIME-Autoconverted: from quoted-printable to 8bit by sucuri.mat.puc-rio.br id OAA09356 Sender: owner-obm-l@sucuri.mat.puc-rio.br Precedence: bulk Reply-To: obm-l@mat.puc-rio.br estou com uma duvida nessa questao,porque eu fiz usando o 1ºlema de Kaplansky,e meu professor disse que nao pode: (IME-75/76) Considere uma turma com n alunos,numerados de 1 a n.Deseja-se organizar uma comissao de 3 alunos.De quantas maneiras pode ser formada esta comissao,de modo que nao facam parte da mesma 2 ou 3 alunos designados por numeros consecutivos ? []'s. Adriano. __________________________________________________________________________ Encontre sempre uma linha desocupada com o Discador BOL! http://www.bol.com.br/discador Ainda não tem AcessoBOL? Assine já! http://www.bol.com.br/acessobol ========================================================================= Instruções para entrar na lista, sair da lista e usar a lista em http://www.mat.puc-rio.br/~nicolau/olimp/obm-l.html O administrador desta lista é ========================================================================= From owner-obm-l@sucuri.mat.puc-rio.br Sat Sep 21 15:18:04 2002 Return-Path: Received: (from majordom@localhost) by sucuri.mat.puc-rio.br (8.9.3/8.9.3) id PAA10217 for obm-l-MTTP; Sat, 21 Sep 2002 15:17:54 -0300 Received: (from nicolau@localhost) by sucuri.mat.puc-rio.br (8.9.3/8.9.3) id PAA10212 for obm-l@mat.puc-rio.br; Sat, 21 Sep 2002 15:17:53 -0300 Date: Sat, 21 Sep 2002 15:17:53 -0300 From: "Nicolau C. Saldanha" To: obm-l@mat.puc-rio.br Subject: Re: [obm-l] combinatoria Message-ID: <20020921151753.A10100@sucuri.mat.puc-rio.br> References: Mime-Version: 1.0 Content-Type: text/plain; charset=iso-8859-1 Content-Disposition: inline Content-Transfer-Encoding: 8bit User-Agent: Mutt/1.2.5i In-Reply-To: ; from adr.scr.m@bol.com.br on Sat, Sep 21, 2002 at 02:28:22PM -0300 Sender: owner-obm-l@sucuri.mat.puc-rio.br Precedence: bulk Reply-To: obm-l@mat.puc-rio.br On Sat, Sep 21, 2002 at 02:28:22PM -0300, adr.scr.m wrote: > estou com uma duvida nessa questao,porque eu > fiz usando o 1ºlema de Kaplansky,e meu > professor disse que nao pode: > (IME-75/76) Considere uma turma com n > alunos,numerados de 1 a n.Deseja-se > organizar uma comissao de 3 alunos.De > quantas maneiras pode ser formada esta > comissao,de modo que nao facam parte da > mesma 2 ou 3 alunos designados por numeros > consecutivos ? > []'s. > Adriano. Eu contaria todas as comissões, o que dá binomial(n,3) = n(n-1)(n-2)/6, e depois excluiria as da forma j,k,k+1. Para contá-las, eu primeiro escolho k (temos (n-1) valores possíveis) e depois j ((n-2) valores possíveis), o que dá (n-1)(n-2); note entretanto que as n-2 comissões da forma i,i+1,i+2 foram eliminadas duas vezes (j=i,k=i+1 e j=i+2,k=i) e isso deve ser corrigido. Assim, temos n(n-1)(n-2)/6 - (n-1)(n-2) + (n-2) = (n-2)(n(n-1) - 6(n-2))/6 o que dá os valores certos para n=2,3,4 (0), n=5 (1) e n=6 (4). Como a resposta obviamente é um polinômio de grau <= 3 estes 5 testes são bem convincentes. :-) []s, N. ========================================================================= Instruções para entrar na lista, sair da lista e usar a lista em http://www.mat.puc-rio.br/~nicolau/olimp/obm-l.html O administrador desta lista é ========================================================================= From owner-obm-l@sucuri.mat.puc-rio.br Sat Sep 21 15:22:01 2002 Return-Path: Received: (from majordom@localhost) by sucuri.mat.puc-rio.br (8.9.3/8.9.3) id PAA10293 for obm-l-MTTP; Sat, 21 Sep 2002 15:22:00 -0300 Received: from hotmail.com (f136.law4.hotmail.com [216.33.149.136]) by sucuri.mat.puc-rio.br (8.9.3/8.9.3) with ESMTP id PAA10289 for ; Sat, 21 Sep 2002 15:21:58 -0300 Received: from mail pickup service by hotmail.com with Microsoft SMTPSVC; Sat, 21 Sep 2002 11:23:35 -0700 Received: from 200.163.7.178 by lw4fd.law4.hotmail.msn.com with HTTP; Sat, 21 Sep 2002 18:23:35 GMT X-Originating-IP: [200.163.7.178] From: "rafael dowsley" To: obm-l@mat.puc-rio.br Subject: [obm-l] Geometria Date: Sat, 21 Sep 2002 18:23:35 +0000 Mime-Version: 1.0 Content-Type: text/plain; charset=iso-8859-1; format=flowed Message-ID: X-OriginalArrivalTime: 21 Sep 2002 18:23:35.0631 (UTC) FILETIME=[FFA035F0:01C2619B] Sender: owner-obm-l@sucuri.mat.puc-rio.br Precedence: bulk Reply-To: obm-l@mat.puc-rio.br Onde posso encontrar um material bom sobre geometria(com demonstrações dos principais teoremas)? Rafael Baião Dowsley _________________________________________________________________ Tenha você também um MSN Hotmail, o maior webmail do mundo: http://www.hotmail.com/br ========================================================================= Instruções para entrar na lista, sair da lista e usar a lista em http://www.mat.puc-rio.br/~nicolau/olimp/obm-l.html O administrador desta lista é ========================================================================= From owner-obm-l@sucuri.mat.puc-rio.br Sat Sep 21 17:48:37 2002 Return-Path: Received: (from majordom@localhost) by sucuri.mat.puc-rio.br (8.9.3/8.9.3) id RAA12066 for obm-l-MTTP; Sat, 21 Sep 2002 17:47:14 -0300 Received: from hotmail.com (f34.sea2.hotmail.com [207.68.165.34]) by sucuri.mat.puc-rio.br (8.9.3/8.9.3) with ESMTP id RAA12062 for ; Sat, 21 Sep 2002 17:47:11 -0300 Received: from mail pickup service by hotmail.com with Microsoft SMTPSVC; Sat, 21 Sep 2002 13:48:49 -0700 Received: from 200.220.36.213 by sea2fd.sea2.hotmail.msn.com with HTTP; Sat, 21 Sep 2002 20:48:45 GMT X-Originating-IP: [200.220.36.213] From: "leonardo mattos" To: obm-l@mat.puc-rio.br Subject: Re: [obm-l] Geometria Date: Sat, 21 Sep 2002 20:48:45 +0000 Mime-Version: 1.0 Content-Type: text/plain; charset=iso-8859-1; format=flowed Message-ID: X-OriginalArrivalTime: 21 Sep 2002 20:48:49.0601 (UTC) FILETIME=[498E8310:01C261B0] Sender: owner-obm-l@sucuri.mat.puc-rio.br Precedence: bulk Reply-To: obm-l@mat.puc-rio.br www.kalva.demon.co.uk/ Um abraço,Leonardo >From: "rafael dowsley" >Reply-To: obm-l@mat.puc-rio.br >To: obm-l@mat.puc-rio.br >Subject: [obm-l] Geometria >Date: Sat, 21 Sep 2002 18:23:35 +0000 > > >Onde posso encontrar um material bom sobre geometria(com demonstrações dos >principais teoremas)? > > >Rafael Baião Dowsley > > >_________________________________________________________________ >Tenha você também um MSN Hotmail, o maior webmail do mundo: >http://www.hotmail.com/br > >========================================================================= >Instruções para entrar na lista, sair da lista e usar a lista em >http://www.mat.puc-rio.br/~nicolau/olimp/obm-l.html >O administrador desta lista é >========================================================================= _________________________________________________________________ Converse com seus amigos online, faça o download grátis do MSN Messenger: http://messenger.msn.com.br ========================================================================= Instruções para entrar na lista, sair da lista e usar a lista em http://www.mat.puc-rio.br/~nicolau/olimp/obm-l.html O administrador desta lista é ========================================================================= From owner-obm-l@sucuri.mat.puc-rio.br Sat Sep 21 19:42:39 2002 Return-Path: Received: (from majordom@localhost) by sucuri.mat.puc-rio.br (8.9.3/8.9.3) id TAA13149 for obm-l-MTTP; Sat, 21 Sep 2002 19:41:04 -0300 Received: from traven10.pub1 (traven10.uol.com.br [200.221.4.45]) by sucuri.mat.puc-rio.br (8.9.3/8.9.3) with ESMTP id TAA13145 for ; Sat, 21 Sep 2002 19:41:02 -0300 Received: from u2z7z2 ([200.158.145.174]) by traven10.pub1 (8.9.1/8.9.1) with ESMTP id TAA11492 for ; Sat, 21 Sep 2002 19:41:56 -0300 (BRT) Message-ID: <000b01c261c0$5d1ea620$2101a8c0@u2z7z2> From: "Wagner" To: Subject: [obm-l] Probabilidade Date: Sat, 21 Sep 2002 19:43:50 -0300 Organization: Wagner MIME-Version: 1.0 Content-Type: multipart/alternative; boundary="----=_NextPart_000_0008_01C261A7.359FE820" X-Priority: 3 X-MSMail-Priority: Normal X-Mailer: Microsoft Outlook Express 5.50.4133.2400 X-MimeOLE: Produced By Microsoft MimeOLE V5.50.4133.2400 Sender: owner-obm-l@sucuri.mat.puc-rio.br Precedence: bulk Reply-To: obm-l@mat.puc-rio.br This is a multi-part message in MIME format. ------=_NextPart_000_0008_01C261A7.359FE820 Content-Type: text/plain; charset="iso-8859-1" Content-Transfer-Encoding: quoted-printable Oi para todos ! =C9 poss=EDvel descrever a probabilidade do evento abaixo em uma = f=F3rmula apenas? Uma caneta =E9 girada por uma pessoa de forma aleat=F3ria. Os = movimentos poss=EDveis da caneta s=E3o meia volta para a esquerda e meia = volta para a direita. Qual a probabilidade de que ap=F3s n movimentos = aleat=F3rios a caneta tenha feito pelo menos 2 voltas ou para esquerda = ou para a direita (considere apenas o balan=E7o final, ex: 2 voltas para = esquerda e meia volta para direita deve ser considerado como uma volta e = meia para a esquerda). Andr=E9 T. ------=_NextPart_000_0008_01C261A7.359FE820 Content-Type: text/html; charset="iso-8859-1" Content-Transfer-Encoding: quoted-printable
Oi para todos !
 
      =C9 = poss=EDvel descrever=20 a probabilidade do evento abaixo em uma f=F3rmula apenas?
 
    Uma caneta =E9 = girada por uma=20 pessoa de forma aleat=F3ria. Os movimentos poss=EDveis da caneta s=E3o = meia volta para=20 a esquerda e meia volta para a direita. Qual a probabilidade de que = ap=F3s n=20 movimentos aleat=F3rios a caneta tenha feito pelo menos 2 voltas ou para = esquerda=20 ou para a direita (considere apenas o balan=E7o final, ex: 2 voltas para = esquerda=20 e meia volta para direita deve ser considerado como uma volta e meia = para a=20 esquerda).
 
    Andr=E9=20 T.
------=_NextPart_000_0008_01C261A7.359FE820-- ========================================================================= Instruções para entrar na lista, sair da lista e usar a lista em http://www.mat.puc-rio.br/~nicolau/olimp/obm-l.html O administrador desta lista é ========================================================================= From owner-obm-l@sucuri.mat.puc-rio.br Sat Sep 21 20:43:52 2002 Return-Path: Received: (from majordom@localhost) by sucuri.mat.puc-rio.br (8.9.3/8.9.3) id UAA13926 for obm-l-MTTP; Sat, 21 Sep 2002 20:42:37 -0300 Received: from pina.terra.com.br (pina.terra.com.br [200.176.3.17]) by sucuri.mat.puc-rio.br (8.9.3/8.9.3) with ESMTP id UAA13922 for ; Sat, 21 Sep 2002 20:42:35 -0300 Received: from engenho.terra.com.br (engenho.terra.com.br [200.176.3.42]) by pina.terra.com.br (Postfix) with ESMTP id 287AE52EB5 for ; Sat, 21 Sep 2002 20:44:14 -0300 (EST) Received: from stabel (unknown [200.203.38.140]) (authenticated user dudasta) by engenho.terra.com.br (Postfix) with ESMTP id BC9C868106 for ; Sat, 21 Sep 2002 20:44:13 -0300 (EST) Message-ID: <000901c261c8$cbb7a980$0301a8c0@stabel> From: "Eduardo Casagrande Stabel" To: References: <000b01c261c0$5d1ea620$2101a8c0@u2z7z2> Subject: Re: [obm-l] Probabilidade Date: Sat, 21 Sep 2002 20:44:15 -0300 MIME-Version: 1.0 Content-Type: text/plain; charset="iso-8859-1" Content-Transfer-Encoding: 8bit X-Priority: 3 X-MSMail-Priority: Normal X-Mailer: Microsoft Outlook Express 6.00.2600.0000 X-MimeOLE: Produced By Microsoft MimeOLE V6.00.2600.0000 Sender: owner-obm-l@sucuri.mat.puc-rio.br Precedence: bulk Reply-To: obm-l@mat.puc-rio.br From: Wagner > >Oi para todos ! > > É possível descrever a probabilidade do evento abaixo em uma fórmula apenas? > > Uma caneta é girada por uma pessoa de forma aleatória. Os movimentos possíveis da caneta são >meia volta para a esquerda e meia volta para a direita. Qual a probabilidade de que após n >movimentos aleatórios a caneta tenha feito pelo menos 2 voltas ou para esquerda ou para a direita >(considere apenas o balanço final, ex: 2 voltas para esquerda e meia volta para direita deve ser >considerado como uma volta e meia para a esquerda). > > André T. Caro André T., gosto dos problemas que você envia à lista, eu os considero muito criativos. Este não parece ser difícil, mas é um pouco comprido. Vamos considerar, para não ficar tratando de 1/2 em 1/2, que cada giro dá 1 volta para direita ou para a esquerda, i.e., acrescente +1 ou -1 às voltas totais. Queremos saber qual a probabilidade de a soma de n parcelas "+1" ou "-1" retornar um dos números: -4, -3, -2, -1, 0, 1, 2, 3, 4 (de 4 voltas à esquerda à 4 voltas à direita). Se no total dos n giros, d foram à direita, se terá n - d à esquerda. Vamos separar em casos: CASO o resultado final seja 4 (ou -4). A soma de d à direita com n - d à esquerda (vezes -1) deve ser igual a 4, ou seja, d - (n - d) = 4 e d >=0 e n - d >=0. Quantas são as possibilidades? É preciso que 2d = 4 + n, ou seja, d = (4 - n)/2 que só é inteiro se n é par. Para que d >= 0 é preciso que n >= 4, n - d >= 0 vale também. Deve-se ter, de um total de n escolhas, (4 - n)/2 para a direita e o resto à esquerda. As possibilidades são COMB(n ; (4-n)/2). CASO o resultado final seja 3 (ou -3). De modo similar n >= 3 e precisa ser ímpar, as possibilidades são COMB(n ; (3-n)/2) CASO seja 2 (ou -2), n>=2 par e há COMB(n ; (2-n)/2) possibilidades. E geralmente CASO seja i (ou -i), n>=1 deve ter a paridade de i e há COMB(n ; (i -n)/2) possibilidades. Portanto se n for PAR >=4 o número de possibilidades de acabar em -4,-2,0,2 ou 4 é P(n) = 2*COMB(n ; (4-n)/2) + 2*COMB(n ; (2-n)/2) + COMB(n ; n/2). Se n for ÍMPAR >=3 o número de possibilidades de acabar em -3,-1,1 ou 3 é I(n) = 2*COMB(n ; (3-n)/2) + 2*COMB(n ; (1-n)/2) Para n >= 3 temos a quantidade total de P(n) se n é par e I(n) se n é impar. Como "juntar" essas fórmulas em uma só? Um jeito artificial é o seguinte. [(-1)^n + 1]/2 * P(n) - [(-1)^n - 1]/2 * I(n) Eduardo. Porto Alegre, RS. ========================================================================= Instruções para entrar na lista, sair da lista e usar a lista em http://www.mat.puc-rio.br/~nicolau/olimp/obm-l.html O administrador desta lista é ========================================================================= From owner-obm-l@sucuri.mat.puc-rio.br Sat Sep 21 21:41:03 2002 Return-Path: Received: (from majordom@localhost) by sucuri.mat.puc-rio.br (8.9.3/8.9.3) id VAA14691 for obm-l-MTTP; Sat, 21 Sep 2002 21:39:15 -0300 Received: from lampiao.digi.com.br (dial-up-a-nat-060.digi.com.br [200.241.100.60] (may be forged)) by sucuri.mat.puc-rio.br (8.9.3/8.9.3) with ESMTP id VAA14687 for ; Sat, 21 Sep 2002 21:39:13 -0300 Received: from benedito.digi.com.br (host116.d.digizap.com.br [200.249.6.116]) by lampiao.digi.com.br (8.11.2/8.11.6) with ESMTP id g8M0dk725543 for ; Sat, 21 Sep 2002 21:39:46 -0300 Message-Id: <5.1.0.14.0.20020921211916.009ea240@200.241.100.60> X-Sender: benedito@200.241.100.60 X-Mailer: QUALCOMM Windows Eudora Version 5.1 Date: Sat, 21 Sep 2002 21:28:53 -0300 To: obm-l@mat.puc-rio.br From: benedito Subject: [obm-l] Olimp.doRN In-Reply-To: Mime-Version: 1.0 Content-Type: multipart/mixed; x-avg-checked=avg-ok-16295C2A; boundary="=======69D54B68=======" X-MailScanner: Found to be clean Sender: owner-obm-l@sucuri.mat.puc-rio.br Precedence: bulk Reply-To: obm-l@mat.puc-rio.br --=======69D54B68======= Content-Type: text/plain; x-avg-checked=avg-ok-16295C2A; charset=iso-8859-1; format=flowed Content-Transfer-Encoding: quoted-printable Segue, abaixo, c=F3pia da prova (N=EDvel I) da Olimp=EDada de Matem=E1tica= do Rio=20 Grande do Norte. As dos outros n=EDveis, estou remetendo para Nelly estudar a possibilidade= de=20 coloc=E1-las na p=E1gina da OBM. Benedito XIII OLIMP=CDADA DE MATEM=C1TICA DO RIO GRANDE DO NORTE - ANO 2002 PROVA DA SEGUNDA ETAPA Em 21/09/2002 N=CDVEL I (5a e 6a S=E9ries) Problema 1 Um copo est=E1 cheio de =E1gua e seu peso =E9 de 520 gramas. Joga-se um= quinto=20 da =E1gua fora. Assim, o peso cai para 440 gramas. Qual =E9 o peso do copo vazio? Explique. Problema 2 Camila tem uma faixa de papel de comprimento igual a 2/3 de um metro.=20 Camila precisa de somente meio metro dessa faixa. Explique como Camila deve proceder para obter o que ela quer, se Camila=20 tem uma tesoura e n=E3o possui qualquer instrumento para medir e nem=20 material para colar. Problema 3 Numa corrida de pedestrianismo, Leonardo obteve exatamente a coloca=E7=E3o= do=20 meio dentre a coloca=E7=E3o final de todos os participantes. Andr=E9= colocou-se=20 abaixo de Leonardo, na d=E9cima posi=E7=E3o, e Thiago classificou-se em= d=E9cimo=20 sexto lugar. Qual =E9 n=FAmero m=EDnimo de corredores? Explique sua resposta. Problema 4 Numa escola, cada crian=E7a joga futebol ou t=EAnis. Um s=E9timo dos que= jogam=20 futebol jogam t=EAnis, e um nono dos que jogam t=EAnis tamb=E9m jogam= futebol. =C9 correto afirmar que mais da metade das crian=E7as joga t=EAnis?= Explique. --=======69D54B68======= Content-Type: text/plain; charset=us-ascii; x-avg=cert; x-avg-checked=avg-ok-16295C2A Content-Disposition: inline --- Outgoing mail is certified Virus Free. Checked by AVG anti-virus system (http://www.grisoft.com). Version: 6.0.385 / Virus Database: 217 - Release Date: 4/9/2002 --=======69D54B68=======-- ========================================================================= Instruções para entrar na lista, sair da lista e usar a lista em http://www.mat.puc-rio.br/~nicolau/olimp/obm-l.html O administrador desta lista é ========================================================================= From owner-obm-l@sucuri.mat.puc-rio.br Sat Sep 21 22:04:55 2002 Return-Path: Received: (from majordom@localhost) by sucuri.mat.puc-rio.br (8.9.3/8.9.3) id WAA15072 for obm-l-MTTP; Sat, 21 Sep 2002 22:03:21 -0300 Received: from lampiao.digi.com.br (dial-up-a-nat-060.digi.com.br [200.241.100.60] (may be forged)) by sucuri.mat.puc-rio.br (8.9.3/8.9.3) with ESMTP id WAA15068 for ; Sat, 21 Sep 2002 22:03:19 -0300 Received: from benedito.digi.com.br (host116.d.digizap.com.br [200.249.6.116]) by lampiao.digi.com.br (8.11.2/8.11.6) with ESMTP id g8M14O727325 for ; Sat, 21 Sep 2002 22:04:24 -0300 Message-Id: <5.1.0.14.0.20020921213355.009f8be0@200.241.100.60> X-Sender: benedito@200.241.100.60 X-Mailer: QUALCOMM Windows Eudora Version 5.1 Date: Sat, 21 Sep 2002 21:53:34 -0300 To: obm-l@mat.puc-rio.br From: benedito Subject: Re: [obm-l] Geometria In-Reply-To: Mime-Version: 1.0 Content-Type: multipart/mixed; x-avg-checked=avg-ok-16295C2A; boundary="=======2484794D=======" X-MailScanner: Found to be clean Sender: owner-obm-l@sucuri.mat.puc-rio.br Precedence: bulk Reply-To: obm-l@mat.puc-rio.br --=======2484794D======= Content-Type: text/plain; x-avg-checked=avg-ok-16295C2A; charset=iso-8859-1; format=flowed Content-Transfer-Encoding: quoted-printable A seguir, uma lista de bons livros de Geometria: 1. Geometria Euclidiana Plana - Jo=E3o Lucas M. Barbosa Cole=E7=E3o do Profesor de Matem=E1tica - SBM. 1995 2. Geometria - Vols. I e II - A. C. Morgado/Eduardo Wagner/M. Jorge Livraria Francisco Alves Editora S. A. 1974 3. Elementos de Geometria - Fr=E8re Ignace Chaput (FIC) F. Briguiet & Cia. 1954 4. Curso de Geometria - Paulo Ventura Ara=FAjo Gradiva. 1999 5. Geometry Revisited - H S M. Coxeter/S. L. Greitzer The Mathematical Association of America. 1967 6. Episodes in Nineteenth and Twentieth Century Euclidean Geometry Ross Honsberger The Mathematical Association of America . 1995 7. College Geometry - A Problem-Solving Approach with Applications - Gary=20 L. Musser Prentice Hall . 1994 8. Challenging Problems in Geometry - Alfred S Posamentier/Charles T.= Salkind Dover. 1988 9. Advanced Geometry - Alfred S Posamentier Springer. 2001 10. Notes on Geometry - Dan Br=E2nzei Editura Paralela 45. 1999 Benedito At 18:23 21/9/2002 +0000, you wrote: >Onde posso encontrar um material bom sobre geometria(com demonstra=E7=F5es= dos=20 >principais teoremas)? > > >Rafael Bai=E3o Dowsley > > >_________________________________________________________________ >Tenha voc=EA tamb=E9m um MSN Hotmail, o maior webmail do mundo:=20 >http://www.hotmail.com/br > >=3D=3D=3D=3D=3D=3D=3D=3D=3D=3D=3D=3D=3D=3D=3D=3D=3D=3D=3D=3D=3D=3D=3D=3D=3D= =3D=3D=3D=3D=3D=3D=3D=3D=3D=3D=3D=3D=3D=3D=3D=3D=3D=3D=3D=3D=3D=3D=3D=3D=3D= =3D=3D=3D=3D=3D=3D=3D=3D=3D=3D=3D=3D=3D=3D=3D=3D=3D=3D=3D=3D=3D=3D=3D >Instru=E7=F5es para entrar na lista, sair da lista e usar a lista em >http://www.mat.puc-rio.br/~nicolau/olimp/obm-l.html >O administrador desta lista =E9 >=3D=3D=3D=3D=3D=3D=3D=3D=3D=3D=3D=3D=3D=3D=3D=3D=3D=3D=3D=3D=3D=3D=3D=3D=3D= =3D=3D=3D=3D=3D=3D=3D=3D=3D=3D=3D=3D=3D=3D=3D=3D=3D=3D=3D=3D=3D=3D=3D=3D=3D= =3D=3D=3D=3D=3D=3D=3D=3D=3D=3D=3D=3D=3D=3D=3D=3D=3D=3D=3D=3D=3D=3D=3D > > > > >--- >Incoming mail is certified Virus Free. >Checked by AVG anti-virus system (http://www.grisoft.com). >Version: 6.0.385 / Virus Database: 217 - Release Date: 4/9/2002 --=======2484794D======= Content-Type: text/plain; charset=us-ascii; x-avg=cert; x-avg-checked=avg-ok-16295C2A Content-Disposition: inline --- Outgoing mail is certified Virus Free. Checked by AVG anti-virus system (http://www.grisoft.com). Version: 6.0.385 / Virus Database: 217 - Release Date: 4/9/2002 --=======2484794D=======-- ========================================================================= Instruções para entrar na lista, sair da lista e usar a lista em http://www.mat.puc-rio.br/~nicolau/olimp/obm-l.html O administrador desta lista é ========================================================================= From owner-obm-l@sucuri.mat.puc-rio.br Sun Sep 22 10:31:15 2002 Return-Path: Received: (from majordom@localhost) by sucuri.mat.puc-rio.br (8.9.3/8.9.3) id KAA21507 for obm-l-MTTP; Sun, 22 Sep 2002 10:29:53 -0300 Received: from traven10.pub1 (traven10.uol.com.br [200.221.4.45]) by sucuri.mat.puc-rio.br (8.9.3/8.9.3) with ESMTP id KAA21503 for ; Sun, 22 Sep 2002 10:29:51 -0300 Received: from xxx ([200.191.179.21]) by traven10.pub1 (8.9.1/8.9.1) with ESMTP id KAA13817 for ; Sun, 22 Sep 2002 10:30:40 -0300 (BRT) From: "haroldo" To: Subject: [obm-l] determinantes Date: Sun, 22 Sep 2002 10:33:45 -0300 Message-ID: <000001c2623c$ae5bb240$15b3bfc8@xxx> MIME-Version: 1.0 Content-Type: multipart/alternative; boundary="----=_NextPart_000_0001_01C26223.890E7A40" X-Priority: 3 (Normal) X-MSMail-Priority: Normal X-Mailer: Microsoft Outlook, Build 10.0.2627 Importance: Normal X-MimeOLE: Produced By Microsoft MimeOLE V5.00.2919.6600 Sender: owner-obm-l@sucuri.mat.puc-rio.br Precedence: bulk Reply-To: obm-l@mat.puc-rio.br This is a multi-part message in MIME format. ------=_NextPart_000_0001_01C26223.890E7A40 Content-Type: text/plain; charset="iso-8859-1" Content-Transfer-Encoding: quoted-printable Gostaria de ajuda no c=E1lculo desse determinante: =20 =20 1 1/2 1/3 .... 1/n 1/2 1/3 1/4 .... 1/(n+1) 1/3 1/4 1/5 ... 1/(n+2) .... .................................. ...................................... 1/n 1/(n+1) ... ..... 1/(2n-1) =20 =20 grato=20 HAROLDO. ------=_NextPart_000_0001_01C26223.890E7A40 Content-Type: text/html; charset="iso-8859-1" Content-Transfer-Encoding: quoted-printable

Gostaria de ajuda no c=E1lculo desse = determinante:

 

 

=A0=A0=A0=A0=A0=A0=A0=A0 1=A0=A0 = =A0=A01/2 =A0=A0=A01/3 =A0=A0....=A0=A0 1/n

=A0=A0=A0=A0=A0=A0=A0 1/2 =A0=A01/3=A0=A0=A0 1/4 =A0....=A0=A0=A0 = 1/(n+1)

=A0=A0=A0=A0=A0=A0 = =A01/3=A0 =A01/4 =A0=A0=A01/5=A0=A0=A0 ...=A0 =A01/(n+2)

=

=A0=A0=A0=A0=A0=A0 = ....=A0 = ..................................

=A0=A0=A0=A0=A0=A0=A0 = ......................................

=A0=A0=A0=A0=A0=A0 = =A01/n=A0=A0 1/(n+1)=A0 ... ..... =A01/(2n-1)

 

 

grato

HAROLDO.

------=_NextPart_000_0001_01C26223.890E7A40-- ========================================================================= Instruções para entrar na lista, sair da lista e usar a lista em http://www.mat.puc-rio.br/~nicolau/olimp/obm-l.html O administrador desta lista é ========================================================================= From owner-obm-l@sucuri.mat.puc-rio.br Sun Sep 22 11:32:13 2002 Return-Path: Received: (from majordom@localhost) by sucuri.mat.puc-rio.br (8.9.3/8.9.3) id LAA22193 for obm-l-MTTP; Sun, 22 Sep 2002 11:30:41 -0300 Received: from shen.bol.com.br (shen.bol.com.br [200.221.24.14]) by sucuri.mat.puc-rio.br (8.9.3/8.9.3) with ESMTP id LAA22189 for ; Sun, 22 Sep 2002 11:30:39 -0300 Received: from bol.com.br (200.221.24.131) by shen.bol.com.br (5.1.071) id 3D8BAD73000648C7 for obm-l@mat.puc-rio.br; Sun, 22 Sep 2002 11:31:30 -0300 Date: Sun, 22 Sep 2002 11:29:29 -0300 Message-Id: Subject: Re: [obm-l] combinatoria MIME-Version: 1.0 Content-Type: text/plain;charset="iso-8859-1" From: "adr.scr.m" To: obm-l@mat.puc-rio.br X-XaM3-API-Version: 2.4.3.4.4 X-SenderIP: 200.151.19.39 Content-Transfer-Encoding: 8bit X-MIME-Autoconverted: from quoted-printable to 8bit by sucuri.mat.puc-rio.br id LAA22190 Sender: owner-obm-l@sucuri.mat.puc-rio.br Precedence: bulk Reply-To: obm-l@mat.puc-rio.br Entao estaria errado fazer com Kaplasky ? []'s. Adriano. __________________________________________________________________________ Encontre sempre uma linha desocupada com o Discador BOL! http://www.bol.com.br/discador Ainda não tem AcessoBOL? Assine já! http://www.bol.com.br/acessobol ========================================================================= Instruções para entrar na lista, sair da lista e usar a lista em http://www.mat.puc-rio.br/~nicolau/olimp/obm-l.html O administrador desta lista é ========================================================================= From owner-obm-l@sucuri.mat.puc-rio.br Sun Sep 22 16:11:25 2002 Return-Path: Received: (from majordom@localhost) by sucuri.mat.puc-rio.br (8.9.3/8.9.3) id QAA24240 for obm-l-MTTP; Sun, 22 Sep 2002 16:09:30 -0300 Received: from puma.unisys.com.br (smtp.unisys.com.br [200.220.64.7]) by sucuri.mat.puc-rio.br (8.9.3/8.9.3) with ESMTP id QAA24236 for ; Sun, 22 Sep 2002 16:09:27 -0300 Received: from jf (riohiper01p235.uninet.com.br [200.220.2.235]) by puma.unisys.com.br (8.12.3/8.12.3) with SMTP id g8MJB7F6015471 for ; Sun, 22 Sep 2002 16:11:08 -0300 (EST) X-Spam-Filter: check_local@puma.unisys.com.br by digitalanswers.org Message-ID: <009801c2626c$a38e57c0$eb02dcc8@jf> From: "Jose Francisco Guimaraes Costa" To: "obm-l" Subject: [obm-l] elementos de euclides Date: Sun, 22 Sep 2002 16:17:04 -0300 MIME-Version: 1.0 Content-Type: multipart/alternative; boundary="----=_NextPart_000_0095_01C26253.7D3DB980" X-Priority: 3 X-MSMail-Priority: Normal X-Mailer: Microsoft Outlook Express 6.00.2600.0000 X-MimeOLE: Produced By Microsoft MimeOLE V6.00.2600.0000 Sender: owner-obm-l@sucuri.mat.puc-rio.br Precedence: bulk Reply-To: obm-l@mat.puc-rio.br This is a multi-part message in MIME format. ------=_NextPart_000_0095_01C26253.7D3DB980 Content-Type: text/plain; charset="iso-8859-1" Content-Transfer-Encoding: quoted-printable H=E1 alguns dias algu=E9m perguntou onde poderia encontrar o Principia = de Newton e o Elementos de Euclides.=20 Eu respondi dando refer=EAncias sobre vers=F5es impressas de ambas as = obras. Navegando na net neste domingo chuvoso, =FAltimo dia de inverno, = encontrei o Elementos em = http://aleph0.clarku.edu/~djoyce/java/elements/toc.html.=20 JF ------=_NextPart_000_0095_01C26253.7D3DB980 Content-Type: text/html; charset="iso-8859-1" Content-Transfer-Encoding: quoted-printable
H=E1 alguns dias algu=E9m perguntou = onde poderia=20 encontrar o Principia de Newton e o Elementos de Euclides.
 
Eu respondi dando refer=EAncias sobre = vers=F5es=20 impressas de ambas as obras.
 
Navegando na net neste domingo = chuvoso, =FAltimo=20 dia de inverno, encontrei o Elementos em http://a= leph0.clarku.edu/~djoyce/java/elements/toc.html.=20
 
JF
------=_NextPart_000_0095_01C26253.7D3DB980-- ========================================================================= Instruções para entrar na lista, sair da lista e usar a lista em http://www.mat.puc-rio.br/~nicolau/olimp/obm-l.html O administrador desta lista é ========================================================================= From owner-obm-l@sucuri.mat.puc-rio.br Sun Sep 22 16:46:32 2002 Return-Path: Received: (from majordom@localhost) by sucuri.mat.puc-rio.br (8.9.3/8.9.3) id QAA24769 for obm-l-MTTP; Sun, 22 Sep 2002 16:44:57 -0300 Received: (from nicolau@localhost) by sucuri.mat.puc-rio.br (8.9.3/8.9.3) id QAA24764 for obm-l@mat.puc-rio.br; Sun, 22 Sep 2002 16:44:57 -0300 Date: Sun, 22 Sep 2002 16:44:57 -0300 From: "Nicolau C. Saldanha" To: obm-l@mat.puc-rio.br Subject: Re: [obm-l] Probabilidade Message-ID: <20020922164457.A7940@sucuri.mat.puc-rio.br> References: <000b01c261c0$5d1ea620$2101a8c0@u2z7z2> Mime-Version: 1.0 Content-Type: text/plain; charset=iso-8859-1 Content-Disposition: inline Content-Transfer-Encoding: 8bit User-Agent: Mutt/1.2.5i In-Reply-To: <000b01c261c0$5d1ea620$2101a8c0@u2z7z2>; from timpa@uol.com.br on Sat, Sep 21, 2002 at 07:43:50PM -0300 Sender: owner-obm-l@sucuri.mat.puc-rio.br Precedence: bulk Reply-To: obm-l@mat.puc-rio.br On Sat, Sep 21, 2002 at 07:43:50PM -0300, Wagner wrote: > Oi para todos ! > É possível descrever a probabilidade do evento abaixo em uma fórmula apenas? > > Uma caneta é girada por uma pessoa de forma aleatória. Os movimentos possíveis da caneta são meia volta para a esquerda e meia volta para a direita. Qual a probabilidade de que após n movimentos aleatórios a caneta tenha feito pelo menos 2 voltas ou para esquerda ou para a direita (considere apenas o balanço final, ex: 2 voltas para esquerda e meia volta para direita deve ser considerado como uma volta e meia para a esquerda). > > André T. Devemos separar os casos par e ímpar, e é mais fácil calcular a probabilidade de que a caneta tenha girado *menos* de duas voltas e depois fazer 1-... Se n = 2m, a probabilidade de que a caneta tenha dado menos de 2 voltas é a probabilidade de que ela tenha feito m-1, m ou m+1 movimentos para a esquerda, o que tem probabilidade (binomial(2m,m-1) + binomial(2m,m) + binomial(2m,m+1))/2^(2m) Analogamente, se n = 2m+1, a probabilidade é (binomial(2m+1,m-1) + binomial(2m+1,m) + binomial(2m+1,m+1) + binomial(2m+1,m+2))/2^(2m+1) []s, N. ========================================================================= Instruções para entrar na lista, sair da lista e usar a lista em http://www.mat.puc-rio.br/~nicolau/olimp/obm-l.html O administrador desta lista é ========================================================================= From owner-obm-l@sucuri.mat.puc-rio.br Sun Sep 22 18:35:08 2002 Return-Path: Received: (from majordom@localhost) by sucuri.mat.puc-rio.br (8.9.3/8.9.3) id SAA25859 for obm-l-MTTP; Sun, 22 Sep 2002 18:33:09 -0300 Received: from traven9.pub1 (traven9.uol.com.br [200.221.4.35]) by sucuri.mat.puc-rio.br (8.9.3/8.9.3) with ESMTP id SAA25855 for ; Sun, 22 Sep 2002 18:33:06 -0300 Received: from u2z7z2 ([200.158.144.199]) by traven9.pub1 (8.9.1/8.9.1) with ESMTP id SAB20855 for ; Sun, 22 Sep 2002 18:36:06 -0300 (BRT) Message-ID: <001801c2627f$f8c7af80$2101a8c0@u2z7z2> From: "Wagner" To: References: <000001c2623c$ae5bb240$15b3bfc8@xxx> Subject: Re: [obm-l] determinantes Date: Sun, 22 Sep 2002 18:35:13 -0300 Organization: Wagner MIME-Version: 1.0 Content-Type: multipart/alternative; boundary="----=_NextPart_000_0015_01C26266.CA0C86C0" X-Priority: 3 X-MSMail-Priority: Normal X-Mailer: Microsoft Outlook Express 5.50.4133.2400 X-MimeOLE: Produced By Microsoft MimeOLE V5.50.4133.2400 Sender: owner-obm-l@sucuri.mat.puc-rio.br Precedence: bulk Reply-To: obm-l@mat.puc-rio.br This is a multi-part message in MIME format. ------=_NextPart_000_0015_01C26266.CA0C86C0 Content-Type: text/plain; charset="iso-8859-1" Content-Transfer-Encoding: quoted-printable Oi para todos!=20 O m=E9todo mais r=E1pida para a solu=E7=E3o =E9 usar a Regra de Chi=F3. = Voc=EA tamb=E9m pode multiplicar todos os termos por (2n-1)! (usando uma = calculadora) e depois dividir o resultado por ((2n-1)!)^n (isso ajuda = para valores pequenos de n)=20 Andr=E9 T. ----- Original Message -----=20 From: haroldo=20 To: obm-l@mat.puc-rio.br=20 Sent: Sunday, September 22, 2002 10:33 AM Subject: [obm-l] determinantes=20 Gostaria de ajuda no c=E1lculo desse determinante: =20 =20 1 1/2 1/3 .... 1/n 1/2 1/3 1/4 .... 1/(n+1) 1/3 1/4 1/5 ... 1/(n+2) .... .................................. ...................................... 1/n 1/(n+1) ... ..... 1/(2n-1) =20 =20 grato=20 HAROLDO. ------=_NextPart_000_0015_01C26266.CA0C86C0 Content-Type: text/html; charset="iso-8859-1" Content-Transfer-Encoding: quoted-printable
Oi para todos!
 
O m=E9todo mais r=E1pida para a = solu=E7=E3o =E9 usar a Regra=20 de Chi=F3. Voc=EA tamb=E9m pode multiplicar todos os termos por (2n-1)! = (usando uma=20 calculadora) e depois dividir o resultado por ((2n-1)!)^n (isso ajuda = para=20 valores pequenos de n)
 
Andr=E9 T.
 
----- Original Message -----
From:=20 haroldo=20
Sent: Sunday, September 22, = 2002 10:33=20 AM
Subject: [obm-l] determinantes =

Gostaria de ajuda no = c=E1lculo desse=20 determinante:

 

 

        =20 1  =20   1/2    1/3   ....   = 1/n

        = 1/2   1/3    1/4  ....   =20 1/(n+1)

       =  1/3   1/4    1/5    ...   1/(n+2)

      =20 .... =20 ..................................

        = = ......................................

       =  1/n   1/(n+1)  ... .....  1/(2n-1)

 

 

grato=20

HAROLDO.

------=_NextPart_000_0015_01C26266.CA0C86C0-- ========================================================================= Instruções para entrar na lista, sair da lista e usar a lista em http://www.mat.puc-rio.br/~nicolau/olimp/obm-l.html O administrador desta lista é ========================================================================= From owner-obm-l@sucuri.mat.puc-rio.br Sun Sep 22 19:07:01 2002 Return-Path: Received: (from majordom@localhost) by sucuri.mat.puc-rio.br (8.9.3/8.9.3) id TAA26428 for obm-l-MTTP; Sun, 22 Sep 2002 19:05:14 -0300 Received: from traven9.pub1 (traven9.uol.com.br [200.221.4.35]) by sucuri.mat.puc-rio.br (8.9.3/8.9.3) with ESMTP id TAA26424 for ; Sun, 22 Sep 2002 19:05:11 -0300 Received: from u2z7z2 ([200.158.144.199]) by traven9.pub1 (8.9.1/8.9.1) with ESMTP id TAA17203 for ; Sun, 22 Sep 2002 19:08:15 -0300 (BRT) Message-ID: <001701c26284$7494a920$2101a8c0@u2z7z2> From: "Wagner" To: Subject: [obm-l] =?iso-8859-1?Q?O_n=FAmero_gugol?= Date: Sun, 22 Sep 2002 19:07:22 -0300 Organization: Wagner MIME-Version: 1.0 Content-Type: multipart/alternative; boundary="----=_NextPart_000_0014_01C2626B.479EB000" X-Priority: 3 X-MSMail-Priority: Normal X-Mailer: Microsoft Outlook Express 5.50.4133.2400 X-MimeOLE: Produced By Microsoft MimeOLE V5.50.4133.2400 Sender: owner-obm-l@sucuri.mat.puc-rio.br Precedence: bulk Reply-To: obm-l@mat.puc-rio.br This is a multi-part message in MIME format. ------=_NextPart_000_0014_01C2626B.479EB000 Content-Type: text/plain; charset="iso-8859-1" Content-Transfer-Encoding: quoted-printable Oi pessoal! Li em uma reportagem que um tal de n=FAmero gugol =E9 10^100 e que outro = n=FAmero chamado de gugolplex =E9 igual =E0 gugol^gugol. Fiquei = pensando, o que seria maior, (1gugol)! ou 1 gugolplex. Como acho a = resposta disso? Andr=E9 T. ------=_NextPart_000_0014_01C2626B.479EB000 Content-Type: text/html; charset="iso-8859-1" Content-Transfer-Encoding: quoted-printable
Oi pessoal!
 
Li em uma reportagem que um tal de = n=FAmero=20 gugol =E9 10^100 e que outro n=FAmero chamado de gugolplex =E9 igual =E0 = gugol^gugol.=20 Fiquei pensando, o que seria maior, (1gugol)! ou 1 gugolplex. Como acho = a=20 resposta disso?
 
Andr=E9 T.
------=_NextPart_000_0014_01C2626B.479EB000-- ========================================================================= Instruções para entrar na lista, sair da lista e usar a lista em http://www.mat.puc-rio.br/~nicolau/olimp/obm-l.html O administrador desta lista é ========================================================================= From owner-obm-l@sucuri.mat.puc-rio.br Sun Sep 22 19:18:34 2002 Return-Path: Received: (from majordom@localhost) by sucuri.mat.puc-rio.br (8.9.3/8.9.3) id TAA26686 for obm-l-MTTP; Sun, 22 Sep 2002 19:17:02 -0300 Received: from sr1.terra.com.br (sr1.terra.com.br [200.176.3.16]) by sucuri.mat.puc-rio.br (8.9.3/8.9.3) with ESMTP id TAA26682 for ; Sun, 22 Sep 2002 19:17:00 -0300 Received: from pavuna.terra.com.br (pavuna.terra.com.br [200.176.3.41]) by sr1.terra.com.br (Postfix) with ESMTP id 185C96F0AB for ; Sun, 22 Sep 2002 19:18:43 -0300 (EST) Received: from stabel (unknown [200.203.38.218]) (authenticated user dudasta) by pavuna.terra.com.br (Postfix) with ESMTP id BFC746837A for ; Sun, 22 Sep 2002 19:18:42 -0300 (EST) Message-ID: <000d01c26286$0404fa50$0301a8c0@stabel> From: "Eduardo Casagrande Stabel" To: References: <001701c26284$7494a920$2101a8c0@u2z7z2> Subject: [obm-l] =?iso-8859-1?Q?Re:_=5Bobm-l=5D_O_n=FAmero_gugol?= Date: Sun, 22 Sep 2002 19:18:44 -0300 MIME-Version: 1.0 Content-Type: text/plain; charset="iso-8859-1" Content-Transfer-Encoding: 8bit X-Priority: 3 X-MSMail-Priority: Normal X-Mailer: Microsoft Outlook Express 6.00.2600.0000 X-MimeOLE: Produced By Microsoft MimeOLE V6.00.2600.0000 Sender: owner-obm-l@sucuri.mat.puc-rio.br Precedence: bulk Reply-To: obm-l@mat.puc-rio.br From: Wagner > >Oi pessoal! > >Li em uma reportagem que um tal de número gugol é 10^100 e que outro número chamado de gugolplex é >igual à gugol^gugol. Fiquei pensando, o que seria maior, (1gugol)! ou 1 gugolplex. Como acho a resposta >disso? > >André T. O que é maior: 1 * 2 * 3 * 4 * ... * n ou n * n * n * n * ... * n ( n vezes )? ========================================================================= Instruções para entrar na lista, sair da lista e usar a lista em http://www.mat.puc-rio.br/~nicolau/olimp/obm-l.html O administrador desta lista é ========================================================================= From owner-obm-l@sucuri.mat.puc-rio.br Sun Sep 22 22:06:06 2002 Return-Path: Received: (from majordom@localhost) by sucuri.mat.puc-rio.br (8.9.3/8.9.3) id WAA28254 for obm-l-MTTP; Sun, 22 Sep 2002 22:03:57 -0300 Received: from seki.bol.com.br (seki.bol.com.br [200.221.24.26]) by sucuri.mat.puc-rio.br (8.9.3/8.9.3) with ESMTP id WAA28250 for ; Sun, 22 Sep 2002 22:03:55 -0300 Received: from comp1 (200.221.24.192) by seki.bol.com.br (5.1.071) id 3D8B711600089BAB for obm-l@mat.puc-rio.br; Sun, 22 Sep 2002 22:05:21 -0300 Message-ID: <000801c262ca$26374f30$3e59b7c8@comp1> From: "Caio Augusto" To: References: <001701c26284$7494a920$2101a8c0@u2z7z2> Subject: [obm-l] =?iso-8859-1?Q?Re:_=5Bobm-l=5D_O_n=FAmero_gugol?= Date: Sun, 22 Sep 2002 22:26:25 -0800 MIME-Version: 1.0 Content-Type: text/plain; charset="iso-8859-1" Content-Transfer-Encoding: 8bit X-Priority: 3 X-MSMail-Priority: Normal X-Mailer: Microsoft Outlook Express 5.50.4522.1200 X-MimeOLE: Produced By Microsoft MimeOLE V5.50.4522.1200 X-Sender-IP: 200.183.89.62 Sender: owner-obm-l@sucuri.mat.puc-rio.br Precedence: bulk Reply-To: obm-l@mat.puc-rio.br O que é maior: n^n ou n! ----- Original Message ----- From: Wagner To: obm-l@mat.puc-rio.br Sent: Sunday, September 22, 2002 2:07 PM Subject: [obm-l] O número gugol Quer ter seu próprio endereço na Internet? Garanta já o seu e ainda ganhe cinco e-mails personalizados. DomíniosBOL - http://dominios.bol.com.br Oi pessoal! Li em uma reportagem que um tal de número gugol é 10^100 e que outro número chamado de gugolplex é igual à gugol^gugol. Fiquei pensando, o que seria maior, (1gugol)! ou 1 gugolplex. Como acho a resposta disso? André T. ========================================================================= Instruções para entrar na lista, sair da lista e usar a lista em http://www.mat.puc-rio.br/~nicolau/olimp/obm-l.html O administrador desta lista é ========================================================================= From owner-obm-l@sucuri.mat.puc-rio.br Sun Sep 22 22:16:29 2002 Return-Path: Received: (from majordom@localhost) by sucuri.mat.puc-rio.br (8.9.3/8.9.3) id WAA28405 for obm-l-MTTP; Sun, 22 Sep 2002 22:15:03 -0300 Received: from sidney1.bol.com.br (sidney1.bol.com.br [200.221.24.206]) by sucuri.mat.puc-rio.br (8.9.3/8.9.3) with ESMTP id WAA28401; Sun, 22 Sep 2002 22:15:01 -0300 Received: from bol.com.br (200.221.24.140) by sidney1.bol.com.br (5.1.071) id 3D8D166900025A12; Sun, 22 Sep 2002 22:16:03 -0300 Date: Sun, 22 Sep 2002 22:13:58 -0300 Message-Id: Subject: [obm-l] =?iso-8859-1?q?numero_de_d=EDgitos?= MIME-Version: 1.0 Content-Type: multipart/mixed; boundary="_=__=_XaM3_Boundary.1032743638.2A.110513.42.1482.52.42.101010.1510651536" From: "diegoalonsoteixeira" To: obm-l@mat.puc-rio.br Cc: obm-l@mat.puc-rio.br X-XaM3-API-Version: 2.4.3.4.4 X-SenderIP: 200.206.236.239 Sender: owner-obm-l@sucuri.mat.puc-rio.br Precedence: bulk Reply-To: obm-l@mat.puc-rio.br --_=__=_XaM3_Boundary.1032743638.2A.110513.42.1482.52.42.101010.1510651536 Content-Type: text/plain;charset="iso-8859-1" Content-Transfer-Encoding: quoted-printable Recebi uma quest=E3o que n=E3o consegui fazer: quanto vale a soma pelo produto dos seis primeiros digitos de (7^4600!*7^460!)/(7^10!*7^46!) N=E3o estou acostumado a esse tipo de quest=E3o, quem responde-la por favor fa=E7a detalhadamente. Obrigado __________________________________________________________________________ Encontre sempre uma linha desocupada com o Discador BOL! http://www.bol.com.br/discador Ainda n=E3o tem AcessoBOL? Assine j=E1! http://www.bol.com.br/acessobol --_=__=_XaM3_Boundary.1032743638.2A.110513.42.1482.52.42.101010.1510651536 Content-Type: text/plain; name="00000LH8" Content-Transfer-Encoding: base64 DQpGcm9tOiBXYWduZXINCj4NCj5PaSBwZXNzb2FsIQ0KPg0KPkxpIGVtIHVtYSByZXBvcnRh Z2VtIHF1ZSB1bSB0YWwgZGUgbvptZXJvIGd1Z29sIOkgMTBeMTAwIGUgcXVlIG91dHJvIG76 bWVybw0KY2hhbWFkbyBkZSBndWdvbHBsZXgg6SA+aWd1YWwg4CBndWdvbF5ndWdvbC4gRmlx dWVpIHBlbnNhbmRvLCBvIHF1ZSBzZXJpYQ0KbWFpb3IsICgxZ3Vnb2wpISBvdSAxIGd1Z29s cGxleC4gQ29tbyBhY2hvIGEgcmVzcG9zdGEgPmRpc3NvPw0KPg0KPkFuZHLpIFQuDQoNCk8g cXVlIOkgbWFpb3I6IDEgKiAyICogMyAqIDQgKiAuLi4gKiBuIG91IG4gKiBuICogbiAqIG4g KiAuLi4gKiBuICggbg0KdmV6ZXMgKT8NCg0KPT09PT09PT09PT09PT09PT09PT09PT09PT09 PT09PT09PT09PT09PT09PT09PT09PT09PT09PT09PT09PT09PT09PT09PT09PQ0KSW5zdHJ1 5/VlcyBwYXJhIGVudHJhciBuYSBsaXN0YSwgc2FpciBkYSBsaXN0YSBlIHVzYXIgYSBsaXN0 YSBlbQ0KaHR0cDovL3d3dy5tYXQucHVjLXJpby5ici9+bmljb2xhdS9vbGltcC9vYm0tbC5o dG1sDQpPIGFkbWluaXN0cmFkb3IgZGVzdGEgbGlzdGEg6SA8bmljb2xhdUBtYXQucHVjLXJp by5icj4NCj09PT09PT09PT09PT09PT09PT09PT09PT09PT09PT09PT09PT09PT09PT09PT09 PT09PT09PT09PT09PT09PT09PT09PT09PT0NCg== --_=__=_XaM3_Boundary.1032743638.2A.110513.42.1482.52.42.101010.1510651536-- ========================================================================= Instruções para entrar na lista, sair da lista e usar a lista em http://www.mat.puc-rio.br/~nicolau/olimp/obm-l.html O administrador desta lista é ========================================================================= From owner-obm-l@sucuri.mat.puc-rio.br Sun Sep 22 23:03:15 2002 Return-Path: Received: (from majordom@localhost) by sucuri.mat.puc-rio.br (8.9.3/8.9.3) id XAA29566 for obm-l-MTTP; Sun, 22 Sep 2002 23:01:52 -0300 Received: from traven9.pub1 (traven9.uol.com.br [200.221.4.35]) by sucuri.mat.puc-rio.br (8.9.3/8.9.3) with ESMTP id XAA29562 for ; Sun, 22 Sep 2002 23:01:50 -0300 Received: from xxx ([200.191.167.128]) by traven9.pub1 (8.9.1/8.9.1) with ESMTP id XAA13555 for ; Sun, 22 Sep 2002 23:04:48 -0300 (BRT) From: "haroldo" To: Subject: [obm-l] =?iso-8859-1?Q?RES:_=5Bobm-l=5D_Re:_=5Bobm-l=5D_O_n=FAmero_gugol?= Date: Sun, 22 Sep 2002 23:05:26 -0300 Message-ID: <000001c262a5$b2239f00$80a7bfc8@xxx> MIME-Version: 1.0 Content-Type: text/plain; charset="iso-8859-1" X-Priority: 3 (Normal) X-MSMail-Priority: Normal X-Mailer: Microsoft Outlook, Build 10.0.2627 Importance: Normal In-Reply-To: <000801c262ca$26374f30$3e59b7c8@comp1> X-MimeOLE: Produced By Microsoft MimeOLE V5.00.2919.6600 Content-Transfer-Encoding: 8bit X-MIME-Autoconverted: from quoted-printable to 8bit by sucuri.mat.puc-rio.br id XAA29563 Sender: owner-obm-l@sucuri.mat.puc-rio.br Precedence: bulk Reply-To: obm-l@mat.puc-rio.br . n^n > n ! para n natural > 1 DEMO: Por indução N=2 4 > 2 Supomos que a proposição n^n > n ! é verdadeira para n = 2,3,...,k. Devemos mostrar que a proposição continua verdadeira para n= k+1 (k+1)!= (k+1).k! < (k+1). k^k como k < k +1 a proposição continua verdadeira para todo n > 1. -----Mensagem original----- De: owner-obm-l@sucuri.mat.puc-rio.br [mailto:owner-obm-l@sucuri.mat.puc-rio.br] Em nome de Caio Augusto Enviada em: segunda-feira, 23 de setembro de 2002 03:26 Para: obm-l@mat.puc-rio.br Assunto: [obm-l] Re: [obm-l] O número gugol O que é maior: n^n ou n! ----- Original Message ----- From: Wagner To: obm-l@mat.puc-rio.br Sent: Sunday, September 22, 2002 2:07 PM Subject: [obm-l] O número gugol Quer ter seu próprio endereço na Internet? Garanta já o seu e ainda ganhe cinco e-mails personalizados. DomíniosBOL - http://dominios.bol.com.br Oi pessoal! Li em uma reportagem que um tal de número gugol é 10^100 e que outro número chamado de gugolplex é igual à gugol^gugol. Fiquei pensando, o que seria maior, (1gugol)! ou 1 gugolplex. Como acho a resposta disso? André T. ======================================================================== = Instruções para entrar na lista, sair da lista e usar a lista em http://www.mat.puc-rio.br/~nicolau/olimp/obm-l.html O administrador desta lista é ======================================================================== = ========================================================================= Instruções para entrar na lista, sair da lista e usar a lista em http://www.mat.puc-rio.br/~nicolau/olimp/obm-l.html O administrador desta lista é ========================================================================= From owner-obm-l@sucuri.mat.puc-rio.br Sun Sep 22 23:25:55 2002 Return-Path: Received: (from majordom@localhost) by sucuri.mat.puc-rio.br (8.9.3/8.9.3) id XAA29954 for obm-l-MTTP; Sun, 22 Sep 2002 23:24:23 -0300 Received: from ginsberg.uol.com.br (ginsberg.uol.com.br [200.221.4.48]) by sucuri.mat.puc-rio.br (8.9.3/8.9.3) with ESMTP id XAA29950 for ; Sun, 22 Sep 2002 23:24:20 -0300 Received: from xxx ([200.191.167.128]) by ginsberg.uol.com.br (8.9.1/8.9.1) with ESMTP id WAA13136 for ; Sun, 22 Sep 2002 22:34:41 -0300 (BRT) From: "haroldo" To: Subject: [obm-l] =?iso-8859-1?Q?RES:_=5Bobm-l=5D_determinantes_solu=E7=E3o?= Date: Sun, 22 Sep 2002 22:38:48 -0300 Message-ID: <000301c262a1$f87a0c40$80a7bfc8@xxx> MIME-Version: 1.0 Content-Type: multipart/alternative; boundary="----=_NextPart_000_0004_01C26288.D32CD440" X-Priority: 3 (Normal) X-MSMail-Priority: Normal X-Mailer: Microsoft Outlook, Build 10.0.2627 X-MimeOLE: Produced By Microsoft MimeOLE V5.00.2919.6600 Importance: Normal In-Reply-To: <000001c2623c$ae5bb240$15b3bfc8@xxx> Sender: owner-obm-l@sucuri.mat.puc-rio.br Precedence: bulk Reply-To: obm-l@mat.puc-rio.br This is a multi-part message in MIME format. ------=_NextPart_000_0004_01C26288.D32CD440 Content-Type: text/plain; charset="iso-8859-1" Content-Transfer-Encoding: quoted-printable Algu=E9m conhece a solu=E7=E3o completa do determinante abaixo? O exerc=EDcio =E9 do livro Problems in Higher =C1lgebra(MIR) e n=E3o = =E9 um determinante elementar. Sauda=E7=F5es a todos . =20 -----Mensagem original----- De: owner-obm-l@sucuri.mat.puc-rio.br [mailto:owner-obm-l@sucuri.mat.puc-rio.br] Em nome de haroldo Enviada em: domingo, 22 de setembro de 2002 10:34 Para: obm-l@mat.puc-rio.br Assunto: [obm-l] determinantes=20 =20 =20 =20 1 1/2 1/3 .... 1/n 1/2 1/3 1/4 .... 1/(n+1) 1/3 1/4 1/5 ... 1/(n+2) .... .................................. ...................................... 1/n 1/(n+1) ... ..... 1/(2n-1) =20 =20 grato=20 HAROLDO. ------=_NextPart_000_0004_01C26288.D32CD440 Content-Type: text/html; charset="iso-8859-1" Content-Transfer-Encoding: quoted-printable

Algu=E9m conhece a solu=E7=E3o = completa do determinante abaixo?

O exerc=EDcio =E9 do livro Problems in Higher =C1lgebra(MIR) =A0e n=E3o =E9 um determinante = elementar.

Sauda=E7=F5es a todos .

 

-----Mensagem = original-----
De: = owner-obm-l@sucuri.mat.puc-rio.br [mailto:owner-obm-l@sucuri.mat.puc-rio.br] Em nome de haroldo
Enviada em: domingo, 22 = de setembro de 2002 10:34
Para: obm-l@mat.puc-rio.br
Assunto: [obm-l] = determinantes

 

 

 

=A0=A0=A0=A0=A0=A0=A0=A0 1=A0=A0=A0=A0 1/2=A0=A0=A0 1/3=A0=A0 ....=A0=A0 = 1/n

=A0=A0=A0=A0=A0=A0=A0 1/2=A0=A0 1/3=A0=A0=A0 1/4=A0 ....=A0=A0=A0 = 1/(n+1)

=A0=A0=A0=A0=A0=A0=A0 1/3=A0=A0 1/4=A0=A0=A0 1/5=A0=A0=A0 ...=A0=A0 = 1/(n+2)

=A0=A0=A0=A0=A0=A0 ....=A0 ..................................=

=A0=A0=A0=A0=A0=A0=A0 ......................................

=A0=A0=A0=A0=A0=A0=A0 1/n=A0=A0 1/(n+1)=A0 ... .....=A0 = 1/(2n-1)

 

 

grato =

HAROLDO.

------=_NextPart_000_0004_01C26288.D32CD440-- ========================================================================= Instruções para entrar na lista, sair da lista e usar a lista em http://www.mat.puc-rio.br/~nicolau/olimp/obm-l.html O administrador desta lista é ========================================================================= From owner-obm-l@sucuri.mat.puc-rio.br Sun Sep 22 23:31:20 2002 Return-Path: Received: (from majordom@localhost) by sucuri.mat.puc-rio.br (8.9.3/8.9.3) id XAA30169 for obm-l-MTTP; Sun, 22 Sep 2002 23:30:03 -0300 Received: from ginsberg.uol.com.br (ginsberg.uol.com.br [200.221.4.48]) by sucuri.mat.puc-rio.br (8.9.3/8.9.3) with ESMTP id XAA30164 for ; Sun, 22 Sep 2002 23:30:01 -0300 Received: from xxx ([200.191.167.128]) by ginsberg.uol.com.br (8.9.1/8.9.1) with ESMTP id WAA14440 for ; Sun, 22 Sep 2002 22:35:50 -0300 (BRT) From: "haroldo" To: Subject: RES: [obm-l] determinantes Date: Sun, 22 Sep 2002 22:39:57 -0300 Message-ID: <000801c262a2$21a0b740$80a7bfc8@xxx> MIME-Version: 1.0 Content-Type: multipart/alternative; boundary="----=_NextPart_000_0009_01C26288.FC537F40" X-Priority: 3 (Normal) X-MSMail-Priority: Normal X-Mailer: Microsoft Outlook, Build 10.0.2627 X-MimeOLE: Produced By Microsoft MimeOLE V5.00.2919.6600 Importance: Normal In-Reply-To: <001801c2627f$f8c7af80$2101a8c0@u2z7z2> Sender: owner-obm-l@sucuri.mat.puc-rio.br Precedence: bulk Reply-To: obm-l@mat.puc-rio.br This is a multi-part message in MIME format. ------=_NextPart_000_0009_01C26288.FC537F40 Content-Type: text/plain; charset="iso-8859-1" Content-Transfer-Encoding: quoted-printable Agrade=E7o a sugest=E3o mas n=E3o resolve o determinante. =20 -----Mensagem original----- De: owner-obm-l@sucuri.mat.puc-rio.br [mailto:owner-obm-l@sucuri.mat.puc-rio.br] Em nome de Wagner Enviada em: domingo, 22 de setembro de 2002 18:35 Para: obm-l@mat.puc-rio.br Assunto: Re: [obm-l] determinantes=20 =20 Oi para todos!=20 =20 O m=E9todo mais r=E1pida para a solu=E7=E3o =E9 usar a Regra de Chi=F3. = Voc=EA tamb=E9m pode multiplicar todos os termos por (2n-1)! (usando uma calculadora) e depois dividir o resultado por ((2n-1)!)^n (isso ajuda para valores pequenos de n)=20 =20 Andr=E9 T. =20 ----- Original Message -----=20 From: haroldo =20 To: obm-l@mat.puc-rio.br=20 Sent: Sunday, September 22, 2002 10:33 AM Subject: [obm-l] determinantes=20 =20 Gostaria de ajuda no c=E1lculo desse determinante: =20 =20 1 1/2 1/3 .... 1/n 1/2 1/3 1/4 .... 1/(n+1) 1/3 1/4 1/5 ... 1/(n+2) .... .................................. ...................................... 1/n 1/(n+1) ... ..... 1/(2n-1) =20 =20 grato=20 HAROLDO. ------=_NextPart_000_0009_01C26288.FC537F40 Content-Type: text/html; charset="iso-8859-1" Content-Transfer-Encoding: quoted-printable

Agrade=E7o a sugest=E3o mas n=E3o = resolve o determinante.

 

-----Mensagem = original-----
De: = owner-obm-l@sucuri.mat.puc-rio.br [mailto:owner-obm-l@sucuri.mat.puc-rio.br] Em nome de Wagner
Enviada em: domingo, 22 = de setembro de 2002 18:35
Para: obm-l@mat.puc-rio.br
Assunto: Re: [obm-l] = determinantes

 

Oi para todos! =

 

O = m=E9todo mais r=E1pida para a solu=E7=E3o =E9 usar a Regra de = Chi=F3. Voc=EA tamb=E9m pode multiplicar todos os termos por (2n-1)! (usando uma calculadora) e = depois dividir o resultado por ((2n-1)!)^n (isso ajuda para valores pequenos de = n)

 

Andr=E9 = T.

 

----- Original Message = -----

From: haroldo

To:<= /font> obm-l@mat.puc-rio.br

Sent: Sunday, September 22, 2002 10:33 AM

Subject: [obm-l] determinantes

 

Gostaria de ajuda no = c=E1lculo desse determinante:

 

 

=A0=A0=A0=A0=A0=A0=A0=A0 1=A0=A0=A0=A0 1/2=A0=A0=A0 1/3=A0=A0 ....=A0=A0 = 1/n

=A0=A0=A0=A0=A0=A0=A0 1/2=A0=A0 1/3=A0=A0=A0 1/4=A0 ....=A0=A0=A0 = 1/(n+1)

=A0=A0=A0=A0=A0=A0=A0 1/3=A0=A0 1/4=A0=A0=A0 1/5=A0=A0=A0 ...=A0=A0 = 1/(n+2)

=A0=A0=A0=A0=A0=A0 .... =A0..................................=

=A0=A0=A0=A0=A0=A0=A0 ......................................

=A0=A0=A0=A0=A0=A0=A0 1/n=A0=A0 1/(n+1)=A0 ... .....=A0 = 1/(2n-1)

 

 

grato =

HAROLDO.

------=_NextPart_000_0009_01C26288.FC537F40-- ========================================================================= Instruções para entrar na lista, sair da lista e usar a lista em http://www.mat.puc-rio.br/~nicolau/olimp/obm-l.html O administrador desta lista é ========================================================================= From owner-obm-l@sucuri.mat.puc-rio.br Mon Sep 23 09:35:27 2002 Return-Path: Received: (from majordom@localhost) by sucuri.mat.puc-rio.br (8.9.3/8.9.3) id JAA02302 for obm-l-MTTP; Mon, 23 Sep 2002 09:34:14 -0300 Received: from gorgo.centroin.com.br (gorgo.centroin.com.br [200.225.63.128]) by sucuri.mat.puc-rio.br (8.9.3/8.9.3) with ESMTP id JAA02298 for ; Mon, 23 Sep 2002 09:34:12 -0300 Received: from centroin.com.br (du171c.rjo.centroin.com.br [200.225.58.171]) (authenticated bits=0) by gorgo.centroin.com.br (8.12.2/8.12.1) with ESMTP id g8NCaH5k000971 for ; Mon, 23 Sep 2002 09:36:18 -0300 (BRT) Message-ID: <3D8F0B69.6000106@centroin.com.br> Date: Mon, 23 Sep 2002 09:39:05 -0300 From: Augusto =?ISO-8859-1?Q?C=E9sar?= Morgado User-Agent: Mozilla/5.0 (Windows; U; Win98; en-US; rv:0.9.4.1) Gecko/20020508 Netscape6/6.2.3 X-Accept-Language: en-us MIME-Version: 1.0 To: obm-l@mat.puc-rio.br Subject: Re: [obm-l] numero de =?ISO-8859-1?Q?d=EDgitos?= References: Content-Type: multipart/alternative; boundary="------------040007030801070607000601" Sender: owner-obm-l@sucuri.mat.puc-rio.br Precedence: bulk Reply-To: obm-l@mat.puc-rio.br --------------040007030801070607000601 Content-Type: text/plain; charset=ISO-8859-1; format=flowed Content-Transfer-Encoding: 8bit Que eh soma pelo produto? diegoalonsoteixeira wrote: >Recebi uma questão que não consegui fazer: >quanto vale a soma pelo produto dos seis primeiros >digitos de (7^4600!*7^460!)/(7^10!*7^46!) >Não estou acostumado a esse tipo de questão, quem >responde-la por favor faça detalhadamente. >Obrigado > > > > >__________________________________________________________________________ >Encontre sempre uma linha desocupada com o Discador BOL! >http://www.bol.com.br/discador >Ainda não tem AcessoBOL? Assine já! http://www.bol.com.br/acessobol > > >------------------------------------------------------------------------ > > >From: Wagner > >>Oi pessoal! >> >>Li em uma reportagem que um tal de número gugol é 10^100 e que outro número >> >chamado de gugolplex é >igual à gugol^gugol. Fiquei pensando, o que seria >maior, (1gugol)! ou 1 gugolplex. Como acho a resposta >disso? > >>André T. >> > >O que é maior: 1 * 2 * 3 * 4 * ... * n ou n * n * n * n * ... * n ( n >vezes )? > >========================================================================= >Instruções para entrar na lista, sair da lista e usar a lista em >http://www.mat.puc-rio.br/~nicolau/olimp/obm-l.html >O administrador desta lista é >========================================================================= > --------------040007030801070607000601 Content-Type: text/html; charset=us-ascii Content-Transfer-Encoding: 7bit Que eh soma pelo produto?

diegoalonsoteixeira wrote:
Recebi uma questão que não consegui fazer:
quanto vale a soma pelo produto dos seis primeiros
digitos de (7^4600!*7^460!)/(7^10!*7^46!)
Não estou acostumado a esse tipo de questão, quem
responde-la por favor faça detalhadamente.
Obrigado




__________________________________________________________________________
Encontre sempre uma linha desocupada com o Discador BOL!
http://www.bol.com.br/discador
Ainda não tem AcessoBOL? Assine já! http://www.bol.com.br/acessobol




From: Wagner
Oi pessoal!

Li em uma reportagem que um tal de número gugol é 10^100 e que outro número
chamado de gugolplex é >igual à gugol^gugol. Fiquei pensando, o que seria
maior, (1gugol)! ou 1 gugolplex. Como acho a resposta >disso?
André T.

O que é maior: 1 * 2 * 3 * 4 * ... * n ou n * n * n * n * ... * n ( n
vezes )?

=========================================================================
Instruções para entrar na lista, sair da lista e usar a lista em
http://www.mat.puc-rio.br/~nicolau/olimp/obm-l.html
O administrador desta lista é <nicolau@mat.puc-rio.br>
=========================================================================

--------------040007030801070607000601-- ========================================================================= Instruções para entrar na lista, sair da lista e usar a lista em http://www.mat.puc-rio.br/~nicolau/olimp/obm-l.html O administrador desta lista é ========================================================================= From owner-obm-l@sucuri.mat.puc-rio.br Mon Sep 23 12:06:47 2002 Return-Path: Received: (from majordom@localhost) by sucuri.mat.puc-rio.br (8.9.3/8.9.3) id MAA04737 for obm-l-MTTP; Mon, 23 Sep 2002 12:05:24 -0300 Received: (from nicolau@localhost) by sucuri.mat.puc-rio.br (8.9.3/8.9.3) id MAA04732 for obm-l@mat.puc-rio.br; Mon, 23 Sep 2002 12:05:23 -0300 Date: Mon, 23 Sep 2002 12:05:23 -0300 From: "Nicolau C. Saldanha" To: obm-l@mat.puc-rio.br Subject: [obm-l] Re: =?iso-8859-1?Q?=5Bobm-l=5D_O_n=FAmero_gugol?= Message-ID: <20020923120523.C4477@sucuri.mat.puc-rio.br> References: <001701c26284$7494a920$2101a8c0@u2z7z2> Mime-Version: 1.0 Content-Type: text/plain; charset=iso-8859-1 Content-Disposition: inline Content-Transfer-Encoding: 8bit User-Agent: Mutt/1.2.5i In-Reply-To: <001701c26284$7494a920$2101a8c0@u2z7z2>; from timpa@uol.com.br on Sun, Sep 22, 2002 at 07:07:22PM -0300 Sender: owner-obm-l@sucuri.mat.puc-rio.br Precedence: bulk Reply-To: obm-l@mat.puc-rio.br On Sun, Sep 22, 2002 at 07:07:22PM -0300, Wagner wrote: > Oi pessoal! > > Li em uma reportagem que um tal de número gugol é 10^100 e que outro número > chamado de gugolplex é igual à gugol^gugol. Fiquei pensando, o que seria > maior, (1gugol)! ou 1 gugolplex. Como acho a resposta disso? Acho que o usual é definir um gugolplex como 10^(10^100), mas o fato é que 10^(10^100) < (10^100)! < (10^100)^(10^100) pois (10^100)! ~ (10^100)^(10^100) e^(- 10^100) sqrt(2 pi 10^100) o que é claramente menor que (10^100)^(10^100) pois e^(- 10^100) sqrt(2 pi 10^100) é bem pequeno. Por outro lado, (10^100)! ~ 10^(10^100) (10^99)^(10^100) e^(- 10^100) sqrt(2 pi 10^100) e como 10^99 é muito maior do que e, (10^99)^(10^100) e^(- 10^100) sqrt(2 pi 10^100) é muito maior do que 1. []s, N. ========================================================================= Instruções para entrar na lista, sair da lista e usar a lista em http://www.mat.puc-rio.br/~nicolau/olimp/obm-l.html O administrador desta lista é ========================================================================= From owner-obm-l@sucuri.mat.puc-rio.br Mon Sep 23 14:45:03 2002 Return-Path: Received: (from majordom@localhost) by sucuri.mat.puc-rio.br (8.9.3/8.9.3) id OAA07239 for obm-l-MTTP; Mon, 23 Sep 2002 14:44:03 -0300 Received: from web12908.mail.yahoo.com (web12908.mail.yahoo.com [216.136.174.75]) by sucuri.mat.puc-rio.br (8.9.3/8.9.3) with SMTP id OAA07235 for ; Mon, 23 Sep 2002 14:44:00 -0300 Message-ID: <20020923174543.63199.qmail@web12908.mail.yahoo.com> Received: from [200.206.103.3] by web12908.mail.yahoo.com via HTTP; Mon, 23 Sep 2002 14:45:43 ART Date: Mon, 23 Sep 2002 14:45:43 -0300 (ART) From: =?iso-8859-1?q?Johann=20Peter=20Gustav=20Lejeune=20Dirichlet?= Subject: Re: [obm-l] Geometria To: obm-l@mat.puc-rio.br In-Reply-To: MIME-Version: 1.0 Content-Type: multipart/alternative; boundary="0-1539608119-1032803143=:61022" Content-Transfer-Encoding: 8bit Sender: owner-obm-l@sucuri.mat.puc-rio.br Precedence: bulk Reply-To: obm-l@mat.puc-rio.br --0-1539608119-1032803143=:61022 Content-Type: text/plain; charset=iso-8859-1 Content-Transfer-Encoding: 8bit Meu,referencias tem milhares.Tem o meu "A Demonstraçao em Geometria",do A.I.Fetissov,Editora Mir,Moscou.Depois passo mais. rafael dowsley wrote: Onde posso encontrar um material bom sobre geometria(com demonstrações dos principais teoremas)? Rafael Baião Dowsley _________________________________________________________________ Tenha você também um MSN Hotmail, o maior webmail do mundo: http://www.hotmail.com/br ========================================================================= Instruções para entrar na lista, sair da lista e usar a lista em http://www.mat.puc-rio.br/~nicolau/olimp/obm-l.html O administrador desta lista é ========================================================================= --------------------------------- Yahoo! GeoCities Tudo para criar o seu site: ferramentas fáceis de usar, espaço de sobra e acessórios. --0-1539608119-1032803143=:61022 Content-Type: text/html; charset=iso-8859-1 Content-Transfer-Encoding: 8bit

Meu,referencias tem milhares.Tem o meu "A Demonstraçao em Geometria",do A.I.Fetissov,Editora Mir,Moscou.Depois passo mais.

 rafael dowsley wrote:


Onde posso encontrar um material bom sobre geometria(com demonstrações dos
principais teoremas)?


Rafael Baião Dowsley


_________________________________________________________________
Tenha você também um MSN Hotmail, o maior webmail do mundo:
http://www.hotmail.com/br

=========================================================================
Instruções para entrar na lista, sair da lista e usar a lista em
http://www.mat.puc-rio.br/~nicolau/olimp/obm-l.html
O administrador desta lista é
=========================================================================



Yahoo! GeoCities
Tudo para criar o seu site: ferramentas fáceis de usar, espaço de sobra e acessórios. --0-1539608119-1032803143=:61022-- ========================================================================= Instruções para entrar na lista, sair da lista e usar a lista em http://www.mat.puc-rio.br/~nicolau/olimp/obm-l.html O administrador desta lista é ========================================================================= From owner-obm-l@sucuri.mat.puc-rio.br Mon Sep 23 15:18:05 2002 Return-Path: Received: (from majordom@localhost) by sucuri.mat.puc-rio.br (8.9.3/8.9.3) id PAA07902 for obm-l-MTTP; Mon, 23 Sep 2002 15:14:45 -0300 Received: from toole.pub1 (toole.uol.com.br [200.221.4.26]) by sucuri.mat.puc-rio.br (8.9.3/8.9.3) with ESMTP id PAA07898 for ; Mon, 23 Sep 2002 15:14:25 -0300 Received: from slap ([200.207.152.104]) by toole.pub1 (8.9.1/8.9.1) with SMTP id PAA27488 for ; Mon, 23 Sep 2002 15:10:14 -0300 (BRT) Message-ID: <001b01c2632e$4180ca00$6898cfc8@slap> From: "Afemano" To: Subject: [obm-l] =?iso-8859-1?Q?N=FAmeros_complexos...?= Date: Mon, 23 Sep 2002 15:22:52 -0300 MIME-Version: 1.0 Content-Type: multipart/alternative; boundary="----=_NextPart_000_0018_01C26315.159F0000" X-Priority: 3 X-MSMail-Priority: Normal X-Mailer: Microsoft Outlook Express 6.00.2600.0000 X-MimeOLE: Produced By Microsoft MimeOLE V6.00.2600.0000 Sender: owner-obm-l@sucuri.mat.puc-rio.br Precedence: bulk Reply-To: obm-l@mat.puc-rio.br This is a multi-part message in MIME format. ------=_NextPart_000_0018_01C26315.159F0000 Content-Type: text/plain; charset="iso-8859-1" Content-Transfer-Encoding: quoted-printable Ol=E1 Gostaria de saber se eu posso usar a igualdade : r(cos@ + isen@ )^n =3D r^n [ cos(n@) + isen(n@) ] para uma express=E3o = dos reais do tipo : ( cos@ + xsen@) ^ n.... aplicando o mesmo processo.. Obrigado... ------=_NextPart_000_0018_01C26315.159F0000 Content-Type: text/html; charset="iso-8859-1" Content-Transfer-Encoding: quoted-printable
Ol=E1
 
Gostaria de saber se eu posso usar a = igualdade=20 :
r(cos@ + isen@ )^n =3D r^n [ cos(n@) + = isen(n@) ]=20 para uma express=E3o dos reais do tipo :
( cos@ + xsen@) ^ n.... aplicando o = mesmo=20 processo..
 
Obrigado...
------=_NextPart_000_0018_01C26315.159F0000-- ========================================================================= Instruções para entrar na lista, sair da lista e usar a lista em http://www.mat.puc-rio.br/~nicolau/olimp/obm-l.html O administrador desta lista é ========================================================================= From owner-obm-l@sucuri.mat.puc-rio.br Mon Sep 23 16:20:04 2002 Return-Path: Received: (from majordom@localhost) by sucuri.mat.puc-rio.br (8.9.3/8.9.3) id QAA09638 for obm-l-MTTP; Mon, 23 Sep 2002 16:18:38 -0300 Received: from zeus.opendf.com.br (zeus.opengate.com.br [200.181.71.10]) by sucuri.mat.puc-rio.br (8.9.3/8.9.3) with ESMTP id QAA09634 for ; Mon, 23 Sep 2002 16:18:35 -0300 Received: from localhost (localhost.opengate.com.br [127.0.0.1]) by zeus.opendf.com.br (Postfix) with ESMTP id 8CC313EB47 for ; Mon, 23 Sep 2002 16:20:14 -0300 (BRT) Received: by zeus.opendf.com.br (Postfix, from userid 48) id 55C313EB5A; Mon, 23 Sep 2002 16:20:06 -0300 (BRT) From: "498 - Artur Costa Steiner" To: obm-l@mat.puc-rio.br Subject: Re: [obm-l] Números complexos... X-Mailer: NeoMail 1.25 X-IPAddress: 200.252.155.2 MIME-Version: 1.0 Content-Type: text/plain; charset=iso-8859-1 Message-Id: <20020923192006.55C313EB5A@zeus.opendf.com.br> Date: Mon, 23 Sep 2002 16:20:06 -0300 (BRT) X-Virus-Scanned: by AMaViS new-20020517 Sender: owner-obm-l@sucuri.mat.puc-rio.br Precedence: bulk Reply-To: obm-l@mat.puc-rio.br > This is a multi-part message in MIME format. > > > Olá > > Gostaria de saber se eu posso usar a igualdade : > r(cos@ + isen@ )^n = r^n [ cos(n@) + isen(n@) ] para uma expressão dos reais do tipo : > ( cos@ + xsen@) ^ n.... aplicando o mesmo processo.. > > Obrigado... Não, isto não é válido. Artur > > -- OPEN Internet - o 1º Provedor do DF com anit-virus no servidor de e- mail! ========================================================================= Instruções para entrar na lista, sair da lista e usar a lista em http://www.mat.puc-rio.br/~nicolau/olimp/obm-l.html O administrador desta lista é ========================================================================= From owner-obm-l@sucuri.mat.puc-rio.br Mon Sep 23 16:42:15 2002 Return-Path: Received: (from majordom@localhost) by sucuri.mat.puc-rio.br (8.9.3/8.9.3) id QAA10072 for obm-l-MTTP; Mon, 23 Sep 2002 16:41:41 -0300 Received: (from nicolau@localhost) by sucuri.mat.puc-rio.br (8.9.3/8.9.3) id QAA10067 for obm-l@mat.puc-rio.br; Mon, 23 Sep 2002 16:41:40 -0300 Date: Mon, 23 Sep 2002 16:41:40 -0300 From: "Nicolau C. Saldanha" To: obm-l@mat.puc-rio.br Subject: Re: [obm-l] determinantes Message-ID: <20020923164140.A9868@sucuri.mat.puc-rio.br> References: <000001c2623c$ae5bb240$15b3bfc8@xxx> Mime-Version: 1.0 Content-Type: text/plain; charset=iso-8859-1 Content-Disposition: inline Content-Transfer-Encoding: 8bit User-Agent: Mutt/1.2.5i In-Reply-To: <000001c2623c$ae5bb240$15b3bfc8@xxx>; from divaneto@uol.com.br on Sun, Sep 22, 2002 at 10:33:45AM -0300 Sender: owner-obm-l@sucuri.mat.puc-rio.br Precedence: bulk Reply-To: obm-l@mat.puc-rio.br On Sun, Sep 22, 2002 at 10:33:45AM -0300, haroldo wrote: > Gostaria de ajuda no cálculo desse determinante: > > > 1 1/2 1/3 .... 1/n > 1/2 1/3 1/4 .... 1/(n+1) > 1/3 1/4 1/5 ... 1/(n+2) > .... .................................. > ...................................... > 1/n 1/(n+1) ... ..... 1/(2n-1) Esta é a chamada matriz de Hilbert. Você encontrará mais informações sobre ela em bons livros de álgebra linear ou em http://mathworld.wolfram.com/HilbertMatrix.html Se chamarmos o determinante da matriz de Hilbert nxn de 1/f(n) temos f(n)/f(n-1) = (2n-1) (binomial(2n-2,n-1))^2 o que nos dá uma fórmula um pouco complicada para os determinantes... A seqüência f(n) é formada pelos números inteiros positivos abaixo: 1 12 2160 6048000 266716800000 186313420339200000 2067909047925770649600000 365356847125734485878112256000000 1028781784378569697887052962909388800000000 46206893947914691316295628839036278726983680000000000 33122504897063413755362143627040727106080127672469422080000000000 379106579436304517151885479034796391880188687864118464104324304732160000000000 e esta é a seqüência A005249 na excelente Encyclopaedia of Integer Sequences: http://www.research.att.com/~njas/sequences/ []s, N. ========================================================================= Instruções para entrar na lista, sair da lista e usar a lista em http://www.mat.puc-rio.br/~nicolau/olimp/obm-l.html O administrador desta lista é ========================================================================= From owner-obm-l@sucuri.mat.puc-rio.br Mon Sep 23 17:11:07 2002 Return-Path: Received: (from majordom@localhost) by sucuri.mat.puc-rio.br (8.9.3/8.9.3) id RAA11022 for obm-l-MTTP; Mon, 23 Sep 2002 17:10:43 -0300 Received: from sidney1.bol.com.br (sidney1.bol.com.br [200.221.24.206]) by sucuri.mat.puc-rio.br (8.9.3/8.9.3) with ESMTP id RAA11018; Mon, 23 Sep 2002 17:10:41 -0300 Received: from bol.com.br (200.221.24.136) by sidney1.bol.com.br (5.1.071) id 3D8D16690004E49A; Mon, 23 Sep 2002 17:11:44 -0300 Date: Mon, 23 Sep 2002 17:09:43 -0300 Message-Id: Subject: [obm-l] =?iso-8859-1?q?Re=3A_=5Bobm=2Dl=5D_numero_de_d=EDgitos?= MIME-Version: 1.0 Content-Type: multipart/mixed; boundary="_=__=_XaM3_Boundary.1032811783.2A.534918.42.29720.52.42.101010.1963820080" From: "diegoalonsoteixeira" To: obm-l@mat.puc-rio.br Cc: obm-l@mat.puc-rio.br X-XaM3-API-Version: 2.4.3.4.4 X-SenderIP: 200.206.236.239 Sender: owner-obm-l@sucuri.mat.puc-rio.br Precedence: bulk Reply-To: obm-l@mat.puc-rio.br --_=__=_XaM3_Boundary.1032811783.2A.534918.42.29720.52.42.101010.1963820080 Content-Type: text/plain;charset="iso-8859-1" Content-Transfer-Encoding: quoted-printable faltou o dividido soma dividido pelo produto dos 6 primeiros d=EDgitos de (7^4600!*7^460!)/7^10!*7^46! __________________________________________________________________________ Encontre sempre uma linha desocupada com o Discador BOL! http://www.bol.com.br/discador Ainda n=E3o tem AcessoBOL? Assine j=E1! http://www.bol.com.br/acessobol --_=__=_XaM3_Boundary.1032811783.2A.534918.42.29720.52.42.101010.1963820080 Content-Type: text/plain; name="00000JPN" Content-Transfer-Encoding: base64 UXVlIGVoIHNvbWEgcGVsbyBwcm9kdXRvPw0KDQpkaWVnb2Fsb25zb3RlaXhlaXJhIHdyb3Rl Og0KDQo+UmVjZWJpIHVtYSBxdWVzdONvIHF1ZSBu428gY29uc2VndWkgZmF6ZXI6DQo+cXVh bnRvIHZhbGUgYSBzb21hIHBlbG8gcHJvZHV0byBkb3Mgc2VpcyBwcmltZWlyb3MgDQo+ZGln aXRvcyBkZSAoN140NjAwISo3XjQ2MCEpLyg3XjEwISo3XjQ2ISkNCj5O428gZXN0b3UgYWNv c3R1bWFkbyBhIGVzc2UgdGlwbyBkZSBxdWVzdONvLCBxdWVtIA0KPnJlc3BvbmRlLWxhIHBv ciBmYXZvciBmYedhIGRldGFsaGFkYW1lbnRlLg0KPk9icmlnYWRvDQo+ICAgICAgICAgICAg DQo+DQo+DQo+IA0KPl9fX19fX19fX19fX19fX19fX19fX19fX19fX19fX19fX19fX19fX19f X19fX19fX19fX19fX19fX19fX19fX19fX19fX19fX19fDQo+RW5jb250cmUgc2VtcHJlIHVt YSBsaW5oYSBkZXNvY3VwYWRhIGNvbSBvIERpc2NhZG9yIEJPTCENCj5odHRwOi8vd3d3LmJv bC5jb20uYnIvZGlzY2Fkb3INCj5BaW5kYSBu428gdGVtIEFjZXNzb0JPTD8gQXNzaW5lIGrh ISBodHRwOi8vd3d3LmJvbC5jb20uYnIvYWNlc3NvYm9sDQo+DQo+DQo+LS0tLS0tLS0tLS0t LS0tLS0tLS0tLS0tLS0tLS0tLS0tLS0tLS0tLS0tLS0tLS0tLS0tLS0tLS0tLS0tLS0tLS0t LS0tLS0tDQo+DQo+DQo+RnJvbTogV2FnbmVyDQo+DQo+Pk9pIHBlc3NvYWwhDQo+Pg0KPj5M aSBlbSB1bWEgcmVwb3J0YWdlbSBxdWUgdW0gdGFsIGRlIG76bWVybyBndWdvbCDpIDEwXjEw MCBlIHF1ZSBvdXRybyBu+m1lcm8NCj4+DQo+Y2hhbWFkbyBkZSBndWdvbHBsZXgg6SA+aWd1 YWwg4CBndWdvbF5ndWdvbC4gRmlxdWVpIHBlbnNhbmRvLCBvIHF1ZSBzZXJpYQ0KPm1haW9y LCAoMWd1Z29sKSEgb3UgMSBndWdvbHBsZXguIENvbW8gYWNobyBhIHJlc3Bvc3RhID5kaXNz bz8NCj4NCj4+QW5kcukgVC4NCj4+DQo+DQo+TyBxdWUg6SBtYWlvcjogMSAqIDIgKiAzICog NCAqIC4uLiAqIG4gb3UgbiAqIG4gKiBuICogbiAqIC4uLiAqIG4gKCBuDQo+dmV6ZXMgKT8N Cj4NCj49PT09PT09PT09PT09PT09PT09PT09PT09PT09PT09PT09PT09PT09PT09PT09PT09 PT09PT09PT09PT09PT09PT09PT09PT09DQo+SW5zdHJ15/VlcyBwYXJhIGVudHJhciBuYSBs aXN0YSwgc2FpciBkYSBsaXN0YSBlIHVzYXIgYSBsaXN0YSBlbQ0KPmh0dHA6Ly93d3cubWF0 LnB1Yy1yaW8uYnIvfm5pY29sYXUvb2xpbXAvb2JtLWwuaHRtbA0KPk8gYWRtaW5pc3RyYWRv ciBkZXN0YSBsaXN0YSDpIDxuaWNvbGF1QG1hdC5wdWMtcmlvLmJyPg0KPj09PT09PT09PT09 PT09PT09PT09PT09PT09PT09PT09PT09PT09PT09PT09PT09PT09PT09PT09PT09PT09PT09 PT09PT09PT0NCj4NCg0K --_=__=_XaM3_Boundary.1032811783.2A.534918.42.29720.52.42.101010.1963820080-- ========================================================================= Instruções para entrar na lista, sair da lista e usar a lista em http://www.mat.puc-rio.br/~nicolau/olimp/obm-l.html O administrador desta lista é ========================================================================= From owner-obm-l@sucuri.mat.puc-rio.br Tue Sep 24 09:19:10 2002 Return-Path: Received: (from majordom@localhost) by sucuri.mat.puc-rio.br (8.9.3/8.9.3) id JAA18833 for obm-l-MTTP; Tue, 24 Sep 2002 09:17:09 -0300 Received: from web21507.mail.yahoo.com (web21507.mail.yahoo.com [66.163.169.18]) by sucuri.mat.puc-rio.br (8.9.3/8.9.3) with SMTP id JAA18827 for ; Tue, 24 Sep 2002 09:17:07 -0300 Message-ID: <20020924121848.85129.qmail@web21507.mail.yahoo.com> Received: from [200.137.198.82] by web21507.mail.yahoo.com via HTTP; Tue, 24 Sep 2002 09:18:48 ART Date: Tue, 24 Sep 2002 09:18:48 -0300 (ART) From: =?iso-8859-1?q?bruno=20lima?= Subject: [obm-l] EDO - nenhum palpite? To: OBM lISTA MIME-Version: 1.0 Content-Type: text/plain; charset=iso-8859-1 Content-Transfer-Encoding: 8bit Sender: owner-obm-l@sucuri.mat.puc-rio.br Precedence: bulk Reply-To: obm-l@mat.puc-rio.br Numa dessas provas universitarias por ai apareceu: y''=y*exp(x) y: Vai de [a,b]em R , e y(a)=0 e y(b)=0. Estou errado ou a unica solucáo é a identicamente nula?? Fazendo uma mudança de variaveis vc cai numa de Ricati, mas isso nao resolve muito,pois nao achei nenhuma soluçao particular. _______________________________________________________________________ Yahoo! GeoCities Tudo para criar o seu site: ferramentas fáceis de usar, espaço de sobra e acessórios. http://br.geocities.yahoo.com/ ========================================================================= Instruções para entrar na lista, sair da lista e usar a lista em http://www.mat.puc-rio.br/~nicolau/olimp/obm-l.html O administrador desta lista é ========================================================================= From owner-obm-l@sucuri.mat.puc-rio.br Tue Sep 24 10:01:41 2002 Return-Path: Received: (from majordom@localhost) by sucuri.mat.puc-rio.br (8.9.3/8.9.3) id KAA19833 for obm-l-MTTP; Tue, 24 Sep 2002 10:01:20 -0300 Received: (from nicolau@localhost) by sucuri.mat.puc-rio.br (8.9.3/8.9.3) id KAA19828 for obm-l@mat.puc-rio.br; Tue, 24 Sep 2002 10:01:19 -0300 Date: Tue, 24 Sep 2002 10:01:19 -0300 From: "Nicolau C. Saldanha" To: obm-l@mat.puc-rio.br Subject: Re: [obm-l] EDO - nenhum palpite? Message-ID: <20020924100119.B19600@sucuri.mat.puc-rio.br> References: <20020924121848.85129.qmail@web21507.mail.yahoo.com> Mime-Version: 1.0 Content-Type: text/plain; charset=iso-8859-1 Content-Disposition: inline Content-Transfer-Encoding: 8bit User-Agent: Mutt/1.2.5i In-Reply-To: <20020924121848.85129.qmail@web21507.mail.yahoo.com>; from bbslima@yahoo.com.br on Tue, Sep 24, 2002 at 09:18:48AM -0300 Sender: owner-obm-l@sucuri.mat.puc-rio.br Precedence: bulk Reply-To: obm-l@mat.puc-rio.br On Tue, Sep 24, 2002 at 09:18:48AM -0300, bruno lima wrote: > Numa dessas provas universitarias por ai apareceu: > > y''=y*exp(x) > > y: Vai de [a,b]em R , e y(a)=0 e y(b)=0. > > Estou errado ou a unica solucáo é a identicamente > nula?? Sim, a única solução é identicamente nula. Podemos ver isso resolvendo y(a) = 0, y'(a) = 1. É fácil ver que y(t) > t-a para todo t > a e portanto y(b) > 0. Você tem certeza que a equação é esta? A coisa seria bem diferente se fosse, por exemplo, y'' = - y * exp(x) ou até y'' = exp(y) []s, N. ========================================================================= Instruções para entrar na lista, sair da lista e usar a lista em http://www.mat.puc-rio.br/~nicolau/olimp/obm-l.html O administrador desta lista é ========================================================================= From owner-obm-l@sucuri.mat.puc-rio.br Tue Sep 24 17:02:01 2002 Return-Path: Received: (from majordom@localhost) by sucuri.mat.puc-rio.br (8.9.3/8.9.3) id QAA25459 for obm-l-MTTP; Tue, 24 Sep 2002 16:57:44 -0300 Received: from traven10.pub1 (traven10.uol.com.br [200.221.4.45]) by sucuri.mat.puc-rio.br (8.9.3/8.9.3) with ESMTP id QAA25454 for ; Tue, 24 Sep 2002 16:57:41 -0300 Received: from u2z7z2 ([200.158.145.244]) by traven10.pub1 (8.9.1/8.9.1) with ESMTP id PAA06144 for ; Tue, 24 Sep 2002 15:24:58 -0300 (BRT) Message-ID: <000b01c263f7$fb3631a0$2101a8c0@u2z7z2> From: "Wagner" To: Subject: [obm-l] =?iso-8859-1?Q?Fun=E7=F5es_trigonom=E9tricas?= Date: Tue, 24 Sep 2002 15:26:55 -0300 Organization: Wagner MIME-Version: 1.0 Content-Type: multipart/alternative; boundary="----=_NextPart_000_0008_01C263DE.D0BC8320" X-Priority: 3 X-MSMail-Priority: Normal X-Mailer: Microsoft Outlook Express 5.50.4133.2400 X-MimeOLE: Produced By Microsoft MimeOLE V5.50.4133.2400 Sender: owner-obm-l@sucuri.mat.puc-rio.br Precedence: bulk Reply-To: obm-l@mat.puc-rio.br This is a multi-part message in MIME format. ------=_NextPart_000_0008_01C263DE.D0BC8320 Content-Type: text/plain; charset="iso-8859-1" Content-Transfer-Encoding: quoted-printable Oi para todos! De=EAm uma olhada nessa pergunta: Qual o menor valor poss=EDvel de a para que a fun=E7=E3o = y=3Dsen(x.Pi)+a.x/Pi tenha menos de 2 pontos de m=E1ximo? Andr=E9 T. ------=_NextPart_000_0008_01C263DE.D0BC8320 Content-Type: text/html; charset="iso-8859-1" Content-Transfer-Encoding: quoted-printable
Oi para todos!
 
De=EAm uma olhada nessa = pergunta:
 
Qual o menor valor poss=EDvel de a para = que a fun=E7=E3o=20 y=3Dsen(x.Pi)+a.x/Pi tenha menos de 2 pontos de m=E1ximo?
 
Andr=E9 T.
------=_NextPart_000_0008_01C263DE.D0BC8320-- ========================================================================= Instruções para entrar na lista, sair da lista e usar a lista em http://www.mat.puc-rio.br/~nicolau/olimp/obm-l.html O administrador desta lista é ========================================================================= From owner-obm-l@sucuri.mat.puc-rio.br Tue Sep 24 18:13:26 2002 Return-Path: Received: (from majordom@localhost) by sucuri.mat.puc-rio.br (8.9.3/8.9.3) id SAA26785 for obm-l-MTTP; Tue, 24 Sep 2002 18:11:16 -0300 Received: from lampiao.digi.com.br (dial-up-a-nat-060.digi.com.br [200.241.100.60] (may be forged)) by sucuri.mat.puc-rio.br (8.9.3/8.9.3) with ESMTP id SAA26781 for ; Tue, 24 Sep 2002 18:11:14 -0300 Received: from david (host042.d.digizap.com.br [200.249.6.42]) by lampiao.digi.com.br (8.11.2/8.11.6) with SMTP id g8OLCR719892 for ; Tue, 24 Sep 2002 18:12:27 -0300 Message-ID: <000401c2640f$fc4bac60$2a06f9c8@david> From: "David Ricardo" To: Subject: [obm-l] mdc e mmc Date: Tue, 24 Sep 2002 18:17:32 -0300 MIME-Version: 1.0 Content-Type: text/plain; charset="iso-8859-1" Content-Transfer-Encoding: 8bit X-Priority: 3 X-MSMail-Priority: Normal X-Mailer: Microsoft Outlook Express 5.00.2615.200 X-MimeOLE: Produced By Microsoft MimeOLE V5.00.2615.200 X-MailScanner: Found to be clean Sender: owner-obm-l@sucuri.mat.puc-rio.br Precedence: bulk Reply-To: obm-l@mat.puc-rio.br Eu vi em algum lugar que: mdc(x,y)*mmc(x,y) = x*y Como não havia nenhuma prova disto, resolvi tentar prová-lo. Eu gostaria de saber se essa prova está certa: (1) mdc(x,y) = maior m, tal que x = m*a e y = m*b (2) mmc(x,y) = menor n, tal que n = x*c e n = y*d (*) Podemos concluir que (a, b) e (c, d) são primos entre si. (3) x*y = m^2*a*b (tirando de 1) (4) n = mac (1 em 2) (5) n = mbd (1 em 2) (6) a*c = b*d (de 4 e 5) (7) a/b = d/c (de 6) Devido a (*), a/b e d/c são fracões irredutíveis, então: (8) a = d e b = c (9) n = mab (usando 8 em 4) (10) x*y = m*n (usando 9 em 3) []s David ========================================================================= Instruções para entrar na lista, sair da lista e usar a lista em http://www.mat.puc-rio.br/~nicolau/olimp/obm-l.html O administrador desta lista é ========================================================================= From owner-obm-l@sucuri.mat.puc-rio.br Tue Sep 24 18:46:18 2002 Return-Path: Received: (from majordom@localhost) by sucuri.mat.puc-rio.br (8.9.3/8.9.3) id SAA27502 for obm-l-MTTP; Tue, 24 Sep 2002 18:44:43 -0300 Received: from videira.terra.com.br (videira.terra.com.br [200.176.3.5]) by sucuri.mat.puc-rio.br (8.9.3/8.9.3) with ESMTP id SAA27498 for ; Tue, 24 Sep 2002 18:44:40 -0300 Received: from engenho.terra.com.br (engenho.terra.com.br [200.176.3.42]) by videira.terra.com.br (Postfix) with ESMTP id E3D42E1985 for ; Tue, 24 Sep 2002 18:46:29 -0300 (EST) Received: from stabel (200-180-181-209-paemt7002.dsl.telebrasilia.net.br [200.180.181.209]) (authenticated user dudasta) by engenho.terra.com.br (Postfix) with ESMTP id 6686768085 for ; Tue, 24 Sep 2002 18:46:29 -0300 (EST) Message-ID: <000d01c26413$d7680570$0301a8c0@stabel> From: "Eduardo Casagrande Stabel" To: References: <000401c2640f$fc4bac60$2a06f9c8@david> Subject: Re: [obm-l] mdc e mmc Date: Tue, 24 Sep 2002 18:46:29 -0300 MIME-Version: 1.0 Content-Type: text/plain; charset="iso-8859-1" Content-Transfer-Encoding: 8bit X-Priority: 3 X-MSMail-Priority: Normal X-Mailer: Microsoft Outlook Express 6.00.2600.0000 X-MimeOLE: Produced By Microsoft MimeOLE V6.00.2600.0000 Sender: owner-obm-l@sucuri.mat.puc-rio.br Precedence: bulk Reply-To: obm-l@mat.puc-rio.br David, a mim, parece estar tudo certo. Um outro jeito é analisar o coeficiente de cada primo. Seja pi um primo e ai e bi os coeficientes em x e y, respect. O coeficiente de pi em mdc(x,y) é min(ai,bi). O coeficiente de pi em mmc(x,y) é max(ai,bi). Como min(ai,bi) + max(ai,bi) = ai + bi, conclui-se o que você concluiu. Eduardo. Porto Alegre, RS. From: "David Ricardo" Eu vi em algum lugar que: mdc(x,y)*mmc(x,y) = x*y Como não havia nenhuma prova disto, resolvi tentar prová-lo. Eu gostaria de saber se essa prova está certa: (1) mdc(x,y) = maior m, tal que x = m*a e y = m*b (2) mmc(x,y) = menor n, tal que n = x*c e n = y*d (*) Podemos concluir que (a, b) e (c, d) são primos entre si. (3) x*y = m^2*a*b (tirando de 1) (4) n = mac (1 em 2) (5) n = mbd (1 em 2) (6) a*c = b*d (de 4 e 5) (7) a/b = d/c (de 6) Devido a (*), a/b e d/c são fracões irredutíveis, então: (8) a = d e b = c (9) n = mab (usando 8 em 4) (10) x*y = m*n (usando 9 em 3) []s David ========================================================================= Instruções para entrar na lista, sair da lista e usar a lista em http://www.mat.puc-rio.br/~nicolau/olimp/obm-l.html O administrador desta lista é ========================================================================= ========================================================================= Instruções para entrar na lista, sair da lista e usar a lista em http://www.mat.puc-rio.br/~nicolau/olimp/obm-l.html O administrador desta lista é ========================================================================= From owner-obm-l@sucuri.mat.puc-rio.br Tue Sep 24 23:32:43 2002 Return-Path: Received: (from majordom@localhost) by sucuri.mat.puc-rio.br (8.9.3/8.9.3) id XAA30228 for obm-l-MTTP; Tue, 24 Sep 2002 23:31:00 -0300 Received: from mta2sjc.directnet.com.br (mta2sjc.directnet.com.br [200.152.0.13]) by sucuri.mat.puc-rio.br (8.9.3/8.9.3) with ESMTP id XAA30224 for ; Tue, 24 Sep 2002 23:30:58 -0300 Received: from meu ([200.152.17.12]) by pop.directnet.com.br (iPlanet Messaging Server 5.1 HotFix 1.3 (built Jul 16 2002)) with SMTP id <0H2Z00CCA31NA2@pop.directnet.com.br> for obm-l@mat.puc-rio.br; Tue, 24 Sep 2002 23:32:11 -0300 (EST) Date: Tue, 24 Sep 2002 23:50:11 -0300 From: Daniel Subject: [obm-l] Kaplanky To: Lista OBM Message-id: <000c01c2643e$47a152e0$0c1198c8@directnet.com.br> MIME-version: 1.0 X-MIMEOLE: Produced By Microsoft MimeOLE V5.00.2615.200 X-Mailer: Microsoft Outlook Express 5.00.2615.200 Content-type: multipart/alternative; boundary="Boundary_(ID_377HnO5Mf/3fOV7Y3mBUXg)" X-Priority: 3 X-MSMail-priority: Normal Sender: owner-obm-l@sucuri.mat.puc-rio.br Precedence: bulk Reply-To: obm-l@mat.puc-rio.br This is a multi-part message in MIME format. --Boundary_(ID_377HnO5Mf/3fOV7Y3mBUXg) Content-type: text/plain; charset=iso-8859-1 Content-transfer-encoding: quoted-printable Ol=E1 a todos, Alguem poderia enunciar os dois teoremas de Kaplanky = da an=E1lise combinat=F3ria. 1) O primeiro =E9 referente a seq=FC=EAncias com = sucess=F5es 2) O segundo =E9 do mesmo tipo, mas em uma = disposi=E7=E3o sobre circunfer=EAncia =20 Ex: 1) Em uma urna existem n bolas numeradas de 1 a n. = De quantas formas podemos pegar k bolas, de modo que n=E3o haja bolas = sucessivas? 2) Doze cavaleiros est=E3o dispostos em uma t=E1vola = redonda, de sorte que cavaleiros sucessivos s=E3o inimigos. Quantas = comiss=F5es de 5 cavaleiros podemos formar, de modo que n=E3o haja = cavaleiros inimigos na comiss=E3o? =20 Grato Daniel --Boundary_(ID_377HnO5Mf/3fOV7Y3mBUXg) Content-type: text/html; charset=iso-8859-1 Content-transfer-encoding: quoted-printable
       =20     Ol=E1 a todos,
 
       =20             Alguem poderia = enunciar=20 os dois teoremas de Kaplanky da an=E1lise combinat=F3ria.
       =20            1) =     O=20 primeiro =E9 referente a seq=FC=EAncias com sucess=F5es
          &nbs= p;        2)   &nb= sp; O=20 segundo =E9 do mesmo tipo, mas em uma disposi=E7=E3o sobre = circunfer=EAncia
       =20            
       =20             = Ex:
       =20             1) Em uma urna = existem=20 n bolas numeradas de 1 a n. De quantas formas podemos pegar k bolas, de = modo que=20 n=E3o haja bolas sucessivas?
 
       =20             2) Doze = cavaleiros=20 est=E3o dispostos em uma t=E1vola redonda, de sorte que cavaleiros = sucessivos s=E3o=20 inimigos. Quantas comiss=F5es de 5 cavaleiros podemos formar, de modo = que n=E3o haja=20 cavaleiros inimigos na comiss=E3o?
       =20
Grato
 
       =20         Daniel
--Boundary_(ID_377HnO5Mf/3fOV7Y3mBUXg)-- ========================================================================= Instruções para entrar na lista, sair da lista e usar a lista em http://www.mat.puc-rio.br/~nicolau/olimp/obm-l.html O administrador desta lista é ========================================================================= From owner-obm-l@sucuri.mat.puc-rio.br Wed Sep 25 13:25:31 2002 Return-Path: Received: (from majordom@localhost) by sucuri.mat.puc-rio.br (8.9.3/8.9.3) id NAA04862 for obm-l-MTTP; Wed, 25 Sep 2002 13:21:09 -0300 Received: from web21306.mail.yahoo.com (web21306.mail.yahoo.com [216.136.129.60]) by sucuri.mat.puc-rio.br (8.9.3/8.9.3) with SMTP id NAA04858 for ; Wed, 25 Sep 2002 13:21:06 -0300 Message-ID: <20020925162257.6336.qmail@web21306.mail.yahoo.com> Received: from [200.247.197.140] by web21306.mail.yahoo.com via HTTP; Wed, 25 Sep 2002 13:22:57 ART Date: Wed, 25 Sep 2002 13:22:57 -0300 (ART) From: =?iso-8859-1?q?Jorge=20Paulino?= Subject: [obm-l] QUESTÃO IME To: obm-l@mat.puc-rio.br MIME-Version: 1.0 Content-Type: text/plain; charset=iso-8859-1 Content-Transfer-Encoding: 8bit Sender: owner-obm-l@sucuri.mat.puc-rio.br Precedence: bulk Reply-To: obm-l@mat.puc-rio.br Galera, gostaria de ajuda numa questão do IME, que provavelmente já foi discutida nessa lista. É aquela onde se constrói um octógno no centro de um quadrado de lado a, a partir de semi-retas com origem nos vértices do quadrado... pede-se a área do octógono em função de a. Na resposta não devem aparecer funções trigonométricas... Se alguém tiver disposição para ajudar, agradeço. Um abraço, Jorge "A partir de cada vértice de um quadrado de lado a trace _______________________________________________________________________ Yahoo! GeoCities Tudo para criar o seu site: ferramentas fáceis de usar, espaço de sobra e acessórios. http://br.geocities.yahoo.com/ ========================================================================= Instruções para entrar na lista, sair da lista e usar a lista em http://www.mat.puc-rio.br/~nicolau/olimp/obm-l.html O administrador desta lista é ========================================================================= From owner-obm-l@sucuri.mat.puc-rio.br Wed Sep 25 14:19:04 2002 Return-Path: Received: (from majordom@localhost) by sucuri.mat.puc-rio.br (8.9.3/8.9.3) id OAA06215 for obm-l-MTTP; Wed, 25 Sep 2002 14:17:24 -0300 Received: from mat.puc-rio.br (IDENT:root@perere.mat.puc-rio.br [139.82.27.60]) by sucuri.mat.puc-rio.br (8.9.3/8.9.3) with ESMTP id OAA06211 for ; Wed, 25 Sep 2002 14:17:21 -0300 Received: from localhost (fredpalm@localhost) by mat.puc-rio.br (8.9.3/8.9.3) with ESMTP id GAA08493 for ; Wed, 25 Sep 2002 06:38:50 -0300 Date: Wed, 25 Sep 2002 06:38:42 -0300 (BRT) From: Carlos Frederico Borges Palmeira To: obm-l@mat.puc-rio.br Subject: [obm-l] Fwd: (fwd) Message-ID: MIME-Version: 1.0 Content-Type: TEXT/PLAIN; charset=X-UNKNOWN Content-Transfer-Encoding: 8bit X-MIME-Autoconverted: from QUOTED-PRINTABLE to 8bit by sucuri.mat.puc-rio.br id OAA06212 Sender: owner-obm-l@sucuri.mat.puc-rio.br Precedence: bulk Reply-To: obm-l@mat.puc-rio.br alguem pode ajudar o meu xara' ? Fred palmeira ---------- Forwarded message ---------- Date: Wed, 25 Sep 2002 13:56:07 -0300 From: Fernanda To: Fred Subject: Fwd: Prof Fred, Nao sei se vc podera ajudar o rapaz num prazo tao curto, mas segue abaixo o pedido. Abraco, FJ >X-Originating-IP: [200.227.191.8] >From: "Fred Alexandre dos Santos da Silva Alexandre" >To: >Date: Wed, 25 Sep 2002 11:58:03 -0300 >X-Mailer: MSN Explorer 6.10.0016.1624 >X-OriginalArrivalTime: 25 Sep 2002 14:56:30.0949 (UTC) >FILETIME=[BB973150:01C264A3] > >Olá tb!! > >Primeiro vou me apresentar, meu nome é Fred Alexandre dos Santos da Silva, >tenho 20anos, e estou no segundo ano de Físisca, na FACAP, em São José dos >Campos, e estou com um probelma, gravissimo, presciso da ajuda de vcs. O >problema é o seguinte, tenho que fazer um trabalho completo sobre o >Wronskiano, sua biografia, como ele chegou em tais conclusões, no geral um >trabalho completo, mas tem um porém não consegui quase nada, e tenho que >entregar o trabalho na sexta, e estou muito preocupado, por não ter achado >nada, gostaria de saber de vcs se poderiam estar me ajudando, com algum >documento ou algum link. > >Muito Obrigado pela atenção, > >Fred Alexandre > > > >---------- >Aproveite melhor a Web. Faça o download GRÁTIS do MSN Explorer : >http://explorer.msn.com.br/intl.asp#po ========================================================================= Instruções para entrar na lista, sair da lista e usar a lista em http://www.mat.puc-rio.br/~nicolau/olimp/obm-l.html O administrador desta lista é ========================================================================= From owner-obm-l@sucuri.mat.puc-rio.br Wed Sep 25 14:53:46 2002 Return-Path: Received: (from majordom@localhost) by sucuri.mat.puc-rio.br (8.9.3/8.9.3) id OAA07018 for obm-l-MTTP; Wed, 25 Sep 2002 14:50:36 -0300 Received: from web12905.mail.yahoo.com (web12905.mail.yahoo.com [216.136.174.72]) by sucuri.mat.puc-rio.br (8.9.3/8.9.3) with SMTP id OAA07004 for ; Wed, 25 Sep 2002 14:50:32 -0300 Message-ID: <20020925175223.62423.qmail@web12905.mail.yahoo.com> Received: from [200.206.103.3] by web12905.mail.yahoo.com via HTTP; Wed, 25 Sep 2002 14:52:23 ART Date: Wed, 25 Sep 2002 14:52:23 -0300 (ART) From: =?iso-8859-1?q?Johann=20Peter=20Gustav=20Lejeune=20Dirichlet?= Subject: [obm-l] Racionais diadicos To: obm-l@mat.puc-rio.br, nicolau@sucuri.mat.puc-rio.br MIME-Version: 1.0 Content-Type: multipart/alternative; boundary="0-475713334-1032976343=:61232" Content-Transfer-Encoding: 8bit Sender: owner-obm-l@sucuri.mat.puc-rio.br Precedence: bulk Reply-To: obm-l@mat.puc-rio.br --0-475713334-1032976343=:61232 Content-Type: text/plain; charset=iso-8859-1 Content-Transfer-Encoding: 8bit O que sao racionais diadicos? --------------------------------- Yahoo! GeoCities Tudo para criar o seu site: ferramentas fáceis de usar, espaço de sobra e acessórios. --0-475713334-1032976343=:61232 Content-Type: text/html; charset=iso-8859-1 Content-Transfer-Encoding: 8bit O que sao racionais diadicos?



Yahoo! GeoCities
Tudo para criar o seu site: ferramentas fáceis de usar, espaço de sobra e acessórios. --0-475713334-1032976343=:61232-- ========================================================================= Instruções para entrar na lista, sair da lista e usar a lista em http://www.mat.puc-rio.br/~nicolau/olimp/obm-l.html O administrador desta lista é ========================================================================= From owner-obm-l@sucuri.mat.puc-rio.br Wed Sep 25 16:38:51 2002 Return-Path: Received: (from majordom@localhost) by sucuri.mat.puc-rio.br (8.9.3/8.9.3) id QAA09184 for obm-l-MTTP; Wed, 25 Sep 2002 16:33:26 -0300 Received: from puma.unisys.com.br (smtp.unisys.com.br [200.220.64.7]) by sucuri.mat.puc-rio.br (8.9.3/8.9.3) with ESMTP id QAA09179 for ; Wed, 25 Sep 2002 16:33:21 -0300 Received: from josefran (riopm18p162.uninet.com.br [200.220.16.162]) by puma.unisys.com.br (8.12.3/8.12.3) with SMTP id g8PJZ2F6023145 for ; Wed, 25 Sep 2002 16:35:03 -0300 (EST) X-Spam-Filter: check_local@puma.unisys.com.br by digitalanswers.org Message-ID: <007b01c264cb$6be7fea0$6b10dcc8@josefran> From: "Jose Francisco Guimaraes Costa" To: References: Subject: Re: [obm-l] Fwd: (fwd) Date: Wed, 25 Sep 2002 16:39:59 -0300 MIME-Version: 1.0 Content-Type: text/plain; charset="iso-8859-1" Content-Transfer-Encoding: 8bit X-Priority: 3 X-MSMail-Priority: Normal X-Mailer: Microsoft Outlook Express 5.00.2314.1300 X-MIMEOLE: Produced By Microsoft MimeOLE V5.00.2314.1300 Sender: owner-obm-l@sucuri.mat.puc-rio.br Precedence: bulk Reply-To: obm-l@mat.puc-rio.br Vá até http://www-gap.dcs.st-and.ac.uk/~history/Mathematicians/Wronski.html. É quase nada, mas quem sabe se não ajuda? Para ver um texto de quem não tinha Wronki em bom conceito, vá até http://www.univ-rouen.fr/upresa6085/Vulgarisation/ew.html JF -----Mensagem Original----- De: Carlos Frederico Borges Palmeira Para: Enviada em: Quarta-feira, 25 de Setembro de 2002 06:38 Assunto: [obm-l] Fwd: (fwd) > > alguem pode ajudar o meu xara' ? > Fred palmeira > > ---------- Forwarded message ---------- > Date: Wed, 25 Sep 2002 13:56:07 -0300 > From: Fernanda > To: Fred > Subject: Fwd: > > Prof Fred, > Nao sei se vc podera ajudar o rapaz num prazo tao curto, mas segue abaixo o > pedido. > Abraco, > FJ > >X-Originating-IP: [200.227.191.8] > >From: "Fred Alexandre dos Santos da Silva Alexandre" > >To: > >Date: Wed, 25 Sep 2002 11:58:03 -0300 > >X-Mailer: MSN Explorer 6.10.0016.1624 > >X-OriginalArrivalTime: 25 Sep 2002 14:56:30.0949 (UTC) > >FILETIME=[BB973150:01C264A3] > > > >Olá tb!! > > > >Primeiro vou me apresentar, meu nome é Fred Alexandre dos Santos da Silva, > >tenho 20anos, e estou no segundo ano de Físisca, na FACAP, em São José dos > >Campos, e estou com um probelma, gravissimo, presciso da ajuda de vcs. O > >problema é o seguinte, tenho que fazer um trabalho completo sobre o > >Wronskiano, sua biografia, como ele chegou em tais conclusões, no geral um > >trabalho completo, mas tem um porém não consegui quase nada, e tenho que > >entregar o trabalho na sexta, e estou muito preocupado, por não ter achado > >nada, gostaria de saber de vcs se poderiam estar me ajudando, com algum > >documento ou algum link. > > > >Muito Obrigado pela atenção, > > > >Fred Alexandre > > > > > > > >---------- > >Aproveite melhor a Web. Faça o download GRÁTIS do MSN Explorer : > >http://explorer.msn.com.br/intl.asp#po > > ========================================================================= > Instruções para entrar na lista, sair da lista e usar a lista em > http://www.mat.puc-rio.br/~nicolau/olimp/obm-l.html > O administrador desta lista é > ========================================================================= > ========================================================================= Instruções para entrar na lista, sair da lista e usar a lista em http://www.mat.puc-rio.br/~nicolau/olimp/obm-l.html O administrador desta lista é ========================================================================= From owner-obm-l@sucuri.mat.puc-rio.br Wed Sep 25 18:11:24 2002 Return-Path: Received: (from majordom@localhost) by sucuri.mat.puc-rio.br (8.9.3/8.9.3) id SAA10872 for obm-l-MTTP; Wed, 25 Sep 2002 18:08:30 -0300 Received: from server3.claretianas.com.br (server3.claretianas.com.br [200.245.46.67]) by sucuri.mat.puc-rio.br (8.9.3/8.9.3) with ESMTP id SAA10868 for ; Wed, 25 Sep 2002 18:08:28 -0300 Received: from local (modem76.claretianas.com.br [200.245.46.236]) by server3.claretianas.com.br (8.11.2/8.11.2) with SMTP id g8PL98J23980 for ; Wed, 25 Sep 2002 18:09:09 -0300 Message-ID: <001601c264d8$ee5d4a40$ec2ef5c8@www.claretianas.com.br> From: "Carlos Roberto de Moraes" To: Subject: [obm-l] combinatoria Date: Wed, 25 Sep 2002 18:17:17 -0300 MIME-Version: 1.0 Content-Type: multipart/alternative; boundary="----=_NextPart_000_0013_01C264BF.C8038440" X-Priority: 3 X-MSMail-Priority: Normal X-Mailer: Microsoft Outlook Express 6.00.2600.0000 X-MimeOLE: Produced By Microsoft MimeOLE V6.00.2600.0000 X-MailScanner: Found to be clean Sender: owner-obm-l@sucuri.mat.puc-rio.br Precedence: bulk Reply-To: obm-l@mat.puc-rio.br This is a multi-part message in MIME format. ------=_NextPart_000_0013_01C264BF.C8038440 Content-Type: text/plain; charset="iso-8859-1" Content-Transfer-Encoding: quoted-printable Podem me ajudar? 1)Um tabuleiro quadrado disp=F5e de 9 orificios dispostos em 3 linhas e = 3 colunas. De quantas maneiras podemos colocar 3 bolas de modo que os = orificios ocupados n=E3o fiquem alinhados? Diagonais s=E3o consideradas = tipos de alinhamento. 2) O total de n=FAmeros constituidos de 3 algarismos impares e 2 = algarismos pares que podem ser formados com os algarismos de 1 a 9, sem = repeti=E7=E3o =E9 igual a qto? ------=_NextPart_000_0013_01C264BF.C8038440 Content-Type: text/html; charset="iso-8859-1" Content-Transfer-Encoding: quoted-printable
Podem me ajudar?
 
1)Um tabuleiro quadrado disp=F5e de 9 orificios = dispostos em 3=20 linhas e 3 colunas. De quantas maneiras podemos colocar 3 bolas de modo = que os=20 orificios ocupados n=E3o fiquem alinhados? Diagonais s=E3o consideradas = tipos de=20 alinhamento.
 
2) O total de n=FAmeros constituidos de 3 algarismos = impares e 2=20 algarismos pares que podem ser formados com os algarismos de 1 a 9, sem=20 repeti=E7=E3o =E9 igual a qto?
------=_NextPart_000_0013_01C264BF.C8038440-- ========================================================================= Instruções para entrar na lista, sair da lista e usar a lista em http://www.mat.puc-rio.br/~nicolau/olimp/obm-l.html O administrador desta lista é ========================================================================= From owner-obm-l@sucuri.mat.puc-rio.br Wed Sep 25 19:00:33 2002 Return-Path: Received: (from majordom@localhost) by sucuri.mat.puc-rio.br (8.9.3/8.9.3) id TAA11970 for obm-l-MTTP; Wed, 25 Sep 2002 19:00:15 -0300 Received: (from nicolau@localhost) by sucuri.mat.puc-rio.br (8.9.3/8.9.3) id TAA11965 for obm-l@mat.puc-rio.br; Wed, 25 Sep 2002 19:00:15 -0300 Date: Wed, 25 Sep 2002 19:00:15 -0300 From: "Nicolau C. Saldanha" To: obm-l@mat.puc-rio.br Subject: Re: [obm-l] Racionais diadicos Message-ID: <20020925190015.B11842@sucuri.mat.puc-rio.br> References: <20020925175223.62423.qmail@web12905.mail.yahoo.com> Mime-Version: 1.0 Content-Type: text/plain; charset=iso-8859-1 Content-Disposition: inline Content-Transfer-Encoding: 8bit User-Agent: Mutt/1.2.5i In-Reply-To: <20020925175223.62423.qmail@web12905.mail.yahoo.com>; from peterdirichlet2002@yahoo.com.br on Wed, Sep 25, 2002 at 02:52:23PM -0300 Sender: owner-obm-l@sucuri.mat.puc-rio.br Precedence: bulk Reply-To: obm-l@mat.puc-rio.br On Wed, Sep 25, 2002 at 02:52:23PM -0300, Johann Peter Gustav Lejeune Dirichlet wrote: > O que sao racionais diadicos? Um racional cujo denominador é uma potência de 2. []s, N. ========================================================================= Instruções para entrar na lista, sair da lista e usar a lista em http://www.mat.puc-rio.br/~nicolau/olimp/obm-l.html O administrador desta lista é ========================================================================= From owner-obm-l@sucuri.mat.puc-rio.br Wed Sep 25 20:01:52 2002 Return-Path: Received: (from majordom@localhost) by sucuri.mat.puc-rio.br (8.9.3/8.9.3) id UAA12804 for obm-l-MTTP; Wed, 25 Sep 2002 20:01:32 -0300 Received: from hotmail.com (dav55.pav3.hotmail.com [64.4.38.159]) by sucuri.mat.puc-rio.br (8.9.3/8.9.3) with ESMTP id UAA12800 for ; Wed, 25 Sep 2002 20:01:30 -0300 Received: from mail pickup service by hotmail.com with Microsoft SMTPSVC; Wed, 25 Sep 2002 16:03:21 -0700 X-Originating-IP: [200.163.21.132] From: "Carlos Bergfeld" To: "obm-l" Subject: [obm-l] =?iso-8859-1?Q?Re:_=5Bobm-l=5D_QUEST=C3O_IME?= Date: Wed, 25 Sep 2002 20:03:20 -0300 MIME-Version: 1.0 X-Mailer: MSN Explorer 6.10.0016.1624 Content-Type: multipart/alternative; boundary="----=_NextPart_001_0000_01C264CE.9883CBD0" Message-ID: X-OriginalArrivalTime: 25 Sep 2002 23:03:21.0744 (UTC) FILETIME=[BE94C900:01C264E7] Sender: owner-obm-l@sucuri.mat.puc-rio.br Precedence: bulk Reply-To: obm-l@mat.puc-rio.br ------=_NextPart_001_0000_01C264CE.9883CBD0 Content-Type: text/plain; charset="iso-8859-1" Content-Transfer-Encoding: quoted-printable Que tal usar anal=EDtica Jorge? Assim poderia encontrar todos os v=E9rtic= es e calcular a =E1rea do oct=F3gono. S=F3 n=E3o sei se seria pr=E1tico! -----Mensagem Original----- De: Jorge Paulino Enviado: quarta-feira, 25 de setembro de 2002 14:09 Para: obm-l@mat.puc-rio.br Assunto: [obm-l] QUEST=C3O IME Galera, gostaria de ajuda numa quest=E3o do IME, que =20 provavelmente j=E1 foi discutida nessa lista. =C9 aquela onde se constr=F3i um oct=F3gno no centro de um quadrado de lado a, a partir de semi-retas com origem nos v=E9rtices do quadrado... pede-se a =E1rea do oct=F3gono em fun=E7=E3o de a. Na resposta n=E3o devem aparecer =20 fun=E7=F5es trigonom=E9tricas... Se algu=E9m tiver disposi=E7=E3o para ajudar, agrade=E7o. Um abra=E7o, Jorge "A partir de cada v=E9rtice de um quadrado de lado a trace =20 _______________________________________________________________________ Yahoo! GeoCities Tudo para criar o seu site: ferramentas f=E1ceis de usar, espa=E7o de sob= ra e acess=F3rios. http://br.geocities.yahoo.com/ =3D=3D=3D=3D=3D=3D=3D=3D=3D=3D=3D=3D=3D=3D=3D=3D=3D=3D=3D=3D=3D=3D=3D=3D=3D= =3D=3D=3D=3D=3D=3D=3D=3D=3D=3D=3D=3D=3D=3D=3D=3D=3D=3D=3D=3D=3D=3D=3D=3D=3D= =3D=3D=3D=3D=3D=3D=3D=3D=3D=3D=3D=3D=3D=3D=3D=3D=3D=3D=3D=3D=3D=3D=3D Instru=E7=F5es para entrar na lista, sair da lista e usar a lista em http://www.mat.puc-rio.br/~nicolau/olimp/obm-l.html O administrador desta lista =E9 =3D=3D=3D=3D=3D=3D=3D=3D=3D=3D=3D=3D=3D=3D=3D=3D=3D=3D=3D=3D=3D=3D=3D=3D=3D= =3D=3D=3D=3D=3D=3D=3D=3D=3D=3D=3D=3D=3D=3D=3D=3D=3D=3D=3D=3D=3D=3D=3D=3D=3D= =3D=3D=3D=3D=3D=3D=3D=3D=3D=3D=3D=3D=3D=3D=3D=3D=3D=3D=3D=3D=3D=3D=3DApro= veite melhor a Web. Fa=E7a o download GR=C1TIS do MSN Explorer : http://e= xplorer.msn.com.br/intl.asp#po ------=_NextPart_001_0000_01C264CE.9883CBD0 Content-Type: text/html; charset="iso-8859-1" Content-Transfer-Encoding: quoted-printable
Que tal usar a= nal=EDtica Jorge? Assim poderia encontrar todos os v=E9rtices e calcular = a =E1rea do oct=F3gono. S=F3 n=E3o sei se seria pr=E1tico!
&nb= sp;
-----Mensagem Original-----
De: Jorge Pa= ulino
Enviado: quarta-feira,= 25 de setembro de 2002 14:09
Pa= ra: obm-l@mat.puc-rio.br
Ass= unto: [obm-l] QUEST=C3O IME
 
Galera, gostaria d= e ajuda numa quest=E3o do IME, que
provavelmente j=E1 foi discutida n= essa lista.
=C9 aquela onde se constr=F3i um oct=F3gno no centro de um=
quadrado de lado a, a partir de semi-retas com origem
nos v=E9rtic= es do quadrado... pede-se a =E1rea do oct=F3gono

em fun=E7=E3o de = a. Na resposta n=E3o devem aparecer
fun=E7=F5es trigonom=E9tricas...<= BR>Se algu=E9m tiver disposi=E7=E3o para ajudar, agrade=E7o.
Um abra=E7= o,
Jorge



"A partir de cada v=E9rtice de um quadrado de = lado a
trace

_________________________________________________= ______________________
Yahoo! GeoCities
Tudo para criar o seu site:= ferramentas f=E1ceis de usar, espa=E7o de sobra e acess=F3rios.
http:= //br.geocities.yahoo.com/
=3D=3D=3D=3D=3D=3D=3D=3D=3D=3D=3D=3D=3D=3D=3D= =3D=3D=3D=3D=3D=3D=3D=3D=3D=3D=3D=3D=3D=3D=3D=3D=3D=3D=3D=3D=3D=3D=3D=3D=3D= =3D=3D=3D=3D=3D=3D=3D=3D=3D=3D=3D=3D=3D=3D=3D=3D=3D=3D=3D=3D=3D=3D=3D=3D=3D= =3D=3D=3D=3D=3D=3D=3D=3D
Instru=E7=F5es para entrar na lista, sair da = lista e usar a lista em
http://www.mat.puc-rio.br/~nicolau/olimp/obm-l= .html
O administrador desta lista =E9 <nicolau@mat.puc-rio.br>=3D=3D=3D=3D=3D=3D=3D=3D=3D=3D=3D=3D=3D=3D=3D=3D=3D=3D=3D=3D=3D=3D=3D=3D= =3D=3D=3D=3D=3D=3D=3D=3D=3D=3D=3D=3D=3D=3D=3D=3D=3D=3D=3D=3D=3D=3D=3D=3D=3D= =3D=3D=3D=3D=3D=3D=3D=3D=3D=3D=3D=3D=3D=3D=3D=3D=3D=3D=3D=3D=3D=3D=3D=3D<= BR>


Aproveite melhor a Web.= Fa=E7a o download GR=C1TIS do MSN Explorer : http://explorer.msn.com.br/intl.asp#po
------=_NextPart_001_0000_01C264CE.9883CBD0-- ========================================================================= Instruções para entrar na lista, sair da lista e usar a lista em http://www.mat.puc-rio.br/~nicolau/olimp/obm-l.html O administrador desta lista é ========================================================================= From owner-obm-l@sucuri.mat.puc-rio.br Wed Sep 25 20:44:04 2002 Return-Path: Received: (from majordom@localhost) by sucuri.mat.puc-rio.br (8.9.3/8.9.3) id UAA13530 for obm-l-MTTP; Wed, 25 Sep 2002 20:43:18 -0300 Received: from pretoria.ime.unicamp.br (pretoria.ime.unicamp.br [143.106.22.7]) by sucuri.mat.puc-rio.br (8.9.3/8.9.3) with ESMTP id UAA13526 for ; Wed, 25 Sep 2002 20:43:16 -0300 Received: from VELOZ (lei058.lei.ime.unicamp.br [143.106.118.58]) by pretoria.ime.unicamp.br (8.9.0/8.9.0) with ESMTP id UAA18416 for ; Wed, 25 Sep 2002 20:45:20 -0300 (EST) From: "Mario Salvatierra Junior" To: Subject: [obm-l] =?iso-8859-1?Q?=C1lgebra_Linear?= Date: Wed, 25 Sep 2002 20:45:25 -0300 Message-ID: <000001c264ed$a22efb90$3a766a8f@VELOZ> MIME-Version: 1.0 Content-Type: multipart/alternative; boundary="----=_NextPart_000_0001_01C264D4.7CE1C390" X-Priority: 3 (Normal) X-MSMail-Priority: Normal X-Mailer: Microsoft Outlook, Build 10.0.2627 Importance: Normal X-MimeOLE: Produced By Microsoft MimeOLE V6.00.2600.0000 Sender: owner-obm-l@sucuri.mat.puc-rio.br Precedence: bulk Reply-To: obm-l@mat.puc-rio.br This is a multi-part message in MIME format. ------=_NextPart_000_0001_01C264D4.7CE1C390 Content-Type: text/plain; charset="iso-8859-1" Content-Transfer-Encoding: quoted-printable Algu=E9m pode me informar onde encontro um livro bom de =C1lgebra Linear = (em portugu=EAs ou ingl=EAs ) dispon=EDvel na net em pdf ou ps que n=E3o = tenha muito mais que 200 p=E1ginas? ------=_NextPart_000_0001_01C264D4.7CE1C390 Content-Type: text/html; charset="iso-8859-1" Content-Transfer-Encoding: quoted-printable

Algu=E9m pode me informar onde encontro um livro bom = de =C1lgebra Linear (em portugu=EAs ou ingl=EAs ) = dispon=EDvel na net em pdf ou ps que n=E3o tenha muito mais que 200 = p=E1ginas?

------=_NextPart_000_0001_01C264D4.7CE1C390-- ========================================================================= Instruções para entrar na lista, sair da lista e usar a lista em http://www.mat.puc-rio.br/~nicolau/olimp/obm-l.html O administrador desta lista é ========================================================================= From owner-obm-l@sucuri.mat.puc-rio.br Wed Sep 25 22:43:00 2002 Return-Path: Received: (from majordom@localhost) by sucuri.mat.puc-rio.br (8.9.3/8.9.3) id WAA14789 for obm-l-MTTP; Wed, 25 Sep 2002 22:42:48 -0300 Received: from cassino.digi.com.br (cassino.digi.com.br [200.241.100.140]) by sucuri.mat.puc-rio.br (8.9.3/8.9.3) with ESMTP id WAA14785 for ; Wed, 25 Sep 2002 22:42:46 -0300 Received: from david (host072.d.digizap.com.br [200.249.6.72]) by cassino.digi.com.br (8.11.6/8.11.6) with SMTP id g8Q1jge19339 for ; Wed, 25 Sep 2002 22:45:43 -0300 Message-ID: <08fe01c264fe$f99111a0$4806f9c8@david> From: "David Ricardo" To: References: <000001c264ed$a22efb90$3a766a8f@VELOZ> Subject: [obm-l] =?iso-8859-1?Q?Re:_=5Bobm-l=5D_=C1lgebra_Linear?= Date: Wed, 25 Sep 2002 22:49:27 -0300 MIME-Version: 1.0 Content-Type: text/plain; charset="iso-8859-1" Content-Transfer-Encoding: 8bit X-Priority: 3 X-MSMail-Priority: Normal X-Mailer: Microsoft Outlook Express 5.00.2615.200 X-MimeOLE: Produced By Microsoft MimeOLE V5.00.2615.200 X-MailScanner: Found to be clean Sender: owner-obm-l@sucuri.mat.puc-rio.br Precedence: bulk Reply-To: obm-l@mat.puc-rio.br Vá em http://www.mat.ufmg.br/~regi/ Tem os seguintes livros em PDF: - Matrizes Vetores e Geometria Analítica - Álgebra Linear e Aplicações - Um Curso de Geometria Analítica e Álgebra Linear - Introdução à Álgebra Linear E outras apostilas... Não sei se o material todo dá mais de 200 páginas, mas ele é muito bom. []s David ----- Original Message ----- From: Mario Salvatierra Junior To: obm-l@mat.puc-rio.br Sent: Wednesday, September 25, 2002 8:45 PM Subject: [obm-l] Álgebra Linear Alguém pode me informar onde encontro um livro bom de Álgebra Linear (em português ou inglês ) disponível na net em pdf ou ps que não tenha muito mais que 200 páginas? ========================================================================= Instruções para entrar na lista, sair da lista e usar a lista em http://www.mat.puc-rio.br/~nicolau/olimp/obm-l.html O administrador desta lista é ========================================================================= From owner-obm-l@sucuri.mat.puc-rio.br Wed Sep 25 22:47:40 2002 Return-Path: Received: (from majordom@localhost) by sucuri.mat.puc-rio.br (8.9.3/8.9.3) id WAA14867 for obm-l-MTTP; Wed, 25 Sep 2002 22:47:36 -0300 Received: from hotmail.com (f156.sea1.hotmail.com [207.68.163.156]) by sucuri.mat.puc-rio.br (8.9.3/8.9.3) with ESMTP id WAA14863 for ; Wed, 25 Sep 2002 22:47:33 -0300 Received: from mail pickup service by hotmail.com with Microsoft SMTPSVC; Wed, 25 Sep 2002 18:49:26 -0700 Received: from 200.204.36.111 by sea1fd.sea1.hotmail.msn.com with HTTP; Thu, 26 Sep 2002 01:49:25 GMT X-Originating-IP: [200.204.36.111] From: "David Turchick" To: obm-l@mat.puc-rio.br Subject: Re: [obm-l] Racionais diadicos Date: Wed, 25 Sep 2002 22:49:25 -0300 Mime-Version: 1.0 Content-Type: text/plain; charset=iso-8859-1; format=flowed Message-ID: X-OriginalArrivalTime: 26 Sep 2002 01:49:26.0164 (UTC) FILETIME=[F1D69D40:01C264FE] Sender: owner-obm-l@sucuri.mat.puc-rio.br Precedence: bulk Reply-To: obm-l@mat.puc-rio.br >O que sao racionais diadicos? São os que podem ser escritos na forma n/2^k, para n e k inteiros. _________________________________________________________________ MSN Photos é a maneira mais fácil e prática de editar e compartilhar sua fotos: http://photos.msn.com.br ========================================================================= Instruções para entrar na lista, sair da lista e usar a lista em http://www.mat.puc-rio.br/~nicolau/olimp/obm-l.html O administrador desta lista é ========================================================================= From owner-obm-l@sucuri.mat.puc-rio.br Wed Sep 25 22:52:17 2002 Return-Path: Received: (from majordom@localhost) by sucuri.mat.puc-rio.br (8.9.3/8.9.3) id WAA14959 for obm-l-MTTP; Wed, 25 Sep 2002 22:52:10 -0300 Received: from silva5.uol.com.br (silva5.uol.com.br [200.221.4.52]) by sucuri.mat.puc-rio.br (8.9.3/8.9.3) with ESMTP id WAA14955 for ; Wed, 25 Sep 2002 22:52:08 -0300 Received: from u2z7z2 ([200.158.144.12]) by silva5.uol.com.br (8.9.1/8.9.1) with ESMTP id WAA14062 for ; Wed, 25 Sep 2002 22:56:34 -0300 (EST) Message-ID: <000901c264ff$c4aceb20$0c909ec8@u2z7z2> From: "Wagner" To: References: <000001c2623c$ae5bb240$15b3bfc8@xxx> Subject: [obm-l] Determinante da matriz de Hilbert (to Haroldo) Date: Wed, 25 Sep 2002 22:55:18 -0300 Organization: Wagner MIME-Version: 1.0 Content-Type: multipart/alternative; boundary="----=_NextPart_000_0006_01C264E6.9EC10220" X-Priority: 3 X-MSMail-Priority: Normal X-Mailer: Microsoft Outlook Express 5.50.4133.2400 X-MimeOLE: Produced By Microsoft MimeOLE V5.50.4133.2400 Sender: owner-obm-l@sucuri.mat.puc-rio.br Precedence: bulk Reply-To: obm-l@mat.puc-rio.br This is a multi-part message in MIME format. ------=_NextPart_000_0006_01C264E6.9EC10220 Content-Type: text/plain; charset="iso-8859-1" Content-Transfer-Encoding: quoted-printable Oi para todos! Consegui arranjar uma resposta para a pergunta: Seja Hn a matriz de Hilbert de ordem n e |A|=3Ddet Hn .Multiplicando as = m-=E9simas linhas e colunas por (m+n-1) vem: |A|=3Dx1.|B|. Em que B =E9 = a matriz ap=F3s as multiplica=E7=F5es e x1=3D((n-1)! / (2n-1)! )^2. ( = =C9 importante n=E3o simplificar os termos das matrizes que aparecerem = no problema para uma melhor compreens=E3o da resposta ).Em B aplique a = regra de Chi=F3 para chegar em |C| de ordem n-1. |A|=3Dx1.a1.|C| . Em = que a1 =E9 o termo a b,b em que b =E9 a ordem do determinante analisado. = Em C, multiplique as m-=E9simas linhas e colunas por (m+n-2). Seja D a = matriz formada pelas multiplica=E7=F5es,=20 |A|=3Dx1.a1.x2.|D|, em que x2=3D((n-2)! / (2n-3)! )^2. Aplique a regra = de Chi=F3. |A|=3Da1.x1.a2.x2.|E|. Mantenha o processo de multiplicar as = m-=E9simas linhas e colunas por=20 (m+o-1), no determinante de ordem o e de aplicar a regra de Chi=F3 = isolando sempre o termo a o,o, at=E9 chegar a uma matriz 1x1. O padr=E3o de multiplica=E7=F5es adotado =E9 tal que ap=F3s a = multiplica=E7=E3o por (m+o-1), a o,o =3D(2o-1).((n-o)! )^2 e xi =3D = ((n-i)! / (2n-2i+1)! )^2 det Hn =3D |A| =3D a1.x1.a2.x2...anxn. ao,o =3D a(n-o+1) =3D> Se P{n}{i=3D1} xi =E9 o produto de n termos xi em = fun=E7=E3o de i, tal que i varia de 1 em 1e i =E9 natural n=E3o nulo e = i
Oi para todos!
 
Consegui arranjar uma resposta para a=20 pergunta:
 
Seja Hn a matriz de Hilbert de ordem n=20 e |A|=3Ddet Hn .Multiplicando as m-=E9simas linhas e colunas por = (m+n-1) vem:=20 |A|=3Dx1.|B|. Em que B =E9 a = matriz ap=F3s as=20 multiplica=E7=F5es e x1=3D((n-1)! / = (2n-1)! )^2. ( =C9=20 importante n=E3o simplificar os termos das matrizes que aparecerem no = problema=20 para uma melhor compreens=E3o da resposta ).Em B aplique a regra de = Chi=F3 para=20 chegar em |C| de ordem n-1. |A|=3Dx1.a1.|C| . Em que = a1 =E9 o termo a b,b = em que b =E9 a ordem do determinante analisado. Em C, = multiplique as=20 m-=E9simas linhas e colunas por (m+n-2). Seja D a matriz formada pelas=20 multiplica=E7=F5es,
|A|=3Dx1.a1.x2.|D|, em que x2=3D((n-2)! / (2n-3)! )^2. Aplique a regra de Chi=F3. = |A|=3Da1.x1.a2.x2.|E|. Mantenha o processo de multiplicar = as m-=E9simas=20 linhas e colunas por
(m+o-1), no determinante = de ordem o=20 e de aplicar a regra de Chi=F3 isolando sempre o termo a = o,o, at=E9 chegar a uma matriz = 1x1.
O padr=E3o de multiplica=E7=F5es = adotado =E9 tal que ap=F3s a=20 multiplica=E7=E3o por (m+o-1), a o,o =3D(2o-1).((n-o)! )^2 e xi =3D=20 ((n-i)! / (2n-2i+1)! )^2
det Hn =3D |A| =3D a1.x1.a2.x2...anxn.
ao,o =3D=20 a(n-o+1) =3D> Se = P{n}{i=3D1} xi =E9 o produto de n termos xi em = fun=E7=E3o de i,=20 tal que i varia de 1 em 1e i =E9 natural n=E3o nulo e i<ou=3Dn, = logo=20 a resposta =E9:
|A| =3D det Hn =3D P{n}{i=3D1} ai.xi, em que=20 ai =3D = (2i-1).((n-i)!)^2  e =20 xi =3D((n-i)! /=20 (2n-2i+1)!)^2
 
OBS: Os padr=F5es de ai = e de xi s=E3o = consequ=EAncias de=20 padr=F5es existente entre os numeradores dos termos da matriz quando = eles n=E3o s=E3o=20 simplificados. A complexidade desse padr=E3o aumenta proporcionalmente = ao n=FAmero=20 de vezes que a regra de Chi=F3 foi aplicada.
 
Andr=E9 T.
 
 
----- Original Message -----
From:=20 haroldo=20
Sent: Sunday, September 22, = 2002 10:33=20 AM
Subject: [obm-l] determinantes =

Gostaria de ajuda no = c=E1lculo desse=20 determinante:

 

 

        =20 1  =20   1/2    1/3   ....   = 1/n

        = 1/2   1/3    1/4  ....   =20 1/(n+1)

       =  1/3   1/4    1/5    ...   1/(n+2)

      =20 .... =20 ..................................

        = = ......................................

       =  1/n   1/(n+1)  ... .....  1/(2n-1)

 

 

grato=20

HAROLDO.

------=_NextPart_000_0006_01C264E6.9EC10220-- ========================================================================= Instruções para entrar na lista, sair da lista e usar a lista em http://www.mat.puc-rio.br/~nicolau/olimp/obm-l.html O administrador desta lista é ========================================================================= From owner-obm-l@sucuri.mat.puc-rio.br Thu Sep 26 10:26:11 2002 Return-Path: Received: (from majordom@localhost) by sucuri.mat.puc-rio.br (8.9.3/8.9.3) id KAA20764 for obm-l-MTTP; Thu, 26 Sep 2002 10:24:35 -0300 Received: from hotmail.com (f130.sea2.hotmail.com [207.68.165.130]) by sucuri.mat.puc-rio.br (8.9.3/8.9.3) with ESMTP id KAA20760 for ; Thu, 26 Sep 2002 10:24:32 -0300 Received: from mail pickup service by hotmail.com with Microsoft SMTPSVC; Thu, 26 Sep 2002 06:26:26 -0700 Received: from 200.193.162.187 by sea2fd.sea2.hotmail.msn.com with HTTP; Thu, 26 Sep 2002 13:26:25 GMT X-Originating-IP: [200.193.162.187] From: =?iso-8859-1?B?ZmlsaXBlIGZhbGPjbw==?= To: obm-l@mat.puc-rio.br Subject: Re: [obm-l] combinatoria Date: Thu, 26 Sep 2002 10:26:25 -0300 Mime-Version: 1.0 Content-Type: text/plain; charset=iso-8859-1; format=flowed Message-ID: X-OriginalArrivalTime: 26 Sep 2002 13:26:26.0114 (UTC) FILETIME=[50789220:01C26560] Sender: owner-obm-l@sucuri.mat.puc-rio.br Precedence: bulk Reply-To: obm-l@mat.puc-rio.br No primeiro exercicio encontre o número total de acontecimentos (9*8*7), depois subtraia os casos de alinhamento (se nao me engano sao 8) e pronto. No segundo basta observar que sao 5 elementos impares e 4 pares, como se quer o total de números constituidos de 3 algarismos impares e 2 algarismos pares temos 5*4*3*4*3, acho que eh isso... []'s Filipe Falcão >From: "Carlos Roberto de Moraes" >Reply-To: obm-l@mat.puc-rio.br >To: >Subject: [obm-l] combinatoria >Date: Wed, 25 Sep 2002 18:17:17 -0300 > >Podem me ajudar? > >1)Um tabuleiro quadrado dispõe de 9 orificios dispostos em 3 linhas e 3 >colunas. De quantas maneiras podemos colocar 3 bolas de modo que os >orificios ocupados não fiquem alinhados? Diagonais são consideradas tipos >de alinhamento. > >2) O total de números constituidos de 3 algarismos impares e 2 algarismos >pares que podem ser formados com os algarismos de 1 a 9, sem repetição é >igual a qto? "Saber e não fazer, ainda é não saber." _________________________________________________________________ Converse com seus amigos online, faça o download grátis do MSN Messenger: http://messenger.msn.com.br ========================================================================= Instruções para entrar na lista, sair da lista e usar a lista em http://www.mat.puc-rio.br/~nicolau/olimp/obm-l.html O administrador desta lista é ========================================================================= From owner-obm-l@sucuri.mat.puc-rio.br Thu Sep 26 10:42:59 2002 Return-Path: Received: (from majordom@localhost) by sucuri.mat.puc-rio.br (8.9.3/8.9.3) id KAA21233 for obm-l-MTTP; Thu, 26 Sep 2002 10:42:25 -0300 Received: from trex.centroin.com.br (trex.centroin.com.br [200.225.63.134]) by sucuri.mat.puc-rio.br (8.9.3/8.9.3) with ESMTP id KAA21226 for ; Thu, 26 Sep 2002 10:42:16 -0300 Received: from trex.centroin.com.br (localhost [127.0.0.1]) by trex.centroin.com.br (8.12.5/8.12.1) with ESMTP id g8QDiIMY021995 for ; Thu, 26 Sep 2002 10:44:33 -0300 (BRT) Received: by trex.centroin.com.br (8.12.5/8.12.5/Submit) id g8QDi8Pe021849; Thu, 26 Sep 2002 10:44:08 -0300 (BRT) Message-Id: <200209261344.g8QDi8Pe021849@trex.centroin.com.br> Received: from 200.165.196.172 by trex.centroin.com.br (CIPWM versao 1.4C1) with HTTPS for ; Thu, 26 Sep 2002 10:44:03 -0300 (BRT) Date: Thu, 26 Sep 2002 10:44:03 -0300 (BRT) From: Augusto Cesar de Oliveira Morgado To: obm-l@mat.puc-rio.br Subject: Re: [obm-l] combinatoria MIME-Version: 1.0 X-Mailer: CentroIn Internet Provider WebMail v. 1.4C1 (http://www.centroin.com.br/) Content-Type: text/plain; charset="iso-8859-1" Content-Transfer-Encoding: 8bit X-MIME-Autoconverted: from quoted-printable to 8bit by sucuri.mat.puc-rio.br id KAA21230 Sender: owner-obm-l@sucuri.mat.puc-rio.br Precedence: bulk Reply-To: obm-l@mat.puc-rio.br Em Wed, 25 Sep 2002 18:17:17 -0300, Carlos Roberto de Moraes disse: > Podem me ajudar? > > 1)Um tabuleiro quadrado dispõe de 9 orificios dispostos em 3 linhas e 3 colunas. De quantas maneiras podemos colocar 3 bolas de modo que os orificios ocupados não fiquem alinhados? Diagonais são consideradas tipos de alinhamento. > > 2) O total de números constituidos de 3 algarismos impares e 2 algarismos pares que podem ser formados com os algarismos de 1 a 9, sem repetição é igual a qto? 1) Supondo as bolas iguais, ha C(9,3)= 84 modos de coloca-las no tabuleiro. Excluindo as 3 horizontais, as 3 verticais e as 2 diagonais, obtemos a resposta 84-8 = 76. Supondo as bolas diferentes, a resposta passa a ser 76 x 3! 2)Podemos escolher 3 impares de C(5,3)=10 modos e 2 pares de C(5,2) = 10 modos. Escolhidos os algarismos, ha 5!=120 modos de arruma-los, e a resposta seria 10 x 10 x 120 = 12 000. Entretanto, devemos excluir os numeros começados por zero que sao 4(numero de modos de escolher o outro algarismo par) x 10 (numero de modos de escolher os algarismos impares) x 4!(numero de modos de arruma-los com o zero no primeiro lugar). A resposta eh 12 000 - 960 = 11 040. ========================================================================= Instruções para entrar na lista, sair da lista e usar a lista em http://www.mat.puc-rio.br/~nicolau/olimp/obm-l.html O administrador desta lista é ========================================================================= From owner-obm-l@sucuri.mat.puc-rio.br Thu Sep 26 12:12:42 2002 Return-Path: Received: (from majordom@localhost) by sucuri.mat.puc-rio.br (8.9.3/8.9.3) id MAA23291 for obm-l-MTTP; Thu, 26 Sep 2002 12:11:29 -0300 Received: from lampiao.digi.com.br (dial-up-a-nat-060.digi.com.br [200.241.100.60] (may be forged)) by sucuri.mat.puc-rio.br (8.9.3/8.9.3) with ESMTP id MAA23287 for ; Thu, 26 Sep 2002 12:11:26 -0300 Received: from david (host072.d.digizap.com.br [200.249.6.72]) by lampiao.digi.com.br (8.11.2/8.11.6) with SMTP id g8QFCj726963 for ; Thu, 26 Sep 2002 12:12:45 -0300 Message-ID: <008e01c26570$12db4e20$4806f9c8@david> From: "David Ricardo" To: References: <000001c264ed$a22efb90$3a766a8f@VELOZ> <08fe01c264fe$f99111a0$4806f9c8@david> Subject: [obm-l] =?iso-8859-1?Q?Re:_=5Bobm-l=5D_Re:_=5Bobm-l=5D_=C1lgebra_Linear?= Date: Thu, 26 Sep 2002 12:17:55 -0300 MIME-Version: 1.0 Content-Type: text/plain; charset="iso-8859-1" Content-Transfer-Encoding: 8bit X-Priority: 3 X-MSMail-Priority: Normal X-Mailer: Microsoft Outlook Express 5.00.2615.200 X-MimeOLE: Produced By Microsoft MimeOLE V5.00.2615.200 X-MailScanner: Found to be clean Sender: owner-obm-l@sucuri.mat.puc-rio.br Precedence: bulk Reply-To: obm-l@mat.puc-rio.br > Não sei se o material todo dá mais de 200 páginas, mas ele é muito bom. Na verdade cada livro tem cerca de 600 paginas... hehehehe :) Foi mal! []s David ========================================================================= Instruções para entrar na lista, sair da lista e usar a lista em http://www.mat.puc-rio.br/~nicolau/olimp/obm-l.html O administrador desta lista é ========================================================================= From owner-obm-l@sucuri.mat.puc-rio.br Thu Sep 26 13:24:04 2002 Return-Path: Received: (from majordom@localhost) by sucuri.mat.puc-rio.br (8.9.3/8.9.3) id NAA24662 for obm-l-MTTP; Thu, 26 Sep 2002 13:22:08 -0300 Received: from salvatore4.bol.com.br ([200.221.24.52]) by sucuri.mat.puc-rio.br (8.9.3/8.9.3) with ESMTP id NAA24658 for ; Thu, 26 Sep 2002 13:22:05 -0300 Received: from bol.com.br (200.221.24.136) by salvatore4.bol.com.br (5.1.071) id 3D8B888000136EB9 for obm-l@mat.puc-rio.br; Thu, 26 Sep 2002 13:23:52 -0300 Date: Thu, 26 Sep 2002 13:21:06 -0300 Message-Id: Subject: [obm-l] =?iso-8859-1?q?Assunto_de_Olimp=EDadas?= MIME-Version: 1.0 Content-Type: text/plain;charset="iso-8859-1" From: "tarsis19" To: obm-l@mat.puc-rio.br X-XaM3-API-Version: 2.4.3.4.4 X-SenderIP: 200.129.136.221 Content-Transfer-Encoding: 8bit X-MIME-Autoconverted: from quoted-printable to 8bit by sucuri.mat.puc-rio.br id NAA24659 Sender: owner-obm-l@sucuri.mat.puc-rio.br Precedence: bulk Reply-To: obm-l@mat.puc-rio.br A Universidade do Estado do Pará esta liberando uma verba para projetos. Estou montando um de uma "Oficina para Olimpíada de MAtemática", esta oficina terá como público alvo estudantes de escolas públicas. Estou encontrando dificuldade para montar os assuntos a serem ministrados para primeiro e segundo nível. Se alguém me fornecer os nomes dos assuntos a serem ministrados, seria grato. __________________________________________________________________________ Encontre sempre uma linha desocupada com o Discador BOL! http://www.bol.com.br/discador Ainda não tem AcessoBOL? Assine já! http://www.bol.com.br/acessobol ========================================================================= Instruções para entrar na lista, sair da lista e usar a lista em http://www.mat.puc-rio.br/~nicolau/olimp/obm-l.html O administrador desta lista é ========================================================================= From owner-obm-l@sucuri.mat.puc-rio.br Thu Sep 26 14:34:12 2002 Return-Path: Received: (from majordom@localhost) by sucuri.mat.puc-rio.br (8.9.3/8.9.3) id OAA26154 for obm-l-MTTP; Thu, 26 Sep 2002 14:32:54 -0300 Received: from mat.puc-rio.br (IDENT:root@perere.mat.puc-rio.br [139.82.27.60]) by sucuri.mat.puc-rio.br (8.9.3/8.9.3) with ESMTP id OAA26150 for ; Thu, 26 Sep 2002 14:32:52 -0300 Received: from localhost (fredpalm@localhost) by mat.puc-rio.br (8.9.3/8.9.3) with ESMTP id GAA09024 for ; Thu, 26 Sep 2002 06:54:35 -0300 Date: Thu, 26 Sep 2002 06:54:34 -0300 (BRT) From: Carlos Frederico Borges Palmeira To: obm-l@mat.puc-rio.br Subject: Re: [obm-l] =?iso-8859-1?q?Assunto_de_Olimp=EDadas?= In-Reply-To: Message-ID: MIME-Version: 1.0 Content-Type: TEXT/PLAIN; charset=X-UNKNOWN Content-Transfer-Encoding: 8bit X-MIME-Autoconverted: from QUOTED-PRINTABLE to 8bit by sucuri.mat.puc-rio.br id OAA26151 Sender: owner-obm-l@sucuri.mat.puc-rio.br Precedence: bulk Reply-To: obm-l@mat.puc-rio.br veja na pagina da obm os programas que foram definidos para a olimpiada estadual do rio de janeiro este ano. Acho que e' um ponto de partida. fred palmeira On Thu, 26 Sep 2002, tarsis19 wrote: > A Universidade do Estado do Pará esta liberando uma > verba para projetos. Estou montando um de uma "Oficina > para Olimpíada de MAtemática", esta oficina terá como > público alvo estudantes de escolas públicas. Estou > encontrando dificuldade para montar os assuntos a serem > ministrados para primeiro e segundo nível. Se alguém me > fornecer os nomes dos assuntos a serem ministrados, seria > grato. > > > __________________________________________________________________________ > Encontre sempre uma linha desocupada com o Discador BOL! > http://www.bol.com.br/discador > Ainda não tem AcessoBOL? Assine já! http://www.bol.com.br/acessobol > > > ========================================================================= > Instruções para entrar na lista, sair da lista e usar a lista em > http://www.mat.puc-rio.br/~nicolau/olimp/obm-l.html > O administrador desta lista é > ========================================================================= > ========================================================================= Instruções para entrar na lista, sair da lista e usar a lista em http://www.mat.puc-rio.br/~nicolau/olimp/obm-l.html O administrador desta lista é ========================================================================= From owner-obm-l@sucuri.mat.puc-rio.br Thu Sep 26 16:39:09 2002 Return-Path: Received: (from majordom@localhost) by sucuri.mat.puc-rio.br (8.9.3/8.9.3) id QAA28903 for obm-l-MTTP; Thu, 26 Sep 2002 16:38:35 -0300 Received: from hotmail.com (f76.sea2.hotmail.com [207.68.165.76]) by sucuri.mat.puc-rio.br (8.9.3/8.9.3) with ESMTP id QAA28899 for ; Thu, 26 Sep 2002 16:38:33 -0300 Received: from mail pickup service by hotmail.com with Microsoft SMTPSVC; Thu, 26 Sep 2002 12:40:28 -0700 Received: from 200.220.36.213 by sea2fd.sea2.hotmail.msn.com with HTTP; Thu, 26 Sep 2002 19:40:28 GMT X-Originating-IP: [200.220.36.213] From: "leonardo mattos" To: obm-l@mat.puc-rio.br Subject: Re:Re: [obm-l] combinatoria Date: Thu, 26 Sep 2002 19:40:28 +0000 Mime-Version: 1.0 Content-Type: text/plain; charset=iso-8859-1; format=flowed Message-ID: X-OriginalArrivalTime: 26 Sep 2002 19:40:28.0100 (UTC) FILETIME=[90EFE040:01C26594] Sender: owner-obm-l@sucuri.mat.puc-rio.br Precedence: bulk Reply-To: obm-l@mat.puc-rio.br Ola Morgado, Quanto a questao 2.Ao inves de C(5,2) nao seriam C(4,2)?!Pq na realidade seriam formados numeros com algarismos de 1 a 9, dando um total de 7200. Um abraço,Leonardo >From: Augusto Cesar de Oliveira Morgado >Reply-To: obm-l@mat.puc-rio.br >To: obm-l@mat.puc-rio.br >Subject: Re: [obm-l] combinatoria >Date: Thu, 26 Sep 2002 10:44:03 -0300 (BRT) > >Em Wed, 25 Sep 2002 18:17:17 -0300, Carlos Roberto de Moraes > disse: > > > Podem me ajudar? > > > > 1)Um tabuleiro quadrado dispõe de 9 orificios dispostos em 3 linhas e 3 >colunas. De quantas maneiras podemos colocar 3 bolas de modo que os >orificios ocupados não fiquem alinhados? Diagonais são consideradas tipos >de alinhamento. > > > > 2) O total de números constituidos de 3 algarismos impares e 2 >algarismos pares que podem ser formados com os algarismos de 1 a 9, sem >repetição é igual a qto? >1) Supondo as bolas iguais, ha C(9,3)= 84 modos de coloca-las no tabuleiro. >Excluindo as 3 horizontais, as 3 verticais e as 2 diagonais, obtemos a >resposta 84-8 = 76. >Supondo as bolas diferentes, a resposta passa a ser 76 x 3! >2)Podemos escolher 3 impares de C(5,3)=10 modos e 2 pares de C(5,2) = 10 >modos. Escolhidos os algarismos, ha 5!=120 modos de arruma-los, e a >resposta seria 10 x 10 x 120 = 12 000. >Entretanto, devemos excluir os numeros começados por zero que sao >4(numero de modos de escolher o outro algarismo par) x 10 (numero de modos >de escolher os algarismos impares) x 4!(numero de modos de arruma-los com o >zero no primeiro lugar). >A resposta eh 12 000 - 960 = 11 040. >========================================================================= >Instruções para entrar na lista, sair da lista e usar a lista em >http://www.mat.puc-rio.br/~nicolau/olimp/obm-l.html >O administrador desta lista é >========================================================================= _________________________________________________________________ MSN Photos é a maneira mais fácil e prática de editar e compartilhar sua fotos: http://photos.msn.com.br ========================================================================= Instruções para entrar na lista, sair da lista e usar a lista em http://www.mat.puc-rio.br/~nicolau/olimp/obm-l.html O administrador desta lista é ========================================================================= From owner-obm-l@sucuri.mat.puc-rio.br Thu Sep 26 17:53:39 2002 Return-Path: Received: (from majordom@localhost) by sucuri.mat.puc-rio.br (8.9.3/8.9.3) id RAA30248 for obm-l-MTTP; Thu, 26 Sep 2002 17:52:00 -0300 Received: from zeus.opendf.com.br (zeus.opengate.com.br [200.181.71.10]) by sucuri.mat.puc-rio.br (8.9.3/8.9.3) with ESMTP id RAA30244 for ; Thu, 26 Sep 2002 17:51:57 -0300 Received: from localhost (localhost.opengate.com.br [127.0.0.1]) by zeus.opendf.com.br (Postfix) with ESMTP id 8F8A23EA7C for ; Thu, 26 Sep 2002 17:53:46 -0300 (BRT) Received: by zeus.opendf.com.br (Postfix, from userid 48) id 702F13EC7E; Thu, 26 Sep 2002 17:36:24 -0300 (BRT) From: "498 - Artur Costa Steiner" To: obm-l@mat.puc-rio.br Subject: [obm-l] Alguns pontos interessantes pouco mencionados X-Mailer: NeoMail 1.25 X-IPAddress: 200.252.155.2 MIME-Version: 1.0 Content-Type: text/plain; charset=iso-8859-1 Message-Id: <20020926203624.702F13EC7E@zeus.opendf.com.br> Date: Thu, 26 Sep 2002 17:36:24 -0300 (BRT) X-Virus-Scanned: by AMaViS new-20020517 Sender: owner-obm-l@sucuri.mat.puc-rio.br Precedence: bulk Reply-To: obm-l@mat.puc-rio.br Olá para todos, Sou engenheiro, formei-me no início dos anos 70. Acho curioso que, na cadeira de Cálculo ministrada durante meu curso de Engenharia, não foram sequer mencionados alguns teoremas e conceitos dos quais, mais tarde, vim a tomar conhecimento simplesmente por gostar de matemática . Não que eles sejam de vital importância para um engenheiro, mas eu gostaria de ouvir opiniões, inclusive de professores universitários, sobre os seguintes pontos que, creio, ainda hoje não são citados em cursos de engenharia: 1) Definição precisa de continuidade, frisando-se, inclusive, o conceito de continuidade uniforme- este último parece-me um conceito importante e interessante. 2) O teorema (creio que é conhecido por Teorema de Darbaux) o qual afirma que derivadas de uma função real, de variável real, sempre apresentam a chamada Propriedade do Valor Intermediário. Parece-me que este interessante resultado, de fácil demonstração, não é muito conhecido. 3)Um outro teorema, de simples demonstração (que me parece não ser também muito conhecido), o qual diz que derivadas em R jamais apresentam descontinuidaes do tipo "salto". 4) Apresentação mais precisa do conceito de diferenciabilidade de funções de R^n em R. Quando fiz Engenharia, o meu curso de Cálculo Integral foi muito mais um curso de como determinar primitivas. Os aspectos conceituais da integral não foram frisados, jamais se falou, por exemplo, em Soma de Riemann. s. Creio que alguém com agilidade algébrica pode fazer isso muito bem sem ter a menor idéia do que seja, de fato, o processo de integração. Espero que hoje não seja mais assim. Artur ========================================================================= Instruções para entrar na lista, sair da lista e usar a lista em http://www.mat.puc-rio.br/~nicolau/olimp/obm-l.html O administrador desta lista é ========================================================================= From owner-obm-l@sucuri.mat.puc-rio.br Thu Sep 26 17:55:31 2002 Return-Path: Received: (from majordom@localhost) by sucuri.mat.puc-rio.br (8.9.3/8.9.3) id RAA30331 for obm-l-MTTP; Thu, 26 Sep 2002 17:55:11 -0300 Received: from zeus.opendf.com.br (zeus.opengate.com.br [200.181.71.10]) by sucuri.mat.puc-rio.br (8.9.3/8.9.3) with ESMTP id RAA30327 for ; Thu, 26 Sep 2002 17:55:09 -0300 Received: from localhost (localhost.opengate.com.br [127.0.0.1]) by zeus.opendf.com.br (Postfix) with ESMTP id EFC703EABB for ; Thu, 26 Sep 2002 17:56:58 -0300 (BRT) Received: by zeus.opendf.com.br (Postfix, from userid 48) id 739243EBB0; Thu, 26 Sep 2002 17:40:51 -0300 (BRT) From: "498 - Artur Costa Steiner" To: obm-l@mat.puc-rio.br Subject: [obm-l] Alguns pontos interessantes pouco mencionados X-Mailer: NeoMail 1.25 X-IPAddress: 200.252.155.2 MIME-Version: 1.0 Content-Type: text/plain; charset=iso-8859-1 Message-Id: <20020926204051.739243EBB0@zeus.opendf.com.br> Date: Thu, 26 Sep 2002 17:40:51 -0300 (BRT) X-Virus-Scanned: by AMaViS new-20020517 Sender: owner-obm-l@sucuri.mat.puc-rio.br Precedence: bulk Reply-To: obm-l@mat.puc-rio.br Olá para todos, Sou engenheiro, formei-me no início dos anos 70. Acho curioso que, na cadeira de Cálculo ministrada durante meu curso de Engenharia, não foram sequer mencionados alguns teoremas e conceitos dos quais, mais tarde, vim a tomar conhecimento simplesmente por gostar de matemática . Não que eles sejam de vital importância para um engenheiro, mas eu gostaria de ouvir opiniões, inclusive de professores universitários, sobre os seguintes pontos que, creio, ainda hoje não são citados em cursos de engenharia: 1) Definição precisa de continuidade, frisando-se, inclusive, o conceito de continuidade uniforme- este último parece-me um conceito importante e interessante. 2) O teorema (creio que é conhecido por Teorema de Darbaux) o qual afirma que derivadas de uma função real, de variável real, sempre apresentam a chamada Propriedade do Valor Intermediário. Parece-me que este interessante resultado, de fácil demonstração, não é muito conhecido. 3)Um outro teorema, de simples demonstração (que me parece não ser também muito conhecido), o qual diz que derivadas em R jamais apresentam descontinuidaes do tipo "salto". 4) Apresentação mais precisa do conceito de diferenciabilidade de funções de R^n em R. Quando fiz Engenharia, o meu curso de Cálculo Integral foi muito mais um curso de como determinar primitivas. Os aspectos conceituais da integral não foram frisados, jamais se falou, por exemplo, em Soma de Riemann. s. Creio que alguém com agilidade algébrica pode fazer isso muito bem sem ter a menor idéia do que seja, de fato, o processo de integração. Espero que hoje não seja mais assim. Artur ========================================================================= Instruções para entrar na lista, sair da lista e usar a lista em http://www.mat.puc-rio.br/~nicolau/olimp/obm-l.html O administrador desta lista é ========================================================================= From owner-obm-l@sucuri.mat.puc-rio.br Thu Sep 26 18:12:25 2002 Return-Path: Received: (from majordom@localhost) by sucuri.mat.puc-rio.br (8.9.3/8.9.3) id SAA31072 for obm-l-MTTP; Thu, 26 Sep 2002 18:10:58 -0300 Received: from hotmail.com (oe153.pav0.hotmail.com [64.4.33.146]) by sucuri.mat.puc-rio.br (8.9.3/8.9.3) with ESMTP id SAA31068 for ; Thu, 26 Sep 2002 18:10:54 -0300 Received: from mail pickup service by hotmail.com with Microsoft SMTPSVC; Thu, 26 Sep 2002 14:12:49 -0700 X-Originating-IP: [200.217.188.1] From: "e isso mesmo" To: Subject: [obm-l] Dica Date: Thu, 26 Sep 2002 18:12:49 -0300 MIME-Version: 1.0 X-Mailer: MSN Explorer 6.10.0016.1624 Content-Type: multipart/alternative; boundary="----=_NextPart_001_0002_01C26588.525ED720" Message-ID: X-OriginalArrivalTime: 26 Sep 2002 21:12:49.0693 (UTC) FILETIME=[77FBDCD0:01C265A1] Sender: owner-obm-l@sucuri.mat.puc-rio.br Precedence: bulk Reply-To: obm-l@mat.puc-rio.br ------=_NextPart_001_0002_01C26588.525ED720 Content-Type: text/plain; charset="iso-8859-1" Content-Transfer-Encoding: quoted-printable Me d=EAem uma dica nesta: x+y=3D8 x^2+y^2 =3D m=EDnimo ObrigadoAproveite melhor a Web. Fa=E7a o download GR=C1TIS do MSN Explore= r : http://explorer.msn.com.br/intl.asp#po ------=_NextPart_001_0002_01C26588.525ED720 Content-Type: text/html; charset="iso-8859-1" Content-Transfer-Encoding: quoted-printable
 
<= DIV>Me d=EAem uma dica  nesta:
 
x+y=3D8<= /DIV>
x^2+y^2 =3D m=EDnimo
 
Obrigado
 


Aproveite melhor a = Web. Fa=E7a o download GR=C1TIS do MSN Explorer : http://explorer.msn.com.br/intl.asp#po

------=_NextPart_001_0002_01C26588.525ED720-- ========================================================================= Instruções para entrar na lista, sair da lista e usar a lista em http://www.mat.puc-rio.br/~nicolau/olimp/obm-l.html O administrador desta lista é ========================================================================= From owner-obm-l@sucuri.mat.puc-rio.br Thu Sep 26 18:46:16 2002 Return-Path: Received: (from majordom@localhost) by sucuri.mat.puc-rio.br (8.9.3/8.9.3) id SAA32277 for obm-l-MTTP; Thu, 26 Sep 2002 18:46:01 -0300 Received: from gorgo.centroin.com.br (gorgo.centroin.com.br [200.225.63.128]) by sucuri.mat.puc-rio.br (8.9.3/8.9.3) with ESMTP id SAA32273 for ; Thu, 26 Sep 2002 18:45:58 -0300 Received: from centroin.com.br (du53c.rjo.centroin.com.br [200.225.58.53]) (authenticated bits=0) by gorgo.centroin.com.br (8.12.2/8.12.1) with ESMTP id g8QLmE5k029511 for ; Thu, 26 Sep 2002 18:48:16 -0300 (BRT) Message-ID: <3D938149.4030002@centroin.com.br> Date: Thu, 26 Sep 2002 18:51:05 -0300 From: Augusto =?ISO-8859-1?Q?C=E9sar?= Morgado User-Agent: Mozilla/5.0 (Windows; U; Win98; en-US; rv:0.9.4.1) Gecko/20020508 Netscape6/6.2.3 X-Accept-Language: en-us MIME-Version: 1.0 To: obm-l@mat.puc-rio.br Subject: Re: [obm-l] combinatoria References: <200209261344.g8QDi8Pe021849@trex.centroin.com.br> Content-Type: multipart/alternative; boundary="------------070608090900070506060307" Sender: owner-obm-l@sucuri.mat.puc-rio.br Precedence: bulk Reply-To: obm-l@mat.puc-rio.br --------------070608090900070506060307 Content-Type: text/plain; charset=ISO-8859-1; format=flowed Content-Transfer-Encoding: 8bit Pisei na bola no exercicio 2. Nao reparei que era de 1 a 9. Corrigindo, Podemos escolher 3 impares de C(5,3)=10 modos e 2 pares de C(4,2) = 6 modos. Escolhidos os algarismos, ha 5!=120 modos de arruma-los, e a resposta seria 10 x 6 x 120 = 7 200. Augusto Cesar de Oliveira Morgado wrote: >Em Wed, 25 Sep 2002 18:17:17 -0300, Carlos Roberto de Moraes disse: > >>Podem me ajudar? >> >>1)Um tabuleiro quadrado dispõe de 9 orificios dispostos em 3 linhas e 3 colunas. De quantas maneiras podemos colocar 3 bolas de modo que os orificios ocupados não fiquem alinhados? Diagonais são consideradas tipos de alinhamento. >> >>2) O total de números constituidos de 3 algarismos impares e 2 algarismos pares que podem ser formados com os algarismos de 1 a 9, sem repetição é igual a qto? >> >1) Supondo as bolas iguais, ha C(9,3)= 84 modos de coloca-las no tabuleiro. Excluindo as 3 horizontais, as 3 verticais e as 2 diagonais, obtemos a resposta 84-8 = 76. >Supondo as bolas diferentes, a resposta passa a ser 76 x 3! >2)Podemos escolher 3 impares de C(5,3)=10 modos e 2 pares de C(5,2) = 10 modos. Escolhidos os algarismos, ha 5!=120 modos de arruma-los, e a resposta seria 10 x 10 x 120 = 12 000. >Entretanto, devemos excluir os numeros começados por zero que sao >4(numero de modos de escolher o outro algarismo par) x 10 (numero de modos de escolher os algarismos impares) x 4!(numero de modos de arruma-los com o zero no primeiro lugar). >A resposta eh 12 000 - 960 = 11 040. >========================================================================= >Instruções para entrar na lista, sair da lista e usar a lista em >http://www.mat.puc-rio.br/~nicolau/olimp/obm-l.html >O administrador desta lista é >========================================================================= > > --------------070608090900070506060307 Content-Type: text/html; charset=us-ascii Content-Transfer-Encoding: 7bit Pisei na bola no exercicio 2. Nao reparei que era de 1 a 9.
Corrigindo,
Podemos escolher 3 impares de C(5,3)=10 modos e 2 pares de C(4,2) = 6 modos. Escolhidos os algarismos, ha 5!=120 modos de arruma-los, e a resposta seria 10 x 6 x 120 = 7 200. 


Augusto Cesar de Oliveira Morgado wrote:
Em Wed, 25 Sep 2002 18:17:17 -0300, Carlos Roberto de Moraes <crmoraes@claretianas.com.br> disse:

Podem me ajudar?

1)Um tabuleiro quadrado dispõe de 9 orificios dispostos em 3 linhas e 3 colunas. De quantas maneiras podemos colocar 3 bolas de modo que os orificios ocupados não fiquem alinhados? Diagonais são consideradas tipos de alinhamento.

2) O total de números constituidos de 3 algarismos impares e 2 algarismos pares que podem ser formados com os algarismos de 1 a 9, sem repetição é igual a qto?
1) Supondo as bolas iguais, ha C(9,3)= 84 modos de coloca-las no tabuleiro. Excluindo as 3 horizontais, as 3 verticais e as 2 diagonais, obtemos a resposta 84-8 = 76.
Supondo as bolas diferentes, a resposta passa a ser 76 x 3!
2)Podemos escolher 3 impares de C(5,3)=10 modos e 2 pares de C(5,2) = 10 modos. Escolhidos os algarismos, ha 5!=120 modos de arruma-los, e a resposta seria 10 x 10 x 120 = 12 000.
Entretanto, devemos excluir os numeros começados por zero que sao
4(numero de modos de escolher o outro algarismo par) x 10 (numero de modos de escolher os algarismos impares) x 4!(numero de modos de arruma-los com o zero no primeiro lugar).
A resposta eh 12 000 - 960 = 11 040.
=========================================================================
Instruções para entrar na lista, sair da lista e usar a lista em
h ttp://www.mat.puc-rio.br/~nicolau/olimp/obm-l.html
O administrador desta lista é <nicolau@mat.puc-rio.br>
=========================================================================



--------------070608090900070506060307-- ========================================================================= Instruções para entrar na lista, sair da lista e usar a lista em http://www.mat.puc-rio.br/~nicolau/olimp/obm-l.html O administrador desta lista é ========================================================================= From owner-obm-l@sucuri.mat.puc-rio.br Thu Sep 26 19:10:06 2002 Return-Path: Received: (from majordom@localhost) by sucuri.mat.puc-rio.br (8.9.3/8.9.3) id TAA00479 for obm-l-MTTP; Thu, 26 Sep 2002 19:08:15 -0300 Received: from pretoria.ime.unicamp.br (pretoria.ime.unicamp.br [143.106.22.7]) by sucuri.mat.puc-rio.br (8.9.3/8.9.3) with ESMTP id TAA00474 for ; Thu, 26 Sep 2002 19:08:12 -0300 Received: from VELOZ (lei058.lei.ime.unicamp.br [143.106.118.58]) by pretoria.ime.unicamp.br (8.9.0/8.9.0) with ESMTP id TAA14744 for ; Thu, 26 Sep 2002 19:10:20 -0300 (EST) From: "Mario Salvatierra Junior" To: Subject: [obm-l] =?iso-8859-1?Q?RES:_=5Bobm-l=5D_Re:_=5Bobm-l=5D_Re:_=5Bobm-l=5D_=C1lg?= =?iso-8859-1?Q?ebra_Linear?= Date: Thu, 26 Sep 2002 19:10:06 -0300 Message-ID: <000801c265a9$8aace420$3a766a8f@VELOZ> MIME-Version: 1.0 Content-Type: text/plain; charset="iso-8859-1" X-Priority: 3 (Normal) X-MSMail-Priority: Normal X-Mailer: Microsoft Outlook, Build 10.0.2627 X-MimeOLE: Produced By Microsoft MimeOLE V6.00.2600.0000 In-Reply-To: <008e01c26570$12db4e20$4806f9c8@david> Importance: Normal Content-Transfer-Encoding: 8bit X-MIME-Autoconverted: from quoted-printable to 8bit by sucuri.mat.puc-rio.br id TAA00476 Sender: owner-obm-l@sucuri.mat.puc-rio.br Precedence: bulk Reply-To: obm-l@mat.puc-rio.br Valeu pela dica............na verdade ontem eu já havia encontrado este livro e achei bom também , porém como meu objetivo é imprimir o livro, me desanimei com o número de 600 páginas. Imagine uma resma de papel A4 (500 folhas) + 100 folhas em forma de livro......será um tijolo............. -----Mensagem original----- De: owner-obm-l@sucuri.mat.puc-rio.br [mailto:owner-obm-l@sucuri.mat.puc-rio.br] Em nome de David Ricardo Enviada em: quinta-feira, 26 de setembro de 2002 12:18 Para: obm-l@mat.puc-rio.br Assunto: [obm-l] Re: [obm-l] Re: [obm-l] Álgebra Linear > Não sei se o material todo dá mais de 200 páginas, mas ele é muito bom. Na verdade cada livro tem cerca de 600 paginas... hehehehe :) Foi mal! []s David ======================================================================== = Instruções para entrar na lista, sair da lista e usar a lista em http://www.mat.puc-rio.br/~nicolau/olimp/obm-l.html O administrador desta lista é ======================================================================== = ========================================================================= Instruções para entrar na lista, sair da lista e usar a lista em http://www.mat.puc-rio.br/~nicolau/olimp/obm-l.html O administrador desta lista é ========================================================================= From owner-obm-l@sucuri.mat.puc-rio.br Thu Sep 26 19:29:05 2002 Return-Path: Received: (from majordom@localhost) by sucuri.mat.puc-rio.br (8.9.3/8.9.3) id TAA01185 for obm-l-MTTP; Thu, 26 Sep 2002 19:28:58 -0300 Received: from hotmail.com (f103.law4.hotmail.com [216.33.149.103]) by sucuri.mat.puc-rio.br (8.9.3/8.9.3) with ESMTP id TAA01181 for ; Thu, 26 Sep 2002 19:28:55 -0300 Received: from mail pickup service by hotmail.com with Microsoft SMTPSVC; Thu, 26 Sep 2002 15:30:50 -0700 Received: from 200.186.138.238 by lw4fd.law4.hotmail.msn.com with HTTP; Thu, 26 Sep 2002 22:30:50 GMT X-Originating-IP: [200.186.138.238] From: "Laurito Alves" To: obm-l@mat.puc-rio.br Subject: Re: [obm-l] Dica Date: Thu, 26 Sep 2002 22:30:50 +0000 Mime-Version: 1.0 Content-Type: text/plain; charset=iso-8859-1; format=flowed Message-ID: X-OriginalArrivalTime: 26 Sep 2002 22:30:50.0383 (UTC) FILETIME=[5DE479F0:01C265AC] Sender: owner-obm-l@sucuri.mat.puc-rio.br Precedence: bulk Reply-To: obm-l@mat.puc-rio.br Se x + y = 8 então y = 8 - x. Assim, A = x^2 + (8-x)^2 deve ser mínimo. Desenvolva A e determine seu vértice. Laurito >From: "e isso mesmo" >Reply-To: obm-l@mat.puc-rio.br >To: >Subject: [obm-l] Dica >Date: Thu, 26 Sep 2002 18:12:49 -0300 > > >Me dêem uma dica nesta: > >x+y=8 >x^2+y^2 = mínimo > >ObrigadoAproveite melhor a Web. Faça o download GRÁTIS do MSN Explorer : >http://explorer.msn.com.br/intl.asp#po _________________________________________________________________ MSN Photos é a maneira mais fácil e prática de editar e compartilhar sua fotos: http://photos.msn.com.br ========================================================================= Instruções para entrar na lista, sair da lista e usar a lista em http://www.mat.puc-rio.br/~nicolau/olimp/obm-l.html O administrador desta lista é ========================================================================= From owner-obm-l@sucuri.mat.puc-rio.br Thu Sep 26 19:41:51 2002 Return-Path: Received: (from majordom@localhost) by sucuri.mat.puc-rio.br (8.9.3/8.9.3) id TAA01559 for obm-l-MTTP; Thu, 26 Sep 2002 19:41:42 -0300 Received: from hotmail.com (f78.sea1.hotmail.com [207.68.163.78]) by sucuri.mat.puc-rio.br (8.9.3/8.9.3) with ESMTP id TAA01546 for ; Thu, 26 Sep 2002 19:41:38 -0300 Received: from mail pickup service by hotmail.com with Microsoft SMTPSVC; Thu, 26 Sep 2002 15:43:33 -0700 Received: from 143.107.45.30 by sea1fd.sea1.hotmail.msn.com with HTTP; Thu, 26 Sep 2002 22:43:33 GMT X-Originating-IP: [143.107.45.30] From: "David Turchick" To: obm-l@mat.puc-rio.br Subject: Re: [obm-l] Dica Date: Thu, 26 Sep 2002 19:43:33 -0300 Mime-Version: 1.0 Content-Type: text/plain; charset=iso-8859-1; format=flowed Message-ID: X-OriginalArrivalTime: 26 Sep 2002 22:43:33.0984 (UTC) FILETIME=[2508BE00:01C265AE] Sender: owner-obm-l@sucuri.mat.puc-rio.br Precedence: bulk Reply-To: obm-l@mat.puc-rio.br >Me dêem uma dica nesta: > >x+y=8 >x^2+y^2 = mínimo > >Obrigado Dica: (x,y) é o ponto da reta Y=-X+8 mais próximo da origem. Outro jeito seria dar um chute (aqui é muito fácil, veja como o problema é simétrico em relação a x e y), digamos (x0,y0), e mostrar que (x0+épsilon)²+(y0-épsilon)² > x0²+y0². David _________________________________________________________________ MSN Photos é a maneira mais fácil e prática de editar e compartilhar sua fotos: http://photos.msn.com.br ========================================================================= Instruções para entrar na lista, sair da lista e usar a lista em http://www.mat.puc-rio.br/~nicolau/olimp/obm-l.html O administrador desta lista é ========================================================================= From owner-obm-l@sucuri.mat.puc-rio.br Thu Sep 26 19:42:02 2002 Return-Path: Received: (from majordom@localhost) by sucuri.mat.puc-rio.br (8.9.3/8.9.3) id TAA01599 for obm-l-MTTP; Thu, 26 Sep 2002 19:42:01 -0300 Received: from hotmail.com (f142.sea2.hotmail.com [207.68.165.142]) by sucuri.mat.puc-rio.br (8.9.3/8.9.3) with ESMTP id TAA01585 for ; Thu, 26 Sep 2002 19:41:55 -0300 Received: from mail pickup service by hotmail.com with Microsoft SMTPSVC; Thu, 26 Sep 2002 15:43:51 -0700 Received: from 200.220.36.213 by sea2fd.sea2.hotmail.msn.com with HTTP; Thu, 26 Sep 2002 22:43:50 GMT X-Originating-IP: [200.220.36.213] From: "leonardo mattos" To: obm-l@mat.puc-rio.br Subject: Re: [obm-l] Dica Date: Thu, 26 Sep 2002 22:43:50 +0000 Mime-Version: 1.0 Content-Type: text/plain; charset=iso-8859-1; format=flowed Message-ID: X-OriginalArrivalTime: 26 Sep 2002 22:43:51.0106 (UTC) FILETIME=[2F3D5A20:01C265AE] Sender: owner-obm-l@sucuri.mat.puc-rio.br Precedence: bulk Reply-To: obm-l@mat.puc-rio.br Jogue a equaçao x+y=8 no plano cartesiano.Verá que esse reta corta o eixo x em x=8 e o eixo y em y=8.Trace agora uma circunferencia centrada na origem que tangencie a reta, e entao determine o raio da mesma que será o minimo. Um abraço,Leonardo >From: "e isso mesmo" >Reply-To: obm-l@mat.puc-rio.br >To: >Subject: [obm-l] Dica >Date: Thu, 26 Sep 2002 18:12:49 -0300 > > >Me dêem uma dica nesta: > >x+y=8 >x^2+y^2 = mínimo > >ObrigadoAproveite melhor a Web. Faça o download GRÁTIS do MSN Explorer : >http://explorer.msn.com.br/intl.asp#po _________________________________________________________________ MSN Photos é a maneira mais fácil e prática de editar e compartilhar sua fotos: http://photos.msn.com.br ========================================================================= Instruções para entrar na lista, sair da lista e usar a lista em http://www.mat.puc-rio.br/~nicolau/olimp/obm-l.html O administrador desta lista é ========================================================================= From owner-obm-l@sucuri.mat.puc-rio.br Thu Sep 26 19:50:22 2002 Return-Path: Received: (from majordom@localhost) by sucuri.mat.puc-rio.br (8.9.3/8.9.3) id TAA01971 for obm-l-MTTP; Thu, 26 Sep 2002 19:50:16 -0300 Received: from sidney1.bol.com.br (sidney1.bol.com.br [200.221.24.206]) by sucuri.mat.puc-rio.br (8.9.3/8.9.3) with ESMTP id TAA01967 for ; Thu, 26 Sep 2002 19:50:13 -0300 Received: from bol.com.br (200.221.24.138) by sidney1.bol.com.br (5.1.071) id 3D8D166900116BFC for obm-l@mat.puc-rio.br; Thu, 26 Sep 2002 19:51:27 -0300 Date: Thu, 26 Sep 2002 19:49:26 -0300 Message-Id: Subject: Re:[obm-l] Dica MIME-Version: 1.0 Content-Type: multipart/mixed; boundary="_=__=_XaM3_Boundary.1033080566.2A.703633.42.31706.52.42.101010.1148639079" From: "diegoalonsoteixeira" To: obm-l@mat.puc-rio.br X-XaM3-API-Version: 2.4.3.4.4 X-SenderIP: 200.206.236.239 Sender: owner-obm-l@sucuri.mat.puc-rio.br Precedence: bulk Reply-To: obm-l@mat.puc-rio.br --_=__=_XaM3_Boundary.1033080566.2A.703633.42.31706.52.42.101010.1148639079 Content-Type: text/plain;charset="iso-8859-1" Content-Transfer-Encoding: quoted-printable x+y=3D8 y=3D8-x x^2+y^2=3Dx^2+(8-x)^2=3Dx^2+64-16x+x^2=3Dmin=3D2x^2-16x+64=3Dy voce quer o menor y =C9 uma par=E1bola, logo x do vertice =3D -b/a=3D-(-16)/2=3D8 assim 2*8^2-16*8+64=3D128-128+64=3D64 min=3D64 __________________________________________________________________________ Encontre sempre uma linha desocupada com o Discador BOL! http://www.bol.com.br/discador Ainda n=E3o tem AcessoBOL? Assine j=E1! http://www.bol.com.br/acessobol --_=__=_XaM3_Boundary.1033080566.2A.703633.42.31706.52.42.101010.1148639079 Content-Type: text/plain; name="00000HUH" Content-Transfer-Encoding: base64 DQpNZSBk6mVtIHVtYSBkaWNhICBuZXN0YToNCg0KeCt5PTgNCnheMit5XjIgPSBt7W5pbW8N Cg0KT2JyaWdhZG9BcHJvdmVpdGUgbWVsaG9yIGEgV2ViLiBGYedhIG8gZG93bmxvYWQgR1LB VElTIGRvIE1TTiBFeHBsb3JlciA6IGh0dHA6Ly9leHBsb3Jlci5tc24uY29tLmJyL2ludGwu YXNwI3BvDQo= --_=__=_XaM3_Boundary.1033080566.2A.703633.42.31706.52.42.101010.1148639079-- ========================================================================= Instruções para entrar na lista, sair da lista e usar a lista em http://www.mat.puc-rio.br/~nicolau/olimp/obm-l.html O administrador desta lista é ========================================================================= From owner-obm-l@sucuri.mat.puc-rio.br Thu Sep 26 20:42:25 2002 Return-Path: Received: (from majordom@localhost) by sucuri.mat.puc-rio.br (8.9.3/8.9.3) id UAA03890 for obm-l-MTTP; Thu, 26 Sep 2002 20:41:43 -0300 Received: from smtp.ieg.com.br (sharon.protocoloweb.com.br [200.226.139.12]) by sucuri.mat.puc-rio.br (8.9.3/8.9.3) with ESMTP id UAA03886 for ; Thu, 26 Sep 2002 20:41:40 -0300 Received: from localhost (200-158-118-125.dsl.telesp.net.br [200.158.118.125]) by smtp.ieg.com.br (IeG relay/8.9.3) with SMTP id g8QNiDRY006869 for ; Thu, 26 Sep 2002 20:44:13 -0300 (BRT) From: Tonik To: obm-l@mat.puc-rio.br Date: Thu, 26 Sep 2002 20:42:49 -0300 X-Priority: 3 (Normal) Organization: Tonik In-Reply-To: Message-Id: Subject: Re:[obm-l] Dica MIME-Version: 1.0 Content-Type: text/plain; charset="windows-1252" X-Mailer: Opera 6.04 build 1135 Sender: owner-obm-l@sucuri.mat.puc-rio.br Precedence: bulk Reply-To: obm-l@mat.puc-rio.br 26/09/02 19:49:26, "diegoalonsoteixeira" wrote: >x+y=8 y=8-x >x^2+y^2=x^2+(8-x)^2=x^2+64-16x+x^2=min=2x^2-16x+64=y > >voce quer o menor y > >É uma parábola, logo x do vertice = -b/a=-(-16)/2=8 >assim 2*8^2-16*8+64=128-128+64=64 >min=64 a abscissa do vertice é -b/2a, q dá 4, resultando em min=32 ========================================================================= Instruções para entrar na lista, sair da lista e usar a lista em http://www.mat.puc-rio.br/~nicolau/olimp/obm-l.html O administrador desta lista é ========================================================================= From owner-obm-l@sucuri.mat.puc-rio.br Thu Sep 26 20:44:37 2002 Return-Path: Received: (from majordom@localhost) by sucuri.mat.puc-rio.br (8.9.3/8.9.3) id UAA03963 for obm-l-MTTP; Thu, 26 Sep 2002 20:44:35 -0300 Received: from toole.uol.com.br (toole.uol.com.br [200.221.4.26]) by sucuri.mat.puc-rio.br (8.9.3/8.9.3) with ESMTP id UAA03959 for ; Thu, 26 Sep 2002 20:44:33 -0300 Received: from u2z7z2 ([200.158.144.224]) by toole.uol.com.br (8.9.1/8.9.1) with ESMTP id UAA18136 for ; Thu, 26 Sep 2002 20:40:51 -0300 (BRT) Message-ID: <005f01c265b7$2098caa0$2101a8c0@u2z7z2> From: "Wagner" To: Subject: [obm-l] Corpos tetradimensionais Date: Thu, 26 Sep 2002 20:47:50 -0300 Organization: Wagner MIME-Version: 1.0 Content-Type: multipart/alternative; boundary="----=_NextPart_000_005C_01C2659D.FAB482C0" X-Priority: 3 X-MSMail-Priority: Normal X-Mailer: Microsoft Outlook Express 5.50.4133.2400 X-MimeOLE: Produced By Microsoft MimeOLE V5.50.4133.2400 Sender: owner-obm-l@sucuri.mat.puc-rio.br Precedence: bulk Reply-To: obm-l@mat.puc-rio.br This is a multi-part message in MIME format. ------=_NextPart_000_005C_01C2659D.FAB482C0 Content-Type: text/plain; charset="iso-8859-1" Content-Transfer-Encoding: quoted-printable Oi para todos! Queria saber se um corpo tridimensional pode ser definido em um plano de = 4 dimens=F5es como sendo a intersec=E7=E3o de 2 corpos = tetradimensionais. Ex: O sistema : x^2+y^2+z^2+w^2=3D16 ( I ) e x.y.z-w = =3D3 Andr=E9 T. ------=_NextPart_000_005C_01C2659D.FAB482C0 Content-Type: text/html; charset="iso-8859-1" Content-Transfer-Encoding: quoted-printable
Oi para todos!
 
Queria saber se um corpo tridimensional = pode ser=20 definido em um plano de 4 dimens=F5es como sendo a intersec=E7=E3o de 2 = corpos=20 tetradimensionais. Ex: O sistema : x^2+y^2+z^2+w^2=3D16 ( I ) e x.y.z-w=20 =3D3
 
Andr=E9 T.
------=_NextPart_000_005C_01C2659D.FAB482C0-- ========================================================================= Instruções para entrar na lista, sair da lista e usar a lista em http://www.mat.puc-rio.br/~nicolau/olimp/obm-l.html O administrador desta lista é ========================================================================= From owner-obm-l@sucuri.mat.puc-rio.br Thu Sep 26 20:58:55 2002 Return-Path: Received: (from majordom@localhost) by sucuri.mat.puc-rio.br (8.9.3/8.9.3) id UAA04434 for obm-l-MTTP; Thu, 26 Sep 2002 20:58:46 -0300 Received: from videira.terra.com.br (videira.terra.com.br [200.176.3.5]) by sucuri.mat.puc-rio.br (8.9.3/8.9.3) with ESMTP id UAA04430 for ; Thu, 26 Sep 2002 20:58:43 -0300 Received: from smtp4-poa.terra.com.br (smtp4-poa.terra.com.br [200.176.3.35]) by videira.terra.com.br (Postfix) with ESMTP id 6BDC2E1216 for ; Thu, 26 Sep 2002 21:00:40 -0300 (EST) Received: from stabel (200-180-163-007-paemt7002.dsl.telebrasilia.net.br [200.180.163.7]) (authenticated user dudasta) by smtp4-poa.terra.com.br (Postfix) with ESMTP id E2EE9AC597 for ; Thu, 26 Sep 2002 21:00:39 -0300 (EST) Message-ID: <006901c265b8$eb14ec40$0301a8c0@stabel> From: "Eduardo Casagrande Stabel" To: References: <20020926204051.739243EBB0@zeus.opendf.com.br> Subject: Re: [obm-l] Alguns pontos interessantes pouco mencionados Date: Thu, 26 Sep 2002 21:00:39 -0300 MIME-Version: 1.0 Content-Type: text/plain; charset="iso-8859-1" Content-Transfer-Encoding: 8bit X-Priority: 3 X-MSMail-Priority: Normal X-Mailer: Microsoft Outlook Express 6.00.2600.0000 X-MimeOLE: Produced By Microsoft MimeOLE V6.00.2600.0000 Sender: owner-obm-l@sucuri.mat.puc-rio.br Precedence: bulk Reply-To: obm-l@mat.puc-rio.br From: "498 - Artur Costa Steiner" > >Olá para todos, > >Sou engenheiro, formei-me no início dos anos 70. Acho curioso que, na >cadeira de Cálculo ministrada durante meu curso de Engenharia, não >foram sequer mencionados alguns teoremas e conceitos dos quais, mais >tarde, vim a tomar conhecimento simplesmente por gostar de matemática . >Não que eles sejam de vital importância para um engenheiro, mas eu >gostaria de ouvir opiniões, inclusive de professores universitários, >sobre os seguintes pontos que, creio, ainda hoje não são citados em >cursos de engenharia: Olá! Eu sou aluno, fiz o curso de Cálculo semestre retrasado, e vou relatar minha experiência. >1) Definição precisa de continuidade, frisando-se, inclusive, o >conceito de continuidade uniforme- este último parece-me um conceito >importante e interessante. No meu curso, foi informado a seção do livro em que a definição formal era estudada, e mais nada se disse a respeito. >2) O teorema (creio que é conhecido por Teorema de Darbaux) o qual >afirma que derivadas de uma função real, de variável real, sempre >apresentam a chamada Propriedade do Valor Intermediário. Parece-me que >este interessante resultado, de fácil demonstração, não é muito >conhecido. É Darboux, eu acho. No curso, este teorema não foi mencionado. >3)Um outro teorema, de simples demonstração (que me parece não ser >também muito conhecido), o qual diz que derivadas em R jamais >apresentam descontinuidaes do tipo "salto". Este também não. >4) Apresentação mais precisa do conceito de diferenciabilidade de >funções de R^n em R. Também não. >Quando fiz Engenharia, o meu curso de Cálculo Integral foi muito mais >um curso de como determinar primitivas. Os aspectos conceituais da >integral não foram frisados, jamais se falou, por exemplo, em Soma de >Riemann. s. Creio que alguém com agilidade algébrica pode fazer isso >muito bem sem ter a menor idéia do que seja, de fato, o processo de >integração. Espero que hoje não seja mais assim. > >Artur Aqui, pelo menos, se falou e bastante sobre as Somas de Riemman. Claro que nada formal, e sem demonstrar os resultados básicos. Mas o conceito de partição, de somas parciais foram bem esclarecidos. Infelizmente, pouquíssima coisa mudou. Praticamente 4/5 da prova exigia exclusivemente habilidade algébrica, bastava saber: subst. trigonométricas, algumas primitivas e derivadas elementares, alguns limites fundamentais, e alguns teoremas de teste de convergência - dos quais quase nenhum foi demonstrado. O 1/5 que resta era pra plotar gráficos, o que eu não considero puramente braçal, apesar de eles haverem reduzido a 5 ou 6 etapas fixas, imexíveis... Felizmente, no curso de matemática, existe a cadeira de Análise que COMPENSA a de Cálculo. Perde-se tempo redobrado, pois reaprende-se tudo que foi visto só que formalmente. Tudo que foi visto, e mais alguns teoremas, como os que você mencionou, que são importantes. Já no curso de Engenharia, nada se vê no curso e nada se vê depois dele no sentido de formalizar o assunto. Eu considero uma pena, um fato lastimável, visto que se perde um semestre aprendendo a fazer o que o Maple faz 1.000.000 de vezes melhor que qualquer aluno - não que não seja importante as contas, mas SÓ elas é demais. Esse relato é sobre o curso da UFRGS, onde estudo. Um grande abraço a todos! Eduardo. Porto Alegre, RS. ========================================================================= Instruções para entrar na lista, sair da lista e usar a lista em http://www.mat.puc-rio.br/~nicolau/olimp/obm-l.html O administrador desta lista é ========================================================================= From owner-obm-l@sucuri.mat.puc-rio.br Thu Sep 26 21:19:00 2002 Return-Path: Received: (from majordom@localhost) by sucuri.mat.puc-rio.br (8.9.3/8.9.3) id VAA05464 for obm-l-MTTP; Thu, 26 Sep 2002 21:18:40 -0300 Received: from data2.poli.usp.br (data2.poli.usp.br [143.107.106.108]) by sucuri.mat.puc-rio.br (8.9.3/8.9.3) with ESMTP id VAA05459 for ; Thu, 26 Sep 2002 21:18:38 -0300 Received: from WEB02.poli.usp.br ([143.107.106.102]) by data2.poli.usp.br with Microsoft SMTPSVC(5.0.2195.4453); Thu, 26 Sep 2002 21:20:31 -0300 Received: from apl03.poli.usp.br ([143.107.106.15]) by WEB02.poli.usp.br with Microsoft SMTPSVC(5.0.2195.5329); Thu, 26 Sep 2002 21:20:10 -0300 X-MimeOLE: Produced By Microsoft Exchange V6.0.5762.3 content-class: urn:content-classes:message MIME-Version: 1.0 Content-Type: multipart/mixed; boundary="----_=_NextPart_001_01C265BB.A3A754D2" Subject: RE: [obm-l] Dica Date: Thu, 26 Sep 2002 21:20:09 -0300 Message-ID: <2B184DFE97456744924ACF58987D941D01935307@apl03.poli.usp.br> X-MS-Has-Attach: X-MS-TNEF-Correlator: <2B184DFE97456744924ACF58987D941D01935307@apl03.poli.usp.br> Thread-Topic: [obm-l] Dica Thread-Index: AcJls7Avrv1Y4WPmTcSsJOWtoG9E3AABwqDT From: "Edilon Ribeiro da Silva" To: X-OriginalArrivalTime: 27 Sep 2002 00:20:10.0374 (UTC) FILETIME=[A3F3AA60:01C265BB] Sender: owner-obm-l@sucuri.mat.puc-rio.br Precedence: bulk Reply-To: obm-l@mat.puc-rio.br This is a multi-part message in MIME format. ------_=_NextPart_001_01C265BB.A3A754D2 Content-Type: text/plain; charset="iso-8859-1" Content-Transfer-Encoding: quoted-printable Sendo x+y =3D 8, ent=E3o y =3D x-8. Chamando a =3D x^2 + y^2, temos: =20 a =3D x^2 + (8-x)^2 a =3D 2x^2 - 16x + 64 ( i ) =20 Derivando 'a' em rela=E7=E3o a 'x' tem-se: =20 (da/dx) =3D 4x - 16 =20 Igualando (da/dx) a zero [isto nos d=E1 um ponto de m=EDnimo, pois a = segunda derivada de 'a' com rela=E7=E3o a 'x' =E9 maior que zero], = temos: =20 4x - 16 =3D 0 , implica x =3D 4. =20 Substituindo em ( i ) temos: =20 a(min) =3D 2*4^2 - 16*4 - 64 =3D 32 =20 ------------------------------------------ Obs: O x do v=E9rtice de uma par=E1bola =E9 -b/(2a) =3D -(-16)/(2*2) = =3D 4. =20 =20 -----Original Message-----=20 From: diegoalonsoteixeira [mailto:diegoalonsoteixeira@bol.com.br]=20 Sent: Thu 9/26/2002 7:49 PM=20 To: obm-l@mat.puc-rio.br=20 Cc:=20 Subject: Re:[obm-l] Dica =09 =09 x+y=3D8 y=3D8-x x^2+y^2=3Dx^2+(8-x)^2=3Dx^2+64-16x+x^2=3Dmin=3D2x^2-16x+64=3Dy =09 voce quer o menor y =09 =C9 uma par=E1bola, logo x do vertice =3D -b/a=3D-(-16)/2=3D8 assim 2*8^2-16*8+64=3D128-128+64=3D64 min=3D64 =09 =09 = _________________________________________________________________________= _ Encontre sempre uma linha desocupada com o Discador BOL! http://www.bol.com.br/discador Ainda n=E3o tem AcessoBOL? Assine j=E1! http://www.bol.com.br/acessobol =09 ------_=_NextPart_001_01C265BB.A3A754D2 Content-Type: application/ms-tnef; name="winmail.dat" Content-Transfer-Encoding: base64 eJ8+IgsAAQaQCAAEAAAAAAABAAEAAQeQBgAIAAAA5AQAAAAAAADoAAEIgAcAGAAAAElQTS5NaWNy b3NvZnQgTWFpbC5Ob3RlADEIAQ2ABAACAAAAAgACAAEEgAEAEQAAAFJFOiBbb2JtLWxdIERpY2EA EQUBBYADAA4AAADSBwkAGgAVABQACQAEADIBASCAAwAOAAAA0gcJABoAFQAUAAkABAAyAQEJgAEA IQAAAEU5OEY3RDNEQUFBQTM4NDNCMTgzMjlEODA4OERDNDQzAEsHAQOQBgDoEQAANgAAAAMANgAA AAAAQAA5ANJUp6O7ZcIBHgA9AAEAAAAFAAAAUkU6IAAAAAACAUcAAQAAAC0AAABjPUJSO2E9IDtw PUVQVVNQO2w9QVBMMDMtMDIwOTI3MDAyMDA5Wi05MjIwMgAAAAAeAEkAAQAAABAAAABSZTpbb2Jt LWxdIERpY2EAQABOAAAf2vauZcIBHgBaAAEAAAAUAAAAZGllZ29hbG9uc290ZWl4ZWlyYQACAVsA AQAAAFAAAAAAAAAAgSsfpL6jEBmdbgDdAQ9UAgAAAABkaWVnb2Fsb25zb3RlaXhlaXJhAFNNVFAA ZGllZ29hbG9uc290ZWl4ZWlyYUBib2wuY29tLmJyAAIBXAABAAAAJAAAAFNNVFA6RElFR09BTE9O U09URUlYRUlSQUBCT0wuQ09NLkJSAB4AXQABAAAAFAAAAGRpZWdvYWxvbnNvdGVpeGVpcmEAAgFe AAEAAABQAAAAAAAAAIErH6S+oxAZnW4A3QEPVAIAAAAAZGllZ29hbG9uc290ZWl4ZWlyYQBTTVRQ AGRpZWdvYWxvbnNvdGVpeGVpcmFAYm9sLmNvbS5icgACAV8AAQAAACQAAABTTVRQOkRJRUdPQUxP TlNPVEVJWEVJUkFAQk9MLkNPTS5CUgAeAGYAAQAAAAUAAABTTVRQAAAAAB4AZwABAAAAHwAAAGRp ZWdvYWxvbnNvdGVpeGVpcmFAYm9sLmNvbS5icgAAHgBoAAEAAAAFAAAAU01UUAAAAAAeAGkAAQAA AB8AAABkaWVnb2Fsb25zb3RlaXhlaXJhQGJvbC5jb20uYnIAAB4AcAABAAAADQAAAFtvYm0tbF0g RGljYQAAAAACAXEAAQAAABsAAAABwmWzsC+u/VjhY+ZNxKwk5a2gb0TcAAHCoNMAHgB0AAEAAAAV AAAAb2JtLWxAbWF0LnB1Yy1yaW8uYnIAAAAAHgAaDAEAAAAYAAAARWRpbG9uIFJpYmVpcm8gZGEg U2lsdmEAHgAdDgEAAAANAAAAW29ibS1sXSBEaWNhAAAAAAIBCRABAAAASwsAAEcLAAARMgAATFpG dZRP4/QDAAoAcmNwZzEyNYIyA0NodG1sMQMwPwEDAfcKgAKkA+MCAGNowQrAc2V0MCAHEwKA/xAD AFAEVghVB7IR1Q5RAwHdENcyBgAGwxHVMwRGENlvEusR4wjvCfc7GM8OMDU7EdIMYGMAUAsJAWQz NpMRYAulNCAQAipcDrK9AZBnFPAKoxHjHeg0FPAAPCFET0NUWVAARSBIVE1MIFAAVUJMSUMgIi0g Ly9XM0MhgERUIkQglDMuMiGARU6cIj4e7R6PI8ExOB/wbyCiIw8kHyaQMx2AJXBFfEFEJc0O8Sbv KW8k9DZBDvA8TUVUQQewQTEsYD0iRwnwBJBhdEUFsCIS0E9OVCLQVBMs8AXhRXgQ8W5nZT0GUnYT MS9BAJACICA2QC4wLjU3NzAgOT4xIv4qzyUDMFAf8FRJKFRMRSXONA7wUmUAOltvYm0tbF1UIEQN 4GEkbjUf8C//M18xjyZFNKE24ChPJp86tAI1EWA8Qk9EWSDQZGlyPTrQcjogOpPnACEDMD0xZG8A 4D0xCrH8XHEYsD0xEPADMD2VEWBHOksc8TtPZzk2H/BEPElWPWkAAD+nOmk2NBdC3z/yBmBuPlAg eCuAeSA9IDgsIAnwYHRcJ2UzRoBGwnh4LTguEtARAAOBRnFhdUfSXhTwK0ewSTBHEHTyZQRgczo6 aQHAPXcKovc9dwpxJHwwKBEh4EKrS9h/QC9BP0JPQ19Eb1J7Oks4wR2AJm5ic3ACgD2I/CdhAUBS v0q/S89M303v/1UPUA9RH1RPUz9gP1VfVm//V3tjf2SPZZ9mr2e/aM9p3/9q72v/bQ9uH28vcD9x T3JfH3NvdH91j1dOSNkoOC38eClJMFhPWV9ab1t/XI//XZ9er1+/YM9h34AveB95L/+Gj4efiK+J v4rPi9+M743//48PkB+RL5I/k0+UX5Vvln8vl4+Yn5mvekYySSItIP0sEHhJUTjMgZWFUJrvm/// mi+gr6G/os+j36Tvpf+nD/+oH6kvqj+rT6xfrW+uf6+P/7Cfsa+yv7PPtN+177b/uA//uR+6L7s/ vE+9X75vv3/Aj4/Bn3pCnu+BwiggacLf+8PvV4ope598r32/fs9/3/+A74H/gw+EH4Uvzs/Hz8jf /1gPyw/MH80vzj/PT9Bf0W/X0n/Tj+CLRAZxdkiT13ByJ0cgbSAY0AtgR2E3s0dkSOAneOQwSdEt ETD/Si/ZT9pf22/cf92P3p/fr//gv+HP6r/V39bv1//m/+gP/+kf6i/rP+xP7V/ub+9/8I//8Z/y r/7v//8BDwIfAy8EP/8FTwZfB28IfwmPCp8Lrwy//w3PDt8P7xD/Eg8THxQvFT/B+TEoZGEvZHtQ RtH+NJ6gnmDFz/pyLBD0T/Vf//Zv93/4j/mf+q/7v/zP/d//IE8W/xgP9A8cjx2fHq8fvz8gzyHf Iu8j/yUPMgtJZ/x1YeSgRmIZVSbvJ/8pCg1I4HrjgOPwW2lzdB3j8G5KADzgR2ExIHUZ5GBwb0dA 4/BkZSDibUdhZG5pSfBHEDqA/zlg5VEaTy/y5gA1AEZgSODPOuDjkj3T5ANjb+Rv5XNFR2E5OwBh aW8wAHH2dTrwOQJdSb8qXytvLH//LY8uny+vML8z7zLfSk82D/83HykPQp9Dr0S/Rc9M70fv/0j/ TC9LH1gfVT9OTxV/W5//XK9dv17PX99g72H/Yw9kH/9lL2Y/Z09oX2lvan9rj2yf/22vbr9vz3Df ce9y/3QPdR//di93P3hPeV96b3t/fI99n/9+r3+/gM+B38TfPF9WoBnzH56AneFVUDugO3BwbGnu Yz5wnqAZ0S5QX1FvUn//U49Un1WvVr9Xz1jfWe+OX/+ET4VfUB+Kn4uvjL+Nz47fn4/vkP+SD5Mf lCtTdZYQ4HRpdHVp49LkUccR/8oQQd+Yf5mPmp+br5y/nc//nt+f76D/k/+VD5Yfly+lL/+mP6dP qF+vf6p/q4+uv62v/7qvr8+w34Lfvi+/P8BPwV//wm/Df8SPxZ/Gr8e/yM/J3//K78v/zQ/OH88v 0D/RT9Jff9Nv1H/Vj9af16/Yv4WvYTwobaOAGbKGv7kCMir4NF4yiCPfoBoRvJCJcfwzMrLfs++0 /7YPtx+4L/+5P7pPu1+8b+WP20/cX7Kf/+HP4t/j7+T/5g/nH+gv6T/76k/rWy36P/tP+8XvD/Af //Ev8j/zT/Rf9W/2f/eP+J91AWdP7UA67C/tP4XrT/uJUTVhdkCC/gCJIDrwOuHPOlA8EP5BOgJi bz9gQHRALWIvKDJhGbItlCgtG5ApDfEqMt4P/4eHib/93/7v//8BDwIfAy//BD8FTwZfFV8IvwnP 7r8Rn/8SrxO/FM8V3xbvF/8ZDxof/xsvHD8dTymPKp8rryy/Lc//Lt8v7zD/Mg8zHzQvhk82P/83 Tx7PH98g7yH/Iw8kHyUvASYxQkxPQ0tRVYhPVEUMQGlyPd7QA0GgozB5bGU9Ik0AQVJHSU4tUkkQ R0hUOoiQcHgiPyaZQXJAgkPRDMJD0WNoeFxxbEFgP/9BDw+nMv40hoFBvyZPOs8ob0fYRsCIaTM2 SF0xNDg+wJBGT05URABpekRQYjJMS2ZzMklB+axPKHJpZ6OAYUewTWX0c3NJIGX7w1AtiFFC0OZS RaqJEG5lSHBH3zj3C0LQTEtiR8lGcm9t/Qg8Od/QPtBZij/ZD19Jf0c5QkrRQ5BlZ29UcG9IbnNv pGBpeGFQcpUNYFsMkGne0G86YJ8lYaBADTEuY1tALmL8cl1Vb1Z/V49Yn1mvopb4ZW50W29cf12P Xp9frQBUaHUgOS8yNolvwDAw39A3OjQNsPxQTWRPZV9mb2d/aI9zZR5UYjBqX2tvbHZvYm0ILWxA DJB0LnB19GMtVCBvZAFwz3Hfcu/nc/91D301Q2N2r3e/bG//bX9frXqve798z33ffu+ipzhqZWNq P4F/gohSZaw6W3lTZDBEDBBhha+/hr+Hz4+fkK+Rvz4mNUvh/i9RUo0POx88Lz0/Pk+XD99IX4Qf Sn9OBZuQUEWvTykfTp9Pr1C/Uc9S3Xgrefw9ODiPOZ83v4NvhH9K4LmoES14km+Tf5SMeN/Aq6gA 38A9sQIordEpsVXvTlAOkafwsQE93dGmELEB87KjTlA9ea4Prx+Uj7R/E7WPtpl2bwwhcXVlr3qN rA9f+AuAbWoQb0GgP7Q/uE+2X74fvy+h+ydj8w2wDI0sIGDwYMALRbqwnwwDD0+732AWDdFhPQ51 P7FgqDzBz8Lf3aBUwGltv6hPqV+qb8bfx+/fcjizk/PS0LPyMTKt0NOhs/JOUP/KD8sfwByzMtRf 1W+//9fv39j/2g/bH9wv3Thf4N/h7z/i/+QP5GPd397v3PxFbntj0Gogcgww0E/RX2A0c7hlbXDo 0QyC3NFoDWDTDFBhIGN1maBkDWBj0Tt5QI8Rc49AC3BBoEJPvEwh5X/mj9z/iNxB6eQ10CBo6NBm RGDO0HRwcDovL3fz4GQAY6cvn0OQ7cRFmqbhYmBsZJ1yDmb2YLNARBB7SFlQyEVSTETASyDzb/R9 TH19puH2YHJzD/Bc4GNmMVx1R7r3//SL75wfooqcr4w6QUxA7n/vj33wnEGzQO0R6Q/qH2A0bt0N gTMLgGFA7WBBDCBUwDpv7jE/BFDNEfCQIGq9DPIh8W/yf/vP9JJhCAP/xPH1X/Zv938MHw0k+e/6 //8Qbwzn/t/+r/+/AM8B3wLv/xfvlg8XP6KnmZ6hLxwvmKT/m5Gg8ZqfjNFC9x4fIj+eHxsnjwUH NaVRjOBPRFlvGY1wMClPn1E3m5FFIE0KTExAfS4AAB4ANRABAAAAPQAAADwyQjE4NERGRTk3NDU2 NzQ0OTI0QUNGNTg5ODdEOTQxRDAxOTM1MzA3QGFwbDAzLnBvbGkudXNwLmJyPgAAAAAeAEcQAQAA AA8AAABtZXNzYWdlL3JmYzgyMgAACwDyEAEAAAAfAPMQAQAAAC4AAABSAEUAJQAzAEEAIABbAG8A YgBtAC0AbABdACAARABpAGMAYQAuAEUATQBMAAAAAAALAPYQAAAAAEAABzDil6+6umXCAUAACDBs QLOju2XCAQMA3j+vbwAAAwDxPwkEAAAeAPg/AQAAABgAAABFZGlsb24gUmliZWlybyBkYSBTaWx2 YQACAfk/AQAAAFAAAAAAAAAA3KdAyMBCEBq0uQgAKy/hggEAAAAAAAAAL089RVBVU1AvT1U9RElS RVRPUklBL0NOPVJFQ0lQSUVOVFMvQ049RURJTE9OLlNJTFZBAB4A+j8BAAAAFQAAAFN5c3RlbSBB ZG1pbmlzdHJhdG9yAAAAAAIB+z8BAAAAHgAAAAAAAADcp0DIwEIQGrS5CAArL+GCAQAAAAAAAAAu AAAAAwD9P+QEAAADABlAAAAAAAMAGkAAAAAAAwAdQAAAAAADAB5AAAAAAB4AMEABAAAADQAAAEVE SUxPTi5TSUxWQQAAAAAeADFAAQAAAA0AAABFRElMT04uU0lMVkEAAAAAHgAyQAEAAAAfAAAAZGll Z29hbG9uc290ZWl4ZWlyYUBib2wuY29tLmJyAAAeADNAAQAAAB8AAABkaWVnb2Fsb25zb3RlaXhl aXJhQGJvbC5jb20uYnIAAB4AOEABAAAADQAAAEVESUxPTi5TSUxWQQAAAAAeADlAAQAAAAIAAAAu AAAACwApAAAAAAALACMAAAAAAAMABhBFjWesAwAHEB8DAAADABAQAAAAAAMAERAAAAAAHgAIEAEA AABlAAAAU0VORE9YK1k9OCxFTlTjT1k9WC04Q0hBTUFORE9BPVgyK1kyLFRFTU9TOkE9WDIrKDgt WCkyQT0yWDItMTZYKzY0KEkpREVSSVZBTkRPQUVNUkVMQefjT0FYVEVNLVNFOihEQQAAAAACAX8A AQAAAD0AAAA8MkIxODRERkU5NzQ1Njc0NDkyNEFDRjU4OTg3RDk0MUQwMTkzNTMwN0BhcGwwMy5w b2xpLnVzcC5icj4AAAAA1po= ------_=_NextPart_001_01C265BB.A3A754D2-- ========================================================================= Instruções para entrar na lista, sair da lista e usar a lista em http://www.mat.puc-rio.br/~nicolau/olimp/obm-l.html O administrador desta lista é ========================================================================= From owner-obm-l@sucuri.mat.puc-rio.br Thu Sep 26 21:36:08 2002 Return-Path: Received: (from majordom@localhost) by sucuri.mat.puc-rio.br (8.9.3/8.9.3) id VAA06128 for obm-l-MTTP; Thu, 26 Sep 2002 21:36:03 -0300 Received: from artemis.opendf.com.br (artemis.opengate.com.br [200.181.71.15]) by sucuri.mat.puc-rio.br (8.9.3/8.9.3) with ESMTP id VAA06123 for ; Thu, 26 Sep 2002 21:35:59 -0300 Received: from localhost (localhost [127.0.0.1]) by artemis.opendf.com.br (Postfix) with ESMTP id 17ACA1C882 for ; Thu, 26 Sep 2002 21:37:38 -0300 (BRT) Received: from computer (200-181-90-212-bsace7001.dsl.telebrasilia.net.br [200.181.90.212]) by artemis.opendf.com.br (Postfix) with ESMTP id 9E43B1BDFA for ; Thu, 26 Sep 2002 21:37:33 -0300 (BRT) From: "Artur Costa Steiner" To: Subject: RE: [obm-l] Corpos tetradimensionais Date: Thu, 26 Sep 2002 21:37:53 -0300 Organization: Steiner Consultoria LTDA Message-ID: <002f01c265be$1e0f3560$9865fea9@computer> MIME-Version: 1.0 Content-Type: multipart/alternative; boundary="----=_NextPart_000_0030_01C265A4.F8C1FD60" X-Priority: 3 (Normal) X-MSMail-Priority: Normal X-Mailer: Microsoft Outlook, Build 10.0.2627 Importance: Normal X-MimeOLE: Produced By Microsoft MimeOLE V6.00.2600.0000 In-Reply-To: <005f01c265b7$2098caa0$2101a8c0@u2z7z2> X-Virus-Scanned: by AMaViS new-20020517 Sender: owner-obm-l@sucuri.mat.puc-rio.br Precedence: bulk Reply-To: obm-l@mat.puc-rio.br This is a multi-part message in MIME format. ------=_NextPart_000_0030_01C265A4.F8C1FD60 Content-Type: text/plain; charset="iso-8859-1" Content-Transfer-Encoding: quoted-printable =20 Oi para todos! =20 Queria saber se um corpo tridimensional pode ser definido em um plano de 4 dimens=F5es como sendo a intersec=E7=E3o de 2 corpos = tetradimensionais. Ex: O sistema : x^2+y^2+z^2+w^2=3D16 ( I ) e x.y.z-w =3D3 =20 Andr=E9 T. =20 Na realidade, n=E3o existem corpos de dimens=E3o superior a 2, = entendendo-se como corpo a estrutura alg=E9brica que satisfaz =E0queles conhecidos = axiomas referentes =E0 soma e =E0 multiplica=E7=E3o. No exemplo que vc deu, = temos 2 subconjuntos de um espa=E7o vetorial tetradimensional, cuja = intersec=E7=E3o pode ser vista como um subconjunto de um espa=E7o vetorial = tridimensional, o qual, por sua vez, =E9 um sub-espa=E7o do tetradimensional. Observe = que os dois conjuntos que vc deu n=E3o s=E3o espa=E7os vetoriais, ao menos = com base nas regras que definem os espa=E7os R^n. =20 Artur =20 ------=_NextPart_000_0030_01C265A4.F8C1FD60 Content-Type: text/html; charset="iso-8859-1" Content-Transfer-Encoding: quoted-printable

 

Oi para todos!

 

Queria saber se um corpo tridimensional pode ser definido em um = plano de 4 dimens=F5es como sendo a intersec=E7=E3o de 2 corpos tetradimensionais. Ex: O sistema : = x^2+y^2+z^2+w^2=3D16 ( I ) e x.y.z-w =3D3

 

Andr=E9 = T.

 

Na realidade, n=E3o existem corpos de dimens=E3o superior a 2, = entendendo-se como corpo a estrutura alg=E9brica que satisfaz =E0queles conhecidos axiomas = referentes =E0 soma e =E0 multiplica=E7=E3o. No exemplo que vc deu, temos 2 subconjuntos de = um espa=E7o vetorial =A0tetradimensional, = cuja intersec=E7=E3o pode ser vista como um subconjunto de um espa=E7o vetorial = tridimensional, o qual, por sua vez, =A0=E9 um = sub-espa=E7o do tetradimensional. =A0Observe que os dois conjuntos = que vc deu n=E3o s=E3o espa=E7os vetoriais, ao menos com base nas regras que = definem os espa=E7os R^n.

 

Artur

 

------=_NextPart_000_0030_01C265A4.F8C1FD60-- ========================================================================= Instruções para entrar na lista, sair da lista e usar a lista em http://www.mat.puc-rio.br/~nicolau/olimp/obm-l.html O administrador desta lista é ========================================================================= From owner-obm-l@sucuri.mat.puc-rio.br Thu Sep 26 22:45:12 2002 Return-Path: Received: (from majordom@localhost) by sucuri.mat.puc-rio.br (8.9.3/8.9.3) id WAA07252 for obm-l-MTTP; Thu, 26 Sep 2002 22:44:16 -0300 Received: from gorgo.centroin.com.br (gorgo.centroin.com.br [200.225.63.128]) by sucuri.mat.puc-rio.br (8.9.3/8.9.3) with ESMTP id WAA07248 for ; Thu, 26 Sep 2002 22:44:14 -0300 Received: from centroin.com.br (du106c.rjo.centroin.com.br [200.225.58.106]) (authenticated bits=0) by gorgo.centroin.com.br (8.12.2/8.12.1) with ESMTP id g8R1kU5k014225 for ; Thu, 26 Sep 2002 22:46:32 -0300 (BRT) Message-ID: <3D93B921.2020302@centroin.com.br> Date: Thu, 26 Sep 2002 22:49:21 -0300 From: Augusto =?ISO-8859-1?Q?C=E9sar?= Morgado User-Agent: Mozilla/5.0 (Windows; U; Win98; en-US; rv:0.9.4.1) Gecko/20020508 Netscape6/6.2.3 X-Accept-Language: en-us MIME-Version: 1.0 To: obm-l@mat.puc-rio.br Subject: Re: [obm-l] Kaplanky References: <000c01c2643e$47a152e0$0c1198c8@directnet.com.br> Content-Type: multipart/alternative; boundary="------------040500060704010708070509" Sender: owner-obm-l@sucuri.mat.puc-rio.br Precedence: bulk Reply-To: obm-l@mat.puc-rio.br --------------040500060704010708070509 Content-Type: text/plain; charset=ISO-8859-1; format=flowed Content-Transfer-Encoding: 8bit Kaplansky. Primeiro lema: O número de subconjuntos de tamanho p do conjunto {1, 2,..., n} no qual nao figuram numeros consecutivos eh C(n-p+1, p) Segundo lema: Igual ao anterior, mas considerando 1 e n como consecutivos. O numero de subconjuntos eh [n/(n-p)]*C(n-p, p). Os lemas e suas generalizaçoes estao no Analise Combinatoria e Probabilidade editado pela SBM na Coleçao do Professor de Matematica. Daniel wrote: > Olá a todos, > > > > Alguem poderia enunciar os dois teoremas de > Kaplanky da análise combinatória. > > 1) O primeiro é referente a seqüências com > sucessões > > 2) O segundo é do mesmo tipo, mas em uma > disposição sobre circunferência > > > > Ex: > > 1) Em uma urna existem n bolas numeradas de 1 a n. > De quantas formas podemos pegar k bolas, de modo que não haja bolas > sucessivas? > > > > 2) Doze cavaleiros estão dispostos em uma távola > redonda, de sorte que cavaleiros sucessivos são inimigos. Quantas > comissões de 5 cavaleiros podemos formar, de modo que não haja > cavaleiros inimigos na comissão? > > > > Grato > > > > Daniel > --------------040500060704010708070509 Content-Type: text/html; charset=us-ascii Content-Transfer-Encoding: 7bit Kaplansky.
Primeiro lema:
O número de subconjuntos de tamanho p do conjunto {1, 2,..., n} no qual nao figuram numeros consecutivos eh C(n-p+1, p)
Segundo lema:
Igual ao anterior, mas considerando 1 e n como consecutivos. O numero de subconjuntos eh  [n/(n-p)]*C(n-p, p).

Os lemas e suas generalizaçoes estao no Analise Combinatoria e Probabilidade editado pela SBM na Coleçao do Professor de Matematica.

Daniel wrote:
            Olá a todos,
 
                    Alguem poderia enunciar os dois teoremas de Kaplanky da análise combinatória.
                   1)     O primeiro é referente a seqüências com sucessões
                   2)     O segundo é do mesmo tipo, mas em uma disposição sobre circunferência
                   
                    Ex:
                    1) Em uma urna existem n bolas numeradas de 1 a n. De quantas formas podemos pegar k bolas, de modo que não haja bolas sucessivas?
 
                    2) Doze cavaleiros estão dispostos em uma távola redonda, de sorte que cavaleiros sucessivos são inimigos. Quantas comissões de 5 cavaleiros podemos formar, de modo que não haja cavaleiros inimigos na comissão?
       
Grato
 
                Daniel

--------------040500060704010708070509-- ========================================================================= Instruções para entrar na lista, sair da lista e usar a lista em http://www.mat.puc-rio.br/~nicolau/olimp/obm-l.html O administrador desta lista é ========================================================================= From owner-obm-l@sucuri.mat.puc-rio.br Thu Sep 26 22:57:29 2002 Return-Path: Received: (from majordom@localhost) by sucuri.mat.puc-rio.br (8.9.3/8.9.3) id WAA07602 for obm-l-MTTP; Thu, 26 Sep 2002 22:57:20 -0300 Received: from artemis.opendf.com.br (artemis.opengate.com.br [200.181.71.15]) by sucuri.mat.puc-rio.br (8.9.3/8.9.3) with ESMTP id WAA07598 for ; Thu, 26 Sep 2002 22:57:17 -0300 Received: from localhost (localhost [127.0.0.1]) by artemis.opendf.com.br (Postfix) with ESMTP id C98E61C882 for ; Thu, 26 Sep 2002 22:58:57 -0300 (BRT) Received: from computer (200-181-90-212-bsace7001.dsl.telebrasilia.net.br [200.181.90.212]) by artemis.opendf.com.br (Postfix) with ESMTP id D2B721BDFA for ; Thu, 26 Sep 2002 22:58:52 -0300 (BRT) From: "Artur Costa Steiner" To: Subject: RE: [obm-l] Alguns pontos interessantes pouco mencionados Date: Thu, 26 Sep 2002 22:59:10 -0300 Organization: Steiner Consultoria LTDA Message-ID: <003901c265c9$7aba8070$9865fea9@computer> MIME-Version: 1.0 Content-Type: text/plain; charset="iso-8859-1" X-Priority: 3 (Normal) X-MSMail-Priority: Normal X-Mailer: Microsoft Outlook, Build 10.0.2627 Importance: Normal X-MimeOLE: Produced By Microsoft MimeOLE V6.00.2600.0000 In-Reply-To: <006901c265b8$eb14ec40$0301a8c0@stabel> X-Virus-Scanned: by AMaViS new-20020517 Content-Transfer-Encoding: 8bit X-MIME-Autoconverted: from quoted-printable to 8bit by sucuri.mat.puc-rio.br id WAA07599 Sender: owner-obm-l@sucuri.mat.puc-rio.br Precedence: bulk Reply-To: obm-l@mat.puc-rio.br Obrigado pelas sua contribuição, Eduardo. É, parece que, de fato, pouca coisa mudou nos últimos 30 anos....Mas não devemos ser tão críticos. Afinal de contas, nem todos vibram com matemática, nem todos gostam de epsilons e deltas - quanto mais de assuntos como espaços topológicos (aliás, acho que a maioria das pessoas, inclusive engenheiros, detesta e acha que é frescura, que não serve rigorosamente para nada). É verdade que um engenheiro não precisa mesmo ter um conhecimento rigoroso de Análise. Para a maioria das pessoas que usam matemática, acho que é mesmo suficiente dizer que uma função é contínua se podemos traçar seu gráfico sem tirar o lápis do papel. Julgo, entretanto, que é importante se ter uma noção um pouco mais precisa do que sejam integrais e derivadas. Eu trabalho com modelos ligados à otimização do sistema elétrico brasileiro (operá-lo e expandi-lo de modo a atender aos consumidores com o mínimo custo, observando-se aspectos de ordem política, empresarial e ambiental) e estas nocões tornam mais fácil compreender o problema. Abraços Artur Olá! Eu sou aluno, fiz o curso de Cálculo semestre retrasado, e vou relatar minha experiência. >1) Definição precisa de continuidade, frisando-se, inclusive, o >conceito de continuidade uniforme- este último parece-me um conceito >importante e interessante. No meu curso, foi informado a seção do livro em que a definição formal era estudada, e mais nada se disse a respeito. >2) O teorema (creio que é conhecido por Teorema de Darbaux) o qual >afirma que derivadas de uma função real, de variável real, sempre >apresentam a chamada Propriedade do Valor Intermediário. Parece-me que >este interessante resultado, de fácil demonstração, não é muito >conhecido. É Darboux, eu acho. No curso, este teorema não foi mencionado. >3)Um outro teorema, de simples demonstração (que me parece não ser >também muito conhecido), o qual diz que derivadas em R jamais >apresentam descontinuidaes do tipo "salto". Este também não. >4) Apresentação mais precisa do conceito de diferenciabilidade de >funções de R^n em R. Também não. >Quando fiz Engenharia, o meu curso de Cálculo Integral foi muito mais >um curso de como determinar primitivas. Os aspectos conceituais da >integral não foram frisados, jamais se falou, por exemplo, em Soma de >Riemann. s. Creio que alguém com agilidade algébrica pode fazer isso >muito bem sem ter a menor idéia do que seja, de fato, o processo de >integração. Espero que hoje não seja mais assim. > >Artur Aqui, pelo menos, se falou e bastante sobre as Somas de Riemman. Claro que nada formal, e sem demonstrar os resultados básicos. Mas o conceito de partição, de somas parciais foram bem esclarecidos. Infelizmente, pouquíssima coisa mudou. Praticamente 4/5 da prova exigia exclusivemente habilidade algébrica, bastava saber: subst. trigonométricas, algumas primitivas e derivadas elementares, alguns limites fundamentais, e alguns teoremas de teste de convergência - dos quais quase nenhum foi demonstrado. O 1/5 que resta era pra plotar gráficos, o que eu não considero puramente braçal, apesar de eles haverem reduzido a 5 ou 6 etapas fixas, imexíveis... Felizmente, no curso de matemática, existe a cadeira de Análise que COMPENSA a de Cálculo. Perde-se tempo redobrado, pois reaprende-se tudo que foi visto só que formalmente. Tudo que foi visto, e mais alguns teoremas, como os que você mencionou, que são importantes. Já no curso de Engenharia, nada se vê no curso e nada se vê depois dele no sentido de formalizar o assunto. Eu considero uma pena, um fato lastimável, visto que se perde um semestre aprendendo a fazer o que o Maple faz 1.000.000 de vezes melhor que qualquer aluno - não que não seja importante as contas, mas SÓ elas é demais. Esse relato é sobre o curso da UFRGS, onde estudo. Um grande abraço a todos! Eduardo. Porto Alegre, RS. ======================================================================== = Instruções para entrar na lista, sair da lista e usar a lista em http://www.mat.puc-rio.br/~nicolau/olimp/obm-l.html O administrador desta lista é ======================================================================== = ========================================================================= Instruções para entrar na lista, sair da lista e usar a lista em http://www.mat.puc-rio.br/~nicolau/olimp/obm-l.html O administrador desta lista é ========================================================================= From owner-obm-l@sucuri.mat.puc-rio.br Thu Sep 26 23:33:03 2002 Return-Path: Received: (from majordom@localhost) by sucuri.mat.puc-rio.br (8.9.3/8.9.3) id XAA08610 for obm-l-MTTP; Thu, 26 Sep 2002 23:32:53 -0300 Received: from hotmail.com (f187.law15.hotmail.com [64.4.23.187]) by sucuri.mat.puc-rio.br (8.9.3/8.9.3) with ESMTP id XAA08606 for ; Thu, 26 Sep 2002 23:32:50 -0300 Received: from mail pickup service by hotmail.com with Microsoft SMTPSVC; Thu, 26 Sep 2002 19:34:47 -0700 Received: from 200.151.93.31 by lw15fd.law15.hotmail.msn.com with HTTP; Fri, 27 Sep 2002 02:34:46 GMT X-Originating-IP: [200.151.93.31] From: "Luiz Felipe Constantino" To: obm-l@mat.puc-rio.br Subject: [obm-l] Jogos Date: Thu, 26 Sep 2002 23:34:46 -0300 Mime-Version: 1.0 Content-Type: text/plain; charset=iso-8859-1; format=flowed Message-ID: X-OriginalArrivalTime: 27 Sep 2002 02:34:47.0166 (UTC) FILETIME=[721851E0:01C265CE] Sender: owner-obm-l@sucuri.mat.puc-rio.br Precedence: bulk Reply-To: obm-l@mat.puc-rio.br outro dia, vi em algum lugar o responsável pela tabela dos jogos do Campeonato Brasileiro dizer que com os 26 times não era possível fazer com que todos os times jogassem na mesma rodada durante todo o campeonato. Ou seja, não dava para ter treze jogos em toda rodada, de modo que cada time jogue com o outro apenas uma vez. Gostaria de saber como colocar isso no papel (como provar)... L. Felipe ========================================================================= Instruções para entrar na lista, sair da lista e usar a lista em http://www.mat.puc-rio.br/~nicolau/olimp/obm-l.html O administrador desta lista é ========================================================================= From owner-obm-l@sucuri.mat.puc-rio.br Thu Sep 26 23:58:07 2002 Return-Path: Received: (from majordom@localhost) by sucuri.mat.puc-rio.br (8.9.3/8.9.3) id XAA09089 for obm-l-MTTP; Thu, 26 Sep 2002 23:57:35 -0300 Received: from artemis.opendf.com.br (artemis.opengate.com.br [200.181.71.15]) by sucuri.mat.puc-rio.br (8.9.3/8.9.3) with ESMTP id XAA09085 for ; Thu, 26 Sep 2002 23:57:32 -0300 Received: from localhost (localhost [127.0.0.1]) by artemis.opendf.com.br (Postfix) with ESMTP id 0500D1CAAF for ; Thu, 26 Sep 2002 23:59:14 -0300 (BRT) Received: from computer (200-181-90-212-bsace7001.dsl.telebrasilia.net.br [200.181.90.212]) by artemis.opendf.com.br (Postfix) with ESMTP id C30651BDFA for ; Thu, 26 Sep 2002 23:59:06 -0300 (BRT) From: "Artur Costa Steiner" To: Subject: [obm-l] =?iso-8859-1?Q?Pontos_extremos_de_fun=E7=F5es_reais?= Date: Thu, 26 Sep 2002 23:59:26 -0300 Organization: Steiner Consultoria LTDA Message-ID: <004a01c265d1$e494ea50$9865fea9@computer> MIME-Version: 1.0 Content-Type: multipart/alternative; boundary="----=_NextPart_000_004B_01C265B8.BF47B250" X-Priority: 3 (Normal) X-MSMail-Priority: Normal X-Mailer: Microsoft Outlook, Build 10.0.2627 Importance: Normal X-MimeOLE: Produced By Microsoft MimeOLE V6.00.2600.0000 In-Reply-To: <006901c265b8$eb14ec40$0301a8c0@stabel> X-Virus-Scanned: by AMaViS new-20020517 Sender: owner-obm-l@sucuri.mat.puc-rio.br Precedence: bulk Reply-To: obm-l@mat.puc-rio.br This is a multi-part message in MIME format. ------=_NextPart_000_004B_01C265B8.BF47B250 Content-Type: text/plain; charset="iso-8859-1" Content-Transfer-Encoding: quoted-printable Oi para todos, =20 Eu estou um tanto intrigado com o que consta no excelente livro sobre An=E1lise Real (em R) publicado em 2000 por Robert Bartle e Donald Sherbert (dois grandes autores), "Introduction to Real Analysis".=20 =20 No livro h=E1 uma prova daquele famoso teorema o qual diz que, se f =E9 definida em um intervalo I e a =E9 um ponto interior de I no qual as n-1 primeiras derivadas de f s=E3o nulas e a derivada de ordem n =E9 = diferente de zero, ent=E3o: =20 Se n =E9 par e a n-=E9sima derivada =E9 >0, ent=E3o f tem um m=EDnimo = relativo em a; Se n =E9 par e a n-=E9sima derivada =E9 <0, ent=E3o f tem um m=E1ximo = relativo em a; Se n =E9 =EDmpar, ent=E3o f n=E3o tem nem um m=E1ximo nem um m=EDnimo = relativos em a. =20 A prova baseia-se no Teorema de Taylor e os autores assumem =96 e =E9 = isto justamente o que me intriga =96 que a derivada de ordem n de f existe e = =E9 cont=EDnua em uma vizinhan=E7a de a. Ora, na realidade, parece-me que o teorema n=E3o requer uma hip=F3tese t=E3o forte, n=E3o =E9 necess=E1rio = assumir a continuidade de f^n em um a vizinhan=E7a de a. Nem sequer =E9 = necess=E1rio assumir que f^n exista em toda uma vizinhan=E7a de a. A simples = exist=EAncia de f^n j=E1 =E9 suficiente para o teorema dos pontos extremos ser = v=E1lido. Certo? =20 Eu realmente gostaria de saber porque autores consagrados assumem uma hip=F3tese mais forte do que a necess=E1ria. (Apesar disto, recomendo intensamente este livro, =E9 excelente) =20 Artur ------=_NextPart_000_004B_01C265B8.BF47B250 Content-Type: text/html; charset="iso-8859-1" Content-Transfer-Encoding: quoted-printable

Oi para todos,

 

Eu estou um tanto intrigado com o que consta no excelente livro sobre = An=E1lise Real (em R) publicado em 2000 por Robert Bartle e Donald Sherbert (dois = grandes autores), "Introduction to Real Analysis". =

 = ;

No livro h=E1 uma prova daquele famoso teorema =
o qual diz que, se f =E9 definida em um intervalo I e a =E9 um ponto =
interior de I no qual as n-1 primeiras derivadas de f s=E3o nulas e a =
derivada de ordem n =E9 diferente de zero, =
ent=E3o:
 
Se n =E9 =
par e a n-=E9sima derivada =E9 >0, ent=E3o f tem um m=EDnimo relativo =
em a;
Se n =E9 par e a n-=E9sima derivada =E9 <0, ent=E3o f tem um =
m=E1ximo relativo em a;
Se n =E9 =EDmpar, ent=E3o f n=E3o tem nem um m=E1ximo nem um =
m=EDnimo relativos em a.
 
A prova =
baseia-se no Teorema de Taylor e os autores assumem – e =E9 isto =
justamente o que me intriga – que a derivada de ordem n de f =
existe e =E9 cont=EDnua em uma vizinhan=E7a de a. Ora, na realidade, =
parece-me que o teorema n=E3o requer uma hip=F3tese t=E3o forte, n=E3o =
=E9 necess=E1rio assumir a continuidade de f^n em um a vizinhan=E7a de =
a. Nem sequer =E9 necess=E1rio assumir que f^n exista em toda uma =
vizinhan=E7a de a. A simples exist=EAncia de f^n j=E1 =E9 suficiente =
para o teorema dos pontos extremos ser v=E1lido. =
Certo?
 
Eu =
realmente gostaria de saber porque autores consagrados assumem uma =
hip=F3tese mais forte do que a necess=E1ria. (Apesar disto, recomendo =
intensamente este livro, =E9 =
excelente)
 
Artur
= ------=_NextPart_000_004B_01C265B8.BF47B250-- ========================================================================= Instruções para entrar na lista, sair da lista e usar a lista em http://www.mat.puc-rio.br/~nicolau/olimp/obm-l.html O administrador desta lista é ========================================================================= From owner-obm-l@sucuri.mat.puc-rio.br Fri Sep 27 00:17:16 2002 Return-Path: Received: (from majordom@localhost) by sucuri.mat.puc-rio.br (8.9.3/8.9.3) id AAA09681 for obm-l-MTTP; Fri, 27 Sep 2002 00:17:11 -0300 Received: from proxy.annex.com.br (proxy.annex.com.br [200.248.173.4]) by sucuri.mat.puc-rio.br (8.9.3/8.9.3) with ESMTP id AAA09677 for ; Fri, 27 Sep 2002 00:17:08 -0300 Received: from mario (CF049.annex.com.br [200.248.173.49]) by proxy.annex.com.br (8.11.6/linuxconf) with SMTP id g8R3bJZ18580 for ; Fri, 27 Sep 2002 00:37:19 -0300 Message-ID: <00ed01c265d4$a3161240$31adf8c8@mario> From: =?iso-8859-1?Q?M=E1rio_Pereira?= To: Subject: [obm-l] Date: Fri, 27 Sep 2002 00:19:05 -0300 MIME-Version: 1.0 Content-Type: multipart/alternative; boundary="----=_NextPart_000_00EA_01C265BB.7D089780" X-Priority: 3 X-MSMail-Priority: Normal X-Mailer: Microsoft Outlook Express 5.50.4522.1200 X-MimeOLE: Produced By Microsoft MimeOLE V5.50.4522.1200 Sender: owner-obm-l@sucuri.mat.puc-rio.br Precedence: bulk Reply-To: obm-l@mat.puc-rio.br This is a multi-part message in MIME format. ------=_NextPart_000_00EA_01C265BB.7D089780 Content-Type: text/plain; charset="iso-8859-1" Content-Transfer-Encoding: quoted-printable Preciso de ajuda: A escala termom=E9trica Celsius adota os valores de 0 a 100 para pontos = de fus=E3o do gelo e de ebuli=E7=E3o da =E1gua =E0 press=E3o normal, = respectivamente. A escala Fahrenheit adota os valores 32 e 212 para = esses mesmos pontos. Ent=E3o a partir de uma determinada temperatura o = n=BA lido na escala Fahrenheit =E9 maior que o lido na escala Celsius. = Que temperatura =E9 essa? Obrigado.=20 M=E1rio ------=_NextPart_000_00EA_01C265BB.7D089780 Content-Type: text/html; charset="iso-8859-1" Content-Transfer-Encoding: quoted-printable
Preciso de ajuda:
 
A escala termom=E9trica Celsius adota = os valores de 0=20 a 100 para pontos de fus=E3o do gelo e de ebuli=E7=E3o da =E1gua =E0 = press=E3o normal,=20 respectivamente. A escala Fahrenheit adota os valores 32 e 212 para = esses mesmos=20 pontos. Ent=E3o a partir de uma determinada temperatura o n=BA lido na = escala=20 Fahrenheit =E9 maior que o lido na escala Celsius. Que temperatura =E9=20 essa?
 
Obrigado.
 
M=E1rio
------=_NextPart_000_00EA_01C265BB.7D089780-- ========================================================================= Instruções para entrar na lista, sair da lista e usar a lista em http://www.mat.puc-rio.br/~nicolau/olimp/obm-l.html O administrador desta lista é ========================================================================= From owner-obm-l@sucuri.mat.puc-rio.br Fri Sep 27 00:27:51 2002 Return-Path: Received: (from majordom@localhost) by sucuri.mat.puc-rio.br (8.9.3/8.9.3) id AAA09993 for obm-l-MTTP; Fri, 27 Sep 2002 00:27:42 -0300 Received: from trex.centroin.com.br (trex.centroin.com.br [200.225.63.134]) by sucuri.mat.puc-rio.br (8.9.3/8.9.3) with ESMTP id AAA09985 for ; Fri, 27 Sep 2002 00:27:39 -0300 Received: from trex.centroin.com.br (localhost [127.0.0.1]) by trex.centroin.com.br (8.12.5/8.12.1) with ESMTP id g8R3Two4015712 for ; Fri, 27 Sep 2002 00:29:58 -0300 (BRT) Received: by trex.centroin.com.br (8.12.5/8.12.5/Submit) id g8R3Twdd015711; Fri, 27 Sep 2002 00:29:58 -0300 (BRT) Message-Id: <200209270329.g8R3Twdd015711@trex.centroin.com.br> Received: from 200.225.58.90 by trex.centroin.com.br (CIPWM versao 1.4C1) with HTTPS for ; Fri, 27 Sep 2002 00:29:58 -0300 (BRT) Date: Fri, 27 Sep 2002 00:29:58 -0300 (BRT) From: Augusto Cesar de Oliveira Morgado To: obm-l@mat.puc-rio.br Subject: Re: [obm-l] MIME-Version: 1.0 X-Mailer: CentroIn Internet Provider WebMail v. 1.4C1 (http://www.centroin.com.br/) Content-Type: text/plain; charset="iso-8859-1" Content-Transfer-Encoding: 8bit X-MIME-Autoconverted: from quoted-printable to 8bit by sucuri.mat.puc-rio.br id AAA09986 Sender: owner-obm-l@sucuri.mat.puc-rio.br Precedence: bulk Reply-To: obm-l@mat.puc-rio.br Em Fri, 27 Sep 2002 00:19:05 -0300, Mário_Pereira disse: > Preciso de ajuda: > > A escala termométrica Celsius adota os valores de 0 a 100 para pontos de fusão do gelo e de ebulição da água à pressão normal, respectivamente. A escala Fahrenheit adota os valores 32 e 212 para esses mesmos pontos. Então a partir de uma determinada temperatura o nº lido na escala Fahrenheit é maior que o lido na escala Celsius. Que temperatura é essa? > > Obrigado. > > Mário (F-32)/9 = C/5 Temos F>C sse 32 + 9C/5 > C 32> -4C/5 C > -40 No ponto -40 as leituras sao iguais ( -40C = -40F) A partir daih, a leitura F eh superior a leitura C ========================================================================= Instruções para entrar na lista, sair da lista e usar a lista em http://www.mat.puc-rio.br/~nicolau/olimp/obm-l.html O administrador desta lista é ========================================================================= From owner-obm-l@sucuri.mat.puc-rio.br Fri Sep 27 07:54:15 2002 Return-Path: Received: (from majordom@localhost) by sucuri.mat.puc-rio.br (8.9.3/8.9.3) id HAA13792 for obm-l-MTTP; Fri, 27 Sep 2002 07:53:53 -0300 Received: from traven9.uol.com.br (traven9.uol.com.br [200.221.4.35]) by sucuri.mat.puc-rio.br (8.9.3/8.9.3) with ESMTP id HAA13788 for ; Fri, 27 Sep 2002 07:53:51 -0300 Received: from emmanuela ([200.150.128.234]) by traven9.uol.com.br (8.9.1/8.9.1) with SMTP id HAA25189 for ; Fri, 27 Sep 2002 07:57:33 -0300 (BRT) Message-ID: <002501c26614$1042fd80$ea8096c8@emmanuela> From: "Paulo Rodrigues" To: References: Subject: Re: [obm-l] Jogos Date: Fri, 27 Sep 2002 07:53:07 -0300 MIME-Version: 1.0 Content-Type: text/plain; charset="iso-8859-1" Content-Transfer-Encoding: 8bit X-Priority: 3 X-MSMail-Priority: Normal X-Mailer: Microsoft Outlook Express 6.00.2600.0000 X-MimeOLE: Produced By Microsoft MimeOLE V6.00.2600.0000 Sender: owner-obm-l@sucuri.mat.puc-rio.br Precedence: bulk Reply-To: obm-l@mat.puc-rio.br ----- Original Message ----- From: "Luiz Felipe Constantino" To: Sent: Thursday, September 26, 2002 11:34 PM Subject: [obm-l] Jogos > outro dia, vi em algum lugar o responsável pela tabela dos jogos do > Campeonato Brasileiro dizer que com os 26 times não era possível fazer com > que todos os times jogassem na mesma rodada durante todo o campeonato. Ou > seja, não dava para ter treze jogos em toda rodada, de modo que cada time > jogue com o outro apenas uma vez. Gostaria de saber como colocar isso no > papel (como provar)... > > L. Felipe Supondo que jogam todos contra todos em turno único, a afirmação acima é falsa. Veja a EUREKA! 7 - Páginas 32, 33 - Problema 5 da OBM 99 - Nível 3. (http://www.obm.org.br) ========================================================================= Instruções para entrar na lista, sair da lista e usar a lista em http://www.mat.puc-rio.br/~nicolau/olimp/obm-l.html O administrador desta lista é ========================================================================= From owner-obm-l@sucuri.mat.puc-rio.br Fri Sep 27 10:17:35 2002 Return-Path: Received: (from majordom@localhost) by sucuri.mat.puc-rio.br (8.9.3/8.9.3) id KAA15785 for obm-l-MTTP; Fri, 27 Sep 2002 10:16:42 -0300 Received: from Euler.impa.br (euler.impa.br [147.65.1.3]) by sucuri.mat.puc-rio.br (8.9.3/8.9.3) with ESMTP id KAA15776 for ; Fri, 27 Sep 2002 10:16:38 -0300 Received: from obm-01 (obm-01.impa.br [147.65.2.170]) by Euler.impa.br (8.11.6/8.11.6) with SMTP id g8RDIaF09940 for ; Fri, 27 Sep 2002 10:18:36 -0300 (EST) Message-Id: <3.0.5.32.20020927101917.007ec8d0@pop.impa.br> X-Sender: obm@pop.impa.br X-Mailer: QUALCOMM Windows Eudora Light Version 3.0.5 (32) Date: Fri, 27 Sep 2002 10:19:17 -0300 To: obm-l@mat.puc-rio.br From: Olimpiada Brasileira de Matematica Subject: [obm-l] Treinamento no RJ. Mime-Version: 1.0 Content-Type: text/plain; charset="us-ascii" Sender: owner-obm-l@sucuri.mat.puc-rio.br Precedence: bulk Reply-To: obm-l@mat.puc-rio.br Caros(as) amigos(as) da lista: Segunda-feira 30 de setembro iniciaremos o treinamento para a Terceira Fase da OBM aqui no IMPA. Endereco: Estrada Dona Castorina, 110 Jardim Botanico. (Ponto Final Onibus 409 ou 125) Professores: Luciano Castro/Carlos G. Moreira Horario:14:00horas Interessados dirigirse a sala da OBM (107 - 1 Andar) Tel: 25295077 Abracos, Nelly. ========================================================================= Instruções para entrar na lista, sair da lista e usar a lista em http://www.mat.puc-rio.br/~nicolau/olimp/obm-l.html O administrador desta lista é ========================================================================= From owner-obm-l@sucuri.mat.puc-rio.br Fri Sep 27 11:48:55 2002 Return-Path: Received: (from majordom@localhost) by sucuri.mat.puc-rio.br (8.9.3/8.9.3) id LAA17632 for obm-l-MTTP; Fri, 27 Sep 2002 11:46:56 -0300 Received: (from nicolau@localhost) by sucuri.mat.puc-rio.br (8.9.3/8.9.3) id LAA17627 for obm-l@mat.puc-rio.br; Fri, 27 Sep 2002 11:46:55 -0300 Date: Fri, 27 Sep 2002 11:46:55 -0300 From: "Nicolau C. Saldanha" To: obm-l@mat.puc-rio.br Subject: Re: [obm-l] Corpos tetradimensionais Message-ID: <20020927114655.B17380@sucuri.mat.puc-rio.br> References: <005f01c265b7$2098caa0$2101a8c0@u2z7z2> Mime-Version: 1.0 Content-Type: text/plain; charset=iso-8859-1 Content-Disposition: inline Content-Transfer-Encoding: 8bit User-Agent: Mutt/1.2.5i In-Reply-To: <005f01c265b7$2098caa0$2101a8c0@u2z7z2>; from timpa@uol.com.br on Thu, Sep 26, 2002 at 08:47:50PM -0300 Sender: owner-obm-l@sucuri.mat.puc-rio.br Precedence: bulk Reply-To: obm-l@mat.puc-rio.br On Thu, Sep 26, 2002 at 08:47:50PM -0300, Wagner wrote: > Oi para todos! > > Queria saber se um corpo tridimensional pode ser definido em um plano de 4 > dimensões como sendo a intersecção de 2 corpos tetradimensionais. Ex: O > sistema : x^2+y^2+z^2+w^2=16 ( I ) e x.y.z-w =3 Não, em geral duas equações em R^4 definem um objeto de dimensão 2 e não 3. Note que *uma* equação em 4 variáveis já define um objeto de dimensão 3. Por outro lado, duas *inequações* em R^4 definem um objeto de dimensão 4. []s, N. ========================================================================= Instruções para entrar na lista, sair da lista e usar a lista em http://www.mat.puc-rio.br/~nicolau/olimp/obm-l.html O administrador desta lista é ========================================================================= From owner-obm-l@sucuri.mat.puc-rio.br Fri Sep 27 11:50:40 2002 Return-Path: Received: (from majordom@localhost) by sucuri.mat.puc-rio.br (8.9.3/8.9.3) id LAA17710 for obm-l-MTTP; Fri, 27 Sep 2002 11:50:36 -0300 Received: (from nicolau@localhost) by sucuri.mat.puc-rio.br (8.9.3/8.9.3) id LAA17705 for obm-l@mat.puc-rio.br; Fri, 27 Sep 2002 11:50:35 -0300 Date: Fri, 27 Sep 2002 11:50:34 -0300 From: "Nicolau C. Saldanha" To: obm-l@mat.puc-rio.br Subject: Re: [obm-l] Jogos Message-ID: <20020927115034.C17380@sucuri.mat.puc-rio.br> References: Mime-Version: 1.0 Content-Type: text/plain; charset=iso-8859-1 Content-Disposition: inline Content-Transfer-Encoding: 8bit User-Agent: Mutt/1.2.5i In-Reply-To: ; from luiz_felipe_c@hotmail.com on Thu, Sep 26, 2002 at 11:34:46PM -0300 Sender: owner-obm-l@sucuri.mat.puc-rio.br Precedence: bulk Reply-To: obm-l@mat.puc-rio.br On Thu, Sep 26, 2002 at 11:34:46PM -0300, Luiz Felipe Constantino wrote: > outro dia, vi em algum lugar o responsável pela tabela dos jogos do > Campeonato Brasileiro dizer que com os 26 times não era possível fazer com > que todos os times jogassem na mesma rodada durante todo o campeonato. Ou > seja, não dava para ter treze jogos em toda rodada, de modo que cada time > jogue com o outro apenas uma vez. Gostaria de saber como colocar isso no > papel (como provar)... > > L. Felipe Acho que este senhor usou a palavra 'impossível' no sentido não matemático, querendo dizer que não há campos de futebol de boa qualidade um número suficiente, ou que os jogadores estão ocupados treinando pela seleção, ou alguma outra consideração não matemática do gênero... []s, N. ========================================================================= Instruções para entrar na lista, sair da lista e usar a lista em http://www.mat.puc-rio.br/~nicolau/olimp/obm-l.html O administrador desta lista é ========================================================================= From owner-obm-l@sucuri.mat.puc-rio.br Fri Sep 27 11:51:54 2002 Return-Path: Received: (from majordom@localhost) by sucuri.mat.puc-rio.br (8.9.3/8.9.3) id LAA17762 for obm-l-MTTP; Fri, 27 Sep 2002 11:51:51 -0300 Received: (from nicolau@localhost) by sucuri.mat.puc-rio.br (8.9.3/8.9.3) id LAA17757 for obm-l@mat.puc-rio.br; Fri, 27 Sep 2002 11:51:51 -0300 Date: Fri, 27 Sep 2002 11:51:50 -0300 From: "Nicolau C. Saldanha" To: obm-l@mat.puc-rio.br Subject: Re: [obm-l] Message-ID: <20020927115150.D17380@sucuri.mat.puc-rio.br> References: <00ed01c265d4$a3161240$31adf8c8@mario> Mime-Version: 1.0 Content-Type: text/plain; charset=iso-8859-1 Content-Disposition: inline Content-Transfer-Encoding: 8bit User-Agent: Mutt/1.2.5i In-Reply-To: <00ed01c265d4$a3161240$31adf8c8@mario>; from mpereira@annex.com.br on Fri, Sep 27, 2002 at 12:19:05AM -0300 Sender: owner-obm-l@sucuri.mat.puc-rio.br Precedence: bulk Reply-To: obm-l@mat.puc-rio.br On Fri, Sep 27, 2002 at 12:19:05AM -0300, Mário Pereira wrote: > Preciso de ajuda: > > A escala termométrica Celsius adota os valores de 0 a 100 para pontos de > fusão do gelo e de ebulição da água à pressão normal, respectivamente. A > escala Fahrenheit adota os valores 32 e 212 para esses mesmos pontos. Então a > partir de uma determinada temperatura o nº lido na escala Fahrenheit é maior > que o lido na escala Celsius. Que temperatura é essa? -40 Basta interpolar por funções da forma ax+b. []s, N. ========================================================================= Instruções para entrar na lista, sair da lista e usar a lista em http://www.mat.puc-rio.br/~nicolau/olimp/obm-l.html O administrador desta lista é ========================================================================= From owner-obm-l@sucuri.mat.puc-rio.br Fri Sep 27 11:53:20 2002 Return-Path: Received: (from majordom@localhost) by sucuri.mat.puc-rio.br (8.9.3/8.9.3) id LAA17831 for obm-l-MTTP; Fri, 27 Sep 2002 11:53:16 -0300 Received: (from nicolau@localhost) by sucuri.mat.puc-rio.br (8.9.3/8.9.3) id LAA17826 for obm-l@mat.puc-rio.br; Fri, 27 Sep 2002 11:53:16 -0300 Date: Fri, 27 Sep 2002 11:53:15 -0300 From: "Nicolau C. Saldanha" To: obm-l@mat.puc-rio.br Subject: Re: [obm-l] Corpos tetradimensionais Message-ID: <20020927115315.E17380@sucuri.mat.puc-rio.br> References: <005f01c265b7$2098caa0$2101a8c0@u2z7z2> <002f01c265be$1e0f3560$9865fea9@computer> Mime-Version: 1.0 Content-Type: text/plain; charset=iso-8859-1 Content-Disposition: inline Content-Transfer-Encoding: 8bit User-Agent: Mutt/1.2.5i In-Reply-To: <002f01c265be$1e0f3560$9865fea9@computer>; from artur@opengate.com.br on Thu, Sep 26, 2002 at 09:37:53PM -0300 Sender: owner-obm-l@sucuri.mat.puc-rio.br Precedence: bulk Reply-To: obm-l@mat.puc-rio.br On Thu, Sep 26, 2002 at 09:37:53PM -0300, Artur Costa Steiner wrote: > > Oi para todos! > > Queria saber se um corpo tridimensional pode ser definido em um plano de > 4 dimensões como sendo a intersecção de 2 corpos tetradimensionais. Ex: > O sistema : x^2+y^2+z^2+w^2=16 ( I ) e x.y.z-w =3 > > André T. > > Na realidade, não existem corpos de dimensão superior a 2, entendendo-se > como corpo a estrutura algébrica que satisfaz àqueles conhecidos axiomas > referentes à soma e à multiplicação. Acho que o André *não* estava pensando na definição algébrica de corpo... []s, N. ========================================================================= Instruções para entrar na lista, sair da lista e usar a lista em http://www.mat.puc-rio.br/~nicolau/olimp/obm-l.html O administrador desta lista é ========================================================================= From owner-obm-l@sucuri.mat.puc-rio.br Fri Sep 27 14:28:13 2002 Return-Path: Received: (from majordom@localhost) by sucuri.mat.puc-rio.br (8.9.3/8.9.3) id OAA21873 for obm-l-MTTP; Fri, 27 Sep 2002 14:27:10 -0300 Received: from zeus.opendf.com.br (zeus.opengate.com.br [200.181.71.10]) by sucuri.mat.puc-rio.br (8.9.3/8.9.3) with ESMTP id OAA21868 for ; Fri, 27 Sep 2002 14:27:06 -0300 Received: from localhost (localhost.opengate.com.br [127.0.0.1]) by zeus.opendf.com.br (Postfix) with ESMTP id 667AF3EAA9 for ; Fri, 27 Sep 2002 14:28:47 -0300 (BRT) Received: by zeus.opendf.com.br (Postfix, from userid 48) id 474573EB6E; Fri, 27 Sep 2002 14:28:32 -0300 (BRT) From: "498 - Artur Costa Steiner" To: obm-l@mat.puc-rio.br Subject: Re: [obm-l] Corpos tetradimensionais X-Mailer: NeoMail 1.25 X-IPAddress: 200.252.155.2 MIME-Version: 1.0 Content-Type: text/plain; charset=iso-8859-1 Message-Id: <20020927172832.474573EB6E@zeus.opendf.com.br> Date: Fri, 27 Sep 2002 14:28:32 -0300 (BRT) X-Virus-Scanned: by AMaViS new-20020517 Sender: owner-obm-l@sucuri.mat.puc-rio.br Precedence: bulk Reply-To: obm-l@mat.puc-rio.br > On Thu, Sep 26, 2002 at 09:37:53PM -0300, Artur Costa Steiner wrote: > > > > Oi para todos! > > > > Queria saber se um corpo tridimensional pode ser definido em um plano de > > 4 dimensões como sendo a intersecção de 2 corpos tetradimensionais. Ex: > > O sistema : x^2+y^2+z^2+w^2=16 ( I ) e x.y.z-w =3 > > > > André T. > > > > Na realidade, não existem corpos de dimensão superior a 2, entendendo-se > > como corpo a estrutura algébrica que satisfaz àqueles conhecidos axiomas > > referentes à soma e à multiplicação. > > Acho que o André *não* estava pensando na definição algébrica de corpo... Bom...não faz mal, mesmo assim espero ter ajudado. Já que falamos em corpos, gostaria de tirar a seguinte dúvida. Li no newsgroup sci.math que, embora não existam corpos de dimensão finita >2, há corpos de dimensãoinfinita. Isto é verdade? Se for, como podemos definir um corpo infinito-dimensional? Sei que existem espaços vetoriais de dimensão infinita, mas não sabia que existiam corpos assim. Artur ========================================================================= Instruções para entrar na lista, sair da lista e usar a lista em http://www.mat.puc-rio.br/~nicolau/olimp/obm-l.html O administrador desta lista é ========================================================================= From owner-obm-l@sucuri.mat.puc-rio.br Fri Sep 27 14:44:25 2002 Return-Path: Received: (from majordom@localhost) by sucuri.mat.puc-rio.br (8.9.3/8.9.3) id OAA22264 for obm-l-MTTP; Fri, 27 Sep 2002 14:43:54 -0300 Received: from zeus.opendf.com.br (zeus.opengate.com.br [200.181.71.10]) by sucuri.mat.puc-rio.br (8.9.3/8.9.3) with ESMTP id OAA22239 for ; Fri, 27 Sep 2002 14:43:45 -0300 Received: from localhost (localhost.opengate.com.br [127.0.0.1]) by zeus.opendf.com.br (Postfix) with ESMTP id 87F6E3EB72 for ; Fri, 27 Sep 2002 14:45:28 -0300 (BRT) Received: by zeus.opendf.com.br (Postfix, from userid 48) id B61D13EB54; Fri, 27 Sep 2002 14:43:49 -0300 (BRT) From: "498 - Artur Costa Steiner" To: obm-l@mat.puc-rio.br Subject: Re: [obm-l] Corpos tetradimensionais X-Mailer: NeoMail 1.25 X-IPAddress: 200.252.155.2 MIME-Version: 1.0 Content-Type: text/plain; charset=iso-8859-1 Message-Id: <20020927174349.B61D13EB54@zeus.opendf.com.br> Date: Fri, 27 Sep 2002 14:43:49 -0300 (BRT) X-Virus-Scanned: by AMaViS new-20020517 Sender: owner-obm-l@sucuri.mat.puc-rio.br Precedence: bulk Reply-To: obm-l@mat.puc-rio.br > On Thu, Sep 26, 2002 at 08:47:50PM -0300, Wagner wrote: > > Oi para todos! > > > > Queria saber se um corpo tridimensional pode ser definido em um plano de 4 > > dimensões como sendo a intersecção de 2 corpos tetradimensionais. Ex: O > > sistema : x^2+y^2+z^2+w^2=16 ( I ) e x.y.z-w =3 > > Não, em geral duas equações em R^4 definem um objeto de dimensão 2 e não 3. > Note que *uma* equação em 4 variáveis já define um objeto de dimensão 3. > Por outro lado, duas *inequações* em R^4 definem um objeto de dimensão 4. > > []s, N. Acho que me enrolei um pouco. De modo geral, duas equações em R^n definem um objeto de dimensão de, no máximo, n-2, certo? Artur ========================================================================= Instruções para entrar na lista, sair da lista e usar a lista em http://www.mat.puc-rio.br/~nicolau/olimp/obm-l.html O administrador desta lista é ========================================================================= From owner-obm-l@sucuri.mat.puc-rio.br Fri Sep 27 20:10:59 2002 Return-Path: Received: (from majordom@localhost) by sucuri.mat.puc-rio.br (8.9.3/8.9.3) id UAA25638 for obm-l-MTTP; Fri, 27 Sep 2002 20:09:37 -0300 Received: from traven.uol.com.br (traven.uol.com.br [200.221.4.39]) by sucuri.mat.puc-rio.br (8.9.3/8.9.3) with ESMTP id UAA25634 for ; Fri, 27 Sep 2002 20:09:35 -0300 Received: from Eder ([200.211.159.178]) by traven.uol.com.br (8.9.1/8.9.1) with SMTP id TAA22515 for ; Fri, 27 Sep 2002 19:58:10 -0300 (BRT) Message-ID: <003b01c2667b$9386f0e0$d5d9fea9@Eder> From: "Eder" To: Subject: [obm-l] =?iso-8859-1?B?YW5hbO10aWNh?= Date: Fri, 27 Sep 2002 20:12:52 -0300 MIME-Version: 1.0 Content-Type: multipart/alternative; boundary="----=_NextPart_000_0028_01C26662.426468A0" X-Priority: 3 X-MSMail-Priority: Normal X-Mailer: Microsoft Outlook Express 5.00.2314.1300 X-MimeOLE: Produced By Microsoft MimeOLE V5.00.2314.1300 Sender: owner-obm-l@sucuri.mat.puc-rio.br Precedence: bulk Reply-To: obm-l@mat.puc-rio.br This is a multi-part message in MIME format. ------=_NextPart_000_0028_01C26662.426468A0 Content-Type: text/plain; charset="iso-8859-1" Content-Transfer-Encoding: quoted-printable Ol=E1, Dada a equa=E7=E3o de uma circunfer=EAncia : (x-4)=B2+(y-3)=B2=3D9,como = fa=E7o para achar o ponto sobre a mesma que est=E1 a menor dist=E2ncia = da origem?Algu=E9m poderia dar uma dica? ------=_NextPart_000_0028_01C26662.426468A0 Content-Type: text/html; charset="iso-8859-1" Content-Transfer-Encoding: quoted-printable
Ol=E1,
 
Dada a equa=E7=E3o de uma = circunfer=EAncia :=20 (x-4)=B2+(y-3)=B2=3D9,como fa=E7o para achar o ponto sobre a mesma que = est=E1 a menor=20 dist=E2ncia da origem?Algu=E9m poderia dar uma dica?
 
 
 
 
 
------=_NextPart_000_0028_01C26662.426468A0-- ========================================================================= Instruções para entrar na lista, sair da lista e usar a lista em http://www.mat.puc-rio.br/~nicolau/olimp/obm-l.html O administrador desta lista é ========================================================================= From owner-obm-l@sucuri.mat.puc-rio.br Fri Sep 27 20:22:20 2002 Return-Path: Received: (from majordom@localhost) by sucuri.mat.puc-rio.br (8.9.3/8.9.3) id UAA25761 for obm-l-MTTP; Fri, 27 Sep 2002 20:20:56 -0300 Received: from gorgo.centroin.com.br (gorgo.centroin.com.br [200.225.63.128]) by sucuri.mat.puc-rio.br (8.9.3/8.9.3) with ESMTP id UAA25757 for ; Fri, 27 Sep 2002 20:20:54 -0300 Received: from centroin.com.br (du79c.rjo.centroin.com.br [200.225.58.79]) (authenticated bits=0) by gorgo.centroin.com.br (8.12.2/8.12.1) with ESMTP id g8RNND5k023937 for ; Fri, 27 Sep 2002 20:23:15 -0300 (BRT) Message-ID: <3D94E905.80400@centroin.com.br> Date: Fri, 27 Sep 2002 20:25:57 -0300 From: Augusto =?ISO-8859-1?Q?C=E9sar?= Morgado User-Agent: Mozilla/5.0 (Windows; U; Win98; en-US; rv:0.9.4.1) Gecko/20020508 Netscape6/6.2.3 X-Accept-Language: en-us MIME-Version: 1.0 To: obm-l@mat.puc-rio.br Subject: Re: [obm-l] =?ISO-8859-1?Q?anal=EDtica?= References: <003b01c2667b$9386f0e0$d5d9fea9@Eder> Content-Type: multipart/alternative; boundary="------------050408020705050505090203" Sender: owner-obm-l@sucuri.mat.puc-rio.br Precedence: bulk Reply-To: obm-l@mat.puc-rio.br --------------050408020705050505090203 Content-Type: text/plain; charset=ISO-8859-1; format=flowed Content-Transfer-Encoding: 8bit Ligue a origem ao centro. Os pontos de interseçao com a circunferencia sao os que estao a maior e a menor distancia da origem Eder wrote: > Olá, > > > > Dada a equação de uma circunferência : (x-4)²+(y-3)²=9,como faço para > achar o ponto sobre a mesma que está a menor distância da > origem?Alguém poderia dar uma dica? > > > > > > > > > > > --------------050408020705050505090203 Content-Type: text/html; charset=us-ascii Content-Transfer-Encoding: 7bit Ligue a origem ao centro. Os pontos de interseçao com a circunferencia sao os que estao a maior e a menor distancia da origem

Eder wrote:
Olá,
 
Dada a equação de uma circunferência : (x-4)²+(y-3)²=9,como faço para achar o ponto sobre a mesma que está a menor distância da origem?Alguém poderia dar uma dica?
 
 
 
 
 

--------------050408020705050505090203-- ========================================================================= Instruções para entrar na lista, sair da lista e usar a lista em http://www.mat.puc-rio.br/~nicolau/olimp/obm-l.html O administrador desta lista é ========================================================================= From owner-obm-l@sucuri.mat.puc-rio.br Sat Sep 28 10:52:21 2002 Return-Path: Received: (from majordom@localhost) by sucuri.mat.puc-rio.br (8.9.3/8.9.3) id KAA31534 for obm-l-MTTP; Sat, 28 Sep 2002 10:51:19 -0300 Received: from artemis.opendf.com.br (artemis.opengate.com.br [200.181.71.15]) by sucuri.mat.puc-rio.br (8.9.3/8.9.3) with ESMTP id KAA31530 for ; Sat, 28 Sep 2002 10:51:16 -0300 Received: from localhost (localhost [127.0.0.1]) by artemis.opendf.com.br (Postfix) with ESMTP id 71CBE1CAA9 for ; Sat, 28 Sep 2002 10:52:56 -0300 (BRT) Received: from computer (200-181-89-112-bsace7001.dsl.telebrasilia.net.br [200.181.89.112]) by artemis.opendf.com.br (Postfix) with ESMTP id 37D0B1C881 for ; Sat, 28 Sep 2002 10:52:53 -0300 (BRT) From: "Artur Costa Steiner" To: Subject: [obm-l] =?iso-8859-1?B?RnVu5+NvIEFuYWztdGljYQ==?= Date: Sat, 28 Sep 2002 10:53:20 -0300 Organization: Steiner Consultoria LTDA Message-ID: <002001c266f6$6893f470$9865fea9@computer> MIME-Version: 1.0 Content-Type: text/plain; charset="iso-8859-1" X-Priority: 3 (Normal) X-MSMail-Priority: Normal X-Mailer: Microsoft Outlook, Build 10.0.2627 Importance: Normal In-Reply-To: <20020910164616.C27048@sucuri.mat.puc-rio.br> X-MimeOLE: Produced By Microsoft MimeOLE V6.00.2600.0000 X-Virus-Scanned: by AMaViS new-20020517 Content-Transfer-Encoding: 8bit X-MIME-Autoconverted: from quoted-printable to 8bit by sucuri.mat.puc-rio.br id KAA31531 Sender: owner-obm-l@sucuri.mat.puc-rio.br Precedence: bulk Reply-To: obm-l@mat.puc-rio.br Alguém poderia informar qual o verdadeiro significado do termo "função analítica"? Eu julgava que este termo só se aplicava a funções complexas e que significava uma função diferenciável em um subconjunto aberto do conjunto dos complexos. Mas já vi o termo ser aplicado a funções de R^n em R. Obrigado Artur ========================================================================= Instruções para entrar na lista, sair da lista e usar a lista em http://www.mat.puc-rio.br/~nicolau/olimp/obm-l.html O administrador desta lista é ========================================================================= From owner-obm-l@sucuri.mat.puc-rio.br Sat Sep 28 11:06:16 2002 Return-Path: Received: (from majordom@localhost) by sucuri.mat.puc-rio.br (8.9.3/8.9.3) id LAA31785 for obm-l-MTTP; Sat, 28 Sep 2002 11:06:08 -0300 Received: from proxy.annex.com.br (proxy.annex.com.br [200.248.173.4]) by sucuri.mat.puc-rio.br (8.9.3/8.9.3) with ESMTP id LAA31781 for ; Sat, 28 Sep 2002 11:06:05 -0300 Received: from mario ([200.248.173.35]) by proxy.annex.com.br (8.11.6/linuxconf) with SMTP id g8SEQWV06970 for ; Sat, 28 Sep 2002 11:26:32 -0300 Message-ID: <003901c266f8$7fa77a40$23adf8c8@mario> From: =?iso-8859-1?Q?M=E1rio_Pereira?= To: Subject: [obm-l] =?iso-8859-1?Q?d=FAvida?= Date: Sat, 28 Sep 2002 11:08:18 -0300 MIME-Version: 1.0 Content-Type: multipart/alternative; boundary="----=_NextPart_000_0036_01C266DF.59AAC860" X-Priority: 3 X-MSMail-Priority: Normal X-Mailer: Microsoft Outlook Express 5.50.4522.1200 X-MimeOLE: Produced By Microsoft MimeOLE V5.50.4522.1200 Sender: owner-obm-l@sucuri.mat.puc-rio.br Precedence: bulk Reply-To: obm-l@mat.puc-rio.br This is a multi-part message in MIME format. ------=_NextPart_000_0036_01C266DF.59AAC860 Content-Type: text/plain; charset="iso-8859-1" Content-Transfer-Encoding: quoted-printable Ol=E1, se algu=E9m puder, me d=EA uma dica: Em um jogo de televis=E3o, um candidato deve responder a 10 perguntas. A = primeira vale 1 ponto, a segunda vale 2 pontos, e assim, sucessivamente, = dobrando sempre. O candidato responde a todas as perguntas e ganha os = pontos correspondentes =E0s respostas que acertou, mesmo que erre = algumas. Se o candidato obteve 610 pontos, quantas perguntas acertou? Obrigado,=20 M=E1rio.=20 ------=_NextPart_000_0036_01C266DF.59AAC860 Content-Type: text/html; charset="iso-8859-1" Content-Transfer-Encoding: quoted-printable
 
 
Ol=E1, se algu=E9m puder, me d=EA uma = dica:
 
Em um jogo de televis=E3o, um candidato = deve=20 responder a 10 perguntas. A primeira vale 1 ponto, a segunda vale 2 = pontos, e=20 assim, sucessivamente, dobrando sempre. O candidato responde a todas as=20 perguntas e ganha os pontos correspondentes =E0s respostas que acertou, = mesmo que=20 erre algumas. Se o candidato obteve 610 pontos, quantas perguntas=20 acertou?
 
Obrigado,
 
M=E1rio.
------=_NextPart_000_0036_01C266DF.59AAC860-- ========================================================================= Instruções para entrar na lista, sair da lista e usar a lista em http://www.mat.puc-rio.br/~nicolau/olimp/obm-l.html O administrador desta lista é ========================================================================= From owner-obm-l@sucuri.mat.puc-rio.br Sat Sep 28 11:43:55 2002 Return-Path: Received: (from majordom@localhost) by sucuri.mat.puc-rio.br (8.9.3/8.9.3) id LAA00332 for obm-l-MTTP; Sat, 28 Sep 2002 11:43:44 -0300 Received: from artemis.opendf.com.br (artemis.opengate.com.br [200.181.71.15]) by sucuri.mat.puc-rio.br (8.9.3/8.9.3) with ESMTP id LAA00328 for ; Sat, 28 Sep 2002 11:43:40 -0300 Received: from localhost (localhost [127.0.0.1]) by artemis.opendf.com.br (Postfix) with ESMTP id D9F571CAB3 for ; Sat, 28 Sep 2002 11:45:19 -0300 (BRT) Received: from computer (200-181-89-112-bsace7001.dsl.telebrasilia.net.br [200.181.89.112]) by artemis.opendf.com.br (Postfix) with ESMTP id 4C6B71CAB2 for ; Sat, 28 Sep 2002 11:45:13 -0300 (BRT) From: "Artur Costa Steiner" To: Subject: [obm-l] 0,999..... Date: Sat, 28 Sep 2002 11:45:41 -0300 Organization: Steiner Consultoria LTDA Message-ID: <002b01c266fd$b82f6cb0$9865fea9@computer> MIME-Version: 1.0 Content-Type: multipart/alternative; boundary="----=_NextPart_000_002C_01C266E4.92E234B0" X-Priority: 3 (Normal) X-MSMail-Priority: Normal X-Mailer: Microsoft Outlook, Build 10.0.2627 Importance: Normal X-MimeOLE: Produced By Microsoft MimeOLE V6.00.2600.0000 X-Virus-Scanned: by AMaViS new-20020517 Sender: owner-obm-l@sucuri.mat.puc-rio.br Precedence: bulk Reply-To: obm-l@mat.puc-rio.br This is a multi-part message in MIME format. ------=_NextPart_000_002C_01C266E4.92E234B0 Content-Type: text/plain; charset="iso-8859-1" Content-Transfer-Encoding: quoted-printable Aqueles que costumam acessar o newsgroup internacional sci.math provavvelmente j=E1 notaram que a pergunta "=C9 0,99999.... =3D 1" j=E1 = apareceu zilh=F5es de vezes e que, toda vez que =E9 reapresentada, provoca = intensa pol=EAmica, levando a cadeias de resposta com 100 ou mais mensagens. Interessante que esta discuss=E3o acaba fatalmente indo para aspectos topol=F3gicos, e mesmo filos=F3ficos, ligados =E0 constru=E7=E3o dos = reais e assuntos correlatos. =20 H=E1 motivo para tanta pol=EAmica assim? Na realidade, 0,9999... =E9 = apenas uma representa=E7=E3o gr=E1fica da s=E9rie geom=E9trica 9/10 + 9/100 + ...9/10^n..., cujo limite =E9 1. 0,9999... =3D1 se, conforme = freq=FCentemente se faz, interpretarmos 0,999.... como o limite da s=E9rie, e n=E3o como = a s=E9rie em si, da mesma forma como se diz que 1 + 1/2!... + 1/n!..... = =3De. Qual a raz=E3o para tanta pol=EAmica a respeito deste assunto. =20 Artur =20 ------=_NextPart_000_002C_01C266E4.92E234B0 Content-Type: text/html; charset="iso-8859-1" Content-Transfer-Encoding: quoted-printable

Aqueles que = costumam acessar o newsgroup internacional sci.math provavvelmente j=E1 notaram = que a pergunta "=C9 0,99999.... =3D 1" j=E1 apareceu zilh=F5es de = vezes e que, toda vez que =E9 reapresentada, provoca intensa pol=EAmica, levando a cadeias = de resposta com 100 ou mais mensagens. Interessante que esta discuss=E3o = acaba fatalmente indo para aspectos topol=F3gicos,=A0 e mesmo filos=F3ficos, ligados =E0 constru=E7=E3o dos reais e = assuntos correlatos.

 

H=E1 motivo para = tanta pol=EAmica assim? Na realidade, 0,9999... =E9 apenas uma = representa=E7=E3o gr=E1fica da s=E9rie geom=E9trica 9/10 + 9/100 + ...9/10^n..., cujo limite =E9 1. = 0,9999... =3D1 se, conforme freq=FCentemente se faz, interpretarmos 0,999.... como o limite = da s=E9rie, e n=E3o como a s=E9rie em si, da mesma forma como se diz que 1 = + 1/2!... + 1/n!..... =3De. Qual a raz=E3o para tanta pol=EAmica a respeito deste = assunto.

 

Artur

 

------=_NextPart_000_002C_01C266E4.92E234B0-- ========================================================================= Instruções para entrar na lista, sair da lista e usar a lista em http://www.mat.puc-rio.br/~nicolau/olimp/obm-l.html O administrador desta lista é ========================================================================= From owner-obm-l@sucuri.mat.puc-rio.br Sat Sep 28 12:20:29 2002 Return-Path: Received: (from majordom@localhost) by sucuri.mat.puc-rio.br (8.9.3/8.9.3) id MAA01018 for obm-l-MTTP; Sat, 28 Sep 2002 12:19:52 -0300 Received: from web21109.mail.yahoo.com (web21109.mail.yahoo.com [216.136.227.111]) by sucuri.mat.puc-rio.br (8.9.3/8.9.3) with SMTP id MAA01014 for ; Sat, 28 Sep 2002 12:19:50 -0300 Message-ID: <20020928152151.99967.qmail@web21109.mail.yahoo.com> Received: from [150.161.2.4] by web21109.mail.yahoo.com via HTTP; Sat, 28 Sep 2002 12:21:51 ART Date: Sat, 28 Sep 2002 12:21:51 -0300 (ART) From: =?iso-8859-1?q?Carlos=20Ma=E7aranduba?= Subject: [obm-l] Grafos To: obm-l@mat.puc-rio.br In-Reply-To: <3D7D3AB6.8070306@centroin.com.br> MIME-Version: 1.0 Content-Type: text/plain; charset=iso-8859-1 Content-Transfer-Encoding: 8bit Sender: owner-obm-l@sucuri.mat.puc-rio.br Precedence: bulk Reply-To: obm-l@mat.puc-rio.br Um grafo pode ser hamiltoniano e euleriano ao mesmo tempo??Ou seja ter caminho hamiltoniano e caminho euleriano ao mesmo tempo????e quanto aos ciclos??? Podem coexistir em harmonia??? _______________________________________________________________________ Yahoo! GeoCities Tudo para criar o seu site: ferramentas fáceis de usar, espaço de sobra e acessórios. http://br.geocities.yahoo.com/ ========================================================================= Instruções para entrar na lista, sair da lista e usar a lista em http://www.mat.puc-rio.br/~nicolau/olimp/obm-l.html O administrador desta lista é ========================================================================= From owner-obm-l@sucuri.mat.puc-rio.br Sat Sep 28 12:20:42 2002 Return-Path: Received: (from majordom@localhost) by sucuri.mat.puc-rio.br (8.9.3/8.9.3) id MAA01055 for obm-l-MTTP; Sat, 28 Sep 2002 12:20:41 -0300 Received: from www.zipmail.com.br (smtp.zipmail.com.br [200.187.242.10]) by sucuri.mat.puc-rio.br (8.9.3/8.9.3) with ESMTP id MAA01051 for ; Sat, 28 Sep 2002 12:20:39 -0300 From: ghaeser@zipmail.com.br Received: from [200.158.6.55] by www.zipmail.com.br with HTTP; Sat, 28 Sep 2002 12:22:40 -0300 Message-ID: <3D939CCC00002EE3@www.zipmail.com.br> Date: Sat, 28 Sep 2002 12:22:40 -0300 In-Reply-To: <002b01c266fd$b82f6cb0$9865fea9@computer> Subject: [obm-l] =?iso-8859-1?Q?Bizarrice=20Geom=E9trica?= To: obm-l@mat.puc-rio.br MIME-Version: 1.0 Content-Type: text/plain; charset="iso-8859-1" Content-Transfer-Encoding: 8bit X-MIME-Autoconverted: from quoted-printable to 8bit by sucuri.mat.puc-rio.br id MAA01052 Sender: owner-obm-l@sucuri.mat.puc-rio.br Precedence: bulk Reply-To: obm-l@mat.puc-rio.br O que vocês acham disso: considere duas circunferências concêntricas com raios diferentes. Considere a circunferencia como uma roda no chao, seja A o ponto de contato da circunferencia externa com o chao, trace uma reta perpendicular ao chao passando por A, e seja B o primeiro ponto onde a reta toca a circunferencia externa. gire a circunferencia de uma volta completa de modo que o ponto de contato entre o solo e a circunferencia externa seja ainda A. conclusão: do ponto de vista do ponto A, a distancia entre a posição inicial e final do centro da circunferencua é o perímetro da circunferencia externa, mas do ponto de vista do ponto B, a distancia entre a posição inicial e final é o perímetro da circunferencia interna. Seria isso um paradoxo ? Gabriel Haeser www.gabas.cjb.net "Mathematicus nascitur, non fit" Matemáticos não são feitos, eles nascem --------------------------------------- Gabriel Haeser www.gabas.cjb.net ------------------------------------------ Use o melhor sistema de busca da Internet Radar UOL - http://www.radaruol.com.br ========================================================================= Instruções para entrar na lista, sair da lista e usar a lista em http://www.mat.puc-rio.br/~nicolau/olimp/obm-l.html O administrador desta lista é ========================================================================= From owner-obm-l@sucuri.mat.puc-rio.br Sat Sep 28 12:57:06 2002 Return-Path: Received: (from majordom@localhost) by sucuri.mat.puc-rio.br (8.9.3/8.9.3) id MAA02118 for obm-l-MTTP; Sat, 28 Sep 2002 12:56:42 -0300 Received: from bidu.ime.usp.br (bidu.ime.usp.br [143.107.45.12]) by sucuri.mat.puc-rio.br (8.9.3/8.9.3) with SMTP id MAA02114 for ; Sat, 28 Sep 2002 12:56:40 -0300 Received: (qmail 16809 invoked from network); 28 Sep 2002 15:58:42 -0000 Received: from mafalda.ime.usp.br (HELO jaca.ime.usp.br) (143.107.45.13) by bidu.ime.usp.br with SMTP; 28 Sep 2002 15:58:42 -0000 Received: (qmail 27228 invoked by uid 1604); 28 Sep 2002 15:58:45 -0000 Date: Sat, 28 Sep 2002 12:58:45 -0300 (EST) From: Salvador Addas Zanata X-Sender: sazanata@mafalda To: obm-l@mat.puc-rio.br Subject: Re: [obm-l] =?iso-8859-1?B?RnVu5+NvIEFuYWztdGljYQ==?= In-Reply-To: <002001c266f6$6893f470$9865fea9@computer> Message-ID: MIME-Version: 1.0 Content-Type: TEXT/PLAIN; charset=ISO-8859-1 Content-Transfer-Encoding: 8bit X-MIME-Autoconverted: from QUOTED-PRINTABLE to 8bit by sucuri.mat.puc-rio.br id MAA02115 Sender: owner-obm-l@sucuri.mat.puc-rio.br Precedence: bulk Reply-To: obm-l@mat.puc-rio.br A definicao de analiticidade pra funcoes complexas implica no seguinte fato: Se uma funcao complexa f e analitica num ponto, entao o seu polinomio de taylor centrado nesse ponto converge para f numa bola suficientemente pequena, centrada nesse ponto. Esse fato se obtem por derivacoes da formula integral de Cauchy... Pra funcoes f de R^n em R, por exemplo, diz-se que uma tal e analitica (num ponto) se o seu polinomio de Taylor (centrado nesse ponto) converge para f (numa vizinhanca do ponto). Por exemplo, arctan(x) e analitica em x=0, apesar de que seu polinomio de Taylor: p(x)=x-x^3/3+x^5/5-x^7/7+x^9/9-.... so converge para |x|<1. Por outro lado, f(x)=exp(-1/x^2), se x<>0 f(0)=0 E infinitamente diferenciavel no zero, se definirmos todas as derivadas no zero como sendo zero. (apesar das derivadas nao serem continuas no zero, o limite de todas f'''''''''(x)->0, para x->0). E claro que essa f nao e analitica, porque o seu polinomio de Taylor centrado no zero e identicamente nulo e a funcao f so se anula em x=0. Abraco, Salvador On Sat, 28 Sep 2002, Artur Costa Steiner wrote: > > Alguém poderia informar qual o verdadeiro significado do termo "função > analítica"? Eu julgava que este termo só se aplicava a funções complexas > e que significava uma função diferenciável em um subconjunto aberto do > conjunto dos complexos. Mas já vi o termo ser aplicado a funções de R^n > em R. > > Obrigado > Artur > > ========================================================================= > Instruções para entrar na lista, sair da lista e usar a lista em > http://www.mat.puc-rio.br/~nicolau/olimp/obm-l.html > O administrador desta lista é > ========================================================================= > ========================================================================= Instruções para entrar na lista, sair da lista e usar a lista em http://www.mat.puc-rio.br/~nicolau/olimp/obm-l.html O administrador desta lista é ========================================================================= From owner-obm-l@sucuri.mat.puc-rio.br Sat Sep 28 13:04:17 2002 Return-Path: Received: (from majordom@localhost) by sucuri.mat.puc-rio.br (8.9.3/8.9.3) id NAA02331 for obm-l-MTTP; Sat, 28 Sep 2002 13:04:15 -0300 Received: from salvatore4.bol.com.br ([200.221.24.52]) by sucuri.mat.puc-rio.br (8.9.3/8.9.3) with ESMTP id NAA02327 for ; Sat, 28 Sep 2002 13:04:13 -0300 Received: from bol.com.br (200.221.24.132) by salvatore4.bol.com.br (5.1.071) id 3D8B8880001B0869 for obm-l@mat.puc-rio.br; Sat, 28 Sep 2002 13:06:10 -0300 Date: Sat, 28 Sep 2002 13:03:12 -0300 Message-Id: Subject: [obm-l] =?iso-8859-1?q?Re=3A_Menor_dist=E2ncia_=E0_circunfer=EAncia?= MIME-Version: 1.0 Content-Type: text/plain;charset="iso-8859-1" From: "tarsis19" To: obm-l@mat.puc-rio.br X-XaM3-API-Version: 2.4.3.4.4 X-SenderIP: 198.81.8.1 Content-Transfer-Encoding: 8bit X-MIME-Autoconverted: from quoted-printable to 8bit by sucuri.mat.puc-rio.br id NAA02328 Sender: owner-obm-l@sucuri.mat.puc-rio.br Precedence: bulk Reply-To: obm-l@mat.puc-rio.br Para saber o ponto de menor distância da circunferência (x-4)^2 + (y-3)^2 = 9 à origem, deve-se achar a reta que passa pela origem e pelos pontos (4,3);(y=3x/4); e fazer a interseção dela com a circunferência. __________________________________________________________________________ Encontre sempre uma linha desocupada com o Discador BOL! http://www.bol.com.br/discador Ainda não tem AcessoBOL? Assine já! http://www.bol.com.br/acessobol ========================================================================= Instruções para entrar na lista, sair da lista e usar a lista em http://www.mat.puc-rio.br/~nicolau/olimp/obm-l.html O administrador desta lista é ========================================================================= From owner-obm-l@sucuri.mat.puc-rio.br Sat Sep 28 13:11:59 2002 Return-Path: Received: (from majordom@localhost) by sucuri.mat.puc-rio.br (8.9.3/8.9.3) id NAA02609 for obm-l-MTTP; Sat, 28 Sep 2002 13:11:51 -0300 Received: from sr1.terra.com.br (sr1.terra.com.br [200.176.3.16]) by sucuri.mat.puc-rio.br (8.9.3/8.9.3) with ESMTP id NAA02603 for ; Sat, 28 Sep 2002 13:11:48 -0300 Received: from engenho.terra.com.br (engenho.terra.com.br [200.176.3.42]) by sr1.terra.com.br (Postfix) with ESMTP id 5FDAF706D9 for ; Sat, 28 Sep 2002 13:13:51 -0300 (EST) Received: from stabel (200-180-183-020-paemt7002.dsl.telebrasilia.net.br [200.180.183.20]) (authenticated user dudasta) by engenho.terra.com.br (Postfix) with ESMTP id ED9BE680A4 for ; Sat, 28 Sep 2002 13:13:50 -0300 (EST) Message-ID: <002901c26e1c$87df8e70$0301a8c0@stabel> From: "Eduardo Casagrande Stabel" To: References: <3D939CCC00002EE3@www.zipmail.com.br> Subject: [obm-l] =?iso-8859-1?Q?Re:_=5Bobm-l=5D_Bizarrice_Geom=E9trica?= Date: Mon, 7 Oct 2002 13:13:53 -0300 MIME-Version: 1.0 Content-Type: text/plain; charset="iso-8859-1" Content-Transfer-Encoding: 8bit X-Priority: 3 X-MSMail-Priority: Normal X-Mailer: Microsoft Outlook Express 6.00.2600.0000 X-MimeOLE: Produced By Microsoft MimeOLE V6.00.2600.0000 Sender: owner-obm-l@sucuri.mat.puc-rio.br Precedence: bulk Reply-To: obm-l@mat.puc-rio.br Esse é o famoso Paradoxo da Roda de Aristóteles: http://mathworld.wolfram.com/AristotlesWheelParadox.html. Na verdade não é um paradoxo, é um pseudo-paradoxo. O erro está na parte em que o B percorre a mesma distância. O ponto B não se move como se o chão estivesse tocando a circunferência interna, ele se move mais rápido. Na verdade uma representação biunívoca de pontos de duas linhas não implica que elas tenham o mesmo comprimento, basta ver f:[0,1]->[0,2], f(x) = 2x. Eduardo. From: O que vocês acham disso: considere duas circunferências concêntricas com raios diferentes. Considere a circunferencia como uma roda no chao, seja A o ponto de contato da circunferencia externa com o chao, trace uma reta perpendicular ao chao passando por A, e seja B o primeiro ponto onde a reta toca a circunferencia externa. gire a circunferencia de uma volta completa de modo que o ponto de contato entre o solo e a circunferencia externa seja ainda A. conclusão: do ponto de vista do ponto A, a distancia entre a posição inicial e final do centro da circunferencua é o perímetro da circunferencia externa, mas do ponto de vista do ponto B, a distancia entre a posição inicial e final é o perímetro da circunferencia interna. Seria isso um paradoxo ? Gabriel Haeser www.gabas.cjb.net "Mathematicus nascitur, non fit" Matemáticos não são feitos, eles nascem --------------------------------------- Gabriel Haeser www.gabas.cjb.net ------------------------------------------ Use o melhor sistema de busca da Internet Radar UOL - http://www.radaruol.com.br ========================================================================= Instruções para entrar na lista, sair da lista e usar a lista em http://www.mat.puc-rio.br/~nicolau/olimp/obm-l.html O administrador desta lista é ========================================================================= ========================================================================= Instruções para entrar na lista, sair da lista e usar a lista em http://www.mat.puc-rio.br/~nicolau/olimp/obm-l.html O administrador desta lista é ========================================================================= From owner-obm-l@sucuri.mat.puc-rio.br Sat Sep 28 14:50:36 2002 Return-Path: Received: (from majordom@localhost) by sucuri.mat.puc-rio.br (8.9.3/8.9.3) id OAA04332 for obm-l-MTTP; Sat, 28 Sep 2002 14:50:13 -0300 Received: from hotmail.com (f219.pav2.hotmail.com [64.4.37.219]) by sucuri.mat.puc-rio.br (8.9.3/8.9.3) with ESMTP id OAA04328 for ; Sat, 28 Sep 2002 14:50:10 -0300 Received: from mail pickup service by hotmail.com with Microsoft SMTPSVC; Sat, 28 Sep 2002 10:52:12 -0700 Received: from 200.193.254.5 by pv2fd.pav2.hotmail.msn.com with HTTP; Sat, 28 Sep 2002 17:52:11 GMT X-Originating-IP: [200.193.254.5] From: "ricardo matos" To: obm-l@mat.puc-rio.br Subject: Re: [obm-l] Alguns pontos interessantes pouco mencionados Date: Sat, 28 Sep 2002 14:52:11 -0300 Mime-Version: 1.0 Content-Type: text/plain; charset=iso-8859-1; format=flowed Message-ID: X-OriginalArrivalTime: 28 Sep 2002 17:52:12.0291 (UTC) FILETIME=[C5F59530:01C26717] Sender: owner-obm-l@sucuri.mat.puc-rio.br Precedence: bulk Reply-To: obm-l@mat.puc-rio.br Sou aluno de Matemática Bacharelado do 7 semestre. Muitas vezes pensei sobre que tipo de curso de cálculo deveria ser dado para alunos de engenharia, não cheguei a nenhum tipo e conclusão precisa, concordo com você que estes teoremas e conceitos deveriam ser esclarecidos em curso de cálculo(não conhecia o segundo). Acho também que o conceito de derivada como inclinação de reta que melhor aproxima a função perto de um ponto(de forma mais precisa) deveria ser mais usado, pois com ele a regra da soma e da composição ficam mais claras, a regra do produto com a interpretação de variação de área de uma placa retangular de lados f(t) e g(t) variáveis também acho interessante( e outras interpretações geométricas). Não estou certo em quanto formalismo(acho este tema crucial) é necessário num curso de engenharia mas acho que para a maior parte dos teoremas essencias devem ser dadas "demonstrações intuitivas realmente convincentes" e talvez algumas demonstrações de verdade para que o aluno perceba que o formalismo realmente é necessário para não nos perdermos em nossas intuições. Pelo que vejo muitos alunos de engenharia só querem saber "como calcula e pronto"(o que acho deprimente) os alunos mais inteligentes no entanto se sentem muito melhor(tenho alguns amigos na engenharia) com algum entendimento real sobre o assunto(pelas explicações intuitivas) do que com demonstrações formais que na minha opinião(e na deles) pouco os ajudam. No entanto como já disse mesmo para eles é bom que tenham alguma idéia de como se faz uma demonstração formal(caso eles precisem), sempre acompanhada de alguma visão intuitiva(1 ou 2 demonstrações p/semestre é mais do que o suficiente). >From: "498 - Artur Costa Steiner" >Reply-To: obm-l@mat.puc-rio.br >To: obm-l@mat.puc-rio.br >Subject: [obm-l] Alguns pontos interessantes pouco mencionados >Date: Thu, 26 Sep 2002 17:36:24 -0300 (BRT) > >Olá para todos, > >Sou engenheiro, formei-me no início dos anos 70. Acho curioso que, na >cadeira de Cálculo ministrada durante meu curso de Engenharia, não >foram sequer mencionados alguns teoremas e conceitos dos quais, mais >tarde, vim a tomar conhecimento simplesmente por gostar de matemática . >Não que eles sejam de vital importância para um engenheiro, mas eu >gostaria de ouvir opiniões, inclusive de professores universitários, >sobre os seguintes pontos que, creio, ainda hoje não são citados em >cursos de engenharia: > >1) Definição precisa de continuidade, frisando-se, inclusive, o >conceito de continuidade uniforme- este último parece-me um conceito >importante e interessante. > >2) O teorema (creio que é conhecido por Teorema de Darbaux) o qual >afirma que derivadas de uma função real, de variável real, sempre >apresentam a chamada Propriedade do Valor Intermediário. Parece-me que >este interessante resultado, de fácil demonstração, não é muito >conhecido. > >3)Um outro teorema, de simples demonstração (que me parece não ser >também muito conhecido), o qual diz que derivadas em R jamais >apresentam descontinuidaes do tipo "salto". > >4) Apresentação mais precisa do conceito de diferenciabilidade de >funções de R^n em R. > >Quando fiz Engenharia, o meu curso de Cálculo Integral foi muito mais >um curso de como determinar primitivas. Os aspectos conceituais da >integral não foram frisados, jamais se falou, por exemplo, em Soma de >Riemann. s. Creio que alguém com agilidade algébrica pode fazer isso >muito bem sem ter a menor idéia do que seja, de fato, o processo de >integração. Espero que hoje não seja mais assim. > >Artur > >========================================================================= >Instruções para entrar na lista, sair da lista e usar a lista em >http://www.mat.puc-rio.br/~nicolau/olimp/obm-l.html >O administrador desta lista é >========================================================================= ========================================================================= Instruções para entrar na lista, sair da lista e usar a lista em http://www.mat.puc-rio.br/~nicolau/olimp/obm-l.html O administrador desta lista é ========================================================================= From owner-obm-l@sucuri.mat.puc-rio.br Sat Sep 28 19:57:14 2002 Return-Path: Received: (from majordom@localhost) by sucuri.mat.puc-rio.br (8.9.3/8.9.3) id TAA06188 for obm-l-MTTP; Sat, 28 Sep 2002 19:56:56 -0300 Received: from pina.terra.com.br (pina.terra.com.br [200.176.3.17]) by sucuri.mat.puc-rio.br (8.9.3/8.9.3) with ESMTP id TAA06184 for ; Sat, 28 Sep 2002 19:56:54 -0300 Received: from penha.terra.com.br (penha.terra.com.br [200.176.3.43]) by pina.terra.com.br (Postfix) with ESMTP id DA1A053712 for ; Sat, 28 Sep 2002 19:58:57 -0300 (EST) Received: from stabel (200-180-183-020-paemt7002.dsl.telebrasilia.net.br [200.180.183.20]) (authenticated user dudasta) by penha.terra.com.br (Postfix) with ESMTP id 92D88681A4 for ; Sat, 28 Sep 2002 19:58:57 -0300 (EST) Message-ID: <001501c26e55$1f5ba350$0301a8c0@stabel> From: "Eduardo Casagrande Stabel" To: Subject: [obm-l] =?iso-8859-1?Q?Quest=E3o_de_geometria.?= Date: Mon, 7 Oct 2002 19:58:58 -0300 MIME-Version: 1.0 Content-Type: text/plain; charset="iso-8859-1" Content-Transfer-Encoding: 8bit X-Priority: 3 X-MSMail-Priority: Normal X-Mailer: Microsoft Outlook Express 6.00.2600.0000 X-MimeOLE: Produced By Microsoft MimeOLE V6.00.2600.0000 Sender: owner-obm-l@sucuri.mat.puc-rio.br Precedence: bulk Reply-To: obm-l@mat.puc-rio.br Caros amigos da Lista, É dada uma circunferência C, e dois pontos P e Q que não pertencem à ela. Considere o conjunto dos pontos X, da circunferência, e calcule a soma das distâncias d = XP + XQ. Construir com régua e compasso o ponto X que minimiza d. Abraços, Eduardo. ========================================================================= Instruções para entrar na lista, sair da lista e usar a lista em http://www.mat.puc-rio.br/~nicolau/olimp/obm-l.html O administrador desta lista é ========================================================================= From owner-obm-l@sucuri.mat.puc-rio.br Sat Sep 28 21:01:13 2002 Return-Path: Received: (from majordom@localhost) by sucuri.mat.puc-rio.br (8.9.3/8.9.3) id VAA06989 for obm-l-MTTP; Sat, 28 Sep 2002 21:00:32 -0300 Received: from mail.vetor.com.br (wool.vetor.com.br [200.160.244.7]) by sucuri.mat.puc-rio.br (8.9.3/8.9.3) with ESMTP id VAA06985 for ; Sat, 28 Sep 2002 21:00:29 -0300 Received: from rodrigo (dl-ct-ip98.wb.com.br [200.216.73.98] (may be forged)) by mail.vetor.com.br (8.12.5) with SMTP id g8SNv9Mq017149 for ; Sat, 28 Sep 2002 20:57:10 -0300 Message-ID: <002401c2674b$48a9f060$6249d8c8@rodrigo> From: "Rodrigo Villard Milet" To: Subject: [obm-l] =?iso-8859-1?Q?Re:_=5Bobm-l=5D_d=FAvida?= Date: Sat, 28 Sep 2002 21:00:54 -0300 MIME-Version: 1.0 Content-Type: multipart/alternative; boundary="----=_NextPart_000_0021_01C26732.2260F340" X-Priority: 3 X-MSMail-Priority: Normal X-Mailer: Microsoft Outlook Express 4.72.3110.5 X-MimeOLE: Produced By Microsoft MimeOLE V4.72.3110.3 X-Virus-Scanned: by amavis (http://amavis.org/) Sender: owner-obm-l@sucuri.mat.puc-rio.br Precedence: bulk Reply-To: obm-l@mat.puc-rio.br This is a multi-part message in MIME format. ------=_NextPart_000_0021_01C26732.2260F340 Content-Type: text/plain; charset="iso-8859-1" Content-Transfer-Encoding: quoted-printable Escreva 610 na base 2 : 610 =3D (1001100010)_2. Como sabemos que a = representa=E7=E3o na base 2 =E9 =FAnica, ele acertou as perguntas 2,6,7 = e 10. Villard -----Mensagem original----- De: M=E1rio Pereira Para: obm-l@mat.puc-rio.br Data: S=E1bado, 28 de Setembro de 2002 11:22 Assunto: [obm-l] d=FAvida Ol=E1, se algu=E9m puder, me d=EA uma dica: Em um jogo de televis=E3o, um candidato deve responder a 10 perguntas. = A primeira vale 1 ponto, a segunda vale 2 pontos, e assim, = sucessivamente, dobrando sempre. O candidato responde a todas as = perguntas e ganha os pontos correspondentes =E0s respostas que acertou, = mesmo que erre algumas. Se o candidato obteve 610 pontos, quantas = perguntas acertou? Obrigado,=20 M=E1rio.=20 ------=_NextPart_000_0021_01C26732.2260F340 Content-Type: text/html; charset="iso-8859-1" Content-Transfer-Encoding: quoted-printable
Escreva 610 na base 2 : 610 =3D (1001100010)_2. Como sabemos que a=20 representa=E7=E3o na base 2 =E9 =FAnica, ele acertou as perguntas 2,6,7 = e 10.
Villard
------=_NextPart_000_0021_01C26732.2260F340-- ========================================================================= Instruções para entrar na lista, sair da lista e usar a lista em http://www.mat.puc-rio.br/~nicolau/olimp/obm-l.html O administrador desta lista é ========================================================================= From owner-obm-l@sucuri.mat.puc-rio.br Sat Sep 28 21:12:43 2002 Return-Path: Received: (from majordom@localhost) by sucuri.mat.puc-rio.br (8.9.3/8.9.3) id VAA07209 for obm-l-MTTP; Sat, 28 Sep 2002 21:12:40 -0300 Received: from hotmail.com (f68.sea2.hotmail.com [207.68.165.68]) by sucuri.mat.puc-rio.br (8.9.3/8.9.3) with ESMTP id VAA07205 for ; Sat, 28 Sep 2002 21:12:38 -0300 Received: from mail pickup service by hotmail.com with Microsoft SMTPSVC; Sat, 28 Sep 2002 17:14:40 -0700 Received: from 200.220.36.213 by sea2fd.sea2.hotmail.msn.com with HTTP; Sun, 29 Sep 2002 00:14:40 GMT X-Originating-IP: [200.220.36.213] From: "leonardo mattos" To: obm-l@mat.puc-rio.br Subject: [obm-l] =?iso-8859-1?B?UmU6IFtvYm0tbF0gZPp2aWRh?= Date: Sun, 29 Sep 2002 00:14:40 +0000 Mime-Version: 1.0 Content-Type: text/plain; charset=iso-8859-1; format=flowed Message-ID: X-OriginalArrivalTime: 29 Sep 2002 00:14:40.0442 (UTC) FILETIME=[341F89A0:01C2674D] Sender: owner-obm-l@sucuri.mat.puc-rio.br Precedence: bulk Reply-To: obm-l@mat.puc-rio.br Ola Mario, Escreva o numero 610 na base 2.Obtera desta forma quais questoes foram acertadas e consequentemente o numero de questoes acertadas uma vez que os nº1´s serao as questoes acertadas e os nº0´s as questoes nao acertadas. Um abraço,Leonardo >From: Mário Pereira >Reply-To: obm-l@mat.puc-rio.br >To: >Subject: [obm-l] dúvida >Date: Sat, 28 Sep 2002 11:08:18 -0300 > >Olá, se alguém puder, me dê uma dica: > >Em um jogo de televisão, um candidato deve responder a 10 perguntas. A >primeira vale 1 ponto, a segunda vale 2 pontos, e assim, sucessivamente, >dobrando sempre. O candidato responde a todas as perguntas e ganha os >pontos correspondentes às respostas que acertou, mesmo que erre algumas. Se >o candidato obteve 610 pontos, quantas perguntas acertou? > >Obrigado, > >Mário. _________________________________________________________________ Tenha você também um MSN Hotmail, o maior webmail do mundo: http://www.hotmail.com/br ========================================================================= Instruções para entrar na lista, sair da lista e usar a lista em http://www.mat.puc-rio.br/~nicolau/olimp/obm-l.html O administrador desta lista é ========================================================================= From owner-obm-l@sucuri.mat.puc-rio.br Sun Sep 29 07:39:54 2002 Return-Path: Received: (from majordom@localhost) by sucuri.mat.puc-rio.br (8.9.3/8.9.3) id HAA13400 for obm-l-MTTP; Sun, 29 Sep 2002 07:38:04 -0300 Received: (from nicolau@localhost) by sucuri.mat.puc-rio.br (8.9.3/8.9.3) id HAA13395 for obm-l@mat.puc-rio.br; Sun, 29 Sep 2002 07:38:03 -0300 Date: Sun, 29 Sep 2002 07:38:03 -0300 From: "Nicolau C. Saldanha" To: obm-l@mat.puc-rio.br Subject: Re: [obm-l] Corpos tetradimensionais Message-ID: <20020929073803.C13349@sucuri.mat.puc-rio.br> References: <20020927172832.474573EB6E@zeus.opendf.com.br> Mime-Version: 1.0 Content-Type: text/plain; charset=iso-8859-1 Content-Disposition: inline Content-Transfer-Encoding: 8bit User-Agent: Mutt/1.2.5i In-Reply-To: <20020927172832.474573EB6E@zeus.opendf.com.br>; from artur@opendf.com.br on Fri, Sep 27, 2002 at 02:28:32PM -0300 Sender: owner-obm-l@sucuri.mat.puc-rio.br Precedence: bulk Reply-To: obm-l@mat.puc-rio.br On Fri, Sep 27, 2002 at 02:28:32PM -0300, 498 - Artur Costa Steiner wrote: > Já que falamos em corpos, gostaria de tirar a seguinte dúvida. Li no > newsgroup sci.math que, embora não existam corpos de dimensão finita > >2, há corpos de dimensãoinfinita. Isto é verdade? Se for, como > podemos definir um corpo infinito-dimensional? Sei que existem espaços > vetoriais de dimensão infinita, mas não sabia que existiam corpos assim. Um exemplo é o das funções racionais com coeficientes reais ou complexos. Outro exemplo é o das funções meromorfas em algum aberto A contido em C. Uma função é meromorfa em A se ela for holomorfa em A menos um número finito de pontos sendo que todos estes pontos são polos. []s, N. ========================================================================= Instruções para entrar na lista, sair da lista e usar a lista em http://www.mat.puc-rio.br/~nicolau/olimp/obm-l.html O administrador desta lista é ========================================================================= From owner-obm-l@sucuri.mat.puc-rio.br Sun Sep 29 07:45:40 2002 Return-Path: Received: (from majordom@localhost) by sucuri.mat.puc-rio.br (8.9.3/8.9.3) id HAA13492 for obm-l-MTTP; Sun, 29 Sep 2002 07:44:24 -0300 Received: (from nicolau@localhost) by sucuri.mat.puc-rio.br (8.9.3/8.9.3) id HAA13487 for obm-l@mat.puc-rio.br; Sun, 29 Sep 2002 07:44:23 -0300 Date: Sun, 29 Sep 2002 07:44:23 -0300 From: "Nicolau C. Saldanha" To: obm-l@mat.puc-rio.br Subject: Re: [obm-l] 0,999..... Message-ID: <20020929074423.D13349@sucuri.mat.puc-rio.br> References: <002b01c266fd$b82f6cb0$9865fea9@computer> Mime-Version: 1.0 Content-Type: text/plain; charset=iso-8859-1 Content-Disposition: inline Content-Transfer-Encoding: 8bit User-Agent: Mutt/1.2.5i In-Reply-To: <002b01c266fd$b82f6cb0$9865fea9@computer>; from artur@opengate.com.br on Sat, Sep 28, 2002 at 11:45:41AM -0300 Sender: owner-obm-l@sucuri.mat.puc-rio.br Precedence: bulk Reply-To: obm-l@mat.puc-rio.br On Sat, Sep 28, 2002 at 11:45:41AM -0300, Artur Costa Steiner wrote: > Aqueles que costumam acessar o newsgroup internacional sci.math > provavvelmente já notaram que a pergunta "É 0,99999.... = 1" já apareceu > zilhões de vezes e que, toda vez que é reapresentada, provoca intensa > polêmica, levando a cadeias de resposta com 100 ou mais mensagens. > Interessante que esta discussão acaba fatalmente indo para aspectos > topológicos, e mesmo filosóficos, ligados à construção dos reais e > assuntos correlatos. > > Há motivo para tanta polêmica assim? Na realidade, 0,9999... é apenas > uma representação gráfica da série geométrica 9/10 + 9/100 + > ...9/10^n..., cujo limite é 1. 0,9999... =1 se, conforme freqüentemente > se faz, interpretarmos 0,999.... como o limite da série, e não como a > série em si, da mesma forma como se diz que 1 + 1/2!... + 1/n!..... =e. > Qual a razão para tanta polêmica a respeito deste assunto. Já se falou muito sobre este tema aqui, veja uma lista de links em http://www.mat.puc-rio.br/~nicolau/olimp/obm-l.200208/msg00112.html Aliás obrigado ao Eduardo Casagrande Stabel por preparar esta lista. []s, N. ========================================================================= Instruções para entrar na lista, sair da lista e usar a lista em http://www.mat.puc-rio.br/~nicolau/olimp/obm-l.html O administrador desta lista é ========================================================================= From owner-obm-l@sucuri.mat.puc-rio.br Sun Sep 29 07:49:28 2002 Return-Path: Received: (from majordom@localhost) by sucuri.mat.puc-rio.br (8.9.3/8.9.3) id HAA13555 for obm-l-MTTP; Sun, 29 Sep 2002 07:48:12 -0300 Received: (from nicolau@localhost) by sucuri.mat.puc-rio.br (8.9.3/8.9.3) id HAA13550 for obm-l@mat.puc-rio.br; Sun, 29 Sep 2002 07:48:12 -0300 Date: Sun, 29 Sep 2002 07:48:11 -0300 From: "Nicolau C. Saldanha" To: obm-l@mat.puc-rio.br Subject: [obm-l] Re: =?iso-8859-1?Q?=5Bobm-l=5D_Fun=E7=E3o_Anal=EDtica?= Message-ID: <20020929074811.E13349@sucuri.mat.puc-rio.br> References: <002001c266f6$6893f470$9865fea9@computer> Mime-Version: 1.0 Content-Type: text/plain; charset=iso-8859-1 Content-Disposition: inline Content-Transfer-Encoding: 8bit User-Agent: Mutt/1.2.5i In-Reply-To: ; from sazanata@ime.usp.br on Sat, Sep 28, 2002 at 12:58:45PM -0300 Sender: owner-obm-l@sucuri.mat.puc-rio.br Precedence: bulk Reply-To: obm-l@mat.puc-rio.br On Sat, Sep 28, 2002 at 12:58:45PM -0300, Salvador Addas Zanata wrote: > > A definicao de analiticidade pra funcoes complexas implica no seguinte > fato: > > Se uma funcao complexa f e analitica num ponto, entao o seu polinomio de > taylor centrado nesse ponto converge para f numa bola suficientemente > pequena, centrada nesse ponto. > > > Esse fato se obtem por derivacoes da formula integral de Cauchy... > > Pra funcoes f de R^n em R, por exemplo, diz-se que uma tal e analitica > (num ponto) se o seu polinomio de Taylor (centrado nesse > ponto) converge para f (numa vizinhanca do ponto). > > Por exemplo, arctan(x) e analitica em x=0, apesar de que seu polinomio de > Taylor: > > > p(x)=x-x^3/3+x^5/5-x^7/7+x^9/9-.... > > so converge para |x|<1. > > > Por outro lado, f(x)=exp(-1/x^2), se x<>0 > f(0)=0 > > E infinitamente diferenciavel no zero, se definirmos todas as derivadas no > zero como sendo zero. (apesar das derivadas nao serem continuas no zero, o > limite de todas f'''''''''(x)->0, para x->0). E claro que essa f nao e > analitica, porque o seu polinomio de Taylor centrado no zero e > identicamente nulo e a funcao f so se anula em x=0. Outra definição equivalente é a seguinte: uma função f: A -> R, A um subconjunto aberto de R^n é real analítica se existir uma função complexa analítica g: B -> C, B um aberto de C^n, A contido em B, g restrita a A igual a f. []s, N. ========================================================================= Instruções para entrar na lista, sair da lista e usar a lista em http://www.mat.puc-rio.br/~nicolau/olimp/obm-l.html O administrador desta lista é ========================================================================= From owner-obm-l@sucuri.mat.puc-rio.br Sun Sep 29 07:51:04 2002 Return-Path: Received: (from majordom@localhost) by sucuri.mat.puc-rio.br (8.9.3/8.9.3) id HAA13668 for obm-l-MTTP; Sun, 29 Sep 2002 07:49:48 -0300 Received: (from nicolau@localhost) by sucuri.mat.puc-rio.br (8.9.3/8.9.3) id HAA13663 for obm-l@mat.puc-rio.br; Sun, 29 Sep 2002 07:49:47 -0300 Date: Sun, 29 Sep 2002 07:49:47 -0300 From: "Nicolau C. Saldanha" To: obm-l@mat.puc-rio.br Subject: [obm-l] Re: =?iso-8859-1?Q?=5Bobm-l=5D_Re:_Menor_dist=E2ncia_=E0_circunfer=EAncia?= Message-ID: <20020929074947.F13349@sucuri.mat.puc-rio.br> References: Mime-Version: 1.0 Content-Type: text/plain; charset=iso-8859-1 Content-Disposition: inline Content-Transfer-Encoding: 8bit User-Agent: Mutt/1.2.5i In-Reply-To: ; from tarsis19@bol.com.br on Sat, Sep 28, 2002 at 01:03:12PM -0300 Sender: owner-obm-l@sucuri.mat.puc-rio.br Precedence: bulk Reply-To: obm-l@mat.puc-rio.br On Sat, Sep 28, 2002 at 01:03:12PM -0300, tarsis19 wrote: > Para saber o ponto de menor distância da circunferência > (x-4)^2 + (y-3)^2 = 9 à origem, deve-se achar a reta que > passa pela origem e pelos pontos (4,3);(y=3x/4); e fazer > a interseção dela com a circunferência. Mas se você só quer o valor da distância, basta fazer 5 - 3 = 2, onde 5 é a distância da origem ao centro (4,3) e 3 é o raio... []s, N. ========================================================================= Instruções para entrar na lista, sair da lista e usar a lista em http://www.mat.puc-rio.br/~nicolau/olimp/obm-l.html O administrador desta lista é ========================================================================= From owner-obm-l@sucuri.mat.puc-rio.br Sun Sep 29 14:07:51 2002 Return-Path: Received: (from majordom@localhost) by sucuri.mat.puc-rio.br (8.9.3/8.9.3) id OAA17033 for obm-l-MTTP; Sun, 29 Sep 2002 14:06:24 -0300 Received: from traven10.uol.com.br (traven10.uol.com.br [200.221.4.45]) by sucuri.mat.puc-rio.br (8.9.3/8.9.3) with ESMTP id OAA17029 for ; Sun, 29 Sep 2002 14:06:22 -0300 Received: from Eder ([200.211.159.148]) by traven10.uol.com.br (8.9.1/8.9.1) with SMTP id OAA11194 for ; Sun, 29 Sep 2002 14:07:37 -0300 (BRT) Message-ID: <007601c267da$f72dcaa0$e1e1fea9@Eder> From: "Eder" To: Subject: [obm-l] ??? Date: Sun, 29 Sep 2002 14:09:24 -0300 MIME-Version: 1.0 Content-Type: multipart/alternative; boundary="----=_NextPart_000_0073_01C267C1.D0A330A0" X-Priority: 3 X-MSMail-Priority: Normal X-Mailer: Microsoft Outlook Express 5.00.2314.1300 X-MimeOLE: Produced By Microsoft MimeOLE V5.00.2314.1300 Sender: owner-obm-l@sucuri.mat.puc-rio.br Precedence: bulk Reply-To: obm-l@mat.puc-rio.br This is a multi-part message in MIME format. ------=_NextPart_000_0073_01C267C1.D0A330A0 Content-Type: text/plain; charset="iso-8859-1" Content-Transfer-Encoding: quoted-printable Ol=E1, Gostaria de ajuda na situa=E7=E3o abaixo. Sendo a,b e c inteiros positivos,resolva: (1+1/a)(1+1/b)(1+1/c)=3D2. Eu estava tentando e n=E3o me pareceu haver solu=E7=E3o... ------=_NextPart_000_0073_01C267C1.D0A330A0 Content-Type: text/html; charset="iso-8859-1" Content-Transfer-Encoding: quoted-printable
Ol=E1,
 
 
 
Gostaria de ajuda na situa=E7=E3o = abaixo.
 
 
Sendo a,b e c inteiros = positivos,resolva:=20 (1+1/a)(1+1/b)(1+1/c)=3D2.
 
Eu estava tentando e n=E3o me pareceu = haver=20 solu=E7=E3o...
------=_NextPart_000_0073_01C267C1.D0A330A0-- ========================================================================= Instruções para entrar na lista, sair da lista e usar a lista em http://www.mat.puc-rio.br/~nicolau/olimp/obm-l.html O administrador desta lista é ========================================================================= From owner-obm-l@sucuri.mat.puc-rio.br Sun Sep 29 16:55:50 2002 Return-Path: Received: (from majordom@localhost) by sucuri.mat.puc-rio.br (8.9.3/8.9.3) id QAA18193 for obm-l-MTTP; Sun, 29 Sep 2002 16:54:20 -0300 Received: from pina.terra.com.br (pina.terra.com.br [200.176.3.17]) by sucuri.mat.puc-rio.br (8.9.3/8.9.3) with ESMTP id QAA18189 for ; Sun, 29 Sep 2002 16:54:18 -0300 Received: from penha.terra.com.br (penha.terra.com.br [200.176.3.43]) by pina.terra.com.br (Postfix) with ESMTP id BFDBD53170 for ; Sun, 29 Sep 2002 16:56:24 -0300 (EST) Received: from stabel (200-180-183-020-paemt7002.dsl.telebrasilia.net.br [200.180.183.20]) (authenticated user dudasta) by penha.terra.com.br (Postfix) with ESMTP id 3B02E680D2 for ; Sun, 29 Sep 2002 16:56:24 -0300 (EST) Message-ID: <001e01c26f04$c94eeb60$0301a8c0@stabel> From: "Eduardo Casagrande Stabel" To: References: <007601c267da$f72dcaa0$e1e1fea9@Eder> Subject: Re: [obm-l] ??? Date: Tue, 8 Oct 2002 16:56:25 -0300 MIME-Version: 1.0 Content-Type: multipart/alternative; boundary="----=_NextPart_000_001B_01C26EEB.A38A6080" X-Priority: 3 X-MSMail-Priority: Normal X-Mailer: Microsoft Outlook Express 6.00.2600.0000 X-MimeOLE: Produced By Microsoft MimeOLE V6.00.2600.0000 Sender: owner-obm-l@sucuri.mat.puc-rio.br Precedence: bulk Reply-To: obm-l@mat.puc-rio.br This is a multi-part message in MIME format. ------=_NextPart_000_001B_01C26EEB.A38A6080 Content-Type: text/plain; charset="iso-8859-1" Content-Transfer-Encoding: quoted-printable Caro Eder, a =3D 5, b =3D 4, c =3D 3 =E9 uma solu=E7=E3o. Existem mais solu=E7=F5es, por exemplo, 8, 3, 3 que encontrei com = aux=EDlio do computador. S=F3 n=E3o encontrei um jeito f=E1cil de encontr=E1-las. Um fato simples =E9 que se a > 6, b > 6 e c > 6 ent=E3o 1 + 1/a < 1 + 1/6 =3D 1.1666... < 1.25... =3D 2^(1/3) portanto (1 + 1/a)(1 + 1/b)(1 + 1/c) <=3D 1.666...^3 < 2. As solu=E7=F5es s=E3o com a, b, c "pequenos"... Eduardo. ----- Original Message -----=20 From: Eder=20 To: obm-l@mat.puc-rio.br=20 Sent: Sunday, September 29, 2002 2:09 PM Subject: [obm-l] ??? Ol=E1, Gostaria de ajuda na situa=E7=E3o abaixo. Sendo a,b e c inteiros positivos,resolva: (1+1/a)(1+1/b)(1+1/c)=3D2. Eu estava tentando e n=E3o me pareceu haver solu=E7=E3o... ------=_NextPart_000_001B_01C26EEB.A38A6080 Content-Type: text/html; charset="iso-8859-1" Content-Transfer-Encoding: quoted-printable
Caro Eder,
 
a = =3D 5, b =3D 4, c =3D 3=20 =E9 uma solu=E7=E3o.
 
Existem mais solu=E7=F5es, por exemplo, = 8, 3, 3 que=20 encontrei com aux=EDlio do computador.
S=F3 n=E3o encontrei um jeito f=E1cil = de=20 encontr=E1-las.
 
Um fato simples =E9 que se a > 6, b = > 6 e c=20 > 6 ent=E3o
1 + 1/a < 1 + 1/6 =3D 1.1666... < = 1.25... =3D=20 2^(1/3)
portanto
(1 + 1/a)(1 + 1/b)(1 + 1/c) <=3D = 1.666...^3 <=20 2.
 
As solu=E7=F5es s=E3o com a, b, c=20 "pequenos"...
 
Eduardo.
 
----- Original Message -----
From:=20 Eder
-----Mensagem = original-----
De:=20 M=E1rio Pereira <mpereira@annex.com.br>
Para:=20
obm-l@mat.puc-rio.br = <obm-l@mat.puc-rio.br>
D= ata:=20 S=E1bado, 28 de Setembro de 2002 11:22
Assunto: [obm-l]=20 d=FAvida

 
 
Ol=E1, se algu=E9m puder, me d=EA uma = dica:
 
Em um jogo de televis=E3o, um = candidato deve=20 responder a 10 perguntas. A primeira vale 1 ponto, a segunda vale 2 = pontos, e=20 assim, sucessivamente, dobrando sempre. O candidato responde a todas = as=20 perguntas e ganha os pontos correspondentes =E0s respostas que = acertou, mesmo=20 que erre algumas. Se o candidato obteve 610 pontos, quantas perguntas=20 acertou?
 
Obrigado,
 
M=E1rio. =
Sent: Sunday, September 29, = 2002 2:09=20 PM
Subject: [obm-l] ???

Ol=E1,
 
 
 
Gostaria de ajuda na situa=E7=E3o=20 abaixo.
 
 
Sendo a,b e c inteiros = positivos,resolva:=20 (1+1/a)(1+1/b)(1+1/c)=3D2.
 
Eu estava tentando e n=E3o me pareceu = haver=20 solu=E7=E3o...
------=_NextPart_000_001B_01C26EEB.A38A6080-- ========================================================================= Instruções para entrar na lista, sair da lista e usar a lista em http://www.mat.puc-rio.br/~nicolau/olimp/obm-l.html O administrador desta lista é ========================================================================= From owner-obm-l@sucuri.mat.puc-rio.br Sun Sep 29 17:38:57 2002 Return-Path: Received: (from majordom@localhost) by sucuri.mat.puc-rio.br (8.9.3/8.9.3) id RAA18813 for obm-l-MTTP; Sun, 29 Sep 2002 17:37:47 -0300 Received: from videira.terra.com.br (videira.terra.com.br [200.176.3.5]) by sucuri.mat.puc-rio.br (8.9.3/8.9.3) with ESMTP id RAA18809 for ; Sun, 29 Sep 2002 17:37:45 -0300 Received: from pavuna.terra.com.br (pavuna.terra.com.br [200.176.3.41]) by videira.terra.com.br (Postfix) with ESMTP id 3A223E120E for ; Sun, 29 Sep 2002 17:39:52 -0300 (EST) Received: from stabel (200-180-183-020-paemt7002.dsl.telebrasilia.net.br [200.180.183.20]) (authenticated user dudasta) by pavuna.terra.com.br (Postfix) with ESMTP id AF1D368246 for ; Sun, 29 Sep 2002 17:39:51 -0300 (EST) Message-ID: <003401c26f0a$db5804d0$0301a8c0@stabel> From: "Eduardo Casagrande Stabel" To: References: <002b01c266fd$b82f6cb0$9865fea9@computer> <20020929074423.D13349@sucuri.mat.puc-rio.br> Subject: Re: [obm-l] 0,999..... Date: Tue, 8 Oct 2002 17:39:53 -0300 MIME-Version: 1.0 Content-Type: text/plain; charset="iso-8859-1" Content-Transfer-Encoding: 8bit X-Priority: 3 X-MSMail-Priority: Normal X-Mailer: Microsoft Outlook Express 6.00.2600.0000 X-MimeOLE: Produced By Microsoft MimeOLE V6.00.2600.0000 Sender: owner-obm-l@sucuri.mat.puc-rio.br Precedence: bulk Reply-To: obm-l@mat.puc-rio.br Só para completar, há mais links (os mais interessantes, a meu ver) na minha outra mensagem: http://www.mat.puc-rio.br/~nicolau/olimp/obm-l.200208/msg00114.html. Eduardo. From: "Nicolau C. Saldanha" On Sat, Sep 28, 2002 at 11:45:41AM -0300, Artur Costa Steiner wrote: > Aqueles que costumam acessar o newsgroup internacional sci.math > provavvelmente já notaram que a pergunta "É 0,99999.... = 1" já apareceu > zilhões de vezes e que, toda vez que é reapresentada, provoca intensa > polêmica, levando a cadeias de resposta com 100 ou mais mensagens. > Interessante que esta discussão acaba fatalmente indo para aspectos > topológicos, e mesmo filosóficos, ligados à construção dos reais e > assuntos correlatos. > > Há motivo para tanta polêmica assim? Na realidade, 0,9999... é apenas > uma representação gráfica da série geométrica 9/10 + 9/100 + > ...9/10^n..., cujo limite é 1. 0,9999... =1 se, conforme freqüentemente > se faz, interpretarmos 0,999.... como o limite da série, e não como a > série em si, da mesma forma como se diz que 1 + 1/2!... + 1/n!..... =e. > Qual a razão para tanta polêmica a respeito deste assunto. Já se falou muito sobre este tema aqui, veja uma lista de links em http://www.mat.puc-rio.br/~nicolau/olimp/obm-l.200208/msg00112.html Aliás obrigado ao Eduardo Casagrande Stabel por preparar esta lista. []s, N. ========================================================================= Instruções para entrar na lista, sair da lista e usar a lista em http://www.mat.puc-rio.br/~nicolau/olimp/obm-l.html O administrador desta lista é ========================================================================= ========================================================================= Instruções para entrar na lista, sair da lista e usar a lista em http://www.mat.puc-rio.br/~nicolau/olimp/obm-l.html O administrador desta lista é ========================================================================= From owner-obm-l@sucuri.mat.puc-rio.br Mon Sep 30 01:19:27 2002 Return-Path: Received: (from majordom@localhost) by sucuri.mat.puc-rio.br (8.9.3/8.9.3) id BAA21271 for obm-l-MTTP; Mon, 30 Sep 2002 01:17:08 -0300 Received: from proxy.annex.com.br (proxy.annex.com.br [200.248.173.4]) by sucuri.mat.puc-rio.br (8.9.3/8.9.3) with ESMTP id BAA21267 for ; Mon, 30 Sep 2002 01:17:05 -0300 Received: from mario (CF031.annex.com.br [200.248.173.31]) by proxy.annex.com.br (8.11.6/linuxconf) with SMTP id g8U4bmM19275 for ; Mon, 30 Sep 2002 01:37:49 -0300 Message-ID: <009b01c26838$8f47cd60$1fadf8c8@mario> From: =?iso-8859-1?Q?M=E1rio_Pereira?= To: Subject: [obm-l] =?iso-8859-1?Q?d=FAvida=3F=3F?= Date: Mon, 30 Sep 2002 01:19:23 -0300 MIME-Version: 1.0 Content-Type: multipart/alternative; boundary="----=_NextPart_000_0098_01C2681F.6941F3C0" X-Priority: 3 X-MSMail-Priority: Normal X-Mailer: Microsoft Outlook Express 5.50.4522.1200 X-MIMEOLE: Produced By Microsoft MimeOLE V5.50.4522.1200 Sender: owner-obm-l@sucuri.mat.puc-rio.br Precedence: bulk Reply-To: obm-l@mat.puc-rio.br This is a multi-part message in MIME format. ------=_NextPart_000_0098_01C2681F.6941F3C0 Content-Type: text/plain; charset="iso-8859-1" Content-Transfer-Encoding: quoted-printable Amigos de lista, pe=E7o ajuda:=20 "Uma bola pula cada vez que bate no ch=E3o 2/3 da altura de onde caiu.=20 Deixando-a cair da altura de 12 metros, pergunta-se: a) qual ser=E1 a altura do terceiro pulo? b) Quanto percorreu ao bater no ch=E3o pela terceira vez? Obrigado pelo aux=EDlio.=20 M=E1rio.=20 ------=_NextPart_000_0098_01C2681F.6941F3C0 Content-Type: text/html; charset="iso-8859-1" Content-Transfer-Encoding: quoted-printable
Amigos de lista, pe=E7o ajuda: =
 
 
"Uma bola pula cada vez que bate no = ch=E3o 2/3 da=20 altura de onde caiu.
Deixando-a cair da altura de 12 metros, = pergunta-se:
a) qual ser=E1 a altura do terceiro=20 pulo?
b) Quanto percorreu ao bater no ch=E3o = pela terceira=20 vez?
 
 
Obrigado pelo aux=EDlio.
 
M=E1rio.
------=_NextPart_000_0098_01C2681F.6941F3C0-- ========================================================================= Instruções para entrar na lista, sair da lista e usar a lista em http://www.mat.puc-rio.br/~nicolau/olimp/obm-l.html O administrador desta lista é ========================================================================= From owner-obm-l@sucuri.mat.puc-rio.br Mon Sep 30 08:23:47 2002 Return-Path: Received: (from majordom@localhost) by sucuri.mat.puc-rio.br (8.9.3/8.9.3) id IAA23708 for obm-l-MTTP; Mon, 30 Sep 2002 08:20:55 -0300 Received: from trex.centroin.com.br (trex.centroin.com.br [200.225.63.134]) by sucuri.mat.puc-rio.br (8.9.3/8.9.3) with ESMTP id IAA23704 for ; Mon, 30 Sep 2002 08:20:53 -0300 Received: from trex.centroin.com.br (localhost [127.0.0.1]) by trex.centroin.com.br (8.12.5/8.12.1) with ESMTP id g8UBNOo4019046 for ; Mon, 30 Sep 2002 08:23:24 -0300 (BRT) Received: by trex.centroin.com.br (8.12.5/8.12.5/Submit) id g8UBNO8F019044; Mon, 30 Sep 2002 08:23:24 -0300 (BRT) Message-Id: <200209301123.g8UBNO8F019044@trex.centroin.com.br> Received: from 200.165.196.172 by trex.centroin.com.br (CIPWM versao 1.4C1) with HTTPS for ; Mon, 30 Sep 2002 08:23:23 -0300 (BRT) Date: Mon, 30 Sep 2002 08:23:23 -0300 (BRT) From: Augusto Cesar de Oliveira Morgado To: obm-l@mat.puc-rio.br Subject: =?iso-8859-1?q?Re: [obm-l] d=FAvida???= MIME-Version: 1.0 X-Mailer: CentroIn Internet Provider WebMail v. 1.4C1 (http://www.centroin.com.br/) Content-Type: text/plain; charset="iso-8859-1" Content-Transfer-Encoding: 8bit X-MIME-Autoconverted: from quoted-printable to 8bit by sucuri.mat.puc-rio.br id IAA23705 Sender: owner-obm-l@sucuri.mat.puc-rio.br Precedence: bulk Reply-To: obm-l@mat.puc-rio.br Em Mon, 30 Sep 2002 01:19:23 -0300, Mário_Pereira disse: > Amigos de lista, peço ajuda: > > > "Uma bola pula cada vez que bate no chão 2/3 da altura de onde caiu. > Deixando-a cair da altura de 12 metros, pergunta-se: > a) qual será a altura do terceiro pulo? > b) Quanto percorreu ao bater no chão pela terceira vez? > > > Obrigado pelo auxílio. > > Mário. A primeira queda é da altura 12, a segunda da altura 12*(2/3)=8, a terceira da altura 8*(2/3) = 16/3, a quarta 32/9. a) 32/9 metros b)12+8+8+16/3+16/3 = (38 + 2/3)metros ========================================================================= Instruções para entrar na lista, sair da lista e usar a lista em http://www.mat.puc-rio.br/~nicolau/olimp/obm-l.html O administrador desta lista é ========================================================================= From owner-obm-l@sucuri.mat.puc-rio.br Mon Sep 30 09:11:45 2002 Return-Path: Received: (from majordom@localhost) by sucuri.mat.puc-rio.br (8.9.3/8.9.3) id JAA24619 for obm-l-MTTP; Mon, 30 Sep 2002 09:08:53 -0300 Received: from hotmail.com (f96.sea2.hotmail.com [207.68.165.96]) by sucuri.mat.puc-rio.br (8.9.3/8.9.3) with ESMTP id JAA24615 for ; Mon, 30 Sep 2002 09:08:50 -0300 Received: from mail pickup service by hotmail.com with Microsoft SMTPSVC; Mon, 30 Sep 2002 05:10:58 -0700 Received: from 200.220.36.213 by sea2fd.sea2.hotmail.msn.com with HTTP; Mon, 30 Sep 2002 12:10:58 GMT X-Originating-IP: [200.220.36.213] From: "leonardo mattos" To: obm-l@mat.puc-rio.br Subject: [obm-l] =?iso-8859-1?B?UmU6IFtvYm0tbF0gZPp2aWRhPz8=?= Date: Mon, 30 Sep 2002 12:10:58 +0000 Mime-Version: 1.0 Content-Type: text/plain; charset=iso-8859-1; format=flowed Message-ID: X-OriginalArrivalTime: 30 Sep 2002 12:10:58.0731 (UTC) FILETIME=[6F97BFB0:01C2687A] Sender: owner-obm-l@sucuri.mat.puc-rio.br Precedence: bulk Reply-To: obm-l@mat.puc-rio.br Ola Mario, Perceba que os tamanhos das alturas sao os termos de uma P.G. em que o 1ºtermo é 12 e sua razao vale 2/3, logo a altura do 3ºpulo é 12*(2/3)^3.O caminho percorrido será nada mais nada menos que 12+2*12*(2/3)+2*12*(2/3)^2. Um abraço,Leonardo >From: Mário Pereira >Reply-To: obm-l@mat.puc-rio.br >To: >Subject: [obm-l] dúvida?? >Date: Mon, 30 Sep 2002 01:19:23 -0300 > >Amigos de lista, peço ajuda: > > >"Uma bola pula cada vez que bate no chão 2/3 da altura de onde caiu. >Deixando-a cair da altura de 12 metros, pergunta-se: >a) qual será a altura do terceiro pulo? >b) Quanto percorreu ao bater no chão pela terceira vez? > > >Obrigado pelo auxílio. > >Mário. ========================================================================= Instruções para entrar na lista, sair da lista e usar a lista em http://www.mat.puc-rio.br/~nicolau/olimp/obm-l.html O administrador desta lista é ========================================================================= From owner-obm-l@sucuri.mat.puc-rio.br Mon Sep 30 10:54:44 2002 Return-Path: Received: (from majordom@localhost) by sucuri.mat.puc-rio.br (8.9.3/8.9.3) id KAA26690 for obm-l-MTTP; Mon, 30 Sep 2002 10:52:48 -0300 Received: (from nicolau@localhost) by sucuri.mat.puc-rio.br (8.9.3/8.9.3) id KAA26685 for obm-l@mat.puc-rio.br; Mon, 30 Sep 2002 10:52:47 -0300 Date: Mon, 30 Sep 2002 10:52:46 -0300 From: "Nicolau C. Saldanha" To: obm-l@mat.puc-rio.br Subject: Re: [obm-l] ??? Message-ID: <20020930105246.B25713@sucuri.mat.puc-rio.br> References: <007601c267da$f72dcaa0$e1e1fea9@Eder> Mime-Version: 1.0 Content-Type: text/plain; charset=iso-8859-1 Content-Disposition: inline Content-Transfer-Encoding: 8bit User-Agent: Mutt/1.2.5i In-Reply-To: <007601c267da$f72dcaa0$e1e1fea9@Eder>; from edalbuquerque@uol.com.br on Sun, Sep 29, 2002 at 02:09:24PM -0300 Sender: owner-obm-l@sucuri.mat.puc-rio.br Precedence: bulk Reply-To: obm-l@mat.puc-rio.br On Sun, Sep 29, 2002 at 02:09:24PM -0300, Eder wrote: > Olá, > > > > Gostaria de ajuda na situação abaixo. > > > Sendo a,b e c inteiros positivos,resolva: (1+1/a)(1+1/b)(1+1/c)=2. > > Eu estava tentando e não me pareceu haver solução... Podemos supor 1 < a <= b <= c. Devemos ter a <= 3 pois se a >= 4 temos b, c >= 4 e (1+1/a)(1+1/b)(1+1/c) <= (1+1/4)(1+1/4)(1+1/4) = 125/64 < 128/64 = 2. Vamos estudar separadamente os casos a = 2 e a = 3. Para a = 2 temos 2 <= b <= c e (1+1/b)(1+1/c) = 4/3 donde devemos ter b <= 6 pois se b >= 7 temos c >= 7 e (1+1/b)(1+1/c) <= (1+1/7)(1+1/7) = 64/49 < 64/48 = 4/3. Assim basta testar b = 2, 3, 4, 5, 6 e ver para quais casos temos c inteiro positivo. Os casos b=2 e b=3 falham mas os outros nos dão as soluções: 2,4,15 2,5,9 2,6,7 Para a = 3 temos 3 <= b <= c e (1+1/b)(1+1/c) = 3/2 donde devemos ter b <= 4 pois se b >= 5 temos c >= 5 e (1 + 1/b)(1+1/c) <= (1+1/5)(1+1/5) = 36/25 < 36/24 = 3/2. Novamente basta testar b = 3 e b = 4 para obter as soluções 3,3,8 3,4,5 e estas são (a menos de permutações) todas as soluções. O problema fica mais interessante se permitirmos mais variáveis: (1+1/x1)...(1+1/xn) = 2, 1 < x1 <= ... <= xn inteiros. As soluções com n= 4 são 2,4,16,255 2,4,17,135 2,4,18,95 2,4,19,75 2,4,20,63 2,4,21,55 2,4,23,45 2,4,25,39 2,4,27,35 2,4,30,31 2,5,10,99 2,5,11,54 2,5,12,39 2,5,14,27 2,5,15,24 2,5,18,19 2,6,8,63 2,6,9,35 2,6,11,21 2,6,14,15 2,7,7,48 2,7,8,27 2,7,9,20 2,7,12,13 2,8,9,15 2,9,10,11 3,3,9,80 3,3,10,44 3,3,11,32 3,3,12,26 3,3,14,20 3,3,16,17 3,4,6,35 3,4,7,20 3,4,8,15 3,4,10,11 3,5,5,24 3,5,6,14 3,5,8,9 3,6,7,8 4,4,5,15 4,5,5,9 4,5,6,7 Esta lista foi gerada com maple, eu não sabia que ia ficar tão grande... []s, N. ========================================================================= Instruções para entrar na lista, sair da lista e usar a lista em http://www.mat.puc-rio.br/~nicolau/olimp/obm-l.html O administrador desta lista é ========================================================================= From owner-obm-l@sucuri.mat.puc-rio.br Mon Sep 30 13:15:32 2002 Return-Path: Received: (from majordom@localhost) by sucuri.mat.puc-rio.br (8.9.3/8.9.3) id NAA29212 for obm-l-MTTP; Mon, 30 Sep 2002 13:12:37 -0300 Received: from traven10.uol.com.br (traven10.uol.com.br [200.221.4.45]) by sucuri.mat.puc-rio.br (8.9.3/8.9.3) with ESMTP id NAA29208 for ; Mon, 30 Sep 2002 13:12:35 -0300 Received: from slap ([200.207.152.104]) by traven10.uol.com.br (8.9.1/8.9.1) with SMTP id NAA15791 for ; Mon, 30 Sep 2002 13:13:34 -0300 (BRT) Message-ID: <001601c2689d$91abba20$6898cfc8@slap> From: "Afemano" To: Subject: [obm-l] ONDE EU POSSO ARRANJAR ?? Date: Mon, 30 Sep 2002 13:22:23 -0300 MIME-Version: 1.0 Content-Type: multipart/alternative; boundary="----=_NextPart_000_0012_01C26884.6995EC40" X-Priority: 3 X-MSMail-Priority: Normal X-Mailer: Microsoft Outlook Express 6.00.2600.0000 X-MimeOLE: Produced By Microsoft MimeOLE V6.00.2600.0000 Sender: owner-obm-l@sucuri.mat.puc-rio.br Precedence: bulk Reply-To: obm-l@mat.puc-rio.br This is a multi-part message in MIME format. ------=_NextPart_000_0012_01C26884.6995EC40 Content-Type: text/plain; charset="iso-8859-1" Content-Transfer-Encoding: quoted-printable Onde eu posso arranjar algum programa de matem=E1tica que fa=E7a = gr=E1ficos ?? Ou resolva sistemas etc... algo do tipo da HP mas um pouco = melhor por se tratar de computador. Obrigado !! ------=_NextPart_000_0012_01C26884.6995EC40 Content-Type: text/html; charset="iso-8859-1" Content-Transfer-Encoding: quoted-printable
Onde eu posso arranjar algum programa = de matem=E1tica=20 que fa=E7a gr=E1ficos ?? Ou resolva sistemas etc... algo do tipo da HP = mas um pouco=20 melhor por se tratar de computador.
 
Obrigado !!
------=_NextPart_000_0012_01C26884.6995EC40-- ========================================================================= Instruções para entrar na lista, sair da lista e usar a lista em http://www.mat.puc-rio.br/~nicolau/olimp/obm-l.html O administrador desta lista é ========================================================================= From owner-obm-l@sucuri.mat.puc-rio.br Mon Sep 30 14:04:46 2002 Return-Path: Received: (from majordom@localhost) by sucuri.mat.puc-rio.br (8.9.3/8.9.3) id OAA30367 for obm-l-MTTP; Mon, 30 Sep 2002 14:01:41 -0300 Received: from mat.puc-rio.br (IDENT:root@perere.mat.puc-rio.br [139.82.27.60]) by sucuri.mat.puc-rio.br (8.9.3/8.9.3) with ESMTP id OAA30363 for ; Mon, 30 Sep 2002 14:01:40 -0300 Received: from localhost (fredpalm@localhost) by mat.puc-rio.br (8.9.3/8.9.3) with ESMTP id GAA11992 for ; Mon, 30 Sep 2002 06:24:20 -0300 Date: Mon, 30 Sep 2002 06:24:19 -0300 (BRT) From: Carlos Frederico Borges Palmeira To: obm-l@mat.puc-rio.br Subject: Re: [obm-l] ONDE EU POSSO ARRANJAR ?? In-Reply-To: <001601c2689d$91abba20$6898cfc8@slap> Message-ID: MIME-Version: 1.0 Content-Type: TEXT/PLAIN; charset=X-UNKNOWN Content-Transfer-Encoding: 8bit X-MIME-Autoconverted: from QUOTED-PRINTABLE to 8bit by sucuri.mat.puc-rio.br id OAA30364 Sender: owner-obm-l@sucuri.mat.puc-rio.br Precedence: bulk Reply-To: obm-l@mat.puc-rio.br o mathematics plotting program e' de graca e facil de usar. O mupad versao estudante tambem e' gratis, e e' mais poderoso, mais nao e' tao facil de usar. Fred palmeira On Mon, 30 Sep 2002, Afemano wrote: > Onde eu posso arranjar algum programa de matemática que faça gráficos ?? Ou resolva sistemas etc... algo do tipo da HP mas um pouco melhor por se tratar de computador. > > Obrigado !! > ========================================================================= Instruções para entrar na lista, sair da lista e usar a lista em http://www.mat.puc-rio.br/~nicolau/olimp/obm-l.html O administrador desta lista é ========================================================================= From owner-obm-l@sucuri.mat.puc-rio.br Mon Sep 30 20:30:53 2002 Return-Path: Received: (from majordom@localhost) by sucuri.mat.puc-rio.br (8.9.3/8.9.3) id UAA02138 for obm-l-MTTP; Mon, 30 Sep 2002 20:27:51 -0300 Received: from silva5.uol.com.br (silva5.uol.com.br [200.221.4.52]) by sucuri.mat.puc-rio.br (8.9.3/8.9.3) with ESMTP id UAA02134 for ; Mon, 30 Sep 2002 20:27:49 -0300 Received: from u2z7z2 ([200.158.144.21]) by silva5.uol.com.br (8.9.1/8.9.1) with ESMTP id UAA22507 for ; Mon, 30 Sep 2002 20:32:35 -0300 (EST) Message-ID: <000a01c268d9$7ff8d100$2101a8c0@u2z7z2> From: "Wagner" To: Subject: [obm-l] =?iso-8859-1?Q?Re:=5Bobm-l=5D_d=FAvida=3F=3F_?= Date: Mon, 30 Sep 2002 20:31:27 -0300 Organization: Wagner MIME-Version: 1.0 Content-Type: multipart/alternative; boundary="----=_NextPart_000_0007_01C268C0.5A3F42A0" X-Priority: 3 X-MSMail-Priority: Normal X-Mailer: Microsoft Outlook Express 5.50.4133.2400 X-MimeOLE: Produced By Microsoft MimeOLE V5.50.4133.2400 Sender: owner-obm-l@sucuri.mat.puc-rio.br Precedence: bulk Reply-To: obm-l@mat.puc-rio.br This is a multi-part message in MIME format. ------=_NextPart_000_0007_01C268C0.5A3F42A0 Content-Type: text/plain; charset="iso-8859-1" Content-Transfer-Encoding: quoted-printable Oi pessoal ! M=E1rio wrote:=20 Amigos de lista, pe=E7o ajuda:=20 "Uma bola pula cada vez que bate no ch=E3o 2/3 da altura de onde caiu.=20 Deixando-a cair da altura de 12 metros, pergunta-se: a) qual ser=E1 a altura do terceiro pulo? b) Quanto percorreu ao bater no ch=E3o pela terceira vez? As alturas m=E1ximas a cada pulo est=E3o em progress=E3o geom=E9trica = de raz=E3o 2/3 e termo inicial 12, assim como a dist=E2ncia percorrida a = cada pulo tamb=E9m forma uma PG de raz=E3o 2/3 e termo inicial 24. Logo: a) x =3D 12.(2/3)^2 =3D 16/3 metros b) Considerando que a pergunta seja quanto ela percorreu desde que =E9 = abandonada da altura inicial : y =3D 24.((2/3)^3 - 1)/(2/3 -1) =3D 24.(-19/27)/(-1/3) =3D 24.3.19/27 = =3D 152/3 metros ------=_NextPart_000_0007_01C268C0.5A3F42A0 Content-Type: text/html; charset="iso-8859-1" Content-Transfer-Encoding: quoted-printable
Oi pessoal !
 
 
M=E1rio wrote: 
 
Amigos de lista, pe=E7o ajuda: =
 
 
"Uma bola pula cada vez que bate no = ch=E3o 2/3 da=20 altura de onde caiu.
Deixando-a cair da altura de 12 metros, = pergunta-se:
a) qual ser=E1 a altura do terceiro=20 pulo?
b) Quanto percorreu ao bater no ch=E3o = pela terceira=20 vez?
 
 
 As alturas m=E1ximas a cada pulo = est=E3o em=20 progress=E3o geom=E9trica de raz=E3o 2/3 e termo inicial 12, assim como = a dist=E2ncia=20 percorrida a cada pulo tamb=E9m forma uma PG de raz=E3o 2/3 e termo = inicial 24.=20 Logo:
 
a) x =3D 12.(2/3)^2 =3D 16/3 = metros
b) Considerando que a pergunta seja = quanto ela=20 percorreu desde que =E9 abandonada da altura inicial :
y =3D 24.((2/3)^3 - 1)/(2/3 -1) =3D = 24.(-19/27)/(-1/3)=20 =3D 24.3.19/27 =3D 152/3 metros
------=_NextPart_000_0007_01C268C0.5A3F42A0-- ========================================================================= Instruções para entrar na lista, sair da lista e usar a lista em http://www.mat.puc-rio.br/~nicolau/olimp/obm-l.html O administrador desta lista é ========================================================================= From owner-obm-l@sucuri.mat.puc-rio.br Mon Sep 30 20:42:06 2002 Return-Path: Received: (from majordom@localhost) by sucuri.mat.puc-rio.br (8.9.3/8.9.3) id UAA02284 for obm-l-MTTP; Mon, 30 Sep 2002 20:39:28 -0300 Received: from silva5.uol.com.br (silva5.uol.com.br [200.221.4.52]) by sucuri.mat.puc-rio.br (8.9.3/8.9.3) with ESMTP id UAA02280 for ; Mon, 30 Sep 2002 20:39:26 -0300 Received: from slap ([200.207.152.104]) by silva5.uol.com.br (8.9.1/8.9.1) with SMTP id UAA07658 for ; Mon, 30 Sep 2002 20:44:12 -0300 (EST) Message-ID: <000c01c268db$ec087920$6898cfc8@slap> From: "Afemano" To: References: Subject: Re: [obm-l] ONDE EU POSSO ARRANJAR ?? Date: Mon, 30 Sep 2002 20:48:47 -0300 MIME-Version: 1.0 Content-Type: text/plain; charset="iso-8859-1" Content-Transfer-Encoding: 8bit X-Priority: 3 X-MSMail-Priority: Normal X-Mailer: Microsoft Outlook Express 6.00.2600.0000 X-MimeOLE: Produced By Microsoft MimeOLE V6.00.2600.0000 Sender: owner-obm-l@sucuri.mat.puc-rio.br Precedence: bulk Reply-To: obm-l@mat.puc-rio.br Bom, vou tentar achar aqui nuns sites de busca. Mas você não tem algum lugar certo pra puxar não ?! Valeu por enquanto !! ----- Original Message ----- From: "Carlos Frederico Borges Palmeira" To: Sent: Monday, September 30, 2002 6:24 AM Subject: Re: [obm-l] ONDE EU POSSO ARRANJAR ?? > o mathematics plotting program e' de graca e facil de usar. O mupad versao > estudante tambem e' gratis, e e' mais poderoso, mais nao e' tao facil de > usar. > Fred palmeira > > On Mon, 30 Sep 2002, Afemano wrote: > > > Onde eu posso arranjar algum programa de matemática que faça gráficos ?? Ou resolva sistemas etc... algo do tipo da HP mas um pouco melhor por se tratar de computador. > > > > Obrigado !! > > > > ========================================================================= > Instruções para entrar na lista, sair da lista e usar a lista em > http://www.mat.puc-rio.br/~nicolau/olimp/obm-l.html > O administrador desta lista é > ========================================================================= ========================================================================= Instruções para entrar na lista, sair da lista e usar a lista em http://www.mat.puc-rio.br/~nicolau/olimp/obm-l.html O administrador desta lista é ========================================================================= From owner-obm-l@sucuri.mat.puc-rio.br Mon Sep 30 21:20:14 2002 Return-Path: Received: (from majordom@localhost) by sucuri.mat.puc-rio.br (8.9.3/8.9.3) id VAA03387 for obm-l-MTTP; Mon, 30 Sep 2002 21:16:58 -0300 Received: from toole.uol.com.br (toole.uol.com.br [200.221.4.26]) by sucuri.mat.puc-rio.br (8.9.3/8.9.3) with ESMTP id VAA03383 for ; Mon, 30 Sep 2002 21:16:56 -0300 Received: from Eder ([200.211.161.166]) by toole.uol.com.br (8.9.1/8.9.1) with SMTP id VAA14385 for ; Mon, 30 Sep 2002 21:13:26 -0300 (BRT) Message-ID: <004a01c268e0$83a9c6e0$082efea9@Eder> From: "Eder" To: Subject: [obm-l] Mais uma... Date: Mon, 30 Sep 2002 21:20:42 -0300 MIME-Version: 1.0 Content-Type: multipart/alternative; boundary="----=_NextPart_000_0047_01C268C7.3B8EF380" X-Priority: 3 X-MSMail-Priority: Normal X-Mailer: Microsoft Outlook Express 5.00.2314.1300 X-MimeOLE: Produced By Microsoft MimeOLE V5.00.2314.1300 Sender: owner-obm-l@sucuri.mat.puc-rio.br Precedence: bulk Reply-To: obm-l@mat.puc-rio.br This is a multi-part message in MIME format. ------=_NextPart_000_0047_01C268C7.3B8EF380 Content-Type: text/plain; charset="iso-8859-1" Content-Transfer-Encoding: quoted-printable Essa foi da olimp=EDada russa: "Sabendo que a quadr=E1tica x=B2+ax+b+1 tem ra=EDzes inteiras e = positivas,mostre que a=B2+b=B2 =E9 composto." Bom,supondo X1 e X2 ra=EDzes,analisei as possibilidades de serem ambas = =EDmpares,uma par e outra =EDmpar e as duas pares.O =FAnico problema que = encontrei foi para o =FAltimo caso.X1 e X2 pares implica b =EDmpar e a = par.Diferente dos outros casos,n=E3o pude ou n=E3o vi como concluir que = a=B2+b=B2 =E9 composto.Aguardo coment=E1rios. ------=_NextPart_000_0047_01C268C7.3B8EF380 Content-Type: text/html; charset="iso-8859-1" Content-Transfer-Encoding: quoted-printable
Essa foi da olimp=EDada = russa:
 
 
"Sabendo que a quadr=E1tica x=B2+ax+b+1 = tem ra=EDzes=20 inteiras e positivas,mostre que a=B2+b=B2 =E9 composto."
 
Bom,supondo X1 e X2 ra=EDzes,analisei = as=20 possibilidades de serem ambas =EDmpares,uma par e outra =EDmpar e as = duas pares.O=20 =FAnico problema que encontrei foi para o =FAltimo caso.X1 e X2 pares = implica b=20 =EDmpar e a par.Diferente dos outros casos,n=E3o pude ou n=E3o vi como = concluir que=20 a=B2+b=B2 =E9 composto.Aguardo coment=E1rios.
------=_NextPart_000_0047_01C268C7.3B8EF380-- ========================================================================= Instruções para entrar na lista, sair da lista e usar a lista em http://www.mat.puc-rio.br/~nicolau/olimp/obm-l.html O administrador desta lista é ========================================================================= From owner-obm-l@sucuri.mat.puc-rio.br Mon Sep 30 21:52:24 2002 Return-Path: Received: (from majordom@localhost) by sucuri.mat.puc-rio.br (8.9.3/8.9.3) id VAA03990 for obm-l-MTTP; Mon, 30 Sep 2002 21:49:19 -0300 Received: from hotmail.com (f131.sea2.hotmail.com [207.68.165.131]) by sucuri.mat.puc-rio.br (8.9.3/8.9.3) with ESMTP id VAA03986 for ; Mon, 30 Sep 2002 21:49:16 -0300 Received: from mail pickup service by hotmail.com with Microsoft SMTPSVC; Mon, 30 Sep 2002 17:51:26 -0700 Received: from 200.220.36.213 by sea2fd.sea2.hotmail.msn.com with HTTP; Tue, 01 Oct 2002 00:51:25 GMT X-Originating-IP: [200.220.36.213] From: "leonardo mattos" To: obm-l@mat.puc-rio.br Subject: [obm-l] =?iso-8859-1?B?UmU6IFtvYm0tbF0gUmU6W29ibS1sXSBk+nZpZGE/Pw==?= Date: Tue, 01 Oct 2002 00:51:25 +0000 Mime-Version: 1.0 Content-Type: text/plain; charset=iso-8859-1; format=flowed Message-ID: X-OriginalArrivalTime: 01 Oct 2002 00:51:26.0196 (UTC) FILETIME=[ABAE6340:01C268E4] Sender: owner-obm-l@sucuri.mat.puc-rio.br Precedence: bulk Reply-To: obm-l@mat.puc-rio.br Ola, Quanto a letra a).Pq nao seria 12*(2/3)^3?!Considerando que no 1ºpulo ela alcança 12*2/3,2ºpulo 12*(2/3)^2 e no 3º12*(2/3)^3... Um abraço,Leonardo >From: "Wagner" >Reply-To: obm-l@mat.puc-rio.br >To: >Subject: [obm-l] Re:[obm-l] dúvida?? Date: Mon, 30 Sep 2002 20:31:27 -0300 > >Oi pessoal ! > > >Mário wrote: > >Amigos de lista, peço ajuda: > > >"Uma bola pula cada vez que bate no chão 2/3 da altura de onde caiu. >Deixando-a cair da altura de 12 metros, pergunta-se: >a) qual será a altura do terceiro pulo? >b) Quanto percorreu ao bater no chão pela terceira vez? > > > As alturas máximas a cada pulo estão em progressão geométrica de razão >2/3 e termo inicial 12, assim como a distância percorrida a cada pulo >também forma uma PG de razão 2/3 e termo inicial 24. Logo: > >a) x = 12.(2/3)^2 = 16/3 metros >b) Considerando que a pergunta seja quanto ela percorreu desde que é >abandonada da altura inicial : >y = 24.((2/3)^3 - 1)/(2/3 -1) = 24.(-19/27)/(-1/3) = 24.3.19/27 = 152/3 >metros _________________________________________________________________ Converse com seus amigos online, faça o download grátis do MSN Messenger: http://messenger.msn.com.br ========================================================================= Instruções para entrar na lista, sair da lista e usar a lista em http://www.mat.puc-rio.br/~nicolau/olimp/obm-l.html O administrador desta lista é ========================================================================= From owner-obm-l@sucuri.mat.puc-rio.br Mon Sep 30 22:20:13 2002 Return-Path: Received: (from majordom@localhost) by sucuri.mat.puc-rio.br (8.9.3/8.9.3) id WAA04437 for obm-l-MTTP; Mon, 30 Sep 2002 22:17:38 -0300 Received: from hotmail.com (f161.sea2.hotmail.com [207.68.165.161]) by sucuri.mat.puc-rio.br (8.9.3/8.9.3) with ESMTP id WAA04424 for ; Mon, 30 Sep 2002 22:17:35 -0300 Received: from mail pickup service by hotmail.com with Microsoft SMTPSVC; Mon, 30 Sep 2002 18:19:45 -0700 Received: from 200.220.36.213 by sea2fd.sea2.hotmail.msn.com with HTTP; Tue, 01 Oct 2002 01:19:45 GMT X-Originating-IP: [200.220.36.213] From: "leonardo mattos" To: obm-l@mat.puc-rio.br Subject: Re: [obm-l] Mais uma... Date: Tue, 01 Oct 2002 01:19:45 +0000 Mime-Version: 1.0 Content-Type: text/plain; charset=iso-8859-1; format=flowed Message-ID: X-OriginalArrivalTime: 01 Oct 2002 01:19:45.0458 (UTC) FILETIME=[A0853120:01C268E8] Sender: owner-obm-l@sucuri.mat.puc-rio.br Precedence: bulk Reply-To: obm-l@mat.puc-rio.br Ola Eder, Se vc encontrou para X1 e X2 pares,(b) impar e (a) par, entao a^2 é par e b^2 é impar,logo a^2 + b^2 é par diferente de 2 concluindo entao que a^2 + b^2 é composto. Um abraço,Leonardo >From: "Eder" >Reply-To: obm-l@mat.puc-rio.br >To: >Subject: [obm-l] Mais uma... >Date: Mon, 30 Sep 2002 21:20:42 -0300 > >Essa foi da olimpíada russa: > > >"Sabendo que a quadrática x²+ax+b+1 tem raízes inteiras e positivas,mostre >que a²+b² é composto." > >Bom,supondo X1 e X2 raízes,analisei as possibilidades de serem ambas >ímpares,uma par e outra ímpar e as duas pares.O único problema que >encontrei foi para o último caso.X1 e X2 pares implica b ímpar e a >par.Diferente dos outros casos,não pude ou não vi como concluir que a²+b² é >composto.Aguardo comentários. _________________________________________________________________ Tenha você também um MSN Hotmail, o maior webmail do mundo: http://www.hotmail.com/br ========================================================================= Instruções para entrar na lista, sair da lista e usar a lista em http://www.mat.puc-rio.br/~nicolau/olimp/obm-l.html O administrador desta lista é ========================================================================= From owner-obm-l@sucuri.mat.puc-rio.br Mon Sep 30 22:44:43 2002 Return-Path: Received: (from majordom@localhost) by sucuri.mat.puc-rio.br (8.9.3/8.9.3) id WAA04990 for obm-l-MTTP; Mon, 30 Sep 2002 22:42:02 -0300 Received: from bidu.ime.usp.br (bidu.ime.usp.br [143.107.45.12]) by sucuri.mat.puc-rio.br (8.9.3/8.9.3) with SMTP id WAA04986 for ; Mon, 30 Sep 2002 22:42:00 -0300 Received: (qmail 2256 invoked from network); 1 Oct 2002 01:44:11 -0000 Received: from mafalda.ime.usp.br (HELO jaca.ime.usp.br) (143.107.45.13) by bidu.ime.usp.br with SMTP; 1 Oct 2002 01:44:11 -0000 Received: (qmail 13419 invoked by uid 1604); 1 Oct 2002 01:44:11 -0000 Date: Mon, 30 Sep 2002 22:44:11 -0300 (EST) From: Salvador Addas Zanata X-Sender: sazanata@mafalda To: obm-l@mat.puc-rio.br Subject: [obm-l] Problema dos pontos e do circulo In-Reply-To: <009b01c26838$8f47cd60$1fadf8c8@mario> Message-ID: MIME-Version: 1.0 Content-Type: TEXT/PLAIN; charset=US-ASCII Sender: owner-obm-l@sucuri.mat.puc-rio.br Precedence: bulk Reply-To: obm-l@mat.puc-rio.br Oi pessoal, Acho que o Eduardo Casagrande mandou um problema pra lista, mais ou menos assim: Dados dois pontos P e Q e um circulo C, achar o ponto X do circulo, tal que XP+XQ seja minimo. Com regua e compasso. E claro que podemos supor que PQ nao intersecta C, caso contrario X e qualquer ponto de interseccao. Por analitica, com um pouco de calculo sai. Primeiro, Coloque um sistema de coordenadas de forma que o centro do circulo seja a origem. Agora, escolha o eixo x de modo que ou P ou Q caia sobre ele. Isso so pra facilitar as contas. Ai e so escrever a expressao de XP+XQ, por exemplo como funcao de teta, angulo sobre o circulo. Dai derivando em rel. a teta, etc, voce consegue obter uma forma simples pra tan(teta), como funcao do raio do circulo e das coordenadas dos pontos. Ai e so tracar a radial correspondente a esse teta e obter X. Nao pensei muito, entao espero nao estar falando bobagem. Abraco, Salvador ========================================================================= Instruções para entrar na lista, sair da lista e usar a lista em http://www.mat.puc-rio.br/~nicolau/olimp/obm-l.html O administrador desta lista é ========================================================================= From owner-obm-l@sucuri.mat.puc-rio.br Mon Sep 30 23:10:59 2002 Return-Path: Received: (from majordom@localhost) by sucuri.mat.puc-rio.br (8.9.3/8.9.3) id XAA05734 for obm-l-MTTP; Mon, 30 Sep 2002 23:07:49 -0300 Received: from ginsberg.uol.com.br (ginsberg.uol.com.br [200.221.4.48]) by sucuri.mat.puc-rio.br (8.9.3/8.9.3) with ESMTP id XAA05730 for ; Mon, 30 Sep 2002 23:07:47 -0300 Received: from emmanuela ([200.150.128.234]) by ginsberg.uol.com.br (8.9.1/8.9.1) with SMTP id XAA24119 for ; Mon, 30 Sep 2002 23:08:10 -0300 (BRT) Message-ID: <002b01c268ef$17b670a0$ea8096c8@emmanuela> From: "Paulo Rodrigues" To: References: <004a01c268e0$83a9c6e0$082efea9@Eder> Subject: Re: [obm-l] Mais uma... Date: Mon, 30 Sep 2002 23:06:00 -0300 MIME-Version: 1.0 Content-Type: multipart/alternative; boundary="----=_NextPart_000_0028_01C268D5.F1982D00" X-Priority: 3 X-MSMail-Priority: Normal X-Mailer: Microsoft Outlook Express 6.00.2600.0000 X-MimeOLE: Produced By Microsoft MimeOLE V6.00.2600.0000 Sender: owner-obm-l@sucuri.mat.puc-rio.br Precedence: bulk Reply-To: obm-l@mat.puc-rio.br This is a multi-part message in MIME format. ------=_NextPart_000_0028_01C268D5.F1982D00 Content-Type: text/plain; charset="iso-8859-1" Content-Transfer-Encoding: quoted-printable Sendo m e n as ra=EDzes, temos a=3D -(m+n) =3D> a^2=3Dm^2+2mn+n^2 b+1=3Dmn =3D> b^2=3D(mn-1)^2=3Dm^2n^2-2mn+1 Logo a^2+b^2=3Dm^2n^2 + m^2 + n^2 +1 =3D (m^2+1)(n^2+1) ----- Original Message -----=20 From: Eder=20 To: obm-l@mat.puc-rio.br=20 Sent: Monday, September 30, 2002 9:20 PM Subject: [obm-l] Mais uma... Essa foi da olimp=EDada russa: "Sabendo que a quadr=E1tica x=B2+ax+b+1 tem ra=EDzes inteiras e = positivas,mostre que a=B2+b=B2 =E9 composto." Bom,supondo X1 e X2 ra=EDzes,analisei as possibilidades de serem ambas = =EDmpares,uma par e outra =EDmpar e as duas pares.O =FAnico problema que = encontrei foi para o =FAltimo caso.X1 e X2 pares implica b =EDmpar e a = par.Diferente dos outros casos,n=E3o pude ou n=E3o vi como concluir que = a=B2+b=B2 =E9 composto.Aguardo coment=E1rios. ------=_NextPart_000_0028_01C268D5.F1982D00 Content-Type: text/html; charset="iso-8859-1" Content-Transfer-Encoding: quoted-printable
Sendo m e n as ra=EDzes, temos
 
a=3D -(m+n) =3D> a^2=3Dm^2+2mn+n^2
b+1=3Dmn =3D> = b^2=3D(mn-1)^2=3Dm^2n^2-2mn+1
Logo
a^2+b^2=3Dm^2n^2 + m^2 + n^2 +1 =3D = (m^2+1)(n^2+1)
----- Original Message -----
From:=20 Eder
Sent: Monday, September 30, = 2002 9:20=20 PM
Subject: [obm-l] Mais = uma...

Essa foi da olimp=EDada = russa:
 
 
"Sabendo que a quadr=E1tica = x=B2+ax+b+1 tem ra=EDzes=20 inteiras e positivas,mostre que a=B2+b=B2 =E9 composto."
 
Bom,supondo X1 e X2 ra=EDzes,analisei = as=20 possibilidades de serem ambas =EDmpares,uma par e outra =EDmpar e as = duas pares.O=20 =FAnico problema que encontrei foi para o =FAltimo caso.X1 e X2 pares=20 implica b =EDmpar e a par.Diferente dos outros casos,n=E3o pude = ou n=E3o vi=20 como concluir que a=B2+b=B2 =E9 composto.Aguardo=20 coment=E1rios.
------=_NextPart_000_0028_01C268D5.F1982D00-- ========================================================================= Instruções para entrar na lista, sair da lista e usar a lista em http://www.mat.puc-rio.br/~nicolau/olimp/obm-l.html O administrador desta lista é ========================================================================= From owner-obm-l@sucuri.mat.puc-rio.br Mon Sep 30 23:47:37 2002 Return-Path: Received: (from majordom@localhost) by sucuri.mat.puc-rio.br (8.9.3/8.9.3) id XAA06534 for obm-l-MTTP; Mon, 30 Sep 2002 23:44:37 -0300 Received: from salvatore4.bol.com.br ([200.221.24.52]) by sucuri.mat.puc-rio.br (8.9.3/8.9.3) with ESMTP id XAA06530 for ; Mon, 30 Sep 2002 23:44:35 -0300 Received: from bol.com.br (200.221.24.136) by salvatore4.bol.com.br (5.1.071) id 3D8B888000231E01 for obm-l@mat.puc-rio.br; Mon, 30 Sep 2002 23:46:42 -0300 Date: Mon, 30 Sep 2002 23:43:39 -0300 Message-Id: Subject: Re:[obm-l] Mais uma... MIME-Version: 1.0 Content-Type: multipart/mixed; boundary="_=__=_XaM3_Boundary.1033440219.2A.364994.42.1321.52.42.101010.1071079306" From: "tarsis19" To: obm-l@mat.puc-rio.br X-XaM3-API-Version: 2.4.3.4.4 X-SenderIP: 198.81.9.3 Sender: owner-obm-l@sucuri.mat.puc-rio.br Precedence: bulk Reply-To: obm-l@mat.puc-rio.br --_=__=_XaM3_Boundary.1033440219.2A.364994.42.1321.52.42.101010.1071079306 Content-Type: text/plain;charset="iso-8859-1" Content-Transfer-Encoding: quoted-printable substitua a por -(X1 + X2) e b por X1X2 a express=E3o a=B2+ b=B2 fica (X1)=B2+ (X2)=B2 + (X1X2)=B2 + 1 (X1)=B2[(X2)=B2 + 1] + (X2)=B2 + 1 Colocando (X2)=B2 + 1 em evid=EAncia, temos: [(X1)=B2 + 1][(X2)=B2 + 1] =3D a=B2 + b=B2 __________________________________________________________________________ Encontre sempre uma linha desocupada com o Discador BOL! http://www.bol.com.br/discador Ainda n=E3o tem AcessoBOL? Assine j=E1! http://www.bol.com.br/acessobol --_=__=_XaM3_Boundary.1033440219.2A.364994.42.1321.52.42.101010.1071079306 Content-Type: text/plain; name="00000MKF" Content-Transfer-Encoding: base64 RXNzYSBmb2kgZGEgb2xpbXDtYWRhIHJ1c3NhOg0KDQoNCiJTYWJlbmRvIHF1ZSBhIHF1YWRy 4XRpY2EgeLIrYXgrYisxIHRlbSByYe16ZXMgaW50ZWlyYXMgZSBwb3NpdGl2YXMsbW9zdHJl IHF1ZSBhsitisiDpIGNvbXBvc3RvLiINCg0KQm9tLHN1cG9uZG8gWDEgZSBYMiByYe16ZXMs YW5hbGlzZWkgYXMgcG9zc2liaWxpZGFkZXMgZGUgc2VyZW0gYW1iYXMg7W1wYXJlcyx1bWEg cGFyIGUgb3V0cmEg7W1wYXIgZSBhcyBkdWFzIHBhcmVzLk8g+m5pY28gcHJvYmxlbWEgcXVl IGVuY29udHJlaSBmb2kgcGFyYSBvIPpsdGltbyBjYXNvLlgxIGUgWDIgcGFyZXMgaW1wbGlj YSBiIO1tcGFyIGUgYSBwYXIuRGlmZXJlbnRlIGRvcyBvdXRyb3MgY2Fzb3MsbuNvIHB1ZGUg b3UgbuNvIHZpIGNvbW8gY29uY2x1aXIgcXVlIGGyK2KyIOkgY29tcG9zdG8uQWd1YXJkbyBj b21lbnThcmlvcy4NCg== --_=__=_XaM3_Boundary.1033440219.2A.364994.42.1321.52.42.101010.1071079306-- ========================================================================= Instruções para entrar na lista, sair da lista e usar a lista em http://www.mat.puc-rio.br/~nicolau/olimp/obm-l.html O administrador desta lista é ========================================================================= From owner-obm-l@sucuri.mat.puc-rio.br Mon Sep 30 23:59:47 2002 Return-Path: Received: (from majordom@localhost) by sucuri.mat.puc-rio.br (8.9.3/8.9.3) id XAA06816 for obm-l-MTTP; Mon, 30 Sep 2002 23:57:11 -0300 Received: from smtp-3.ig.com.br (smtp-3.ig.com.br [200.226.132.152]) by sucuri.mat.puc-rio.br (8.9.3/8.9.3) with SMTP id XAA06812 for ; Mon, 30 Sep 2002 23:57:09 -0300 Received: (qmail 12334 invoked from network); 1 Oct 2002 02:58:58 -0000 Received: from 200-140-80-021-bsace7026.dsl.telebrasilia.net.br (HELO henrique) (200.140.80.21) by smtp-3.ig.com.br with SMTP; 1 Oct 2002 02:58:58 -0000 Message-ID: <015101c268f6$8077bed0$019da8c0@henrique> From: "Henrique Branco" To: References: <002401c2674b$48a9f060$6249d8c8@rodrigo> Subject: [obm-l] =?iso-8859-1?Q?Re:_=5Bobm-l=5D_Re:_=5Bobm-l=5D_d=FAvida?= Date: Mon, 30 Sep 2002 23:59:02 -0300 MIME-Version: 1.0 Content-Type: text/plain; charset="iso-8859-1" Content-Transfer-Encoding: 8bit X-Priority: 3 X-MSMail-Priority: Normal X-Mailer: Microsoft Outlook Express 6.00.2600.0000 X-MimeOLE: Produced By Microsoft MimeOLE V6.00.2600.0000 Sender: owner-obm-l@sucuri.mat.puc-rio.br Precedence: bulk Reply-To: obm-l@mat.puc-rio.br Pergunta besta... Mas se 610 = (1001100010)_2, por que ele acertou as perguntas 2, 6, 7, 10? Desculpem pelo "nivel primario" da pergunta... Mas... Grato, Henrique. ----- Original Message ----- From: Rodrigo Villard Milet To: obm-l@mat.puc-rio.br Sent: Saturday, September 28, 2002 9:00 PM Subject: [obm-l] Re: [obm-l] dúvida Escreva 610 na base 2 : 610 = (1001100010)_2. Como sabemos que a representação na base 2 é única, ele acertou as perguntas 2,6,7 e 10. Villard -----Mensagem original----- De: Mário Pereira Para: obm-l@mat.puc-rio.br Data: Sábado, 28 de Setembro de 2002 11:22 Assunto: [obm-l] dúvida Olá, se alguém puder, me dê uma dica: Em um jogo de televisão, um candidato deve responder a 10 perguntas. A primeira vale 1 ponto, a segunda vale 2 pontos, e assim, sucessivamente, dobrando sempre. O candidato responde a todas as perguntas e ganha os pontos correspondentes às respostas que acertou, mesmo que erre algumas. Se o candidato obteve 610 pontos, quantas perguntas acertou? Obrigado, Mário. ========================================================================= Instruções para entrar na lista, sair da lista e usar a lista em http://www.mat.puc-rio.br/~nicolau/olimp/obm-l.html O administrador desta lista é =========================================================================
-----Mensagem= Original-----
De: Silvio
Enviado: quarta-feira, 11 de setembro de 2002 00:45
Para: obm-l@mat.puc-rio.br
Assunto: [obm-l] Ajuda Algebra linear (Off Topi= c)
 
Caros amigos,

    Preciso= apresentar um trabalho sobre algebra linear aplicada, gostaria
de sug= est=F5es de sites e/ou livros e at=E9 mesmo quais das aplica=E7=F5es de a= lg.
linear seria interessante de apresentar,"E ,claro, seja f=E1cil de= encontrar
documenta=E7=E3o".

Agrade=E7o desde j=E1.

Silvio
srtb@bol.com.br


=3D=3D=3D=3D=3D=3D=3D=3D=3D=3D=3D=3D= =3D=3D=3D=3D=3D=3D=3D=3D=3D=3D=3D=3D=3D=3D=3D=3D=3D=3D=3D=3D=3D=3D=3D=3D=3D= =3D=3D=3D=3D=3D=3D=3D=3D=3D=3D=3D=3D=3D=3D=3D=3D=3D=3D=3D=3D=3D=3D=3D=3D=3D= =3D=3D=3D=3D=3D=3D=3D=3D=3D=3D=3D
Instru=E7=F5es para entrar na lista,= sair da lista e usar a lista em
http://www.mat.puc-rio.br/~nicolau/ol= imp/obm-l.html
O administrador desta lista =E9 <nicolau@mat.puc-rio= .br>
=3D=3D=3D=3D=3D=3D=3D=3D=3D=3D=3D=3D=3D=3D=3D=3D=3D=3D=3D=3D=3D= =3D=3D=3D=3D=3D=3D=3D=3D=3D=3D=3D=3D=3D=3D=3D=3D=3D=3D=3D=3D=3D=3D=3D=3D=3D= =3D=3D=3D=3D=3D=3D=3D=3D=3D=3D=3D=3D=3D=3D=3D=3D=3D=3D=3D=3D=3D=3D=3D=3D=3D= =3D=3D